(Davis's Success Series.) Colgrove, Kathryn Cadenhead - Med-Surg Success - A Q & A Review Applying Critical Thinking To Test Taking-F.a. Davis Company (2017) 1

You might also like

Download as pdf or txt
Download as pdf or txt
You are on page 1of 801

Med-Surg Success

A Q&A Review Applying Critical Thinking


to Test Taking
Third Edition

00_Colgrove_FM.indd 1 03/06/16 12:56 PM


STRESSED ABOUT THE

NCLEX ?
®

Study smarter with Davis Edge NCLEX-RN.®


You’ll have 10,000+ questions at your fingertips to practice from your
mobile device or laptop. Immediate feedback will highlight the content
areas you are struggling in so that you know where to spend more time
studying. Davis Edge allows you to create targeted practice quizzes to
improve those areas—before you get to the NCLEX.

Davis Edge is an incredible asset that is


essential in preparing for an exam.
- Michal W., Hill College

Comprehensive, helpful, and a total life saver.


– Lauren R., ECPI

Davis Edge NCLEX-RN® gives you the practice


you need to confidently pass your boards.
• Over 10,000 NCLEX-style questions to create
practice quizzes on-the-go
• Comprehensive Exam that simulates the NCLEX
• Rationales for correct and incorrect answers
• Anxiety-reducing test-taking tips

SAVE 20%
Use Promo Code: ORANGE5
START STUDYING
TODAY!
www.FADavis.com
Promotion subject to change without notice. Offer valid for purchases from www.FADavis.com in the US only.

Lagerquist In-book ad_2016--PMS661.indd 1 4/13/16 9:36 AM

00_Colgrove_FM.indd 2 03/06/16 12:56 PM


Med-Surg Success
A Q&A Review Applying Critical Thinking
to Test Taking
Third Edition

Kathryn Cadenhead Colgrove, RN, MSN, CNS

00_Colgrove_FM.indd 3 03/06/16 12:56 PM


F. A. Davis Company
1915 Arch Street
Philadelphia, PA 19103
www.fadavis.com

Copyright © 2017 by F.A. Davis Company

Copyright © 2017 by F.A. Davis Company. All rights reserved. This book is protected by copyright.
No part of it may be reproduced, stored in a retrieval system, or transmitted in any form or by any
means, electronic, mechanical, photocopying, recording, or otherwise, without written permission
from the publisher.

Printed in the United States of America

Last digit indicates print number: 10 9 8 7 6 5 4 3 2 1

Publisher, Nursing: Terri Wood Allen


Content Project Manager: Julia L. Curcio
Electronic Project Manager: Sandra A. Glennie
Design and Illustrations Manager: Carolyn O’Brien

As new scientific information becomes available through basic and clinical research, recommended
treatments and drug therapies undergo changes. The author(s) and publisher have done everything
possible to make this book accurate, up to date, and in accord with accepted standards at the time of
publication. The author(s), editors, and publisher are not responsible for errors or omissions or for
consequences from application of the book, and make no warranty, expressed or implied, in regard to
the contents of the book. Any practice described in this book should be applied by the reader in accor-
dance with professional standards of care used in regard to the unique circumstances that may apply in
each situation. The reader is advised always to check product information (package inserts) for changes
and new information regarding dose and contraindications before administering any drug. Caution is
especially urged when using new or infrequently ordered drugs.

Library of Congress Cataloging-in-Publication Data

Names: Colgrove, Kathryn Cadenhead, author.


Title: Med-surg success : a Q&A review applying critical thinking to test
taking / Kathryn Cadenhead Colgrove.
Description: Third edition. | Philadelphia, PA : F.A. Davis Company, [2017] |
Includes bibliographical references and index.
Identifiers: LCCN 2016019304 | ISBN 9780803644021
Subjects: | MESH: Nursing | Nursing Care | Test Taking Skills | Problem
Solving | Examination Questions
Classification: LCC RD99.25 | NLM WY 18.2 | DDC 617/.0231076--dc23 LC record available at
https://lccn.loc.gov/2016019304

Authorization to photocopy items for internal or personal use, or the internal or personal use of
specific clients, is granted by F. A. Davis Company for users registered with the Copyright Clearance
Center (CCC) Transactional Reporting Service, provided that the fee of $.25 per copy is paid directly
to CCC, 222 Rosewood Drive, Danvers, MA 01923. For those organizations that have been granted a
photocopy license by CCC, a separate system of payment has been arranged. The fee code for users of
the Transactional Reporting Service is: 8036-1169-2/04 0 + $.25.

00_Colgrove_FM.indd 4 03/06/16 12:56 PM


The authors dedicate this book to all nursing students who provide safe, competent,
and “caring” nursing care which makes a difference in their clients’ lives. We
hope this book helps these students to be successful in their nursing program,
successful on their NCLEX-RN, and successful in the nursing profession. Our
continued thanks to Bob Martone, who took a chance on us, and to Julia Curcio,
who continues to support our endeavors in the publishing world. We would also like
to thank Kara Evans for her editorial support during the production of the third
edition. We hope the nursing students who use this book have as long, wonderful,
and exciting a career as we have had in the nursing profession.

I would like to dedicate this book to the memory of my mother, Mary Cadenhead,
and grandmother, Elsie Rogers. They always said that I could accomplish anything
I wanted to accomplish. I would also like to dedicate this book to my husband,
Larry, children Laurie and Todd and Larry Jr. and Mai, and grandchildren
Chris, Ashley, Justin C., Justin A., Connor, Sawyer, and Carson. Without their
support and patience the book would not have been possible.
Kathryn Colgrove

This book is dedicated to the memory of my husband, Bill, and my parents, T/Sgt.
Leo and Nancy Hargrove, who are the rocks on which my life is built. I would like
to thank my sisters, Gail and Debbie, my nephew Benjamin, and Paula for their
support and encouragement through the good times and the bad. My children,
Teresa and Aaron, are the most important people in my life and I want to thank
them for always believing in me.
Ray Hargrove-Huttel

(This was Ray’s dedication. Ray lost her battle with cancer on December 23, 2012.
She is much loved and greatly missed. Kathy Colgrove)

00_Colgrove_FM.indd 5 03/06/16 12:56 PM


00_Colgrove_FM.indd 6 03/06/16 12:56 PM
Reviewers
Beth Batturs Martin, RN, MSN Christina Flint, RN, MSN, MBA
Director of Nursing and Healthcare Assistant Professor
Initiatives University of Indianapolis
Anne Arundel Community College Indianapolis, Indiana
Arnold, Maryland
Rebecca Fountain, RN, PhD
Glenda Bondurant, MSN, RN Assistant Professor
Dean of Allied Health/Sciences University of Texas at Tyler
Wilson Community College Tyler, Texas
Wilson, North Carolina
Charlene Beach Gagliardi, RN,
Johnnie A. Bratton, MSN, BSN, RN BSN, MSN
ADN Instructor Assistant Professor
Durham Technical Community College Mount Saint Mary’s University
Durham, North Carolina Los Angeles, California
Marsha Cannon, EdD, MSN, RN
Carol Girocco, RN, MSN, OCN
Associate Professor
Professor of Nursing
University of West Alabama
Lone Star College – Montgomery
Livingston, Alabama
Conroe, Texas
Barbara A. Caton, MSN, RN, CNE
Assistant Professor Cheryl Harrington, MSN/MHA
Missouri State University – West Plains Assistant Professor of Nursing
West Plains, Missouri Morningside College
Sioux City, Iowa
Lorraine Smirle Collins, RN,
MSN, CNOR Jackie Harris, MNSc, RN, CNE
Assistant Professor, Nursing Assistant Professor of Nursing
Piedmont Virginia Community College Harding University Carr College
Charlottesville, Virginia of Nursing
Searcy, Arkansas
Pamela K. DeMoss, MSN, RN
Assistant Professor Stephanie D. Huff, RN, DNP, FNP-BC
University of Dubuque Nursing Faculty
Dubuque, Iowa Southern Union State Community
College
MaryAnn Edelman, RN, MS Opelika, Alabama
Professor, Department of Nursing
Kingsborough Community College Patricia A. Kelly, MSN, RN, APRN,
Brooklyn, New York FNP-BC
Lecturer
Karen Elsea, MSN, RN Southern Illinois University
Assistant Professor Edwardsville
University of Indianapolis Edwardsville, Illinois
Indianapolis, Indiana
Julie Kolker, MSN, BS, RN
Marie Everhart, MSN, RN, CNE
Associate Degree Nursing Instructor
Associate Professor
North Iowa Area Community College
Northampton Community College
Mason City, Iowa
Bethlehem, Pennsylvania

vii

00_Colgrove_FM.indd 7 03/06/16 12:56 PM


viii R eviewers

Mary Jo Konkloski, MSN, RN, ANP Gina Purdue, DNP, RN, CNE
Instructor Associate Professor
Pomeroy College of Nursing at Crouse Eastern Kentucky University
Hospital Richmond, Kentucky
Syracuse, New York
Lillian Rafeldt, RN, MA, CNE
Karen Kulhanek, BSN, MEd, MSN Professor of Nursing
Nursing Professor Three Rivers Community College
Kellogg Community College Norwich, Connecticut
Battle Creek, Michigan
Lori Riden, MSN, RN, CNE
Donna E. McCabe, DNP, APRN-BC, Nursing Professor
GNP Yavapai College
Clinical Assistant Professor Prescott, Arizona
New York University College of Nursing
New York, New York Brenda Sloan, RN, MA
Assistant Professor
Ann Motycka, RN, MSN, CNE Indiana Wesleyan University
Professor Marion, Indiana
Ivy Tech Community College
Evansville, Indiana Cheryl Sorge, MA, BSN, RN
Associate Professor, Tenured
Michelle M. Murphy-Rozanski, Indiana University-Purdue University
PhD, MSN, RN, CRNP Fort Wayne
Senior Level Coordinator Fort Wayne, Indian
Temple University Health System-
Northeastern School of Nursing Beryl Stetson, RNBC, MSN, CNE,
Philadelphia, Pennsylvania LCCE, CLC
Chairperson, Health Science Education
Lee Anne Nichols, PhD, RN Department
Associate Professor Associate Professor, Nursing
University of Tulsa Raritan Valley Community College
Tulsa, Oklahoma Branchburg, New Jersey

Laura C. Parker, MSN, RN, Stephanie L. Turrise, PhD, RN, BC,


CCRN, CNE APRN, CNE
Associate Professor, Nursing Assistant Professor
County College of Morris University of North Carolina, Wilmington
Randolph, New Jersey Wilmington, North Carolina

Susan M. Perry, MS, RN Renée Wright, EdD, RN


Assistant Professor of Nursing Assistant Professor
SUNY Adirondack York College
Queensbury, New York Jamaica, New York

00_Colgrove_FM.indd 8 03/06/16 12:56 PM


Editors and Contributors
to Past Editions
Judy Callicoatt, RN, MS, CNS Leslie Prater, RN, MS, CNS, CDE
Associate Degree Nursing Instructor Clinical Diabetes Educator
Trinity Valley Community College Associate Degree Nursing Instructor
Kaufman, Texas Trinity Valley Community College
Kaufman, Texas
Joan L. Consullo, RN, MS, CNRN
Advanced Clinical Nurse, Neuroscience Helen Reid, RN, PhD
St. Luke’s Episcopal Hospital Dean, Health Occupations
Houston, Texas Trinity Valley Community College
Kaufman, Texas
Michelle L. Edwards, RN, MSN,
ACNP, FNP Elester E. Stewart, RRT, RN,
Advanced Practice Nurse, Cardiology MSN, FNP
Acute Care Nurse Practitioner/Family Advanced Practice Nurse, Pulmonary
Nurse Practitioner Family Nurse Practitioner
St. Luke’s Episcopal Hospital St. Luke’s Episcopal Hospital
Houston, Texas Houston, Texas

Gail F. Graham, APRN, MS, NP-C


Advanced Practice Nurse, Internal
Medicine
Adult Nurse Practitioner
St. Luke’s Episcopal Hospital
Houston, Texas

ix

00_Colgrove_FM.indd 9 03/06/16 12:56 PM


00_Colgrove_FM.indd 10 03/06/16 12:56 PM
Table of Contents

1 Test Taking ......................................................................................................................................... 1


INTRODUCTION ...................................................................................................................................... 1
GUIDELINES FOR USING THIS BOOK ............................................................................................ 1
PREPARING FOR LECTURE ................................................................................................................ 2
Sample Study Guide ......................................................................................................................... 2
PREPARING FOR AN EXAMINATION ................................................................................................ 3
Study .................................................................................................................................................... 3
Understanding What the Test Taker Does Not Know ................................................................ 3
The Night Before the Exam ............................................................................................................ 4
The Day of the Exam ........................................................................................................................ 4
Test-Taking Anxiety ........................................................................................................................... 4
TAKING THE EXAM ................................................................................................................................ 4
Test-Taking Hints for the Computerized NCLEX-RN Examination ......................................... 4
Understanding the Types of Nursing Questions ......................................................................... 5
THE RACE MODEL: THE APPLICATION OF CRITICAL THINKING
TO MULTIPLE-CHOICE QUESTIONS .............................................................................................. 6
CONCEPT-FOCUSED QUESTIONS ..................................................................................................... 6

2 Neurological Disorders ............................................................................................................. 9


KEYWORDS/ABBREVIATIONS ............................................................................................................. 9
PRACTICE QUESTIONS ...................................................................................................................... 10
Cerebrovascular Accident (Stroke) .............................................................................................. 10
Head Injury ....................................................................................................................................... 11
Spinal Cord Injury (Sci) ................................................................................................................ 12
Seizures ............................................................................................................................................. 14
Brain Tumors .................................................................................................................................... 15
Meningitis ......................................................................................................................................... 16
Parkinson’s Disease ........................................................................................................................ 17
Substance Abuse ............................................................................................................................. 19
Amyotrophic Lateral Sclerosis (ALS or Lou Gehrig’s Disease) ............................................. 20
Encephalitis ...................................................................................................................................... 21
CONCEPTS ............................................................................................................................................. 22
PRACTICE QUESTIONS ANSWERS AND RATIONALES ............................................................. 26
Cerebrovascular Accident (Stroke) .............................................................................................. 26
Head Injury ....................................................................................................................................... 29
Spinal Cord Injury ........................................................................................................................... 32
Seizures ............................................................................................................................................. 34
Brain Tumors .................................................................................................................................... 37
Meningitis ......................................................................................................................................... 40
Parkinson’s Disease ........................................................................................................................ 43
Substance Abuse ............................................................................................................................. 45
Amyotrophic Lateral Sclerosis (ALS or Lou Gehrig’s Disease) ............................................. 48
Encephalitis ...................................................................................................................................... 50
CONCEPTS ............................................................................................................................................. 53
NEUROLOGICAL DISORDERS COMPREHENSIVE EXAMINATION .......................................... 57
NEUROLOGICAL DISORDERS COMPREHENSIVE EXAMINATION
ANSWERS AND RATIONALES ........................................................................................................ 63

xi

00_Colgrove_FM.indd 11 03/06/16 12:56 PM


xii T able of C ontents

3 Cardiac Disorders ....................................................................................................................... 71


KEYWORDS/ABBREVIATIONS ........................................................................................................... 71
PRACTICE QUESTIONS ...................................................................................................................... 72
Congestive Heart Failure ............................................................................................................... 72
Angina/Myocardial Infarction ........................................................................................................ 73
Coronary Artery Disease ................................................................................................................. 74
Valvular Heart Disease ................................................................................................................... 75
Dysrhythmias and Conduction Problems ................................................................................... 77
Inflammatory Cardiac Disorders .................................................................................................. 78
CONCEPTS ............................................................................................................................................. 79
PRACTICE QUESTIONS ANSWERS AND RATIONALES ............................................................. 82
Congestive Heart Failure ............................................................................................................... 82
Angina/Myocardial Infarction ........................................................................................................ 84
Coronary Artery Disease ................................................................................................................. 87
Valvular Heart Disease ................................................................................................................... 90
Dysrhythmias and Conduction Problems ................................................................................... 92
Inflammatory Cardiac Disorders .................................................................................................. 95
CONCEPTS ............................................................................................................................................. 98
CARDIAC DISORDERS COMPREHENSIVE EXAMINATION ..................................................... 102
CARDIAC DISORDERS COMPREHENSIVE EXAMINATION ANSWERS
AND RATIONALES .......................................................................................................................... 106

4 Peripheral Vascular Disorders ....................................................................................... 111


KEYWORDS/ABBREVIATIONS ........................................................................................................ 111
PRACTICE QUESTIONS ................................................................................................................... 112
Arterial Hypertension ................................................................................................................... 112
Arterial Occlusive Disease .......................................................................................................... 113
Atherosclerosis .............................................................................................................................. 114
Abdominal Aortic Aneurysm ...................................................................................................... 115
Deep Vein Thrombosis ................................................................................................................ 117
Peripheral Venous Disease ......................................................................................................... 118
CONCEPTS .......................................................................................................................................... 119
PRACTICE QUESTIONS ANSWERS AND RATIONALES .......................................................... 122
Arterial Hypertension ................................................................................................................... 122
Arterial Occlusive Disease .......................................................................................................... 124
Atherosclerosis .............................................................................................................................. 127
Abdominal Aortic Aneurysm ...................................................................................................... 129
Deep Vein Thrombosis ................................................................................................................ 132
Peripheral Venous Disease ......................................................................................................... 134
CONCEPTS .......................................................................................................................................... 137
PERIPHERAL VASCULAR DISORDERS COMPREHENSIVE EXAMINATION ....................... 141
PERIPHERAL VASCULAR DISORDERS COMPREHENSIVE EXAMINATION
ANSWERS AND RATIONALES ..................................................................................................... 145

5 Hematological Disorders .................................................................................................... 149


KEYWORDS/ABBREVIATIONS ........................................................................................................ 149
PRACTICE QUESTIONS ................................................................................................................... 150
Leukemia ........................................................................................................................................ 150
Lymphoma ...................................................................................................................................... 151
Anemia ............................................................................................................................................ 152
Bleeding Disorders ....................................................................................................................... 153
Blood Transfusions ....................................................................................................................... 154
Sickle Cell Anemia ....................................................................................................................... 156
CONCEPTS .......................................................................................................................................... 157
PRACTICE QUESTIONS ANSWERS AND RATIONALES .......................................................... 159
Leukemia ........................................................................................................................................ 159
Lymphoma ...................................................................................................................................... 161

00_Colgrove_FM.indd 12 03/06/16 12:56 PM


T able of C ontents xiii

Anemia ............................................................................................................................................ 164


Bleeding Disorders ....................................................................................................................... 167
Blood Transfusions ....................................................................................................................... 169
Sickle Cell Anemia ....................................................................................................................... 173
CONCEPTS .......................................................................................................................................... 176
HEMATOLOGICAL DISORDERS COMPREHENSIVE EXAMINATION ..................................... 179
HEMATOLOGICAL DISORDERS COMPREHENSIVE EXAMINATION
ANSWERS AND RATIONALES ..................................................................................................... 183

6 Respiratory Disorders ........................................................................................................... 189


KEYWORDS/ABBREVIATIONS ........................................................................................................ 189
PRACTICE QUESTIONS ................................................................................................................... 190
Upper Respiratory Infection (URI) ........................................................................................... 190
Lower Respiratory Infection ....................................................................................................... 191
Chronic Obstructive Pulmonary Disease (Copd) .................................................................. 192
Reactive Airway Disease (Asthma) ........................................................................................... 194
Lung Cancer .................................................................................................................................. 195
Cancer of the Larynx ................................................................................................................... 198
Pulmonary Embolus ..................................................................................................................... 199
Chest Trauma ................................................................................................................................ 200
Acute Respiratory Distress Syndrome ..................................................................................... 201
CONCEPTS .......................................................................................................................................... 203
PRACTICE QUESTIONS ANSWERS AND RATIONALES .......................................................... 206
Upper Respiratory Infection ...................................................................................................... 206
Lower Respiratory Infection ....................................................................................................... 208
Chronic Obstructive Pulmonary Disease (COPD) .................................................................. 211
Reactive Airway Disease (Asthma) ........................................................................................... 214
Lung Cancer .................................................................................................................................. 216
Cancer of the Larynx ................................................................................................................... 219
Pulmonary Embolus ..................................................................................................................... 222
Chest Trauma ................................................................................................................................ 224
Acute Respiratory Distress Syndrome (ARDS) ...................................................................... 227
RESPIRATORY DISORDERS COMPREHENSIVE EXAMINATION ........................................... 233
RESPIRATORY DISORDERS COMPREHENSIVE EXAMINATION
ANSWERS AND RATIONALES ..................................................................................................... 239

7 Gastrointestinal Disorders ................................................................................................ 249


KEYWORDS/ABBREVIATIONS ........................................................................................................ 249
PRACTICE QUESTIONS ................................................................................................................... 250
Gastroesophageal Reflux (GERD) ............................................................................................. 250
Inflammatory Bowel Disease ..................................................................................................... 251
Peptic Ulcer Disease ................................................................................................................... 252
Colorectal Disease ........................................................................................................................ 253
Diverticulosis/Diverticulitis ......................................................................................................... 254
Gallbladder Disorders .................................................................................................................. 255
Liver Failure ................................................................................................................................... 256
Hepatitis ......................................................................................................................................... 258
Gastroenteritis ............................................................................................................................... 259
Abdominal Surgery ....................................................................................................................... 260
Eating Disorders ........................................................................................................................... 261
Constipation/Diarrhea Disorders ............................................................................................... 262
CONCEPTS .......................................................................................................................................... 263
PRACTICE QUESTIONS ANSWERS AND RATIONALES .......................................................... 265
Gastroesophageal Reflux (GERD) ............................................................................................. 265
Inflammatory Bowel Disease ..................................................................................................... 267
Peptic Ulcer Disease ................................................................................................................... 269
Colorectal Disease ........................................................................................................................ 272

00_Colgrove_FM.indd 13 03/06/16 12:56 PM


xiv T able of C ontents

Diverticulosis/Diverticulitis ......................................................................................................... 275


Gallbladder Disorders .................................................................................................................. 277
Liver Failure ................................................................................................................................... 280
Hepatitis ......................................................................................................................................... 282
Gastroenteritis ............................................................................................................................... 285
Abdominal Surgery ....................................................................................................................... 287
Eating Disorders ........................................................................................................................... 290
Constipation/Diarrhea Disorders ............................................................................................... 292
GASTROINTESTINAL DISORDERS COMPREHENSIVE EXAMINATION .............................. 298
GASTROINTESTINAL DISORDERS COMPREHENSIVE EXAMINATION
ANSWERS AND RATIONALES ..................................................................................................... 304

8 Endocrine Disorders .............................................................................................................. 313


KEYWORDS/ABBREVIATIONS ........................................................................................................ 313
PRACTICE QUESTIONS ................................................................................................................... 314
Diabetes Mellitus ......................................................................................................................... 314
Pancreatitis .................................................................................................................................... 316
Cancer of the Pancreas ............................................................................................................... 317
Adrenal Disorders ......................................................................................................................... 319
Pituitary Disorders ........................................................................................................................ 320
Thyroid Disorders ......................................................................................................................... 321
CONCEPTS .......................................................................................................................................... 322
PRACTICE QUESTIONS ANSWERS AND RATIONALES .......................................................... 325
Diabetes Mellitus ......................................................................................................................... 325
Pancreatitis .................................................................................................................................... 330
Cancer of the Pancreas ............................................................................................................... 332
Adrenal Disorders ......................................................................................................................... 335
Pituitary Disorders ........................................................................................................................ 337
Thyroid Disorders ......................................................................................................................... 339
CONCEPTS .......................................................................................................................................... 343
ENDOCRINE DISORDERS COMPREHENSIVE EXAMINATION .............................................. 346
ENDOCRINE DISORDERS COMPREHENSIVE EXAMINATION
ANSWERS AND RATIONALES ..................................................................................................... 350

9 Genitourinary Disorders ...................................................................................................... 355


KEYWORDS/ABBREVIATIONS ........................................................................................................ 355
PRACTICE QUESTIONS ................................................................................................................... 356
Acute Renal Failure (ARF) ......................................................................................................... 356
Chronic Kidney Disease (CKD) .................................................................................................. 357
Fluid and Electrolyte Disorders ................................................................................................. 358
Urinary Tract Infection (Uti) ..................................................................................................... 359
Benign Prostatic Hypertrophy ................................................................................................... 361
Renal Calculi ................................................................................................................................. 362
Cancer of the Bladder ................................................................................................................. 363
CONCEPTS .......................................................................................................................................... 365
PRACTICE QUESTIONS ANSWERS AND RATIONALES .......................................................... 368
Acute Renal Failure (ARF) ......................................................................................................... 368
Chronic Kidney Disease (CKD) .................................................................................................. 370
Fluid and Electrolyte Disorders ................................................................................................. 373
Urinary Tract Infection (UTI) ..................................................................................................... 376
Benign Prostatic Hypertrophy ................................................................................................... 378
Renal Calculi ................................................................................................................................. 380
Cancer of the Bladder ................................................................................................................. 383
CONCEPTS .......................................................................................................................................... 386
GENITOURINARY DISORDERS COMPREHENSIVE EXAMINATION ..................................... 389
GENITOURINARY DISORDERS COMPREHENSIVE EXAMINATION
ANSWERS AND RATIONALES ..................................................................................................... 392

00_Colgrove_FM.indd 14 03/06/16 12:56 PM


T able of C ontents xv

10 Reproductive Disorders ...................................................................................................... 397


KEYWORDS/ABBREVIATIONS ........................................................................................................ 397
PRACTICE QUESTIONS ................................................................................................................... 398
Breast Disorders ........................................................................................................................... 398
Pelvic Floor Relaxation Disorders ............................................................................................. 399
Uterine Disorders ......................................................................................................................... 400
Ovarian Disorders ......................................................................................................................... 401
Prostate Disorders ........................................................................................................................ 403
Testicular Disorders ..................................................................................................................... 404
Sexually Transmitted Diseases .................................................................................................. 405
CONCEPTS .......................................................................................................................................... 406
PRACTICE QUESTIONS ANSWERS AND RATIONALES .......................................................... 409
Breast Disorders ........................................................................................................................... 409
Pelvic Floor Relaxation Disorders ............................................................................................. 412
Uterine Disorders ......................................................................................................................... 414
Ovarian Disorders ......................................................................................................................... 416
Prostate Disorders ........................................................................................................................ 419
Testicular Disorders ..................................................................................................................... 422
Sexually Transmitted Diseases .................................................................................................. 424
CONCEPTS .......................................................................................................................................... 427
REPRODUCTIVE DISORDERS COMPREHENSIVE EXAMINATION ....................................... 430
REPRODUCTIVE DISORDERS COMPREHENSIVE EXAMINATION
ANSWERS AND RATIONALES ..................................................................................................... 434

11 Musculoskeletal Disorders ............................................................................................... 439


KEYWORDS/ABBREVIATIONS ........................................................................................................ 439
PRACTICE QUESTIONS ................................................................................................................... 440
Degenerative/Herniated Disk Disease ...................................................................................... 440
Osteoarthritis ................................................................................................................................. 441
Osteoporosis .................................................................................................................................. 442
Amputation .................................................................................................................................... 443
Fractures ........................................................................................................................................ 444
Joint Replacements ..................................................................................................................... 446
CONCEPTS .......................................................................................................................................... 447
PRACTICE QUESTIONS ANSWERS AND RATIONALES .......................................................... 450
Degenerative/Herniated Disk Disease ...................................................................................... 450
Osteoarthritis ................................................................................................................................. 452
Osteoporosis .................................................................................................................................. 454
Amputation .................................................................................................................................... 457
Fractures ........................................................................................................................................ 459
Joint Replacements ..................................................................................................................... 461
CONCEPTS .......................................................................................................................................... 464
MUSCULOSKELETAL DISORDERS COMPREHENSIVE EXAMINATION .............................. 467
MUSCULOSKELETAL DISORDERS COMPREHENSIVE EXAMINATION
ANSWERS AND RATIONALES ..................................................................................................... 470

12 Integumentary Disorders ................................................................................................... 475


KEYWORDS/ABBREVIATIONS ........................................................................................................ 475
PRACTICE QUESTIONS ................................................................................................................... 475
Burns ............................................................................................................................................... 475
Pressure Ulcers ............................................................................................................................. 477
Skin Cancer ................................................................................................................................... 478
Bacterial Skin Infection .............................................................................................................. 479
Viral Skin Infection ...................................................................................................................... 481
Fungal/Parasitic Skin Infection ................................................................................................. 482
CONCEPTS .......................................................................................................................................... 484
PRACTICE QUESTIONS ANSWERS AND RATIONALES .......................................................... 486

00_Colgrove_FM.indd 15 03/06/16 12:56 PM


xvi T able of C ontents

Burns ............................................................................................................................................... 486


Pressure Ulcers ............................................................................................................................. 488
Skin Cancer ................................................................................................................................... 491
Bacterial Skin Infection .............................................................................................................. 493
Viral Skin Infection ...................................................................................................................... 496
Fungal/Parasitic Skin Infection ................................................................................................. 499
CONCEPTS .......................................................................................................................................... 501
INTEGUMENTARY DISORDERS COMPREHENSIVE EXAMINATION .................................... 504
INTEGUMENTARY DISORDERS COMPREHENSIVE EXAMINATION
ANSWERS AND RATIONALES ..................................................................................................... 507

13 Immune System Disorders ............................................................................................... 513


KEYWORDS/ABBREVIATIONS ........................................................................................................ 513
PRACTICE QUESTIONS ................................................................................................................... 514
Multiple Sclerosis ......................................................................................................................... 514
Guillain-Barré Syndrome ............................................................................................................ 515
Myasthenia Gravis ........................................................................................................................ 516
Systemic Lupus Erythematosus ................................................................................................ 518
Acquired Immunodeficiency Syndrome .................................................................................. 519
Allergies and Allergic Reactions ............................................................................................... 521
Rheumatoid Arthritis (RA) .......................................................................................................... 522
CONCEPTS .......................................................................................................................................... 523
PRACTICE QUESTIONS ANSWERS AND RATIONALES .......................................................... 526
Multiple Sclerosis ......................................................................................................................... 526
Guillain-Barré Syndrome ............................................................................................................ 528
Myasthenia Gravis ........................................................................................................................ 531
Systemic Lupus Erythematosus ................................................................................................ 533
Acquired Immunodeficiency Syndrome .................................................................................. 536
Allergies and Allergic Reactions ............................................................................................... 538
Rheumatoid Arthritis (RA) .......................................................................................................... 541
CONCEPTS .......................................................................................................................................... 544
IMMUNE SYSTEM DISORDERS COMPREHENSIVE EXAMINATION .................................... 547
IMMUNE SYSTEM DISORDERS COMPREHENSIVE EXAMINATION
ANSWERS AND RATIONALES ..................................................................................................... 551

14 Sensory Deficits ....................................................................................................................... 557


KEYWORDS/ABBREVIATIONS ........................................................................................................ 557
PRACTICE QUESTIONS ................................................................................................................... 558
Eye Disorders ................................................................................................................................ 558
Ear Disorders ................................................................................................................................. 559
CONCEPTS .......................................................................................................................................... 560
PRACTICE QUESTIONS ANSWERS AND RATIONALES .......................................................... 562
Eye Disorders ................................................................................................................................ 562
Ear Disorders ................................................................................................................................. 564
CONCEPTS .......................................................................................................................................... 567
SENSORY DEFICITS COMPREHENSIVE EXAMINATION ......................................................... 569
SENSORY DEFICITS COMPREHENSIVE EXAMINATION ANSWERS
AND RATIONALES .......................................................................................................................... 572

15 Emergency Nursing ............................................................................................................... 577


KEYWORDS/ABBREVIATIONS ........................................................................................................ 577
PRACTICE QUESTIONS ................................................................................................................... 578
Shock .............................................................................................................................................. 578
Bioterrorism ................................................................................................................................... 579
Codes ............................................................................................................................................... 580
Disasters/Triage ............................................................................................................................. 581
Poisoning ........................................................................................................................................ 582
Violence, Physical Abuse, and Neglect ................................................................................... 584

00_Colgrove_FM.indd 16 03/06/16 12:56 PM


T able of C ontents xvii

CONCEPTS .......................................................................................................................................... 585


PRACTICE QUESTIONS ANSWERS AND RATIONALES .......................................................... 588
Shock .............................................................................................................................................. 588
Bioterrorism ................................................................................................................................... 591
Codes ............................................................................................................................................... 594
Disasters/Triage ............................................................................................................................. 596
Poisoning ........................................................................................................................................ 599
Violence, Physical Abuse, and Neglect ................................................................................... 602
CONCEPTS .......................................................................................................................................... 605
EMERGENCY NURSING COMPREHENSIVE EXAMINATION .................................................. 607
EMERGENCY NURSING COMPREHENSIVE EXAMINATION ANSWERS
AND RATIONALES .......................................................................................................................... 610

16 Perioperative Care .................................................................................................................. 615


KEYWORDS/ABBREVIATIONS ........................................................................................................ 615
PRACTICE QUESTIONS ................................................................................................................... 616
Preoperative ................................................................................................................................... 616
Intraoperative ................................................................................................................................ 617
Postoperative ................................................................................................................................. 618
Acute Pain ..................................................................................................................................... 619
CONCEPTS .......................................................................................................................................... 620
PRACTICE QUESTIONS ANSWERS AND RATIONALES .......................................................... 623
Preoperative ................................................................................................................................... 623
Intraoperative ................................................................................................................................ 625
Postoperative ................................................................................................................................. 628
Acute Pain ..................................................................................................................................... 630
CONCEPTS .......................................................................................................................................... 633
PERIOPERATIVE CARE COMPREHENSIVE EXAMINATION .................................................... 635
PERIOPERATIVE CARE COMPREHENSIVE EXAMINATION ANSWERS
AND RATIONALES .......................................................................................................................... 638

17 Cultural and Spiritual Nursing and Alternative Health Care ............... 643
KEYWORDS/ABBREVIATIONS ........................................................................................................ 643
PRACTICE QUESTIONS ................................................................................................................... 644
CONCEPTS .......................................................................................................................................... 647
PRACTICE QUESTIONS ANSWERS AND RATIONALES .......................................................... 649
CONCEPTS ANSWERS AND RATIONALES ................................................................................. 655
CULTURAL NURSING AND ALTERNATIVE HEALTH CARE
COMPREHENSIVE EXAMINATION ............................................................................................. 658
CULTURAL NURSING AND ALTERNATIVE HEALTH CARE COMPREHENSIVE
EXAMINATION ANSWERS AND RATIONALES ........................................................................ 661

18 End-of-Life Issues .................................................................................................................. 667


ABBREVIATIONS ............................................................................................................................... 667
PRACTICE QUESTIONS ................................................................................................................... 668
Advance Directives ....................................................................................................................... 668
Death and Dying ........................................................................................................................... 669
Chronic Pain .................................................................................................................................. 670
Ethical/Legal Issues ..................................................................................................................... 672
Organ/Tissue Donation ................................................................................................................ 673
CONCEPTS .......................................................................................................................................... 675
PRACTICE QUESTIONS ANSWERS AND RATIONALES .......................................................... 676
Advance Directives ....................................................................................................................... 676
Death and Dying ........................................................................................................................... 678
Chronic Pain .................................................................................................................................. 681
Ethical/Legal Issues ..................................................................................................................... 684
Organ/Tissue Donation ................................................................................................................ 687

00_Colgrove_FM.indd 17 03/06/16 12:56 PM


xviii T able of C ontents

CONCEPTS ANSWERS AND RATIONALES ................................................................................. 690


END-OF-LIFE ISSUES COMPREHENSIVE EXAMINATION ..................................................... 692
END-OF-LIFE ISSUES COMPREHENSIVE EXAMINATION ANSWERS
AND RATIONALES .......................................................................................................................... 696

19 Pharmacology ............................................................................................................................. 701


KEYWORDS/ABBREVIATIONS ........................................................................................................ 701
TEST-TAKING HINTS FOR PHARMACOLOGY QUESTIONS .................................................... 702
Drug Cards ..................................................................................................................................... 702
Sample Drug Cards.......................................................................................................... 704
Example #1: Digoxin ................................................................................................................... 705
Example #2: Furosemide ........................................................................................................... 705
MEDICATIONS ADMINISTRATION IN A MEDICAL-SURGICAL SETTING
COMPREHENSIVE EXAMINATION ............................................................................................. 706
MEDICATIONS ADMINISTRATION IN A MEDICAL-SURGICAL SETTING
COMPREHENSIVE EXAMINATION ANSWERS AND RATIONALES .................................... 717

20 Comprehensive Final Examination ............................................................................ 733


COMPREHENSIVE FINAL EXAMINATION ................................................................................... 733
COMPREHENSIVE FINAL EXAMINATION ANSWERS AND RATIONALES .......................... 744

Glossary of English Words Commonly Encountered


on Nursing Examinations .................................................................................................. 761
Appendix A: Normal Laboratory Values ................................................................ 765
Appendix B: Sample Nursing Concept Care Map ........................................ 767
Illustration Credits .................................................................................................................. 771
Index .................................................................................................................................................. 773

00_Colgrove_FM.indd 18 03/06/16 12:56 PM


INTRODUCTION
Test Taking 1
This book is part of a series of books published by the F.A. Davis Company designed to assist
the student nurse to be successful in nursing school. This book focuses on critical thinking in
regard to test taking. There are the usual test questions found in review books, but the test
taker will also find test-taking hints in 19 of 20 chapters. Table 1-1 indicates the breakdown
of the content found on the NCLEX-RN. This book has attempted to follow this blueprint.
The 2016–2019 NCLEX-RN blueprint includes additional types of alternative test ques-
tions. The end of each chapter includes these new graphic types of test questions.
The most important aspect of taking any examination is to become knowledgeable about
the subject matter the test will cover. There is no substitute for studying the material.
Book one of this series—Fundamentals Success: A Q & A Review Applying Critical Thinking
to Test Taking, 4th edition, by Patricia M. Nugent and Barbara A. Vitale—defines critical
thinking and the RACE model for applying critical thinking to test-taking skills, and the
specific topics in that volume will not be repeated in this book. This book will also assist
the test taker to apply critical-thinking skills directly to the questions found on nursing
examinations.

GUIDELINES FOR USING THIS BOOK


This book is designed to assist the nursing student when preparing for and taking medical-
surgical examinations as well as the graduate nurse who is preparing to take the national
licensure examination. The book is divided into chapters according to body systems and
includes chapters on pharmacology, end-of-life issues, emergency nursing, and alternative
therapy; a 100-question final examination; and the DavisPlus site containing 150 additional
questions. Chapters 2 through 18 are further divided into disease processes to more easily
help the test taker identify specific content. A comprehensive final including other content
areas as well as disease processes is included at the end of each chapter.
In the Answers and Rationales sections of the chapters, an explanation for the c­ orrect
­answer as well as the incorrect distracters is provided. In Chapters 2 through 18, a Test-Taking
Hint is provided for every question in the content area. Each question is coded according
to Bloom’s Taxonomy for categorizing levels of abstraction required for the test taker to
­arrive at the correct answer. As specified by the National Council of State Boards of ­Nursing
(­NCSBN) testing information, the questions are coded by Content, Integrated Nursing
Process, ­Client Needs, Cognitive Level (Bloom’s Taxonomy), and Concept. There are no
test-taking hints provided for the comprehensive final examinations or the questions on
­DavisPlus. In C­ hapter 19, the pharmacology chapter, the test-taking hints are discussed in
the beginning of the chapter.

01_Colgrove_Ch01.indd 1 01/06/16 3:32 PM


2 M ed -S urg S uccess

Table 1-1 Client Needs Categories and Percentage of Items


Client Needs Percentage of Items

SAFE AND EFFECTIVE CARE ENVIRONMENT


• Management of Care 17–23%
• Safety and Infection Control 9–15%
• Health Promotion and Maintenance 6–12%
• Psychosocial Integrity 6–12%

PHYSIOLOGICAL INTEGRITY
• Basic Care and Comfort 6–12%
• Pharmacological and Parenteral Therapies 12–18%
• Reduction of Risk Potential 9–15%
• Physiological Adaption 11–17%

The National Council of State Boards of Nursing, Inc., Chicago, IL, with permission.

PREPARING FOR LECTURE


In preparation for attending class on a specific topic, the student should have read the as-
signment and prepared notes to take to class. Any information the student does not under-
stand should be highlighted in order to clarify the information if the instructor does not
cover it in class or if, after the lecture, the student still does not understand the concept.
A piece of paper, or study guide, divided into categories of information should be suf-
ficient for most disease processes. If the student is unable to limit the information to one
page, the student is probably not being discriminatory when reading. The idea is not to
rewrite the textbook but rather to glean from the textbook the important, need-to-know
information.

Sample Study Guide

Medical Diagnosis: Definition:


DIAGNOSTIC TESTS: SIGNS AND SYMPTOMS: NURSING INTERVENTIONS:
(List normal values) (Include teaching)

PROCEDURES AND NURSING


IMPLICATIONS:

MEDICAL INTERVENTIONS:

Complete the study guide in one color of pen but take a different color or a pencil to class
along with a highlighter and the study guide. Whatever the instructor emphasizes during
the lecture on the study guide should be highlighted. Whatever information the instruc-
tor emphasizes in the lecture that the student did not include on the study guide should be

01_Colgrove_Ch01.indd 2 01/06/16 3:32 PM


C hapter 1  T est T aking 3

written in the different color pen or pencil. The student should reread the information in
the textbook that was included in lecture but was not included on the study guide. When
studying for the examination, the student can identify the information obtained from the
textbook and the information obtained in class. The information on the study guide that is
highlighted represents information that the student thought was important from reading the
textbook and the instructor emphasized during the lecture. This is important, need-to-know
information for the examination. Please note, however, the instructor may not emphasize
laboratory tests and values but still expects the student to realize the importance of this
information.
The completed study guides can be carried with the student in a folder and reviewed dur-
ing children’s sports practices, when waiting for an appointment, or at any time the student
finds a minute that is spent idly. This is making the most of limited time. The study guides
should also be carried to clinical assignments to use when caring for clients in the hospital.
Students who prepare before attending class will find the lecture easier to understand
and, as a result, those students will score higher on examinations. Being prepared allows the
student to listen to the instructor and not sit in class trying to write every word from the
overhead presentation.
The student should recognize the importance of the instructor’s hints during the l­ ecture.
The instructor may emphasize information by highlighting areas on overhead slides, re-
peating information, or emphasizing a particular fact, which usually means the instructor
thinks the information is very important. Important information usually finds its way onto tests
at some point.

PREPARING FOR AN EXAMINATION


Study
The student should plan to study three (3) hours for every one (1) hour of class. For example,
a course that is three (3) hours of credit requires nine (9) hours of study each week. Cram-
ming immediately before the test usually indicates the student is at risk for being unsuc-
cessful on the examination. The information acquired during cramming is not learned and
will be quickly forgotten. Nursing examinations include material required by the registered
nurse when caring for clients at the bedside. The knowledge required to care for clients
builds on ALL previous knowledge as well as information newly acquired.

Understanding What the Test Taker Does Not Know


The first time many students realize they do not understand the information is during the
examination, when it is too late. Nursing examinations contain high-level application ques-
tions requiring the test taker not only to have memorized information but also to be able
to interpret the data and make a judgment as to the correct course of action. Test takers
must recognize their own areas of weakness before seeing an examination for the first time.
This book is designed to assist test takers to identify their areas of weakness before the
examination.
Two (2) to three (3) days before the examination, the student should compose a practice
test and take the examination. If a topic of study proves to be an area of strength, as evi-
denced by selecting the correct answer to the question, then the student should proceed to
study other areas identified as weaknesses. Missing the question identifies a weakness. If the
test taker does not understand the rationale for the correct answer, the test taker should read
the appropriate textbook and try to understand the rationale for the correct answer. The
test taker should be cautious about reading rationales for the incorrect distracters. During
the examination, the test taker may remember reading the information but could become
confused about whether the information applied to the correct answer or to the incorrect
distracter.

01_Colgrove_Ch01.indd 3 01/06/16 3:32 PM


4 M ed -S urg S uccess

The Night Before the Exam


The night before the examination, the student should quit studying by 6:00 to 7:00 p.m. The
student should do something fun or relaxing up until bedtime and get a good night’s rest
before taking the examination. Studying until bedtime or in an all-night cram session leaves
the student tired and sleepy during the examination, which is just when the mind should be
at its top performance.

The Day of the Exam


The student should eat a meal before an examination; a source of carbohydrate for energy
along with a protein source makes a good meal. Skipping a meal before the examination
leaves the brain without nourishment. A bagel with peanut butter and milk is an excel-
lent meal; it provides a source of protein and sustained release of carbohydrates. Do not
eat donuts or drink soft drinks. The energy from these is quickly available but will not last
throughout the time required for an examination. Excessive fluid intake may cause the need
to urinate during the examination and make it hard for the test taker to concentrate.

Test-Taking Anxiety
If the student has test-taking anxiety, then it is advisable for the student to arrive at the test-
ing site 45 minutes before the examination. Find a seat for the examination and place books
there to reserve the desk. The student should walk for 15 minutes at a fast pace away from
the testing site and at the end of the 15 minutes the student should turn and walk back. This
exercise literally walks anxiety away.
If other test takers getting up and leaving the room bothers the test taker, the test taker
should try to get a desk away from the group, in front of the room or facing a wall. Most
schools allow students to wear hunter’s ear plugs during a test if noise bothers the student.
Most NCLEX-RN test sites will provide ear plugs if the graduate requests them.

TAKING THE EXAM


During the examination, if the test taker finds a question that contains totally unknown in-
formation and the test taker is taking a pencil-and-paper test, the test taker should circle the
question and skip it. Another question may help to answer the skipped question. Delaying
moving on and worrying over a question will place the next few questions in jeopardy. The
mind will not let go of the worry, and the test taker may miss important information in the
subsequent questions.
During the NCLEX-RN computerized test, the test taker should take some deep breaths
and then select an answer. The computer does not allow the test taker to return to a question.
Test takers who become anxious during an examination should stop, put their hands in
their lap, shut their eyes, and take a minimum of five (5) deep breaths before resuming the
examination. The test taker must become aware of personal body signals that indicate in-
creasing stress levels. Some people get gastrointestinal symptoms and others feel a tighten-
ing of muscles.

Test-Taking Hints for the Computerized NCLEX-RN Examination


A computer administers the NCLEX-RN examination. Here are some test-taking hints that
specifically apply to examination by computer.
• The test is composed of 75 to 265 questions. The computer determines with a 95% cer-
tainty that the test taker’s ability is above the passing standard before the examination will
conclude. The minimum number of questions the test taker will receive is 75 questions.

01_Colgrove_Ch01.indd 4 01/06/16 3:32 PM


C hapter 1  T est T aking 5

• The examination comprises multiple-choice questions and may include several types of
alternate questions:
• Fill-in-the-blank questions, which test math abilities.
• “Select all that apply” questions, which require the test taker to select more than
one distracter as the correct answer. In a “Select All” question, a minimum of two an-
swers will be correct and in some questions all of the options may be correct. This is a
new revision of the NCLEX-RN examination. Previously, there had to be at least one
incorrect option. Click-and-drag questions, which require the test taker to identify a
specific area of the body as the correct answer.
• An audio component, which requires the test taker to identify body sounds.
Examples of most of these types of questions are included in this book. In an attempt to
illustrate the click-and-drag questions, this book has pictures with lines to delineate A,
B, C, and D. A fifth type of question which prioritizes the answers 1, 2, 3, 4, and 5 in
order of when the nurse would implement the intervention is also included in this
book. Finally, the audio questions can be found on the F.A. Davis Web site.
• Test takers should not be overly concerned if they possess rudimentary computer skills.
The test taker must use the mouse to select the correct answer. Every question asks for a
confirmation before being submitted as the answer.
• Other than typing pertinent personal information, the test taker must be able to type
numbers and use the drop-down computer calculator. The test taker can request an erase
slate to calculate math problems by hand.
• The test taker should practice taking tests on the computer before taking the NCLEX-RN
examination. Many textbooks contain computer disks with test questions, and there are
many online review opportunities.
• The test taker should refer to the Web site for the National Council of State Boards
of Nursing (http://www.ncsbn.org) for additional information on the NCLEX-RN
examination.

Understanding the Types of Nursing Questions

Components of a Multiple-Choice Question


A multiple-choice question is called an item. Each item has two parts. The stem is the
part that contains the information that identifies the topic and its parameters and then asks
a question. The second part consists of one or more possible responses, which are called
­options. One of the options is the correct answer and the others are the wrong answers,
called distracters.

The client diagnosed with angina complains of chest pain while ambulating
Stem
in the hall. Which intervention should the nurse implement first?

a. Have the client sit down. Correct answer

Options
b. Monitor the pulse oximeter reading.
c. Administer sublingual nitroglycerin. Distracters
d. Apply oxygen via nasal cannula.

Cognitive Levels of Nursing Questions


Questions on nursing examinations reflect a variety of thinking processes that nurses use
when caring for clients. These thinking processes are part of the cognitive domain, and they
progress from the simple to the complex, from the concrete to the abstract, and from the

01_Colgrove_Ch01.indd 5 01/06/16 3:32 PM


6 M ed -S urg S uccess

tangible to the intangible. There are four types of thinking processes represented by nursing
questions:
• Knowledge Questions—The emphasis is on recalling remembered information.
• Comprehension Questions—The emphasis is on understanding the meaning and intent
of remembered information.
• Application Questions—The emphasis is on remembering understood information and
utilizing the information in new situations.
• Analysis Questions—The emphasis is on comparing and contrasting a variety of elements
of information.

THE RACE MODEL: THE APPLICATION OF CRITICAL


THINKING TO MULTIPLE-CHOICE QUESTIONS
Answering a test question is like participating in a race. Of course, the test taker wants
to come in first and be the winner. However, the thing to remember about a race is that
success is not based just on speed but also on strategy and tactics. The same is true about
nursing examinations. Although speed may be a variable that must be considered when tak-
ing a timed test, so that the amount of time spent on each question is factored into the
test strategy, the emphasis on RACE is the use of critical-thinking techniques to answer
multiple-choice questions. The RACE Model presented next is a critical-thinking strategy
to use when a­ nswering nursing multiple-choice questions. If the test taker follows the RACE
model every time when examining a test question, its use will become second nature. This
methodical approach will improve the test taker’s abilities to critically analyze a test question
and improve chances of selecting the correct answer.
The RACE Model has four steps to answering a test question. The best way to remember
the four steps is to refer to the acronym RACE.
R—Recognize what information is in the stem.
• Recognize the key words in the stem.
• Recognize who the client is in the stem.
• Recognize what the topic is about.
A—Ask what is the question asking?
• Ask what are the key words in the stem that indicate the need for a response?
• Ask what is the question asking the nurse to implement?
C—Critically analyze the options in relation to the question asked in the stem.
• Critically scrutinize each option in relation to the information in the stem.
• Critically identify a rationale for each option.
• Critically compare and contrast the options in relation to the information in the stem
and their relationships to one another.
E—Eliminate as many options as possible.
• Eliminate one option at a time.
The text Fundamentals Success: A Q & A Review Applying Critical Thinking to Test Taking,
4th edition, by Patricia M. Nugent and Barbara A. Vitale, includes an in-depth discus-
sion exploring the RACE Model in relation to the thinking processes as represented in
multiple-choice nursing questions.

CONCEPT-FOCUSED QUESTIONS
A number of states and nursing programs within the states have chosen to use a Concept-based
curriculum approach to nursing education. Oregon, North Carolina, and Texas are among
these states. This is a change from the disease-based model of nursing education that followed
a ­disease-based approach. Over the years of curriculum development in nursing, the number of
topics has expanded under the disease-based curriculum to become burdensome and unwieldy

01_Colgrove_Ch01.indd 6 01/06/16 3:32 PM


C hapter 1  T est T aking 7

for students and faculty. These schools have transitioned to teaching “concepts” of client needs,
utilizing specific exemplars of commonly occurring disease processes to represent the concept to
students. This is not a new idea but rather a return to a previous method of nursing curriculum.
For example, if the curriculum being taught is Oxygenation, it would be taught using
the exemplars of pneumonia or chronic obstructive pulmonary disease (COPD). Under the
concept, there are similarities of nursing assessment guidelines and nursing interventions.
Assessing the client’s lung fields, elevating the head of the bed for maximum lung expansion,
and administering oxygen are applicable for both exemplars and also for asthma, respiratory
distress, myasthenia gravis with respiratory involvement, and others that require these same
interventions. The exemplars require the nurse to determine what makes this Oxygenation
problem different from a client with myasthenia. The concept problem focus is designed to
encourage critical thinking and problem solving. This book includes questions at the end of
most chapters that focus on the larger concept with the use of exemplars.

01_Colgrove_Ch01.indd 7 01/06/16 3:32 PM


01_Colgrove_Ch01.indd 8 01/06/16 3:32 PM
Neurological
Disorders
The first step toward knowledge is to know that we are not
ignorant.
2
—Richard Cecil

Test-taking hints are useful to discriminate information, but they cannot substitute for
knowledge. The student should refer to Chapter 1 for assistance in preparing for class,
studying, and taking an examination.
This chapter focuses on disorders that affect the neurological system. It provides a list
of keywords and abbreviations, practice questions focused on disease processes, and a com-
prehensive examination that includes other content areas involving the neurological system
and the disease processes addressed in the practice questions. Answers and reasons why the
answer options provided are either correct or incorrect are also provided, as are test-taking
hints. Subsequent chapters (Chapters 3 through 14) focus on disorders that affect other
body systems and function.

KEYWORDS     ABBREVIATIONS
Agnosia Activities of daily living (ADLs)
Akinesia Amyotrophic lateral sclerosis (ALS)
Aphasia Blood pressure (BP)
Apraxia Cerebrovascular accident (CVA)
Areflexia Computed tomography (CT)
Ataxia Electroencephalogram (EEG)
Autonomic dysreflexia Electromyogram (EMG)
Bradykinesia Emergency department (ED)
Decarboxylase Enzyme-linked immunosorbent assay (ELISA)
Diplopia Health-care provider (HCP)
Dysarthria Intensive care department (ICD)
Dysphagia Intracranial pressure (ICP)
Echolalia Intravenous (IV)
Epilepsy Magnetic resonance imaging (MRI)
Papilledema Nonsteroidal anti-inflammatory drug (NSAID)
Paralysis Nothing by mouth (NPO)
Paresthesia Parkinson’s disease (PD)
Paroxysms Pulse (P)
Penumbra Range of motion (ROM)
Postictal Respiration (R)

02_Colgrove_Ch02.indd 9 02/06/16 7:59 PM


10 M ed -S urg S uccess

Rule out (R/O)


Spinal cord injury (SCI)
STAT—immediately (STAT)
Temperature (T)
Transient ischemic attack (TIA)
Traumatic brain injury (TBI)
Unlicensed assistive personnel (UAP)

Please note: The term health-care provider, as used in this text, refers to a nurse practitioner
(NP), a physician (MD), an osteopath (DO), or a physician assistant (PA) who has prescrip-
tive authority. These providers are responsible for directing the care and providing orders
for the clients.

PRACTICE QUESTIONS
Cerebrovascular Accident (Stroke) 4. The client diagnosed with a right-sided
cerebrovascular accident is admitted to the
1. A 78-year-old client is admitted to the emergency rehabilitation unit. Which interventions should be
department (ED) with numbness and weakness of included in the nursing care plan? Select all that
the left arm and slurred speech. Which nursing apply.
intervention is priority? 1. Position the client to prevent shoulder
1. Prepare to administer recombinant tissue adduction.
plasminogen activator (rt-PA). 2. Turn and reposition the client every shift.
2. Discuss the precipitating factors that caused 3. Encourage the client to move the affected side.
the symptoms. 4. Perform quadriceps exercises three (3) times a
3. Schedule for a STAT computed tomography day.
(CT) scan of the head. 5. Instruct the client to hold the fingers in a fist.
4. Notify the speech pathologist for an emergency 5. The nurse is planning care for a client experiencing
consult. agnosia secondary to a cerebrovascular accident.
2. The nurse is assessing a client experiencing motor Which collaborative intervention will be included
loss as a result of a left-sided cerebrovascular in the plan of care?
accident (CVA). Which 1. Observe the client swallowing for possible
clinical manifestation would the nurse document? aspiration.
1. Hemiparesis of the client’s left arm and apraxia. 2. Position the client in a semi-Fowler’s position
2. Paralysis of the right side of the body and when sleeping.
ataxia. 3. Place a suction setup at the client’s bedside
3. Homonymous hemianopsia and diplopia. during meals.
4. Impulsive behavior and hostility toward family. 4. Refer the client to an occupational therapist for
evaluation.
3. Which client would the nurse identify as being
most at risk for experiencing a cerebrovascular 6. The nurse and an unlicensed assistive personnel
accident (CVA)? (UAP) are caring for a client with right-sided
1. A 55-year-old African American male. paralysis. Which action by the UAP requires the
2. An 84-year-old Japanese female. nurse to intervene?
3. A 67-year-old Caucasian male. 1. The assistant places a gait belt around the
4. A 39-year-old pregnant female. client’s waist prior to ambulating.
2. The assistant places the client on the back with
the client’s head to the side.
3. The assistant places a hand under the client’s
right axilla to move up in bed.
4. The assistant praises the client for attempting
to perform ADLs independently.

02_Colgrove_Ch02.indd 10 02/06/16 7:59 PM


C hapter 2   N eurological D isorders 11

7. The client diagnosed with atrial fibrillation has Head Injury


experienced a transient ischemic attack (TIA).
Which medication would the nurse anticipate 13. The client diagnosed with a mild concussion
being ordered for the client on discharge? is being discharged from the emergency
1. An oral anticoagulant medication. department. Which discharge instruction should
2. A beta blocker medication. the nurse teach the client’s significant other?
3. An anti-hyperuricemic medication. 1. Awaken the client every two (2) hours.
4. A thrombolytic medication. 2. Monitor for increased intracranial
8. The client has been diagnosed with a pressure (ICP).
cerebrovascular accident (stroke). The 3. Observe frequently for hypervigilance.
client’s wife is concerned about her husband’s 4. Offer the client food every three (3) to four
generalized weakness. Which home modification (4) hours.
should the nurse suggest to the wife prior to 14. The resident in a long-term care facility fell
discharge? during the previous shift and has a laceration in
1. Obtain a rubber mat to place under the dinner the occipital area that has been closed with steri-
plate. strips. Which signs/symptoms would warrant
2. Purchase a long-handled bath sponge for transferring the resident to the emergency
showering. department?
3. Purchase clothes with Velcro closure devices. 1. A 4-cm area of bright red drainage on the
4. Obtain a raised toilet seat for the client’s dressing.
bathroom. 2. A weak pulse, shallow respirations, and cool
9. The client is diagnosed with expressive aphasia. pale skin.
Which psychosocial client problem would the 3. Pupils that are equal, react to light, and
nurse include in the plan of care? accommodate.
1. Potential for injury. 4. Complaints of a headache that resolves with
2. Powerlessness. medication.
3. Disturbed thought processes. 15. The nurse is caring for several clients. Which
4. Sexual dysfunction. client would the nurse assess first after receiving
10. Which assessment data would indicate to the the shift report?
nurse that the client would be at risk for a 1. The 22-year-old male client diagnosed with
hemorrhagic stroke? a concussion who is complaining someone is
1. A blood glucose level of 480 mg/dL. waking him up every two (2) hours.
2. A right-sided carotid bruit. 2. The 36-year-old female client admitted with
3. A blood pressure (BP) of 220/120 mm Hg. complaints of left-sided weakness who is
4. The presence of bronchogenic carcinoma. scheduled for a magnetic resonance imaging
(MRI) scan.
11. The 85-year-old client diagnosed with a stroke 3. The 45-year-old client admitted with blunt
is complaining of a severe headache. Which trauma to the head after a motorcycle
intervention should the nurse implement first? accident who has a Glasgow Coma Scale
1. Administer a nonnarcotic analgesic. (GCS) score of 6.
2. Prepare for STAT magnetic resonance 4. The 62-year-old client diagnosed with a
imaging (MRI). cerebrovascular accident (CVA) who has
3. Start an intravenous infusion with D5W at expressive aphasia.
100 mL/hr.
4. Complete a neurological assessment. 16. The client has sustained a severe closed head
injury and the neurosurgeon is determining if
12. A client diagnosed with a subarachnoid the client is “brain dead.” Which data support
hemorrhage has undergone a craniotomy that the client is brain dead?
for repair of a ruptured aneurysm. Which 1. When the client’s head is turned to the right,
intervention will the intensive care nurse the eyes turn to the right.
implement? 2. The electroencephalogram (EEG) has
1. Administer a stool softener bid. identifiable waveforms.
2. Encourage the client to cough hourly. 3. No eye activity is observed when the cold
3. Monitor neurological status every shift. caloric test is performed.
4. Maintain the dopamine drip to keep BP 4. The client assumes decorticate posturing
at 160/90. when painful stimuli are applied.

02_Colgrove_Ch02.indd 11 02/06/16 7:59 PM


12 M ed -S urg S uccess

17. The client is admitted to the medical floor with provider to respond to the accident. Which
a diagnosis of closed head injury. Which nursing intervention should be implemented first?
intervention has priority? 1. Assess the client’s level of consciousness.
1. Assess neurological status. 2. Organize onlookers to remove the client from
2. Monitor pulse, respiration, and blood the lake.
pressure. 3. Perform a head-to-toe assessment to
3. Initiate an intravenous access. determine injuries.
4. Maintain an adequate airway. 4. Stabilize the client’s cervical spine.
18. The client diagnosed with a closed head injury 23. The client is diagnosed with a closed head
is admitted to the rehabilitation department. injury and is in a coma. The nurse writes the
Which medication order would the nurse client problem as “high risk for immobility
question? complications.” Which intervention would be
1. A subcutaneous anticoagulant. included in the plan of care?
2. An intravenous osmotic diuretic. 1. Position the client with the head of the bed
3. An oral anticonvulsant. elevated at intervals.
4. An oral proton pump inhibitor. 2. Perform active range-of-motion (ROM)
exercises every four (4) hours.
19. The client diagnosed with a gunshot wound to
3. Turn the client every shift and massage bony
the head assumes decorticate posturing when the
prominences.
nurse applies painful stimuli. Which assessment
4. Explain all procedures to the client before
data obtained three (3) hours later would
performing them.
indicate the client is improving?
1. Purposeless movement in response to painful 24. The 29-year-old client who was employed as
stimuli. a forklift operator sustains a traumatic brain
2. Flaccid paralysis in all four extremities. injury (TBI) secondary to a motor-vehicle
3. Decerebrate posturing when painful stimuli accident. The client is being discharged from the
are applied. rehabilitation unit after three (3) months and has
4. Pupils that are 6 mm in size and nonreactive cognitive deficits. Which goal would be most
on painful stimuli. realistic for this client?
1. The client will return to work within six (6)
20. The nurse is caring for a client diagnosed
months.
with an epidural hematoma. Which nursing
2. The client is able to focus and stay on task for
interventions should the nurse implement?
10 minutes.
Select all that apply.
3. The client will be able to dress self without
1. Maintain the head of the bed at 60 degrees of
assistance.
elevation.
4. The client will regain bowel and bladder
2. Administer stool softeners daily.
control.
3. Ensure the pulse oximeter reading is higher
than 93%.
4. Perform deep nasal suction every two (2) Spinal Cord Injury (SCI)
hours.
5. Administer mild sedatives. 25. The nurse arrives at the site of a one-car motor-
21. The client with a closed head injury has clear vehicle accident and stops to render aid. The
fluid draining from the nose. Which action driver of the car is unconscious. After stabilizing
should the nurse implement first? the client’s cervical spine, which action should
1. Notify the health-care provider immediately. the nurse take next?
2. Prepare to administer an antihistamine. 1. Carefully remove the driver from the car.
3. Test the drainage for presence of glucose. 2. Assess the client’s pupils for reaction.
4. Place a 2 × 2 gauze under the nose to collect 3. Assess the client’s airway.
drainage. 4. Attempt to wake the client up by shaking him.

22. The nurse is enjoying a day at the lake and 26. In assessing a client with a Thoracic SCI, which
witnesses a water skier hit the boat ramp. The clinical manifestation would the nurse expect
water skier is in the water not responding to to find to support the diagnosis of neurogenic
verbal stimuli. The nurse is the first health-care shock?
1. No reflex activity below the waist.
2. Inability to move upper extremities.
3. Complaints of a pounding headache.
4. Hypotension and bradycardia.

02_Colgrove_Ch02.indd 12 02/06/16 7:59 PM


C hapter 2   N eurological D isorders 13

27. The rehabilitation nurse caring for the client 32. The client with a cervical fracture is being
with an Lumbar SCI is developing the nursing discharged in a halo device. Which teaching
care plan. Which intervention should the nurse instruction should the nurse discuss with the
implement? client?
1. Keep oxygen via nasal cannula on at 1. Discuss how to correctly remove the insertion
all times. pins.
2. Administer low-dose subcutaneous 2. Instruct the client to report reddened or
anticoagulants. irritated skin areas.
3. Perform active lower extremity ROM 3. Inform the client that the vest liner cannot be
exercises. changed.
4. Refer to a speech therapist for ventilator- 4. Encourage the client to remain in the recliner
assisted speech. as much as possible.
28. The nurse in the neurointensive care unit is 33. The intensive care nurse is caring for a client
caring for a client with a new Cervical SCI with a T1 SCI. When the nurse elevates the head
who is breathing independently. Which nursing of the bed 30 degrees, the client complains of
interventions should be implemented? Select all light-headedness and dizziness. The client’s vital
that apply. signs are T 99.2°F, P 98, R 24, and BP 84/40.
1. Monitor the pulse oximetry reading. Which action should the nurse implement?
2. Provide pureed foods six (6) times a day. 1. Notify the health-care provider as soon as
3. Encourage coughing and deep breathing. possible (ASAP).
4. Assess for autonomic dysreflexia. 2. Calm the client down by talking
5. Administer intravenous corticosteroids. therapeutically.
3. Increase the IV rate by 50 mL/hour.
29. The home health nurse is caring for a 28-year-
4. Lower the head of the bed immediately.
old client with a T10 SCI who says, “I can’t
do anything. Why am I so worthless?” Which 34. The nurse is caring for clients on the
statement by the nurse would be the most rehabilitation unit. Which clients should the
therapeutic? nurse assess first after receiving the change-
1. “This must be very hard for you. You’re of-shift report?
feeling worthless?” 1. The client with a C6 SCI who is complaining
2. “You shouldn’t feel worthless—you are still of dyspnea and has crackles in the lungs.
alive.” 2. The client with an L4 SCI who is crying and
3. “Why do you feel worthless? You still have the very upset about being discharged home.
use of your arms.” 3. The client with an L2 SCI who is
4. “If you attended a work rehab program you complaining of a headache and feeling
wouldn’t feel worthless.” very hot.
4. The client with a T4 SCI who is unable to
30. The client is diagnosed with an SCI and is
move the lower extremities.
scheduled for a magnetic resonance imaging
(MRI) scan. Which question would be most 35. Which nursing task would be most appropriate
appropriate for the nurse to ask prior to taking for the nurse to delegate to the unlicensed
the client to the diagnostic test? assistive personnel?
1. “Do you have trouble hearing?” 1. Teach Credé’s maneuver to the client needing
2. “Are you allergic to any type of dairy to void.
products?” 2. Administer the tube feeding to the client who
3. “Have you eaten anything in the last eight (8) is quadriplegic.
hours?” 3. Assist with bowel training by placing the
4. “Are you uncomfortable in closed spaces?” client on the bedside commode.
4. Observe the client demonstrating self-
31. The client with a C6 SCI is admitted to the
catheterization technique.
emergency department complaining of a severe
pounding headache and has a BP of 180/110.
Which intervention should the emergency
department nurse implement?
1. Keep the client flat in bed.
2. Dim the lights in the room.
3. Assess for bladder distention.
4. Administer a narcotic analgesic.

02_Colgrove_Ch02.indd 13 02/06/16 7:59 PM


14 M ed -S urg S uccess

36. The 34-year-old male client with an SCI is 3. Determine if the client is incontinent of urine
sharing with the nurse that he is worried about or stool.
finding employment after being discharged 4. Provide the client with privacy during the
from the rehabilitation unit. Which intervention seizure.
should the nurse implement?
41. The client who just had a three (3)-minute
1. Refer the client to the American Spinal Cord
seizure has no apparent injuries and is oriented
Injury Association (ASIA).
to name, place, and time but is very lethargic and
2. Refer the client to the state rehabilitation
just wants to sleep. Which intervention should
commission.
the nurse implement?
3. Ask the social worker (SW) about applying for
1. Perform a complete neurological assessment.
disability.
2. Awaken the client every 30 minutes.
4. Suggest that the client talk with his significant
3. Turn the client to the side and allow the client
other about this concern.
to sleep.
4. Interview the client to find out what caused
Seizures the seizure.
42. The unlicensed assistive personnel (UAP) is
37. The male client is sitting in the chair and attempting to put an oral airway in the mouth
his entire body is rigid with his arms and of a client having a tonic-clonic seizure. Which
legs contracting and relaxing. The client is action should the primary nurse take?
not aware of what is going on and is making 1. Help the UAP to insert the oral airway in the
guttural sounds. Which action should the nurse mouth.
implement first? 2. Tell the UAP to stop trying to insert anything
1. Push aside any furniture. in the mouth.
2. Place the client on his side. 3. Take no action because the UAP is handling
3. Assess the client’s vital signs. the situation.
4. Ease the client to the floor. 4. Notify the charge nurse of the situation
38. The occupational health nurse is concerned immediately.
about preventing occupation-related acquired 43. The client is prescribed phenytoin (Dilantin), an
seizures. Which intervention should the nurse anticonvulsant, for a seizure disorder.
implement? Which statement indicates the client
1. Ensure that helmets are worn in appropriate understands the discharge teaching concerning
areas. this medication?
2. Implement daily exercise programs for the 1. “I will brush my teeth after every meal.”
staff. 2. “I will check my Dilantin level daily.”
3. Provide healthy foods in the cafeteria. 3. “My urine will turn orange while on
4. Encourage employees to wear safety glasses. Dilantin.”
39. The client is scheduled for an electroenceph­ 4. “I won’t have any seizures while on this
alogram (EEG) to help diagnose a seizure medication.”
disorder. Which preprocedure teaching should 44. The client is admitted to the intensive care unit
the nurse implement? (ICU) experiencing status epilepticus. Which
1. Tell the client to take any routine antiseizure collaborative intervention should the nurse
medication prior to the EEG. anticipate?
2. Tell the client not to eat anything for eight (8) 1. Assess the client’s neurological status every
hours prior to the procedure. hour.
3. Instruct the client to stay awake for 24 hours 2. Monitor the client’s heart rhythm via
prior to the EEG. telemetry.
4. Explain to the client that there will be some 3. Administer an anticonvulsant medication by
discomfort during the procedure. intravenous push.
40. The nurse enters the room as the client is 4. Prepare to administer a glucocorticosteroid
beginning to have a tonic-clonic seizure. What orally.
action should the nurse implement first?
1. Note the first thing the client does in the
seizure.
2. Assess the size of the client’s pupils.

02_Colgrove_Ch02.indd 14 02/06/16 7:59 PM


C hapter 2   N eurological D isorders 15

45. The client has been newly diagnosed with 50. The client has been diagnosed with a brain
epilepsy. Which discharge instructions should be tumor. Which presenting signs and symptoms
taught to the client? Select all that apply. help to localize the tumor position?
1. Keep a record of seizure activity. 1. Widening pulse pressure and bounding pulse.
2. Take tub baths only; do not take showers. 2. Diplopia and decreased visual acuity.
3. Avoid over-the-counter medications. 3. Bradykinesia and scanning speech.
4. Have anticonvulsant medication serum levels 4. Hemiparesis and personality changes.
checked regularly.
51. The male client diagnosed with a brain tumor
5. Do not drive alone; have someone in the car.
is scheduled for a magnetic resonance imaging
46. Which statement by the female client indicates (MRI) scan in the morning. The client tells the
that the client understands factors that may nurse that he is scared. Which response by the
precipitate seizure activity? nurse indicates an appropriate therapeutic
1. “It is all right for me to drink coffee for response?
breakfast.” 1. “MRIs are loud but there will not be any
2. “My menstrual cycle will not affect my seizure invasive procedure done.”
disorder.” 2. “You’re scared. Tell me about what is scaring
3. “I am going to take a class in stress you.”
management.” 3. “This is the least thing to be scared about—
4. “I should wear dark glasses when I am out in there will be worse.”
the sun.” 4. “I can call the MRI tech to come and talk to
you about the scan.”
47. The nurse asks the male client with epilepsy if
he has auras with his seizures. The client says, 52. The client diagnosed with breast cancer has
“I don’t know what you mean. What are auras?” developed metastasis to the brain. Which
Which statement by the nurse would be the best prophylactic measure should the nurse
response? implement?
1. “Some people have a warning that the seizure 1. Institute aspiration precautions.
is about to start.” 2. Refer the client to Reach to Recovery.
2. “Auras occur when you are physically and 3. Initiate seizure precautions.
psychologically exhausted.” 4. Teach the client about mastectomy care.
3. “You’re concerned that you do not have auras
53. The significant other of a client diagnosed
before your seizures?”
with a brain tumor asks the nurse for help
4. “Auras usually cause you to be sleepy after you
identifying resources. Which would be the most
have a seizure.”
appropriate referral for the nurse to make?
48. The nurse educator is presenting an in-service 1. Social worker.
on seizures. Which disease process is the leading 2. Chaplain.
cause of seizures in the elderly? 3. Health-care provider.
1. Alzheimer’s disease. 4. Occupational therapist.
2. Parkinson’s disease (PD).
54. The nurse has written a care plan for a client
3. Cerebral Vascular Accident (CVA, stroke).
diagnosed with a brain tumor. Which is an
4. Brain atrophy due to aging.
important goal regarding self-care deficit?
1. The client will maintain body weight within
Brain Tumors two (2) pounds.
2. The client will execute an advance directive.
49. The client is being admitted to rule out a 3. The client will be able to perform three (3)
brain tumor. Which classic triad of symptoms ADLs with assistance.
supports a diagnosis of brain tumor? 4. The client will verbalize feeling of loss by the
1. Nervousness, metastasis to the lungs, and end of the shift.
seizures.
2. Headache, vomiting, and papilledema.
3. Hypotension, tachycardia, and tachypnea.
4. Abrupt loss of motor function, diarrhea, and
changes in taste.

02_Colgrove_Ch02.indd 15 02/06/16 7:59 PM


16 M ed -S urg S uccess

55. The client diagnosed with a brain tumor 3. Radiation therapy will have fewer side effects
was admitted to the intensive care unit with than chemotherapy.
decorticate posturing. Which indicates that the 4. Metastatic tumors become resistant to
client’s condition is becoming worse? chemotherapy and it becomes useless.
1. The client has purposeful movement with
60. The client is being discharged following a
painful stimuli.
transsphenoidal hypophysectomy. Which
2. The client has assumed adduction of the
discharge instructions should the nurse teach the
upper extremities.
client? Select all that apply.
3. The client is aimlessly thrashing in the bed.
1. Sleep with the head of the bed elevated.
4. The client has become flaccid and does not
2. Keep a humidifier in the room.
respond to stimuli.
3. Use caution when performing oral care.
56. The client is diagnosed with a pituitary 4. Stay on a full liquid diet until seen by
tumor and is scheduled for a transsphenoidal the HCP.
hypophysectomy. Which preoperative 5. Notify the HCP if developing a cold or fever.
instruction is important for the nurse to teach?
1. There will be a large turban dressing around
the skull after surgery. Meningitis
2. The client will not be able to eat for four (4)
or five (5) days postop. 61. The wife of the client diagnosed with septic
3. The client should not blow the nose for two meningitis asks the nurse, “I am so scared. What
(2) weeks after surgery. is meningitis?” Which statement would be the
4. The client will have to lie flat for 24 hours most appropriate response by the nurse?
following the surgery. 1. “There is bleeding into his brain causing
irritation of the meninges.”
57. The client has undergone a craniotomy 2. “A virus has infected the brain and meninges,
for a brain tumor. Which data indicate a causing inflammation.”
complication of this surgery? 3. “It is a bacterial infection of the tissues that
1. The client complains of a headache at “3” to cover the brain and spinal cord.”
“4” on a 1-to-10 scale. 4. “It is an inflammation of the brain
2. The client has an intake of 1,000 mL and an parenchyma caused by a mosquito bite.”
output of 3,500 mL.
3. The client complains of a raspy, sore throat. 62. The public health nurse is giving a lecture on
4. The client experiences dizziness when trying potential outbreaks of infectious meningitis.
to get up too quickly. Which population is most at risk for an
outbreak?
58. The client diagnosed with a brain tumor has a 1. Clients recently discharged from the hospital.
diminished gag response and weakness on the 2. Residents of a college dormitory.
left side of the body. Which intervention should 3. Individuals who visit a third world country.
the nurse implement? 4. Employees in a high-rise office building.
1. Make the client NPO until seen by the
health-care provider. 63. The nurse is assessing the client diagnosed
2. Position the client in low Fowler’s position for with bacterial meningitis. Which clinical
all meals. manifestations would support the diagnosis of
3. Place the client on a mechanically ground bacterial meningitis?
diet. 1. Positive Babinski’s sign and peripheral
4. Teach the client to direct food and fluid paresthesia.
toward the right side. 2. Negative Chvostek’s sign and facial tingling.
3. Positive Kernig’s sign and nuchal rigidity.
59. The client is diagnosed with a metastatic brain 4. Negative Trousseau’s sign and nystagmus.
tumor, and radiation therapy is scheduled.
The client asks the nurse, “Why not try 64. The nurse is assessing the client diagnosed with
chemotherapy first? It has helped my other meningococcal meningitis. Which assessment
tumors.” The nurse’s response is based on which data would warrant notifying the HCP?
scientific rationale? 1. Purpuric lesions on the face.
1. Chemotherapy is only used as a last resort in 2. Complaints of light hurting the eyes.
caring for clients with brain tumors. 3. Dull, aching, frontal headache.
2. The blood–brain barrier prevents medications 4. Not remembering the day of the week.
from reaching the brain.

02_Colgrove_Ch02.indd 16 02/06/16 7:59 PM


C hapter 2   N eurological D isorders 17

65. Which type of precautions should the nurse 70. Which statement best describes the scientific
implement for the client diagnosed with septic rationale for alternating a nonnarcotic
meningitis? antipyretic and a nonsteroidal anti-inflammatory
1. Standard Precautions. drug (NSAID) every two (2) hours to a female
2. Airborne Precautions. client diagnosed with bacterial meningitis?
3. Contact Precautions. 1. This regimen helps to decrease the purulent
4. Droplet Precautions. exudate surrounding the meninges.
2. These medications will decrease intracranial
66. The nurse is developing a plan of care for
pressure and brain metabolism.
a client diagnosed with aseptic meningitis
3. These medications will increase the client’s
secondary to a brain tumor. Which nursing
memory and orientation.
goal would be most appropriate for the client
4. This will help prevent a yeast infection
problem “altered cerebral tissue perfusion”?
secondary to antibiotic therapy.
1. The client will be able to complete activities
of daily living. 71. The client diagnosed with septic meningitis is
2. The client will be protected from injury if admitted to the medical floor at noon. Which
seizure activity occurs. health-care provider’s order would have the
3. The client will be afebrile for 48 hours prior highest priority?
to discharge. 1. Administer an intravenous antibiotic.
4. The client will have elastic tissue turgor with 2. Obtain the client’s lunch tray.
ready recoil. 3. Provide a quiet, calm, and dark room.
4. Weigh the client in hospital attire.
67. The nurse is preparing a client diagnosed with
rule-out meningitis for a lumbar puncture. 72. The 29-year-old client is admitted to the
Which interventions should the nurse medical floor diagnosed with meningitis. Which
implement? Select all that apply. assessment by the nurse has priority?
1. Obtain an informed consent from the client 1. Assess lung sounds.
or significant other. 2. Assess the six cardinal fields of gaze.
2. Have the client empty the bladder prior to the 3. Assess apical pulse.
procedure. 4. Assess level of consciousness.
3. Place the client in a side-lying position with
the back arched.
4. Instruct the client to breathe rapidly and Parkinson’s Disease
deeply during the procedure.
5. Explain to the client what to expect during 73. The client diagnosed with Parkinson’s disease
the procedure. (PD) is being admitted with a fever and patchy
infiltrates in the lung fields on the chest x-ray.
68. The nurse is caring for a client diagnosed with Which clinical manifestations of PD would
meningitis. Which collaborative intervention explain these assessment data?
should be included in the plan of care? 1. Masklike facies and shuffling gait.
1. Administer antibiotics. 2. Difficulty swallowing and immobility.
2. Obtain a sputum culture. 3. Pill rolling of fingers and flat affect.
3. Monitor the pulse oximeter. 4. Lack of arm swing and bradykinesia.
4. Assess intake and output.
74. The client diagnosed with PD is being
69. The client is diagnosed with meningococcal discharged on carbidopa/levodopa (Sinemet),
meningitis. Which preventive measure would the an antiparkinsonian drug. Which statement is
nurse expect the health-care provider to order the scientific rationale for combining these
for the significant others in the home? medications?
1. The Haemophilus influenzae vaccine. 1. There will be fewer side effects with this
2. Antimicrobial chemoprophylaxis. combination than with carbidopa alone.
3. A 10-day dose pack of corticosteroids. 2. Dopamine D requires the presence of both of
4. A gamma globulin injection. these medications to work.
3. Carbidopa makes more levodopa available to
the brain.
4. Carbidopa crosses the blood–brain barrier to
treat Parkinson’s disease.

02_Colgrove_Ch02.indd 17 02/06/16 7:59 PM


18 M ed -S urg S uccess

75. The nurse caring for a client diagnosed 79. The nurse researcher is working with clients
with Parkinson’s disease writes a problem diagnosed with Parkinson’s disease. Which is an
of “impaired nutrition.” Which nursing example of an experimental therapy?
intervention would be included in the plan of 1. Stereotactic pallidotomy/thalamotomy.
care? 2. Dopamine receptor agonist medication.
1. Consult the occupational therapist for 3. Physical therapy for muscle strengthening.
adaptive appliances for eating. 4. Fetal tissue transplantation.
2. Request a low-fat, low-sodium diet from the
80. The client diagnosed with Parkinson’s disease is
dietary department.
being discharged. Which statement made by the
3. Provide three (3) meals per day that include
significant other indicates an understanding of
nuts and whole-grain breads.
the discharge instructions?
4. Offer six (6) meals per day with a soft
1. “All of my spouse’s emotions will slow down
consistency.
now just like his body movements.”
76. The nurse and the unlicensed assistive personnel 2. “My spouse may experience hallucinations
(UAP) are caring for clients on a medical- until the medication starts working.”
surgical unit. Which task should not be assigned 3. “I will schedule appointments late in the
to the UAP? morning after his morning bath.”
1. Feed the 69-year-old client diagnosed with 4. “It is fine if we don’t follow a strict medication
Parkinson’s disease who is having difficulty schedule on weekends.”
swallowing.
81. The nurse is admitting a client with the
2. Turn and position the 89-year-old client
diagnosis of Parkinson’s disease. Which
diagnosed with a pressure ulcer secondary to
assessment data support this diagnosis?
Parkinson’s disease.
1. Crackles in the upper lung fields and jugular
3. Assist the 54-year-old client diagnosed with
vein distention.
Parkinson’s disease with toilet-training
2. Muscle weakness in the upper extremities and
activities.
ptosis.
4. Obtain vital signs on a 72-year-old client
3. Exaggerated arm swinging and scanning
diagnosed with pneumonia secondary to
speech.
Parkinson’s disease.
4. Masklike facies and a shuffling gait.
77. The charge nurse is making assignments. Which
82. Which is a common cognitive problem
client should be assigned to the new graduate
associated with Parkinson’s disease?
nurse?
1. Emotional lability.
1. The client diagnosed with aseptic meningitis
2. Depression.
who is complaining of a headache and the
3. Memory deficits.
light bothering his eyes.
4. Paranoia.
2. The client diagnosed with Parkinson’s disease
who fell during the night and is complaining 83. The nurse is conducting a support group for
of difficulty walking. clients diagnosed with Parkinson’s disease and
3. The client diagnosed with a cerebrovascular their significant others. Which information
accident whose vitals signs are P 60, R 14, and regarding psychosocial needs should be included
BP 198/68. in the discussion?
4. The client diagnosed with a brain tumor who 1. The client should discuss feelings about being
has a new complaint of seeing spots before the placed on a ventilator.
eyes. 2. The client may have rapid mood swings and
become easily upset.
78. The nurse is planning the care for a client
3. Pill-rolling tremors will become worse when
diagnosed with Parkinson’s disease. Which
the medication is wearing off.
would be a therapeutic goal of treatment for
4. The client may automatically start to repeat
the disease process?
what another person says.
1. The client will experience periods of akinesia
throughout the day.
2. The client will take the prescribed
medications correctly.
3. The client will be able to enjoy a family
outing with the spouse.
4. The client will be able to carry out activities
of daily living.

02_Colgrove_Ch02.indd 18 02/06/16 7:59 PM


C hapter 2   N eurological D isorders 19

84. The nurse is caring for clients on a medical- 88. A 20-year-old female client who tried lysergic
surgical floor. Which clients should be assessed acid diethylamide (LSD) as a teen tells the nurse
first? that she has bad dreams that make her want to
1. The 65-year-old client diagnosed with kill herself. Which is the explanation for this
seizures who is complaining of a headache occurrence?
that is a “2” on a 1-to-10 scale. 1. These occurrences are referred to as
2. The 24-year-old client diagnosed with a T10 “holdover reactions” to the drug.
spinal cord injury who cannot move his toes. 2. These are flashbacks to a time when the client
3. The 58-year-old client diagnosed with had a “bad trip.”
Parkinson’s disease who is crying and worried 3. The drug is still in the client’s body and
about her facial appearance. causing these reactions.
4. The 62-year-old client diagnosed with a 4. The client is suicidal and should be on ­
cerebrovascular accident who has a resolving one-to-one precautions.
left hemiparesis.
89. The nurse observes a coworker acting erratically.
The clients assigned to this coworker don’t
Substance Abuse seem to get relief when pain medications are
administered. Which action should the nurse
85. The friend of an 18-year-old male client implement?
brings the client to the emergency department 1. Try to help the coworker by confronting the
(ED). The client is unconscious and his coworker with the nurse’s suspicions.
breathing is slow and shallow. Which action 2. Tell the coworker that the nurse will give all
should the nurse implement first? narcotic medications from now on.
1. Ask the friend what drugs the client has been 3. Report the nurse’s suspicions to the nurse’s
taking. supervisor or the facility’s peer review.
2. Initiate an IV infusion at a keep-open rate. 4. Do nothing until the nurse can prove the
3. Call for a ventilator to be brought to the ED. coworker has been using drugs.
4. Apply oxygen at 100% via nasal cannula. 90. The client is diagnosed with Wernicke-
86. The chief executive officer (CEO) of a large Korsakoff syndrome as a result of chronic
manufacturing plant presents to the occupational alcoholism. For which symptoms would the
health clinic with chronic rhinitis and requesting nurse assess?
medication. On inspection, the nurse notices 1. Insomnia and anxiety.
holes in the septum that separates the nasal 2. Visual or auditory hallucinations.
passages. The nurse also notes dilated pupils 3. Extreme tremors and agitation.
and tachycardia. The facility has a “No Drug” 4. Ataxia and confabulation.
policy. Which intervention should the nurse 91. The client diagnosed with delirium tremens
implement? when trying to quit drinking cold turkey is
1. Prepare to complete a drug screen urine test. admitted to the medical unit. Which medications
2. Discuss the client’s use of illegal drugs. would the nurse anticipate administering?
3. Notify the client’s supervisor about the 1. Thiamine (vitamin B6) and librium, a
situation. benzodiazepine.
4. Give the client an antihistamine and say 2. Dilantin, an anticonvulsant, and Feosol, an
nothing. iron preparation.
87. The nurse is working with several clients in a 3. Methadone, a synthetic narcotic, and
substance abuse clinic. Client A tells the nurse Depakote, a mood stabilizer.
that another client, Client B, has “started 4. Mannitol, an osmotic diuretic, and Ritalin, a
using again.” Which action should the nurse stimulant.
implement? 92. The client is withdrawing from a heroin
1. Tell Client A the nurse cannot discuss addiction. Which interventions should the nurse
Client B with him. implement? Select all that apply.
2. Find out how Client A got this information. 1. Initiate seizure precautions.
3. Inform the HCP that Client B is using again. 2. Check vital signs every eight (8) hours.
4. Get in touch with Client B and have the client 3. Place the client in a quiet, calm atmosphere.
come to the clinic. 4. Have a consent form signed for HIV testing.
5. Provide the client with sterile needles.

02_Colgrove_Ch02.indd 19 02/06/16 7:59 PM


20 M ed -S urg S uccess

93. The wife of the client diagnosed with chronic Amyotrophic Lateral Sclerosis
alcoholism tells the nurse, “I have to call his (ALS or Lou Gehrig’s Disease)
work just about every Monday to let them
know he is ill or he will lose his job.” Which 97. Which diagnostic test is used to confirm the
would be the nurse’s best response? diagnosis of Amyotropic Lateral Sclerosis
1. “I am sure that this must be hard for you. (ALS)?
Tell me about your concerns.” 1. Electromyogram (EMG).
2. “You are afraid he will lose his source of 2. Muscle biopsy.
income.” 3. Serum creatine kinase (CK).
3. “Why would you call in for your husband? 4. Pulmonary function test.
Can’t he do this?”
4. “Are you aware that when you do this you 98. The client is diagnosed with ALS. Which client
are enabling him?” problem would be most appropriate for this
client?
94. The nurse caring for a client who has been 1. Disuse syndrome.
abusing amphetamines writes a problem of 2. Altered body image.
“cardiovascular compromise.” Which nursing 3. Fluid and electrolyte imbalance.
interventions should be implemented? 4. Alteration in pain.
1. Monitor the telemetry and vital signs every
four (4) hours. 99. The client is being evaluated to rule out ALS.
2. Encourage the client to verbalize the reason Which signs/symptoms would the nurse note
for using drugs. to confirm the diagnosis?
3. Provide a quiet, calm atmosphere for the 1. Muscle atrophy and flaccidity.
client to rest. 2. Fatigue and malnutrition.
4. Place the client on bedrest and a low- 3. Slurred speech and dysphagia.
sodium diet. 4. Weakness and paralysis.
95. The client diagnosed with substance abuse 100. The client diagnosed with ALS asks the
is being discharged from a drug and alcohol nurse, “I know this disease is going to kill
rehabilitation facility. Which information me. What will happen to me in the end?”
should the nurse teach the client? Which statement by the nurse would be most
1. “Do not go anyplace where you can be appropriate?
tempted to use again.” 1. “You are afraid of how you will die?”
2. “It is important that you attend a 12-step 2. “Most people with ALS die of respiratory
meeting regularly.” failure.”
3. “Now that you are clean, your family will be 3. “Don’t talk like that. You have to stay
willing to see you again.” positive.”
4. “You should explain to all your coworkers 4. “ALS is not a killer. You can live a long life.”
what has happened.” 101. The client with ALS is admitted to the medical
96. The nurse is working with clients and their unit with shortness of breath, dyspnea, and
families regarding substance abuse. Which respiratory complications. Which intervention
statement is the scientific rationale for teaching should the nurse implement first?
the children new coping mechanisms? 1. Elevate the head of the bed 30 degrees.
1. The child needs to realize that the parent 2. Administer oxygen via nasal cannula.
will be changing behaviors. 3. Assess the client’s lung sounds.
2. The child will need to point out to the 4. Obtain a pulse oximeter reading.
parent when the parent is not coping. 102. The client is to receive a 100-mL intravenous
3. Children tend to mimic behaviors of parents antibiotic over 30 minutes via an intravenous
when faced with similar situations. pump. At what rate should the nurse set the IV
4. Children need to feel like they are a part of pump? ___________
the parent’s recovery.

02_Colgrove_Ch02.indd 20 02/06/16 7:59 PM


C hapter 2   N eurological D isorders 21

103. The nurse is caring for several clients on a 108. The client diagnosed with ALS is prescribed
medical unit. Which client should the nurse an antiglutamate, riluzole (Rilutek). Which
assess first? instruction should the nurse discuss with the
1. The client with ALS who is refusing to turn client?
every two (2) hours. 1. Take the medication with food.
2. The client with abdominal pain who is 2. Do not eat green, leafy vegetables.
complaining of nausea. 3. Use SPF 30 when going out in the sun.
3. The client with pneumonia who has a pulse 4. Report any febrile illness.
oximeter reading of 90%.
4. The client who is complaining about not Encephalitis
receiving any pain medication.
104. The client is diagnosed with ALS. As the 109. The nurse is admitting the client for rule-
disease progresses, which intervention should out encephalitis. Which interventions should
the nurse implement? the nurse assess to support the diagnosis of
1. Discuss the need to be placed in a long-term encephalitis? Select all that apply.
care facility. 1. Determine if the client has recently received
2. Explain how to care for a sigmoid any immunizations.
colostomy. 2. Ask the client if he or she has had a cold in
3. Assist the client to prepare an advance the last week.
directive. 3. Check to see if the client has active herpes
4. Teach the client how to use a motorized simplex 1.
wheelchair. 4. Find out if the client has traveled to the
Great Lakes region.
105. The client is in the terminal stage of ALS. 5. Assess for exposure to soil with fungal spores.
Which intervention should the nurse
implement? 110. The nurse is assessing the client admitted with
1. Perform passive ROM every two (2) hours. encephalitis. Which data require immediate
2. Maintain a negative nitrogen balance. nursing intervention? The client has bilateral
3. Encourage a low-protein, soft-mechanical facial palsies.
diet. 1. The client has a recurrent temperature of
4. Turn the client and have him cough and 100.6°F.
deep breathe every shift. 2. The client has a decreased complaint of
headache.
106. The son of a client diagnosed with ALS asks 3. The client comments that the meal has no
the nurse, “Is there any chance that I could get taste.
this disease?” Which statement by the nurse
would be most appropriate? 111. The client admitted to the hospital to rule out
1. “It must be scary to think you might get this encephalitis is being prepared for a lumbar
disease.” puncture. Which instructions should the nurse
2. “No, this disease is not genetic or teach the client regarding care postprocedure?
contagious.” 1. Instruct that all invasive procedures require
3. “ALS does have a genetic factor and runs in a written permission.
families.” 2. Explain that this allows analysis of a sample
4. “If you are exposed to the same virus, you of the cerebrospinal fluid.
may get the disease.” 3. Tell the client to increase fluid intake to
300 mL for the next 48 hours.
107. The client with end-stage ALS requires a 4. Discuss that lying supine with the head flat
gastrostomy tube feeding. Which finding will prevent all hematomas.
would require the nurse to hold a bolus tube
feeding? 112. The nurse is caring for a client diagnosed
1. A residual of 125 mL. with encephalitis. Which is an expected
2. The abdomen is soft. outcome for the client?
3. Three episodes of diarrhea. 1. The client will regain as much neurological
4. The potassium level is 3.4 mEq/L. function as possible.
2. The client will have no short-term memory
loss.
3. The client will have improved renal function.
4. The client will apply hydrocortisone cream
daily.

02_Colgrove_Ch02.indd 21 02/06/16 7:59 PM


22 M ed -S urg S uccess

113. Which intervention should the nurse implement 118. Which collaborative intervention should the
when caring for the client diagnosed with nurse implement when caring for the client
encephalitis? Select all that apply. with West Nile virus?
1. Turn the client every two (2) hours. 1. Complete neurovascular examinations every
2. Encourage the client to increase fluids. eight (8) hours.
3. Keep the client in the supine position. 2. Maintain accurate intake and output at the
4. Assess for deep vein thrombosis (DVT). end of each shift.
5. Assess for any alterations in elimination. 3. Assess the client’s symptoms to determine if
there is improvement.
114. The nurse is caring for the client with
4. Administer intravenous fluids while
encephalitis. Which intervention should
assessing for overload.
the nurse implement first if the client is
experiencing a complication? 119. The nurse is caring for the client diagnosed
1. Examine pupil reactions to light. with West Nile virus. Which assessment data
2. Assess level of consciousness. would require immediate intervention from
3. Observe for seizure activity. the nurse?
4. Monitor vital signs every shift. 1. The vital signs are documented as T
100.2°F, P 80, R 18, and BP 136/78.
115. The public health department nurse is preparing
2. The client complains of generalized body
a lecture on prevention of West Nile virus.
aches and pains.
Which information should the nurse include?
3. Positive results are reported from the
1. Change water daily in pet dishes and
enzyme-linked immunosorbent assay
birdbaths.
(ELISA).
2. Wear thick, dark clothing when outside to
4. The client becomes lethargic and is difficult
avoid bites.
to arouse using verbal stimuli.
3. Apply insect repellent over face and arms
only. 120. Which rationale explains the transmission of
4. Explain that mosquitoes are more prevalent the West Nile virus?
in the morning. 1. Transmission occurs through exchange of
body fluids when sneezing and coughing.
116. Which problem is the highest priority for the
2. Transmission occurs only through mosquito
client diagnosed with West Nile virus?
bites and not between humans.
1. Alteration in body temperature.
3. Transmission can occur from human to
2. Altered tissue perfusion.
human in blood products and breast milk.
3. Fluid volume excess.
4. Transmission occurs with direct contact
4. Altered skin integrity.
from the maculopapular rash drainage.
117. The nurse is developing a plan of care for a
client diagnosed with West Nile virus. Which
intervention should the nurse include in this plan?
1. Monitor the client’s respirations frequently.
2. Refer to a dermatologist for treatment of
maculopapular rash.
3. Treat hypothermia by using ice packs under
the client’s arms.
4. Teach the client to report any swollen lymph
glands.

CONCEPTS
In this section of the practice questions, the focus is on a particular concept. The concepts of intracranial
regulation, functional ability, and cognition along with the exemplars (example disease processes) are directed
toward the test taker recognizing the commonality that exists between one exemplar and another under the
concept. There are also interrelated concepts that may be presented in a question to help the test taker under-
stand how the concepts intertwine. The test taker must also recognize that in order to perform individualized
client care, all aspects of the client’s beliefs, developmental stage, and culture as well as the disease process must
be considered.

02_Colgrove_Ch02.indd 22 02/06/16 7:59 PM


C hapter 2   N eurological D isorders 23

121. The male client is admitted to the emergency 125. The concept of intracranial regulation is
department following a motorcycle accident. identified for a client diagnosed with a brain
The client was not wearing a helmet and struck tumor. Which intervention should the nurse
his head on the pavement. The nurse identifies include in the client’s plan of care?
the concept as impaired intracranial regulation. 1. Tell the client to remain on bedrest.
Which interventions should the emergency 2. Maintain the intravenous rate at 150 mL/hour.
department nurse implement in the first 3. Provide a soft, bland diet with three (3)
five (5) minutes? Select all that apply. snacks per day.
1. Stabilize the client’s neck and spine. 4. Place the client on seizure precautions.
2. Contact the organ procurement
126. The 80-year-old male client on an Alzheimer’s
organization to speak with the family.
unit is agitated and asking the nurse to get
3. Elevate the head of the bed to 70 degrees.
his father to come and see him. Which is the
4. Perform a Glasgow Coma Scale assessment.
nurse’s best response?
5. Ensure the client has a patent peripheral
1. Tell the client his father is dead and cannot
venous catheter in place.
come to see him.
6. Check the client’s driver’s license to see if he
2. Give the client the phone and have him
will accept blood.
attempt to call his father.
122. The client diagnosed with atrial fibrillation 3. Ask the client to talk about his father with
complains of numbness and tingling of her left the nurse.
arm and leg. The nurse assesses facial drooping 4. Call the family so they can tell the client
on the left side and slight slurring of speech. why his father cannot come to see him.
Which nursing interventions should the nurse
127. The 28-year-old client is on the rehabilitation
implement first?
unit post spinal cord injury at level T10. Which
1. Schedule a STAT Magnetic Resonance
collaborative team members should participate
Imaging of the brain.
with the nurse at the case conference? Select
2. Call a Code STROKE.
all that apply.
3. Notify the health-care provider (HCP).
1. Occupational Therapist (OT).
4. Have the client swallow a glass of water.
2. Physical therapist (PT).
123. The nurse identifies the concept of intracranial 3. Registered dietitian (RD).
regulation disturbance in a client diagnosed 4. Rehabilitation physician.
with Parkinson’s Disease. Which priority 5. Social Worker (SW).
intervention should the nurse implement? 6. Patient care tech (PCT).
1. Keep the bed low and call light in reach.
128. Which diagnostic evaluation tool would
2. Provide a regular diet of three (3) meals
the nurse use to assess the client’s cognitive
per day.
functioning? Select all that apply.
3. Obtain an order for home health to see the
1. The Geriatric Depression Scale (GDS).
client.
2. The St. Louis University Mental Status
4. Perform the Braden scale skin assessment.
(SLUMS) scale.
124. The client newly diagnosed with Parkinson’s 3. The Mini-Mental Status Examination
Disease (PD) asks the nurse, “Why can’t I (MMSE) scale.
control these tremors?” Which is the nurse’s 4. The Manic Depression vs Elderly
best response? Depression (MDED) scale.
1. “You can control the tremors when you 5. The Functional Independence Measurement
learn to concentrate and focus on the cause.” Scale (FIMS).
2. “The tremors are caused by a lack of the
129. Which priority goal would the nurse identify
chemical dopamine in the brain; medication
for a client diagnosed with Parkinson’s
may help.”
Disease (PD)?
3. “You have too much acetylcholine in your
1. The client will be able to maintain mobility
brain causing the tremors but they will get
and swallow without aspiration.
better with time.”
2. The client will verbalize feelings about the
4. “You are concerned about the tremors? If
diagnosis of Parkinson’s Disease.
you want to talk I would like to hear how
3. The client will understand the purpose of
you feel.”
medications administered for PD.
4. The client will have a home health agency
for monitoring at home.

02_Colgrove_Ch02.indd 23 02/06/16 7:59 PM


24 M ed -S urg S uccess

130. The intensive care unit nurse is admitting a client with a traumatic brain injury. Which health-care
provider medication order would the nurse question?

Client Number Allergies Diagnosis


Client Name: ABCD 1234567 NKA Angina

Date Medication 0701–1500 1501–2300 2301–0700


Today Dexamethasone 4 mg IVP 2400
every 6 hours 0600
1200
1800
Today 0.9% saline continuous 1600
infusion at 25 mL/hour
Today Nicotine patch 15 mg per day 0900
Apply at 0800; discontinue at 2100 2100
Today Morphine sulfate 4 mg IVP
­every 3 hours prn
Signature of Nurse: Day Nurse RN/DN

1. Dexamethasone.
2. 0.9% NS.
3. Nicotine patch.
4. Morphine sulfate.
131. The charge nurse on a medical-surgical unit is reviewing client diagnostic reports. Which report
warrants immediate intervention?
1. Client A—Male 64.

Complete Blood
Count Client Value Normal Values
RBC (106)    5 Male: 4.7–6.1 (106)
Female: 4.2–5.4 (106)
Hemoglobin   13.4 Male: 13.5–17.5 g/dL
Female: 11.5–15.5 g/dL
Hematocrit   39.5 Male: 40%­–52%
Female: 36%–48%
Platelet (103) 110 150–400 (103)/mm3
WBC (103)     9.2 4.5–11 (103)/mm3

2. Client B—Female 34.

Chemistry Client Value Normal Values


Sodium 142 135–145 mEq/L
Potassium    3.5 3.5–5.5 mEq/L
Chloride   98 97–107 mEq/L
CO2   26 22–30 mEq/L

3. Client C—Male 45.

Laboratory Test Client Value Normal Values


Glucose 94 70–110 mg/dL
Hemoglobin A1C 13.2 4%–6%

4. Client D—Female 56.

Patient MRI Findings


MRI of brain with MRI brain—Finding of 5 3 4 cm mass located in the L frontal lobe;
and without a shift in the brain tissue ­exerting pressure on the cerebellum and
contrast occluding the a ­ nterior portion of the L lateral ventricle.

02_Colgrove_Ch02.indd 24 02/06/16 7:59 PM


C hapter 2   N eurological D isorders 25

132. The nurse is caring for a client with increased 134. The client diagnosed with a brain abscess is
intracranial pressure (ICP) who has secretions experiencing a tonic-clonic seizure. Which
pooled in the throat. Which intervention interventions should the nurse implement?
should the nurse implement first? Rank in order of performance.
1. Set the ventilator to hyperventilate the 1. Assess the client’s mouth.
client in preparation for suctioning. 2. Loosen restrictive clothing.
2. Assess the client’s lung sounds and check for 3. Administer phenytoin IVP.
peripheral cyanosis. 4. Turn the client to the side.
3. Turn the client to the side to allow the 5. Protect the client’s head from injury.
secretions to drain from the mouth.
135. Which intervention should the nurse
4. Suction the client using the in-line suction,
implement to decrease increased intracranial
wait 30 seconds, and repeat.
pressure (ICP) for a client on a ventilator?
133. The nurse is performing a Glascow Coma Select all that apply.
Scale (GCS) assessment on a client with a 1. Position the client with the head of the bed
problem with intracranial regulation. The up 30 degrees.
client’s GCS one (1) hour ago was scored 2. Cluster activities of care.
at 10. Which datum indicates the client is 3. Suction the client every three (3) hours.
improving? 4. Administer soapsuds enemas until clear.
1. The current GSC rating is 3. 5. Place the client in Trendelenburg position.
2. The current GSC rating is 9.
3. The current GSC rating is 10.
4. The current GSC rating is 12.

02_Colgrove_Ch02.indd 25 02/06/16 7:59 PM


PRACTICE QUESTIONS ANSWERS
AND RATIONALES

This section provides the answers for the questions side. Ataxia is an impaired ability to coordi-
given in the previous section. The correct answers and nate movement.
why they are correct are provided in blue boldface. 3. Homonymous hemianopsia (loss of half of the
The reason why each of the other answer options is visual field of each eye) and diplopia (double
not the correct or the best answer is also given. vision) are visual field deficits that a client with
a CVA may experience, but they are not motor
losses.
Cerebrovascular Accident (Stroke) 4. Personality disorders occur in clients with a
right-sided CVA and are cognitive deficits;
1. 1. The drug rt-PA may be administered, but a hostility is an emotional deficit.
cerebrovascular accident (CVA) must be veri-
TEST TAKING HINT: Be sure to always notice
fied by diagnostic tests prior to ­administering
adjectives describing something. In this case,
it. The drug rt-PA helps dissolve a blood clot,
“left-sided” describes the type of CVA. Also
and it may be administered if an ischemic CVA
be sure to identify exactly what the question
is verified; rt-PA is not given if the client is ex-
is asking—in this case, about “motor loss,”
periencing a hemorrhagic stroke.
which will help rule out many of the possible
2. Teaching is important to help prevent another
answer options.
CVA, but it is not the priority intervention
on admission to the emergency department. Content – Medical: Integrated Nursing Process –
­Implementation: Client Needs – Safe Effective Care
Slurred speech indicates problems that may
Environment, Management of Care: Cognitive Level –
interfere with teaching. Application: Concept – Neurologic Regulation.
3. A CT scan will determine if the client is
having a stroke or has a brain tumor or 3. 1. African Americans have twice the rate of
another neurological disorder. If a CVA is CVAs as Caucasians and men have a higher
diagnosed, the CT scan can determine if it incidence than women; African Americans
is a hemorrhagic or an ischemic also suffer more extensive damage from
accident and guide treatment. a CVA than do people of other cultural
4. The client may be referred for speech deficits groups.
and/or swallowing difficulty, but referrals are 2. Females are less likely to have a CVA than
not the priority in the emergency department. males, but advanced age does increase the risk
TEST TAKING HINT: When “priority” is used for CVA. The Asian population has a lower
in the stem, all answer options may be ap- risk, possibly as a result of their relatively high
propriate for the client situation, but only one intake of omega-3 fatty acids, antioxidants
option is the priority. The client must have found in fish.
a documented d ­ iagnosis before treatment is 3. Caucasians have a lower risk of CVA than do
started. African Americans, Hispanics, and Native Pa-
cific Islanders.
Content – Medical: Integrated Nursing Process –
Implementation: Client Needs – Safe Effective Care Envi-
4. Pregnancy is a minimal risk for having a CVA.
ronment, Management of Care: Cognitive Level – Synthesis: TEST TAKING HINT: Note the age of the cli-
Concept – Neurologic Regulation. ent if this information is given, but take this
information in context with the additional in-
2. 1. A left-sided cerebrovascular accident (CVA)
formation provided in the answer options. The
will result in right-sided motor deficits; hemi-
84-year-old may appear to be the best answer
paresis is weakness of one-half of the body, not
but not if the client is a female and Asian,
just the upper extremity. Apraxia, the inability
which rules out this option for the client most
to perform a previously learned task, is a com-
at risk.
munication loss, not a motor loss.
2. The most common motor dysfunction of Content – Medical: Integrated Nursing Process –
Assessment: Client Needs – Health Promotion and Mainte-
a CVA is paralysis of one side of the body,
nance: Cognitive Level – Knowledge: Concept – Neurologic
hemiplegia; in this case with a left-sided Regulation.
CVA, the paralysis would affect the right

26

02_Colgrove_Ch02.indd 26 02/06/16 7:59 PM


C hapter 2   N eurological D isorders 27

4. 1. Placing a small pillow under the shoulder 6. 1. Placing a gait belt prior to ambulating is an
will prevent the shoulder from adduct- appropriate action for safety and would not
ing toward the chest and developing a require the nurse to intervene.
contracture. 2. Placing the client in a supine position with the
2. The client should be repositioned at least head turned to the side is not a problem posi-
every two (2) hours to prevent contractures, tion, so the nurse does not need to intervene.
pneumonia, skin breakdown, and other com- 3. This action is inappropriate and would
plications of immobility. require intervention by the nurse because
3. The client should not ignore the para- pulling on a flaccid shoulder joint could
lyzed side, and the nurse must encourage cause shoulder dislocation; the client
the client to move it as much as possible; should be pulled up by placing the arm
a written schedule may assist the client in underneath the back or using a lift sheet.
exercising. 4. The client should be encouraged and praised
4. These exercises are recommended, but they for attempting to perform any activities inde-
must be done at least five (5) times a day for pendently, such as combing hair or brushing
10 minutes to help strengthen the muscles for teeth.
walking. TEST TAKING HINT: This type of question has
5. The fingers are positioned so that they are barely three answer options that do not require a
flexed to help prevent contracture of the hand. nurse to intervene to correct a subordinate.
TEST TAKING HINT: Be sure to look at the in- Remember to read every possible answer
tervals of time for any intervention; note that option before deciding on a correct one.
“every shift” and “three (3) times a day” are Content – Medical: Integrated Nursing Process –
not appropriate time intervals for this client. Implementation: Client Needs – Safe Effective Care Envi-
Because this is a “select all that apply” ques- ronment, Management of Care: Cognitive Level – Synthesis:
tion, the test taker must read each answer Concept – Neurologic Regulation.
option and decide if it is correct; one will not
7. 1. The nurse would anticipate an oral antico-
eliminate another.
agulant, warfarin (Coumadin), to be pre-
Content – Medical: Integrated Nursing Process – scribed to help prevent thrombi formation
Planning: Client Needs – Safe Effective Care Environment, in the atria secondary to atrial fibrillation.
Management of Care: Cognitive Level – Synthesis: Concept –
The thrombi can become embolic and may
Neurologic Regulation.
cause a TIA or CVA (stroke).
5. 1. Agnosia is the failure to recognize familiar 2. Beta blockers slow the heart rate and decrease
objects; therefore, observing the client for pos- blood pressure but would not be an anticipated
sible aspiration is not appropriate. medication to help prevent a TIA secondary to
2. A semi-Fowler’s position is appropriate for atrial fibrillation.
sleeping, but agnosia is the failure to recognize 3. An anti-hyperuricemic medication is admin-
familiar objects; therefore, this intervention is istered for a client experiencing gout and de-
inappropriate. creases the formation of tophi.
3. Placing suction at the bedside will help if the 4. A thrombolytic medication is administered to
client has dysphagia (difficulty swallowing), not dissolve a clot, and it may be ordered during
agnosia, which is failure to recognize familiar the initial presentation for a client with a CVA
objects. but not on discharge.
4. A collaborative intervention is an inter­ TEST TAKING HINT: In the stem of this
vention in which another health-care ­question, there are two disease processes
discipline—in this case, occupational mentioned—atrial fibrillation and TIA. The
therapy—is used in the care of the client. reader must determine how one process
TEST TAKING HINT: Be sure to look at what ­affects the other before answering the ques-
the question is asking and see if the answer tion. In this question, the test taker must
can be determined even if some terms are not know atrial fibrillation predisposes the c­ lient
understood. In this case, note that the ques- to the formation of thrombi, and, therefore,
tion refers to “collaborative intervention.” the nurse should anticipate the health-care
Only option “4” refers to collaboration with provider ordering a medication to prevent clot
another discipline. formation, an anticoagulant.
Content – Medical: Integrated Nursing Process –
Planning: Client Needs – Safe Effective Care Environment,
Management of Care: Cognitive Level – Synthesis:
Concept – Neurologic Regulation.

02_Colgrove_Ch02.indd 27 02/06/16 7:59 PM


28 M ed -S urg S uccess

Content – Medical: Integrated Nursing Process – 10. 1. This glucose level is elevated and could pre-
Planning: Client Needs – Physiological Integrity, Pharma- dispose the client to ischemic neurological
cological and Parenteral Therapies: Cognitive Level – changes due to blood viscosity, but it is not
Synthesis: Concept – Neurologic Regulation. a risk factor for a hemorrhagic stroke.
8. 1. The rubber mat will stabilize the plate and 2. A carotid bruit predisposes the client to an
prevent it from slipping away from the client embolic or ischemic stroke but not to a hem-
learning to feed himself, but this does not ad- orrhagic stroke.
dress generalized weakness. 3. Uncontrolled hypertension is a risk ­factor
2. A long-handled bath sponge will assist the for hemorrhagic stroke, which
client when showering hard-to-reach areas, is a ruptured blood vessel inside the
but it is not a home modification nor will it cranium.
help with generalized weakness. 4. Cancer is not a precursor to developing a
3. Clothes with Velcro closures will make dress- hemorrhagic stroke.
ing easier, but they do not constitute TEST TAKING HINT: Both options “1” and “2”
a home modification and do not address gen- are risk factors for an ischemic or embolic
eralized weakness. type of stroke. Knowing this, the test taker
4. Raising the toilet seat is modifying the can rule out these options as incorrect.
home and addresses the client’s weakness Content – Medical: Integrated Nursing Process –
in being able to sit down and get up with- Assessment: Client Needs – Physiological Integrity,
out straining muscles or requiring lifting Reduction of Risk Potential: Cognitive Level – Analysis:
assistance from the wife. Concept – Perfusion.
TEST TAKING HINT: The test taker must read 11. 1. The nurse should not administer any medi-
the stem of the question carefully and note cation to a client without first assessing the
that the intervention must be one in which cause of the client’s complaint or problem.
the home is modified in some way. This 2. An MRI scan may be needed, but the nurse
would eliminate three of the options, leaving must determine the client’s neurological
the correct answer. ­status prior to diagnostic tests.
Content – Medical: Integrated Nursing Process – 3. Starting an IV infusion is appropriate, but it
Planning: Client Needs – Physiological Integrity, is not the action the nurse should implement
Physiological Adaptation: Cognitive Level – Synthesis: when assessing pain, and 100 mL/hr might be
Concept – Neurologic Regulation. too high a rate for an 85-year-old client.
9. 1. Potential for injury is a physiological, not a 4. The nurse must complete a neurological
psychosocial, problem. assessment to help determine the cause
2. Expressive aphasia means that the client of the headache before taking any further
cannot communicate thoughts but under- action.
stands what is being communicated; this TEST TAKING HINT: The test taker should al-
leads to frustration, anger, depression, and ways apply the nursing process when answer-
the inability to verbalize needs, which, in ing questions. If the test taker narrows down
turn, causes the client to have a lack of the choices to two possible answer options,
control and feel powerless. always select the assessment option as the
3. A disturbance in thought processes is a cogni- first intervention.
tive problem; with expressive aphasia the cli- Content – Medical: Integrated Nursing Process –
ent’s thought processes are intact. Implementation: Client Needs – Safe Effective Care
4. Sexual dysfunction can have a psychosocial or Environment, Management of Care: Cognitive Level –
physical component, but it is not related to Synthesis: Concept – Neurologic Regulation.
expressive aphasia.
12. 1. The client is at risk for increased intra-
TEST TAKING HINT: The test taker should cranial pressure whenever performing the
always make sure that the choice selected as Valsalva maneuver, which will occur when
the c­ orrect answer matches what the ques- straining during defecation. Therefore,
tion is asking. The stem has the adjective stool softeners would be appropriate.
“psychosocial,” so the ­correct answer must 2. Coughing increases intracranial pressure and
address psychosocial needs. is discouraged for any client who has had a
Content – Medical: Integrated Nursing Process – craniotomy. The client is encouraged to turn
Diagnosis: Client Needs – Psychosocial Integrity: Cognitive and breathe deeply but not to cough.
Level – Analysis: Concept – Neurologic Regulation.

02_Colgrove_Ch02.indd 28 02/06/16 7:59 PM


C hapter 2   N eurological D isorders 29

3. Monitoring the neurological status is appro- 14. 1. The scalp is a very vascular area and a mod-
priate for this client, but it should be done erate amount of bleeding would be expected.
much more frequently than every shift. 2. These signs/symptoms—weak pulse, shal-
4. Dopamine is used to increase blood pressure low respirations, cool pale skin—indicate
or to maintain renal perfusion, and a BP of increased intracranial pressure from cere-
160/90 is too high for this client. bral edema secondary to the fall, and they
TEST TAKING HINT: The test taker should require immediate attention.
always notice if an answer option has a time 3. This is a normal pupillary response and
frame—every shift, every four (4) hours, or would not warrant intervention.
daily. Whether or not the time frame is cor- 4. A headache that resolves with medication is
rect may lead the test taker to the correct not an emergency situation, and the nurse
answer. would expect the client to have a headache
after the fall; a headache not relieved with
Content – Medical: Integrated Nursing Process –
Implementation: Client Needs – Safe Effective Care Tylenol would warrant further investigation.
Environment, Management of Care: Cognitive Level – TEST TAKING HINT: The test taker is looking
Implementation: Concept – Neurologic Regulation. for an answer option that is not normal for
the client’s situation. Of the options listed,
three would be expected and would not war-
Head Injury rant a trip to the emergency department.
Content – Medical: Integrated Nursing Process –
13. 1. Awakening the client every two (2) Assessment: Client Needs – Safe Effective Care
hours allows the identification of head- Environment, Management of Care: Cognitive Level –
ache, dizziness, lethargy, irritability, and Analysis: Concept – Neurologic Regulation.
anxiety—all signs of post-concussion
syndrome—that would warrant the signifi- 15. 1. A client with a head injury must be awakened
cant other’s taking the client back to the every two (2) hours to determine alertness;
emergency department. decreasing level of consciousness is the first
2. The nurse should monitor for signs of in- indicator of increased intracranial pressure.
creased intracranial pressure (ICP), but a lay- 2. A diagnostic test, MRI, would be an expected
man, the significant other, would not know test for a client with left-sided weakness and
what these signs and medical terms mean. would not require immediate attention.
3. Hypervigilance, increased alertness and 3. The Glasgow Coma Scale is used to de-
super-awareness of the surroundings, is a termine a client’s response to stimuli (eye-
sign of amphetamine or cocaine abuse, but it opening response, best verbal response,
would not be expected in a client with a head and best motor response) secondary to a
injury. neurological problem; scores range from
4. The client can eat food as tolerated, but 3 (deep coma) to 15 (intact neurological
feeding the client every three (3) to four (4) function). A client with a score of 6 should
hours does not affect the development of be assessed first by the nurse.
postconcussion syndrome, the signs of which 4. The nurse would expect a client diagnosed
are what should be taught to the significant with a CVA (stroke) to have some sequelae of
other. the problem, including the inability to speak.
TEST TAKING HINT: Remember to pay close TEST TAKING HINT: This is a prioritiz-
attention to answer options that have times ing question that asks the test taker to
(e.g., “every two [2] hours,” “every three [3] ­determine which client has priority when
to four [4] hours”). Also consider the likeli- assessing all four clients. The nurse should
hood of the options listed. Would a nurse assess the client who has abnormal data for
teach the significant other terms such as the disease process.
increased intracranial pressure or hypervigi- Content – Medical: Integrated Nursing Process –
lance? Probably not, so options “2” and “3” Assessment: Client Needs – Safe Effective Care Environ-
should be eliminated. ment, Management of Care: Cognitive Level – Synthesis:
Concept – Neurologic Regulation.
Content – Medical: Integrated Nursing Process –
Planning: Client Needs – Physiological Integrity, Physiologi-
cal Adaptation: Cognitive Level – Synthesis:
Concept – Neurologic Regulation.

02_Colgrove_Ch02.indd 29 02/06/16 7:59 PM


30 M ed -S urg S uccess

16. 1. This is an oculocephalic test (doll’s eye move- 3. Clients with head injuries are at risk for
ment) that determines brain activity. If the post-traumatic seizures; thus an oral anticon-
eyes move with the head, it means the brain- vulsant would be administered for seizure
stem is intact and there is no brain death. prophylaxis.
2. Waveforms on the EEG indicate that there is 4. The client is at risk for a stress ulcer; there-
brain activity. fore, an oral proton pump inhibitor would be
3. The cold caloric test, also called the ocu- an appropriate medication.
lovestibular test, is a test used to deter- TEST TAKING HINT: The client is in the reha-
mine if the brain is intact or dead. No eye bilitation unit and therefore must be stable.
activity indicates brain death. If the cli- The use of any intravenous medication
ent’s eyes moved, that would indicate that should be questioned under those circum-
the brainstem is intact. stances, even if the test taker is not sure why
4. Decorticate posturing after painful stimuli are the medication may be considered.
applied indicates that the brainstem is intact; Content – Medical: Integrated Nursing Process –
flaccid paralysis is the worse neurological Implementation: Client Needs – Safe Effective Care
response when assessing a client with a head Environment, Management of Care: Cognitive Level –
injury. Application: Concept – Neurologic Regulation.
TEST TAKING HINT: The test taker needs to
19. 1. Purposeless movement indicates that the cli-
know what the results of the cold caloric test ent’s cerebral edema is decreasing. The best
signify—in this case, no eye activity indicates motor response is purposeful movement, but
brain death. purposeless movement indicates an improve-
Content – Medical: Integrated Nursing Process – ment over decorticate movement, which, in
Assessment: Client Needs – Safe Effective Care Environ- turn, is an improvement over decerebrate
ment, Management of Care: Cognitive Level – Analysis:
movement or flaccidity.
Concept – Neurologic Regulation.
2. Flaccidity would indicate a worsening of the
17. 1. Assessing the neurological status is impor- client’s condition.
tant, but ensuring an airway is a priority over 3. Decerebrate posturing would indicate a wors-
assessment. ening of the client’s condition.
2. Monitoring vital signs is important, but main- 4. The eyes respond to light, not painful stimuli,
taining an adequate airway is higher priority. but a 6-mm nonreactive pupil indicates severe
3. Initiating an IV access is an intervention the neurological deficit.
nurse can implement, but it is not the priority TEST TAKING HINT: The test taker must have
intervention. strong assessment skills and know what spe-
4. The most important nursing goal in cific signs/symptoms signify for each of the
the management of a client with a head body systems—in this case, the significance
injury is to establish and maintain an of different stages of p
­ osturing/
adequate airway. movement in assessing n ­ eurological status.
TEST TAKING HINT: If the question asks for a Content – Medical: Integrated Nursing Process –
priority intervention, it means that all of the Assessment: Client Needs – Safe Effective Care Environ-
options would be appropriate for the client ment, Management of Care: Cognitive Level – Analysis:
but only one intervention is priority. Always Concept – Trauma.
apply Maslow’s hierarchy of needs—an ad- 20. 1. The head of the bed should be elevated
equate airway is first. no more than 30 degrees to help decrease ce-
Content – Medical: Integrated Nursing Process – rebral edema by gravity.
Implementation: Client Needs – Safe Effective Care 2. Stool softeners are initiated to prevent the
Environment, Management of Care: Cognitive Level –
Valsalva maneuver, which increases intra-
Synthesis: Concept – Neurologic Regulation.
cranial pressure.
18. 1. The client in rehabilitation is at risk for the 3. Oxygen saturation higher than 93%
development of deep vein thrombosis; there- ensures oxygenation of the brain tissues;
fore, this is an appropriate medication. decreasing oxygen levels increase cerebral
2. An osmotic diuretic would be ordered in edema.
the acute phase to help decrease cerebral 4. Noxious stimuli, such as suctioning, increase
edema, but this medication would not intracranial pressure and should be avoided.
be expected to be ordered in a rehabilita-
tion unit.

02_Colgrove_Ch02.indd 30 02/06/16 7:59 PM


C hapter 2   N eurological D isorders 31

5. Mild sedatives will reduce the client’s 3. Assessing the client for further injury is ap-
agitation; strong narcotics would not be propriate, but the first intervention is to sta-
administered because they decrease the bilize the spine because the impact was strong
client’s level of consciousness. enough to render the client unconsciousness.
TEST TAKING HINT: In “select all that apply” 4. The nurse should always assume that a
questions, the test taker should look at each client with traumatic head injury may have
answer option as a separate entity. In option sustained spinal cord injury. Moving the
“1” the test taker should attempt to get a client could further injure the spinal cord
mental picture of the client’s position in the and cause paralysis; therefore, the nurse
bed. A 60-degree angle is almost upright in should stabilize the cervical spinal cord
the bed. Would any client diagnosed with a as best as possible prior to removing the
head injury be placed this high? The client ­client from the water.
would be at risk for slumping over because TEST TAKING HINT: When two possible an-
of the inability to control the body position. swer options contain the same directive
Nasal suctioning, option “4,” which ­increases word—in this case, “assess”—the test taker
intracranial pressure, should also be avoided. can either rule out these two as incorrect
Content – Medical: Integrated Nursing Process – or prioritize between the two assessment
Implementation: Client Needs – Safe Effective Care responses.
Environment, Management of Care: Cognitive Level – Content – Medical: Integrated Nursing Process –
Application: Concept – Neurologic Regulation. Implementation: Client Needs – Safe Effective Care
Environment, Management of Care: Cognitive Level –
21. 1. Prior to notifying the HCP, the nurse should
Synthesis: Concept – Neurologic Regulation.
always make sure that all the needed assess-
ment information is available to discuss with 23. 1. The head of the client’s bed should be el-
the HCP. evated to help the lungs expand and prevent
2. With head injuries, any clear drainage may stasis of secretions that could lead to pneu-
indicate a cerebrospinal fluid leak; the nurse monia, a complication of immobility.
should not assume the drainage is secondary 2. Active range-of-motion exercises require that
to allergies and administer an antihistamine. the client participate in the activity. This is
3. The presence of glucose in drainage from not possible because the client is in a coma.
the nose or ears indicates cerebrospinal 3. The client is at risk for pressure ulcers and
fluid, and the HCP should be notified im- should be turned more frequently than every
mediately once this is determined. shift, and research now shows that massaging
4. This would be appropriate, but it is not the bony prominences can increase the risk for
first intervention. The nurse must determine tissue breakdown.
where the fluid is coming from. 4. The nurse should always talk to the client,
TEST TAKING HINT: The question is asking even if he or she is in a coma, but this will not
which intervention should be implemented address the problem of immobility.
first, and the nurse should always ­assess the TEST TAKING HINT: Whenever a client prob-
situation before calling the HCP or taking an lem is written, interventions must address
action. the specific problem, not the disease. Posi-
Content – Medical: Integrated Nursing Process – tioning the client addresses the possibility of
Implementation: Client Needs – Safe Effective Care immobility complications, whereas talking to
Environment, Management of Care: Cognitive Level – a comatose client ­addresses communication
Synthesis: Concept – Neurologic Regulation. deficit and psychosocial needs, not immobil-
ity ­issues.
22. 1. Assessment is important, but with clients with
head injury the nurse must assume Content – Medical: Integrated Nursing Process –
spinal cord injury until it is ruled out with Diagnosis: Client Needs – Safe Effective Care Environ-
ment, Management of Care: ­Cognitive Level – Analysis:
x-ray; therefore, stabilizing the spinal cord
Concept – Neurologic Regulation.
is the priority.
2. Removing the client from the water is an ap- 24. 1. The client is at risk for seizures and does not
propriate intervention, but the nurse must process information appropriately. Allowing
assume spinal cord injury until it is ruled out him to return to his occupation as a forklift
with x-ray; therefore, stabilizing the spinal operator is a safety risk for him and other em-
cord is the priority. ployees. Vocational training may be required.

02_Colgrove_Ch02.indd 31 02/06/16 7:59 PM


32 M ed -S urg S uccess

2. “Cognitive” pertains to mental processes 3. Complaints of a pounding headache are not


of comprehension, judgment, memory, typical of a T12 spinal injury.
and reasoning. Therefore, an appropriate 4. Hypotension (low blood pressure) and tachy-
goal would be for the client to stay on task cardia (rapid heart rate) are signs of
for 10 minutes. hypovolemic or septic shock, but these do not
3. The client’s ability to dress self addresses self- occur in spinal shock.
care problems, not a cognitive problem. TEST TAKING HINT: If the test taker does not
4. The client’s ability to regain bowel and have any idea what the answer is, an attempt
bladder control does not address cognitive to relate the anatomical position of keywords
deficits. in the question stem to words in the answer
TEST TAKING HINT: The test taker must note options is appropriate. In this case, T12,
adjectives closely. The question is asking ­mentioned in the stem, is around the waist,
about “cognitive” deficits; ­therefore, the cor- so a­ nswer options involving the anatomy
rect answer must ­address cognition. above that level (e.g., the u ­ pper ­extremities)
Content – Medical: Integrated Nursing Process – can be eliminated.
Diagnosis: Client Needs – Physiological Integrity, Physi- Content – Medical: Integrated Nursing Process –
ological Adaptation: Cognitive Level – Analysis: Assessment: Client Needs – Safe Effective Care
Concept – Neurologic Regulation. ­Environment, Management of Care: Cognitive Level –
Analysis: Concept – Mobility.

Spinal Cord Injury 27. 1. Oxygen is administered initially to maintain


a high arterial partial pressure of oxygen
25. 1. The nurse should stabilize the client’s neck (PaO2) because hypoxemia can worsen a neu-
prior to removal from the car. rological deficit to the spinal cord initially,
2. The nurse must stabilize the client’s neck but this client is in the rehabilitation depart-
before doing any further assessment. Most ment and thus not in the initial stages of the
nurses don’t carry penlights, and the cli- injury.
ent’s pupil reaction can be determined after 2. Deep vein thrombosis (DVT) is a poten-
stabilization. tial complication of immobility, which
3. The nurse must maintain a patent airway. can occur because the client cannot move
Airway is the first step in resuscitation. the lower extremities as a result of the L1
4. Shaking the patient could cause further dam- SCI. Low-dose anticoagulation therapy
age, possibly leading to paralysis. (Lovenox) helps prevent blood from co-
agulating, thereby preventing DVTs.
TEST TAKING HINT: Remember that, in a
3. The client is unable to move the lower
question asking about which action should
­extremities. The nurse should do passive
be taken first, all of the answers are interven-
ROM exercises.
tions, but only one should be implemented
4. A client with a spinal injury at C4 or above
first. There are very few “always” situations
would be dependent on a ventilator for
in the health-care profession, but in this
breathing, but a client with an L1 SCI
situation, unless the client’s car is on fire or
would not.
under water, stabilizing the client’s neck is
always the priority, followed by airway. TEST TAKING HINT: The test taker should
Content – Medical: Integrated Nursing Process –
notice any adjectives such as “rehabilitation,”
Implementation: Client Needs – Safe Effective Care which should clue the test taker into rul-
­Environment, Management of Care: Cognitive Level – ing out oxygen, which is for the acute phase.
Synthesis: Concept – Neurologic Regulation. The test taker should also be very selective
if choosing an ­answer with a definitive word
26. 1. Neurogenic shock associated with SCI such as “all” (option “1”).
represents a sudden depression of re-
Content – Medical: Integrated Nursing Process –
flex activity below the level of the injury. Implementation: Client Needs – Safe Effective Care
T12 is just above the waist; therefore, no ­Environment, Management of Care: Cognitive Level –
reflex activity below the waist would be Application: Concept – Mobility.
expected.
2. Assessment of the movement of the upper 28. 1. Oxygen is administered initially to prevent
extremities would be more appropriate with hypoxemia, which can worsen the spinal
a higher level injury; an injury in the cervical cord injury; therefore, the nurse should
area might cause an inability to move the up- determine how much oxygen is reaching
per extremities. the periphery.

02_Colgrove_Ch02.indd 32 02/06/16 7:59 PM


C hapter 2   N eurological D isorders 33

2. A C6 injury would not affect the client’s abil- TEST TAKING HINT: The nurse must be
ity to chew and swallow, so pureed food is not knowledgeable of diagnostic tests to
necessary. ­prepare the client for the tests safely. The
3. Breathing exercises are supervised by test taker must be realistic in d
­ etermining
the nurse to increase the strength and ­answers—is there any test in which a hear-
endurance of inspiratory muscles, ing problem would make the diagnostic test
especially those of the diaphragm. ­contraindicated?
4. Autonomic dysreflexia occurs during the Content – Medical: Integrated Nursing Process –
rehabilitation phase, not the acute phase. Implementation: Client Needs – Physiological Integrity,
5. Corticosteroids are administered to de- Reduction of Risk Potential: Cognitive Level – Application:
crease inflammation, which will decrease Concept – Neurologic Regulation.
edema, and help prevent edema from 31. 1. This action will not address the client’s
ascending up the spinal cord, causing pounding headache and hypertension.
breathing difficulties. 2. Dimming the lights will not help the client’s
TEST TAKING HINT: The test taker must no- condition.
tice where the client is receiving care, which 3. This is an acute emergency caused by ex-
may be instrumental in being able to rule aggerated autonomic responses to stimuli
out incorrect answer options and help in and only occurs after spinal shock has
identifying the correct answer. Remember resolved in the client with a spinal cord
Maslow’s hierarchy of needs—oxygen and injury above T6. The most common cause
breathing are priority nursing interventions. is a full bladder.
Content – Medical: Integrated Nursing Process – 4. The nurse should always assess the client be-
Implementation: Client Needs – Safe Effective Care fore administering medication.
­Environment, Management of Care: Cognitive Level –
TEST TAKING HINT: The test taker should
Application: Concept – Oxygenation.
apply the nursing process when answering
29. 1. Therapeutic communication addresses questions, and assessing the client comes
the client’s feelings and attempts to allow first, before administering any type of
the client to verbalize feelings; the nurse medication.
should be a therapeutic listener. Content – Medical: Integrated Nursing Process –
2. This is belittling the client’s feelings. Implementation: Client Needs – Safe Effective Care
3. The client does not owe the nurse an ­Environment, Management of Care: Cognitive Level –
explanation of his feelings; “why” is never Application: Concept – Neurologic Regulation.
therapeutic. 32. 1. The halo device is applied by inserting pins
4. This is advising the client and is not into the skull, and the client cannot remove
therapeutic. them; the pins should be checked for signs of
TEST TAKING HINT: When the question re- infection.
quests a therapeutic response, the test taker 2. Reddened areas, especially under the brace,
should ­select the answer option that has must be reported to the HCP because
“feelings” in the response. pressure ulcers can occur when wearing
Content – Medical: Integrated Nursing Process – this appliance for an extended period.
Implementation: Client Needs – Psychosocial Integrity: 3. The vest liner should be changed for ­hygiene
Cognitive Level – Application: Concept – Mood. reasons, but the halo part is not removed.
30. 1. The machine is very loud and the technician 4. The client should be encouraged to ambulate
will offer the client ear plugs, but hearing dif- to prevent complications of immobility.
ficulty will not affect the MRI scan. TEST TAKING HINT: The test taker would
2. Allergies to dairy products will not affect the need basic knowledge about a halo ­device to
MRI scan. answer this question easily, but some clues
3. The client does not need to be NPO for this are in the stem. A c­ ervical fracture is in the
procedure. upper portion of the spine or neck area,
4. MRI scans are often done in a very con- and most p ­ eople u­ nderstand that a halo is
fined space; many people who have claus- ­something that surrounds the f­ orehead or
trophobia must be medicated or even higher. So the test taker could get a mental
rescheduled for the procedure in an open image of a device that must span this area of
MRI machine, which may be available if the body and maintain a­ lignment of the neck.
needed. If an HCP ­attaches pins into the head, then
the test taker could assume that they were

02_Colgrove_Ch02.indd 33 02/06/16 7:59 PM


34 M ed -S urg S uccess

not to be ­removed by the client. Redness 2. Tube feedings should be treated as if they
­usually ­indicates some sort of problem with were medications, and this task cannot be
the skin. delegated.
Content – Medical: Integrated Nursing Process – 3. The assistant can place the client on the
Planning: Client Needs – Physiological Integrity, Reduction bedside commode as part of bowel train-
of Risk Potential: Cognitive Level – Synthesis: ing; the nurse is responsible for the train-
Concept – Mobility. ing but can delegate this task.
4. Evaluating the client’s ability to self-­
33. 1. This is not an emergency; therefore, the
catheterize must be done by the nurse.
nurse should not notify the health-care
provider. TEST TAKING HINT: Although each state has
2. A physiological change in the client requires its own delegation rules, teaching, assessing,
more than a therapeutic conversation. evaluating, and medication administration
3. Increasing the IV rate will not address the are nursing interventions that cannot be del-
cause of the problem. egated to unlicensed assistive personnel.
4. For the first two (2) weeks after an SCI Content – Medical: Integrated Nursing Process –
above T7, the blood pressure tends to be Planning: Client Needs – Safe Effective Care Environment,
unstable and low; slight elevations of the Management of Care: Cognitive Level – Synthesis:
head of the bed can cause profound hypo- Concept – Nursing Roles.
tension; therefore, the nurse should lower 36. 1. The ASIA is an appropriate referral for living
the head of the bed immediately. with this condition, but it does not help find
TEST TAKING HINT: The test taker should gainful employment after the injury.
notice that the only answer option that ad- 2. The rehabilitation commission of each
dresses the “bed” is the correct answer. This state will help evaluate and determine if
does not always help identify the correct the client can receive training or educa-
answer, but it is a hint that should be used if tion for another occupation after injury.
the test taker has no idea what the correct 3. The client is not asking about disability; he is
answer is. concerned about employment. Therefore, the
Content – Medical: Integrated Nursing Process – nurse needs to make a referral to the appro-
Implementation: Client Needs – Physiological Integrity, priate agency.
Physiological Adaptation: Cognitive Level – Application: 4. This does not address the client’s concern
Concept – Mobility. about gainful employment.
34. 1. This client has signs/symptoms of a TEST TAKING HINT: If the question
respiratory complication and should be as- ­ entions a specific age for a client, the
m
sessed first. nurse should consider it when attempt-
2. This is a psychosocial need and should be ad- ing to answer the question. This is a
dressed, but it does not have priority over a young person who needs to find gainful
physiological problem. ­employment. Remember Erickson’s stages
3. A client with a lower SCI would not be at risk of growth and development.
for autonomic dysreflexia; therefore, a com- Content – Medical: Integrated Nursing Process –
plaint of headache and feeling hot would not Implementation: Client Needs – Psychosocial Integrity:
be a priority over an airway problem. Cognitive Level – Application: Concept – Health Care
4. The client with a T4 SCI would not be System.
expected to move the lower extremities.
TEST TAKING HINT: The nurse should assess Seizures
the client who is at risk for dying or having
some type of complication that requires in- 37. 1. The nurse needs to protect the client from
tervention. Remember Maslow’s hierarchy of injury. Moving furniture would help ensure
needs, in which physiological problems are that the client would not hit something
always priority and airway is the top physi- accidentally, but this is not done first.
ological problem. 2. This is done to help keep the airway patent,
Content – Medical: Integrated Nursing Process – but it is not the first intervention in this
Assessment: Client Needs – Safe Effective Care Environ- specific situation.
ment, Management of Care: Cognitive Level – Analysis: 3. Assessment is important but when the cli-
Concept – Ph Regulation. ent is having a seizure, the nurse should not
35. 1. The nurse cannot delegate assessment or touch him.
teaching.

02_Colgrove_Ch02.indd 34 02/06/16 7:59 PM


C hapter 2   N eurological D isorders 35

4. The client should not remain in the chair Reduction of Risk Potential: Cognitive Level – Synthesis:
during a seizure. He should be brought Concept – Neurologic Regulation.
safely to the floor so that he will have 40. 1. Noticing the first thing the client does
room to move the extremities. during a seizure provides information and
TEST TAKING HINT: All of the answer options clues as to the location of the seizure in the
are possible interventions, so the test taker brain. It is important to document whether
should go back to the stem of the question the beginning of the seizure was observed.
and note that the question asks which inter- 2. Assessment is important, but during the sei-
vention has priority. “In the chair” is the key zure the nurse should not attempt to restrain
to this question because the nurse should the head to assess the eyes; muscle contrac-
always think about safety, and a client having tions are strong, and restraining the client
a seizure is not safe in a chair. could cause injury.
Content – Medical: Integrated Nursing Process – 3. This should be done, but it is not the first
Implementation: Client Needs – Safe Effective Care intervention when walking into a room where
­Environment, Management of Care: Cognitive Level – the client is beginning to have a seizure.
Synthesis: Concept – Neurologic Regulation. 4. The client should be protected from onlook-
38. 1. Head injury is one of the main reasons for ers, but the nurse should always address the
epilepsy that can be prevented through client first.
occupational safety precautions and high- TEST TAKING HINT: This is a prioritizing
way safety programs. question that asks the test taker which in-
2. Sedentary lifestyle is not a cause of epilepsy. tervention to implement first. All four inter-
3. Dietary concerns are not a cause of epilepsy. ventions would be appropriate, but only one
4. Safety glasses will help prevent eye injuries, should be implemented first. If the test taker
but such injuries are not a cause of epilepsy. cannot decide between two choices, always
TEST TAKING HINT: The nurse must be aware select the one that directly affects the client
of risk factors that cause diseases. If the test or the condition; privacy is important, but
taker does not know the correct answer, helping determine the origin of the seizure is
thinking about which body system the ques- priority.
tion is asking about may help rule out or rule Content – Medical: Integrated Nursing Process –
in some of the answer options. Only options Implementation: Client Needs – Safe Effective Care
“1” and “4” have anything to do with the ­Environment, Management of Care: Cognitive Level –
Synthesis: Concept – Neurologic Regulation.
head, and only helmets on the head are
connected with the neurological system. 41. 1. The client is exhausted from the seizure and
Content – Medical: Integrated Nursing Process – should be allowed to sleep.
Implementation: Client Needs – Health Promotion and 2. Awakening the client every 30 minutes pos-
Maintenance: Cognitive Level – Application: Concept – sibly could induce another seizure as a result
Neurologic Regulation. of sleep deprivation.
39. 1. Antiseizure drugs, tranquilizers, stimulants, 3. During the postictal (after-seizure) phase,
and depressants are withheld before an EEG the client is very tired and should be
because they may alter the brain wave patterns. allowed to rest quietly; placing the client
2. Meals are not withheld because altered blood on the side will help prevent aspiration
glucose level can cause changes in brain wave and maintain a patent airway.
patterns. 4. The client must rest, and asking questions
3. The goal is for the client to have a seizure about the seizure will keep the client awake,
during the EEG. Sleep deprivation, hyper- which may induce another seizure as a result
ventilating, or flashing lights may induce a of sleep deprivation.
seizure. TEST TAKING HINT: Options “1,” “2,” and “4”
4. Electrodes are placed on the client’s scalp, all have something to do with keeping the
but there are no electroshocks or any type of client awake. This might lead the test taker
discomfort. to choose the option that is different from
TEST TAKING HINT: The test taker should the other three.
highlight the words “diagnose a seizure Content – Medical: Integrated Nursing Process –
disorder” in the stem and ask which answer Implementation: Client Needs – Safe Effective Care
option would possibly cause a seizure. Environment, Management of Care: Cognitive Level –
Application: Concept – Neurologic Regulation.
Content – Medical: Integrated Nursing Process –
Planning: Client Needs – Physiological Integrity,

02_Colgrove_Ch02.indd 35 02/06/16 7:59 PM


36 M ed -S urg S uccess

42. 1. Once the seizure has started, no one should 44. 1. Assessment is an independent nursing
attempt to put anything in the client’s mouth. action, not a collaborative one.
2. The nurse should tell the UAP to stop 2. All clients in the ICD will be placed on
trying to insert anything in the mouth of telemetry, which does not require an order by
the client experiencing a seizure. Broken another health-care provider or collaboration
teeth and injury to the lips and tongue with one.
may result from trying to place anything 3. Administering an anticonvulsant medi-
in the clenched jaws of a client having a cation by intravenous push requires the
tonic-clonic seizure. nurse to have an order or confer with an-
3. The primary nurse is responsible for the ac- other member of the health-care team.
tion of the UAP and should stop the UAP 4. A glucocorticoid is a steroid and is not used
from doing anything potentially dangerous to treat seizures.
to the client. No one should attempt to pry TEST TAKING HINT: The keyword in the stem
open the jaws that are clenched in a spasm to of this question is the adjective “collabora-
insert anything. tive.” The test taker would eliminate the
4. The primary nurse must correct the action of options “1” and “2” because these do not
the UAP immediately, prior to any injury oc- require collaboration with another member
curring to the client and before notifying the of the health-care team and would eliminate
charge nurse. option “4” because it is not used to treat
TEST TAKING HINT: The nurse is responsible seizures.
for the actions of the unlicensed assistive Content – Medical: Integrated Nursing Process –
personnel and must correct the behavior Planning: Client Needs – Safe Effective Care
immediately. Environment, Management of Care: Cognitive Level –
Content – Medical: Integrated Nursing Process – Synthesis: Concept – Neurologic Regulation.
Implementation: Client Needs – Safe Effective Care 45. 1. Keeping a seizure and medication chart
Environment, Management of Care: Cognitive Level –
will be helpful when keeping follow-up
Application: Concept – Neurologic Regulation.
appointments with the health-care pro-
43. 1. Thorough oral hygiene after each meal, vider and in identifying activities that may
gum massage, daily flossing, and regu- trigger a seizure.
lar dental care are essential to prevent 2. The client should take showers, rather than
or control gingival hyperplasia, which is tub baths, to avoid drowning if a seizure
a common occurrence in clients taking occurs. The nurse should also instruct the cli-
Dilantin. ent never to swim alone.
2. A serum (venipuncture) Dilantin level is 3. Over-the-counter medications may contain
checked monthly at first and then, after ingredients that will interact with antisei-
a therapeutic level is attained, every zure medications or, in some cases, as with
six (6) months. use of stimulants, possibly cause a seizure.
3. Dilantin does not turn the urine orange. 4. Most of the anticonvulsant medications
4. The use of Dilantin does not ensure that the have therapeutic serum levels that should
client will not have any seizures, and, in some be maintained, and regular checks of the se-
instances, the dosage may need to be adjusted rum levels help to ensure the correct level.
or another medication may need to be used. 5. A newly diagnosed client would have just
TEST TAKING HINT: The test taker should been put on medication, which may cause
realize that monitoring blood glucose levels drowsiness. Therefore, the client should avoid
using a glucometer is about the only level activities that require alertness and coordi-
that is monitored daily; therefore, option “2,” nation and should not be driving at all until
which calls for daily monitoring of Dilantin after the effects of the medication have been
levels, could be eliminated. Remember, there evaluated.
are very few absolutes in the health-care TEST TAKING HINT: The test taker must select
field; therefore, option “4” could be ruled all interventions that are appropriate for the
out because “won’t have any” is an absolute. question. A key word is the adverb “newly.”
Content – Medical: Integrated Nursing Process – Content – Medical: Integrated Nursing Process –
Evaluation: Client Needs – Physiological Integrity, Planning: Client Needs – Physiological Integrity,
Pharmacological and Parenteral Therapies: Cognitive Physiological Adaptation: Cognitive Level – Synthesis:
Level – Evaluation: Concept – Nursing Roles. Concept – Nursing Roles.

02_Colgrove_Ch02.indd 36 02/06/16 7:59 PM


C hapter 2   N eurological D isorders 37

46. 1. The client with a seizure disorder should TEST TAKING HINT: All four answer ­options
avoid stimulants, such as caffeine. are associated with the brain, neurological
2. The onset of menstruation can cause seizure system, and aging. However, options “1,” “2,”
activity in the female client. and “4” usually occur over time, with the
3. Tension states, such as anxiety and frus- condition gradually getting worse, and thus
tration, induce seizures in some clients, can be eliminated as a cause of seizures,
so stress management may be helpful in which are usually sudden.
preventing seizures. Content – Medical: Integrated Nursing Process –
4. Bright flickering lights, television viewing, Implementation: Client Needs – Health Promotion and
and some other photic (light) stimulation may Maintenance: Cognitive Level – Knowledge: Concept –
cause seizures but sunlight does not. Wearing Neurologic Regulation.
dark glasses or covering one eye during po-
tential seizure-stimulating activities may help
prevent seizures. Brain Tumors
TEST TAKING HINT: Caffeine is a stimulant
49. 1. Nervousness is not a symptom of a brain
and its use is not recommended in many dis- tumor, and brain tumors rarely metastasize
ease processes. Menstrual cycle changes are outside of the cranium. Brain tumors kill by
known to affect seizure disorders. Therefore, occupying space and increasing intracranial
options “1” and “2” can be eliminated, as can pressure. Although seizures are not uncom-
option “4” because sunlight does not cause mon with brain tumors, seizures are not part
seizures. of the classic triad of symptoms.
Content – Medical: Integrated Nursing Process – 2. The classic triad of symptoms suggesting
Evaluation: Client Needs – Physiological Integrity, a brain tumor includes a headache that is
Physiological Adaptation: Cognitive Level – Evaluation:
dull, unrelenting, and worse in the morn-
Concept – Neurologic Regulation.
ing; vomiting unrelated to food intake; and
47. 1. An aura is a visual, an auditory, or an edema of the optic nerve ­(papilledema),
olfactory occurrence that takes place prior which occurs in 70% to 75% of clients
to a seizure and warns the client a seizure diagnosed with brain ­tumors. Papilledema
is about to occur. The aura often allows causes visual disturbances such as de-
time for the client to lie down on the floor creased visual acuity and diplopia.
or find a safe place to have the seizure. 3. Hypertension and bradycardia, not hypoten-
2. An aura is not dependent on the client being sion and tachycardia, occur with increased
physically or psychologically exhausted. intracranial pressure resulting from pressure
3. This is a therapeutic response, reflecting feel- on the cerebrum. Tachypnea does not occur
ings, which is not an appropriate response with brain tumors.
when answering a client’s question. 4. Abrupt loss of motor function occurs with a
4. Sleepiness after a seizure is very common, but stroke; diarrhea does not occur with a brain
the aura does not itself cause the sleepiness. tumor, and the client with a brain tumor does
TEST TAKING HINT: If the stem of the ques- not experience a change in taste.
tion has the client asking a question, then the TEST TAKING HINT: The test taker can rule
nurse needs to give factual information, and out option “4” because of the symptom of
option “3,” a therapeutic response, would not diarrhea, which is a gastrointestinal symp-
be appropriate. Neither would option “2” or tom, not a neurological one. Considering the
“4” because these options are worded in such other three possible choices, the symptom
a way as to imply incorrect information. of “headache” would make sense for a client
Content – Medical: Integrated Nursing Process – with a brain tumor.
Implementation: Client Needs – Physiological Integrity, Content – Medical: Integrated Nursing Process –
Physiological Adaptation: Cognitive Level – Application: Assessment: Client Needs – Physiological Integrity,
Concept – Neurologic Regulation. ­Physiological Adaptation: Cognitive Level – Analysis:
Concept – Cellular Regulation.
48. 1. Alzheimer’s disease does not lead to seizures.
2. Parkinson’s disease does not cause seizures. 50. 1. A widening pulse pressure and bounding
3. A CVA (stroke) is the leading cause of sei- pulse indicate increased intracranial pressure
zures in the elderly; increased intracranial but do not localize the tumor.
pressure associated with the stroke can 2. Diplopia and decreased visual pressure are
lead to seizures. symptoms indicating papilledema, a general
4. Brain atrophy is not associated with seizures. symptom in the majority of all brain tumors.

02_Colgrove_Ch02.indd 37 02/06/16 7:59 PM


38 M ed -S urg S uccess

3. Bradykinesia is slowed movement, a ­symptom stem; in this case, “prophylactic” is the key to
of Parkinson’s disease, and scanning speech is answering the question correctly.
symptomatic of multiple sclerosis. Content – Medical: Integrated Nursing Process –
4. Hemiparesis would localize a tumor to a Planning: Client Needs – Physiological Integrity, Reduction
motor area of the brain, and personality of Risk Potential: Cognitive Level – Synthesis:
changes localize a tumor to the frontal Concept – Cellular Regulation.
lobe. 53. 1. A social worker is qualified to assist the client
TEST TAKING HINT: The test taker could arrive with referrals to any agency or
at the correct answer if the test taker realized personnel that is needed.
that specific regions of the brain control motor 2. The chaplain should be referred if spiritual
function and hemiparesis and that other re- guidance is required, but the stem did not
gions are involved in personality changes. specify this need.
Content – Medical: Integrated Nursing Process – 3. The HCP also can refer to the social
Assessment: Client Needs – Physiological Integrity, worker, but the nurse can make this referral
Physiological Adaptation: Cognitive Level – Analysis: independently.
Concept – Cellular Regulation. 4. The occupational therapist assists with
51. 1. This is providing information and is not com- cognitive functioning, activities of daily
pletely factual. MRIs are loud, but frequently living (ADLs), and modification of the home,
the client will require an IV access (an inva- but the stem did not define these needs.
sive procedure) to be started for a contrast TEST TAKING HINT: The test taker must
medium to be injected. decide what each discipline has to offer the
2. This is restating and offering self. Both client; an SW has the broadest range of
are therapeutic responses. ­referral capabilities.
3. This statement is belittling the client’s Content – Medical: Integrated Nursing Process –
feeling. Implementation: Client Needs – Safe Effective Care
4. This is not dealing with the client’s concerns Environment, Management of Care: Cognitive Level –
and is passing the buck. The nurse should ex- Application: Concept – Health Care System.
plore the client’s feeling to determine what is
54. 1. Maintaining weight is a nutritional goal.
concerning the client. The MRI may or may
2. Completing an advance directive is an
not be the problem. The client may be afraid
end-of-life or psychosocial goal.
of the results of the MRI.
3. Performing activities of daily living is a
TEST TAKING HINT: When the question asks goal for self-care deficit.
the test taker for a therapeutic response, the 4. Verbalizing feelings is a psychosocial goal.
test taker should choose the response that TEST TAKING HINT: The test taker should
directly addresses the client’s feelings. read the stem of the question carefully. All
Content – Medical: Integrated Nursing Process – of the goals could be appropriate for a client
Implementation: Client Needs – Physiological Integrity, diagnosed with a brain tumor, but only one
Reduction of Risk Potential: Cognitive Level – Application:
applies to self-care deficit.
Concept – Cellular Regulation.
Content – Medical: Integrated Nursing Process –
52. 1. Nothing in the stem indicates the client has a Planning: Client Needs – Physiological Integrity,
problem with swallowing, so aspiration pre- Physiological Adaptation: Cognitive Level – Synthesis:
cautions are not needed. Concept – Cellular Regulation.
2. Reach to Recovery is an American Cancer 55. 1. Purposeful movement following painful
Society–sponsored program for clients with stimuli would indicate an improvement in the
breast cancer, but it is not prophylactic. client’s condition.
3. The client diagnosed with metastatic 2. Adducting the upper extremities while inter-
lesions to the brain is at high risk for nally rotating the lower extremities is decor-
seizures. ticate positioning and would indicate that the
4. Teaching about mastectomy care is not client’s condition had not changed.
prophylactic, and the stem did not indicate 3. Aimless thrashing would indicate an improve-
whether the client had a mastectomy. ment in the client’s condition.
TEST TAKING HINT: The test taker should not 4. The most severe neurological impair-
read into a question—for example, about ment result is flaccidity and no response
mastectomy care. The test taker should be to stimuli. This indicates that the client’s
careful to read the descriptive words in the condition has worsened.

02_Colgrove_Ch02.indd 38 02/06/16 7:59 PM


C hapter 2   N eurological D isorders 39

TEST TAKING HINT: Neurological assessment 58. 1. Making the client NPO will not help the
includes assessing the client for levels of ­client to swallow.
consciousness; the nurse must memorize the 2. A low Fowler’s position would make it easier
stages of neurological progression toward a for the client to aspirate.
coma and death. 3. The consistency of the food is not an issue;
Content – Medical: Integrated Nursing Process – Assess- the client will have difficulty swallowing this
ment: Client Needs – Safe Effective Care Environment, food as well as regular-consistency food.
Management of Care: Cognitive Level – Analysis: 4. To decrease the risk of aspiration, the cli-
Concept – Cellular Regulation. ent should direct food to the unaffected
side of the throat; this helps the client to
56. 1. A transsphenoidal hypophysectomy is done
be able to use the side of the throat that is
by an incision above the gumline and through
functioning.
the sinuses to reach the sella turcica, where
the pituitary is located. TEST TAKING HINT: The test taker should
2. The client will be given regular food when try to visualize the position of the client
awake and able to tolerate food. in the bed. A mostly recumbent position
3. Blowing the nose creates increased (low Fowler’s) would increase the chance
intracranial pressure and could result in a of aspiration; thus option “2” should be
cerebrospinal fluid leak. eliminated.
4. The client will return from surgery with Content – Medical: Integrated Nursing Process –
the head of the bed elevated to about Implementation: Client Needs – Safe Effective Care
30 degrees; this allows for gravity to assist Environment, Management of Care: Cognitive Level –
in draining the cerebrospinal fluid. Application: Concept – Cellular Regulation.

TEST TAKING HINT: The test taker must know 59. 1. Chemotherapy is systemic therapy that is
the procedures for specific disease processes used extensively in the care of clients diag-
to answer this question, but anatomical posi- nosed with cancer. However, most drugs have
tioning of the pituitary gland (just above the difficulty in crossing the blood–brain barrier
sinuses) could help to eliminate option “1,” and are not useful in treating brain tumors
which calls for a large turban dressing. unless delivered by direct placement into the
Content – Surgical: Integrated Nursing Process – spinal column or directly to the ventricles
Planning: Client Needs – Physiological Integrity, of the brain by a device called an Omaya
Physiological Adaptation: Cognitive Level – Synthesis: reservoir.
Concept – Nursing Roles. 2. The blood–brain barrier is the body’s de-
fense mechanism for protecting the brain
57. 1. A headache after this surgery would be an ex-
from chemical effects; in this case, it pre-
pected occurrence, not a complication.
vents the chemotherapy from being able
2. An output much larger than the intake
to work on the tumor in the brain.
could indicate the development of diabe-
3. Radiation has about the same amount of side
tes insipidus. Pressure on the pituitary
effects as chemotherapy, but the effects of ra-
gland can result in decreased production
diation tend to last for a much longer time.
of vasopressin, the antidiuretic hormone
4. Some tumors do become resistant to the che-
(ADH).
motherapy agents used. When this happens,
3. A raspy sore throat is common after surgery
the oncologist switches to different drugs.
due to the placement of the endotracheal
tube during anesthesia. TEST TAKING HINT: The test taker can elimi-
4. Dizziness on arising quickly is expected; the nate option “1” as a possible answer because
client should be taught to rise slowly and call it states that chemotherapy is used to treat
for assistance for safety. brain tumors, but it does not tell the client
why it is not being used. Option “2” is the
TEST TAKING HINT: The test taker could
only one that actually informs the client of a
eliminate options “1” and “3” as expected
medical reason for not administering chemo-
occurrences following the surgery and
therapy for a brain tumor.
not complications. Option “4” can also be
expected. Content – Medical: Integrated Nursing Process –
Diagnosis: Client Needs – Physiological Integrity,
Content – Surgical: Integrated Nursing Process –
Pharmacological and Parenteral Therapies: Cognitive
Assessment: Client Needs – Physiological Integrity, Reduc-
Level – Analysis: Concept – Cellular Regulation.
tion of Risk Potential: Cognitive Level – Analysis:
Concept – Cellular Regulation.

02_Colgrove_Ch02.indd 39 02/06/16 7:59 PM


40 M ed -S urg S uccess

60. 1. The client should sleep with the head groups such as college campuses, jails, and
of the bed elevated to promote drainage of military installations.
the cerebrospinal fluid. 3. Third world countries do not pose a risk fac-
2. Humidified air will prevent drying of the tor for meningitis. They provide a risk for
nasal passages. hepatitis or tuberculosis.
3. Because the incision for this surgery is 4. Employees in a high-rise building do not
just above the gumline, the client should live together and they have their own space;
not brush the front teeth. Oral care therefore, they are not at risk for developing
should be performed using a sponge meningitis.
until the incision has healed. TEST TAKING HINT: The test taker must re-
4. The client can eat a regular diet. member that the NCLEX-RN tests all areas
5. The HCP should be notified if the cli- of nursing, so always notice the type of nurse
ent develops an infection of any kind. A if this is mentioned in the stem. A public
cold with sinus involvement and sneezing health nurse would not be concerned with
places the client at risk for opening the third world countries.
incision and developing a brain infection. Content – Medical: Integrated Nursing Process – Plan-
TEST TAKING HINT: The test taker could ning: Client Needs – Health Promotion and Maintenance:
choose option “5” because this is a standard Cognitive Level – Synthesis Concept – Nursing Roles.
instruction for any surgery. The test taker
63. 1. Babinski’s sign is used to assess brainstem
should look for more than one correct an-
activity, and paresthesia is tingling, which
swer in an alternative-type question.
is not a clinical manifestation of bacterial
Content – Surgical: Integrated Nursing Process – meningitis.
Planning: Client Needs – Physiological Integrity, Physi-
2. Chvostek’s sign is used to assess for hypocal-
ological Adaptation: Cognitive Level – Synthesis:
Concept – Nursing Roles.
cemia, and facial tingling is a sign of hypo-
calcemia. It is not used to assess for bacterial
meningitis.
Meningitis 3. A positive Kernig’s sign (client unable to
extend leg when lying flat) and nuchal
61. 1. This is a definition of aseptic meningitis, rigidity (stiff neck) are signs of bacterial
which refers to irritated meninges from viral meningitis, occurring because the menin-
or noninfectious sources. ges surrounding the brain and spinal col-
2. This is another example of aseptic meningitis, umn are irritated.
which refers to irritated meninges from viral 4. Trousseau’s sign is used to assess for hypo-
or noninfectious sources. calcemia, and nystagmus is abnormal eye
3. Septic meningitis refers to meningitis movement. Neither of these is a clinical
caused by bacteria; the most common manifestation of bacterial meningitis.
form of bacterial meningitis is caused by TEST TAKING HINT: If two answer options
the Neisseria meningitides bacteria. test for the same thing (Trousseau’s and Ch-
4. This is the explanation for encephalitis. vostek’s signs), then the test taker can rule
TEST TAKING HINT: The nurse should explain out these as possible answers because there
the client’s diagnosis in layperson’s terms cannot be two correct answers in the ques-
when the stem is identifying the significant tion, unless the question tells the test taker
other as asking the question. Be sure to no- that it is a “select all that apply” question.
tice that the adjective “septic” is the key to Content – Medical: Integrated Nursing Process –
answering this question, ruling out ­options Assessment: Client Needs – Safe Effective Care Environ-
“1” and “2.” ment, Management of Care: Cognitive Level – Analysis:
Concept – Neurologic Regulation.
Content – Medical: Integrated Nursing Process –
Implementation: Client Needs – Physiological Integrity, 64. 1. In clients with meningococcal meningitis,
Physiological Adaptation: Cognitive Level – Application: purpuric lesions over the face and extremity
Concept – Neurologic Regulation. are the signs of a fulminating infection that
62. 1. Clients who have been hospitalized are weak- can lead to death within a few hours.
ened, but they are not at risk for contracting 2. Photophobia is a common clinical manifesta-
any type of meningitis. tion of all types of meningitis and would be
2. Outbreaks of infectious meningitis are expected.
most likely to occur in dense community 3. Inflammation of the meninges results in in-
creased intracranial pressure, which causes a

02_Colgrove_Ch02.indd 40 02/06/16 7:59 PM


C hapter 2   N eurological D isorders 41

headache. This would be an expected occur- 3. This would be an appropriate goal for the cli-
rence and would not warrant notifying the ent who has a problem of infection.
HCP. 4. This would be an appropriate goal for the cli-
4. A client not being able to identify the day of ent who has a problem of dehydration.
the week would not in itself warrant notifying TEST TAKING HINT: The goal must be related
the HCP. to the problem—in this case, “altered cere-
TEST TAKING HINT: The stem is asking the bral tissue perfusion.”
nurse to identify which assessment data are Content – Medical: Integrated Nursing Process –
abnormal for the disease process and require Diagnosis: Client Needs – Physiological Integrity, Physi-
an immediate medical intervention to pre- ological Adaptation: Cognitive Level – Analysis:
vent the client from experiencing a complica- Concept – Cellular Regulation.
tion or possible death. 67. 1. A lumbar puncture is an invasive procedure;
Content – Medical: Integrated Nursing Process – therefore, an informed consent is required.
Assessment: Client Needs – Safe Effective Care Environ- 2. This could be offered for client comfort
ment, Management of Care: Cognitive Level – Synthesis:
during the procedure.
Concept – Neurologic Regulation.
3. This position increases the space between
65. 1. Standard Precautions are mandated for all the vertebrae, which allows the HCP eas-
clients, but a client with septic meningitis ier entry into the spinal column.
will require more than the Standard 4. The client is encouraged to relax and breathe
Precautions. normally; hyperventilation may lower an
2. Airborne Precautions are for contagious elevated cerebrospinal fluid pressure.
organisms that are spread on air currents 5. The nurse should always explain to the
and require the hospital personnel to wear client what is happening prior to and dur-
an ultra-high filtration mask; these precau- ing a procedure.
tions would be applied for diseases such as TEST TAKING HINT: This is an alternative-type
tuberculosis. question, which requires the test taker to se-
3. Contact Precautions are for contagious or- lect more than one answer option.
ganisms that are spread by blood and body Content – Medical: Integrated Nursing Process –
fluids, such as those that occur with wounds Implementation: Client Needs – Safe Effective Care
or diarrhea. Environment, Management of Care: Cognitive Level –
4. Droplet Precautions are respiratory pre- Analysis: Concept – Neurologic Regulation.
cautions used for organisms that have a
limited span of transmission. Precautions 68. 1. A nurse administering antibiotics is a col-
include staying at least four (4) feet away laborative intervention because the HCP
from the client or wearing a standard iso- must write an order for the intervention;
lation mask and gloves when coming in nurses cannot prescribe medications un-
close contact with the client. Clients are less they have additional education and
in isolation for 24 to 48 hours after initia- licensure and are nurse practitioners with
tion of antibiotics. prescriptive authority.
2. The nurse needs an order to send a culture to
TEST TAKING HINT: The test taker must know
the laboratory for payment purposes, but the
the types of isolation precautions used for nurse can obtain a specimen without an order.
different diseases and note the adjective— A sputum specimen is not appropriate for
“septic”—in the stem of the question. meningitis.
Content – Medical: Integrated Nursing Process – 3. A pulse oximeter measures the amount of
Implementation: Client Needs – Safe Effective Care Envi- oxygen in the periphery and does not require
ronment, Safety and Infection Control: Cognitive Level –
an HCP to order.
Application: Concept – Neurologic Regulation.
4. Intake and output are independent nursing
66. 1. This goal is not related to altered cerebral tis- interventions and do not require an HCP’s
sue perfusion, but it would be a goal for self- order.
care deficit. TEST TAKING HINT: The test taker must note
2. A client with a problem of altered cere- adjectives and understand that a collabora-
bral tissue perfusion is at risk for seizure tive nursing intervention is dependent on
activity secondary to focal areas of cortical another member of the health-care team; an
irritability; therefore, the client should be independent nursing intervention does not
on seizure precautions. require collaboration.

02_Colgrove_Ch02.indd 41 02/06/16 7:59 PM


42 M ed -S urg S uccess

Content – Medical: Integrated Nursing Process – Content – Medical: Integrated Nursing Process –
Diagnosis: Client Needs – Physiological Integrity, Physi- Diagnosis: Client Needs – Physiological Integrity,
ological Adaptation: Cognitive Level – Analysis: ­Pharmacological and Parenteral Therapies: Cognitive
Concept – Neurologic Regulation. Level – Analysis: Concept – Neurologic Regulation.

69. 1. This vaccine must be administered prior to 71. 1. The antibiotic has the highest priority be-
exposure to build up an immunity to pre- cause failure to treat a bacterial infection can
vent meningitis resulting from Haemophilus result in shock, systemic sepsis, and death.
influenzae. 2. The lunch tray is important and may ­actually
2. Chemoprophylaxis includes administering arrive prior to the antibiotic, but the ­priority
medication that will prevent infection or for the nurse must be the medication.
eradicate the bacteria and the develop- 3. The client’s room should be kept dark be-
ment of symptoms in people who have cause of photophobia, but photophobia is a
been in close proximity to the client. symptom that is not life threatening.
Medications include rifampin (Rifadin), 4. Knowledge of the client’s weight is necessary,
ciprofloxacin (Cipro), and ceftriaxone but initial antibiotic therapy can be initi-
(Rocephin). ated without knowing the client’s admission
3. Steroids are used as an adjunct therapy in weight.
treatment of clients diagnosed with acute TEST TAKING HINT: The nurse must know
bacterial meningitis. They would not be given how to prioritize care. Which intervention
as a prophylactic measure to others has the potential to avoid a complication re-
in the home. lated to the disease process? Remember the
4. Gamma globulin provides passive immunity word “priority.”
to clients who have been exposed to hepatitis. Content – Medical: Integrated Nursing Process –
It is not appropriate in this situation. Planning: Client Needs – Safe Effective Care Environment,
TEST TAKING HINT: The key word in the stem Management of Care: Cognitive Level – Synthesis:
is “preventive.” The test taker must pay close Concept – Neurologic Regulation.
attention to the adjectives. 72. 1. The client’s lung sounds should be clear with
Content – Medical: Integrated Nursing Process – meningitis, and nothing in the question stem
Planning: Client Needs – Safe Effective Care ­Environment, indicates a comorbid condition. Therefore,
Safety and Infection Control: Cognitive Level – Synthesis:
assessing lung sounds is not a priority.
Concept – Neurologic Regulation.
2. The client may experience photophobia and
70. 1. Antibiotics would help decrease the bacterial visual disturbances, but assessing the six fields
infection in meningitis, which would cause of gaze will not affect the client’s condition.
the exudate. The drugs mentioned in the 3. The client’s cardiac status is not affected by
question would not. meningitis. Therefore, the apical pulse would
2. Fever increases cerebral metabolism and not be a priority.
intracranial pressure. Therefore, measures 4. Meningitis directly affects the client’s
are taken to reduce body temperature as brain. Therefore, assessing the neurologi-
soon as possible, and alternating Tylenol cal status would have priority for
and Motrin would be appropriate. this client.
3. A nonnarcotic antipyretic (Tylenol) and an TEST TAKING HINT: The test taker should
NSAID (Motrin) will not address the client’s apply a systemic approach to discerning the
memory or orientation. priority response. Maslow’s hierarchy of
4. These medications do not prevent or treat a needs would put option “1” as correct, but
yeast infection. the disease process of meningitis does not
TEST TAKING HINT: The test taker must have include signs or symptoms of a respiratory
a basic knowledge of the disease process and component. The next highest priority would
medications that are prescribed to treat a be the neurological component, and menin-
disease. Purulent drainage would require an gitis definitely is a neurological disease.
antibiotic. Therefore, option “1” should be Content – Medical: Integrated Nursing Process –
eliminated as a possible answer because the Assessment: Client Needs – Safe Effective Care Environ-
question is asking about NSAIDs and a non- ment, Management of Care: Cognitive Level – Synthesis:
narcotic antipyretic (Tylenol). Concept – Neurologic Regulation.

02_Colgrove_Ch02.indd 42 02/06/16 7:59 PM


C hapter 2   N eurological D isorders 43

Parkinson’s Disease 75. 1. Adaptive appliances will not help the client’s
shaking movements and are not used for
73. 1. Masklike facies is responsible for lack of clients with Parkinson’s disease.
expression and is part of the motor mani- 2. Clients with Parkinson’s disease are placed on
festations of Parkinson’s disease but is not high-calorie, high-protein, soft or liquid diets.
related to the symptoms listed. Shuffling is Supplemental feedings may also be ordered.
also a motor deficit and does pose a risk for If liquids are ordered because of difficulty
falling, but fever and patchy infiltrates on a chewing, then the liquids should be thickened
chest x-ray do not result from a gait prob- to a honey or pudding consistency.
lem. They are manifestations of a pulmonary 3. Nuts and whole-grain food would require ex-
complication. tensive chewing before swallowing and would
2. Difficulty swallowing places the client at not be good for the client. Three large meals
risk for aspiration. Immobility predisposes would get cold before the client can consume
the client to pneumonia. Both clinical the meal, and one-half or more of the food
manifestations place the client at risk for would be wasted.
pulmonary complications. 4. The client’s energy levels will not
3. Pill rolling of fingers and flat affect do not sustain eating for long periods. Offering
have an impact on the development of pul- frequent and easy-to-chew (soft) meals
monary complications. of small proportions is the preferred
4. Arm swing and bradykinesia are motor dietary plan.
deficits. TEST TAKING HINT: The correct answer for a
TEST TAKING HINT: The nurse must recog- nursing problem question must address the
nize the clinical manifestations of a disease actual problem.
and the resulting bodily compromise. In this Content – Medical: Integrated Nursing Process –
situation, fever and patchy infiltrates on a Diagnosis: Client Needs – Physiological Integrity,
chest x-ray indicate a pulmonary complica- ­Physiological Adaptation: Cognitive Level – Analysis:
tion. Options “1,” “3,” and “4” focus on mo- Concept – Neurologic Regulation.
tor problems and could be ruled out as too 76. 1. The nurse should not delegate feeding
similar. Only option “2” includes dissimilar a client who is at risk for complications
information. during feeding. This requires judgment
Content – Medical: Integrated Nursing Process – that the UAP is not expected to possess.
Assessment: Client Needs – Physiological Integrity, 2. Unlicensed assistive personnel can turn and
­Physiological Adaptation: Cognitive Level – Analysis: position clients with pressure ulcers. The
Concept – Neurologic Regulation. nurse should assist in this at least once during
74. 1. Carbidopa is never given alone. Carbidopa the shift to assess the wound area.
is given together with levodopa to help the 3. The UAP can assist the client to the bath-
levodopa cross the blood–brain barrier. room every two (2) hours and document the
2. Levodopa is a form of dopamine given orally results of the attempt.
to clients diagnosed with PD. 4. The UAP can obtain the vital signs on a
3. Carbidopa enhances the effects of stable client.
levodopa by inhibiting decarboxylase in TEST TAKING HINT: When reading the answer
the periphery, thereby making more options in a question in which the nurse is
levodopa available to the central nervous delegating to an unlicensed-assistive person-
system. Sinemet is the most effective nel, read the stem carefully. Is the question
treatment for PD. asking what to delegate or what not to del-
4. Carbidopa does not cross the blood–brain egate? Anything requiring professional judg-
barrier. ment should not be delegated.
TEST TAKING HINT: The nurse must be Content – Medical: Integrated Nursing Process –
knowledgeable of the rationale for adminis- Planning: Client Needs – Safe Effective Care Environment,
tering a medication for a specific disease. Management of Care: Cognitive Level – Synthesis:
Concept – Nursing Roles.
Content – Medical: Integrated Nursing Process –
Diagnosis: Client Needs – Physiological Integrity,
Pharmacological and Parenteral Therapies: Cognitive
Level – Analysis: Concept – Neurologic Regulation.

02_Colgrove_Ch02.indd 43 02/06/16 7:59 PM


44 M ed -S urg S uccess

77. 1. Headache and photophobia are expected 3. Physical therapy is a standard therapy used
clinical manifestations of meningitis. to improve the quality of life for clients diag-
The new graduate could care for this nosed with PD.
client. 4. Fetal tissue transplantation has shown
2. This client has had an unusual occurrence some success in PD, but it is an ex-
(fall) and now has a potential complication perimental and highly controversial
(a fracture). The experienced nurse should procedure.
take care of this client. TEST TAKING HINT: The test taker should not
3. These vital signs indicate increased overlook the adjective “experimental.” This
­intracranial pressure. The more experienced would eliminate at least option “3,” physi-
nurse should care for this client. cal therapy, and option “2,” which refers to
4. This could indicate a worsening of the standard dopamine treatment, even if the
tumor. This client is at risk for seizures and test taker was not familiar with all of the
herniation of the brainstem. The more expe- procedures.
rienced nurse should care for this client. Content – Medical: Integrated Nursing Process –
TEST TAKING HINT: The test taker should de- Planning: Client Needs – Physiological Integrity, Physi-
termine if the clinical manifestations are ex- ological Adaptation: Cognitive Level – Synthesis:
pected as part of the disease process. If they Concept – Neurologic Regulation.
are, a new graduate can care for the client; 80. 1. The emotions of a person diagnosed with PD
if they are not expected occurrences, a more are labile. The client has rapid mood swings
experienced nurse should care for the client. and is easily upset.
Content – Medical: Integrated Nursing Process – 2. Hallucinations are a sign that the client is ex-
Planning: Client Needs – Safe Effective Care Environment, periencing drug toxicity.
Management of Care: Cognitive Level – Synthesis:
3. Scheduling appointments late in the
Concept – Nursing Roles.
morning gives the client a chance to com-
78. 1. Akinesia is lack of movement. The goal in plete ADLs without pressure and allows
treating PD is to maintain mobility. the medications time to give the best
2. This could be a goal for a problem of non- benefits.
compliance with the treatment regimen but 4. The client should take the prescribed medica-
not a goal for treating the disease process. tions at the same time each day to provide a
3. This might be a goal for a psychosocial prob- continuous drug level.
lem of social isolation. TEST TAKING HINT: The test taker could elim-
4. The major goal of treating PD is to inate option “2” because hallucinations are
maintain the ability to function. Clients never an expected part of legal medication
diagnosed with PD experience slow, jerky administration.
movements and have difficulty performing Content – Medical: Integrated Nursing Process –
routine daily tasks. Evaluation: Client Needs – Physiological Integrity,
TEST TAKING HINT: The test taker should ­Physiological Adaptation: Cognitive Level – Evaluation:
match the goal to the problem. A “thera- Concept – Neurologic Regulation.
peutic goal” is the key to answering this 81. 1. Crackles and jugular vein distention indicate
question. heart failure, not PD.
Content – Medical: Integrated Nursing Process – 2. Upper extremity weakness and ptosis are clin-
Planning: Client Needs – Safe Effective Care Environment, ical manifestations of myasthenia gravis.
Management of Care: Cognitive Level – Synthesis:
3. The client has very little arm swing, and scan-
Concept – Neurologic Regulation.
ning speech is a clinical manifestation
79. 1. A stereotactic pallidotomy and/or thalamot- of multiple sclerosis.
omy are surgeries that use CT or MRI scans 4. Masklike facies and a shuffling gait are
to localize specific areas of the brain in which two clinical manifestations of PD.
to produce lesions in groups of brain cells TEST TAKING HINT: Option “3” refers to
through electrical stimulation or thermoco- arm swing and speech, both of which are
agulation. These procedures are done when affected by PD. The test taker needs to de-
medication has failed to control tremors. cide if the adjectives used to describe these
2. Dopamine receptor agonists are medications ­activities—“exaggerated” and “scanning”—
that activate the dopamine receptors in the are appropriate. They are not, but masklike
striatum of the brain. facies and shuffling gait are.

02_Colgrove_Ch02.indd 44 02/06/16 7:59 PM


C hapter 2   N eurological D isorders 45

Content – Medical: Integrated Nursing Process – TEST TAKING HINT: At times a psychological
Assessment: Client Needs – Physiological Integrity, Physi- problem can have priority. All the physi-
ological Adaptation: Cognitive Level – Analysis: cal problems are expected and are not life
Concept – Neurologic Regulation. threatening or life altering.
82. 1. Emotional lability is a psychosocial problem, Content – Medical: Integrated Nursing Process –
not a cognitive one. Assessment: Client Needs – Safe Effective Care Environ-
2. Depression is a psychosocial problem. ment, Management of Care: Cognitive Level – Analysis:
3. Memory deficits are cognitive impair- Concept – Neurologic Regulation.
ments. The client may also develop a
dementia.
4. Paranoia is a psychosocial problem.
Substance Abuse
TEST TAKING HINT: The test taker must know 85. 1. This should be done so that appropriate care
the definitions of common medical terms. can be provided, but it is not a priority action.
“Cognitive” refers to mental capacity to 2. This should be done before the client ceases
function. breathing and a cardiac arrest follows, but it is
Content – Medical: Integrated Nursing Process – not the first action.
Assessment: Client Needs – Physiological Integrity, Physi- 3. This would be a good step to take to prepare
ological Adaptation: Cognitive Level – Analysis: for the worst-case scenario, but it can be done
Concept – Neurologic Regulation. last among these answer options.
83. 1. This is information that should be discussed 4. Applying oxygen would be the priority ac-
when filling out an advance directive form. tion for this client. The client’s breathing
A ventilator is used to treat a physiological is slow and shallow. The greater amount
problem. of inhaled oxygen, the better the client’s
2. These are psychosocial manifestations of prognosis.
PD. These should be discussed in the sup- TEST TAKING HINT: When the test taker is de-
port meeting. ciding on a priority, some guidelines should
3. The reduction in the unintentional pill-­ be used. Maslow’s hierarchy of needs places
rolling movement of the hands is controlled oxygen at the top of the priority list.
at times by the medication; this is a physi- Content – Medical: Integrated Nursing Process –
ological problem. Implementation: Client Needs – Safe Effective Care
4. Echolalia is a speech deficit in which the ­Environment, Management of Care: Cognitive Level –
client automatically repeats the words or Synthesis: Concept – Neurologic Regulation.
sentences of another person; this is a physi-
86. 1. No employee of a facility is above ­certain
ological problem.
rules. In a company with a “No Drugs”
TEST TAKING HINT: Psychosocial problems policy, this includes the CEO. This client
should address the client’s feelings or inter- is exhibiting symptoms of cocaine abuse.
actions with another person. 2. The nurse does not have a definitive knowl-
Content – Medical: Integrated Nursing Process – edge that the client is using drugs until a
­Planning: Client Needs – Psychosocial Integrity: positive drug screen result is obtained. If the
Cognitive Level – Synthesis: Concept – Neurologic nurse is not a trained substance abuse coun-
Regulation. selor, this intervention would be out of the
84. 1. A headache of “2” on a 1-to-10 scale is a mild realm of the nurse’s expertise.
headache. 3. The client is the CEO of the facility; only
2. A spinal cord injury at T10 involves deficits the board of directors or parent company is
at approximately the waist area. Inability to above this client in supervisory rank.
move the toes would be expected. 4. Giving an antihistamine is prescribing with-
3. Body image is a concern for clients out a license, and the nurse is obligated to
diagnosed with PD. This client is the one intervene in this situation.
client who is not experiencing expected TEST TAKING HINT: The title of the ­client—
sequelae of the disease. CEO—eliminates option “3.” The nurse has
4. This client is getting better; “resolving” noted a potential illegal situation.
indicates an improvement in the client’s clini- Content – Medical: Integrated Nursing Process –
cal manifestations. Implementation: Client Needs – Safe Effective Care
­Environment, Management of Care: Cognitive Level –
Application: Concept – Addiction.

02_Colgrove_Ch02.indd 45 02/06/16 7:59 PM


46 M ed -S urg S uccess

87. 1. The Health Insurance Portability and suspicions so that the suspicions can be
­Accountability Act (HIPAA) requires that investigated. This is a client safety and
a health-care professional not divulge care concern.
information about one person to an unau- 4. The nurse is obligated to report suspicious
thorized person. behavior to protect the clients the coworker
2. This would be discussing Client B and a is caring for.
violation of HIPAA. TEST TAKING HINT: The test taker can
3. The nurse does not know Client B is eliminate option “4” based on “do noth-
using drugs, so notifying the HCP is not ing.” In this instance, direct confrontation
appropriate. is not recommended, but the nurse must do
4. Client B would require an explanation ­something—namely, report the suspicions
for coming to the clinic, for which, if the to the supervisor or peer review.
nurse has not violated HIPAA, there is no Content – Medical: Integrated Nursing Process –
explanation. Implementation: Client Needs – Safe Effective Care
TEST TAKING HINT: Nurses are required to Environment, Management of Care: Cognitive Level –
practice within the laws of the state and Application: Concept – Communication.
within federal laws. HIPAA is a federal law 90. 1. Insomnia and anxiety are symptoms of al-
and applies to all health-care professionals cohol withdrawal, not Wernicke-Korsakoff
in the United States. Legally the nurse can- syndrome.
not use the information provided by Client 2. Visual and auditory hallucinations are symp-
A, but morally the nurse might try to identify toms of delirium tremens.
behavior in Client B that would warrant the
3. Extreme tremors and agitation are symptoms
nurse’s intervention.
of delirium tremens.
Content – Medical: Integrated Nursing Process – 4. Ataxia, or lack of coordination, and con-
Implementation: Client Needs – Safe Effective Care fabulation, making up elaborate stories to
Environment, Management of Care: Cognitive Level –
explain lapses in memory, are both symp-
Application: Concept – Legal.
toms of Wernicke-Korsakoff syndrome.
88. 1. These reactions are called “flashbacks.” TEST TAKING HINT: The test taker can elimi-
2. Flashback reactions occur after the use of nate options “2” and “3” if the test taker
hallucinogens in which the client relives a knows the symptoms of delirium tremens.
bad episode that occurred while using the Content – Medical: Integrated Nursing Process –
drug. Assessment: Client Needs – Physiological Integrity,
3. The drug is gone from the body, but the Physiological Adaptation: Cognitive Level – Analysis:
mind-altering effects can occur at any time in Concept – Addiction.
the form of memory flashbacks.
4. The client stated that the dreams are causing 91. 1. Thiamine is given in high doses to de-
her distress. She is asking for help with the crease the rebound effect on the nervous
dreams, not planning her suicide. system as it adjusts to the absence of alco-
hol, and a benzodiazepine is given in high
TEST TAKING HINT: The client is 20 years old
doses and titrated down over several days
and took the drug in her teens; drugs do not for the tranquilizing effect to prevent de-
stay in the body for extended periods. This lirium tremens.
eliminates option “3.” 2. The client may have seizures, but Valium
Content – Medical: Integrated Nursing Process – would control this. The client does not need a
Assessment: Client Needs – Physiological Integrity, long-term anticonvulsant medication (Dilan-
­Physiological Adaptation: Cognitive Level – Analysis:
tin), and it is not known that the client needs
Concept – Addiction.
an iron preparation (Feosol). The vitamin
89. 1. The nurse is not the coworker’s supervisor, deficiency associated with delirium tremens is
and confronting the coworker about the sus- lack of thiamine, not iron.
picions could lead to problems if the nurse is 3. Methadone is used for withdrawing clients
not trained to deal with substance abusers. from heroin, and Depakote can be used as a
2. This is circumventing the problem. The co- mood stabilizer in bipolar disorder or as an
worker will find another source of drugs if anticonvulsant.
needed, and it is finding the coworker guilty 4. The client does not need a diuretic, and a
without due process. stimulant would produce an effect opposite to
3. The coworker’s supervisor or peer review what is desired.
committee should be aware of the nurse’s

02_Colgrove_Ch02.indd 46 02/06/16 7:59 PM


C hapter 2   N eurological D isorders 47

TEST TAKING HINT: Option “3” could be elim- Amphetamine use causes tachycardia, vaso-
inated if the test taker knew the treatment constriction, hypertension, and arrhythmias.
for heroin withdrawal, and option “4” could 2. This might be an intervention for a problem
be reasoned out because a stimulant would of altered coping.
produce an undesired effect. 3. This would be an intervention for a problem
Content – Medical: Integrated Nursing Process – of insomnia.
Planning: Client Needs – Physiological Integrity, 4. These are interventions for heart failure.
­Pharmacological and Parenteral Therapies: Cognitive TEST TAKING HINT: The correct answer
Level – Synthesis: Concept – Addiction. must address the problem of cardiovascular
92. 1. Chills, sweats, and gooseflesh occur with compromise, which eliminates options
heroin withdrawal, but seizures do not “2” and “3.”
­usually occur, so seizure precautions are Content – Medical: Integrated Nursing Process –
not necessary. Implementation: Client Needs – Physiological Integrity,
2. Vital signs should be taken more frequently, Physiological Adaptation: Cognitive Level – Application:
every two (2) to four (4) hours, depending on Concept – Addiction.
the client’s condition. 95. 1. This is unrealistic. Most restaurants serve
3. The client should be in an atmosphere some form of alcoholic beverage. It is good
with little stimulation. The client will be advice for the client to try to avoid situations
irritable and fearful. that provide the temptation to use drugs or
4. Heroin is administered intravenously. alcohol again.
Heroin addicts are at high risk for HIV as 2. The client will require a follow-up
a result of shared needles and thus should ­program such as 12-step meetings if the
be tested for HIV. client is not to relapse.
5. The client is withdrawing from heroin, so 3. The nurse does not know that this is true.
providing needles is inappropriate. Providing 4. The client should discuss the history with the
sterile needles to IV drug users is controver- people the client chooses.
sial, but it attempts to decrease the incidence
TEST TAKING HINT: The test taker must no-
of HIV among drug users.
tice descriptive words such as “all” or “do not
TEST TAKING HINT: A “select all that ­apply” go anywhere.” These words or phrases are
question will usually have more than one absolutes that should cause the test taker to
correct answer. One option cannot eliminate eliminate the options containing them.
another. Content – Medical: Integrated Nursing Process –
Content – Medical: Integrated Nursing Process – Planning: Client Needs – Physiological Integrity,
Implementation: Client Needs – Safe Effective Care Physiological Adaptation: Cognitive Level – Analysis:
­Environment, Management of Care: Cognitive Level – Concept – Nursing Roles.
Application: Concept – Addiction.
96. 1. The child will realize the changed behaviors
93. 1. This is a therapeutic response. The spouse is when and if they happen.
not expressing feelings but is ­stating a fact. 2. This could cause problems between the par-
The nurse should address the problem. ent and child.
2. This is a therapeutic response. The spouse is 3. Most coping behaviors are learned from
not expressing feelings but is ­stating a fact. parents and guardians. Children of sub-
The nurse should address the problem. stance abusers tend to cope with life situ-
3. The spouse is not required to give an expla- ations by becoming substance abusers
nation to the nurse. unless taught healthy coping mechanisms.
4. The spouse’s behavior is enabling the cli- 4. Children can be a part of the parent’s recov-
ent to continue to drink until he cannot ery, but this is not the rationale for teaching
function. new coping mechanisms.
TEST TAKING HINT: The stem of the question TEST TAKING HINT: Most parents do not like
did not ask for a therapeutic response but did to be corrected by their child; this could
ask for the nurse’s best response. The best eliminate option “2.” The correct answer
response is to address the problem. must address a reason for teaching new cop-
Content – Medical: Integrated Nursing Process – ing strategies.
Implementation: Client Needs – Psychosocial Integrity: Content – Medical: Integrated Nursing Process –
Cognitive Level – Application: Concept – Addiction. Planning: Client Needs – Psychosocial Integrity:
94. 1. Telemetry and vital signs would be done Cognitive Level – Synthesis: Concept – Nursing Roles.
to monitor cardiovascular compromise.

02_Colgrove_Ch02.indd 47 02/06/16 7:59 PM


48 M ed -S urg S uccess

Amyotrophic Lateral Sclerosis answer being early signs/symptoms. The test


(ALS or Lou Gehrig’s Disease) taker could rule out option “1” because of
atrophy, which is a long-term occurrence;
97. 1. EMG is done to differentiate a neuropathy rule out option “2” because these symptoms
from a myopathy, but it does not confirm will occur as the disease progresses; and rule
ALS. out option “3” because these are late signs/
2. Biopsy confirms changes consistent with symptoms.
atrophy and loss of muscle fiber, both Content – Medical: Integrated Nursing Process –
characteristic of ALS. Assessment: Client Needs – Physiological Integrity, Physi-
ological Adaptation: Cognitive Level – Analysis:
3. CK may or may not be elevated in ALS so it
Concept – Neurologic Regulation.
cannot confirm the diagnosis of ALS.
4. This is done as ALS progresses to determine 100. 1. This is a therapeutic response, but the client
respiratory involvement, but it does not con- is asking for specific information.
firm ALS. 2. About 50% of clients die within two (2)
TEST TAKING HINT: The test taker must be to five (5) years from respiratory failure,
clear as to what the question is asking. The aspiration pneumonia, or another infec-
word “confirm” is the key to answering this tious process.
question correctly. The test taker would need 3. The nurse should allow the client to talk
to know that this disease affects the muscle freely about the disease process and should
tissue to correctly identify the answer. provide educational and emotional support.
Content – Medical: Integrated Nursing Process –
4. This is incorrect information; ALS is a dis-
Diagnosis: Client Needs – Physiological Integrity, ease that results in death within 5 years in
Reduction of Risk Potential: Cognitive Level – Analysis: most cases.
Concept – Neurologic Regulation. TEST TAKING HINT: When the client is ask-
98. 1. Disuse syndrome is associated with com- ing for factual information, the nurse should
plications of bedrest. Clients with ALS provide accurate and truthful information.
cannot move and reposition themselves, This helps foster a trusting client–nurse
and they frequently have altered nutri- relationship. A therapeutic response (option
tional and hydration status. “1”) should be used when the client needs
2. The client does not usually have a change in to ventilate feelings and is not asking spe-
body image. cific questions about the disease process.
3. ALS is a disease affecting the muscles, not the Content – Medical: Integrated Nursing Process –
kidneys or circulatory system. Implementation: Client Needs – Safe Effective Care
Environment, Management of Care: Cognitive Level –
4. ALS is not painful.
Application: Concept – Neurologic Regulation.
TEST TAKING HINT: The test taker would have
to be knowledgeable about ALS to answer 101. 1. Elevating the head of the bed will en-
this question. This disease is chronic and hance lung expansion, but it is not the first
debilitating over time and leads to wasting of intervention.
the muscles. 2. Oxygen should be given immediately
Content – Medical: Integrated Nursing Process –
to help alleviate the difficulty breathing.
­Diagnosis: Client Needs – Physiological Integrity, Remember that oxygenation is a priority.
Physiological Adaptation: Cognitive Level – Analysis: 3. Assessment is the first part of the nursing
Concept – Neurologic Regulation. process and is a priority, but assessment will
not help the client breathe easier.
99. 1. These signs and symptoms occur during 4. This is an appropriate intervention, but
the course of ALS, but they are not early obtaining the pulse oximeter reading will
symptoms. not alleviate the client’s respiratory distress.
2. These signs and symptoms will occur as the
TEST TAKING HINT: The test taker should
disease progresses.
3. These are late signs/symptoms of ALS. not automatically select assessment. Make
4. ALS results from the degeneration and sure that there is not another intervention
demyelination of motor neurons in the that will directly help the client, especially if
spinal cord, which results in paralysis and the client is experiencing a life-threatening
weakness of the muscles. complication.
Content – Medical: Integrated Nursing Process –
TEST TAKING HINT: This is an application
Implementation: Client Needs – Safe Effective Care
question in which the test taker must know ­Environment, Management of Care: Cognitive
that ruling out of ALS would result in the Level – Synthesis: Concept – Neurologic Regulation.

02_Colgrove_Ch02.indd 48 02/06/16 7:59 PM


C hapter 2   N eurological D isorders 49

102. 200 mL/hr. Content – Medical: Integrated Nursing Process –


This is a basic math question. The IV pump is Implementation: Client Needs – Physiological Integrity,
calculated in mL/hr, so the nurse must double Physiological Adaptation: Cognitive Level – Application:
the rate to infuse the IV solution in 30 minutes. Concept – Neurologic Regulation.

TEST TAKING HINT: This is a basic calcula- 105. 1. Contractures can develop within a week
tion that the nurse should be able to make because extensor muscles are weaker
even without a calculator. than flexor muscles. If the client cannot
Content – Medical: Integrated Nursing Process – perform ROM exercises, then the nurse
Implementation: Client Needs – Physiological Integrity, must do it for him—passive ROM.
Pharmacological and Parenteral Therapies: Cognitive 2. The client should maintain a positive
Level – Application: Concept – Medication. nitrogen balance to promote optimal body
functioning.
103. 1. Refusing to turn needs to be addressed by
3. Adequate protein is required to maintain
the nurse, but it is not a priority over a life-
osmotic pressure and prevent edema.
threatening condition.
4. The client is usually on bedrest in the last
2. Nausea needs to be assessed by the nurse,
stages and should be turned and told to
but it is not a priority over an oxygenation
cough and deep breathe more often than
problem.
every shift.
3. A pulse oximeter reading of less
than 93% indicates that the client is TEST TAKING HINT: “Terminal stage” is the
experiencing hypoxemia, which is a key term in the stem that should cause the
life-threatening emergency. This client test taker to look for an option addressing
should be assessed first. immobility issues—option “1.” An interven-
4. The nurse must address the client’s com- tion implemented only once in every shift
plaints, but it is not a priority over a physi- should be eliminated as a possible answer
ological problem. when addressing immobility issues.
TEST TAKING HINT: The test taker should Content – Medical: Integrated Nursing Process –
Implementation: Client Needs – Physiological Integrity,
apply Maslow’s hierarchy of needs, in which
Basic Care and Comfort: Cognitive Level – Application:
oxygenation is a priority. The nurse must
Concept – Neurologic Regulation.
know normal parameters for diagnostic
tools and laboratory data. 106. 1. The son is not sure if he may get ALS,
Content – Medical: Integrated Nursing Process – so this is not an appropriate response.
Assessment: Client Needs – Safe Effective Care Environ- 2. This is incorrect information.
ment, Management of Care: Cognitive Level – Analysis: 3. There is a genetic factor with ALS
Concept – Neurologic Regulation. that is linked to a chromosome 21
defect.
104. 1. With assistance, the client may be able to
4. ALS is not caused by a virus. The exact
stay at home. Therefore, placement in a
etiology is unknown, but studies indicate
long-term care facility should not be dis-
that some environmental factors may lead
cussed until the family can no longer care
to ALS.
for the client in the home.
2. There is no indication that a client with ALS TEST TAKING HINT: This question requires
will need a sigmoid colostomy. knowledge of ALS. There are some ques-
3. A client with ALS usually dies within tions for which test-taking hints are not
five (5) years. Therefore, the nurse available.
should offer the client the opportunity Content – Medical: Integrated Nursing Process –
to determine how he/she wants to die. Implementation: Client Needs – Safe Effective Care
4. ALS affects both upper and lower extremi- ­Environment, Management of Care: Cognitive Level –
ties and leads to a debilitating state, so the Application: Concept – Neurologic Regulation.
client will not be able to transfer into and 107. 1. A residual (aspirated gastric contents) of
operate a wheelchair. greater than 50 to 100 mL indicates that
TEST TAKING HINT: The nurse should always the tube feeding is not being digested
help the client prepare for death in disease and that the feeding should be held.
processes that are terminal and should dis- 2. A soft abdomen is normal; a distended abdo-
cuss advance directives, which include both men would be cause to hold the feeding.
a durable power of attorney for health care 3. Diarrhea is a common complication of tube
and a living will. feedings, but it is not a reason to hold the
feeding.

02_Colgrove_Ch02.indd 49 02/06/16 7:59 PM


50 M ed -S urg S uccess

4. The potassium level is low and needs inter- TEST TAKING HINT: Encephalitis is inflam-
vention, but this would not indicate a need mation of the brain caused by either a hy-
to hold the bolus tube feeding. persensitivity reaction or a postinfectious
TEST TAKING HINT: Knowing normal assess- state in which a virus reproduces in the
ment data would lead the test taker to elimi- brain. Encephalitis can be a life-threatening
nate option “2” as a possible correct answer. disease process. History is vital in the
Diarrhea and hypokalemia would not cause diagnosis.
the client to not receive a feeding. Even if Content – Medical: Integrated Nursing Process –
the test taker did not know what “residual” Assessment: Client Needs – Safe Effective Care Environ-
means, this would be the best option. ment, Management of Care: Cognitive Level – Analysis:
Concept – Assessment.
Content – Medical: Integrated Nursing Process –
Assessment: Client Needs – Physiological Integrity, Phar- 110. 1. Bilateral facial palsies are a common initial
macological and Parenteral Therapies: Cognitive sign and symptom of encephalitis.
Level – Analysis: Concept – Neurologic Regulation. 2. Fever is usually one of the first signs and
108. 1. The medication should be given without symptoms the client experiences.
food at the same time each day. 3. A decrease in the client’s headache does not
2. This medication is not affected by green, indicate that the client’s condition is becom-
leafy vegetables. (The anticoagulant warfarin ing worse and thus does not warrant imme-
[Coumadin] is a well-known medication that diate intervention.
is affected by eating green, leafy vegetables.) 4. The absence of smell and taste indicates
3. This medication is not affected by the sun. that the cranial nerves may be involved.
4. The medication can cause blood dyscra- The client’s condition is becoming more
sias. Therefore, the client is monitored serious.
for liver function, blood count, blood TEST TAKING HINT: This question requires
chemistries, and alkaline phosphatase. the test taker to select an option that in-
The client should report any febrile ill- dicates the disease is progressing and the
ness. This is the first medication devel- client is at risk. Option “3” indicates that
oped to treat ALS. the client is improving, and options “1” and
TEST TAKING HINT: Blood dyscrasias oc- “2” are common early manifestations of the
cur with many medications, and this might disease. The only option that reflects cranial
prompt the test taker to select option “4” nerve involvement, a sign that the client’s
as the correct option. Otherwise, the test condition is becoming worse and requires
taker must be knowledgeable of medication immediate intervention, is option “4.”
administration. Content – Medical: Integrated Nursing Process –
Content – Medical: Integrated Nursing Process – Assessment: Client Needs – Safe Effective Care Environ-
Planning: Client Needs – Physiological Integrity, ment, Management of Care: Cognitive Level – Synthesis:
­Pharmacological and Parenteral Therapies: Cognitive Concept – Assessment.
Level – Synthesis: Concept – Medication. 111. 1. A written consent is given for all invasive
procedures, but this would reflect care
before the lumbar puncture, not after.
Encephalitis 2. This is information that would be shared
with the client about the reason the proce-
109. 1. A complication of immunizations for
dure would be done but not care after.
measles, mumps, and rubella can be
3. The nurse should teach this information
encephalitis.
to prevent the severe, throbbing, “spinal
2. Upper respiratory tract illnesses can be a
headache” caused by the decrease in
precursor to encephalitis.
cerebrospinal fluid.
3. The herpes simplex virus, specifically
4. The client should lie with the head of the
type 1, can lead to encephalitis.
bed flat for four (4) to eight (8) hours after
4. Fungal encephalitis is known to occur in
the lumbar puncture, but this position would
certain regions, and the nurse should as-
not prevent all hematomas.
sess for recent trips to areas where these
fungal spores exist, but the common areas TEST TAKING HINT: When the test taker is
are the southwest United States and central ­trying to eliminate options, any that have
California. absolute words, such as “all,” “never,” and
5. Exposure to spores does not lead to “always,” are usually wrong and can be
encephalitis. eliminated quickly. Rarely is any activity
­always or never done.

02_Colgrove_Ch02.indd 50 02/06/16 7:59 PM


C hapter 2   N eurological D isorders 51

Content – Medical: Integrated Nursing Process – Content – Medical: Integrated Nursing Process –
Planning: Client Needs – Physiological Integrity, Reduc- Implementation: Client Needs – Physiological Integrity,
tion of Risk Potential: Cognitive Level – Synthesis: Physiological Adaptation: Cognitive Level – Application:
Concept – Nursing Roles. Concept – Nursing Roles.

112. 1. Clients diagnosed with encephalitis have 114. 1. This is an important area to assess for neuro­
neurological deficits while the inflammation logical deterioration, but it is not the first
is present. The therapeutic plan is to treat indication of increased intracranial pressure.
the disease process, decrease the edema, 2. This is the most important assessment
and return the client to an optimal level of data. A change in level of consciousness
wellness. is usually the first sign of neurological
2. The client may have short-term memory deterioration.
loss from a previous condition. 3. Seizures can occur with inflammation from
3. Renal function is not affected by encepha- encephalitis, but their occurrence does not
litis. Only immobility would affect this indicate that the client has increased intra-
system. cranial pressure resulting from a worsening
4. There is no reason to apply hydrocortisone condition.
cream for encephalitis. 4. This is important information to assess,
TEST TAKING HINT: The test taker should but changes in vital signs are not the first
look at the option that reflects the body sys- sign and symptom of increased intracranial
tem that is involved with the disease. Refer pressure.
to medical terminology; encephalon means TEST TAKING HINT: The word “first” asks
“the brain.” the test taker to prioritize the interventions.
Content – Medical: Integrated Nursing Process – Usually all the options are interventions
Diagnosis: Client Needs – Physiological Integrity, Physi- that the nurse should do, but the question
ological Adaptation: Cognitive Level – Analysis: implies that the client may be deteriorating.
Concept – Neurologic Regulation. Level of consciousness is the most sensitive
indicator of neurological deficit.
113. 1. Clients with encephalitis should be treated
for the disease process and also to prevent Content – Medical: Integrated Nursing Process –
complications of immobility. Turning the Implementation: Client Needs – Safe Effective Care
­Environment, Management of Care: Cognitive
client will prevent skin breakdown.
Level – Synthesis: Concept – Neurologic Regulation.
2. Increasing fluids helps prevent urinary
tract infections and mobilize secretions 115. 1. Mosquitoes breed in standing water,
in the lungs. even pet dishes and birdbaths. All areas
3. The client would be maintained in a slightly that collect water should be emptied,
elevated position, semi-Fowler’s, for gravity removed, covered, or turned over. Rain
to assist the body in decreasing intracranial gutters should be cleaned.
pressure. 2. Light-colored, long-sleeved, and loose-
4. Immobility causes clients to be at risk for fitting clothing should be worn to avoid
deep vein thrombosis. Therefore, clients mosquito bites.
with encephalitis should be assessed for 3. Insect repellent may irritate the eyes, but it
deep vein thrombosis. should be applied over clothing and on all
5. Immobility causes the gastrointestinal exposed areas.
tract to slow, resulting in constipation. 4. Mosquitoes are more prevalent at dusk,
Clients can have difficulty emptying their dawn, and early evening.
bladders, which can cause retention and TEST TAKING HINT: Words such as “only”
urinary tract infections and stones. As- (option “3”) should clue the test taker to
sessing these systems can identify prob- eliminate that option. Rarely are these ab-
lems early. solute terms correct.
TEST TAKING HINT: Each option should be Content – Medical: Integrated Nursing Process –
read carefully. If the test taker does not read Planning: Client Needs – Health Promotion and
each one carefully, the test taker could miss ­Maintenance: Cognitive Level – Synthesis: Concept –
important words, such as “supine” in option Promoting Health.
“3,” resulting in an incorrect answer. 116. 1. An alteration in body temperature in a client
with West Nile virus would not be the high-
est priority.

02_Colgrove_Ch02.indd 51 02/06/16 7:59 PM


52 M ed -S urg S uccess

2. Altered tissue perfusion would be the TEST TAKING HINT: When reading test ques-
highest priority because it could be life tions, the test taker should pay attention to
threatening. adjectives. In this question, the word “col-
3. A problem of fluid volume excess would not laborative” makes all the options incorrect
apply for the client with West Nile virus. except option “4.” Collaborative interven-
These clients are at risk for fluid volume def- tions require an order from a health-care
icit from nausea, vomiting, and hyperthermia. provider, but the nurse uses judgment and
4. A problem with skin integrity could apply to intuition within the scope of practice.
the client with immobility caused by West Content – Medical: Integrated Nursing Process –
Nile virus, but it would not be the highest Implementation: Client Needs – Safe Effective Care
priority problem. ­Environment, Management of Care: Cognitive Level –
TEST TAKING HINT: When prioritizing client Application: Concept – Neurologic Regulation.
problems, oxygenation is the highest prior- 119. 1. These vital signs are within normal ranges.
ity problem according to Maslow, and tissue The temperature is slightly elevated and
perfusion is oxygenation. may require an antipyretic but not as an
Content – Medical: Integrated Nursing Process – ­immediate need.
Diagnosis: Client Needs – Physiological Integrity, 2. This is a common complaint requiring
Physiological Adaptation: Cognitive Level – Analysis: medication but not immediately.
Concept – Neurologic Regulation. 3. This test is used to differentiate West Nile
117. 1. Clients with West Nile virus should be virus from other types of encephalitis and
continuously assessed for alteration in gas would not require immediate intervention.
exchanges or patterns. Supportive care is given for West Nile virus.
2. A rash will resolve when the disease causing No definitive treatment is available.
the rash is treated. 4. These assessment data may indicate that
3. Hypothermia is not treated with ice packs the client’s condition is deteriorating and
but with warming blankets. require immediate intervention to pre-
4. Lymph glands are edematous early in the vent complications.
disease process. There is no reason to teach TEST TAKING HINT: The word “immediate”
the client to report this condition. means that the nurse must recognize and
TEST TAKING HINT: The test taker needs intervene before complications occur. The
to read words carefully. Prefixes such as test taker should eliminate any option that
“hypo-” and “hyper-” are important in de- contains normal assessment data.
termining if an option is correct. Even if the Content – Medical: Integrated Nursing Process –
test taker did not know if the client is hy- ­Assessment: Client Needs – Safe Effective Care Environ-
pothermic or hyperthermic, “hypo-” means ment, Management of Care: Cognitive Level – Synthesis:
“less than normal” so hypothermia would Concept – Neurologic Regulation.
not be treated with ice packs. A client with 120. 1. Transmission does not occur through expo-
West Nile virus usually has a fever that sure with sneezed or coughed secretions.
should be reduced. Thus, a treatment for 2. The most common transmission of the West
hypothermia is not needed. Nile virus to humans is through the bite of
Content – Medical: Integrated Nursing Process – an infected mosquito.
Planning: Client Needs – Safe Effective Care Environ- 3. The West Nile virus can be transmitted
ment, Management of Care: Cognitive Level – Synthesis: through breast milk, blood products, and
Concept – Nursing Roles. organ transplants. This is a vector-borne
118. 1. This intervention is independent, not disease. It is transmitted to mosquitoes
collaborative. that bite infected birds. The incubation
2. This is an independent nursing intervention. period is around 15 days.
3. Assessment is an independent nursing 4. Maculopapular rashes do not drain. Drain-
intervention. ing is a characteristic of a vesicle.
4. Administering an IV fluid is collabora- TEST TAKING HINT: The test taker should
tive because it requires an order from a eliminate option “2” because of the abso-
health-care provider. It does, however, lute word “only.”
require the nurse to assess the rate, fluid, Content – Medical: Integrated Nursing Process –
and site for complications. Diagnosis: Client Needs – Safe Effective Care Environ-
ment, Management of Care: Cognitive Level – Analysis:
Concept – Neurologic Regulation.

02_Colgrove_Ch02.indd 52 02/06/16 7:59 PM


CONCEPTS
121. 1. The first nursing action is to ensure that the droops. A stands for Arms: if both arms are
client does not sustain further damage to raised, does one drift to the side? S stands for
the spinal cord. The nurse does this by plac- Speech: is it slurred, or strange? And T stands
ing sandbags around the client’s head or by for Time: don’t waste time before calling 911 if
someone has started to show any of these signs.
maintaining the client on a backboard with
The American Stroke Association says that
the head securely affixed to the board. during a stroke, “Time lost is brain lost.” An
2. This will not occur until a full assessment is estimated two million brain cells die every
made and brain death is imminent. minute during a stroke, increasing risk of per-
3. The head of the bed has to be kept flat with manent brain damage, disability and death.
the client’s head stabilized until spinal dam- (Gaitan, 2014)
age has been ruled out.
3. The first nursing intervention is to call
4. The Glasgow Coma Scale is a systematic
the Code STROKE, then the health-care
tool used to assess a client’s neurologi-
­provider would be notified next.
cal status. It gives health-care workers a
4. Having the client swallow could be an as-
standard method to determine the prog-
sessment step but not a glass of water and
ress of a client’s condition.
with standby suction available in case the
5. The client should have an access to
client is unable to swallow.
be able to administer emergency
medications. TEST TAKING HINT: The test taker should
6. In an emergency the nurse must concentrate remember that certain physiological pro-
on the immediate care of the client. If the cesses carry risks that have to be contended
client requires a blood transfusion, it will with. Atrial fibrillation can cause the blood
take time to have the type and crossmatch to become stagnant and coat the atrial inte-
completed. rior surfaces. If this coating of blood breaks
loose, then the result can be an intracranial
TEST TAKING HINT: The nurse must re-
embolus.
member that safety is a priority when car-
ing for clients who are incapable or unable Content – Medical: Integrated Nursing Process –
to protect themselves, as in option “1.” Intervention: Client Needs – Physiological Integrity,
Physiological Adaptation: Cognitive Level – Synthesis:
Option “3” is an appropriate assessment
Concept – Neurologic Regulation.
method for a traumatic brain injury be-
cause the first step in the nursing process is 123. 1. Safety is always a priority intervention when
assessment. working with a client whose physical func-
Content – Medical: Integrated Nursing Process – tioning is impaired or when the client’s cog-
Intervention: Client Needs – Physiological Integrity, nitive judgment is compromised.
Physiological Adaptation: Cognitive Level – Synthesis: 2. The client should receive six (6) small meals
Concept – Neurologic Regulation. each day. The client’s swallowing ability may
be impaired and the client will be unable to
122. 1. This may be needed once the client is sta-
consume the meal before it gets cold. The
ble, but the first action is to get the needed
consistency should be soft to not require
personnel to intervene to prevent lasting
extended chewing.
damage for the client.
3. Home health may be needed but the prior-
2. A Code STROKE (for an RRT related
ity intervention is safety.
to a stroke) has been instituted in most
4. A skin assessment is not priority over keep-
facilities to have personnel to respond
ing the patient safe.
so that there is no delay in initiating
interventions, thus reducing the impact TEST TAKING HINT: The test taker should
of a cerebrovascular accident (stroke) on remember that basic nursing care is ap-
a client. propriate for client protection. Maslow’s
hierarchy of needs lists safety in the second
According to Reuters:
highest priority tier. Physiological needs
The National Stroke Association suggests us- that involve life-threatening or life-altering
ing the word FAST to help recognize the signs complications are the only things that are
of a stroke. F stands for Face: ask the person more important than safety.
to smile, and see whether one side of the face
53

02_Colgrove_Ch02.indd 53 02/06/16 7:59 PM


54 M ed -S urg S uccess

Content – Medical: Integrated Nursing Process – complications are the only things that are
Intervention: Client Needs – Safety: Cognitive more important than safety.
Level – Analysis: Concept – Neurologic Regulation.
Content – Medical: Integrated Nursing Process – I
124. 1. Sometimes a client can temporarily over- ntervention: Client Needs – Safety: Cognitive
come a freezing of motion or a tremor by Level – Analysis: Concept – Neurologic Regulation.
making an intentional movement, but the 126. 1. This is presumably true but it is not an ap-
issue is not enough of the neurotransmit- propriate response to someone with a cogni-
ter, dopamine, in the brain. Concentration tive impairment. Rational thought processes
or focusing will not increase the amount of do not apply.
dopamine available in the brain. 2. The client would become increasingly
2. This is the cause of the tremors, agitated when unable to utilize the phone
cogwheel motion of movement, and and/or be unable to reach the father.
bradykinesia, and so forth. It is also 3. The client is focused on his father. Let-
in layman’s terms that the client can ting the client talk about his father will
understand and provides some measure allow him to focus on his father while
of hope that something can be done distracting him from his impossible-to-
without giving false reassurance. fulfill request.
3. The issue is dopamine. The acetylcholine 4. This is called “passing the buck.” A nurse on
effects are caused by the dopamine not be- an Alzheimer’s unit should be able to assess
ing available to counteract the acetylcholine. and intervene in this type of situation.
4. This is a therapeutic response and the client
TEST TAKING HINT: Arguing with a client
is asking for information.
with cognitive impairments only produces
TEST TAKING HINT: The test taker should frustrations for the client and nurse. The
read the stem of the question carefully and nurse must remember the disease process
determine what the client is requesting. and respond accordingly.
The client is newly diagnosed and wants to Content – Medical: Integrated Nursing Process –
know about the disease. The nurse should Intervention: Client Needs – Psychological Needs:
respond to the client’s question. Cognitive Level – Synthesis: Concept – Cognition.
Content – Medical: Integrated Nursing Process –
Intervention: Client Needs – Physiological Needs: 127. 1. This client is 28 years old and needs to learn
Cognitive Level – Analysis: Concept – Neurologic how to function in the home to be able to
Regulation. manage activities of daily living. The OT
works with clients to help them attain the
125. 1. The client can be up as he/she wishes but highest level of functionality.
the nurse should assess this and determine 2. The PT will work with the client to de-
if the client has the functional ability to be velop upper body strength.
able to accomplish this without assistance. 3. The RD will make sure that nutritional
2. A client with a brain tumor would be at risk needs are being met.
for increased intracranial pressure (ICP). 4. The rehabilitation physician (physiatrist)
Fluids should be limited to decrease the is a rehabilitation specialist and an expert
amount of cerebrospinal fluids produced in bone, muscle, and nerves and treats
by the body. injuries or illnesses that affect how a
3. The client can have the diet of choice. The client moves.
tumor occupies space and increases the 5. The SW can assist the client with finan-
pressure on the brain, which can cause vom- cial matters and can direct the client to
iting. This vomiting is not associated with programs that will help the client to re-
the diet; it is caused by the pressure. ceive training in a skill(s) that will assist
4. Clients with brain issues are at risk in job placement.
for electrical misfiring of the neurons, 6. The RN represents nursing in the case con-
a seizure. The nurse should institute ference, not the PCT.
measures to protect the client during
TEST TAKING HINT: The test taker can elimi-
a seizure.
nate option “6” by understanding the roles
TEST TAKING HINT: The test taker should of the staff members. Nurses are with the
remember that basic nursing care is ap- client 24 hours a day in an inpatient care
propriate for client protection. Maslow’s facility and frequently are the coordinators
hierarchy of needs lists safety in the second of the client’s care. The nurse must know
highest priority tier. Physiological needs which discipline should be consulted.
that involve life-threatening or life-altering

02_Colgrove_Ch02.indd 54 02/06/16 7:59 PM


C hapter 2   N eurological D isorders 55

Content – Mobility: Integrated Nursing Process – intravenous, can mask signs and symp-
Implementation: Client Needs – Physiological and toms of deterioration of the client’s
Psychosocial Integrity, Management of Care: Cognitive status.
Level – Synthesis, Concept – Collaboration.
TEST TAKING HINT: The test taker must
128. 1. The GDS assesses the older client for know the actions of medications in order
depression, not cognitive functioning. to administer them safely. Maslow’s hierar-
2. The SLUMS scale is a measurement tool chy of needs lists safety as a high priority.
for cognitive functioning. Many medications will only be listed on the
3. The MMSE scale is another tool to NCLEX-RN examination using only the
assess cognitive functioning. generic name. Dexamethasone’s trade name
4. There is no MDED scale and, in addition, is Decadron.
depression is not cognitive functioning. Content – Neurologic Regulation: Integrated
5. The FIMS measures how well the client can Nursing Process – Intervention: Client Needs –
perform activities of daily living, not cogni- Pharmacological Therapies: Cognitive Level – Synthesis:
tive functioning. Concept – Comfort.
TEST TAKING HINT: The test taker could 131. 1. This complete blood count has findings that
eliminate options “1,” “4,” and “5” based are within normal limits.
on the word “cognitive” in the stem of the 2. This chemistry report has findings that are
question. The test taker should highlight within normal limits.
any word that gives a clue as to what the 3. This glucose level and Hgb A1C are within
question is asking. Words matter. normal limits.
Content – Cognition: Integrated Nursing Process – 4. This client has abnormal findings which
Implementation: Client Needs – Physiological Integrity, could result in serious issues for the
Management of Care: Cognitive Level – Application: client. The nurse should first initiate
Concept – Neurologic Regulation. seizure precautions, then notify the HCP
129. 1. The priority goal is for the client to main- of the results and discuss the code status
tain functional ability. This improves quality of the client.
and quantity of life. TEST TAKING HINT: The test taker can elimi-
2. Verbalizing feelings is a good goal but feel- nate options “1,” “2,” and “3” based on the
ing will not impact stabilizing the physi- expected results. These values are within
ological deterioration of the client. the ranges listed. The only option with ab-
3. There is no way to measure the client’s normal data is option “4.”
understanding. Content – Medical: Integrated Nursing Process –
4. Having a home health agency does not Implementation: Client Needs – Physiological and
ensure that functional ability is maintained. ­Psychosocial Integrity, Management of Care: Cognitive
TEST TAKING HINT: Using Maslow’s hierar- Level – Synthesis: Concept – Neurologic Regulation.
chy of needs, physiological needs are higher 132. 1. When suctioning a client on a ventilator it
than psychosocial needs, so the test taker is good to hyperventilate the client before
can eliminate option “2.” The nurse cannot suctioning because suctioning the secretions
determine or measure “understanding,” so would also suction the oxygen from the cli-
option “3” can be eliminated. ent. However, suctioning a client who has
Content – Mobility: Integrated Nursing Process – ICP increases the ICP. The nurse should
Planning: Client Needs – Physiological Integrity, attempt to remove the secretions without
­Management of Care: Cognitive Level – Analysis: having to suction the client.
Concept – Neurologic Regulation. 2. The secretions pooling in the back of the
130. 1. Dexamethasone is a steroid medication and throat would not be assessed by listening
is the steroid of choice to reduce cerebral to lung sounds or checking for peripheral
edema. perfusion.
2. An IV of NS at a keep-open rate (25 mL per 3. Secretions can drain if the client is
hour is a keep-open rate) would not be ques- turned to the side unless the secretions
tioned. It is needed for emergency access. are too heavy. The first action is to at-
3. A nicotine patch would be administered if tempt to relieve the situation without
the client is a smoker and unable to smoke increasing the ICP even further.
during hospitalization. 4. If suctioning is absolutely needed, then a
4. A narcotic analgesic is contraindicated minimum of 1 minute is needed between
until it is known that the client is neu- attempts to suction.
rologically stable. Narcotics, especially

02_Colgrove_Ch02.indd 55 02/06/16 7:59 PM


56 M ed -S urg S uccess

TEST TAKING HINT: The test taker can elimi- into the throat and occluding the airway.
nate options or decide between two of the (Padded tongue blades are NOT forced
options based on the fact that options “1” into the mouth because they can break
and “4” both involve suctioning the client. teeth and cause aspiration of the teeth.)
Either the nurse will perform suctioning 5. The client’s head should be protected
or it is contraindicated. Assessment is the from hitting the side rails or other
first step of the nursing process but the test objects.
taker must decide if the nurse is assessing 2. Clothing should be loosened to prevent
the correct situation. Pooled oral secretion airway difficulties.
is not lung sounds. 3. The medications to control the seizures
Content – Neurologic Regulation: Integrated Nursing should be administered to stop the
Process – Implementation: Client Needs – Physiological seizure.
and Psychosocial Integrity, Management of Care: Cogni- 1. Assessment in this instance is last be-
tive Level – Synthesis: Concept – Oxygenation. cause of the crisis that is occurring. The
133. 1. The lowest ranking possible on the GCS nurse should assess the mouth to deter-
is 3. The client would be considered brain mine if the client bit the tongue or buc-
dead, not improving. cal mucosa during the seizure or if teeth
2. The lower the numbers are on the GCS, the were chipped or broken.
worse the client’s functioning; this client is TEST TAKING HINT: Rank order questions
not improving. can be difficult to answer. The test taker
3. This GCS rating indicates the client is the should remember safety. Which interven-
same as one (1) hour ago. tion will keep the client safe the fastest?
4. The GCS rating is going up, which Also important is if in stress, do not assess:
means the client is improving. Perform an intervention.
TEST TAKING HINT: The test taker can Content – Safety: Integrated Nursing Process –
eliminate options “1” and “2” because they Implementation: Client Needs – Physiological and
­Psychosocial Integrity, Management of Care: Cognitive
both present a lower score; both cannot be
Level – Analysis: Concept – Neurologic Regulation.
correct in a multiple-choice question and,
here, both indicate a worsening of the cli- 135. 1. Elevating the head of the bed 30 degrees
ent’s condition. Likewise, option “3” is the will decrease ICP by using gravity to
same score as 1 hour ago, so the test taker drain cerebrospinal fluid.
should be careful to note this. Therefore, 2. Minimizing disturbing the client and
because only one answer can show an im- allowing rest in between activities will
proving condition, the test taker can deduce decrease ICP.
that it is ­option “4.” 3. Suctioning increases ICP and should not be
Content – Neurologic Regulation: Integrated Nursing performed unless absolutely necessary.
Process – Implementation: Client Needs – Physiological 4. Soapsuds enemas increase intra-abdominal
and Psychosocial Integrity, Management of Care: Cogni- pressure, which, in turn, increases ICP.
tive Level – Synthesis: Concept – Neurologic Regulation. 5. Trendelenburg position is head down, feet
up. This would increase ICP.
134. In order of priority: 4, 5, 2, 3, 1.
4. The client should be turned to the side
to prevent the tongue from falling back

02_Colgrove_Ch02.indd 56 02/06/16 7:59 PM


C hapter 2   N eurological D isorders 57

NEUROLOGICAL DISORDERS
COMPREHENSIVE EXAMINATION
1. The client is admitted with a diagnosis of 6. Which assessment data would make the nurse
trigeminal neuralgia. Which assessment data suspect that the client has amyotrophic lateral
would the nurse expect to find in this client? sclerosis?
1. Joint pain of the neck and jaw. 1. History of a cold or gastrointestinal upset in
2. Unconscious grinding of the teeth during the last month.
sleep. 2. Complaints of double vision and drooping
3. Sudden severe unilateral facial pain. eyelids.
4. Progressive loss of calcium in the nasal 3. Fatigue, progressive muscle weakness, and
septum. twitching.
4. Loss of sensation below the level of the
2. The client recently has been diagnosed with
umbilicus.
trigeminal neuralgia. Which intervention is
most important for the nurse to implement 7. The client is scheduled for an MRI of the brain
with the client? to confirm a diagnosis of Creutzfeldt-Jakob
1. Assess the client’s sense of smell and taste. disease. Which intervention should the nurse
2. Teach the client how to care for the eyes. implement prior to the procedure?
3. Instruct the client to have carbamazepine 1. Determine if the client has claustrophobia.
(Tegretol) levels monitored regularly. 2. Obtain a signed informed consent form.
4. Assist the client to identify factors that trigger 3. Determine if the client is allergic to egg yolks.
an attack. 4. Start an intravenous line in both hands.
3. The client comes to the clinic and reports a 8. Which should be the nurse’s first intervention
sudden drooping of the left side of the face and with the client diagnosed with Bell’s palsy?
complains of pain in that area. The nurse notes 1. Explain that this disorder will resolve within
that the client cannot wrinkle the forehead or a month.
close the left eye. Which condition should the 2. Tell the client to apply heat to the involved
nurse suspect? side of the face.
1. Bell’s palsy. 3. Encourage the client to eat a soft diet.
2. Right-sided stroke. 4. Teach the client to protect the affected eye
3. Tetany. from injury.
4. Mononeuropathy.
9. The client asks the nurse, “What causes
4. The client comes to the clinic for treatment of Creutzfeldt-Jakob disease?” Which statement
a dog bite. Which intervention should the clinic would be the nurse’s best response?
nurse implement first? 1. “The person must have been exposed to an
1. Prepare the client for a series of rabies infected prion.”
injections. 2. “It is mad cow disease, and eating
2. Notify the local animal control shelter. contaminated meat is the cause.”
3. Administer a tetanus toxoid in the deltoid. 3. “This disease is caused by a virus that is in
4. Determine if the animal has had its stagnant water.”
vaccinations. 4. “A fungal spore from the lungs infects the
brain tissue.”
5. The client has glossopharyngeal nerve (cranial
nerve IX) paralysis secondary to a stroke. Which 10. The client is diagnosed with Creutzfeldt-Jakob
referral would be most appropriate for this disease. Which referral would be the most
client? appropriate?
1. Hospice nurse. 1. Alzheimer’s Association.
2. Speech therapist. 2. Creutzfeldt-Jakob Disease Foundation.
3. Physical therapist. 3. Hospice care.
4. Occupational therapist. 4. A neurosurgeon.

02_Colgrove_Ch02.indd 57 02/06/16 7:59 PM


58 M ed -S urg S uccess

11. The client is diagnosed with arboviral 16. The client is undergoing post-thrombolytic
encephalitis. Which priority intervention should therapy for a stroke. The health-care provider
the nurse implement? has ordered heparin to be infused at 1,000 units
1. Place the client in strict isolation. per hour. The solution comes as 25,000 units of
2. Administer IV antibiotics. heparin in 500 mL of D5W. At what rate will the
3. Keep the client in the supine position. nurse set the pump? ___________
4. Institute seizure precautions.
17. Which finding is considered to be one of the
12. The client is diagnosed with a brain warning signs of developing Alzheimer’s
abscess. Which sign/symptom is the most disease?
common? 1. Difficulty performing familiar tasks.
1. Projectile vomiting. 2. Problems with orientation to date, time,
2. Disoriented behavior. and place.
3. Headaches, worse in the morning. 3. Having problems focusing on a task.
4. Petit mal seizure activity. 4. Atherosclerotic changes in the vessels.
13. The client diagnosed with a brain abscess has 18. Which information should be shared with the
become lethargic and difficult to arouse. Which client diagnosed with stage I Alzheimer’s disease
intervention should the nurse implement first? who is prescribed donepezil (Aricept),
1. Implement seizure precautions. a cholinesterase inhibitor?
2. Assess the client’s neurological status. 1. The client must continue taking this
3. Close the drapes and darken the room. medication forever to maintain function.
4. Prepare to administer an IV steroid. 2. The drug may delay the progression of the
disease, but it does not cure it.
14. The client is diagnosed with Huntington’s
3. A serum drug level must be obtained monthly
chorea. Which interventions should the
to evaluate for toxicity.
nurse implement with the family? Select all
4. If the client develops any muscle aches, the
that apply.
HCP should be notified.
1. Refer to the Huntington’s Chorea
Foundation. 19. The spouse of a recently retired man tells the
2. Explain the need for the client to wear nurse, “All my husband does is sit around and
football padding. watch television all day long. He is so irritable
3. Discuss how to cope with the client’s and moody. I don’t want to be around him.”
messiness. Which action should the nurse implement?
4. Provide three (3) meals a day and no 1. Encourage the wife to leave the client alone.
between-meal snacks. 2. Tell the wife that he is probably developing
5. Teach the family how to perform chest Alzheimer’s disease.
percussion. 3. Recommend that the client see an HCP for
an antidepressant medication.
15. The nurse is discussing psychosocial
4. Instruct the wife to buy him some arts and
implications of Huntington’s chorea with the
crafts supplies.
adult child of a client diagnosed with the disease.
Which psychosocial intervention should the 20. The nurse in a long-term care facility has
nurse implement? noticed a change in the behavior of one of the
1. Refer the child for genetic counseling as soon clients. The client no longer participates in
as possible. activities and prefers to stay in his room. Which
2. Teach the child to use a warming tray under intervention should the nurse implement first?
the food during meals. 1. Insist that the client go to the dining room
3. Discuss the importance of not abandoning for meals.
the parent. 2. Notify the family of the change in behavior.
4. Allow the child to talk about the fear of 3. Determine if the client wants another
getting the disease. roommate.
4. Complete a Geriatric Depression Scale.

02_Colgrove_Ch02.indd 58 02/06/16 7:59 PM


C hapter 2   N eurological D isorders 59

21. A family member brings the client to the 24. Which intervention has the highest priority
emergency department reporting that the for the client in the emergency department
78-year-old father has suddenly become very who has been in a motorcycle collision with an
confused and thinks he is living in 1942, that he automobile and has a fractured left leg?
has to go to war, and that someone is trying to 1. Assessing the neurological status.
poison him. Which question should the nurse 2. Immobilizing the fractured leg.
ask the family member? 3. Monitoring the client’s output.
1. “Has your father been diagnosed with 4. Starting an 18-gauge saline lock.
dementia?”
25. The nurse writes the nursing diagnosis
2. “What medication has your father taken
“altered body temperature related to damaged
today?”
temperature regulating mechanism” for a client
3. “What have you given him that makes him
with a head injury. Which would be the most
think it’s poison?”
appropriate goal?
4. “Does your father like to watch old movies on
1. Administer acetaminophen (Tylenol) for
television?”
elevated temperature.
22. The student nurse asks the nurse, “Why do 2. The client’s temperature will remain less than
you ask the client to identify how many 100°F.
fingers you have up when the client hit the 3. Maintain the hypothermia blanket at 99°F for
front of the head, not the back?” The nurse 24 hours.
would base the response on which scientific 4. The basal metabolic temperature will
rationale? fluctuate no more than two (2) degrees.
1. This is part of the routine neurological
26. Which potential pituitary complication should
examination.
the nurse assess for in the client diagnosed with
2. This is done to determine if the client has
a traumatic brain injury (TBI)?
diplopia.
1. Diabetes mellitus type 2 (DM 2).
3. This assesses the amount of brain damage.
2. Seizure activity.
4. This is done to indicate if there is a rebound
3. Syndrome of inappropriate antidiuretic
effect on the brain.
hormone (SIADH).
4. Cushing’s disease.

23. The ambulance brings the client with a head injury to the emergency department. The client responds to
painful stimuli by opening the eyes, muttering, and pulling away from the nurse. How would the nurse rate
this client on the Glasgow Coma Scale?
1. 3
2. 8
3. 10
4. 15

Glasgow Coma Scale


Eye Opening (E) Verbal Response (V) Motor Response (M)
4 = Spontaneous 5 = Normal conversation 6 = Normal
3 = To voice 4 = Disoriented conversation 5 = Localizes to pain
2 = To pain 3 = Words, but not coherent 4 = Withdraws to pain
1 = None 2 = No words ... only sounds 3 = Decorticate posture
1 = None 2 = Decerebrate
1 = None
Total = E+V+M

From Teasdale G, Jennett B. Assessment of coma and impaired consciousness. A practical scale. Lancet 1974;2:81–84. Reprinted with
permission.

02_Colgrove_Ch02.indd 59 02/06/16 7:59 PM


60 M ed -S urg S uccess

27. The nurse is discussing seizure prevention with 2. The client diagnosed with meningitis who
a female client who was just diagnosed with complains of photosensitivity.
epilepsy. Which statement indicates the client 3. The client with a brain tumor who has
needs more teaching? projectile vomiting.
1. “I will take calcium supplements daily and 4. The client with epilepsy who complains of
drink milk.” tender gums.
2. “I will see my HCP to have my blood levels
33. The client is reporting neck pain, fever, and a
drawn regularly.”
headache. The nurse elicits a positive Kernig’s
3. “I should not drink any type of alcohol while
sign. Which diagnostic test procedure should
taking the medication.”
the nurse anticipate the HCP ordering to
4. “I am glad that my periods will not affect my
confirm a diagnosis?
epilepsy.”
1. A computed tomography (CT).
28. The unlicensed assistive personnel (UAP) is 2. Blood cultures times two (2).
caring for a client who is having a seizure. 3. Electromyogram (EMG).
Which action by the UAP would warrant 4. Lumbar puncture (LP).
immediate intervention by the nurse?
34. Which behavior is a risk factor for developing
1. The assistant attempts to insert an oral
and spreading bacterial meningitis?
airway.
1. An upper respiratory infection (URI).
2. The assistant turns the client on the right
2. Unprotected sexual intercourse.
side.
3. Chronic alcohol consumption.
3. The assistant has all the side rails padded and
4. Use of tobacco products.
up.
4. The assistant does not leave the client’s 35. Which assessment data should the nurse
bedside. expect to observe for the client diagnosed with
Parkinson’s disease?
29. The nurse is preparing the male client for
1. Ascending paralysis and pain.
an electroencephalogram (EEG). Which
2. Masklike facies and pill rolling.
intervention should the nurse implement?
3. Diplopia and ptosis.
1. Explain that this procedure is not painful.
4. Dysphagia and dysarthria.
2. Premedicate the client with a benzodiazepine
drug. 36. The client diagnosed with Parkinson’s disease
3. Instruct the client to shave all facial hair. is prescribed carbidopa/levodopa (Sinemet).
4. Tell the client it will cause him to see Which intervention should the nurse implement
“floaters.” prior to administering the medication?
1. Discuss how to prevent orthostatic
30. Which assessment data indicate that the
hypotension.
client with a traumatic brain injury (TBI)
2. Take the client’s apical pulse for one (1) full
exhibiting decorticate posturing on admission is
minute.
responding effectively to treatment?
3. Inform the client that this medication is for
1. The client has flaccid paralysis.
short-term use.
2. The client has purposeful movement.
4. Tell the client to take the medication on an
3. The client has decerebrate posturing with
empty stomach.
painful stimuli.
4. The client does not move the extremities. 37. The client diagnosed with amyotrophic
lateral sclerosis (Lou Gehrig’s disease) is
31. The intensive care nurse is caring for the
prescribed medications that require intravenous
client who has had intracranial surgery. Which
access. The HCP has ordered a primary
interventions should the nurse implement?
intravenous line at a keep-vein-open (KVO) rate
Select all that apply.
at 25 mL/hr. The drop factor is
1. Assess for deep vein thrombosis.
10 gtts/mL. At what rate should the nurse set
2. Administer intravenous anticoagulant.
the IV tubing? ___________
3. Monitor intake and output strictly.
4. Apply warm compresses to the eyes. 38. Which intervention should the nurse take with
5. Perform passive range-of-motion exercises. the client recently diagnosed with amyotrophic
lateral sclerosis (Lou Gehrig’s disease)?
32. Which client should the nurse assess first after
1. Discuss a percutaneous gastrostomy tube.
receiving the shift report?
2. Explain how a fistula is accessed.
1. The client diagnosed with a stroke who has
3. Provide an advance directive.
right-sided paralysis.
4. Refer to a physical therapist for leg braces.

02_Colgrove_Ch02.indd 60 02/06/16 7:59 PM


C hapter 2   N eurological D isorders 61

39. The public health nurse is discussing St. Louis 44. Which assessment data would make the nurse
encephalitis with a group in the community. suspect that the client with a C7 spinal cord
Which instruction should the nurse provide injury is experiencing autonomic dysreflexia?
to help prevent an outbreak? 1. Abnormal diaphoresis.
1. Yearly vaccinations for the disease. 2. A severe throbbing headache.
2. Advise that the city should spray for 3. Sudden loss of motor function.
mosquitoes. 4. Spastic skeletal muscle movement.
3. The use of gloves when gardening.
45. The nurse stops at the scene of a motor-vehicle
4. Not going out at night.
accident and provides emergency first aid at the
40. The husband of a client who is an alcoholic scene. Which law protects the nurse as a first
tells the nurse, “I don’t know what to do. I don’t responder?
know how to deal with my wife’s problem.” 1. The First Aid Law.
Which response would be most appropriate by 2. Ombudsman Act.
the nurse? 3. Good Samaritan Act.
1. “It must be difficult. Maybe you should think 4. First Responder Law.
about leaving.”
46. The nurse writes the problem “high risk for
2. “I think you should attend Alcoholics
impaired skin integrity” for the client with an
Anonymous.”
L5-6 spinal cord injury. Which intervention
3. “I think that Alanon might be very helpful for
should the nurse include in the plan of care?
you.”
1. Perform active range-of-motion exercise.
4. “You should not enable your wife’s
2. Massage the legs and trochanters every shift.
alcoholism.”
3. Arrange for a Roho cushion in the wheelchair.
41. The client is brought to the emergency 4. Apply petroleum-based lotion to the
department by the police for public extremities.
disorderliness. The client reports feeling no
47. The nurse is preparing to administer
pain and is unconcerned that the police have
acetaminophen (Tylenol) to a client diagnosed
arrested him. The nurse notes the client has
with a stroke who is complaining of a headache.
epistaxis and nasal congestion. Which substance
Which intervention should the nurse implement
should the nurse suspect the client has abused?
first?
1. Marijuana.
1. Administer the medication in pudding.
2. Heroin.
2. Check the client’s armband.
3. Ecstasy.
3. Crush the tablet and dissolve in juice.
4. Cocaine.
4. Have the client sip some water.
42. The client with a history of migraine headaches
48. The nurse is caring for clients on a medical
comes to the clinic and reports that a migraine is
unit. Which client would be most at risk for
coming because the client is experiencing bright
experiencing a stroke?
spots before the eyes. Which phase of migraine
1. A 92-year-old client who is an alcoholic.
headaches is the client experiencing?
2. A 54-year-old client diagnosed with hepatitis.
1. Prodrome phase.
3. A 60-year-old client who has a Greenfield
2. Aura phase.
filter.
3. Headache phase.
4. A 68-year-old client with chronic atrial
4. Recovery phase.
fibrillation.
43. The client with a history of migraine
49. The charge nurse is making client assignments
headaches comes to the emergency department
for a neuro-medical floor. Which client should
complaining of a migraine headache. Which
be assigned to the most experienced nurse?
collaborative treatment should the nurse
1. The elderly client who is experiencing a
anticipate?
stroke in evolution.
1. Administer an injection of sumatriptan
2. The client diagnosed with a transient
(Imitrex), a triptan.
ischemic attack 48 hours ago.
2. Prepare for a computed tomography (CT) of
3. The client diagnosed with Guillain-Barré
the head.
syndrome who complains of leg pain.
3. Place the client in a quiet room with the
4. The client with Alzheimer’s disease who is
lights off.
wandering in the halls.
4. Administer propranolol (Inderal), a beta
blocker.

02_Colgrove_Ch02.indd 61 02/06/16 7:59 PM


62 M ed -S urg S uccess

50. The nurse is assessing a client who is 51. The nurse arrives at the scene of a motor-
experiencing anosmia on a neurological floor. vehicle accident and the car is leaking gasoline.
Which area should the nurse assess for cranial The client is in the driver’s seat of the car
nerve I that is pertinent to anosmia? complaining of not being able to move the legs.
1. A Which actions should the nurse implement? List
2. B in order of priority.
3. C 1. Move the client safely out of the car.
4. D 2. Assess the client for other injuries.
3. Stabilize the client’s neck.
4. Notify the emergency medical system.
5. Place the client in a functional anatomical
A
position.

D
C B

52. The elderly male client diagnosed with Parkinson’s disease has undergone a minor surgical procedure and
is receiving cefazolin (Ancef) IV piggyback every six (6) hours. Which laboratory data should the nurse
report to the health-care provider?
1.

Complete Blood Count Client Value Normal Values


6
RBCs (3 10 )   6 Male: 4.7–6.1 (3 106)/mm3
Female: 4.2–5.4 (3 106)/mm3
Hemoglobin 16.6 Male: 13.5–17.5 g/dL
Female: 11.5–15.5 g/dL
Hematocrit 48 Male: 40%–52%
Female: 36%–48%
3
Platelets (3 10 )   140 150–400 (3 103)/mm3
WBCs (× 103) 10.3 4.5–11 (3 103)/mm3

2.

Chemistry Client Value Normal Values


Sodium 137 135–145 mEq/L
Potassium 4.2 3.5–5.5 mEq/L
Chloride 103 97–107 mEq/L
CO2 30 22–30 mEq/L

3.

Client Value Normal Values


Glucose 115 76–110 mg/dL
Hemoglobin A1c 6.2 4%–6%

4.

Site Client Culture Sensitivity Resistant


Surgical wound Methicillin-resistant Vancomycin Penicillin
Staphylococcus aureus Cephalosporin
Intravenous line Negative

02_Colgrove_Ch02.indd 62 02/06/16 7:59 PM


NEUROLOGICAL DISORDERS COMPREHENSIVE
EXAMINATION ANSWERS AND RATIONALES

1. 1. Joint pain is usually associated with some type Content – Medical: Integrated Nursing Process –
of arthritis. Assessment: Client Needs – Physiological Integrity,
2. Unconscious grinding of the teeth during Physiological Adaptation: Cognitive Level – Analysis:
sleep is usually associated with temporoman- Concept – Neurologic Regulation.
dibular joint (TMJ) disorder. 4. 1. This may be needed if it is determined the
3. Trigeminal neuralgia affects the 5th cra- dog has not had its shots or if the dog cannot
nial nerve and is characterized by par- be found, but it is not the first intervention.
oxysms of pain in the area innervated by 2. This is an appropriate action if the client does
the three branches of the nerve. The uni- not know who owns the dog so that the dog
lateral nature of the pain is an important can be found and quarantined.
diagnostic characteristic. The disorder is 3. If the client has not had a tetanus booster in
also known as tic douloureux. the last 10 years, one must be administered,
4. The nasal structure is not made up of bone. but it is usually the last action taken before
Content – Medical: Integrated Nursing Process – the client is discharged from the clinic.
Assessment: Client Needs – Physiological Integrity, 4. This is a priority because if the dog has
Physiological Adaptation: Cognitive Level – Analysis: had its vaccinations, the client will not
Concept – Neurologic Regulation. have to undergo a series of very painful
2. 1. The client’s sense of smell and taste are not injections. The nurse must obtain infor-
affected. mation about the dog, which is assessment
2. The cornea is at risk for abrasions because of of the situation.
the twitching, which causes irritation. There- Content – Medical: Integrated Nursing Process –
fore, the nurse must teach the client how to Implementation: Client Needs – Safe Effective
care for the eye, but the most important in- Care Environment, Management of Care: Cognitive
tervention is to prevent the attacks. Level – Synthesis: Concept – Trauma.
3. Tegretol is the treatment of choice for tri- 5. 1. Clients are referred to hospice when there is
geminal neuralgia, but it is not the most im- a life expectancy of less than six (6) months.
portant intervention when the client is first This client has difficulty swallowing, which is
diagnosed with this condition. not life threatening.
4. Stimulating specific areas of the face, 2. Speech therapists address the needs
called trigger zones, many initiate the on- of ­clients who have difficulty with the
set of pain. Therefore, the nurse should ­innervations and musculature of the
help the client identify situations that face and neck. This includes the swallow-
exacerbate the condition, such as chewing ing reflex.
gum, eating, brushing the teeth, or being 3. The physical therapist assists the client to
exposed to a draft of cold air. ambulate and transfer (e.g., from bed to chair)
Content – Medical: Integrated Nursing Process – and with muscle strength training.
Assessment: Client Needs – Safe Effective Care 4. The occupational therapist focuses on cogni-
Environment, Management of Care: Cognitive tive disability and activities of daily living.
Level – Synthesis: Concept – Neurologic Regulation.
Content – Medical: Integrated Nursing Process –
3. 1. Bell’s palsy, called facial paralysis, is a Planning: Client Needs – Safe Effective Care
disorder of the 7th cranial nerve (­facial Environment, Management of Care: Cognitive
nerve) characterized by unilateral ­paralysis Level – Synthesis: Concept – Neurologic Regulation.
of facial muscles. 6. 1. A history of a cold or gastrointestinal upset in
2. These are symptoms of a left-sided stroke. the last month would be assessment data that
3. Tetany is due to low calcium levels. In this would make the nurse suspect Guillain-Barré
disorder, the face twitches when touched; this syndrome.
is known as a positive Chvostek’s sign. 2. Complaints of double vision and droop-
4. Mononeuropathy is limited to a single pe- ing eyelids would make the nurse suspect
ripheral nerve and its branches and occurs ­myasthenia gravis.
because the trunk of the nerve is compressed,
such as in carpal tunnel syndrome.
63

02_Colgrove_Ch02.indd 63 02/06/16 7:59 PM


64 M ed -S urg S uccess

3. Fatigue, progressive muscle weakness, and Content – Medical: Integrated Nursing Process –
twitching are signs of ALS, a progressive Implementation: Client Needs – Safe Effective Care
neurological disease in which there is a Environment, Management of Care: Cognitive
loss of motor neurons. There is no cure, Level – Application: Concept – Neurologic Regulation.
but recently a medication to slow the 10. 1. Creutzfeldt-Jakob disease is not Alzheimer’s
­deterioration of the motor neurons has disease, although the presenting symptoms
been found. may mimic Alzheimer’s disease.
4. Loss of sensation would make the nurse 2. There is no foundation for Creutzfeldt-
­suspect some type of spinal cord injury. Jakob disease, but if there were, the sig-
Content – Medical: Integrated Nursing Process – nificant other would not be referred to this
Assessment: Client Needs – Physiological Integrity, organization because the disease progresses
Reduction of Risk Potential: Cognitive Level – so rapidly that the client would not get any
Analysis: Concept – Neurologic Regulation. benefit from the organization.
7. 1. For an MRI scan, the client is placed in a 3. This disease is usually fatal within a year,
very narrow tube. If the client is claustro- and the symptoms progress rapidly to
phobic, he or she may need medication dementia.
or an open MRI machine may need to be 4. The nurse does not refer the client to a neu-
considered. rosurgeon and the primary HCP would not
2. An MRI scan is not an invasive procedure; refer to a neurosurgeon because there is no
therefore, informed consent is not needed. surgical or medical treatment for this disease.
3. The nurse would need to determine allergies Content – Medical: Integrated Nursing Process –
to shellfish or iodine, not to egg yolks. Planning: Client Needs – Safe Effective Care
4. The client will need one saline lock, not two Environment, Management of Care: Cognitive
intravenous lines. Often the MRI tech is the Level – Synthesis: Concept – Health Care System.
person who inserts the IV line. 11. 1. Arboviral encephalitis is a viral infection
Content – Medical: Integrated Nursing Process – transmitted by mosquito bites, and isolation is
Implementation: Client Needs – Physiological not required.
Integrity, Reduction of Risk Potential: Cognitive 2. Antibiotics are prescribed for bacterial infec-
Level – Application: Concept – Nursing Roles. tions, not viral infections. There is no antivi-
8. 1. This is correct information, but it is not ral medication to treat this disease.
a priority when discussing Bell’s palsy. 3. Keeping the client supine would increase in-
2. Heat will help promote comfort and tracranial pressure, which is a concern when
increase blood flow to the muscles, but safety caring for clients with brain diseases.
of the client’s eye is a priority. 4. Seizure precautions should be instituted
3. The client may have difficulty chewing on the because any inflammation of the brain tis-
affected side, so a soft diet should be sue will put the client at risk for seizures.
encouraged, but it is not a priority teaching. Content – Medical: Integrated Nursing Process –
4. Teaching the client to protect the eye is Implementation: Client Needs – Safe Effective Care
a priority because the eye does not close Environment, Safety and Infection Control: Cognitive
completely and the blink reflex is dimin- Level – Synthesis: Concept – Nursing Roles.
ished, making the eye vulnerable to ­injury. 12. 1. Vomiting may occur, but it is not projectile
The client should wear an eye patch and it is not the most common.
at night and wraparound sunglasses or 2. Disoriented behavior may occur, but it is not
goggles during the day; he or she may also the most common.
need artificial tears. 3. The most common and prevailing symp-
Content – Medical: Integrated Nursing Process – tom of a brain abscess is a headache that
Implementation: Client Needs – Physiological is worse in the morning because of in-
Integrity, Physiological Adaptation: Cognitive Level – creased intracranial pressure as a result of
Synthesis: Concept – Nursing Roles. lying flat (gravity).
9. 1. Would a layperson know what a prion is? 4. The client with a brain abscess may have
This is using medical jargon, which is not the seizure activity, but it is usually tonic-clonic
nurse’s best response. (grand mal) and it is not the most common.
2. This is the cause of this disease and would Content – Medical: Integrated Nursing Process –
be the best response. ­Assessment: Client Needs– Physiological Integrity,
3. A virus is not the cause of Creutzfeldt-Jakob Physiological Adaptation: Cognitive Level – Analysis:
disease. Concept – Neurologic Regulation.
4. Fungal spores do not cause this disease.

02_Colgrove_Ch02.indd 64 02/06/16 7:59 PM


C hapter 2   N eurological D isorders 65

13. 1. This is an appropriate intervention, but it is Content – Medical: Integrated Nursing Process –
not the first. Implementation: Client Needs – Psychosocial Integrity:
2. Remember, assessment is the first step of Cognitive Level – Application: Concept – Neurologic
the nursing process and should be imple- Regulation.
mented first whenever there is a change 16. 20 mL/hr.
in the client’s behavior. To arrive at the answer, the test taker must
3. This helps prevent stimulation that could divide 25,000 units by 500 mL = 50 units
initiate a seizure, but it is not the first in 1 mL. Divide 1,000 units by 50 units =
intervention. 20 mL/hr.
4. Steroids may be administered to clients with Content – Medical: Integrated Nursing Process –
brain abscesses to decrease inflammation, but Implementation: Client Needs – Physiological I­ ntegrity,
assessment is the first intervention. Pharmacological and Parenteral Therapies: Cognitive
Content – Medical: Integrated Nursing Process – Level – Application: Concept – Medication.
Implementation: Client Needs – Safe Effective Care
Environment, Management of Care: Cognitive
17. 1. The client may experience minor difficulty
Level – Synthesis: Concept – Neurologic Regulation. in work or social activities but has adequate
cognitive ability to hide the loss and continue
14. 1. Foundations offer the family and client to function independently.
information about the disease, support 2. Disorientation to time and place is
groups, and up-to-date information on a warning sign.
current research. 3. Not being able to focus on a task is more
2. The use of padding will help prevent likely a sign of attention deficit-­hyperactivity
­injury from the constant movement that disorder.
occurs with this disease. 4. Atherosclerotic changes are not warning
3. The constant movement causes the ­client signs of Alzheimer’s disease. Amyloid protein
to be messy when eating, dressing, or plaques do appear to have something to do
­performing activities of daily living. with the disease, but they are not found until
4. The constant movements expend more calo- autopsy.
ries; therefore, the client should have three Content – Medical: Integrated Nursing Process – ­
(3) meals plus between-meal snacks. Assessment: Client Needs – Physiological Integrity, Physi-
5. The client is at risk for choking; therefore, ological Adaptation: Cognitive Level – Analysis:
teaching the Heimlich maneuver is appropri- Concept – Neurologic Regulation.
ate, but teaching chest percussion is not.
18. 1. This is not a true statement. The client will
Content – Medical: Integrated Nursing Process – ­
no longer be prescribed this medication
Implementation: Client Needs – Safe Effective Care
Environment, Management of Care: Cognitive Level –
as the disease progresses and it becomes
Application: Concept – Nursing Roles. ineffective.
2. This medication does not cure
15. 1. Referring the child is not a psychosocial ­Alzheimer’s disease, and at some point
intervention. The gene that determines if a it will become ineffective as the disease
client has Huntington’s chorea has been iden- progresses.
tified, and genetic counseling could rule out 3. There is no monthly drug level to be
or confirm that the client’s child will ­monitored. Toxicity includes jaundice and
develop Huntington’s chorea. gastrointestinal distress.
2. This is an appropriate intervention, but it is 4. Muscle aches are an adverse effect of the
not a psychosocial intervention. (Read the lipid-lowering medications, not of Aricept.
stem closely.) Content – Medical: Integrated Nursing Process – ­
3. This is placing a lot of responsibility on the Planning: Client Needs – Physiological Integrity,
child concerning the parent’s debilitating, ­Pharmacological and Parenteral Therapies: Cognitive
chronic, and devastating disease. The client Level – Synthesis: Concept – Medication.
may need to be in a long-term care facility,
and the child should not feel guilty if this is 19. 1. If the wife could leave the client alone, she
necessary. would not be sharing her concerns with the
4. The child will develop this disease if he or nurse. The nurse needs to address the wife’s
she inherited the gene. It can be frighten- concerns as well as the husband’s.
ing for children to watch a parent prog- 2. This is not the typical signs/symptoms of
ress through this disease and understand stage I Alzheimer’s disease.
that they too may get it.

02_Colgrove_Ch02.indd 65 02/06/16 7:59 PM


66 M ed -S urg S uccess

3. This behavior indicates the client is de- 4. When the client hits the front of the head,
pressed and should be treated with anti- there is a rebound effect known as “coup-
depressants. A major lifestyle change has contrecoup” in which the brain hits the
occurred, and he may need short-term back of the skull. The occipital lobe is in
medication therapy, depending on how the the back of the head, and an injury to it
client adjusts to retirement. may be manifested by seeing double.
4. The client may not want to participate in arts Content – Medical: Integrated Nursing Process –
and crafts. Assessment: Client Needs – Physiological Integrity,
Content – Medical: Integrated Nursing Process – Reduction of Risk Potential: Cognitive Level –
Planning: Client Needs – Safe Effective Care Environ- Analysis: Concept – Neurologic Regulation.
ment, Management of Care: Cognitive Level – Synthesis:
23. 1. A score of 3 is the lowest score and indicates
Concept – Neurologic Regulation.
deep coma and impending brain death.
20. 1. The nurse cannot insist that the client do 2. A score of 8 indicates severe increased
anything. The nurse can encourage but, intracranial pressure, but with appropri-
remember, this is the client’s home. ate care the client may survive. The nurse
2. The family may need to be notified, but the would rate the client at an 8: 1 for open-
nurse should first assess what is happening ing the eyes; 3 for verbal response; and
that is causing this change in behavior. 4 for motor response.
3. There is nothing that indicates the client is 3. A score of 10 indicates moderately increased
unhappy with the roommate. In fact, the cli- intracranial pressure.
ent wants to stay in the room, which does not 4. A score of 15 is the highest score a client can
indicate a need for a room change. receive, indicating normal function.
4. A change in behavior may indicate de- Content – Medical: Integrated Nursing Process –
pression. The Geriatric Depression Scale Assessment: Client Needs – Physiological Integrity,
measures satisfaction with life’s accom- Reduction of Risk Potential: Cognitive Level –
plishments. The elderly should be in Erik- Analysis: Concept – Neurologic Regulation.
son’s generativity versus stagnation stage
24. 1. Assessment is the first step in the ­nursing
of life.
process, and a client in a ­motorcycle acci-
Content – Medical: Integrated Nursing Process – dent must be assessed for a head injury.
Implementation: Client Needs – Safe Effective Care Envi- 2. Neurological assessment is a priority over a
ronment, Management of Care: Cognitive Level – Analysis:
fractured leg.
Concept – Nursing Roles.
3. The client’s urinary output is not a priority
21. 1. Dementia involves behavior changes that are over an assessment.
irreversible and occur over time. Delirium, 4. An 18-gauge IV access should be started in
however, occurs suddenly (as in this man’s case the client has to go to surgery, but it is
symptom onset), is caused by an acute event, not a priority over an assessment.
and is reversible. Content – Medical: Integrated Nursing Process –
2. Drug toxicity and interactions are com- Implementation: Client Needs – Safe Effective Care
mon causes of delirium in the elderly. Environment, Management of Care: Cognitive
3. This is blaming the family member for the Level – Synthesis: Concept – Trauma.
client’s paranoid ideation. 25. 1. Administering acetaminophen is an interven-
4. Watching old movies on television will not tion, which is not a goal.
cause delirium. 2. This is an appropriate goal. It ­
Content – Medical: Integrated Nursing Process – addresses the client, addresses the
Implementation: Client Needs – Safe Effective Care problem (­temperature elevation), and
Environment, Management of Care: Cognitive
is measurable.
Level – Application: Concept – Neurologic Regulation.
3. Maintaining the blanket temperature is a
22. 1. This is part of the neurological examination, nursing intervention, which should eliminate
but this is not the scientific rationale for why this as a possible answer.
it is done. The nurse must understand what is 4. The basal metabolic temperature is evaluated
being assessed to interpret the data. for a woman trying to get pregnant; it helps
2. Diplopia, double vision, is a sign of head indicate ovulation.
injury, but it is not the scientific rationale. Content – Medical: Integrated Nursing Process –
3. The procedure does assess for brain damage, Planning: Client Needs – Safe Effective Care
but this answer does not explain why. Environment, Management of Care: Cognitive
Level – Synthesis: Concept – Neurologic Regulation.

02_Colgrove_Ch02.indd 66 02/06/16 7:59 PM


C hapter 2   N eurological D isorders 67

26. 1. Diabetes mellitus type 2 is a pancreatic dis- client will need to wash the hair after the
ease that has nothing to do with the pituitary procedure.
gland or head injury. 2. Antianxiety medication would make the client
2. Seizure activity is a possible complication of drowsy and could cause a false EEG reading.
TBI, but it is not a pituitary complication. 3. There is no reason for facial hair to be
3. The pituitary gland produces vasopressin, shaved.
the antidiuretic hormone (ADH), and any 4. This procedure measures the electrical con-
injury that causes increased intracranial ductivity in the brain and does not cause the
pressure will exert pressure on the pitu- client to see “floaters” (spots before the eyes).
itary gland and can cause the syndrome Flashing bright lights may be used in an at-
of inappropriate antidiuretic hormone tempt to evoke a seizure.
(SIADH). Content – Medical: Integrated Nursing Process –
4. Cushing’s disease is caused by an excess pro- Implementation: Client Needs – Physiological
duction of glucocorticoids and mineralocorti- Integrity, Reduction of Risk Potential: Cognitive
coids from the adrenal gland. Level – Application: Concept – Nursing Roles.
Content – Medical: Integrated Nursing Process – 30. 1. Flaccid paralysis indicates a worsening of the
Assessment: Client Needs – Physiological Integrity, increased ICP.
Reduction of Risk Potential: Cognitive Level –
2. Purposeful movement indicates the client
Analysis: Concept – Neurologic Regulation.
is getting better and is responding to the
27. 1. Because of bone loss associated with long- treatment.
term use of anticonvulsants, the client should 3. Decerebrate positioning indicates a worsen-
increase calcium intake to reduce the risk of ing of the increased intracranial pressure.
osteoporosis. 4. This is the same as flaccid paralysis and indi-
2. Anticonvulsant medications have a narrow cates a worsening of the increased intracranial
range of therapeutic value and the levels pressure.
should be checked regularly. Content – Medical: Integrated Nursing Process –
3. Alcohol interferes with anticonvulsant medi- Assessment: Client Needs – Physiological Integrity,
cation and should be avoided. Reduction of Risk Potential: Cognitive Level –
4. Women with epilepsy note an increase Analysis: Concept – Neurologic Regulation.
in the frequency of seizures during men-
31. 1. Assessing for deep vein thrombosis, which
ses. This is thought to be linked to the
is a complication of immobility, would be
increase in sex hormones that alter the
appropriate for this client.
excitability of the neurons in the brain.
2. Anticoagulants may cause bleeding; therefore,
Content – Medical: Integrated Nursing Process – the client who has had surgery would not be
Evaluation: Client Needs – Physiological Integrity, Physi- prescribed this medication.
ological Adaptation: Cognitive Level – Evaluation:
3. Monitoring of intake and output helps
Concept – Nursing Roles.
to detect possible complications of the
28. 1. The nurse must intervene to stop pituitary gland, which include diabetes
the UAP because the client’s jaws are insipidus and syndrome of inappropriate
clenched. Attempting to insert anything antidiuretic hormone (SIADH).
into the mouth could cause injury to the 4. The nurse should apply cool compresses to
client or to the UAP. alleviate periocular edema.
2. Side-lying positions help to prevent aspira- 5. The nurse does not want the client to be
tion and are an appropriate intervention. active and possibly increase intracranial
3. The client’s safety is priority, and this will pressure; therefore, the nurse should
help protect the client from injury. perform passive range-of-motion for the
4. Staying with the client is an appropriate client.
behavior that would not warrant intervention Content – Surgical: Integrated Nursing Process –
by the nurse. Implementation: Client Needs – Safe Effective Care
Content – Medical: Integrated Nursing Process – Environment, Management of Care: Cognitive
Implementation: Client Needs – Safe Effective Care Level – Analysis: Concept – Nursing Roles.
Environment, Management of Care: Cognitive
32. 1. Paralysis is an expected occurrence with a
Level – Synthesis: Concept – Neurologic Regulation.
­client who has had a stroke.
29. 1. This procedure is not painful, although 2. Photosensitivity is an expected sign of
electrodes are attached to the scalp. The meningitis.

02_Colgrove_Ch02.indd 67 02/06/16 7:59 PM


68 M ed -S urg S uccess

3. Projectile vomiting indicates that in- 4. Dysphagia and dysarthria are signs/
creased intracranial pressure is exerting symptoms of myasthenia gravis.
pressure on the vomiting center of the Content – Medical: Integrated Nursing Process –
brain. Assessment: Client Needs – Physiological Integrity,
4. Tender gums could be secondary to medica- Physiological Adaptation: Cognitive Level – Analysis:
tion given for epilepsy. The client may need Concept – Neurologic Regulation.
to see a dentist, but this client does not need 36. 1. Because carbidopa/levodopa has been
to be assessed first. linked to hypotension, teaching a ­client
Content – Medical: Integrated Nursing Process – given the medication ways to help ­prevent
Assessment: Client Needs – Safe Effective Care a drop in blood pressure when standing—
Environment, Management of Care: Cognitive
orthostatic hypotension—­decreases the
Level – Analysis: Concept – Nursing Roles.
risks associated with ­hypotension and
33. 1. The symptoms and a positive Kernig’s sign falling.
suggest meningitis, but a CT scan is not 2. The medication will not cause the heart rate
diagnostic of meningitis. to change, so taking the client’s apical pulse
2. Blood cultures determine septicemia or infec- for one (1) minute is not a priority.
tions of the bloodstream, not meningitis. 3. This medication is prescribed for the client
3. An electromyogram (EMG) evaluates electri- the rest of his or her life unless the medica-
cal conductivity through the muscle. tion stops working or the client experiences
4. The client’s symptoms, along with a adverse side effects.
positive Kernig’s sign, should make the 4. The medication should be administered with
nurse suspect meningitis. The definitive food to help prevent gastrointestinal distress.
diagnostic test for meningitis is a lumbar Content – Medical: Integrated Nursing Process –
puncture to obtain cerebrospinal fluid for Planning: Client Needs – Physiological Integrity,
culture. Pharmacological and Parenteral Therapies: Cognitive
Content – Medical: Integrated Nursing Process – Level – Synthesis: Concept – Medication.
Planning: Client Needs – Physiological Integrity, 37. 4 gtts/min.
Reduction of Risk Potential: Cognitive Level –
The nurse must know the formula for
Synthesis: Concept – Neurologic Regulation.
regulating IV drips: the amount to
34. 1. An upper respiratory infection (URI) is not a infuse (25 mL/hr) times the drop factor
behavior. The question asked which behavior (10 gtts/mL) divided by the minutes. Thus,
was a risk factor, so this option can be ruled 25 × 10 = 250 ÷ 60 = 4.11 or 4 gtts/min.
out. However, a URI is a risk factor for de- Content – Medical: Integrated Nursing Process –
veloping and spreading bacterial meningitis Implementation: Client Needs – Physiological
because of increased droplet production. Integrity, Pharmacological and Parenteral Therapies:
2. Unprotected sexual intercourse is a risk factor Cognitive Level– Application: Concept – Medication.
for sexually transmitted diseases (STDs) but 38. 1. The client was diagnosed recently and at
not for meningitis. some point may need a percutaneous endo-
3. Chronic alcohol consumption can cause pan- scopic gastrostomy (PEG) tube, but it is too
creatitis or hepatitis but not meningitis. early for this discussion.
4. Tobacco use increases respiratory secre- 2. A fistula is used for hemodialysis, and ALS
tions and droplet production and thus is does not cause renal dysfunction.
a risk factor for developing and spreading
3. It is never too early to discuss advance
bacterial meningitis.
directives with a client diagnosed with a
Content – Medical: Integrated Nursing Process – terminal illness.
Assessment: Client Needs – Safe Effective Care 4. A client with ALS does not have leg braces as
Environment, Safety and Infection Control: Cognitive
part of the therapeutic regimen.
Level – Analysis: Concept – Neurologic Regulation.
Content – Medical: Integrated Nursing Process –
35. 1. The spread of pain and paralysis are signs/ Implementation: Client Needs – Safe Effective Care
symptoms of Guillain-Barré syndrome. Environment, Management of Care: Cognitive
2. Masklike facies and pill rolling are signs/ Level – Application: Concept – Neurologic Regulation.
symptoms of Parkinson’s disease, along 39. 1. There is no vaccine for preventing
with cogwheeling, postural instability, and encephalitis.
stooped and shuffling gait. 2. Mosquitoes are the vectors that spread
3. Diplopia and ptosis are signs/symptoms the disease, and spraying to kill mosquito
of myasthenia gravis. larvae will help prevent an outbreak in the
community.

02_Colgrove_Ch02.indd 68 02/06/16 7:59 PM


C hapter 2   N eurological D isorders 69

3. Gloves will not protect a person from being a physician or other health-care provider
bitten by a mosquito. who has prescriptive authority.
4. Mosquitoes are more prevalent at night, but 2. This is a collaborative intervention, but it is
this is an unrealistic intervention and will not not routinely ordered because the client re-
help prevent an outbreak. ports having a history of migraine headaches.
Content – Medical: Integrated Nursing Process – 3. This is an appropriate independent nursing
Planning: Client Needs – Health Promotion and Mainte- intervention.
nance: Cognitive Level – Synthesis: Concept – Neurologic 4. Propranolol is not used for acute migraine
Regulation. headaches; it is prescribed for long-term pro-
phylaxis of migraines, so the nurse should not
40. 1. This advice might be appropriate at some
anticipate its use in this situation.
point from a professional counselor but not
from the nurse. Content – Medical: Integrated Nursing Process –
2. Alcoholics Anonymous is the support group Planning: Client Needs – Safe Effective Care
Environment, Management of Care: Cognitive
that alcoholics—in this case, the wife, not the
Level – Synthesis: Concept – Neurologic Regulation.
husband—should attend.
3. Alanon is the support group for signifi- 44. 1. Sweating is not a sign of autonomic
cant others of alcoholics. Al-A-Teen is for dysreflexia.
teenage children of alcoholics. 2. A throbbing headache is the classic sign of
4. This statement is making a judgment that is autonomic dysreflexia, which is caused by
not given in the stem and is not applicable to a stimulus such as a full bladder.
all husbands of alcoholic wives. 3. Sudden loss of motor function occurs with
Content – Medical: Integrated Nursing Process – the original injury. Autonomic dysreflexia
Implementation: Client Needs – Safe Effective Care does not occur until spinal shock has resolved;
Environment, Management of Care: Cognitive it usually occurs in the rehabilitation phase.
Level – Application: Concept – Neurologic Regulation. 4. Spastic skeletal muscle movement could be
secondary to the reflex arc in lower motor
41. 1. Symptoms of marijuana use are apathy,
neuron injuries.
­delayed time, and not wanting to eat.
2. Heroin symptoms include pupil changes and Content – Medical: Integrated Nursing Process –
respiratory depression. Assessment: Client Needs – Physiological Integrity,
Physiological Adaptation: Cognitive Level – Analysis:
3. Ecstasy is a hallucinogen that is an “upper.”
Concept – Neurologic Regulation.
4. Disorderly behavior and the symptoms of
epistaxis and nasal congestion would make 45. 1. There is no such law known as the First Aid
the nurse suspect cocaine abuse. Law.
Content – Medical: Integrated Nursing Process – 2. The Ombudsman Act addresses many areas,
Evaluation: Client Needs – Safe Effective Care such as advance directives, elderly advocacy,
Environment, Management of Care: Cognitive and several other areas.
Level – Analysis: Concept – Addiction. 3. The Good Samaritan Act protects nurses
from judgment against them when in an
42. 1. The prodrome phase occurs hours to days
emergency situation in which the nurse
before the migraine headache.
is not receiving compensation for the
2. This is the aura phase, which is character-
skills and expertise rendered. The nurse
ized by focal neurological symptoms.
is held to a different standard than a lay-
3. The headache phase occurs when vasodilation
person; the nurse must act as any reason-
occurs in the brain, along with a decline in se-
able and prudent nurse would in the same
rotonin levels, causing a throbbing headache.
situation.
4. The recovery phase is when the pain begins
4. There is no such law known as the First
to gradually subside.
­Responder Law.
Content – Medical: Integrated Nursing Process –
Assessment: Client Needs – Physiological Integrity, Content – Fundamentals: Integrated Nursing
Physiological Adaptation: Cognitive Level – Analysis: Process – Implementation: Client Needs – Safe ­Effective
­Concept – Neurologic Regulation. Care Environment, Management of Care: Cognitive
Level – Knowledge: Concept – Trauma.
43. 1. Sumatriptan is a medication of choice for
46. 1. A patient with an L5-6 spinal cord injury is
migraine headaches. It constricts blood
paralyzed and cannot perform active ROM
vessels and reduces inflammation. The
exercises.
nurse administering the medication is part
2. Massaging bony prominences can cause
of a collaborative effort because the nurse
trauma to the underlying blood vessels and
must act on the order or prescription of
will increase the risk for skin breakdown.

02_Colgrove_Ch02.indd 69 02/06/16 7:59 PM


70 M ed -S urg S uccess

3. The nurse must realize that the client Content – Medical: Integrated Nursing Process –
is at risk for skin breakdown even when Planning: Client Needs – Safe Effective Care
sitting in the chair. A Roho cushion is an Environment, Management of Care: Cognitive
air-filled cushion that provides reduced Level – Synthesis: Concept – Nursing Roles.
pressure on the ischium. 50. 1. The eyes, indicated by A, would be assessed
4. Lotion will not prevent skin breakdown and if checking cranial nerves II, III, IV, or VI.
should be water based, not petroleum based. 2. The tongue located in the mouth, indicated
Content – Medical: Integrated Nursing Process – by B, would be assessed if checking cranial
Implementation: Client Needs – Physiological I­ ntegrity, nerves IX, X, or XII.
Basic Care and Comfort: Cognitive Level – Application: 3. The cheek, indicated by C, would be assessed
Concept – Neurologic Regulation. if checking for cranial nerve V, the trigeminal
47. 1. The medication can be administered in pud- nerve.
ding, but it is not the first intervention. 4. Anosmia, the loss of the sense of smell,
2. The armband should be checked but not would require the nurse to assess for cranial
before determining if the client can swallow. nerve I, the olfactory nerve, indicated by D.
3. Tylenol comes in liquid form, and the nurse Content – Medical: Integrated Nursing Process –
should request this before crushing a very bit- Assessment: Client Needs – Physiological Integrity,
ter tablet. Physiological Adaptation: Cognitive Level – Analysis:
4. Asking the client to sip some water as- ­Concept – Neurologic Regulation.
sesses the client’s ability to swallow, which 51. In order of priority: 3, 2, 1, 5, 4.
is a priority when placing anything in the 3. Stabilizing the client’s neck is a priority
mouth of the client who has had a stroke. action to prevent further injury to the cli-
Content – Medical: Integrated Nursing Process – ent, and it must be done prior to
Implementation: Client Needs – Physiological I­ ntegrity, moving the client.
Pharmacological and Parenteral Therapies: Cognitive
Level – Application: Concept – Nursing Roles. 2. The nurse should assess for any other
injuries prior to moving the client from
48. 1. An alcoholic is not at risk for having a the vehicle.
stroke anymore than someone in the general 1. Because the vehicle is leaking fuel and there
population. is potential for an explosion or fire, the cli-
2. A client with hepatitis is not at risk for having ent should be moved to an area of safety.
a stroke anymore than someone in the gen- 5. Placing the client in a functional anatomi-
eral population. cal position is an attempt to prevent fur-
3. A Greenfield filter is positioned in the infe- ther spinal cord injury.
rior vena cava to prevent an embolism result- 4. Because the vehicle is leaking fuel, the
ing from deep vein thrombosis; these filters priority is to remove the client and then
prevent strokes and pulmonary emboli. obtain emergency medical assistance.
4. A client with atrial fibrillation is at high
Content – Medical: Integrated Nursing Process –
risk to have a stroke and is usually given Intervention: Client Needs – Physiological Integrity:
oral anticoagulants to prevent a stroke. Cognitive Level – Application: Concept – Trauma.
Content – Medical: Integrated Nursing Process –
Assessment: Client Needs – Physiological Integrity, 52. 1. These values are within normal range for a
Reduction of Risk Potential: Cognitive Level – ­Analysis: male client.
Concept – Neurologic Regulation. 2. These values are within normal range.
3. These values are slightly above range for the
49. 1. This client is experiencing a progressing client but not alarmingly so.
stroke, is at risk for dying, and should be 4. The wound has methicillin-resistant
cared for by the most experienced nurse. S. aureus (MRSA) in it, and the client is
2. A TIA by definition lasts less than 24 hours, receiving an antibiotic that the MRSA
so this client should be stable at this time. is resistant to; the nurse should notify
3. Pain is expected in clients with Guillain-Barré the HCP for a medication change to
syndrome, and symptoms are on the lower vancomycin.
half of the body, which does not affect the
Content – Medical: Integrated Nursing Process –
airway. Therefore, a less experienced nurse
Assessment: Client Needs – Physiological Integrity,
could care for this client. Physiological Adaptation: Cognitive Level – Synthesis: Con-
4. The charge nurse could delegate much of the cept – Medication.
care of this client to a UAP.

02_Colgrove_Ch02.indd 70 02/06/16 7:59 PM


Cardiac Disorders
If you have knowledge, let others light their candles in it.
—Margaret Fuller
3
Heart disease is the leading cause of death in the United States. A nurse must have a thor-
ough knowledge of the signs/symptoms of cardiac disorders and of what to expect in assess-
ing and treating clients with heart-related problems.

KEYWORDS ABBREVIATIONS
Atelectasis Activities of daily living (ADLs)
Buccal Angiotensin-converting enzyme (ACE)
Cardiac tamponade Basal metabolic panel (BMP)
Crackles Blood pressure (BP)
Dyspnea Blood urea nitrogen (BUN)
Dysrhythmia B-type natriuretic peptide (BNP)
Eupnea Capillary refill time (CRT)
Exacerbation Cardiopulmonary resuscitation (CPR)
Intermittent claudication Chest x-ray (CXR)
Nocturia Chronic obstructive pulmonary disease (COPD)
Orthostatic hypotension Congestive heart failure (CHF)
Paresthesia Coronary artery disease (CAD)
Petechiae Dyspnea on exertion (DOE)
Pulse oximeter Electrocardiogram (ECG)
Pulsus paradoxus Endotracheal (ET)
Pyrosis Health-care provider (HCP)
Splinter hemorrhages Implantable cardioverter defibrillator (ICD)
Telemetry International normalized ratio (INR)
Intravenous push (IVP)
Jugular vein distention (JVD)
Myocardial infarction (MI)
Nonsteroidal anti-inflammatory drugs (NSAIDs)
Nothing by mouth (NPO)
Partial thromboplastin time (PTT)
Percutaneous transluminal coronary angioplasty
(PTCA)
Premature ventricular contraction (PVC)
Pulmonary embolus (PE)
Rule out (R/O)
Ventilation/perfusion (V/Q)
White blood cells (WBCs)

71

03_Colgrove_Ch03.indd 71 02/06/16 7:53 PM


72 M ed -S urg S uccess

PRACTICE QUESTIONS
Congestive Heart Failure 5. The nurse is assessing the client diagnosed with
congestive heart failure. Which signs/symptoms
1. The client is admitted to the telemetry unit would indicate that the medical treatment has
diagnosed with acute exacerbation of congestive been effective?
heart failure (CHF). Which signs/symptoms 1. The client’s peripheral pitting edema has gone
would the nurse expect to find when assessing this from 3+ to 4+.
client? 2. The client is able to take the radial pulse
1. Apical pulse rate of 110 and 4+ pitting edema accurately.
of feet. 3. The client is able to perform ADLs without
2. Thick white sputum and crackles that clear dyspnea.
with cough. 4. The client has minimal jugular vein distention.
3. The client sleeping with no pillow and eupnea. 6. The nurse is assessing the client diagnosed
4. Radial pulse rate of 90 and CRT less than three with congestive heart failure. Which laboratory
(3) seconds. data would indicate that the client is in severe
2. The nurse is developing a nursing care plan congestive heart failure?
for a client diagnosed with congestive heart 1. An elevated B-type natriuretic peptide (BNP).
failure. A nursing diagnosis of “decreased cardiac 2. An elevated creatine kinase (CK-MB).
output related to inability of the heart to pump 3. A positive D-dimer.
effectively” is written. Which short-term goal 4. A positive ventilation/perfusion (V/Q) scan.
would be best for the client? 7. The health-care provider has ordered an
1. The client will be able to ambulate in the hall angiotensin-converting enzyme (ACE) inhibitor
by date of discharge. for the client diagnosed with congestive heart
2. The client will have an audible S1 and S2 with failure. Which discharge instructions should the
no S3 heard by end of shift. nurse include?
3. The client will turn, cough, and deep breathe 1. Instruct the client to take a cough suppressant
every two (2) hours. if a cough develops.
4. The client will have a SaO2 reading of 98% by 2. Teach the client how to prevent orthostatic
day two (2) of care. hypotension.
3. The nurse is developing a discharge-teaching plan 3. Encourage the client to eat bananas to increase
for the client diagnosed with congestive heart potassium level.
failure. Which interventions should be included in 4. Explain the importance of taking the
the plan? Select all that apply. medication with food.
1. Notify the health-care provider of a weight 8. The nurse on the telemetry unit has just received
gain of more than one (1) pound in a week. the a.m. shift report. Which client should the
2. Teach the client how to count the radial pulse nurse assess first?
when taking digoxin, a cardiac glycoside. 1. The client diagnosed with myocardial
3. Instruct the client to remove the saltshaker infarction who has an audible S3 heart sound.
from the dinner table. 2. The client diagnosed with congestive heart
4. Encourage the client to monitor urine output failure who has 4+ sacral pitting edema.
for change in color to become dark. 3. The client diagnosed with pneumonia who has
5. Discuss the importance of taking the loop a pulse oximeter reading of 94%.
diuretic furosemide at bedtime. 4. The client with chronic renal failure who has
4. The nurse enters the room of the client diagnosed an elevated creatinine level.
with congestive heart failure. The client is lying
in bed gasping for breath, is cool and clammy, and
has buccal cyanosis. Which intervention would
the nurse implement first?
1. Sponge the client’s forehead.
2. Obtain a pulse oximetry reading.
3. Take the client’s vital signs.
4. Assist the client to a sitting position.

03_Colgrove_Ch03.indd 72 02/06/16 7:53 PM


C hapter 3  C ardiac D isorders 73

9. The nurse and an unlicensed assistive personnel Angina/Myocardial Infarction


(UAP) are caring for four clients on a telemetry
unit. Which nursing task would be best for the 13. Which cardiac enzyme would the nurse expect
nurse to delegate to the UAP? to elevate first in a client diagnosed with a
1. Assist the client to go down to the smoking myocardial infarction (MI)?
area for a cigarette. 1. Creatine kinase (CK-MB).
2. Transport the client to the intensive care unit 2. Lactate dehydrogenase (LDH).
(ICU) via a stretcher. 3. Troponin.
3. Provide the client going home discharge- 4. White blood cells (WBCs).
teaching instructions.
4. Help position the client who is having a 14. Along with persistent, crushing chest pain, which
portable x-ray done. signs/symptoms would make the nurse suspect
that the client is experiencing a myocardial
10. The charge nurse is making shift assignments infarction?
for the medical floor. Which client should be 1. Midepigastric pain and pyrosis.
assigned to the most experienced registered 2. Diaphoresis and cool, clammy skin.
nurse? 3. Intermittent claudication and pallor.
1. The client diagnosed with congestive heart 4. Jugular vein distention and dependent edema.
failure who is being discharged in the
morning. 15. The client diagnosed with rule-out myocardial
2. The client who is having frequent incontinent infarction is experiencing chest pain while
liquid bowel movements and vomiting. walking to the bathroom. Which action should
3. The client with an apical pulse rate of 116, a the nurse implement first?
respiratory rate of 26, and a blood pressure of 1. Administer sublingual nitroglycerin.
94/62. 2. Obtain a STAT electrocardiogram (ECG).
4. The client who is complaining of chest pain 3. Have the client sit down immediately.
on inspiration and a nonproductive cough. 4. Assess the client’s vital signs.

11. The client diagnosed with congestive heart 16. The nurse is caring for a client diagnosed with a
failure is complaining of leg cramps at night. myocardial infarction who is experiencing chest
Which nursing interventions should be pain. Which interventions should the nurse
implemented? implement? Select all that apply.
1. Check the client for peripheral edema and 1. Administer morphine intramuscularly.
make sure the client takes a diuretic early in 2. Administer an aspirin orally.
the day. 3. Apply oxygen via a nasal cannula.
2. Monitor the client’s potassium level and assess 4. Place the client in a supine position.
the client’s intake of bananas and orange juice. 5. Administer nitroglycerin subcutaneously.
3. Determine if the client has gained weight and 17. The client who has had a myocardial infarction
instruct the client to keep the legs elevated. is admitted to the telemetry unit from intensive
4. Instruct the client to ambulate frequently and care. Which referral would be most appropriate
perform calf-muscle stretching exercises daily. for the client?
12. The nurse has written an outcome goal 1. Social worker.
“demonstrates tolerance for increased activity” 2. Physical therapy.
for a client diagnosed with congestive heart 3. Cardiac rehabilitation.
failure. Which intervention should the nurse 4. Occupational therapy.
implement to assist the client to achieve this 18. The client is one (1) day postoperative coronary
outcome? artery bypass surgery. The client complains of
1. Measure intake and output. chest pain. Which intervention should the nurse
2. Provide two (2)g sodium diet. implement first?
3. Weigh the client daily. 1. Medicate the client with intravenous
4. Plan for frequent rest periods. morphine.
2. Assess the client’s chest dressing and vital
signs.
3. Encourage the client to turn from side to side.
4. Check the client’s telemetry monitor.

03_Colgrove_Ch03.indd 73 02/06/16 7:53 PM


74 M ed -S urg S uccess

19. The client diagnosed with a myocardial 23. The client diagnosed with a myocardial
infarction is six (6) hours post–right femoral infarction asks the nurse, “Why do I have to
percutaneous transluminal coronary angioplasty rest and take it easy? My chest doesn’t hurt
(PTCA), also known as balloon surgery. Which anymore.” Which statement would be the nurse’s
assessment data would require immediate best response?
intervention by the nurse? 1. “Your heart is damaged and needs about four
1. The client is keeping the affected extremity (4) to six (6) weeks to heal.”
straight. 2. “There is necrotic myocardial tissue that puts
2. The pressure dressing to the right femoral you at risk for dysrhythmias.”
area is intact. 3. “Your doctor has ordered bedrest. Therefore,
3. The client is complaining of numbness in the you must stay in the bed.”
right foot. 4. “Just because your chest doesn’t hurt anymore
4. The client’s right pedal pulse is 3+ and doesn’t mean you are out of danger.”
bounding.
24. The client has just returned from a cardiac
20. The intensive care department nurse is assessing catheterization. Which assessment data would
the client who is 12 hours post–myocardial warrant immediate intervention from the
infarction. The nurse assesses an S3 heart nurse?
sound. Which intervention should the nurse 1. The client’s BP is 110/70 and pulse is 90.
implement? 2. The client’s groin dressing is dry and intact.
1. Notify the health-care provider immediately. 3. The client refuses to keep the leg straight.
2. Elevate the head of the client’s bed. 4. The client denies any numbness and tingling.
3. Document this as a normal and expected
finding.
4. Administer morphine intravenously. Coronary Artery Disease
21. The nurse is administering a calcium channel 25. The male client is diagnosed with coronary
blocker to the client diagnosed with a artery disease (CAD) and is prescribed
myocardial infarction. Which assessment data sublingual nitroglycerin. Which statement
would cause the nurse to question administering indicates the client needs more teaching?
this medication? 1. “I should keep the tablets in the dark-colored
1. The client’s apical pulse is 64. bottle they came in.”
2. The client’s calcium level is elevated. 2. “If the tablets do not burn under my tongue,
3. The client’s telemetry shows occasional PVCs. they are not effective.”
4. The client’s blood pressure is 90/58. 3. “I should keep the bottle with me in my
22. The client diagnosed with a myocardial pocket at all times.”
infarction is on bedrest. The unlicensed assistive 4. “If my chest pain is not gone with one tablet, I
personnel (UAP) is encouraging the client to will go to the ER.”
move the legs. Which action should the nurse 26. The client with coronary artery disease asks
implement? the nurse, “Why do I get chest pain?” Which
1. Instruct the UAP to stop encouraging the leg statement would be the most appropriate
movements. response by the nurse?
2. Report this behavior to the charge nurse as 1. “Chest pain is caused by decreased oxygen to
soon as possible. the heart muscle.”
3. Praise the UAP for encouraging the client to 2. “There is ischemia to the myocardium as a
move the legs. result of hypoxemia.”
4. Take no action concerning the UAP’s 3. “The heart muscle is unable to pump
behavior. effectively to perfuse the body.”
4. “Chest pain occurs when the lungs cannot
adequately oxygenate the blood.”

03_Colgrove_Ch03.indd 74 02/06/16 7:53 PM


C hapter 3  C ardiac D isorders 75

27. The client is scheduled for a right femoral 33. The nurse is discussing angina with a client
cardiac catheterization. Which nursing who is diagnosed with coronary artery disease.
intervention should the nurse implement after Which action should the client take first when
the procedure? experiencing angina?
1. Perform passive range-of-motion exercises. 1. Put a nitroglycerin tablet under the tongue.
2. Assess the client’s neurovascular status. 2. Stop the activity immediately and rest.
3. Keep the client in high Fowler’s position. 3. Document when and what activity caused
4. Assess the gag reflex prior to feeding the angina.
client. 4. Notify the health-care provider immediately.
28. The nurse is preparing to administer a beta 34. The client with coronary artery disease is
blocker to the client diagnosed with coronary prescribed a Holter monitor. Which intervention
artery disease. Which assessment data would should the nurse implement?
cause the nurse to question administering the 1. Instruct the client to keep a diary of activity,
medication? especially when having chest pain.
1. The client has a BP of 110/70. 2. Discuss the need to remove the Holter
2. The client has an apical pulse of 56. monitor during a.m. care and showering.
3. The client is complaining of a headache. 3. Explain that all medications should be
4. The client’s potassium level is 4.5 mEq/L. withheld while wearing a Holter monitor.
4. Teach the client the importance of decreasing
29. Which intervention should the nurse implement
activity while wearing the monitor.
when administering a loop diuretic to a client
diagnosed with coronary artery disease? 35. Which statement by the client diagnosed with
1. Assess the client’s radial pulse. coronary artery disease indicates that the client
2. Assess the client’s serum potassium level. understands the discharge teaching concerning
3. Assess the client’s glucometer reading. diet?
4. Assess the client’s pulse oximeter reading. 1. “I will not eat more than six (6) eggs a week.”
2. “I should bake or grill any meats I eat.”
30. Which client teaching should the nurse
3. “I will drink eight (8) ounces of whole milk a
implement for the client diagnosed with
day.”
coronary artery disease? Select all that apply.
4. “I should not eat any type of pork products.”
1. Encourage a low-fat, low-cholesterol diet.
2. Instruct the client to walk 30 minutes a day. 36. The charge nurse is making assignments for
3. Decrease the salt intake to two (2) g a day. clients on a cardiac unit. Which client should the
4. Refer to a counselor for stress reduction charge nurse assign to a new graduate nurse?
techniques. 1. The 44-year-old client diagnosed with a
5. Teach the client to increase fiber in the diet. myocardial infarction.
2. The 65-year-old client admitted with unstable
31. The elderly client has coronary artery disease.
angina.
Which question should the nurse ask the client
3. The 75-year-old client scheduled for a cardiac
during the client teaching?
catheterization.
1. “Do you have a daily bowel movement?”
4. The 50-year-old client complaining of chest
2. “Do you get yearly chest x-rays (CXRs)?”
pain.
3. “Are you sexually active?”
4. “Have you had any weight change?”
32. The nurse is discussing the importance of Valvular Heart Disease
exercise with the client diagnosed with coronary
artery disease. Which intervention should the 37. A client is being seen in the clinic to rule out
nurse implement? (R/O) mitral valve stenosis. Which assessment
1. Perform isometric exercises daily. data would be most significant?
2. Walk for 15 minutes three (3) times a week. 1. The client complains of shortness of breath
3. Do not walk outside if it is less than 40˚F. when walking.
4. Wear open-toed shoes when ambulating. 2. The client has jugular vein distention and 3+
pedal edema.
3. The client complains of chest pain after eating
a large meal.
4. The client’s liver is enlarged and the abdomen
is edematous.

03_Colgrove_Ch03.indd 75 02/06/16 7:53 PM


76 M ed -S urg S uccess

38. Which assessment data would the nurse expect 44. The client who has just had a percutaneous
to auscultate in the client diagnosed with mitral balloon valvuloplasty is in the recovery room.
valve insufficiency? Which intervention should the Post Anesthesia
1. A loud S1, S2 split, and a mitral opening snap. Care Unit nurse implement?
2. A holosystolic murmur heard best at the 1. Assess the client’s chest tube output.
cardiac apex. 2. Monitor the client’s chest dressing.
3. A midsystolic ejection click or murmur heard 3. Evaluate the client’s endotracheal (ET) lip
at the base. line.
4. A high-pitched sound heard at the third left 4. Keep the client’s affected leg straight.
intercostal space.
45. The client with a mechanical valve replacement
39. The client has just received a mechanical valve asks the nurse, “Why do I have to take
replacement. Which behavior by the client antibiotics before getting my teeth cleaned?”
indicates the client needs more teaching? Which response by the nurse is most
1. The client takes prophylactic antibiotics. appropriate?
2. The client uses a soft-bristle toothbrush. 1. “You are at risk of developing an infection in
3. The client takes an enteric-coated aspirin your heart.”
daily. 2. “Your teeth will not bleed as much if you have
4. The client alternates rest with activity. antibiotics.”
3. “This procedure may cause your valve to
40. The nurse is teaching a class on valve
malfunction.”
replacements. Which statement identifies a
4. “Antibiotics will prevent vegetative growth on
disadvantage of having a biological tissue valve
your valves.”
replacement?
1. The client must take lifetime anticoagulant 46. The client had open-heart surgery to replace
therapy. the mitral valve. Which intervention should the
2. The client’s infections are easier to treat. intensive care unit nurse implement?
3. There is a low incidence of thromboembolism. 1. Restrict the client’s fluids as ordered.
4. The valve has to be replaced frequently. 2. Keep the client in the supine position.
3. Maintain oxygen saturation at 90%.
41. The nurse is preparing to administer warfarin
4. Monitor the total parenteral nutrition.
(Coumadin), an oral anticoagulant, to a client
with a mechanical valve replacement. The 47. Which client would the nurse suspect of having
client’s INR is 2.7. Which action should the a mitral valve prolapse?
nurse implement? 1. A 60-year-old female with congestive heart
1. Administer the medication as ordered. failure.
2. Prepare to administer vitamin K 2. A 23-year-old male with Marfan’s
(AquaMephyton). syndrome.
3. Hold the medication and notify the HCP. 3. An 80-year-old male with atrial fibrillation.
4. Assess the client for abnormal bleeding. 4. A 33-year-old female with Down
syndrome.
42. Which signs/symptoms should the nurse assess
in any client who has a long-term valvular heart 48. The charge nurse is making shift assignments.
disease? Select all that apply. Which client would be most appropriate for the
1. Paroxysmal nocturnal dyspnea. charge nurse to assign to a new graduate who
2. Orthopnea. just completed orientation to the medical floor?
3. Cough. 1. The client admitted for diagnostic tests to
4. Pericardial friction rub. rule out valvular heart disease.
5. Pulsus paradoxus. 2. The client three (3) days post–myocardial
infarction being discharged tomorrow.
43. The client is being evaluated for valvular heart
3. The client exhibiting supraventricular
disease. Which information would be most
tachycardia (SVT) on telemetry.
significant?
4. The client diagnosed with atrial fibrillation
1. The client has a history of coronary artery
who has an INR of five (5).
disease.
2. There is a family history of valvular heart
disease.
3. The client has a history of smoking for 10 years.
4. The client has a history of rheumatic heart
disease.

03_Colgrove_Ch03.indd 76 02/06/16 7:53 PM


C hapter 3  C ardiac D isorders 77

Dysrhythmias and Conduction Problems 54. The client has chronic atrial fibrillation. Which
discharge teaching should the nurse discuss with
49. The telemetry nurse is unable to read the the client?
telemetry monitor at the nurse’s station. 1. Instruct the client to use a soft-bristle
Which intervention should the telemetry nurse toothbrush.
implement first? 2. Discuss the importance of getting a monthly
1. Go to the client’s room to check the client. partial thromboplastin time (PTT).
2. Instruct the primary nurse to assess the client. 3. Teach the client about signs of pacemaker
3. Contact the client on the client call system. malfunction.
4. Request the nursing assistant to take the crash 4. Explain to the client the procedure for
cart to the client’s room. synchronized cardioversion.
50. The client shows ventricular fibrillation on the 55. The client is exhibiting ventricular tachycardia.
telemetry at the nurse’s station. Which action Which intervention should the nurse implement
should the telemetry nurse implement first? first?
1. Administer epinephrine IVP. 1. Administer amiodarone , an antidysrhythmic,
2. Prepare to defibrillate the client. IVP.
3. Call a STAT code. 2. Prepare to defibrillate the client.
4. Start cardiopulmonary resuscitation (CPR). 3. Assess the client’s apical pulse and blood
pressure.
51. The client is experiencing multifocal premature 4. Start basic cardiopulmonary resuscitation.
ventricular contractions. Which antidysrhythmic
medication would the nurse expect the health- 56. The client is in complete heart block. Which
care provider to order for this client? intervention should the nurse implement first?
1. Amiodarone. 1. Prepare to insert a pacemaker.
2. Atropine. 2. Administer atropine, an antidysrhythmic.
3. Digoxin. 3. Obtain a STAT electrocardiogram (ECG).
4. Adenosine. 4. Notify the health-care provider.
52. The client is exhibiting sinus bradycardia, is 57. The client is in ventricular fibrillation. Which
complaining of syncope and weakness, and has interventions should the nurse implement?
a BP of 98/60. Which collaborative treatment Select all that apply.
should the nurse anticipate being implemented? 1. Start cardiopulmonary resuscitation.
1. Administer a thrombolytic medication. 2. Prepare to administer the antidysrhythmic
2. Assess the client’s cardiovascular status. adenosine IVP.
3. Prepare for insertion of a pacemaker. 3. Prepare to defibrillate the client.
4. Obtain a permit for synchronized 4. Bring the crash cart to the bedside.
cardioversion. 5. Prepare to administer the antidysrhythmic
amiodarone IVP.
53. Which intervention should the nurse implement
when defibrillating a client who is in ventricular 58. The client who is one (1) day postoperative
fibrillation? coronary artery bypass surgery is exhibiting
1. Defibrillate the client at 50, 100, and sinus tachycardia. Which intervention should the
200 joules. nurse implement?
2. Do not remove the oxygen source during 1. Assess the apical heart rate for one (1) full
defibrillation. minute.
3. Place petroleum jelly on the defibrillator pads. 2. Notify the client’s cardiac surgeon.
4. Shout “all clear” prior to defibrillating the 3. Prepare the client for synchronized
client. cardioversion.
4. Determine if the client is having pain.
59. The client’s telemetry reading shows a P wave
before each QRS complex and the rate is 78.
Which action should the nurse implement?
1. Document this as normal sinus rhythm.
2. Request a 12-lead electrocardiogram.
3. Prepare to administer the cardiotonic
digoxin PO.
4. Assess the client’s cardiac enzymes.

03_Colgrove_Ch03.indd 77 02/06/16 7:53 PM


78 M ed -S urg S uccess

60. Which client problem has priority for the client 66. The client diagnosed with pericarditis is
with a cardiac dysrhythmia? experiencing cardiac tamponade. Which
1. Alteration in comfort. collaborative intervention should the nurse
2. Decreased cardiac output. anticipate for this client?
3. Impaired gas exchange. 1. Prepare for a pericardiocentesis.
4. Activity intolerance. 2. Request STAT cardiac enzymes.
3. Perform a 12-lead electrocardiogram.
4. Assess the client’s heart and lung sounds.
Inflammatory Cardiac Disorders
67. The female client is diagnosed with rheumatic
61. The client is diagnosed with pericarditis. Which fever and prescribed penicillin, an antibiotic.
are the most common signs/symptoms the Which statement indicates the client needs
nurse would expect to find when assessing the more teaching concerning the discharge
client? teaching?
1. Pulsus paradoxus. 1. “I must take all the prescribed antibiotics.”
2. Complaints of fatigue and arthralgias. 2. “I may get a vaginal yeast infection with
3. Petechiae and splinter hemorrhages. penicillin.”
4. Increased chest pain with inspiration. 3. “I will have no problems as long as I take my
medication.”
62. The client is diagnosed with acute pericarditis. 4. “My throat culture was positive for a
Which sign/symptom warrants immediate streptococcal infection.”
attention by the nurse?
1. Muffled heart sounds. 68. Which potential complication should the nurse
2. Nondistended jugular veins. assess for in the client with infective endocarditis
3. Bounding peripheral pulses. who has embolization of vegetative lesions from
4. Pericardial friction rub. the mitral valve?
1. Pulmonary embolus (PE).
63. The client is admitted to the medical unit to rule 2. Cerebrovascular accident.
out carditis. Which question should the nurse 3. Hemoptysis.
ask the client during the admission interview to 4. Deep vein thrombosis.
support this diagnosis?
1. “Have you had a sore throat in the last 69. Which nursing diagnosis would be priority for
month?” the client diagnosed with myocarditis?
2. “Did you have rheumatic fever as a child?” 1. Anxiety related to possible long-term
3. “Do you have a family history of carditis?” complications.
4. “What over-the-counter (OTC) medications 2. High risk for injury related to antibiotic
do you take?” therapy.
3. Increased cardiac output related to valve
64. The client with pericarditis is prescribed a regurgitation.
nonsteroidal anti-inflammatory drug (NSAID). 4. Activity intolerance related to impaired
Which teaching instruction should the nurse cardiac muscle function.
discuss with the client?
1. Explain the importance of tapering off the 70. The client diagnosed with pericarditis is being
medication. discharged home. Which intervention should the
2. Discuss that the medication will make the nurse include in the discharge teaching?
client drowsy. 1. Be sure to allow for uninterrupted rest and
3. Instruct the client to take the medication with sleep.
food. 2. Refer the client to outpatient occupational
4. Tell the client to take the medication when therapy.
the pain level is around “8.” 3. Maintain oxygen via nasal cannula at two
(2) L/min.
65. The client diagnosed with pericarditis 4. Discuss upcoming valve replacement surgery.
is complaining of increased pain. Which
intervention should the nurse implement first?
1. Administer oxygen via nasal cannula.
2. Evaluate the client’s urinary output.
3. Assess the client for cardiac complications.
4. Encourage the client to use the incentive
spirometer.

03_Colgrove_Ch03.indd 78 02/06/16 7:53 PM


C hapter 3  C ardiac D isorders 79

71. The client has just had a pericardiocentesis. 72. The client with infective endocarditis is admitted
Which interventions should the nurse to the medical department. Which health-care
implement? Select all that apply. provider’s order should be implemented first?
1. Monitor vital signs every 15 minutes for the 1. Administer intravenous antibiotic.
first hour. 2. Obtain blood cultures times two (2).
2. Assess the client’s heart and lung sounds. 3. Schedule an echocardiogram.
3. Record the amount of fluid removed as 4. Encourage bedrest with bathroom privileges.
output.
4. Evaluate the client’s cardiac rhythm.
5. Keep the client in the supine position.

CONCEPTS
The concepts covered in this chapter focus on the cardiac-related areas of nursing; perfusion-related problems of
myocardial infarction, dysrhythmias, heart failure, and hypertension; and the interrelated concepts of the nursing
process and critical thinking. The concept of nursing process is displayed throughout the questions. The concept
of critical thinking is presented in any prioritizing or “first” question.
73. The nurse enters the client’s room and notes 5. Determine if the client is on an antiplatelet or
an unconscious client with an absence of anticoagulant medication.
respirations and no pulse or blood pressure. 6. Assess the client’s neurological status every
The concept of perfusion is identified by the shift and prn.
nurse. Which should the nurse implement
76. The nurse identifies the concept of perfusion for
first?
a client diagnosed with congestive heart failure.
1. Notify the health care provider.
Which assessment data support this concept?
2. Call a rapid response team (RRT).
1. The client has a large abdomen and a positive
3. Determine the telemetry monitor reading.
tympanic wave.
4. Push the Code Blue button.
2. The client has paroxysmal nocturnal dyspnea.
74. The 45-year-old male client diagnosed with 3. The client has 2+ glucose in the urine.
essential hypertension has decided not to take 4. The client has a comorbid condition of
his medications. The client’s BP is 178/94, myocardial infarction.
indicating a perfusion issue. Which question
77. The nurse is caring for a client diagnosed with
should the nurse ask the client first?
coronary artery disease (CAD). Which should
1. “Do you have the money to buy your
the nurse teach the client prior to discharge?
medication?”
1. Carry your nitroglycerin tablets in a brown
2. “Does the medication give unwanted side
bottle.
effects?”
2. Swallow a nitroglycerin tablet at the first sign
3. “Did you quit taking the medications because
of angina.
you don’t feel bad?”
3. If one nitroglycerin tablet does not work in 10
4. “Can you tell me why you stopped taking the
minutes, take another.
medication?”
4. Nitroglycerin tablets have a fruity odor if they
75. The nurse identifies the concept of altered tissue are potent.
perfusion related to a client admitted with atrial
78. The nurse is caring for a client who suddenly
fibrillation. Which interventions should the
complains of crushing substernal chest pain
nurse implement? Select all that apply.
while ambulating in the hall. Which nursing
1. Monitor the client’s blood pressure and apical
action should the nurse implement first?
rate every four (4) hours.
1. Call a Code Blue.
2. Place the client on intake and output every
2. Assess the telemetry reading.
shift.
3. Take the client’s apical pulse.
3. Require the client to sleep with the head of
4. Have the client sit down.
the bed elevated.
4. Teach the patient to perform Buerger Allen
exercises daily.

03_Colgrove_Ch03.indd 79 02/06/16 7:53 PM


80 M ed -S urg S uccess

79. The client’s telemetry reading is below.


Which should the nurse implement?
1. Take the client’s apical pulse and blood pressure.
2. Prepare to administer amiodarone IVPB.
3. Continue to monitor.
4. Place oxygen on the client via a nasal cannula.

80. The nurse is functioning in the role of 83. The nurse is admitting a client diagnosed with
medication nurse during a code. Which should coronary artery disease (CAD) and angina.
the nurse implement when administering Which concept is priority?
amiodarone for ventricular tachycardia? 1. Sleep, rest, activity.
1. Mix the medication in 100 mL of fluid and 2. Comfort.
administer rapidly. 3. Oxygenation.
2. Push the amiodarone directly into the nearest 4. Perfusion.
IV port and raise the arm.
84. The home health nurse is assigned a client
3. Question the physician’s order because it is
diagnosed with heart failure. Which should the
not ACLS recommended.
nurse implement? Select all that apply.
4. Administer via an IV pump based on
1. Request a dietary consult for a sodium-
mg/kg/min.
restricted diet.
81. The client diagnosed with an ST elevation 2. Instruct the client to elevate the feet during
myocardial infarction (STEMI) has developed the day.
2+ edema bilaterally of the lower extremities 3. Teach the client to weigh every morning
and has crackles in all lung fields. Which should wearing the same type of clothing.
the nurse implement first? 4. Assess for edema in dependent areas of the
1. Notify the health care provider (HCP). body.
2. Assess what the client ate at the last meal. 5. Encourage the client to drink at least
3. Request a STAT 12 lead electrocardiogram. 3,000 mL of fluid per day.
4. Administer furosemide IVP. 6. Have the client repeat back instructions to the
nurse.
82. The nurse is administering morning medications
to clients on a telemetry unit. Which medication
would the nurse question?
1. Furosemide IVP to a client with a potassium
level of 3.6 mEq/L.
2. Digoxin orally to a client diagnosed with
rapid atrial fibrillation.
3. Enalapril orally to a client whose BP is 86/64
and apical pulse is 65.
4. Morphine IVP to a client complaining of
chest pain and who is diaphoretic.

03_Colgrove_Ch03.indd 80 02/06/16 7:53 PM


C hapter 3  C ardiac D isorders 81

85. The telemetry monitor tech notifies the nurse of the strip shown below.
Which should the nurse implement first?
1. Instruct the unlicensed assistive personnel (UAP) to check the client.
2. Go to the client’s room and assess the client personally.
3. Have the monitor tech check the client using a different lead.
4. Call for the Code Blue team and perform cardiopulmonary resuscitation.

86. The nurse is working with a group of new 89. The nurse is administering morning
graduates on a medical-surgical unit. Which medications. Which medication should be
should the nurse explain about completing first administered first?
morning rounds on clients? 1. The cardiac glycoside medication, digoxin, to
1. Perform a “down and dirty” assessment on a client diagnosed with heart failure and who
each client soon after receiving report. has 2+ edema of the feet.
2. Determine which client should have a bath 2. The sliding scale insulin to a client with a
and inform the unlicensed assistive personnel. fasting blood glucose of 345 mg/dL who is
3. Give all the clients a wet wash to wash the demanding breakfast.
face and a toothbrush and toothpaste. 3. The loop diuretic, furosemide, to a client with
4. Pick up any paper on the floor and get the room a 24-hour intake of 986 mL and an output of
ready for morning physician rounds. 1,400 mL.
4. The ARB medication to a client whose blood
87. The nurse has received shift report. Which
pressure was reported by the unlicensed
client should the nurse assess first?
assistive personnel as 142/76.
1. The client diagnosed with coronary artery
disease complaining of severe indigestion. 90. The nurse identifies the concept of tissue
2. The client diagnosed with congestive heart perfusion as a client problem. Which is an
failure who has 3+ pitting edema. antecedent of tissue perfusion?
3. The client diagnosed with atrial fibrillation 1. The client has a history of coronary artery
whose apical rate is 110 and irregular. disease (CAD).
4. The client diagnosed with sinus bradycardia 2. The client has a history of diabetes insipidus
who is complaining of being constipated. (DI).
3. The client has a history of chronic obstructive
88. The client diagnosed with a myocardial
pulmonary disease (COPD).
infarction (MI) is being discharged. Which
4. The client has a history of multiple fractures
discharge instruction(s) should the nurse teach
from a motor-vehicle accident.
the client?
1. Call the health care provider (HCP) if any
chest pain happens.
2. Discuss when the client can resume sexual
activity.
3. Explain the pharmacology of nitroglycerin
tablets.
4. Encourage the client to sleep with the head of
the bed elevated.

03_Colgrove_Ch03.indd 81 02/06/16 7:53 PM


PRACTICE QUESTIONS
ANSWERS AND RATIONALES

Congestive Heart Failure taker should look for an answer that addresses
the ability of the heart to pump blood.
1. 1. The client with CHF would exhibit tachy- Content – Medical: Integrated Nursing Process –
cardia (apical pulse rate of 110), dependent Planning: Client Needs – Safe Effective Care Environment,
edema, fatigue, third heart sounds, lung Management of Care: Cognitive Level – Synthesis:
congestion, and change in mental status. Concept – Perfusion.
2. The client with CHF usually has pink frothy 3. 1. The client should notify the HCP of weight
sputum and crackles that do not clear with gain of more than two (2) or three (3) pounds
coughing. in one (1) day.
3. The client with CHF would report sleeping 2. The client should not take digoxin if the
on at least two pillows, if not sleeping in an radial pulse is less than 60.
upright position, and labored breathing, not 3. The client should be on a low-sodium diet
eupnea, which means normal breathing. to prevent water retention.
4. In a client diagnosed with heart failure, the 4. The color of the urine should not change to a
apical pulse, not the radial pulse, is the best dark color; if anything, it might become lighter
place to assess the cardiac status. and the amount will increase with diuretics.
TEST-TAKING HINT: In option “3,” the word 5. Instruct the client to take the diuretic in the
“no” is an absolute term and, usually, ab- morning to prevent nocturia.
solutes, such as “no,” “never,” “always,” and TEST-TAKING HINT: This is an alternative-type
“only,” are incorrect because there is no room question—in this case, “Select all that apply.” If
for any other possible answer. If the test taker the test taker missed this statement, it is pos-
is looking for abnormal data, then the test sible to jump at the first correct answer. This is
taker should exclude the options that have one reason that it is imperative to read all op-
normal values in them, such as eupnea, pulse tions before deciding on the correct one(s). This
rate of 90, and capillary refill time (CRT) less could be a clue to reread the question for clarity.
than three (3) seconds. Another hint that this is an alternative question
Content – Medical: Integrated Nursing Process – is the number of options. The other questions
Assessment: Client Needs – Physiological Integrity, have four potential answers; this one has five.
Physiological Adaptation: Cognitive Level – Analysis: Numbers in an answer option are always im-
Concept – Perfusion. portant. Is one (1) pound enough to indicate a
2. 1. Ambulating in the hall by day of discharge problem that should be brought to the attention
would be a more appropriate goal for an of the health-care provider?
activity-intolerance nursing diagnosis. Content – Medical: Integrated Nursing Process – Planning:
2. Audible S1 and S2 sounds are normal for a Client Needs – Physiological Integrity, Physiological Adaptation:
heart with adequate output. An audible S3 Cognitive Level – Synthesis: Concept – Nursing Roles.
sound might indicate left ventricular fail- 4. 1. Sponging dry the client’s forehead would be
ure, which could be life threatening. appropriate, but it is not the first intervention.
3. This is a nursing intervention, not a short- 2. Obtaining a pulse oximeter reading would be
term goal, for this client. appropriate, but it is not the first intervention.
4. A pulse oximeter reading would be a goal for 3. Taking the vital signs would be appropriate,
impaired gas exchange, not for cardiac output. but it is not the first intervention.
TEST-TAKING HINT: When reading a nursing 4. The nurse must first put the client in a sit-
diagnosis or problem, the test taker must be ting position to decrease the workload of
sure that the answer selected addresses the the heart by decreasing venous return and
problem. An answer option may be appropri- maximizing lung expansion. Then, the nurse
ate care for the disease process but may not could take vital signs and check the pulse ox-
fit with the problem or etiology. Remember, imeter and then sponge the client’s forehead.
when given an etiology in a nursing diagnosis, TEST-TAKING HINT: In a question that asks
the answer will be doing something about the the nurse to set priorities, all the answer op-
problem (etiology). In this question the test tions can be appropriate actions by the nurse
82

03_Colgrove_Ch03.indd 82 02/06/16 7:53 PM


C hapter 3  C ardiac D isorders 83

for a given situation. The test taker should 7. 1. If a cough develops, the client should notify
apply some guidelines or principles, such as the health-care provider because this is an ad-
Maslow’s hierarchy, to determine what will verse reaction and the HCP will discontinue
give the client the most immediate assistance. the medication.
Content – Medical: Integrated Nursing Process – 2. Orthostatic hypotension may occur with
Implementation: Client Needs – Safe Effective Care ACE inhibitors as a result of vasodilation.
Environment, Management of Care: Cognitive Level – Therefore, the nurse should instruct the
Synthesis: Concept – Perfusion. client to rise slowly and sit on the side of
the bed until equilibrium is restored.
5. 1. Pitting edema changing from 3+ to 4+
3. ACE inhibitors may cause the client to retain
­indicates a worsening of the CHF.
potassium; therefore, the client should not in-
2. The client’s ability to take the radial pulse
crease potassium intake.
would evaluate teaching, not medical
4. An ACE inhibitor should be taken one (1) hour
treatment.
before meals or two (2) hours after a meal to
3. Being able to perform activities of daily living
increase absorption of the medication.
(ADLs) without shortness of breath (dys-
pnea) would indicate the ­client’s condition is TEST-TAKING HINT: If the test taker knows that
improving. The client’s heart is a more effec- an ACE inhibitor is also given for hyperten-
tive pump and can oxygenate the body better sion, then looking at answer options referring
without increasing fluid in the lungs. to hypotension would be appropriate.
4. Any jugular vein distention indicates that the Content – Medical: Integrated Nursing Process – Planning:
right side of the heart is failing, which would Client Needs – Physiological Integrity, Pharmacological and
not indicate effective medical treatment. Parenteral Therapies: Cognitive Level – Synthesis: Concept –
Perfusion.
TEST-TAKING HINT: When asked to determine
whether treatment is effective, the test taker 8. 1. An S3 heart sound indicates left ventricu-
must know the signs and symptoms of the dis- lar failure, and the nurse must assess this
ease being treated. An improvement in the signs client first because it is an emergency
and symptoms indicates effective treatment. situation.
Content – Medical: Integrated Nursing Process – 2. The nurse would expect a client with CHF to
Assessment: Client Needs – Physiological Integrity, have sacral edema of 4+; the client with an S3
Reduction of Risk Potential: Cognitive Level – Analysis: would be in a more life-threatening situation.
Concept – Perfusion. 3. A pulse oximeter reading of greater than 93%
is considered normal.
6. 1. BNP is a specific diagnostic test. Levels
4. An elevated creatinine level is expected in a cli-
higher than normal indicate congestive
ent diagnosed with chronic renal failure.
heart failure, with the higher the number,
the more severe the CHF. TEST-TAKING HINT: Because the nurse will be
2. An elevated CK-MB would indicate a myocar- assessing each client, the test taker must de-
dial infarction, not severe CHF. CK-MB is an termine which client is a priority. A general
isoenzyme. guideline for this type of question is for the
3. A positive D-dimer would indicate a pulmo- test taker to ask “Is this within normal lim-
nary embolus. its?” or “Is this expected for the disease pro-
4. A positive ventilation/perfusion (V/Q) scan cess?” If the answer is yes to either question,
(ratio) would indicate a pulmonary embolus. then the test taker can eliminate these options
and look for abnormal data that would make
TEST-TAKING HINT: This question requires the
that client a priority.
test taker to discriminate among CHF, MI,
and PE. If unsure of the answer of this type of Content – Medical: Integrated Nursing Process –
question, the test taker should eliminate any Assessment: Client Needs – Safe Effective Care Environ-
ment, Management of Care: Cognitive Level – Evaluation:
answer options that the test taker knows are
Concept – Perfusion.
wrong. For example, the test taker may not
know about pulmonary embolus but might 9. 1. Allowing the UAP to take a client down to
know that CK-MB data are used to monitor smoke is not cost effective and is not sup-
MI and be able to eliminate option “2” as a portive of the medical treatment regimen that
possibility. Then, there is a 1:3 chance of get- discourages smoking.
ting the correct answer. 2. The client going to the ICU would be unstable,
Content – Medical: Integrated Nursing Process – and the nurse should not delegate to a UAP any
Assessment: Client Needs – Physiological Integrity, nursing task that involves an unstable client.
Reduction of Risk Potential: Cognitive Level – Analysis: 3. The nurse cannot delegate teaching.
Concept – Perfusion.

03_Colgrove_Ch03.indd 83 02/06/16 7:53 PM


84 M ed -S urg S uccess

4. The UAP can assist the x-ray technician return to the central circulation, but these in-
in positioning the client for the portable terventions would not help with leg cramps.
x-ray. This does not require judgment. 4. Ambulating frequently and performing leg-
TEST-TAKING HINT: The test taker must be stretching exercises will not be effective in
knowledgeable about the individual state’s alleviating the leg cramps.
Nurse Practice Act regarding what a nurse TEST-TAKING HINT: The timing “at night” in
may delegate to unlicensed assistive person- this question was not important in answering
nel. Generally, the answer options that re- the question, but it could have made the test
quire higher level of knowledge or ability are taker jump at option “1.” Be sure to read all
reserved for licensed staff. answer ­options before deciding on an answer.
Content – Medical: Integrated Nursing Process – Answering this question correctly requires
Planning: Client Needs – Safe Effective Care Environ- knowledge of the side effects of treatments
ment, Management of Care: Cognitive Level – Synthesis: used for CHF.
Concept – Nursing Roles. Content – Medical: Integrated Nursing Process –
10. 1. This client is stable because discharge is ­Implementation: Client Needs – Physiological Integrity,
­Physiological Adaptation: Cognitive Level – Application:
scheduled for the following day. Therefore,
Concept – Perfusion.
this client does not need to be assigned to the
most experienced registered nurse. 12. 1. Measuring the intake and output is an
2. This client is more in need of custodial nurs- ­appropriate intervention to implement for
ing care than care from the most experienced a client with CHF, but it does not address
registered nurse. Therefore, the charge nurse ­getting the client to tolerate activity.
could assign a less experienced nurse to this 2. Dietary sodium is restricted in clients
client. with CHF, but this is an intervention for
3. This client is exhibiting signs/symptoms ­decreasing fluid volume, not for increasing
of shock, which makes this client the most tolerance for activity.
unstable. An experienced nurse should 3. Daily weighing monitors fluid volume ­status,
care for this client. not activity tolerance.
4. These complaints usually indicate muscular 4. Scheduling activities and rest periods
or pleuritic chest pain; cardiac chest pain does allows the client to participate in his or
not fluctuate with inspiration. This client her own care and addresses the desired
does not require the care of an experienced outcome.
nurse as much as does the client with signs of TEST-TAKING HINT: With questions involving
shock. nursing diagnoses or goals and outcomes,
TEST-TAKING HINT: When deciding on an the test taker should realize that all activi-
answer for this type of question, the test ties referred to in the answer options may be
taker should reason as to which client is appropriate for the disease but may not be
stable and which has a potentially higher specific for the desired outcome.
level of need. Content – Medical: Integrated Nursing Process –
Content – Medical: Integrated Nursing Process – ­Implementation: Client Needs – Safe Effective Care ­
Planning: Client Needs – Safe Effective Care Environ- Environment, Management of Care: Cognitive Level –
ment, Management of Care: Cognitive Level – Synthesis: Application: Concept – Perfusion.
Concept – Perfusion.

11. 1. The client with peripheral edema will experi- Angina/Myocardial Infarction
ence calf tightness but would not have leg
cramping, which is the result of low potas- 13. 1. CK-MB elevates in 12 to 24 hours.
sium levels. The timing of the diuretic will 2. Lactic dehydrogenase (LDH) elevates in
not change the side effect of leg cramping 24 to 36 hours.
resulting from low potassium levels. 3. Troponin is the enzyme that elevates
2. The most probable cause of the leg within 1 to 2 hours.
cramping is potassium excretion as a re- 4. WBCs elevate as a result of necrotic tissue,
sult of diuretic medication. Bananas and but this is not a cardiac enzyme.
orange juice are foods that are high in
TEST-TAKING HINT: The test taker should
potassium.
be aware of the words “cardiac enzyme,”
3. Weight gain is monitored in clients with
which would eliminate option “4” as a pos-
CHF, and elevating the legs would decrease
sible answer. The word in the stem is “first.”
peripheral edema by increasing the rate of

03_Colgrove_Ch03.indd 84 02/06/16 7:53 PM


C hapter 3  C ardiac D isorders 85

This question­­requires the test taker to have Content – Medical: Integrated Nursing Process –
knowledge of ­laboratory values. Implementation: Client Needs – Safe Effective Care
Environment, Management of Care: Cognitive Level –
Content – Medical: Integrated Nursing Process –
Synthesis: Concept – Perfusion.
Assessment: Client Needs – Physiological Integrity,
­Reduction of Risk Potential: Cognitive Level – ­Analysis: 16. 1. Morphine should be administered intrave-
Concept – Perfusion. nously, not intramuscularly.
14. 1. Midepigastric pain would support a d ­ iagnosis 2. Aspirin is an antiplatelet medication and
of peptic ulcer disease; pyrosis is belching. should be administered orally.
2. Diaphoresis (sweating) is a systemic reac- 3. Oxygen will help decrease myocardial
tion to the MI. The body ­vasoconstricts ischemia, thereby decreasing pain.
to shunt blood from the periphery to the 4. The supine position will increase respiratory
trunk of the body; this, in turn, leads to effort, which will increase myocardial oxy-
cold, clammy skin. gen consumption; the client should be in the
3. Intermittent claudication is leg pain semi-Fowler’s position.
­secondary to decreased oxygen to the muscle, 5. Nitroglycerin, a coronary vasodilator, is ad-
and pallor is paleness of the skin as a result of ministered sublingually, not subcutaneously.
decreased blood supply. Neither is an early TEST-TAKING HINT: This is an alternate-type
sign of MI. question that requires the test taker to select
4. Jugular vein distension (JVD) and dependent all options that are applicable. The test taker
edema are signs/symptoms of congestive must identify all correct answer options to
heart failure, not of MI. receive credit for a correct answer; no partial
TEST-TAKING HINT: The stem already ad- credit is given. Remember to read the ques-
dresses chest pain; therefore, the test taker tion carefully—it is not meant to be tricky.
could eliminate option “1” as a possible an- Content – Medical: Integrated Nursing Process –
swer. Intermittent claudication, option “3,” is Implementation: Client Needs – Safe Effective Care
the classic sign of arterial occlusive disease, Environment, Management of Care: Cognitive Level –
and JVD is very specific to congestive heart Analysis: Concept – Perfusion.
failure. The nurse must be able to identify at 17. 1. The social worker addresses financial con-
least two or three signs/symptoms of disease cerns or referrals after discharge, which are
processes. not indicated for this client.
Content – Medical: Integrated Nursing Process – 2. Physical therapy addresses gait problems,
Assessment: Client Needs – Physiological Integrity, lower extremity strength building, and assist-
Reduction of Risk Potential: Cognitive Level – Analysis: ing with transfer, which are not required for
Concept – Perfusion. this client.
15. 1. The nurse must assume the chest pain 3. Cardiac rehabilitation is the most
is secondary to decreased oxygen to the ­appropriate referral. The client can start
myocardium and administer a sublingual rehabilitation in the hospital and then
nitroglycerin tablet, which is a coronary ­attend an outpatient cardiac ­rehabilitation
­vasodilator, but this is not the first action. clinic, which includes progressive exercise,
2. An ECG should be ordered, but it is not the diet teaching, and classes on modifying
first intervention. risk factors.
3. Stopping all activity will decrease the need 4. Occupational therapy assists the client in
of the myocardium for oxygen and may regaining activities of daily living and covers
help decrease the chest pain. mainly fine motor activities.
4. Assessment is often the first nursing interven- TEST-TAKING HINT: The test taker must be
tion, but when the client has chest pain and a familiar with the responsibilities of the other
possible MI, the nurse must first take care of members of the health-care team. If the test
the client. Taking vital signs would not help taker had no idea which would be the most
relieve chest pain. appropriate referral, the word “cardiac,”
TEST-TAKING HINT: Whenever the test taker which means “heart,” should help the test
wants to select an assessment interven- taker in deciding that this is the most sen-
tion, be sure to think about whether that sible option because the client had a myocar-
intervention will help the client, especially dial infarction, a “heart attack.”
if the client is experiencing pain. Do not au- Content – Medical: Integrated Nursing Process –
tomatically select the answer ­option that is Planning: Client Needs – Safe Effective Care Environ-
assessment. ment, Management of Care: Cognitive Level – Synthesis:
Concept – Collaboration.

03_Colgrove_Ch03.indd 85 02/06/16 7:53 PM


86 M ed -S urg S uccess

18. 1. The nurse should medicate the client as 3. This is not a normal finding; it indicates heart
needed, but it is not the first intervention. failure.
2. The nurse must always assess the client to 4. Morphine is administered for chest pain, not
determine if the chest pain that is occur- for heart failure, which is suggested by the S3
ring is expected postoperatively or if it is a sound.
complication of the surgery. TEST-TAKING HINT: There are some situations
3. Turning will help decrease complications in which the nurse must notify the health-
from immobility, such as pneumonia, but it care provider, and the test taker should not
will not help relieve the client’s pain. automatically eliminate this as a possible
4. The nurse, not a machine, should always take correct answer. The test taker must decide if
care of the client. any of the other three options will help cor-
TEST-TAKING HINT: The stem asks the nurse rect a life-threatening complication. Normal
to identify the first intervention that should assessment concepts should help identify the
be implemented. Therefore, the test taker correct option. The normal heart sounds are
should apply the nursing process and select S1 and S2 (“lubb-dupp”); S3 is abnormal.
an assessment intervention. Both options “2” Content – Medical: Integrated Nursing Process –
and “4” ­involve ­assessment, but the nurse— Implementation: Client Needs – Physiological Integrity,
not a machine or ­diagnostic test—should Reduction of Risk Potential: Cognitive Level – Synthesis:
always assess the client. Concept – Perfusion.
Content – Surgical: Integrated Nursing Process – 21. 1. The apical pulse is within normal limits—
Implementation: Client Needs – Physiological Integrity, 60 to 100 beats per minute.
Reduction of Risk Potential: Cognitive Level – Synthesis:
2. The serum calcium level is not monitored
Concept – Assessment.
when calcium channel blockers are given.
19. 1. After PTCA, the client must keep the right 3. Occasional PVCs would not warrant immedi-
leg straight for at least six (6) to eight (8) ate intervention prior to administering this
hours to prevent any arterial bleeding from medication.
the insertion site in the right femoral artery. 4. The client’s blood pressure is low, and a
2. A pressure dressing is applied to the insertion calcium channel blocker could cause the
site to help prevent arterial bleeding. blood pressure to bottom out.
3. Any neurovascular assessment data that TEST-TAKING HINT: The test taker must know
are abnormal require intervention by the when to question administering medications.
nurse; numbness may indicate decreased The test taker is trying to select an option
blood supply to the right foot. that, if the medication is administered, would
4. A bounding pedal pulse indicates that ad- cause serious harm to the client.
equate circulation is getting to the right foot; Content – Medical: Integrated Nursing Process –
therefore, this would not require immediate ­Assessment: Client Needs – Physiological Integrity,
intervention. Pharmacological and Parenteral Therapies: Cognitive
TEST-TAKING HINT: This question requires Level – Analysis: Concept – Medication.
the test taker to identify abnormal, unex- 22. 1. Leg movement is an appropriate action,
pected, or life-threatening data. The nurse and the UAP should not be told to stop
must know that a PTCA is performed by ­encouraging it.
placing a catheter in the femoral artery and 2. This behavior is not unsafe or dangerous and
that internal or external bleeding is the most should not be reported to the charge nurse.
common complication. 3. The nurse should praise and encourage
Content – Surgical: Integrated Nursing Process – UAPs to participate in the client’s care.
Implementation: Client Needs – Safe Effective Care Clients on bedrest are at risk for deep
Environment, Management of Care: Cognitive Level –
vein thrombosis, and moving the legs will
Synthesis: Concept – Perfusion.
help prevent this from occurring.
20. 1. An S3 indicates left ventricular failure and 4. The nurse should praise subordinates for ap-
should be reported to the health-care propriate behavior, especially when it is help-
provider. It is a potential life-threatening ing to prevent life-threatening complications.
complication of a myocardial infarction.
2. Elevating the head of the bed will not do any-
thing to help a failing heart.

03_Colgrove_Ch03.indd 86 02/06/16 7:53 PM


C hapter 3  C ardiac D isorders 87

TEST-TAKING HINT: This is a management may occur. Therefore, the test taker must
question. The test taker must know the chain select an answer option that is abnormal or
of command and when to report behavior. unsafe. In the data listed, there are three
The test taker could eliminate options “1” normal findings and one abnormal finding.
and “2” with the knowledge that moving the Content – Medical: Integrated Nursing Process –
legs is a safe activity for the client. When Assessment: Client Needs – Safe Effective Care
having to choose between options “3” and Environment, Management of Care: Cognitive Level –
“4,” the test taker should select doing some- Synthesis: Concept – Perfusion.
thing positive, instead of taking no ­action.
This is a management concept.
Content – Medical: Integrated Nursing Process –
Coronary Artery Disease
­Implementation: Client Needs – Safe Effective Care
­Environment, Management of Care: Cognitive Level – 25. 1. If the tablets are not kept in a dark bottle,
Application: Concept – Nursing Roles. they will lose their potency.
2. The tablets should burn or sting when put
23. 1. The heart tissue is dead, stress or activity under the tongue.
may cause heart failure, and it does take 3. The client should keep the tablets with him
about six (6) weeks for scar tissue to form. in case of chest pain.
2. The nurse should talk to the client in lay- 4. The client should take one tablet every
person’s terms, not medical terms. Medical five (5) minutes and, if no relief occurs
terminology is a foreign language to most after the third tablet, have someone
clients. drive him to the emergency department
3. This is not answering the client’s question. or call 911.
The nurse should take any opportunity to
TEST-TAKING HINT: This question is an
teach the client.
“­except” question, requiring the test taker to
4. This is a condescending response, and telling
identify which statement indicates the client
the client that he or she is not out of danger
doesn’t understand the teaching. Sometimes
is not an appropriate response.
the test taker could restate the question and
TEST-TAKING HINT: When attempting to think which statement indicates the client
a­ nswer a client’s question, the nurse should understands the teaching.
provide factual information in simple, under- Content – Medical: Integrated Nursing Process –
standable terms. The test taker should select ­Evaluation: Client Needs – Physiological Integrity,
the answer option that provides this type of Pharmacological and Parenteral Therapies: Cognitive
information. Level – Application: Concept – Perfusion.
Content – Medical: Integrated Nursing Process –
­Implementation: Client Needs – Physiological Integrity,
26. 1. This is a correct statement presented
­Physiological Adaptation: Cognitive Level – Application: in layman’s terms. When the coronary
Concept – Nursing Roles. ­arteries cannot supply adequate oxygen to
the heart muscle, there is chest pain.
24. 1. These vital signs are within normal lim- 2. This is the explanation in medical terms that
its and would not require any immediate should not be used when explaining medical
intervention. conditions to a client.
2. The groin dressing should be dry and intact. 3. This explains congestive heart failure but
3. If the client bends the leg, it could cause does not explain why chest pain occurs.
the insertion site to bleed. This is arterial 4. Respiratory compromise occurs when the
blood and the client could bleed to death lungs cannot oxygenate the blood, such as
very quickly, so this requires immediate occurs with altered level of consciousness,
intervention. cyanosis, and increased respiratory rate.
4. The nurse must check the neurovascular
TEST-TAKING HINT: The nurse must select the
­assessment, and paresthesia would warrant
option that best explains the facts in terms
immediate intervention, but no numbness
a client who does not have medical training
and tingling is a good sign.
can understand.
TEST-TAKING HINT: “Warrants immediate in-
Content – Medical: Integrated Nursing Process –
tervention” means the nurse should probably ­Implementation: Client Needs – Physiological Integrity,
notify the health-care provider or do some- ­Physiological Adaptation: Cognitive Level – Application:
thing independently because a complication Concept – Perfusion.

03_Colgrove_Ch03.indd 87 02/06/16 7:53 PM


88 M ed -S urg S uccess

27. 1. The client’s right leg should be kept straight 29. 1. The nurse should always assess the apical (not
to prevent arterial bleeding from the femoral radial) pulse, but the pulse is not affected by a
insertion site for the catheter used to perform loop diuretic.
the catheterization. 2. Loop diuretics cause potassium to be lost
2. The nurse must make sure that blood is in the urine output. Therefore, the nurse
circulating to the right leg, so the client should assess the client’s potassium level, and
should be assessed for pulses, paresthesia, if the client is hypokalemic, the nurse should
paralysis, coldness, and pallor. question administering this medication.
3. The head of the bed should be elevated no 3. The glucometer provides a glucose level,
more than 10 degrees. The client should which is not affected by a loop diuretic.
be kept on bedrest, flat with the affected 4. The pulse oximeter reading evaluates periph-
­extremity straight, to help decrease the eral oxygenation and is not affected by a loop
chance of femoral artery bleeding. diuretic.
4. The gag reflex is assessed if a scope is in- TEST-TAKING HINT: Knowing that diuretics in-
serted down the trachea (bronchoscopy) or crease urine output would lead the test taker
esophagus (endoscopy) because the throat is to eliminate glucose level and oxygenation
numbed when inserting the scope. A catheter (options “3” and “4”). In very few instances
is inserted in the femoral or brachial artery does the nurse assess the radial pulse; the
when performing a cardiac catheterization. apical pulse is assessed.
TEST-TAKING HINT: The nurse should apply Content – Medical: Integrated Nursing Process –
the nursing process when determining the Implementation: Client Needs – Physiological Integrity,
correct answer. Therefore, either option “2” Pharmacological and Parenteral Therapies: Cognitive
or option “4” could possibly be the correct Level – Application: Concept – Medication.
answer. The test taker then should apply
30. 1. A low-fat, low-cholesterol diet will help
anatomy concepts—where is the left femoral
decrease the buildup of atherosclerosis in
artery? Neurovascular assessment is per-
the arteries.
formed on extremities.
2. Walking will help increase collateral
Content – Surgical: Integrated Nursing Process – circulation.
Planning: Client Needs – Physiological Integrity, 3. Salt should be restricted in the diet of a client
Reduction of Risk Potential: Cognitive Level – Synthesis:
with hypertension, not coronary artery disease.
Concept – Perfusion.
4. Stress reduction is encouraged for clients
28. 1. This blood pressure is normal and the nurse with CAD because this helps prevent
would administer the medication. ­excess stress on the heart muscle.
2. A beta blocker decreases sympathetic 5. Increasing fiber in the diet will help
stimulation to the heart, thereby decreas- r­emove cholesterol via the gastrointestinal
ing the heart rate. An apical rate less than system.
60 indicates a lower-than-normal heart TEST-TAKING HINT: This is an alternate-type
rate and should make the nurse question question where the test taker must select all
administering this medication because it interventions that are applicable to the situ-
will further decrease the heart rate. ation. Coronary artery disease is a common
3. A headache will not affect administering the disease, and the nurse must be knowledge-
medication to the client. able about ways to modify risk factors.
4. The potassium level is within normal lim- Content – Medical: Integrated Nursing Process – ­
its, but it is usually not monitored prior to Planning: Client Needs – Physiological Integrity,
­administering a beta blocker. ­Physiological Adaptation: Cognitive Level – Synthesis:
TEST-TAKING HINT: If the test taker does not ­Concept – Nursing Roles.
know when to question the use of a certain 31. 1. Bowel movements are important, but they are
medication, the test taker should evaluate not pertinent to coronary artery disease.
the options to determine if any options in- 2. Chest x-rays are usually done for respiratory
clude abnormal data based on normal param- problems, not for coronary artery disease.
eters. This would make the test taker select 3. Sexual activity is a risk factor for angina
option “2” because the normal apical pulse in resulting from coronary artery disease.
an adult is 60 to 100. The client’s being elderly should not af-
Content – Medical: Integrated Nursing Process – fect the nurse’s assessment of the client’s
Assessment: Client Needs – Physiological Integrity, concerns about sexual activity.
Pharmacological and Parenteral Therapies: Cognitive
4. Weight change is not significant in a client
Level – Analysis: Concept – Medication.
with coronary artery disease.

03_Colgrove_Ch03.indd 88 02/06/16 7:53 PM


C hapter 3  C ardiac D isorders 89

TEST-TAKING HINT: Remember, if the client is 34. 1. The Holter monitor is a 24-hour elec-
described with an adjective such as “elderly,” trocardiogram, and the client must keep
this may be the key to selecting the correct an accurate record of activity so that the
answer. The nurse must not be judgmental health-care provider can compare the ECG
about the elderly, especially about issues recordings with different levels of activity.
concerning sexual activity. 2. The Holter monitor should not be removed
Content – Medical: Integrated Nursing Process – for any reason.
Assessment: Client Needs – Physiological Integrity, 3. All medications should be taken as prescribed.
Reduction of Risk Potential: Cognitive Level – Analysis: 4. The client should perform all activity as usual
Concept – Perfusion. while wearing the Holter monitor so the
HCP can get an accurate account of heart
32. 1. Isometric exercises are weight lifting–type
function during a 24-hour period.
exercises. A client with CAD should perform
isotonic exercises, which increase muscle TEST-TAKING HINT: In some instances, the test
tone, not isometric exercises. taker must be knowledgeable about diagnos-
2. The client should walk at least 30 minutes a tic tests and there are no test-taking hints.
day to increase collateral circulation. The test taker might eliminate option “3” by
3. When it is cold outside, vasoconstriction realizing that, unless the client is NPO for a
occurs, and this will decrease oxygen to the test or surgery, medications are usually taken.
heart muscle. Therefore, the client should Content – Medical: Integrated Nursing Process –
not exercise when it is cold outside. Implementation: Client Needs – Physiological Integrity,
4. The client should wear good, supportive te­ Reduction of Risk Potential: Cognitive Level – Application:
nnis shoes when ambulating, not sandals or Concept – Perfusion.
other open-toed shoes. 35. 1. According to the American Heart Association,
TEST-TAKING HINT: The test taker should be the client should not eat more than three (3)
aware of adjectives such as “isometric,” which eggs a week, especially the egg yolk.
makes option “1” incorrect, and “open-toed,” 2. The American Heart Association recom-
which makes option “4” incorrect. mends a low-fat, low-cholesterol diet for
Content – Medical: Integrated Nursing Process – a client with coronary artery disease. The
­Implementation: Client Needs – Physiological Integrity, ­client should avoid any fried foods, especially
­Physiological Adaptation: Cognitive Level – Application: meats, and bake, broil, or grill any meat.
Concept – Perfusion. 3. The client should drink low-fat milk, not
whole milk.
33. 1. The client should take the coronary vasodila-
4. Pork products (bacon, sausage, ham) are high
tor nitroglycerin sublingually, but it is not the
in sodium, which is prohibited in a low-salt
first intervention.
diet, not a low-cholesterol, low-fat diet.
2. Stopping the activity decreases the heart’s
need for oxygen and may help decrease TEST-TAKING HINT: The test taker must
the angina (chest pain). be knowledgeable of prescribed diets for
3. The client should keep a diary of when an- s­pecific disease processes. This is mainly
gina occurs, what activity causes it, and how memorizing facts. There is no test-taking
many nitroglycerin tablets are taken before hint to help eliminate any of the options.
chest pain is relieved. Content – Medical: Integrated Nursing Process –
4. If the chest pain (angina) is not relieved with Evaluation: Client Needs – Physiological Integrity,
three (3) nitroglycerin tablets, the client Physiological Adaptation: Cognitive Level – Analysis:
should call 911 or have someone take him Concept – Nutrition.
to the emergency department. Notifying the 36. 1. This client is at high risk for complications
HCP may take too long. related to necrotic myocardial tissue and will
TEST-TAKING HINT: The question is asking which need extensive teaching, so this client should
action the client should take first. This implies not be assigned to a new graduate.
that more than one of the answer options could 2. Unstable angina means this client is at risk
be appropriate for the chest pain, but that only for life-threatening complications and should
one is done first. The test taker should select not be assigned to a new graduate.
the answer that will help the client directly and 3. A new graduate should be able to complete
quickly—and that is stopping the activity. a preprocedure checklist and get this client
Content – Medical: Integrated Nursing Process – to the catheterization laboratory.
Planning: Client Needs – Physiological Integrity, Physiologi- 4. Chest pain means this client could be having
cal Adaptation: Cognitive Level – Synthesis: a myocardial infarction and should not be as-
Concept – Perfusion. signed to a new graduate.

03_Colgrove_Ch03.indd 89 02/06/16 7:53 PM


90 M ed -S urg S uccess

TEST-TAKING HINT: “New graduate” is the 39. 1. Prophylactic antibiotics before invasive pro-
key to answering this question correctly. cedures prevent infectious endocarditis.
What type of client should be assigned to an 2. The client is undergoing anticoagulant ther-
inexperienced nurse? The test taker should apy and should use a soft-bristle toothbrush
not assign the new graduate a client who to help prevent gum trauma and bleeding.
is unstable or at risk for a life-threatening 3. Aspirin and nonsteroidal anti-
complication. inflammatory drugs (NSAIDs) interfere
Content – Medical: Integrated Nursing Process – with clotting and may potentiate the ef-
Planning: Client Needs – Safe Effective Care Environ- fects of the anticoagulant therapy, which
ment, Management of Care: Cognitive Level – Synthesis: the client with a mechanical valve will be
Concept – Nursing Roles. prescribed. Therefore, the client should
not take aspirin daily.
Valvular Heart Disease 4. The client should alternate rest with activity
to prevent fatigue to help decrease the work-
37. 1. Dyspnea on exertion (DOE) is typically load of the heart.
the earliest manifestation of mitral valve TEST-TAKING HINT: The stem asks the test
stenosis. taker to identify which behavior means
2. Jugular vein distention (JVD) and 3+ pedal the client does not understand the teach-
edema are signs/symptoms of right-sided ing. Therefore, the test taker should select
heart failure and indicate worsening of the the distracter that does not agree with the
mitral valve stenosis. These signs would not condition. There is no condition for which
be expected in a client with early manifesta- alternating rest with activity would not be
tions of mitral valve stenosis. recommended.
3. Chest pain rarely occurs with mitral valve Content – Medical: Integrated Nursing Process –
stenosis. Evaluation: Client Needs – Physiological Integrity,
4. An enlarged liver and edematous abdomen Physiological Adaptation: Cognitive Level – Synthesis:
are late signs of right-sided heart failure that Concept – Perfusion.
can occur with long-term untreated mitral
40. 1. An advantage of having a biological valve re-
valve stenosis.
placement is that no anticoagulant therapy is
TEST-TAKING HINT: Whenever the test taker needed. Anticoagulant therapy is needed with
reads “rule out,” the test taker should look a mechanical valve replacement.
for data that would not indicate a severe 2. This is an advantage of having a biologi-
condition of the body system that is affected. cal valve replacement; infections are harder
Chest pain, JVD, and pedal edema are late to treat in clients with mechanical valve
signs of heart problems. replacement.
Content – Medical: Integrated Nursing Process – 3. This is an advantage of having a biological
Assessment: Client Needs – Physiological Integrity, valve replacement; there is a high incidence
Reduction of Risk Potential: Cognitive Level – Analysis: of thromboembolism in clients with mechani-
Concept – Perfusion. cal valve replacement.
38. 1. This would be expected with mitral valve 4. Biological valves deteriorate and need to
stenosis. be replaced frequently; this is a disadvan-
2. The murmur associated with mitral tage of them. Mechanical valves do not
valve insufficiency is loud, high pitched, deteriorate and do not have to be replaced
rumbling, and holosystolic (occurring often.
throughout systole) and is heard best at TEST-TAKING HINT: This is an “except”
the cardiac apex. ­question. The test taker might reverse the
3. This would be expected with mitral valve question and ask, “Which is an advantage
prolapse. of a biological valve?”—which might make
4. This would be expected with aortic ­answering the question easier.
regurgitation. Content – Medical: Integrated Nursing Process –
TEST-TAKING HINT: This is a knowledge-based ­Diagnosis: Client Needs – Physiological Integrity,
question and there is no test-taking hint to ­Physiological Adaptation: Cognitive Level – Knowledge:
help the test taker rule out distracters. Concept – Perfusion.
Content – Medical: Integrated Nursing Process –
Assessment: Client Needs – Physiological Integrity,
Reduction of Risk Potential: Cognitive Level – Analysis:
Concept – Perfusion.

03_Colgrove_Ch03.indd 90 02/06/16 7:53 PM


C hapter 3  C ardiac D isorders 91

41. 1. The therapeutic range for most clients’ 43. 1. An acute myocardial infarction can damage
INR is 2 to 3, but for a client with a heart valves, causing tearing, ischemia, or
­mechanical valve replacement it is 2 to damage to heart muscles that affects valve
3.5. The medication should be given as leaflet function, but coronary heart disease
­ordered and not withheld. does not cause valvular heart disease.
2. Vitamin K is the antidote for an overdose of 2. Valvular heart disease does not show a ­genetic
warfarin, but 2.7 is within the therapeutic etiology.
range. 3. Smoking can cause coronary artery disease,
3. This laboratory result is within the but it does not cause valvular heart disease.
­therapeutic range, INR 2 to 3, and the 4. Rheumatic heart disease is the most com-
­medication does not need to be withheld. mon cause of valvular heart disease.
4. There is no need for the nurse to assess for TEST-TAKING HINT: The test taker could rule
bleeding because 2.7 is within the ­therapeutic out option “1” because of knowledge of anat-
range. omy: Coronary artery disease has to do with
TEST-TAKING HINT: The test taker has to blood supply to heart muscle, whereas the
know the therapeutic range for the INR to valves are a part of the anatomy of the heart.
be able to answer this question correctly. The Content – Medical: Integrated Nursing Process –
test taker should keep a list of normal and Assessment: Client Needs – Physiological Integrity,
therapeutic laboratory values that must be Reduction of Risk Potential: Cognitive Level – Analysis:
remembered. Concept – Perfusion.
Content – Medical: Integrated Nursing Process – 44. 1. Percutaneous balloon valvuloplasty is not an
­Implementation: Client Needs – Physiological Integrity, open-heart surgery; therefore, the chest will
­Pharmacological and Parenteral Therapies: Cognitive
not be open and the client will not have a
Level – Analysis: Concept – Medication.
chest tube.
42. 1. Paroxysmal nocturnal dyspnea is a ­sudden 2. This is not an open-heart surgery; therefore,
attack of respiratory distress, usually oc- the client will not have a chest dressing.
curring at night because of the reclining 3. The endotracheal (ET) tube is inserted if the
position, and occurs in valvular disorders. client is on a ventilator, and this surgery does
2. This is an abnormal condition in which a not require putting the client on a ventilator.
client must sit or stand to breathe com- 4. In this invasive procedure, performed in a
fortably and occurs in valvular disorders. cardiac catheterization laboratory, the cli-
3. Coughing occurs when the client with ent has a catheter inserted into the femo-
long-term valvular disease has difficulty ral artery. Therefore, the client must keep
breathing when walking or performing the leg straight to prevent hemorrhaging
any type of activity. at the insertion site.
4. Pericardial friction rub is a sound auscultated TEST-TAKING HINT: If the test taker knows
in clients with pericarditis, not valvular heart that the word “percutaneous” means “via
disease. the skin,” then options “1” and “2” could be
5. Pulsus paradoxus is a marked decrease in eliminated as possible correct answers.
amplitude during inspiration. It is a sign of Content – Surgical: Integrated Nursing Process –
cardiac tamponade, not valvular heart disease. Implementation: Client Needs – Physiological Integrity,
TEST-TAKING HINT: The test taker should Reduction of Risk Potential: Cognitive Level – Application:
­notice that options “1,” “2,” and “3” are all Concept – Perfusion.
signs/­symptoms that have something to do 45. 1. The client is at risk for developing endo-
with the lungs. It would be a good choice to carditis and should take prophylactic anti-
select these three as correct answers. They biotics before any invasive procedure.
are similar in description. 2. Antibiotics have nothing to do with how
Content – Medical: Integrated Nursing Process – much the teeth bleed during a cleaning.
Assessment: Client Needs – Physiological Integrity, 3. Teeth cleaning will not cause the valve to
Reduction of Risk Potential: Cognitive Level – Analysis:
malfunction.
Concept – Perfusion.
4. Vegetation develops on valves secondary to
bacteria that cause endocarditis, but the client
may not understand “vegetative growth on
your valves”; therefore, this is not the most
appropriate answer.

03_Colgrove_Ch03.indd 91 02/06/16 7:53 PM


92 M ed -S urg S uccess

TEST-TAKING HINT: The test taker should that possible complications of these problems
s­ elect an option that answers the client’s do not ­include mitral valve prolapse.
question in the easiest and most understand- Content – Medical: Integrated Nursing Process –
able terms, not in medical jargon. This would Assessment: Client Needs – Physiological Integrity,
cause the test taker to eliminate option “4” Physiological Adaptation: Cognitive Level – Analysis:
as a possible correct answer. The test taker Concept – Perfusion.
should know antibiotics do not ­affect bleed- 48. 1. This client requires teaching and an under-
ing and so can eliminate option “2.” standing of the preprocedure interventions for
Content – Surgical: Integrated Nursing Process – diagnostic tests; therefore, a more experienced
­Implementation: Client Needs – Physiological Integrity, nurse should be assigned to this client.
­Physiological Adaptation: Cognitive Level – Application:
2. Because this client is being discharged, it
Concept – Perfusion.
would be an appropriate assignment for
46. 1. Fluid intake may be restricted to reduce the new graduate.
the cardiac workload and pressures within 3. Supraventricular tachycardia (SVT) is not
the heart and pulmonary circuit. life threatening, but the client requires in-
2. The head of the bed should be elevated to travenous medication and close monitoring
help improve alveolar ventilation. and therefore should be assigned to a more
3. Oxygen saturation should be no less than ­experienced nurse.
93%; 90% indicates an arterial oxygen satura- 4. A client with atrial fibrillation is usually tak-
tion of around 60 (normal is 80 to 100). ing the anticoagulant warfarin (Coumadin),
4. Total parenteral nutrition would not be and the therapeutic INR is 2 to 3. An INR of
prescribed for a client with mitral valve 5 is high and the client is at risk for bleeding.
replacement. It is ordered for clients with TEST-TAKING HINT: The test taker must real-
malnutrition, gastrointestinal disorders, or ize that a new graduate must be assigned the
conditions in which increased calories are least critical client. Remember, teaching is a
needed, such as burns. primary responsibility of the nurse; physical
TEST-TAKING HINT: A client with a heart or lung care is not ­always the criterion that should be
problem should never have the head of the bed used when making client assignments.
in a flat (supine) position; therefore, option “2” Content – Medical: Integrated Nursing Process – ­
should be eliminated as a possible correct an- Planning: Client Needs – Safe Effective Care Environ-
swer. The test taker must know normal values ment, Management of Care: Cognitive Level – Synthesis:
for monitoring techniques such as pulse oxim- ­Concept – Nursing Roles.
eters and keep a list of normal values.
Content – Surgical: Integrated Nursing Process –
Implementation: Client Needs – Safe Effective Care
Dysrhythmias and Conduction Problems
Environment, Management of Care: Cognitive Level –
Application: Concept – Perfusion. 49. 1. The telemetry nurse should not leave the
monitors unattended at any time.
47. 1. Congestive heart failure does not predispose 2. The telemetry nurse must have someone
the female client to having a mitral valve go assess the client, but this is not the first
prolapse. intervention.
2. Clients with Marfan’s syndrome have life- 3. If the client answers the call light and is
threatening cardiovascular problems, in- not experiencing chest pain, then there is
cluding mitral valve prolapse, progressive probably a monitor artifact, which is not a
dilation of the aortic valve ring, and weak- life-threatening emergency. After talking
ness of the arterial walls, and they usually with the client, send a nurse to the room
do not live past the age of 40 because of to check the monitor.
dissection and rupture of the aorta. 4. The crash cart should be taken to a room
3. Atrial fibrillation does not predispose a client when the client is experiencing a code.
to mitral valve prolapse.
TEST-TAKING HINT: When the test taker sees
4. A client with Down syndrome may have con-
the word “first,” the test taker must realize that
genital heart anomalies but not mitral valve
more than one answer option may be a pos-
prolapse.
sible intervention but that only one should be
TEST-TAKING HINT: The test taker could elim- ­implemented first. The test taker should try to
inate options “1” and “3” based on knowledge determine which intervention directly affects
that these are commonly occurring cardiovas- the client.
cular problems, and the nurse should know

03_Colgrove_Ch03.indd 92 02/06/16 7:53 PM


C hapter 3  C ardiac D isorders 93

Content – Medical: Integrated Nursing Process – TEST-TAKING HINT: The key to answering
Implementation: Client Needs – Safe Effective Care this question is the adjective “collaborative,”
Environment, Management of Care: Cognitive Level – which means the treatment requires obtain-
Synthesis: Concept – Perfusion. ing a health-care provider’s order or work-
50. 1. There are many interventions that should ing with another member of the health-care
be implemented prior to administering team. This would cause the test taker to elim-
medication. inate option “2” as a possible correct answer.
2. The treatment of choice for ventricular fibril- Content – Medical: Integrated Nursing Process –
lation is defibrillation, but it is not the first Planning: Client Needs – Physiological Integrity,
action. Reduction of Risk Potential: Cognitive Level – Synthesis:
3. The nurse must call a code that activates Concept – Perfusion.
the crash cart being brought to the room 53. 1. The adult client should be defibrillated at
and a team of health-care providers that 360 joules.
will care for the client according to an es- 2. The oxygen source should be removed
tablished protocol. to prevent any type of spark during
4. The first person at the bedside should start defibrillation.
cardiopulmonary resuscitation (CPR), but the 3. The nurse should use defibrillator pads or
telemetry nurse should call a code so that all defibrillator gel to prevent any type of skin
necessary equipment and personnel are at the burns while defibrillating the client.
bedside. 4. If any member of the health-care team
TEST-TAKING HINT: The test taker must realize is touching the client or the bed during
that ventricular fibrillation is life threatening defibrillation, that person could possibly
and immediate action must be implemented. be shocked. Therefore, the nurse should
Remember, when the question asks “first,” all shout “all clear.”
options could be appropriate interventions TEST-TAKING HINT: The test taker should al-
but only one should be implemented first. ways consider the safety of the client and the
Content – Medical: Integrated Nursing Process – health-care team. Options “2” and “3” put the
Implementation: Client Needs – Safe Effective Care Envi- client at risk for injury during defibrillation.
ronment, Management of Care: Cognitive Level –
Content – Medical: Integrated Nursing Process –
Synthesis: Concept – Perfusion.
Implementation: Client Needs – Safe Effective Care
51. 1. Amiodarone suppresses ventricular ectopy Environment, Management of Care: Cognitive Level –
and is the drug of choice for ­ventricular Application: Concept – Perfusion.
dysrhythmias. 54. 1. A client with chronic atrial fibrillation will
2. Atropine decreases vagal stimulation and is be taking an anticoagulant to help prevent
the drug of choice for asystole. clot formation. Therefore, the client is at
3. Digoxin slows heart rate and increases ­cardiac risk for bleeding and should be instructed
contractility and is the drug of choice for to use a soft-bristle toothbrush.
atrial fibrillation. 2. The client will need a regularly scheduled
4. Adenosine is the drug of choice for supraven- INR to determine the therapeutic level for
tricular tachycardia. the anticoagulant warfarin (Coumadin); PTT
TEST-TAKING HINT: This is a knowledge-based levels are monitored for heparin.
question, and the test taker must know the 3. A client with symptomatic sinus bradycardia,
answer. The nurse must know what medica- not a client with atrial fibrillation, may need a
tions treat specific dysrhythmias. pacemaker.
Content – Medical: Integrated Nursing Process – 4. Synchronized cardioversion may be pre-
Planning: Client Needs – Physiological Integrity, scribed for new-onset atrial fibrillation but
Pharmacological and Parenteral Therapies: Cognitive not for chronic atrial fibrillation.
Level – Synthesis: Concept – Medication. TEST TAKING HINT: In order to choose the
52. 1. A thrombolytic medication is administered for correct answer for this question the test
a client experiencing a myocardial infarction. taker must recognize the disease process,
2. Assessment is an independent nursing a­ ction, then know what complications are possible,
not a collaborative treatment. and finally the test-taker must know how
3. The client is symptomatic and will ­require the client can possibly be treated so that the
a pacemaker. complication does not occur.
4. Synchronized cardioversion is used for Content – Medical: Integrated Nursing Process – Planning:
­ventricular tachycardia with a pulse or atrial Client Needs – Physiological Integrity, Physiological Adapta-
fibrillation. tion: Cognitive Level – Synthesis: Concept – Nursing Roles.

03_Colgrove_Ch03.indd 93 02/06/16 7:53 PM


94 M ed -S urg S uccess

55. 1. Amiodarone is the drug of choice for ven- 3. Defibrillation is the treatment of choice
tricular tachycardia, but it is not the first for ventricular fibrillation.
intervention. 4. The crash cart has the defibrillator and is
2. Defibrillation may be needed, but it is not the used when performing advanced cardio-
first intervention. pulmonary resuscitation.
3. The nurse must assess the apical pulse and 5. Amiodarone is an antidysrhythmic that is
blood pressure to determine if the client is used in ventricular dysrhythmias.
in cardiac arrest and then treat as ventricu- TEST-TAKING HINT: This is an alternate-type
lar fibrillation. If the client’s heart is beating, question that requires the test taker to pos-
the nurse would then administer lidocaine. sibly select more than one option. To receive
4. CPR is only performed on a client who is credit, the test taker must select all correct
not breathing and does not have a pulse. The options; partial credit is not given for this
nurse must establish if this is occurring first type of question.
prior to taking any other action. Content – Medical: Integrated Nursing Process –
TEST-TAKING HINT: When the stem asks the Implementation: Client Needs – Safe Effective Care
test taker to select the first intervention, all Environment, Management of Care: Cognitive Level –
answer options could be plausible interven- Analysis: Concept – Perfusion.
tions, but only one is implemented first. The 58. 1. The telemetry reading is accurate, and there
test taker should use the nursing process to is no need for the nurse to assess the client’s
answer the question and select the interven- heart rate.
tion that addresses assessment, which is the
2. There is no reason to notify the surgeon for a
first step in the nursing process.
client exhibiting sinus tachycardia.
Content – Medical: Integrated Nursing Process – 3. Synchronized cardioversion is prescribed for
Implementation: Client Needs – Safe Effective Care clients in acute atrial fibrillation or ventricu-
Environment, Management of Care: Cognitive Level –
lar fibrillation with a pulse.
Synthesis: Concept – Perfusion.
4. Sinus tachycardia means the sinoatrial
56. 1. A pacemaker will have to be inserted, but it is node is the pacemaker, but the rate is
not the first intervention. greater than 100 because of pain, anxiety,
2. Atropine will decrease vagal stimulation or fever. The nurse must determine the
and increase the heart rate. Therefore, it cause and treat appropriately. There is no
is the first intervention. specific medication for sinus tachycardia.
3. A STAT ECG may be done, but the telem- TEST-TAKING HINT: The test taker must use
etry reading shows complete heart block, the nursing process to determine the correct
which is a life-threatening dysrhythmia and option and select an option that addresses as-
must be treated. sessment, the first step of the nursing process.
4. The HCP will need to be notified but not Because both option “1” and option “4” address
prior to administering a medication. The test assessment, the test taker must determine
taker must assume the nurse has the ­order which option is more appropriate. How will
to administer medication. Many telemetry taking the apical pulse help treat sinus tachy-
­departments have standing protocols. cardia? Determining the cause for sinus tachy-
TEST-TAKING HINT: The test taker must select cardia is the most appropriate intervention.
the intervention that should be implemented Content – Surgical: Integrated Nursing Process –
first and will directly affect the dysrhythmia. Implementation: Client Needs – Safe Effective Care
Medication is the first intervention, and Environment, Management of Care: Cognitive Level –
then pacemaker ­insertion. The test taker Application: Concept – Perfusion.
should not eliminate an option because the 59. 1. The P wave represents atrial contraction,
test taker thinks there is not an order by a and the QRS complex represents ven-
health-care provider. tricular contraction—a normal telemetry
Content – Medical: Integrated Nursing Process – ­ reading. A rate between 60 and 100 in-
Implementation: Client Needs – Safe Effective Care dicates normal sinus rhythm. Therefore,
Environment, Management of Care: Cognitive Level –
the nurse should document this as normal
Synthesis: Concept – Perfusion.
sinus rhythm and not take any action.
57. 1. Ventricular fibrillation indicates the ­client 2. A 12-lead ECG should be requested for chest
does not have a heartbeat. Therefore, pain or abnormal dysrhythmias.
CPR should be instituted. 3. Digoxin is used to treat atrial fibrillation.
2. Adenosine, an antidysrhythmic, is the drug of 4. Cardiac enzymes are monitored to determine
choice for supraventricular tachycardia, not if the client has had a myocardial infarction.
for ventricular fibrillation.

03_Colgrove_Ch03.indd 94 02/06/16 7:53 PM


C hapter 3  C ardiac D isorders 95

Nothing in the stem indicates the client has s­ ystem, specifically the outside of the heart.
had an MI. The test taker should select the option that
TEST-TAKING HINT: The test taker must know has something to do with the heart, which is
­normal sinus rhythm, and there are no test- either option “1” or option “4.”
taking hints to help eliminate incorrect op- Content – Medical: Integrated Nursing Process –
tions. The test taker should not automatically Assessment: Client Needs – Physiological Integrity,
select assessment as the correct answer, but Reduction of Risk Potential: Cognitive Level – Analysis:
if the test taker had no idea of the answer, re- Concept – Perfusion.
member assessment of ­laboratory data is not 62. 1. Acute pericardial effusion interferes
the same as assessing the client. with normal cardiac filling and pumping,
Content – Medical: Integrated Nursing Process – ­causing venous congestion and decreased
­Implementation: Client Needs – Safe Effective Care cardiac output. Muffled heart sounds,
Environment, Management of Care: Cognitive Level – indicative of acute pericarditis, must be
Application: Concept – Perfusion. reported to the health-care provider.
60. 1. Not every cardiac dysrhythmia causes 2. Distended, not nondistended, jugular veins
­alteration in comfort; angina is caused by would warrant immediate intervention.
decreased oxygen to the myocardium. 3. Decreasing quality of peripheral pulses, not
2. Any abnormal electrical activity of the bounding peripheral pulses, would warrant
heart causes decreased cardiac output. immediate intervention.
3. Impaired gas exchange is the result of pulmo- 4. A pericardial friction rub is a classic symptom
nary complications, not cardiac dysrhythmias. of acute pericarditis, but it would not warrant
4. Not all clients with cardiac dysrhythmias have immediate intervention.
activity intolerance. TEST-TAKING HINT: This is a priority setting
TEST-TAKING HINT: Option “2” has the word question, the test taker should determine if
“cardiac,” which refers to the heart. Therefore, the data provided is abnormal or expected for
even if the test taker had no idea what the cor- the the disease process. If so, then the test
rect answer was, this would be an appropriate taker can consider the option as being the
option. The test taker should use medical ter- correct answer. If the data is within normal
minology to help identify the correct option. limits aor expected for the disease process
then the option is not a priority.
Content – Medical: Integrated Nursing Process –
Diagnosis: Client Needs – Physiological Integrity, Content – Medical: Integrated Nursing Process –
Physiological Adaptation: Cognitive Level – Analysis: Assessment: Client Needs – Safe Effective Care
Concept – Perfusion. Environment, Management of Care: Cognitive Level –
Synthesis: Concept – Perfusion.

63. 1. A sore throat in the last month would not


Inflammatory Cardiac Disorders support the diagnosis of carditis.
2. Rheumatic fever, a systemic inflammatory
61. 1. Pulsus paradoxus is the hallmark of cardiac
disease caused by an abnormal immune re-
tamponade; a paradoxical pulse is markedly
sponse to pharyngeal infection by group A
decreased in amplitude during inspiration.
beta-hemolytic streptococci, causes cardi-
2. Fatigue and arthralgias are nonspecific
tis in about 50% of people who develop it.
signs/symptoms that usually occur with
3. Carditis is not a genetic or congenital ­disease
myocarditis.
process.
3. Petechiae on the trunk, conjunctiva, and
4. This is an appropriate question to ask any
mucous membranes and hemorrhagic streaks
client, but OTC medications do not cause
under the fingernails or toenails occur with
carditis.
endocarditis.
4. Chest pain is the most common symp- TEST-TAKING HINT: This is a knowledge-based
tom of pericarditis, usually has an abrupt question, but the test taker could eliminate
onset, and is aggravated by respiratory ­option “4,” realizing this is a question to ask
movements (deep inspiration, coughing), any client, and the stem asks which question
changes in body position, and swallowing. will ­support the diagnosis of carditis.
TEST-TAKING HINT: The test taker who has no Content – Medical: Integrated Nursing Process –
Assessment: Client Needs – Physiological Integrity,
idea what the answer is should apply the test-
Reduction of Risk Potential: Cognitive Level – Analysis:
taking strategy of asking which body sys- Concept – Perfusion.
tem is affected. In this case, it is the cardiac

03_Colgrove_Ch03.indd 95 02/06/16 7:53 PM


96 M ed -S urg S uccess

64. 1. Steroids, such as prednisone, not NSAIDs, TEST-TAKING HINT: “Collaborative” means
must be tapered off to prevent adrenal another member of the health-care team
insufficiency. must order or participate in the intervention.
2. NSAIDs will not make clients drowsy. Therefore, option “4” could be eliminated as
3. NSAIDs must be taken with food, milk, or a possible correct answer.
antacids to help decrease gastric distress. Content – Medical: Integrated Nursing Process –
NSAIDs reduce fever, inflammation, and Planning: Client Needs – Physiological Integrity,
pericardial pain. Reduction of Risk Potential: Cognitive Level – Synthesis:
4. NSAIDs should be taken regularly around Concept – Perfusion.
the clock to help decrease inflammation, 67. 1. The full course of antibiotics must be taken
which, in turn, will decrease pain. to help ensure complete destruction of strep-
TEST-TAKING HINT: The test taker must re- tococcal infection.
member NSAIDs and steroids cause gastric 2. Antibiotics kill bacteria but also destroy
distress to the point of causing peptic ulcer normal body flora in the vagina, bowel, and
disease. These ­medications are administered mouth, leading to a superinfection.
for a variety of conditions and diseases. 3. Even with antibiotic treatment for
Content – Medical: Integrated Nursing Process – ­rheumatic fever, the client may experience
Planning: Client Needs – Physiological Integrity, bacterial endocarditis in later years and
Pharmacological and Parenteral Therapies: Cognitive should know this may occur.
Level – Synthesis: Concept – Medication. 4. A throat culture is taken to diagnose group A
65. 1. Oxygen may be needed, but it is not the first beta-hemolytic streptococcus and is positive
intervention. in 25% to 40% of clients with acute rheu-
2. This would be appropriate to determine if the matic fever.
urine output is at least 30 mL/hr, but it is not TEST-TAKING HINT: The question is asking
the first intervention. the test taker to identify which statement
3. The nurse must assess the client to deter- indicates the client does not understand the
mine if the pain is expected secondary to teaching; this is an “except” question. The
pericarditis or if the pain is indicative of test taker can ask which statement indicates
a complication that requires intervention the teaching is effective and choose the one
from the health-care provider. option that is not appropriate.
4. Using the incentive spirometer will increase Content – Medical: Integrated Nursing Process –
the client’s alveolar ventilation and help Evaluation: Client Needs – Physiological Integrity,
prevent atelectasis, but it is not the first Pharmacological and Parenteral Therapies: Cognitive
intervention. Level – Synthesis: Concept – Medication.
TEST-TAKING HINT: The test taker must ap- 68. 1. Pulmonary embolus would occur with an
ply the nursing process when determining embolization of vegetative lesions from the
the correct answer and select the option tricuspid valve on the right side of the heart.
that addresses the first step in the nursing 2. Bacteria enter the bloodstream from inva-
process—assessment. sive procedures, and sterile platelet-fibrin
Content – Medical: Integrated Nursing Process – vegetation forms on heart valves. The
Implementation: Client Needs – Safe Effective Care mitral valve is on the left side of the heart
Environment, Management of Care: Cognitive Level – and, if the vegetation breaks off, it will
Synthesis: Concept – Perfusion. go through the left ventricle into the sys-
66. 1. A pericardiocentesis removes fluid from temic circulation and may lodge in the
the pericardial sac and is the emergency brain, kidneys, or peripheral tissues.
treatment for cardiac tamponade. 3. Coughing up blood (hemoptysis) occurs
2. Cardiac enzymes may be slightly elevated be- when the vegetation breaks off the tricuspid
cause of the inflammatory process, but evalu- valve in the right side of the heart and enters
ation of these would not be ordered to treat the pulmonary artery.
or evaluate cardiac tamponade. 4. Deep vein thrombosis is a complication of
3. A 12-lead ECG would not help treat the immobility, not of a vegetative embolus from
medical emergency of cardiac tamponade. the left side of the heart.
4. Assessment by the nurse is not collaborative; TEST-TAKING HINT: If the test taker does not
it is an independent nursing action. know the answer, knowledge of anatomy may
help determine the answer. The mitral valve
is on the left side of the heart and any emboli

03_Colgrove_Ch03.indd 96 02/06/16 7:53 PM


C hapter 3  C ardiac D isorders 97

would not enter the lung first, thereby Content – Medical: Integrated Nursing Process –
eliminating options “1” and “3” as possible Planning: Client Needs – Physiological Integrity,
correct answers. Physiological Adaptation: Cognitive Level – Synthesis:
Concept – Nursing Roles.
Content – Medical: Integrated Nursing Process –
Assessment: Client Needs – Physiological Integrity, 71. 1. The nurse should monitor the vital signs
Reduction of Risk Potential: Cognitive Level – Analysis: for any client who has just undergone
Concept – Perfusion. surgery.
69. 1. Anxiety is a psychosocial nursing diagnosis, 2. A pericardiocentesis involves entering
which is not a priority over a physiological the pericardial sac. Assessing heart and
nursing diagnosis. lung sounds allows assessment for ­cardiac
2. Antibiotic therapy does not result in injury to failure.
the client. 3. The pericardial fluid is documented as
3. Myocarditis does not result in valve damage output.
(endocarditis does), and there would be de- 4. Evaluating the client’s cardiac rhythm
creased, not increased, cardiac output. allows the nurse to assess for car-
4. Activity intolerance is priority for the diac failure, which is a complication of
­client with myocarditis, an inflammation pericardiocentesis.
of the heart muscle. Nursing care is aimed 5. The client should be in the semi-Fowler’s po-
at decreasing myocardial work and main- sition, not in a flat position, which increases
taining cardiac output. the workload of the heart.
TEST-TAKING HINT: If the test taker has no TEST-TAKING HINT: This is an alternate-type
idea which is the correct answer, then “myo,” question that requires the test taker to select
which refers to muscle, and “card,” which re- possibly more than one option as a correct
fers to the heart, should lead the test taker to answer.
the only ­option which has both muscle and Content – Medical: Integrated Nursing Process – ­
heart in it, o
­ ption “4.” Implementation: Client Needs – Safe Effective Care
Environment, Management of Care: Cognitive Level –
Content – Medical: Integrated Nursing Process –
Application: Concept – Perfusion.
Diagnosis: Client Needs – Physiological Integrity,
Physiological Adaptation: Cognitive Level – Analysis: 72. 1. The nurse must obtain blood cultures prior
Concept – Perfusion. to administering antibiotics.
70. 1. Uninterrupted rest and sleep help de- 2. Blood cultures must be done before ad-
crease the workload of the heart and help ministering antibiotics so that an a­ dequate
ensure the restoration of physical and number of organisms can be obtained to
emotional health. culture and identify.
2. Occupational therapy addresses activities of 3. An echocardiogram allows visualization of
daily living. The client should be referred to vegetations and evaluation of valve function.
physical therapy to develop a realistic and However, antibiotic therapy is a priority be-
progressive plan of activity. fore diagnostic tests, and blood cultures must
3. The client with pericarditis is not usually be obtained before administering medication.
prescribed oxygen, and 2 L/min is a low 4. Bedrest should be implemented, but the first
dose of oxygen that is prescribed for a client intervention should be obtaining blood cul-
with chronic obstructive pulmonary disease tures so that antibiotic therapy can be started
(COPD). as soon as possible.
4. Endocarditis, not pericarditis, may lead to TEST-TAKING HINT: The test taker must iden-
surgery for valve replacement. tify the first of the HCP’s orders to be imple-
TEST-TAKING HINT: A concept that the test mented. “Infective” should indicate that this
taker must remember with any client being is an infection, which requires antibiotics,
discharged from the hospital should be to but the nurse should always assess for aller-
alternate rest with activity to avoid problems gies and obtain cultures prior to administer-
associated with immobility. If the test taker ing any antibiotic.
does not know the answer to a question, us- Content – Medical: Integrated Nursing Process –
ing basic concepts is the best option. Implementation: Client Needs – Safe Effective Care
Environment, Management of Care: Cognitive Level –
Application: Concept – Perfusion.

03_Colgrove_Ch03.indd 97 02/06/16 7:53 PM


CONCEPTS

73. 1. The HCP will be notified but the first action 75. 1. The client should be monitored for any
is to call for the Code Blue team and initiate cardiovascular changes.
CPR. 2. The client should be monitored for the
2. A Rapid Response Team is called to prevent development of heart failure as a result
an arrest situation from occurring. This client of increased strain on the heart from the
is in an arrest situation. atria not functioning as they should.
3. The client has clinical signs of death; CPR 3. There is no evidence that the client requires
must be initiated and the code team notified. to sleep in the orthopneic position.
4. The first action is to immediately no- 4. Buerger Allen exercises are useful for clients
tify the code team and initiate CPR per who have peripheral artery disease but do not
protocol. have an effect on atrial fibrillation.
TEST-TAKING HINT: The test taker should 5. Clients who have been diagnosed with
remember “If in stress do not assess.” The atrial fibrillation are at risk for develop-
nurse has enough information given in the ing emboli from the stasis of blood in the
stem of the question to initiate an action. The atria. If an emboli breaks loose from the
question asks for a first; all of the options lining of the atria then it can travel to the
may be implemented but only one is first. lungs (right) or to the brain (left).
Content – Medical: Integrated Nursing Process –
6. Clients who have been diagnosed with
Implementation: Client Needs – Physiological Integrity, atrial fibrillation are at risk for develop-
Physiological Adaptation: Cognitive Level – Synthesis: ing emboli from the stasis of blood in the
Concept – Perfusion. atria. If an emboli breaks loose from the
lining of the atria then it can travel to the
74. 1. Although this might be the cause of noncom- lungs (right) or to the brain (left).
pliance, actual side effects of antihypertensive
TEST-TAKING HINT: To answer “Select all that
medications may be more likely. Evidence indi-
­apply” questions the test taker should look
cates that the side effect of erectile dysfunction
at each option independently of the others.
is a major reason of noncompliance for males.
Each option becomes a true/false question.
2. This is a mild way of introducing the sub-
ject of side effects to a client not wishing Content – Medical: Integrated Nursing Process –
to admit the medication causes unwanted Implementation: Client Needs – Pharmacologic Integrity:
Cognitive Level – Analysis: Concept – Perfusion/Medication.
effects. It opens the door to more probing
assessment questions. The nurse should 76. 1. This indicates ascites, which can happen in
bring up the subject in order to allow the heart failure but does not necessarily do so;
client to be forthcoming with the issues of it can also be liver failure or another issue.
why he is not taking his medication. 2. Dyspnea occurring at night when the
3. This would be the second question to ask if the ­client is in a recumbent position indicates
client denies any problems with side effects. that the cardiac muscle is not able to
4. Although in this case the nurse can ask “why” compensate for extra fluid returning to
because it is an interview question and not the heart during sleep.
therapeutic conversation being requested in 3. This could indicate diabetes but not heart
the stem, a more direct question will open the failure.
conversation up better. 4. The client is at risk for heart failure as a result
TEST-TAKING HINT: To answer this question of the MI, but it does not happen with all MI
the test taker must remember that all medi- clients and does not support the diagnosis.
cations have potential side effects. Antihy- TEST-TAKING HINT: The test taker should
pertensive medications can cause erectile read the stem of the question carefully. It is
dysfunction in males, frequently resulting ­asking for assessment data to support the
in noncompliance with the medication regi- client is in heart failure. Three of the answer
men. The issue is a psychological as well as a options give assessment data; therefore,
physiological one. option “4” can be eliminated. Only one of the
Content – Medical: Integrated Nursing Process – other three gives an option that only occurs
Assessment: Client Needs – Pharmacologic Integrity: Cog- with heart failure.
nitive Level – Analysis: Concept – Perfusion/Medication.
98

03_Colgrove_Ch03.indd 98 02/06/16 7:53 PM


C hapter 3  C ardiac D isorders 99

Content – Medical: Integrated Nursing Process – 4. This strip indicates normal sinus rhythm;
Assessment: Client Needs – Physiological Integrity, there is no need for further action based on
Physiological Adaptation: Cognitive Level – Synthesis: the strip.
Concept – Perfusion.
TEST-TAKING HINT: The test taker should
77. 1. Nitroglycerin tablets are dispensed in recognize normal values and results in order
small brown bottles to preserve the po- to recognize abnormals. A normal result can
tency. The client should not change the rule out an answer in a “which do you assess
tablets to another container. first” question; an abnormal value automati-
2. The tablets are placed under the tongue to cally elevates the need to see that client be-
dissolve and thereby work more rapidly. fore another one.
3. The client is taught to take one (1) tablet ev- Content – Medical: Integrated Nursing Process –
ery 5 minutes and if the angina is not relieved Implementation: Client Needs – Physiological Integrity,
to call 911. Physiological Adaptation: Cognitive Level – Synthesis:
4. The tablets do not have a fruity odor; they Concept – Perfusion.
sting when placed under the tongue if they
80. 1. Amiodarone is administered during a code
are potent.
rapidly after being mixed in 100 mL of
TEST-TAKING HINT: The test taker should be fluid.
knowledgeable of common medications and 2. Amiodarone is not pushed; lidocaine is
what to teach the client. ­administered by this method. Amiodarone is
Content – Medical: Integrated Nursing Process – replacing the use of lidocaine during a code
Implementation: Client Needs – Physiological ­Integrity, because of evidence-based practice.
Physiological Adaptation: Cognitive Level – Synthesis: 3. Amiodarone is ACLS recommended.
Concept – Perfusion. 4. Dopamine is administered via mg/kg/min.
78. 1. The client has not arrested. The nurse might The time to calculate this kind of dosage is
call the rapid response team (RRT) but not a not taken until after the code is concluded
code blue. and the client is placed on a vasopressor
2. The client is in distress; the nurse should ­medication such as dopamine.
implement a procedure that will alleviate the TEST-TAKING HINT: The test taker should be
distress. knowledgeable of common medications and
3. The client is in distress; the nurse should basic rules of administration.
implement a procedure that will alleviate the Content – Medical: Integrated Nursing Process –
distress. Implementation: Client Needs – Pharmacological
4. The client began to have a problem Adaptation: Cognitive Level – Synthesis: Concept –
during physical exertion. Stopping the Perfusion/Medication.
exertion should be the first action taken
81. 1. “Has developed” indicates a new issue;
by the nurse.
the nurse should notify the HCP of the
TEST-TAKING HINT: The test taker should re- assessment findings, which indicate that
member “If in stress do not assess.” The nurse the client has developed heart failure.
has enough information given in the stem of 2. What the client ate has no bearing on the
the question to initiate an action. The ques- new development of the clinical manifesta-
tion asks for a first. All of the options may be tions of heart failure.
implemented but only one is first. 3. A 12-lead ECG will not treat heart failure.
Content – Medical: Integrated Nursing Process – 4. A diuretic may need to be administered but
Implementation: Client Needs – Physiological Integrity, notifying the HCP is first.
Physiological Adaptation: Cognitive Level – Synthesis:
TEST-TAKING HINT: The test taker should
Concept – Perfusion.
read every word in the stem of the question;
79. 1. This strip indicates normal sinus rhythm; “has developed” indicates a newly occurring
there is no need for further action based on situation for the client. The nurse must no-
the strip. tify the HCP when new issues occur in order
2. This strip indicates normal sinus rhythm; to intervene before a failure to rescue issue
there is no need for further action based on occurs.
the strip. Content – Medical: Integrated Nursing Process –
3. This strip indicates normal sinus rhythm; Implementation: Client Needs – Physiological Integrity,
there is no need for further action based Physiological Adaptation: Cognitive Level – Synthesis:
on the strip. The nurse should continue to Concept – Perfusion.
monitor the client.

03_Colgrove_Ch03.indd 99 02/06/16 7:53 PM


100 M ed -S urg S uccess

82. 1. The potassium level is within normal range; clothing (gown, underwear, jeans, etc.)
this medication would not be questioned. and report a weight gain of 3 pounds in a
2. Digoxin is given to clients with rapid atrial week to the HCP.
fibrillation to slow the heart rate; this medi- 4. The nurse should not assess for edema in
cation would not be questioned. the feet and lower legs, but if the client
3. Enalapril, an ACE inhibitor, will lower is in bed the lowest part of the body may
the blood pressure even more. The nurse be in the sacral area. Whichever area is
should hold the medication and notify the dependent is where the nurse should look
HCP that the medication is being held. for edema.
4. This would be the first medication to be 5. The client should drink enough fluids to
administered because it indicates a potential maintain body function, but 3,000 mL is
cardiac muscle perfusion issue. excessive.
TEST-TAKING HINT: The test taker should 6. Whenever the nurse is instructing a client,
recognize normal values and results in order the nurse should determine if the client
to recognize abnormals. A normal result can heard and understood the instructions.
rule out an answer in “Which do you assess Having the client repeat the instructions
first?” or “Which would the nurse question?” is one way of determining “hearing.”
An abnormal value indicates a need for some ­Having the client return demonstrate is a
action on the part of the nurse. method of determining understanding.
Content – Medical: Integrated Nursing Process – TEST-TAKING HINT: The new NCLEX-RN
Assessment: Client Needs – Pharmacological Integrity: Cog- test plan report states that “Select all that ap-
nitive Level – Synthesis: Concept – Perfusion/Medication. ply” questions may have five (5) to six (6) op-
tions, and one option must be correct but all
83. 1. Activity intolerance is a result of lack of per-
may be correct. In order to answer a “Select
fusion of the cardiac muscle, but the priority
all that apply” question each option is con-
is to get the muscle perfused.
sidered separately as a true/false question.
2. Pain does not kill anyone; the reason behind
the pain could. In the case of chest pain the Content – Medical: Integrated Nursing Process –
Planning: Client Needs – Physiological Integrity,
cardiac muscle is not being perfused, which
Physiological Adaptation: Cognitive Level – Synthesis:
causes the pain. Concept – Perfusion.
3. The problem is not having enough oxygen
available to the body but that the oxygen is 85. 1. This could be nothing serious (artifact) but
not being perfused to the cardiac muscle. from the appearance of the strip, the nurse
4. The cardiac muscle is not perfused when cannot tell if the client is in an arrest situ-
there is a narrowing of the arteries caused ation; the nurse must personally assess the
by CAD or when an embolus or a throm- situation. The nurse cannot delegate an un-
bus occludes the artery. Adequate perfu- stable client.
sion will supply oxygen to the cardiac 2. The nurse must determine the situation
muscle, allow for increased activity, and personally; this could be artifact or ven-
decrease pain. tricular fibrillation.
TEST-TAKING HINT: The test taker should 3. This could be nothing serious (artifact) but
remember basic pathophysiology to answer from the appearance of the strip, the nurse
this priority question. The other three inter- cannot tell if the client is in an arrest situa-
related concepts are based on the issue of tion; the nurse must personally assess the sit-
tissue perfusion. uation. Telemetry monitoring is accomplished
using lead II; the monitor tech does not have
Content – Medical: Integrated Nursing Process –
Planning: Client Needs – Physiological Integrity,
the ability to change the lead placement.
Physiological Adaptation: Cognitive Level – Synthesis: 4. The nurse must assess the client prior to
Concept – Perfusion. making a decision as to whether to notify the
code team or not.
84. 1. A dietitian can assist the nurse in
TEST-TAKING HINT: The rules for delegation
­explaining the sodium restrictions to the
are the nurse cannot delegate an unstable
client as well as hidden sources of sodium.
client, teaching, assessment, evaluation, or
2. This will help the client’s body to return
medication administration to a UAP. This
excess fluid to the heart for removal from
would eliminate option “1.” Calling a code
the body by the kidneys.
without assessing the client would eliminate
3. The client should weigh himself/­herself
option “4.”
every morning in the same type of

03_Colgrove_Ch03.indd 100 02/06/16 7:53 PM


C hapter 3  C ardiac D isorders 101

Content – Medical: Integrated Nursing Process – 2. The nurse should make sure the client is
Implementation: Client Needs – Physiological Integrity, aware of when sexual activity can be safely
Physiological Adaptation: Cognitive Level – Synthesis: resumed.
Concept – Perfusion/Collaboration. 3. The client needs to know how to take nitro-
86. 1. “Down and dirty” rounds include assess- glycerin but not the pharmacology of how the
ing each client for the main focus of the medication works.
client’s admission or any new issue that is 4. The client can sleep in any position of comfort.
reported from the shift report and assess- TEST-TAKING HINT: The test taker should
ing all lines and tubes going into or com- recognize certain words such as “any,” “all,”
ing out of the client. Once this is done the “never,” or “always.” These absolutes will de-
nurse knows then that the client is stable termine if an ­option is incorrect or correct.
and a full head-to-toe assessment can be Content – Medical: Integrated Nursing Process –
done at a later time. Planning: Client Needs – Physiological Integrity,
2. The UAP will determine when and how Physiological Adaptation: Cognitive Level – Synthesis:
to accomplish the job; the nurse may assist Concept – Perfusion.
the UAP by informing the UAP of situa-
89. 1. Digoxin is a routine medication that will be
tions which may impact the timing of the
administered at 0900 in most hospitals.
baths, but this is not the purpose of morning
2. The client intends on eating breakfast and
rounds.
this is a scheduled medication for before
3. This is the UAP’s job.
meals.
4. This is not the purpose of morning rounds.
3. This client is showing that the diuretic is
TEST-TAKING HINT: Option “3” has the word ­doing what it should do. This medication will
“all,” which could eliminate it from consider- be given at 0900.
ation because rarely does an “all” apply. 4. This is a slightly abnormal blood pressure but
Options “2” and “3” are doing the UAP’s job is in acceptable range for someone prescribed
and option “4” is the housekeeping’s job. an ARB, angiotensin receptor blocker. The
Content – Medical: Integrated Nursing Process – medication can be administered at 0900.
Assessment: Client Needs – Physiological Integrity, TEST-TAKING HINT: The test taker should be
Physiological Adaptation: Cognitive Level – Synthesis:
knowledgeable of common medications and
Concept – Perfusion/Collaboration.
the basic rules of administration.
87. 1. A complaint of indigestion could be car- Content – Medical: Integrated Nursing Process –
diac chest pain. The nurse should assess ­Implementation: Client Needs – Pharmacological
this client because of the diagnosis of Adaptation: Cognitive Level – Synthesis: Concept –
CAD and the word “severe” in the option. Perfusion/Medication.
2. Edema is expected for the client diag-
90. 1. CAD narrows the arteries of the heart,
nosed with heart failure, and it is not life
causing the tissues not to be perfused,
threatening.
­especially when an embolus or a thrombus
3. An irregular heart rate is not life threaten-
occurs.
ing, and 110 is abnormal but also not life
2. DI is a disease of the pituitary gland or the
threatening.
kidneys; it is not a perfusion issue.
4. Constipation is not life threatening albeit
3. COPD is an oxygenation issue, not a perfu-
uncomfortable.
sion one.
TEST-TAKING HINT: A first makes the test 4. Multiple fractures do not cause perfusion
taker determine which client has the greatest issues unless an interrelated issue occurs.
need. Expected and not life-threatening is-
TEST-TAKING HINT: The test taker should
sues do not require being a priority.
remember basic pathophysiology and the
Content – Medical: Integrated Nursing Process – resulting problems associated with different
Assessment: Client Needs – Physiological Integrity, pathology.
Physiological Adaptation: Cognitive Level – Synthesis:
Concept – Perfusion. Content – Medical: Client Needs – Physiological Integrity,
Physiological Adaptation: Cognitive Level – Synthesis:
88. 1. The word “any” makes this a wrong option. ­Concept – Perfusion.
The nurse should teach the client what to do
if chest pain occurs. Take one nitroglycerin
tablet every 5 minutes times three (3), and if
not relieved call 911.

03_Colgrove_Ch03.indd 101 02/06/16 7:53 PM


102 M ed -S urg S uccess

CARDIAC DISORDERS COMPREHENSIVE


EXAMINATION
1. Which population is at a higher risk for dying 6. To what area should the nurse place the
from a myocardial infarction? stethoscope to best auscultate the apical pulse?
1. Caucasian males.
2. Hispanic females.
3. Asian males.
4. African American females.
2. Which preprocedure information should be
taught to the female client having an exercise A B
stress test in the morning? C
1. Wear open-toed shoes to the stress test. D
2. Inform the client not to wear a bra.
3. Do not eat anything for four (4) hours.
4. Take the beta blocker one (1) hour before the
test.
3. Which intervention should the nurse implement
with the client diagnosed with dilated
cardiomyopathy? 1. A
1. Keep the client in the supine position with the 2. B
legs elevated. 3. C
2. Discuss a heart transplant, which is the 4. D
definitive treatment.
7. The telemetry nurse notes a peaked T wave for
3. Prepare the client for coronary artery bypass
the client diagnosed with congestive heart failure.
graft.
Which laboratory data should the nurse assess?
4. Teach the client to take a calcium channel
1. CK-MB.
blocker in the morning.
2. Troponin.
4. Which medical client problem should the nurse 3. BNP.
include in the plan of care for a client diagnosed 4. Potassium.
with cardiomyopathy?
8. The client comes to the emergency department
1. Heart failure.
saying, “I am having a heart attack.” Which
2. Activity intolerance.
question is most pertinent when assessing the
3. Powerlessness.
client?
4. Anticipatory grieving.
1. “Can you describe your chest pain?”
5. The client has an implantable cardioverter 2. “What were you doing when the pain started?”
defibrillator (ICD). Which discharge instructions 3. “Did you have a high-fat meal today?”
should the nurse teach the client? 4. “Does the pain get worse when you lie down?”
1. Do not lift or carry more than 23 kg.
2. Have someone drive the car for the rest of your
life.
3. Carry the cell phone on the opposite side of
the ICD.
4. Avoid using the microwave oven in the home.

03_Colgrove_Ch03.indd 102 02/06/16 7:53 PM


C hapter 3  C ardiac D isorders 103

9. The client with coronary artery disease is 14. Which laboratory data confirm the diagnosis of
prescribed transdermal nitroglycerin, a coronary congestive heart failure?
vasodilator. Which behavior indicates the client 1. Chest x-ray (CXR).
understands the discharge teaching concerning 2. Liver function tests.
this medication? 3. Blood urea nitrogen (BUN).
1. The client places the medication under the 4. Beta-type natriuretic peptide (BNP).
tongue.
15. What is the priority problem in the client
2. The client removes the old patch before
diagnosed with congestive heart failure?
placing the new.
1. Fluid volume overload.
3. The client applies the patch to a hairy area.
2. Decreased cardiac output.
4. The client changes the patch every 36 hours.
3. Activity intolerance.
10. Which client would most likely be 4. Knowledge deficit.
misdiagnosed for having a myocardial
16. Which data would cause the nurse to question
infarction?
administering digoxin to a client diagnosed with
1. A 55-year-old Caucasian male with crushing
congestive heart failure?
chest pain and diaphoresis.
1. The potassium level is 3.2 mEq/L.
2. A 60-year-old Native American male with an
2. The digoxin level is 1.2 mcg/mL.
elevated troponin level.
3. The client’s apical pulse is 64.
3. A 40-year-old Hispanic female with a normal
4. The client denies yellow haze.
electrocardiogram.
4. An 80-year-old Peruvian female with a normal 17. The nurse is caring for clients on a cardiac floor.
CK-MB at 12 hours. Which client should the nurse assess first?
1. The client with three (3) unifocal PVCs in
11. Which meal would indicate the client
one (1) minute.
understands the discharge teaching concerning
2. The client diagnosed with coronary artery
the recommended diet for coronary artery
disease who wants to ambulate.
disease?
3. The client diagnosed with mitral valve
1. Baked fish, steamed broccoli, and garden
prolapse with an audible S3.
salad.
4. The client diagnosed with pericarditis who is
2. Enchilada dinner with fried rice and refried
in normal sinus rhythm.
beans.
3. Tuna salad sandwich on white bread and 18. The nurse is told in report the client has aortic
whole milk. stenosis. Which anatomical position should the
4. Fried chicken, mashed potatoes, and gravy. nurse auscultate to assess the murmur?
1. Second intercostal space, right sternal notch.
12. The unlicensed assistive personnel (UAP) tells
2. Erb’s point.
the primary nurse that the client diagnosed with
3. Second intercostal space, left sternal notch.
coronary artery disease is having chest pain.
4. Fourth intercostal space, left sternal border.
Which action should the nurse take first?
1. Tell the UAP to go take the client’s vital signs. 19. The nurse is caring for a client who goes into
2. Ask the UAP to have the telemetry nurse read ventricular tachycardia. Which intervention
the strip. should the nurse implement first?
3. Notify the client’s health-care provider. 1. Call a code immediately.
4. Go to the room and assess the client’s chest 2. Assess the client for a pulse.
pain. 3. Begin chest compressions.
4. Continue to monitor the client.
13. Which interventions should the nurse discuss
with the client diagnosed with coronary artery 20. The nurse is assisting with a synchronized
disease? Select all that apply. cardioversion on a client in atrial fibrillation.
1. Instruct the client to stop smoking. When the machine is activated, there is a pause.
2. Encourage the client to exercise three (3) days What action should the nurse take?
a week. 1. Wait until the machine discharges.
3. Teach about coronary vasodilators. 2. Shout “all clear” and don’t touch the bed.
4. Prepare the client for a carotid 3. Make sure the client is all right.
endarterectomy. 4. Increase the joules and redischarge.
5. Eat foods high in monosaturated fats.

03_Colgrove_Ch03.indd 103 02/06/16 7:53 PM


104 M ed -S urg S uccess

21. The client is diagnosed with pericarditis. When 24. The client is three (3) hours post–myocardial
assessing the client, the nurse is unable to infarction. Which data would warrant
auscultate a friction rub. Which action should immediate intervention by the nurse?
the nurse implement? 1. Bilateral peripheral pulses 2+.
1. Notify the health-care provider. 2. The pulse oximeter reading is 96%.
2. Document that the pericarditis has resolved. 3. The urine output is 240 mL in the last four
3. Ask the client to lean forward and listen again. (4) hours.
4. Prepare to insert a unilateral chest tube. 4. Cool, clammy, diaphoretic skin.
22. The nurse assessing the client with pericardial 25. The nurse is transcribing the doctor’s orders for
effusion at 1600 notes the apical pulse is 72 and a client with congestive heart failure. The order
the BP is 138/94. At 1800, the client has neck reads 2.5 mg of Lanoxin daily. Which action
vein distention, the apical pulse is 70, and the should the nurse implement?
BP is 106/94. Which action would the nurse 1. Discuss the order with the health-care
implement first? provider.
1. Stay with the client and use a calm voice. 2. Take the client’s apical pulse rate before
2. Notify the health-care provider immediately. administering.
3. Place the client left lateral recumbent. 3. Check the client’s potassium level before
4. Administer morphine intravenous push slowly. giving the medication.
4. Determine if a digoxin level has been drawn.
23. The client is admitted to the emergency
department, and the nurse suspects a cardiac 26. According to the 2010 American Heart
problem. Which assessment interventions should Association Guidelines, which steps of
the nurse implement? Select all that apply. cardiopulmonary resuscitation for an adult
1. Obtain a midstream urine specimen. suffering from a cardiac arrest should the nurse
2. Attach the telemetry monitor to the client. teach individuals in the community? Rank in
3. Start a saline lock in the right arm. order of performance.
4. Draw a basal metabolic panel (BMP). 1. Place the hands over the lower half of the
5. Request an order for a STAT 12-lead ECG. sternum.
2. Look for obvious signs of breathing.
3. Begin compressions at a ratio of 30:2.
4. Call for and AED immediately.
5. Position the victim on the back.

03_Colgrove_Ch03.indd 104 02/06/16 7:53 PM


C hapter 3  C ardiac D isorders 105

27. The nurse has received report when the telemetry technician notifies the nurse of the
telemetry readings. Which client should the nurse assess first?
1.
6 seconds

2.

6 seconds

3.

6 seconds

4.
6 seconds

03_Colgrove_Ch03.indd 105 02/06/16 7:54 PM


CARDIAC DISORDERS COMPREHENSIVE
EXAMINATION ANSWERS AND RATIONALES

1. 1. Caucasian males have a high rate of coro- 4. Calcium channel blockers are contraindicated
nary artery disease, but they do not delay in clients with dilated cardiomyopathy because
seeking health care as long as some other they interfere with the contractility of the
ethnic groups. The average delay time is five heart.
(5) hours. Content – Medical: Integrated Nursing Process –
2. Hispanic females are at higher risk for diabetes Implementation: Client Needs – Physiological ­Integrity,
than for dying from a myocardial infarction. Physiological Adaptation: Cognitive Level – Application:
3. Asian males have fewer cardiovascular events, Concept – Perfusion.
which is attributed to their diet, which is high 4. 1. Medical client problems indicate the nurse
in fiber and omega-3 fatty acids. and the physician must collaborate to care
4. African American females are 35% more for the client; the client must have medica-
likely to die from coronary artery disease tions for heart failure.
than any other population. This population 2. The nurse can instruct the client to pace
has significantly higher rates of hyperten- ­activities and can teach about rest versus activ-
sion and it occurs at a younger age. The ity without a health care provider order.
higher risk of death from an MI is also at- 3. This is a psychosocial client problem that does
tributed to a delay in seeking emergency not require a physician’s order to effectively
care—an average of 11 hours.
care for the client.
Content – Medical: Integrated Nursing Process – 4. Anticipatory grieving involves the nurse ad-
Assessment: Client Needs – Health Promotion and Mainte- dressing issues that will occur based on the
nance: Cognitive Level – Knowledge: Concept – Perfusion.
knowledge of the poor prognosis of this
2. 1. The client should wear firm-fitting, solid disease.
­athletic shoes. Content – Medical: Integrated Nursing Process –
2. The client should wear a bra to provide Diagnosis: Client Needs – Physiological Integrity,
­adequate support during the exercise. Physiological Adaptation: Cognitive Level – Analysis:
3. NPO decreases the chance of aspiration in Concept – Perfusion.
case of emergency. In addition, if the client 5. 1. Clients should not lift more than 5 to
has just had a meal, the blood supply will 10 pounds because it puts a strain on the
be shunted to the stomach for digestion heart; 23 kg is more than 50 pounds.
and away from the heart, perhaps leading 2. There may be driving restrictions, but the
to an inaccurate test result. ­client should be able to drive independently.
4. A beta blocker is not taken prior to the stress 3. Cell phones may interfere with the func-
test because it will decrease the pulse rate tioning of the ICD if they are placed too
and blood pressure by direct parasympathetic close to it.
stimulation to the heart. 4. Microwave ovens should not cause problems
Content – Medical: Integrated Nursing Process – with the ICD.
Planning: Client Needs – Physiological Integrity,
Content – Medical: Integrated Nursing Process –
­Reduction of Risk Potential: Cognitive Level – ­Synthesis:
Planning: Client Needs – Physiological Integrity,
Concept – Perfusion.
Physiological Adaptation: Cognitive Level – Synthesis:
3. 1. Most clients with dilated cardiomyopathy pre- Concept – Perfusion.
fer to sit up with their legs in the dependent 6. 1. This is the best place to auscultate the aortic
position. This position causes pooling of blood valve, the second intercostal space, right
in the periphery and reduces preload. sternal notch.
2. Without a heart transplant, this client 2. This is the best place to auscultate the
will end up in end-stage heart failure. A pulmonic valve, the second intercostal space,
­transplant is the only treatment for a ­client left sternal notch.
with dilated cardiomyopathy. 3. This is the best place to auscultate the tricus-
3. A bypass is the treatment of choice for a c­ lient pid valve, the third intercostal space, left ster-
with occluded coronary arteries. nal border.

106

03_Colgrove_Ch03.indd 106 02/06/16 7:54 PM


C hapter 3  C ardiac D isorders 107

4. The best place to auscultate the apical Content – Medical: Integrated Nursing Process –
pulse is over the mitral valve area, which Evaluation: Client Needs – Physiological Integrity, Pharma-
is the fifth intercostal space, midclavicular cological and Parenteral Therapies: Cognitive Level – Analy-
line. sis: Concept – Medication.
Content – Medical: Integrated Nursing Process – 10. 1. Crushing pain and sweating are classic signs
Assessment: Client Needs – Physiological Integrity, of an MI and should not be misdiagnosed.
­Reduction of Risk Potential: Cognitive Level – ­Analysis: 2. An elevated troponin level is a benchmark
Concept – Perfusion. in diagnosing an MI and should not be
7. 1. CK-MB is assessed to determine if the client misdiagnosed.
has had a myocardial infarction. The electri- 3. The clients who are misdiagnosed
cal activity of the heart will not be affected by ­concerning MIs usually present with atyp-
elevation of this enzyme. ical symptoms. They tend to be ­female, be
2. Troponin is assessed to determine if the client younger than 55 years old, be members
has had a myocardial infarction. The electri- of a minority group, and have normal
cal activity of the heart will not be affected by electrocardiograms.
elevation of this enzyme. 4. CK-MB may not elevate until up to 24 hours
3. Beta-type natriuretic peptide (BNP) is elevated after onset of chest pain.
in clients with congestive heart failure, but Content – Medical: Integrated Nursing Process –
it does not affect the electrical activity of the Diagnosis: Client Needs – Safe Effective Care
heart. Environment, Management of Care: Cognitive
4. Hyperkalemia will cause a peaked T wave; Level – Knowledge: Concept – Perfusion.
therefore, the nurse should check these 11. 1. The recommended diet for CAD is low
laboratory data. fat, low cholesterol, and high fiber. The
Content – Medical: Integrated Nursing Process – diet described is a diet that is low in fat
Assessment: Client Needs – Safe Effective Care and cholesterol.
Environment, Management of Care: Cognitive 2. This is a diet very high in fat and cholesterol.
Level – Analysis: Concept – Perfusion. 3. The word “salad” implies something has been
8. 1. The chest pain for an MI usually is de- mixed with the tuna, usually mayonnaise,
scribed as an elephant sitting on the chest which is high in fat, but even if the test taker
or a belt squeezing the substernal midchest, did not know this, white bread is low in fiber
often radiating to the jaw or left arm. and whole milk is high in fat.
2. This helps to identify if it is angina ­(resulting 4. Meats should be baked, broiled, or grilled—
from activity) or MI (not necessarily brought not fried. Gravy is high in fat.
on by activity). Content – Medical: Integrated Nursing Process –
3. Learning about a client’s intake of a high-fat Evaluation: Client Needs – Physiological Integrity,
meal would help the nurse to identify a gall- Basic Care and Comfort: Cognitive Level – Synthesis:
bladder attack. Concept – Nutrition.
4. This is a question the nurse might ask the 12. 1. The client with CAD who is having chest
­client with reflux esophagitis. pain is unstable and requires further judg-
Content – Medical: Integrated Nursing Process – ment to determine appropriate actions
Assessment: Client Needs – Safe Effective Care to take, and the UAP does not have that
Environment, Management of Care: Cognitive knowledge.
Level – Analysis: Concept – Assessment. 2. The UAP could go ask the telemetry nurse,
9. 1. The client does not understand how to apply but this is not the first action.
this medication; it is placed on the skin, not 3. The client’s HCP may need to be notified,
under the tongue. but this is not the first intervention.
2. This behavior indicates the client under- 4. Assessment is the first step in the nursing
stands the discharge teaching. process and should be implemented first;
3. The patch needs to be in a nonhairy place so it chest pain is priority.
makes good contact with the skin. Content – Medical: Integrated Nursing Process –
4. The patch should be changed every 12 or Implementation: Client Needs – Safe Effective Care
24 hours but never every two (2) hours. It Environment, Management of Care: Cognitive
takes two (2) hours for the patch to warm up Level – Synthesis: Concept – Assessment.
and begin delivering the optimum dose of
medication.

03_Colgrove_Ch03.indd 107 02/06/16 7:54 PM


108 M ed -S urg S uccess

13. 1. Smoking is the one risk factor that must be 2. This digoxin level is within therapeutic
stopped totally; there is no compromise. range, 0.5 to 2 mcg/mL.
2. Exercising helps develop collateral circu- 3. The nurse would question the medication
lation and decrease anxiety; it also helps if the apical pulse were less than 60.
clients to lose weight. 4. Yellow haze is a sign of digoxin toxicity.
3. Clients with coronary artery disease are Content – Medical: Integrated Nursing Process –
usually prescribed nitroglycerin, which is Assessment: Client Needs – Physiological Integrity,
the treatment of choice for angina. Pharmacological and Parenteral Therapies: Cognitive
4. Carotid endarterectomy is a procedure to re- Level – Analysis: Concept – Medication.
move atherosclerotic plaque from the carotid 17. 1. Three (3) unifocal PVCs in one (1) minute is
arteries, not the coronary arteries. not life threatening.
5. The client should eat polyunsaturated fats, 2. The client wanting to ambulate is not a prior-
not monosaturated fats, to help decrease ity over a client with a physiological problem.
atherosclerosis. 3. An audible S3 indicates the client is devel-
Content – Medical: Integrated Nursing Process – oping left-sided heart failure and needs to
Planning: Client Needs – Physiological Integrity, be assessed immediately.
Physiological Adaptation: Cognitive Level – Synthesis:
4. A client in normal sinus rhythm will not be a
Concept – Perfusion.
priority over someone with a potentially life-
14. 1. The CXR will show an enlarged heart, but threatening situation.
it is not used to confirm the diagnosis of Content – Medical: Integrated Nursing Process –
­congestive heart failure. Assessment: Client Needs – Safe Effective Care
2. Liver function tests may be ordered to evalu- Environment, Management of Care: Cognitive
ate the effects of heart failure on the liver, but Level – Analysis: Concept – Assessment.
they do not confirm the diagnosis.
18. 1. The second intercostal space, right ster-
3. The BUN is elevated in heart failure, dehy- nal notch, is the area on the chest where
dration, and renal failure, but it is not used to
the aorta can best be heard opening and
confirm congestive heart failure.
closing.
4. BNP is a hormone released by the heart
2. Erb’s point allows the nurse to hear the
muscle in response to changes in blood
­opening and closing of the tricuspid valve.
volume and is used to diagnose and grade
3. The second intercostal space, left sternal
heart failure.
notch, is the area on the chest where the pul-
Content – Medical: Integrated Nursing Process – monic valve can best be heard opening and
Assessment: Client Needs – Physiological Integrity, closing.
Reduction of Risk Potential: Cognitive Level –
4. The fourth intercostal space, left sternal bor-
Analysis: Concept – Perfusion.
der, is another area on the chest that can as-
15. 1. Fluid volume overload is a problem in cli- sess the tricuspid valve.
ents with congestive heart failure, but it is Content – Medical: Integrated Nursing Process –
not priority because, if the cardiac output Assessment: Client Needs – Physiological Integrity,
is ­improved, then the kidneys are perfused, Reduction of Risk Potential: Cognitive Level –
which leads to elimination of excess fluid Analysis: Concept – Perfusion.
from the body. 19. 1. The nurse should call a code if the client does
2. Decreased cardiac output is responsible not have vital signs.
for all the signs/symptoms associated with 2. The nurse must first determine if the
CHF and eventually causes death, which is client has a pulse. Pulseless ventricular
why it is the priority problem. tachycardia is treated as ventricular fibril-
3. Activity intolerance alters quality of life, but
lation. Stable ventricular tachycardia is
it is not life threatening. treated with medications.
4. Knowledge deficit is important, but it is not 3. Chest compression is only done if the client is
priority over a physiological problem. not breathing and has no pulse.
Content – Medical: Integrated Nursing Process – 4. Ventricular tachycardia is a potentially life-
Diagnosis: Client Needs – Physiological Integrity, threatening dysrhythmia and needs to be
Physiological Adaptation: Cognitive Level – Analysis:
treated immediately.
Concept – Perfusion.
Content – Medical: Integrated Nursing Process –
16. 1. This potassium level is below normal levels; Implementation: Client Needs – Safe Effective Care
hypokalemia can potentiate digoxin toxicity Environment, Management of Care: Cognitive
and lead to cardiac dysrhythmias. Level – Synthesis: Concept – Perfusion.

03_Colgrove_Ch03.indd 108 02/06/16 7:54 PM


C hapter 3  C ardiac D isorders 109

20. 1. Cardioversion involves the delivery of a 4. Morphine would be given to a client with
timed electrical current. The electrical pain from myocardial infarction; it is not a
impulse discharges during ventricular treatment option for cardiac tamponade.
depolarization and, therefore, there might Content – Medical: Integrated Nursing Process –
be a short delay. The nurse should wait Implementation: Client Needs – Safe Effective Care
until it discharges. Environment, Management of Care: Cognitive Level –
2. Calling “all clear” and not touching the bed Synthesis: Concept – Perfusion.
should be done prior to activating the ma- 23. 1. A midstream urine specimen is ordered for a
chine to discharge the electrical current. client with a possible urinary tract infection,
3. A pause is an expected event, and asking if not for a client with cardiac problems.
the client is all right may worry the client 2. Anytime a nurse suspects cardiac prob-
unnecessarily. lems, the electrical conductivity of the
4. Increasing joules and redischarging is imple- heart should be assessed.
mented during defibrillation, not during syn- 3. Emergency medications for heart prob-
chronized cardioversion. lems are primarily administered intra-
Content – Medical: Integrated Nursing Process – venously, so starting a saline lock in the
Implementation: Client Needs – Safe Effective Care right arm is appropriate.
Environment, Management of Care: Cognitive
4. This serum blood test is not specific to assess
Level – Application: Concept – Perfusion.
cardiac problems. A BMP evaluates potas-
21. 1. These assessment data are not life threaten- sium, sodium, glucose, and more.
ing and do not warrant notifying the HCP. 5. A 12-lead ECG evaluates the electrical
2. The nurse should attempt to hear the friction conductivity of the heart from all planes.
rub in multiple ways before documenting that Content – Medical: Integrated Nursing Process –
it is not heard. The nurse does not determine Implementation: Client Needs – Safe Effective Care
if pericarditis has resolved. Environment, Management of Care: Cognitive
3. Having the client lean forward and to the Level – Analysis: Concept – Assessment.
left uses gravity to force the heart nearer
24. 1. This pulse indicates the heart is pumping ad-
to the chest wall, which allows the friction
equately. Normal pulses should be 2+ to 3+.
rub to be heard.
2. A pulse oximeter reading of greater than 93%
4. Chest tubes are not the treatment of choice indicates the heart is perfusing the periphery.
for not hearing a friction rub. 3. An output of 30 mL/hr indicates the heart is
Content – Medical: Integrated Nursing Process – perfusing the kidneys adequately.
Implementation: Client Needs – Physiological 4. Cold, clammy skin is an indicator of car-
Integrity, Reduction of Risk Potential: Cognitive
diogenic shock, which is a complication of
Level – Application: Concept – Perfusion.
MI and warrants immediate intervention.
22. 1. This is a medical emergency; the nurse Content – Medical: Integrated Nursing Process –
should stay with the client, keep him calm, Assessment: Client Needs – Physiological Integrity,
and call the nurses’ station to notify the Reduction of Risk Potential: Cognitive Level – Synthesis:
health-care provider. Cardiac output de- Concept – Perfusion.
clines with each contraction as the peri- 25. 1. This dosage is 10 times the normal dose
cardial sac constricts the myocardium. for a client with CHF. This dose is poten-
2. The client’s signs/symptoms would make the tially lethal.
nurse suspect cardiac tamponade, a medical 2. No other action can be taken because of the
emergency. The pulse pressure is narrowing, incorrect dose.
and the client is experiencing severe rising 3. No other action can be taken because of the
central venous pressure as evidenced by neck incorrect dose.
vein distention. Notifying the health-care 4. No other action can be taken because of the
provider is important, but the nurse should incorrect dose.
stay with the client first.
Content – Medical: Integrated Nursing Process –
3. A left lateral recumbent position is used when
Implementation: Client Needs – Physiological Integrity,
administering enemas. Pharmacological and Parenteral Therapies: Cognitive
Level – Application: Concept – Medication.

03_Colgrove_Ch03.indd 109 02/06/16 7:54 PM


110 M ed -S urg S uccess

26. In order of performance: 5, 2, 4, 1, 3 27. 1. Atrial fibrillation is not life threatening.


5. The victim is positioned on the back for 2. Unifocal PVCs are common in most clients
assessment and for the rescuer to be able and are not life threatening. Up to six (6)
to begin cardiopulmonary resuscitation. per minute may occur before the nurse must
2. Although we now perform a quick look to intervene.
determine if the victim is breathing there 3. This is complete heart block with brady-
is no longer a “look, listen, feel” step. The cardia, a potentially life-threatening situ-
victim may not be breathing at all or may ation. The nurse should assess this client
be having agonal respirations. first and make interventions accordingly.
4. For adults the rescuer should immedi- 4. Sinus tachycardia may occur for different
ately call for an AED or 911. Research reasons, such as ambulating to the bathroom,
has proven the faster that defibrillation is fever, or anemia. This is not life threatening.
­performed the better the chance of sur- Content – Medical: Integrated Nursing Process –
vival for the victim. Assessment: Client Needs – Physiological Integrity,
1. Compressions are initiated immediately ­Physiological Adaptation: Cognitive Level – Synthesis:
because the victim will have some residual ­Concept – Perfusion.
oxygen in the lungs. Breathing is not
initiated unless there is a barrier device
available.
3. The compression rate is 30:2.
Content – Fundamentals: Integrated Nursing
Process – Implementation: Client Needs – Safe
Effective Care Environment, Management of Care:
Cognitive Level – Knowledge: Concept – Perfusion.

03_Colgrove_Ch03.indd 110 02/06/16 7:54 PM


Peripheral Vascular
Disorders
Knowledge is a process of piling up facts; wisdom lies in their
simplification.
4
—Martin Fischer

Peripheral vascular disorders (PVDs) are very common disorders, affecting many people
in the United States. Among the most common of these are hypertension, atherosclerosis,
arterial occlusive disease, abdominal aortic aneurysms (AAA), deep vein thrombosis (DVT),
and venous insufficiency. These disorders cause pain and an interruption of activities and
sometimes lead to life-threatening complications.

KEYWORDS     ABBREVIATIONS
Bruit Abdominal aortic aneurysm (AAA)
Dehiscence Angiotensin-converting enzyme (ACE)
Dependent position Blood pressure (BP)
Doppler device Blood urea nitrogen (BUN)
Evisceration Capillary refill time (CRT)
Intermittent claudication Coronary artery disease (CAD)
Neuropathy Deep vein thrombosis (DVT)
Petechiae Electrocardiogram (ECG)
Sequential compression device Guillain-Barré (GB) syndrome
Health-care provider (HCP)
Heparin-induced thrombocytopenia (HIT)
High-density lipoprotein (HDL)
International normalized ratio (INR)
Intravenous (IV)
Low-density lipoprotein (LDL)
Low molecular weight heparin (LMWH)
Nothing by mouth (NPO)
Partial thromboplastin time (PTT)
Peripheral vascular disease (PVD)
Physical therapy (PT)
Prothrombin time (PT)
Range of motion (ROM)
Unlicensed assistive personnel (UAP)
When required, as needed (prn)

111

04_Colgrove_Ch04.indd 111 02/06/16 7:57 PM


112 M ed -S urg S uccess

PRACTICE QUESTIONS
Arterial Hypertension 5. The male client diagnosed with essential
hypertension has been prescribed an
1. The 66-year-old male client has his blood alpha-adrenergic blocker. Which intervention
pressure (BP) checked at a health fair. The BP is should the nurse discuss with the client?
168/98. Which action should the nurse implement 1. Eat at least one (1) banana a day to help
first? increase the potassium level.
1. Recommend that the client have his blood 2. Explain that impotence is an expected side
pressure checked in one (1) month. effect of the medication.
2. Instruct the client to see his health-care 3. Take the medication on an empty stomach to
provider as soon as possible. increase absorption.
3. Discuss the importance of eating a low-salt, 4. Change position slowly when going from a
low-fat, low-cholesterol diet. lying to sitting position.
4. Explain that this BP is within the normal range 6. The nurse just received the a.m. shift report.
for an elderly person. Which client should the nurse assess first?
2. The nurse is teaching the client recently 1. The client diagnosed with coronary artery
diagnosed with essential hypertension. Which disease who has a BP of 170/100.
instruction should the nurse provide when 2. The client diagnosed with DVT who is
discussing heart healthy exercise? complaining of chest pain.
1. Walk at least 30 minutes a day on flat surfaces. 3. The client diagnosed with pneumonia who has
2. Perform light weight lifting three (3) times a a pulse oximeter reading of 98%.
week. 4. The client diagnosed with ulcerative colitis
3. Recommend high-intensity aerobics daily. who has non-bloody diarrhea.
4. Encourage the client to swim laps once a week. 7. The client diagnosed with essential hypertension
3. The health-care provider prescribes an asks the nurse, “Why do I have high blood
angiotensin-converting enzyme (ACE) pressure?” Which response by the nurse would be
inhibitor for the client diagnosed with essential most appropriate?
hypertension. Which statement is the most 1. “You probably have some type of kidney
appropriate rationale for administering this disease that causes the high BP.”
medication? 2. “More than likely you have had a diet high in
1. ACE inhibitors prevent beta receptor salt, fat, and cholesterol.”
stimulation in the heart. 3. “There is no specific cause for
2. This medication blocks the alpha receptors in hypertension, but there are many
the vascular smooth muscle. known risk factors.”
3. ACE inhibitors prevent vasoconstriction and 4. “You are concerned that you have high blood
sodium and water retention. pressure. Let’s sit down and talk.”
4. ACE inhibitors decrease blood pressure by 8. The nurse is teaching the Dietary Approaches
relaxing vascular smooth muscle. to Stop Hypertension (DASH) diet to a
4. The nurse is administering a beta blocker to the client diagnosed with essential hypertension.
client diagnosed with essential hypertension. Which statement indicates that the client
Which intervention should the nurse implement? understands teaching concerning the
1. Notify the health-care provider if the DASH diet?
potassium level is 3.8 mEq. 1. “I should eat at least four (4) to five (5) servings
2. Question administering the medication if the of vegetables a day.”
BP is less than 90/60 mm Hg. 2. “I should eat meat that has a lot of white
3. Do not administer the medication if the client’s streaks in it.”
radial pulse is greater than 100. 3. “I should drink no more than two (2) glasses of
4. Monitor the client’s BP while he or she is lying, whole milk a day.”
standing, and sitting. 4. “I should decrease my grain intake to no more
than twice a week.”

04_Colgrove_Ch04.indd 112 02/06/16 7:57 PM


C hapter 4  P eripheral V ascular D isorders 113

9. The client diagnosed with essential hypertension 14. Which instruction should the nurse include
is taking a loop diuretic daily. Which assessment when providing discharge instructions to a client
data would require immediate intervention by diagnosed with peripheral arterial disease?
the nurse? 1. Encourage the client to use a heating pad on
1. The telemetry reads normal sinus rhythm. the lower extremities.
2. The client has a weight gain of 2 kg within 2. Demonstrate to the client the correct way to
1 to 2 days. apply elastic support hose.
3. The client’s blood pressure is 148/92. 3. Instruct the client to walk daily for at least 30
4. The client’s serum potassium level is minutes.
4.5 mEq. 4. Tell the client to check both feet for red areas
at least once a week.
10. The client diagnosed with essential hypertension
asks the nurse, “I don’t know why the doctor 15. The nurse is teaching the client diagnosed with
is worried about my blood pressure. I feel just arterial occlusive disease. Which interventions
great.” Which statement by the nurse would be should the nurse include in the teaching? Select
the most appropriate response? all that apply.
1. “Damage can be occurring to your heart and 1. Wash legs and feet daily in warm water.
kidneys even if you feel great.” 2. Apply moisturizing cream to feet.
2. “Unless you have a headache, your blood 3. Buy shoes in the morning hours only.
pressure is probably within normal limits.” 4. Do not wear any type of knee stocking.
3. “When is the last time you saw your doctor? 5. Wear clean white cotton socks.
Does he know you are feeling great?”
16. Which assessment data would warrant
4. “Your blood pressure reflects how well your
immediate intervention in the client diagnosed
heart is working.”
with arterial occlusive disease?
11. The intensive care department nurse is 1. The client has 2+ pedal pulses.
calculating the total intake for a client diagnosed 2. The client is able to move the toes.
with hypertensive crisis. The client has received 3. The client has numbness and tingling.
880 mL of D5W, IVPB of 100 mL of 0.9% NS, 4. The client’s feet are red when standing.
8 ounces of water, 4 ounces of milk, and 6 ounces
17. Which client problem would be priority in a
of chicken broth. The client has had a urinary
client diagnosed with arterial occlusive disease
output of 1,480 mL. What is the total intake for
who is admitted to the hospital with a foot ulcer?
this client? _______
1. Impaired skin integrity.
12. The nurse is teaching a class on arterial essential 2. Activity intolerance.
hypertension. Which modifiable risk factors 3. Ineffective health maintenance.
would the nurse include when preparing this 4. Risk for peripheral neuropathy.
presentation?
18. The client diagnosed with arterial occlusive
1. Include information on retinopathy and
disease is one (1) day postoperative right
nephropathy.
femoral-popliteal bypass. Which intervention
2. Discuss sedentary lifestyle and smoking
should the nurse implement?
cessation.
1. Keep the right leg in the dependent position.
3. Include discussions on family history and
2. Apply sequential compression devices to
gender.
lower extremities.
4. Provide information on a low-fiber and high-
3. Monitor the client’s pedal pulses every shift.
salt diet.
4. Assess the client’s leg dressing every four (4)
hours.
Arterial Occlusive Disease 19. The nurse is unable to assess a pedal pulse in the
client diagnosed with arterial occlusive disease.
13. The client comes to the clinic complaining of Which intervention should the nurse implement
muscle cramping and pain in both legs when first?
walking for short periods of time. Which 1. Complete a neurovascular assessment.
medical term would the nurse document in the 2. Use the Doppler device.
client’s record? 3. Instruct the client to hang the feet off the side
1. Peripheral vascular disease. of the bed.
2. Intermittent claudication. 4. Wrap the legs in a blanket.
3. Deep vein thrombosis.
4. Dependent rubor.

04_Colgrove_Ch04.indd 113 02/06/16 7:57 PM


114 M ed -S urg S uccess

20. The wife of a client with arterial occlusive should the nurse discuss when teaching about
disease tells the nurse, “My husband says he atherosclerosis?
is having rest pain. What does that mean?” 1. Stress.
Which statement by the nurse would be most 2. Age.
appropriate? 3. Gender.
1. “It describes the type of pain he has when he 4. Family history.
stops walking.”
26. The client asks the nurse, “My doctor just told
2. “His legs are deprived of oxygen during
me that atherosclerosis is why my legs hurt
periods of inactivity.”
when I walk. What does that mean?” Which
3. “You are concerned that your husband is
response by the nurse would be the best
having rest pain.”
response?
4. “This term is used to support that his
1. “The muscle fibers and endothelial lining of
condition is getting better.”
your arteries have become thickened.”
21. The nurse is assessing the client diagnosed with 2. “The next time you see your HCP, ask what
long-term arterial occlusive disease. Which atherosclerosis means.”
assessment data support the diagnosis? 3. “The valves in the veins of your legs are
1. Hairless skin on the legs. incompetent so your legs hurt.”
2. Brittle, flaky toenails. 4. “You have a hardening of your arteries that
3. Petechiae on the soles of feet. decreases the oxygen to your legs.”
4. Nonpitting ankle edema.
27. The client diagnosed with peripheral vascular
22. The health-care provider ordered a femoral disease is overweight, has smoked two (2) packs
angiogram for the client diagnosed with arterial of cigarettes a day for 20 years, and sits behind a
occlusive disease. Which intervention should the desk all day. What is the strongest factor in the
nurse implement? development of atherosclerotic lesions?
1. Explain that this procedure will be done at the 1. Being overweight.
bedside. 2. Sedentary lifestyle.
2. Discuss with the client that he or she will be 3. High-fat, high-cholesterol diet.
on bedrest with bathroom privileges. 4. Smoking cigarettes.
3. Inform the client that no intravenous access
28. The client tells the nurse that his cholesterol
will be needed.
level is 240 mg/dL. Which action should the
4. Inform the client that fluids will be increased
nurse implement?
after the procedure.
1. Praise the client for having a normal
23. Which medication should the nurse expect cholesterol level.
the health-care provider to order for a client 2. Explain that the client needs to lower the
diagnosed with arterial occlusive disease? cholesterol level.
1. An anticoagulant medication. 3. Discuss dietary changes that could help
2. An antihypertensive medication. increase the level.
3. An antiplatelet medication. 4. Allow the client to ventilate feelings about the
4. A muscle relaxant. blood test result.
24. The nurse and an unlicensed assistive personnel 29. The nurse is discussing the pathophysiology of
(UAP) are caring for a 64-year-old client who is atherosclerosis with a client who has a normal
four (4) hours postoperative bilateral femoral- high-density lipoprotein (HDL) level. Which
popliteal bypass surgery. Which nursing task information should the nurse discuss with the
should be delegated to the UAP? client concerning HDLs?
1. Monitor the continuous passive motion 1. A normal HDL is good because it has a
machine. protective action in the body.
2. Assist the client to the bedside commode. 2. The HDL level measures the free fatty acids
3. Feed the client the evening meal. and glycerol in the blood.
4. Elevate the foot of the client’s bed. 3. HDLs are the primary transporters of
cholesterol into the cell.
4. The client needs to decrease the amount of
Atherosclerosis cholesterol and fat in the diet.

25. The nurse is teaching a class on coronary


artery disease. Which modifiable risk factor

04_Colgrove_Ch04.indd 114 02/06/16 7:57 PM


C hapter 4  P eripheral V ascular D isorders 115

30. Which assessment data would cause the nurse to 35. Which interventions should the nurse discuss
suspect the client has atherosclerosis? with the client diagnosed with atherosclerosis?
1. Change in bowel movements. Select all that apply.
2. Complaints of a headache. 1. Include the significant other in the discussion.
3. Intermittent claudication. 2. Stop smoking or using any type of tobacco
4. Venous stasis ulcers. products.
3. Maintain a sedentary lifestyle as much as
31. The nurse is teaching a class on atherosclerosis.
possible.
Which statement describes the scientific
4. Avoid stressful situations.
rationale as to why diabetes is a risk factor for
5. Daily isometric exercises are
developing atherosclerosis?
important.
1. Glucose combines with carbon monoxide,
instead of with oxygen, and this leads to 36. The nurse is caring for clients on a telemetry
oxygen deprivation of tissues. floor. Which nursing task would be most
2. Diabetes stimulates the sympathetic appropriate to delegate to an unlicensed
nervous system, resulting in peripheral assistive personnel (UAP)?
constriction that increases the development of 1. Teach the client how to perform a glucometer
atherosclerosis. check.
3. Diabetes speeds the atherosclerotic process by 2. Assist feeding the client diagnosed with
thickening the basement membrane of both congestive heart failure.
large and small vessels. 3. Check the cholesterol level for the client
4. The increased glucose combines with the diagnosed with atherosclerosis.
hemoglobin, which causes deposits of plaque 4. Assist the nurse to check the unit of blood at
in the lining of the vessels. the client’s bedside.
32. The nurse is discussing the importance of
exercising with a client who is diagnosed with Abdominal Aortic Aneurysm
coronary artery disease. Which statement
best describes the scientific rationale for 37. Which assessment data would the nurse
encouraging 30 minutes of walking daily to help recognize to support the diagnosis of abdominal
prevent complications of atherosclerosis? aortic aneurysm (AAA)?
1. Exercise promotes the development of 1. Shortness of breath.
collateral circulation. 2. Abdominal bruit.
2. Isometric exercises help develop the client’s 3. Ripping abdominal pain.
muscle mass. 4. Decreased urinary output.
3. Daily exercise helps prevent plaque from
developing in the vessel. 38. Which medical treatment would be prescribed
4. Isotonic exercises promote the transport of for the client with an AAA less than 3 cm?
glucose into the cell. 1. Ultrasound every six (6) months.
2. Intravenous pyelogram yearly.
33. The HCP prescribes an HMG-CoA reductase 3. Assessment of abdominal girth
inhibitor (statin) medication to a client with monthly.
CAD. Which should the nurse teach the client 4. Repair of abdominal aortic aneurysm.
about this medication?
1. Take this medication on an empty stomach. 39. Which client would be most likely to develop
2. This medication should be taken in the an abdominal aortic aneurysm?
evening. 1. A 45-year-old female with a history of
3. Do not be concerned if muscle pain occurs. osteoporosis.
4. Check your cholesterol level daily. 2. An 80-year-old female with congestive heart
failure.
34. The nurse knows the client understands the 3. A 69-year-old male with peripheral vascular
teaching concerning a low-fat, low-cholesterol disease.
diet when the client selects which meal? 4. A 30-year-old male with a genetic
1. Fried fish, garlic mashed potatoes, and iced tea. predisposition to AAA.
2. Ham and cheese on white bread and whole milk.
3. Baked chicken, baked potato, and skim milk.
4. A hamburger, French fries, and carbonated
beverage.

04_Colgrove_Ch04.indd 115 02/06/16 7:57 PM


116 M ed -S urg S uccess

40. The client is diagnosed with an abdominal aortic 45. The client is diagnosed with a small abdominal
aneurysm. Which statement would the nurse aortic aneurysm. Which interventions should be
expect the client to make during the admission included in the discharge teaching? Select all
assessment? that apply.
1. “I have stomach pain every time I eat a big, 1. Tell the client to exercise three (3) times a
heavy meal.” week for 30 minutes.
2. “I don’t have any abdominal pain or any type 2. Encourage the client to eat a low-fat, low-
of problems.” cholesterol diet.
3. “I have periodic episodes of constipation and 3. Instruct the client to decrease tobacco use.
then diarrhea.” 4. Discuss the importance of losing weight with
4. “I belch a lot, especially when I lie down after the client.
eating.” 5. Teach the client to wear a truss at all times.
41. The client is admitted for surgical repair of an 46. Which assessment data would require
8-cm abdominal aortic aneurysm (AAA). Which immediate intervention by the nurse for
sign/symptom would make the nurse suspect the the client who is six (6) hours postoperative
client has an expanding AAA? abdominal aortic aneurysm repair?
1. Complaints of low back pain. 1. Absent bilateral pedal pulses.
2. Weakened radial pulses. 2. Complaints of pain at the site of the incision.
3. Decreased urine output. 3. Distended, tender abdomen.
4. Increased abdominal girth. 4. An elevated temperature of 100˚F.
42. The client is one (1) day postoperative 47. The nurse is discussing discharge teaching with
abdominal aortic aneurysm repair. Which the client who is three (3) days postoperative
information from the unlicensed assistive abdominal aortic aneurysm repair. Which
personnel (UAP) would require immediate discharge instructions should the nurse include
intervention from the nurse? when teaching the client?
1. The client refuses to turn from the back to 1. Notify the HCP of any redness or irritation of
the side. the incision.
2. The client’s urinary output is 90 mL in six (6) 2. Do not lift anything that weighs more than 20
hours. pounds.
3. The client wants to sit on the side of the bed. 3. Inform the client there may be pain not
4. The client’s vital signs are T 98, P 90, R 18, relieved with pain medication.
and BP 130/70. 4. Stress the importance of having daily bowel
movements.
43. The client had an abdominal aortic aneurysm
repair two (2) days ago. Which intervention 48. On which area would the nurse place the bell of
should the nurse implement first? the stethoscope when assessing the client with an
1. Assess the client’s bowel sounds. abdominal aortic aneurysm?
2. Administer an IV prophylactic antibiotic.
3. Encourage the client to splint the incision. A
4. Ambulate the client in the room with C
assistance.
B
44. Which health-care provider’s order should the
nurse question in a client diagnosed with an D
expanding abdominal aortic aneurysm who is
scheduled for surgery in the morning?
1. Type and crossmatch for two (2) units of 1. A
blood. 2. B
2. Tap water enema until clear fecal return. 3. C
3. Bedrest with bathroom privileges. 4. D
4. Keep nothing by mouth (NPO) after
midnight.

04_Colgrove_Ch04.indd 116 02/06/16 7:57 PM


C hapter 4  P eripheral V ascular D isorders 117

Deep Vein Thrombosis 53. The client diagnosed with a DVT is placed on
a medical unit. Which nursing interventions
49. The nurse is discharging a client diagnosed should be implemented? Select all that apply.
with DVT from the hospital. Which discharge 1. Place sequential compression devices on both
instructions should be provided to the client? legs.
1. Have the PTT levels checked weekly until 2. Instruct the client to stay in bed and not
therapeutic range is achieved. ambulate.
2. Staying at home is best, but if traveling, 3. Encourage fluids and a diet high in roughage.
airplanes are better than automobiles. 4. Monitor IV site every four (4) hours and prn.
3. Avoid green, leafy vegetables and notify the 5. Assess Homans’ sign every 24 hours.
HCP of red or brown urine. 54. The nurse is caring for a client receiving heparin
4. Wear knee stockings with an elastic band sodium via constant infusion. The heparin
around the top. protocol reads to increase the IV rate by 100
50. The nurse is caring for clients on a surgical floor. units/hr if the PTT is less than 50 seconds. The
Which client should be assessed current PTT level is 46 seconds. The heparin
first? comes in 500 mL of D5W with 25,000 units of
1. The client who is four (4) days postoperative heparin added. The current rate on the IV pump
abdominal surgery and is complaining of left is 18 mL/hr. At what rate should the nurse set
calf pain when ambulating. the pump? _________
2. The client who is one (1) day postoperative 55. Which assessment data would warrant
hernia repair who has just been able to void immediate intervention by the nurse?
550 mL of clear amber urine. 1. The client diagnosed with DVT who
3. The client who is five (5) days postoperative complains of pain on inspiration.
open cholecystectomy who has a T-tube and is 2. The immobile client who has refused to turn
being discharged. for the last three (3) hours.
4. The client who is 16 hours post–abdominal 3. The client who had an open cholecystectomy
hysterectomy and is complaining of who refuses to breathe deeply.
abdominal pain and is expelling flatus. 4. The client who has had an inguinal hernia
51. The male client is diagnosed with Guillain-Barré repair who must void before discharge.
(GB) syndrome and is in the intensive care unit 56. The client diagnosed with a DVT is on a heparin
on a ventilator. Which cardiovascular rationale drip at 1,400 units per hour, and Coumadin
explains implementing passive range-of-motion (warfarin sodium, also an anticoagulant) 5 mg
(ROM) exercises? daily. Which intervention should the nurse
1. Passive ROM exercises will prevent implement first?
contractures from developing. 1. Check the PTT and PT/INR.
2. The client will feel better if he is able to 2. Check with the HCP to see which drug
exercise and stretch his muscles. should be discontinued.
3. ROM exercises will help alleviate the pain 3. Administer both medications.
associated with GB syndrome. 4. Discontinue the heparin because the client is
4. They help to prevent DVTs by movement of receiving Coumadin.
the blood through the veins.
57. Which actions should the surgical scrub
52. The nurse and an unlicensed assistive personnel nurse take to prevent personally developing
(UAP) are bathing a bedfast client. Which action a DVT?
by the UAP warrants immediate intervention? 1. Keep the legs in a dependent position and
1. The UAP closes the door and cubicle curtain stand as still as possible.
before undressing the client. 2. Flex the leg muscles and change the leg
2. The UAP begins to massage and rub lotion positions frequently.
into the client’s calf. 3. Wear white socks and shoes that have an
3. The UAP tests the temperature of the water elevated heel.
with the wrist before starting. 4. Ask the surgeon to allow the nurse to take a
4. The UAP collects all the linens and supplies break midway through each surgery.
and brings them to the room.

04_Colgrove_Ch04.indd 117 02/06/16 7:57 PM


118 M ed -S urg S uccess

58. The client receiving low molecular weight heparin the nurse include when discussing how to
(LMWH) subcutaneously to prevent DVT prevent varicose veins?
following hip replacement surgery complains to 1. Wear low-heeled, comfortable shoes.
the nurse that there are small purple hemorrhagic 2. Wear clean white cotton socks.
areas on the upper abdomen. Which action should 3. Move the legs back and forth often.
the nurse implement? 4. Wear graduated compression hose.
1. Notify the HCP immediately.
64. The client with varicose veins asks the nurse,
2. Check the client’s PTT level.
“What caused me to have these?” Which
3. Explain this results from the medication.
statement by the nurse would be most
4. Assess the client’s vital signs.
appropriate?
59. The home health nurse is admitting a client 1. “You have incompetent valves in your legs.”
diagnosed with a DVT. Which action by the 2. “Your legs have decreased oxygen to the
client warrants immediate intervention by the muscle.”
nurse? 3. “There is an obstruction in the saphenous
1. The client takes a stool softener every day at vein.”
dinnertime. 4. “Your blood is thick and can’t circulate
2. The client is wearing a Medic Alert properly.”
bracelet.
65. The nurse is caring for the client with chronic
3. The client takes vitamin E over-the-counter
venous insufficiency. Which statement indicates
medication.
the client understands the discharge teaching?
4. The client has purchased a new recliner that
1. “I shouldn’t cross my legs for more than
will elevate the legs.
15 minutes.”
60. The client is being admitted with Coumadin 2. “I need to elevate the foot of my bed while
(warfarin, an anticoagulant) toxicity. Which sleeping.”
laboratory data should the nurse monitor? 3. “I should take a baby aspirin every day with
1. Blood urea nitrogen (BUN) levels. food.”
2. Bilirubin levels. 4. “I should increase my fluid intake to 3,000 mL
3. International normalized ratio (INR). a day.”
4. Partial thromboplastin time (PTT).
66. The unlicensed assistive personnel (UAP) is
caring for the client diagnosed with chronic
Peripheral Venous Disease venous insufficiency. Which action would warrant
immediate intervention from the nurse?
61. The nurse is teaching a class on venous 1. Removing compression stockings before
insufficiency. The nurse would identify which assisting the client to bed.
condition as the most serious complication of 2. Taking the client’s blood pressure manually
chronic venous insufficiency? after using the machine.
1. Arterial thrombosis. 3. Assisting the client by opening the milk
2. Deep vein thrombosis. carton on the lunch tray.
3. Venous ulcerations. 4. Calculating the client’s shift intake and output
4. Varicose veins. with a pen and paper.

62. Which assessment data would support that the 67. The 80-year-old client is being discharged home
client has a venous stasis ulcer? after having surgery to débride a chronic venous
1. A superficial pink open area on the medial ulcer on the right ankle. Which referral would
part of the ankle. be most appropriate for the client?
2. A deep pale open area over the top side of the 1. Occupational therapist.
foot. 2. Social worker.
3. A reddened blistered area on the heel of the 3. Physical therapist.
foot. 4. Cardiac rehabilitation.
4. A necrotic gangrenous area on the dorsal side 68. Which assessment data would the nurse expect
of the foot. to find in the client diagnosed with chronic
63. The client is employed in a job that requires venous insufficiency?
extensive standing. Which intervention should 1. Decreased pedal pulses.
2. Cool skin temperature.
3. Intermittent claudication.
4. Brown discolored skin.

04_Colgrove_Ch04.indd 118 02/06/16 7:57 PM


C hapter 4  P eripheral V ascular D isorders 119

69. Which client would be most at risk for 71. The nurse has just received the a.m. shift report.
developing varicose veins? Which client would the nurse assess first?
1. A Caucasian female who is a nurse. 1. The client with a venous stasis ulcer who is
2. An African American male who is a bus driver. complaining of pain.
3. An Asian female with no children. 2. The client with varicose veins who has dull,
4. An elderly male with diabetes. aching muscle cramps.
3. The client with arterial occlusive disease who
70. The client with varicose veins is six (6) hours
cannot move the foot.
postoperative vein ligation. Which nursing
4. The client with deep vein thrombosis who has
intervention should the nurse implement
a positive Homans’ sign.
first?
1. Assist the client to dangle the legs off the side 72. The nurse is completing a neurovascular
of the bed. assessment on the client with chronic venous
2. Assess and maintain pressure bandages on the insufficiency. What should be included in this
affected leg. assessment? Select all that apply.
3. Apply a sequential compression device to the 1. Assess for paresthesia.
affected leg. 2. Assess for pedal pulses.
4. Administer the prescribed prophylactic 3. Assess for paralysis.
intravenous antibiotic. 4. Assess for pallor.
5. Assess for polar (temperature).

CONCEPTS
The concepts covered in this chapter focus on clotting and the perfusion-related areas of nursing issues. Clotting
and tissue perfusion-related problems of deep vein thrombosis and peripheral vascular disease as well as interrelated
concepts of the nursing process and critical thinking are also covered. The concept of nursing process is displayed
throughout the questions. The concept of critical thinking is presented in any prioritizing or “first” question.
73. The nurse is admitting a client diagnosed with Based on the laboratory results, which
peripheral vascular disease. Which data support intervention should the nurse implement?
a diagnosis of venous insufficiency? 1. Request a change of medication to a
1. The client has bright red skin on the lower subcutaneous anticoagulant.
extremities. 2. Administer AquaMephyton (vitamin K) IM.
2. The client has a brownish purple area on the 3. Have the dietary department remove all
lower legs. green, leafy vegetables from the trays.
3. The client complains of pain after ambulating 4. Administer the IV as ordered.
for short distances.
76. The nurse is caring for a client diagnosed with
4. The client has nonhealing wounds on the toes
deep vein thrombosis. Which information
and ankles.
reported to the nurse by the unlicensed
74. The nurse identifies the concept of clotting for assistive personnel (UAP) requires immediate
a client diagnosed with a deep vein thrombosis. intervention?
Which clinical manifestations support the 1. The UAP informed the nurse the client is
diagnosis? complaining of chest pain.
1. Brown-purple discoloration on the calf. 2. The UAP notified the nurse the client’s blood
2. Bright red skin on the lower legs. pressure is 100/66.
3. Swelling in the calf, warmth, and tenderness. 3. The UAP reported the client is requesting to
4. Pain after walking for short distances that be able to take a shower.
resolve with rest. 4. The UAP tells the nurse the client is asking
for medication for a headache.
75. The client diagnosed with a deep vein
thrombosis is prescribed heparin via continuous
infusion. The client’s laboratory data are:
PT 12.2 aPTT 48
Control 1.4 Control 32
INR 1

04_Colgrove_Ch04.indd 119 02/06/16 7:57 PM


120 M ed -S urg S uccess

77. The client presents to the outpatient clinic 3. Constipation.


complaining of calf pain. The client reports 4. Myocardial infarction.
returning from an airplane trip the previous day.
82. The nurse is teaching a class to clients diagnosed
Which should the nurse assess first?
with hypertension. Which should the nurse
1. The nurse should auscultate the lung fields
teach the clients?
and heart sounds.
1. The blood pressure target range should be
2. The nurse should determine the length of the
120/80.
airplane trip.
2. Take the medication even when feeling well.
3. The nurse should determine if the client has
3. Get up quickly when rising from a recumbent
had chest pain.
position.
4. The nurse should measure the calf and
4. Consume a 3,000-mg sodium diet.
palpate the calf for warmth.
83. Which should the nurse include in the plan of
78. The client prescribed rivaroxaban (Xarelto), an
care for a client diagnosed with venous stasis
anticoagulant, is complaining of dark, tarry stool.
ulcers? Select all that apply.
Which should the nurse implement first?
1. Elevate the legs while sitting.
1. Notify the health-care provider (HCP).
2. Wear antiembolism compression
2. Ask the client to provide a stool sample.
stockings.
3. Ask the client when the rivaroxaban was last
3. Avoid injury to the lower limbs.
taken.
4. Trim the toenails straight across.
4. Assess the client for ecchymotic areas and
5. Do not apply moisturizer to the lower legs.
bleeding.
6. Allow the legs to hang over the bed in a
79. The nurse is planning care for a client diagnosed dependent position.
with arterial occlusive disease. Which exercise
84. The client on the telemetry unit diagnosed with
instructions would the nurse teach the client?
a thromboembolism is complaining of chest
1. Have the client perform isometric exercises 30
pain and anxiety. Which action should the nurse
minutes each day.
implement first?
2. Tell the client to start exercising on a stair
1. Stay with the client and call the Rapid
stepper for 15 minutes.
Response Team (RRT).
3. Inform the client that warm-up exercises are
2. Assess the client’s vital signs.
not necessary.
3. Have the unlicensed assistive personnel (UAP)
4. Teach the client to walk in well-fitting shoes
stay with the client.
on level ground.
4. Check the client’s telemetry reading.
80. The nurse is teaching the client diagnosed with
85. Which arterial anticoagulant medication would
deep vein thrombosis and prescribed warfarin.
the nurse anticipate being prescribed for a client
Which should the nurse teach the client? Select
diagnosed with arterial occlusive
all that apply.
disease?
1. Keep a constant amount of green, leafy
1. Clopidogrel.
vegetables in the diet.
2. Streptokinase.
2. Instruct the client to have regular INR
3. Protamine sulfate.
laboratory work done.
4. Enoxaparin.
3. Tell the client to go to the hospital
immediately for any bleeding. 86. The nurse is caring for a male client diagnosed
4. Inform the client to notify the HCP if having with essential hypertension. Which information
dark, tarry stools. regarding antihypertensive medication should
5. Encourage the client to avoid all green the nurse teach?
vegetables. 1. Teach the client to take his blood pressure
6. Have the client take iron orally to prevent four (4) times each day.
bleeding. 2. Instruct the client to have regular blood levels
of the medication checked.
81. Which complication of anticoagulant therapy
3. Explain the need to rise slowly from a lying or
should the nurse teach the client to report to the
sitting position.
health-care provider?
4. Demonstrate how to use a blood glucose
1. Gastric upset.
meter daily.
2. Bleeding from any site.

04_Colgrove_Ch04.indd 120 02/06/16 7:57 PM


C hapter 4  P eripheral V ascular D isorders 121

87. The nurse is demonstrating the use of a blood 89. The client is at risk for a myocardial infarction
pressure sphygmomanometer to a client newly due to decreased tissue perfusion as a result
diagnosed with hypertension. Which should the of atherosclerosis. Which instructions can the
nurse teach the client? Select all that apply. nurse provide the client to reduce the risk?
1. Tell the client to make sure the cuff is placed 1. Teach the client to control the blood pressure
over an artery. to less than 140/90.
2. Teach the client to notify the health-care 2. Instruct the client to exercise 30 minutes a day
provider if the BP is >160/100. three (3) times a week.
3. Instruct the client about orthostatic 3. Demonstrate how to take the blood pressure
hypotension. using a battery-operated cuff.
4. Encourage the client to keep a record of the 4. Inform the client to limit fat intake and which
blood pressure readings. foods have a higher fat content.
5. Explain that even when the blood pressure is
90. The nurse is caring for a client who is receiving
within normal limits the medication should
heparin therapy intravenously. Which assessment
still be taken.
data would indicate to the nurse the client is
88. The client diagnosed with a deep vein developing heparin-induced thrombocytopenia
thrombosis (DVT) is prescribed a heparin (HIT)? Select all that apply.
drip. The solution is 40,000 units in 500 mL 1. The client has spontaneous bleeding from
of D5W. The health-care provider ordered around the IV site.
the client to receive 1,200 units per hour. At 2. The client complains of chest pain on
which rate should the nurse set the IV pump? inspiration and has become restless.
______________________ 3. The client’s platelet count on admission was
420 (103) and now is 200 (103).
4. The client complains that the gums bleed
when brushing the teeth.
5. The client has developed skin lesions at the
IV site.

04_Colgrove_Ch04.indd 121 02/06/16 7:57 PM


PRACTICE QUESTIONS ANSWERS
AND RATIONALES

Arterial Hypertension 2. Alpha-adrenergic blockers, not ACE inhibitors,


block alpha receptors in the vascular smooth
1. 1. This BP is elevated, and the client should have muscle, which decreases vasomotor tone and
his BP checked frequently but not before seek- vasoconstriction.
ing medical treatment. 3. Angiotensin-converting enzyme (ACE)
2. The diastolic blood pressure should be less inhibitors prevent the conversion of
than 85 mm Hg according to the American ­angiotensin I to angiotensin II and this, in
Heart Association; therefore, this client turn, prevents vasoconstriction and ­sodium
should see the health-care provider. and water retention.
3. Teaching is important, but the nurse must 4. Vasodilators, not ACE inhibitors, reduce blood
first make sure the client sees the health-care pressure by relaxing vascular smooth muscle,
provider for a thorough checkup and antihy- especially in the arterioles.
pertensive medication prescription. Diet alone TEST-TAKING HINT: The test taker needs to
should not be recommended by the nurse. understand how the major classifications of
4. This is not the normal range for an elderly medications work to answer this question.
person’s blood pressure; the diastolic should be Content – Medical: Integrated Nursing Process –
less than 85 mm Hg. Diagnosis: Client Needs – Physiological Integrity,
TEST-TAKING HINT: Remember, the question Pharmacological and Parenteral Therapies: Cognitive
asks which action should be implemented Level – Knowledge: Concept – Medication.
first. Therefore, more than one answer is 4. 1. The potassium level is within normal limits
­appropriate but the first to be implemented (3.5 to 5.5 mEq/L), and it is not usually
should be the one that directly affects the checked prior to administering beta blockers.
client. 2. The nurse should question administer-
Content – Medical: Integrated Nursing Process – ing the beta blocker if the BP is low be-
­Implementation: Client Needs – Safe Effective Care cause this medication will cause the blood
Environment, Management of Care: Cognitive Level – pressure to drop even lower, leading to
Application: Concept – Perfusion. hypotension.
2. 1. Walking 30 to 45 minutes a day will help 3. The nurse would not administer the medica-
to reduce blood pressure, weight, and tion if the apical (not radial) pulse were less
stress and will increase a feeling of ­overall than 60 beats per minute.
well-being. 4. The nurse needs to assess the blood pressure
2. Isometric exercises (such as weight lifting) only once prior to administering the medica-
should be discouraged because performing tion (not take all three blood pressures prior to
them can raise the systolic blood pressure. administering the medication).
3. The client should walk, cycle, jog, or swim TEST-TAKING HINT: Be sure to read the ­entire
daily, but high-level aerobic exercise may question and all the answer options and note
­increase the client’s blood pressure. the specific numbers that are identified. The
4. Swimming laps is recommended, but it should test taker must know normal laboratory data
be daily, not once a week. and assessment findings.
TEST-TAKING HINT: Remember to look at the Content – Medical: Integrated Nursing Process –
frequency of interventions; it makes a differ- Implementation: Client Needs – Physiological Integrity,
ence when selecting the correct answers. Pharmacological and Parenteral Therapies: Cognitive
Level – Application: Concept – Medication.
Content – Medical: Integrated Nursing Process –
Planning: Client Needs – Health Promotion and 5. 1. The potassium level is not affected by an
Maintenance: Cognitive Level – Synthesis: Concept – alpha-adrenergic blocker.
Nursing Roles. 2. Impotence is a major cause of noncompliance
3. 1. Beta-adrenergic blocking agents, not ACE in- with taking prescribed medications for hyper-
hibitors, prevent the beta receptor stimulation tension in male clients. The noncompliance
in the heart, which decreases heart rate and should be reported to the HCP immediately
cardiac output. so the medication can be changed.
122

04_Colgrove_Ch04.indd 122 02/06/16 7:57 PM


C hapter 4  P eripheral V ascular D isorders 123

3. The medication can be taken on an empty 4. This is a therapeutic reply that is inappropriate
or a full stomach, depending on whether the because the client needs facts.
client becomes nauseated after taking the TEST-TAKING HINT: When clients request in-
medication. formation, the exchange should not address
4. Orthostatic hypotension may occur when emotions. Just facts should be given. There-
the blood pressure is decreasing and may fore, option “4” can be eliminated as a correct
lead to dizziness and light-headedness, so answer.
the client should change position slowly. Content – Medical: Integrated Nursing Process –
TEST-TAKING HINT: The test taker should un- ­Implementation: Client Needs – Physiological Integrity,
derstand the side effects of medications. The ­Physiological Adaptation: Cognitive Level – Application:
test taker who does not know the answer may Concept – Perfusion.
realize that hypertension is being treated and 8. 1. The DASH diet has proved beneficial in
that hypotension is the opposite of hyperten- lowering blood pressure. It recommends
sion and might be a complication of treating eating a diet high in vegetables and fruits.
hypertension. Only option “4” refers to hy- 2. The DASH diet recommends two (2) or fewer
potension, providing advice on how to avoid servings of lean meats, which have very few
orthostatic hypotension. white streaks; the white streaks
Content – Medical: Integrated Nursing Process – indicate the meat is high in fat.
­Planning: Client Needs – Physiological Integrity, 3. The DASH diet recommends two (2) to three
Pharmacological and Parenteral Therapies: Cognitive
(3) servings of nonfat or low-fat milk, not
Level – Synthesis: Concept – Medication.
whole milk.
6. 1. This blood pressure is elevated, but it is not 4. The DASH diet recommends seven (7) to
life threatening. eight (8) servings of grain a day.
2. The chest pain could be a pulmonary em- TEST-TAKING HINT: The test taker is looking
bolus secondary to deep vein thrombosis for correct information about the DASH diet.
and requires immediate intervention by the A recommended diet for hypertension would
nurse. limit fatty meats and whole milk.
3. A pulse oximeter reading of greater than 93% Content – Medical: Integrated Nursing Process –
is within normal limits. Evaluation: Client Needs – Physiological Integrity,
4. Nonbloody diarrhea is an expected sign Basic Care and Comfort: Cognitive Level – Evaluation:
of ulcerative colitis and would not require Concept – Perfusion.
­immediate intervention by the nurse.
9. 1. Normal sinus rhythm indicates that the ­client’s
TEST-TAKING HINT: The nurse should assess
heart is working normally.
the client who has abnormal assessment data 2. Rapid weight gain—for example, 2 kg
or a life-threatening condition first when in one (1) to two (2) days—indicates
­determining which client is priority. that the loop diuretic is not working
Content – Medical: Integrated Nursing Process – effectively; 2 kg equals 4.4 lb; 1 L of
Assessment: Client Needs – Safe Effective Care Environment, fluid weighs l kg.
Management of Care: Cognitive Level – Synthesis:
3. This blood pressure is not life threaten-
Concept – Assessment.
ingly high and does not require immediate
7. 1. Kidney disease leads to secondary hyperten- intervention.
sion; secondary hypertension is elevated blood 4. Loop diuretics cause an increase in potassium
pressure resulting from an identifiable under- excretion in the urine; therefore, the potassium
lying process. level should be assessed, but 4.5 mEq/L is
2. A high-salt, high-fat, high-cholesterol diet is a within normal limits (3.5 to 5.5 mEq/L).
risk factor for essential hypertension, but it is TEST-TAKING HINT: The phrase “requires
not the only cause; therefore, this would be an immediate intervention” should make the
incorrect answer. test taker think that the correct answer will
3. There is no known cause for essential be abnormal assessment data that require
hypertension, but many factors—both medical intervention or indicate conditions
modifiable (obesity, smoking, diet) and that are life threatening.
nonmodifiable (family history, age, Content – Medical: Integrated Nursing Process –
gender)—are risk factors for essential ­Assessment: Client Needs – Physiological Integrity,
hypertension. Pharmacological and Parenteral Therapies: Cognitive
Level – Synthesis: Concept – Medication.

04_Colgrove_Ch04.indd 123 02/06/16 7:57 PM


124 M ed -S urg S uccess

10. 1. Even if the client feels great, the blood 4. A low-salt diet is recommended because
pressure can be elevated, causing damage increased salt intake causes water retention,
to the heart, kidney, and blood vessels. which increases the workload of the heart.
2. A headache may indicate an elevated blood A high-fiber diet is recommended because it
pressure, but the client with essential hyper- helps decrease cholesterol levels.
tension can be asymptomatic and still have a TEST-TAKING HINT: Remember to look at the
very high blood pressure reading. adjectives. The stem of the question is asking
3. This response does not answer the client’s about “modifiable risk factors.”
question as to why the doctor is worried Content – Medical: Integrated Nursing Process –
about the client’s blood pressure. Planning: Client Needs – Health Promotion and
4. The blood pressure does not necessarily Maintenance: Cognitive Level – Synthesis: Concept –
reflect how well the heart is working. Many Promoting Health.
other diagnostic tests assess how well the
heart is working, including an electrocardio-
gram (ECG), an ultrasound, and a chest x-ray. Arterial Occlusive Disease
TEST-TAKING HINT: The test taker should se-
lect the option that provides the client with 13. 1. Peripheral vascular disease is a broad term
correct information in a nonthreatening, that encompasses both venous and arterial
nonjudgmental approach. peripheral problems of the lower extremities.
2. This is the classic symptom of arterial
Content – Medical: Integrated Nursing Process –
­Implementation: Client Needs – Health Promotion
o­cclusive disease.
and Maintenance: Cognitive Level – Application: 3. This is characterized by calf tenderness, calf
Concept – Perfusion. edema, and a positive Homans’ sign.
4. This term is a sign of arterial occlusive dis-
11. 1,520 mL total intake. ease; the legs are pale when elevated but are
The urinary output is not used in this calcula- dark red when in the dependent position.
tion. The nurse must add up both intravenous
TEST-TAKING HINT: The test taker could
fluids and oral fluids to obtain the total intake
eliminate options “1” and “3” as possible
for this client:
answers if the words “medical term” were
880 + 100 = 980 IV fluids
noted. Both ­options “1” and “3” are disease
Oral fluids (1 ounce = 30 mL): 8 ounces × 30 mL
processes, not medical terms.
= 240 mL, 4 ounces × 30 mL = 120 mL,
6 ounces × 30 mL = 180 mL Content – Medical: Integrated Nursing Process –
­Implementation: Client Needs – Health Promotion
240 + 120 + 180 = 540 mL oral fluids
and Maintenance: Cognitive Level – Application:
Total intake is 980 + 540 = 1,520 mL. Concept – Perfusion.
TEST-TAKING HINT: The reader must know
the conversion equivalents before perform- 14. 1. External heating devices are avoided to
ing any type of math problems. The key to reduce the risk of burns.
this answer is 1 ounce = 30 mL. 2. Elastic support hose reduce the circulation to
the skin and are avoided.
Content – Medical: Integrated Nursing Process –
­Implementation: Client Needs – Physiological Integrity,
3. Walking promotes the development of
Pharmacological and Parenteral Therapies: Cognitive collateral circulation to ischemic tissue
Level – Application: Concept – Perfusion. and slows the process of atherosclerosis.
4. The feet must be checked daily, not weekly.
12. 1. Retinopathy and nephropathy are compli-
TEST-TAKING HINT: The test taker must note
cations of uncontrolled hypertension, not
the noun “week” in option “4,” which could
­modifiable risk factors.
eliminate this distracter as a possible answer.
2. Sedentary lifestyle is discouraged in
­clients with hypertension; daily isotonic Content – Medical: Integrated Nursing Process –
Planning: Client Needs – Physiological Integrity,
exercises are recommended. Smoking
Physiological Adaptation: Cognitive Level – Synthesis:
(cigars have problems too) increases the Concept – Perfusion.
atherosclerotic process in vessels; causes
vasoconstriction of vessels; and adheres to 15. 1. Cold water causes vasoconstriction and
hemoglobin, decreasing oxygen levels. hot water may burn the client’s feet;
3. Family history and gender are nonmodifiable therefore, warm (tepid) water should be
risk factors. The question is asking for infor- recommended.
mation on modifiable risk factors. 2. Moisturizing prevents drying of the feet.

04_Colgrove_Ch04.indd 124 02/06/16 7:57 PM


C hapter 4  P eripheral V ascular D isorders 125

3. Shoes should be purchased in the afternoon 18. 1. The right leg should be elevated to decrease
when the feet are the largest. edema, not flat or hanging off the side of the
4. This will further decrease circulation to bed (dependent).
the legs. 2. The left leg could have a sequential compres-
5. Colored socks have dye, and dirty socks sion device to prevent deep vein thrombosis,
may cause foot irritation that may lead to but it should not be on the leg with an opera-
breaks in the skin. tive incision site.
TEST-TAKING HINT: The test taker must 3. The client is one (1) day postoperative, and
­select all appropriate interventions; option the pedal pulses must be assessed more than
“3” could be eliminated as a correct answer once every eight (8) or 12 hours.
because of “only,” which is an absolute word. 4. The leg dressing needs to be assessed for
There are very few absolutes in health care. hemorrhaging or signs of infection.
Content – Medical: Integrated Nursing Process – TEST-TAKING HINT: The test taker must be
Planning: Client Needs – Physiological Integrity, observant of time (“one (1) day postopera-
Physiological Adaptation: Cognitive Level – Synthesis: tive”), and doing an intervention every “shift”
Concept – Nursing Roles. should cause the test taker to eliminate this
distracter. The test taker must know terms
16. 1. These are normal pedal pulses and would not
used to describe positioning, such as depen-
require any intervention.
dent, prone, and supine.
2. Moving the toes is a good sign in a client with
arterial occlusive disease. Content – Surgical: Integrated Nursing Process –
3. Numbness and tingling are paresthesia, ­Implementation: Client Needs – Physiological Integrity, ­
Reduction of Risk Potential: Cognitive Level – Application:
which is a sign of a severely decreased
­Concept – Perfusion.
blood supply to the lower extremities.
4. Reddened extremities are expected secondary 19. 1. An absent pulse is not uncommon in a
to increased blood supply when the legs are client diagnosed with arterial occlusive
in the dependent position. disease, but the nurse must ensure that
TEST-TAKING HINT: “Warrants immediate in- the feet can be moved and are warm,
tervention” indicates that the test taker must which indicates adequate blood supply to
select the distracter that is abnormal, unex- the feet.
pected, or life threatening for the client’s dis- 2. To identify the location of the pulse, the
ease process. Sometimes if the test taker flips nurse should use a Doppler device to amplify
the question and thinks which assessment the sound, but it is not the first intervention.
data are normal for the disease process, it is 3. This position will increase blood flow and
easier to identify the correct answer. may help the nurse palpate the pulse, but it is
not the first intervention.
Content – Medical: Integrated Nursing Process –
Assessment: Client Needs – Physiological Integrity, 4. Cold can cause vasoconstriction and decrease
Reduction of Risk Potential: Cognitive Level – Analysis: the ability to palpate the pulse, and warm-
Concept – Perfusion. ing will dilate the arteries, helping the nurse
find the pedal pulse, but it is not the first
17. 1. The client has a foot ulcer; therefore, the intervention.
protective lining of the body—the skin—
TEST-TAKING HINT: The stem asks the test
has been impaired.
taker to identify the first intervention, and
2. This is an appropriate problem but would not
the test taker should apply the ­nursing
take priority over impaired skin integrity.
process and implement an assessment
3. The client needs teaching, but it does not
intervention.
take priority over a physiological problem.
4. The client has peripheral neuropathy, not Content – Medical: Integrated Nursing Process – ­
a risk for it; this is the primary pathologi- Implementation: Client Needs – Safe Effective Care
Environment, Management of Care: Cognitive Level –
cal change in a client with arterial occlusive
Synthesis: Concept – Perfusion.
disease.
TEST-TAKING HINT: Remember Maslow’s 20. 1. The pain stops when the client quits walking;
hierarchy of needs; physiological needs are therefore, it is not rest pain.
priority. 2. Rest pain indicates a worsening of the
­arterial occlusive disease; the muscles of
Content – Medical: Integrated Nursing Process –
Diagnosis: Client Needs – Safe Effective Care the legs are not getting enough oxygen
Environment, Management of Care: Cognitive Level – when the client is resting to prevent
Analysis: Concept – Perfusion. ­muscle ischemia.

04_Colgrove_Ch04.indd 125 02/06/16 7:57 PM


126 M ed -S urg S uccess

3. This is a therapeutic response and does not femoral artery would require pressure at the
answer the wife’s question. site to prevent bleeding—this information
4. Rest pain indicates that the arterial occlusive could help the test taker to eliminate option
disease is getting worse. “2” as a possible answer. Very few diagnostic
TEST-TAKING HINT: The nurse should answer tests are done at the bedside.
questions with factual information; there- Content – Medical: Integrated Nursing Process –
fore, option “3” could be eliminated as a ­Implementation: Client Needs – Physiological Integrity,
possible answer. Pain usually does not indi- Reduction of Risk Potential: Cognitive Level – Application:
cate that a condition is getting better, which Concept – Perfusion.
would cause the test taker to eliminate 23. 1. An anticoagulant medication is prescribed
option “4.” for venous problems, such as deep vein
Content – Medical: Integrated Nursing Process – thrombosis.
­
Implementation: Client Needs – Physiological Integrity, 2. Arterial occlusive disease is caused by athero-
­Physiological Adaptation: Cognitive Level – Application: sclerosis, which may cause hypertension as
Concept –Perfusion. well, but antihypertensive medications are not
21. 1. The decreased oxygen over time causes prescribed for arterial occlusive disease.
the loss of hair on the tops of the feet and 3. Antiplatelet medications, such as aspirin
ascending both legs. or clopidogrel (Plavix), inhibit platelet
2. The toenails are usually thickened as a result a­ggregations in the arterial blood.
of hypoxemia. 4. A muscle relaxant will not help the leg pain
3. Petechiae are tiny purple or red spots that because the origin of the pain is decreased
appear on the skin as a result of minute hem- oxygen to the muscle.
orrhages within the dermal layer; this does TEST-TAKING HINT: The test taker should
not occur with arterial occlusive disease. a­pply the knowledge learned in anatomy and
4. There may be edema but it is usually pitting; physiology class. Platelets are part of the
nonpitting edema resolves with elevation but arterial blood; therefore, this would be an
not in clients with arterial occlusive disease. ­excellent selection if the test taker did not
TEST-TAKING HINT: The test taker should have any idea about the answer.
­apply the pathophysiological concept that Content – Medical: Integrated Nursing Process –
arterial blood supplies oxygen and nutrients ­Implementation: Client Needs – Physiological Integrity,
and, if the hair cannot get nutrients, it will Reduction of Risk Potential: Cognitive Level – Application:
not grow. Concept – Medication.

Content – Medical: Integrated Nursing Process – 24. 1. A continuous passive motion machine is used
Assessment: Client Needs – Physiological Integrity, for a client with a total knee replacement, not
Reduction of Risk Potential: Cognitive Level – Analysis: for this type of surgery.
Concept – Perfusion. 2. The client will be on bedrest at four (4) hours
22. 1. This procedure will be done in a catheteriza- after the surgery. Remember, the client had
tion laboratory or special room, not at the bilateral surgery on the legs.
bedside, because machines are used to visual- 3. There is nothing in the stem that would
ize the extent of the arterial occlusion. ­indicate the client could not feed himself
2. The client will have to keep the leg straight or herself. The nurse should encourage
for at least six (6) hours after the procedure to ­independence as much as possible.
prevent bleeding from the femoral artery. 4. After the surgery, the client’s legs will
3. An intravenous contrast medium is injected be elevated to help decrease edema. The
and vessels are visualized using fluoroscopy surgery has corrected the decreased blood
and x-rays. supply to the lower legs.
4. Fluids will help flush the contrast dye out of TEST-TAKING HINT: A concept that is appli-
the body and help prevent kidney damage. cable to surgery is decreasing edema, and
TEST-TAKING HINT: The test taker must be extremity surgeries usually include elevat-
knowledgeable of diagnostic tests. If not, ing the affected extremity. The test taker
the test taker could dissect the word “angio- must apply basic concepts when answering
gram”; angio- means “vessel,” which could questions.
help eliminate option “3” as a possible an- Content – Surgical: Integrated Nursing Process –
swer because some type of dye would have Planning: Client Needs – Physiological Integrity, Reduction
to be used to visualize a vessel. Adjectives of Risk Potential: Cognitive Level – Synthesis: Concept –
should be noted—anything done in the Nursing Roles.

04_Colgrove_Ch04.indd 126 02/06/16 7:57 PM


C hapter 4  P eripheral V ascular D isorders 127

Atherosclerosis Nicotine decreases blood flow to the


extremities and increases heart rate and
25. 1. A modifiable risk factor is a risk factor blood pressure. It also increases the risk
that can possibly be altered by modifying of clot formation by increasing the aggre-
or changing behavior, such as developing gation of platelets.
new ways to deal with stress. TEST-TAKING HINT: The test taker should look
2. The client cannot do anything about getting at the answer options closely to determine if
older, so it cannot be modified. any are similar. This will help eliminate two
3. Gender is a risk factor that cannot be options—“1” and “3”—as possible answers.
changed. An unhealthy diet will cause the client to be
4. Having a family history of coronary artery overweight.
disease predisposes the client to a higher risk, Content – Medical: Integrated Nursing Process –
but this cannot be changed by the client. Diagnosis: Client Needs – Health Promotion and
TEST-TAKING HINT: The test taker needs to Maintenance: Cognitive Level – Knowledge:
key in on adjectives when reading the stem Concept – Perfusion.
of a question. The word “modifiable” should 28. 1. The cholesterol level should be less than
cause the test taker to select “stress” because 200 mg/dL.
it is the only answer option referring to 2. The client needs to be taught ways to
something that can be changed or modified. lower the cholesterol level.
Content – Medical: Integrated Nursing Process – 3. The client should be taught a low-fat, low-
Planning: Client Needs – Health Promotion and cholesterol diet to help lower the cholesterol
Maintenance: Cognitive Level – Synthesis: Concept – level.
Promoting Health. 4. The nurse needs to discuss facts concern-
26. 1. The nurse should assume the client is a ing the cholesterol level and teach the client.
­layperson and should not explain disease A therapeutic conversation would not be
­processes using medical terminology. appropriate.
2. This is passing the buck; the nurse should TEST-TAKING HINT: The nurse needs to know
have the knowledge to answer this question. normal laboratory test findings. If the test
3. Atherosclerosis involves the arteries, not taker is not aware of normal cholesterol
the veins. ­levels, he or she could only guess the answer
4. This response explains in plain terms why to the question.
the client’s legs hurt from atherosclerosis. Content – Medical: Integrated Nursing Process –
TEST-TAKING HINT: If the test taker knows ­Implementation: Client Needs – Safe Effective Care
medical terminology, option “3” could be Environment, Management of Care: Cognitive Level –
eliminated because athero- means “arteries,” Application: Concept – Perfusion.
not veins. The test taker should be very cau- 29. 1. A normal HDL level is good because
tious when choosing an option that asks the HDL transports cholesterol away
health-care provider to answer questions that from the tissues and cells of the arte-
nurses should be able to answer. rial wall to the liver for excretion. This
Content – Medical: Integrated Nursing Process – helps d­ecrease the development of
­Implementation: Client Needs – Physiological Integrity, atherosclerosis.
Physiological Adaptation: Cognitive Level – Application: 2. The normal HDL level was the result of a test
Concept – Perfusion. measuring high-density lipoproteins, not free
27. 1. Being overweight is not a risk factor for fatty acids and glycerol in the blood, which
­atherosclerotic lesions, but it does indicate are measured by the serum triglyceride level.
that the client does not eat a healthy diet or Triglycerides are a source of energy.
exercise as needed. 3. Low-density lipoproteins (LDLs), not HDLs,
2. Lack of exercise is a risk factor, but it is not are the primary transporters of cholesterol
the strongest. into the cell. They have the harmful effect of
3. Although the stem did not explicitly identify depositing cholesterol into the walls of the
diet, the nurse should assume that a client arterial vessels.
who is obese would not eat a low-fat, low- 4. A normal HDL level is good and the client
cholesterol diet. does not need to change the diet.
4. Tobacco use is the strongest factor in the
development of atherosclerotic lesions.

04_Colgrove_Ch04.indd 127 02/06/16 7:57 PM


128 M ed -S urg S uccess

TEST-TAKING HINT: If the test taker has no 2. Isometric (weight-lifting) exercises help de-
idea what the correct answer is, the test taker velop muscle mass, but this type of exercise
should look at the specific words in the an- does not help decrease complications of
swer options. Normal laboratory data would atherosclerosis.
probably be good for the client; therefore, 3. A low-fat, low-cholesterol diet may help
­option “1” would be a probable correct answer. d­ecrease the plaque formation, but exercise
Content – Medical: Integrated Nursing Process – will not do this.
Planning: Client Needs – Health Promotion and 4. Isotonic exercises, such as walking and swim-
Maintenance: Cognitive Level – Synthesis: ming, promote the movement of glucose
Concept – Perfusion. across the cell membrane, but this is not why
30. 1. A change in bowel movements may indicate such exercises are recommended for preven-
cancer but not atherosclerosis. tion of atherosclerotic complications.
2. A headache is not a sign/symptom of TEST-TAKING HINT: The test taker must un-
atherosclerosis. derstand what the stem of the question is
3. Intermittent claudication is a sign of gen- asking and note the words “the complications
eralized atherosclerosis and is a marker of of atherosclerosis.” “Isometric,” which refers
atherosclerosis. to “muscle” (remember “m”), and “isotonic,”
4. Atherosclerosis indicates arterial involvement, which refers to “tone” (remember “t”), exer-
not venous involvement. cises do not directly help prevent the compli-
cations of atherosclerosis, so options “2” and
TEST-TAKING HINT: Knowledge of medical
“4” can be eliminated.
terminology—in this case, knowing that
“­atherosclerosis” refers to arteries—would Content – Medical: Integrated Nursing Process –
allow the test taker to rule out all of the Diagnosis: Client Needs – Physiological Integrity,
Physiological Adaptation: Cognitive Level – Analysis:
answer options except option “3,” even if the
Concept – Perfusion.
test taker does not know what “intermittent
claudication” means. 33. 1. A statin medication can be taken with food or
Content – Medical: Integrated Nursing Process – on an empty stomach.
Assessment: Client Needs – Physiological Integrity, 2. Statin medications should be taken in
Reduction of Risk Potential: Cognitive Level – Analysis: the evening for best results because the
Concept – Perfusion. enzyme that destroys cholesterol works
best in the evening and the medication
31. 1. Glucose does not combine with carbon
enhances this process.
monoxide.
3. Cholesterol-reducing medications can cause
2. Vasoconstriction is not a risk factor for
serious liver problems, and if a client has
­developing atherosclerosis.
muscle pain, it is an adverse effect that should
3. This is the scientific rationale for why
be reported to the HCP.
diabetes mellitus is a modifiable risk
4. The cholesterol level is checked every few
­factor for atherosclerosis.
months, not on a daily basis.
4. When glucose combines with the hemoglobin
in a laboratory test called glycosylated hemoglo- TEST-TAKING HINT: The test taker must be
bin, the result can determine the client’s average aware of adverbs such as “daily.” Cholesterol
glucose level over the past three (3) months. is not monitored daily, so option “4” can be
TEST-TAKING HINT: The nurse must under-
eliminated. There are only a few medications
stand the reason “why,” or the scientific taken on an empty stomach; most medica-
rationale, for teaching in addition to nursing tions can and should be administered with
interventions. This is critical thinking. food to help prevent gastric irritation.
Content – Medical: Integrated Nursing Process –
Content – Medical: Integrated Nursing Process –
Planning: Client Needs – Physiological Integrity,
Diagnosis: Client Needs – Physiological Integrity, Reduction
Pharmacological and Parenteral Therapies: Cognitive
of Risk Potential: Cognitive Level – Analysis: Concept –
Level – Synthesis: Concept – Medication.
Fluid and Electrolyte Balance.
34. 1. Fried foods are high in fat and cholesterol.
32. 1. Collateral circulation is the development
2. White bread is not high in fiber; wheat bread
of blood supply around narrowed arteries;
should be recommended because it is high
it helps prevent complications of athero-
in fiber. Whole milk is high in fat; skim milk
sclerosis, including myocardial infarction,
should be used.
cerebrovascular accidents, and peripheral
3. Baked, broiled, or grilled meats are
vascular disease. Exercise promotes the
recommended; a plain baked potato is
development of collateral circulation.
appropriate; and skim milk is low in

04_Colgrove_Ch04.indd 128 02/06/16 7:57 PM


C hapter 4  P eripheral V ascular D isorders 129

fat—so this meal is appropriate for a Abdominal Aortic Aneurysm


low-fat, low-cholesterol diet.
4. Hamburger meat is high in fat, French fries 37. 1. Shortness of breath indicates a respiratory
are usually cooked in oil (which is high in fat), problem or possibly a thoracic aneurysm, not
and carbonated beverages are high in calories. an AAA.
TEST-TAKING HINT: The nurse must be aware 2. A systolic bruit over the abdomen is a
of special diets, and a low-fat, low-cholesterol diagnostic indication of an AAA.
diet is often prescribed for clients with ath- 3. Ripping or tearing pain indicates a dissecting
erosclerosis. Remember, baked, broiled, and aneurysm.
grilled meats are lower in fat and cholesterol 4. Urine output is not diagnostic of an AAA.
than fried meats. TEST-TAKING HINT: The test taker who has
Content – Medical: Integrated Nursing Process – no idea of the answer should note that two
­Evaluation: Client Needs – Physiological Integrity, of the options—“2” and “3”—have the word
Basic Care and Comfort: Cognitive Level – Evaluation: “abdominal” and choose between them,
Concept – Nutrition. ­ruling out options “1” and “4,” both of
35. 1. Adherence to lifestyle modifications is en- which refer to other systems of the body
hanced when the client receives support (­respiratory and urinary).
from significant others. Content – Medical: Integrated Nursing Process –
2. Tobacco use is the most significant modi- Assessment: Client Needs – Physiological Integrity,
fiable risk factor that contributes to the Reduction of Risk Potential: Cognitive Level – Analysis:
development of atherosclerosis. Concept – Perfusion.
3. A sedentary lifestyle should be discouraged; 38. 1. When the aneurysm is small (less than
daily walking or swimming is encouraged. 5 to 6 cm), an abdominal sonogram will
4. This is an unrealistic intervention. The nurse be done every six (6) months until the
needs to help the client learn ways to deal with aneurysm reaches a size at which surgery
stressful situations, not avoid the situations. to prevent rupture is of more benefit than
5. Isometric exercises are weight-lifting exer- possible complications of an abdominal
cises, which should be discouraged; isotonic aortic aneurysm repair.
exercises, such as walking or swimming, are 2. An intravenous pyelogram evaluates the kidney.
encouraged. 3. The abdomen will not distend as the AAA
TEST-TAKING HINT: This type of alternate enlarges.
question requires the test taker to select all 4. This AAA is too small to perform surgery to
answer options that apply. Some interven- remove it.
tions are universal to all teaching, such as TEST-TAKING HINT: The AAA less than 3 cm
including significant others. Be careful with should make the test taker assume surgery is
words such as “avoid.” not an option; therefore, option “4” could be
Content – Medical: Integrated Nursing Process – ruled out as a correct answer. Pyelo- means
Planning: Client Needs – Physiological Integrity, “kidney,” so option “2” could be ruled out.
Physiological Adaptation: Cognitive Level – Synthesis: The term “medical treatment” in the stem of
Concept – Perfusion. the question should cause the test taker to
36. 1. Teaching cannot be delegated to a UAP. rule out abdominal girth.
2. The UAP can feed a client. Content – Medical: Integrated Nursing Process –
3. The UAP cannot assess the client and does not Diagnosis: Client Needs – Physiological Integrity,
have the education to interpret laboratory data. Physiological Adaptation: Cognitive Level – Knowledge:
4. A unit of blood must be checked by two (2) Concept – Perfusion.
registered nurses at the bedside. 39. 1. AAAs affect men four (4) times more often
TEST-TAKING HINT: Many states have rules than women.
concerning what tasks can be delegated to 2. AAAs affect men four (4) times more often
unlicensed assistive personnel, but even than women.
those states that don’t have delegation rules 3. The most common cause of AAA is
agree that teaching and assessing an unstable a­therosclerosis (which is the cause of
client cannot be delegated to unlicensed ­peripheral vascular disease); it occurs in
a­ssistive personnel. men four (4) times more often than in
Content – Nursing Management: Integrated Nursing women and primarily in Caucasians.
Process – Planning: Client Needs – Safe Effective Care 4. AAAs occur most often in elderly men, and
Environment, Management of Care: Cognitive Level – there is no genetic predisposition.
Synthesis: Concept – Nursing Roles.

04_Colgrove_Ch04.indd 129 02/06/16 7:57 PM


130 M ed -S urg S uccess

TEST-TAKING HINT: If the test taker knew that 2. The client must have 30 mL of urinary
AAA and peripheral vascular disease both output every hour. Clients who are
occur with atherosclerosis, it might possibly post-AAA are at high risk for renal failure
lead to the selection of option “3” as the because of the anatomical location of the
correct answer. AAA near the renal arteries.
Content – Medical: Integrated Nursing Process – 3. The client can sit on the bed the first day
­Assessment: Client Needs – Health Promotion and Mainte- postoperation; this is, in fact, encouraged.
nance: Cognitive Level – Analysis: Concept – Perfusion. 4. These vital signs would not warrant immedi-
ate intervention by the nurse.
40. 1. This statement would not make the nurse
suspect an AAA. TEST-TAKING HINT: A basic concept that the
2. Only about two-fifths of clients with test taker should remember is that any urine
AAA have symptoms; the remainder are output less than 30 mL/hr should be cause
asymptomatic. for investigation.
3. Periodic episodes of constipation and diar- Content – Medical: Integrated Nursing Process –
rhea may indicate colon cancer but do not ­Evaluation: Client Needs – Safe Effective Care
support a diagnosis of AAA. Environment, Management of Care: Cognitive Level –
4. Belching does not support a diagnosis of Synthesis: Concept – Perfusion.
AAA, but it could possibly indicate gastro- 43. 1. Assessment is the first part of the nursing
esophageal reflux or a hiatal hernia. process and is the first intervention the
TEST-TAKING HINT: The test taker must nurse should implement.
­remember not all disease processes or condi- 2. Administering an antibiotic is an appropri-
tions have signs/symptoms. The test taker ate intervention, but it is not a priority over
should attempt to determine what disease assessment.
processes the other answer options are 3. The client should splint the incision when
describing. coughing and deep breathing to help decrease
Content – Medical: Integrated Nursing Process – the pain, but this intervention is not a priority
Assessment: Client Needs – Physiological Integrity, over assessment.
Reduction of Risk Potential: Cognitive Level – Analysis: 4. Ambulating the client as soon as possible is
Concept – Perfusion. an appropriate intervention to help decrease
complications from immobility, but it is not a
41. 1. Low back pain is present because of
priority over assessment.
the pressure of the aneurysm on the
lumbar nerves; this is a serious symptom, TEST-TAKING HINT: If the test taker has
usually indicating that the aneurysm is difficulty in determining the first interven-
­expanding rapidly and about to rupture. tion, the test taker should always rely on the
2. If any pulses were affected, it would be the nursing process and select the assessment
pedal pulses, not the radial pulses. intervention if the intervention is appropri-
3. Decreased urine output would not indicate an ate for the disease process or condition.
expanding AAA, but decreased urine output Content – Surgical: Integrated Nursing Process – ­
may occur when the AAA ruptures, causing Implementation: Client Needs – Safe Effective Care
hypovolemia. Environment, Management of Care: Cognitive Level –
4. The abdominal girth would not increase for Synthesis: Concept – Perfusion.
an expanding AAA, but it might increase with 44. 1. The client is at risk for bleeding; therefore,
a ruptured AAA. this order would not be questioned.
TEST-TAKING HINT: If the test taker knows 2. Increased pressure in the abdomen
the anatomical position of the abdominal ­secondary to a tap water enema could
aorta and understands the term “expanding,” cause the AAA to rupture.
it may lead the test taker to select low back 3. The client should be able to ambulate to the
pain as the correct answer. bathroom without any problems.
Content – Medical: Integrated Nursing Process – ­ 4. Clients are NPO prior to surgery to help
Assessment: Client Needs – Physiological Integrity, prevent aspiration or problems from general
­Reduction of Risk Potential: Cognitive Level – Analysis: anesthesia.
Concept – Perfusion. TEST-TAKING HINT: An expanding AAA
42. 1. The nurse needs to intervene, but it does not should cause the test taker to realize that no
require immediate intervention. a­dditional pressure should be placed on the
AAA and that, therefore, selecting option “2”

04_Colgrove_Ch04.indd 130 02/06/16 7:57 PM


C hapter 4  P eripheral V ascular D isorders 131

would be the most appropriate answer. which information warrants immediate


­Options “1,” “3,” and “4” would be appropri- intervention.
ate for a client scheduled for an AAA repair Content – Surgical: Integrated Nursing Process –
or most types of ­surgeries. Remember basic Assessment: Client Needs – Safe Effective Care
concepts. Environment, Management of Care: Cognitive Level –
Content – Surgical: Integrated Nursing Process – ­ Analysis: Concept – Perfusion.
Implementation: Client Needs – Safe Effective Care 47. 1. Redness or irritation of the incision
Environment, Management of Care: Cognitive Level –
­indicates infection and should be reported
Application: Concept – Perfusion.
immediately to the HCP.
45. 1. The most common cause of AAA is 2. The client should not lift anything heavier
­atherosclerosis, so teaching should than five (5) pounds because it may cause
­address this area. dehiscence or evisceration of the bowel.
2. A low-fat, low-cholesterol diet will help 3. The pain medication should keep the client
decrease development of atherosclerosis. comfortable; if it doesn’t, the client should
3. The client should not decrease tobacco call the HCP.
use—he or she must quit totally. Smoking 4. Some clients do not have daily bowel move-
is the one modifiable risk factor that is not ments, but the nurse should instruct the cli-
negotiable. ent not to allow himself or herself to become
4. Losing weight will help decrease the constipated, which will increase pressure on
­pressure on the AAA and will help address the incision.
decreasing the cholesterol level. TEST-TAKING HINT: The test taker should use
5. A truss is worn by a client with a hernia, not basic concepts to answer questions. Signs/­
an AAA. symptoms of incisional infection or systemic
TEST-TAKING HINT: “Select all that apply” infection should always be reported to the
questions are an alternate-type question HCP. Remember, the test taker should not
that requires the nurse to select all interven- eliminate an option as a possible answer just
tions that are applicable to the question. The because he or she thinks it is too easy an
NCLEX-RN does not give partial credit; the answer.
test taker must select all appropriate answers Content – Surgical: Integrated Nursing Process –
to receive credit for the question. Planning: Client Needs – Physiological Integrity,
Content – Medical: Integrated Nursing Process – Physiological Adaptation: Cognitive Level – Synthesis:
­Planning: Client Needs – Physiological Integrity, Concept – Nursing Roles.
Physiological Adaptation: Cognitive Level – Synthesis:
48. 1. Organs in the right upper quadrant include
Concept – Perfusion.
the liver and gallbladder.
46. 1. Any neurovascular abnormality in the 2. The aorta traverses the abdomen in the
c­lient’s lower extremities indicates the midline position, and that is the best
graft is occluded or possibly bleeding ­location to hear an abdominal bruit.
and requires immediate intervention by The bell should be placed midline above
the nurse. the umbilicus to best auscultate an
2. The nurse would expect the client to have in- a­bdominal bruit.
cisional pain six (6) hours after surgery, so this 3. Organs in the left upper quadrant include the
is not a priority over a complication. stomach, pancreas, and spleen.
3. The nurse would expect the client to have 4. Organs in the left lower quadrant are the
a distended, tender abdomen as a result of ­colon and ovaries in females.
­postoperative edema. TEST-TAKING HINT: There may be questions
4. A slightly elevated temperature would not be on the NCLEX-RN that require the
uncommon in a client who has had surgery. test taker to point to a specific area. The
TEST-TAKING HINT: Any time the test taker test taker must know correct anatomical
has an answer option that has the word ­positions for body parts and organs.
“a­bsent” in it, the test taker should deter- Content – Fundamentals: Integrated Nursing Process –
mine if this is normal for any client and, if Assessment: Client Needs – Physiological Integrity, Reduction
not, consider it for the correct answer, espe- of Risk Potential: Cognitive Level – Knowledge: Concept –
cially in this case where the stem is asking Perfusion.

04_Colgrove_Ch04.indd 131 02/06/16 7:57 PM


132 M ed -S urg S uccess

Deep Vein Thrombosis Content – Surgical: Integrated Nursing Process –


Assessment: Client Needs – Safe Effective Care
Environment, Management of Care: Cognitive Level –
49. 1. The client will be taking an oral anticoagu-
Analysis: Concept – Clotting.
lant, warfarin (Coumadin). Prothrombin
time (PT) and international normalized 51. 1. Passive range-of-motion exercises are recom-
r­atio (INR) levels, not partial thrombo- mended to prevent contracture formation and
plastin time (PTT), are monitored when muscle atrophy, but this is a musculoskeletal
this medication is taken. The client should complication, not a cardiovascular one.
be in therapeutic range before discharge. 2. If the client is on a ventilator, then the paraly-
The HCP will determine how often to sis associated with GB syndrome has moved
monitor the levels, usually in two (2) to up the spinal column to include the muscles
three (3) weeks and then at three (3)- to six of respiration. Passive range-of-motion exer-
(6)-month intervals. cises are done by the staff; the client will not
2. The client is not restricted to the home. The be able to do active ROM.
client should not take part in any activity that 3. Range-of-motion exercises will not alleviate
does not allow frequent active and passive leg the pain of GB syndrome.
exercises. In an airplane, the client should be 4. One reason for performing range-of-­
instructed to drink plenty of fluids, move the motion exercises is to assist the blood
legs up and down, and flex the muscles. If in vessels in the return of blood to the heart,
an automobile, the client should stop to take preventing DVT.
frequent breaks to walk around. TEST-TAKING HINT: The question is asking for
3. Green, leafy vegetables contain vitamin K, a cardiovascular reason for range-of-motion
which is the antidote for warfarin. These exercises. Options “1,” “2,” and “3” do not
foods will interfere with the action of have any cardiovascular component. Only
warfarin. Red or brown urine may indicate ­option “4” discusses veins and blood.
bleeding.
Content – Medical: Integrated Nursing Process –
4. The client should be instructed to wear ­Implementation: Client Needs – Physiological Integrity,
stockings that do not constrict any area of Basic Care and Comfort: Cognitive Level – Analysis:
the leg. Concept – Clotting.
TEST-TAKING HINT: The test taker must know
52. 1. This protects the client’s privacy.
laboratory data for specific medications. The
2. The UAP could dislodge a blood clot in
INR and PT are monitored for oral antico-
the leg when massaging the calf. The UAP
agulants. Remember: “PT boats go to war”
can apply lotion gently, being sure not to
(­warfarin). PTT monitors heparin (“tt” is
massage the leg.
like an H for heparin).
3. Testing the temperature of the water prevents
Content – Medical: Integrated Nursing Process – scalding the client with water that is too hot
­Planning: Client Needs – Physiological Integrity, or making the client uncomfortable with wa-
Physiological Adaptation: Cognitive Level – Synthesis:
ter that is too cold.
Concept – Clotting.
4. Collecting supplies needed before beginning
50. 1. A complication of immobility after the bath is using time wisely and avoids inter-
s­urgery is developing a DVT. This rupting the bath to go and get items needed.
­client with left calf pain should be TEST-TAKING HINT: This is an “except” ques-
assessed for a DVT. tion, so all options except one (1) will be
2. This is an expected finding. actions that should be encouraged. The test
3. Clients who require an open cholecystectomy taker should not jump to the first option and
frequently are discharged with a T-tube. This choose it as the correct answer.
client needs to know how to care for the tube
Content – Medical: Integrated Nursing Process –
before leaving, but this is not a priority over a Evaluation: Client Needs – Safe Effective Care
possible surgical complication. Environment, Management of Care: Cognitive Level –
4. This is expected for this client. Synthesis: Concept – Clotting.
TEST-TAKING HINT: In priority-setting ques-
53. 1. Sequential compression devices provide gen-
tions, the test taker must decide if the in-
tle compression of the legs to prevent DVT,
formation in the answer option is expected
but they are not used to treat DVT because
or abnormal for the situation. Based on this,
the compressions could cause the clot to
­options “2,” “3,” and “4” can be eliminated.
break loose.

04_Colgrove_Ch04.indd 132 02/06/16 7:57 PM


C hapter 4  P eripheral V ascular D isorders 133

2. Clients should be on bedrest for five (5) to TEST-TAKING HINT: The test taker should
seven (7) days after diagnosis to allow time ­ etermine which option contains i­nformation
d
for the clot to adhere to the vein wall, that indicates a potentially life-threatening
thereby preventing embolization. situation. This is the priority client.
3. Bedrest and limited activity predispose Content – Medical: Integrated Nursing Process –
the client to constipation. Fluids and diets Assessment: Client Needs – Safe Effective Care
high in fiber will help prevent constipa- Environment, Management of Care: Cognitive Level –
tion. Fluids will also help provide ad- Synthesis: Concept – Clotting.
equate fluid volume in the vasculature.
56. 1. The nurse should check the laboratory
4. The client will be administered a heparin
values pertaining to the medications
IV drip, which should be monitored.
­before administering the medications.
5. Homans’ sign is assessed to determine if a
2. The client will be administered an oral medi-
DVT is present. This client has already been
cation while still receiving a heparin drip to
diagnosed with a DVT. Manipulating the leg
allow time for the client to achieve a thera-
to determine the presence of Homans’ sign
peutic level of the oral medication before
could dislodge the clot.
discontinuing the heparin. The effects of oral
TEST-TAKING HINT: Two (2) of the answer medications take three (3) to five (5) days to
options are used to determine if a DVT is become therapeutic.
present or to prevent one. The test taker 3. The laboratory values should be noted
should not become confused about treat- ­before administering the medications.
ment and prevention or early diagnosis. 4. The heparin will be continued for three (3) to
Content – Medical: Integrated Nursing Process – ­ five (5) days before being discontinued.
Implementation: Client Needs – Safe Effective Care TEST-TAKING HINT: Knowing the actions of
Environment, Management of Care: Cognitive Level –
each medication, as well as the laboratory
Application: Concept – Clotting.
tests that monitor the safe range of dosing,
54. 20 mL/hr. is important. Remember, assessment is first.
To determine the rate, the test taker must Assess blood levels and then administer the
first determine how many units are in each medication.
mL of fluid; 25,000 divided by 500 = 50 units Content – Medical: Integrated Nursing Process –
of heparin in each mL of fluid, and 50 divided ­Implementation: Client Needs – Physiological Integrity,
into 100 = 2, and 2 + 18 = 20. Pharmacological and Parenteral Therapies: Cognitive
TEST-TAKING HINT: This math question is Level – Synthesis: Clotting – Medication.
worked in several steps, but every step is 57. 1. Keeping the legs dependent and standing
simple addition or division. still will promote the development of a
Content – Medical: Integrated Nursing Process – DVT.
­Implementation: Client Needs – Physiological Integrity, 2. Flexing the leg muscles and changing
Pharmacological and Parenteral Therapies: Cognitive positions assist the blood to return to
Level – Application: Concept – Medication. the heart and move out of the peripheral
55. 1. A potentially life-threatening complica- vessels.
tion of DVT is a pulmonary embolus, 3. The nurse should wear support stockings,
which causes chest pain. The nurse should not socks, and change the types of shoes
determine if the client has “thrown” a worn from day to day, varying the type
­pulmonary embolus. of heels.
2. An immobile client should be turned at least 4. This is not in the client’s best interest.
every two (2) hours, but a pressure area is not TEST-TAKING HINT: The test taker can
life threatening. ­eliminate option “4” by imagining the
3. This is expected in a client who has a large ­reaction of the HCP if this were done.
upper abdominal incision. It hurts to breathe Thewords “dependent” and “still” make
deeply. The nurse should address this but has o ­ ption “1” wrong.
some time. The life-threatening complication Content – Medical: Integrated Nursing Process –
is priority. Diagnosis: Client Needs – Health Promotion and
4. Clients who have had inguinal hernia repair Maintenance: Cognitive Level – Knowledge:
often have difficulty voiding afterward. This Concept – Clotting.
is expected.

04_Colgrove_Ch04.indd 133 02/06/16 7:57 PM


134 M ed -S urg S uccess

58. 1. This occurs from the administration of the TEST-TAKING HINT: The test taker should
low molecular weight heparin and is not a devise some sort of memory-jogging mne-
reason to notify the HCP. monic or aid to remember which laboratory
2. A therapeutic range will not be achieved with test monitors for which condition. Try “PT
LMWH, and PTT levels are usually not boats go to war,” to recall that PT monitors
done. warfarin.
3. This is not hemorrhaging, and the cli- Content – Medical: Integrated Nursing Process –
ent should be reassured that this is a side ­Assessment: Client Needs – Physiological Integrity,
­effect of the medication. Pharmacological and ­Parenteral Therapies: Cognitive
4. Assessing the vital signs will not provide any Level – Analysis: Concept – Medication.
pertinent information to help answer the
­client’s question.
Peripheral Venous Disease
TEST-TAKING HINT: Before selecting “Notify
the HCP,” the test taker should ask, “What 61. 1. Venous insufficiency is a venous problem, not
will the HCP do with this information? What an arterial problem.
can the HCP order or do to help the purple 2. Deep vein thrombosis is not a complication
hemorrhaged areas?” This would cause of chronic venous insufficiency, but it may be
the test taker to eliminate option “1” as a a cause.
p
­ ossible answer. 3. Venous ulcerations are the most serious
Content – Surgical: Integrated Nursing Process – ­ complication of chronic venous insuffi-
Implementation: Client Needs – Safe Effective Care ciency. It is very difficult for these ulcer-
Environment, Management of Care: Cognitive Level – ations to heal, and often clients must be
Synthesis: Concept – Medication.
seen in wound care clinics for treatment.
59. 1. There is nothing that contraindicates the 4. Varicose veins may lead to chronic venous
use of a stool softener, and use of one may be insufficiency, but they are not a complication.
recommended if the client is prone to consti- TEST-TAKING HINT: The test taker must
pation and hard stool that could cause some use knowledge of anatomy, which would
bleeding from hemorrhoids. eliminate option “1” because “venous” and
2. A Medic Alert bracelet notifies any emer- “­arterial” refer to different parts of the
gency HCP of the client’s condition and ­vascular system. The test taker must key in
medications. on the most serious complication to select
3. Vitamin E can affect the action of warfa- the correct answer.
rin. The nurse should explain to the client Content – Medical: Integrated Nursing Process –
that these and other medications could Planning: Client Needs – Health Promotion and
potentiate the action of warfarin. Maintenance: Cognitive Level – Synthesis:
4. This would be recommended for the client Concept – Clotting.
if the footrest does not restrict blood flow in
the calves. 62. 1. The medial part of the ankle usually
­ulcerates because of edema that leads to
TEST-TAKING HINT: The test taker can elimi- stasis, which, in turn, causes the skin to
nate option “1” by realizing that a stool break down.
softener would not cause a problem and 2. A deep, pale, open area over the top side of
could help with an unrelated problem. Medic the foot describes an arterial ulcer.
Alert bracelets are frequently recommended 3. A reddened blistered area on the heel
for many clients with certain diseases and ­describes a blister that may result from
conditions. ­wearing shoes that are too tight or that rub
Content – Medical: Integrated Nursing Process – on the heel.
Evaluation: Client Needs – Safe Effective Care 4. Gangrene does not usually occur with venous
Environment, Management of Care: Cognitive Level – problems; it occurs with arterial ulcers.
Synthesis: Concept – Clotting.
TEST-TAKING HINT: There are some questions
60. 1. BUN laboratory tests are measurements of that require the test taker to be knowledge-
renal functioning. able of the disease process.
2. Bilirubin is a liver function test. Content – Medical: Integrated Nursing Process –
3. PT/INR is a test to monitor warfarin Assessment: Client Needs – Physiological Integrity,
(Coumadin) action in the body. Reduction of Risk Potential: Cognitive Level – Analysis:
4. PTT levels monitor heparin activity. Concept – Clotting.

04_Colgrove_Ch04.indd 134 02/06/16 7:57 PM


C hapter 4  P eripheral V ascular D isorders 135

63. 1. Low-heeled, comfortable shoes should be correct answer. Venous blood goes back
recommended to help decrease foot pain, but to the heart, so elevating the feet will help
they will not help prevent varicose veins. ­return it. Options “3” and “4” do not have
2. Wearing clean white socks will help prevent anything to do with the extremities.
irritation to the feet, but they will not help Content – Medical: Integrated Nursing Process –
prevent varicose veins. Evaluation: Client Needs – Physiological Integrity,
3. Moving the legs back and forth often may Physiological Adaptation: Cognitive Level – Evaluation:
help prevent deep vein thrombosis, but it will Concept – Clotting.
not prevent varicose veins. 66. 1. Research shows that removing the com-
4. Graduated compression hose help pression stockings while the client is in
­decrease edema and increase the circula- bed promotes perfusion of the subcutane-
tion back to the heart; this helps prevent ous tissue. The foot of the bed should be
varicose veins. elevated.
TEST-TAKING HINT: Options “1” and “2” could 2. The UAP can take the blood pressure with
be eliminated as possible answers if the test a machine or manually; therefore, the nurse
taker knows that the varicose veins are in the would not need to intervene.
leg, ­because options “1” and “2” are address- 3. The UAP can help the client with meals as
ing the feet. long as the client is stable.
Content – Medical: Integrated Nursing Process – 4. The UAP can calculate the intake and out-
Planning: Client Needs – Health Promotion and Mainte- put, but the nurse must evaluate the data to
nance: Cognitive Level – Synthesis: Concept – Clotting. determine if they are normal for the client.
64. 1. Varicose veins are irregular, tortuous veins TEST-TAKING HINT: This is a backward
with incompetent valves that do not allow ­“except” question. Flipping the question and
the venous blood to ascend the saphenous asking which actions would be appropriate
vein. for the UAP to implement might make it eas-
2. Decreased oxygen to the muscle occurs with ier for the test taker to answer the question.
arterial occlusive disease. Content – Medical: Integrated Nursing Process –
3. This is the explanation for a deep vein Evaluation: Client Needs – Safe Effective Care
thrombosis. Environment, Management of Care: Cognitive
4. Thick, poorly circulating blood could be an Level – Synthesis: Concept – Clotting.
explanation for diabetic neuropathy. 67. 1. The occupational therapist assists the client
TEST-TAKING HINT: Knowing that veins have with activities of daily living skills, such as
valves and arteries do not might help the eating, bathing, or brushing teeth.
test taker select the correct answer. The test 2. The social worker would assess the client
taker should use knowledge of anatomy and to determine if home health-care services
physiology to determine the answer. or financial interventions were appropri-
Content – Medical: Integrated Nursing Process – ate for the client. The client is elderly,
­Implementation: Client Needs – Safe Effective Care immobility is a concern, and wound care
Environment, Management of Care: Cognitive must be a concern when the client is
Level – Analysis: Concept – Clotting. discharged home.
65. 1. The client should not cross the legs at all 3. The physical therapist addresses gait training
because this further impedes the blood from and transferring.
ascending the saphenous vein. 4. Cardiac rehabilitation helps clients who have
2. Elevating the foot of the bed while had myocardial infarctions, cardiac bypass
­sleeping helps the venous blood return to surgery, or congestive heart failure recover.
the heart and decreases pressure in the TEST-TAKING HINT: The test taker must be
lower extremity. aware of the responsibilities of other mem-
3. Antiplatelet therapy is for arterial blood, not bers of the health-care team. “Discharge”
venous blood. is the key word in the stem. The test taker
4. Fluid intake will not help prevent or improve should select an answer that will help the
chronic venous insufficiency. ­client in the home.
TEST-TAKING HINT: Knowing about the Content – Surgical: Integrated Nursing Process –
v­enous and arterial blood systems will help Planning: Client Needs – Safe Effective Care Environment,
the test taker eliminate or identify the Management of Care: Cognitive Level – Synthesis:
Concept – Skin Integrity.

04_Colgrove_Ch04.indd 135 02/06/16 7:57 PM


136 M ed -S urg S uccess

68. 1. Pedal pulses are normal in venous insuffi- TEST-TAKING HINT: When the question asks
ciency, but pulses are decreased or absent in the test taker to implement the first inter-
arterial insufficiency. vention, two or more of the answer options
2. The skin is warm in venous insufficiency; the could be possible interventions, but only
skin is cool in arterial insufficiency. one is implemented first. Apply the nursing
3. Intermittent claudication, pain that ­occurs process and select the intervention that ad-
when walking, is a symptom of arterial dresses assessment, which is the first part of
insufficiency. the nursing process.
4. Chronic venous insufficiency leads to Content – Surgical: Integrated Nursing Process – ­
chronic edema that, in turn, causes a Implementation: Client Needs – Safe Effective Care
brownish pigmentation to the skin. Environment, Management of Care: Cognitive
TEST-TAKING HINT: The test taker could Level – Synthesis: Concept – Clotting.
­apply anatomical concepts to eliminate 71. 1. The client with a venous stasis ulcer should
both options “1” and “2” because it is the have pain, so this would be expected.
arteries that have pulses and control the 2. Dull, aching muscle cramps are expected with
temperature of the skin. varicose veins.
Content – Medical: Integrated Nursing Process – 3. The inability to move the foot means that
Assessment: Client Needs – Physiological Integrity, a severe neurovascular compromise has
Reduction of Risk Potential: Cognitive Level – Analysis: occurred, and the nurse should assess this
Concept – Clotting. client first.
69. 1. Varicose veins are more common in white 4. A positive Homans’ sign is expected in a
females in occupations that involve client diagnosed with deep vein thrombosis.
prolonged standing. TEST-TAKING HINT: The nurse should first
2. Driving a bus does not require prolonged assess the client who is experiencing an
standing, which is a risk factor for developing abnormal, unexpected, or life-threatening
varicose veins. complication of the disease process. Do not
3. Studies suggest that the increased risk for automatically select the client with pain; pain
varicose veins is common during pregnancy in many instances is not life threatening or
and may be the result of venous stasis. unexpected.
4. Diabetes may lead to diabetic neuropathy and Content – Medical: Integrated Nursing Process –
arterial occlusive disease, but it does not lead Assessment: Client Needs – Safe Effective Care
to varicose veins. Environment, Management of Care: Cognitive
TEST-TAKING HINT: The test taker must know Level – Analysis: Concept – Clotting.
that prolonged standing is a risk factor for 72. 1. The nurse should determine if the client
varicose veins and identify which occupations has any numbness or tingling.
include being on the feet most of the time. 2. The nurse should determine if the client
Content – Medical: Integrated Nursing Process – has pulses, the presence of which indi-
Assessment: Client Needs – Health Promotion and Mainte- cates there is no circulatory compromise.
nance: Cognitive Level – Analysis: Concept – Clotting. 3. The nurse should determine if the client
70. 1. Because the saphenous vein is removed dur- can move the feet and legs.
ing vein ligation, standing and sitting are 4. The nurse should determine if the ­client’s
prohibited during the initial recovery period feet are pink or pale.
to prevent increased pressure in the lower 5. The nurse should assess the feet to
extremities. ­determine if they are cold or warm.
2. Pressure bandages are applied for up to TEST-TAKING HINT: These five (5) assessment
six (6) weeks after vein ligation to help interventions, along with assessing for pain,
prevent bleeding and to help venous re- are known as the 6 Ps, which is the neurovas-
turn from the lower extremities when in cular assessment.
the standing or sitting position. Content – Medical: Integrated Nursing Process –
3. Sequential compression devices are used to Assessment: Client Needs – Physiological Integrity,
help prevent deep vein thrombosis. Reduction of Risk Potential: Cognitive Level – Analysis:
4. Antibiotics would be ordered prophylactically Concept – Clotting.
for surgery, but it is not the first intervention.

04_Colgrove_Ch04.indd 136 02/06/16 7:57 PM


CONCEPTS

73. 1. Bright red tissue indicates an arterial problem. commonly administered medications.
2. Purplish-brown areas on the skin indicate This will allow the test taker to make
venous stasis of the blood in the legs. decisions as to the appropriate nursing
3. This indicates arterial occlusive disease. actions to implement.
4. This could indicate diabetic foot/ulcers but Content – Medical: Integrated Nursing Process –
does not mean venous problems. Intervention: Client Needs – Pharmacological Integrity:
TEST-TAKING HINT: The test taker should Cognitive Level – ­Application: Concept – Perfusion.
apply pathophysiology knowledge to answer 76. 1. The UAP has informed the nurse that a
this question. Venous blood is darker than client is having chest pain. A DVT can
arterial blood, which is brighter red. break loose and become an embolism,
Content – Medical: Integrated Nursing Process – which can cause life-threatening problems
Assessment: Client Needs – Safe Effective Care for the client.
Environment, Management of Care: Cognitive 2. The blood pressure is WNL.
Level – Application: Concept – Perfusion. 3. This is not an unusual request and the
74. 1. Indicates a venous insufficiency issue, not a clot. nurse should discuss the need for limited
2. Indicates an arterial issue, not a clot. activity, but it does not require immediate
3. A clot disrupts the blood flow; swelling intervention.
and warmth, along with pain, indicate a 4. A headache is not life threatening. It does not
potential blood clot. require immediate intervention.
4. Indicates arterial occlusive disease usually TEST-TAKING HINT: The test taker should
caused by narrowing of the arteries as a result ask himself/herself which could be the most
of atherosclerosis. serious of the situations listed for the
TEST-TAKING HINT: The test taker should client. This is the one that requires
apply pathophysiology knowledge to answer immediate intervention.
this question. Venous blood is returning to Content – Medical: Integrated Nursing Process –
the heart to pick up oxygen from the lungs Intervention: Client Needs – Safety: Cognitive Level –
and be redistributed to the body, making it a Synthesis: Concept – Perfusion.
bright red color; oxygen in the blood makes 77. 1. Auscultation of heart and lung sounds are
the blood bright red. This could eliminate part of a full assessment, but the first action
both options “1” and “2.” is to focus on the client’s chief complaint; the
Content – Medical: Integrated Nursing Process – results of this assessment will guide the re-
Assessment: Client Needs – Safe Effective Care mainder of the nurse’s actions.
Environment, Management Care: Cognitive 2. The length of the trip is implicated in the
Level – Application: Concept – Perfusion.
client developing a blood clot because of
75. 1. The client is in therapeutic range for intrave- the lower moisture content in the air being
nous heparin; the nurse has no reason to ask breathed and the lack of movement of the
for a change of medication. lower extremities during the flight; however,
2. Heparin is administered based on the aPPT if the client has developed a clot, then that is
results; vitamin K is the antidote for warfarin the issue now.
(Coumadin). 3. If the client had experienced chest pain, that
3. Green, leafy vegetables apply to warfarin, not would have been the chief complaint; the
heparin, and all green, leafy vegetables are nurse needs to assess the client for a DVT.
not prohibited. The client should consume a 4. Measuring the client’s calf and assessing for
consistent amount of green, leafy vegetables warmth are part of a focused assessment
in order for the INR levels to maintain a for deep vein thrombosis, which the client’s
therapeutic range. flight placed him/her at risk for DVT.
4. The client is in therapeutic range for TEST-TAKING HINT: The test taker should ap-
intravenous heparin; the nurse should ply decision-making questions when deter-
administer the heparin as ordered. mining which to do first. One such question
TEST-TAKING HINT: The test taker should be is: Which option will give the nurse the most
aware of therapeutic levels as they apply to needed information the fastest? The nurse
137

04_Colgrove_Ch04.indd 137 02/06/16 7:57 PM


138 M ed -S urg S uccess

should acquire information which will elimi- leafy vegetables will change the antico-
nate or support the suspected diagnosis. agulant effects on the body.
Content – Medical: Integrated Nursing Process – 2. The INR is the level that the HCP will
Assessment: Client Needs – Safe Effective Care use to gauge the anticoagulant effects
Environment, Management Care: Cognitive Level – occurring in the body and should be
Application: Concept – Perfusion. monitored regularly.
3. The nurse should teach the client to apply
78. 1. The nurse must complete the assessment
pressure on any bleeding for five (5) minutes
before notifying the HCP.
to see if a minor cut will stop bleeding on its
2. This can be done but most clients cannot
own. The client should not go to the hospital
provide a stool sample immediately.
unless bleeding cannot be stopped.
3. The timing is not important at this time;
4. Dark, tarry stools indicate upper GI
the client has been taking the medication.
bleeding and the HCP should be
4. Ecchymotic areas (bruising) indicate
informed.
bleeding; the nurse should determine
5. The client should consume a consistent
the extent of the client’s bleeding before
amount of green, leafy vegetables; they are
notifying the HCP.
part of a healthy diet.
TEST-TAKING HINT: When answering a ques- 6. Iron does not prevent bleeding.
tion and the test taker wishes to choose “no-
TEST-TAKING HINT: When answering a ­“Select
tify the health-care provider (HCP),” the test
all that apply” question, the test taker should
taker should read all the options carefully; if
read each option as a true/false option. If the
any option has needed information to report
option is true then the test taker should
to the HCP, then that option comes before
choose it. All of the options may be correct
notifying the HCP.
or only one may be.
Content – Medical: Integrated Nursing Process – ­
Intervention: Client Needs – Safe Effective Care Environ- Content – Medical: Integrated Nursing Process –
ment, Management Care: Cognitive Level – ­Application: Planning: Client Needs – Safe Effective Care Environment,
Concept – Perfusion. Management Care: Cognitive Level – Application:
Concept – Perfusion.
79. 1. Isometric exercises are “pumping iron” or
81. 1. Gastric upset is not an issue for a client
body-building exercises. Isotonic exercises are
­receiving anticoagulant therapy.
what the nurse teaches the client to perform.
2. Anticoagulant therapy reduces the c­ lient’s
2. A stair stepper places considerable require-
ability to form clots; bleeding is the most
ments for lower body strength on the client.
important issue to discuss with the client.
The client with arterial occlusive disease may
3. Constipation is not caused by anticoagulant
not have the ability to use this machine.
therapy.
3. Warm-up exercises are needed to prevent
4. A myocardial infarction is not caused
injury to the client.
by ­anticoagulant therapy but it could be
4. The client should walk as the preferred
­prevented by it.
form of exercise, the shoes should fit the
client without causing blisters, and walk- TEST-TAKING HINT: When answering ques-
ing on level ground decreases the risk of tions about medications, the nurse must be
injury or excessive stress on the muscles. aware of common instructions. In the case
TEST-TAKING HINT: The test taker could
of any medication that involves changing the
eliminate option “1” if aware of the definition body’s ability to form a clot, bleeding must
of isometric; the test taker must be knowl- always be an issue.
edgeable of medical terminology. Content – Medical: Integrated Nursing Process –
Planning: Client Needs – Pharmacological Integrity:
Content – Medical: Integrated Nursing Process –
Cognitive Level – Application: Concept – Perfusion.
Planning: Client Needs – Safe Effective Care Environment,
Management Care: Cognitive Level – Application: 82. 1. Clients diagnosed with hypertension must
Concept – Perfusion. have had a BP of 140/90 on three separate
80. 1. Green, leafy vegetables contain vitamin K, occasions unless the client has diabetes and
which is the antidote for warfarin; when the BP reading is 135/85. The numbers may
achieving a therapeutic range for the INR, reach much higher levels. Over 160 systolic
a change in the consumption of green, and 90 to 95 diastolic places the client at risk
for cardiovascular events. The client’s BP

04_Colgrove_Ch04.indd 138 02/06/16 7:57 PM


C hapter 4  P eripheral V ascular D isorders 139

should be controlled according to the HCP help as soon as possible. The rules of the
instructions but 120/80 is not a realistic goal. RRT are that anyone can call an RRT if
2. Many clients decide that because they a concern is noted, and no one will suffer
do not feel ill, medication is not needed. consequences because one was called and
Hypertension is called the silent killer be- it was determined that the client was not
cause damage to the body can occur with- in serious danger.
out the client realizing it. 2. The nurse’s first action is to stay with the cli-
3. The client should be taught to rise slowly ent and call for help.
from a recumbent positon to prevent dizzi- 3. The UAP cannot be assigned an unstable
ness and falls. client.
4. The client should limit sodium in the diet to 4. The telemetry reading is not important in
2,000 mg/day. regard to the current clinical manifestations.
TEST-TAKING HINT: The test taker should TEST-TAKING HINT: Remember, “if in stress,
be aware of commonly administered medi- do not assess”; the nurse must implement an
cations and which instructions apply. This intervention that will directly affect the cli-
will allow the test taker to make decisions ent outcome.
as to the appropriate nursing actions to Content – Medical: Integrated Nursing Process – ­
implement. Intervention: Client Needs – Safe Effective Care
Content – Medical: Integrated Nursing Process – Environment, Management Care: Cognitive
Planning: Client Needs – Pharmacological Integrity: Level – A
­ pplication: Concept – Perfusion.
Cognitive Level – Application: Concept – Perfusion.
85. 1. Clopidogrel (Plavix) is an arterial anti-
83. 1. Elevating the legs will assist the blood to platelet that prevents clots from occurring
return to the heart. in the lower extremity arteries.
2. Antiembolism stockings compress the 2. Streptokinase is an enzyme that breaks down
veins and prevent stasis in the legs. existing clots, a “clot buster.”
3. The circulation in the legs is compro- 3. Protamine sulfate is the antidote for heparin.
mised. Injuries will take longer to heal. 4. Enoxaparin is a subcutaneous low molecular
4. Trimming the toenails straight across weight heparin and not usually administered
prevents ingrown toenails. for arterial occlusive disease.
5. The client may apply moisturizer to the legs. TEST-TAKING HINT: The test taker should
6. The legs being in a dependent position ap- be aware of commonly administered medi-
plies to clients diagnosed with arterial oc- cations and which instructions apply. This
clusive disease. This assists the arterial flow will allow the test taker to make decisions
to the feet. The problem with venous stasis is as to the appropriate nursing actions to
getting the blood back to the heart; the feet implement.
should be elevated. Content – Medical: Integrated Nursing Process –
TEST-TAKING HINT: When answering a Intervention: Client Needs – Pharmacological Integrity:
“­Select all that apply” question the test taker Cognitive Level – Application: Concept – Perfusion.
should read each option as a true/false op- 86. 1. The client should occasionally monitor his
tion. If the option is true then the test taker blood pressure but not four times a day.
should choose it. All of the options may be 2. The antihypertensive medications do not
correct or only one may be. By knowing the have established therapeutic blood levels.
pathophysiology of blood circulation the test 3. Antihypertensive medications can cause
taker can choose option “1” because venous a drop in the blood pressure when the
circulation is returning blood to the heart. client changes positions from a sitting
The test taker could eliminate option “6” or lying position to an upright position
b­ecause it is the opposite of option “1.” because of gravity and relaxed blood ves-
Content – Medical: Integrated Nursing ­Process – sels. This is orthostatic hypotension; the
Planning: Client Needs – Safe Effective Care Environment, blood vessels will adjust if the client rises
Management Care: Cognitive Level – ­Application:
slowly or sits on the side of the bed for a
Concept – Perfusion.
short time.
84. 1. These clinical manifestations could 4. Blood glucose meters measure glucose levels,
i­ndicate a pulmonary embolus. The nurse which is not what should be monitored for
should not leave the client but should get hypertension.

04_Colgrove_Ch04.indd 139 02/06/16 7:57 PM


140 M ed -S urg S uccess

TEST-TAKING HINT: The test taker should be 89. 1. The usual guideline is to keep the blood pres-
aware of commonly administered medications sure to less than 160/100; 140/90 is the low
and which instructions apply. This will allow number to diagnose hypertension.
the test taker to make decisions as to the ap- 2. This will improve cardiovascular fitness
propriate nursing actions to implement. but will not directly cause the increase in
Content – Medical: Integrated Nursing Process – atherosclerosis.
Intervention: Client Needs – Pharmacological Integrity: 3. This will allow the client to monitor the
Cognitive Level – A­ pplication: Concept – Perfusion. blood pressure but does not reduce the risk of
an MI.
87. 1. Blood pressure readings measure arterial
4. Atherosclerosis is caused by plaque
pressures; the cuff should be placed so the
buildup in the arteries. Plaque is primarily
pressure in an artery can be read.
caused by fat in the diet compounded by
2. The client should be instructed as to when
clotting mechanisms.
to notify the HCP. BP readings over 160
systolic or 100 diastolic indicate the blood TEST-TAKING HINT: The test taker must be
pressure is not controlled and the medica- aware of the recommended treatment for the
tion regimen might need to be adjusted. prevention of the problems associated with
3. Antihypertensive medications can cause a disease processes in order to teach the client
drop in the blood pressure when the client about the disease.
changes from a sitting or lying position to Content – Medical: Integrated Nursing Process –
an upright position because of gravity and Intervention: Client Needs – Safe Effective Care
relaxed blood vessels. This is orthostatic Environment, Management Care: Cognitive
hypotension; the blood vessels will adjust Level – Application: Concept – Perfusion.
if the client rises slowly or sits on the side 90. 1. Type 2 HIT is an immune mediated ­disorder
of the bed for a short time. that typically occurs after exposure to heparin
4. A record of the blood pressure reading for 4 to 10 days and has life-threatening and
obtained by the client can assist the HCP limb-threatening thrombotic complications.
in planning the suggested regimen. The client clots rather than bleeds. Spontane-
5. Many clients decide that if they are WNL ous bleeding is not associated with HIT. In
the medication is no longer needed. The general practice type 2 HIT is referred to as
client should understand if the readings simple HIT.
are WNL it is because of the medication 2. HIT is not manifested by bleeding but
and stopping the medication will stop the by the development of clots, either deep
desired effect. venous or pulmonary, and sometimes
TEST-TAKING HINT: The test taker should arterially, which can cause a myocardial
be aware of commonly administered medi- infarction. These are symptoms of a
cations and which instructions apply. This pulmonary embolus.
will allow the test taker to make decisions 3. HIT is a decrease in baseline platelet
as to the appropriate nursing actions to count by 50% of baseline.
implement. 4. Bleeding is not associated with HIT.
Content – Medical: Integrated Nursing Process – 5. Clinically, HIT may manifest itself as skin
Intervention: Client Needs – Pharmacological Integrity: lesions at the site of heparin ­injections or
Cognitive Level – A­ pplication: Concept – Perfusion. chills, fever, dyspnea, or chest pain.
88. 15 mL per hour. The nurse must first deter- TEST-TAKING HINT: The test taker must
mine how many units are in each mL of fluid. remember diseases and syndromes that do
40,000 ÷ 500 mL = 80 units per mL. Next not match with the general picture of what is
the nurse must determine how many mL occurring. In HIT the thrombocytopenia is
should be administered each hour: not associated with bleeding but rather with
1200 ÷ 80 = 15 mL per hour. clotting. This could eliminate options “1”
and “4” because both are options indicating
TEST-TAKING HINT: The test taker should be
the client is bleeding.
aware of common mathematical equations
Content – Medical: Integrated Nursing Process –
in order to administer the correct dose of
Assessment: Client Needs – Safe Effective Care
medication. Environment, Management of Care: Cognitive
Content – Medical: Integrated Nursing Process – Level – Application: Concept – Perfusion.
Intervention: Client Needs – Pharmacological Integrity/
Mathematics: Cognitive Level – Application: Concept –
Perfusion.

04_Colgrove_Ch04.indd 140 02/06/16 7:57 PM


C hapter 4  P eripheral V ascular D isorders 141

PERIPHERAL VASCULAR DISORDERS


COMPREHENSIVE EXAMINATION
1. Which client behavior would be a causative 7. The client diagnosed with diabetes mellitus type 2
factor for developing Buerger’s disease is admitted to the hospital with cellulitis of the
(thromboangiitis obliterans)? right foot secondary to an insect bite. Which
1. Drinking alcohol daily. intervention should the nurse implement first?
2. Eating a high-fat diet. 1. Administer intravenous antibiotics.
3. Chewing tobacco. 2. Apply warm moist packs every two (2) hours.
4. Inhaling gasoline fumes. 3. Elevate the right foot on two (2) pillows.
4. Teach the client about skin and foot care.
2. Which signs/symptoms would the nurse expect
to find when assessing a client diagnosed with 8. Which discharge instruction should the nurse
subclavian steal syndrome? discuss with the client to prevent recurrent
1. Complaints of arm tiredness with exertion. episodes of cellulitis?
2. Complaints of shortness of breath while 1. Soak your feet daily in Epsom salts for
resting. 20 minutes.
3. Jugular vein distention when sitting at a 2. Wear thick white socks when working in the
35-degree angle. yard.
4. Dilated blood vessels above the nipple line. 3. Use a mosquito repellent when going outside.
4. Inspect the feet between the toes for cracks in
3. Which question should the nurse ask the male
the skin.
client diagnosed with aortoiliac disease during the
admission interview? 9. Which discharge instruction should the nurse
1. “Do you have trouble sitting for long periods teach the client diagnosed with varicose veins
of time?” who has received sclerotherapy?
2. “How often do you have a bowel movement 1. Walk 15 to 20 minutes three (3) times a day.
and urinate?” 2. Keep the legs in the dependent position when
3. “When you lie down, do you feel throbbing in sitting.
your abdomen?” 3. Remove compression bandages before going
4. “Have you experienced any problems having to bed.
sexual intercourse?” 4. Perform Buerger-Allen exercises four (4)
times a day.
4. The client is four (4) hours postoperative
abdominal aortic aneurysm repair. Which nursing 10. The nurse is teaching the client with peripheral
intervention should be implemented for this vascular disease. Which interventions should
client? the nurse discuss with the client? Select all that
1. Assist the client to ambulate. apply.
2. Assess the client’s bilateral pedal pulses. 1. Wash your feet in antimicrobial soap.
3. Maintain a continuous IV heparin drip. 2. Wear comfortable, well-fitting shoes.
4. Provide a clear liquid diet to the client. 3. Cut your toenails in an arch.
4. Keep the area between the toes dry.
5. Which referral would be most appropriate
5. Use a heating pad when feet are cold.
for the client diagnosed with thoracic outlet
syndrome? 11. The unlicensed assistive personnel (UAP) is
1. The physical therapist. applying elastic compression stockings to the
2. The thoracic surgeon. client. Which action by the UAP would warrant
3. The occupational therapist. immediate intervention by the nurse?
4. The social worker. 1. The UAP is putting the stockings on while
the client is in the chair.
6. Which instruction should the nurse discuss
2. The UAP inserted two (2) fingers under the
with the client diagnosed with Raynaud’s
proximal end of the stocking.
phenomenon?
3. The UAP elevated the feet while lying down
1. Explain exacerbations will not occur in the
prior to putting on the stockings.
summer.
4. The UAP made sure the toes were warm after
2. Use nicotine gum to help quit smoking.
putting the stockings on.
3. Wear extra-warm clothing during cold
exposure.
4. Avoid prolonged exposure to direct sunlight.

04_Colgrove_Ch04.indd 141 02/06/16 7:57 PM


142 M ed -S urg S uccess

12. The nurse is administering a beta blocker to the 17. The client is receiving prophylactic low
client diagnosed with essential hypertension. molecular weight heparin. There are no
Which data would cause the nurse to question PT/PTT or INR results on the client’s chart
administering the medication? since admission three (3) days ago. Which action
1. The client’s BP is 110/70. should the nurse implement?
2. The client’s potassium level is 3.4 mEq/L. 1. Administer the medication as ordered.
3. The client has a barky cough. 2. Notify the health-care provider immediately.
4. The client’s apical pulse is 56. 3. Obtain the PT/PTT and INR prior to
administering the medication.
13. The client diagnosed with acute deep vein
4. Hold the medication until the HCP makes
thrombosis is receiving a continuous heparin
rounds.
drip, an intravenous anticoagulant. The health-
care provider orders warfarin (Coumadin), an 18. The client diagnosed with atherosclerosis asks
oral anticoagulant. Which action should the the nurse, “I have heard of atherosclerosis for
nurse take? many years but I never really knew what it
1. Discontinue the heparin drip prior to meant. Am I going to die?” Which statement
initiating the Coumadin. would be the nurse’s best response?
2. Check the client’s INR prior to beginning 1. “This disease process will not kill you, so
Coumadin. don’t worry.”
3. Clarify the order with the health-care 2. “The blood supply to your brain is being
provider as soon as possible. cut off.”
4. Administer the Coumadin along with the 3. “It is what caused you to have your high blood
heparin drip as ordered. pressure.”
4. “Atherosclerosis is a buildup of plaque in your
14. The nurse is caring for the client on strict
arteries.”
bedrest. Which intervention is priority when
caring for this client? 19. Which signs/symptoms would the nurse expect
1. Encourage the client to drink liquids. to find in the female client diagnosed with
2. Perform active range-of-motion exercises. Marfan’s syndrome?
3. Elevate the head of the bed to 45 degrees. 1. Xerostomia, dry eyes, and complaints of a dry
4. Provide a high-fiber diet to the client. vagina.
2. A triad of arthritis, conjunctivitis, and
15. The nurse is caring for clients on a medical floor.
urethritis.
Which client will the nurse assess first?
3. Very tall stature and long bones in the hands
1. The client with an abdominal aortic aneurysm
and feet.
who is constipated.
4. Spinal deformities of the vertebral column
2. The client on bedrest who ambulated to the
and malaise.
bathroom.
3. The client with essential hypertension who 20. The nurse is preparing to administer 7.5 mg of
has epistaxis and a headache. an oral anticoagulant. The medication available
4. The client with arterial occlusive disease who is 5 mg per tablet. How many tablets should the
has a decreased pedal pulse. nurse administer? _________
16. The client with peripheral venous disease 21. Which dietary selection indicates the client
is scheduled to go to the whirlpool for a with essential hypertension understands the
dressing change. Which is the nurse’s priority discharge teaching?
intervention? 1. Fried pork chops, a loaded baked potato, and
1. Escort the client to the physical therapy coffee.
department. 2. Spaghetti and meatballs, garlic bread, and iced
2. Medicate the client 30 minutes before going tea.
to the whirlpool. 3. Baked ham, macaroni and cheese, and milk.
3. Obtain the sterile dressing supplies for the 4. Broiled fish, steamed broccoli, and garden
client. salad.
4. Assist the client to the bathroom prior to the
treatment.

04_Colgrove_Ch04.indd 142 02/06/16 7:57 PM


C hapter 4  P eripheral V ascular D isorders 143

22. The client diagnosed with Buerger’s disease 25. The client is four (4) hours postoperative
(thromboangiitis obliterans) asks the nurse, femoral-popliteal bypass surgery. Which
“What is the worst thing that could happen pulse would be best for the nurse to assess for
if I don’t quit smoking? I love my cigarettes.” complications related to an occluded vessel?
Which statement is the nurse’s best response?
1. “You are concerned about quitting smoking.
Let’s sit down and talk about it.”
2. “Many clients end up having to have an
amputation, especially a leg.”
3. “You should consider attending a smoking
cessation program.”
4. “Your coronary arteries could block and cause
a heart attack.”
23. The client diagnosed with subclavian steal
syndrome has undergone surgery. Which
assessment data would warrant immediate
intervention by the nurse? A
1. The client’s pedal pulse on the left leg is
absent. B
2. The client complains of numbness in the right
hand.
3. The client’s brachial pulse is strong and
bounding.
4. The client’s capillary refill time (CRT) is less C
than three (3) seconds.
24. The client with a left-sided mastectomy is
diagnosed with elephantiasis of the left arm.
Which signs/symptoms should the nurse expect
D
to assess?
1. Edematous arm from the axillary area to
the fingertips.
2. Painful, edematous, reddened lower 1. A
forearm. 2. B
3. Tented skin turgor over the entire left arm. 3. C
4. Nipple retraction and peau d’orange skin. 4. D
26. The client admitted with a diagnosis of
pneumonia complains of tenderness and pain
in the left calf, and the nurse assesses a positive
Homans’ sign. Which interventions should the
nurse implement? List in order of priority.
1. Notify the health-care provider.
2. Initiate an intravenous line.
3. Monitor the client’s PTT.
4. Administer a continuous heparin infusion.
5. Instruct the client not to get out of the bed.

04_Colgrove_Ch04.indd 143 02/06/16 7:57 PM


144 M ed -S urg S uccess

27. The nurse is caring for clients on a medical-surgical floor. Which information warrants immediate
investigation?

1.
Intake and Output Record
Oral Intravenous Urine Nasogastric Other (Specify)
Day One (in mL) (in mL) (in mL) Tube (in mL) (in mL)
0701–1500 3,280 1,575 2,400
1501–2300 1,250 2,500 1,000
2301–0700   450 2,000   870
TOTAL 4,980 6,075 4,270

2.
Intake and Output Record
Oral Intravenous Urine Nasogastric Other (Specify)
Day One (in mL) (in mL) (in mL) Tube (in mL) (in mL)
0701–1500   600 1,250 2,000 200
1501–2300   700   600 1,100   0
2301–0700   100   800 1,400   0
TOTAL 1,400 2,650 4,500 200

3.
Intake and Output Record
Oral Intravenous Urine Nasogastric Other (Specify)
Day One (in mL) (in mL) (in mL) Tube (in mL) (in mL)
0701–1500 1,000 1,000 1,550 JP drain 50
1501–2300   750 1,000   800 30
2301–0700      0 1,000 1,600 28
TOTAL 1,750 3,000 3,950 108

4.
Intake and Output Record
Oral Intravenous Urine Nasogastric Other (Specify)
Day One (in mL) (in mL) (in mL) Tube (in mL) (in mL)
0701–1500 1,260 100 1,300
1501–2300   637 50   485
2301–0700   120 100   600
TOTAL 2,017 250 2,385

04_Colgrove_Ch04.indd 144 02/06/16 7:57 PM


PERIPHERAL VASCULAR DISORDERS COMPREHENSIVE
EXAMINATION ANSWERS AND RATIONALES

1. 1. Drinking alcohol daily is not a cause of 4. 1. At four (4) hours postoperative, the client
­Buerger’s disease. would not be able to ambulate and would be in
2. Eating a high-fat diet is not a cause of the intensive care department. The client usu-
­Buerger’s disease. ally ambulates the first day postoperative.
3. Heavy smoking or chewing tobacco is 2. A neurovascular assessment is priority to
a causative or an aggravating factor for make sure the graft is perfusing the lower
Buerger’s disease. Cessation of tobacco use extremities.
may cause cessation of the disease process 3. Intravenous anticoagulant therapy would cause
in some clients. the client to bleed postoperatively. Prophylac-
4. Inhaling gasoline fumes is not a cause of tic anticoagulant therapy may be ordered to
Buerger’s disease; however, this can cause neu- help prevent deep vein thrombosis.
rological damage. 4. The client will be NPO and have a nasogastric
 ontent – Medical: Integrated Nursing Process –
C tube for at least 24 hours.
Assessment: Client Needs – Health Promotion and Mainte- Content – Surgical: Integrated Nursing Process –
nance: Cognitive Level – Analysis: Concept – Clotting. Implementation: Client Needs – Safe Effective Care
Environment, Management of Care: Cognitive Level –
2. 1. Subclavian steal syndrome occurs in the Application: Concept – Perfusion.
upper extremities from a subclavian ar-
tery occlusion or stenosis, which causes 5. 1. Thoracic outlet syndrome is a compression
arm tiredness, paresthesia, and exercise- of the subclavian artery at the thoracic out-
induced pain in the forearm when the arms let by an anatomical structure such as a rib
are elevated. This is also known as upper or muscle. Physical therapy, exercises, and
extremity arterial occlusive disease. avoiding aggravating positions are recom-
2. Shortness of breath could be a sign of a variety mended treatments.
of diseases, including congestive heart failure, 2. The nurse does not refer to a surgeon.
pneumonia, or chronic obstructive pulmonary 3. The occupational therapist helps with activi-
disease. ties of daily living and cognitive disorders that
3. Jugular vein distention could indicate conges- do not occur with this syndrome.
tive heart failure. 4. The social worker would not need to be con-
4. Dilated vessels above the nipple line are a sign sulted for a client with this syndrome.
of superior vena cava syndrome, an oncologic Content – Medical: Integrated Nursing Process –
emergency. Implementation: Client Needs – Safe Effective Care
Content – Medical: Integrated Nursing Process – Environment, Management of Care: Cognitive Level –
Assessment: Client Needs – Physiological Integrity, Application: Concept – Clotting.
Reduction of Risk Potential: Cognitive Level – Analysis: 6. 1. During the summer, a sweater should be avail-
Concept – Perfusion.
able when entering an air-conditioned room.
3. 1. The client would not experience problems sit- 2. Nicotine of any type causes vasoconstriction
ting; walking causes the pain. and may induce attacks.
2. The client does not have elimination 3. Raynaud’s phenomenon is a form of in-
dysfunction. termittent arteriolar vasoconstriction that
3. This would indicate an abdominal aortic results in coldness, pain, and pallor of fin-
­aneurysm but not an aortoiliac problem. gertips or toes; therefore, the client should
4. Aortoiliac disease is caused by atheroscle- keep warm to prevent vasoconstriction of
rosis of the aortoiliac arch, which causes the extremities.
pain in the lower back and buttocks and 4. Sunlight does not cause an exacerbation of this
impotence in men. disease.
Content – Medical: Integrated Nursing Process – Content – Medical: Integrated Nursing Process –
Assessment: Client Needs – Physiological Integrity, Planning: Client Needs – Physiological Integrity,
Reduction of Risk Potential: Cognitive Level – Physiological Adaptation: Cognitive Level – Synthesis:
Analysis: Concept – Perfusion. Concept – Perfusion.

145

04_Colgrove_Ch04.indd 145 02/06/16 7:57 PM


146 M ed -S urg S uccess

7. 1. This would be an appropriate intervention 4. Moisture between the toes increases fun-
but not before elevating the foot. gal growth, leading to skin breakdown.
2. Warm, moist packs cause necessary dilation, 5. The client with PVD has decreased sensation
but it is not the first intervention. in the feet and should not use a heating pad.
3. Elevating the foot above the heart will Content – Medical: Integrated Nursing Process –
decrease edema and thereby help decrease Planning: Client Needs – Physiological Integrity,
the pain. It is the easiest and first inter- Physiological Adaptation: Cognitive Level – Synthesis:
vention for the nurse to implement. Concept – Perfusion.
4. Educating the client is important, but it is not 11. 1. Stockings should be applied after the legs
the first intervention. The client will be have been elevated for a period of time,
hospitalized for seven (7) to 14 days. when the amount of blood in the leg veins
Content – Medical: Integrated Nursing Process – is at its lowest; therefore, the nurse should
Implementation: Client Needs – Physiological Integrity, intervene when the UAP is putting them
Physiological Adaptation: Cognitive Level –
on while the client is in the chair.
Application: Concept – Skin Integrity.
2. The top of the stocking should not be tight;
8. 1. Soaking the feet will not help prevent celluli- being able to insert two (2) fingers indicates it
tis; salt would be drying to the tissue, causing is not too tight.
cracks that may lead to cellulitis. 3. This is the correct way to apply stockings.
2. Thick white socks do not prevent cellulitis. 4. Warm toes mean the stockings are not too
3. Mosquito repellent would help to prevent tight and that there is adequate circulation.
encephalitis but not cellulitis. Content – Medical: Integrated Nursing Process –
4. The key to preventing cellulitis is identi- Evaluation: Client Needs – Safe Effective Care Environ-
fying the sites of bacterial entry. The most ment, Management of Care: Cognitive Level –
commonly overlooked areas are the cracks Synthesis: Concept – Clotting.
and fissures that occur between the toes.
12. 1. This BP indicates the medication is effec-
Content – Medical: Integrated Nursing Process – tive and in the desired range, and the nurse
Planning: Client Needs – Physiological Integrity, should not question administering the
Physiological Adaptation: Cognitive Level – Synthesis:
medication.
Concept – Skin Integrity.
2. This potassium level is low, but a beta blocker
9. 1. After sclerotherapy, clients are taught to does not affect the potassium level.
perform walking activities to maintain 3. A barky cough is associated with an ACE
blood flow in the leg and enhance dilution inhibitor, not a beta blocker.
of the sclerosing agent. 4. The beta blocker decreases sympathetic
2. The legs should be elevated to decrease stimulation to the beta cells of the heart.
edema. The legs should not be below the Therefore, the nurse should question ad-
level of the heart, which is the dependent ministering the medication if the apical
position. pulse is less than 60 beats per minute.
3. The compression bandages should be kept on Content – Medical: Integrated Nursing Process –
at all times for at least five (5) days or until Implementation: Client Needs – Physiological Integrity,
the health-care provider removes them for Pharmacological and Parenteral Therapies: Cognitive
the first time. Level – Application: Concept – Perfusion.
4. These exercises are recommended for clients 13. 1. This is an inappropriate intervention.
with arterial occlusive disease, not a venous 2. The INR is checked to determine if the med-
disease. ication is within therapeutic range, not prior
Content – Medical: Integrated Nursing Process – to administering the first dose.
Planning: Client Needs – Physiological Integrity, 3. The order does not need to be clarified.
Physiological Adaptation: Cognitive Level – Synthesis:
4. It will require several days for the
Concept – Clotting.
Coumadin to reach therapeutic levels; the
10. 1. Antimicrobial soap is harsh and can dry the client will continue receiving the heparin
skin; the client should use mild soap and drip until the therapeutic range can be
room-temperature water. attained.
2. Shoes must be comfortable to prevent Content – Medical: Integrated Nursing Process –
blisters or ulcerations of the feet. Implementation: Client Needs – Physiological Integrity,
3. The toenails should be cut straight across; Pharmacological and Parenteral Therapies: Cognitive
cutting in an arch increases the risk for Level – Application: Concept – Medications.
i­ngrown toenails.

04_Colgrove_Ch04.indd 146 02/06/16 7:57 PM


C hapter 4  P eripheral V ascular D isorders 147

14. 1. Fluids will help prevent dehydration and 18. 1. This is not a true statement. Atherosclerosis
­renal calculi, but this is not the priority nurs- causes strokes, heart attacks, and hyperten-
ing intervention. sion, which are all possibly lethal diseases.
2. Preventing deep vein thrombosis is the 2. This may occur, but it does not answer the
priority nursing intervention because the client’s question.
client is on strict bedrest; ROM exercises 3. This may occur, but it does not answer the
should be done every four (4) hours. client’s question.
3. Elevating the head of the bed will not help 4. A buildup of plaque in the arteries is oc-
prevent immobility complications. curring in the body when the client has
4. High-fiber diets will help prevent constipa- atherosclerosis. The nurse cannot tell the
tion, but this is not a priority. client the atherosclerosis will lead to his
Content – Medical: Integrated Nursing Process – or her death because there are many other
Implementation: Client Needs – Safe Effective Care things that could cause the client’s death.
Environment, Management of Care: Cognitive The statement does inform the client
Level – Synthesis: Concept – Clotting. what atherosclerosis is.
15. 1. This is not a priority unless the client is ac- Content – Medical: Integrated Nursing Process –
tively having a bowel movement and is per- Implementation: Client Needs – Physiological Integrity,
Physiological Adaptation: Cognitive Level –
forming the Valsalva maneuver.
Application: Concept – Perfusion.
2. This client needs to be addressed and orders
need to be clarified, but this is not a priority. 19. 1. Xerostomia, dry eyes, and dry vagina are
3. A bloody nose and a headache indicate symptoms of Sjögren’s syndrome.
the client is experiencing very high blood 2. A triad of arthritis, conjunctivitis, and urethri-
pressure and should be assessed first be- tis are symptoms of Reiter’s syndrome.
cause of a possible myocardial infarction 3. Clients with Marfan’s syndrome are very
or stroke. tall and have long bones in the hands and
4. A decreased pedal pulse is expected in this feet. They also have abnormalities of the
client. cardiovascular system, resulting in valvular
Content – Medical: Integrated Nursing Process – problems and aneurysms, which are the
Assessment: Client Needs – Safe Effective Care leading cause of death during the late 20s.
Environment, Management of Care: Cognitive Level – 4. Spinal deformities and malaise are symptoms
Synthesis: Concept – Clotting. of ankylosing spondylitis.
16. 1. The physical therapy department comes and Content – Medical: Integrated Nursing Process –
gets the client; if not, the UAP should escort Assessment: Client Needs – Physiological Integrity,
Reduction of Risk Potential: Cognitive Level – Analysis:
the client to physical therapy.
Concept – Perfusion.
2. The client’s pain is priority, and the nurse
should premedicate prior to treatment. 20. 1.5 tablets.
3. The physical therapy department will have The nurse must score and divide one tablet
the supplies for dressing the wound. and administer one and a half tablets to en-
4. This could be done, but it is not a priority for sure that the correct dose is administered.
the nurse. Content – Medical: Integrated Nursing Process –
Content – Medical: Integrated Nursing Process – Implementation: Client Needs – Physiological Integrity,
Implementation: Client Needs – Physiological Integrity, Pharmacological and Parenteral Therapies: ­Cognitive
Reduction of Risk Potential: Cognitive Level – Synthesis: Level – Application: Concept – Medication.
Concept – Perfusion.
21. 1. Fried foods have a lot of grease, a loaded
17. 1. Subcutaneous heparin will not achieve baked potato is high in fat, and coffee has
a therapeutic level because of the short caffeine. This is a high-calorie, high-sodium
half-life of the medication; therefore, the diet that is not recommended.
nurse should administer the medication. 2. Spaghetti is high in carbohydrates, meat is
2. There is no need to notify the HCP. high in fat, garlic bread is high in carbohy-
3. There is no need to monitor these drates, and iced tea has caffeine.
laboratory tests. 3. Ham is cured meat and is high in sodium, but
4. There is no need to notify the HCP. many clients do not realize this; cheese and
Content – Medical: Integrated Nursing Process – milk are high in sodium.
Implementation: Client Needs – Physiological 4. The client should be eating a low-fat,
Integrity, Pharmacological and Parenteral Therapies: low-cholesterol, low-sodium diet. This
Cognitive Level – Synthesis: Concept – Medication. meal reflects this diet.

04_Colgrove_Ch04.indd 147 02/06/16 7:57 PM


148 M ed -S urg S uccess

Content – Medical: Integrated Nursing Process – the nurse needs to determine if the blood is
Evaluation: Client Needs – Physiological Integrity, getting to the most distal area of the leg.
Basic Care and Comfort: Cognitive Level – Evaluation: 3. The popliteal pulse would be an appropriate
Concept – Perfusion. pulse to assess, but it is not the best because
22. 1. This is a therapeutic response and does not the nurse needs to determine if the blood is
answer the client’s question. getting to the most distal area of the leg.
2. Smoking aggravates Buerger’s disease. 4. The pedal pulse is the best pulse to assess
Aggravated or severe Buerger’s disease because it indicates if there is adequate
can lead to arterial occlusion caused by circulation to the most distal site of the
superficial thrombophlebitis resulting in extremity. The bypass extends from the
poor wound healing and poor circulation. midthigh to the midcalf area.
This can lead to the need for amputation. Content – Surgical: Integrated Nursing Process –
3. This is an appropriate referral, but it does not Assessment: Client Needs – Physiological Integrity,
answer the question. Reduction of Risk Potential: Cognitive Level – Analysis:
4. Buerger’s disease affects the arteries and veins Concept – Perfusion.
of the upper and lower extremities, but it 26. In order of priority: 5, 1, 2, 4, 3.
does not affect the coronary arteries. 5. The nurse should suspect a deep vein
Content – Medical: Integrated Nursing Process – thrombosis and should not allow the
Implementation: Client Needs – Physiological Integrity, ­client to get out of the bed.
Physiological Adaptation: Cognitive Level – Application: 1. This is a medical emergency, and the
Concept – Perfusion. HCP should be notified as soon as
23. 1. Subclavian steal syndrome affects the upper possible.
extremities only. 2. The client needs an intravenous line so
2. Subclavian steal syndrome occurs in the that IV heparin can be administered.
upper extremities from a subclavian artery 4. The treatment for DVT is to prevent fur-
occlusion or stenosis; therefore, any ther coagulation until the clot dissolves.
abnormal neurovascular assessment would 3. The client’s PTT is monitored when
warrant intervention. receiving heparin therapy.
3. This is a normal finding and would not Content – Medical: Integrated Nursing Process –
­warrant immediate intervention. Implementation: Client Needs – Safe Effective Care
4. This is a normal finding and would not war- Environment, Management of Care: Cognitive
rant immediate intervention. Level – Analysis: Concept – Clotting.
Content – Surgical: Integrated Nursing Process – 27. 1. This client has had 11,055 mL of intake
Assessment: Client Needs – Safe Effective Care Environ- compared to 4,270 mL of output; even
ment, Management of Care: Cognitive Level – with insensible loss, the nurse should as-
Synthesis: Concept – Perfusion. sess the client to determine if the client is
24. 1. Elephantiasis is obstruction of the lym- becoming fluid volume overloaded.
phatic vessels that causes chronic fibrosis, 2. This client has had 4,050 mL of intake com-
thickening of the subcutaneous tissue, and pared to 4,700 mL of output. This client has
hypertrophy of the skin; this condition a deficit but not one to be concerned with at
causes chronic edema of the extremity this time.
that recedes only slightly with elevation. 3. This client has had 4,750 mL of intake com-
2. This may be cellulitis. pared to 4,058 mL of output. The insensible
3. This may indicate dehydration of the tissue. loss would account for the difference and
4. Nipple retraction and peau d’orange skin make this client’s intake and output basically
would be an indication of late-stage breast even.
cancer. 4. This client has had 2,267 mL of intake
Content – Medical: Integrated Nursing Process – compared to 2,385 mL of output, basically
Assessment: Client Needs – Physiological Integrity, even.
Reduction of Risk Potential: Cognitive Level – Analysis: Content – Surgical: Integrated Nursing Process –
Concept – Clotting. Assessment: Client Needs – Physiological Integrity,
Physiological Adaptation: Cognitive Level – Synthesis:
25. 1. An absent radial pulse would not indicate a Concept – Perfusion.
complication of a lower extremity surgery.
2. The femoral pulse would be an appropriate
pulse to assess, but it is not the best because

04_Colgrove_Ch04.indd 148 02/06/16 7:57 PM


Hematological
Disorders
You can lead a man to the university but you can’t make him think.
—Finley Peter Dunne
5
Hematological disorders include a broad range of blood dyscrasias, which may be ­hereditary
or have an unknown etiology; some may be fatal, and some clients may live a normal life.
Among the most serious blood disorders—types of cancer—are leukemia and lymphoma.
Others include bleeding disorders, both hereditary (hemophilia and von Willebrand’s dis-
ease) and nonhereditary (thrombocytopenic purpura); clotting disorders (disseminated in-
travascular coagulation); and anemias characterized by abnormal red blood cells (sickle cell
anemia, thalassemia). The nurse must know the signs/symptoms of these disorders, what is
expected with the disorder, and when immediate intervention is necessary. The questions
and answers that follow cover these topics.

KEYWORDS     ABBREVIATIONS
Angiogram Absolute neutrophil count (ANC)
Apoptosis Activities of daily living (ADLs)
Autologous Acute respiratory distress syndrome (ARDS)
Epistaxis American Cancer Society (ACS)
Hemarthrosis Arterial blood gases (ABG)
Hematemesis Arterial partial pressure of oxygen (Pao2)
Hematoma As needed (prn)
Hematuria Capillary refill time (CRT)
Leukocytosis Chronic lymphocytic leukemia (CLL)
Leukopenia Complete blood count (CBC)
Melena Continuous passive motion (CPM)
Menorrhagia Disseminated intravascular coagulation (DIC)
Neutropenia End-stage renal disease (ESRD)
Pancytopenia Gastrointestinal (GI) tract
Placenta previa Health-care provider (HCP)
Purpura Hematocrit (Hct)
Hemoglobin (Hb)
Hemoglobin and hematocrit (H&H)
Human leukocyte antigen (HLA)
Idiopathic thrombocytopenic purpura (ITP)
Immediately (STAT)
Intramuscular (IM)
Intravenous (IV)
Intravenous push (IVP)

149

05_Colgrove_Ch05.indd 149 02/06/16 7:58 PM


150 M ed -S urg S uccess

Medication administration record (MAR)


Nonsteroidal anti-inflammatory drug (NSAID)
Normal saline (NS)
Over-the-counter (OTC)
Packed red blood cells (PRBCs)
Partial pressure of carbon dioxide (Pco2)
Partial thromboplastin time (PTT)
Post-anesthesia care unit (PACU)
Prothrombin time (PT)
Red blood cell (RBC)
Sickle cell anemia (SCA)
Unlicensed assistive personnel (UAP)
White blood cell (WBC)

PRACTICE QUESTIONS
Leukemia 4. The client diagnosed with leukemia is scheduled
for bone marrow transplantation. Which
1. The nurse is caring for clients on an oncology interventions should be implemented to prepare
unit. Which neutropenia precautions should be the client for this procedure? Select all that
implemented? apply.
1. Hold all venipuncture sites for at least 1. Administer high-dose chemotherapy.
five (5) minutes. 2. Teach the client about autologous transfusions.
2. Limit fresh fruits and flowers. 3. Have the family members’ HLA typed.
3. Place all clients in reverse isolation. 4. Monitor the complete blood cell count daily.
4. Have the clients use a soft-bristle toothbrush. 5. Provide central line care per protocol.
2. The nurse is assessing a client diagnosed with 5. The client is diagnosed with chronic lymphocytic
acute myeloid leukemia. Which assessment data leukemia (CLL) after routine laboratory tests
support this diagnosis? during a yearly physical. Which is the scientific
1. Fever and infections. rationale for the random nature of discovering
2. Nausea and vomiting. the illness?
3. Excessive energy and high platelet counts. 1. CLL is not serious, and clients die from other
4. Cervical lymph node enlargement and positive causes first.
acid-fast bacillus. 2. There are no symptoms with this form of
leukemia.
3. The client diagnosed with leukemia has central 3. This is a childhood illness and is self-limiting.
nervous system involvement. Which instruction 4. In early stages of CLL, the client may be
should the nurse teach? asymptomatic.
1. Sleep with the HOB elevated to prevent
increased intracranial pressure. 6. The client diagnosed with leukemia is
2. Take an analgesic medication for pain only being admitted for an induction course of
when the pain becomes severe. chemotherapy. Which laboratory values indicate a
3. Explain radiation therapy to the head may diagnosis of leukemia?
result in permanent hair loss. 1. A left shift in the white blood cell (WBC)
4. Discuss end-of-life decisions prior to cognitive count differential.
deterioration. 2. A large number of WBCs that decreases after
the administration of antibiotics.
3. An abnormally low hemoglobin (Hb) and
hematocrit (Hct) level.
4. Red blood cells (RBCs) that are larger than
normal.

05_Colgrove_Ch05.indd 150 02/06/16 7:58 PM


C hapter 5  H ematological D isorders 151

7. Which medication is contraindicated for Lymphoma


a client diagnosed with leukemia?
1. Bactrim, a sulfa antibiotic. 13. The client diagnosed with non-Hodgkin’s
2. Morphine, a narcotic analgesic. lymphoma is scheduled for a lymphangiogram.
3. Epogen, a biologic response modifier. Which information should the nurse teach?
4. Gleevec, a genetic blocking agent. 1. The scan will identify any malignancy in the
8. The laboratory results for a male client vascular system.
diagnosed with leukemia include RBC count 2. Radiopaque dye will be injected between
2.1 × 106/mm3, WBC count 150 × 103/mm3, the toes.
platelets 22 × 103/mm3, K+ 3.8 mEq/L, and 3. The test will be done similar to a cardiac
Na+ 139 mEq/L. Based on these results, which angiogram.
intervention should the nurse teach the client? 4. The test will be completed in about
1. Encourage the client to eat foods high in iron. five (5) minutes.
2. Instruct the client to use an electric razor 14. The client asks the nurse, “They say I have
when shaving. cancer. How can they tell if I have Hodgkin’s
3. Discuss the importance of limiting sodium in disease from a biopsy?” The nurse’s answer is
the diet. based on which scientific rationale?
4. Instruct the family to limit visits to once 1. Biopsies are nuclear medicine scans that can
a week. detect cancer.
9. The nurse writes a nursing problem of “altered 2. A biopsy is a laboratory test that detects
nutrition” for a client diagnosed with leukemia cancer cells.
who has received a treatment regimen of 3. It determines which kind of cancer the
chemotherapy and radiation. Which nursing client has.
intervention should be implemented? 4. The HCP takes a small piece out of the tumor
1. Administer an antidiarrheal medication prior and looks at the cells.
to meals. 15. The nurse is admitting a client with a diagnosis
2. Monitor the client’s serum albumin levels. of rule-out Hodgkin’s lymphoma. Which
3. Assess for signs and symptoms of infection. assessment data support this diagnosis?
4. Provide skin care to irradiated areas. 1. Night sweats and fever without “chills.”
10. The nurse and the licensed practical nurse (LPN) 2. Edematous lymph nodes in the groin.
are caring for clients on an oncology floor. Which 3. Malaise and complaints of an upset stomach.
client should not be assigned to the LPN? 4. Pain in the neck area after a fatty meal.
1. The client newly diagnosed with chronic 16. Which client is at the highest risk for
lymphocytic leukemia. developing a lymphoma?
2. The client who is four (4) hours 1. The client diagnosed with chronic lung
postprocedure bone marrow biopsy. disease who is taking a steroid.
3. The client who received two (2) units of 2. The client diagnosed with breast cancer who
(PRBCs) on the previous shift. has extensive lymph involvement.
4. The client who is receiving multiple 3. The client who received a kidney transplant
intravenous piggyback medications. several years ago.
11. The nurse is completing a care plan for a client 4. The client who has had ureteral stent
diagnosed with leukemia. Which independent placements for a neurogenic bladder.
problem should be addressed? 17. The female client recently diagnosed with
1. Infection. Hodgkin’s lymphoma asks the nurse about
2. Anemia. her prognosis. Which is the nurse’s best
3. Nutrition. response?
4. Grieving. 1. Survival for Hodgkin’s disease is relatively
12. The nurse is caring for a client diagnosed with good with standard therapy.
acute myeloid leukemia. Which assessment data 2. Survival depends on becoming involved in an
warrant immediate intervention? investigational therapy program.
1. T 99, P 102, R 22, and BP 132/68. 3. Survival is poor, with more than 50% of
2. Hyperplasia of the gums. clients dying within six (6) months.
3. Weakness and fatigue. 4. Survival is fine for primary Hodgkin’s, but
4. Pain in the left upper quadrant. secondary cancers occur within a year.

05_Colgrove_Ch05.indd 151 02/06/16 7:58 PM


152 M ed -S urg S uccess

18. The nurse writes the problem of “grieving” for a 23. The nurse and an unlicensed assistive personnel
client diagnosed with non-Hodgkin’s lymphoma. (UAP) are caring for clients in a bone marrow
Which collaborative intervention should be transplantation unit. Which nursing task should
included in the plan of care? the nurse delegate?
1. Encourage the client to talk about feelings of 1. Take the hourly vital signs on a client
loss. receiving blood transfusions.
2. Arrange for the family to plan a memorable 2. Monitor the infusion of antineoplastic
outing. medications.
3. Refer the client to the American Cancer 3. Transcribe the HCP’s orders onto the
Society’s Dialogue group. medication administration record (MAR).
4. Have the chaplain visit with the client. 4. Determine the client’s response to the
therapy.
19. Which test is considered diagnostic for
Hodgkin’s lymphoma? 24. The 33-year-old client diagnosed with Stage IV
1. A magnetic resonance image (MRI) of the Hodgkin’s lymphoma is at the five (5)-year
chest. remission mark. Which information should the
2. A computed tomography (CT) scan of the nurse teach the client?
cervical area. 1. Instruct the client to continue scheduled
3. An erythrocyte sedimentation rate (ESR). screenings for cancer.
4. A biopsy of the cervical lymph nodes. 2. Discuss the need for follow-up appointments
every five (5) years.
20. Which client should be assigned to the
3. Teach the client that the cancer risk is the
experienced medical-surgical nurse who is in
same as for the general population.
the first week of orientation to the oncology
4. Have the client talk with the family about
floor?
funeral arrangements.
1. The client diagnosed with non-Hodgkin’s
lymphoma who is having daily radiation
treatments. Anemia
2. The client diagnosed with Hodgkin’s
disease who is receiving combination 25. The nurse is admitting a 24-year-old African
chemotherapy. American female client with a diagnosis of rule-
3. The client diagnosed with leukemia who out anemia. The client has a history of gastric
has petechiae covering both anterior and bypass surgery for obesity four (4) years ago.
posterior body surfaces. Current assessment findings include height 5′5′′;
4. The client diagnosed with diffuse histolytic weight 75 kg; P 110, R 27, and BP 104/66; pale
lymphoma who is to receive two (2) units mucous membranes and dyspnea on exertion.
of packed red blood cells. Which type of anemia would the nurse suspect
the client has developed?
21. Which information about reproduction should
1. Vitamin B12 deficiency.
be taught to the 27-year-old female client
2. Folic acid deficiency.
diagnosed with Hodgkin’s disease?
3. Iron deficiency.
1. The client’s reproductive ability will be the
4. Sickle cell anemia.
same after treatment is completed.
2. The client should practice birth control for at 26. The client diagnosed with menorrhagia
least two (2) years following therapy. complains to the nurse of feeling listless and
3. All clients become sterile from the therapy tired all the time. Which scientific rationale
and should plan to adopt. would explain why these symptoms occur?
4. The therapy will temporarily interfere with 1. The pain associated with the menorrhagia
the client’s menstrual cycle. does not allow the client to rest.
2. The client’s symptoms are unrelated to the
22. Which clinical manifestation of Stage I non-
diagnosis of menorrhagia.
Hodgkin’s lymphoma would the nurse expect to
3. The client probably has been exposed to a
find when assessing the client?
virus that causes chronic fatigue.
1. Enlarged lymph tissue anywhere in the body.
4. Menorrhagia has caused the client to have
2. Tender left upper quadrant.
decreased levels of hemoglobin.
3. No symptom in this stage.
4. Elevated B-cell lymphocytes on the CBC.

05_Colgrove_Ch05.indd 152 02/06/16 7:58 PM


C hapter 5  H ematological D isorders 153

27. The nurse writes a diagnosis of altered tissue 2. The client diagnosed with pernicious anemia
perfusion for a client diagnosed with anemia. who is receiving vitamin B12 intramuscularly.
Which interventions should be included in the 3. The client diagnosed with aplastic anemia
plan of care? Select all that apply. who has developed pancytopenia.
1. Monitor the client’s hemoglobin and 4. The client diagnosed with renal disease
hematocrit. who has a deficiency of erythropoietin.
2. Move the client to a room near the nurse’s
33. The client diagnosed with anemia begins to
desk.
complain of dyspnea when ambulating in the
3. Limit the client’s dietary intake of green
hall. Which intervention should the nurse
vegetables.
implement first?
4. Assess the client for numbness and tingling.
1. Apply oxygen via nasal cannula.
5. Allow for rest periods during the day for
2. Get a wheelchair for the client.
the client.
3. Assess the client’s lung fields.
28. The client diagnosed with iron-deficiency 4. Assist the client when ambulating in the hall.
anemia is prescribed ferrous gluconate orally.
34. The nurse is transcribing the HCP’s order for
Which should the nurse teach the client?
an iron supplement on the MAR. At which time
1. Take Imodium, an antidiarrheal, over-the-
should the nurse schedule the daily dose?
counter (OTC) for diarrhea.
1. 0900.
2. Limit exercise for several weeks until a
2. 1000.
tolerance is achieved.
3. 1200.
3. The stools may be very dark, and this can
4. 1630.
mask blood.
4. Eat only red meats and organ meats for 35. The nurse is discharging a client diagnosed with
protein. anemia. Which discharge instruction should the
nurse teach?
29. The nurse and an unlicensed assistive personnel
1. Take the prescribed iron until it is completely
(UAP) are caring for clients on a medical unit.
gone.
Which task should the nurse delegate to the
2. Monitor pulse and blood pressure at a local
UAP?
pharmacy weekly.
1. Check on the bowel movements of a client
3. Have a complete blood count checked at the
diagnosed with melena.
HCP’s office.
2. Take the vital signs of a client who received
4. Perform isometric exercise three (3) times
blood the day before.
a week.
3. Evaluate the dietary intake of a client who has
been noncompliant with eating. 36. The nurse writes a client problem of “activity
4. Shave the client diagnosed with severe intolerance” for a client diagnosed with anemia.
hemolytic anemia. Which intervention should the nurse implement?
1. Pace activities according to tolerance.
30. The client is diagnosed with congestive heart
2. Provide supplements high in iron and
failure and anemia. The HCP ordered a
vitamins.
transfusion of two (2) units of packed red blood
3. Administer packed red blood cells.
cells. The unit has 250 mL of red blood cells
4. Monitor vital signs every four (4) hours.
plus 45 mL of additive. At what rate should the
nurse set the IV pump to infuse each unit of
packed red blood cells? ____________ Bleeding Disorders
31. The client is being admitted with folic acid
37. The charge nurse in the intensive care unit is
deficiency anemia. Which would be the most
making client assignments. Which client should
appropriate referral?
the charge nurse assign to the graduate nurse who
1. Alcoholics Anonymous.
has just finished the three (3)-month orientation?
2. Leukemia Society of America.
1. The client with an abdominal peritoneal
3. A hematologist.
resection who has a colostomy.
4. A social worker.
2. The client diagnosed with pneumonia who
32. The charge nurse is making assignments on a has acute respiratory distress syndrome.
medical floor. Which client should be assigned 3. The client with a head injury developing
to the most experienced nurse? disseminated intravascular coagulation.
1. The client diagnosed with iron-deficiency 4. The client admitted with a gunshot wound
anemia who is prescribed iron supplements. who has an H&H of 7 and 22.

05_Colgrove_Ch05.indd 153 02/06/16 7:58 PM


154 M ed -S urg S uccess

38. Which client would be most at risk for 3. The client has prolonged bleeding following
developing disseminated intravascular surgery.
coagulation (DIC)? 4. The female client developed abruptio
1. A 35-year-old pregnant client with placenta placentae.
previa.
45. The client with hemophilia A is experiencing
2. A 42-year-old client with a pulmonary
hemarthrosis. Which intervention should the
embolus.
nurse recommend to the client?
3. A 60-year-old client receiving hemodialysis
1. Alternate aspirin and acetaminophen to help
three (3) days a week.
with the pain.
4. A 78-year-old client diagnosed with
2. Apply cold packs for 24 to 48 hours to the
septicemia.
affected area.
39. The client admitted with full-thickness burns 3. Perform active range-of-motion exercise on
may be developing DIC. Which signs/symptoms the extremity.
would support the diagnosis of DIC? 4. Put the affected extremity in the dependent
1. Oozing blood from the IV catheter site. position.
2. Sudden onset of chest pain and frothy sputum.
46. Which sign would the nurse expect to assess
3. Foul-smelling, concentrated urine.
in the client diagnosed with idiopathic
4. A reddened, inflamed central line catheter
thrombocytopenic purpura (ITP)?
site.
1. Petechiae on the anterior chest, arms, and neck.
40. Which laboratory result would the nurse expect 2. Capillary refill of less than three (3) seconds.
in the client diagnosed with DIC? 3. An enlarged spleen.
1. A decreased prothrombin time. 4. Pulse oximeter reading of 95%.
2. A low fibrinogen level.
3. An increased platelet count. 47. The nurse is caring for the following clients.
4. An increased white blood cell count. Which client should the nurse assess first?
1. The client whose partial thromboplastin time
41. Which collaborative treatment would the nurse (PTT) is 38 seconds.
anticipate for the client diagnosed with DIC? 2. The client whose hemoglobin is 14 g/dL and
1. Administer oral anticoagulants. hematocrit is 45%.
2. Prepare for plasmapheresis. 3. The client whose platelet count is 75,000 per
3. Administer frozen plasma. cubic millimeter of blood.
4. Calculate the intake and output. 4. The client whose red blood cell count is
42. The unlicensed assistive personnel (UAP) asks 4.8 × 106/mm3.
the primary nurse, “How does someone get 48. Which nursing interventions should the nurse
hemophilia A?” Which statement would be the implement when caring for a client diagnosed
primary nurse’s best response? with hemophilia A? Select all that apply.
1. “It is an inherited X-linked recessive disorder.” 1. Instruct the client to use a razor blade to shave.
2. “There is a deficiency of the clotting 2. Avoid administering enemas to the client.
factor VIII.” 3. Encourage participation in noncontact sports.
3. “The person is born with hemophilia A.” 4. Teach the client how to apply direct pressure
4. “The mother carries the gene and gives it to if bleeding occurs.
the son.” 5. Explain the importance of not flossing the
43. Which sign/symptom should the nurse gums.
expect to assess in the client diagnosed with
hemophilia A? Blood Transfusions
1. Epistaxis.
2. Petechiae. 49. The client has a hematocrit of 22.3% and a
3. Subcutaneous emphysema. hemoglobin of 7.7 g/dL. The HCP has ordered
4. Intermittent claudication. two (2) units of packed red blood cells to be
44. Which situation might cause the nurse to transfused. Which interventions should the
think that the client has von Willebrand’s (vW) nurse implement? Select all that apply.
disease? 1. Obtain a signed consent.
1. The client has had unexplained episodes of 2. Initiate a 22-gauge IV.
hematemesis. 3. Assess the client’s lungs.
2. The client has microscopic blood in the urine. 4. Check for allergies.
5. Hang a keep-open IV of D5W.

05_Colgrove_Ch05.indd 154 02/06/16 7:58 PM


C hapter 5  H ematological D isorders 155

50. The client is admitted to the emergency 3. Replace the unit with a continuous passive
department after a motor-vehicle accident. The motion (CPM) unit and start it on low.
nurse notes profuse bleeding from a right-sided 4. Have another nurse verify the unit number
abdominal injury. Which intervention should the prior to reinfusing the blood.
nurse implement first?
55. Which statement is the scientific rationale
1. Type and crossmatch for red blood cells
for infusing a unit of blood in less than
immediately (STAT).
four (4) hours?
2. Initiate an IV with an 18-gauge needle and
1. The blood will coagulate if left out of
hang normal saline.
the refrigerator for greater than four
3. Have the client sign a consent for an
(4) hours.
exploratory laparotomy.
2. The blood has the potential for bacterial
4. Notify the significant other of the client’s
growth if allowed to infuse longer.
admission.
3. The blood components begin to break down
51. The nurse is working in a blood bank facility after four (4) hours.
procuring units of blood from donors. Which 4. The blood will not be affected; this is a
client would not be a candidate to donate blood? laboratory procedure.
1. The client who had wisdom teeth removed a
56. The HCP orders two (2) units of blood to
week ago.
be administered over eight (8) hours each
2. The nursing student who received a measles
for a client diagnosed with heart failure.
immunization two (2) months ago.
Which intervention(s) should the nurse
3. The mother with a six (6)-week-old
implement?
newborn.
1. Call the HCP to question the order
4. The client who developed an allergy to aspirin
because blood must infuse within
in childhood.
four (4) hours.
52. The client with O+ blood is in need of an 2. Retrieve the blood from the laboratory
emergency transfusion but the laboratory and run each unit at an eight (8)-hour rate.
does not have any O+ blood available. Which 3. Notify the laboratory to split each unit
potential unit of blood could be given to the into half-units and infuse each half for
client? four (4) hours.
1. The O– unit. 4. Infuse each unit for four (4) hours, the
2. The A+ unit. maximum rate for a unit of blood.
3. The B+ unit.
57. The client receiving a unit of PRBCs begins to
4. Any Rh+ unit.
chill and develops hives. Which action should be
53. The client is scheduled to have a total hip the nurse’s first response?
replacement in two (2) months and has chosen 1. Notify the laboratory and health-care
to prepare for autologous transfusions. Which provider.
medication would the nurse administer to 2. Administer the histamine-1 blocker,
prepare the client? Benadryl, IV.
1. Prednisone, a glucocorticoid. 3. Assess the client for further complications.
2. Zithromax, an antibiotic. 4. Stop the transfusion and change the tubing at
3. Ativan, a tranquilizer. the hub.
4. Epogen, a biologic response modifier.
58. The nurse and an unlicensed assistive personnel
54. The client undergoing knee replacement (UAP) are caring for clients on an oncology
surgery has a “cell saver” apparatus attached to floor. Which nursing task would be delegated to
the knee when he arrives in the post-anesthesia the UAP?
care unit (PACU). Which intervention should 1. Assess the urine output on a client who has
the nurse implement to care for this drainage had a blood transfusion reaction.
system? 2. Take the first 15 minutes of vital signs on a
1. Infuse the drainage into the client when a client receiving a unit of PRBCs.
prescribed amount fills the chamber. 3. Auscultate the lung sounds of a client prior to
2. Attach an hourly drainage collection bag to a transfusion.
the unit and discard the drainage. 4. Assist a client who received 10 units of
platelets in brushing the teeth.

05_Colgrove_Ch05.indd 155 02/06/16 7:58 PM


156 M ed -S urg S uccess

59. The nurse is caring for clients on a medical 63. The client diagnosed with sickle cell
floor. After the shift report, which client should anemia comes to the emergency department
be assessed first? complaining of joint pain throughout the
1. The client who is two-thirds of the way body. The oral temperature is 102.4°F and
through a blood transfusion and has had no the pulse oximeter reading is 91%. Which
complaints of dyspnea or hives. action should the emergency department nurse
2. The client diagnosed with leukemia who has implement first?
a hematocrit of 18% and petechiae covering 1. Request arterial blood gases STAT.
the body. 2. Administer oxygen via nasal cannula.
3. The client with peptic ulcer disease who 3. Start an IV with an 18-gauge angiocath.
called over the intercom to say that he is 4. Prepare to administer analgesics as ordered.
vomiting blood.
64. The client diagnosed with sickle cell anemia is
4. The client diagnosed with Crohn’s disease
experiencing a vaso-occlusive sickle cell crisis
who is complaining of perineal discomfort.
secondary to an infection. Which medical
60. The client received two (2) units of packed treatment should the nurse anticipate the HCP
red blood cells of 250 mL with 63 mL of ordering for the client?
preservative each during the shift. There was 1. Administer meperidine (Demerol)
240 mL of saline remaining in the 500-mL bag intravenously.
when the nurse discarded the blood tubing. How 2. Admit the client to a private room and keep in
many milliliters of fluid should be documented reverse isolation.
on the intake and output record? ____________ 3. Infuse D5W 0.33% NS at 150 mL/hr via
pump.
4. Insert a 22-French Foley catheter with a
Sickle Cell Anemia urimeter.

61. The student nurse asks the nurse, “What is 65. The nurse is assessing an African American
sickle cell anemia?” Which statement by the client diagnosed with sickle cell crisis. Which
nurse would be the best answer to the student’s assessment datum is most pertinent when
question? assessing for cyanosis in clients with
1. “There is some written material at the desk dark skin?
that will explain the disease.” 1. Assess the client’s oral mucosa.
2. “It is a congenital disease of the blood in 2. Assess the client’s metatarsals.
which the blood does not clot.” 3. Assess the client’s capillary refill time.
3. “The client has decreased synovial fluid that 4. Assess the sclera of the client’s eyes.
causes joint pain.” 66. The client is diagnosed with sickle cell
4. “The blood becomes thick when the client is crisis. The nurse is calculating the client’s intake
deprived of oxygen.” and output (I&O) for the shift. The client
62. The client’s nephew has just been diagnosed had 20 ounces of water, eight (8) ounces of
with sickle cell anemia (SCA). The client apple juice, three (3) cartons of milk with four
asks the nurse, “How did my nephew get this (4) ounces each, 1,800 mL of IV fluids
disease?” Which statement would be the best for the last 12 hours, and a urinary output of
response by the nurse? 1,200. What is the client’s total intake for this
1. “Sickle cell anemia is an inherited autosomal shift? _____________
recessive disease.” 67. The nurse is caring for the female client
2. “He was born with it and both his parents recovering from a sickle cell crisis. The client
were carriers of the disease.” tells the nurse her family is planning a trip this
3. “At this time, the cause of sickle cell anemia is summer to Yellowstone National Park. Which
unknown.” response would be best for the nurse?
4. “Your sister was exposed to a virus while she 1. “That sounds like a wonderful trip to take this
was pregnant.” summer.”
2. “Have you talked to your doctor about taking
the trip?”
3. “You really should not take a trip to areas with
high altitudes.”
4. “Why do you want to go to Yellowstone
National Park?”

05_Colgrove_Ch05.indd 156 02/06/16 7:58 PM


C hapter 5  H ematological D isorders 157

68. Which is a potential complication that is it?” Which statement is the best response
occurs specifically to a male client diagnosed by the nurse?
with sickle cell anemia during a sickle cell 1. “It is a foundation that deals primarily with
crisis? research for a cure for SCA.”
1. Chest syndrome. 2. “It provides information on the disease and on
2. Compartment syndrome. support groups in this area.”
3. Priapism. 3. “I recommend joining any organization that
4. Hypertensive crisis. will help deal with your disease.”
4. “The foundation arranges for families that
69. The nurse is completing discharge teaching
have children with sickle cell to meet.”
for the client diagnosed with a sickle cell crisis.
The nurse recommends the client getting the 71. Which sign/symptom will the nurse expect
flu and pneumonia vaccines. The client asks, to assess in the client diagnosed with a
“Why should I take those shots? I hate shots.” vaso-occlusive sickle cell crisis?
Which statement by the nurse is the best 1. Lordosis.
response? 2. Epistaxis.
1. “These vaccines promote health in clients 3. Hematuria.
with chronic illnesses.” 4. Petechiae.
2. “You are susceptible to infections. These shots
72. The male client with sickle cell anemia comes to
may help prevent a crisis.”
the emergency department with a temperature
3. “The vaccines will help your blood from
of 101.4°F and tells the nurse that he is having
sickling secondary to viruses.”
a sickle cell crisis. Which diagnostic test should
4. “The doctor wanted to make sure that I
the nurse anticipate the emergency department
discussed the vaccines with you.”
doctor ordering for the client?
70. The client diagnosed with sickle cell anemia 1. Spinal tap.
asks the nurse, “Should I join the Sickle Cell 2. Hemoglobin electrophoresis.
Foundation? I received some information from 3. Sickle-turbidity test (Sickledex).
the Sickle Cell Foundation. What kind of group 4. Blood cultures.

CONCEPTS
The concepts covered in this chapter focus on hematological regulation problems such as leukemia, lymphoma,
and clotting issues such as sickle cell anemia and bleeding. Interrelated concepts of the nursing process and critical
thinking are covered throughout the questions. The test taker will be challenged to think critically, particularly in
the questions about priority interventions or “first” choices.
73. The nurse has identified the concept of cellular 75. The nurse is caring for a client diagnosed with
deviation for a client diagnosed with chronic sickle cell disease. Which should the nurse
myelogenous leukemia. Which intervention should include in the client’s plan of care?
the nurse implement? Select all that apply. 1. Teach the client to limit fluids.
1. Screen visitors for infection before allowing 2. Discuss interventions to maintain hydration.
them to enter the room. 3. Measure the client’s calf for swelling.
2. Assess the client’s vital signs every four (4) 4. Have the client take narcotic pain medication
hours. every four (4) hours.
3. Do not allow fresh fruits and vegetables on
76. The nurse identified clotting as a concept related
diet trays.
to sickle cell disease. Which intervention should
4. Monitor the client’s white blood cell count.
the nurse implement?
5. Place the client on droplet isolation.
1. Assess for cerebrovascular symptoms.
6. Check the client’s bone marrow results daily.
2. Keep the head of the bed elevated.
74. The client is diagnosed with non-Hodgkin’s 3. Order a 2,000-mg sodium diet.
lymphoma. Which nursing concept should the 4. Apply antiembolism stockings.
nurse identify as priority?
1. Immunity.
2. Grieving.
3. Perfusion.
4. Clotting.

05_Colgrove_Ch05.indd 157 02/06/16 7:58 PM


158 M ed -S urg S uccess

77. The client is diagnosed with Hodgkin’s disease. 80. The nurse is assessing a client diagnosed with
Which data are diagnostic for Hodgkin’s vaso-occlusive crisis. Which indicates the client
disease? is not meeting an appropriate stage of growth
1. Night sweats and low-grade fever. and development according to Erikson?
2. Cavitation noted on the chest x-ray. 1. The 32-year-old client does not have a
3. Reed-Sternberg cells found on biopsy. significant other and is on disability.
4. Weight loss and palpable inguinal lymph 2. The 28-year-old client is actively involved in
nodes. the care of a six (6)-year-old child.
3. The 40-year-old client has a full-time job and
78. Which interrelated psychological concept
cares for an aged parent.
is priority for the nurse caring for a client
4. The 19-year-old client is a full-time college
diagnosed with leukemia?
student and has many friends.
1. Comfort.
2. Stress. 81. The nurse identifies a concept of hematologic
3. Grieving. regulation for a client diagnosed with leukemia.
4. Coping. Which clinical manifestations support the
concept?
79. The client diagnosed with sickle cell disease
1. The client has petechiae on the trunk and
is experiencing a vaso-occlusive crisis. Which
extremities.
priority interventions should the nurse
2. The client complains of pain and swelling
implement?
in the joints.
1. Maintain IV fluids and administer pain
3. The client has an Hbg of 9.7 and Hct of 32%.
medication prn.
4. The client complains of a headache and
2. Encourage frequent ambulation in the
slurred speech.
hallways.
3. Administer oxygen via nasal cannula at 82. Which concepts could the nurse identify for a
10 LPM. client diagnosed with lymphoma? Select all that
4. Monitor the client’s red blood cell count every apply.
four (4) hours. 1. Coping.
2. Hematologic regulation.
3. Tissue perfusion.
4. Clotting.
5. Clinical judgment.

05_Colgrove_Ch05.indd 158 02/06/16 7:58 PM


PRACTICE QUESTIONS ANSWERS
AND RATIONALES

Leukemia normally drains energy reserves; it does not


increase them.
1. 1. This would be done for thrombocytopenia Content – Medical: Integrated Nursing Process –
(low platelets), not neutropenia (low white Assessment: Client Needs – Physiological Integrity,
blood cells). Reduction of Risk Potential: Cognitive Level – Analysis:
2. Fresh fruits and flowers may carry bacte- Concept – Hematologic Regulation.
ria or insects on the skin of the fruit or 3. 1. Sleeping with the head of the bed elevated
dirt on the flowers and leaves, so they are might relieve some intracranial pressure, but it
restricted around clients with low white will not prevent increased intracranial pressure
blood cell counts. from occurring.
3. Clients with severe neutropenia may be placed 2. Analgesic medications for clients with can-
in reverse isolation, but not all clients with cer are given on a scheduled basis with a
neutropenia will be placed in reverse isola- fast-acting analgesic administered prn for
tion. Clients are at a greater risk for infecting breakthrough pain.
themselves from endogenous fungi and bacte- 3. Radiation therapy to the head and scalp
ria than from being exposed to noninfectious area is the treatment of choice for central
individuals. nervous system involvement of any cancer.
4. This is an intervention for thrombocytopenia. Radiation therapy has longer lasting side
TEST-TAKING HINT: The test taker must match effects than chemotherapy. If the radiation
the problem with the answer option. Options therapy destroys the hair follicles, the hair
“1” and “4” would probably be implemented will not grow back.
for the client with a bleeding disorder. Option 4. Cognitive deterioration does not usually occur.
“3” has the word “all” in it, which would make TEST-TAKING HINT: The test taker must be
the test taker not select this option. aware of the treatments used for the disease
Content – Medical: Integrated Nursing Process – processes to answer this question but might
Implementation: Client Needs – Safe Effective Care eliminate option “2” because it violates basic
Environment, Management of Care: Cognitive Level – principles of pain management.
Application: Concept – Hematologic Regulation.
Content – Medical: Integrated Nursing Process –
2. 1. Fever and infection are hallmark symptoms Planning: Client Needs – Physiological Integrity,
of leukemia. They occur because the bone Physiological Adaptation: Cognitive Level – Synthesis:
marrow is unable to produce white blood Concept – Hematologic Regulation.
cells of the number and 4. 1. All of the bone marrow cells must be de-
maturity needed to fight infection. stroyed prior to “implanting” the healthy
2. Nausea and vomiting are symptoms related to bone marrow. High-dose chemotherapy
the treatment of cancer but not to the and full-body irradiation therapy are used
diagnosis of leukemia. to accomplish this.
3. The clients are frequently fatigued and have 2. Autologous transfusions are infusions from
low platelet counts. The platelet count is low the client himself or herself. This client has a
as a result of the inability of the bone marrow cancer involving blood tissue. To reinfuse the
to produce the needed cells. In some forms client’s own tissues would be to purposefully
of leukemia, the bone marrow is not produc- give the client cancer cells.
ing cells at all, and in others, the bone mar- 3. The best bone marrow donor comes from
row is stuck producing tens of thousands of an identical twin; next best comes from a
immature cells. sibling who matches. The most complica-
4. Cervical lymph node enlargement is tions occur from a matched unrelated do-
associated with Hodgkin’s lymphoma, and nor (MUD). The client’s body recognizes
positive acid-fast bacillus is diagnostic for the marrow as foreign and tries to reject
tuberculosis. it, resulting in graft-versus-host disease
TEST-TAKING HINT: Option “3” could be elimi- (GVHD).
nated because of the excessive energy. Illness
159

05_Colgrove_Ch05.indd 159 02/06/16 7:58 PM


160 M ed -S urg S uccess

4. The CBC must be monitored daily 4. Red blood cells larger than normal occur in
to assess for infections, anemia, and macrocytic anemias (vitamin B12 and folic
thrombocytopenia. acid deficiency). They are not characteristic of
5. Clients will have at least one multiple- leukemia.
line central venous access. These clients TEST-TAKING HINT: The test taker should rec-
are ­seriously ill and require multiple ognize elevated WBCs resolve with antibiot-
­transfusions and antibiotics. ics as an infection. Option “3” is not specific
TEST-TAKING HINT: If the test taker knows the enough to be the correct answer.
definition of “autologous,” then option “2” Content – Medical: Integrated Nursing Process –
could be eliminated. Assessment: Client Needs – Physiological Integrity,
Content – Surgical: Integrated Nursing Process – Reduction of Risk Potential: Cognitive Level – Analysis:
Implementation: Client Needs – Physiological Integrity, Concept – Hematologic Regulation.
Reduction of Risk Potential: Cognitive Level – Application:
7. 1. Because of the ineffective or nonexistent
Concept – Hematologic Regulation.
WBCs (segmented neutrophils) characteristic
5. 1. All types of leukemia are serious and can cause of leukemia, the body cannot fight infections,
death. The chronic types of leukemia are and antibiotics are given to treat infections.
more insidious in the onset of symptoms and 2. Leukemic infiltrations into the organs or the
can have a slower progression of the disease. central nervous system cause pain. Morphine
Chronic types of leukemia are more common is the drug of choice for most clients with
in the adult population. cancer.
2. The symptoms may have a slower onset, but 3. Epogen is a biologic response modifier that
anemia causing fatigue and weakness and stimulates the bone marrow to produce red
thrombocytopenia causing bleeding can be blood cells. The bone marrow is the area
present (usually in the later stages of the dis- of malignancy in leukemia. Stimulating the
ease). Organ enlargement from infiltration bone marrow would be generally ineffective
may be present. Secondary symptoms of fe- for the desired results and would have the
ver, night sweats, and weight loss may also be potential to stimulate malignant growth.
present. 4. Gleevec is a drug that specifically works in
3. This disease is usually found in adults. leukemic cells to block the expression of the
4. In this form of leukemia, the cells seem to BCR-ABL protein, preventing the cells from
escape apoptosis (programmed cell death), growing and dividing.
which results in many thousands of mature TEST-TAKING HINT: If the test taker were not
cells clogging the body. Because the cells familiar with the drug mentioned in
are mature, the client may be asymptomatic o­ption “4,” then this option would not be a
in the early stages. good choice. Options “1” and “2” are common
TEST-TAKING HINT: The test taker can elimi- drugs and should not be chosen as the answer
nate option “1” based on the words “not seri- unless the test taker knows for sure they are
ous”; common sense lets the test taker know contraindicated.
this is not true. Content – Medical: Integrated Nursing Process –
Content – Medical: Integrated Nursing Process – Implementation: Client Needs – Physiological Integrity,
Diagnosis: Client Needs – Physiological Integrity, Reduction Pharmacological and Parenteral Therapies: Cognitive Level –
of Risk Potential: Cognitive Level – Analysis: Concept – Application: Concept – Medication.
Hematologic Regulation.
8. 1. The anemia that occurs in leukemia is not
6. 1. A left shift indicates immature white blood ­related to iron deficiency and eating foods
cells are being produced and high in iron will not help.
released into the circulating blood 2. The platelet count of 22 3 103/mm3
volume. This should be investigated for the i­ndicates a platelet count of 22,000. The
malignant process of leukemia. definition of thrombocytopenia is a count
2. Leukocytosis (elevated WBCs) is normal in the less than 100,000. This client is at risk for
presence of an infection, but it should decrease bleeding. Bleeding precautions include
as the infection clears. decreasing the risk by using soft-bristle
3. A low hemoglobin and hematocrit level indi- toothbrushes and electric razors and hold-
cates anemia and can be caused by a number of ing all venipuncture sites for a minimum of
factors. Anemia does occur in leukemia, but it five (5) minutes.
is not diagnostic for leukemia.

05_Colgrove_Ch05.indd 160 02/06/16 7:58 PM


C hapter 5  H ematological D isorders 161

3. The sodium level is within normal limits. The 11. 1. Treating infections, which require HCP
client is encouraged to eat whatever he or she orders for cultures and antibiotics, is a
wants to eat unless some other disease process collaborative problem.
limits food choices. 2. The treatment of anemia is a collaborative
4. The client is at risk for infection, but un- problem.
less the family or significant others are ill, 3. The provision of adequate nutrition requires
they should be encouraged to visit whenever collaboration among the nurse, HCP, and
possible. dietitian.
TEST-TAKING HINT: The test taker could 4. Grieving is an independent problem, and
eliminate option “3” based on a normal the nurse can assess and treat this prob-
­laboratory value. The RBC, WBC, and lem with or without collaboration.
­platelet values are all not in normal range. TEST-TAKING HINT: The stem of the question
The correct answer option must address asks for an independent intervention. If the
one of these values. test taker understands the problem and the
Content – Medical: Integrated Nursing Process – treatment needs, options “1,” “2,” and “3”
Planning: Client Needs – Physiological Integrity, Reduction can be eliminated.
of Risk Potential: Cognitive Level – Synthesis: Concept – Content – Medical: Integrated Nursing Process –
Hematologic Regulation. Diagnosis: Client Needs – Psychosocial Integrity: Cognitive
Level – Analysis: Concept – Nursing Roles.
9. 1. The nurse should administer an antiemetic
prior to meals, not an antidiarrheal medication. 12. 1. These vital signs are not alarming. The vital
2. Serum albumin is a measure of the pro- signs are slightly elevated and indicate moni-
tein content in the blood that is derived toring at intervals, but they do not indicate an
from the foods eaten; albumin monitors immediate need.
nutritional status. 2. Hyperplasia of the gums is a symptom of my-
3. Assessment of the nutritional status is indi- eloid leukemia, but it is not an emergency.
cated for this problem, not assessment of the 3. Weakness and fatigue are symptoms of the
signs and symptoms of infections. disease and are expected.
4. This addresses an altered skin integrity 4. Pain is expected, but it is a priority, and pain
problem. control measures should be implemented.
TEST-TAKING HINT: The stem of the question TEST-TAKING HINT: If all the answer ­options
asks for interventions for “altered nutrition.” contain expected events, the test taker
Assessment is the first step of the nursing pro- must decide which is priority—and pain is a
cess, but option “3” is not assessing nutrition. p
­ riority need.
Content – Medical: Integrated Nursing Process – Content – Medical: Integrated Nursing Process –
Implementation: Client Needs – Physiological Integrity, Assessment: Client Needs – Physiological Integrity,
Basic Care and Comfort: Cognitive Level – Application: Reduction of Risk Potential: Cognitive Level – Analysis:
Concept – Hematologic Regulation. Concept – Hematologic Regulation.
10. 1. The newly diagnosed client will need to
be taught about the disease and about Lymphoma
treatment options. The registered nurse
cannot delegate teaching to an LPN. 13. 1. The scan detects abnormalities in the lym-
2. This client is postprocedure and could be phatic system, not the vascular system.
cared for by the LPN. 2. Dye is injected between the toes of both
3. This client has already received the blood feet and then scans are performed in a few
products; this client requires routine moni- hours, at 24 hours, and then possibly once
toring, which the LPN could perform. a day for several days.
4. The LPN can administer antibiotic 3. Cardiac angiograms are performed through
medications. the femoral or brachial arteries and are com-
TEST-TAKING HINT: The nurse cannot assign pleted in one session.
assessment, teaching, or evaluation. The cli- 4. The test takes 30 minutes to one (1) hour and
ents in options “2,” “3,” and “4” are stable or then is repeated at intervals.
have expected situations. TEST-TAKING HINT: The test taker must be
Content – Nursing Management: Integrated Nursing aware of diagnostic tests used to diagnose
Process – Planning: Client Needs – Safe Effective Care specific diseases. Options “1” and “3” could
Environment, Management of Care: Cognitive Level – be eliminated because of the words “vascu-
Synthesis: Concept – Nursing Roles.
lar” and “cardiac”; these words pertain to

05_Colgrove_Ch05.indd 161 02/06/16 7:58 PM


162 M ed -S urg S uccess

the cardiovascular system, not the ­lymphatic Reduction of Risk Potential: Cognitive Level – Analysis:
system. Concept – Hematologic Regulation.
Content – Medical: Integrated Nursing Process – 16. 1. Long-term steroid use suppresses the
Planning: Client Needs – Physiological Integrity, Reduction immune system and has many side effects, but
of Risk Potential: Cognitive Level – Synthesis: Concept – it is not the highest risk for the development
Nursing Roles.
of lymphoma.
14. 1. Biopsies are surgical procedures requiring 2. This client would be considered to be
needle aspiration or excision of the area; they in late-stage breast cancer. Cancers are
are not nuclear medicine scans. described by the original cancerous tissue.
2. The biopsy specimen is sent to the pathology This client has breast cancer that has metas-
laboratory for the pathologist to determine tasized to the lymph system.
the type of cell. “Laboratory test” refers to 3. Clients who have received a transplant
tests of body fluids performed by a laboratory must take immunosuppressive medica-
technician. tions to prevent rejection of the organ.
3. A biopsy is used to determine if the client has This immunosuppression blocks the
cancer and, if so, what kind. However, this re- immune system from protecting the body
sponse does not answer the client’s question. against cancers and other diseases. There
4. A biopsy is the removal of cells from a is a high incidence of lymphoma among
mass and examination of the tissue under transplant recipients.
a microscope to determine if the cells 4. A neurogenic bladder is a benign disease;
are cancerous. Reed-Sternberg cells are stent placement would not put a client at risk
diagnostic for Hodgkin’s disease. If these for cancer.
cells are not found in the biopsy, the HCP TEST-TAKING HINT: To answer this question,
can rebiopsy to make sure the specimen the test taker must be aware of the function
provided the needed sample or, depending of the immune system in the body and of the
on involvement of the tissue, diagnose a treatments of the disease processes.
non-Hodgkin’s lymphoma. Content – Medical: Integrated Nursing Process –
TEST-TAKING HINT: Option “1” can be elimi- Assessment: Client Needs – Health Promotion and
nated if the test taker knows what the word Maintenance: Cognitive Level – Analysis: Concept –
“biopsy” means. Option “3” does not answer Hematologic Regulation.
the question and can be eliminated for this 17. 1. Up to 90% of clients respond well to
reason. standard treatment with chemotherapy
Content – Medical: Integrated Nursing Process – and radiation therapy, and those who
Diagnosis: Client Needs – Physiological Integrity, Reduction relapse usually respond to a change of
of Risk Potential: Cognitive Level – Knowledge: Concept – chemotherapy medications. Survival
Hematologic Regulation.
depends on the individual client and the
15. 1. Clients with Hodgkin’s disease experience stage of disease at diagnosis.
drenching diaphoresis, especially at night; 2. Investigational therapy regimens would not
fever without chills; and unintentional be recommended for clients initially diag-
weight loss. Early stage disease is indi- nosed with Hodgkin’s disease because of the
cated by a painless enlargement of a lymph expected prognosis with standard therapy.
node on one side of the neck (cervical 3. Clients usually achieve a significantly
area). Pruritus is also a common symptom. longer survival rate than six (6) months.
2. Lymph node enlargement with Hodgkin’s Many clients survive to develop long-term
disease is in the neck area. secondary complications.
3. Malaise and stomach complaints are not 4. Secondary cancers can occur as long as
a­ssociated with Hodgkin’s disease. 20 years after a remission of the Hodgkin’s
4. Pain in the neck area at the site of the cancer disease has occurred.
occurs in some clients after the ingestion of TEST-TAKING HINT: The test taker must have
alcohol. The cause for this is unknown. a basic knowledge of the disease process but
TEST-TAKING HINT: The test taker must could rule out option “2” based on the word
­notice the descriptive words, such as “groin” “investigational.”
and “fatty,” to decide if these options could Content – Medical: Integrated Nursing Process –
be correct. Implementation: Client Needs – Physiological Integrity,
Content – Medical: Integrated Nursing Process – Physiological Adaptation: Cognitive Level – Application:
Assessment: Client Needs – Physiological Integrity, Concept – Hematologic Regulation.

05_Colgrove_Ch05.indd 162 02/06/16 7:58 PM


C hapter 5  H ematological D isorders 163

18. 1. Encouraging the client to talk about his 20. 1. This client is receiving treatments that can
or her feelings is an independent nursing have life-threatening side effects; the nurse is
intervention. not experienced with this type of client.
2. Discussing activities that will make pleas- 2. Chemotherapy is administered only by nurses
ant memories and planning a family outing who have received training in chemotherapy
improve the client’s quality of life and assist medications and their effects on the body and
the family in the grieving process after the are aware of necessary safety precautions; this
client dies, but this is an independent nursing nurse is in the first week of orientation.
intervention. 3. This is expected in a client with leukemia,
3. Nurses can and do refer clients diagnosed but it indicates a severely low platelet count;
with cancer to the American Cancer Society– a nurse with more experience should care for
sponsored groups independently. Dialogue is this client.
a group support meeting that focuses on deal- 4. This client is receiving blood. The nurse
ing with the feelings associated with a cancer with experience on a medical-surgical
diagnosis. floor should be able to administer blood
4. Collaborative interventions involve other and blood products.
departments of the health-care facility. A TEST-TAKING HINT: The key to this question
chaplain is a referral that can be made, and is the fact, although the nurse is an experi-
the two disciplines should work together enced medical-surgical nurse, the nurse is
to provide the needed interventions. not experienced in oncology. The client who
TEST-TAKING HINT: The stem of the question could receive a treatment on a medical-sur-
asks for a collaborative intervention, which gical floor should be assigned to the nurse.
means that another health-care discipline Content – Nursing Management: Integrated Nursing
must be involved. Options “1,” “2,” and “3” Process – Planning: Client Needs – Safe Effective Care
are all interventions the nurse can do with- Environment, Management of Care: Cognitive Level –
out another discipline being involved. Synthesis: Concept – Nursing.
Content – Medical: Integrated Nursing Process – 21. 1. This is a false promise. Many clients undergo
Diagnosis: Client Needs – Psychosocial Integrity: Cognitive premature menopause as a result of the can-
Level – Analysis: Concept – Hematologic Regulation.
cer therapy.
19. 1. An MRI of the chest area will determine 2. The client should be taught to practice
numerous disease entities, but it cannot birth control during treatment and for
determine the specific morphology of at least two (2) years after treatment has
Reed-Sternberg cells, which are diagnostic ceased. The therapies used to treat the can-
for Hodgkin’s disease. cer can cause cancer. Antineoplastic medica-
2. A CT scan will show tumor masses in the tions are carcinogenic, and radiation therapy
area, but it is not capable of pathological has proved to be a precursor to leukemia. A
diagnosis. developing fetus would be subjected to the
3. ESR laboratory tests are sometimes used internal conditions of the mother.
to monitor the progress of the treatment 3. Some clients—but not all—do become sterile.
of Hodgkin’s disease, but ESR levels can be The client must understand the risks of ther-
e­levated in several disease processes. apy, but the nurse should give a realistic pic-
4. Cancers of all types are definitively ture of what the client can expect. It is correct
diagnosed through biopsy procedures. procedure to tell the client the nurse does not
The pathologist must identify Reed- know the absolute outcome of ­therapy. This is
Sternberg cells for a diagnosis of the ethical principle of veracity.
Hodgkin’s disease. 4. The therapy may interfere with the client’s
TEST-TAKING HINT: The test taker can elimi- menses, but it may be temporary.
nate the first three (3) answer options based TEST-TAKING HINT: Option “3” can be
on these tests giving general information on ­eliminated on the basis that it says “all”
multiple diseases. A biopsy procedure of the ­clients; if the test taker can think of one
involved tissues is the only procedure that case where “all” does not apply, then the
provides a definitive diagnosis. ­option is incorrect.
Content – Medical: Integrated Nursing Process – Content – Medical: Integrated Nursing Process –
Diagnosis: Client Needs – Physiological Integrity, Planning: Client Needs – Physiological Integrity,
Reduction of Risk Potential: Cognitive Level – Knowledge: Physiological Adaptation: Cognitive Level – Synthesis:
Concept – Hematologic Regulation. Concept – Nursing Roles.

05_Colgrove_Ch05.indd 163 02/06/16 7:58 PM


164 M ed -S urg S uccess

22. 1. Enlarged lymph tissue would occur in malignancies and there may be a genetic
Stage III or IV Hodgkin’s lymphoma. predisposition for the client to develop
2. A tender left upper quadrant would indicate cancer. The client should continue to be
spleen infiltration and occurs at a later stage. tested regularly.
3. Stage I lymphoma presents with no symp- 2. Follow-up appointments should be at least
toms; for this reason, clients are usually not yearly.
diagnosed until the later stages of lymphoma. 3. The client’s risk for developing cancer has in-
4. B-cell lymphocytes are the usual lymphocytes creased as a result of the therapies undergone
involved in the development of lymphoma, for the lymphoma.
but a serum blood test must be done specifi- 4. This client is in remission, and death is not
cally to detect B cells. They are not tested on imminent.
a CBC. TEST-TAKING HINT: The test taker should
TEST-TAKING HINT: Most cancers are staged look at the time frames in the answer op-
from 0 to IV. Stage 0 is microinvasive and tions. It would be unusual for a client to be
Stage I is minimally invasive, progressing to told to have a checkup every five (5) years.
Stage IV, which is large tumor load or distant Option “4” can be eliminated by the stem,
disease. If the test taker noted the “Stage I,” which clearly indicates the client is progress-
then choosing the option that presented ing well at the five (5)-year remission mark.
with the least amount of known disease—­ Content – Medical: Integrated Nursing Process –
option “3”—would be a good choice. Planning: Client Needs – Health Promotion and
Content – Medical: Integrated Nursing Process – Maintenance: Cognitive Level – Synthesis:
Planning: Client Needs – Physiological Integrity, Concept – Nursing Roles.
Physiological Adaptation: Cognitive Level – Synthesis:
Concept – Hematologic Regulation. Anemia
23. 1. After the first 15 minutes during which the
client tolerates the blood transfusion, it is 25. 1. The rugae in the stomach produce intrinsic
appropriate to ask the UAP to take the vital factor, which allows the body to use vitamin
signs as long as the UAP has been given B12 from the foods eaten. Gastric bypass
specific parameters for the vital signs. Any surgery reduces the amount of rugae dras-
vital sign outside the normal parameters tically. Clients develop pernicious anemia
must have an intervention by the nurse. (vitamin B12 deficiency). Other symptoms
2. Antineoplastic medication infusions must be of anemia include dizziness and the tachy-
monitored by a chemotherapy-certified, com- cardia and dyspnea listed in the stem.
petent nurse. 2. Folic acid deficiency is usually associated with
3. This is the responsibility of the ward secre- chronic alcohol intake.
tary or the nurse, not the unlicensed assistive 3. Iron deficiency is the result of chronic blood
personnel. loss or inadequate dietary intake of iron.
4. This represents the evaluation portion of the 4. Sickle cell anemia is associated with African
nursing process and cannot be delegated. Americans, but the symptoms and history in-
dicate a different anemia.
TEST-TAKING HINT: The test taker must decide
what is within the realm of duties of a UAP. TEST-TAKING HINT: The question did not give
Three (3) of the options have the UAP do- a lifetime history of anemia, which would be
ing some action with medications. This could associated with sickle cell disease. The stem
eliminate all of these. Option “1” did not say related a history of obesity and surgery. The
monitor or evaluate or decide on a nursing test taker should look for an answer related
action; this option only says the UAP can take to intake of vitamins and minerals. A review
vital signs on a client who is presumably stable of the anatomy of the stomach is the key to
because the infusion has been going long the question.
enough to reach the hourly time span. Content – Medical: Integrated Nursing Process –
Content – Medical: Integrated Nursing Process – Assessment: Client Needs – Physiological Integrity,
Planning: Client Needs – Safe Effective Care Environment, Physiological Adaptation: Cognitive Level – Analysis:
Management of Care: Cognitive Level – Synthesis: Concept – Hematologic Regulation.
Concept – Nursing Roles. 26. 1. Menorrhagia (excessive blood loss during
24. 1. The five (5)-year mark is a time for cel- menses) does not cause pain. Fibroids or
ebration for clients diagnosed with cancer, other factors that cause the menorrhagia may
but the therapies can cause secondary cause pain, but lack of rest or sleep is not
­responsible for the listlessness or fatigue.

05_Colgrove_Ch05.indd 164 02/06/16 7:58 PM


C hapter 5  H ematological D isorders 165

2. The symptoms are the direct result of the ex- 3. The stool will be a dark green–black,
cessive blood loss. which can mask the appearance of blood
3. Some viruses do cause a chronic fatigue syn- in the stool.
drome, but there is a direct cause and effect 4. The client should eat a well-balanced diet
from the menorrhagia. high in iron, vitamins, and protein. Fowl and
4. Menorrhagia is excessive blood loss dur- fish are encouraged.
ing menses. If the blood loss is severe, then TEST-TAKING HINT: The test taker could
the client will not have the blood’s oxygen- ­eliminate option “4” as a possible ­answer
carrying capacity needed for daily activities. because of the absolute word “only.”
The most frequent symptom and complica- ­Health-care professionals usually encourage
tion of anemia is fatigue. It frequently has the client to limit red and organ meats.
the greatest impact on the client’s ability to Content – Medical: Integrated Nursing Process –
function and quality of life. Planning: Client Needs – Physiological Integrity,
TEST-TAKING HINT: Three (3) of the ­answer op- Pharmacological and Parenteral Therapies: Cognitive
tions have the word “menorrhagia” in them, Level – Synthesis: Concept – Medication.
but option “3” detours from the subject to talk 29. 1. The nurse should evaluate the stools of a
about viruses; this could eliminate it as a con- client diagnosed with melena (dark, tarry
sideration. This question requires the test taker stools indicate blood) as part of the ongoing
to understand medical terminology—in this assessment.
case, what menorrhagia means—and then de- 2. The UAP can take the vital signs of a
cide what results from that process in the body. client who is stable; this client received
Content – Medical: Integrated Nursing Process – the blood the day before.
Implementation: Client Needs – Physiological Integrity, 3. Evaluation is the nurse’s responsibility; the
Physiological Adaptation: Cognitive Level – Analysis:
nurse should know exactly what the client
Concept – Hematologic Regulation.
is eating.
27. 1. The nurse should monitor the hemoglo- 4. A client with severe hemolytic anemia
bin and hematocrit in all clients diagnosed would have pancytopenia and is at risk for
with anemia. bleeding.
2. Because decreased oxygenation levels to TEST-TAKING HINT: Melena indicates a prob-
the brain can cause the client to become lem with the bowel movements and would
confused, a room where the client can indicate a need for nursing judgment. A
be observed frequently—near the nurse’s nurse cannot ­delegate judgment decisions.
desk—is a safety issue. Content – Medical: Integrated Nursing Process –
3. The client should include leafy, green veg- Planning: Client Needs – Safe Effective Care Environment,
etables in the diet. These are high in iron. Management of Care: Cognitive Level – Synthesis:
4. Numbness and tingling may occur Concept – Nursing Roles.
in anemia as a result of neurological
involvement. 30. 74 mL/hr.
5. Fatigue is the number-one presenting Pumps are set at an hourly rate. The client
symptom of anemia. in congestive heart failure should receive
blood at the slowest possible rate to prevent
TEST-TAKING HINT: This is an alternative-type
the client from further complications of fluid
question requiring the test taker to select volume overload. Each unit of blood must be
multiple correct answers. The test taker infused within four (4) hours of initiation of
could eliminate option “3” because the only the infusion.
clients told to limit green, leafy vegetables 250 mL + 45 mL = 295 mL
are those receiving Coumadin, an oral 295 mL ÷ 4 = 73 ¾ mL/hr, which rounded
anticoagulant. is 74 mL/hr.
Content – Medical: Integrated Nursing Process –
TEST-TAKING HINT: The test taker must think
Diagnosis: Client Needs – Safe Effective Care
Environment, Management of Care: Cognitive about the disease process and the normal
Level – Analysis: Concept – Hematologic Regulation. r­equirement for administering blood to
­arrive at the correct answer.
28. 1. Iron is constipating; an antidiarrheal is Content – Medical: Integrated Nursing Process –
­contraindicated for this drug. Implementation: Client Needs – Physiological Integrity,
2. Iron can cause gastrointestinal distress and Pharmacological and Parenteral Therapies: Cognitive
tolerance to it is built up gradually; exercise Level – Application: Concept – Medication.
has nothing to do with tolerating iron.

05_Colgrove_Ch05.indd 165 02/06/16 7:58 PM


166 M ed -S urg S uccess

31. 1. Most clients diagnosed with folic acid 33. 1. The client may need oxygen, but getting the
deficiency anemia have developed the client a wheelchair and getting the client back
­anemia from chronic alcohol abuse. to bed is priority.
Alcohol consumption increases the use 2. The client is experiencing dyspnea on ex-
of folates, and the alcoholic diet is usually ertion, which is common for clients with
deficient in folic acid. A referral to Alco- anemia. The client needs a wheelchair to
holics Anonymous would be appropriate. limit the exertion.
2. There is no connection between folic acid de- 3. The problem with this client is not a pulmo-
ficiency and leukemia; therefore, this ­referral nary one; it is a lack of hemoglobin.
would not be appropriate. 4. Even if the nurse helps the client ambulate,
3. A hematologist may see the client, but nurses the client still will not have the needed oxy-
usually don’t make this kind of referral; the gen for the tissues.
HCP would make this referral if the HCP TEST-TAKING HINT: The test taker should ask,
felt incapable of caring for the client. “What is causing the distress and what will
4. The social worker is not the most appropriate alleviate the distress the fastest?” The dis-
referral. tress is caused by exertion and it is occurring
TEST-TAKING HINT: The stem asks the test in the hallway. The most expedient interven-
taker to decide which referral is most appro- tion is to have the client stop the activity that
priate. ­Option “4” might be appropriate for is causing the distress; a chair will accom-
a number of different clients, but nothing in plish this, but the client should be returned
the stem indicates a specific need for social to the room, so it needs to be a wheelchair.
services. The test taker must decide which Content – Medical: Integrated Nursing Process –
could be a possible cause of folic acid defi- ­Implementation: Client Needs – Safe Effective Care
ciency anemia. Environment, Management of Care: Cognitive Level –
Content – Medical: Integrated Nursing Process – Application: Concept – Hematologic Regulation.
Planning: Client Needs – Safe Effective Care Environment, 34. 1. The usual medication dosing time for daily
Management of Care: Cognitive Level – Synthesis:
medications is 0900, but this is only an hour
Concept – Health-Care System.
after the breakfast meal. Iron absorption is
32. 1. Any nurse should be able to adminis- reduced when taken with food.
ter iron supplements, which are oral iron 2. This is approximately two (2) hours after
preparations. breakfast and is the correct dosing time
2. Any nurse should be able to give an intramus- for iron to achieve the best effects. Iron
cular medication. preparations should be administered one
3. Pancytopenia is a situation that develops (1) hour before a meal or two (2) hours
in clients diagnosed with aplastic anemia after a meal. Iron can cause gastrointesti-
because the bone marrow is not able to nal upset, but if administered with a meal,
produce cells of any kind. The client has absorption can be diminished by as much
anemia, thrombocytopenia, and leukope- as 50%.
nia. This client could develop an infection 3. This is the usual time health-care facilities
or hemorrhage, go into congestive heart serve lunch.
failure, or have a number of other com- 4. This time would be very close to the evening
plications develop. This client needs the meal and would decrease the absorption of
most experienced nurse. the iron.
4. A deficiency of erythropoietin is common TEST-TAKING HINT: The test taker could
in clients diagnosed with renal disease. The eliminate options “3” and “4” if the test taker
current treatment for this is to administer realized both of these times are very close
erythropoietin, a biologic response modifier, to mealtimes, so, with this in common, food
subcutaneously or, if the anemia is severe must pose a problem for the medication.
enough, a blood transfusion. Content – Medical: Integrated Nursing Process –
TEST-TAKING HINT: The test taker could ­Implementation: Client Needs – Safe Effective Care
eliminate options “1” and “2” because of the Environment, Management of Care: Cognitive Level –
words “most experienced nurse” in the stem. Application: Concept – Medication.
Content – Medical: Integrated Nursing Process – 35. 1. This is an instruction for antibiotics, not iron.
Planning: Client Needs – Safe Effective Care Environment, The client will take iron for an indefinite
Management of Care: Cognitive Level – Synthesis:
period.
Concept – Nursing Roles.

05_Colgrove_Ch05.indd 166 02/06/16 7:58 PM


C hapter 5  H ematological D isorders 167

2. Pulse is indirectly affected by anemia when emergency surgery; therefore, this client
the body attempts to compensate for the lack should be assigned to a more experienced
of oxygen supply, but this is an indirect mea- nurse.
sure, and blood pressure is not monitored for TEST-TAKING HINT: The test taker must think
anemia. about what type of client a new graduate
3. The client should have a complete blood should be assigned. The least critical client is
count regularly to determine the status of the correct choice.
the anemia.
Content – Medical: Integrated Nursing Process –
4. Isometric exercises are bodybuilding exer- Planning: Client Needs – Safe Effective Care Environment,
cises, and the client should not be exerting Management of Care: Cognitive Level – Synthesis:
himself or herself in this manner. Concept – Nursing Roles.
TEST-TAKING HINT: The test taker could
38. 1. DIC is a complication in many obstetric
­ liminate option “1” because this applies
e problems, including septic abortion, abruptio
to antibiotics and option “4” because it is placentae, amniotic fluid embolus, and re-
isometric exercise. tained dead fetus, but it is not a complication
Content – Medical: Integrated Nursing Process – of placenta previa.
­Planning: Client Needs – Physiological Integrity, 2. A client with a fat embolus is at risk for DIC,
Physiological Adaptation: Cognitive Level – Synthesis:
but a client with a pulmonary embolus is not.
Concept – Nursing Roles.
3. Hemodialysis is not a risk factor for develop-
36. 1. The client’s problem is activity intoler- ing DIC.
ance, and pacing of activities directly 4. DIC is a clinical syndrome that devel-
affects the diagnosis. ops as a complication of a wide variety
2. This is an appropriate intervention for iron of other disorders, with sepsis being the
or vitamin deficiency, but it is not for activity most common cause of DIC.
intolerance. TEST-TAKING HINT: The test taker could elim-
3. This may be done but not specifically for the inate option “3” if the test taker knew that
diagnosis. DIC is a complication of another disorder.
4. This would not help activity intolerance. Age is not a risk factor for developing DIC.
TEST-TAKING HINT: The test taker should read Content – Medical: Integrated Nursing Process –
the stem closely and choose only interven- Assessment: Client Needs – Physiological Integrity,
tions that directly affect an activity. The word Reduction of Risk Potential: Cognitive Level – Analysis:
“activity” is in the diagnosis and in the correct Concept – Clotting.
answer. When an answer option matches the 39. 1. The signs/symptoms of DIC result from
stem, it is a good choice. clotting and bleeding, ranging from ooz-
Content – Medical: Integrated Nursing Process – ing blood to bleeding from every body
Implementation: Client Needs – Physiological Integrity, orifice and into the tissues.
Physiological Adaptation: Cognitive Level – Application:
2. Chest pain and frothy sputum may indicate a
Concept – Hematologic Regulation.
pulmonary embolus.
3. Foul-smelling, concentrated urine may
Bleeding Disorders ­indicate dehydration or urinary tract
infection.
37. 1. This is a major surgery but has a predictable 4. A reddened, inflamed central line catheter
course with no complications identified in site indicates a possible infection.
the stem, and a colostomy is expected with TEST-TAKING HINT: If the test taker realized
this type of surgery. The graduate nurse that coagulation deals with blood, then the
could be assigned this client. only answer option that addresses any type of
2. Acute respiratory distress syndrome (ARDS) bleeding is option “1.”
is a potentially life-threatening complication Content – Medical: Integrated Nursing Process –
and should be assigned to a more experienced Assessment: Client Needs – Physiological Integrity,
nurse. Reduction of Risk Potential: Cognitive Level – Analysis:
3. Disseminated intravascular coagulation (DIC) Concept – Clotting.
is life threatening. The client is u
­ nstable and
40. 1. The prothrombin time (PT), along with
should be assigned to a more experienced nurse.
the partial thromboplastin time (PTT) and
4. This client is experiencing hypovolemia,
thrombin time, are prolonged or increased in
which means hemorrhaging and potential
a client with DIC.

05_Colgrove_Ch05.indd 167 02/06/16 7:58 PM


168 M ed -S urg S uccess

2. The fibrinogen level helps predict bleed- 2. This is a true statement, but it refers to the
ing in DIC. As it becomes lower, the risk pathophysiology of hemophilia A and does
of bleeding increases. not explain how someone gets the disease.
3. The platelet count is decreased in DIC. The 3. This is a true statement, but it does not an-
platelets are used up because clotting and swer the question of how someone gets it.
bleeding are occurring simultaneously. 4. This is a true statement and explains
4. White blood cell counts increase as a result of ­exactly how someone gets hemophilia
infection, not from DIC. A: The mother passes it to the son.
TEST-TAKING HINT: An understanding of TEST-TAKING HINT: When the stem has the
bleeding may help the test taker rule out word “best” in it, then all four answer options
options “3” and “4” because platelets are could be correct and, in this case, statements
needed for clotting, so an increased platelet that the nurse could reply, but only one an-
count would not cause bleeding, and WBCs swer option is best. The test taker needs to
are associated with infection. If the test taker evaluate the stem and identify terms that may
thinks about oral anticoagulant therapy and help select the best option; in this case, a UAP
remembers that PT is prolonged with bleed- is asking the question and a direct answer
ing, it might lead to eliminating option “1” as without using medical jargon should be given.
a possible correct answer. Content – Medical: Integrated Nursing Process –
Content – Medical: Integrated Nursing Process – Implementation: Client Needs – Safe Effective Care
Assessment: Client Needs – Physiological Integrity, Environment, Management of Care: Cognitive Level –
Reduction of Risk Potential: Cognitive Level – Analysis: Application: Concept – Clotting.
Concept – Clotting.
43. 1. Nosebleeds, along with hemarthrosis,
41. 1. Heparin, a parenteral anticoagulant, is admin- cutaneous hematoma formation, bleed-
istered to interfere with the clotting cascade ing gums, hematemesis, occult blood,
and may prevent further clotting factor con- and hematuria, are all signs/symptoms of
sumption as a result of uncontrolled bleeding, hemophilia.
but its use is controversial. Oral anticoagu- 2. Petechiae are tiny purple or red spots that
lants are not administered. appear on the skin as a result of minute hem-
2. Plasmapheresis involves the removal of orrhages within the dermal or submucosal
plasma from withdrawn blood by centrifuga- layers, but they are not signs of hemophilia.
tion, reconstituting it in an isotonic solution, 3. Subcutaneous emphysema is air under the
and then reinfusing the solution back into skin, which may occur with chest tube or tra-
the body, but it is not a treatment for DIC. cheostomy insertion.
3. Fresh frozen plasma and platelet concen- 4. Intermittent claudication is severe leg pain
trates are administered to restore clotting that occurs from decreased oxygenation to
factors and platelets. the leg muscles.
4. Calculating the intake and output is adding TEST-TAKING HINT: If the test taker knows
up how much oral and intravenous fluids went that hemophilia is a bleeding disorder, the
into the client and how much fluid came out test taker should eliminate any answer
of the client. It does not require an HCP’s or- options that don’t address bleeding—in this
der and thus is not a collaborative treatment. case, options “3” and “4.”
TEST-TAKING HINT: “Collaborative” means Content – Medical: Integrated Nursing Process –
another health-care discipline must order or Assessment: Client Needs – Physiological Integrity,
perform the intervention. A test-taking hint Reduction of Risk Potential: Cognitive Level – Analysis:
that may help with unit examinations, but not Concept – Clotting.
with the NCLEX-RN, is that, if the test taker 44. 1. Vomiting blood is not a situation that would
has studied the assigned content and has never indicate the client has von Willebrand’s
heard of one of the words in the answer options, disease.
the test taker should not select that answer—in 2. Microscopic blood in the urine is not a sign
this case, perhaps the word “plasmapheresis.” of von Willebrand’s disease.
Content – Medical: Integrated Nursing Process – 3. von Willebrand’s disease is a type of
Planning: Client Needs – Physiological Integrity, hemophilia. The most common ­hereditary
Reduction of Risk Potential: Cognitive Level – Synthesis:
bleeding disorder, it is caused by a defi-
Concept – Clotting.
ciency in von Willebrand’s (vW) factor and
42. 1. This is a true statement, but it is medical jar- is often diagnosed after ­prolonged bleed-
gon explaining how someone gets hemophilia ing following surgery or dental extraction.
A and so is not the best response.

05_Colgrove_Ch05.indd 168 02/06/16 7:58 PM


C hapter 5  H ematological D isorders 169

4. Abruptio placentae is not a situation that Options “2” and “4” could be eliminated as
might cause von Willebrand’s disease. possible answers based on the knowledge
TEST-TAKING HINT: The test taker must be that these are normal values.
knowledgeable about vW factor to be able to Content – Medical: Integrated Nursing Process –
answer this question. Risk factors or situa- Assessment: Client Needs – Physiological Integrity,
tions are facts that need to be memorized. Reduction of Risk Potential: Cognitive Level – Analysis:
Concept – Hematologic Regulation.
Content – Medical: Integrated Nursing Process –
Assessment: Client Needs – Physiological Integrity, 47. 1. A range for the normal PTT is 32 to
Reduction of Risk Potential: Cognitive Level – Analysis: 39 seconds.
Concept – Clotting. 2. These are normal hemoglobin/hematocrit
45. 1. The client should avoid prescription and levels for either a male or female client.
over-the-counter drugs containing aspirin 3. A platelet count of less than 100,000
because these drugs may have an antiplatelet per cubic millimeter of blood indicates
effect, leading to bleeding. thrombocytopenia.
2. Hemarthrosis is bleeding into the joint. 4. This is a normal red blood cell count.
Applying ice to the area can cause TEST-TAKING HINT: The test taker must be
­vasoconstriction, which can help ­decrease ­knowledgeable of normal laboratory values.
bleeding. The test taker should write these normal
3. The joint should be immobilized for 24 to values on a 3 × 5 card, carry it with him or
48 hours after the bleeding starts. her, and memorize the values.
4. Dependent position is putting the extremity Content – Medical: Integrated Nursing Process –
below the level of the heart; the extremity Assessment: Client Needs – Safe Effective Care
should be elevated if possible. Environment, Management of Care: Cognitive
TEST-TAKING HINT: If the test taker does Level – Analysis: Concept – Hematologic Regulation.
not know the answer, the test taker could 48. 1. The client should use an electric razor, which
apply ­medical ­terminology; in this case, the minimizes the opportunity to develop super-
terminology contains hem-, which refers to ficial cuts that may result in bleeding.
“blood.” A basic concept with bleeding is 2. Enemas, rectal thermometers, and
that cold causes vasoconstriction; therefore, intramuscular injections can pose a risk
option “2” would be a good choice to select. of tissue and vascular trauma that can
Content – Medical: Integrated Nursing Process – p­recipitate bleeding.
Planning: Client Needs – Physiological Integrity, 3. Even minor trauma can lead to serious
Physiological Adaptation: Cognitive Level – Synthesis: bleeding episodes; safer activities such as
Concept – Clotting. swimming or golf should be recommended.
46. 1. ITP is caused by bleeding from small ves- 4. Direct pressure occludes bleeding vessels.
sels and mucous membranes. Petechiae, 5. There is no reason why the client can’t floss
tiny purple or red spots that appear on the the teeth.
skin as a result of minute hemorrhages TEST-TAKING HINT: This type of question re-
within the dermal or submucosal layers, quires the test taker to select all interventions
and purpura, hemorrhaging into the tissue that apply. Bleeding is the priority concern with
beneath the skin and mucous membranes, hemophilia A, so all interventions should be
are the first signs of ITP. based around activities that can potentially
2. A capillary refill time (CRT) less than three cause bleeding and ways to treat bleeding.
(3) seconds is a normal assessment finding Content – Medical: Integrated Nursing Process –
and would not indicate ITP. Implementation: Client Needs – Safe Effective Care
3. A splenectomy is the treatment of choice if Environment, Management of Care: Cognitive
glucocorticosteroid therapy does not treat the Level – Application: Concept – Clotting.
ITP, but the spleen is not enlarged.
4. ITP causes bleeding, but it does not affect the
oxygen that gets to the periphery. Blood Transfusions
TEST-TAKING HINT: Knowing medical
49. 1. The client must give permission to r­ eceive
­­terminology—in this case, that thrombo- blood or blood products because of the
refers to “platelets”—the test taker could nature of potential complications.
conclude that an answer option that includes 2. Most blood products require at least a
something about bleeding would be the most 20-gauge IV because of the size of the cells.
appropriate selection for the correct answer.

05_Colgrove_Ch05.indd 169 02/06/16 7:58 PM


170 M ed -S urg S uccess

RBCs are best infused through an 18-gauge test taker must not read into a question—for
IV. If unable to achieve cannulation with an ­example, that consent is needed to send a cli-
18-gauge, a 20-gauge is the smallest accept- ent to surgery to correct the problem, so that
able IV. Smaller IVs damage the cell walls of could be first. Only one answer option has
the RBCs and reduce the life expectancy of the potential to stabilize the client.
the RBCs. Content – Medical: Integrated Nursing Process –
3. Because infusing IV fluids can cause a Implementation: Client Needs – Safe Effective Care
fluid volume overload, the nurse must as- Environment, Management of Care: Cognitive
sess for congestive heart failure. Assess- Level – Synthesis: Concept – Trauma.
ing the lungs includes auscultating for 51. 1. Oral surgeries are associated with transient
crackles and other signs of left-sided heart bacteremia, and the client cannot donate for
failure. Additional assessment findings of 72 hours after an oral surgery.
jugular vein distention, peripheral edema, 2. The client cannot donate blood for one
and liver engorgement indicate right- (1) month following rubella immunization.
sided failure. 3. The client cannot donate blood for six (6)
4. Checking for allergies is important prior months after a pregnancy because of the
to administering any medication. Some nutritional demands on the mother.
medications are administered prior to 4. Recent allergic reactions prevent donation
blood administration. because passive transference of hypersensitiv-
5. A keep-open IV of 0.9% saline would be ity can occur. This client has an allergy devel-
hung. D5W causes red blood cells to hemo- oped during childhood.
lyze in the tubing.
TEST-TAKING HINT: All of the answer options
TEST-TAKING HINT: This is an alternative-type
have a given time period, and these time
question. This type of question can appear frames make each option correct or incorrect.
a­ nywhere on the NCLEX-RN examination. The test taker must pay particular attention
Each answer option must be evaluated on its whenever an option contains time frames. Is
own merit. One will not rule out another. As- it long enough or not frequent enough?
sessing is the first step of the nursing process.
Content – Medical: Integrated Nursing Process –
Unless the test taker is absolutely sure that an
Evaluation: Client Needs – Safe Effective Care
option is wrong, the test taker could select an Environment, Management of Care: Cognitive
option based on “assessing,” such as options Level – Knowledge: Concept – Promoting Health.
“3” and “4.” E
­ thically speaking, informed con-
sent should ­always be given for any procedure 52. 1. O2 (O-negative) blood is considered
unless an emergency life-or-death situation the universal donor because it does not
exists. The other options require knowledge contain the antigens A, B, or Rh. (AB1 is
of blood and blood product administration. considered the universal recipient ­because
Content – Medical: Integrated Nursing Process – a person with this blood type has all the
Implementation: Client Needs – Safe Effective Care antigens on the blood.)
Environment, Management of Care: Cognitive 2. A+ blood contains the antigen A that the cli-
Level – Application: Concept – Hematologic Regulation. ent will react to, causing the development of
antibodies. The unit being Rh+ is compatible
50. 1. This should be done, but the client requires with the client.
the IV first. This client is at risk for shock. 3. B+ blood contains the antigen B that the cli-
2. The first action in a situation in which ent will react to, causing the development of
the nurse suspects the client has a fluid antibodies. The unit being Rh+ is compatible
volume loss is to replace the volume as with the client.
quickly as possible. 4. This client does not have antigens A or B
3. The client will probably need to have surgery
on the blood. Administration of these types
to correct the source of the bleeding, but sta-
would cause an antigen-antibody r­ eaction
bilizing the client with fluid resuscitation is within the client’s body, resulting in massive
first priority. hemolysis of the client’s blood and death.
4. This is the last thing on this list in order of
priority. TEST-TAKING HINT: This is a knowledge-based
question that requires memorization of
TEST-TAKING HINT: The question requires
the particular facts regarding blood typing.
the test taker to decide which of the actions Three of the possible answer options have a
comes first. Only one of the options actu- positive (1) Rh factor; only one has a nega-
ally has the nurse treating the client. The tive (2) Rh factor.

05_Colgrove_Ch05.indd 170 02/06/16 7:58 PM


C hapter 5  H ematological D isorders 171

Content – Medical: Integrated Nursing Process – ­ the outside of the bandage and started if the
Implementation: Client Needs – Safe Effective Care surgeon so orders, but this has nothing to do
Environment, Management of Care: Cognitive with the blood.
Level – Analysis: Concept – H
­ ematologic Regulation. 4. The blood has not been crossmatched so
53. 1. A steroid could delay healing time after the there is not a crossmatch number.
surgery and has no effect on the production TEST-TAKING HINT: The test taker could dis-
of red blood cells. card option “4” if the test taker realized that
2. An antibiotic does not increase the produc- the ­laboratory is not involved with this blood
tion of red blood cells. Orthopedic surgeries at all. The test taker must have basic knowl-
frequently involve blood loss. The client is edge of ­surgical care.
wishing to donate blood to himself or herself Content – Surgical: Integrated Nursing Process –
(autologous). ­Implementation: Client Needs – Safe Effective Care
3. Tranquilizers do not affect the production of Environment, Management of Care: Cognitive
red blood cells. Level – Application: Concept – Hematologic Regulation.
4. Epogen and Procrit are forms of eryth-
55. 1. Blood will coagulate if left out for an
ropoietin, the substance in the body that
­extended period, but blood is stored with a
stimulates the bone marrow to produce
preservative that prevents this and prolongs
red blood cells. A client may be prescribed
the life of the blood.
iron preparations to prevent depletion
2. Blood is a medium for bacterial growth,
of iron stores and erythropoietin to in-
and any bacteria contaminating the unit
crease RBC production. A unit of blood
will begin to grow if left outside of a con-
can be withdrawn once a week beginning
trolled refrigerated temperature for lon-
at six (6) weeks prior to surgery. No phle-
ger than four (4) hours, placing the client
botomy will be done within 72 hours of
at risk for septicemia.
surgery.
3. Blood components are stable and do not
TEST-TAKING HINT: The test taker should break down after four (4) hours.
examine the key words “autologous” and 4. These are standard nursing and laboratory
“transfusion.” If the test taker did not know procedures to prevent the complication of
the meaning of the word “autologous,” septicemia.
“auto-” as a prefix refers to “self,” such as
TEST-TAKING HINT: The test taker must know
an autobiography is one’s own story. Pairing
the rationale behind nursing interventions to
“self” with “transfusion” then should make
be able to answer this question.
the test taker look for an option that would
directly affect the production of blood cells. Content – Medical: Integrated Nursing Process –
­Implementation: Client Needs – Safe Effective Care
Content – Medical: Integrated Nursing Process – Environment, Management of Care: Cognitive Level –
­Implementation: Client Needs – Physiological Integrity, Knowledge: Concept – Hematologic Regulation.
Pharmacological and Parenteral Therapies: Cognitive
Level – Application: Concept – Medication. 56. 1. The HCP has written an appropriate order
for this client, who has heart failure, and does
54. 1. A cell saver is a device to catch the blood
not need to be called to verify the order be-
lost during orthopedic surgeries to rein-
fore the nurse implements it.
fuse into the client, rather than giving the
2. Blood or blood components have a specified
client donor blood products. The cells are
amount of infusion time, and this is not eight
washed with saline and reinfused through
(8) hours. The time constraints are for the
a filter into the client. The salvaged cells
protection of the client.
cannot be stored and must be used within
3. The correct procedure for administering
four (4) hours or discarded because of
a unit of blood over eight (8) hours is to
bacterial growth.
have the unit split into halves. Each half-
2. The cell saver has a measuring device; an
unit is treated as a new unit and checked
hourly drainage bag is part of a urinary drain-
accordingly. This slower administration al-
age system. A cell saver is a sterile system that
lows the compromised client, such as one
should not be broken until ready to discon-
with heart failure, to assimilate the extra
nect for reinfusion.
fluid volume.
3. The post-anesthesia care unit nurse would
4. This rate has already been determined by the
not replace the cell saver; it is inserted into
HCP to be unsafe for this client.
the surgical wound. A continuous passive
motion (CPM) machine can be attached on

05_Colgrove_Ch05.indd 171 02/06/16 7:58 PM


172 M ed -S urg S uccess

TEST-TAKING HINT: The key to this ques- nurse cannot ­delegate assessment or any
tion is the time frame of eight (8) hours intervention ­requiring nursing judgment.
and the client’s diagnosis of heart failure. ­Options “1,” “2,” and “3” ­require judgment
Basic knowledge of heart failure allows the and cannot be delegated to a UAP.
test taker to realize that fluid volume is the Content – Medical: Integrated Nursing Process –
­problem. Only one option addresses admin- Planning: Client Needs – Safe Effective Care Environment,
istering a smaller volume at a time. Management of Care: Cognitive Level – Synthesis:
Content – Medical: Integrated Nursing Process – Concept – Nursing Roles.
Implementation: Client Needs – Safe Effective Care 59. 1. The likelihood of a client who has ­already
Environment, Management of Care: Cognitive
received more than half of the blood ­product
Level – Application: Concept – Hematologic Regulation.
having a transfusion reaction is slim. The first
57. 1. This should be done but after preventing any 15 minutes have passed and to this point the
more of the PRBCs from infusing. client is tolerating the blood.
2. Benadryl may be administered to reduce the 2. Clients diagnosed with leukemia have a
severity of the transfusion reaction, but it is ­cancer involving blood cell production. These
not first priority. are expected findings in a client d­ iagnosed
3. The nurse should assess the client, but in with leukemia.
this case the nurse has all the assessment data 3. This client has a potential for ­hemorrhage
needed to stop the transfusion. and is reporting blood in the vomitus.
4. The priority in this situation is to ­prevent This client should be a­ ssessed first.
a further reaction if possible. Stopping 4. Crohn’s disease involves frequent diarrhea
the transfusion and changing the fluid stools, leading to perineal irritation and skin
out at the hub will prevent any more of excoriation. This is expected and not life
the transfusion from entering the client’s threatening. Clients “1,” “2,” and “3” should
bloodstream. be seen before this client.
TEST-TAKING HINT: In a question that re- TEST-TAKING HINT: In a prioritizing question,
quires the test taker to determine a prior- the test taker should be able to rank in order
ity action, the test taker must decide what which client to see first, second, third, and
will have the most impact on the client. fourth. ­Expected but not immediately life-
­Option “4” does this. All the options are threatening situations are seen after a situa-
interventions that should be taken, but only tion in which the client has a life-threatening
one will be first. problem.
Content – Medical: Integrated Nursing Process – Content – Medical: Integrated Nursing Process –
­Implementation: Client Needs – Safe Effective Care Assessment: Client Needs – Safe Effective Care
Environment, Management of Care: Cognitive Level – Environment, Management of Care: Cognitive
Synthesis: Concept – ­Hematologic Regulation. Level – Analysis: Concept – Hematologic Regulation.

58. 1. UAPs cannot assess. The nurse cannot 60. 886 mL of fluid has infused.
­delegate assessment. 250 mL + 63 mL = 313 mL per unit
2. The likelihood of a reaction is the greatest 313 + 313 = 626 mL
during the first 15 minutes of a transfusion. 500 mL of saline – 240 mL remaining =
The nurse should never leave the client until 260 mL infused.
after this time. The nurse should take and as- 626 mL + 260 mL = 886 mL of fluid
sess the vital signs during this time. infused.
3. Auscultation of the lung sounds and admin- TEST-TAKING HINT: This problem has several
istering blood based on this information are steps but only requires basic addition and
the nurse’s responsibility. Any action requir- subtraction. The test taker should use the
ing nursing judgment cannot be delegated. drop-down calculator on the computer to
4. The UAP can assist a client to brush the check or double-check the answer to make
teeth. Instructions about using soft-bristle sure that simple mistakes are not made.
toothbrushes and the need to report to Content – Medical: Integrated Nursing Process –
the nurse any pink or bleeding gums Implementation: Client Needs – Physiological Integrity,
should be given prior to delegating the Pharmacological and ­Parenteral Therapies: Cognitive
procedure. Level – Application: Concept – ­Hematologic Regulation.
TEST-TAKING HINT: The test taker must
be aware of delegation guidelines. The

05_Colgrove_Ch05.indd 172 02/06/16 7:58 PM


C hapter 5  H ematological D isorders 173

Sickle Cell Anemia 2. A pulse oximeter reading of less than 93%


indicates hypoxia, which warrants oxygen
61. 1. Offering the nursing student written ­material administration.
is appropriate, but the nurse’s best statement 3. The nurse should start an 18-gauge IV
would be to answer the student’s question. ­catheter because the client may need blood,
2. The problem in sickle cell anemia is the but this is not the nurse’s first intervention.
blood clots inappropriately when there is a 4. The medication will be administered intra-
decrease in oxygenation. venously, so the IV will have to be started
3. This is a true statement, but it is not the best before administering the medication.
response because it is not answering the nurs- TEST-TAKING HINT: If the test taker can elimi-
ing student’s question. nate two (2) options and cannot decide
4. Sickle cell anemia is a disorder of the red between the other two (2), the test taker
blood cells characterized by abnormally should apply a rule such as Maslow’s hierar-
shaped red cells that sickle or clump chy of needs and select the intervention that
­together, leading to oxygen deprivation addresses oxygenation or airway.
and resulting in crisis and severe pain. Content – Medical: Integrated Nursing Process – ­
TEST-TAKING HINT: When answering a Implementation: Client Needs – Safe Effective Care
­question, the test taker should really close Environment, Management of Care: Cognitive
the eyes and answer the question exactly Level – Synthesis: Concept – Hematologic Regulation.
how he or she would answer in the clini- 64. 1. Demerol is no longer recommended for
cal setting. Most nurses would answer the long-term pain management because the ac-
student’s question and not offer written cumulative effect of normeperidine can cause
material. seizures.
Content – Medical: Integrated Nursing Process – 2. The client does not need to be in reverse iso-
­Implementation: Client Needs – Safe Effective Care lation because other people will not affect the
Environment, Management of Care: Cognitive crisis.
Level – Application: Concept – Hematologic Regulation. 3. Increased intravenous fluid reduces the
62. 1. This is the etiology for sickle cell anemia viscosity of blood, thereby preventing fur-
(SCA), but a layperson would not understand ther sickling as a result of dehydration.
this explanation. 4. The client needs to be receiving intake and
2. This explains the etiology in terms that a output monitoring, but there is no reason to
layperson could understand. When both insert a Foley catheter.
parents are carriers of the disease, each TEST-TAKING HINT: The test taker would have
pregnancy has a 25% chance of producing to know that sickling is caused by decreased
a child who has sickle cell anemia. oxygen or dehydration to know this answer.
3. The cause of SCA is known, and genetic Possibly looking at the word “infection” and
counseling can explain it to the prospective increasing fluids may help the test taker
parent. s­elect this option.
4. A virus does not cause sickle cell anemia. Content – Medical: Integrated Nursing Process –
TEST-TAKING HINT: When discussing disease Planning: Client Needs – Physiological Integrity, Reduction
­processes with laypersons, the nurse should of Risk Potential: Cognitive Level – Synthesis:
explain the facts in terms that the person can Concept – Hematologic Regulation.
understand. Would a layperson know what 65. 1. To assess for cyanosis (blueness) in indi-
“­autosomal recessive” means? The test taker viduals with dark skin, the oral mucosa
should consider terminology when selecting and conjunctiva should be assessed be-
an answer. cause cyanosis cannot be assessed in the
Content – Medical: Integrated Nursing Process – lips or fingertips.
­Implementation: Client Needs – Safe Effective Care 2. Metacarpals are the fingers, which should not
Environment, Management of Care: Cognitive be assessed for cyanosis.
Level – Application: Concept – Hematologic Regulation. 3. Capillary refill time is not a reliable indicator
63. 1. The health-care provider could order STAT for cyanosis in individuals with dark skin.
ABGs, but caring directly for the client is al- 4. The nurse should assess the conjunctiva for
ways the first priority. paleness, indicating hypoxemia; the sclera is
assessed for jaundice.

05_Colgrove_Ch05.indd 173 02/06/16 7:58 PM


174 M ed -S urg S uccess

TEST-TAKING HINT: The test taker must real- 2. Compartment syndrome is a complication of
ize that assessing different nationalities or a cast that has been applied too tightly or a
races requires different assessment tech- fracture in which there is edema in a muscle
niques and data. compartment.
Content – Medical: Integrated Nursing Process – 3. This is a term that means painful and
Assessment: Client Needs – Physiological Integrity, constant penile erection that can occur in
Reduction of Risk Potential: Cognitive Level – Analysis: male clients with SCA during a sickle cell
Concept – Hematologic Regulation. crisis.
4. A hypertensive crisis is potentially fatal, but it
66. 3,000 mL.
is not a complication of SCA. The client with
The key is knowing that 1 ounce is
sickle cell anemia usually has cardiomegaly or
equal to 30 mL. Then, 20 ounces
(20 × 30) = 600 mL, 8 ounces systolic murmurs; both genders have this.
(8 × 30) = 240 mL, and 4 ounces TEST-TAKING HINT: This is a knowledge-
(4 × 30) = 120 × 3 cartons = 360 mL for based question, but if the test taker realized
a total of 600 + 240 + 360 = 1,200 mL of that priapism could only occur in males, this
oral fluids. That, plus 1,800 mL of IV fluids, might help the test taker select option “3” as
makes the total intake for this shift 3,000 mL. a correct answer. Whenever there is a gender
for the client, it usually has something to do
TEST-TAKING HINT: The test taker must
with the correct answer.
memorize equivalents such as how many mil-
liliters are in an ounce. This is the only way Content – Medical: Integrated Nursing Process –
that the test taker can convert the client’s in- Diagnosis: Client Needs – Physiological Integrity,
Physiological Adaptation: Cognitive Level – Analysis:
take to be able to assess the balance between
Concept – Hematologic Regulation.
intake and output.
Content – Medical: Integrated Nursing Process – ­ 69. 1. Health promotion is important in clients with
Implementation: Client Needs – Safe Effective Care chronic illnesses, but the best answer should
­Environment, Management of Care: Cognitive address the client’s specific disease process,
Level – Application: Concept – Nutrition. not chronic illnesses in general.
2. An individual with SCA has a reduction
67. 1. Whenever an opportunity presents itself, the
in splenic activity from infarcts occurring
nurse should teach the client about his or her
during crises. This situation progresses to
condition. This client should not go to areas
the spleen no longer being able to func-
that have decreased oxygen, such as Yellow-
tion, and this increases the client’s suscep-
stone National Park, which is at high altitude.
tibility to infection.
2. This is passing the buck. The nurse can re-
3. These vaccines do nothing to prevent sickling
spond to this comment.
of the blood cells.
3. High altitudes have decreased oxygen,
4. Teaching the client is an independent nursing
which could lead to a sickle cell crisis.
intervention, and the nurse does not need to
4. It is none of the nurse’s business why the cli-
rely on the HCP to designate what should be
ent wants to go to Yellowstone National Park.
taught to the client.
The client’s safety comes first, and the nurse
needs to teach the client. TEST-TAKING HINT: Vaccines are recom-
mended to clients with chronic problems to
TEST-TAKING HINT: Even if the test taker did
help prevent the flu and pneumonia. The test
not know the answer to this question, the
taker should see if a widely accepted concept
only answer option that does any teaching is
could help answer the question and not get
option “3,” which would be the best choice.
caught up in the client’s ­disease process.
Yellowstone National Park, because of its al-
titude, has something to do with the answer. Content – Medical: Integrated Nursing Process –
­Implementation: Client Needs – Health Promotion
Content – Medical: Integrated Nursing Process –
and Maintenance: Cognitive Level – Application:
­Implementation: Client Needs – Physiological Integrity,
Concept – ­Hematologic Regulation.
­Physiological Adaptation: Cognitive Level – Application:
Concept – Hematologic Regulation. 70. 1. Research and a search for a cure are not the
missions of the Sickle Cell Foundation.
68. 1. Chest syndrome refers to chest pain, fever,
2. The foundation’s mission is to pro-
and a dry, hacking cough with or without pre-
vide information about the disease and
existing pneumonia and is not a fatal compli-
about support groups in the area. This
cation. It can occur in either gender.

05_Colgrove_Ch05.indd 174 02/06/16 7:58 PM


C hapter 5  H ematological D isorders 175

information helps decrease the client’s important part of being able to answer
and significant others’ feelings of frustra- NCLEX-RN questions. These terms can
tion and helplessness. help eliminate or select an answer option.
3. The nurse should not force personal thoughts Content – Medical: Integrated Nursing Process –
on the client. The nurse should provide infor- Assessment: Client Needs – Physiological Integrity,
mation and let the client make his or her own Reduction of Risk Potential: Cognitive Level – Analysis:
decision. This empowers the client. Concept – Hematologic Regulation.
4. The nurse can arrange for families to meet, 72. 1. A spinal tap is a test used to diagnose
but this is not the mission of the foundation. meningitis.
TEST-TAKING HINT: The nurse should know 2. Hemoglobin electrophoresis is a test used
about organizations for specific disease pro- to help diagnose sickle cell anemia; it is the
cesses in the geographic area, but most orga- “fingerprinting” of the protein, which detects
nizations provide information on the disease homozygous and heterozygous forms of the
process and support groups. disease.
Content – Medical: Integrated Nursing Process – 3. The Sickledex test is a screening test com-
Implementation: Client Needs – Safe Effective Care monly used to screen for sickle cell anemia. It
Environment, Management of Care: Cognitive is performed by a finger stick with results in
Level – Application: Concept – Promoting Health. three (3) minutes.
71. 1. Skeletal deformities, such as lordosis or 4. The elevated temperature is the first sign
­kyphosis, are common. They are secondary of bacteremia. Bacteremia leads to a sickle
to chronic vaso-occlusive crisis, not an acute cell crisis. Therefore, the bacteria must be
crisis. identified so the appropriate antibiotics
2. A bloody nose is not an expected finding in a can be prescribed to treat the infection.
client diagnosed with an acute vaso-occlusive Blood cultures assist in determining the
crisis. type and source of infection so that it can
3. Vaso-occlusive crisis, the most frequent be treated appropriately.
crisis, is characterized by organ infarction, TEST-TAKING HINT: The client’s temperature
which will result in bloody urine second- in the stem is the key to selecting blood cul-
ary to kidney infarction. tures as the correct answer.
4. Petechiae are small pinpoint blood spots on Content – Medical: Integrated Nursing Process –
the skin, but they are not signs/symptoms of a Planning: Client Needs – Physiological Integrity, Reduction
vaso-occlusive crisis. of Risk Potential: Cognitive Level – Synthesis: Concept –
TEST-TAKING HINT: Understanding medi- Hematologic Regulation.
cal ­terminology of assessment data is an

05_Colgrove_Ch05.indd 175 02/06/16 7:58 PM


CONCEPTS

73. 1. The client is at risk of infection because Option “2” is a psychological problem; physi-
of the lack of mature WBCs in the circu- ological issues are priority over psychological
lating blood. Leukemia is a disease pro- problems.
cess involving the hematological system. Content – Medical: Integrated Nursing Process –
White blood cells are produced in the Implementation: Client Needs – Safe Effective Care
bone marrow. The nurse screens visitors Environment, Management of Care: Cognitive
for infection because there are insufficient Level – Application: Concept – Hematologic Regulation.
functioning white blood cells to protect 75. 1. The client should be encouraged to push
the client from developing an infection if fluids to maintain hydration. When the client
exposed to a virus or bacteria. becomes dehydrated, then sickling of the ab-
2. The client should be monitored for infec- normal red blood cells will occur.
tion; one way to do this is to monitor the 2. The client should maintain hydration
temperature. ­status to prevent sickling of the red
3. Fresh fruits and vegetables may have bac- blood cells.
teria on the skins, which could expose the 3. Swelling from sickling cells usually occurs
client to infection. in the joints and into the organs. Measur-
4. The WBC count indicates the presence or ing the calf would be assessing for deep vein
absence of granulocytes, also called neu- thrombosis.
trophils or segmented neutrophils. 4. The client should receive pain medication on
5. The client might be placed on Reverse Iso- a prn basis, not routinely around the clock.
lation or Neutropenic precautions because
the client is at risk from anyone entering the TEST-TAKING HINT: The test taker must be
room; however, there is no reason to suspect aware of the cause of physiological changes
the client is a risk to others, which droplet in the body. The sickling of cells occurs when
precautions is initiated to prevent. the ­client ­experiences lack of oxygen to the
6. Bone marrow biopsies are not performed cells or ­becomes dehydrated.
daily. Content – Medical: Integrated Nursing Process –
Implementation: Client Needs – Safe Effective Care
TEST-TAKING HINT: The test taker must an-
Environment, Management of Care: Cognitive
swer “Select all that apply” questions by Level – Application: ­Concept – Hematologic Regulation.
viewing each option as true/false.
Content – Medical: Integrated Nursing Process – 76. 1. The client is at risk for forming clots,
Implementation: Client Needs – Safe Effective Care which can lead to cerebrovascular
Environment, Management of Care: Cognitive ­accidents (stroke).
Level – Application: Concept – Hematologic Regulation. 2. There is no reason to keep the HOB elevated.
3. A low-sodium diet is for fluid volume
74. 1. Immunity is compromised when the he- ­overload, not dehydration.
matopoietic system is impaired. The bone 4. Antiembolism hose are not needed for sickle
marrow is part of the initial response by cell crisis.
the immune system to prevent disease.
2. Grieving is a possible interrelated concept TEST-TAKING HINT: The test taker must be
because the client must adjust to the diag- aware of the cause of physiological changes
nosis but is not priority over a physiological in the body. The sickling of cells occurs when
problem. the client experiences lack of oxygen to the
3. Perfusion is not a concept normally associ- cells or becomes dehydrated. The results of
ated with lymphoma. clot formation can lead to strokes and other
4. Clotting is not a concept normally associated cerebrovascular accidents.
with lymphoma. Content – Medical: Integrated Nursing Process –
Implementation: Client Needs – Safe Effective Care
TEST-TAKING HINT: The test taker must match
Environment, Management of Care: Cognitive
the problem with the answer option. Options Level – Application:Concept – Hematologic Regulation.
“3” and “4” would probably be implemented
for the client with a bleeding disorder.

176

05_Colgrove_Ch05.indd 176 02/06/16 7:58 PM


C hapter 5  H ematological D isorders 177

77. 1. Night sweats and low-grade fever do occur in discomfort if the cause of the pain is known
Hodgkin’s disease but also occur in diseases and no other complications are occurring.
associated with an HIV infection. Content – Medical: Integrated Nursing Process –
2. Cavitation occurs with tuberculosis. Intervention: Client Needs – Safe Effective Care
3. Reed-Sternberg cells found on biopsy are Environment, Management of Care: Cognitive
diagnostic for Hodgkin’s disease. Level – Application: Concept – Hematologic Regulation.
4. Weight loss and palpable lymph nodes 80. 1. According to Erikson’s growth and
shuld be investigated but are not defini- ­development stages, the 32-year-old
tive in the diagnosis of Hodgkin’s disease. should be in the Intimacy versus Isola-
TEST-TAKING HINT: The test taker must be tion stage. In this stage the client should
aware of the cause of physiological changes have developed a close relationship with
in the body. The test taker should read every another human being, have a job, and be-
word; “diagnostic” means not just signs and come self-supporting. This client is not
symptoms, which could be attributed to sev- meeting Erikson’s expected stage of devel-
eral different diseases, but something that is opment for his age.
only attributed to the disease in question. 2. This client is caring for a child; based on
Content – Medical: Integrated Nursing Process – this statement the client is meeting Erikson’s
Assessment: Client Needs – Safe Effective Care growth and development stages.
Environment, Management of Care: Cognitive 3. This client is caring for a parent and working.
Level – Application: Concept – Hematologic Regulation. Based on this statement the client is meeting
78. 1. Comfort is appropriate for the disease but is Erikson’s growth and development stages.
not the priority concept. Pain alerts the cli- 4. This client is actively involved in preparing
ent that a problem is occurring but is not life for a career and has friends. Based on this
threatening. statement this client is meeting Erikson’s
2. Stress could be a concept but coping growth and development stages.
­problems are priority. TEST-TAKING HINT: The test taker must be
3. Grieving is an interrelated concept but is lim- aware of the cause of psychological changes
ited in the scope of the client’s needs. as growth across the life span occurs. Basic
4. Coping includes dealing with stress, psychology principles are frequently used
anxiety, and grief. The nurse can help the by nurses to assess the client’s status. The
client most by assisting in identifying the test taker has to memorize the stages of
client’s coping mechanisms. development.
TEST-TAKING HINT: The test taker must be Content – Medical: Integrated Nursing Process –
aware of concepts that are interrelated. Assessment: Client Needs – Psychological Integrity/
­Coping encompasses stress and grieving. Management of Care: Cognitive Level – Application:
The word “psychological” can rule out Concept – Hematologic Regulation.
option “1” because comfort is pain. 81. 1. Petechiae indicate a lack of clotting
Content – Medical: Integrated Nursing Process – ­ability caused by decreased production of
Planning: Client Needs – Safe Effective Care Environment, platelets.
Management of Care: Cognitive Level – Application: 2. Pain and joint swelling could indicate several
Concept – Hematologic Regulation. different disease processes.
79. 1. The nurse’s priority is to treat the cause 3. The H&H are low but not indicative of
of the crisis and the pain. leukemia.
2. During a crisis and the administration of nar- 4. A headache and slurred speech could indicate
cotic medication frequent ambulation is not a cerebrovascular accident (CVA) or other
encouraged. disease processes.
3. The client has the ability to oxygenate the TEST-TAKING HINT: The test taker must be
cells; if used then the rate would not be at aware of the cause of physiological changes
10 LPM. in the body in order to answer this option.
4. The client’s RBCs would be monitored but Platelets are produced by the bone marrow
not every four (4) hours. and a deficient number results in bleeding
TEST-TAKING HINT: The test taker must be (small pinpoint areas of bleeding appear on
aware of the cause of physiological changes the body).
in the body. The nurse must treat a client’s Content – Medical: Integrated Nursing Process – ­
Assessment: Client Needs – Safe Effective Care

05_Colgrove_Ch05.indd 177 02/06/16 7:58 PM


178 M ed -S urg S uccess

Environment, Management of Care: Cognitive Level – TEST-TAKING HINT: The test taker must con-
Application: Concept – Hematologic Regulation. sider each option in a “Select all that apply”
82. 1. Coping could be applied to any diagnosis question as a true/false option. The lympho-
of a cancer. cytes are produced on the bone marrow; the
2. Hematologic regulation is the top prior- test taker has to remember pathophysiology
ity identified for a client with lymphoma of the disease process.
because the client has a problem with the Content – Medical: Integrated Nursing Process –
cells produced by the bone marrow. Planning: Client Needs – Safe Effective Care Environment,
3. Tissue perfusion is not associated with Management of Care: Cognitive Level – Application:
Concept – Hematologic Regulation.
lymphoma.
4. Clotting is not associated with lymphoma.
5. Clinical judgment is a concept for the nurse,
not the client.

05_Colgrove_Ch05.indd 178 02/06/16 7:58 PM


C hapter 5  H ematological D isorders 179

HEMATOLOGICAL DISORDERS
COMPREHENSIVE EXAMINATION
1. The client is diagnosed with severe iron- 6. The client diagnosed with cancer has been
deficiency anemia. Which statement is the undergoing systemic treatments and has red
scientific rationale regarding oral replacement blood cell deficiency. Which signs and symptoms
therapy? should the nurse teach the client to manage?
1. Iron supplements are well tolerated without 1. Nausea associated with cancer treatment.
side effects. 2. Shortness of breath and fatigue.
2. There is no benefit from oral preparations; the 3. Controlling mucositis and diarrhea.
best route is IV. 4. The emotional aspects of having cancer.
3. Oral iron preparations cause diarrhea if not
7. The nurse is assisting the HCP with a
taken with food.
bone marrow biopsy. Which intervention
4. Very little of the iron supplement will be
postprocedure has priority?
absorbed by the body.
1. Apply pressure to site for five (5) to
2. The client’s laboratory values are RBCs 10 minutes.
5.5 (×106)/mm3, WBCs 8.9 (×103)/mm3, and 2. Medicate for pain with morphine slow IVP.
platelets 189 (×103)/mm3. Which intervention 3. Maintain head of bed in high Fowler’s
should the nurse implement? position.
1. Prepare to administer packed red blood cells. 4. Apply oxygen via nasal cannula at five
2. Continue to monitor the client. (5) L/min.
3. Request an order for Neupogen, a biologic
8. The client diagnosed with end-stage renal
response modifier.
disease (ESRD) has developed anemia. Which
4. Institute bleeding precautions.
would the nurse anticipate the HCP prescribing
3. The client diagnosed with anemia is admitted to for this client?
the emergency department with dyspnea, cool 1. Place the client in reverse isolation.
pale skin, and diaphoresis. Which assessment data 2. Discontinue treatments until blood count
warrant immediate intervention? improves.
1. The vital signs are T 98.6° F, P 116, R 28, and 3. Monitor CBC daily to assess for bleeding.
BP 88/62. 4. Give client erythropoietin, a biologic response
2. The client is allergic to multiple antibiotic modifier.
medications.
9. The nurse is planning the care of a client
3. The client has a history of receiving
diagnosed with aplastic anemia. Which
chemotherapy.
interventions should be taught to the client?
4. ABGs are pH 7.35, Pco2 44, Hco3 22,
Select all that apply.
Pao2 92.
1. Avoid alcohol.
4. The client diagnosed with anemia has an Hb of 2. Pace activities.
6.1 g/dL. Which complication should the nurse 3. Stop smoking.
assess for? 4. Eat a balanced diet.
1. Decreased pulmonary functioning. 5. Use a safety razor.
2. Impaired muscle functioning.
10. The nurse is caring for a client in a sickle cell
3. Congestive heart failure.
crisis. Which is the pain regimen of choice to
4. Altered gastric secretions.
relieve the pain?
5. The nurse writes a diagnosis of “activity 1. Frequent aspirin (acetylsalicylic acid) and a
intolerance” for a client diagnosed with anemia. nonnarcotic analgesic.
Which intervention should the nurse implement? 2. Motrin (ibuprofen), a nonsteroidal anti-
1. Encourage isometric exercises. inflammatory drug (NSAID), prn.
2. Assist the client with activities of daily living 3. Demerol (meperidine), a narcotic analgesic,
(ADLs). every four (4) hours.
3. Provide a high-protein diet. 4. Morphine, a narcotic analgesic, every two (2)
4. Refer to the physical therapist. to three (3) hours prn.

05_Colgrove_Ch05.indd 179 02/06/16 7:58 PM


180 M ed -S urg S uccess

11. The client is diagnosed with hereditary 17. The client is placed on neutropenia precautions.
spherocytosis. Which treatment/procedure Which information should the nurse teach the
would the nurse prepare the client to receive? client?
1. Bone marrow transplant. 1. Shave with an electric razor and use a soft
2. Splenectomy. toothbrush.
3. Frequent blood transfusions. 2. Eat plenty of fresh fruits and vegetables.
4. Liver biopsy. 3. Perform perineal care after every bowel
movement.
12. Which is the primary goal of care for a client
4. Some blood in the urine is not unusual.
diagnosed with sickle cell anemia?
1. The client will call the HCP if feeling ill. 18. The client is diagnosed with chronic myeloid
2. The client will be compliant with medical leukemia and leukocytosis. Which signs/
regimen. symptoms would the nurse expect to find when
3. The client will live as normal a life as possible. assessing this client?
4. The client will verbalize understanding of 1. Frothy sputum and jugular vein distention.
treatments. 2. Dyspnea and slight confusion.
3. Right upper quadrant tenderness and nausea.
13. The client diagnosed with thalassemia, a
4. Increased appetite and weight gain.
hereditary anemia, is to receive a transfusion
of packed RBCs. The crossmatch reveals 19. The client’s CBC indicates an RBC
the presence of antibodies that cannot be 6 (× 106)/mm3, Hb 14.2 g/dL, Hct 42%, and
crossmatched. Which precaution should platelets 69 (× 103)/mm3. Which intervention
the nurse implement when initiating the should the nurse implement?
transfusion? 1. Teach the client to use a soft-bristle
1. Start the transfusion at 10 to 15 mL/hr for 15 toothbrush.
to 30 minutes. 2. Monitor the client for elevated temperature.
2. Re-crossmatch the blood until the antibodies 3. Check the client’s blood pressure.
are identified. 4. Hold venipuncture sites for one (1) minute.
3. Have the client sign a permit to receive
20. The 24-year-old female client is diagnosed with
uncrossmatched blood.
idiopathic thrombocytopenic purpura (ITP).
4. Have the unlicensed assistive personnel stay
Which question would be important for the
with the client.
nurse to ask during the admission interview?
14. The client is diagnosed with polycythemia vera. 1. “Do you become short of breath during
The nurse would prepare to perform which activity?”
intervention? 2. “How heavy are your menstrual periods?”
1. Type and crossmatch for a transfusion. 3. “Do you have a history of deep vein
2. Assess for petechiae and purpura. thrombosis?”
3. Perform phlebotomy of 500 mL of blood. 4. “How often do you have migraine
4. Monitor for low hemoglobin and hematocrit. headaches?”
15. The client diagnosed with leukemia has received 21. The client is diagnosed with hemophilia. Which
a bone marrow transplant. The nurse monitors safety precaution should the nurse encourage?
the client’s absolute neutrophil count (ANC). 1. Wear helmets and pads during contact sports.
What is the client’s neutrophil count if the 2. Take antibiotics prior to any dental work.
WBCs are 2.2 (× 103)/mm3, neutrophils are 3. Keep clotting factor VIII on hand at all times.
25%, and bands are 5%? ____________ 4. Use ibuprofen, an NSAID, for mild pain.
16. The client is diagnosed with leukemia and has
leukocytosis. Which laboratory value would the
nurse expect to assess?
1. An elevated hemoglobin.
2. A decreased sedimentation rate.
3. A decreased red cell distribution width.
4. An elevated white blood cell count.

05_Colgrove_Ch05.indd 180 02/06/16 7:58 PM


C hapter 5  H ematological D isorders 181

22. The nurse writes a diagnosis of “potential for 25. The HCP has ordered one (1) unit of packed
fluid volume deficit related to bleeding” for a RBCs for the client who is right-handed. Which
client diagnosed with disseminated intravascular area would be the best place to insert the
coagulation (DIC). Which would be an intravenous catheter?
appropriate goal for this client?
1. The client’s clot formations will resolve in two
(2) days.
2. The saturation of the client’s dressings will be
documented.
3. The client will use lemon-glycerin swabs for
oral care.
4. The client’s urine output will be greater than
30 mL per hour.
23. The client diagnosed with atrial fibrillation is
B
admitted with warfarin (Coumadin) toxicity.
Which HCP order would the nurse anticipate? C
1. Protamine sulfate, an anticoagulant antidote.
2. Heparin sodium, an anticoagulant. A
3. Lovenox, a low molecular weight D
anticoagulant.
4. Vitamin K, an anticoagulant agonist.
24. Fifteen minutes after the nurse has initiated a 1. A
transfusion of packed red blood cells, the client 2. B
becomes restless and complains of itching on the 3. C
trunk and arms. Which intervention should the 4. D
nurse implement first?
26. The nurse is administering a transfusion of
1. Collect urine for analysis.
packed red blood cells to a client. Which
2. Notify the laboratory of the reaction.
interventions should the nurse implement? List
3. Administer diphenhydramine, an
in order of performance.
antihistamine.
1. Start the transfusion slowly.
4. Stop the transfusion at the hub.
2. Have the client sign a permit.
3. Assess the IV site for size and patency.
4. Check the blood with another nurse at the
bedside.
5. Obtain the blood from the laboratory.

05_Colgrove_Ch05.indd 181 02/06/16 7:58 PM


182 M ed -S urg S uccess

27. The charge nurse is reviewing the laboratory values on clients on a medical floor. Which laboratory data
should be reported to the health-care provider (HCP) immediately?
1.

Complete Blood Count Patient Value Normal Values


RBCs (× 106) 4.7 Male: 4.7–6.1 (× 106)/mm3
Female: 4.2–5.4 (× 106)/mm3
Hemoglobin 12.8 Male: 13.5–17.5 g/dL
Female: 11.5–15.5 g/dL
Hematocrit 35 Male: 40%–52%
Female: 36%–48%
Platelets (× 103) 39 150–400 (× 103)/mm3
3
WBCs (× 10 ) 10.8 4.5–11 (× 103)/mm3

2.

Complete Blood Count Patient Value Normal Values


6
RBCs (× 10 ) 4.2 Male: 4.7–6.1 (× 106)/mm3
Female: 4.2–5.4 (× 106)/mm3
Hemoglobin 10.6 Male: 13.5–17.5 g/dL
Female: 11.5–15.5 g/dL
Hematocrit 30.4 Male: 40%–52%
Female: 36%–48%
Platelets (× 103) 148 150–400 (× 103)/mm3
WBCs (× 103) 4.3 4.5–11 (× 103)/mm3

3.

Complete Blood Count Patient Value Normal Values


6
RBCs (× 10 ) 7.2 Male: 4.7–6.1 (× 106)/mm3
Female: 4.2–5.4 (× 106)/mm3
Hemoglobin 18.9 Male: 13.5–17.5 g/dL
Female: 11.5–15.5 g/dL
Hematocrit 56 Male: 40%–52%
Female: 36%–48%
Platelets (× 103) 125 150–400 (× 103)/mm3
3
WBCs (× 10 ) 4.3 4.5–11 (× 103)/mm3

4.

Complete Blood Count Patient Value Normal Values


6
RBCs (× 10 ) 5.5 Male: 4.7–6.1 (× 106)/mm3
Female: 4.2–5.4 (× 106)/mm3
Hemoglobin 15 Male: 13.5–17.5 g/dL
Female: 11.5–15.5 g/dL
Hematocrit 45 Male: 40%–52%
Female: 36%–48%
3
Platelets (× 10 ) 200 150–400 (× 103)/mm3
3
WBCs (× 10 ) 6.3 4.5–11 (× 103)/mm3

05_Colgrove_Ch05.indd 182 02/06/16 7:58 PM


HEMATOLOGICAL DISORDERS COMPREHENSIVE
EXAMINATION ANSWERS AND RATIONALES

1. 1. Iron supplements can be poorly tolerated. The Management of Care: Cognitive Level – Synthesis:
supplements can cause nausea, abdominal dis- Concept – Hematologic Regulation.
comfort, and constipation. 4. 1. A low hemoglobin level will not decrease pul-
2. There is benefit from oral preparations, but in monary functioning. In fact, the lungs will try
severe cases of iron deficiency, the client may to compensate for the anemia by speeding up
need parenteral replacement. respirations to oxygenate the red blood cells
3. Iron supplements cause constipation and should and provide oxygen to the tissues.
be taken one (1) hour before a meal or two (2) 2. The client may have difficulty with activity in-
hours after a meal for the best absorption of the tolerance, but this will be from lack of oxygen,
medication. As much as 50% of the iron is not not lack of ability of the muscles to function.
absorbed when taken with food. 3. General complications of severe anemia
4. At best only about 20% to 35% of the include heart failure, paresthesias, and con-
medication is absorbed through the fusion. The heart tries to compensate for
­gastrointestinal (GI) tract. the lack of oxygen in the tissues by becom-
Content – Medical: Integrated Nursing Process – ing tachycardic. The heart will be able to
Diagnosis: Client Needs – Physiological Integrity, maintain this compensatory mechanism for
­Reduction of Risk Potential: Cognitive Level – ­Analysis: only so long and then will show evidence of
Concept – Hematologic Regulation.
failure.
2. 1. The normal RBC is 4.7 to 6.1 (× 106) for 4. The gastric secretions will not be altered. The
males and 4.2 to 5.4 (× 106) for females. The blood supply to the stomach may be shunted
RBC is within normal limits. to the more vital organs, leaving the acidic
2. All the laboratory values are within normal stomach to deal with the production of acid,
limits. The nurse should continue to moni- and this could cause the client to develop a
tor the client. gastric ulcer.
3. The normal WBC is 4.5 to 11 (× 103), so a Content – Medical: Integrated Nursing Process –
biologic response modifier to increase the Assessment: Client Needs – Physiological Integrity,
numbers of WBCs is not needed. Reduction of Risk Potential: Cognitive Level –
4. Thrombocytopenia does not occur until the Analysis: Concept – Hematologic Regulation.
client’s platelet count is less than 100 (× 103); 5. 1. Isometric exercises are bodybuilding exercises;
there is no reason to institute bleeding these types of exercises would deplete the cli-
precautions. ent’s energy stores.
Content – Medical: Integrated Nursing Process – 2. The client with activity intolerance will
Implementation: Client Needs – Safe Effective Care need assistance to perform activities of
Environment, Management of Care: Cognitive Level – daily living.
Application: Concept – Hematologic Regulation.
3. A high-protein diet may be needed, but this
3. 1. The pulse of 116 and BP of 88/62 in does not address activity intolerance.
­addition to the other symptoms indicate 4. Activity intolerance is not a problem for a
the client is in shock. This is an emergency physical therapist. The client’s blood counts
situation. must increase to increase the ability to perform
2. The client is in shock; allergies to medications activities.
are not the most important thing. Content – Medical: Integrated Nursing Process –
3. This may be the cause of the anemia, and Implementation: Client Needs – Physiological ­Integrity,
complications from the chemotherapy may be Physiological Adaptation: Cognitive Level – Application:
causing the shock, but the priority intervention Concept – Hematologic Regulation.
is to treat the shock, regardless of the cause. 6. 1. Red blood cell deficiency is anemia. The ­client
4. These are normal blood gases, so no immedi- should be taught to deal with the ­effects of
ate intervention regarding arterial blood gases anemia, not nausea.
is needed. 2. Anemia causes the client to experience
Content – Medical: Integrated Nursing Process – dyspnea and fatigue. Teaching the client
Assessment: Client Needs – Safe Effective Care Environment, to pace activities and rest often, to eat a
183

05_Colgrove_Ch05.indd 183 02/06/16 7:58 PM


184 M ed -S urg S uccess

balanced diet, and to cope with changes in 3. Although all clients should be told to stop
lifestyle is needed. smoking, smoking will not directly affect the
3. Mucositis and diarrhea may occur with chemo- client’s diagnosis.
therapy administration, but this client’s prob- 4. The client should eat a well-balanced diet
lem is anemia. to be able to manufacture blood cells.
4. All clients with cancer should be assisted to 5. The client should use an electric razor to
discuss the impact of cancer on their lives, but diminish the risk of cuts and bleeding.
this client’s problem is anemia. Content – Medical: Integrated Nursing Process –
Content – Medical: Integrated Nursing Process – Planning: Client Needs – Physiological Integrity, Physiologi-
Planning: Client Needs – Physiological Integrity, cal Adaptation: Cognitive Level – Synthesis: Concept –
Physiological Adaptation: Cognitive Level – Synthesis: Hematologic Regulation.
Concept – Hematologic Regulation.
10. 1. Aspirin is an option for mild pain, but it
7. 1. After a bone marrow biopsy, it is important would not be strong enough during a crisis.
that the client form a clot to prevent bleed- 2. NSAIDs are helpful because they relieve
ing. The nurse should hold direct pressure pain and decrease inflammation, but they are
on the site for five (5) to 10 minutes. not used during a crisis because they are not
2. The nurse might premedicate for pain, but strong enough.
once the procedure is completed, a mild oral 3. Demerol breaks down in the body into
medication is usually sufficient to relieve any normeperidine, which can cause seizures in
residual discomfort. large doses.
3. The head of the bed can be in any position of 4. Morphine is the drug of choice for a crisis;
comfort for the client. it does not have a ceiling effect and can
4. The procedure is performed on the iliac crest be given in large amounts and frequent
or the sternum and does not cause respiratory doses.
distress. Content – Medical: Integrated Nursing Process –
Content – Surgical: Integrated Nursing Process – Diagnosis: Client Needs – Physiological Integrity,
Implementation: Client Needs – Safe Effective Care Physiological Adaptation: Cognitive Level –
Environment, Management of Care: Cognitive Knowledge: Concept – Medication.
Level – Synthesis: Concept – Perioperative.
11. 1. A bone marrow transplant is a treatment op-
8. 1. The client is anemic, not neutropenic, so tion for aplastic anemia and some clients with
r­everse isolation is not needed. thalassemia, but not for spherocytosis.
2. This client will be receiving a form of dialysis 2. Hereditary spherocytosis is a relatively
to maintain life. Discontinuing treatments un- common hemolytic anemia (1:5,000
til the counts improve would be resigning the people) characterized by an abnormal
client to death. permeability of the red blood cell, which
3. The red blood cell count would not apprecia- permits it to become spherical in shape.
bly improve from one day to the next unless a The spheres are then destroyed by the
transfusion had been given. spleen. A splenectomy is the treatment of
4. Erythropoietin is a biologic response modi- choice.
fier produced by the kidneys in ­response 3. The client may require blood transfusions,
to a low red blood cell count in the body. but preventing the destruction of RBCs by
It stimulates the body to produce more the spleen is the better option.
RBCs. 4. A liver biopsy will not affect the client’s
Content – Medical: Integrated Nursing Process – condition.
Planning: Client Needs – Physiological Integrity, Content – Surgical: Integrated Nursing Process –
Reduction of Risk Potential: Cognitive Level – Planning: Client Needs – Physiological Integrity,
Synthesis: Concept – Hematologic Regulation. Reduction of Risk Potential: Cognitive Level –
Synthesis: Concept – Hematologic Regulation.
9. 1. Alcohol consumption interferes with the
absorption of nutrients. 12. 1. All clients should know when to notify the
2. The client will be short of breath with HCP. This is not the primary goal of living
activity and therefore should pace with a chronic illness.
activities. 2. The client should be compliant with recom-
mendations, but this is a lifestyle choice and
not the primary goal.

05_Colgrove_Ch05.indd 184 02/06/16 7:58 PM


C hapter 5  H ematological D isorders 185

3. The primary goal for any client coping is used to determine a client’s risk of develop-
with a chronic illness is that the client ing an infection.
will be able to maintain as normal a life as Content – Medical: Integrated Nursing Process –
possible. Implementation: Client Needs – Safe Effective Care
4. This is a goal for a client with knowledge- Environment, Management of Care: Cognitive Level –
deficit problems. Application: Concept – Hematologic Regulation.
Content – Medical: Integrated Nursing Process – 16. 1. Clients with leukemia are usually anemic
Planning: Client Needs – Physiological Integrity, because of the bone marrow’s inability to
Physiological Adaptation: Cognitive Level – Synthesis:
produce cells or, in this case, the bone mar-
Concept – Hematologic Regulation.
row, stuck in high gear, producing immature
13. 1. It can be difficult to crossmatch blood white blood cells that are unable to function
when antibodies are present. If imper- normally.
fectly crossmatched blood must be trans- 2. Sedimentation rates increase in some diseases,
fused, the nurse must start the blood very such as rheumatoid arthritis, but there will be
slowly and stay with the ­client, monitor- no change in leukemia.
ing frequently for signs of a ­hemolytic 3. The red cell distribution width (RDW),
reaction. shown in the CBC report, reports on the size
2. The antibodies have been identified. The of the red blood cells.
donor blood does not crossmatch perfectly to 4. An elevated white blood cell count is
the blood of the client. what is being described in the term
3. The blood has been crossmatched. Client “­leukocytosis”—leuko- means “white” and
permits regarding uncrossmatched blood are cyto- refers to “cell.” Leukocytosis is the
used for emergency transfusions when time opposite of leukopenia.
does not allow an attempt to crossmatch. In Content – Medical: Integrated Nursing Process –
such a case, O– blood, the universal donor, Assessment: Client Needs – Physiological Integrity,
will be used. Reduction of Risk Potential: Cognitive Level –
4. The nurse cannot delegate an unstable Analysis: Concept – Hematologic Regulation.
­client to an unlicensed assistive personnel. 17. 1. Shaving with an electric razor and ­using
The nurse must stay with the client.
a soft toothbrush are interventions for
Content – Medical: Integrated Nursing Process – thrombocytopenia.
Implementation: Client Needs – Safe Effective Care 2. Fresh fruits and vegetables are limited
Environment, Management of Care: Cognitive
­because they may harbor microbes.
Level – Application: Concept – Hematologic Regulation.
3. Perineal care after each bowel movement,
14. 1. The client has too many red blood cells and preferably with an antimicrobial soap, is
does not need more. performed to reduce bacteria on the skin.
2. Petechiae and purpura occur when a client 4. Blood in the urine would indicate a
does not have adequate platelets. ­complication and is not expected.
3. The client has too many red blood cells, Content – Medical: Integrated Nursing Process –
which can cause as much damage as too Planning: Client Needs – Physiological Integrity,
few. The treatment for this disease is to Physiological Adaptation: Cognitive Level – Synthesis:
remove the excess blood; 500 mL at a time Concept – Hematologic Regulation.
is removed. 18. 1. Frothy sputum and jugular vein distention are
4. The client’s hemoglobin and hematocrit are symptoms of heart failure, which could occur
high, not low. as a complication of anemia.
Content – Medical: Integrated Nursing Process – 2. Clients with leukocytosis may be short of
Implementation: Client Needs – Safe Effective Care breath and somewhat confused as a result
Environment, Management of Care: Cognitive Level –
of decreased capillary perfusion to the
Synthesis: Concept – Hematologic Regulation.
lung and brain from excessive amounts of
15. 660 ANC. WBCs inhibiting blood flow through the
To determine the absolute neutrophil count, capillaries.
first the WBC count must be determined: 3. The client may have left upper quadrant pain
2.2 multiplied by 1,000 (103) = 2,200. and tenderness from WBC infiltration of the
­Multiply that by 30 (25% neutrophils + 5% spleen, but the client will not have right up-
bands) to obtain 66,000 and divide that by per quadrant tenderness.
100 to determine the ANC of 660. The ANC 4. The client may be anorectic and lose weight.

05_Colgrove_Ch05.indd 185 02/06/16 7:58 PM


186 M ed -S urg S uccess

Content – Medical: Integrated Nursing Process – 4. The problem is addressing the potential
Assessment: Client Needs – Physiological Integrity, for hemorrhage, and a urine output of
­Reduction of Risk Potential: Cognitive Level – ­Analysis: greater than 30 mL/hr indicates the
Concept – Hematologic Regulation. ­kidneys are being adequately perfused and
19. 1. The client has a low platelet count the body is not in shock.
(thrombocytopenia) and should be on Content – Medical: Integrated Nursing Process –
bleeding precautions, such as using a soft- Planning: Client Needs – Physiological Integrity,
bristle toothbrush. Physiological Adaptation: Cognitive Level – Synthesis:
2. Monitoring for a fever, a sign of infection, Concept – Clotting.
would be an intervention for low WBCs. 23. 1. Protamine sulfate is the antidote for a heparin
3. Assessing the blood pressure is not ­indicated overdose, not for warfarin toxicity.
for any of the blood cell count abnormalities. 2. Administering heparin would increase the
4. Holding the venipuncture site would be done ­client’s risk of bleeding.
for a minimum of five (5) minutes. 3. Lovenox is a low molecular weight hepa-
Content – Medical: Integrated Nursing Process – rin and would increase the client’s risk of
Implementation: Client Needs – Safe Effective Care bleeding.
Environment, Management of Care: Cognitive 4. The antidote for warfarin (Coumadin) is
Level – Application: Concept – Hematologic Regulation. vitamin K, an anticoagulant agonist.
20. 1. Thrombocytopenia is low platelets and would Content – Medical: Integrated Nursing Process –
not cause shortness of breath. Planning: Client Needs – Physiological Integrity,
2. Because thrombocytopenia causes bleed- Reduction of Risk Potential: Cognitive Level –
ing, the nurse should assess for any type Synthesis: Concept – Hematologic Regulation.
of bleeding that may be occurring. A 24. 1. The question asks for a first action; urine will
young female client would present with be collected for analysis, but this is not the
excessive menstrual bleeding. first intervention.
3. The problems associated with ITP are 2. The question asks for a first action. The labo-
­bleeding, not clotting. ratory should be notified, but this is not the
4. ITP does not cause migraine headaches. first action to take.
Content – Medical: Integrated Nursing Process – 3. Administering an antihistamine may be done,
Assessment: Client Needs – Physiological Integrity, but it is more important to stop the transfu-
Reduction of Risk Potential: Cognitive Level – sion causing the reaction.
Analysis: Concept – Hematologic Regulation. 4. Anytime the nurse suspects the client is
21. 1. The client should not participate in contact having a reaction to blood or blood prod-
sports because even minimal injury can cause ucts, the nurse should stop the infusion at
massive bleeding. the spot closest to the client and not allow
2. The client should have factor VIII on hand any more of the blood to enter the client’s
for any dental procedure to make sure that he body.
or she clots, but antibiotics are not needed. Content – Medical: Integrated Nursing Process –
3. The client must have the clotting fac- Implementation: Client Needs – Safe Effective Care
tor on hand in case of injury to prevent Environment, Management of Care: Cognitive Level –
­massive bleeding. Synthesis: Concept – Hematologic Regulation.
4. NSAIDs prolong bleeding and should be 25. 1. The client is right-handed; therefore, the
avoided. nurse should attempt to place the IV catheter
Content – Medical: Integrated Nursing Process – in the nondominant arm.
Planning: Client Needs – Safe Effective Care 2. The antecubital area should not be used as an
Environment, Management of Care: Cognitive Level – IV site because the movement of the elbow
Synthesis: Concept – Hematologic Regulation. will crimp the cannula and it is uncomfort-
22. 1. The body dissolves clots over a period of sev- able for the client to keep the arm straight all
eral days to a week or more. The problem in the time.
DIC is that the body is bleeding and clotting 3. The left forearm is the best site to start
simultaneously. the IV because it has larger veins that will
2. This is a nursing goal, not a client goal. accommodate an 18-gauge catheter, which
3. Lemon-glycerin swabs are drying to the should be used when administering blood.
­mucosa and should be avoided because they This area is less likely to have extravasa-
will increase bleeding from the mucosa. tion because there is no joint movement,

05_Colgrove_Ch05.indd 186 02/06/16 7:58 PM


C hapter 5  H ematological D isorders 187

and this site is on the client’s nondomi- 1. After all of the previous steps are com-
nant side. pleted, then the nurse should start the
4. The hand area usually has smaller veins that infusion of the blood slowly for the first
do not accommodate a larger gauge catheter, 15 minutes to determine if the client is
and hand movement can cause extravasation going to have a reaction.
more readily than forearm movement. Content – Medical: Integrated Nursing Process –
Content – Medical: Integrated Nursing Process – Implementation: Client Needs – Safe Effective Care Envi-
Implementation: Client Needs – Safe Effective Care ronment, Management of Care: Cognitive Level –
Environment, Management of Care: Cognitive Level – Analysis: Concept – Hematologic Regulation.
Knowledge: Concept – Hematologic Regulation.
27. 1. This client has a critically low ­platelet
26. In order of performance: 2, 3, 5, 4, 1. count, even though the other labora-
2. The client must give consent prior to re- tory values are in a normal or near-­
ceiving blood; therefore, this is the first normal range. This client is at risk for
intervention. hemorrhaging.
3. Blood products should be administered 2. This client has low values but not danger-
within 30 minutes of obtaining the blood ously so. The HCP can be shown the results
from the laboratory; therefore, the nurse on rounds.
should determine the IV is patent and the 3. This client’s laboratory values are high, but
catheter is large enough to administer not dangerously so; these can be shown to the
blood, preferably an 18-gauge catheter, HCP on rounds.
before obtaining the blood. 4. This client has normal laboratory values.
5. The nurse must then obtain the blood Content – Medical: Integrated Nursing Process –
from the laboratory. Assessment: Client Needs – Physiological Integrity,
4. Blood must be checked by two (2) regis- Physiological Adaptation: Cognitive Level – Synthesis:
tered nurses at the bedside to check the Concept – Hematologic Regulation.
client’s crossmatch bracelet with the unit
of blood.

05_Colgrove_Ch05.indd 187 02/06/16 7:58 PM


05_Colgrove_Ch05.indd 188 02/06/16 7:58 PM
Respiratory
Disorders
Any piece of knowledge I acquire today has a value at this
moment exactly proportional to my skill to deal with it.
6
Tomorrow when I know more, I recall that piece of knowledge
and use it better.
—Mark Van Doren

Respiratory disorders include some of the most common disorders that nurses and other
health-care providers encounter. They range from the simple cold to chronic conditions
such as asthma to life-threatening diseases such as lung cancer. Whether it is teaching a
­client the correct way to blow the nose; administering oxygen, antibiotics, or pain-relieving
medications; or checking ventilators, chest tubes, and other equipment used in the treat-
ment of clients with respiratory disorders, the nurse must know the correct procedures,
medications, and interventions to use. One of the most important interventions—and per-
haps, the most important—is assessment and monitoring of the client’s breathing status and
­oxygenation level. Oxygenation is the first priority in Maslow’s hierarchy—and in terms of
saving lives.

KEYWORDS ABBREVIATIONS
Adenoidectomy Acute respiratory distress syndrome (ARDS)
Ambu Arterial blood gases (ABG)
Antral Blood pressure (BP)
Aphonia Capillary refill time (CRT)
Bronchoscopy Chronic obstructive pulmonary disease (COPD)
Caldwell-Luc Continuous positive airway pressure (CPAP)
Dorsiflexion Directly observed treatment (DOT)
Enteral Electrocardiogram (ECG)
Eupnea Endotracheal tube (ET)
Exacerbation Exercise-induced asthma (EIA)
Extrinsic Health-care provider (HCP)
Hypoxemia Hemoglobin and hematocrit (H&H)
Iatrogenic International normalized ratio (INR)
Intrinsic Intravenous (IV)
Laryngectomy Intravenous piggyback (IVPB)
Laryngoscopy Intravenous push (IVP)
Lobectomy Licensed practical nurse (LPN)
Pleurodesis Liters per minute (LPM)
Pneumonectomy Magnetic resonance imaging (MRI)
Polysomnography Nothing by mouth (NPO)
Rhinitis Partial thromboplastin time (PTT)
Rhinorrhea Percutaneous endoscopic gastrostomy (PEG) tube

189

06_Colgrove_Ch06.indd 189 01/06/16 5:16 PM


190 M ed -S urg S uccess

Rhonchi Prothrombin time (PT)


Thoracentesis Rule out (R/O)
Thoracotomy Severe acute respiratory syndrome (SARS)
Shortness of breath (SOB)
Total parenteral nutrition (TPN)
Unlicensed assistive personnel (UAP)
Upper respiratory infection (URI)
When required; as needed (prn)

PRACTICE QUESTIONS
Upper Respiratory Infection (URI) 4. The client has been diagnosed with chronic
sinusitis. Which sign/symptom alerts the nurse to
1. The home health-care nurse is talking on the a potentially life-threatening complication?
telephone to a male client diagnosed with 1. Muscle weakness.
hypertension and hears the client sneezing. The 2. Purulent sputum.
client tells the nurse he has been blowing his nose 3. Nuchal rigidity.
frequently. Which question should the nurse ask 4. Intermittent loss of muscle control.
the client? 5. The client diagnosed with tonsillitis is scheduled
1. “Have you had the flu shot in the last two (2) to have surgery in the morning. Which assessment
weeks?” data should the nurse notify the health-care
2. “Are there any small children in the home?” provider about prior to surgery?
3. “Are you taking over-the-counter medicine for 1. The client has a hemoglobin of 12.2 g/dL and
these symptoms?” hematocrit of 36.5%.
4. “Do you have any cold sores associated with 2. The client has an oral temperature of 100.2˚F
your sneezing?” and a dry cough.
2. The school nurse is presenting a class to students 3. There are one (1) to two (2) white blood cells
at a primary school on how to prevent the (WBCs) in the urinalysis.
transmission of the common cold virus. Which 4. The client’s current international normalized
information should the nurse discuss? ratio (INR) is 1.
1. Instruct the children to always keep a tissue or 6. The influenza vaccine is in short supply. Which
handkerchief with them. group of clients would the public health nurse
2. Explain that children current with consider priority when administering the vaccine?
immunizations will not get a cold. 1. Elderly and chronically ill clients.
3. Tell the children they should go to the doctor if 2. Child-care workers and children less than four
they get a cold. (4) years of age.
4. Demonstrate to the students how to wash 3. Hospital chaplains and health-care workers.
hands correctly. 4. Schoolteachers and students living in a
3. Which information should the nurse teach the dormitory.
client diagnosed with acute sinusitis? 7. The client diagnosed with chronic sinusitis who
1. Instruct the client to complete all the ordered has undergone a Caldwell-Luc procedure is
antibiotics. complaining of pain. Which intervention should
2. Teach the client how to irrigate the nasal the nurse implement first?
passages. 1. Administer the narcotic analgesic intravenous
3. Have the client demonstrate how to blow the push (IVP).
nose. 2. Perform gentle oral hygiene.
4. Give the client samples of a narcotic analgesic 3. Place the client in semi-Fowler’s position.
for the headache. 4. Assess the client’s pain.

06_Colgrove_Ch06.indd 190 01/06/16 5:16 PM


C hapter 6  R espiratory D isorders 191

8. The charge nurse on a surgical floor is 12. The nurse is caring for a client diagnosed with
making assignments. Which client should be a cold. Which is an example of an alternative
assigned to the most experienced registered therapy?
nurse (RN)? 1. Vitamin C, 2,000 mg daily.
1. The 36-year-old client who has undergone an 2. Strict bedrest.
antral irrigation for sinusitis yesterday and has 3. Humidification of the air.
moderate pain. 4. Decongestant therapy.
2. The six-(6)-year-old client scheduled
for a tonsillectomy and adenoidectomy
this morning who will not swallow Lower Respiratory Infection
medication.
3. The 18-year-old client who had a Caldwell- 13. The nurse is assessing a 79-year-old client
Luc procedure three (3) days ago and has diagnosed with pneumonia. Which signs and
purulent drainage on the drip pad. symptoms should the nurse expect to assess in
4. The 45-year-old client diagnosed with a the client?
peritonsillar abscess who requires VPB 1. Confusion and lethargy.
antibiotic therapy four (4) times a day. 2. High fever and chills.
3. Frothy sputum and edema.
9. The client diagnosed with influenza A is being 4. Bradypnea and jugular vein distention.
discharged from the emergency department with
a prescription for antibiotics. Which statement 14. The nurse is planning the care of a client
by the client indicates an understanding of this diagnosed with pneumonia and writes a problem
prescription? of “impaired gas exchange.” Which is an
1. “These pills will make me feel better fast and expected outcome for this problem?
I can return to work.” 1. Performs chest physiotherapy three (3) times
2. “The antibiotics will help prevent me from a day.
developing a bacterial pneumonia.” 2. Able to complete activities of daily living.
3. “If I had gotten this prescription sooner, 3. Ambulates in the hall several times during
I could have prevented this illness.” each shift.
4. “I need to take these pills until I feel better; 4. Alert and oriented to person, place, time, and
then I can stop taking the rest.” events.

10. The nurse is developing a plan of care for a 15. The nurse in a long-term care facility is planning
client diagnosed with laryngitis and identifies the care for a client with a percutaneous
the client problem “altered communication.” endoscopic gastrostomy (PEG) feeding tube
Which intervention should the nurse used for bolus feedings. Which intervention
implement? should the nurse include in the plan of care?
1. Instruct the client to drink a mixture of 1. Inspect the insertion line at the naris prior to
brandy and honey several times a day. instilling formula.
2. Encourage the client to whisper instead of 2. Elevate the head of the bed (HOB) after
trying to speak at a normal level. feeding the client.
3. Provide the client with a blank note pad for 3. Place the client in the Sims position following
writing any communication. each feeding.
4. Explain that the client’s aphonia may become 4. Change the dressing on the feeding tube
a permanent condition. every three (3) days.

11. Which task is most appropriate for the nurse 16. The client diagnosed with a community-
to delegate to an unlicensed assistive personnel acquired pneumonia is being admitted to the
(UAP)? medical unit. Which nursing intervention has
1. Feed a client who is postoperative the highest priority?
tonsillectomy the first meal of clear liquids. 1. Administer the ordered oral antibiotic
2. Encourage the client diagnosed with a cold to immediately (STAT).
drink a glass of orange juice. 2. Order the meal tray to be delivered as soon as
3. Obtain a throat culture on a client diagnosed possible.
with bacterial pharyngitis. 3. Obtain a sputum specimen for culture and
4. Escort the client diagnosed with laryngitis sensitivity.
outside to smoke a cigarette. 4. Have the unlicensed assistive personnel weigh
the client.

06_Colgrove_Ch06.indd 191 01/06/16 5:16 PM


192 M ed -S urg S uccess

17. The 56-year-old client diagnosed with 21. The day shift charge nurse on a medical unit is
tuberculosis (Tb) is being discharged. Which making rounds after report. Which client should
statement made by the client indicates an be seen first?
understanding of the discharge instructions? 1. The 65-year-old client diagnosed with
1. “I will take my medication for the full three tuberculosis who has a sputum specimen to
(3) weeks prescribed.” be sent to the laboratory.
2. “I must stay on the medication for months 2. The 76-year-old client diagnosed with
if I am to get well.” aspiration pneumonia who has a clogged
3. “I can be around my friends because I have feeding tube.
started taking antibiotics.” 3. The 45-year-old client diagnosed with
4. “I should get a Tb skin test every three (3) pneumonia who has a pulse oximetry reading
months to determine if I am well.” of 92%.
4. The 39-year-old client diagnosed with
18. The employee health nurse is administering
bronchitis who has an arterial oxygenation
tuberculin skin testing to employees who have
level of 89%.
possibly been exposed to a client with active
tuberculosis (Tb). Which statement indicates 22. The client is admitted with a diagnosis of
the need for radiological evaluation instead rule-out tuberculosis. Which type of isolation
of skin testing? procedures should the nurse implement?
1. The client’s first skin test indicates a purple 1. Standard Precautions.
flat area at the site of injection. 2. Contact Precautions.
2. The client’s second skin test indicates a red 3. Droplet Precautions.
area measuring four (4) mm. 4. Airborne Precautions.
3. The client’s previous skin test was read as
23. The nurse observes the unlicensed assistive
positive.
personnel (UAP) entering an airborne isolation
4. The client has never shown a reaction to the
room and leaving the door open. Which action
tuberculin medication.
is the nurse’s best response?
19. The nurse is caring for the client diagnosed with 1. Close the door and discuss the UAP’s action
pneumonia. Which information should the nurse after coming out of the room.
include in the teaching plan? Select all that 2. Make the UAP come back outside the room
apply. and then reenter, closing the door.
1. Place the client on oxygen delivered by nasal 3. Say nothing to the UAP but report the
cannula. incident to the nursing supervisor.
2. Plan for periods of rest during activities of 4. Enter the client’s room and discuss the matter
daily living. with the UAP immediately.
3. Place the client on a fluid restriction of
24. The client is admitted to a medical unit with
1,000 mL/day.
a diagnosis of pneumonia. Which signs and
4. Restrict the client’s smoking to two (2) to
symptoms should the nurse assess in the client?
three (3) cigarettes per day.
1. Pleuritic chest discomfort and anxiety.
5. Monitor the client’s pulse oximetry readings
2. Asymmetrical chest expansion and pallor.
every four (4) hours.
3. Leukopenia and CRT <three (3) seconds.
20. The nurse is feeding a client diagnosed 4. Substernal chest pain and diaphoresis.
with aspiration pneumonia who becomes
dyspneic, begins to cough, and is turning blue.
Which nursing intervention should the nurse Chronic Obstructive Pulmonary
implement first? Disease (COPD)
1. Suction the client’s nares.
2. Turn the client to the side. 25. The nurse is assessing the client with COPD.
3. Place the client in Trendelenburg position. Which health promotion information is most
4. Notify the health-care provider. important for the nurse to obtain?
1. Number of years the client has smoked.
2. Risk factors for complications.
3. Ability to administer inhaled medication.
4. Willingness to modify lifestyle.

06_Colgrove_Ch06.indd 192 01/06/16 5:16 PM


C hapter 6  R espiratory D isorders 193

26. The client diagnosed with an exacerbation of 31. Which client problems are appropriate for the
COPD is in respiratory distress. Which nurse to include in the plan of care for the
intervention should the nurse implement first? client diagnosed with COPD? Select all that
1. Assist the client into a sitting position at apply.
90 degrees. 1. Impaired gas exchange.
2. Administer oxygen at six (6) LPM via nasal 2. Inability to tolerate temperature extremes.
cannula. 3. Activity intolerance.
3. Monitor vital signs with the client sitting 4. Inability to cope with changes in roles.
upright. 5. Alteration in nutrition.
4. Notify the health-care provider about the
32. Which outcome is appropriate for the client
client’s status.
problem “ineffective gas exchange” for the client
27. The nurse is assessing the client diagnosed recently diagnosed with COPD?
with COPD. Which data require immediate 1. The client demonstrates the correct way
intervention by the nurse? to pursed-lip breathe.
1. Large amounts of thick white sputum. 2. The client lists three (3) signs/symptoms
2. Oxygen flowmeter set on eight (8) liters. to report to the HCP.
3. Use of accessory muscles during inspiration. 3. The client will drink at least 2,500 mL of
4. Presence of a barrel chest and dyspnea. water daily.
4. The client will be able to ambulate 100 feet
28. The nurse is caring for the client diagnosed with
with dyspnea.
COPD. Which outcome requires a revision in
the plan of care? 33. The nurse observes the unlicensed assistive
1. The client has no signs of respiratory distress. personnel (UAP) removing the nasal cannula
2. The client shows an improved respiratory from the client diagnosed with COPD while
pattern. ambulating the client to the bathroom. Which
3. The client demonstrates intolerance to action should the nurse implement?
activity. 1. Praise the UAP because this prevents
4. The client participates in establishing goals. the client from tripping on the oxygen
tubing.
29. The nurse is caring for the client diagnosed
2. Place the oxygen back on the client
with end-stage COPD. Which data warrant
while sitting in the bathroom and say
immediate intervention by the nurse?
nothing.
1. The client’s pulse oximeter reading is 92%.
3. Explain to the UAP in front of the
2. The client’s arterial blood gas level is 74.
client oxygen must be left in place at all
3. The client has SOB when walking to the
times.
bathroom.
4. Discuss the UAP’s action with the
4. The client’s sputum is rusty colored.
charge nurse so appropriate action can
30. Which statement made by the client diagnosed be taken.
with chronic bronchitis indicates to the nurse
34. Which clinical manifestation should the
more teaching is required?
nurse expect to assess in the client recently
1. “I should contact my health-care provider if
diagnosed with COPD?
my sputum changes color or amount.”
1. Clubbing of the client’s fingers.
2. “I will take my bronchodilator regularly to
2. Infrequent respiratory infections.
prevent having bronchospasms.”
3. Chronic sputum production.
3. “This metered-dose inhaler gives a precise
4. Nonproductive hacking cough.
amount of medication with each dose.”
4. “I need to return to the HCP to have my
blood drawn with my annual physical.”

06_Colgrove_Ch06.indd 193 01/06/16 5:16 PM


194 M ed -S urg S uccess

35. Which statement made by the client indicates 40. The client is diagnosed with mild intermittent
the nurse’s discharge teaching is effective for asthma. Which medication should the nurse
the client diagnosed with COPD? discuss with the client?
1. “I need to get an influenza vaccine each year, 1. Daily inhaled corticosteroids.
even when there is a shortage.” 2. Use of a “rescue inhaler.”
2. “I need to get a vaccine for pneumonia each 3. Use of systemic steroids.
year with my influenza shot.” 4. Leukotriene agonists.
3. “If I reduce my cigarettes to six (6) a day,
41. Which statement indicates to the nurse the
I won’t have difficulty breathing.”
client diagnosed with asthma understands
4. “I need to restrict my drinking liquids to keep
the teaching regarding mast cell stabilizer
from having so much phlegm.”
medications?
36. Which referral is most appropriate for a client 1. “I should take two (2) puffs when I begin to
diagnosed with end-stage COPD? have an asthma attack.”
1. The Asthma Foundation of America. 2. “I must taper off the medications and not stop
2. The American Cancer Society. taking them abruptly.”
3. The American Lung Association. 3. “These drugs will be most effective if taken at
4. The American Heart Association. bedtime.”
4. “These drugs are not good at the time of an
attack.”
Reactive Airway Disease (Asthma)
42. The client diagnosed with asthma is admitted
37. The nurse is completing the admission to the emergency department with difficulty
assessment on a 13-year-old client diagnosed breathing and a blue color around the mouth.
with an acute exacerbation of asthma. Which Which diagnostic test will be ordered to
signs and symptoms would the nurse expect determine the status of the client?
to find? 1. Complete blood count.
1. Fever and crepitus. 2. Pulmonary function test.
2. Rales and hives. 3. Allergy skin testing.
3. Dyspnea and wheezing. 4. Drug cortisol level.
4. Normal chest shape and eupnea. 43. The nurse and a licensed practical nurse (LPN)
38. The nurse is planning the care of a client are caring for five (5) clients on a medical unit.
diagnosed with asthma and has written Which clients would the nurse assign to the
a problem of “anxiety.” Which nursing LPN? Select all that apply.
intervention should be implemented? 1. The 32-year-old female diagnosed with
1. Remain with the client. exercise-induced asthma who has a forced
2. Notify the health-care provider. vital capacity of 1,000 mL.
3. Administer an anxiolytic medication. 2. The 45-year-old male with adult-onset
4. Encourage the client to drink fluids. asthma who is complaining of difficulty
completing all of the ADLs at one time.
39. The case manager is arranging a care planning 3. The 92-year-old client diagnosed with
meeting regarding the care of a 65-year-old respiratory difficulty who is beginning to be
client diagnosed with adult-onset asthma. Which confused and keeps climbing out of bed.
health-care disciplines should participate in the 4. The 6-year-old client diagnosed with intrinsic
meeting? Select all that apply. asthma who is scheduled for discharge and
1. Nursing. the mother needs teaching about the
2. Pharmacy. medications.
3. Social work. 5. The 20-year-old client diagnosed with asthma
4. Occupational therapy. who has a pulse oximetry reading of 95% and
5. Speech therapy. wants to sleep all the time.

06_Colgrove_Ch06.indd 194 01/06/16 5:16 PM


C hapter 6  R espiratory D isorders 195

44. The charge nurse is making rounds. Which 48. The nurse is discharging a client newly
client should the nurse assess first? diagnosed with restrictive airway disease
1. The 29-year-old client diagnosed with (asthma). Which statement indicates the
reactive airway disease who is complaining client understands the discharge
the nurse caring for him was rude. instructions?
2. The 76-year-old client diagnosed with heart 1. “I will call 911 if my medications don’t
failure who has 2+ edema of the lower control an attack.”
extremities. 2. “I should wash my bedding in warm water.”
3. The 15-year-old client diagnosed with diabetic 3. “I can still eat at the Chinese restaurant when
ketoacidosis after a bout with the flu who has a I want.”
blood glucose reading of 189 mg/dL. 4. “If I get a headache, I should take a
4. The 62-year-old client diagnosed with COPD nonsteroidal anti-inflammatory drug.”
and pneumonia who is receiving O2 by nasal
cannula at two (2) liters per minute.
Lung Cancer
45. The client diagnosed with exercise-induced
asthma (EIA) is being discharged. Which 49. The nurse is taking the social history from a
information should the nurse include in the client diagnosed with small cell carcinoma of the
discharge teaching? lung. Which information is significant for this
1. Take two (2) puffs on the rescue inhaler and disease?
wait five (5) minutes before exercise. 1. The client worked with asbestos for a short
2. Warm-up exercises will increase the potential time many years ago.
for developing the asthma attacks. 2. The client has no family history for this type
3. Use the bronchodilator inhaler immediately of lung cancer.
prior to beginning to exercise. 3. The client has numerous tattoos covering
4. Increase dietary intake of food high in both upper and lower arms.
monosodium glutamate (MSG). 4. The client has smoked two (2) packs of
46. The client diagnosed with restrictive airway cigarettes a day for 20 years.
disease (asthma) has been prescribed a 50. The nurse writes a problem of “impaired gas
glucocorticoid inhaled medication. Which exchange” for a client diagnosed with cancer
information should the nurse teach regarding of the lung. Which interventions should be
this medication? included in the plan of care? Select all that
1. Do not abruptly stop taking this medication; apply.
it must be tapered off. 1. Apply O2 via nasal cannula.
2. Immediately rinse the mouth following 2. Have the dietitian plan for six (6) small meals
administration of the drug. per day.
3. Hold the medication in the mouth for 3. Place the client in respiratory isolation.
15 seconds before swallowing. 4. Assess vital signs for fever.
4. Take the medication immediately when an 5. Listen to lung sounds every shift.
attack starts.
47. The nurse is discussing the care of a child
diagnosed with asthma with the parent. Which
referral is important to include in the teaching?
1. Referral to a dietitian.
2. Referral for allergy testing.
3. Referral to the developmental psychologist.
4. Referral to a home health nurse.

06_Colgrove_Ch06.indd 195 01/06/16 5:16 PM


196 M ed -S urg S uccess

51. The nurse is discussing cancer statistics with a group from the community. Which information about death
rates from lung cancer is accurate?

Trends in Age-adjusted Cancer Death Rates* by Site, Males U.S., 1930–2012


100

Lung & bronchus

80
Rate per 100,000 male population

60

Stomach Prostate
Colon & rectum
40

20
Liver† Pancreas†

Leukemia
0
1930 1940 1950 1960 1970 1980 1990 2000 2012
*Per 100,000 age-adjusted to the 2000 U.S. standard population. †Mortality
rates for pancreatic and liver cancer are increasing.
Note: Due to changes in ICD coding, numerator information has changed over time. Rates for cancers of the liver, lung & bronchus,
and colon & rectum are affected by these coding changes.
Source: U.S. Mortality Volumes 1930 to 1959 and U.S. Mortality Data 1960 to 2012, National Center for Health Statistics, Centers
for Disease Control and Prevention.
©2016, American Cancer Society Inc., Surveillance Research.

Trends in Age-adjusted Cancer Death Rates* by Site, Females U.S., 1930–2012


100

80
Rate per 100,000 male population

60

Uterus† Breast Lung & bronchus


40

Stomach Colon & rectum


20
Pancreas‡
Liver‡

0
1930 1940 1950 1960 1970 1980 1990 2000 2012
*Per 100,000 age-adjusted to the 2000 U.S. standard population. †Uterus refers to uterine cervix and uterine corpus combined.
‡Mortality rates for pancreatic and liver cancers are increasing.

Note: Due to changes in ICD coding, numerator information has changed over time. Rates for cancers of the liver, lung & bronchus,
and colon & rectum are affected by these coding changes.
Source: U.S. Mortality Volumes 1930 to1959, National Center for Health Statistics, Centers for Disease Control and Prevention.
©2016, American Cancer Society Inc., Surveillance Research.

06_Colgrove_Ch06.indd 196 01/06/16 5:16 PM


C hapter 6  R espiratory D isorders 197

1. Lung cancer has a low mortality rate because 55. The nursing staff on an oncology unit is
of new treatment options. interviewing applicants for the unit manager
2. Lung cancer is the number-one cause of position. Which type of organizational structure
cancer deaths in both men and women. does this represent?
3. Lung cancer deaths are not significant in 1. Centralized decision making.
relation to other cancers. 2. Decentralized decision making.
4. Lung cancer deaths have continued to 3. Shared governance.
increase in the male population. 4. Pyramid with filtered-down decisions.
52. The nurse and an unlicensed assistive personnel 56. The client diagnosed with lung cancer is being
(UAP) are caring for a group of clients on a discharged. Which statement made by the client
medical unit. Which information provided by indicates more teaching is required?
the UAP warrants immediate intervention by 1. “It doesn’t matter if I smoke now. I already
the nurse? have cancer.”
1. The client diagnosed with cancer of the 2. “I should see the oncologist at my scheduled
lung has a small amount of blood in the appointment.”
sputum collection cup. 3. “If I begin to run a fever, I should notify
2. The client diagnosed with chronic the HCP.”
emphysema is sitting on the side of the bed 4. “I should plan for periods of rest throughout
and leaning over the bedside table. the day.”
3. The client receiving Procrit, a biologic
57. The clinic nurse is interviewing clients. Which
response modifier, has a T 99.2˚F, P 68, R 24,
information provided by a client warrants
and BP of 198/102.
further investigation?
4. The client receiving prednisone, a steroid, is
1. The client uses Vicks VapoRub every night
complaining of an upset stomach after eating
before bed.
breakfast.
2. The client has had an appendectomy.
53. The client diagnosed with lung cancer has 3. The client takes a multiple vitamin pill
been told the cancer has metastasized to the every day.
brain. Which intervention should the nurse 4. The client has been coughing up blood in the
implement? mornings.
1. Discuss implementing an advance directive.
58. The client is four (4) hours post-lobectomy
2. Explain the use of chemotherapy for brain
for cancer of the lung. Which assessment data
involvement.
warrant immediate intervention by the nurse?
3. Teach the client to discontinue driving.
1. The client has an intake of 1,500 mL IV and
4. Have the significant other make decisions for
an output of 1,000 mL.
the client.
2. The client has 450 mL of bright-red drainage
54. The client diagnosed with lung cancer is in an in the chest tube.
investigational program and receiving a vaccine 3. The client is complaining of pain at a “10” on
to treat the cancer. Which information regarding a 1-to-10 scale.
investigational regimens should the nurse 4. The client has absent lung sounds on the side
teach? of the surgery.
1. Investigational regimens provide a better
59. The client is admitted to the outpatient surgery
chance of survival for the client.
center for a bronchoscopy to rule out cancer
2. Investigational treatments have not been
of the lung. Which information should the nurse
proven to be helpful to clients.
teach?
3. Clients will be paid to participate in an
1. The test will confirm the results of the MRI.
investigational program.
2. The client can eat and drink immediately after
4. Only clients who are dying qualify for
the test.
investigational treatments.
3. The HCP can do a biopsy of the tumor
through the scope.
4. There is no discomfort associated with this
procedure.

06_Colgrove_Ch06.indd 197 01/06/16 5:16 PM


198 M ed -S urg S uccess

60. The client diagnosed with oat cell carcinoma of 65. The client has had a total laryngectomy. Which
the lung tells the nurse, “I am so tired of all this. referral is specific for this surgery?
I might as well just end it all.” Which statement 1. CanSurmount.
should be the nurse’s first response? 2. Dialogue.
1. Say, “This must be hard for you. Would you 3. Lost Chord Club.
like to talk?” 4. SmokEnders.
2. Tell the HCP of the client’s statement.
66. The nurse and an unlicensed assistive personnel
3. Refer the client to a social worker or spiritual
(UAP) are caring for a group of clients on a
advisor.
surgical floor. Which information provided by
4. Find out if the client has a plan to carry out
the UAP requires immediate intervention by
suicide.
the nurse?
1. There is a small, continuous amount of
Cancer of the Larynx bright-red drainage coming out from under
the dressing of the client who had a radical
61. The nurse is admitting a client with a diagnosis neck dissection.
of rule-out cancer of the larynx. Which 2. The client who has had a right upper
information should the nurse teach? lobectomy is complaining that the patient-
1. Demonstrate the proper method of gargling controlled analgesia (PCA) pump is not
with normal saline. providing any relief.
2. Perform voice exercises for 30 minutes three 3. The client diagnosed with cancer of the
(3) times a day. lung is complaining of being tired and short
3. Explain that a lighted instrument will be of breath.
placed in the throat to biopsy the area. 4. The client admitted with chronic obstructive
4. Teach the client to self-examine the larynx pulmonary disease is making a whistling
monthly. sound with every breath.

62. The client is diagnosed with cancer of the larynx 67. The charge nurse is assigning clients for the
and is to have radiation therapy to the area. shift. Which client should be assigned to the
Which prophylactic procedure will the nurse new graduate nurse?
prepare the client for? 1. The client diagnosed with cancer of the lung
1. Removal of the client’s teeth and fitting for who has chest tubes.
dentures. 2. The client diagnosed with laryngeal spasms
2. Take antiemetic medications every four who has stridor.
(4) hours. 3. The client diagnosed with laryngeal cancer
3. Wear sunscreen on the area at all times. who has multiple fistulas.
4. Placement of a nasogastric feeding tube. 4. The client who is two (2) hours post–partial
laryngectomy.
63. The client is three (3) days post–partial
laryngectomy. Which type of nutrition should 68. The nurse is writing a care plan for a client
the nurse offer the client? newly diagnosed with cancer of the larynx.
1. Total parenteral nutrition. Which problem is the highest priority?
2. Soft, regular diet. 1. Wound infection.
3. Partial parenteral nutrition. 2. Hemorrhage.
4. Clear liquid diet. 3. Respiratory distress.
4. Knowledge deficit.
64. The nurse is preparing the client diagnosed
with laryngeal cancer for a laryngectomy in 69. The male client has had a radial neck dissection
the morning. Which intervention is the nurse’s for cancer of the larynx. Which action by the
priority? client indicates a disturbance in body image?
1. Take the client to the intensive care unit for a 1. The client requests a consultation by the
visit. speech therapist.
2. Explain that the client will need to ask for 2. The client has a towel placed over the mirror.
pain medication. 3. The client is attempting to shave himself.
3. Demonstrate the use of an antiembolism hose. 4. The client practices neck and shoulder
4. Find out if the client can read and write. exercises.

06_Colgrove_Ch06.indd 198 01/06/16 5:16 PM


C hapter 6  R espiratory D isorders 199

70. The HCP has recommended a total 74. The client is suspected of having a pulmonary
laryngectomy for a male client diagnosed with embolus. Which diagnostic test suggests the
cancer of the larynx but the client refuses. presence of a pulmonary embolus and requires
Which intervention by the nurse illustrates the further investigation?
ethical principle of nonmalfeasance? 1. Plasma D-dimer test.
1. The nurse listens to the client explain why he 2. Arterial blood gases.
is refusing surgery. 3. Chest x-ray (CXR).
2. The nurse and significant other insist that the 4. Magnetic resonance imaging (MRI).
client have the surgery.
75. Which nursing assessment data support that the
3. The nurse refers the client to a counselor for
client has experienced a pulmonary embolism?
help with the decision.
1. Calf pain with dorsiflexion of the foot.
4. The nurse asks a cancer survivor to come and
2. Sudden onset of chest pain and dyspnea.
discuss the surgery with the client.
3. Left-sided chest pain and diaphoresis.
71. The client diagnosed with cancer of the larynx 4. Bilateral crackles and low-grade fever.
has had four (4) weeks of radiation therapy to the
76. The client diagnosed with a pulmonary
neck. The client is complaining of severe pain
embolus is in the intensive care department.
when swallowing. Which scientific rationale
Which assessment data warrant immediate
explains the pain?
intervention from the nurse?
1. The cancer has grown to obstruct the
1. The client’s ABGs are pH 7.36, Pao2 95,
esophagus.
Paco2 38, Hco3 24.
2. The treatments are working on the cancer
2. The client’s telemetry exhibits occasional
and the throat is edematous.
premature ventricular contractions (PVCs).
3. Cancers are painful and this is expected.
3. The client’s pulse oximeter reading is 90%.
4. The treatments are also affecting the
4. The client’s urinary output for the 12-hour
esophagus, causing ulcerations.
shift is 800 mL.
72. The client who has undergone a radical neck
77. The client is admitted to the medical unit
dissection and tracheostomy for cancer of the
diagnosed with a pulmonary embolus. Which
larynx is being discharged. Which discharge
intervention should the nurse implement?
instructions should the nurse teach? Select all
1. Administer oral anticoagulants.
that apply.
2. Assess the client’s bowel sounds.
1. The client will be able to speak again after the
3. Prepare the client for a thoracentesis.
surgery area has healed.
4. Institute and maintain bedrest.
2. The client should wear a protective covering
over the stoma when showering. 78. The nurse is preparing to administer the oral
3. The client should clean the stoma and then anticoagulant warfarin (Coumadin) to a client
apply a petroleum-based ointment. who has a PT/PTT of 22/39 and an INR of 2.8.
4. The client should use a humidifier in the What action should the nurse implement?
room. 1. Assess the client for abnormal bleeding.
5. The client can get a special telephone for 2. Prepare to administer vitamin K
communication. (AquaMephyton).
3. Administer the medication as ordered.
4. Notify the HCP to obtain an order to
Pulmonary Embolus increase the dose.

73. The client is diagnosed with a pulmonary 79. The nurse identified the client problem
embolus (PE) and is receiving a heparin drip. “decreased cardiac output” for the client
The bag hanging is 20,000 units/500 mL diagnosed with a pulmonary embolus. Which
of D5W infusing at 22 mL/hr. How many intervention should be included in the plan
units of heparin is the client receiving each of care?
hour? ________ 1. Monitor the client’s arterial blood gases
2. Assess skin color and temperature.
3. Check the client for signs of bleeding.
4. Keep the client in the Trendelenburg position.

06_Colgrove_Ch06.indd 199 01/06/16 5:16 PM


200 M ed -S urg S uccess

80. Which nursing interventions should the Chest Trauma


nurse implement for the client diagnosed
with a pulmonary embolus who is undergoing 85. The client is admitted to the emergency
thrombolytic therapy? Select all that apply. department with chest trauma. Which signs/
1. Keep protamine sulfate readily available. symptoms indicate to the nurse the diagnosis
2. Avoid applying pressure to venipuncture of pneumothorax?
sites. 1. Bronchovesicular lung sounds and bradypnea.
3. Assess for overt and covert signs of bleeding. 2. Unequal lung expansion and dyspnea.
4. Avoid invasive procedures and injections. 3. Frothy, bloody sputum and consolidation.
5. Administer stool softeners as ordered. 4. Barrel chest and polycythemia.
81. Which statement by the client diagnosed with 86. The client had a right-sided chest tube inserted
a pulmonary embolus indicates the discharge two (2) hours ago for a pneumothorax. Which
teaching is effective? action should the nurse implement if there
1. “I am going to use a regular-bristle is no fluctuation (tidaling) in the water-seal
toothbrush.” compartment?
2. “I will take antibiotics prior to having my 1. Obtain an order for a STAT chest x-ray.
teeth cleaned.” 2. Increase the amount of wall suction.
3. “I can take enteric-coated aspirin for my 3. Check the tubing for kinks or clots.
headache.” 4. Monitor the client’s pulse oximeter reading.
4. “I will wear a Medic Alert band at all times.”
87. Which intervention should the nurse implement
82. The client diagnosed with a pulmonary embolus for a male client who has had a left-sided chest
is being discharged. Which intervention should tube for six (6) hours and who refuses to take
the nurse discuss with the client? deep breaths because of the pain?
1. Increase fluid intake to two (2) to three 1. Medicate the client and have the client take
(3) L/day. deep breaths.
2. Eat a low-cholesterol, low-fat diet. 2. Encourage the client to take shallow breaths
3. Avoid being around large crowds. to help with the pain.
4. Receive pneumonia and flu vaccines. 3. Explain deep breaths do not have to be taken
83. The nurse is preparing to administer at this time.
medications to the following clients. Which 4. Tell the client if he doesn’t take deep breaths,
medication should the nurse question he could die.
administering? 88. The unlicensed assistive personnel (UAP) assists
1. The oral coagulant warfarin (Coumadin) the client with a chest tube to ambulate to the
to the client with an INR of 1.9. bathroom. Which situation warrants immediate
2. Regular insulin to a client with a blood intervention from the nurse?
glucose level of 218 mg/dL. 1. The UAP keeps the chest tube below chest
3. Hang the heparin bag on a client with a level.
PT/PTT of 12.9/98. 2. The UAP has the chest tube attached to suction.
4. A calcium channel blocker to the client with a 3. The UAP allowed the client out of the bed.
BP of 112/82. 4. The UAP uses a bedside commode for the
84. The client is getting out of bed and becomes client.
very anxious and has a feeling of impending 89. The nurse is caring for a client with a right-
doom. The nurse thinks the client may be sided chest tube that is accidentally pulled out of
experiencing a pulmonary embolism. Which the pleural space. Which action should the nurse
action should the nurse implement first? implement first?
1. Administer oxygen 10 L via nasal cannula. 1. Notify the health-care provider to have chest
2. Place the client in high Fowler’s position. tubes reinserted STAT.
3. Obtain a STAT pulse oximeter reading. 2. Instruct the client to take slow shallow breaths
4. Auscultate the client’s lung sounds. until the tube is reinserted.
3. Take no action and assess the client’s
respiratory status every 15 minutes.
4. Tape a petroleum jelly occlusive dressing on
three (3) sides to the insertion site.

06_Colgrove_Ch06.indd 200 01/06/16 5:16 PM


C hapter 6  R espiratory D isorders 201

90. The nurse is presenting a class on chest tubes. 95. The alert and oriented client is diagnosed with
Which statement best describes a tension a spontaneous pneumothorax, and the health-
pneumothorax? care provider is preparing to insert a left-sided
1. A tension pneumothorax develops when an air- chest tube. Which intervention should the nurse
filled bleb on the surface of the lung ruptures. implement first?
2. When a tension pneumothorax occurs, the air 1. Gather the needed supplies for the
moves freely between the pleural space and procedure.
the atmosphere. 2. Obtain a signed informed consent form.
3. The injury allows air into the pleural space 3. Assist the client into a side-lying position.
but prevents it from escaping from the pleural 4. Discuss the procedure with the client.
space.
96. Which intervention should the nurse implement
4. A tension pneumothorax results from a
first for the client diagnosed with a hemothorax
puncture of the pleura during a central line
who has had a right-sided chest tube for three
placement.
(3) days and has no fluctuation (tidaling) in the
91. Which action should the nurse implement for water compartment?
the client with a hemothorax who has a right- 1. Assess the client’s bilateral lung sounds.
sided chest tube with excessive bubbling in the 2. Obtain an order for a STAT chest x-ray.
water-seal compartment? 3. Notify the health-care provider as soon as
1. Check the amount of wall suction being possible.
applied. 4. Document the findings in the client’s chart.
2. Assess the tubing for any blood clots.
3. Milk the tubing proximal to distal.
4. Encourage the client to cough forcefully. Acute Respiratory Distress Syndrome
92. Which assessment data indicate to the nurse 97. The unlicensed assistive personnel (UAP)
the chest tubes inserted three (3) days ago have is bathing the client diagnosed with acute
been effective in treating the client with a respiratory distress syndrome (ARDS). The bed
hemothorax? is in a high position with the opposite side rail in
1. Gentle bubbling in the suction compartment. the low position. Which action should the nurse
2. No fluctuation (tidaling) in the water-seal implement?
compartment. 1. Demonstrate the correct technique for giving
3. The drainage compartment has 250 mL of a bed bath.
blood 2. Encourage the UAP to put the bed in the
4. The client is able to deep breathe without lowest position.
any pain. 3. Instruct the UAP to get another person to
93. The nurse is caring for a client with a right-sided help with the bath.
chest tube secondary to a pneumothorax. Which 4. Provide praise for performing the bath safely
interventions should the nurse implement when for the client and the UAP.
caring for this client? Select all that apply. 98. The client diagnosed with ARDS is transferred
1. Place the client in the low Fowler’s position. to the intensive care department and placed on a
2. Assess chest tube drainage system frequently. ventilator. Which intervention should the nurse
3. Maintain strict bedrest for the client. implement first?
4. Secure a loop of drainage tubing to the sheet. 1. Confirm that the ventilator settings are
5. Observe the site for subcutaneous emphysema. correct.
94. The charge nurse is making client assignments 2. Verify that the ventilator alarms are
on a medical floor. Which client should the functioning properly.
charge nurse assign to the licensed practical 3. Assess the respiratory status and pulse
nurse (LPN)? oximeter reading.
1. The client with pneumonia who has a pulse 4. Monitor the client’s arterial blood gas
oximeter reading of 91%. results.
2. The client with a hemothorax who has Hb of
9 g/dL and Hct of 20%.
3. The client with chest tubes who has jugular
vein distention and BP of 96/60.
4. The client who is two (2) hours post–
bronchoscopy procedure.

06_Colgrove_Ch06.indd 201 01/06/16 5:16 PM


202 M ed -S urg S uccess

99. The nurse suspects the client may be 103. The client with ARDS is on a mechanical
developing ARDS. Which assessment data ventilator. Which intervention should be
confirm the diagnosis of ARDS? included in the nursing care plan addressing
1. Low arterial oxygen when administering the endotracheal tube (ET) care?
high concentration of oxygen. 1. Do not move or touch the ET tube.
2. The client has dyspnea and tachycardia and 2. Obtain a chest x-ray daily.
is feeling anxious. 3. Determine if the ET cuff is deflated.
3. Bilateral breath sounds clear and pulse 4. Ensure that the ET tube is secure.
oximeter reading is 95%.
104. Which medication should the nurse anticipate
4. The client has jugular vein distention and
the health-care provider ordering for the client
frothy sputum.
diagnosed with ARDS?
100. The client who smokes two (2) packs of 1. An aminoglycoside antibiotic.
cigarettes a day develops ARDS after a near- 2. A synthetic surfactant.
drowning. The client asks the nurse, “What is 3. A potassium cation.
happening to me? Why did I get this?” Which 4. A nonsteroidal anti-inflammatory drug.
statement by the nurse is most appropriate?
105. The client diagnosed with ARDS is in
1. “Most people who almost drown end up
respiratory distress and the ventilator is
developing ARDS.”
malfunctioning. Which intervention should
2. “Platelets and fluid enter the alveoli due to
the nurse implement first?
permeability instability.”
1. Notify the respiratory therapist
3. “Your lungs are filling up with fluid, causing
immediately.
breathing problems.”
2. Ventilate with a manual resuscitation bag.
4. “Smoking has caused your lungs to become
3. Request STAT arterial blood gases.
weakened, so you got ARDS.”
4. Auscultate the client’s lung sounds.
101. Which assessment data indicate to the
106. The nurse is caring for the client diagnosed
nurse the client diagnosed with ARDS has
with ARDS. Which interventions should the
experienced a complication secondary to the
nurse implement? Select all that apply.
ventilator?
1. Assess the client’s level of consciousness.
1. The client’s urine output is 100 mL in four
2. Monitor urine output every shift.
(4) hours.
3. Turn the client every two (2) hours.
2. The pulse oximeter reading is greater
4. Maintain intravenous fluids as ordered.
than 95%.
5. Place the client in the Fowler’s position.
3. The client has asymmetrical chest
expansion. 107. Which instruction is priority for the nurse to
4. The telemetry reading shows sinus discuss with the client diagnosed with ARDS
tachycardia. who is being discharged from the hospital?
1. Avoid smoking and exposure to smoke.
102. The health-care provider ordered STAT
2. Do not receive flu or pneumonia vaccines.
arterial blood gases (ABGs) for the client
3. Avoid any type of alcohol intake.
diagnosed with ARDS. The ABG results are
4. It will take about one (1) month to
pH 7.38, Pao2 92, Paco2 38, Hco3 24. Which
recuperate.
action should the nurse implement?
1. Continue to monitor the client without 108. The client diagnosed with ARDS is on a
taking any action. ventilator and the high alarm indicates an
2. Encourage the client to take deep breaths increase in the peak airway pressure. Which
and cough. intervention should the nurse implement first?
3. Administer one (1) ampule of sodium 1. Check the tubing for any kinks.
bicarbonate IVP. 2. Suction the airway for secretions.
4. Notify the respiratory therapist of the ABG 3. Assess the lip line of the ET tube.
results. 4. Sedate the client with a muscle relaxant.

06_Colgrove_Ch06.indd 202 01/06/16 5:16 PM


C hapter 6  R espiratory D isorders 203

CONCEPTS
The concepts covered in this chapter focus on oxygenation and acid–base problems. Exemplars that are covered
are chronic obstructive pulmonary disease (COPD), pneumonia, lung cancer, and respiratory distress. Interrelated
concepts of the nursing process and critical thinking are covered throughout the questions. The concept of critical
thinking is presented in the prioritizing or “first” questions.

109. The client is admitted to emergency 111. The client diagnosed with community-
department complaining of shortness of breath acquired pneumonia is admitted to the medical
and fever. The vital signs are T 100.40, P 94, unit. Which health-care provider order should
R 26 and BP 134/86. Which concept should the nurse implement first?
the nurse identify and a concern for the client? 1. Start IV with 1,000 mL 0.9% saline.
Select all that apply. 2. Ceftriaxone 1 gm IVPB every 12 hours.
1. Clotting. 3. Obtain sputum and blood cultures.
2. Oxygenation. 4. CBC and basic metabolic panel.
3. Infection.
112. The client diagnosed with chronic obstructive
4. Perfusion.
pulmonary disease (COPD) is admitted to
5. Coping.
the medical unit. The client has thin
110. The nurse observes the client sitting on the extremities, truncal obesity, and multiple
side of the bed with the arms propped on the ecchymotic areas on the arms. Based on the
over-bed table. The chest is barrel shaped and assessment data, which question should the
the client is breathing though lips spaced close nurse ask the client?
together and is exhaling slowly. Which concept 1. “Do you take prednisone?”
is priority for this client? 2. “Can you tell me who hurts you?”
1. Mobility. 3. “May I check your coccyx for pressure
2. Nutrition. areas?”
3. Activity intolerance. 4. “Do you sleep with the head of the bed
4. Oxygenation. elevated?”
113. The client is diagnosed with chronic obstructive pulmonary disease (COPD) and pneumonia. Which
medication should the day nurse question administering?
1. Methylprednisolone.
2. Ceftriaxone.
3. Tessalon Perles.
4. Morphine.

Admitting Diagnosis:
Client Name: A.B. COPD/Pneumonia

Account Number: 1234567 Med Rec #:00 66 77 Allergies: Cephalosporins


Date Medication 2301–0700 0701–1500 1501–2300
Yesterday Methylprednisolone 100 mg 2400 0800 1600
IVP every 8 hours
Yesterday Ceftriaxone 1 gr. IVPB 0900 2100
every 12 hours
Yesterday Benzonatate (Tessalon Perles) orally 0900 2100
every 12 hours
Yesterday Morphine 30 mg orally every day 0900
Signature of Nurse: Day Nurse RN/DN

06_Colgrove_Ch06.indd 203 01/06/16 5:16 PM


204 M ed -S urg S uccess

114. The nurse is reviewing the charts of the clients. Which data should the nurse report to the health-care
provider?
A. Client A—Diagnosis: Pneumonia

Complete Blood Count Patient Value Normal Values


6
RBC (10 ) 5.2 Male: 4.7 to 5.1 106 cells/mm3
Female: 4.2 to 4.8 106 cells/mm3
Hemoglobin 38 Male: 43% to 49%
Female: 38% to 44%
Hematocrit 12.1 Male: 13.2 to 17.3 g/dL
Female: 11.7 to 15.5 g/dL
Platelet (103) 582 150 to 450 103/µL/mm3
WBC (103) 18.3 4.5 to 11.0 1,000/mm3

B. Client B—Diagnosis: Respiratory Distress

Arterial Blood Gas Client Normal Values


pH 7.35 7.35 to 7.45
Pco2 45 35 to 45 mm Hg
Hco3 28 22 to 26 mEq/L
Pao2 82 80 to 100 mm Hg

C. Client C—Diagnosis: Acute Exacerbation of COPD

Glucose 132 70–110 mg/dL


Hemoglobin A1C 6.9 4–6

D. Client D—Diagnosis: Sepsis

Site Patient Culture Sensitivity


Sputum Culture Positive for gram-negative Keflex -
Streptococcus Ceftriaxone -
Gram Stain Gram-negative organism

1. Client A.
2. Client B.
3. Client C.
4. Client D.
115. Which intervention should the nurse institute for the client diagnosed with COPD and cor pulmonale
based on the intake and output record?
Oral Intravenous Urine Nasogastric Tube Other (Specify)
Day One (in mL) (in mL) (in mL) (in mL) (in mL)
0701–1500 1275 1250 1000
1501–2300 680 1000 700
2301–0700 260 1000 1100
Total 2215 3250 2800

1. Administer methylprednisolone IVP.


2. Administer guaifenesin orally.
3. Request an order to reduce the IV rate.
4. Continue to monitor the client.

06_Colgrove_Ch06.indd 204 01/06/16 5:16 PM


C hapter 6  R espiratory D isorders 205

116. The client diagnosed with respiratory distress 119. The nurse is caring for clients on a medical
has arterial blood gases of pH 7.45; Paco2 54; unit. Which assessment data indicates a critical
Hco3 25; Pao2 52. Which should the nurse oxygenation problem for the client?
implement? Select all that apply. 1. The client with an anterior upper left chest
1. Apply oxygen via nonrebreather mask. tube is splinting the dressing with a pillow.
2. Call the rapid response team (RRT). 2. The male client on oxygen is coughing
3. Elevate the head of the bed. forcefully, making it hard to catch his breath.
4. Stay with the client. 3. The client who is at rest has circumoral
5. Notify the health-care provider (HCP). cyanosis and is difficult to arouse.
4. The female client complains of shortness of
117. The nurse is applying oxygen via nasal
breath while ambulating in the hallway.
cannula to a client diagnosed with chronic
obstructive pulmonary disease (COPD). 120. The 75-year-old male nursing home resident
The client complains of extreme shortness is found wandering in the hall and has a new
of breath. At which rate should the nurse set onset of confusion. Which should the nurse
the flowmeter? implement first?
1. 2 LPM. 1. Assess the resident’s lung fields and
2. 4 LPM. temperature.
3. 6 LPM. 2. Have the resident return to his room.
4. 10 LPM. 3. Notify the resident’s family to come and sit
with him.
118. The charge nurse receives morning laboratory
4. Ask the unlicensed assistive personnel (UAP)
and respiratory data on the clients. Which data
to push fluids.
requires immediate intervention?
1. ABG results of pH 7.35, Paco2 56, Hco3 29,
Pao2 78 for a client diagnosed with COPD.
2. Pulse oximetry reading of 89% on a two-day
postsurgical total knee replacement client.
3. Hgb of 9 g/dL and Hct of 28% on a client
who is receiving the second unit of blood.
4. B-type natriuretic peptide (BNP) of 100 on
a client diagnosed with stage 4 congestive
heart failure.

06_Colgrove_Ch06.indd 205 01/06/16 5:16 PM


PRACTICE QUESTIONS ANSWERS
AND RATIONALES

Upper Respiratory Infection 3. 1. The client should be taught to take all


­antibiotics as ordered. Discontinuing
1. 1. Influenza is a viral illness that might cause antibiotics prior to the full dose results
these symptoms; however, an immunization in the development of antibiotic-resistant
should not give the client the illness. bacteria. Sinus infections are difficult to
2. Coming into contact with small children in- treat and may become chronic and will
creases the risk of contracting colds and the then require several weeks of therapy or
flu, but the client has a problem—not just a possibly surgery to control.
potential one. 2. If the sinuses are irrigated, it is done under
3. A client diagnosed with hyperten- anesthesia by a health-care provider.
sion should not take many of the 3. Blowing the nose will increase pressure in
over-the-counter medications because they the sinus cavities and will cause the client
work by causing vasoconstriction, which increased pain.
will increase the hypertension. 4. The nurse is not licensed to prescribe medi-
4. Cold sores are actually an infection by the her- cations, so this is not in the nurse’s scope of
pes simplex virus. Colds and cold-like symp- practice. Also, narcotic analgesic medications
toms are caused by the rhinovirus or influenza are controlled substances and require writ-
virus. The term “cold sore” is a common term ten documentation of being prescribed by the
that still persists in the populace. health-care provider; samples are not generally
available.
TEST-TAKING HINT: The keys to answering
this question are the words “hypertension” TEST-TAKING HINT: Note in this situation an
and “cold.” Any time a client has a chronic “all” is in the correct answer. There are very
illness, the client should discuss over-the- few cases in which absolute adjectives will de-
counter medications with the health-care scribe the correct answer. The test taker must
provider or a pharmacist. Many of the routine be aware that general rules will not always
medications for chronic illnesses interact with apply.
over-the-counter medications. Content – Medical: Integrated Nursing Process –
Content – Medical: Integrated Nursing Process – Planning: Client Needs – Physiological Integrity, Physiologi-
Assessment: Client Needs – Physiological Integrity, cal Adaptation: Cognitive Level – Synthesis: Concept –
Reduction of Risk Potential: Cognitive Level – Analysis: Infection.
Concept – Perfusion. 4. 1. Muscle weakness is a sign/symptom of
2. 1. It is not feasible for a child to always have a myalgia, but it is not a life-threatening
tissue or handkerchief available. complication of sinusitis.
2. There is no immunization for the common 2. Purulent sputum would be a sign/symptom of
cold. Colds are actually caused by at least a lung infection, but it is not a life-
200 separate viruses and the viruses mutate threatening complication of sinusitis.
frequently. 3. Nuchal rigidity is a sign/symptom of men-
3. Colds are caused by a virus and antibiotics do ingitis, which is a life-threatening potential
not treat a virus; therefore, there is no need to complication of sinusitis resulting from the
go to a health-care provider. close proximity of the sinus cavities to the
4. Hand washing is the single most useful meninges.
technique for prevention of disease. 4. Intermittent loss of muscle control can be a
symptom of multiple sclerosis, but it would not
TEST-TAKING HINT: Option “1” contains the
be a life-threatening complication of sinusitis.
word “always,” an absolute word, and in most
questions, absolutes such as “always,” “never,” TEST-TAKING HINT: A basic knowledge of
and “only” make that answer option incorrect. anatomy and physiology would help to answer
Content – Medical: Integrated Nursing Process – this question. The sinuses lie in the head and
Planning: Client Needs – Health Promotion and Main- surround the orbital cavity. Options “1” and
tenance: Cognitive Level – Synthesis: Concept – Nursing “4” refer to muscle problems, so both could
Roles. be ruled out as wrong.

206

06_Colgrove_Ch06.indd 206 01/06/16 5:16 PM


C hapter 6  R espiratory D isorders 207

Content – Medical: Integrated Nursing Process – 7. 1. The client has complained of pain, and the
Assessment: Client Needs – Physiological Integrity, Reduction nurse, after determining the severity of the
of Risk Potential: Cognitive Level – Analysis: Concept – pain and barring any complications in the
Infection. client, will administer pain medication after
5. 1. The hemoglobin and hematocrit (H&H) given completion of the assessment.
are within normal range. This would not war- 2. Oral hygiene helps to prevent the development
rant notifying the health-care provider. of infections and promotes comfort, but it will
2. A low-grade temperature and a cough not relieve the pain.
could indicate the presence of an 3. Placing the client in the semi-Fowler’s position
infection, in which case the health-care will reduce edema of inflamed sinus tissue, but
provider would not want to subject the it will not immediately affect the client’s per-
client to anesthesia and the possibility of ception of pain.
further complications. The surgery would 4. Prior to intervening, the nurse must
be postponed. assess to determine the amount of pain
3. One (1) to two (2) WBCs in a urinalysis is not and possible complications occurring that
uncommon because of the normal flora in the could be masked if narcotic medication is
bladder. administered.
4. The INR indicates that the client’s bleeding TEST-TAKING HINT: Whenever there is an as-
time is within normal range. sessment answer among the answer options,
TEST-TAKING HINT: In this question, all the the test taker should look carefully at what
answer options contain normal data except for is being assessed. If the option says to assess
one. The nurse would not call the health-care the problem identified in the question, it will
provider to notify him or her of normal values. usually be the correct answer. Remember, the
Content – Medical: Integrated Nursing Process – first step in the nursing process is assessment.
Assessment: Client Needs – Safe Effective Care Environment, Content – Surgical: Integrated Nursing Process –
Management of Care: Cognitive Level – Analysis: Concept – Implementation: Client Needs – Safe Effective Care Envi-
Infection. ronment, Management of Care: Cognitive Level –
Application: Concept – Comfort.
6. 1. The elderly and chronically ill are at great-
est risk for developing serious complica- 8. 1. This client is one (1) day postoperative and has
tions if they contract the influenza virus. moderate pain, which is to be expected after
2. It is recommended people in contact with chil- surgery. A less experienced nurse can care for
dren receive the flu vaccine whenever possible, this client.
but these clients should be able to withstand a 2. A child about to go to surgery involving the
bout with the flu if their immune systems are throat area can be expected to have painful
functioning normally. swallowing. This does not require the most
3. It is probable these clients will be exposed to experienced nurse.
the virus, but they are not as likely to develop 3. The postoperative client with purulent
severe complications with intact functioning drainage could be developing an infection.
immune systems. The experienced nurse would be needed to
4. During flu season, the more people the assess and monitor the client’s condition.
individual comes into contact with, the greater 4. Any nurse who is capable of administering
the risk the client will be exposed to the in- IVPB medications can care for this client.
fluenza virus, but this group of people would TEST-TAKING HINT: In this type of question,
not receive the vaccine before the elderly and the test taker must determine if the situation
chronically ill. described is expected or within normal limits
TEST-TAKING HINT: Sometimes the test taker (WNL). The option that gives unexpected
may think the answer is too easy and obvious, data or data not within normal limits requires
but the test taker should not try to second- a nurse with more knowledge and experience.
guess the question. Item writers are not try- Content – Surgical: Integrated Nursing Process –
ing to trick the test taker; they are trying to Planning: Client Needs – Safe Effective Care Environment,
evaluate knowledge. Management of Care: Cognitive Level – Synthesis:
Concept – Medication.
Content – Medical: Integrated Nursing Process –
Assessment: Client Needs – Health Promotion and Main-
tenance: Cognitive Level – Analysis: Concept – Promoting
Health.

06_Colgrove_Ch06.indd 207 01/06/16 5:16 PM


208 M ed -S urg S uccess

9. 1. A person with a viral infection should not re- 3. Throat swabs for culture must be done
turn to work until the virus has run its course correctly or false-negative results can occur.
because the antibiotics help prevent compli- The nurse should obtain the swab.
cations of the virus, but they do not make the 4. Clients with laryngitis are instructed not to
client feel better faster. smoke. Smoking is discouraged in all health-
2. Secondary bacterial infections often care facilities. Sending nursing personnel out-
accompany influenza, and antibiotics side encourages an unhealthy practice, which
are often prescribed to help prevent the is not the best use of the personnel.
development of a bacterial infection re- TEST-TAKING HINT: Interventions requiring
sulting from a weakened client immune assessment, teaching, and evaluation cannot
system. be delegated. Levels of activities being del-
3. Antibiotics will not prevent the flu. Only the egated should be appropriate for the level of
flu vaccine will prevent the flu. training of the staff member carrying out the
4. When people take portions of the antibiotic task. Tasks delegated must conform to safe
prescription and stop taking the remainder, health-care practice.
an antibiotic-resistant strain of bacteria may Content – Nursing Management: Integrated Nursing
develop, and the client may experience a Process – Planning: Client Needs – Safe Effective Care
return of symptoms—but this time, the Environment, Management of Care: Cognitive Level –
antibiotics will not be effective. Synthesis: Concept – Nursing Roles.
TEST-TAKING HINT: Knowing drug classifi-
12. 1. Alternative therapies are therapies not
cations and how the drugs within the clas- accepted as standard medical practice.
sification work would assist the test taker These may be encouraged as long as they
to determine the correct answer. ­Antibiotics do not interfere with the ­medical regimen.
work to destroy bacterial invasions of the Vitamin C in large doses is thought to
body. improve the immune system’s functions.
Content – Medical: Integrated Nursing Process – 2. Bedrest is accepted standard advice for a
Evaluation: Client Needs – Physiological Integrity, client with a cold.
Pharmacological and Parenteral Therapies: Cognitive
3. Humidifying the air helps to relieve conges-
Level – Evaluation: Concept – Medication.
tion and is a standard practice.
10. 1. The client with laryngitis is instructed to 4. Decongestant therapy is standard therapy for
avoid all alcohol. Alcohol causes increased a cold.
­irritation of the throat. TEST-TAKING HINT: Only one of the answer
2. Whispering places added strain on the larynx. options is not common advice for a cli-
3. Voice rest is encouraged for the client ex- ent with a cold. When all options but one
periencing laryngitis. (1) match each other, then the odd option
4. Aphonia, or inability to speak, is a temporary should be selected as the correct answer.
condition associated with laryngitis. Content – Medical: Integrated Nursing Process –
TEST-TAKING HINT: Encouraging the use of Diagnosis: Client Needs – Physiological Integrity, Pharma-
alcohol, with the exception of a glass of red cological and Parenteral Therapies: Cognitive Level –
wine, is not accepted medical practice; Knowledge: Concept – Oxygenation.
therefore, option “1” could be eliminated.
Option “4” has an absolute—“permanent”—
in it and therefore could be eliminated from Lower Respiratory Infection
consideration.
13. 1. The elderly client diagnosed with
Content – Medical: Integrated Nursing Process –
Implementation: Client Needs – Physiological Integrity,
pneumonia may present with weakness,
Basic Care and Comfort: Cognitive Level – Application: fatigue, lethargy, confusion, and poor ap-
Concept – Infection. petite but not have any of the classic signs
and symptoms of pneumonia.
11. 1. Tonsillectomies cause throat edema and 2. Fever and chills are classic symptoms of
difficulty swallowing; the nurse must pneumonia, but they are usually absent in
observe the client’s ability to swallow the elderly client.
before this task can be delegated. 3. Frothy sputum and edema are signs and
2. Clients with colds are encouraged to symptoms of heart failure, not pneumonia.
drink 2,000 mL of liquids a day. The 4. The client has tachypnea (fast respirations),
UAP could do this. not bradypnea (slow respirations), and jugular
vein distention accompanies heart failure.

06_Colgrove_Ch06.indd 208 01/06/16 5:16 PM


C hapter 6  R espiratory D isorders 209

TEST-TAKING HINT: The question provides given, three (3) days, is the correct time in-
an age range—“elderly”—so age can be ex- terval for performing this intervention.
pected to affect the disease process—in this Content – Medical: Integrated Nursing Process –
case, causing atypical symptoms. The prefix Diagnosis: Client Needs – Physiological Integrity, Basic
brady- means “slow” when attached to a Care and Comfort: Cognitive Level – Analysis: Concept –
word. Knowing the definition of medical pre- Nursing.
fixes can assist the test taker in determining
16. 1. Broad-spectrum IV antibiotics are priority,
the correct answer.
but before antibiotics are administered, it is
Content – Medical: Nursing Process – Assessment: Client important to obtain culture specimens to de-
Needs – Physiological Integrity, Reduction of Risk Potential:
termine the correct antibiotic for the client’s
Cognitive Level – Analysis: Concept – Infection.
infection. Clients are placed on oral medica-
14. 1. Clients do not perform chest physiotherapy; tions only after several days of IVPB therapy.
this is normally done by the respiratory ther- 2. Meal trays are not priority over cultures.
apist. This is a staff goal, not a client goal. 3. To determine the antibiotic that will
2. This would be a goal for self-care deficit but effectively treat an infection, specimens
not for impaired gas exchange. for culture are taken prior to beginning
3. This would be a goal for the problem of the medication. Administering antibiotics
activity intolerance. prior to cultures may make it impossible
4. Impaired gas exchange results in hypoxia, to determine the actual agent causing the
the earliest sign/symptom of which is a pneumonia.
change in the level of consciousness. 4. Admission weights are important to deter-
TEST-TAKING HINT: The test taker should mine appropriate dosing of medication, but
match the answer option to the listed nurs- they are not priority over sputum collection.
ing problem. Option “1” is a staff goal to ac- TEST-TAKING HINT: Option “1” has a medica-
complish. When writing goals for the client, tion classification and a route, and the test
it is important to remember they are written taker should question if the route is appro-
in terms of what is expected of the client. priate for the client being admitted. Clients
Options “2” and “3” are appropriately writ- will not die from a delayed meal, but a client
ten client goals, but they do not evaluate gas could die from delayed IV antibiotic therapy.
exchange. Content – Medical: Integrated Nursing Process –
Content – Medical: Integrated Nursing Process – Implementation: Client Needs – Safe Effective Care
Diagnosis: Client Needs – Physiological Integrity, Physi- Environment, Management of Care: Cognitive Level –
ological Adaptation: Cognitive Level – Analysis: Concept – Synthesis: Concept – Infection.
Infection.
17. 1. Clients diagnosed with Tb will need to
15. 1. A gastrostomy tube is placed directly into the take the medications for six (6) months to
stomach through the abdominal wall; the a year.
naris is the opening of the nostril. 2. Compliance with treatment plans for
2. Elevating the head of the bed uses Tb includes multidrug therapy for six (6)
gravity to keep the formula in the gastric months to one (1) year for the client to be
cavity and help prevent it from refluxing free of the Tb bacteria.
into the esophagus, which predisposes the 3. Clients are no longer contagious when three
client to aspiration. (3) morning sputum specimens are cultured
3. The Sims position is the left lateral side-lying negative, but this will not occur until after
flat position. This position is used for admin- several weeks of therapy.
istering enemas and can be used to prevent 4. The Tb skin test only determines possible
aspiration in clients sedated by anesthesia. The exposure to the bacteria, not active disease.
sedated client would not have a full stomach. TEST-TAKING HINT: The test taker should
4. Dressings on PEG tubes should be changed determine if the time of three (3) weeks in
at least daily. If there is no dressing, the option “1,” months in option “2,” or im-
­insertion site is still assessed daily. mediately in ­option “3” is the correct time
TEST-TAKING HINT: The test taker should try interval.
to picture the positioning of the client to Content – Medical: Nursing Process – Evaluation: Client
determine the correct answer. In option “4,” Needs – Physiological Integrity, Physiological Adaptation:
the test taker should question if the time Cognitive Level – Evaluation: Concept – Oxygenation.

06_Colgrove_Ch06.indd 209 01/06/16 5:16 PM


210 M ed -S urg S uccess

18. 1. A purple flat area indicates that the client 3. Placing the client in the Trendelenburg
­became bruised when the intradermal position increases the risk of aspiration.
injection was given, but it has no bearing 4. An immediate action is needed to protect the
on whether the test is positive. client.
2. A positive skin test is 10 mm or greater with TEST-TAKING HINT: In a question requiring
induration, not redness. the test taker to determine the first action,
3. If the client has ever reacted positively, all the answer options may be correct for
then the client should have a chest x-ray the situation. The test taker must determine
to look for causation and inflammation. which has the greatest p ­ otential for improv-
4. These are negative findings and do not ing the client’s condition.
indicate the need to have x-ray determination Content – Medical: Integrated Nursing Process –
of disease. Implementation: Client Needs – Safe Effective Care
TEST-TAKING HINT: The test taker should Environment, Management of Care: Cognitive Level –
note descriptive terms such as “purple,” Application: Concept – Infection.
“flat,” or “4 mm” before determining the 21. 1. The specimen needs to be taken to the labo-
correct answer. Option “4” has the absolute ratory within a reasonable time frame, but a
word “never,” and absolutes usually indicate UAP can take specimens to the laboratory.
incorrect answers. 2. Clogged feeding tubes occur with some regu-
Content – Medical: Integrated Nursing Process – larity. Delay in feeding a client will not result
Evaluation: Client Needs – Safe Effective Care Environ- in permanent damage.
ment, Management of Care: Cognitive Level – Analysis:
3. A pulse oximetry reading of 92% means
Concept – Oxygenation.
that the arterial blood oxygen saturation
19. 1. The client diagnosed with pneumonia will is somewhere around 60% to 70%.
have some degree of gas-exchange defi- 4. Arterial oxygenation normal values are 80%
cit. Administering oxygen would help the to 100%.
client. TEST-TAKING HINT: Be sure to read all the
2. Activities of daily living require energy answer options. Pulse oximetry readings do
and therefore oxygen consumption. Spac- not give the same information as arterial
ing the activities allows the client blood gas readings.
to rebuild oxygen reserves between Content – Medical: Integrated Nursing Process –
activities. Assessment: Client Needs – Safe Effective Care Environ-
3. Clients are encouraged to drink at least ment, Management of Care: Cognitive Level – Analysis:
2,000 mL daily to thin secretions. Concept – Oxygenation.
4. Cigarette smoking depresses the action of
the cilia in the lungs. Any smoking should 22. 1. Standard Precautions are used to prevent
be prohibited. exposure to blood and body secretions on all
5. Pulse oximetry readings provide the nurse clients. Tuberculosis is caused by airborne
with an estimate of oxygenation in the bacteria.
periphery. 2. Contact Precautions are used for wounds.
3. Droplet Precautions are used for infections
TEST-TAKING HINT: Maslow’s hierarchy of
spread by sneezing or coughing but not
needs lists oxygenation as the top priority. transmitted over distances of more than
Therefore, the test taker should select inter- three (3) to four (4) feet.
ventions addressing oxygenation. 4. Tuberculosis bacteria are capable of dis-
Content – Medical: Integrated Nursing Process – seminating over long distances on air cur-
Planning: Client Needs – Physiological Integrity, rents. Clients with tuberculosis are placed
Physiological Adaptation: Cognitive Level – Synthesis:
in negative air pressure rooms where the
Concept – Infection.
air in the room is not allowed to cross-
20. 1. The nares are the openings of the nostrils. contaminate the air in the hallway.
Suctioning, if done, would be of the posterior TEST-TAKING HINT: Standard Precautions and
pharynx. Contact Precautions can be ruled out as the
2. Turning the client to the side allows for correct answer if the test taker is aware that
the food to be coughed up and come out Tb is usually a respiratory illness. This at
of the mouth, rather than be aspirated least gives the reader a 1:2 chance of select-
into the lungs. ing the correct answer if the answer is not
known.

06_Colgrove_Ch06.indd 210 01/06/16 5:16 PM


C hapter 6  R espiratory D isorders 211

Content – Medical: Integrated Nursing Process – Chronic Obstructive Pulmonary


Implementation: Client Needs – Safe Effective Care
Environment, Safety and Infection Control: Cognitive Disease (COPD)
Level – Application: Concept – Oxygenation.
25. 1. The number of years of smoking is informa-
23. 1. Closing the door reestablishes the nega- tion needed to treat the client but not the
tive air pressure, which prevents the air most important in health promotion.
from entering the hall and contaminating 2. The risk factors for complications are impor-
the hospital environment. When correct- tant in planning care.
ing an individual, it is always best to do so 3. Assessing the ability to deliver medications
in a private manner. is an important consideration when teaching
2. The employee is an adult and as such the client.
should be treated with respect and corrected 4. The client’s attitude toward lifestyle
accordingly. changes is the most important consid-
3. Problems should be taken care of at the eration in health promotion, in this case
lowest level possible. The nurse is responsible smoking cessation. The nurse should as-
for any task delegated, including the appro- sess if the client is willing to consider ces-
priate handling of isolation. sation of smoking and carry out the plan.
4. Correcting staff should never be done in
TEST-TAKING HINT: The test taker should
the presence of the client. This undermines
read the stem for words such as “health pro-
the UAP and creates doubt of the staff’s com-
motion.” These words make all the other
petency in the client’s mind.
answer options incorrect because they do not
TEST-TAKING HINT: An action must be taken; promote health.
the test taker must determine which action Content – Medical: Integrated Nursing Process –
would have the desired results with the least Assessment: Client Needs – Safe Effective Care Environ-
amount of disruption to client care. Correct- ment, Management of Care: Cognitive Level – Analysis:
ing the UAP in this manner has the greatest Concept – Oxygenation.
chance of creating a win–win situation.
26. 1. The client should be assisted into a
Content – Medical: Integrated Nursing Process –
sitting position either on the side of the
Implementation: Client Needs – Safe Effective Care Envi-
ronment, Safety and Infection Control: Cognitive Level – bed or in the bed. This position decreases
Application: Concept – Oxygenation. the work of breathing. Some clients find it
easier sitting on the side of the bed lean-
24. 1. Pleuritic chest pain and anxiety from ing over the bed table. The nurse needs to
diminished oxygenation occur along maintain the client’s safety.
with fever, chills, dyspnea, and cough. 2. Oxygen will be applied as soon as possible, but
2. Asymmetrical chest expansion occurs if the least amount possible. If levels of oxygen
the client has a collapsed lung from a are too high, the client may stop breathing.
pneumothorax or hemothorax, and the 3. Vital signs need to be monitored, but this is
client would be cyanotic from decreased not the first priority. If the equipment is not
oxygenation. in the room, another member of the health-
3. The client would have leukocytosis, not care team should bring it to the nurse. The
leukopenia, and a capillary refill time nurse should stay with the client.
(CRT) of less than 3 seconds is normal. 4. The health-care provider needs to be
4. Substernal chest pain and diaphoresis are notified, but the client must be treated first.
symptoms of myocardial infarction. The nurse should get assistance if possible
TEST-TAKING HINT: Options “1” and “4” have so the nurse can treat this client quickly.
chest pain as part of the answer. The adjec- TEST-TAKING HINT: When a question asks for
tives describing the chest pain determine the the test taker to choose the intervention to
correct answer. implement first, the test taker should select
Content – Medical: Integrated Nursing Process – an ­intervention directly caring for the client.
Assessment: Client Needs – Physiological Integrity, ­Remember: in distress do not assess.
Reduction of Risk Potential: Cognitive Level – Analysis:
Content – Medical: Integrated Nursing Process –
Concept – Infection.
Implementation: Client Needs – Safe Effective Care Envi-
ronment, Management of Care: Cognitive Level –
Application: Concept – Oxygenation.

06_Colgrove_Ch06.indd 211 01/06/16 5:16 PM


212 M ed -S urg S uccess

27. 1. A large amount of thick sputum is a common TEST-TAKING HINT: The test taker could rule
symptom of COPD. There is no cause for out options “1” and “2” as correct answers
immediate intervention. because both describe the same data of de-
2. The nurse should decrease the oxygen creased oxygen, which is characteristic of
rate to two (2) to three (3) liters. Hypox- COPD.
emia is the stimulus for breathing in the Content – Medical: Integrated Nursing Process –
client with COPD. If the hypoxemia Assessment: Client Needs – Safe Effective Care Environ-
improves and the oxygen level increases, ment, Management of Care: Cognitive Level – Synthesis:
the drive to breathe may be eliminated. Concept – Oxygenation.
3. It is common for clients with COPD to use 30. 1. When sputum changes in color or amount,
accessory muscles when inhaling. These or both, this indicates infection, and the
clients tend to lean forward. client should report this information to the
4. In clients with COPD, there is a characteris- health-care provider. This statement indicates
tic barrel chest from chronic hyperinflation, the client understands the teaching.
and dyspnea is common. 2. Bronchodilators should be taken routinely to
TEST-TAKING HINT: This question requires prevent bronchospasms. This statement indi-
interpreting the data to determine which cates the client understands the teaching.
are abnormal or unexpected and require 3. Clients use metered-dose inhalers because
intervention. Options “1,” “3,” and “4” are they deliver a precise amount of medication
expected for the client’s disease process. with correct use. This statement indicates the
Content – Medical: Integrated Nursing Process – client understands the teaching.
Assessment: Client Needs – Safe Effective Care Environ- 4. Clients should have blood levels drawn
ment, Management of Care: Cognitive Level – Analysis: every six (6) months when taking bron-
Concept – Oxygenation. chodilators, not yearly. This indicates
28. 1. The expected outcome showing no signs of the client needs more teaching.
respiratory distress indicates the plan of care TEST-TAKING HINT: When evaluating whether
is effective and should be continued. the client has learned the information pre-
2. An improved respiratory pattern indicates the sented, the test taker is observing for incor-
plan should be continued. rect information. The test taker should pay
3. The expected outcome should be that the close attention to time frames such as “every
client has tolerance for activity; because 12 months.”
the client is not meeting the expected out- Content – Medical: Integrated Nursing Process –
come, the plan of care needs revision. Evaluation: Client Needs – Physiological Integrity, Physi-
4. The client should participate in planning the ological Adaptation: Cognitive Level – Evaluation:
course of care. The client is meeting the ex- Concept – Infection.
pected outcome. 31. 1. The client diagnosed with COPD has
TEST-TAKING HINT: This question is an “ex- difficulty exchanging oxygen with carbon
cept” question. Three of the options indicate dioxide, which is manifested by physical
desired outcomes and only one (1) option signs such as fingernail clubbing and re-
indicates the need for improvement. spiratory acidosis as seen on arterial
Content – Medical: Integrated Nursing Process – blood gases.
Diagnosis: Client Needs – Physiological Integrity, 2. The client should avoid extremes in tem-
Physiological Adaptation: Cognitive Level – Analysis: peratures. Warm temperatures cause an
Concept – Oxygenation. increase in the metabolism and increase
29. 1. The client with end-stage COPD has de- the need for oxygen. Cold temperatures
creased peripheral oxygen levels; therefore, this cause bronchospasms.
would not warrant immediate intervention. 3. The client has increased respiratory effort
2. The client’s ABGs would normally indicate during activities and can be fatigued. Ac-
a low oxygen level; therefore, this would not tivities should be timed so rest periods are
warrant immediate intervention. scheduled to prevent fatigue.
3. The client who develops dyspnea on exertion 4. The client may have difficulty adapting to
should stop the exertion but does not require the role changes brought about because
intervention by the nurse if the dyspnea of the disease process. Many cannot main-
resolves. tain the activities involved in meeting re-
4. Rusty-colored sputum indicates blood in sponsibilities at home and at work. Clients
the sputum and requires further assess- should be assessed for these issues.
ment by the nurse. 5. Clients often lose weight because of the
effort expended to breathe.

06_Colgrove_Ch06.indd 212 01/06/16 5:16 PM


C hapter 6  R espiratory D isorders 213

TEST-TAKING HINT: This is an example of an 34. 1. Clubbing of the fingers is the result of chronic
alternate-type question. There may be more hypoxemia, which is expected with chronic
than one (1) correct answer. The test taker COPD but not recently diagnosed COPD.
should consider all options independently 2. These clients have frequent respiratory
and understand that the question is not a infections.
trick. 3. Sputum production, along with cough and
Content – Medical: Integrated Nursing Process – dyspnea on exertion, are the early signs/
Diagnosis: Client Needs – Physiological Integrity, Physi- symptoms of COPD.
ological Adaptation: Cognitive Level – Analysis: Concept – 4. These clients have a productive cough, not
Oxygenation. a nonproductive cough.
32. 1. Pursed-lip breathing helps keep the TEST-TAKING HINT: The test taker must be
alveoli open to allow for better oxygen observant of terms such as “recently diag-
and carbon dioxide exchange. nosed,” which help to rule out incorrect
2. This would be an appropriate outcome for answers such as option “1.” Option “2” has
a knowledge-deficit problem. the word “infrequent.” The test taker must
3. This outcome does not ensure the client notice these words.
has an effective airway; increasing fluid Content – Medical: Integrated Nursing Process –
does not ensure an effective airway. Assessment: Client Needs – Physiological Integrity, Reduc-
4. This is not an appropriate outcome for tion of Risk Potential: Cognitive Level – Analysis:
any client problem because the client Concept – Oxygenation.
should be able to ambulate without 35. 1. Clients diagnosed with COPD should
dyspnea for 100 feet. receive the influenza vaccine each year.
TEST-TAKING HINT: The test taker needs to If there is a shortage, these clients have
identify the outcome for the client problem top priority.
cited—namely, “ineffective gas exchange.” 2. The pneumococcal vaccine should be admin-
The only answer option addressing the air- istered every five (5) to seven (7) years.
way is option “1,” pursed-lip breathing. 3. Reducing the number of cigarettes smoked
Content – Medical: Integrated Nursing Process – does not stop the progression of COPD, and
Diagnosis: Client Needs – Physiological Integrity, Physi- the client will continue to experience signs
ological Adaptation: Cognitive Level – Analysis: Concept – and symptoms such as shortness of breath or
Oxygenation. dyspnea on exertion.
4. Clients diagnosed with COPD should in-
33. 1. The client diagnosed with COPD needs
crease their fluid intake unless contrain-
oxygen at all times, especially when
dicated for another health condition. The
exerting energy such as ambulating to the
increased fluid assists the client in expectorat-
bathroom.
ing the thick sputum.
2. The client needs the oxygen, and the
nurse should not correct the UAP in front TEST-TAKING HINT: Nurses are expected to
of the client; it is embarrassing for the serve as community resources. The nurse
UAP and the client loses confidence in the should be knowledgeable about health-
staff. promotion activities such as immunizations.
3. The nurse should not verbally correct a UAP One (1) option describes a desired goal, but
in front of the client; the nurse should cor- the other three (3) do not.
rect the behavior and then talk to the UAP in Content – Medical: Integrated Nursing Process –
private. Evaluation: Client Needs – Health Maintenance and
4. The primary nurse should confront the UAP Promotion: Cognitive Level – Synthesis: Concept –
and take care of the situation. Continued un- Oxygenation.
safe client care would warrant notifying the 36. 1. The Asthma Foundation of America is not ap-
charge nurse. propriate for a client in this stage of COPD.
TEST-TAKING HINT: The test taker must know 2. The American Cancer Society is helpful for
management concepts, and the nurse should a client with lung cancer but not for a client
first address the behavior with the person with COPD.
directly, then follow the chain of command. 3. The American Lung Association has in-
Content – Medical: Integrated Nursing Process – formation helpful for a client with COPD.
Implementation: Client Needs – Safe Effective Care Envi- 4. Many clients with COPD end up with heart
ronment, Management of Care: Cognitive Level – problems, but the American Heart Associa-
Application: Concept – Nursing Roles. tion does not have information for clients
with COPD.

06_Colgrove_Ch06.indd 213 01/06/16 5:16 PM


214 M ed -S urg S uccess

TEST-TAKING HINT: The test taker should be TEST-TAKING HINT: Before choosing an an-
familiar with organizations, but if the test swer option that directs the test taker to
taker had no idea what the answer was, the notify a health-care provider, the test taker
only option containing a word referring to should determine if the option is describing
r­ espiration—“lung”—is option “3.” an expected event or data for the disease pro-
Content – Medical: Integrated Nursing Process – cess being discussed. If it is expected, then
Implementation: Client Needs – Safe Effective Care Envi- notifying the health-care provider would not
ronment, Management of Care: Cognitive Level – be the correct answer.
Application: Concept – Health Care System. Content – Medical: Integrated Nursing Process –
Implementation: Client Needs – Safe Effective Care Envi-
ronment, Management of Care: Cognitive Level –
Reactive Airway Disease (Asthma) Application: Concept – Oxygenation.

37. 1. Fever is a sign of infection, and crepitus is air 39. 1. Nursing is the one discipline remaining
trapped in the layers of the skin. with the client around the clock. There-
2. Rales indicate fluid in the lung, and hives are fore, nurses have knowledge of the client
a skin reaction to a stimulus such as occurs that other disciplines might not know.
with an allergy to a specific substance. 2. The pharmacist will be able to discuss
3. During an asthma attack, the muscles sur- the medication regimen the client is
rounding the bronchioles constrict, caus- receiving and make suggestions regard-
ing a narrowing of the bronchioles. The ing other medications or medication
lungs then respond with the production of interactions.
secretions that further narrow the lumen. 3. The social worker may be able to assist
The resulting symptoms include wheez- with financial information or home care
ing from air passing through the narrow, arrangements.
clogged spaces and dyspnea. 4. Occupational therapists help clients with
4. During an attack, the chest will be expanded activities of daily living and modifications to
from air being trapped and not being home environments; nothing in the stem in-
exhaled. A chest x-ray will reveal a lowered dicates a need for these services.
diaphragm and hyperinflated lungs. 5. Speech therapists assist clients with speech
and swallowing problems; nothing in the
TEST-TAKING HINT: The test taker must have
stem indicates a need for these services.
a basic knowledge of common medical terms
to answer this question. Dyspnea, wheez- TEST-TAKING HINT: Cost containment issues
ing, and rales are common terms used when are always a concern the nurse must address.
describing respiratory function and lung The use of limited resources (health-care
sounds. Crepitus and eupnea are not as com- personnel) should be on an as-needed basis
monly used, but they are also terms that de- only. Cost containment must be considered
scribe respiratory processes and problems. when using other disciplines or supplies.
Content – Medical: Integrated Nursing Process – Content – Medical: Integrated Nursing Process –
Assessment: Client Needs – Physiological Integrity, Reduc- Implementation: Client Needs – Safe Effective Care Envi-
tion of Risk Potential: Cognitive Level – Analysis: ronment, Management of Care: Cognitive Level –
Concept – Oxygenation. Application: Concept – Health Care System.

38. 1. Anxiety is an expected sequela of being 40. 1. Daily inhaled steroids are used for mild,
unable to meet the oxygen needs of the moderate, or severe persistent asthma, not
body. Staying with the client lets the for intermittent asthma.
client know the nurse will intervene 2. Clients with intermittent asthma will
and the client is not alone. have exacerbations treated with rescue
2. Because anxiety is an expected occurrence ­inhalers. Therefore, the nurse should
with asthma, it is not necessary to notify the teach the client about rescue inhalers.
health-care provider. 3. Systemic steroids are used frequently by
3. An anxiolytic medication could decrease clients with severe persistent asthma, not
respiratory drive and increase the respiratory with mild intermittent asthma.
distress. Also, the medication will require a 4. Leukotriene agonists are prescribed for
delayed time period to begin to work. clients diagnosed with mild persistent
4. Drinking fluids will not treat an asthma asthma.
attack or anxiety.

06_Colgrove_Ch06.indd 214 01/06/16 5:16 PM


C hapter 6  R espiratory D isorders 215

TEST-TAKING HINT: In the stem, there are two only one (1) option has the word “pulmo-
(2) words giving the test taker a clue about nary” in it.
the correct answer. “Mild” and “intermit- Content – Medical: Integrated Nursing Process –
tent” are words that indicate the client is not Implementation: Client Needs – Physiological Integrity,
experiencing frequent or escalating symp- Reduction of Risk Potential: Cognitive Level – Application:
toms. Steroid medications can have multiple Concept – Oxygenation.
side effects. 43. 1. A forced vital capacity of 1,000 mL is con-
Content – Medical: Integrated Nursing Process – sidered normal for most females; there-
Planning: Client Needs – Physiological Integrity, Pharma- fore, the LPN could care for this client.
cological and Parenteral Therapies: Cognitive Level –
2. The client should be encouraged to pace
Synthesis: Concept – Medication.
the activities of daily living; this is ex-
41. 1. Mast cell stabilizers require 10 to 14 days pected for a client diagnosed with asthma,
to be effective. Some clients diagnosed with so the LPN could care for this client.
exercise-induced asthma derive benefit from 3. Confusion could be a sign of decreased oxy-
taking the drugs immediately before exercis- gen to the brain and requires the RN’s exper-
ing, but these drugs must be in the system for tise. This client should not be assigned to the
a period of time before effectiveness can be LPN.
achieved. 4. The client’s mother requires teaching, which
2. Tapering of medications is done for systemic is the nurse’s responsibility and cannot be as-
steroids because of adrenal functioning. signed to an LPN.
3. The drugs are taken daily or before exercise, 5. A pulse oximetry level of 95% is normal,
or both. so this client could be assigned to an
4. Mast cell drugs are routine maintenance LPN.
medications and do not treat an attack. TEST-TAKING HINT: The nurse cannot assign a
TEST-TAKING HINT: The test taker must be licensed practical nurse assessment, teaching,
knowledgeable about medications. There are evaluation, or an unstable client.
not many test-taking hints. If the test taker Content – Nursing Management: Integrated Nursing
knows that a specific option applies to a Process – Planning: Client Needs – Safe Effective Care
medication other than the one (1) mentioned Environment, Management of Care: Cognitive Level –
in the stem, the test taker can eliminate that Synthesis: Concept – Nursing Roles.
option. 44. 1. The charge nurse is responsible for all
Content – Medical: Integrated Nursing Process – clients. At times it is necessary to see
Evaluation: Client Needs – Physiological Integrity, Phar- clients with a psychosocial need before
macological and Parenteral Therapies: Cognitive Level –
other clients who have expected and non–
Evaluation: Concept – Medication.
life-threatening situations.
42. 1. A complete blood count determines the 2. Two (2)+ edema of the lower extremities
oxygen-carrying capacity of the hemoglobin is expected in a client diagnosed with heart
in the body, but it will not identify the failure.
immediate problem. 3. A blood glucose reading of 189 mg/dL is not
2. Pulmonary function tests are completed within normal range, but it is not in a range
to determine the forced vital capacity indicating the client is catabolizing the fats
(FVC), the forced expiratory capacity and proteins in the body. No ketones will be
in the first second (FEV1), and the peak produced at this blood glucose level, so the
expiratory flow (PEF). A decline in the ketoacidosis has resolved itself.
FVC, FEV1, and PEF indicates respiratory 4. Most clients diagnosed with COPD are
compromise. receiving oxygen at a low level.
3. Allergy skin testing will be done to determine TEST-TAKING HINT: All the options with
triggers for allergic asthma, but it is not done physiological data are expected and not life
during an attack. threatening, so the nurse should address the
4. Drug cortisol levels do not relate to asthma. client with a psychosocial problem.
TEST-TAKING HINT: If the test taker is unsure Content – Medical: Integrated Nursing Process –
about the correct response, it is helpful to Assessment: Client Needs – Safe Effective Care Environ-
choose the option that directly relates to the ment, Management of Care: Cognitive Level – Analysis:
topic. Asthma is a pulmonary problem, and Concept – Mood.

06_Colgrove_Ch06.indd 215 01/06/16 5:16 PM


216 M ed -S urg S uccess

45. 1. Rescue inhalers are used to treat attacks, not informed of the child’s diagnosis, and the par-
prevent them, so this should not be adminis- ents must know the individual caring for the
tered prior to exercising. child is prepared to intervene during an attack.
2. Warm-up exercises decrease the risk of TEST-TAKING HINT: The test taker must be
developing an asthma attack. aware of the disease process, determine
3. Using a bronchodilator immediately causes, and then make a decision based on
prior to exercising will help reduce interventions required.
bronchospasms. Content – Medical: Integrated Nursing Process –
4. Monosodium glutamate, a food preservative, Planning: Client Needs – Safe Effective Care Environment,
has been shown to initiate asthma attacks. Management of Care: Cognitive Level – Synthesis:
TEST-TAKING HINT: Option “1” has two words Concept – Health Care System.
that are opposed—“rescue” and “wait”— 48. 1. The client must be able to recognize
which might lead the test taker to eliminate a life-threatening situation and initiate
this option. Remember basic concepts, the correct procedure.
which are contradicted in option “2.” There 2. Bedding is washed in hot water to kill dust
are a few disease processes that encourage mites.
intake of sodium, but asthma is not one of 3. Many Chinese dishes are prepared with
them, which would cause option “4” to be monosodium glutamate, an ingredient that
eliminated. can initiate an asthma attack.
Content – Medical: Integrated Nursing Process – 4. Nonsteroidal anti-inflammatory medications,
Planning: Client Needs – Physiological Integrity, Physi- aspirin, and beta blockers have been known to
ological Adaptation: Cognitive Level – Synthesis:
initiate asthma attacks.
Concept – Oxygenation.
TEST-TAKING HINT: Dietary questions or
46. 1. This applies to systemically administered answer options should be analyzed for the
steroids, not to inhaled steroids. content. The test taker should decide, “What
2. The steroids must pass through the oral about Chinese foods could be a problem for
cavity before reaching the lungs. Allowing a client diagnosed with asthma?” or “What
the medication to stay within the oral might be good for the client about this diet?”
cavity will suppress the normal flora found Content – Medical: Integrated Nursing Process –
there, and the client could develop a yeast Evaluation: Client Needs – Physiological Integrity, Physi-
infection of the mouth (oral candidiasis). ological Adaptation: Cognitive Level – Synthesis:
3. Holding the medication in the mouth in- Concept – Oxygenation.
creases the risk of an oral yeast infection, and
the medication is inhaled, not swallowed.
4. Inhaled steroids are not used first; the beta- Lung Cancer
adrenergic inhalers are used for acute attack.
TEST-TAKING HINT: Option “3” suggests that 49. 1. Working with asbestos is significant for
an inhaled medication is swallowed; the two mesothelioma of the lung, a cancer with a
(2) terms do not match. very poor prognosis, but not for small cell
carcinoma.
Content – Medical: Integrated Nursing Process –
Planning: Client Needs – Physiological Integrity, Pharma-
2. Family history is not the significant risk
cological and Parenteral Therapies: Cognitive Level – factor for small cell carcinoma. Smoking is
Synthesis: Concept – Medication. the number-one risk factor.
3. Tattoos may be implicated in the develop-
47. 1. A child with asthma can eat a regular diet if ment of blood-borne pathogen disease
the child is not allergic to the components of (if sterile needles were not used), but they
the diet. do not have any association with cancer.
2. Because asthma can be a reaction to an 4. Smoking is the number-one risk factor
allergen, it is important to determine for developing cancer of the lung. More
which substances may trigger an attack. than 85% of lung cancers are attribut-
3. The stem did not indicate the child is able to inhalation of chemicals. There are
developmentally delayed. more than 400 chemicals in each puff of
4. The child does not require a home health cigarette smoke, 17 of which are known to
nurse solely on the basis of asthma; the school cause cancer.
nurse or any child-care provider should be

06_Colgrove_Ch06.indd 216 01/06/16 5:16 PM


C hapter 6  R espiratory D isorders 217

TEST-TAKING HINT: If the test taker did not 4. Lung cancer deaths have remained relatively
know this information, option “3” has no an- stable among the male population but have
atomical connection to the lungs and could continued to steadily increase among females.
be eliminated. This information has been TEST-TAKING HINT: The nurse must be able to
widely disseminated in the media for more interpret data to the public. The nurse will
than 40 years since the Surgeon General’s of- be asked about incidence and severity of
fice first warned about the dangers of smok- diseases.
ing in the early 1960s. Content – Medical: Integrated Nursing Process –
Content – Medical: Integrated Nursing Process – Diagnosis: Client Needs – Health Promotion and Mainte-
Diagnosis: Client Needs – Physiological Integrity, Physi- nance: Cognitive Level – Comprehension: Concept –
ological Adaptation: Cognitive Level – Analysis: Promoting Health.
Concept – Cellular Regulation.
52. 1. This is expected from this client and does not
50. 1. Respiratory distress is a common finding warrant immediate attention.
in clients diagnosed with lung cancer. As 2. This is called a three (3)-point stance. It is a
the tumor grows and takes up more space position many clients with lung disease will
or blocks air movement, the client may assume because it assists in the expansion of
need to be taught positioning for lung the lung.
expansion. The administration of oxygen 3. Biologic response modifiers stimulate the
will help the client to use the lung capac- bone marrow and can increase the cli-
ity that is available to get oxygen to the ent’s blood pressure to dangerous levels.
tissues. This BP is high and warrants immediate
2. Clients with lung cancer frequently be- attention.
come fatigued trying to eat. Providing six 4. This client can be seen after taking care of
(6) small meals spaces the amount of food the client in “3.” The nurse should intervene,
the client eats throughout the day. but it is not a life-threatening situation.
3. Cancer is not communicable, so the client
TEST-TAKING HINT: Even if the test taker did
does not need to be in isolation.
not know the side effects of Procrit, a BP of
4. Clients with cancer of the lung are at risk
198/102 warrants immediate attention.
for developing an infection from lowered
resistance as a result of treatments or Content – Medical: Integrated Nursing Process – ­
Implementation: Client Needs – Safe Effective Care Envi-
from the tumor blocking secretions in
ronment, ­Management of Care: Cognitive Level –
the lung. Therefore, monitoring for the Synthesis: Concept – Nursing Roles.
presence of fever, a possible indication of
infection, is important. 53. 1. This situation indicates a terminal
5. Assessment of the lungs should be ­process, and the client should make
completed on a routine and prn basis. ­decisions for the end of life.
TEST-TAKING HINT: This alternative-type 2. Radiation therapy is used for tumors in the
question is an all-or-nothing situation. The brain. Chemotherapy as a whole will not
NCLEX-RN examination requires the test cross the blood–brain barrier.
taker to answer each option correctly to re- 3. There is no indication the client cannot drive
ceive credit for the question. Each option has at this point. Clients may develop seizures
the potential to be right or wrong. from the tumors at some point.
4. The client should make decisions for himself
Content – Medical: Integrated Nursing Process –
Planning: Client Needs – Physiological Integrity, Physi-
or herself as long as possible. However, the
ological Adaptation: Cognitive Level – Synthesis: client should discuss personal wishes with
Concept – Cellular Regulation. the person named in an advance directive to
make decisions.
51. 1. Lung cancer is the number-one cause of can-
TEST-TAKING HINT: The ethical principle of
cer deaths in the United States.
autonomy could help the test taker to discard
2. Lung cancers are responsible for almost
option “4” as a correct answer.
twice as many deaths among males as any
other cancer and more deaths than breast Content – Medical: Integrated Nursing Process –
Implementation: Client Needs – Safe Effective Care Envi-
cancer in females.
ronment, Management of Care: Cognitive Level –
3. Lung cancers are the most deadly cancers Application: Concept – Cellular Regulation.
among the U.S. population.

06_Colgrove_Ch06.indd 217 01/06/16 5:16 PM


218 M ed -S urg S uccess

54. 1. If the investigational regimen proves to therefore, the client does not need more
be effective, then this statement is true. How- teaching.
ever, many investigational treatments have 3. Clients diagnosed with cancer and undergo-
not proved to be efficacious. ing treatment are at risk for developing infec-
2. Investigational treatments are just that— tions, so more teaching is not needed.
treatments being investigated to determine 4. Lung cancers produce fatigue as a result of
if they are effective in the care of clients physiological drains on the body in the areas
diagnosed with cancer. There is no guaran- of lack of adequate oxygen to the tissues, the
tee the treatments will help the client. tumor burden on the body, and the toll taken
3. Clients receive medical care and associated by the effects of the treatments. Cancer-related
treatments and laboratory tests at no cost, fatigue syndrome is a very real occurrence, so
but the client is not paid. Paying the client is the client understands the teaching.
unethical. TEST-TAKING HINT: Two (2) options—“2” and
4. Frequently clients who have failed standard “3”—are instructions given to all clients re-
treatments and who have no other hope of a gardless of disease process.
treatment are the clients involved in investi- Content – Medical: Integrated Nursing Process –
gational protocols, but the protocols can be Evaluation: Client Needs – Safe Effective Care Environ-
used for any client who volunteers for investi- ment, Management of Care: Cognitive Level – Synthesis:
gational treatment. Concept – Cellular Regulation.
TEST-TAKING HINT: The test taker should
57. 1. This is an individual cultural/familial situa-
think about the word “investigational” and tion and should be encouraged unless it in-
understand its meaning. Investigational terferes with the medical treatment plan. The
means looking into something. This should nurse should be nonjudgmental when clients
lead the test taker to choose option “2.” discuss their cultural practices if the nurse
Content – Medical: Integrated Nursing Process – expects the clients to be honest about health
Planning: Client Needs – Physiological Integrity, Physi- practices.
ological Adaptation: Cognitive Level – Synthesis:
2. An appendectomy in the past should be
Concept – Informatics.
documented, but no further information is
55. 1. A centralized system of organization means required.
decisions are made at the top and given to the 3. Many clients take a multivitamin, so this
staff underneath to accept and implement. would not warrant intervention.
2. A decentralized decision-making pattern 4. Coughing up blood is not normal and is
means there is a fairly flat accountability cause for investigation. It could indicate
chart. The unit manager has more autonomy lung cancer.
in managing the unit, but this does not mean TEST-TAKING HINT: The test taker should
the staff has input into decisions. read all distracters carefully. “Further investi-
3. Shared governance is a system where gation” means something abnormal is occur-
the staff is empowered to make decisions ring. Coughing up blood is always abnormal.
such as scheduling and hiring of certain Content – Medical: Integrated Nursing Process –
staff. Staff members are encouraged to Assessment: Client Needs – Safe Effective Care Environ-
participate in developing policies and pro- ment, Management of Care: Cognitive Level –
cedures to reach set goals. Analysis: Concept – Assessment.
4. A pyramid decision-making tree is an
example of a centralized system. 58. 1. This is an adequate output. After a major
surgery, clients will frequently have an intake
TEST-TAKING HINT: Answer options “1” and
greater than the output because of the fluid
“4” basically say the same thing and could be shift occurring as a result of trauma to the
eliminated for this reason. body.
Content – Nursing Management: Integrated Nursing 2. This is about a pint of blood loss and could
Process – Planning: Client Needs – Safe Effective Care indicate the client is hemorrhaging.
Environment, Management of Care: Cognitive Level –
3. The nurse should intervene and medicate the
Synthesis: Concept – Nursing Roles.
client, but pain, although a client comfort is-
56. 1. Research indicates smoking will still in- sue, is not life threatening.
terfere with the client’s response to treat- 4. The client will have a chest tube to assist
ment, so more teaching is needed. in reinflation of the lung, and absent lung
2. It is expected for the client to follow up with sounds are expected at this point in the cli-
a specialist regarding subsequent treatment; ent’s recovery.

06_Colgrove_Ch06.indd 218 01/06/16 5:16 PM


C hapter 6  R espiratory D isorders 219

TEST-TAKING HINT: Blood is always a priority. Cancer of the Larynx


Content – Surgical: Integrated Nursing Process –
Assessment: Client Needs – Safe Effective Care Environ- 61. 1. Gargling with salt water is good for sore
ment, Management of Care: Cognitive Level – Synthesis: throats, but it does not diagnose cancer of the
Concept – Cellular Regulation. larynx.
59. 1. A bronchoscopy is not performed to confirm 2. Clients thought to have a vocal cord problem
another test; it is performed to confirm diag- are encouraged to practice voice rest. Vocal
noses, such as cancer, Pneumocystis pneumo- cord exercises would not assist in the diagno-
nia, tuberculosis, fungal infections, and other sis of cancer.
lung diseases. 3. A laryngoscopy will be performed to
2. The client’s throat will be numbed with a allow for visualization of the vocal cords
local anesthetic to prevent gagging during the and to obtain a biopsy for pathological
procedure. The client will not be able to eat diagnosis.
or drink until this medication has worn off. 4. There is no monthly self-examination of the
3. The HCP can take biopsies and perform a larynx. To visualize the vocal cords, the HCP
washing of the lung tissue for pathological must numb the throat and pass a fiberoptic
diagnosis during the procedure. instrument through the throat and into the
4. Most HCPs use an anesthetic procedure trachea.
called twilight sleep to perform endoscopies, TEST-TAKING HINT: The test taker must un-
but there is no guarantee the client will not derstand that, if the question states the cli-
experience some discomfort. ent is admitted to “rule out” a disease, then
TEST-TAKING HINT: The test taker must read diagnostic tests and procedures will be done
each stem and answer option carefully. When to determine if, in fact, the client has the di-
in medicine does one (1) test confirm an- agnosis that the HCP suspects.
other? Tests are done to confirm diagnoses. Content – Medical: Integrated Nursing Process –
Option “4” has a false promise. Planning: Client Needs – Physiological Integrity, Reduction
Content – Surgical: Integrated Nursing Process – of Risk Potential: Cognitive Level – Synthesis: Concept –
Planning: Client Needs – Physiological Integrity, Reduction Cellular Regulation.
of Risk Potential: Cognitive Level – Synthesis: Concept – 62. 1. The teeth will be in the area of radiation
Cellular Regulation. and the roots of teeth are highly sensi-
60. 1. The nurse might enter into a therapeutic tive to radiation, which results in root
conversation, but client safety is the priority. abscesses. The teeth are removed and
2. The nurse must first assess the seriousness of the client is fitted for dentures prior to
the client’s statement and whether he or she radiation.
has a plan to carry out suicide. ­Depending 2. An antiemetic on a routine scheduled basis
on the client’s responses, the nurse will notify is prophylactic for nausea, and the schedule
the HCP. is 30 minutes before a meal. Radiation to
3. The client can be referred for assistance in the throat does not encompass nausea-
dealing with the disease and its ramifications, producing areas.
but this is not the priority. 3. Sunscreen is used to prevent the penetration
4. The priority action anytime a client makes of ultraviolet (UV) rays into the dermis.
a statement regarding taking his or her Radiation is gamma rays.
own life is to determine if the ­client has 4. The client may receive a PEG tube for severe
thought it through enough to have a plan. esophagitis from irradiation of the esophagus.
A plan indicates an emergency situation. The client does not have a nasogastric tube.
TEST-TAKING HINT: In a question that asks TEST-TAKING HINT: The test taker could elimi-
for a first response, all answer options may nate option “4” as a form of nutritional treat-
be actions the nurse would take. Ranking ment, not prophylaxis. The test taker must
the options in order of action—“4,” “1,” “2,” recognize which anatomical structures would
“3”—may help the test taker to make a deci- lie within the radiation beam. The teeth of the
sion. Applying Maslow’s hierarchy of needs, lower jaw definitely are within the port, and
safety comes first. the upper teeth possibly would be in range.
Content – Medical: Integrated Nursing Process – Content – Surgical: Integrated Nursing Process –
Implementation: Client Needs – Psychosocial Integrity: Implementation: Client Needs – Physiological Integrity,
Cognitive Level – Application: Concept – Mood. Reduction of Risk Potential: Cognitive Level – Application:
Concept – Cellular Regulation.

06_Colgrove_Ch06.indd 219 01/06/16 5:16 PM


220 M ed -S urg S uccess

63. 1. The client is three (3) days postoperative 65. 1. CanSurmount is a program of cancer survi-
partial removal of the larynx and should be vors who volunteer to talk to clients about
eating by this time. having cancer or what the treatments ­involve.
2. The client should be eating normal foods This group is based on the success of Reach
by this time. The consistency should be to Recovery for breast cancer, but it is not
soft to allow for less chewing of the food specific to a particular cancer.
and easier swallowing because a portion of 2. Dialogue is a cancer support group that
the throat musculature has been removed. brings together clients diagnosed with
The client should be taught to turn the cancer to discuss the feelings associated
head toward the affected side when swal- with having cancer.
lowing to help prevent aspiration. 3. The Lost Chord Club is an American
3. The client should be capable of enteral Cancer Society–sponsored group of
nutrition at this time. survivors of larynx cancer. These clients
4. The client should have progressed to a diet are able to discuss the feelings and needs
with a more normal consistency and amount. of clients who have had laryngectomies
TEST-TAKING HINT: The keys to this question because they have all had this particular
are “three (3) days” and “partial.” Clients are surgery.
progressed rapidly after surgery to as normal 4. SmokEnders is a group of clients working
a life as possible. together to stop smoking. It is a group where
anyone who smokes could be referred.
Content – Surgical: Integrated Nursing Process –
Implementation: Client Needs – Physiological Integrity, TEST-TAKING HINT: The larynx is commonly
Basic Care and Comfort: Cognitive Level – Application: referred to as the “vocal chords.” If the test
Concept – Nutrition. taker were not aware of the various support
groups, option “3” has “lost” and “chords” in
64. 1. This is an appropriate preoperative interven-
it and is the best choice.
tion, but it is not priority.
2. The client should be taught about pain Content – Surgical: Integrated Nursing Process –
medication administration, but this is not the Implementation: Client Needs – Safe Effective Care Envi-
ronment, Management of Care: Cognitive Level –
highest priority.
Application: Concept – Health Care System.
3. The client should be told about an antiembo-
lism hose, but it is not necessary to demon- 66. 1. The most serious complication resulting
strate the hose because the nurse will apply from a radical neck dissection is rupture
and remove the hose initially. of the carotid artery. Continuous bright-
4. The client is having the vocal cords red drainage indicates bleeding, and this
removed and will be unable to speak. client should be assessed immediately.
Communication is a high priority for 2. Pain is a priority but not over hemorrhaging.
this client. If the client is able to read 3. Clients with cancer of the lung have fatigue
and write, a Magic Slate or pad of paper and are short of breath; these are expected
should be provided. If the client is illiter- findings.
ate, the nurse and the client should de- 4. Clients with chronic lung problems are
velop a method of communication using taught pursed-lip breathing to assist in
pictures. expelling air. This type of breathing often
TEST-TAKING HINT: Questions addressing produces a whistling sound.
highest priority include all of the options TEST-TAKING HINT: If the test taker is not
being possible interventions, but only one is sure of the answer and airway compromise is
priority. Use Maslow’s hierarchy of needs to not one of the answer options, then an op-
answer the question—safety is priority. tion dealing with bleeding is the best choice.
Content – Surgical: Integrated Nursing Process – Content – Surgical: Integrated Nursing Process – ­
Implementation: Client Needs – Physiological Integrity, Implementation: Client Needs – Safe Effective Care
Physiological Adaptation: Cognitive Level – Synthesis: Environment, Management of Care: Cognitive Level –
Concept – Cellular Regulation. Synthesis: Concept – Nursing Roles.

06_Colgrove_Ch06.indd 220 01/06/16 5:16 PM


C hapter 6  R espiratory D isorders 221

67. 1. Chest tubes are part of the nursing educa- TEST-TAKING HINT: The test taker must try to
tion curriculum. The new graduate should match the problem with the answer choices.
be capable of caring for this client or at This would eliminate options “1” and “4.”
least knowing when to get assistance. Shaving himself is a positive action, and the
2. This client is in respiratory compromise, and question asks for an action indicating a dis-
an experienced nurse should care for the client. turbance in body image.
3. A client with multiple fistulas in the neck area is Content – Surgical: Integrated Nursing Process –
at high risk for airway compromise and should Evaluation: Client Needs – Psychosocial Integrity: Cogni-
be assigned to a more experienced nurse. tive Level – Synthesis: Concept – Cellular Regulation.
4. This client is at risk for developing edema
70. 1. This is an example of nonmalfeasance,
of the neck area and should be cared for by
where the nurse “does no harm.” In
a more experienced nurse.
­attempting to discuss the client’s refusal,
TEST-TAKING HINT: The question is asking for the nurse is not trying to influence the
the least compromised or most stable client. client; the nurse is merely attempting to
The client in option “1” already has chest listen therapeutically.
tubes in place and is presumably stable. 2. This is an example of paternalism, telling the
Content – Nursing Management: Nursing Process – client what he should do, and it is also coer-
Planning: Client Needs – Safe Effective Care Environment, cion, an unethical action.
Management of Care: Cognitive Level – Synthesis: 3. This is an example of beneficence, “to do
Concept – Nursing Roles. good”; it is a positive action and a step up
68. 1. Wound infection is a concern, but in the list from nonmalfeasance.
of the answer options it is not the highest 4. This is an example of beneficence.
priority. A wound infection can be treated, TEST-TAKING HINT: If the test taker were not
but a client who is not breathing is in a life- aware of the terms of ethical principles, then
threatening situation and the problem must dissecting the word “nonmalfeasance” into
be addressed immediately. its portions might help. Non- means “noth-
2. Hemorrhage is normally a priority, but bleed- ing” or “none,” and mal- means “bad,” so “no
ing is not priority over not breathing. bad ­action” could be inferred. This would
3. Respiratory distress is the highest priority. eliminate option “2.”
Hemorrhaging and infection are serious Content – Surgical: Integrated Nursing Process –
problems, but airway is priority. Implementation: Client Needs – Safe Effective Care Envi-
4. Knowledge deficit is the lowest on this prior- ronment, Management of Care: Cognitive Level – Compre-
ity list. It is a psychosocial problem, and these hension: Concept – Cellular Regulation.
problems rank lower in priority than physi-
71. 1. The cancer may have grown, but this
ological ones.
would not be indicated by the type of pain
TEST-TAKING HINT: In cardiopulmonary described.
resuscitation the steps are airway, breath- 2. Painful swallowing is caused by esophageal
ing, and then ­circulation. This concurs with irritation.
Maslow’s hierarchy of needs, which places 3. Most cancers are not painful unless obstruct-
oxygenation at the top of the hierarchy. ing an organ or pressing on a nerve. This is
Content – Medical: Integrated Nursing Process – not what is being described.
Diagnosis: Client Needs – Safe Effective Care Environ- 4. The esophagus is extremely radiosensi-
ment, Management of Care: Cognitive Level – Analysis: tive, and esophageal ulcerations are com-
Concept – Cellular Regulation. mon. The pain can become so severe the
69. 1. This request indicates the client is accepting client cannot swallow saliva. This is a
the situation and trying to deal with it. situation in which the client will be admit-
2. Placing a towel over the mirror indicates ted to the hospital for IV narcotic pain
the client is having difficulty looking at medication and possibly total parenteral
his reflection, a body-image problem. nutrition.
3. In attempting to shave himself, the client is TEST-TAKING HINT: The test taker must re-
participating in self-care activities and also member not to jump to conclusions and to
must look at his neck in the mirror, both realize what a word is actually saying. Swal-
good steps toward adjustment. lowing is an action involving the esophagus,
4. Neck and shoulder exercises are done to so the best choice would be either option
strengthen the remaining musculature, but “1” or option “4,” both of which contain the
they have nothing to do with body image. word “esophagus.”

06_Colgrove_Ch06.indd 221 01/06/16 5:16 PM


222 M ed -S urg S uccess

Content – Surgical: Integrated Nursing Process – 74. 1. The plasma D-dimer test is highly spe-
Assessment: Client Needs – Physiological Integrity, Physi- cific for the presence of a thrombus; an
ological Adaptation: Cognitive Level – Synthesis: elevated D-dimer indicates a thrombus
Concept – Cellular Regulation. formation and lysis. This result would
72. 1. This surgery removed the client’s vocal cords, require a CT or V/Q scan to then confirm
so he or she will not be able to speak again the diagnosis.
unless the client learns esophageal speech, 2. An ABG evaluates oxygenation level, but it
uses an electric larynx, or has a surgically does not diagnose a pulmonary embolus (PE).
created transesophageal puncture. 3. A CXR shows pulmonary infiltration and
2. The client breathes through a stoma in pleural effusions, but it does not diagnose
the neck. Care should be taken not to a PE.
allow water to enter the stoma. 4. An MRI is a noninvasive test that detects
3. The stoma should be cleaned, but petroleum- a deep vein thrombosis (DVT), but it does
based products should not be allowed near not diagnose a pulmonary embolus. A com-
the stoma. A petroleum-based product is con- puted tomography (CT) scan or ventilation/
traindicated because it is not water-soluble, perfusion (V/Q) scan would be used to
could contribute to an occlusion, and is confirm the diagnosis.
flammable. TEST-TAKING HINT: The key to answering this
4. The client has lost the use of the nasal question is “confirms the diagnosis.” The
passages to humidify the inhaled air, and test taker should eliminate options “2” and
artificial humidification is useful until the “3” based on the fact these are diagnostic
client’s body adapts to the change. tests used for many disease processes and
5. Special equipment is available for clients conditions.
who cannot hear or speak. Content – Medical: Integrated Nursing Process –
TEST-TAKING HINT: Option “1” can be elimi- Diagnosis: Client Needs – Physiological Integrity, Reduction
nated as a false promise that would under- of Risk Potential: Cognitive Level – Analysis:
mine the nurse–client relationship. Concept – Oxygenation.
Content – Surgical: Integrated Nursing Process – 75. 1. This is a sign of a deep vein thrombosis,
Planning: Client Needs – Physiological Integrity, Physi- which is a precursor to a pulmonary em-
ological Adaptation: Cognitive Level – Synthesis: bolism, but it is not a sign of a pulmonary
Concept – Cellular Regulation. embolism.
2. The most common signs of a pulmonary
embolism are sudden onset of chest pain
Pulmonary Embolus
when taking a deep breath and shortness
of breath.
73. 880 units.
3. These are signs of a myocardial infarction.
If there are 20,000 units of heparin in 500
4. These could be signs of pneumonia or other
mL of D5W, there are 40 units in each mL:
pulmonary complications but not specifically
20,000 ÷ 500 = 40 units
a pulmonary embolism.
If 22 mL are infused per hour, then 880 units
of heparin are infused each hour: TEST-TAKING HINT: The key to selecting
40 × 22 = 880 option “2” as the correct answer is sudden
onset. The test taker would need to note
TEST-TAKING HINT: The test taker must know
“left-sided” in option “3” to eliminate this as
how to calculate heparin drips from two as-
a possible correct answer, and option “4” is
pects: the question may give the mL/hr and
nonspecific for a
the test taker has to determine units/hr, or
pulmonary embolism.
the question may give units/hr and the test
taker has to determine mL/hr. Remember to Content – Medical: Integrated Nursing Process –
learn how to use the drop-down calculator Assessment: Client Needs – Physiological Integrity, Reduc-
tion of Risk Potential: Cognitive Level – Analysis:
on the computer. During the NCLEX-RN,
Concept – Oxygenation.
the test taker can request an erase slate.
Content – Medical: Integrated Nursing Process – 76. 1. The ABGs are within normal limits and
Implementation: Client Needs – Physiological Integrity, would not warrant immediate intervention.
Pharmacological and Parenteral Therapies: Cognitive 2. Occasional premature ventricular contrac-
Level – Application: Concept – Medication. tions are not unusual for any client and would
not warrant immediate intervention.

06_Colgrove_Ch06.indd 222 01/06/16 5:16 PM


C hapter 6  R espiratory D isorders 223

3. The normal pulse oximeter reading is TEST-TAKING HINT: The test taker must know
93% to 100%. A reading of 90% indicates normal laboratory values; this is the only way
the client has an arterial oxygen level of the test taker will be able to answer this
around 60. question. The test taker should make a list
4. A urinary output of 800 mL over 12 hours in- of laboratory values that must be memorized
dicates an output of greater than 30 mL/hour for successful test taking.
and would not warrant immediate interven- Content – Medical: Integrated Nursing Process –
tion by the nurse. Implementation: Client Needs – Physiological Integrity,
TEST-TAKING HINT: This question is asking Pharmacological and Parenteral Therapies: Cognitive
the test taker to select abnormal, unex- Level – Application: Concept – Medication.
pected, or life-threatening assessment data 79. 1. Arterial blood gases would be included in the
in relationship to the client’s disease process. client problem “impaired gas exchange.”
A pulse oximeter reading of less than 93% 2. These assessment data monitor tissue
indicates severe hypoxia and requires imme- perfusion, which evaluates for decreased
diate intervention. cardiac output.
Content – Medical: Integrated Nursing Process – 3. This would be appropriate for the client
Assessment: Client Needs – Safe Effective Care Environ- problem “high risk for bleeding.”
ment, Management of Care: Cognitive Level – Synthesis: 4. The client should not be put in a position
Concept – Oxygenation. with the head lower than the legs because this
77. 1. The intravenous anticoagulant heparin will would increase difficulty breathing.
be administered immediately after diagnosis TEST-TAKING HINT: The test taker must think
of a PE, not oral anticoagulants. about which answer option addresses the
2. The client’s respiratory system will be problem of the heart’s inability to pump
assessed, not the gastrointestinal system. blood. Decreased blood to the extremities
3. A thoracentesis is used to aspirate fluid from results in cyanosis and cold extremities.
the pleural space; it is not a treatment for a Content – Medical: Integrated Nursing Process –
PE. Diagnosis: Client Needs – Safe Effective Care Environ-
4. Bedrest reduces the risk of another clot ment, Management of Care: Cognitive Level – Analysis:
becoming an embolus leading to a pulmo- Concept – Oxygenation.
nary embolus. Bedrest reduces metabolic
80. 1. Heparin is administered during throm-
demands and tissue needs for oxygen in
bolytic therapy, and the antidote is prot-
the lungs.
amine sulfate and should be available to
TEST-TAKING HINT: The test taker must be reverse the effects of the anticoagulant.
aware of adjectives such as “oral” in option 2. Firm pressure reduces the risk for bleeding
“1,” which makes this option incorrect. The into the tissues.
test taker should apply the body system of 3. Obvious (overt) as well as hidden (covert)
the disease process to eliminate option “2” signs of bleeding should be assessed for.
as a correct answer. 4. Invasive procedures increase the risk of
Content – Medical: Integrated Nursing Process – tissue trauma and bleeding.
Implementation: Client Needs – Physiological Integrity, 5. Stool softeners help prevent constipa-
Physiological Adaptation: Cognitive Level – Application: tion and straining, which may precipitate
Concept – Oxygenation. bleeding from hemorrhoids.
78. 1. The client would not be experiencing abnor- TEST-TAKING HINT: Thrombolytic therapy is
mal bleeding with this INR. ordered to help dissolve the clot resulting in
2. This is the antidote for an overdose of antico- the PE. Therefore, all nursing interventions
agulant and the INR does not indicate this. should address bleeding tendencies. The test
3. A therapeutic INR is 2 to 3; therefore, taker must select all interventions applicable
the nurse should administer the in these ­alternative questions.
medication. Content – Medical: Integrated Nursing Process –
4. There is no need to increase the dose; this Implementation: Client Needs – Safe Effective Care Envi-
result is within the therapeutic range. ronment, Management of Care: Cognitive Level –
Application: Concept – Oxygenation.

06_Colgrove_Ch06.indd 223 01/06/16 5:16 PM


224 M ed -S urg S uccess

81. 1. The client should use a soft-bristle tooth- 83. 1. An INR of 2 to 3 is therapeutic; therefore,
brush to reduce the risk of bleeding, so the the nurse would administer this medication.
teaching is not effective. 2. This is an elevated blood glucose level; there-
2. This is appropriate for a client with a fore, the nurse should administer the insulin.
­mechanical valve replacement, not a client 3. A normal PTT is 39 seconds, and for hepa-
­receiving anticoagulant therapy, so the teach- rin to be therapeutic, it should be 1.5 to
ing is not effective. 2 times the normal value, or 58 to 78. A
3. Aspirin, enteric-coated or not, is an antiplate- PTT of 98 indicates the client is not clot-
let, which may increase bleeding tendencies ting and the medication should be held.
and should be avoided, so the teaching is not 4. This is a normal blood pressure and the nurse
effective. should administer the medication.
4. The client should wear a Medic Alert TEST-TAKING HINT: This question is asking
band at all times so that, if any accident or the test taker to select a distracter with
situation occurs, the health-care providers assessment data that are unsafe for admin-
will know the client is receiving antico- istering the medication. The test taker must
agulant therapy. The client understands know normal laboratory values to administer
the teaching. medication safely.
TEST-TAKING HINT: This is a higher level Content – Medical: Integrated Nursing Process –
question in which the test taker must know Implementation: Client Needs – Physiological Integrity,
clients with a pulmonary embolus are pre- Pharmacological and Parenteral Therapies: Cognitive
scribed anticoagulant therapy on discharge Level – Application: Concept – Medication.
from the hospital. If the test taker had no
84. 1. The client needs oxygen, but the nurse can
idea of the answer, the option stating “wear a
intervene to help the client before applying
Medic Alert band” is a good choice because
oxygen.
many disease processes require the client to
2. Placing the client in this position facili-
take long-term medication and a health-care
tates maximal lung expansion and reduces
provider should be aware of this.
venous return to the right side of the
Content – Medical: Integrated Nursing Process – heart, thus lowering pressures in the
Evaluation: Client Needs – Physiological Integrity, Physi- ­pulmonary vascular system.
ological Adaptation: Cognitive Level – Synthesis: Concept –
3. A pulse oximeter reading is needed, but it is
Oxygenation.
not the first intervention.
82. 1. Increasing fluids will help increase fluid 4. Assessing the client is indicated, but it is not
volume, which will, in turn, help prevent the first intervention in this situation.
the development of deep vein thrombosis, TEST-TAKING HINT: The test taker must select
the most common cause of PE. the option that will directly help the client
2. Pulmonary emboli are not caused by athero- breathe easier. Therefore, assessment is not
sclerosis; this is not an appropriate discharge the first intervention and option “4” can be
instruction for a client with a pulmonary eliminated as the correct answer. When the
embolus. client is in distress, do not assess.
3. Infection does not cause a PE; this is not an Content – Medical: Integrated Nursing Process –
appropriate teaching instruction. Implementation: Client Needs – Safe Effective Care Envi-
4. Pneumonia and flu do not cause pulmonary ronment, Management of Care: Cognitive Level –
embolism. Synthesis: Concept – Oxygenation.
TEST-TAKING HINT: The test taker must
know deep vein thrombosis is the most
common cause of pulmonary embolus and Chest Trauma
preventing dehydration is an important in-
tervention. The test taker can attempt to 85. 1. The client with pneumothorax has absent
eliminate answers by trying to determine breath sounds and tachypnea.
which disease process is appropriate for the 2. Unequal lung expansion and dyspnea
intervention. indicate a pneumothorax.
3. Consolidation occurs when there is no air mov-
Content – Medical: Integrated Nursing Process –
Planning: Client Needs – Physiological Integrity, Physi-
ing through the alveoli, as in pneumonia; frothy
ological Adaptation: Cognitive Level – Synthesis: sputum occurs with congestive heart failure.
Concept – Oxygenation. 4. Barrel chest and polycythemia are signs of
chronic obstructive pulmonary disease.

06_Colgrove_Ch06.indd 224 01/06/16 5:16 PM


C hapter 6  R espiratory D isorders 225

TEST-TAKING HINT: The test taker can use should be eliminated based on being a very
“chest trauma” or “pneumothorax” to help rude and threatening comment.
select the correct answer. Both of these terms Content – Medical: Integrated Nursing Process –
should cause the test taker to select option Implementation: Client Needs – Physiological Integrity,
“2” because unequal chest expansion would Physiological Adaptation: Cognitive Level – Application:
result from trauma. Concept – Oxygenation.
Content – Medical: Integrated Nursing Process – 88. 1. Keeping the drainage system lower than the
Assessment: Client Needs – Physiological Integrity, Reduc- chest promotes drainage and prevents reflux.
tion of Risk Potential: Cognitive Level – Analysis:
2. The chest tube system can function as a
Concept – Trauma.
result of gravity and does not have to be
86. 1. A STAT chest x-ray would not be needed to attached to suction. Keeping it attached to
determine why there is no fluctuation in the suction could cause the client to trip and
water-seal compartment. fall. Therefore, this is a safety issue and
2. Increasing the amount of wall suction does the nurse should intervene and explain
not address why there is no fluctuation in the this to the UAP.
water-seal compartment. 3. Ambulation facilitates lung ventilation and ex-
3. The key to the answer is “2 hours.” The pansion; drainage systems are portable to al-
air from the pleural space is not able to low ambulation while chest tubes are in place.
get to the water-seal compartment, and 4. The client should ambulate, but getting up
the nurse should try to determine why. and using the bedside commode is better
Usually the client is lying on the tube, it is than staying in the bed, so no action would be
kinked, or there is a dependent loop. needed.
4. The stem does not state the client is in respi- TEST-TAKING HINT: “Warrants immediate in-
ratory distress, and a pulse oximeter reading tervention” means the test taker must iden-
detects hypoxemia but does not address any tify the situation in which the nurse should
fluctuation in the water-seal compartment. correct the action, demonstrate a skill, or
TEST-TAKING HINT: The test taker should somehow intervene with the UAP’s behavior.
apply the nursing process to answer the Content – Medical: Integrated Nursing Process –
question correctly. The first step in the Implementation: Client Needs – Safe Effective Care Environ-
nursing process is assessment, and “check” ment, Management of Care: Cognitive Level – Synthesis:
(option “3”) is a word that can be used syn- Concept – Nursing Roles.
onymously for “assess.” Monitoring (op- 89. 1. The health-care provider will have to be noti-
tion “4”) is also assessing, but the test taker fied, but this is not the first intervention. Air
should not check a diagnostic test result be- must be prevented from entering the pleural
fore caring for the client. space from the outside atmosphere.
Content – Medical: Integrated Nursing Process – 2. The client should breathe regularly or take
Implementation: Client Needs – Physiological Integrity, deep breaths until the tubes are reinserted.
Reduction of Risk Potential: Cognitive Level – Application:
3. The nurse must take action and prevent air
Concept – Oxygenation.
from entering the pleural space.
87. 1. The client must take deep breaths to help 4. Taping on three sides prevents the de-
push the air out of the pleural space into velopment of a tension pneumothorax by
the water-seal drainage, and deep breaths inhibiting air from entering the wound
will help prevent the client from develop- during inhalation but allowing it to escape
ing pneumonia or atelectasis. during exhalation.
2. The client must take deep breaths; shallow TEST-TAKING HINT: The words “implement
breaths could lead to complications. first” in the stem of the question indicate to
3. Deep breaths must be taken to prevent the test taker that possibly more than one (1)
complications. intervention could be warranted in the situ-
4. This is a cruel intervention; the nurse can ation but only one (1) is implemented first.
medicate the client and then encourage deep Remember, do not select assessment first
breathing. without reading the question. If the client is in
TEST-TAKING HINT: If the test taker reads any type of crisis, then the nurse should first
options “2” and “3” and notices that both do something to help the client’s situation.
reflect the same idea—namely, that deep Content – Medical: Integrated Nursing Process –
breaths are not necessary—then both can Implementation: Client Needs – Safe Effective Care Envi-
either be eliminated as incorrect answers or ronment, Management of Care: Cognitive Level – Analysis:
kept as possible correct answers. Option “4” Concept – Oxygenation.

06_Colgrove_Ch06.indd 225 01/06/16 5:16 PM


226 M ed -S urg S uccess

90. 1. This statement describes a spontaneous 2. At three (3) days postinsertion, no fluctua-
pneumothorax. tion (tidaling) indicates the lung has reex-
2. This statement describes an open panded, which indicates the treatment has
pneumothorax. been effective.
3. This describes a tension pneumothorax. 3. Blood in the drainage bottle is expected for a
It is a medical emergency requiring hemothorax but does not indicate the chest
immediate intervention to preserve life. tubes have reexpanded the lung.
4. This is called an iatrogenic pneumothorax, 4. Taking a deep breath without pain is
which also may be caused by thoracentesis or good, but it does not mean the lungs have
lung biopsy. A tension pneumothorax could reexpanded.
occur from this procedure, but the statement TEST-TAKING HINT: The test taker must be
does not describe a tension pneumothorax. knowledgeable about chest tubes to be able
TEST-TAKING HINT: The test taker must to answer this question. The test taker must
­always be clear about what the question is know the normal time frame and what is
asking before answering the question. If the expected for each compartment of the chest
test taker can eliminate options “1” and “2” tube drainage system.
and can’t decide between options “3” and Content – Medical: Integrated Nursing Process –
“4,” the test taker must go back to the stem Assessment: Client Needs – Physiological Integrity, Reduc-
and clarify what the question is asking. tion of Risk Potential: Cognitive Level – Analysis:
Content – Medical: Integrated Nursing Process – Concept – Oxygenation.
Diagnosis: Client Needs – Physiological Integrity, Physi- 93. 1. The client should be in the high Fowler’s
ological Adaptation: Cognitive Level – Comprehension:
position to facilitate lung expansion.
Concept – Promoting Health.
2. The system must be patent and intact
91. 1. Checking to see if someone has increased to function properly.
the suction rate is the simplest and a 3. The client can have bathroom privileges, and
noninvasive action for the nurse to imple- ambulation facilitates lung ventilation and
ment; if it is not on high, then the nurse expansion.
must check to see if the problem is with 4. Looping the tubing prevents direct pres-
the client or the system. sure on the chest tube itself and keeps tub-
2. No fluctuation (tidaling) would cause the ing off the floor, addressing both a safety
nurse to assess the tubing for a blood clot. and a potential clogging of the tube.
3. The tube is milked to help dislodge a blood 5. Subcutaneous emphysema is air under the
clot that may be blocking the chest tube, skin, which is a common occurrence at the
causing no fluctuation (tidaling) in the water- chest tube insertion site.
seal compartment. The chest tube is never TEST-TAKING HINT: The test taker should be
stripped, which creates a negative air pressure careful with adjectives. In option “1,” the
and could suck lung tissue into the chest tube. word “low” makes it incorrect; in option “3,”
4. Encouraging the client to cough forcefully the word “strict” makes this option incorrect.
will help dislodge a blood clot blocking the
Content – Medical: Integrated Nursing Process –
chest tube, causing no fluctuation (tidaling) in Implementation: Client Needs – Safe Effective Care Environ-
the water-seal compartment. ment, Management of Care: Cognitive Level – Application:
TEST-TAKING HINT: The test taker should al- Concept – Oxygenation.
ways think about assessing the client if there
94. 1. This pulse oximeter reading indicates the
is a problem and the client is not in immedi-
client is hypoxic and therefore is not stable
ate danger. This would cause the test taker
and should be assigned to an RN.
to eliminate options “3” and “4.” If the test
2. This H&H are very low; therefore, the client is
taker thinks about bubbling, he or she should
not stable and should be assigned to an RN.
know it has to do with suctioning.
3. Jugular vein distention and hypotension are
Content – Medical: Integrated Nursing Process – signs of a tension pneumothorax, which is a
Implementation: Client Needs – Safe Effective Care Envi- medical emergency, and the client should be
ronment, Management of Care: Cognitive Level –
assigned to an RN.
Application: Concept – Oxygenation.
4. A client two (2) hours post–bronchoscopy
92. 1. This is an expected finding in the suction procedure could safely be assigned to
compartment of the drainage system, indicat- an LPN.
ing adequate suctioning is being applied.

06_Colgrove_Ch06.indd 226 01/06/16 5:16 PM


C hapter 6  R espiratory D isorders 227

TEST-TAKING HINT: The test taker must un- Acute Respiratory Distress Syndrome
derstand that the LPN should be assigned (ARDS)
the least critical client or the client who
is stable and not exhibiting any complica- 97. 1. The opposite side rail should be elevated
tions secondary to the admitting disease or so the client will not fall out of the bed.
condition. Safety is priority, the nurse should dem-
Content – Medical: Integrated Nursing Process – onstrate the proper way to bathe a client
Planning: Client Needs – Safe Effective Care Environ- in the bed.
ment, Management of Care: Cognitive Level – Synthesis:
2. The bed should be at a comfortable height
Concept – Nursing Roles.
for the UAP to bathe the client, not in the
95. 1. The nurse should gather a thoracotomy tray lowest position.
and the chest tube drainage system and take 3. The UAP can bathe a client without assis-
it to the client’s bedside, but it is not the first tance if the client’s safety can be ensured.
intervention. 4. The UAP is not ensuring the client’s safety
2. The insertion of a chest tube is an because the opposite side rail is not elevated
invasive procedure and requires informed to prevent the client from falling out of
consent. Without a consent form, this the bed.
procedure should not be done on an alert TEST-TAKING HINT: Although the test taker
and oriented client. should always be concerned with the disease
3. This is a correct position to place the client process of the client, many times the answer
in for a chest tube insertion, but it is not the may be selected based on basic nursing skills
first intervention. not requiring an understanding of the client’s
4. The health-care provider will discuss the pro- disease process.
cedure with the client, then informed consent Content – Medical: Integrated Nursing Process –
must be obtained, and the nurse can do fur- Implementation: Client Needs – Safe Effective Care Envi-
ther teaching. ronment, Management of Care: Cognitive Level –
TEST-TAKING HINT: The test taker must know Synthesis: Concept – Oxygenation.
invasive procedures require informed con- 98. 1. Maintaining ventilator settings and check-
sent, and legally it must be obtained first be- ing to ensure they are specifically set as pre-
fore anyone can touch the client. scribed is appropriate, but it is not the first
Content – Medical: Integrated Nursing Process – intervention.
Implementation: Client Needs – Safe Effective Care Envi- 2. Making sure alarms are functioning properly
ronment, Management of Care: Cognitive Level –
is appropriate, but checking a machine is not
Synthesis: Concept – Trauma.
priority.
96. 1. Assessment of the lung sounds could indi- 3. Assessment is the first part of the
cate the client’s lung has reexpanded be- nursing process and is the first interven-
cause it has been three (3) days since the tion the nurse should implement when
chest tube has been inserted. caring for a client on a ventilator.
2. This should be done to ensure the lung has 4. Monitoring laboratory results is an appropri-
reexpanded, but it is not the first intervention. ate intervention for the client on a ventilator,
3. The HCP will need to be notified so the but monitoring laboratory data is not the
chest tube can be removed, but it is not the priority intervention.
first intervention. TEST-TAKING HINT: The test taker should
4. This situation needs to be documented, but it apply the nursing process, which identifies
is not the first intervention. assessment as the first step. Therefore, if the
TEST-TAKING HINT: When the stem asks the test taker is not sure of the answer, the best
test taker to identify the first intervention, educated choice is to select an option ad-
all four (4) answer options could be interven- dressing assessment data.
tions appropriate for the situation, but only Content – Medical: Integrated Nursing Process –
one (1) is the first intervention. Remember Implementation: Client Needs – Safe Effective Care Envi-
to apply the nursing process: the first step is ronment, Management of Care: Cognitive Level –
assessment. Synthesis: Concept – Oxygenation.
Content – Medical: Integrated Nursing Process –
Implementation: Client Needs – Safe Effective Care Envi-
ronment, Management of Care: Cognitive Level –
Synthesis: Concept – Assessment.

06_Colgrove_Ch06.indd 227 01/06/16 5:16 PM


228 M ed -S urg S uccess

99. 1. The classic sign of ARDS is decreased 3. Asymmetrical chest expansion indicates
arterial oxygen level (Pao2) while admin- the client has had a pneumothorax, which
istering high levels of oxygen; the oxygen is a complication of mechanical ventilation.
is unable to cross the alveolar membrane. 4. An increased heart rate does not indicate a
2. These are early signs of ARDS, but they complication; this could result from numer-
could also indicate pneumonia, atelectasis, ous reasons, not specifically because of the
and other pulmonary complications, so they ventilator.
do not confirm the diagnosis of ARDS. TEST-TAKING HINT: The test taker should
3. Clear breath sounds and the oxygen satura- be looking for an answer option describing
tion indicate the client is not experiencing abnormal, unexpected, or life-threatening
any respiratory difficulty or compromise. data because of the word “complication” in
4. These are signs of congestive heart failure; the stem. Understanding medical suffixes
ARDS is noncardiogenic (without signs of and prefixes could help the test taker select
cardiac involvement) pulmonary edema. the correct answer. Any time there is an
TEST-TAKING HINT: If the test taker does not “a” before the word, it means “without,” as
know the signs/symptoms of ARDS, the test in “asymmetrical,” which means “without
taker should eliminate option “2” because symmetry.”
this could be any respiratory disorder, op- Content – Medical: Integrated Nursing Process –
tion “3” because these are normal data, and Assessment: Client Needs – Physiological Integrity, Reduc-
option “4” because jugular vein distention tion of Risk Potential: Cognitive Level – Analysis:
usually occurs with heart problems. Concept – Oxygenation.
Content – Medical: Integrated Nursing Process – 102. 1. These arterial blood gases are within
Assessment: Client Needs – Physiological Integrity, Reduc- normal limits, and, therefore, the nurse
tion of Risk Potential: Cognitive Level – Analysis:
should not take any action except to
Concept – Oxygenation.
continue to monitor the client.
100. 1. This is an incorrect statement. ARDS has 2. The nurse would recommend deep breaths
multiple etiologies, such as hemorrhagic and coughing if the client’s ABGs revealed
shock, septic shock, drug overdose, burns, respiratory acidosis.
and near-drowning. Many people with near- 3. Sodium bicarbonate is administered when
drowning do not develop ARDS. the client is in metabolic acidosis.
2. The layperson may not know what the term 4. This is a normal ABG and the respiratory
alveoli means, and the near-drowning is the therapist does not need to be notified.
initial insult that caused the ARDS. TEST-TAKING HINT: This question requires
3. This is a basic layperson’s terms explana- the test taker to know normal arterial blood
tion of ARDS and explains why the client gas results: pH 7.35 to 7.45, Pao2 80 to
is having trouble breathing. 100, Paco2 35 to 45, Hco3 22 to 26. The
4. Smoking does not increase the risk of devel- test taker must know how to evaluate the
oping ARDS. The etiology is unknown, but results.
an initial insult occurs 24 to 48 hours before Content – Medical: Integrated Nursing Process –
the development of ARDS. Implementation: Client Needs – Safe Effective Care
TEST-TAKING HINT: The test taker should se- Environment, Management of Care: Cognitive Level –
lect the answer option presenting facts and Application: Concept – Oxygenation.
easiest for the client to understand. The test 103. 1. Alternating the ET tube position will help
taker should not select the distracter with prevent a pressure ulcer on the client’s
medical jargon the client may not under- tongue and mouth.
stand. The test taker as a rule can eliminate 2. A CXR is performed immediately after
any distracter using medical jargon in the insertion of the ET tube but not daily.
answer. 3. The cuff should be inflated but no more
Content – Medical: Integrated Nursing Process – than 25 cm H2O to ensure no air leakage
Implementation: Client Needs – Safe Effective Care and must be checked every four (4) to eight
Environment, Management of Care: Cognitive Level –
(8) hours, not daily.
Application: Concept – Oxygenation.
4. The ET tube should be secure to ensure
101. 1. A urine output of 30 mL/hr indicates the it does not enter the right main bronchus.
kidneys are functioning properly or 120 mL The ET tube should be one (1) inch above
in four (4) hours. the bifurcation of the bronchi.
2. This indicates the client is being adequately
oxygenated.

06_Colgrove_Ch06.indd 228 01/06/16 5:16 PM


C hapter 6  R espiratory D isorders 229

TEST-TAKING HINT: The test taker must be Content – Medical: Integrated Nursing Process –
knowledgeable about ventilator care. Radia- Implementation: Client Needs – Safe Effective Care
tion is dangerous; therefore, a daily CXR Environment, Management of Care: Cognitive Level –
(option “2”) may be eliminated as the cor- Synthesis: Concept – Oxygenation.
rect answer. 106. 1. Altered level of consciousness is the
Content – Medical: Integrated Nursing Process – earliest sign of hypoxemia.
Planning: Client Needs – Physiological Integrity, Basic 2. Urine output of less than 30 mL/hr indi-
Care and Comfort: Cognitive Level – Synthesis: cates decreased cardiac output, which re-
Concept – Oxygenation. quires immediate intervention; it should be
104. 1. Unless the initial insult is an infection, an ami- assessed every one (1) or two (2) hours, not
noglycoside antibiotic would not be a medica- once during a shift.
tion the nurse would anticipate being ordered. 3. The client is at risk for complications
2. Surfactant therapy may be prescribed to of immobility; therefore, the nurse
reduce the surface tension in the alveoli. should turn the client at least every two
The surfactant helps maintain open (2) hours to prevent pressure ulcers.
alveoli, decreases the work of breathing, 4. The client is at risk for fluid volume
improves compliance, and helps prevent overload, so the nurse should monitor
atelectasis. and maintain the fluid intake.
3. A potassium cation, such as Kayexalate, helps 5. Fowler’s position facilitates lung ex-
remove potassium from the bloodstream in pansion and reduces the workload of
the gastrointestinal tract and would not be breathing.
prescribed for a client with ARDS. TEST-TAKING HINT: The client with ARDS
4. NSAIDs are under investigation for treating is critically ill, and interventions should
ARDS because they block the inflammatory address complications of immobility, de-
response, but the nurse should not anticipate creased cardiac output, and respiratory dis-
this being prescribed by the health-care tress. Remember, how often an intervention
provider. is implemented is important when selecting
TEST-TAKING HINT: Acute respiratory dis- the correct answer for the question. More
tress syndrome (ARDS) would indicate a than one (1) answer is possible in these
problem with the lungs; if the test taker alternate-type questions.
knew surfactant is needed by the lungs to Content – Medical: Integrated Nursing Process –
expand, this would be an appropriate selec- Implementation: Client Needs – Safe Effective Care
tion as the correct answer. Environment, Management of Care: Cognitive Level –
Application: Concept – Oxygenation.
Content – Medical: Integrated Nursing Process –
Planning: Client Needs – Physiological Integrity, Pharma- 107. 1. Not smoking is vital to prevent further
cological and Parenteral Therapies: Cognitive Level – lung damage.
Synthesis: Concept – Medication. 2. The client should get vaccines to help
105. 1. The nurse must first address the client’s prevent further episodes of serious respira-
acute respiratory distress and then notify tory distress.
other members of the multidisciplinary team. 3. Avoiding alcohol intake is appropriate for
2. If the ventilator system malfunctions, many serious illnesses, but it is not the most
the nurse must ventilate the client with important when discussing ARDS.
a manual resuscitation (Ambu) bag until 4. It usually takes about six (6) months to
the problem is resolved. ­recover maximal respiratory function after
3. The nurse must first address the client’s ARDS.
respiratory distress before requesting any TEST-TAKING HINT: ARDS means some-
laboratory data. thing is wrong with the respiratory system.
4. Assessment is not always priority. In this Therefore, the test taker should
situation, the client is in obvious acute respi- select an answer option addressing lungs
ratory distress; therefore, the nurse needs to and possible lung damage.
intervene to help the client breathe. Content – Medical: Integrated Nursing Process –
TEST-TAKING HINT: When the question asks Planning: Client Needs – Physiological Integrity, Physi-
the test taker to select the first intervention ological Adaptation: Cognitive Level – Synthesis:
and the client is in a life-threatening situa- Concept – Oxygenation.
tion, the nurse should ­select an intervention
that directly helps the client.

06_Colgrove_Ch06.indd 229 01/06/16 5:16 PM


230 M ed -S urg S uccess

108. 1. When peak airway pressure is increased, 3. The symptoms are seen in clients diagnosed
the nurse should implement the inter- with COPD. Oxygenation is the highest
vention least invasive for the client. This priority. Activity intolerance can be an issue
alarm goes off with a plugged airway, but is not priority.
“bucking” in the ventilator, ­decreasing 4. The symptoms are seen in clients
lung compliance, kinked tubing, or diagnosed with COPD. Oxygenation
pneumothorax. is the highest priority.
2. The alarm may indicate the client needs TEST-TAKING HINT: The test taker should
suctioning, but the nurse should always do use Maslow’s hierarchy of needs to answer
the least invasive procedure when trouble- this question. The lowest level of the hierar-
shooting a ventilator alarm. chy pyramid is physiological needs; the cli-
3. The lip line on the ET tube determines how ent will die quickly if tissues do not receive
far the ET tube is in the trachea. It should oxygen.
always stay at the same number, but it would Content – Medical: Integrated Nursing Process –
not have anything to do with the ventilator Planning: Client Needs – Physiological Integrity, Reduc-
alarms. tion of Risk Potential: Cognitive Level – Analysis:
4. This may be needed, but the nurse should not Concept – Oxygenation.
sedate the client unless absolutely necessary.
111. 1. This is the second order to implement; an
TEST-TAKING HINT: If the test taker has no
IVPB cannot be administered without an IV
idea what peak airway pressure is, the test access.
taker should not give up on being able to
2. This is the third order to implement; the
figure out the correct answer. When some-
goal to initiate the IV antibiotic is one (1)
thing is going wrong with a machine, the to two (2) hours from when the order was
nurse should either assess or do something written by the HCP.
for the client. Always select the easiest and 3. Culture specimens should be obtained
least invasive intervention to help the client.
prior to the initiation of antibiotic
Content – Medical: Integrated Nursing Process – medication to prevent skewing of
Implementation: Client Needs – Safe Effective Care Envi- results.
ronment, Management of Care: Cognitive Level –
4. This is the fourth order for the nurse to
Synthesis: Concept – Oxygenation.
initiate.
109. 1. These symptoms indicate a respiratory TEST TAKING HINT: The test taker must be
and infection problem, not clotting. aware of basic steps in medication delivery.
2. Shortness of breath and a low-grade In the case of an antibiotic any or all cul-
fever indicate pneumonia; the oxygen- tures must be obtained prior to initiating
ation concept applies to this client. the antibiotic or the culture results will be
3. Shortness of breath and a low-grade skewed.
fever indicate pneumonia; the infection Content – Medical: Integrated Nursing Process –
concept applies to this client. Implementation: Client Needs – Physiological Integrity,
4. Perfusion does not apply for this client Reduction of Risk Potential: Cognitive Level – Analysis:
based on the presenting symptoms. Concept – Oxygenation/Infection.
5. Coping does not apply for this client based
on the presenting symptoms. 112. 1. The symptoms described indicate
Cushing’s syndrome, developed as a
TEST-TAKING HINT: The keys to answering
result of long-term steroid use. The
this question the client’s presenting symp-
steroid of choice for home administra-
toms and the vital signs.
tion is prednisone. The client must con-
Content – Medical: Integrated Nursing Process – tinue to receive a form of glucocorticoid
Planning: Client Needs – Physiological Integrity, Reduc- medication or the client may develop
tion of Risk Potential: Cognitive Level – Analysis:
symptoms of cortisol insufficiency.
Concept – Oxygenation/Infection.
2. The symptoms indicate steroid-induced
110. 1. The symptoms are seen in clients diagnosed Cushing’s syndrome, not a potential abuse
with COPD. Oxygenation is the highest situation.
priority. Mobility can be an issue but is not 3. Nothing indicates the client is immobile or
priority. has an issue with the coccyx.
2. The symptoms are seen in clients diagnosed 4. Whether or not the client sleeps with the
with COPD. Oxygenation is the highest HOB elevated does not address the de-
priority. Nutrition can be an issue but is not scribed symptoms.
priority.

06_Colgrove_Ch06.indd 230 01/06/16 5:16 PM


C hapter 6  R espiratory D isorders 231

TEST-TAKING HINT: The keys to answering TEST-TAKING HINT: The test taker should
this question are the client’s presenting not read into a question, such as option “4,”
symptoms; the test taker must be knowl- based on the knowledge that cultures take
edgeable of treatments for certain diseases 24 to 48 hours. Then the nurse would be
and the side effects of those treatments. aware that the client has been receiving an-
Content – Medical: Integrated Nursing Process – tibiotic therapy currently.
Assessment: Client Needs – Pharmacology, Reduction of Content – Medical: Integrated Nursing Process –
Risk Potential: Cognitive Level – Analysis: Concept – Implementation: Client Needs – Physiological Integrity,
Oxygenation. Reduction of Risk Potential: Cognitive Level – Analysis:
Concept – Oxygenation.
113. 1. The client should be on a form of glucocor-
ticosteroid medication. The nurse would not 115. 1. This chart graphic indicates the client is not
question administering this medication. processing the amount of fluid being taken
2. A client diagnosed with COPD and in. A steroid medication will not affect this
pneumonia would be placed on an situation.
antibiotic, but the client lists an allergy 2. Guaifenesin is an antitussive medication.
to cephalosporin antibiotics; ceftriaxone This medication will not affect fluid balance.
is a third generation cephalosporin. The 3. The client is not processing the amount
nurse should investigate further prior to of fluid he or she is receiving via oral or
administering the medication. parenteral route. The nurse should
3. Tessalon Perles is an antitussive cough request the HCP to change the fluid to
suppressant; the nurse would not question a saline lock. Cor pulmonale is heart
this medication. failure resulting from increased pressure
4. Continuous release morphine is prescribed in the lungs from COPD.
for the mild bronchodilating effect of mor- 4. The nurse should take action to prevent
phine over a sustained period of time. The further fluid overload.
nurse would not question administering this TEST-TAKING HINT: The test taker should
medication. recognize an imbalance between the intake
TEST-TAKING HINT: The keys to answering and output numbers and make a decision
this question is for the test taker to read based on this fact.
all portions of the graph (MAR) carefully. Content – Medical: Integrated Nursing Process –
Noting the allergy to a “ceph” medication Implementation: Client Needs – Physiological Integrity,
would have provided a clue as to choosing a Reduction of Risk Potential: Cognitive Level – Analysis:
“ceph” answer. Concept – Oxygenation.
Content – Medical: Integrated Nursing Process – 116. 1. The Pao2 level is very low; this client
Implementation: Client Needs – Pharmacology, Reduction should be placed on a ventilator. The
of Risk Potential: Cognitive Level – Analysis: Concept –
nurse should provide as much oxygen as
Oxygenation.
possible until this can be done.
114. 1. This client has an elevated platelet count, 2. The RRT is called when an individual
which is not an expected result for a cli- identifies a situation that requires imme-
ent diagnosed with pneumonia. The nurse diate intervention to prevent the client
should notify the HCP so that the client from going into an arrest situation.
can be evaluated for a comorbid problem. 3. Elevating the HOB allows for better lung
The WBC is high but it is expected in a expansion.
client diagnosed with pneumonia. 4. The nurse should not leave the client but
2. The only value that is abnormal is the Hco3 should direct care from the bedside.
level, which indicates that complete com- 5. The HCP should be notified of the cli-
pensation has occurred by the client’s body. ent’s status.
This is a good ABG report. There is no rea- TEST-TAKING HINT: The keys to answering
son to notify the HCP. this question are knowledge of normal val-
3. A client diagnosed with an exacerbation of ues. The nurse makes decisions based on
COPD would be on steroid medication, deviations from normal.
which can cause an increase in blood glucose. Content – Medical: Integrated Nursing Process –
4. Cultures take 24 to 48 hours to obtain results. Implementation: Client Needs – Physiological Integrity,
The client should already be on antibiotic Reduction of Risk: Concept – Oxygenation.
medication. The information provided did
not mention which antibiotics the client is on.

06_Colgrove_Ch06.indd 231 01/06/16 5:16 PM


232 M ed -S urg S uccess

117. 1. The nurse should set the flow rate for 119. 1. Chest tubes are painful; splinting the inser-
2 LPM (20%) because of the diagnosis tion site can help to lessen the pain.
of COPD. The client with COPD devel- 2. Coughing indicates the ability to move air
ops carbon dioxide narcosis, which does in and out of the lungs. This is not a critical
not allow the brain to recognize high issue.
levels of carbon dioxide as a stimulus to 3. This client with a lack of oxygenation
breathe. The only remaining stimulus at rest, blueness around the mouth, and
for breathing is oxygen hunger. Giving who is difficult to arouse indicates a
high levels of oxygen removes the client’s ­decrease in neurological functioning.
stimulus to breathe. 4. Dyspnea on exertion is not a critical issue.
2. The client should receive 2 LPM only. TEST-TAKING HINT: The key to answering
3. The client should receive 2 LPM only. this question is reading every word in the
4. The client should receive 2 LPM only. stem. “Critical” should indicate that a non–
TEST-TAKING HINT: The test taker must life-threatening issue cannot be the correct
recognize safety precautions in relation answer.
to therapies that are being administered. Content – Medical: Integrated Nursing Process –
Knowledge of basic pathophysiology of the Assessment: Client Needs – Physiological Integrity, Reduc-
brain’s respiratory center will help the test tion of Risk Potential: Cognitive Level – Analysis:
taker choose the correct option. Concept – Oxygenation.
Content – Medical: Integrated Nursing Process – 120. 1. The only clinical manifestation of
Implementation: Client Needs – Physiological Integrity, pneumonia in the elderly may be con-
Reduction of Risk Potential: Cognitive Level – Analysis:
fusion caused by lack of oxygen to the
Concept – Oxygenation.
brain. The nurse should assess the client
118. 1. The body has compensated for the abnor- for a respiratory cause for the new onset
mally high level of carbon dioxide (acid) in of confusion and wandering.
the blood by holding on to the base (Hco3) 2. The nurse should assist the resident in
and the pH is within normal range. This is returning to the room so an assessment can
an expected blood gas for the client with be performed but should not tell him to
COPD. return to the room without the nurse.
2. This pulse oximetry reading indicates an 3. This may be done, but assessment is first
arterial blood oxygen of less than 60. The priority.
client should be seen immediately to prevent 4. Pushing fluids may help to liquefy secre-
respiratory failure. tions, but assessment is first.
3. This client is receiving blood to correct the TEST-TAKING HINT: The test taker should
lower levels of H&H. use the nursing process to answer this first
4. A BNP of less than 100 is considered WNL. question. Assess is the first step of the nurs-
A BNP of 100 would not be a concern to ing process.
report for a client in stage 4 of heart failure. Content – Medical: Integrated Nursing Process –
TEST-TAKING HINT: The test taker must be Implementation: Client Needs – Physiological Integrity,
aware of critical laboratory and respiratory Reduction of Risk Potential: Cognitive Level – Analysis:
results to determine the correct answer to Concept – Oxygenation/Infection.
this question.
Content – Medical: Integrated Nursing Process –
Implementation: Client Needs – Physiological Integrity,
Reduction of Risk Potential: Cognitive Level – Analysis:
Concept – Oxygenation.

06_Colgrove_Ch06.indd 232 01/06/16 5:16 PM


C hapter 6  R espiratory D isorders 233

RESPIRATORY DISORDERS
COMPREHENSIVE EXAMINATION
1. Which diagnostic test should the nurse anticipate 6. The nurse suspects the client admitted with a
the health-care provider ordering to rule out near-drowning is developing acute respiratory
the diagnosis of asthma in clients diagnosed with distress syndrome (ARDS). Which data support
chronic obstructive pulmonary disease (COPD)? the nurse’s suspicion?
1. A bronchoscopy. 1. The client’s arterial blood gases are within
2. An immunoglobulin E. normal limits.
3. An arterial blood gas. 2. The client appears anxious, has dyspnea, and
4. A bronchodilator reversibility test. is tachypneic.
3. The client has intercostal retractions and is
2. Which statement indicates the client diagnosed
using accessory muscles.
with asthma needs more teaching concerning the
4. The client’s bilateral lung sounds have
medication regimen?
crackles and rhonchi.
1. “I will take Singulair, a leukotriene, every day
to prevent allergic asthma attacks.” 7. Which arterial blood gas (ABG) results support
2. “I need to use my Intal, cromolyn inhaler, the diagnosis of acute respiratory distress
15 minutes before I begin my exercise.” syndrome (ARDS) after the client has received
3. “I need to take oral glucocorticoids every day O2 at 10 LPM?
to prevent my asthma attacks.” 1. pH 7.38, Pao2 94, Paco2 44, Hco3 24.
4. “If I have an asthma attack, I need to use my 2. pH 7.46, Pao2 82, Paco2 34, Hco3 22.
albuterol, a beta2 agonist, inhaler.” 3. pH 7.48, Pao2 59, Paco2 30, Hco3 26.
4. pH 7.33, Pao2 94, Paco2 44, Hco3 20.
3. Which intervention should the emergency
department nurse implement first for the client 8. The nurse is planning the activities for the client
admitted for an acute asthma attack? diagnosed with asbestosis. Which activity should
1. Administer glucocorticoids intravenously. the nurse schedule at 0900 if breakfast is served
2. Administer oxygen 5 L per nasal cannula. at 0800?
3. Establish and maintain a 20-gauge saline lock. 1. Assist with the client’s bath and linen change.
4. Assess breath sounds every 15 minutes. 2. Administer an inhalation bronchodilator
treatment.
4. Which isolation procedure should be instituted
3. Provide the client with a one (1)-hour rest
for the client admitted to rule out severe acute
period.
respiratory syndrome (SARS)?
4. Have respiratory therapy perform chest
1. Airborne isolation.
physiotherapy.
2. Droplet isolation
3. Reverse isolation. 9. Which datum requires immediate intervention
4. Strict isolation. by the nurse for the client diagnosed with
asbestosis?
5. The client is admitted with a diagnosis of rule-out
1. The client develops an S3 heart sound.
severe acute respiratory syndrome (SARS). Which
2. The client has clubbing of the fingers.
information is most important for the nurse to ask
3. The client is fatigued in the afternoon.
related to this diagnosis?
4. The client has basilar crackles in all lobes.
1. Current prescription and over-the-counter
medication use. 10. Which clinical manifestation would the nurse
2. Dates of and any complications associated with assess in the client newly diagnosed with
recent immunizations. intrinsic lung cancer?
3. Any problems with recent or past use of blood 1. Dysphagia.
or blood products. 2. Foul-smelling breath.
4. Recent travel to mainland China, Hong Kong, 3. Hoarseness.
or Taiwan. 4. Weight loss.

06_Colgrove_Ch06.indd 233 01/06/16 5:16 PM


234 M ed -S urg S uccess

11. Which priority intervention should the nurse 16. Which statement indicates to the nurse the
implement for the client diagnosed with coal client diagnosed with sleep apnea needs further
workers’ pneumoconiosis? teaching?
1. Monitor the client’s intake and output. 1. “If I lose weight I may not need treatment for
2. Assess for black-streaked sputum. sleep apnea.”
3. Monitor the white blood cell count daily. 2. “The CPAP machine holds my airway open
4. Assess the client’s activity level every shift. with pressure.”
3. “The CPAP will help me stay awake during
12. Which statement indicates the client with
the day while I am at work.”
a total laryngectomy requires more
4. “It is all right to have a couple of beers
teaching concerning the care of the
because I have this CPAP machine.”
tracheostomy?
1. “I must avoid hair spray and powders.” 17. The nurse is preparing to administer influenza
2. “I should take a shower instead of a vaccines to a group of elderly clients in a long-
tub bath.” term care facility. Which client should the nurse
3. “I will need to cleanse around the stoma question receiving the vaccine?
daily.” 1. The client diagnosed with congestive heart
4. “I can use an electric larynx to speak.” failure.
2. The client with a documented allergy to eggs.
13. The occupational nurse for a mining company
3. The client who has had an anaphylactic
is planning a class on the risks of working with
reaction to penicillin.
toxic substances to comply with the “Right to
4. The client who has an elevated blood pressure
Know” law. Which information should the nurse
and pulse.
include in the presentation? Select all that
apply. 18. The nurse is preparing the client for a
1. A client who smokes cigarettes has a polysomnography to confirm sleep apnea.
drastically increased risk for lung cancer. Which preprocedure instruction should the
2. Floors need to be clean and dust needs to be nurse include?
wet to prevent transfer of dust. 1. The client should not eat or drink past
3. The air needs to be monitored at specific midnight.
times to evaluate for exposure. 2. The client will receive a sedative for relaxation.
4. Surface areas need to be painted every year to 3. The client will sleep in a laboratory for
prevent the accumulation of dust. evaluation.
5. Employees should wear the appropriate 4. The client will wear a monitor at home for
personal protective equipment. this test.
14. Which data are significant when assessing a 19. The client in the intensive care unit diagnosed
client diagnosed with rule-out Legionnaires’ with end-stage chronic obstructive pulmonary
disease? disease has a Swan-Ganz mean pulmonary artery
1. The amount of cigarettes smoked a day and pressure of 35 mm Hg. Which health-care
the age when started. provider order would the nurse question?
2. Symptoms of aching muscles, high fever, 1. Administer intravenous fluids of normal saline
malaise, and coughing. at 125 mL/hr.
3. Exposure to a saprophytic water bacterium 2. Provide supplemental oxygen per nasal
transmitted into the air. cannula at 2 L/min.
4. Decreased bilateral lung sounds in the lower 3. Continuous telemetry monitoring with strips
lobes. every four (4) hours.
4. Administer a loop diuretic intravenously every
15. Which assessment data indicate to the nurse the
six (6) hours.
client diagnosed with Legionnaires’ disease is
experiencing a complication? 20. The nurse and an unlicensed assistive personnel
1. The client has an elevated body temperature. (UAP) are caring for an elderly client diagnosed
2. The client has less than 30 mL urine output with emphysema. Which nursing tasks could be
an hour. delegated to the UAP to improve gas exchange?
3. The client has a decrease in body aches. Select all that apply.
4. The client has an elevated white blood cell 1. Keep the head of the bed elevated.
count. 2. Encourage deep breathing exercises.
3. Record pulse oximeter reading.
4. Assess level of conscious.
5. Auscultate breath sounds.

06_Colgrove_Ch06.indd 234 01/06/16 5:16 PM


C hapter 6  R espiratory D isorders 235

21. The nurse is preparing the plan of care for 27. Which information should the nurse include
the client who had a pleurodesis. Which in the teaching plan for the mother of a child
collaborative intervention should the nurse diagnosed with cystic fibrosis (CF)? Select all
include? that apply.
1. Monitor the amount and color of drainage 1. Perform postural drainage and percussion
from the chest tube. every four (4) hours.
2. Perform a complete respiratory assessment 2. Modify activities to accommodate daily
every two (2) hours. physiotherapy.
3. Administer morphine sulfate, an opioid 3. Increase fluid intake to one (1) liter daily to
analgesic, intravenously. thin secretions.
4. Keep a sterile dressing and bottle of sterile 4. Recognize and report signs and symptoms of
normal saline at the bedside. respiratory infections.
5. Avoid anyone suspected of having an upper
22. Which problem is appropriate for the nurse
respiratory infection.
to identify for the client who is one (1) day
postoperative thoracotomy? 28. Which clinical manifestation indicates to the
1. Alteration in comfort. nurse the child has cystic fibrosis?
2. Altered level of conscious. 1. Wheezing with a productive cough.
3. Alteration in elimination pattern. 2. Excessive salty sweat secretions.
4. Knowledge deficit. 3. Multiple vitamin deficiencies.
4. Clubbing of all fingers.
23. The nurse is caring for the client diagnosed
with bacterial pneumonia. Which priority 29. The client is diagnosed with bronchiolitis
intervention should the nurse implement? obliterans. Which data indicate the
1. Assess respiratory rate and depth. glucocorticoid therapy is effective?
2. Provide for adequate rest period. 1. The client has an elevation in the blood
3. Administer oxygen as prescribed. glucose.
4. Teach slow abdominal breathing. 2. The client has a decrease in sputum
production.
24. The nurse is caring for a client diagnosed with
3. The client has an increase in the temperature.
pneumonia who is having shortness of breath
4. The client appears restless and is
and difficulty breathing. Which intervention
irritable.
should the nurse implement first?
1. Take the client’s vital signs. 30. The nurse is discharging the client
2. Check the client’s pulse oximeter reading. diagnosed with bronchiolitis obliterans.
3. Administer oxygen via a nasal cannula. Which priority intervention should the nurse
4. Notify the respiratory therapist STAT. include?
1. Refer the client to the American Lung
25. The post-anesthesia care nurse is caring for
Association.
the client diagnosed with lung cancer who had
2. Notify the physical therapy department to
a thoracotomy and is experiencing frequent
arrange for activity training.
premature ventricular contractions (PVCs).
3. Arrange for oxygen therapy to be used at
Which intervention should the nurse implement
home.
first?
4. Discuss advance directives with the client.
1. Request STAT arterial blood gases.
2. Administer lidocaine intravenous push. 31. The client admitted for recurrent aspiration
3. Assess for possible causes. pneumonia is at risk for bronchiectasis. Which
4. Request a STAT electrocardiogram. intervention should the nurse anticipate the
health-care provider to order?
26. The nurse is caring for the postoperative client
1. Administer intravenous antibiotics for seven
diagnosed with lung cancer recovering from a
(7) days.
thoracotomy. Which data require immediate
2. Insert a subclavian line and initiate total
intervention by the nurse?
parenteral nutrition.
1. The client refuses to perform shoulder
3. Provide a low-calorie and low-sodium
exercises.
restricted diet.
2. The client complains of a sore throat and
4. Encourage the client to turn, cough, and deep
is hoarse.
breathe frequently.
3. The client has crackles that clear with cough.
4. The client is coughing up pink frothy sputum.

06_Colgrove_Ch06.indd 235 01/06/16 5:16 PM


236 M ed -S urg S uccess

32. Which collaborative intervention should the 37. The nurse is preparing to administer warfarin
nurse implement when caring for the client (Coumadin), an oral anticoagulant, to a client
diagnosed with bronchiectasis? diagnosed with a pulmonary embolus. Which
1. Prepare the client for an emergency data would cause the nurse to question
tracheostomy. administering the medication?
2. Discuss postoperative teaching for a 1. The client’s partial thromboplastin time
lobectomy. (PTT) is 38.
3. Administer bronchodilators with postural 2. The client’s international normalized ratio
drainage. (INR) is 5.
4. Obtain informed consent form for chest tube 3. The client’s prothrombin time (PT) is 22.
insertion. 4. The client’s erythrocyte sedimentation rate
(ESR) is 10.
33. The nurse is discussing the results of a
tuberculosis skin test. Which explanation should 38. The nurse is caring for a client diagnosed with a
the nurse provide the client? pneumothorax who had chest tubes inserted four
1. A red area is a positive reading that means the (4) hours ago. There is no fluctuating (tidaling)
client has tuberculosis. in the water-seal compartment of the closed
2. The skin test is the only procedure needed to chest drainage system. Which action should the
diagnose tuberculosis. nurse implement first?
3. A positive reading means exposure to the 1. Milk the chest tube.
tuberculosis bacilli. 2. Check the tubing for kinks.
4. Do not get another skin test for one (1) year if 3. Instruct the client to cough.
the skin test is positive. 4. Assess the insertion site.
34. The client diagnosed with tuberculosis has been 39. The health-care provider has ordered
treated with antitubercular medications for a continuous intravenous infusion of
six (6) weeks. Which data would indicate the aminophylline. The client weighs 165 pounds.
medications have been effective? The infusion order is 0.3 mg/kg/hr. The bag
1. A decrease in the white blood cells in the is mixed with 500 mg of aminophylline in
sputum. 250 mL of D5W. At which rate should the nurse
2. The client’s symptoms are improving. set the pump? _______
3. No change in the chest x-ray.
40. The client diagnosed with a cold is taking an
4. The skin test is now negative.
antihistamine. Which statement indicates to
35. The nurse is caring for a client on a ventilator the nurse the client needs more teaching
and the alarm goes off. Which action should the concerning the medication?
nurse implement first? 1. “If my mouth gets dry I will suck on hard
1. Notify the respiratory therapist immediately. candy.”
2. Check the ventilator to determine the cause. 2. “I will not drink beer or any type of alcohol.”
3. Elevate the head of the client’s bed. 3. “I need to be careful when I drive my car.”
4. Assess the client’s oxygen saturation. 4. “This medication will make me sleepy.”
36. The client diagnosed with deep vein thrombosis 41. The client with a cold asks the nurse, “Is it all
(DVT) suddenly complains of severe chest right to take echinacea for my cold?” Which
pain and a feeling of impending doom. Which statement is the nurse’s best response?
complication should the nurse suspect the client 1. “You should discuss that with your health-care
has experienced? provider.”
1. Myocardial infarction. 2. “No, you should not take any type of herbal
2. Pneumonia. medicine.”
3. Pulmonary embolus. 3. “Yes, but do not take it for more than 3 days.”
4. Pneumothorax. 4. “Echinacea may help with the symptoms of
your cold.”

06_Colgrove_Ch06.indd 236 01/06/16 5:16 PM


C hapter 6  R espiratory D isorders 237

42. Which intervention should the nurse implement 47. Which intervention should the nurse implement
for the client experiencing bronchospasms? first when caring for a client with a respiratory
1. Administer intravenous epinephrine, a disorder?
bronchodilator. 1. Administer a respiratory treatment.
2. Administer albuterol, a bronchodilator, via 2. Check the client’s radial pulses daily.
nebulizer. 3. Monitor the client’s vital signs daily.
3. Request a STAT portable chest x-ray at the 4. Assess the client’s capillary refill time.
bedside.
48. The nurse is preparing to hang the next bag
4. Insert a small nasal trumpet in the right
of aminophylline, a bronchodilator, for the
nostril.
client diagnosed with asthma. The current
43. The nurse is caring for a female client who theophylline level is 18 mcg/mL. Which
is anxious, has a respiratory rate of 40, and is intervention should the nurse implement?
complaining of her fingers tingling and her lips 1. Hang the next bag and continue the infusion.
feeling numb. Which intervention should the 2. Do not hang the next bag and decrease the
nurse implement first? rate.
1. Have the client take slow, deep breaths. 3. Notify the health-care provider of the level.
2. Instruct her to put her head between her legs. 4. Confirm the current serum theophylline level.
3. Determine why she is feeling so anxious.
49. Which intervention should the nurse implement
4. Administer Xanax, an antianxiety agent.
first when administering the first dose of
44. The client diagnosed with pneumonia has intravenous antibiotic to the client diagnosed
arterial blood gases of pH 7.33, Pao2 94, with a respiratory infection?
Paco2 47, Hco3 25. Which intervention should 1. Monitor the client’s current temperature.
the nurse implement? 2. Monitor the client’s white blood cells.
1. Administer sodium bicarbonate. 3. Determine if a culture has been collected.
2. Administer oxygen via nasal cannula. 4. Determine the compatibility of fluids.
3. Have the client cough and deep breathe.
50. Which nursing interventions should the nurse
4. Instruct the client to breathe into a paper bag.
implement for the client who has a respiratory
45. The nurse is assessing the client diagnosed with disorder? Select all that apply.
a lung abscess. Which information supports this 1. Administer oxygen via a nasal cannula.
diagnosis of lung abscess? 2. Assess the client’s lung sounds.
1. Tympanic sounds elicited by percussion over 3. Encourage the client to cough and deep
the site. breathe.
2. Inspiratory and expiratory wheezes heard over 4. Monitor the client’s pulse oximeter reading.
the upper lobes. 5. Increase the client’s fluid intake.
3. Decreased breath sounds with a pleural
51. The client in the intensive care unit (ICU) on a
friction rub.
mechanical ventilator is bucking the ventilator,
4. Asymmetric movement of the chest wall with
causing the alarms to sound, and is in respiratory
inspiration.
distress. Which assessment data should the nurse
46. The public health department nurse is caring for obtain? List in order of priority.
the client diagnosed with active tuberculosis who 1. Assess the ventilator alarms.
has been placed on directly observed therapy 2. Assess the client’s pulse oximetry reading.
(DOT). Which statement best describes this 3. Assess the client’s lung sounds.
therapy? 4. Assess for symmetry of the client’s chest
1. The nurse accounts for all medications expansion.
administered to the client. 5. Assess the client’s endotracheal tube for
2. The nurse must complete federal, state, and secretions.
local forms for this client.
3. The nurse must report the client to the
Centers for Disease Control.
4. The nurse must watch the client take the
medication daily.

06_Colgrove_Ch06.indd 237 01/06/16 5:16 PM


238 M ed -S urg S uccess

52. The nurse is checking the vital signs graphic sheet at 0800 on a client diagnosed with pneumonia secondary
to chronic obstructive pulmonary disease who has been receiving prednisone, a glucocorticoid, for several
years. Which graphic should the nurse expect to see?
1. 2.
Client Graphic Client Graphic
Date: Date: Date: Date: Date: Date:
2 Days Ago Yesterday Today 2 Days Ago Yesterday Today
24 04 08 12 16 20 24 04 08 12 16 20 24 04 08 12 16 20 24 04 08 12 16 20 24 04 08 12 16 20 24 04 08 12 16 20

102 102

101 101

100 100

99 99
98.6 98.6

98 98

97 97

96 96

3. 4.
Client Graphic Client Graphic
Date: Date: Date: Date: Date: Date:
2 Days Ago Yesterday Today 2 Days Ago Yesterday Today
24 04 08 12 16 20 24 04 08 12 16 20 24 04 08 12 16 20 24 04 08 12 16 20 24 04 08 12 16 20 24 04 08 12 16 20

102 102

101 101

100 100

99 99
98.6 98.6

98 98

97 97

96 96

06_Colgrove_Ch06.indd 238 01/06/16 5:16 PM


RESPIRATORY DISORDERS COMPREHENSIVE
EXAMINATION ANSWERS AND RATIONALES
1. 1. A bronchoscope visualizes the bronchial tree 4. 1. Airborne isolation is used when the infectious
under sedation, but it does not confirm the agent can remain in the air and be trans-
diagnosis of asthma. ported greater than three (3) feet and includes
2. An immunoglobulin E is a blood test for the wearing specially fitted masks to prevent
presence of an antibody protein indicating transmission.
allergic reactions. 2. SARS is an influenza-type virus transmitted
3. Arterial blood gases analyze levels providing by particle droplets. Therefore, the client
information about the exchange of oxygen and should be placed in droplet isolation.
carbon dioxide, but they are not diagnostic of 3. “Reverse isolation” is a term for using equip-
asthma. ment to prevent the client from being exposed
4. During a bronchodilator reversibility test, to organisms from other people.
the client’s positive response to a broncho- 4. “Strict isolation” is an old term used to de-
dilator confirms the diagnosis of asthma. It scribe isolation to prevent transmission of
is useful in clients with COPD because air- organisms to other people. It is vague and not
way reversibility is characteristic of asthma used today in infection control standards.
but no emphysema or bronchitis. Content – Medical: Integrated Nursing Process –
Content – Medical: Integrated Nursing Process – Implementation: Client Needs – Safe Effective Care Envi-
Planning: Client Needs – Physiological Integrity, Reduction ronment, Safety and Infection Control: Cognitive Level –
of Risk Potential: Cognitive Level – Synthesis: Synthesis: Concept – Oxygenation.
Concept – Oxygenation.
5. 1. This information is important during an ad-
2. 1. Leukotrienes, such as Singulair, should be mission interview but is not specific to SARS.
taken daily to prevent an asthma attack trig- 2. The information would not be specific to the
gered by an allergen response. diagnosis of SARS.
2. Cromolyn inhalers, such as Intal, are used to 3. This would be important to ask prior to the
prevent exercise-induced asthma attacks. administration of any blood products, but it is
3. Glucocorticoids are administered orally or not specific for SARS.
intravenously during acute exacerbations of 4. Recent travel to mainland China, Taiwan,
asthma, not on a daily basis and Hong Kong is a risk factor for con-
because of the long-term complications of tracting SARS.
steroid therapy. Content – Medical: Integrated Nursing Process –
4. Albuterol, a beta2 agonist, is used during Assessment: Client Needs – Physiological Integrity, Reduction
attacks because of the fast action. of Risk Potential: Cognitive Level – Analysis: Concept –
Content – Medical: Integrated Nursing Process – Oxygenation.
Evaluation: Client Needs – Physiological Integrity, Pharma- 6. 1. The client would have low arterial oxygen
cological and Parenteral Therapies: Cognitive Level –
when developing ARDS.
Evaluation: Concept – Medication.
2. Initial clinical manifestations of ARDS usu-
3. 1. Glucocorticoids are a treatment of choice, but ally develop 24 to 48 hours after the initial
they are not the first intervention. insult leading to hypoxia and include anxi-
2. The client is in distress so the nurse must do ety, dyspnea, and tachypnea.
something for the client’s airway. 3. As ARDS progresses, the client has more diffi-
3. A saline lock is needed for intravenous fluids, culty breathing, resulting in intercostal retrac-
but it is not the first intervention. tions and use of accessory muscles.
4. Assessment is the first step of the nursing 4. Lungs are initially clear; crackles and
process, but in distress do not assess. rhonchi develop in later stages of ARDS.
Content – Medical: Integrated Nursing Process – Content – Medical: Integrated Nursing Process –
Implementation: Client Needs – Safe Effective Care Envi- Assessment: Client Needs – Physiological Integrity, Reduction
ronment, Management of Care: Cognitive Level – Synthesis: of Risk Potential: Cognitive Level – Analysis: Concept –
Concept – Oxygenation. Oxygenation.

239

06_Colgrove_Ch06.indd 239 01/06/16 5:16 PM


240 M ed -S urg S uccess

7. 1. This ABG is within normal limits and would 10. 1. Dysphagia is a late sign of intrinsic lung cancer.
not be expected in a client with ARDS. 2. Foul-smelling breath is a late sign of intrinsic
2. These ABG levels indicate respiratory lung cancer.
alkalosis, but the oxygen level is within 3. Hoarseness is an early clinical manifesta-
normal limits and would not be expected tion of intrinsic lung cancer. “Intrinsic”
in a client with ARDS. means the tumor is on the vocal cord.
3. ABGs initially show hypoxemia with a Pao2 4. Weight loss is a late sign of most types of
of less than 60 mm Hg and respiratory al- cancers, not just intrinsic lung cancer.
kalosis resulting from tachypnea in a client Content – Medical: Integrated Nursing Process –
with ARDS. Assessment: Client Needs – Physiological Integrity,
4. This ABG is metabolic acidosis and would not Reduction of Risk Potential: Cognitive Level –
be expected in a client with ARDS. Analysis: Concept – Cellular Regulation.
Content – Medical: Integrated Nursing Process – 11. 1. Fluids should be encouraged to help to
Assessment: Client Needs – Physiological Integrity, liquefy sputum; therefore, intake and output
Reduction of Risk Potential: Cognitive Level –
should be monitored, but this is not the
Analysis: Concept – Oxygenation.
priority intervention.
8. 1. Bathing is too strenuous an activity immedi- 2. Black-streaked sputum is a classic
ately after eating. sign of coal workers’ pneumoconiosis
2. Inhalation bronchodilators should be (black lung), and the sputum should be
administered one (1) hour after meals. The assessed for color and amount. Remember
nurse should realize the client will require Maslow’s hierarchy of needs when answer-
20 to 30 minutes to eat, and scheduling these ing priority questions.
activities at 0900 would not give the client suf- 3. The client’s white blood cells should be
ficient time to digest the food or rest before monitored to assess for infection, but it is not
the activity. priority and is not done daily.
3. Periods of rest should be alternated with 4. Activity tolerance is important to assess for
periods of activity. clients with all respiratory diseases, but it is
4. Chest physiotherapy should be performed not the priority intervention.
at least one (1) hour after meals. The nurse Content – Medical: Integrated Nursing Process –
should realize the client will require 20 to Implementation: Client Needs – Safe Effective Care Envi-
30 minutes to eat, and scheduling these ronment, Management of Care: Cognitive Level –
activities at 0900 would not give the client suf- Synthesis: Concept – Oxygenation.
ficient time to digest the food or rest
12. 1. The client should not let any spray or powder
before the activity.
enter the stoma because it goes directly into
Content – Medical: Integrated Nursing Process – the lung.
Assessment: Client Needs – Physiological Integrity, 2. The client should not allow water to enter
Reduction of Risk Potential: Cognitive Level – Synthesis:
the stoma; therefore, the client should
Concept – Oxygenation.
take a tub bath, not a shower.
9. 1. The appearance of S3 heart sounds indi- 3. The stoma site should be cleansed to help
cates the client is developing heart failure, prevent infection.
which is a medical emergency. 4. The client’s vocal cords were removed; there-
2. Clubbing of the fingers indicates the client has fore, the client must use an alternate form of
a chronic respiratory condition, but this would communication.
not require immediate intervention. Content – Medical: Integrated Nursing Process –
3. Fatigue is a common occurrence in clients Evaluation: Client Needs – Safe Effective Care Environ-
with respiratory conditions, such as asbestosis, ment, Management of Care: Cognitive Level – Evaluation:
as a result of the effort required to breathe. Concept – Oxygenation.
4. Bibasilar crackles are common symptoms 13. 1. Clients who smoke cigarettes and work
experienced by clients with asbestosis and do with toxic substances have increased risk
not require immediate intervention. Bibasilar of lung cancer because many of the sub-
means basilar on both sides. stances are carcinogenic.
Content – Medical: Integrated Nursing Process – 2. When floors and surfaces are kept clean,
Assessment: Client Needs – Physiological Integrity, toxic dust particles, such as asbestos and
Reduction of Risk Potential: Cognitive Level –
silica, are controlled and this decreases
Analysis: Concept – Perfusion.
exposure. Covering areas with water con-
trols dust.

06_Colgrove_Ch06.indd 240 01/06/16 5:16 PM


C hapter 6  R espiratory D isorders 241

3. The quality of air is monitored to deter- 2. Many clients need a continuous positive air-
mine what toxic substances are present way pressure (CPAP) machine, which contin-
and in what amount. The information uously administers positive pressure to assist
is then used in efforts to minimize the sleep during the night.
amount of exposure. 3. When clients have sleep apnea, the buildup
4. Applying paint to a surface does not eliminate of carbon dioxide causes the client to arouse
or minimize exposure and can trap more dust. constantly from sleep to breathe. This, in turn,
5. Employees must wear protective coverings, causes the client to be sleepy during the day.
goggles, and other equipment needed to 4. Drinking alcohol before sleep sedates
eliminate exposure to the toxic substances. the client, causing the muscles to relax,
Content – Medical: Integrated Nursing Process – which, in turn, causes an obstruction
Planning: Client Needs – Safe Effective Care Environ- of the client’s airway. Drinking alcohol
ment, Management of Care: Cognitive Level – Synthesis: should be avoided even if the client uses
Concept – Promoting Health. a CPAP machine.
14. 1. Smoking cigarettes is important to assess in Content – Medical: Integrated Nursing Process –
any respiratory disease. Legionnaires’ dis- Evaluation: Client Needs – Safe Effective Care Environ-
ment, Management of Care: Cognitive Level – Synthesis:
ease is contracted with a bacterium, not by
Concept – Oxygenation.
smoking.
2. Aching muscles, high fever, malaise, and 17. 1. There would be no reason to question ad-
coughing are symptoms of most respiratory ministering a vaccine to a client with heart
illnesses, including influenza and pneumonia, failure.
but these symptoms are not specific to 2. In clients who are allergic to egg protein,
Legionnaires’ disease. a significant hypersensitivity response may
3. Legionnaires’ disease is caused by a occur when they are receiving the influ-
saprophytic water bacterium that is trans- enza vaccine.
mitted through the air from places where 3. There would be no reason to question
these bacteria are found: rivers, lakes, administering a vaccine to a client who has
evaporative condensers, respiratory appa- had a reaction to penicillin.
ratuses, or water distribution centers. 4. There would be no reason to question
4. Abnormal breath sounds can be heard in administering a vaccine to a client who has
many respiratory illnesses. elevated blood pressure and pulse.
Content – Medical: Integrated Nursing Process – Content – Medical: Integrated Nursing Process –
Assessment: Client Needs – Physiological Integrity, Implementation: Client Needs – Safe Effective Care Envi-
Reduction of Risk Potential: Cognitive Level – ronment, Management of Care: Cognitive Level –
Analysis: Concept – Oxygenation. Application: Concept – Promoting Health.

15. 1. The temperature is elevated and does not in- 18. 1. Preparation for the polysomnography does
dicate a complication. not require having nothing by mouth (NPO)
2. Multiple organ failure is a common com- after midnight.
plication of Legionnaires’ disease. Renal 2. The examination is a recording of the natural
failure should be suspected as a compli- sleep of the client. No sedative is administered.
cation if the client does not have a urine 3. The polysomnography is completed in a
output of 30 mL/hr. sleep laboratory to observe all the stages
3. A decrease in body aches does not indicate a of sleep. Equipment is attached to the
complication. client to monitor depth and stage of sleep
4. An elevation of white blood cells is expected and movement, respiratory effort, and
in a client with Legionnaires’ disease and is oxygen saturation level during sleep.
not a complication. 4. The client could perform this test at home,
Content – Medical: Integrated Nursing Process – but the evaluation is not as reliable and the
Assessment: Client Needs – Physiological Integrity, Reduc- visual observation is not included; therefore,
tion of Risk Potential: Cognitive Level – Analysis: this is not recommended.
Concept – Urinary Elimination. Content – Surgical: Integrated Nursing Process –
16. 1. The contributing factors to developing sleep Planning: Client Needs – Physiological Integrity, Reduction
of Risk Potential: Cognitive Level – Synthesis: Concept –
apnea are obesity, smoking, drinking alcohol,
Oxygenation.
and a short neck. In some situations, modify-
ing lifestyle will improve sleep apnea.

06_Colgrove_Ch06.indd 241 01/06/16 5:16 PM


242 M ed -S urg S uccess

19. 1. Normal mean pulmonary artery pressure 22. 1. Pain and discomfort are major problems
is about 15 mm Hg and an elevation for a client who had a thoracotomy
indicates right ventricular heart failure or because the chest wall has been opened
cor pulmonale, which is a comorbid con- and closed.
dition of chronic obstructive pulmonary 2. The client would be on a mechanical ventila-
disease. The nurse should question this tor and have an adequate airway; therefore,
order because the rate is too high. altered consciousness would not be an appro-
2. Supplemental oxygen should be administered priate client problem.
at the lowest amount; therefore, this order 3. Altered elimination problem is not specific
should not be questioned. for the client with a thoracotomy.
3. Clients with hypoxia and cor pulmonale are 4. A knowledge deficit problem is not an
at risk for dysrhythmias, so monitoring the appropriate problem for the client who is one
electrocardiogram (ECG) is an appropriate (1) day postoperative thoracotomy because
intervention. the client is on a ventilator.
4. Loop diuretics are administered to decrease Content – Surgical: Integrated Nursing Process –
the fluid and decrease the circulatory load on Diagnosis: Client Needs – Physiological Integrity, Physi-
the right side of the heart; therefore, this or- ological Adaptation: Cognitive Level – Analysis: Concept –
der would not be questioned. Perioperative.
Content – Surgical: Integrated Nursing Process – 23. 1. The assessment of respiratory rate
Implementation: Client Needs – Safe Effective Care Envi- and depth is the priority intervention be-
ronment, Management of Care: Cognitive Level –
cause tachypnea and dyspnea may be early
Application: Concept – Oxygenation.
indicators of respiratory compromise.
20. 1. Keeping the head of the bed elevated 2. Rest reduces metabolic demands, fatigue,
maximizes lung excursion and improves and the work of breathing, which promotes a
gas exchange and can be delegated. more effective breathing pattern, but it is not
2. Encouraging breathing exercises can priority over assessment.
be delegated. 3. Oxygen therapy increases the alveolar oxygen
3. Recording pulse oximeter readings can be concentration, reducing hypoxia and anxiety,
delegated. Evaluating is the responsibility but it is not priority over assessment.
of the nurse. 4. This breathing pattern promotes lung expan-
4. Assessment cannot be delegated. Confusion sion, but it is not priority over assessment.
is one of the first symptoms of hypoxia. Content – Medical: Integrated Nursing Process –
5. Auscultation is a technique of assessment and Implementation: Client Needs – Safe Effective Care Envi-
cannot be delegated. ronment, Management of Care: Cognitive Level –
Content – Medical: Integrated Nursing Process – Synthesis: Concept – Infection.
Planning: Client Needs – Safe Effective Care Environ- 24. 1. Taking the client’s vital signs will not help the
ment, Management of Care: Cognitive Level – Synthesis:
client’s shortness of breath and difficulty in
Concept – Oxygenation.
breathing.
21. 1. Monitoring the amount and color of drainage 2. Checking the pulse oximeter reading will not
from the chest tube is an independent nursing help the client’s shortness of breath and dif-
intervention. ficulty breathing.
2. Respiratory assessment is an independent 3. After elevating the head of the bed, the
intervention. nurse should administer oxygen to the
3. Administering medication is a collabora- client who is in respiratory difficulty.
tive intervention because it requires a 4. Notifying the respiratory therapist will not
health-care provider’s order. help the client’s shortness of breath and
4. Keeping supplies at the bedside is an inde- difficulty breathing.
pendent intervention the nurse may take in Content – Medical: Integrated Nursing Process –
case the chest tube becomes dislodged. Implementation: Client Needs – Safe Effective Care Envi-
Content – Surgical: Integrated Nursing Process – ronment, Management of Care: Cognitive Level –
Diagnosis: Client Needs – Physiological Integrity, Physi- Synthesis: Concept – Infection.
ological Adaptation: Cognitive Level – Analysis: Concept –
Oxygenation.

06_Colgrove_Ch06.indd 242 01/06/16 5:16 PM


C hapter 6  R espiratory D isorders 243

25. 1. ABGs may show hypoxia, which is a cause 2. The excessive excretion of salt from the
of PVCs, but it is not the first intervention sweat glands is specific to cystic fibrosis.
the nurse should implement. Repeated values greater than 60 mEq/L of
2. Lidocaine is the treatment of choice for sweat chloride is diagnostic for CF.
PVCs, but it is not the first intervention. 3. Multiple vitamin deficiencies are experienced
3. The nurse should assess for possible with some pulmonary diseases, but they are
causes of the PVCs; these causes may not specific to cystic fibrosis.
include hypoxia or hypokalemia. 4. Clubbing of the fingers is an indicator of
4. An ECG further evaluates the heart function, chronic hypoxia, but it is not specific to the
but it is not the first intervention. diagnosis of cystic fibrosis.
Content – Surgical: Integrated Nursing Process – Content – Medical: Integrated Nursing Process –
Implementation: Client Needs – Safe Effective Care Envi- Assessment: Client Needs – Physiological Integrity,
ronment, Management of Care: Cognitive Level – Reduction of Risk Potential: Cognitive Level –
Synthesis: Concept – Cellular Regulation. Analysis: Concept – Oxygenation.

26. 1. The client refusing to perform shoulder exer- 29. 1. An elevation in the blood glucose level is a
cises is pertinent, but it does not require im- common side effect of corticosteroids and
mediate intervention. does not indicate effectiveness of the treat-
2. Sore throats and hoarseness are common pos- ment for bronchiolitis obliterans.
tintubation and would not require immediate 2. A decrease in sputum production indi-
intervention. cates that the client is improving and the
3. Crackles that clear with coughing would not medication is effective; long-term use of
require immediate intervention. corticosteroids is indicated for a client
4. Pink frothy sputum indicates pulmonary with bronchiolitis obliterans.
edema and would require immediate 3. An elevated temperature indicates the client
intervention. is becoming worse; therefore, the medication
Content – Surgical: Integrated Nursing Process – is not effective.
Assessment: Client Needs – Safe Effective Care Environ- 4. Restlessness and irritability can be side ef-
ment, Management of Care: Cognitive Level – Synthesis: fects of treatment with corticosteroids or they
Concept – Cellular Regulation. could be signs of hypoxemia, which would
indicate the medication is not effective.
27. 1. Clients and family members should be
taught chest physiotherapy, including Content – Medical: Integrated Nursing Process –
postural drainage, chest percussion, Assessment: Client Needs – Physiological Integrity, Phar-
macological and Parenteral Therapies: Cognitive Level –
and vibration and breathing techniques
Analysis: Concept – Oxygenation.
to keep the lungs clear of the copious
secretions. 30. 1. The American Lung Association is an excel-
2. Daily activities should be modified to lent resource for educational material, but it
accommodate the client’s treatments. is not the priority intervention for the client.
3. Clients should increase fluids up to 3,000 mL 2. Physical therapy is an appropriate interven-
each day to thin secretions and ease tion, but it is not the priority intervention.
expectoration. 3. The client with bronchiolitis obliterans
4. Clients should be taught the signs and will need long-term use of oxygen.
symptoms of infections to report to the 4. Advance directives are an important interven-
health-care provider. tion but are not the priority intervention.
5. Clients with CF are susceptible to respira- Content – Medical: Integrated Nursing Process –
tory infections and should avoid anyone Planning: Client Needs – Physiological Integrity, Physi-
who is suspected of having an infection. ological Adaptation: Cognitive Level – Synthesis:
Content – Medical: Integrated Nursing Process – Concept – Oxygenation.
Planning: Client Needs – Physiological Integrity, Physi- 31. 1. Antibiotics should be administered
ological Adaptation: Cognitive Level – Synthesis:
intravenously for seven (7) to 10 days.
Concept – Oxygenation.
Bronchiectasis is an irreversible condi-
28. 1. Wheezing and productive coughs are symp- tion caused by repeated damage to the
toms experienced by clients with respiratory bronchial walls secondary to repeated
diseases, but they are not specific to cystic aspiration of gastric contents and release
fibrosis. of inflammatory mediators by the body to
combat the foreign substances.

06_Colgrove_Ch06.indd 243 01/06/16 5:16 PM


244 M ed -S urg S uccess

2. Total parenteral nutrition is not an expected 35. 1. The nurse needs to notify the respiratory
treatment for a client with bronchiectasis. therapist to check the ventilator, but it is not
3. Clients should have a high-calorie and high- the first intervention.
protein diet as a result of high expenditure of 2. The nurse must determine what is
energy used to breathe and tissue healing. causing the alarm; a high or low alarm will
4. Turning, coughing, and deep breathing are make a difference in the nurse’s action.
appropriate independent nursing interven- 3. Elevating the head of the bed will help lung
tions but do not require a health-care pro- expansion, but it is not the first intervention.
vider’s order. 4. The ventilator alarm indicates something is
Content – Medical: Integrated Nursing Process – wrong, and the nurse must first determine
Planning: Client Needs – Safe Effective Care if the problem is with the ventilator or the
Environment, Management of Care: Cognitive client.
Level – Synthesis: Concept – Oxygenation. Content – Medical: Integrated Nursing Process –
32. 1. Medical treatment for bronchiectasis does Implementation: Client Needs – Safe Effective Care
Environment, Management of Care: Cognitive
not include a tracheostomy.
Level – Synthesis: Concept – Oxygenation.
2. Removing a lobe of the lung is not the ex-
pected medical treatment for a client with 36. 1. The nurse would not suspect a myocardial
bronchiectasis. infarction for a client with a DVT who
3. Administering bronchodilators is a col- suddenly has chest pain.
laborative intervention (requiring an order 2. These signs and symptoms should not make
from a health-care provider) appropriate the nurse think the client has pneumonia.
for this client. 3. Part of the clot in the deep veins of the legs
4. Insertion of chest tubes is not an expected dislodges and travels up the inferior vena
treatment for bronchiectasis. cava, lodges in the pulmonary arterial sys-
Content – Medical: Integrated Nursing Process – tem, and causes the chest pain; the client
Implementation: Client Needs – Safe Effective Care often feels as if he or she is ­going to die.
Environment, Management of Care: Cognitive 4. Chest pain is a sign of pneumothorax, but it is
Level – Application: Concept – Oxygenation. not a complication of deep vein thrombosis.
33. 1. A red and raised area at the injection site Content – Medical: Integrated Nursing Process –
indicates that the client has been exposed to Assessment: Client Needs – Physiological Integrity,
Reduction of Risk Potential: Cognitive Level –
tuberculosis bacilli, but it does not indicate
Analysis: Concept – Oxygenation.
active disease.
2. The skin test indicates the client has been 37. 1. The PTT is not monitored to determine a
exposed to the tuberculosis bacilli, but further serum therapeutic level for warfarin; normal
tests must be performed to confirm the diag- is 30 to 45.
nosis of tuberculosis. 2. The INR therapeutic range is two (2) to
3. A positive reading indicates the client has three (3) for a client receiving warfarin.
been exposed to the bacilli. The INR may be allowed to go to 3.5 if
4. Once a positive reading occurs, then the cli- the client has a mechanical cardiac valve,
ent should never receive a Tb skin test again. but nothing in the stem of the question
Content – Medical: Integrated Nursing Process – indicates this.
Implementation: Client Needs – Physiological Integrity, 3. The PT is monitored for oral anticoagulant
Reduction of Risk Potential: Cognitive Level – Application: therapy and should be 1.5 to 2 times the nor-
Concept – Oxygenation. mal of 12; therefore, 22 is within therapeutic
range and would not warrant the nurse ques-
34. 1. Antitubercular medications target the tuber-
tioning administering this medication.
cular bacilli, not white blood cells.
4. The ESR is not monitored for oral antico-
2. As the bacilli are being destroyed, the
agulant therapy.
client should begin to feel better and have
fewer symptoms. Content – Medical: Integrated Nursing Process –
3. At six (6) weeks, the chest x-ray may not have Assessment: Client Needs – Physiological Integrity, Phar-
macological and Parenteral Therapies: Cognitive Level –
changes.
Analysis: Concept – Oxygenation.
4. The skin test will always be positive.
Content – Medical: Integrated Nursing Process – 38. 1. No fluctuation in the water-seal chamber
Assessment: Client Needs – Physiological Integrity, Phar- four (4) hours postinsertion indicates the tub-
macological and Parenteral Therapies: Cognitive Level – ing is blocked; the nurse should not milk the
Analysis: Concept – Oxygenation. chest tube.

06_Colgrove_Ch06.indd 244 01/06/16 5:16 PM


C hapter 6  R espiratory D isorders 245

2. The nurse should implement the least in- 2. The nurse should not be judgmental. If the
vasive intervention first. The nurse should client does not have comorbid conditions, is
check to see if the tubing is kinked, caus- not taking other medications, or is not preg-
ing a blockage between the pleural space nant, herbal medications may be helpful in
and the water-seal bottle. treating the common cold.
3. Coughing may help push a clot in the tub- 3. Echinacea should not be taken for more than
ing into the drainage bottle, but the first two (2) weeks, not three (3) days. Nothing
intervention is to check and see if the client cures the common cold; the cold must run its
is lying on the tubing or the tube is kinked course.
somewhere. 4. Echinacea is an herb that may reduce the
4. The insertion site can be assessed, but it will duration and symptoms of the common
not help determine why there is no fluctua- cold, but nothing cures the common cold.
tion in the water-seal drainage compartment. If the client does not have comorbid con-
Content – Medical: Integrated Nursing Process – ditions, is not taking other medications,
Implementation: Client Needs – Safe Effective Care Envi- and is not pregnant, herbal medications
ronment, Management of Care: Cognitive Level – may be helpful in treating the common
Application: Concept – Oxygenation. cold.
11 mL/hr.
39. Content – Medical: Integrated Nursing Process –
First, convert pounds to kilograms: Implementation: Client Needs – Physiological Integrity,
Pharmacological and Parenteral Therapies: Cognitive
165 pounds ÷ 2.2 = 75 kg
Level – Application: Concept – Oxygenation.
Then, determine how many milligrams
of aminophylline per hour should be 42. 1. Epinephrine is administered intravenously
administered: during an arrest in a code situation, but it is
0.3 mg × 75 kg = 22.5 mg/hr not a treatment of choice for bronchospasms.
Then, determine how much aminophylline is 2. Albuterol given via nebulizer is adminis-
delivered per milliliter: tered to stop the bronchospasms. If the
500 mg ÷ 250 mL = 2 mg/1 mL client continues to have the broncho-
If 2 mg/1 mL is delivered, then to deliver the spasms, intubation may be needed.
prescribed 22.5 mg/hr, the rate must be set at: 3. A STAT portable x-ray will be ordered, but
22.5 ÷ 2 = 11.25 mL/hr the goal is to prevent respiratory arrest.
Less than 0.5 should be rounded down, 0.5 4. Nasal trumpet airways would not be helpful
and above is rounded up. in stopping the bronchospasm and respiratory
Content – Medical: Integrated Nursing Process – arrest.
Implementation: Client Needs – Physiological Integrity, Content – Medical: Integrated Nursing Process –
Pharmacological and Parenteral Therapies: Cognitive Implementation: Client Needs – Physiological Integrity,
Level – Application: Concept – Medication. Pharmacological and Parenteral Therapies: Cognitive
Level – Application: Concept – Oxygenation.
40. 1. Antihistamines dry respiratory secretions
through an anticholinergic effect; therefore, 43. 1. The client is hyperventilating and blowing
the client will have a dry mouth. off too much CO2, which is why her fin-
2. Antihistamines cause drowsiness; therefore, gers are tingling and her mouth is numb;
the client should not drink any type of alcohol. she needs to retain CO2 by taking slow,
3. Antihistamines cause drowsiness, so the deep breaths.
client should not drive or operate any type 2. Putting the head between the legs sometimes
of machinery. helps a client who is going to faint, but it is
4. Antihistamines cause drowsiness; therefore, not the first intervention.
the client understands the teaching. 3. The client is hyperventilating; determining
Content – Medical: Integrated Nursing Process – why is not appropriate at this time.
Evaluation: Client Needs – Physiological Integrity, Phar- 4. Medications take up to 30 minutes to one (1)
macological and Parenteral Therapies: Cognitive Level – hour to work and are not the first interven-
Synthesis: Concept – Oxygenation. tion for the client who is hyperventilating.
41. 1. The nurse can answer the client’s questions Content – Medical: Integrated Nursing Process –
concerning herbal medication. Passing the Implementation: Client Needs – Physiological Integrity,
Physiological Adaptation: Cognitive Level – Application:
buck should be eliminated as a possible cor-
Concept – Oxygenation.
rect answer.

06_Colgrove_Ch06.indd 245 01/06/16 5:16 PM


246 M ed -S urg S uccess

44. 1. Sodium bicarbonate is administered for meta- 3. Daily vital signs would not indicate the respi-
bolic acidosis. ratory status of the client.
2. The arterial oxygen level is within normal 4. Assessing the client’s capillary refill time
limits (80 to 100); therefore, the client does has the highest priority for the nurse be-
not need oxygen. cause it indicates the oxygenation of the
3. The client is retaining CO2, which causes client.
respiratory acidosis, and the nurse should Content – Medical: Integrated Nursing Process –
help the client remove the CO2 by in- Implementation: Client Needs – Safe Effective Care
structing the client to cough and deep Environment, Management of Care: Cognitive Level –
breathe. Synthesis: Concept – Oxygenation.
4. Breathing into a paper bag is not recom- 48. 1. The therapeutic level is 10 to 20 mcg/mL;
mended for clients in respiratory acidosis. therefore, the nurse should hang the bag
Content – Medical: Integrated Nursing Process – and continue the infusion to maintain the
Implementation: Client Needs – Physiological Integrity, aminophylline level.
Physiological Adaptation: Cognitive Level – Application:
2. There is no reason not to hang the next bag
Concept – Oxygenation.
of aminophylline.
45. 1. Dull sounds would be heard over the site of a 3. There is no need to notify the health-care
lung abscess as a result of the solid mass. provider for a level of 18 mcg/mL.
2. Crackles may be heard, but wheezes indicate 4. There is no need for the nurse to confirm the
a narrowing of airways, not exudates-filled laboratory results.
airways. Content – Medical: Integrated Nursing Process –
3. Diminished or absent sounds are heard Implementation: Client Needs – Physiological Integrity,
with intermittent pleural friction rubs. Pharmacological and Parenteral Therapies: Cognitive
A lung abscess is the accumulation of pus Level – Application: Concept – Medication.
in an area where pneumonia was present
49. 1. The client’s current temperature would not
that becomes encapsulated and can extend
affect the administration of the antibiotic.
to the bronchus or pleural space.
2. The client’s white blood cells may be elevated
4. Even with a lung abscess, the chest should because of the infection, but this would not
move symmetrically.
affect administering the medication.
Content – Medical: Integrated Nursing Process – 3. A culture needs to be collected prior to
Assessment: Client Needs – Physiological Integrity, Reduc- the first dose of antibiotic, or the culture
tion of Risk Potential: Cognitive Level – Analysis:
and sensitivity will be skewed and the ap-
Concept – Oxygenation.
propriate antibiotic needed to treat the re-
46. 1. Nurses are responsible for accounting for spiratory infection may not be identified.
medications, but it is not the rationale 4. Compatibility of fluids should be assessed
for DOT. prior to administering each intravenous anti-
2. Nurses complete forms as required by all biotic, but when administering the first dose
governmental agencies, but this is not the of an antibiotic, the nurse must check to make
­rationale for DOT. sure the sputum culture was obtained.
3. Documentation of events concerning the Content – Medical: Integrated Nursing Process –
­client’s treatment is completed, but this is Implementation: Client Needs – Physiological Integrity,
not the rationale for DOT. Pharmacological and Parenteral Therapies: Cognitive
4. To ensure the compliance with all medi- Level – Application: Concept – Infection.
cations regimens, the health department 50. 1. A client with a respiratory disorder may
has adapted a directly observed therapy have decreased oxygen saturation; there-
(DOT) where the nurse actually observes fore, administering oxygen via a nasal can-
the client taking the medication every day. nula is appropriate.
Content – Medical: Integrated Nursing Process – 2. The client’s lung sounds should be
Diagnosis: Client Needs – Safe Effective Care Environ- assessed to determine how much air is be-
ment, Management of Care: Cognitive Level – Analysis:
ing exchanged in the lungs.
Concept – Oxygenation.
3. Coughing and deep breathing will help
47. 1. The nurse should gather data before imple- the client expectorate sputum, thus clear-
menting an intervention. ing the bronchial tree.
2. The radial pulse would indicate the cardio- 4. The pulse oximeter evaluates how much
vascular status of the client, not the respira- oxygen is reaching the periphery.
tory status, and the nurse should assess the 5. Increasing fluids will help thin secretions,
apical pulse. making them easier to expectorate.

06_Colgrove_Ch06.indd 246 01/06/16 5:16 PM


C hapter 6  R espiratory D isorders 247

Content – Medical: Integrated Nursing Process – not in distress, then the nurse should as-
Implementation: Client Needs – Physiological Integrity, sess the machine first to determine which
Physiological Adaptation: Cognitive Level – Application: alarm is sounding.
Concept – Oxygenation.
Content – Medical: Integrated Nursing Process –
51. In order of priority: 5, 2, 3, 4, 1. Assessment: Client Needs – Physiological Integrity, Reduc-
5. The most common cause of bucking the tion of Risk Potential: Cognitive Level – Analysis:
ventilator is obstructed airway, which Concept – Oxygenation.
could be secondary to secretions in the 52. 1. This client has an immune system that is re-
airway, so assessing the client would be sponding to an infection, not one suppressed
most appropriate. by steroids.
2. Clients in the ICD are constantly moni- 2. This client’s graphic indicates that the client
tored by pulse oximetry; therefore, the was admitted without an infection and now
nurse should determine if the client has has developed one and the immune system is
decreased oxygen saturation and if so, the responding.
nurse should start to “bag” the client. The 3. This client’s pattern of temperatures does not
client is in respiratory distress. indicate an infection exists.
3. The nurse should assess the client’s lung 4. The client diagnosed with COPD and on
fields to determine if air movement is oc- long-term steroids has a suppressed im-
curring because the client is in respiratory mune system. Temperatures are in the
distress. normal range because steroids mask the
4. A complication of mechanical ventilation symptoms of infection by suppressing the
is a pneumothorax, and the nurse should immune system. The only common symp-
assess for this because the client is in re- toms would be rusty-colored sputum or a
spiratory distress. change in sputum color and confusion.
1. The machine is alerting the nurse there Content – Medical: Integrated Nursing Process –
is a problem with the client; because the Assessment: Client Needs – Physiological Integrity, Physi-
client is in respiratory distress, the client ological Adaptation: Cognitive Level – Synthesis:
should be assessed first. If the client were Concept – Infection.

06_Colgrove_Ch06.indd 247 01/06/16 5:16 PM


06_Colgrove_Ch06.indd 248 01/06/16 5:16 PM
Gastrointestinal
Disorders
Science is knowledge; wisdom is organized life.
—Immanuel Kant
7
The many organs making up the gastrointestinal system—the mouth, esophagus, stom-
ach, upper and lower intestine, and related organs—are subject to many disorders/diseases.
Some are relatively minor, such as temporary constipation or a short bout of diarrhea and
gastroenteritis. Others, such as diverticulosis and inflammatory bowel disease, may be
chronic, requiring the client to follow a specific diet and other lifestyle modifications. Some
chronic diseases, including gastroesophageal reflux, may eventually lead to life-threatening
problems such as esophageal cancer. Still other diseases affect the gastrointestinal tract.
One—colon cancer—is one of the most common cancers in the United States. In addition,
eating disorders rooted in psychological problems can be serious if not addressed promptly
and effectively. Because gastrointestinal diseases/disorders are so common, the nurse must
be aware of the signs/symptoms of each, what is considered normal or abnormal for the
disease process, and how the specific problem is treated.

KEYWORDS abbreviations
Ascites Acquired immunodeficiency syndrome (AIDS)
Asterixis Blood pressure (BP)
Borborygmus Body mass index (BMI)
Caput medusae Esophagogastroduodenoscopy (EGD)
Cathartic Gastroesophageal reflux disease (GERD)
Cruciferous Gastrointestinal (GI)
Dyspepsia Head of bed (HOB)
Dysphagia Health-care provider (HCP)
Eructation Inflammatory bowel disease (IBD)
Esophagogastroduodenoscopy Intake and output (I&O)
Evisceration International normalized ratio (INR)
Exacerbation Intravenous (IV)
Feces Nasogastric (N/G) tube
Hematemesis Nonsteroidal anti-inflammatory drugs (NSAIDs)
Hypoalbuminemia Nothing by mouth (NPO)
Jaundice Partial thromboplastin time (PTT)
Lower esophageal sphincter Patient-controlled analgesia (PCA)
Melena Percutaneous endoscopic gastrostomy (PEG)
Nosocomial Prothrombin time (PT)
Odynophagia Pulse (P)
Oligomenorrhea Red blood cells (RBCs)
Peritonitis Three times a day (tid)
Pruritus Total parenteral nutrition (TPN)
Pyrosis

249

07_Colgrove_Ch07.indd 249 03/06/16 12:58 PM


250 M ed -S urg S uccess

Sedentary Unlicensed assistive personnel (UAP)


Steatorrhea When required, as needed (prn)
Tenesmus White blood cells (WBCs)
Water brash Within normal limits (WNL)

PRACTICE QUESTIONS
Gastroesophageal Reflux (GERD) 3. Eat four (4) to six (6) small meals a day and
limit fluids during mealtimes.
1. The male client tells the nurse he has been 4. Encourage the client to consume a glass of red
experiencing “heartburn” at night that awakens wine with one (1) meal a day.
him. Which assessment question should the 5. The nurse is caring for a client diagnosed with
nurse ask? GERD. Which nursing interventions should be
1. “How much weight have you gained recently?” implemented?
2. “What have you done to alleviate the heartburn?” 1. Place the client prone in bed and administer
3. “Do you consume many milk and dairy nonsteroidal anti-inflammatory medications.
products?” 2. Have the client remain upright at all times and
4. “Have you been around anyone with a stomach walk for 30 minutes three (3) times a week.
virus?” 3. Instruct the client to maintain a right lateral
2. The nurse caring for a client diagnosed with side-lying position and take antacids before
GERD writes the client problem of “behavior meals.
modification.” Which intervention should be 4. Elevate the head of the bed (HOB) 30 degrees
included for this problem? and discuss lifestyle modifications with the
1. Teach the client to sleep with a foam wedge client.
under the head. 6. The nurse is caring for an adult client diagnosed
2. Encourage the client to decrease the amount with GERD. Which condition is the most
of smoking. common comorbid disease associated with
3. Instruct the client to take over-the-counter GERD?
medication for relief of pain. 1. Adult-onset asthma.
4. Discuss the need to attend Alcoholics 2. Pancreatitis.
Anonymous to quit drinking. 3. Peptic ulcer disease.
3. The nurse is preparing a client diagnosed 4. Increased gastric emptying.
with GERD for discharge following an 7. The nurse is administering morning medications
esophagogastroduodenoscopy (EGD). Which at 0730. Which medication should have priority?
statement indicates the client understands the 1. A proton pump inhibitor.
discharge instructions? 2. A nonnarcotic analgesic.
1. “I should not eat for at least one (1) day 3. A histamine receptor antagonist.
following this procedure.” 4. A mucosal barrier agent.
2. “I can lie down whenever I want after a meal.
It won’t make a difference.” 8. The nurse is preparing a client diagnosed with
3. “The stomach contents won’t bother my GERD for surgery. Which information warrants
esophagus but will make me nauseous.” notifying the HCP?
4. “I should avoid orange juice and eating 1. The client’s Bernstein esophageal test was
tomatoes until my esophagus heals.” positive.
2. The client’s abdominal x-ray shows a hiatal
4. The nurse is planning the care of a client diagnosed hernia.
with lower esophageal sphincter dysfunction. Which 3. The client’s WBC count is 14,000/mm3.
dietary modifications should be included in the plan 4. The client’s hemoglobin is 13.8 g/dL.
of care?
1. Allow any of the client’s favorite foods as long
as the amount is limited.
2. Have the client perform eructation exercises
several times a day.

07_Colgrove_Ch07.indd 250 03/06/16 12:58 PM


C hapter 7  G astrointestinal D isorders 251

9. The charge nurse is making assignments. 14. The client with type 2 diabetes is prescribed
Staffing includes a registered nurse with five prednisone, a steroid, for an acute exacerbation
(5) years of medical-surgical experience, a of inflammatory bowel disease (IBD). Which
newly graduated registered nurse, and two intervention should the nurse discuss with the
(2) unlicensed assistive personnel (UAPs). client?
Which client should be assigned to the most 1. Take this medication on an empty stomach.
experienced nurse? 2. Notify the HCP if experiencing a moon face.
1. The 39-year-old client diagnosed with lower 3. Take the steroid medication as prescribed.
esophageal dysfunction who is complaining of 4. Notify the HCP if the blood glucose is
pyrosis. over 160.
2. The 54-year-old client diagnosed with
15. The client diagnosed with inflammatory
Barrett’s esophagus who is scheduled to have
bowel disease has a serum potassium level of
an endoscopy this morning.
3.4 mEq/L. Which action should the nurse
3. The 46-year-old client diagnosed with
implement first?
gastroesophageal reflux disease who has
1. Notify the health-care provider (HCP).
wheezes in all five (5) lobes.
2. Assess the client for muscle weakness.
4. The 68-year-old client who is three (3) days
3. Request telemetry for the client.
postoperative for hiatal hernia and needs to be
4. Prepare to administer potassium IV.
ambulated four (4) times today.
16. The client is diagnosed with an acute
10. Which statement made by the client indicates
exacerbation of ulcerative colitis. Which
to the nurse the client may be experiencing
intervention should the nurse implement?
GERD?
1. Provide a low-residue diet.
1. “My chest hurts when I walk up the stairs in
2. Rest the client’s bowel.
my home.”
3. Assess vital signs daily.
2. “I take antacid tablets with me wherever I go.”
4. Administer antacids orally.
3. “My spouse tells me I snore very loudly at
night.” 17. The client diagnosed with IBD is prescribed
4. “I drink six (6) to seven (7) soft drinks every day.” total parenteral nutrition (TPN). Which
intervention should the nurse implement?
11. The nurse is performing an admission
1. Check the client’s glucose level.
assessment on a client diagnosed with GERD.
2. Administer an oral hypoglycemic.
Which signs and symptoms would indicate
3. Assess the peripheral intravenous site.
GERD?
4. Monitor the client’s oral food intake.
1. Pyrosis, water brash, and flatulence.
2. Weight loss, dysarthria, and diarrhea. 18. The client is diagnosed with an acute
3. Decreased abdominal fat, proteinuria, and exacerbation of IBD. Which priority
constipation. intervention should the nurse implement?
4. Midepigastric pain, positive H. pylori test, and 1. Weigh the client daily and document in the
melena. client’s chart.
2. Teach coping strategies such as dietary
12. Which disease is the client diagnosed with
modifications.
GERD at greater risk for developing?
3. Record the frequency, amount, and color of
1. Hiatal hernia.
stools.
2. Gastroenteritis.
4. Monitor the client’s oral fluid intake every shift.
3. Esophageal cancer.
4. Gastric cancer. 19. The client diagnosed with Crohn’s disease
is crying and tells the nurse, “I can’t take it
anymore. I never know when I will get sick and
Inflammatory Bowel Disease end up here in the hospital.” Which statement is
the nurse’s best response?
13. Which sign/symptom should the nurse expect to
1. “I understand how frustrating this must be
find in a client diagnosed with ulcerative colitis?
for you.”
1. Twenty bloody stools a day.
2. “You must keep thinking about the good
2. Oral temperature of 102°F.
things in your life.”
3. Hard, rigid abdomen.
3. “I can see you are very upset. I’ll sit down and
4. Urinary stress incontinence.
we can talk.”
4. “Are you thinking about doing anything like
committing suicide?”

07_Colgrove_Ch07.indd 251 03/06/16 12:58 PM


252 M ed -S urg S uccess

20. The client diagnosed with ulcerative colitis has an Peptic Ulcer Disease
ileostomy. Which statement indicates the client
needs more teaching concerning the ileostomy? 25. Which assessment data supports the client’s
1. “My stoma should be pink and moist.” diagnosis of gastric ulcer to the nurse?
2. “I will irrigate my ileostomy every morning.” 1. Presence of blood in the client’s stool for the
3. “If I get a red, bumpy, itchy rash I will call my past month.
HCP.” 2. Reports of a burning sensation moving like
4. “I will change my pouch if it starts leaking.” a wave.
21. The client diagnosed with IBD is prescribed 3. Sharp pain in the upper abdomen after eating
sulfasalazine (Asulfidine), a sulfonamide a heavy meal.
antibiotic. Which statement best describes the 4. Complaints of epigastric pain 30 to
rationale for administering this medication? 60 minutes after ingesting food.
1. It is administered rectally to help decrease 26. The nurse is caring for a client diagnosed
colon inflammation. with rule-out peptic ulcer disease. Which test
2. This medication slows gastrointestinal (GI) confirms this diagnosis?
motility and reduces diarrhea. 1. Esophagogastroduodenoscopy.
3. This medication kills the bacteria causing the 2. Magnetic resonance imaging (MRI).
exacerbation. 3. Occult blood test.
4. It acts topically on the colon mucosa to 4. Gastric acid stimulation.
decrease inflammation.
27. Which specific data should the nurse obtain
22. The client is diagnosed with Crohn’s disease, from the client who is suspected of having peptic
also known as regional enteritis. Which ulcer disease?
statement by the client supports this diagnosis? 1. History of side effects experienced from all
1. “My pain goes away when I have a bowel medications.
movement.” 2. Use of nonsteroidal anti-inflammatory drugs
2. “I have bright red blood in my stool all the (NSAIDs).
time.” 3. Any known allergies to drugs and
3. “I have episodes of diarrhea and constipation.” environmental factors.
4. “My abdomen is hard and rigid and I have 4. Medical histories of at least three (3)
a fever.” generations.
23. The client diagnosed with ulcerative colitis 28. Which physical examination should the nurse
is prescribed a low-residue diet. Which meal implement first when assessing the client
selection indicates the client understands the diagnosed with peptic ulcer disease?
diet teaching? 1. Auscultate the client’s bowel sounds in all four
1. Grilled hamburger on a wheat bun and fried quadrants.
potatoes. 2. Palpate the abdominal area for tenderness.
2. A chicken salad sandwich and lettuce and 3. Percuss the abdominal borders to identify
tomato salad. organs.
3. Roast pork, white rice, and plain custard. 4. Assess the tender area progressing to
4. Fried fish, whole grain pasta, and fruit salad. nontender.
24. The client with ulcerative colitis is scheduled for an 29. Which problems should the nurse include in
ileostomy. The nurse is aware the client’s stoma will the plan of care for the client diagnosed with
be located in which area of the abdomen? peptic ulcer disease to observe for physiological
complications?
D
C 1. Alteration in bowel elimination patterns.
2. Knowledge deficit in the causes of ulcers.
3. Inability to cope with changing family roles.
4. Potential for alteration in gastric emptying.
A B

1. A
2. B
3. C
4. D

07_Colgrove_Ch07.indd 252 03/06/16 12:58 PM


C hapter 7  G astrointestinal D isorders 253

30. The nurse is caring for a client diagnosed 36. The client with a history of peptic ulcer disease
with hemorrhaging duodenal ulcer. Which is admitted into the intensive care department
collaborative interventions should the nurse with frank gastric bleeding. Which priority
implement? Select all that apply. intervention should the nurse implement?
1. Perform a complete pain assessment. 1. Maintain a strict record of intake and output.
2. Assess the client’s vital signs frequently. 2. Insert a nasogastric (N/G) tube and begin
3. Administer a proton pump inhibitor saline lavage.
intravenously. 3. Assist the client with keeping a detailed
4. Obtain permission and administer blood calorie count.
products. 4. Provide a quiet environment to promote rest.
5. Monitor the intake of a soft, bland diet.
31. Which expected outcome should the nurse
Colorectal Disease
include for a client diagnosed with peptic ulcer
37. The occupational health nurse is preparing a
disease?
presentation to a group of factory workers about
1. The client’s pain is controlled with the use of
preventing colon cancer. Which information
NSAIDs.
should be included in the presentation?
2. The client maintains lifestyle modifications.
1. Wear a high-filtration mask when around
3. The client has no signs and symptoms of
chemicals.
hemoptysis.
2. Eat several servings of cruciferous vegetables
4. The client takes antacids with each meal.
daily.
32. The nurse has been assigned to care for a client 3. Take a multiple vitamin every day.
diagnosed with peptic ulcer disease. Which 4. Do not engage in high-risk sexual behaviors.
assessment data require further intervention?
38. The nurse is admitting a client to a medical
1. Bowel sounds auscultated 15 times in one (1)
floor with a diagnosis of adenocarcinoma of
minute.
the rectosigmoid colon. Which assessment data
2. Belching after eating a heavy and fatty meal
support this diagnosis?
late at night.
1. The client reports up to 20 bloody stools
3. A decrease in systolic blood pressure (BP) of
per day.
20 mm Hg from lying to sitting.
2. The client has a feeling of fullness after
4. A decreased frequency of distress located in
a heavy meal.
the epigastric region.
3. The client has diarrhea alternating with
33. Which oral medication should the nurse constipation.
question before administering to the client 4. The client complains of right lower quadrant
with peptic ulcer disease? pain.
1. E-mycin, an antibiotic.
39. The 85-year-old male client diagnosed with
2. Prilosec, a proton pump inhibitor.
cancer of the colon asks the nurse, “Why did I
3. Flagyl, an antimicrobial agent.
get this cancer?” Which statement is the nurse’s
4. Tylenol, a nonnarcotic analgesic.
best response?
34. The nurse has administered an antibiotic, a 1. “Research shows a lack of fiber in the diet can
proton pump inhibitor, and Pepto-Bismol for cause colon cancer.”
peptic ulcer disease secondary to H. pylori. 2. “It is not common to get colon cancer at your
Which data would indicate to the nurse the age; it is usually in young people.”
medications are effective? 3. “No one knows why anyone gets cancer, it
1. A decrease in alcohol intake. just happens to certain people.”
2. Maintaining a bland diet. 4. “Women usually get colon cancer more often
3. A return to previous activities. than men but not always.”
4. A decrease in gastric distress.
40. The nurse is planning the care of a client who
35. Which assessment data indicate to the nurse has had an abdominal–perineal resection for
the client’s gastric ulcer has perforated? cancer of the colon. Which interventions should
1. Complaints of sudden, sharp, substernal pain. the nurse implement? Select all that apply.
2. Rigid, boardlike abdomen with rebound 1. Provide meticulous skin care to stoma.
tenderness. 2. Assess the flank incision.
3. Frequent, clay-colored, liquid stool. 3. Maintain the indwelling catheter.
4. Complaints of vague abdominal pain in the 4. Irrigate the (JP) drains every shift.
right upper quadrant. 5. Position the client semirecumbent.

07_Colgrove_Ch07.indd 253 03/06/16 12:58 PM


254 M ed -S urg S uccess

41. The client who has had an abdominal perineal 46. The nurse is caring for clients in an outpatient
resection is being discharged. Which discharge clinic. Which information should the nurse
information should the nurse teach? teach regarding the American Cancer Society’s
1. The stoma should be a white, blue, or purple recommendations for the early detection of
color. colon cancer?
2. Limit ambulation to prevent the pouch from 1. Beginning at age 60, a digital rectal
coming off. examination should be done yearly.
3. Take pain medication when the pain level is at 2. After reaching middle age, a yearly fecal
an “8.” occult blood test should be done.
4. Empty the pouch when it is one-third to 3. Have a colonoscopy at age 50 and then once
one-half full. every five (5) to 10 years.
4. A flexible sigmoidoscopy should be done
42. The nurse caring for a client one (1) day
yearly after age 40.
postoperative sigmoid resection notes a moderate
amount of dark reddish brown drainage on the 47. The nurse writes a psychosocial problem of
midline abdominal incision. Which intervention “risk for altered sexual functioning related to
should the nurse implement first? new colostomy.” Which intervention should the
1. Mark the drainage on the dressing with the nurse implement?
time and date. 1. Tell the client there should be no intimacy for
2. Change the dressing immediately using sterile at least three (3) months.
technique. 2. Ensure the client and significant other are
3. Notify the health-care provider immediately. able to change the ostomy pouch.
4. Reinforce the dressing with a sterile gauze pad. 3. Demonstrate with charts possible sexual
positions for the client to assume.
43. The client complains to the nurse of unhappiness
4. Teach the client to protect the pouch from
with the health-care provider. Which intervention
becoming dislodged during sex.
should the nurse implement next?
1. Call the HCP and suggest he or she talk with 48. The client presents with a complete blockage of
the client. the large intestine from a tumor. Which health-
2. Determine what about the HCP is bothering care provider’s order would the nurse question?
the client. 1. Obtain consent for a colonoscopy and biopsy.
3. Notify the nursing supervisor to arrange 2. Start an IV of 0.9% saline at 125 mL/hr.
a new HCP to take over. 3. Administer 3 liters of GoLYTELY.
4. Explain the client cannot request another 4. Give tap water enemas until it is clear.
HCP until after discharge.
44. The client with a new colostomy is being Diverticulosis/Diverticulitis
discharged. Which statement made by the client
indicates the need for further teaching? 49. The client diagnosed with diverticulitis is
1. “If I notice any skin breakdown, I will call the complaining of severe pain in the left lower
HCP.” quadrant and has an oral temperature of
2. “I should drink only liquids until the 100.6°F. Which intervention should the nurse
colostomy starts to work.” implement first?
3. “I should not take a tub bath until the HCP 1. Notify the health-care provider.
okays it.” 2. Document the findings in the chart.
4. “I should not drive or lift more than five 3. Administer an oral antipyretic.
(5) pounds.” 4. Assess the client’s abdomen.
45. The nurse is preparing to hang a new bag of 50. The nurse is teaching the client diagnosed with
total parenteral nutrition for a client with an diverticulosis. Which instruction should the
abdominal perineal resection. The bag has nurse include in the teaching session?
1,500 mL of 50% dextrose, 10 mL of trace 1. Discuss the importance of drinking 1,000 mL
elements, 20 mL of multivitamins, 20 mL of water daily.
of potassium chloride, and 500 mL of lipids. 2. Instruct the client to exercise at least three
The bag is to infuse over the next 24 hours. (3) times a week.
At what rate should the nurse set the pump? 3. Teach the client about eating a low-residue diet.
_________ 4. Explain the need to have daily bowel
movements.

07_Colgrove_Ch07.indd 254 03/06/16 12:58 PM


C hapter 7  G astrointestinal D isorders 255

51. The client is admitted to the medical unit with 57. The client with acute diverticulitis has
a diagnosis of acute diverticulitis. Which health- a nasogastric tube draining green liquid
care provider’s order should the nurse question? bile. Which intervention should the nurse
1. Insert a nasogastric tube. implement?
2. Start an IV with D5W at 125 mL/hr. 1. Document the findings as normal.
3. Put the client on a clear liquid diet. 2. Assess the client’s bowel sounds.
4. Place the client on bedrest with bathroom 3. Determine the client’s last bowel movement.
privileges. 4. Insert the N/G tube at least two (2) more
inches.
52. The nurse is discussing the therapeutic diet for
the client diagnosed with diverticulosis. Which 58. The nurse is teaching a class on diverticulosis.
meal indicates the client understands the Which interventions should the nurse discuss
discharge teaching? when teaching ways to prevent an acute
1. Fried fish, mashed potatoes, and iced tea. exacerbation of diverticulosis? Select all that
2. Ham sandwich, applesauce, and whole milk. apply.
3. Chicken salad on whole-wheat bread and 1. Eat a high-fiber diet.
water. 2. Increase fluid intake.
4. Lettuce, tomato, and cucumber salad and 3. Elevate the HOB after eating.
coffee. 4. Walk 30 minutes a day.
5. Take an antacid every two (2) hours.
53. The client is two (2) hours post colonoscopy.
Which assessment data warrant intermediate 59. The nurse is working in an outpatient clinic.
intervention by the nurse? Which client is most likely to have a diagnosis
1. The client has a soft, nontender abdomen. of diverticulosis?
2. The client has a loose, watery stool. 1. A 60-year-old male with a sedentary lifestyle.
3. The client has hyperactive bowel sounds. 2. A 72-year-old female with multiple childbirths.
4. The client’s pulse is 104 and BP is 98/60. 3. A 63-year-old female with hemorrhoids.
4. A 40-year-old male with a family history of
54. The nurse is preparing to administer the initial
diverticulosis.
dose of an aminoglycoside antibiotic to the
client diagnosed with acute diverticulitis. Which 60. The client is admitted to the medical floor
intervention should the nurse implement? with acute diverticulitis. Which collaborative
1. Obtain a serum trough level. intervention should the nurse anticipate the
2. Ask about drug allergies. health-care provider ordering?
3. Monitor the peak level. 1. Administer total parenteral nutrition.
4. Assess the vital signs. 2. Maintain NPO and nasogastric tube.
3. Maintain on a high-fiber diet and increase
55. The client diagnosed with acute diverticulitis
fluids.
is complaining of severe abdominal pain.
4. Obtain consent for abdominal surgery.
On assessment, the nurse finds a hard, rigid
abdomen and T 102°F. Which intervention
should the nurse implement?
1. Notify the health-care provider.
Gallbladder Disorders
2. Prepare to administer a Fleet’s enema.
61. The client is four (4) hours postoperative
3. Administer an antipyretic suppository.
open cholecystectomy. Which data warrant
4. Continue to monitor the client closely.
immediate intervention by the nurse?
56. The nurse is preparing to administer 250 mL 1. Absent bowel sounds in all four (4)
of intravenous antibiotic to the client. The quadrants.
medication must infuse in one (1) hour. An 2. The T-tube has 60 mL of green drainage.
intravenous pump is not available and the nurse 3. Urine output of 100 mL in the past three
must administer the medication via gravity with (3) hours.
IV tubing at 10 gtts/min. At what rate should 4. Refusal to turn, deep breathe, and cough.
the nurse infuse the medication? _________

07_Colgrove_Ch07.indd 255 03/06/16 12:58 PM


256 M ed -S urg S uccess

62. The client two (2) hours postoperative 3. Assess for return of a gag reflex.
laparoscopic cholecystectomy is complaining of 4. Administer held medications.
severe pain in the right shoulder. Which nursing
68. Which outcome should the nurse identify
intervention should the nurse implement?
for the client scheduled to have a
1. Apply a heating pad to the abdomen for 15 to
cholecystectomy?
20 minutes.
1. Decreased pain management.
2. Administer morphine sulfate intravenously
2. Ambulate first day postoperative.
after diluting with saline.
3. No break in skin integrity.
3. Contact the surgeon for an order to x-ray the
4. Knowledge of postoperative care.
right shoulder.
4. Apply a sling to the right arm, which was 69. Which assessment data indicate to the nurse the
injured during surgery. client recovering from an open cholecystectomy
may require pain medication?
63. The nurse is teaching a client recovering from a
1. The client’s pulse is 65 beats per minute.
laparoscopic cholecystectomy. Which statement
2. The client has shallow respirations.
indicates the discharge teaching is effective?
3. The client’s bowel sounds are 20 per minute.
1. “I will take my lipid-lowering medicine at the
4. The client uses a pillow to splint when
same time each night.”
coughing.
2. “I may experience some discomfort when I eat
a high-fat meal.” 70. The charge nurse is monitoring client laboratory
3. “I need someone to stay with me for about a values. Which value is expected in the client with
week after surgery.” cholecystitis who has chronic inflammation?
4. “I should not splint my incision when I deep 1. An elevated white blood cell (WBC) count.
breathe and cough.” 2. A decreased lactate dehydrogenase (LDH).
3. An elevated alkaline phosphatase.
64. Which signs and symptoms should the nurse
4. A decreased direct bilirubin level.
report to the health-care provider for the client
recovering from an open cholecystectomy? 71. Which problem is highest priority for the
Select all that apply. nurse to identify in the client who had an open
1. Clay-colored stools. cholecystectomy surgery?
2. Yellow-tinted sclera. 1. Alteration in nutrition.
3. Amber-colored urine. 2. Alteration in skin integrity.
4. Wound approximated. 3. Alteration in urinary pattern.
5. Abdominal pain. 4. Alteration in comfort.
65. The nurse is caring for the immediate 72. The nurse assesses a large amount of red
postoper­ative client who had a laparoscopic drainage on the dressing of a client who is six
cholecystectomy. Which task could the nurse (6) hours postoperative open cholecystectomy.
delegate to the unlicensed assistive personnel Which intervention should the nurse
(UAP)? implement?
1. Check the abdominal dressings for bleeding. 1. Measure the abdominal girth.
2. Increase the IV fluid if the blood pressure 2. Palpate the lower abdomen for a mass.
is low. 3. Turn client onto side to assess for further
3. Ambulate the client to the bathroom. drainage.
4. Auscultate the breath sounds in all lobes. 4. Remove the dressing to determine the source.
66. Which data should the nurse expect to assess
in the client who had an upper gastrointestinal Liver Failure
(UGI) series?
1. Chalky white stools. 73. The client diagnosed with end-stage liver failure
2. Increased heart rate. is admitted with esophageal bleeding. The HCP
3. A firm, hard abdomen. inserts and inflates a triple-lumen nasogastric
4. Hyperactive bowel sounds. tube (Sengstaken-Blakemore). Which nursing
intervention should the nurse implement for this
67. The client is one (1) hour post–endoscopic
treatment?
retrograde cholangiopancreatogram (ERCP).
1. Assess the gag reflex every shift.
Which intervention should the nurse include
2. Stay with the client at all times.
in the plan of care?
3. Administer the laxative lactulose (Chronulac).
1. Instruct the client to cough forcefully.
4. Monitor the client’s ammonia level.
2. Encourage early ambulation.

07_Colgrove_Ch07.indd 256 03/06/16 12:58 PM


C hapter 7  G astrointestinal D isorders 257

74. The client has had a liver biopsy. Which 79. The client in end-stage liver failure has vitamin
postprocedure intervention should the nurse K deficiency. Which interventions should the
implement? nurse implement? Select all that apply.
1. Instruct the client to void immediately. 1. Avoid rectal temperatures.
2. Keep the client NPO for eight (8) hours. 2. Use only a soft toothbrush.
3. Place the client on the right side. 3. Monitor the platelet count.
4. Monitor blood urea nitrogen (BUN) and 4. Use small-gauge needles.
creatinine level. 5. Assess for asterixis.
75. The client diagnosed with end-stage liver failure 80. Which gastrointestinal assessment data should
is admitted with hepatic encephalopathy. Which the nurse expect to find when assessing the client
dietary restriction should be implemented by the in end-stage liver failure?
nurse to address this complication? 1. Hypoalbuminemia and muscle wasting.
1. Restrict sodium intake to 2 g/day. 2. Oligomenorrhea and decreased body hair.
2. Limit oral fluids to 1,500 mL/day. 3. Clay-colored stools and hemorrhoids.
3. Decrease the daily fat intake. 4. Dyspnea and caput medusae.
4. Reduce protein intake to 60 to 80 g/day.
81. Which assessment question is priority for the
76. The client diagnosed with end-stage renal nurse to ask the client diagnosed with end-stage
failure and ascites is scheduled for a paracentesis. liver failure secondary to alcoholic cirrhosis?
Which client teaching should the nurse discuss 1. “How many years have you been drinking
with the client? alcohol?”
1. Explain the procedure will be done in the 2. “Have you completed an advance directive?”
operating room. 3. “When did you have your last alcoholic drink?”
2. Instruct the client a Foley catheter will have 4. “What foods did you eat at your last meal?”
to be inserted.
82. The client has end-stage liver failure secondary
3. Tell the client vital signs will be taken
to alcoholic cirrhosis. Which complication
frequently after the procedure.
indicates the client is at risk for developing
4. Provide instructions on holding the breath
hepatic encephalopathy?
when the HCP inserts the catheter.
1. Gastrointestinal bleeding.
77. The client diagnosed with liver failure is 2. Hypoalbuminemia.
experiencing pruritus secondary to severe 3. Splenomegaly.
jaundice. Which action by the unlicensed 4. Hyperaldosteronism.
assistive personnel (UAP) warrants intervention
83. The client is diagnosed with end-stage liver
by the nurse?
failure. The client asks the nurse, “Why is my
1. The UAP is assisting the client to take a hot,
doctor decreasing the doses of my medications?”
soapy shower.
Which statement is the nurse’s best response?
2. The UAP applies an emollient to the client’s
1. “You are worried because your doctor has
legs and back.
decreased the dosage.”
3. The UAP puts mittens on both hands of the
2. “You really should ask your doctor. I am sure
client.
there is a good reason.”
4. The UAP pats the client’s skin dry with a
3. “You may have an overdose of the medications
clean towel.
because your liver is damaged.”
78. The nurse identifies the client problem 4. “The half-life of the medications is altered
“excess fluid volume” for the client in liver because the liver is damaged.”
failure. Which short-term goal would be most
84. The client is admitted with end-stage liver
appropriate for this problem?
failure and is prescribed the laxative lactulose
1. The client will not gain more than two (2) kg
(Chronulac). Which statement indicates the
a day.
client needs more teaching concerning this
2. The client will have no increase in abdominal
medication?
girth.
1. “I should have two to three soft stools a day.”
3. The client’s vital signs will remain within
2. “I must check my ammonia level daily.”
normal limits.
3. “If I have diarrhea, I will call my doctor.”
4. The client will receive a low-sodium diet.
4. “I should check my stool for any blood.”

07_Colgrove_Ch07.indd 257 03/06/16 12:58 PM


258 M ed -S urg S uccess

Hepatitis 91. The client with hepatitis asks the nurse, “I went
to an herbalist, who recommended I take milk
85. The client is in the preicteric phase of hepatitis. thistle. What do you think about the herb?”
Which signs/symptoms should the nurse expect Which statement is the nurse’s best response?
the client to exhibit during this phase? 1. “You are concerned about taking an herb.”
1. Clay-colored stools and jaundice. 2. “The herb has been used to treat liver disease.”
2. Normal appetite and pruritus. 3. “I would not take anything that is not
3. Being afebrile and left upper quadrant pain. prescribed.”
4. Complaints of fatigue and diarrhea. 4. “Why would you want to take any herbs?”
86. The public health nurse is teaching day-care 92. The nurse writes the problem “imbalanced
workers. Which type of hepatitis is transmitted nutrition: less than body requirements” for
by the fecal–oral route via contaminated food, the client diagnosed with hepatitis. Which
water, or direct contact with an infected person? intervention should the nurse include in the
1. Hepatitis A. plan of care?
2. Hepatitis B. 1. Provide a high-calorie intake diet.
3. Hepatitis C. 2. Discuss total parenteral nutrition (TPN).
4. Hepatitis D. 3. Instruct the client to decrease salt intake.
4. Encourage the client to increase water intake.
87. Which type of precaution should the nurse
implement to protect from being exposed to any 93. The female nurse sticks herself with a
of the hepatitis viruses? contaminated needle. Which action should the
1. Airborne Precautions. nurse implement first?
2. Standard Precautions. 1. Notify the infection control nurse.
3. Droplet Precautions. 2. Cleanse the area with soap and water.
4. Exposure Precautions. 3. Request postexposure prophylaxis.
4. Check the hepatitis status of the client.
88. The school nurse is discussing methods to
prevent an outbreak of hepatitis A with a group 94. The client diagnosed with liver problems asks
of high school teachers. Which action is the the nurse, “Why are my stools clay-colored?”
most important to teach the high school On which scientific rationale should the nurse
teachers? base the response?
1. Do not allow students to eat or drink after 1. There is an increase in serum ammonia level.
each other. 2. The liver is unable to excrete bilirubin.
2. Drink bottled water as much as possible. 3. The liver is unable to metabolize fatty foods.
3. Encourage protected sexual activity. 4. A damaged liver cannot detoxify vitamins.
4. Sing the happy birthday song while washing 95. Which statement by the client diagnosed with
hands. hepatitis warrants immediate intervention by
89. Which instruction should the nurse discuss the clinic nurse?
with the client who is in the icteric phase 1. “I will not drink any type of beer or mixed
of hepatitis C? drink.”
1. Decrease alcohol intake. 2. “I will get adequate rest so I don’t get
2. Encourage rest periods. exhausted.”
3. Eat a large evening meal. 3. “I had a big hearty breakfast this morning.”
4. Drink diet drinks and juices. 4. “I took some cough syrup for this nasty head
cold.”
90. The public health nurse is discussing hepatitis
B with a group in the community. Which health 96. Which task is most appropriate for the
promotion activities should the nurse discuss nurse to delegate to the unlicensed assistive
with the group? Select all that apply. personnel (UAP)?
1. Do not share needles or equipment. 1. Draw the serum liver function test.
2. Use barrier protection during sex. 2. Evaluate the client’s intake and output.
3. Get the hepatitis B vaccine. 3. Perform the bedside glucometer check.
4. Obtain immune globulin injections. 4. Help the ward clerk transcribe orders.
5. Avoid any type of hepatotoxic medications.

07_Colgrove_Ch07.indd 258 03/06/16 12:58 PM


C hapter 7  G astrointestinal D isorders 259

Gastroenteritis 2. An aminoglycoside antibiotic.


3. An antitoxin medication.
97. The female client came to the clinic 4. An ACE inhibitor medication.
complaining of abdominal cramping and at 103. Which nursing problem is priority for
least 10 episodes of diarrhea every day for the the 76-year-old client diagnosed with
last two (2) days. The client just returned from gastroenteritis from staphylococcal food
a trip to Mexico. Which intervention should poisoning?
the nurse implement? 1. Fluid volume deficit.
1. Instruct the client to take a cathartic laxative 2. Nausea.
daily. 3. Risk for aspiration.
2. Encourage the client to drink lots of Gatorade. 4. Impaired urinary elimination.
3. Discuss the need to increase protein in the diet.
4. Explain the client should weigh herself daily. 104. Which data should the nurse expect to assess in
the client diagnosed with acute gastroenteritis?
98. Which intervention should the nurse include 1. Decreased gurgling sounds on auscultation
when discussing ways to help prevent potential of the abdominal wall.
episodes of gastroenteritis from Clostridium 2. A hard, firm, edematous abdomen on
botulism? palpation.
1. Make sure all hamburger meat is well cooked. 3. Frequent, small melena-type liquid bowel
2. Ensure all dairy products are refrigerated. movements.
3. Discuss why campers should drink only 4. Bowel assessment reveals loud, rushing
bottled water. bowel sounds.
4. Discard damaged canned goods.
105. The 79-year-old client diagnosed with acute
99. The client is diagnosed with salmonellosis gastroenteritis is admitted to the medical unit.
secondary to eating some slightly cooked Which task would be most appropriate for
hamburger meat. Which clinical manifestations the nurse to delegate to the unlicensed assistive
should the nurse expect the client to report? personnel (UAP)?
1. Abdominal cramping, nausea, and vomiting. 1. Evaluate the client’s intake and output.
2. Neuromuscular paralysis and dysphagia. 2. Take the client’s vital signs.
3. Gross amounts of explosive bloody diarrhea. 3. Change the client’s intravenous solution.
4. Frequent “rice water stool” with no fecal odor. 4. Assess the client’s perianal area.
100. The client is diagnosed with gastroenteritis. 106. Which statement indicates to the emergency
Which laboratory data warrant immediate department nurse the client diagnosed
intervention by the nurse? with acute gastroenteritis understands the
1. A serum sodium level of 137 mEq/L. discharge teaching?
2. Arterial blood gases of pH 7.37, Pao2 95, 1. “I will probably have some leg cramps while
Paco2 43, HCO3 24. I have gastroenteritis.”
3. A serum potassium level of 3.3 mEq/L. 2. “I should decrease my fluid intake until the
4. A stool sample positive for fecal leukocytes. diarrhea subsides.”
101. The client diagnosed with gastroenteritis 3. “I should reintroduce solid foods very slowly
is being discharged from the emergency back into my diet.”
department. Which intervention should the 4. “I should only drink bottled water until the
nurse include in the discharge teaching? abdominal cramping stops.”
1. If diarrhea persists for more than 96 hours, 107. Which nursing interventions should be
contact the health-care provider. included in the care plan for the 84-year-old
2. Instruct the client to wash hands thoroughly client diagnosed with acute gastroenteritis?
before handling any type of food. Select all that apply.
3. Explain the importance of decreasing 1. Assess the skin turgor on the back of the
steroids gradually as instructed. client’s hands.
4. Discuss how to collect all stool samples for 2. Monitor the client for orthostatic hypotension.
the next 24 hours. 3. Record the frequency and characteristics of
102. Which medication should the nurse sputum.
expect the HCP to order to treat the client 4. Use Standard Precautions when caring for
diagnosed with botulism secondary to eating the client.
contaminated canned goods? 5. Institute safety precautions when ambulating
1. An antidiarrheal medication. the client.

07_Colgrove_Ch07.indd 259 03/06/16 12:58 PM


260 M ed -S urg S uccess

108. The nurse has received the a.m. shift report. 113. The client is diagnosed with peritonitis. Which
Which client should the nurse assess first? assessment data indicate to the nurse the
1. The 44-year-old client diagnosed with client’s condition is improving?
peptic ulcer disease who is complaining of 1. The client is using more pain medication on
acute epigastric pain. a daily basis.
2. The 74-year-old client diagnosed with acute 2. The client’s nasogastric tube is draining
gastroenteritis who has had four (4) diarrhea coffee-ground material.
stools during the night. 3. The client has a decrease in temperature and
3. The 65-year-old client diagnosed with IBD a soft abdomen.
who has tented skin turgor and dry mucous 4. The client has had two (2) soft-formed
membranes. bowel movements.
4. The 15-year-old client diagnosed with food
114. The client developed a paralytic ileus after
poisoning who has vomited several times
abdominal surgery. Which intervention should
during the night shift.
the nurse include in the plan of care?
1. Administer a laxative of choice.
2. Encourage the client to increase oral fluids.
Abdominal Surgery 3. Encourage the client to take deep breaths.
4. Maintain a patent nasogastric tube.
109. The male client had abdominal surgery and
the nurse suspects the client has peritonitis. 115. The client who had an abdominal surgery
Which assessment data support the diagnosis has a Jackson Pratt (JP) drainage tube.
of peritonitis? Which assessment data warrant immediate
1. Absent bowel sounds and potassium level intervention by the nurse?
of 3.9 mEq/L. 1. The bulb is round and has 40 mL of fluid.
2. Abdominal cramping and hemoglobin of 2. The drainage tube is taped to the dressing.
14 g/dL. 3. The JP insertion site is pink and has no
3. Profuse diarrhea and stool specimen shows drainage.
Campylobacter. 4. The JP bulb has suction and is sunken in.
4. Hard, rigid abdomen and white blood cell
116. The postanesthesia care nurse is caring for
count 22,000/mm3.
a client who had abdominal surgery and is
110. The client who had abdominal surgery tells complaining of nausea. Which intervention
the nurse, “I felt something give way in my should the nurse implement first?
stomach.” Which intervention should the nurse 1. Medicate the client with a narcotic analgesic
implement first? (IVP).
1. Notify the surgeon immediately. 2. Assess the nasogastric tube for patency.
2. Instruct the client to splint the incision. 3. Check the temperature for elevation.
3. Assess the abdominal wound incision. 4. Hyperextend the neck to prevent stridor.
4. Administer pain medication intravenously.
117. The nurse is assessing the client recovering
111. The client is one (1) day postoperative major from abdominal surgery who has a patient-
abdominal surgery. Which client problem is controlled analgesia (PCA) pump. The client
priority? has shallow respirations and refuses to deep
1. Impaired skin integrity. breathe. Which intervention should the nurse
2. Fluid and electrolyte imbalance. implement?
3. Altered bowel elimination. 1. Insist the client take deep breaths.
4. Altered body image. 2. Notify the surgeon to request a chest x-ray.
3. Determine the last time the client used the
112. The client has an eviscerated abdominal
PCA pump.
wound. Which intervention should the nurse
4. Administer oxygen at 2 L/min via nasal
implement?
cannula.
1. Apply sterile normal saline dressing.
2. Use sterile gloves to replace protruding 118. The client has a nasogastric tube. The health-
parts. care provider orders IV fluid replacement
3. Place the client in reverse Trendelenburg based on the previous hour’s output plus the
position. baseline IV fluid ordered of 125 mL/hr. From
4. Administer intravenous antibiotic 0800 to 0900 the client’s N/G tube drained 45
immediately (STAT). mL. At 0900, what rate should the nurse set for
the IV pump? _______

07_Colgrove_Ch07.indd 260 03/06/16 12:58 PM


C hapter 7  G astrointestinal D isorders 261

119. The nurse is caring for clients on a surgical 124. The nurse writes a problem “low self-esteem”
unit. Which client should the nurse assess first? for a 16-year-old client diagnosed with
1. The client who had an inguinal hernia repair anorexia. Which client goal should be included
and has not voided in four (4) hours. in the plan of care?
2. The client who was admitted with 1. The client will spend one (1) hour a day
abdominal pain who suddenly has no pain. with the parents.
3. The client four (4) hours postoperative 2. The client eats 50% of the meals provided.
abdominal surgery with no bowel sounds. 3. Dietary will provide high-protein milk
4. The client who is one (1) day shakes (tid).
postappendectomy and is being discharged. 4. The client will verbalize one positive attribute.
120. The 84-year-old client comes to the clinic 125. The female client diagnosed with anorexia
complaining of right lower abdominal pain. nervosa is admitted to the hospital. The client
Which question is most appropriate for the is 67 inches tall and weighs 40 kg. Which client
nurse to ask the client? problem has the highest priority?
1. “When was your last bowel movement?” 1. Altered nutrition.
2. “Did you have a high-fat meal last night?” 2. Low self-esteem.
3. “Can you describe the type of pain?” 3. Disturbed body image.
4. “Have you been experiencing any gas?” 4. Altered sexuality.
126. Which diagnostic test should the nurse
monitor for the client diagnosed with severe
Eating Disorders anorexia nervosa?
1. Liver function tests.
121. The female client presents to the clinic for
2. Kidney function tests.
an examination because she has not had a
3. Cardiac function tests.
menstrual cycle for several months and wonders
4. Bone density scan.
if she could be pregnant. The client is 5′10″
tall and weighs 45 kg. Which assessment data 127. The female client is more than 10% over ideal
should the nurse obtain first? body weight. Which nursing intervention
1. Ask the client to recall what she ate for the should the nurse implement first?
last 24 hours. 1. Ask the client why she is eating too much.
2. Determine what type of birth control the 2. Refer the client to a gymnasium for exercise.
client has been using. 3. Have the client set a realistic weight loss goal.
3. Reweigh the client to confirm the data. 4. Determine the client’s eating patterns.
4. Take the client’s pulse and blood pressure.
128. The client who is morbidly obese has
122. The occupational health nurse observes the undergone gastric bypass surgery. Which
chief financial officer eat large lunch meals. The immediate postoperative intervention has the
client disappears into the restroom after a meal greatest priority?
for about 20 minutes. Which observation by the 1. Monitor respiratory status.
nurse would indicate the client has bulimia? 2. Weigh the client daily.
1. The client jogs two (2) miles a day. 3. Teach a healthy diet.
2. The client has not gained weight. 4. Assist in behavior modification.
3. The client’s teeth are a green color.
129. The client who is obese presents to the clinic
4. The client has smooth knuckles.
before beginning a weight loss program.
123. The nurse is caring for a client diagnosed with Which interventions should the nurse teach?
bulimia nervosa. Which nursing intervention Select all that apply.
should the nurse implement after the client’s 1. Walk for 30 minutes three (3) times a day.
evening meal? 2. Determine situations that initiate eating
1. Praise the client for eating all the food on behavior.
the tray. 3. Weigh at the same time every day.
2. Stay with the client for 45 minutes to an hour. 4. Limit sodium in the diet.
3. Allow the client to work out on the treadmill. 5. Refer to a weight support group.
4. Place the client on bedrest until morning.

07_Colgrove_Ch07.indd 261 03/06/16 12:58 PM


262 M ed -S urg S uccess

130. The 22-year-old female who is obese is 135. The client has been experiencing difficulty
discussing weight loss programs with the nurse. and straining when expelling feces. Which
Which information should the nurse teach? intervention should the nurse discuss with the
1. Jog for two (2) to three (3) hours every day. client?
2. Lifestyle behaviors must be modified. 1. Explain some blood in the stool will be
3. Eat one (1) large meal every day in the normal for the client.
evening. 2. Instruct the client in manual removal of
4. Eat 1,000 calories a day and don’t take feces.
vitamins. 3. Encourage the client to use a cathartic
laxative on a daily basis.
131. The 36-year-old female client diagnosed with
4. Place the client on a high-fiber diet.
anorexia nervosa tells the nurse “I am so fat.
I won’t be able to eat today.” Which response 136. The client has dark, watery, and shiny-appearing
by the nurse is most appropriate? stool. Which intervention should the nurse
1. “Can you tell me why you think you are fat?” implement first?
2. “You are skinny. Many women wish they 1. Check for a fecal impaction.
had your problem.” 2. Encourage the client to drink fluids.
3. “If you don’t eat, we will have to restrain 3. Check the chart for sodium and potassium
you and feed you.” levels.
4. “Not eating might cause physical problems.” 4. Apply a protective barrier cream to the
perianal area.
132. The client is being admitted to the outpatient
psychiatric clinic diagnosed with bulimia. 137. The charge nurse has just received the shift
Which question should the nurse ask to report. Which client should the nurse see first?
identify behaviors suggesting bulimia? 1. The client diagnosed with Crohn’s disease
1. “When was the last time you exercised?” who had two (2) semiformed stools on the
2. “What over-the-counter medications do you previous shift.
take?” 2. The elderly client admitted from another
3. “How long have you had a positive facility who is complaining of constipation.
self-image?” 3. The client diagnosed with AIDS who had a
4. “Do you eat a lot of high-fiber foods for 200-mL diarrhea stool and has elastic skin
bowel movements?” tissue turgor.
4. The client diagnosed with hemorrhoids who
had some spotting of bright red blood on
Constipation/Diarrhea Disorders the toilet tissue.
138. The dietitian and the nurse in a long-term care
133. The client being admitted from the emergency
facility are planning the menu for the day. Which
department is diagnosed with a fecal impaction.
foods should be recommended for the immobile
Which nursing intervention should be
clients for whom swallowing is not an issue?
implemented?
1. Cheeseburger and milk shake.
1. Administer an antidiarrheal medication
2. Canned peaches and a sandwich on whole-
every day and prn.
wheat bread.
2. Perform bowel training every two (2) hours.
3. Mashed potatoes and mechanically ground
3. Administer an oil retention enema.
red meat.
4. Prepare for an upper gastrointestinal (UGI)
4. Biscuits and gravy with bacon.
series x-ray.
139. The client diagnosed with AIDS is
134. The nurse is caring for a client who uses
experiencing voluminous diarrhea. Which
cathartics frequently. Which statement made
interventions should the nurse implement?
by the client indicates an understanding of the
Select all that apply.
discharge teaching?
1. Monitor diarrhea, charting amount,
1. “In the future I will eat a banana every time
character, and consistency.
I take the medication.”
2. Assess the client’s tissue turgor every day.
2. “I don’t have to have a bowel movement
3. Encourage the client to drink carbonated
every day.”
soft drinks.
3. “I should limit the fluids I drink with my
4. Weigh the client daily in the same clothes
meals.”
and at the same time.
4. “If I feel sluggish, I will eat a lot of cheese
5. Assist the client with a warm sitz bath PRN.
and dairy products.”

07_Colgrove_Ch07.indd 262 03/06/16 12:58 PM


C hapter 7  G astrointestinal D isorders 263

140. The nurse, a licensed practical nurse (LPN), meat at a fast-food restaurant. Which
and an unlicensed assistive personnel (UAP) intervention should be implemented first?
are caring for clients on a medical floor. Which 1. Obtain a stool sample from the client.
nursing task would be most appropriate to 2. Initiate antibiotic therapy intravenously.
assign to the LPN? 3. Have the laboratory draw a complete blood
1. Assist the UAP to learn to perform blood count.
glucose checks. 4. Administer the antidiarrheal medication
2. Monitor the potassium levels of a client with Lomotil.
diarrhea.
143. The clinic nurse is talking on the phone to a
3. Administer a bulk laxative to a client
client who has diarrhea. Which intervention
diagnosed with constipation.
should the nurse discuss with the client?
4. Assess the abdomen of a client who has had
1. Tell the client to measure the amount of stool.
complaints of pain.
2. Recommend the client come to the clinic
141. The client is placed on percutaneous immediately.
endoscopic gastrostomy (PEG) tube feedings. 3. Explain the client should follow the BRAT diet.
Which occurrence warrants immediate 4. Discuss taking an over-the-counter
intervention by the nurse? histamine-2 blocker.
1. The client tolerates the feedings being
144. The nurse is caring for clients on a medical unit.
infused at 50 mL/hr.
Which client information should be brought to
2. The client pulls the nasogastric feeding
the attention of the HCP immediately?
tube out.
1. A serum sodium of 128 mEq/L in a client
3. The client complains of being thirsty.
diagnosed with obstipation.
4. The client has green, watery stool.
2. The client diagnosed with fecal impaction
142. The client presents to the emergency who had two (2) hard formed stools.
department experiencing frequent watery, 3. A serum potassium level of 3.8 mEq/L in
bloody stools after eating some undercooked a client diagnosed with diarrhea.
4. The client with diarrhea who had two
(2) semiliquid stools totaling 300 mL.

CONCEPTS
The concepts covered in this chapter focus on nutrition and acid–base problems. Exemplars that are covered are
eating disorders/dietary recommendations and gastrointestinal disorders. Interrelated concepts of the nursing
process and critical thinking are covered throughout the questions. The concept of critical thinking is presented
in the prioritizing or “first” questions.
145. The nurse is caring for a postoperative client 146. The 70-year-old client is admitted to the
with a nasogastric tube to low intermittent medical unit diagnosed with acute diverticulitis.
suction. Which intervention should the nurse Which interventions should the nurse
implement first based on the blood gas results? implement? Select all that apply.
1. Tell the client not to eat or drink.
Arterial Blood Gas Results 2. Start an intravenous line.
3. Assess the client for abdominal tenderness.
Client Value Normal Values
4. Have the dietitian consult for a low-residue
pH 7.48 7.35–7.45 diet.
O2 Saturation 96 80–100 5. Place the client on bedrest with bathroom
Paco2 46 35–45 privileges.
HCO3 20 22–26 147. The nurse is admitting a client diagnosed
with protein calorie malnutrition. Which
1. Assess the output in the suction canister. interventions should the nurse implement?
2. Apply oxygen by nasal cannula. Select all that apply.
3. Have the client take slow, deep breaths. 1. Place the client on a 72-hour calorie count.
4. Place the client on stool specimen collection. 2. Ask the client to describe the stools.
3. Have the UAP weigh the client.
4. Obtain a list of current medications.
5. Make a referral to the dietitian.

07_Colgrove_Ch07.indd 263 03/06/16 12:58 PM


264 M ed -S urg S uccess

148. The client at the eating disorder clinic weighs 154. The weight loss clinic nurse identifies the
35 kg and is 5 ft 7 inches tall. Which would concept of nutrition for a client diagnosed with
the nurse document as the Body Mass Index obesity. Which interventions should the nurse
(BMI)? ________________ implement? Select all that apply.
1. Ask the client about previous diet attempts.
149. The client presents to the outpatient clinic
2. Refer the client to the dietitian.
complaining of diarrhea for two (2) days. Which
3. Discuss maintaining a sedentary lifestyle.
laboratory data should the nurse monitor?
4. Weigh the client.
1. The sodium level.
5. Assist the client to set a realistic weight
2. The albumin level.
loss goal.
3. The potassium level.
4. The glucose level. 155. The client diagnosed with bulimia has a BMI
of 20. The nurse understands which scientific
150. The clinic nurse is returning client calls.
rationale explains this finding?
Which client should the nurse call first?
1. The 39-year-old client complaining of Body Mass Index
headache pain with a 3 on the pain scale.
2. The 45-year-old client who needs a BMI Lower BMI Upper
prescription refill for warfarin. Category Range Range
3. The 54-year-old client diagnosed with Underweight Greater than 19 —
diabetes type 1 who has been vomiting.
Ideal weight 19 24.9
4. The 60-year-old client who cannot afford to
Overweight 25 30
buy food and needs assistance.
Obese 30.1 —
151. The occupational health nurse has had five
(5) clients come to the clinic complaining of 1. The BMI is low because the client does not
abdominal cramping, nausea, and vomiting. eat and exercises frequently.
Which information should the nurse teach 2. The BMI is within normal range because
the employees to decrease the spread of this the client’s therapy is effective.
condition? 3. The BMI is WNL because the client vomits
1. Teach the employees to cough into the sleeve. or uses laxatives to prevent weight gain.
2. Teach the housekeepers to use an 4. The BMI is high and the client needs to try
antibacterial soap. new methods of weight control.
3. Teach the coworkers to get a hepatitis vaccine.
4. Teach the employees to wash their hands 156. The nurse is teaching the American Diabetes
frequently. Association diet to a client diagnosed with
diabetes mellitus type 2. Which should the
152. Which diagnostic data should be reported to nurse teach the client?
the health-care provider (HCP) immediately? 1. Instruct the client to weigh all food before
1. The ABG result of pH 7.11, Paco2 45, cooking it.
HCO3 20, and Pao2 98 for a client 2. Teach the client to eat only carbohydrates
diagnosed with type 1 diabetes. if the blood glucose is low.
2. Sodium 137 mEq/L, potassium 4 mEq/L, 3. Demonstrate how to determine the amount
glucose 120 mg/dL for a client diagnosed of carbohydrates being eaten.
with gastroenteritis. 4. Explain that proteins should be 75% of the
3. Hemoglobin 9.4 g/dL and hematocrit 29% recommended diet.
for a client who received a blood transfusion
of the previous shift.
4. A pulse oximetry reading of 93% for a
client diagnosed with chronic obstructive
pulmonary disease (COPD).
153. The parents of a female toddler bring the
child to the pediatrician’s office with nausea,
vomiting, and diarrhea. Which intervention
should the nurse implement first?
1. Ask the parent about the child’s diet.
2. Assess the child’s tissue turgor.
3. Give the child a sucker if she is “good.”
4. Notify the HCP the child is waiting to be seen.

07_Colgrove_Ch07.indd 264 03/06/16 12:58 PM


PRACTICE QUESTIONS ANSWERS
AND RATIONALES
Gastroesophageal Reflux (GERD) 3. 1. The client is allowed to eat as soon as the gag
reflex has returned.
1. 1. Clients with heartburn are frequently diag- 2. An esophagogastroduodenoscopy is a
nosed as having GERD. GERD can occasion- ­diagnostic procedure, not a cure. Therefore,
ally cause weight loss but not weight gain. the client still has GERD and should be in-
2. Most clients with GERD have been structed to stay in an upright position for
self-medicating with over-the-counter two (2) to three (3) hours after eating.
medications prior to seeking advice from 3. Stomach contents are acidic and will erode the
a health-care provider. It is important to esophageal lining.
know what the client has been using to 4. Orange juice and tomatoes are acidic, and
treat the problem. the client diagnosed with GERD should
3. Milk and dairy products contain lactose, which avoid acidic foods until the ­esophagus has
are important if considering lactose intoler- had a chance to heal.
ance but are not important for “heartburn.” TEST-TAKING HINT: This question assumes the
4. Heartburn is not a symptom of a viral illness. test taker has knowledge of diagnostic proce-
TEST-TAKING HINT: Clients will use common dures for specific disease processes.
terms such as “heartburn” to describe symp- Content – Medical: Integrated Nursing Process –
toms. The nurse must be able to interpret or Evaluation: Client Needs – Physiological Integrity, Physi-
clarify the meaning of terms used with the ological Adaptation: Cognitive Level – Synthesis: Concept –
client. Part of the assessment of a symptom Digestion.
requires determining what aggravates and 4. 1. The client is instructed to avoid spicy
alleviates the symptom. and acidic foods and any food producing
Content – Medical: Integrated Nursing Process – symptoms.
Assessment: Client Needs – Physiological Integrity, Reduction 2. Eructation means belching, which is a symp-
of Risk Potential: Cognitive Level – Analysis: Concept – tom of GERD.
Digestion. 3. Clients should eat small, frequent meals
2. 1. The client should elevate the head of the and limit fluids with the meals to prevent
bed on blocks or use a foam wedge to use reflux into the esophagus from a distended
gravity to help keep the gastric acid in the stomach.
stomach and prevent reflux into the esoph- 4. Clients are encouraged to forgo all alcoholic
agus. Behavior modification is changing beverages because alcohol relaxes the lower
one’s behavior. esophageal sphincter and increases the risk
2. The client should be encouraged to quit smok- of reflux.
ing altogether. Referral to support groups for TEST-TAKING HINT: The word “any” in option
smoking cessation should be made. “1” should give the test taker a clue that, un-
3. The nurse should be careful when recom- less there are absolutely no dietary restric-
mending OTC medications. This is not the tions, this is an incorrect answer. Option “2”
most appropriate intervention for a client with requires knowledge of medical terminology.
GERD. Content – Medical: Integrated Nursing Process –
4. The client should be instructed to discontinue Diagnosis: Client Needs – Physiological Integrity, Basic
using alcohol, but the stem does not indicate Care and Comfort: Cognitive Level – Analysis: ­Concept –
the client is an alcoholic. Digestion.
TEST-TAKING HINT: Clients are encouraged to 5. 1. The client is encouraged to lie with the head
quit, not decrease, smoking. Current research of the bed elevated, but this is difficult to
indicates smoking is damaging to many body achieve when on the stomach. NSAIDs inhibit
systems, including the gastrointestinal system. prostaglandin synthesis in the stomach, which
The test taker should not assume anything places the client at risk for developing gastric
not in the stem of a question. ulcers. The client is already experiencing gas-
Content – Medical: Integrated Nursing Process – Diagnosis: tric acid difficulty.
Client Needs – Physiological Integrity, Physiological Adaptation: 2. The client will need to lie down at some time,
Cognitive Level – Analysis: Concept – Digestion. and walking will not help with GERD.
265

07_Colgrove_Ch07.indd 265 03/06/16 12:58 PM


266 M ed -S urg S uccess

3. If lying on the side, the left side-lying position, The test taker should memorize these specific
not the right side, will allow less chance of re- medications.
flux into the esophagus. Antacids are taken one Content – Medical: Integrated Nursing Process –
(1) and three (3) hours after a meal. Implementation: Client Needs – Physiological Integrity,
4. The head of the bed should be elevated to Pharmacological and Parenteral Therapies: Cognitive
allow gravity to help in preventing reflux. Level – Application: Concept – Medication.
Lifestyle modifications of losing weight, 8. 1. In a Bernstein test, acid is instilled into the
making dietary modifications, ­attempting ­distal esophagus, causing immediate heart-
smoking cessation, discontinuing the use of burn for a client diagnosed with GERD.
alcohol, and not stooping or bending at the This would not warrant notifying the HCP.
waist all help to decrease reflux. 2. Hiatal hernias are frequently the cause of
TEST-TAKING HINT: Option “2” has an “all,” GERD; therefore, this finding would not
which should alert the test taker to eliminate warrant notifying the HCP.
this option. If the test taker has no idea of the 3. The client’s WBC count is elevated,
answer, lifestyle modifications are an educated ­indicating a possible infection, which
guess for most chronic problems. warrants notifying the HCP.
Content – Medical: Integrated Nursing Process – ­ 4. This is a normal hemoglobin result and
Implementation: Client Needs – Safe Effective Care would not warrant notifying the HCP.
Environment, Management of Care: Cognitive Level –
TEST-TAKING HINT: When the test taker is
Application: Concept – Digestion.
deciding when to notify a health-care pro-
6. 1. Of adult-onset asthma cases, 80% to 90% vider, the answer should be data not normal
are caused by gastroesophageal reflux for the disease process or signaling a potential
­disease (GERD). or life-threatening complication.
2. Pancreatitis is not related to GERD. Content – Surgical: Integrated Nursing Process –
3. Peptic ulcer disease is related to H. pylori bac- Implementation: Client Needs – Safe Effective Care Environ-
terial infections and can lead to increased lev- ment, Management of Care: Cognitive Level – Application:
els of gastric acid, but it is not related to reflux. Concept – Perioperative.
4. GERD is not related to increased gastric emp- 9. 1. Pyrosis is heartburn and is expected in a
tying. Increased gastric emptying would be a client diagnosed with GERD. The new grad-
benefit to a client with decreased functioning uate can care for this client.
of the lower esophageal sphincter. 2. Barrett’s esophagus is a complication of
TEST-TAKING HINT: Some questions are knowl- GERD; new graduates can prepare a client
edge based. There are no test-taking strate- for a diagnostic procedure.
gies for knowledge-based questions. 3. This client is exhibiting symptoms of
Content – Medical: Integrated Nursing Process – asthma, a complication of GERD. This
Diagnosis: Client Needs – Safe Effective Care Environment, client should be assigned to the most
Management of Care: Cognitive Level – Knowledge: experienced nurse.
Concept – Digestion. 4. This client can be cared for by the new grad-
7. 1. Proton pump inhibitors can be administered uate, and ambulating can be delegated to the
at routine dosing times, usually 0900 or after unlicensed assistive personnel (UAP).
breakfast. TEST-TAKING HINT: The most experienced
2. Pain medication is important, but a nonnar- nurse should be assigned to the client whose
cotic medication, such as Tylenol, can be ad- assessment and care require more experience
ministered after a medication, which must be and knowledge about the disease process,
timed. potential complications, and medications.
3. A histamine receptor antagonist can be The term “most experienced” in the stem is
­administered at routine dosing times. the key to answering this question.
4. A mucosal barrier agent must be adminis- Content – Medical: Integrated Nursing Process –
tered on an empty stomach for the medica- Planning: Client Needs – Safe Effective Care Environ-
tion to coat the stomach. ment, Management of Care: Cognitive Level – Synthesis:
TEST-TAKING HINT: Basic knowledge of how Concept – Nursing Roles.
medications work is required to administer 10. 1. Pain in the chest when walking up stairs
medications for peak effectiveness. There are ­indicates angina.
very few medications requiring a specific time. 2. Frequent use of antacids indicates an acid
reflux problem.

07_Colgrove_Ch07.indd 266 03/06/16 12:58 PM


C hapter 7  G astrointestinal D isorders 267

3. Snoring loudly could indicate sleep apnea but Content – Medical: Integrated Nursing Process – Diagnosis:
not GERD. Client Needs – Health Promotion and Maintenance:
4. Carbonated beverages increase stomach pres- Cognitive Level – Knowledge: Concept – Digestion.
sure. Six (6) to seven (7) soft drinks a day
would not be tolerated by a client with GERD.
Inflammatory Bowel Disease
TEST-TAKING HINT: The stem of the question
indicates an acid problem. The drug classifi- 13. 1. The colon is ulcerated and unable to
cation of antacid, or “against acid,” provides ­absorb water, resulting in bloody diarrhea.
the test taker a hint as to the correct answer. Ten (10) to 20 bloody diarrhea stools is
Content – Medical: Integrated Nursing Process – the most common symptom of ulcerative
Assessment: Client Needs – Physiological Integrity, colitis.
Reduction of Risk Potential: Cognitive Level – Analysis: 2. Inflammation usually causes an elevated tem-
Concept – Digestion. perature but is not expected in the client with
11. 1. Pyrosis is heartburn, water brash is the ulcerative colitis.
feeling of saliva secretion as a result of 3. A hard, rigid abdomen indicates peritonitis,
reflux, and flatulence is gas—all symptoms which is a complication of ulcerative colitis
of GERD. but not an expected symptom.
2. Gastroesophageal reflux disease does not 4. Stress incontinence is not a symptom of colitis.
cause weight loss. TEST-TAKING HINT: If the test taker is not
3. There is no change in abdominal fat, no pro- sure of the answer, the test taker should use
teinuria (the result of a filtration problem in knowledge of anatomy and physiology to
the kidney), and no alteration in bowel elimi- help identify the correct answer. The colon
nation for the client diagnosed with GERD. is responsible for absorbing water, and if the
4. Midepigastric pain, a positive H. pylori test, and colon can’t do its job, then water will not be
melena are associated with gastric ulcer disease. absorbed, causing diarrhea (option “1”).
TEST-TAKING HINT: Frequently, incorrect Colitis is inflammation of the colon; there-
answer options will contain the symptoms fore, option “4” referring to the urinary
of a disease of the same organ system. ­system can be eliminated.
Content – Medical: Integrated Nursing Process – Content – Medical: Integrated Nursing Process –
Assessment: Client Needs – Physiological Integrity, Assessment: Client Needs – Physiological Integrity,
Reduction of Risk Potential: Cognitive Level – Analysis: Reduction of Risk Potential: Cognitive Level – Analysis:
Concept – Digestion. Concept – Digestion.

12. 1. A hiatal hernia places the client at risk for 14. 1. Steroids can cause erosion of the stomach and
GERD; GERD does not predispose the should be taken with food.
­client for developing a hiatal hernia. 2. A moon face is an expected side effect of
2. Gastroenteritis is an inflammation of the steroids.
stomach and intestine, usually caused by 3. This medication must be tapered off to
a virus. prevent adrenal insufficiency; therefore,
3. Barrett’s esophagus results from long- the client must take this medication as
term erosion of the esophagus as a result prescribed.
of reflux of stomach contents secondary to 4. Steroids may increase the client’s blood glu-
GERD. This is a precursor to esophageal cose, but diabetic medication regimens are
cancer. usually not altered for the short period of
4. The problems associated with GERD result time the client with an acute exacerbation is
from the reflux of acidic stomach contents prescribed steroids.
into the esophagus, which is not a precursor TEST-TAKING HINT: The test taker should
to gastric cancer. know few medications must be taken on an
TEST-TAKING HINT: The test taker may associ- empty stomach, which would cause option
ate hiatal hernia with GERD. One can be a “1” to be eliminated. All medications should
result of the other, and this can confuse the be taken as prescribed—don’t think the an-
test taker. If the test taker did not have any swer is too easy.
idea of the correct answer, option “3” has the Content – Medical: Integrated Nursing Process –
word “esophageal” in it, as does the stem of Planning: ­Client Needs – Physiological Integrity,
the question, and, therefore, the test taker Pharmacological and ­Parenteral Therapies: Cognitive
should select this as the correct answer. Level – Synthesis: Concept – Medication.

07_Colgrove_Ch07.indd 267 03/06/16 12:58 PM


268 M ed -S urg S uccess

15. 1. The HCP should be notified so potassium 4. The client is NPO to put the bowel at rest,
supplements can be ordered, but this is not which is the rationale for administering the
the first intervention. TPN.
2. Muscle weakness may be a sign of hypo- TEST-TAKING HINT: The test taker may want
kalemia; hypokalemia can lead to cardiac to select option “3” because it has the word
dysrhythmias and can be life threatening. “assess,” but the test taker should remem-
Assessment is priority for a potassium ber to note the adjective “peripheral,” which
level just below normal level, which is makes this option incorrect. Remember,
3.5 to 5.5 mEq/L. the words “check” and “monitor” are words
3. Hypokalemia can lead to cardiac dysrhyth- meaning “assess.”
mias; therefore, requesting telemetry is ap- Content – Medical: Integrated Nursing Process –
propriate, but it is not the first intervention. Implementation: Client Needs – Physiological Integrity,
4. The client will need potassium to cor- Pharmacological and Parenteral Therapies: Cognitive
rect the hypokalemia, but it is not the first Level – Application: Concept – Nutrition.
intervention.
18. 1. Weighing the client daily will help identify
TEST-TAKING HINT: When the question asks
if the client is experiencing malnutrition, but
which action should be implemented first, it is not the priority intervention during an
remember assessment is the first step in the acute exacerbation.
nursing process. If the answer option ad- 2. Coping strategies help develop healthy ways
dressing assessment is appropriate for the to deal with this chronic disease, which has
situation in the question, then the test taker remissions and exacerbations, but it is not the
should select it as the correct answer. priority intervention.
Content – Medical: Integrated Nursing Process – 3. The severity of the diarrhea helps deter-
Implementation: Client Needs – Safe Effective Care Envi- mine the need for fluid replacement. The
ronment, Management of Care: Cognitive Level –
liquid stool should be measured as part of
Synthesis: Concept – Bowel Elimination.
the total output.
16. 1. The client’s bowel should be placed on rest 4. The client will be NPO when there is an
and no foods or fluids should be introduced acute exacerbation of IBD to allow the bowel
into the bowel. to rest.
2. Whenever a client has an acute exacerba- TEST-TAKING HINT: The test taker can apply
tion of a gastrointestinal disorder, the first Maslow’s hierarchy of needs and select the
intervention is to place the bowel on rest. option addressing a physiological need.
The client should be NPO with intrave- Content – Medical: Integrated Nursing Process –
nous fluids to prevent dehydration. Implementation: Client Needs – Safe Effective Care Environ-
3. The vital signs must be taken more often ment, Management of Care: Cognitive Level – Synthesis:
than daily in a client who is having an acute Concept – Bowel Elimination.
exacerbation of ulcerative colitis.
4. The client will receive anti-inflammatory and 19. 1. The nurse should never tell a client he or she
antidiarrheal medications, not antacids, which understands what the client is going through.
are used for gastroenteritis. 2. Telling the client to think about the good
things is not addressing the client’s feelings.
TEST-TAKING HINT: “Acute exacerbation” is
3. The client is crying and is expressing feel-
the key phrase in the stem of the question. ings of powerlessness; therefore, the nurse
The word “acute” should cause the test taker should allow the client to talk.
to eliminate any daily intervention. 4. The client is crying and states “I can’t take it
Content – Medical: Integrated Nursing Process – anymore,” but this is not a suicidal comment
Implementation: Client Needs – Safe Effective Care Envi- or situation.
ronment, Management of Care: Cognitive Level –
Application: Concept – Digestion. TEST-TAKING HINT: There are rules applied
to therapeutic responses. Do not say “under-
17. 1. TPN is high in dextrose, which is glucose; stand” and do not ask “why.” The test taker
therefore, the client’s blood glucose level should select an option where some type of
must be monitored closely. feeling is being reflected in the statement.
2. The client may be on sliding-scale regular Content – Medical: Integrated Nursing Process –
insulin coverage for the high glucose level. Implementation: Client Needs – Psychosocial Integrity:
3. The TPN must be administered via a subcla- Cognitive Level – Application: Concept – Bowel
vian line because of the high glucose level. Elimination.

07_Colgrove_Ch07.indd 268 03/06/16 12:58 PM


C hapter 7  G astrointestinal D isorders 269

20. 1. A pink and moist stoma indicates viable tis- 4. A fever and hard, rigid abdomen are signs/
sue and adequate circulation. A purple stoma symptoms of peritonitis, a complication of
indicates necrosis. Crohn’s disease.
2. An ileostomy will drain liquid all the time TEST-TAKING HINT: The test taker should
and should not routinely be irrigated. eliminate option “2” because of the word
A sigmoid colostomy may need daily “all,” which is an absolute. There are very
­irrigation to evacuate feces. few absolutes in the health-care arena.
3. A red, bumpy, itchy rash indicates infection
Content – Medical: Integrated Nursing Process –
with the yeast Candida albicans, which should Assessment: Client Needs – Physiological Integrity, Reduc-
be treated with medication. tion of Risk Potential: Cognitive Level – Analysis:
4. The ileostomy drainage has enzymes and bile Concept – Bowel Elimination.
salts, which are irritating and harsh to the
skin; therefore, the pouch should be changed 23. 1. Fried potatoes, along with pastries and pies,
if any leakage occurs. should be avoided.
2. Raw vegetables should be avoided because
TEST-TAKING HINT: This is an “except” ques-
this is roughage.
tion, and the test taker must identify which
3. A low-residue diet is a low-fiber diet.
option is not a correct action for the nurse
Products made of refined flour or finely
to implement. Sometimes flipping the
milled grains, along with roasted, baked,
­question—“Which interventions indicate
or broiled meats, are recommended.
the client understands the teaching?”—can
4. Fried foods should be avoided, and whole
assist in identifying the correct answer.
grain is high in fiber. Nuts and fruits with
Content – Medical: Integrated Nursing Process – peels should be avoided.
Evaluation: Client Needs – Physiological Integrity,
Physiological Adaptation: Cognitive Level – Synthesis: TEST-TAKING HINT: The test taker must know
Concept – Bowel Elimination. about therapeutic diets prescribed by health-
care providers. Remember, low residue is the
21. 1. Asulfidine cannot be administered rectally. same as low fiber.
Corticosteroids may be administered by
Content – Medical: Integrated Nursing Process –
enema for the local effect of decreasing in- Evaluation: Client Needs – Physiological Integrity, Basic
flammation while minimizing the systemic Care and Comfort: Cognitive Level – Analysis:
effects. Concept – Nutrition.
2. Antidiarrheal agents slow the gastrointestinal
motility and reduce diarrhea. 24. 1. The cure for ulcerative colitis is a total
3. IBD is not caused by bacteria. colectomy, which is removing the entire
4. Asulfidine is poorly absorbed from the large colon and bringing the terminal end
gastrointestinal tract and acts topically of the ileum up to the abdomen in the
on the colonic mucosa to inhibit the right lower quadrant. This is an ileostomy.
­inflammatory process. 2. This site is the left lower quadrant.
3. This site is the transverse colon.
TEST-TAKING HINT: If the test taker doesn’t
4. This site is the right upper quadrant.
know the answer, then the test taker could
eliminate options “2” and “3” because they TEST-TAKING HINT: The test taker must
do not contain the word “inflammation”; identify the area by using the computer
IBD is inflammatory bowel disease. mouse. These are called “hot spots” on the
NCLEX-RN.
Content – Medical: Integrated Nursing Process –
Implementation: Client Needs – Physiological Integrity, Content – Medical: Integrated Nursing Process –
Pharmacological and Parenteral Therapies: Cognitive Diagnosis: Client Needs – Safe Effective Care Environ-
Level – Comprehension: Concept – Medication. ment, Management of Care: Cognitive Level –
Comprehension: Concept – Bowel Elimination.
22. 1. The terminal ileum is the most common
site for regional enteritis, which causes
right lower quadrant pain that is relieved Peptic Ulcer Disease
by defecation.
2. Stools are liquid or semiformed and usually 25. 1. The presence of blood does not specifically
do not contain blood. indicate diagnosis of an ulcer. The client
3. Episodes of diarrhea and constipation may be could have hemorrhoids or cancer resulting
a sign/symptom of colon cancer, not Crohn’s in the presence of blood.
disease. 2. A wavelike burning sensation is a symptom of
gastroesophageal reflux.

07_Colgrove_Ch07.indd 269 03/06/16 12:58 PM


270 M ed -S urg S uccess

3. Sharp pain in the upper abdomen after eat- protective mechanism to prevent damage
ing a heavy meal is a symptom of gallbladder from hydrochloric acid.
disease. 3. Allergies are included for safety, but this is
4. In a client diagnosed with a gastric ulcer, not specific for peptic ulcer disease.
pain usually occurs 30 to 60 minutes after 4. Information needs to be obtained about past
eating but not at night. In contrast, a cli- generations so the nurse can analyze any
ent with a duodenal ulcer has pain during potential health problems, but this is not spe-
the night often relieved by eating food. cific for peptic ulcer disease.
Pain occurs one (1) to three (3) hours TEST-TAKING HINT: The words “specific data”
­after meals. ­indicate there will be appropriate data in one
TEST-TAKING HINT: This question asks the (1) or more of the answer options but only
test taker to identify assessment data spe- one (1) is specific to peptic ulcer disease.
cific to the disease process. Many diseases Content – Medical: Integrated Nursing Process –
have similar symptoms, but the timing of Assessment: Client Needs – Physiological Integrity,
symptoms or their location may help rule out Reduction of Risk Potential: Cognitive Level – Analysis:
some diseases and provide the health-care Concept – Infection.
provider with a key to diagnose a specific 28. 1. Auscultation should be used prior to
disease—in this case, peptic ulcer disease. ­palpation or percussion when assess-
Nurses are usually the major source for ing the abdomen. Manipulation of the
­information to the health-care team. ­abdomen can alter bowel sounds and
Content – Medical: Integrated Nursing Process – give false information.
Assessment: Client Needs – Physiological Integrity, 2. Palpation gives appropriate information
Reduction of Risk Potential: Cognitive Level – Analysis:
the nurse needs to collect, but if done
Concept – Digestion.
prior to auscultation, the sounds will
26. 1. The esophagogastroduodenoscopy (EGD) be altered.
is an invasive diagnostic test that visualizes 3. Percussion of the abdomen does not give
the esophagus, stomach, and duodenum to ­specific information about peptic ulcer
accurately diagnose an ­ulcer and evaluate disease.
the effectiveness of the client’s treatment. 4. Tender areas should be assessed last to pre-
2. Magnetic resonance imaging (MRI) shows vent guarding and altering the assessment.
cross-sectional images of tissue or blood flow. This includes palpation, which should be
3. An occult blood test shows the presence of done after auscultation.
blood but not the source. TEST-TAKING HINT: The word “first” requires
4. A gastric acid stimulation test is used to un- the test taker to rank in order the interven-
derstand the pathophysiology of ulcer d ­ isease, tions needing to be performed. The test
but it has limited usefulness. taker should visualize caring for the client.
TEST-TAKING HINT: If the test taker has no This will assist the test taker in making the
idea what the correct answer is, knowledge correct choice.
of anatomy can help identify the answer. Content – Medical: Integrated Nursing Process –
A peptic ulcer is an ulcer in the stomach, and Implementation: Client Needs – Physiological Integrity,
in option “1” the word “esophagogastroduo- Reduction of Risk Potential: Cognitive Level – Synthesis:
denoscopy” has “gastro,” which refers to the Concept – Infection.
stomach. Therefore, this would be the best 29. 1. There is no indication from the question
option to select as the correct answer. there is a problem or potential problem with
Content – Medical: Integrated Nursing Process – bowel elimination.
Diagnosis: Client Needs – Physiological Integrity, Reduction 2. Knowledge deficit does not address physi-
of Risk Potential: Cognitive Level – Knowledge: Concept –
ological complications.
Infection.
3. This client may have problems from chang-
27. 1. A history of problems the client has experi- ing roles within the family, but the question
enced with medications is taken during the asks for potential physiological complications,
admission interview. This information does not psychosocial problems.
not specifically address peptic ulcer disease. 4. Potential for alteration in gastric ­emptying
2. Use of NSAIDs places the client at risk is caused by edema or scarring associated
for peptic ulcer disease and hemorrhage. with an ulcer, which may cause a feeling of
NSAIDs suppress the production of “fullness,” vomiting of undigested food, or
prostaglandin in the stomach, which is a abdominal distention.

07_Colgrove_Ch07.indd 270 03/06/16 12:58 PM


C hapter 7  G astrointestinal D isorders 271

TEST-TAKING HINT: This question asks the 32. 1. The range for normoactive bowel sounds is
test taker to identify a physiological problem from five (5) to 35 times per minute. This
identifying a complication of the disease pro- would require no intervention.
cess. Therefore, options “2” and “3” could 2. Belching after a heavy, fatty meal is a symptom
be eliminated because they do not address of gallbladder disease. Eating late at night may
physiological problems. cause symptoms of esophageal disorders.
Content – Medical: Integrated Nursing Process – 3. A decrease of 20 mm Hg in blood pressure
Diagnosis: Client Needs – Physiological Integrity, ­Reduction after changing position from lying, to sit-
of Risk Potential: Cognitive Level – Analysis: Concept – ting, to standing is orthostatic hypotension.
Infection. This could indicate the client is bleeding.
30. 1. A pain assessment is an independent interven- 4. A decrease in the quality and quantity of dis-
tion the nurse should implement frequently. comfort shows an improvement in the client’s
2. Evaluating vital signs is an independent in- condition. This would not require further
tervention the nurse should implement. If intervention.
the client is able, BPs should be taken lying, TEST-TAKING HINT: When the question asks
sitting, and standing to assess for orthostatic about further intervention, the test taker
hypotension. should examine the answer options for an
3. This is a collaborative intervention the unexpected outcome requiring further
nurse should implement. It requires an assessment.
order from the HCP. Content – Medical: Integrated Nursing Process –
4. Administering blood products is collab- Evaluation: Client Needs – Physiological Integrity, Reduc-
orative, requiring an order from the HCP. tion of Risk Potential: Cognitive Level – Analysis:
5. The diet requires an order by the health-care Concept – Infection.
provider, but a diet will not be ordered be- 33. 1. E-mycin is irritating to the stomach,
cause the client is NPO. and its use in a client with peptic ulcer
TEST-TAKING HINT: Descriptive words such as disease should be questioned.
“collaborative” or “independent” can be the 2. Prilosec, a proton pump inhibitor, decreases
­deciding factor when determining if an an- gastric acid production, and its use should not
swer option is correct or incorrect. These are be questioned by the nurse.
key words the test taker should identify. 3. Flagyl, an antimicrobial, is administered
Content – Medical: Integrated Nursing Process – to treat peptic ulcer disease secondary to
Implementation: Client Needs – Safe Effective Care Envi- H. pylori bacteria.
ronment, Management of Care: Cognitive Level – 4. Tylenol can be safely administered to a ­client
Application: Concept – Infection. with peptic ulcer disease.
31. 1. Use of NSAIDs increases and causes prob- TEST-TAKING HINT: The test taker needs to
lems associated with peptic ulcer disease. understand how medications work, adverse
2. Maintaining lifestyle changes such as fol- effects of medications, when to question ad-
lowing an appropriate diet and reducing ministering a specific medication, and how to
stress indicate the client is complying administer the medication safely. By learning
with the medical regimen. Compli- classifications, the test taker should be able to
ance is the goal of treatment to prevent make a knowledgeable selection in most cases.
complications. Content – Medical: Integrated Nursing Process –
3. Hemoptysis is coughing up blood, which is Implementation: Client Needs – Physiological Integrity,
not a sign or symptom of peptic ulcer disease. Pharmacological and ­Parenteral Therapies: Cognitive
This would not be an expected outcome. Level – Application: Concept – Medication.
4. Antacids should be taken one (1) to three 34. 1. Decreasing the alcohol intake indicates the
(3) hours after meals, not with each meal. client is making some lifestyle changes.
TEST-TAKING HINT: Expected outcomes are 2. The client with peptic ulcer disease (PUD)
positive completion of goals; maintaining is prescribed a regular diet, but the type of
lifestyle modifications would be an appro- diet does not determine if the medication is
priate goal for any client with any chronic effective.
illness. 3. The return to previous activities indicates the
Content – Medical: Integrated Nursing Process – client has not adapted to the lifestyle changes
Planning: Client Needs – Safe Effective Care Environ- and has returned to the previous behaviors
ment, Management of Care: Cognitive Level – Synthesis: which precipitated the peptic ­ulcer disease.
Concept – Infection.

07_Colgrove_Ch07.indd 271 03/06/16 12:58 PM


272 M ed -S urg S uccess

4. Antibiotics, proton pump inhibitors, and nasogastric tube and beginning lavage is
Pepto-Bismol are administered to decrease solving a circulation or fluid deficit problem.
the irritation of the ulcerative area and Content – Medical: Integrated Nursing Process –
cure the ulcer. A decrease in gastric dis- Implementation: Client Needs – Safe Effective Care Envi-
tress indicates the medication is effective. ronment, Management of Care: Cognitive Level –
TEST-TAKING HINT: To determine the effec- Synthesis: Concept – Infection.
tiveness of a medication, the test taker must
know the scientific rationale for adminis- Colorectal Disease
tering the medication. Peptic ulcer disease
causes gastric distress. If gastric distress is 37. 1. Some cancers have a higher risk of develop-
relieved, then the medication is effective. ment when the client is occupationally ex-
Content – Medical: Integrated Nursing Process – posed to chemicals, but cancer of the colon is
Assessment: ­Client Needs – Physiological Integrity, Pharma- not one of them.
cological and ­Parenteral Therapies: Cognitive Level –
2. Cruciferous vegetables, such as broccoli,
Analysis: Concept – Medication.
cauliflower, and cabbage, are high in fiber.
35. 1. Sudden sharp pain felt in the substernal area One of the risks for cancer of the colon
indicates angina or myocardial infarction. is a high-fat, low-fiber, and high-protein
2. A rigid, boardlike abdomen with rebound diet. The longer the transit time (the time
tenderness is the classic sign/symptom of from ingestion of the food to the elimina-
peritonitis, which is a complication of a tion of the waste products), the greater the
perforated gastric ulcer. chance of developing ­cancer of the colon.
3. Clay-colored stools indicate liver disorders, 3. A multiple vitamin may improve immune
such as hepatitis. system function, but it does not prevent c­ olon
4. Clients with gallbladder disease report vague cancer.
to sharp abdominal pain in the right upper 4. High-risk sexual behavior places the client at
quadrant. risk for sexually transmitted diseases. A his-
TEST-TAKING HINT: The only two (2) answer tory of multiple sexual partners and initial
options that refer to the abdomen are op- sexual experience at an early age does increase
tions “2” and “4.” Therefore, the test taker the risk for the development of ­cancer of the
should select one (1) of these two (2) because cervix in females.
a gastric ulcer involves the stomach. TEST-TAKING HINT: The colon processes
Content – Medical: Integrated Nursing Process – waste products from eating foods, and op-
Assessment: Client Needs – Physiological Integrity, Reduc- tion “2” is the only option to mention food.
tion of Risk Potential: Cognitive Level – Analysis: Therefore, option “2” would be the best op-
Concept – Infection. tion to select if the test taker did not know
the correct answer.
36. 1. Maintaining a strict record of intake and out-
put is important to evaluate the progression Content – Medical: Integrated Nursing Process –
of the client’s condition, but it is not the most Planning: Client Needs – Health Promotion and Mainte-
nance: Cognitive Level – Synthesis: Concept – Promoting
important intervention.
Health.
2. Inserting a nasogastric tube and lavaging
the stomach with saline is the most im- 38. 1. Frequent bloody stools are a symptom of in-
portant intervention because this directly flammatory bowel disease (IBD). IBD is a risk
stops the bleeding. factor for cancer of the colon, but the symp-
3. A calorie count is important information toms are different when the colon becomes
assisting in the prevention and treatment of a cancerous.
nutritional deficit, but this intervention does 2. Most people have a feeling of fullness after
not address the client’s immediate and life- a heavy meal; this does not indicate cancer.
threatening problem. 3. The most common symptom of colon can-
4. Promoting a quiet environment aids in the cer is a change in bowel habits, specifically
reduction of stress, which can cause further diarrhea alternating with constipation.
bleeding, but this will not stop the bleeding. 4. Lower right quadrant pain with rebound
TEST-TAKING HINT: The test taker is required ­tenderness would indicate appendicitis.
to rank the importance of interventions TEST-TAKING HINT: The test taker could
in the question. Using Maslow’s hierarchy eliminate option “4” based on anatomical
of needs to rank physiological needs first, ­position. The rectosigmoid colon is in the
the test taker should realize inserting a left lower quadrant.

07_Colgrove_Ch07.indd 272 03/06/16 12:58 PM


C hapter 7  G astrointestinal D isorders 273

Content – Medical: Integrated Nursing Process – 2. The stoma should be pouched securely for
Assessment: Client Needs – Physiological Integrity, Reduc- the client to be able to participate in normal
tion of Risk Potential: Cognitive Level – Analysis: daily activities. The client should be encour-
Concept – Cellular Regulation. aged to ambulate to aid in recovery.
39. 1. A long history of low-fiber, high-fat, and 3. Pain medication should be taken before the
high-protein diets results in a prolonged pain level reaches a “5.” Delaying taking med-
transit time. This allows the carcinogenic ication will delay the onset of pain relief and
agents in the waste products to have a the client will not receive full benefit from
greater exposure to the lumen of the colon. the medication.
2. The older the client, the greater the risk of 4. The pouch should be emptied when it is
developing cancer of the colon. one-third to one-half full to prevent the
3. Risk factors for cancer of the colon include contents from becoming too heavy for the
increasing age; family history of colon cancer seal to hold and to prevent leakage from
or polyps; history of IBD; genital or breast occurring.
cancer; and eating a high-fat, high-protein, TEST-TAKING HINT: Normal mucosa is pink,
low-fiber diet. not white, and clients are always encouraged
4. Males have a slightly higher incidence of to ambulate after surgery to prevent the
­colon cancers than do females. complications related to immobility. Remem-
TEST-TAKING HINT: The test taker should ber basic concepts when answering questions,
realize cancers in general have an increasing especially about postoperative nursing care.
incidence with age. Cancer etiologies are not Content – Surgical: Integrated Nursing Process –
an exact science, but most cancers have some Planning: Client Needs – Physiological Integrity, Physiologi-
risk factor, if only advancing age. cal Adaptation: Cognitive Level – Synthesis: Concept –
Bowel Elimination.
Content – Medical: Integrated Nursing Process – ­
Implementation: Client Needs – Safe Effective Care Envi- 42. 1. The nurse should mark the drainage on the
ronment, Management of Care: Cognitive Level – dressing to determine if active bleeding
Application: Concept – Cellular Regulation. is occurring, because dark ­reddish-brown
40. 1. Colostomy stomas are openings through drainage indicates old blood. This al-
the abdominal wall into the colon, through lows the nurse to assess what is actually
which feces exit the body. ­Feces can be happening.
irritating to the abdominal skin, so careful 2. Surgical dressings are initially changed by
and thorough skin care is needed. the surgeon; the nurse should not remove the
2. There are midline and perineal incisions, not dressing until the surgeon orders the dressing
flank incisions. change to be done by the nurse.
3. Because of the perineal wound, the client 3. The nurse should assess the situation before
will have an indwelling catheter to keep notifying the HCP.
urine out of the incision. 4. The nurse may need to reinforce the dress-
4. Jackson Pratt drains are emptied every shift, ing if the dressing becomes saturated, but
but they are not irrigated. this would be after a thorough assessment is
5. The client should not sit upright because completed.
this causes pressure on the perineum. TEST-TAKING HINT: The question is asking the
TEST-TAKING HINT: The test taker could test taker to determine which intervention
eliminate option “2” because flank and must be implemented first, and assessment is
abdominal–perineal are not in the same the first step of the nursing process. Options
areas. This is an alternative-type question “2,” “3,” and “4” would not be implemented
requiring the test taker to choose more prior to assessing. Marking the dressing
than one (1) option. ­allows the nurse to assess the dressing and
Content – Surgical: Integrated Nursing Process – ­ determine if active bleeding is occurring.
Implementation: Client Needs – Safe Effective Care Envi- Content – Surgical: Integrated Nursing Process –
ronment, Management of Care: Cognitive Level – ­Implementation: Client Needs – Physiological Integrity,
Application: Concept – Bowel Elimination. ­Physiological Adaptation: Cognitive Level – Application:
Concept – Perioperative.
41. 1. The stoma should be light to a medium pink,
the color of the intestines. A blue or purple 43. 1. The nurse should first assess the situation
color indicates a lack of circulation to the prior to informing the HCP of the client’s
stoma and is a medical emergency. concerns and then allow the HCP and client
to discuss the situation.

07_Colgrove_Ch07.indd 273 03/06/16 12:58 PM


274 M ed -S urg S uccess

2. The nurse should determine what is con- Content – Surgical: Integrated Nursing Process –
cerning the client. It could be a misun- ­Implementation: Client Needs – Physiological Integrity,
derstanding or a real situation where the Pharmacological and Parenteral Therapies: Cognitive
client’s care is unsafe or inadequate. Level – Application: Concept – Medication.
3. If a new HCP is to be arranged, it is the 46. 1. A digital rectal examination is done to detect
HCP’s responsibility to arrange for another prostate cancer and should be started at age
HCP to assume responsibility for the care of 40 years.
the client. 2. “Middle age” is a relative term; specific ages
4. The choice of HCP is ultimately the client’s. are used for recommendation.
If the HCP cannot arrange for another HCP, 3. The American Cancer Society recom-
the client may be discharged and obtain a mends a colonoscopy at age 50 and every
new health-care provider. five (5) to 10 years thereafter, and a flex-
TEST-TAKING HINT: The nurse should assess ible sigmoidoscopy and a barium enema
the situation; the first step in the nursing every five (5) years.
process is assessment. 4. A flexible sigmoidoscopy should be done
Content – Nursing Management: Integrated Nursing at five (5)-year intervals between the
Process – Implementation: Client Needs – Safe Effective colonoscopy.
Care Environment, Management of Care: Cognitive TEST-TAKING HINT: A digital examination is
Level – Application: ­Concept – Management. an examination performed by the examiner’s
44. 1. If the tissue around the stoma becomes exco- finger and does not examine the entire colon.
riated, the client will be unable to pouch the Content – Medical: Integrated Nursing Process –
stoma adequately, resulting in discomfort and Planning: Client Needs – Health Promotion and
leakage. The client understands the teaching. Maintenance: Cognitive Level – Synthesis: Concept –
2. The client should be on a regular diet, Cellular Regulation.
and the colostomy will have been working 47. 1. Intimacy involves more than sexual inter-
for several days prior to discharge. The course. The client can be sexually active
client’s statement indicates the need for whenever the wounds are healed sufficiently
further teaching. to not cause pain.
3. Until the incision is completely healed, the 2. This is an appropriate nursing intervention
client should not sit in bath water because of for home care, but it has nothing to do with
the potential contamination of the wound by sexual activity.
the bath water. The client understands the 3. The nurse is not a sexual counselor who
teaching. would have these types of charts. The nurse
4. The client has had major surgery and should should address sexuality with the client but
limit lifting to minimal weight. The client would not be considered an expert capable of
understands the teaching. explaining the advantages and disadvantages
TEST-TAKING HINT: This is an abdominal sur- of sexual positioning.
gery and all instructions for major surgery 4. A pouch that becomes dislodged during
apply. This is an “except” question; therefore, the sexual act would cause embarrassment
three (3) options would indicate the client for the client, whose body image has
understands the teaching. ­already been dealt a blow.
Content – Surgical: Integrated Nursing Process – TEST-TAKING HINT: Option “2” does not ad-
Evaluation: Client Needs – Physiological Integrity, Physi- dress the issue and option “3” is outside of
ological Adaptation: Cognitive Level – Analysis: Concept – the nurse’s professional expertise. Option
Bowel Elimination. “1” could be eliminated because of the word
45. 85 mL/hr. “no,” which is an absolute word.
First determine the total amount to be Content – Surgical: Integrated Nursing Process –
­infused over 24 hours: Implementation: Client Needs – Psychosocial Integrity:
1500 + 500 + 20 + 20 = 2,040 mL over Cognitive Level – Application: Concept – Self.
24 hours 48. 1. The client will need to have diagnostic tests,
Then, determine the rate per hour: so this is an appropriate intervention.
2,040 ÷ 24 = 85 mL/hr 2. The client who has an intestinal blockage will
TEST-TAKING HINT: Check and recheck cal- need to be hydrated.
culations. Division should be carried out 3. This client has an intestinal blockage
to the second or third decimal place before from a solid tumor blocking the colon.
rounding. Although the client needs to be cleaned

07_Colgrove_Ch07.indd 274 03/06/16 12:58 PM


C hapter 7  G astrointestinal D isorders 275

out for the colonoscopy, GoLYTELY TEST-TAKING HINT: The test taker must be
could cause severe cramping without a careful to distinguish between -osis and -itis.
reasonable benefit to the client and could Diverticulosis is the condition of having
cause a medical emergency. small pouches in the colon, and preventing
4. Tap water enemas until clear would be constipation is the most important action
­instilling water from below the tumor to try the client can take to prevent diverticulitis
to rid the colon of any feces. The client can (inflammation of the diverticulum).
expel this water. Content – Medical: Integrated Nursing Process –
TEST-TAKING HINT: The stem states a “com- Planning: Client Needs – Physiological Integrity, Physiologi-
plete blockage,” which indicates the client cal Adaptation: Cognitive Level – Synthesis: Concept –
needs surgery. Therefore, options “1” and Bowel Elimination.
“2” are appropriate for surgery. The stem 51. 1. The client will have a nasogastric tube because
asks the test taker which order would be the client will be NPO, which will decompress
questioned, so this is an “except” question. the bowel and remove hydrochloric acid.
Content – Surgical: Integrated Nursing Process – 2. Preventing dehydration is a priority with the
Implementation: Client Needs – Safe Effective Care Envi- client who is NPO.
ronment, Management of Care: Cognitive Level – 3. The nurse should question a clear liquid
Application: Concept – Cellular Regulation. diet because the bowel must be put on
­total rest, which means NPO.
4. The client is in severe pain and should be on
Diverticulosis/Diverticulitis bedrest, which will help rest the bowel.
49. 1. These are classic signs/symptoms of diver- TEST-TAKING HINT: This is an “except” ques-
ticulitis; therefore, the HCP does not need to tion. Therefore, the test taker must identify
be notified. which answer option is incorrect for the
2. These are normal findings for a client diag- stem. Sometimes flipping the question helps
nosed with diverticulitis, but on admission in selecting the correct answer. In this ques-
the nurse should assess the client and docu- tion, the test taker could ask, “Which HCP
ment the findings in the client’s chart. orders would be expected for a client diag-
3. The nurse should not administer any food or nosed with diverticulitis?” The unexpected
medications. option would be the correct answer.
4. The nurse should assess the client to Content – Medical: Integrated Nursing Process –
determine if the abdomen is soft and Implementation: Client Needs – Safe Effective Care Envi-
nontender. A rigid tender abdomen may ronment, Management of Care: Cognitive Level –
indicate peritonitis. Application: Concept – Digestion.

TEST-TAKING HINT: The test taker must re- 52. 1. Fried foods increase cholesterol. Mashed po-
member to apply the nursing process when tatoes do not have the peel, which is needed
answering test questions. Assessment is the for increased fiber.
first step in the nursing process. Although 2. Applesauce does not have the peel, which is
the signs/symptoms are normal and could be needed for increased fiber, and the option
documented, the nurse should always assess. does not identify which type of bread; whole
Content – Medical: Integrated Nursing Process – milk is high in fat.
Implementation: Client Needs – Physiological Integrity, 3. Chicken salad, which has vegetables
Reduction of Risk Potential: Cognitive Level – Synthesis: such as celery, grapes, and apples, and
Concept – Assessment. whole-wheat bread are high in fiber,
which is the therapeutic diet prescribed
50. 1. The client should drink at least 3,000 mL of
for clients with diverticulosis. An adequate
water daily to help prevent constipation.
intake of water helps prevent constipation.
2. The client should exercise daily to help pre-
4. Tomatoes and cucumbers have seeds, and many
vent constipation.
health-care providers recommend clients with
3. The client should eat a high-fiber diet to help
diverticulosis avoid seeds because of the pos-
prevent constipation.
sibility of the seeds entering the diverticulum
4. The client should have regular bowel
and becoming trapped, leading to peritonitis.
movements, preferably daily. Constipation
may cause diverticulitis, which is a po- TEST-TAKING HINT: The test taker must know
tentially life-threatening complication of a high-fiber diet is prescribed for diverticu-
diverticulosis. losis and at least five (5) to six (6) foods are
encouraged or discouraged for the different

07_Colgrove_Ch07.indd 275 03/06/16 12:58 PM


276 M ed -S urg S uccess

types of diets. High-fiber foods are foods Content – Medical: Integrated Nursing Process –
with peels (potato, apple) and whole-wheat ­Implementation: Client Needs – Physiological Integrity,
products. Pharmacological and Parenteral Therapies: Cognitive
Level – Application: Concept – Medication.
Content – Medical: Integrated Nursing Process –
Evaluation: Client Needs – Physiological Integrity, Basic 55. 1. These are signs of peritonitis, which is
Care and Comfort: Cognitive Level – Synthesis: Concept – life threatening. The health-care provider
Medication. should be notified immediately.
53. 1. The client’s abdomen should be soft and 2. A Fleet’s enema will not help a life-
nontender; therefore, this finding would not threatening complication of diverticulitis.
require immediate intervention. 3. A medication administered to help decrease
2. The client had to clean the bowel prior to the client’s temperature will not help a
the colonoscopy; therefore, watery stool is life-threatening complication.
expected. 4. These are signs/symptoms indicating a pos-
3. The client was NPO and received bowel sible life-threatening situation and require
preparation prior to the colonoscopy; there- immediate intervention.
fore, hyperactive bowel sounds might occur TEST-TAKING HINT: In most instances, the test
and do not warrant immediate intervention. taker should not select the option stating to
4. Bowel perforation is a potential complica- notify the HCP immediately, but in some
tion of a colonoscopy. Therefore, signs of situations, it is the correct answer. The test
hypotension—decreased BP and increased taker should look at all the other options and
pulse—warrant immediate intervention determine if the option is information the
from the nurse. HCP requires or if it is an independent inter-
TEST-TAKING HINT: This is an “except” ques- vention which will help the client.
tion. The test taker is being asked to select Content – Medical: Integrated Nursing Process –
which data are abnormal for a procedure. Implementation: Client Needs – Safe Effective Care Envi-
The test taker should remember any invasive ronment, Management of Care: Cognitive Level –
procedure could possibly lead to hemorrhag- Synthesis: Concept – Infection.
ing, and signs of shock should always be con- 56. 42 gtts/min.
sidered a possible correct answer. The nurse must use the formula:
Content – Surgical: Integrated Nursing Process – amount to be infused × drops per minute
Assessment: Client Needs – Safe Effective Care Environ-
ment, Management of Care: Cognitive Level – Synthesis: minutes for infusion
Concept – Perioperative. 250 mL × 10 gtts
54. 1. Peak and trough levels are drawn after the 60 minutes
client has received at least three (3) to four (4) or, 2,500 ÷ 60 minutes = 41.66 gtts/min,
doses of medication, not after the initial dose which should be rounded up to 42 gtts/min.
because the client has just been admitted. TEST-TAKING HINT: The test taker must know
2. The nurse should always ask about aller- how to calculate dosage and calculation ques-
gies to medication when administering tions. Remember to use the drop-down cal-
medications, but especially when adminis- culator if needed; the test taker can ask for an
tering antibiotics, which are notorious for erase slate during state board examinations.
allergic reactions. Content – Medical: Integrated Nursing Process –
3. The peak and trough levels are not drawn Implementation: Client Needs – Physiological Integrity,
prior to the first dose; they are ordered after Pharmacological and Parenteral Therapies: Cognitive
multiple doses. Level – Application: Concept – Medication.
4. The nurse should question when to adminis-
57. 1. Green bile contains hydrochloric acid and
ter the medication, but there is no vital sign
should be draining from the N/G tube;
preventing the nurse from administering an
therefore, the nurse should take no action
antibiotic.
and document the findings.
TEST-TAKING HINT: The test taker must read 2. There is no reason for the nurse to assess the
the stem closely to realize the client is re- client’s bowel sounds because the drainage is
ceiving the initial dose, causing the test taker normal.
to eliminate options “1” and “3” as possible 3. The client’s last bowel movement would not
correct answers. Both options “2” and “4” are affect the N/G drainage.
assessment data, but the test taker should 4. Bile draining from the N/G tube indicates
ask which one will directly affect the admin- the tube is in the stomach and there is no
istration of the medication. need to advance the tube further.

07_Colgrove_Ch07.indd 276 03/06/16 12:58 PM


C hapter 7  G astrointestinal D isorders 277

TEST-TAKING HINT: The test taker must know Content – Medical: Integrated Nursing Process –
what drainage is normal for tubes inserted Diagnosis: Client Needs – Health Promotion and Main-
into the body. Any type of blood or coffee- tenance: Cognitive Level – Analysis: Concept – Bowel
ground drainage would be abnormal and re- Elimination.
quire intervention by the nurse. 60. 1. Total parenteral nutrition is not an expected
Content – Medical: Integrated Nursing Process – ­ order for this client.
Implementation: Client Needs – Safe Effective Care Envi- 2. The bowel must be put at rest. Therefore,
ronment, Management of Care: Cognitive Level – the nurse should anticipate orders for
Application: Concept – Digestion. maintaining the client NPO and a naso-
58. 1. A high-fiber diet will help to prevent con- gastric tube.
stipation, which is the primary reason for 3. These orders would be instituted when the
diverticulitis. client is getting better and the bowel is not
2. Increased fluids will help keep the stool inflamed.
soft and prevent constipation. 4. Surgery is not the first consideration when
3. This will not do anything to help prevent the client is admitted into the hospital.
diverticulitis. TEST-TAKING HINT: “Collaborative” means
4. Exercise will help prevent constipation. the nurse must care for the client with an-
5. No medications are prescribed to prevent an other discipline, and the health-care provider
acute exacerbation of diverticulitis. Antacids would have to order all of the distracters.
are used to neutralize hydrochloric acid in the The test taker should remember food and
stomach. fluid probably should be stopped in the client
TEST-TAKING HINT: This is an alternate-type with lower gastrointestinal problems.
question where the test taker must select Content – Medical: Integrated Nursing Process – ­
more than one option. To identify the cor- Implementation: Client Needs – Safe Effective Care Envi-
rect answers, the test taker should think ronment, Management of Care: Cognitive Level –
about what part of the GI system is affected. Application: Concept – Digestion.
Knowing diverticulosis occurs in the sigmoid
colon would help eliminate options “3” and
“5” because these would be secondary to
Gallbladder Disorders
stomach disorders.
61. 1. After abdominal surgery, it is not uncommon
Content – Medical: Integrated Nursing Process – for bowel sounds to be absent.
Planning: Client Needs – Health Promotion and Mainte-
2. This is a normal amount and color of drainage.
nance: Cognitive Level – Synthesis: Concept – Promoting
Health.
3. The minimum urine output is 30 mL/hr.
4. Refusing to turn, deep breathe, and cough
59. 1. A sedentary lifestyle may lead to obesity and places the client at risk for pneumonia.
contribute to hypertension or heart disease This client needs immediate intervention
but usually not to diverticulosis. to prevent complications.
2. Multiple childbirths are not a risk factor for TEST-TAKING HINT: The test taker should
developing diverticulosis. recognize normal data such as the normal
3. Hemorrhoids would indicate the client urine output and normal data for postopera-
has chronic constipation, which is a strong tive clients. The test taker should apply basic
risk factor for diverticulosis. Constipation concepts when answering questions. Normal
increases the intraluminal pressure in the or expected outcomes do not require action.
sigmoid colon, leading to weakness in the
Content – Surgical: Integrated Nursing Process – ­
intestinal lining, which, in turn, causes
Implementation: Client Needs – Safe Effective Care Envi-
outpouchings, or diverticula. ronment, Management of Care: Cognitive Level –
4. A family history is not a risk factor. Having Synthesis: Concept – Perioperative.
daily bowel movements and preventing con-
stipation will decrease the chance of develop- 62. 1. A heating pad should be applied for 15 to
ing diverticulosis. 20 minutes to assist the migration of the
CO2 used to insufflate the abdomen.
TEST-TAKING HINT: The test taker must know
Shoulder pain is an expected occurrence.
constipation is the leading risk factor for di-
2. Morphine sulfate does not affect the etiology
verticulosis, and if the test taker knows hem-
of the pain.
orrhoids are caused by constipation, it would
3. The surgeon would not order an x-ray for
lead the test taker to select option “3” as the
this condition.
correct answer.

07_Colgrove_Ch07.indd 277 03/06/16 12:58 PM


278 M ed -S urg S uccess

4. There is no indication an injury occurred dur- question. All answer options have something
ing surgery. A sling would not benefit the mi- to do with the abdominal area, and the com-
gration of the CO2. Shoulder pain is expected. mon bile duct is anatomically near the hepatic
TEST-TAKING HINT: The test taker must un- duct, which causes liver signs/symptoms.
derstand laparoscopic surgery to be able to Content – Surgical: Integrated Nursing Process –
answer this question. Option “4” could be ­Implementation: Client Needs – Safe Effective Care Envi-
eliminated b­ ecause of the phrase “injured ronment, Management of Care: Cognitive Level – Analysis:
during surgery.” Concept – Perioperative.

Content – Surgical: Integrated Nursing Process – ­ 65. 1. This is assessment and cannot be delegated.
Implementation: Client Needs – Safe Effective Care Envi- 2. This intervention would require nursing
ronment, Management of Care: Cognitive Level – judgment, and increasing IV fluid is medi-
Application: Concept – Perioperative. cation administration; neither task can be
63. 1 This surgery does not require lipid-lowering delegated.
medications, but eating high-fat meals may 3. A day surgery client can be ambulated
cause discomfort. to the bathroom, so this task can be
2. After removal of the gallbladder, some ­delegated to the UAP.
clients experience abdominal discomfort 4. This would require assessment and cannot be
when eating fatty foods. delegated.
3. Laparoscopic cholecystectomy surgeries are TEST-TAKING HINT: The nurse cannot del-
performed in day surgery, and clients usually egate teaching, assessing, medication admin-
do not need assistance for a week. istration, and evaluating or any task for an
4. Using a pillow to splint the abdomen unstable client to a UAP.
­provides support for the incision and should Content – Surgical: Integrated Nursing Process –
be continued after discharge. Planning: Client Needs – Safe Effective Care Environ-
TEST-TAKING HINT: When answering questions ment, Management of Care: Cognitive Level – Synthesis:
stating “teaching is effective,” the test taker Concept – Nursing Roles.
should look for the correct information. Basic 66. 1. A UGI series requires the client to
concepts should help the test taker answer ­swallow barium, which passes through
questions, and because pain often occurs the ­intestines, making the stools a chalky
after surgeries, option “2” would probably be white color.
a correct answer. 2. Increased heart rate is abnormal data and
Content – Surgical: Integrated Nursing Process – would be cause for further assessment.
Evaluation: Client Needs – Physiological Integrity, Physi- 3. A firm, hard abdomen is not expected from
ological Adaptation: Cognitive Level – Synthesis: Concept – the UGI series.
Perioperative. 4. Hyperactive bowel sounds is not an expected
64. 1. Clay-colored stools are caused by recur- sequela of a UGI series.
ring stricture of the common bile duct, TEST-TAKING HINT: Option “2” could be elim-
which is a sign of post–cholecystectomy inated because it does not have anything to
syndrome. do with the gastrointestinal system. A firm,
2. Yellow-tinted sclera and skin indicate hard abdomen is seldom ever expected, so
residual effects of stricture of the com- option “3” could be eliminated.
mon bile duct, which is a sign of post– Content – Surgical: Integrated Nursing Process –
cholecystectomy syndrome. Assessment: Client Needs – Physiological Integrity, Reduc-
3. Amber-colored urine is a normal finding for tion of Risk Potential: Cognitive Level – Analysis:
a client recovering from an open cholecystec- Concept – Digestion.
tomy, so this does not warrant intervention by 67. 1. Asking the client to cough forcefully may
the nurse. ­irritate the client’s throat.
4. An approximated wound indicates the inci- 2. Early ambulation does not enhance safety
sion is intact and does not warrant interven- because the client will be sedated.
tion by the nurse. 3. The ERCP requires an anesthetic spray
5. Abdominal pain indicates a residual e ­ ffect be used prior to insertion of the endo-
of a stricture of the common bile duct, scope. If medications, food, or fluid are
inflammation, or calculi, which is a sign of given orally prior to the return of the gag
post–cholecystectomy syndrome. reflex, the client may aspirate.
TEST-TAKING HINT: The test taker must 4. Medications are not administered until the
use knowledge of anatomy to answer this gag reflex has returned.

07_Colgrove_Ch07.indd 278 03/06/16 12:58 PM


C hapter 7  G astrointestinal D isorders 279

TEST-TAKING HINT: The test taker must no- 70. 1. The white blood cell count should be ele-
tice adjectives such as “endoscopic,” which vated in clients with chronic inflammation.
means the procedure includes going down 2. A decreased lactate dehydrogenase (LDH)
the mouth; option “3” is the only option that indicates liver abnormalities.
has anything to do with the mouth. Selecting 3. An elevated alkaline phosphatase indicates
a distracter addressing assessment would be liver abnormalities.
appropriate because assessment is the first 4. A decreased bilirubin indicates an obstructive
step of the nursing process. process.
Content – Surgical: Integrated Nursing Process – TEST-TAKING HINT: If the test taker does not
Planning: Client Needs – Safe Effective Care Environ- know what the values mean, the test taker
ment, Management of Care: Cognitive Level – Synthesis: should look to the disease process. The -itis
Concept – Perioperative.
means inflammation, and an educated guess
68. 1. The expected outcome is pain control for would be elevated WBCs in an inflammatory
both preoperative and postoperative care. process.
2. Postoperative care includes ambulation. Content – Medical: Integrated Nursing Process –
3. Prevention of an additional impaired skin Assessment: Client Needs – Physiological Integrity, Reduc-
integrity is a desired postoperative out- tion of Risk Potential: Cognitive Level – Analysis:
come. The incision would be a break in skin Concept – Inflammation.
integrity. 71. 1. Alteration in nutrition may be an appropriate
4. This would be an expected outcome for client problem, but it is not priority.
the client scheduled for surgery. This 2. Alteration in skin integrity may be an appro-
indicates preoperative teaching has been priate client problem but is not priority.
effective. 3. Alteration in urinary elimination may be an
TEST-TAKING HINT: The time element is im- appropriate client problem but is not priority.
portant in this question. The expected out- 4. Acute pain management is the highest
come is required for the preoperative period. priority client problem after surgery be-
Option “1” is incorrect because of the adjec- cause pain may indicate a life-threatening
tive “decreased.” Adjectives commonly deter- problem.
mine the accuracy of the options. TEST-TAKING HINT: When a question asks for
Content – Surgical: Integrated Nursing Process – the highest priority problem, the test taker
Diagnosis: Client Needs – Physiological Integrity, Reduction should look for a life-threatening complica-
of Risk Potential: Cognitive Level – Analysis: Concept – tion. Pain may be expected, but it may indi-
Perioperative.
cate a complication.
69. 1. An increased pulse is expected in the client Content – Surgical: Integrated Nursing Process –
who is in acute pain. Diagnosis: Client Needs – Safe Effective Care Environ-
2. An open cholecystectomy requires a ment, Management of Care: Cognitive Level – Synthesis:
large incision under the diaphragm. Concept – Perioperative.
Deep breathing places pressure on the 72. 1. Measuring the abdominal girth helps further
diaphragm and the incision, causing assess internal bleeding, not external bleeding.
pain. Shallow respirations indicate inad- 2. Palpating the lower abdomen assesses the
equate pain control, and the nurse should bladder, not bleeding.
intervene. 3. Turning the client to the side to assess the
3. Twenty bowel sounds a minute is normal data amount of drainage and possible bleeding is
and does not require further action. important prior to contacting the surgeon.
4. Splinting the abdomen allows the client to in- 4. The first dressing change is usually done
crease the strength of the cough by increasing by the surgeon; the nurse can reinforce the
comfort and does not indicate a need for pain dressing.
medication.
TEST-TAKING HINT: The adjectives “large” and
TEST-TAKING HINT: The stem asks which data “red” indicate the client is bleeding, and as-
would warrant pain medication. Therefore, sessment is always priority when the client is
the test taker should select an answer not having a possible complication of a surgery.
expected or not normal for clients who are Remember, assessment is the first step in the
postoperative abdominal surgery. nursing process.
Content – Surgical: Integrated Nursing Process – Content – Surgical: Integrated Nursing Process –
Assessment:Client Needs – Physiological Integrity, Pharma- Assessment: Client Needs – Physiological Integrity, Reduc-
cological and ­Parenteral Therapies: Cognitive Level – tion of Risk Potential: Cognitive Level – Analysis:
Analysis: Concept – Medication. Concept – Perioperative.

07_Colgrove_Ch07.indd 279 03/06/16 12:58 PM


280 M ed -S urg S uccess

Liver Failure electrolyte values; fluids do not affect hepatic


encephalopathy.
73. 1. The client’s throat is not anesthetized during 3. A diet high in calories and moderate in fat
the insertion of a nasogastric tube, so the gag intake is recommended to promote healing.
reflex does not need to be assessed. 4. Ammonia is a by-product of protein
2. While the balloons are inflated, the client metabolism and contributes to hepatic
must not be left unattended in case they encephalopathy. Reducing protein intake
become dislodged and occlude the airway. should decrease ammonia levels.
This is a safety issue. TEST-TAKING HINT: The test taker could
3. This laxative is administered to decrease the eliminate options “1” and “2” based on the
ammonia level, but the question does not say knowledge sodium and water work together
the client’s ammonia level is elevated. and address edema, not encephalopathy. The
4. Esophageal bleeding does not cause the am- test taker’s knowledge of biochemistry—­
monia level to be elevated. protein breaks down to ammonia, carbohy-
TEST-TAKING HINT: In most cases, the test drates break down to ­glucose, and fat breaks
taker should not select an option containing down to ketones—may be helpful in selecting
the word “all,” but in some instances, it may the correct answer.
be the correct answer. Although the ammo- Content – Medical: Integrated Nursing Process –
nia level is elevated in liver failure, the test ­Implementation: Client Needs – Physiological Integrity,
taker must be clear as to what the question is ­Basic Care and Comfort: Cognitive Level – Analysis:
asking. “Inflate” is the key to answering the Concept – Nutrition.
question correctly. 76. 1. The procedure is done in the client’s room,
Content – Medical: Integrated Nursing Process – ­ with the client seated either on the side of the
Implementation: Client Needs – Safe Effective Care bed or in a chair.
­Environment, Management of Care: Cognitive Level – 2. The client should empty the bladder prior to
Application: Concept – Nursing Roles. the procedure to avoid bladder puncture, but
74. 1. The client should empty the bladder imme- there is no need for an indwelling catheter to
diately prior to the liver biopsy, not after the be inserted.
procedure. 3. The client is at risk for hypovolemia;
2. Foods and fluids are usually withheld two (2) therefore, vital signs will be assessed
hours after the biopsy, after which the client frequently to monitor for signs of
can resume the usual diet. hemorrhaging.
3. Direct pressure is applied to the site, and 4. The client does not have to hold the breath
then the client is placed on the right side when the catheter is inserted into the peri-
to maintain site pressure. toneum; this is done when obtaining a liver
4. Blood urea nitrogen (BUN) and creatinine biopsy.
levels are monitored for kidney function, not TEST-TAKING HINT: If the test taker had no
liver function, and the renal system is not idea what the answer is, knowing vital signs
­affected with the liver biopsy. are assessed after all procedures should make
TEST-TAKING HINT: The adjective “postproce- the test taker select this option.
dure” should help the test taker rule out op- Content – Surgical: Integrated Nursing Process –
tion “1.” Knowing the anatomical position of Planning: Client Needs – Physiological Integrity, Reduction
the liver should help the test taker select op- of Risk Potential: Cognitive Level – Synthesis: Concept –
tion “3” as the correct answer. The test taker Assessment.
must know laboratory data for each organ, 77. 1. Hot water increases pruritus, and soap
which helps rule out option “4” as a possible will cause dry skin, which increases pru-
correct answer. ritus; therefore, the nurse should discuss
Content – Surgical: Integrated Nursing Process – ­ this with the UAP.
Implementation: Client Needs – Safe Effective Care Envi- 2. Applying emollient lotion will help prevent
ronment, Management of Care: Cognitive Level – dry skin, which will help decrease pruritus;
Application: Concept – Perioperative. therefore, this would not require any inter-
75. 1. Sodium is restricted to reduce ascites and gen- vention by the nurse.
eralized edema, not for hepatic encephalopathy. 3. Mittens will help prevent the client from
2. Fluids are calculated based on di- scratching the skin and causing skin break-
uretic therapy, urine output, and serum down. This would not require intervention by
the nurse.

07_Colgrove_Ch07.indd 280 03/06/16 12:58 PM


C hapter 7  G astrointestinal D isorders 281

4. The skin should be patted dry, not rubbed, 5. Asterixis is a flapping tremor of the hands
because rubbing the skin will cause increased when the arms are extended and indicates an
irritation. This action does not ­require inter- elevated ammonia level not associated with
vention by the nurse. vitamin K deficiency.
TEST-TAKING HINT: A concept accepted for TEST-TAKING HINT: The test taker must know
most clients during a.m. care is not to use the function of specific vitamins. Vitamin K
hot water because it causes dilation of ves- is responsible for blood clotting. This is an
sels, which may cause orthostatic hypoten- alternate-type question, which requires the
sion. This is not the rationale for not using test taker to select all applicable interven-
hot water with a client who has pruritus, but tions; the test taker should select interven-
sometimes the test taker can apply broad tions addressing bleeding.
concepts when answering questions. Content – Medical: Integrated Nursing Process –
Content – Medical: Integrated Nursing Process – ­ Implementation: Client Needs – Physiological Integrity,
Implementation: Client Needs – Safe Effective Care Envi- Physiological Adaptation: Cognitive Level – Application:
ronment, Management of Care: Cognitive Level – Concept – Nutrition.
Synthesis: Concept – Inflammation.
80. 1. Hypoalbuminemia (decreased albumin) and
78. 1. Two (2) kg is more than four (4) pounds, muscle wasting are metabolic effects, not
which indicates severe fluid retention and is gastrointestinal effects.
not an appropriate goal. 2. Oligomenorrhea is no menses, which is a
2. Excess fluid volume could be secondary reproductive effect, and decreased body hair
to portal hypertension. Therefore, no is an integumentary effect.
increase in abdominal girth would be an 3. Clay-colored stools and hemorrhoids are
appropriate short-term goal, indicating gastrointestinal effects of liver failure.
no excess of fluid volume. 4. Dyspnea is a respiratory effect, and caput
3. Vital signs are appropriate to monitor, but medusae (dilated veins around the umbilicus)
they do not yield specific information about is an integumentary effect, although it is on
fluid volume status. the abdomen.
4. Having the client receive a low-sodium diet TEST-TAKING HINT: The adjective “gastro-
does not ensure the client will comply with intestinal” is the key word guiding the test
the diet. The short-term goal must evaluate taker to select the correct answer. The test
if the fluid volume is within normal limits. taker must rule out options not addressing
TEST-TAKING HINT: Remember, goals evaluate gastrointestinal symptoms. Although liver
the interventions; therefore, option “4” could failure affects every body system, the ques-
be eliminated as the correct answer because tion asks for a gastrointestinal effect.
it is an intervention, not a goal. Short-term Content – Medical: Integrated Nursing Process –
weight fluctuations tend to reflect fluid Assessment: Client Needs – Physiological Integrity, Reduc-
balance, and any weight gain in 24 hours tion of Risk Potential: Cognitive Level – Analysis:
indicates retention of fluid, which is not an Concept – Bowel Elimination.
appropriate goal. 81. 1. It really doesn’t matter how long the client
Content – Medical: Integrated Nursing Process – has been drinking alcohol. The diagnosis of
Diagnosis: Client Needs – Physiological Integrity, Physi- alcoholic cirrhosis indicates the client has
ological Adaptation: Cognitive Level – Analysis: Concept –
probably been drinking for many years.
Digestion.
2. An advance directive is important for the
79. 1. Vitamin K deficiency causes impaired co- client who is terminally ill, but it is not the
agulation; therefore, rectal thermometers priority question.
should be avoided to prevent bleeding. 3. The nurse must know when the client
2. Soft-bristle toothbrushes will help ­prevent had the last alcoholic drink to be able
bleeding of the gums. to determine when and if the client will
3. Platelet count, partial thromboplastin experience delirium tremens, the physical
time/prothrombin time (PTT/PT), and in- withdrawal from alcohol.
ternational normalized ratio (INR) should 4. This is not a typical question asked by the
be monitored to assess coagulation status. nurse unless the client is malnourished, which
4. Injections should be avoided, if at all pos- is not information provided in the stem.
sible, because the client is unable to clot, TEST-TAKING HINT: Because the word “alco-
but if they are absolutely necessary, the hol” is in the stem of the question, if the test
nurse should use small-gauge needles. taker had no idea what the correct answer is,

07_Colgrove_Ch07.indd 281 03/06/16 12:58 PM


282 M ed -S urg S uccess

the test taker should select an option with 2. There is no instrument used at home to
the word “alcohol” in it and look closely at test daily ammonia levels. The ammonia
options “1” and “3.” level is a serum level requiring venipunc-
Content – Medical: Integrated Nursing Process – ture and laboratory diagnostic equipment.
Assessment: Client Needs – Physiological Integrity, Reduc- 3. Diarrhea indicates an overdosage of the
tion of Risk Potential: Cognitive Level – Analysis: medication, possibly requiring the dosage to
Concept – Digestion. be decreased. The HCP needs to make this
change in dosage, so the client understands
82. 1. Blood in the intestinal tract is digested as
the teaching.
a protein, which increases serum ammonia
4. The client should check the stool for bright-
levels and increases the risk of developing
red blood as well as dark, tarry stool.
hepatic encephalopathy.
2. Decreased albumin causes the client to de- TEST-TAKING HINT: This is an “except” ques-
velop ascites. tion. The test taker must realize three (3)
3. An enlarged spleen increases the rate at which options indicate an understanding of the
red blood cells (RBCs), WBCs, and platelets teaching. If the test taker does not know the
are destroyed, causing the client to develop answer, notice that all the options except “2”
anemia, leukopenia, and thrombocytopenia, have something to do with stool, and laxative
but not hepatic encephalopathy. affects the stool.
4. An increase in aldosterone causes sodium and Content – Medical: Integrated Nursing Process –
water retention, resulting in the development Evaluation: Client Needs – Physiological Integrity, Phar-
of ascites and generalized edema. macological and ­Parenteral Therapies: Cognitive Level –
Synthesis: Concept – Digestion.
TEST-TAKING HINT: Some questions require
the test taker to have specific knowledge to
be able to identify the correct answer. This is Hepatitis
one of those questions.
Content – Medical: Integrated Nursing Process – 85. 1. Clay-colored stools and jaundice occur in the
Assessment: Client Needs – Physiological Integrity, Reduc- icteric phase of hepatitis.
tion of Risk Potential: Cognitive Level – Analysis: 2. Normal appetite and itching occur in the ic-
Concept – Digestion. teric phase of hepatitis.
83. 1. This is a therapeutic response and is used to 3. Fever subsides in the icteric phase, and the
encourage the client to verbalize feelings but pain is in the right upper quadrant.
does not provide factual information. 4. “Flu-like” symptoms are the first com-
2. This is passing the buck; the nurse should be plaints of the client in the preicteric phase
able to answer this question. of hepatitis, which is the initial phase and
3. This is the main reason the HCP decreases may begin abruptly or insidiously.
the client’s medication dose and is an ex- TEST-TAKING HINT: The test taker must use
planation appropriate for the client. anatomy knowledge in ruling out incorrect
4. This is the medical explanation as to why the answers; option “3” could be ruled out be-
medication dose is decreased, but it should cause the liver is in the right upper quadrant.
not be used to explain to a layperson. Content – Medical: Integrated Nursing Process –
TEST-TAKING HINT: The test taker should Assessment: Client Needs – Physiological Integrity, Reduc-
provide factual information when the client tion of Risk Potential: Cognitive Level – Analysis:
asks “why.” Therefore, options “1” and “2” Concept – Bowel Elimination.
could be eliminated as possible correct an- 86. 1. The hepatitis A virus is in the stool of
swers. Both options “3” and “4” explain the infected people and takes up to two (2)
rationale for decreasing the medication dose, weeks before symptoms develop.
but the nurse should answer in terms the cli- 2. Hepatitis B virus is spread through contact
ent can understand. Would a layperson know with infected blood and body fluids.
what “half-life” means? 3. Hepatitis C virus is transmitted through in-
Content – Medical: Integrated Nursing Process – fected blood and body fluids.
­Implementation: Client Needs – Physiological Integrity, 4. Hepatitis D virus only causes infection in peo-
­Pharmacological and Parenteral Therapies: Cognitive ple who are also infected with hepatitis B or C.
Level – ­Application: Concept – Medication.
TEST-TAKING HINT: This is a knowledge ques-
84. 1. Two to soft three stools a day indicates the tion; the nurse must be aware of how the var-
medication is effective. ious types of hepatitis virus are transmitted.

07_Colgrove_Ch07.indd 282 03/06/16 12:58 PM


C hapter 7  G astrointestinal D isorders 283

Content – Medical: Integrated Nursing Process – 89. 1. The client must avoid alcohol altogether, not
Diagnosis: Client Needs – Health Promotion and Main- decrease intake, to prevent further liver dam-
tenance: Cognitive Level – Knowledge: Concept – Bowel age and promote healing.
Elimination. 2. Adequate rest is needed for maintaining
87. 1. Airborne Precautions are required for trans- optimal immune function.
mission occurring by dissemination of either 3. Clients are more often anorexic and nause-
airborne droplet nuclei or dust particles con- ated in the afternoon and evening; therefore,
taining the infectious agent. the main meal should be in the morning.
2. Standard Precautions apply to blood, all 4. Diet drinks and juices provide few calories,
body fluids, secretions, and excretions, and the client needs an increased-calorie diet
except sweat, regardless of whether they for healing.
contain visible blood. TEST-TAKING HINT: The test taker must be
3. Droplet transmission involves contact of aware of key words in both the stem and
the conjunctivae of the eyes or mucous answer options. The “icteric” phase means
membranes of the nose or mouth with large- the acute phase. The word “decrease” should
particle droplets generated during coughing, cause the test taker to eliminate option “1”
sneezing, talking, or suctioning. as a possible correct answer, and “large”
4. Exposure Precautions is not a designated iso- should cause the test taker to eliminate “3”
lation category. as a possible correct answer.
TEST-TAKING HINT: The test taker must know Content – Medical: Integrated Nursing Process –
Standard Precautions are used by all health- Planning: Client Needs – Physiological Integrity, Physiologi-
care workers who have direct contact with cal Adaptation: Cognitive Level – Synthesis: Concept –
clients or with their body fluids or have indi- Digestion.
rect contact with objects used by clients who 90. 1. Hepatitis B can be transmitted by sharing
are infected, such as would be involved in any type of needles, especially those used
emptying trash, changing linens, or cleaning by drug abusers.
the room. 2. Hepatitis B can be transmitted through
Content – Medical: Integrated Nursing Process – sexual activity; therefore, the nurse should
­Implementation: Client Needs – Safe Effective Care recommend abstinence, mutual monog-
Environment, Safety and Infection Control: Cognitive amy, or barrier protection.
Level – Application: ­Concept – Safety. 3. Three doses of hepatitis B vaccine provide
88. 1. Eating after each other should be discour- immunity in 90% of healthy adults.
aged, but it is not the most important 4. Immune globulin injections are administered
intervention. as postexposure prophylaxis (after being ex-
2. Only bottled water should be consumed in posed to hepatitis B), but encouraging these
third world countries, but this precaution is injections is not a health promotion activity.
not necessary in American high schools. 5. Hepatotoxic medications should be avoided
3. Hepatitis B and C, not hepatitis A, are trans- in clients who have hepatitis or who have had
mitted by sexual activity. hepatitis. The health-care provider prescribes
4. Hepatitis A is transmitted via the fecal– medications, and the layperson does not know
oral route. Good hand washing helps which medications are hepatotoxic.
to prevent its spread. Singing the happy TEST-TAKING HINT: In this “Select all that
birthday song takes approximately apply” type of question, there may be only
30 seconds, which is how long an individ- one (1) correct answer, there may be several,
ual should wash his or her hands. or all five options may be correct answers.
TEST-TAKING HINT: The test taker must real- Content – Medical: Integrated Nursing Process –
ize good hand washing is the most important Planning: Client Needs – Health Promotion and Mainte-
action in preventing transmission of hepatitis nance: Cognitive Level – Synthesis: Concept – Promoting
A virus. Often, the test taker will not select Health.
the answer option that seems too easy—but 91. 1. This is a therapeutic response, and the nurse
remember, do not overlook the obvious. should provide factual information.
Content – Medical: Integrated Nursing Process – 2. Milk thistle has an active ingredient,
Planning: Client Needs – Health Promotion and Mainte- ­silymarin, which has been used to treat
nance: Cognitive Level – Synthesis: Concept – Promoting liver disease for more than 2,000 years.
Health. It is a powerful oxidant and promotes
liver cell growth.

07_Colgrove_Ch07.indd 283 03/06/16 12:58 PM


284 M ed -S urg S uccess

3. The nurse should not discourage comple- intervention will directly help the nurse—and
mentary therapies. that is to clean the area.
4. This is a judgmental statement, and the nurse Content – Nursing Management: Integrated Nursing
should encourage the client to ask questions. ­Process – Implementation: Client Needs – Safe Effective
TEST-TAKING HINT: The test taker may not Care Environment, Management of Care: Cognitive
have any idea what milk thistle is but should Level – Application: Concept – Safety.
apply test-taking strategies, including not se- 94. 1. The serum ammonia level is increased in liver
lecting options with “why” unless interview- failure, but it is not the cause of clay-colored
ing the client. Only use therapeutic responses stools.
when unable to provide factual information. 2. Bilirubin, the by-product of red blood cell
At times, the test taker may not like any an- destruction, is metabolized in the liver
swer option but should always apply the rules and excreted via the feces, which causes
to help determine the correct answer. the feces to be brown in color. If the liver
Content – Medical: Integrated Nursing Process – is damaged, the bilirubin is excreted via
­Implementation: Client Needs – Physiological Integrity, the urine and skin.
­Pharmacological and Parenteral Therapies: Cognitive 3. The liver excretes bile into the gallbladder
Level – Synthesis: Concept – Medication. and the body uses the bile to digest fat, but it
92. 1. Sufficient energy is required for healing. does not affect the feces.
Adequate carbohydrate intake can spare 4. Vitamin deficiency, resulting from the liver’s
protein. The client should eat approxi- inability to detoxify vitamins, may cause steat-
mately 16 carbohydrate kilocalories for orrhea, but it does not cause clay-colored stool.
each kilogram of ideal body weight daily. TEST-TAKING HINT: The test taker should have
2. TPN is not routinely prescribed for the client a grasp of physiology to help answer this ques-
with hepatitis; the client must lose a large of tion. Clay-colored stool indicates no color in
amount of weight and be unable to eat any- the feces. Because color in the feces is caused
thing for TPN to be ordered. by bilirubin, lack of color would be the result
3. Salt intake does not affect the healing of the of the liver’s inability to excrete bilirubin.
liver. Content – Medical: Integrated Nursing Process –
4. Water intake does not affect healing of the Diagnosis: Client Needs – Physiological Integrity, Physi-
liver, and the client should not drink so much ological Adaptation: Cognitive Level – Analysis: Concept –
water as to decrease caloric food intake. Bowel Elimination.
TEST-TAKING HINT: The test taker should key 95. 1. The client should avoid alcohol to prevent
in on “less than body requirements” in the further liver damage and promote healing.
stem and select the answer option addressing 2. Rest is needed for healing of the liver and to
increasing calories, which eliminates options promote optimum immune function.
“3” and “4.” 3. Clients with hepatitis need increased caloric
Content – Medical: Integrated Nursing Process – intake, so this is a good statement.
Planning: Client Needs – Physiological Integrity, Basic 4. The client needs to understand some
Care and Comfort: Cognitive Level – Synthesis: types of cough syrup have alcohol and
Concept – Nutrition. all alcohol must be avoided to prevent
93. 1. The nurse must notify the infection con- further injury to the liver; therefore, this
trol nurse as soon as possible so treatment statement requires intervention.
can start if needed, but this is not the first TEST-TAKING HINT: If the test taker did not
intervention. know the answer, the test taker could apply
2. The nurse should first clean the needle the rule of avoiding any over-the-counter
stick with soap and water and attempt (OTC) medications unless approved by a
stick bleed to help remove any virus in- health-care provider.
jected into the skin. Content – Medical: Integrated Nursing Process – ­
3. Postexposure prophylaxis may be needed, but Implementation: Client Needs – Safe Effective Care Envi-
this is not the first action. ronment, Management of Care: Cognitive Level –
4. The infection control/employee health nurse Synthesis: Concept – Addiction.
will check the status of the client whom the 96. 1. The laboratory technician draws serum blood
needle was used on before the nurse stuck studies, not the UAP.
herself. 2. The UAP can obtain the intake and out-
TEST-TAKING HINT: The question requires put, but the nurse must evaluate the data to
the test taker to identify the first interven- ­determine if the results are normal for the cli-
tion. The test taker should think about which ent’s disease process or condition.

07_Colgrove_Ch07.indd 284 03/06/16 12:58 PM


C hapter 7  G astrointestinal D isorders 285

3. The UAP can perform a bedside gluco­ TEST-TAKING HINT: The test taker should be
meter check, but the nurse must evaluate careful with words such as “all,” “only,” and
the result and determine any ­action needed. “never”; few absolutes exist in the health-
4. The ward clerk has specific training that allows care field.
the transcribing of health-care provider orders. Content – Medical: Integrated Nursing Process –
TEST-TAKING HINT: The test taker must be Planning: Client Needs – Health Promotion and Mainte-
knowledgeable of delegation rules; the nurse nance: Cognitive Level – Synthesis: Concept – Nutrition.
cannot delegate assessing, teaching, medica- 99. 1. Symptoms develop 8 to 48 hours after
tion administration, evaluating, and any task ingesting the Salmonella bacteria and
for an unstable client. include diarrhea, abdominal cramping,
Content – Nursing Management: Integrated Nursing nausea, and vomiting, along with low-
Process – Planning: Client Needs – Safe Effective Care grade fever, chills, and weakness.
Environment, Management of Care: Cognitive Level – 2. Neuromuscular paralysis and dysphagia
Synthesis: Concept – Nursing Roles. occur with botulism, a severe life-
threatening form of food poisoning caused
by Clostridium botulinum.
Gastroenteritis
3. Gross explosive bloody diarrhea is a clinical
manifestation of hemorrhagic colitis caused
97. 1. The client would be taking antidiarrheal
by Escherichia coli.
medication, not medications to stimulate
4. Gray-cloudy diarrhea with no fecal odor,
bowel movements.
blood, or pus is caused by cholera, which is
2. The client probably has traveler’s diar-
endemic in parts of Asia, the Middle East,
rhea, and oral rehydration is the preferred
and Africa.
choice for replacing fluids lost as a result
of diarrhea. An oral glucose electrolyte TEST-TAKING HINT: Often when two (2)
solution, such as Gatorade, All-Sport, or ­options have the same clinical manifesta-
Pedialyte, is recommended. tion, such as diarrhea and stool, this should
3. The client should be encouraged to stay on make the test taker realize either one of
liquids and eat bland foods of all three (3) food these two (2) options is correct, so the other
groups—carbohydrates, proteins, and fats. two (2) options can be eliminated, or both
4. There is no need for the client to weigh her- are incorrect.
self daily. Symptoms usually resolve within two Content – Medical: Integrated Nursing Process –
(2) to three (3) days without complications. Assessment: Client Needs – Physiological Integrity, Reduc-
tion of Risk Potential: Cognitive Level – Analysis:
TEST-TAKING HINT: Be sure to note the adjec-
Concept – Bowel Elimination.
tives and adverbs in the stem and the answer
options, such as “cathartic” laxative and 100. 1. The normal serum sodium level is 135 to
weigh “daily.” These words are very often im- 145 mEq/L; therefore, an intervention by
portant in ruling out answers and identifying the nurse is not needed.
the correct answer. 2. These are normal arterial blood gas results;
Content – Medical: Integrated Nursing Process – therefore, the nurse would not need to
Implementation: Client Needs – Safe Effective Care Envi- intervene.
ronment: Cognitive Level – Application: Concept – Bowel 3. In gastroenteritis, diarrhea often results
Elimination. in metabolic acidosis and loss of potas-
98. 1. Well-cooked meat will help prevent gastro- sium. The normal serum potassium level
enteritis secondary to staphylococcal food is 3.5 to 5.5 mEq/L; therefore, a level
poisoning. of 3.3 mEq/L would require immediate
­intervention. Hypokalemia can lead to
2. Refrigerating dairy products will help prevent
gastroenteritis secondary to eating foods kept life-threatening cardiac dysrhythmias.
4. A stool specimen showing fecal leukocytes
at room temperature, causing staphylococcal
supports the diagnosis of gastroenteritis and
food poisoning.
does not warrant immediate intervention by
3. Drinking bottled water will help prevent gas-
the nurse.
troenteritis secondary to Escherichia coli found
in contaminated water. TEST-TAKING HINT: The test taker should
4. Any discolored food, food from a damaged read the stem and be certain he or she un-
can or jar, or food from a can or jar not derstands what the question is asking—in
having a tight seal should be ­destroyed this case, which data require “immediate
without tasting or touching it. intervention”? Therefore, the test taker is

07_Colgrove_Ch07.indd 285 03/06/16 12:58 PM


286 M ed -S urg S uccess

identifying an answer not normal for the 2. Nausea may occur, but it is not priority.
disease process. However, excessive vomiting could lead to
Content – Medical: Integrated Nursing Process – potential complications.
Assessment: Client Needs – Safe Effective Care Environ- 3. Risk for aspiration could result from vom-
ment, Management of Care: Cognitive Level – Synthesis: iting; however, vomiting does not usually
Concept – Bowel Elimination. occur in food poisoning, but it may be sec-
101. 1. If the diarrhea persists more than 48 hours, ondary to botulism.
4. Impaired urinary elimination is not a prior-
the client should notify the HCP. Diarrhea for
ity. The client has diarrhea, not urine output
more than 96 hours could lead to metabolic
problems.
acidosis, hypokalemia, and possible death.
2. Washing hands should be done by the cli- TEST-TAKING HINT: Always notice the client’s
ent at all times, but especially when the age because it is usually a significant clue as
client has gastroenteritis. The bacteria in to the correct answer. Prioritizing questions
feces may be transferred to other people may have more than one (1) potential ap-
via food if hands are not washed properly. propriate nursing problem but only one (1)
3. Steroids are not used in the treatment of has priority. Remember Maslow’s hierarchy
gastroenteritis; antidiarrheal medication is of needs.
usually prescribed. Content – Medical: Integrated Nursing Process –
4. The client may be asked to provide a stool Diagnosis: Client Needs – Safe Effective Care Environ-
specimen for culture, ova, parasites, and ment, Management of Care: Cognitive Level – Analysis:
­fecal leukocytes, but the client is not asked Concept – Bowel Elimination.
for a 24-hour stool collection. 104. 1. The client would have increased gur-
TEST-TAKING HINT: If the test taker did not gling sounds, revealing hyperactive bowel
know the answer to this question, hand movements.
washing should be selected because it is the 2. A hard, firm, edematous abdomen is not
number-one intervention for preventing expected in a client with gastroenteritis; this
any type of contamination or nosocomial would indicate a possible complication and
infection. require further assessment.
Content – Medical: Integrated Nursing Process – 3. The client has increased liquid bowel
Planning: Client Needs – Physiological Integrity, Physi- movements (diarrhea) but should not have
ological Adaptation: Cognitive Level – Synthesis: blood in the stool, which is the definition
Concept – Bowel Elimination. of melena.
4. Borborygmi, or loud, rushing bowel
102. 1. Antidiarrheal medications are contraindi-
sounds, indicates increased peristalsis,
cated with botulism because the toxin needs
which occurs in clients with diarrhea
to be expelled from the body.
and is the primary clinical manifesta-
2. Aminoglycoside antibiotics will not be or-
tion in a client diagnosed with acute
dered because there is no bacterium with
gastroenteritis.
botulism; it is caused by a neurotoxin.
3. A botulism antitoxin neutralizes the TEST-TAKING HINT: The test taker should
­circulating toxin and is prescribed for a realize that, in an acute condition, the as-
client with botulism. sessment data would be abnormal, which
4. An angiotensin-converting enzyme (ACE) may help select the correct answer for some
inhibitor is prescribed for a client diagnosed questions.
with cardiovascular disease. Content – Medical: Integrated Nursing Process –
TEST-TAKING HINT: The key word in this Assessment: Client Needs – Physiological Integrity, Reduc-
tion of Risk Potential: Cognitive Level – Analysis:
question is “treat.” Because botulism does
Concept – Bowel Elimination.
not end in -itis, and thus is not an infection,
the use of an antibiotic can be eliminated. 105. 1. The UAP can calculate the client’s intake and
Content – Medical: Integrated Nursing Process – output, but the nurse must evaluate the data
Planning: Client Needs – Physiological Integrity, Pharma- to determine if it is normal for the elderly
cological and ­Parenteral Therapies: Cognitive Level – client diagnosed with acute gastroenteritis.
Synthesis: Concept – Medication. 2. The UAP can take the vital signs for
a client who is stable; the nurse must
103. 1. Fluid volume deficit secondary to
­interpret and evaluate the vital signs.
­diarrhea is the priority because of the
3. The UAP cannot administer medications,
potential for metabolic acidosis and
and IV solutions are considered to be
­hypokalemia, which are both life threat-
medications.
ening, especially in the elderly.

07_Colgrove_Ch07.indd 286 03/06/16 12:58 PM


C hapter 7  G astrointestinal D isorders 287

4. The nurse cannot delegate assessment. The 5. The elderly client is at risk for ortho-
client may have an excoriated perianal area static hypotension; therefore, safety pre-
secondary to diarrhea; therefore, the nurse cautions should be instituted to ensure
should assess the client. the client doesn’t fall as a result of a
TEST-TAKING HINT: The nurse should not ­decrease in blood pressure.
delegate any nursing task requiring judg- TEST-TAKING HINT: This is an alternate-type
ment or assessment and cannot delegate question requiring the test taker to choose
the administration of medications. Words all interventions that apply. The test taker
such as “evaluate” mean the same thing as should look at each option and consider if
“assess”; therefore, options “1,” “3,” and “4” this is an intervention for an “elderly” cli-
can be eliminated. ent. The elderly are a special population
Content – Medical: Integrated Nursing Process – usually requiring specific interventions ad-
Planning: Client Needs – Safe Effective Care Environ- dressing the aging process no matter what
ment, Management of Care: Cognitive Level – Synthesis: the disease process.
Concept – Bowel Elimination. Content – Medical: Integrated Nursing Process –
106. 1. Leg cramps could indicate hypokalemia, Diagnosis: Client Needs – Safe Effective Care Environ-
ment, Management of Care: Cognitive Level – Analysis:
which is a potential complication of exces-
Concept – Bowel Elimination.
sive diarrhea and should be reported to the
health-care provider. 108. 1. Epigastric pain is expected in a client diag-
2. The client should increase the fluid intake nosed with peptic ulcer disease.
because oral rehydration is the primary 2. Four (4) diarrheal stools are not unusual in a
treatment for gastroenteritis to replace lost client diagnosed with gastroenteritis.
fluid as a result of diarrhea and to prevent 3. Tented skin turgor and dry mucous
dehydration. membranes indicate dehydration, which
3. Reintroducing solid foods slowly, in small warrants the nurse assessing this client
amounts, will allow the bowel to rest and first.
the mucosa to return to normal func- 4. Vomiting is expected in a client diagnosed
tioning after acute gastroenteritis. with food poisoning.
4. Bottled water should be consumed when TEST-TAKING HINT: When managing clients,
contaminated water is suspected, and an oral the nurse must be able to prioritize care.
glucose–electrolyte solution, such as Gato- Therefore, the test taker must be able to
rade or Pedialyte, should be recommended. determine which client’s complaints, signs,
TEST-TAKING HINT: Both options “2” and or symptoms are not expected of the disease
“4” refer to fluids, which should make the process. The test taker should always look
test taker either eliminate both of these or at the client’s age because it may help deter-
select from one (1) of these two (2) as the mine the best answer.
right answer. Content – Medical: Integrated Nursing Process –
Content – Medical: Integrated Nursing Process – Assessment: Client Needs – Safe Effective Care Environ-
Evaluation: Client Needs – Physiological Integrity, Physi- ment, Management of Care: Cognitive Level – Analysis:
ological Adaptation: Cognitive Level – Synthesis: Concept – Digestion.
Concept – Bowel Elimination.

107. 1. The nurse should assess skin turgor over the Abdominal Surgery
sternum in the elderly client because loss
of subcutaneous fat associated with aging 109. 1. Absent bowel sounds indicate a paralytic
makes skin turgor assessment on the arms ileus, not peritonitis, and the potassium level
less reliable. is within normal limits (3.5 to 5.5 mEq/L).
2. Orthostatic hypotension indicates fluid 2. Abdominal cramping would not make the
volume deficit, which can occur in an nurse suspect peritonitis, and the hemoglo-
elderly client who is having many episodes bin is normal (13 to 17 g/dL).
of diarrhea. 3. Campylobacter is a cause of profuse diar-
3. The nurse should record frequency and rhea, but it does not support a diagnosis of
characteristics of stool, not sputum, in the peritonitis.
client diagnosed with gastroenteritis. 4. A hard, rigid abdomen indicates an
4. Standard Precautions, including wearing ­inflamed peritoneum (abdominal wall
gloves and hand washing, help prevent cavity) resulting from an infection, which
the spread of the infection to others. results in an elevated WBC level.

07_Colgrove_Ch07.indd 287 03/06/16 12:58 PM


288 M ed -S urg S uccess

TEST-TAKING HINT: The -itis of peritonitis 112. 1. Evisceration is a life-threatening condi-


means inflammation, and if the test taker tion in which the abdominal contents
has no idea what the answer is, an elevated protrude through the ruptured incision.
WBC count should provide a key to select- The nurse must protect the bowel from
ing option “4” as the correct answer. the environment by placing a sterile nor-
Content – Surgical: Integrated Nursing Process – mal saline gauze on it, which prevents the
Assessment: Client Needs – Physiological Integrity, Reduc- intestines from drying out and necrosing.
tion of Risk Potential: Cognitive Level – Analysis: 2. The nurse should not attempt to replace the
Concept – Inflammation. protruding bowel.
3. This position places the client with the head
110. 1. The nurse may notify the surgeon if war-
of the bed elevated, which will make the
ranted, but it is not the first intervention.
situation worse.
2. The nurse should instruct the client to
4. Antibiotics will not protect the protruding
splint the incision when coughing, then
bowels, which must be priority. Antibiotics
take ­further action.
will be administered at a later time to pre-
3. Assessing the surgical incision is the first
vent infection, but this is not urgent.
intervention because this may indicate
the client has wound dehiscence. TEST-TAKING HINT: The test taker must un-
4. The nurse should never administer pain derstand the word “evisceration” to answer
medication without assessing for potential this question.
complications. Content – Surgical: Integrated Nursing Process – ­
TEST-TAKING HINT: The stem is asking which Implementation: Client Needs – Safe Effective Care
­Environment, Management of Care: Cognitive Level –
intervention is first. This means all four
Application: Concept – Perioperative.
(4) answer options could be possible ac-
tions but only one (1) is first. The test taker 113. 1. The client needing more pain medication
should use the nursing process and select indicates the client’s condition is getting
the option addressing assessment because it worse.
is the first step in the nursing process. 2. Coffee-ground material indicates old blood
Content – Surgical: Integrated Nursing Process – ­ from the gastrointestinal system.
Implementation: Client Needs – Safe Effective Care 3. Because the signs of peritonitis are el-
­Environment, Management of Care: Cognitive Level – evated temperature and rigid abdomen, a
Analysis: Concept – Perioperative. reversal of these signs indicates the client
is getting better.
111. 1. The client has a surgical incision, which
4. Two soft-formed bowel movements are nor-
­impairs the skin integrity, but it is not the
mal, but this does not have anything to do
priority because it is sutured under sterile
with peritonitis.
conditions.
2. After abdominal surgery, the body TEST-TAKING HINT: The -itis of peritonitis
­distributes fluids to the affected area as means inflammation, which is associated
part of the healing process. These fluids with an elevated temperature. A decrease
are shifted from the intravascular com- in temperature would be a sign the client is
partment to the interstitial space, which improving.
causes potential fluid and electrolyte Content – Surgical: Integrated Nursing Process –
imbalance. Assessment: Client Needs – Physiological Integrity, Reduc-
3. Bowel elimination is a problem, but after tion of Risk Potential: Cognitive Level – Analysis:
general anesthesia wears off, the bowel Concept – Inflammation.
sounds will return, and this is not a life- 114. 1. The client is NPO; therefore, no medication
threatening problem. would be administered.
4. Psychosocial problems are not priority over 2. The client is NPO so no food or fluids are
actual physiological problems. allowed.
TEST-TAKING HINT: When identifying prior- 3. Deep breathing will help prevent pulmonary
ity problems, the test taker can eliminate complications but does not address the cli-
any psychosocial problem as a potential ent’s paralytic ileus.
correct answer if there are applicable physi- 4. A paralytic ileus is the absence of peri-
ological problems. stalsis; therefore, the bowel will be
Content – Surgical: Integrated Nursing Process – unable to process any oral intake. A na-
Diagnosis: Client Needs – Safe Effective Care sogastric tube is inserted to decompress
Environment, Management of Care: Cognitive the bowel until surgical intervention or
Level – Analysis: Concept – Perioperative. until bowel sounds return spontaneously.

07_Colgrove_Ch07.indd 288 03/06/16 12:58 PM


C hapter 7  G astrointestinal D isorders 289

TEST-TAKING HINT: If the test taker realizes interventions. Temperature does not corre-
the stem of the question says part of the late with nausea. Medication may be admin-
gastrointestinal system, the ileus, is para- istered but it would be an antiemetic, not a
lyzed, the test taker should know allowing narcotic analgesic.
the client to take anything by mouth would Content – Surgical: Integrated Nursing Process – ­
be an inappropriate action, so options “1” Implementation: Client Needs – Safe Effective Care
and “2” could be eliminated. Deep breath- Environment, Management of Care: Cognitive Level –
ing addresses the respiratory system, not Synthesis: Concept – Perioperative.
the gastrointestinal system, so option “3” 117. 1. The nurse cannot force the client to do any-
could also be eliminated. thing; this would be considered assault.
Content – Surgical: Integrated Nursing Process – 2. There are no data to support the need for
­Implementation: Client Needs – Physiological Integrity, a chest x-ray.
Physiological Adaptation: Cognitive Level – Analysis:
3. Shallow respirations and refusal to deep
Concept – Perioperative.
breathe could be the result of abdominal
115. 1. The JP bulb should be depressed, which pain. The nurse should assess the client
indicates suction is being applied. A for pain and determine the last time the
round bulb indicates the bulb is full PCA pump was used.
and needs to be emptied and suction 4. Based on the information given, the client
reapplied. does not need oxygen.
2. The tube should be taped to the dressing to TEST-TAKING HINT: If the test taker is unsure
prevent accidentally pulling the drain out of of the answer, identifying key words in the
the insertion site. stem—“abdominal surgery” and “PCA”—
3. The insertion site should be pink and with- should guide the test taker to select an op-
out any signs of infection, which include tion related to one (1) of these key words.
drainage, warmth, and redness. “Determine” can be substituted for the
4. The JP bulb should be sunken in or word “assess,” which is the first step of the
­depressed, indicating suction is being nursing process.
applied.
Content – Surgical: Integrated Nursing Process – ­
TEST-TAKING HINT: The stem is asking which Implementation: Client Needs – Safe Effective Care
data need intervention by the nurse. Option ­Environment, Management of Care: Cognitive Level –
“2” can be ruled out because all tubes and ­Application: Concept – Perioperative.
drains should be secured. A pink insertion 118. 170 mL/hr.
site with no drainage is expected, which The N/G tube drainage of 45 mL must
would cause the test taker to eliminate op- be added to the 125 mL/hr IV rate, which
tion “3” as a possible correct answer. equals 170 (125 + 45 = 170). The nurse
Content – Surgical: Integrated Nursing Process – should infuse 170 mL in the next hour.
Assessment: Client Needs – Physiological Integrity, Reduc-
tion of Risk Potential: Cognitive Level – Synthesis: TEST-TAKING HINT: The stem states the pre-
Concept – Perioperative. vious hour’s N/G tube output plus the base-
line IV rate. The test taker must observe
116. 1. Medicating the client with an analgesic the key words in the stem. Don’t forget to
could increase the client’s nausea unless use the drop-down calculator when taking
the nausea is caused by pain. The nurse the NCLEX-RN.
should assess the etiology to determine the Content – Surgical: Integrated Nursing Process –
interventions. ­Implementation: Client Needs – Physiological Integrity,
2. A client who had abdominal surgery ­Pharmacological and Parenteral Therapies: Cognitive
usually has a nasogastric (N/G) tube in Level – Application: Concept – Perioperative.
place. If the N/G tube is not patent, this
will cause nausea. Irrigating the N/G 119. 1. A client who has not voided within four (4)
tube may relieve nausea. hours after any surgery is not priority. This
3. Checking the temperature will not treat the is an acceptable occurrence, but if the client
nausea. hasn’t voided for eight (8) hours, then the
4. Hyperextending the neck will assist the nurse should assess further.
­client to breathe but will not treat nausea. 2. A sudden cessation of pain may indicate
a ruptured appendix, which could lead to
TEST-TAKING HINT: Assessment is the
peritonitis, a life-threatening complica-
first step in the nursing process. Check- tion; therefore, the nurse should assess
ing the N/G tube for patency and taking this client first.
the temperature are the only assessment

07_Colgrove_Ch07.indd 289 03/06/16 12:58 PM


290 M ed -S urg S uccess

3. Bowel sounds should return within 24 hours TEST-TAKING HINT: The stem of the ques-
after abdominal surgery. Absent bowel tion provides information about the client’s
sounds at four (4) hours postoperative is not height and weight, and the test taker must
of great concern to the nurse. determine if this is important information.
4. The client being discharged is stable and not Information in the stem must be eliminated
a priority for the nurse. as not pertinent to the question or closely
TEST-TAKING HINT: The stem is asking which regarded to let the test taker know what the
client the nurse should see first. Therefore, question is asking.
the test taker should look for life-threatening Content – Medical: Integrated Nursing Process –
or serious complications or abnormal Assessment: Client Needs – Physiological Integrity,
assessment data for the disease process. Reduction of Risk Potential: Cognitive Level – Analysis:
Concept – Nutrition.
Content – Surgical: Integrated Nursing Process –
Assessment: Client Needs – Safe Effective Care Environ- 122. 1. Many clients jog one (1) to two (2) miles per
ment, Management of Care: Cognitive Level – Analysis: day as part of their exercise program. This
Concept – Perioperative. does not indicate bulimia.
120. 1. The last bowel movement does not help 2. Not gaining weight may be an end result of
identify the cause of the client’s right lower bulimia, but it does not identify bulimia.
abdominal pain. This might be appropriate 3. Bulimia is characterized by bingeing
for a client with left lower abdominal pain. and purging by inducing vomiting after
2. Information about a high-fat meal would be a meal. Stomach contents are acidic and
asked if the nurse suspected the client had a the acid wears away the enamel on the
gallbladder problem. teeth, leaving the teeth a green color.
3. An elderly client may experience a 4. The client has calluses on the knuckles from
ruptured appendix with minimal pain; pushing them into the throat to induce
therefore, the nurse should assess the vomiting.
characteristics of the pain. TEST-TAKING HINT: The question requires
4. The passage of flatus (gas) does not help deter- the nurse to be knowledgeable of the signs/
mine the cause of right lower abdominal pain. symptoms of bulimia. Vomiting should lead
TEST-TAKING HINT: The test taker should go the test taker to green teeth secondary to
back to basics and assess the client. hydrochloric acid.
Content – Medical: Integrated Nursing Process – ­ Content – Medical: Integrated Nursing Process –
Implementation: Client Needs – Safe Effective Care Evaluation: Client Needs – Safe Effective Care
Environment, Management of Care: Cognitive Level – ­Environment, Management of Care: Cognitive Level –
Application: Concept – Inflammation. Evaluation: Concept – Nutrition.

123. 1. Clients diagnosed with bulimia will eat the


entire meal and more food if available. This
Eating Disorders is not unusual behavior for a client diag-
nosed with bulimia.
121. 1. This client is 5'10" tall and weighs 99
2. By having someone stay with the client
pounds (45 kg 3 2.2 5 99). Menses will
for 45 minutes to one (1) hour after a
cease if the client is severely emaciated.
meal, the client will be prevented from
A 24-hour dietary recall is a step toward
inducing vomiting and ridding the body
assessing the client’s eating patterns.
of the meal before it can be metabolized.
2. The type of birth control could be asked, but
3. Clients diagnosed with anorexia nervosa
the client is asking about missing menstrual
tend to overexercise to prevent weight gain
periods. Birth control does not interfere with
and to lose imagined excess weight.
having a period; if anything, some forms
4. Bedrest is not needed for this client.
of birth control will make the cycles more
regular. TEST-TAKING HINT: The test taker must be
3. The nurse can look at the client and see able to differentiate between bulimia and
a very thin young woman, which should anorexia. It can be difficult to keep these
confirm more assessment is needed, not processes separate, especially because some
reweighing. clients have both anorexia and bulimia.
4. The pulse and blood pressure will not pro- Content – Medical: Integrated Nursing Process – ­
vide the nurse any information as to why Implementation: Client Needs – Safe Effective Care
the client’s menstrual cycles have ceased. ­Environment, Management of Care: Cognitive Level –
Application: Concept – Nutrition.

07_Colgrove_Ch07.indd 290 03/06/16 12:58 PM


C hapter 7  G astrointestinal D isorders 291

124. 1. The goal is written in terms of client behav- 4. The client’s entire body will be involved in
ior; this option is a nursing intervention, not the process as a result of malnutrition, but
a client goal. bone density tests are not done.
2. Eating 50% of meals provided does not TEST-TAKING HINT: The test taker needs to
­address low self-esteem. be aware of the complications associated
3. High-protein shakes are a dietary intervention. with specific disease processes.
4. The problem of “low self-esteem” re-
Content – Medical: Integrated Nursing Process –
quires the client to verbalize psycho- Diagnosis: Client Needs – Physiological Integrity,
social feelings. Identifying one positive ­Reduction of Risk Potential: Cognitive Level – Analysis:
attribute is an appropriate goal. Concept – Nutrition.
TEST-TAKING HINT: The test taker could
127. 1. The client does not owe the nurse an
eliminate distracter “3” as a health-care dis-
explanation.
cipline intervention. Psychosocial problems
2. If the HCP determines it is safe for the cli-
require goals addressing a nonphysiological
ent to exercise, a gymnasium might be rec-
need.
ommended, but walking is the best exercise,
Content – Medical: Integrated Nursing Process – ­ and this can be done in the neighborhood or
Diagnosis: Client Needs – Physiological Integrity, at an enclosed shopping mall.
­Physiological Adaptation: Cognitive Level – Analysis:
3. The client should set realistic weight loss
Concept – Nutrition.
goals. A realistic weight loss goal is one (1) to
125. 1. The client is 67 inches tall (5'7'') and one and one-half (11/2) pounds per week, but
weighs 88 pounds (40 kg 3 2.2 5 88). this should be done after assessing the client.
This client is severely underweight and 4. Determining the client’s eating patterns
nutrition is the priority. and what triggers the client to eat—stress
2. Clients with anorexia have a chronic low or boredom, for example—and where and
self-esteem problem, but this is a psychoso- when the client consumes most of the
cial problem and actual physical problems calories—snacking in front of the TV at
are priority. night, for example—is needed to assist
3. Disturbed body image is a psychosocial the client to change eating behaviors.
problem manifested in a physical one. The TEST-TAKING HINT: This question is an ex-
physical problem is priority; this would be ample of using the nursing process to arrive
an appropriate long-term goal. at the correct answer. Assessing the client
4. This client thinks her body is not appeal- has priority.
ing and this could be a problem, but it is a
Content – Medical: Integrated Nursing Process –
­psychosocial issue and not priority. ­Implementation: Client Needs – Physiological Integrity,
TEST-TAKING HINT: The test taker must de- Basic Care and Comfort: Cognitive Level – Application:
cide which problem is priority when all the Concept – Nutrition.
problems could apply to the client. Unless
128. 1. The morbidly obese client will have a
the client is considering suicide and has a
large abdomen, preventing the lungs
plan to carry it out, physical problems are
from expanding, which predisposes the
priority.
client to respiratory complications.
Content – Medical: Integrated Nursing Process – 2. The client may be weighed daily, but this is
Diagnosis: Client Needs – Physiological Integrity, not priority.
­Physiological Adaptation: Cognitive Level – Analysis:
3. The client should be taught proper nutrition
Concept – Nutrition.
for weight loss, but this is not the priority in
126. 1. The client diagnosed with anorexia will have the immediate postoperative period.
muscle tissue wasting; liver function tests 4. This is very important for the long term, but
will not monitor for this. respiratory status is priority.
2. Kidney function tests will not monitor nu- TEST-TAKING HINT: Regardless of the pro-
trition or muscle wasting. cedure or the size of the client, respiratory
3. The heart is a muscle; in severe an- status is priority in the immediate postop-
orexia (more than 60% under ideal body erative period. The test taker should apply
weight), muscle tissue is catabolized Maslow’s hierarchy of needs.
to provide energy to the body. The cli-
Content – Surgical: Integrated Nursing Process – ­
ent is at risk for death from cardiac Implementation: Client Needs – Safe Effective Care
complications. ­Environment, Management of Care: Cognitive Level –
Synthesis: Concept – Perioperative.

07_Colgrove_Ch07.indd 291 03/06/16 12:58 PM


292 M ed -S urg S uccess

129. 1. Exercise recommendations for weight loss 3. The client is 36 years old and has the right
are to exercise for 30 minutes at least three to refuse to eat, even to the detriment of her
(3) times per week, not three (3) times a day. body. Restraining the client could be consid-
2. The client should be aware of situations ered assault unless the psychiatric team had
triggering the consumption of food a court order.
when the client is not hungry, such as 4. “Might cause physical problems” is a
anger, boredom, and stress. Food-seeking factual statement to the client about
behaviors are not associated only with the possible results if the client refuses
hunger in the client who is obese. nourishment.
3. The client should weigh himself or herself TEST-TAKING HINT: The test taker could
about once a week. If weight loss is not ob- eliminate answer option “1” based on “why.”
served, the client becomes discouraged and Option “2” is not therapeutic or factual
feels powerless to control the weight. This information.
can lead to diet failure.
Content – Medical: Integrated Nursing Process –
4. Sodium is limited in clients with hyperten- ­Implementation: Client Needs – Psychosocial Integrity:
sion, not obesity. Cognitive Level – Application: Concept – Nutrition.
5. Weight loss support groups such as
Weight Watchers or Take Off Pounds Sen- 132. 1. Clients diagnosed with anorexia exercise
sibly (TOPS) are helpful to keep the client ­excessively; clients diagnosed with bulimia
participating in a weight loss program. do not.
2. Clients diagnosed with bulimia frequently
TEST-TAKING HINT: This is an alternate-type
take cathartic laxatives to prevent absorp-
question. The test taker must judge each
tion of calories from the food consumed.
answer option for itself; one (1) option does
3. Clients diagnosed with bulimia and anorexia
not eliminate another. The NCLEX-RN
have low self-esteem. The client feels ugly
gives credit for the entire question. The test
or unlovable if he or she is overweight (by
taker must identify all the right answers or
his or her perception).
the answer will be counted as incorrect.
4. High-fiber foods do help the body to pro-
Content – Medical: Integrated Nursing Process – duce larger stools, but this client would use a
Planning: Client Needs – Physiological Integrity,
cathartic laxative.
­Physiological Adaptation: Cognitive Level – Synthesis:
Concept – Nutrition. TEST-TAKING HINT: The test taker must dis-
tinguish between bulimia and anorexia to
130. 1. Jogging is not an appropriate exercise for a answer this question. Clients with anorexia
client who is obese: there is too much stress are usually underweight, whereas clients
on the heart and joints. with bulimia may be of a normal or slightly
2. If lifestyle behaviors such as patterns larger size.
of eating and daily exercise are not modi-
Content – Medical: Integrated Nursing Process –
fied, the client who loses weight will re-
Assessment: Client Needs – Safe Effective Care Environ-
gain the weight and usually more. ment, Management of Care: Cognitive Level – Analysis:
3. The client should eat frequent small meals Concept – Nutrition.
during the day to keep from being hungry.
Breakfast should not be skipped.
4. Diets containing fewer than 1,200 calories Constipation/Diarrhea Disorders
per day need to be supplemented with a
multivitamin to provide the body with the 133. 1. An antidiarrheal medication would slow
nutrients needed to stay healthy. down the peristalsis in the colon, worsening
TEST-TAKING HINT: The test taker could the problem.
eliminate answer option “4” because health- 2. The client has an immediate need to evacu-
care professionals should not discourage ate the bowel, not a need for bowel training.
health promotion activities. 3. Oil retention enemas will help to soften
Content – Medical: Integrated Nursing Process – ­
the feces and evacuate the stool.
Planning: Client Needs – Physiological Integrity, 4. A UGI series adds barium to the already
­Physiological Adaptation: Cognitive Level – Synthesis: hardened stool in the colon. Barium enemas
Concept – Nutrition. x-ray the colon; a UGI series x-rays the
stomach and jejunum.
131. 1. The client does not have to explain her actions
TEST-TAKING HINT: If the test taker under-
to the nurse; the nurse should not ask “why.”
2. Telling the client she is skinny is belittling stands fecal impaction is the opposite of
the client. diarrhea, then answer option “1” can be

07_Colgrove_Ch07.indd 292 03/06/16 12:58 PM


C hapter 7  G astrointestinal D isorders 293

eliminated. Knowledge of anatomy and ­Physiological Adaptation: Cognitive Level – Synthesis:


physiology eliminates option “4” because Concept – Bowel Elimination.
stool is formed in the colon and transported 136. 1. This is a symptom of diarrhea moving
to the anus, part of the lower gastrointesti- around an impaction higher up in the
nal tract. colon. The nurse should assess for an
Content – Medical: Integrated Nursing Process – ­ ­impaction when observing this finding.
Implementation: Client Needs – Safe Effective Care 2. Encouraging the client to drink fluids
Environment, Management of Care: Cognitive Level – should be done, but it is not the first
Application: Concept – Bowel Elimination.
intervention.
134. 1. Bananas are encouraged for clients with 3. The sodium level is usually not a problem
potassium loss from diuretics; a banana is for clients experiencing diarrhea, but the
not needed for harsh laxative (cathartic) use. potassium level may be checked. However,
Harsh laxatives should be discouraged be- again, this is not the first intervention.
cause they cause laxative dependence and a 4. A protective cream can be applied to an ex-
narrowing of the colon with long-term use. coriated perineum, but first the nurse should
2. It is not necessary to have a bowel move- assess the situation.
ment every day to have normal bowel TEST-TAKING HINT: The first step of the
functioning. nursing process is assessment, after which a
3. Limiting fluids will increase the problem; nursing diagnosis and interventions follow.
the client should be encouraged to increase The nurse should assess first.
the fluids in the diet. Content – Medical: Integrated Nursing Process – ­
4. If the client is feeling “sluggish” from not Implementation: Client Needs – Safe Effective Care
being able to have a bowel movement, these ­Environment, Management of Care: Cognitive Level –
foods increase constipation because they are Application: C
­ oncept – Bowel Elimination.
low in residue (fiber).
137. 1. This client is improving; semiformed stools
TEST-TAKING HINT: The test taker must un- are better than diarrhea.
derstand words such as “cathartic.” Limit- 2. This client has just arrived, so the nurse
ing fluids is used for clients in renal failure does not know if the complaint is valid
or congestive heart failure, but increas- and needs intervention unless assessed.
ing fluids is recommended for most other The elderly have difficulty with consti-
conditions. pation as a result of decreased gastric
Content – Medical: Integrated Nursing Process – motility, medications, poor diet, and
Evaluation: Client Needs – Physiological Integrity, immobility.
­
Physiological Adaptation: Cognitive Level – Evaluation: 3. The client has diarrhea, but only 200 mL,
Concept – Bowel Elimination.
and has elastic tissue turgor, indicating the
135. 1. Blood may indicate a hemorrhoid, but it is client is not dehydrated.
not normal to expel blood when having a 4. This is not normal, but it is expected for a
bowel movement. client with hemorrhoids.
2. Nurses manually remove feces; it is not a TEST-TAKING HINT: The test taker should
self-care activity. notice descriptive words such as “elderly,”
3. Cathartic use on a daily basis creates de- which should alert the test taker to the age
pendence and a narrowing of the lumen of range having an implication in answering
the colon, creating a much more serious the question. Answer options “3” and “4”
problem. are expected for the disease processes.
4. A high-fiber (residue) diet provides Content – Medical: Integrated Nursing Process –
bulk for the colon to use in remov- Assessment: Client Needs – Safe Effective Care
ing the waste products of metabolism. ­Environment, Management of Care: Cognitive Level –
Bulk ­laxatives and fiber from vegetables Analysis: Concept – Bowel Elimination.
and bran assist the colon to work more
effectively. 138. 1. Cheeseburgers and milk shakes are low-­
residue foods and can make constipation
TEST-TAKING HINT: Blood is not normal in worse.
any circumstance. It may be expected but is 2. Canned peaches are soft and can be
not “normal” unless inside a vessel. chewed and swallowed easily while pro-
Content – Medical: Integrated Nursing Process – ­ viding some fiber; whole-wheat bread is
Planning: Client Needs – Physiological Integrity, higher in fiber than white bread. These

07_Colgrove_Ch07.indd 293 03/06/16 12:58 PM


294 M ed -S urg S uccess

foods will be helpful for clients whose judgment, assessment, teaching, or


gastric motility is slowed as a result of evaluation.
lack of exercise or immobility. Content – Medical: Integrated Nursing Process –
3. Mashed potatoes and mechanically ground Planning: Client Needs – Safe Effective Care Environ-
meat do not provide high fiber. ment, Management of Care: Cognitive Level – Synthesis:
4. Biscuits, gravy, and bacon are refined flour Concept – Nursing Roles.
foods or processed meat (fat). These will not 141. 1. The client is tolerating the feeding change,
help clients to prevent constipation. so there is no need for an immediate action.
TEST-TAKING HINT: The test taker must 2. The client has a PEG tube inserted into the
­realize the consequences of immobility stomach through the abdominal wall. The
i­ nclude constipation. client does not have a nasogastric feeding
Content – Medical: Integrated Nursing Process – tube.
­Implementation: Client Needs – Physiological Integrity, 3. Complaints of being thirsty should be ad-
Basic Care and Comfort: Cognitive Level – Application: dressed; the client may require some ice
Concept – Bowel Elimination. chips in the mouth or oral care, but this is
139. 1. It is important to keep track of the not priority over assessing the client’s ability
amounts, color, and other characteristics to swallow.
of body fluids excreted. 4. This client needs to be cleaned immedi-
2. Skin turgor should be assessed at least every ately, the abdomen must be assessed, and
six (6) to eight (8) hours, not daily. a determination must be made regarding
3. Carbonated soft drinks increase flatus in the the type of feeding and the additives and
GI tract, and the increased sugar will act as medications being administered and skin
an osmotic laxative and increase diarrhea. damage occurring. This occurrence is
4. Daily weights are the best method of priority.
­determining fluid loss and gain. TEST-TAKING HINT: The test taker must
5. Sitz baths will assist in keeping the identify assessment data indicating a com-
­client’s perianal area clean without plication secondary to the disease process
­having to rub. The warm water is when the stem asks which occurrence war-
­soothing, providing comfort. rants immediate intervention.
TEST-TAKING HINT: The test taker should Content – Medical: Integrated Nursing Process –
note the time frame for any answer option. Assessment: Client Needs – Safe Effective Care
“Every day” is not often enough to assess Environment, Management of Care: Cognitive
for dehydration in a client who is experienc- Level – Synthesis: Concept – Nutrition.
ing massive (“voluminous”) fluid loss. If the 142. 1. This client may have developed an infec-
test taker were not aware of the definition, tion from the undercooked meat. The
then an associated word, “volume,” would nurse should obtain a stool specimen for
be a hint. the laboratory to analyze.
Content – Medical: Integrated Nursing Process – ­ 2. Antibiotic therapy is initiated in only the
Implementation: Client Needs – Safe Effective Care most serious cases of infectious diarrhea; the
Environment, Management of Care: Cognitive Level – diarrhea must be assessed first. A specimen
Application: Concept – Bowel Elimination. for culture should be obtained before begin-
140. 1. The nurse will be responsible for signing ning medication.
off on the UAP as to being competent to ­ 3. A complete blood count will provide an
perform the blood glucose. The nurse estimate of blood loss, but it is not the first
should do this to determine the competency intervention.
of the UAP. 4. An antidiarrheal medication would be
2. The laboratory values may require the nurse ­administered after the specimen
to interpret and act on the results. The nurse collection.
cannot delegate tasks requiring professional TEST-TAKING HINT: All options in a priority-
judgment. setting question may be interventions
3. The LPN can administer medications the nurse could implement, but the right
such as a laxative. answer will be the one (1) implemented
4. The nurse cannot delegate assessment. first. Collecting a stool sample is assess-
TEST-TAKING HINT: Nurses cannot del- ment, which is the first step in the nursing
egate any activity requiring professional process.

07_Colgrove_Ch07.indd 294 03/06/16 12:58 PM


C hapter 7  G astrointestinal D isorders 295

Content – Medical: Integrated Nursing Process – ­ range, indicating lungs are trying to retain
Implementation: Client Needs – Safe Effective Care acid to help correct the pH but the pH is
Environment, Management of Care: Cognitive Level – not in normal range (partial compensa-
Synthesis: Concept – Bowel Elimination. tion). The N/G tube could be removing
143. 1. The clinic nurse should not ask the client to too much acid from the stomach. The
measure stool at home; this is done in the nurse should assess the NGT output.
acute care setting. 2. This is a metabolic problem, not respiratory.
2. Unless the client has had diarrhea for longer 3. This is a metabolic problem, not respiratory.
than 48 hours, the client does not need to be 4. The client has an NGT removing acidic
seen in the clinic. contents from the stomach; stool specimen
3. The BRAT (bananas, rice, applesauce, or observation is not indicated.
and toast) diet is recommended for a TEST-TAKING HINT: The nurse must be
client with diarrhea because it is low able to interpret common laboratory re-
residue and produces nutrition while not sults. Part of the assessment of a symptom
irritating the GI system. ­requires determining what therapies can
4. Histamine-2 blockers decrease gastric acid ­impact the result.
production and would not be prescribed for Content – Medical: Integrated Nursing Process – ­
a client with diarrhea. Assessment: Client Needs – Physiological Integrity,
TEST-TAKING HINT: The test taker should ­Reduction of Risk Potential: Cognitive Level – Synthesis:
realize diarrhea involves the gastrointestinal Concept – pH Imbalance.
system and selecting an intervention ad- 146. 1. The client should remain NPO until the
dressing the GI system would be an appro- inflammation in the colon resolves.
priate choice. 2. The client should have an IV to maintain
Content – Medical: Integrated Nursing Process – hydration while being NPO.
Diagnosis: Client Needs – Physiological Integrity, 3. The nurse should assess the client for
­Physiological Adaptation: Cognitive Level – Analysis: complications of a ruptured diverticulum.
Concept – Bowel Elimination. 4. The client will be NPO to rest the bowel.
144. 1. Normal serum sodium levels are 135 to 5. The client is kept on bedrest with bath-
152 mEq/L, so the client’s 128 mEq/L room privileges to decrease colon activ-
value requires intervention. ity. Ambulation increases peristalsis.
2. The client diagnosed with a fecal impaction TEST-TAKING HINT: The test taker must
is beginning to move the stool; this indicates make decisions based on basic principles. If
an improvement. the client has a diagnosis of a gastrointes-
3. Normal potassium levels are 3.5 to tinal disease, the treatment will depend on
5.5 mEq/L. A level of 3.8 mEq/L is allowing the bowel to rest. When choosing
within normal limits and does not require options for a “select all that apply” ques-
intervention. tion, each option is treated as a true/false
4. This client has been having diarrhea and question.
now is having semiliquid stools, so this client Content – Medical: Integrated Nursing Process –
is getting better. ­Implementation: Client Needs – Physiological Integrity,
TEST-TAKING HINT: The test taker must ­Reduction of Risk Potential: Cognitive Level – Analysis:
determine if the client is experiencing a Concept – Nutrition.
potentially life-threatening complication, 147. 1. The nurse should assess the client’s
such as potential for seizures. Answer op- ­intake; a 72-hour calorie count will allow
tions “2,” “3,” and “4” are expected for the the nurse to do this.
disease process and are normal or show 2. Protein calorie malnutrition can result
improvement. from several different diseases. Diarrhea
Content – Medical: Integrated Nursing Process – can impart the ability to absorb calories
­Implementation: Client Needs – Physiological Integrity, and nutrition from food.
Reduction of Risk Potential: Cognitive Level – Synthesis: 3. Daily weights will monitor the client’s
Concept – Bowel Elimination. weight loss or gain.
145. 1. This blood gas indicates metabolic al- 4. The nurse should assess medications for
kalosis with partial compensation. The drug and food interactions.
pH level is high, indicating alkalosis. The 5. The dietitian can be invaluable in assist-
HCO3 level is low, indicating an alkalosis ing this client to gain or at least maintain
problem. The Paco2 is outside the normal weight.

07_Colgrove_Ch07.indd 295 03/06/16 12:58 PM


296 M ed -S urg S uccess

TEST-TAKING HINT: To answer “select all that 4. This client needs a referral to a social
a­ pply” questions, the test taker must read worker. The client is not the first one to
each option as a true/false question. One ­receive a return call.
option does not rule out another one. TEST-TAKING HINT: The nurse must be able
Content – Medical: Integrated Nursing Process – to interpret implications of disease pro-
­Implementation: Client Needs – Physiological Integrity, cesses and comorbid conditions (vomiting).
­Reduction of Risk Potential: Cognitive Level – Synthesis: Part of the assessment of a symptom re-
Concept – Nutrition.
quires determining what other diseases can
12.06 BMI. This client is extremely
148. impact the result.
underweight. Content – Medical: Integrated Nursing Process – ­
To figure the BMI the test taker must first Assessment: Client Needs – Physiological Integrity,
multiply the height in inches times the Reduction of Risk Potential: Cognitive Level –
height in inches. This client is 67 inches tall. ­Synthesis: Concept – pH Imbalance.
67 × 67 = 4489 151. 1. This is a gastrointestinal complaint, not a
The next step is to divide the weight in respiratory virus.
pounds by the sum of the height times the 2. Antibacterial soap will not affect a virus.
height: A virus is not a bacterium.
35 × 2.2 = 77 pounds 3. A hepatitis vaccine prevents hepatitis but
77 ÷ 4489 = 0.01715 this is a gastrointestinal viral illness.
Then multiply this number times the 4. Hand washing will prevent the spread
­conversion of 703: of the virus and decrease the risk for the
0. 01715 × 703 = 12.06 employees.
TEST-TAKING HINT: The nurse must be able TEST-TAKING HINT: The test taker should
to work common math problems to deter- ­remember basic infection control standards.
mine the client needs.
Content – Medical: Integrated Nursing Process – ­
Content – Medical: Integrated Nursing Process – Planning: Client Needs – Physiological Integrity,
Assessment: Client Needs – Pharmacological/Mathematics: ­Reduction of Risk Potential: Cognitive Level – Synthesis:
Cognitive Level – Analysis: Concept – Nutrition. Concept – Nutrition.
149. 1. Sodium is retained by the body before po- 152. 1. This blood gas is metabolic acidosis, a
tassium. The sodium level would be in the potentially lethal situation. The nurse
normal range after two (2) days of diarrhea. should notify the HCP immediately.
2. Albumin is the protein level synthesized by 2. These results for sodium and potassium are
the liver; the albumin level should not be WNL. The glucose is at the edge of the
affected. normal.
3. Potassium is excreted through diarrhea; 3. The results of the hemoglobin and hemato-
the nurse should assess the client’s po- crit (H&H) are not in the range to require
tassium level. more blood at this time. The nurse can give
4. The glucose should not be affected. the results to the HCP on rounds.
TEST-TAKING HINT: The nurse must be able 4. This pulse oximetry reading is WNL.
to interpret common laboratory results and TEST-TAKING HINT: The nurse must be able
how they affect the body. Part of the assess- to interpret common laboratory results.
ment of a symptom requires determining Part of the assessment of a symptom re-
what therapies can impact the result. quires knowing the normal ranges.
Content – Medical: Integrated Nursing Process – Content – Medical: Integrated Nursing Process –
Assessment: Client Needs – Physiological Integrity, Assessment: Client Needs – Physiological Integrity,
­Reduction of Risk Potential: Cognitive Level – Synthesis: ­Reduction of Risk Potential: Cognitive Level – Synthesis:
Concept – Nutrition. Concept – pH Imbalance.
150. 1. A three (3) on the pain scale indicates mild 153. 1. The usual diet is not a concern at this time,
pain. This would not be the first client for when the child goes home the nurse will
the nurse to return the call. need to explain what the parents can offer
2. A prescription refill would not be a reason the child to prevent dehydration.
for the nurse to make this client first. 2. Assessing the skin turgor will give the
3. A client who has diabetes type 1 and is nurse information about the hydration
vomiting is at risk for diabetes ketoaci- status of the toddler. The nurse should
dosis. The nurse should have the client
come in immediately.

07_Colgrove_Ch07.indd 296 03/06/16 12:58 PM


C hapter 7  G astrointestinal D isorders 297

perform an assessment based on the 155. 1. The BMI is not low; these symptoms are
p
­ resenting symptoms. ­associated with anorexia.
3. Suckers are sugar based and rarely given to 2. The BMI is WNL but does not indicate
children for “good behavior” anymore. therapy is effective.
4. The nurse should perform his/her own as- 3. The patient diagnosed with bulimia
sessment prior to notifying the HCP to see binge eats and then induces vomiting or
the child. uses laxatives to prevent weight gain.
TEST-TAKING HINT: The nurse must assess 4. The BMI is WNL.
the c­ lient. This is a professional responsibil- TEST-TAKING HINT: The test taker will
ity regardless of other health-care profes- be ­required to interpret graphs on the
sionals and their responsibilities. NCLEX-RN and associate the graph infor-
Content – Medical: Integrated Nursing Process – ­ mation with disease processes.
Assessment: Client Needs – Physiological Integrity, Content – Medical: Integrated Nursing Process –
­Reduction of Risk Potential: Cognitive Level – Synthesis: Assessment: Client Needs – Physiological Integrity,
Concept – Nutrition. ­Reduction of Risk Potential: Cognitive Level – Synthesis:
Concept – Nutrition.
154. 1. Knowledge of previous weight loss
­attempts will assist in planning a weight 156. 1. Weighing the food is no longer recom-
loss program. mended but if the client weighs the food
2. The dietitian will monitor the nutritional then it should be weighed after cooking be-
intake and help in planning a nutrition- cause cooking changes the weight of food.
ally balanced diet. 2. The client should eat some carbohydrates
3. The client should be encouraged to main- but also protein foods to maintain the blood
tain an active lifestyle. glucose levels.
4. The client’s weight will be useful in 3. The client can estimate the amount of
­determining the client’s progress. carbohydrates being consumed by using
5. Clients who desire weight loss frequently the fist as a guideline. The fist is always
want a quick fix. The nurse should as- with the client and is an unobtrusive
sist the client to determine a consistent method of determining the amount of
weight loss goal in order to achieve food and calories being ingested.
behavior modification to maintain the 4. Seventy-five percent of the diet being pro-
weight loss. tein places a great burden on the kidneys
TEST-TAKING HINT: The test taker must and can result in acidosis. This is unhealthy.
read each option of a “select all that apply” TEST-TAKING HINT: The nurse must be able
­question as a true/false question. to provide basic teaching for diets.
Content – Medical: Integrated Nursing Process – Content – Medical: Integrated Nursing Process – ­
­Implementation: Client Needs – Physiological Integrity, ­ Planning: Client Needs – Physiological Integrity,
Reduction of Risk Potential: Cognitive Level – Synthesis: ­Reduction of Risk Potential: Cognitive Level – Synthesis:
Concept – Nutrition. Concept – Nutrition.

07_Colgrove_Ch07.indd 297 03/06/16 12:58 PM


298 M ed -S urg S uccess

GASTROINTESTINAL DISORDERS
COMPREHENSIVE EXAMINATION
1. The nurse is caring for the client with Clostridium 6. The client is diagnosed with ulcerative colitis.
difficile. Which intervention should the nurse Which sign/symptom warrants immediate
implement to prevent health-care associated intervention by the nurse?
infection (HAI) spread to other clients? 1. The client has 20 bloody stools a day.
1. Wash hands with Betadine for two (2) minutes 2. The client’s oral temperature is 99.8°F.
after giving care. 3. The client’s abdomen is hard and rigid.
2. Wear nonsterile gloves when handling GI 4. The client complains of urinating when
excretions. coughing.
3. Clean the perianal area with soap and water
7. Which outcome should the nurse identify for
after each stool.
the client diagnosed with aphthous stomatitis?
4. Flush the commode twice when disposing of
1. The client will be able to cope with perceived
stool.
stress.
2. The client receiving antibiotic therapy complains 2. The client will consume a balanced diet.
of white, cheesy plaques in the mouth. Which 3. The client will deny any difficulty swallowing.
intervention should the nurse implement? 4. The client will take antacids as prescribed.
1. Notify the health-care provider to obtain an
8. The nurse is administering a proton pump
antifungal medication.
inhibitor to a client diagnosed with peptic ulcer
2. Explain the patches will go away naturally in
disease. Which statement supports the rationale
about two (2) weeks.
for administering this medication?
3. Instruct to rinse the mouth with diluted
1. It prevents the final transport of hydrogen
hydrogen peroxide and water daily.
ions into the gastric lumen.
4. Allow the client to verbalize feelings about
2. It blocks receptors controlling hydrochloric
having the plaques.
acid secretion by the parietal cells.
3. Which instruction should be discussed with the 3. It protects the ulcer from the destructive
client diagnosed with gastroesophageal reflux action of the digestive enzyme pepsin.
disease (GERD)? 4. It neutralizes the hydrochloric acid secreted
1. Eat a low-carbohydrate, low-sodium diet. by the stomach.
2. Lie down for 30 minutes after eating.
9. Which task should the nurse delegate to
3. Do not eat spicy foods or acidic foods.
the unlicensed assistive personnel (UAP) to
4. Drink two (2) glasses of water before bedtime.
improve the desire to eat in a 14-year-old client
4. Which data should the nurse report to the health- diagnosed with anorexia?
care provider when assessing the oral cavity of an 1. Administer an antiemetic 30 minutes before
elderly client? the meal.
1. The client’s tongue is rough and beefy red. 2. Provide mouth care with lemon-glycerin
2. The client’s tonsils are +1 on a grading scale. swabs prior to the meal.
3. The client’s mucosa is pink and moist. 3. Create a social atmosphere by interacting with
4. The client’s uvula rises with the mouth open. the client.
4. Encourage the client’s parents to sit with the
5. Which complaint is significant for the nurse to
client during meals.
assess in the adolescent male client who uses oral
tobacco? 10. The client with a diagnosis of rule-out colon
1. The client complains of clear to white sputum. cancer is two (2) hours post–sigmoidoscopy
2. The client has an episodic blister on the procedure. Which assessment data warrant
upper lip. immediate intervention by the nurse?
3. The client complains of a nonhealing sore in 1. The client has hyperactive bowel sounds.
the mouth. 2. The client is eating a hamburger the family
4. The client has bilateral ducts at the second brought.
molars. 3. The client is sleepy and wants to sleep.
4. The client’s BP is 96/60 and apical pulse
is 108.

07_Colgrove_Ch07.indd 298 03/06/16 12:58 PM


C hapter 7  G astrointestinal D isorders 299

11. The nurse identifies the client problem 16. The client diagnosed with a hiatal hernia
“alteration in gastrointestinal system” for the is scheduled for a laparoscopic Nissen
elderly client. Which statement reflects the most fundoplication. Which statement indicates the
appropriate rationale for this problem? nurse’s teaching is effective?
1. Elderly clients have the ability to chew food 1. “I will have four (4) to five (5) small incisions.”
more thoroughly with dentures. 2. “I will be in the hospital for at least one
2. Elderly clients have an increase in digestive (1) week.”
enzymes, which helps with digestion. 3. “I will not have any pain because this is
3. Elderly clients have an increased need for laparoscopic surgery.”
laxatives because of a decrease in bile. 4. “I will be returning to work the day after my
4. Elderly clients have an increase in bacteria in surgery.”
the GI system, resulting in diarrhea.
17. The client is diagnosed with esophageal
12. Which interventions should the nurse discuss diverticula. Which lifestyle modification should
regarding prevention of an acute exacerbation of be taught by the nurse?
diverticulosis? Select all that apply. 1. Raise the foot of the bed to 45 degrees to
1. Eat a low-fiber diet. increase peristalsis.
2. Drink 2,500 mL of water daily. 2. Eat the evening meal at least two (2) hours
3. Avoid eating foods with seeds. prior to bed.
4. Walk 30 minutes a day. 3. Eat a low-fat, low-cholesterol, high-fiber diet.
5. Take an antacid every two (2) hours. 4. Wear an abdominal binder to strengthen the
abdominal muscles.
13. The clinic nurse is caring for a client who is
67 inches tall and weighs 100 kg. The client 18. The client weighs 160 pounds and is 5'1'' tall.
complains of occasional pyrosis, which resolves Calculate the body mass index (BMI) using the
with standing or with taking antacids. Which following formula:
treatment should the nurse expect the HCP to BMI 703 × weight in pounds
order?
(height in inches)2
1. Place the client on a weight loss program.
2. Instruct the client to eat three (3) balanced 19. Based on the client’s body mass index (BMI)
meals. from Question 18, which category should the
3. Tell the client to take an antiemetic before nurse document in the client’s medical records?
each meal. 1. Underweight.
4. Discuss the importance of decreasing alcohol 2. Overweight.
intake. 3. Ideal weight.
4. Obese.
14. The client is being prepared for discharge
after a laparoscopic cholecystectomy. Which Body Mass Index
intervention should the nurse implement?
1. Discuss the need to change the abdominal BMI Lower BMI Upper
dressing daily. Category Range Range
2. Tell the client to check the T-tube output
Underweight Greater than 19 —
every eight (8) hours.
3. Include the significant other in the discharge Ideal weight 19 24.9
teaching. Overweight 25 30
4. Instruct the client to stay off clear liquids for Obese 30.1 —
two (2) days.
20. Which intervention should the nurse implement
15. Which information should the nurse teach the
specifically for the client in end-stage
client post–barium enema procedure?
liver failure who is experiencing hepatic
1. The client should not eat or drink anything
encephalopathy?
for four (4) hours.
1. Assess the client’s neurological status.
2. The client should remain on bedrest until the
2. Prepare to administer a loop diuretic.
sedative wears off.
3. Check the client’s stool for blood.
3. The client should take a mild laxative to help
4. Assess for an abdominal fluid wave.
expel the barium.
4. The client will have normal elimination color
and pattern.

07_Colgrove_Ch07.indd 299 03/06/16 12:58 PM


300 M ed -S urg S uccess

21. Which priority teaching information should the 27. The nurse is assessing the client in end-stage
nurse discuss with the client to help prevent liver failure who is diagnosed with portal
contracting hepatitis B? hypertension. Which intervention should the
1. Explain the importance of good hand washing. nurse include in the plan of care?
2. Recommend the client take the hepatitis B 1. Assess the abdomen for a tympanic wave.
vaccine. 2. Monitor the client’s blood pressure.
3. Tell the client not to ingest unsanitary food or 3. Percuss the liver for size and location.
water. 4. Weigh the client twice each week.
4. Discuss how to implement Standard
28. The nurse is caring for the client diagnosed with
Precautions.
ascites secondary to hepatic cirrhosis. Which
22. The emergency department nurse is working information should the nurse report to the
in a community hospital. During the past two health-care provider?
(2) hours, 15 clients have been admitted with 1. A decrease in the client’s daily weight of one
Salmonella food poisoning. Which information (1) pound.
should the nurse discuss with the clients? 2. An increase in urine output after
1. Explain the incubation period is 48 to 72 hours. administration of a diuretic.
2. Explain the source of this poisoning is 3. An increase in abdominal girth of two (2)
contaminated water. inches.
3. Explain the sources of contamination are eggs 4. A decrease in the serum direct bilirubin to 0.6
and chicken. mg/dL.
4. Explain the bacterial contaminant came from
29. The nurse is caring for the client diagnosed
canned foods.
with hepatic encephalopathy. Which sign and
23. Which intervention should the nurse include symptom indicate the disease is progressing?
when discussing ways to prevent food 1. The client has a decrease in serum ammonia
poisoning? level.
1. Wash hands for 10 seconds after handling raw 2. The client is not able to circle choices on the
meat. menu.
2. Clean all cutting boards between meats and 3. The client is able to take deep breaths as
fruits. directed.
3. Maintain food temperatures at 1408F during 4. The client is able to eat previously restricted
extended servings. food items.
4. Explain fruits do not require washing prior to
30. The client is scheduled for a colostomy
eating or preparing.
secondary to colon cancer, and the surgeon tells
24. Which problem is most appropriate for the the client the stool will be a formed consistency.
nurse to identify for the client with diarrhea? Where would the nurse teach the client the
1. Alteration in skin integrity. stoma will be located?
2. Chronic pain perception.
3. Fluid volume excess. B
4. Ineffective coping. C

25. The nurse is assessing a client complaining A


of abdominal pain. Which data support the
diagnosis of a bowel obstruction? D
1. Steady, aching pain in one specific area.
2. Sharp back pain radiating to the flank.
3. Sharp pain increases with deep breaths. 1. A
4. Intermittent colicky pain near the umbilicus. 2. B
26. The nurse is caring for the client scheduled for 3. C
an abdominal perineal resection for Stage IV 4. D
colon cancer. Which client problem should the
nurse include in the intraoperative care plan?
1. Fluid volume deficit.
2. Impaired tissue perfusion.
3. Infection of surgical site.
4. Risk for immunosuppression.

07_Colgrove_Ch07.indd 300 03/06/16 12:58 PM


C hapter 7  G astrointestinal D isorders 301

31. The nurse is facilitating a support group for 37. The nurse working in a skilled nursing facility is
clients diagnosed with Crohn’s disease. Which collaborating with the dietitian concerning the
information is most important for the nurse to meals of an immobile client. Which foods are
discuss with the clients? most appropriate for this client?
1. Discuss coping skills to assist with adaptation 1. Oatmeal and wheat toast.
to lifestyle modifications. 2. Cream of wheat and biscuits.
2. Teach about drug administration, dosages, and 3. Cottage cheese and canned peaches.
scheduled times. 4. Tuna on a croissant and applesauce.
3. Teach dietary changes necessary to control
38. Which intervention should the nurse implement
symptoms.
when administering a potassium supplement?
4. Explain the care of the ileostomy and
1. Determine the client’s allergies.
necessary equipment.
2. Ask the client about leg cramps.
32. The nurse is caring for a client diagnosed with 3. Monitor the client’s blood pressure.
ulcerative colitis. Which symptom(s) support 4. Monitor the client’s complete blood count.
this diagnosis?
39. The nurse is preparing the client for a fiberoptic
1. Increased appetite and thirst.
colonoscopy for colon polyps. Which task can be
2. Elevated hemoglobin.
delegated to the unlicensed assistive personnel
3. Multiple bloody, liquid stools.
(UAP)?
4. Exacerbations unrelated to stress.
1. Administer the polyethylene glycol electrolyte
33. The client is diagnosed with an acute lavage solution.
exacerbation of inflammatory bowel disease 2. Explain to the client why this morning’s
(IBD). Which food selection would be the best breakfast is withheld.
choice for a meal? 3. Start an intravenous site with 0.9% normal
1. Roast beef on wheat bread and a milk shake. saline fluid.
2. Hamburger, french fries, and a cola. 4. Administer a cleansing enema until the return
3. Pepper steak, brown rice, and iced tea. is clear.
4. Roasted turkey, instant mashed potatoes, and
40. The nurse is caring for the client who is one (1)
water.
day post–upper gastrointestinal (UGI) series.
34. The nurse is caring for an elderly client Which assessment data warrant intervention?
diagnosed with acute gastritis. Which client 1. No bowel movement.
problem is priority for this client? 2. Oxygen saturation 96%.
1. Fluid volume deficit. 3. Vital signs within normal baseline.
2. Altered nutrition: less than body requirements. 4. Intact gag reflex.
3. Impaired tissue perfusion.
41. The client is complaining of painful swallowing
4. Alteration in comfort.
secondary to mouth ulcers. Which statement
35. The nurse is assessing the client diagnosed with indicates the nurse’s teaching is effective?
chronic gastritis. Which symptom(s) support this 1. “I will brush my teeth with a soft-bristle
diagnosis? toothbrush.”
1. Rapid onset of midsternal discomfort. 2. “I will rinse my mouth with Listerine
2. Epigastric pain relieved by eating food. mouthwash.”
3. Dyspepsia and hematemesis. 3. “I will swish with antifungal solution and then
4. Nausea and projectile vomiting. swallow.”
4. “I will avoid spicy foods, tobacco, and alcohol.”
36. The nurse identifies the problem of “fluid
volume deficit” for a client diagnosed with 42. The nurse is assessing the integumentary system
gastritis. Which intervention should be included of the client diagnosed with anorexia nervosa.
in the plan of care? Which finding supports the diagnosis?
1. Obtain permission for a blood transfusion. 1. Preoccupation with calories.
2. Prepare the client for total parenteral nutrition. 2. Thick body hair.
3. Monitor the client’s lung sounds every shift. 3. Sore tongue.
4. Assess the client’s intravenous site. 4. Dry, brittle hair.

07_Colgrove_Ch07.indd 301 03/06/16 12:58 PM


302 M ed -S urg S uccess

43. The nurse is teaching the client diagnosed with 3. Milk the tube inserted during surgery to allow
colon cancer who is scheduled for a colostomy. the passage of flatus.
Which behavior indicates the nurse is utilizing 4. Digitally dilate the rectal sphincter to express
adult learning principles? old blood.
1. The nurse repeats the information as
48. The client is diagnosed with irritable bowel
indicated by the client’s questions.
syndrome (IBS). Which intervention should the
2. The nurse teaches in one session all the
nurse teach the client to reduce
information the client needs.
symptoms?
3. The nurse uses a video so the client can hear
1. Instruct the client to avoid drinking fluids
the medical terms.
with meals.
4. The nurse waits until the client asks questions
2. Explain the need to decrease intake of flatus-
about the surgery.
forming foods.
44. The nurse is caring for the client one (1) day 3. Teach the client how to perform gentle
postoperative sigmoid colostomy. Which perianal care.
independent nursing intervention should the 4. Encourage the client to attend a support
nurse implement? group meeting.
1. Change the infusion rate of the intravenous
49. The nurse at the scene of a knife fight is caring
fluid.
for a young man who has a knife in his abdomen.
2. Encourage the client to ventilate feelings
Which action should the nurse implement?
about body image.
1. Stabilize the knife.
3. Administer opioid narcotic medications for
2. Remove the knife gently.
pain management.
3. Turn the client on the side.
4. Assist the client out of bed to sit in the chair
4. Apply pressure to the insertion site.
twice daily.
50. The nurse writes the problem “risk for impaired
45. The nurse is caring for the client recovering
skin integrity” for a client with a sigmoid
from intestinal surgery. Which assessment
colostomy. Which expected outcome would be
finding requires immediate intervention?
appropriate for this client?
1. Presence of thin, pink drainage in the Jackson
1. The client will have intact skin around the
Pratt.
stoma.
2. Guarding when the nurse touches the
2. The client will be able to change the ostomy
abdomen.
bag.
3. Tenderness around the surgical site during
3. The client will express anxiety about the body
palpation.
changes.
4. Complaints of chills and feeling feverish.
4. The client will maintain fluid balance.
46. The nurse is caring for the client diagnosed with
51. The client is admitted to the emergency
hemorrhoids. Which statement indicates further
department complaining of acute epigastric pain
teaching is needed?
and reports vomiting a large amount of bright-
1. “I should increase fruits, bran, and fluids in
red blood at home. Which interventions should
my diet.”
the nurse implement? List in order of priority.
2. “I will use warm compresses and take sitz
1. Assess the client’s vital signs.
baths daily.”
2. Insert a nasogastric tube.
3. “I must take a laxative every night and have a
3. Begin iced saline lavage.
stool daily.”
4. Start an IV with an 18-gauge needle.
4. “I can use an analgesic ointment or
5. Type and crossmatch for a blood
suppository for pain.”
transfusion.
47. The nurse is preparing the postoperative nursing
care plan for the client recovering from a
hemorrhoidectomy. Which intervention should
the nurse implement?
1. Establish rapport with the client to decrease
embarrassment of assessing site.
2. Encourage the client to lie in the lithotomy
position twice a day.

07_Colgrove_Ch07.indd 302 03/06/16 12:58 PM


C hapter 7  G astrointestinal D isorders 303

52. The male client diagnosed with a peptic ulcer who is being admitted to the surgical unit admits to using
multiple over-the-counter medications to control his symptoms, including omeprazole (Prilosec, a proton
pump inhibitor) daily; ranitidine (Zantac, an H2 receptor blocker) once or twice a day; and several different
brands of antacids at least every 2 hours. Which ABG results should the nurse expect to find on the chart?
1. Arterial Blood Gas Results
Client Value Normal Values
pH 7.34 7.35–7.45
O2 Saturation 96 80–100
Paco2 36 35–45
HCO3 21 22–26

2. Arterial Blood Gas Results


Client Value Normal Values
pH 7.4 7.35–7.45
O2 Saturation 98 80–100
Paco2 40 35–45
HCO3 24 22–26

3. Arterial Blood Gas Results


Client Value Normal Values
pH 7.49 7.35–7.45
O2 Saturation 95 80–100
Paco2 44 35–45
HCO3 28 22–26

4. Arterial Blood Gas Results


Client Value Normal Values
pH   7.25 7.35–7.45
O2 Saturation 100 80–100
Paco2 32 35–45
HCO3 23 22–26

07_Colgrove_Ch07.indd 303 03/06/16 12:58 PM


GASTROINTESTINAL DISORDERS
COMPREHENSIVE EXAMINATION
ANSWERS AND RATIONALES

1. 1. The nurse should use soap and water for 15 to alcohol, caffeine, and tobacco, because they
30 seconds before and after caring for the cli- increase gastric secretions.
ent. Betadine is surgical scrub. 4. The client should avoid food or drink two (2)
2. Clean gloves should be worn when hours before bedtime or lying down
­providing care to prevent the transfer of after eating.
the bacteria found in the stool. This will Content – Medical: Integrated Nursing Process –
prevent the spread of the bacteria to other Planning: Client Needs – Physiological Integrity, ­
clients in the health-care facility (nosoco- Physiological Adaptation: Cognitive Level – Synthesis:
mial). But this is not a substitute for good Concept – Digestion.
hand hygiene. 4. 1. A rough, beefy-red tongue may indicate the
3. The nurse should clean the perianal area or in- client has pernicious anemia and should be
struct the client to clean the area, but this will evaluated by the health-care provider.
not prevent the spread of the bacteria to other 2. A score of +1 on the tonsil grading scale
clients. shows the tonsils are extending to the
4. Flushing the commode twice is not necessary ­haryngopalatine arch, which is normal.
when disposing of stool and will not prevent a 3. Mucosa should be pink and moist; therefore,
nosocomial infection. the nurse would not need to notify the health-
Content – Medical: Integrated Nursing Process – care provider.
Implementation: Client Needs – Safe Effective Care 4. Symmetrical movement of the uvula is normal
­Environment, Safety and Infection Control: Cognitive
and should not be reported to the health-care
Level – Application: Concept – Bowel Elimination.
provider.
2. 1. Candidiasis, or thrush, presents as white, Content – Medical: Integrated Nursing Process –
cheesy plaques, which bleed when rubbed Implementation: Client Needs – Safe Effective Care
and is a side effect of antibiotic therapy. ­Environment, Management of Care: Cognitive Level –
Candidiasis is treated with antifungal solu- Application: Concept – Hematologic Regulation.
tion, swished around the mouth, held for at 5. 1. Clear to white sputum is not significant in
least one (1) minute, and then swallowed.
the client using oral tobacco.
Candidiasis can be prevented if Lactobacil- 2. Episodic blisters on the lips are herpes simplex
lus acidophilus is administered concurrently 1 and are not specific to this client.
with antibiotic therapy. 3. Presence of any nonhealing sore on the lips
2. White painless patches disappearing in ap- or mouth may be oral cancer. Oral cancer
proximately two (2) weeks are leukoplakia, risk increases by using oral tobacco.
caused by tobacco use, which may be cancer- 4. Bilateral Stensen’s ducts visible at the site of
ous and should be evaluated by an HCP. the second molars are normal assessment data.
3. A solution of hydrogen peroxide is not recom-
Content – Medical: Integrated Nursing Process –
mended to treat candidiasis.
Assessment: Client Needs – Physiological Integrity, ­Reduction
4. The nurse needs to treat the client’s mouth, of Risk Potential: Cognitive Level – A
­ nalysis: Concept –
not use therapeutic communication. ­Cellular Regulation.
Content – Medical: Integrated Nursing Process –
Implementation: Client Needs – Physiological ­Integrity, 6. 1. The colon is ulcerated and unable to absorb
Pharmacological and Parenteral Therapies: ­Cognitive water; 10 to 20 bloody diarrhea stools is the
Level – Application: Concept – Skin Integrity. most common symptom of ulcerative colitis
and does not warrant immediate intervention.
3. 1. The client should eat a low-fat, high-fiber diet. 2. This is not an elevated temperature and does
2. The client should not lie down for at least two not warrant immediate intervention
(2) hours after each meal to prevent g ­ astric by the nurse.
reflux. 3. A hard, rigid abdomen indicates peritoni-
3. The client should avoid irritants, such tis, a complication of ulcerative colitis, and
as spicy foods or acidic foods, as well as warrants immediate intervention.

304

07_Colgrove_Ch07.indd 304 03/06/16 12:58 PM


C hapter 7  G astrointestinal D isorders 305

4. Stress incontinence is not a symptom of colitis 10. 1. The client has been NPO and had laxatives;
and does not warrant immediate intervention. therefore, hyperactive bowel sounds do not
Content – Medical: Integrated Nursing Process – warrant immediate intervention.
Assessment: Client Needs – Physiological Integrity, ­Reduction 2. The client is able to eat after the proce-
of Risk Potential: Cognitive Level – ­Synthesis: Concept – dure, so this does not warrant immediate
Bowel Elimination. intervention.
3. The client received sedation during the
7. 1. The cause of canker sores (aphthous sto-
procedure and may have been up during the
matitis) is unknown. The small ulcerations
night having bowel movements, resulting in
of the soft oral tissue are linked to stress,
the client being exhausted and sleepy.
trauma, allergies, viral infections, and meta-
4. These are signs/symptoms of hypovolemic
bolic disorders. Therefore, being able to
shock requiring immediate intervention
cope with stress is a desired outcome.
by the nurse.
2. The client with recurrent erythematous mac-
ule cankers will not have malnutrition; there- Content – Surgical: Integrated Nursing Process –
fore, a balanced diet is not applicable to this Assessment: Client Needs – Physiological Integrity,
Reduction of Risk Potential: Cognitive Level –
client.
Synthesis: Concept – Perioperative.
3. The client with cankers should not have
­difficulty swallowing. 11. 1. Dentures are not an improvement over the
4. Antacids are not a treatment for canker sores. client’s own teeth in mastication.
Content – Medical: Integrated Nursing Process – 2. The secretion of digestive enzymes and bile
Diagnosis: Client Needs – Physiological Integrity, is decreased in the elderly, resulting in an al-
­Physiological Adaptation: Cognitive Level – Analysis: teration in nutrition and elimination.
­Concept – Skin Integrity. 3. Bile does not affect motility of the intestines.
The elderly client’s perception of the need for
8. 1. This statement is the rationale for proton
laxatives is caused by the client’s misunder-
pump inhibitors.
standing about normal bowel function.
2. This statement explains the rationale for hista-
4. When the motility of the gastrointestinal
mine receptor antagonists.
tract decreases, bacteria remain in the
3. This statement describes how mucosal protec-
gut longer and multiply, which results in
tive agents work in the body.
diarrhea.
4. This statement is the rationale for antacids.
Content – Medical: Integrated Nursing Process –
Content – Medical: Integrated Nursing Process –
Diagnosis: Client Needs – Physiological Integrity,
Implementation: Client Needs – Physiological
­Physiological Adaptation: Cognitive Level – Analysis:
Integrity, Pharmacological and Parenteral Therapies:
Concept – Digestion.
­Cognitive Level – Analysis: Concept – Medication.
12. 1. A high-fiber diet will prevent constipation,
9. 1. Unlicensed assistive personnel (UAP) cannot
the primary reason for diverticulosis/divertic-
administer medications, and this is not appro-
ulitis. A low-fiber (residue) diet is prescribed
priate for a client with anorexia.
for acute diverticulitis.
2. Mouth care should be provided before and
2. Increased fluids help to keep the stool soft
after meals but not with alcohol-based mouth
and prevent constipation.
wash and lemon-glycerin swabs, which can
3. It is controversial if seeds cause an exac-
­decrease the appetite.
erbation of diverticulosis, but this is an
3. The UAP assisting the client with meals
appropriate intervention to teach until
should increase interaction to improve the
proven otherwise.
client’s appetite and make it an e ­ njoyable
4. Exercise helps to prevent constipa-
occasion.
4. Often the parents are the cause of the ­client’s tion, which can cause an exacerbation of
diverticulitis.
stress and anxiety, which may have led to the
5. There are no medications used to help pre-
client’s anorexia; therefore, the parents should
not be asked to stay with the client. vent an acute exacerbation of diverticulosis/
diverticulitis. Antacids are used to neutralize
Content – Medical: Integrated Nursing Process –
hydrochloric acid in the stomach.
Planning: Client Needs – Safe Effective Care Environment,
Management of Care: Cognitive Level – Synthesis: Content – Medical: Integrated Nursing Process –
Concept – Nursing Roles. Planning: Client Needs – Health Promotion and
­Maintenance: Cognitive Level – Synthesis: Concept –
Bowel Elimination.

07_Colgrove_Ch07.indd 305 03/06/16 12:58 PM


306 M ed -S urg S uccess

13. 1. Obesity increases the risk of pyrosis 2. Many clients come through the day surgery
(heartburn); therefore, losing weight department and go home the same day. Some
could help decrease the incidence. clients may remain in the hospital for one (1)
2. Eating small, frequent meals along with de- or two (2) days but not for a week.
creased intake of spicy foods have been linked 3. All surgeries will result in pain for the client.
to the prevention of heartburn (pyrosis). 4. The client should not return to work the next
3. Antiemetics decrease nausea, which does not day; the client should wait at least one (1)
occur with heartburn. Antacids neutralize week before returning to work.
the acid of the stomach and are used to treat Content – Surgical: Integrated Nursing Process –
heartburn. Evaluation: Client Needs – Physiological Integrity,
4. Drinking alcoholic beverages increases heart- ­Physiological Adaptation: Cognitive Level – ­Evaluation:
burn and should be avoided, not decreased. Concept – Perioperative.
Content – Medical: Integrated Nursing Process – 17. 1. The client should elevate the head, not the
Planning: Client Needs – Health Promotion and foot, of the bed to prevent the reflux of stom-
­Maintenance: Cognitive Level – Synthesis: Concept –
ach contents.
Nutrition.
2. The evening meal should be eaten at least
14. 1. The client has three (3) to four (4) incisions two (2) hours prior to retiring. Small, fre-
with Band-Aids in the upper quadrant, not an quent meals and semisoft foods ease the
abdominal dressing. passage of food, which decreases signs and
2. The client will not have a T-tube with a lapa- symptoms of the disease process.
roscopic cholecystectomy. 3. This diet is recommended for a client with
3. A laparoscopic cholecystectomy is done coronary artery disease, not for esophageal
in day surgery. The nurse must make sure diverticula.
the significant others taking care of the 4. Restrictive clothing should be avoided, and
client are knowledgeable of postoperative abdominal binders do not strengthen muscles
care. and would not benefit this client.
4. The client will be on a regular diet after be- Content – Medical: Integrated Nursing Process –
ing discharged from the day surgery clinic. Planning: Client Needs – Physiological Integrity,
Content – Surgical: Integrated Nursing Process – ­Physiological Adaptation: Cognitive Level – Synthesis:
Implementation: Client Needs – Safe Effective Care ­Concept – Digestion.
­Environment, Management of Care: Cognitive
18. BMI of 30.2.
Level – Application: Concept – Perioperative.
BMI 703 × weight in pounds
15. 1. The client may resume the regular diet. (height in inches)2
2. The client will not be sedated for this proce-
dure; therefore, the client does not need to be 112,480 × 30.22 BMI
on bedrest. 3,721
3. The nurse needs to teach the client to Content – Fundamentals: Integrated Nursing
take a mild laxative to help evacuate the Process – Implementation: Client Needs – Safe
barium and return to the client’s normal Effective Care Environment, Management of Care:
bowel routine. Failure to pass the barium Cognitive Level – Application: Concept – Self.
could cause constipation when the barium
hardens. 19. 1. A client with less than a 19 BMI is
4. The client can expect to pass white- or light- underweight.
colored stools until the barium has com- 2. A BMI between 25 and 30 is considered
pletely been evacuated. overweight.
3. The ideal weight for a client is a BMI
Content – Surgical: Integrated Nursing Process –
Planning: Client Needs – Physiological Integrity,
­between 19 and 24.9.
­Physiological Adaptation: Cognitive Level – Synthesis: 4. This client is obese, with a BMI greater
Concept – Bowel Elimination. than 30.
Content – Fundamentals: Integrated Nursing Process –
16. 1. In a laparoscopic Nissen fundoplication, Implementation: Client Needs – Safe Effective Care
there are four (4) to five (5) incisions ap- ­Environment, Management of Care: Cognitive Level –
proximately one (1) inch apart allowing Application: Concept – Self.
for the passage of equipment to visualize
the abdominal organs and perform the 20. 1. The increased serum ammonia level
operation. ­associated with liver failure causes the he-
patic encephalopathy, which, in turn, leads
to neurological deficit.

07_Colgrove_Ch07.indd 306 03/06/16 12:58 PM


C hapter 7  G astrointestinal D isorders 307

2. Administering a loop diuretic is an appro- 24. 1. When clients have multiple liquid stools,
priate intervention for ascites and portal the rectal area can become irritated. The
hypertension. integrity of the skin can be impaired.
3. Checking the stool for bleeding is an 2. Pain experienced by this client would be
­appropriate intervention for esophageal acute, rather than chronic.
­varices and decreased vitamin K. 3. Fluid volume deficit is appropriate, rather
4. Assessing the abdominal fluid wave is an than fluid volume excess.
­appropriate intervention for ascites and ­portal 4. Ineffective coping is a psychosocial prob-
hypertension. lem and is not appropriate for a client with
Content – Medical: Integrated Nursing Process – diarrhea.
Implementation: Client Needs – Physiological Integrity, Content – Medical: Integrated Nursing Process –
Physiological Adaptation: Cognitive Level – Application: Diagnosis: Client Needs – Safe Effective Care E
­ nvironment,
Concept – Digestion. Management of Care: Cognitive Level – Analysis:
Concept – Bowel Elimination.
21. 1. This intervention would be appropriate for
prevention of hepatitis A. 25. 1. Steady, aching pain is associated with a
2. The hepatitis B vaccine will prevent ­peritoneal inflammation, which may be
the client from contracting this ­secondary to a ruptured spleen or perforated
disease. ulcer or other abdominal organ.
3. This intervention would be appropriate for 2. Sharp pain in the back and flank indicates
prevention of hepatitis A. kidney involvement.
4. The nurse uses Standard Precautions, not the 3. Sharp pain increasing with deep breaths indi-
client. cates muscular involvement.
Content – Medical: Integrated Nursing Process – 4. Intermittent and colicky pain located near
Planning: Client Needs – Safe Effective Care the umbilicus is indicative of a small bowel
Environment, Safety and Infection Control: Cognitive obstruction; lumbar pain is indicative of
Level – Synthesis: Concept – Digestion. colon involvement.
22. 1. The incubation period for Salmonella food Content – Medical: Integrated Nursing Process –
poisoning is 8 to 48 hours. Assessment: Client Needs – Physiological Integrity,
­Reduction of Risk Potential: Cognitive Level – Analysis:
2. Salmonellae bacteria are not transmitted to
Concept – Bowel Elimination.
humans via water.
3. Eggs, poultry, and pet turtles are sources 26. 1. Fluid deficit is a potential problem, not an
of the Salmonellae bacteria, which cause actual problem. The client’s fluid balance
food poisoning. should be managed by intravenous fluids.
4. Clostridium botulinum is transmitted via 2. The perfusion of the surgical site is
­improperly canned food. ­compromised as a result of the surgical
Content – Medical: Integrated Nursing Process – incision, especially when a graft is used.
Planning: Client Needs – Safe Effective Care ­Environment, 3. Infection is a potential problem but not at the
Safety and Infection Control: Cognitive Level – Synthesis: time of surgery.
Concept – Digestion. 4. After surgery, not during surgery, the client
may require chemotherapy, which can cause
23. 1. Hand washing for 10 seconds is not long
immunosuppression.
enough to remove any bacteria. Hands should
be washed for at least 30 seconds ­before han- Content – Surgical: Integrated Nursing Process –
dling food or eating. Diagnosis: Client Needs – Safe Effective Care ­
Environment, Management of Care: Cognitive Level –
2. Cutting surfaces used for meats should
Analysis: Concept – Perioperative.
be different from those used for fruits and
­vegetables to prevent contamination. 27. 1. A client who has been diagnosed with
3. Foods being served for an extended portal hypertension should be assessed
time should be kept at 140°F because food for a tympanic (fluid) wave to check for
sitting at less than this temperature allows ascites.
for bacterial growth. 2. High blood pressure is not the etiology of
4. All fruits and vegetables should be washed portal hypertension.
before eating or preparing. 3. In portal hypertension, percussion is difficult
Content – Medical: Integrated Nursing Process – and will not provide pertinent information
Planning: Client Needs – Health Promotion and about the client’s condition.
­Maintenance: Cognitive Level – Synthesis: Concept – 4. Weighing the client should be done daily, not
Digestion. twice each week.

07_Colgrove_Ch07.indd 307 03/06/16 12:58 PM


308 M ed -S urg S uccess

Content – Medical: Integrated Nursing Process – 2. Drug administration, dosage, and scheduled
Diagnosis: Client Needs – Safe Effective Care ­ times should be discussed in the hospital
Environment, Management of Care: Cognitive prior to discharge or in the health-care pro-
Level – Analysis: Concept – Digestion. vider’s office; therefore, this is not a priority
28. 1. A decrease in weight indicates a loss in fluid at the support group meeting.
and is not data necessary to report to the 3. Dietary changes should be taught at the time
health-care provider. the disease is diagnosed, but this is not a pri-
2. An increase in urine output indicates the di- ority at the support group meeting.
uretic was effective. 4. An ileostomy may be the surgical option for
3. An increase in abdominal girth indicates clients who do not respond to medical treat-
the ascites is increasing, meaning the cli- ment, but other nonsurgical treatments would
ent’s condition is becoming more serious be topics of discussions during support group
and should be reported to the health-care meetings.
provider. Content – Medical: Integrated Nursing Process –
4. The normal direct bilirubin value is 0.1 to Planning: Client Needs – Physiological Integrity,
0.4 mg/dL; therefore, a decrease in the value, ­Physiological Adaptation: Cognitive Level – Synthesis:
although it is still elevated, would not be Concept – Bowel Elimination.
reported. 32. 1. Clients suffering from ulcerative colitis expe-
Content – Medical: Integrated Nursing Process – rience anorexia, not an increased appetite.
Implementation: Client Needs – Safe Effective Care 2. The hemoglobin and hematocrit are de-
Environment, Management of Care: Cognitive creased, not elevated, as a result of blood loss.
Level – Application: Concept – Digestion. 3. Clients report as many as 10 to 20 liquid,
29. 1. An increase in serum ammonia levels is seen bloody stools in a day.
in clients diagnosed with hepatic encepha- 4. Stressful events have been linked to an in-
lopathy and coma. crease in symptoms. The nurse needs to
2. The inability to circle food items on the assess for perceived stress in the client’s life
menu may indicate deterioration in the producing symptoms.
client’s cognitive status. The client’s neu- Content – Medical: Integrated Nursing Process –
rological status is impaired with hepatic Assessment: Client Needs – Physiological Integrity,
encephalopathy; the nurse should investi- ­Reduction of Risk Potential: Cognitive Level – ­Analysis:
gate this behavior. Concept – Bowel Elimination.
3. The client being able to follow commands 33. 1. Wheat bread and whole grains should be
indicates the client’s neurological status is avoided, and most clients cannot tolerate milk
intact. products.
4. Consuming foods providing adequate nutri- 2. Fried foods such as hamburger and french
tion indicates the client is getting better and fries should be avoided. Raw fruits and
able to follow client teaching. ­vegetables such as lettuce and tomatoes are
Content – Medical: Integrated Nursing Process – usually not tolerated.
Assessment: Client Needs – Physiological Integrity, 3. Whole grains such as brown rice should be
Reduction of Risk Potential: Cognitive Level – avoided. White rice can be eaten. Spicy meats
Analysis: Concept – Digestion. and foods should be avoided.
30. 1. Stools are liquid in the ascending colon. 4. Meats can be eaten if prepared by roast-
2. Stools are mushy in the right transverse ing, baking, or broiling. Vegetables should
colon. be cooked, not raw, and skins should be
3. The left transverse colon has semi-mushy removed. Instant mashed potatoes do not
stool. have the skin. A low-residue diet should
4. The sigmoid colon is located in the left be eaten.
lower quadrant, and the client expels solid Content – Medical: Integrated Nursing Process –
feces. Planning: Client Needs – Physiological Integrity,
Content – Surgical: Integrated Nursing Process – ­Physiological Adaptation: Cognitive Level – Synthesis:
Planning: Client Needs – Physiological Integrity, Concept – Bowel Elimination.
­Physiological Adaptation: Cognitive Level – Synthesis: 34. 1. Pediatric and geriatric clients have an in-
Concept – Perioperative. creased risk for fluid volume and electro-
31. 1. The objectives for support groups are to lyte imbalances. The nurse should always
help members cope with chronic diseases be alert to this possible complication.
and help manage symptom control.

07_Colgrove_Ch07.indd 308 03/06/16 12:58 PM


C hapter 7  G astrointestinal D isorders 309

2. Altered nutrition may be appropriate, Content – Medical: Integrated Nursing Process –


­depending on how long the client has been Planning: Client Needs – Physiological Integrity,
unable to eat, but it is not priority over Basic Care and Comfort: Cognitive Level – Synthesis:
fluid volume deficit. Concept – Nutrition.
3. Impaired tissue perfusion may be appropriate 38. 1. The nurse should inquire about drug allergies
if the mucosal lining of the stomach is unable before administering all medications, not just
to heal, but it is not priority over fluid volume potassium.
deficit. 2. Leg cramps occur when serum potassium
4. Alteration in comfort may be appropriate, but levels are too low or too high. If the cli-
it is not priority over fluid volume deficit. ent has leg cramps, this could indicate an
Content – Medical: Integrated Nursing Process – imbalance, which could lead to cardiac
Diagnosis: Client Needs – Safe Effective Care ­ dysrhythmias.
Environment, Management of Care: Cognitive Level – 3. The blood pressure does not evaluate for
Analysis: Concept – Digestion. dysrhythmias, a possible result of abnormal
35. 1. Acute gastritis is characterized by sudden potassium levels.
epigastric pain or discomfort, not midsternal 4. The complete blood count does not include
chest pain. the potassium level; a chemistry panel is
2. Chronic pain in the epigastric area needed.
relieved by ingesting food is a sign Content – Medical: Integrated Nursing Process –
of chronic gastritis. Implementation: Client Needs – Physiological
3. Dyspepsia (heartburn) and hematemesis Integrity, Pharmacological and Parenteral Therapies:
(vomiting blood) are frequent symptoms Cognitive Level – Application: Concept – Nutrition.
of acute gastritis. 39. 1. The polyethylene glycol electrolyte lavage
4. Projectile vomiting is not a sign of chronic solution is a medication and cannot be
gastritis. delegated.
Content – Medical: Integrated Nursing Process – 2. Teaching is the responsibility of the nurse and
Assessment: Client Needs – Physiological Integrity, cannot be delegated.
Reduction of Risk Potential: Cognitive Level – 3. Starting an intravenous site and managing the
Analysis: Concept – Digestion. delivery of the fluid is the responsibility of
36. 1. There are no data to suggest the client needs the nurse and cannot be delegated.
a blood transfusion. 4. The administration of enemas can be
2. TPN is not a treatment for a client with fluid ­delegated to the unlicensed assistive
volume deficit. TPN provides calories for nu- ­personnel (UAP).
tritional deficits, not fluid deficits. Content – Surgical: Integrated Nursing Process –
3. If the client’s problem were fluid volume Planning: Client Needs – Safe Effective Care
excess, assessing lung sounds would be Environment, Management of Care: Cognitive
appropriate. Level – Synthesis: Concept – Nursing Roles.
4. Fluid administration is the medical treat- 40. 1. The nurse should monitor the client for
ment for dehydration, so the nurse must the first bowel movement to document
monitor and ensure the IV site is patent. elimination of barium, which should be
Content – Medical: Integrated Nursing Process – eliminated within two (2) days. If the cli-
Diagnosis: Client Needs – Safe Effective Care ent does not have a bowel movement, a
Environment, Management of Care: Cognitive laxative may be needed to help the client
Level – Analysis: Concept – Digestion. to eliminate the barium before it becomes
37. 1. Oatmeal and wheat toast are high- too hard to pass.
fiber foods and are recommended for 2. An oxygen saturation of 96% is acceptable
clients who are immobile to help prevent and does not require intervention.
constipation. 3. Vital signs should be monitored to recognize
2. Cream of wheat and biscuits are low-fiber and treat complications before the client is in
foods. danger. Baseline is a desired outcome.
3. Cottage cheese and canned peaches are 4. The client’s throat is not anesthetized for this
­low-fiber foods. procedure, so the gag reflex is not pertinent
4. Tuna is a good source of protein for the information in this procedure.
­client, but croissants have a high fat content Content – Medical: Integrated Nursing Process –
and are a factor in weight gain if consistently Assessment: Client Needs – Safe Effective Care
eaten. Applesauce is low in fiber. Environment, Management of Care: Cognitive
Level – Synthesis: Concept – Perioperative.

07_Colgrove_Ch07.indd 309 03/06/16 12:58 PM


310 M ed -S urg S uccess

41. 1. A soft-bristle toothbrush will not affect pain- 3. Medication administration is a collaborative
ful swallowing. intervention because it requires an order by
2. An alcohol-based mouthwash (Listerine) is the health-care provider.
irritating to the oral cavity and can increase 4. Activity level immediately postoperative re-
pain. quires an order by the health-care provider.
3. An antifungal medication should be used with Content – Medical: Integrated Nursing Process –
candidiasis and is not effective treatment for Implementation: Client Needs – Safe Effective Care
plain mouth ulcers. ­Environment, Management of Care: Cognitive
4. Irritating substances should be avoided Level – Application: Concept – Perioperative.
during the outbreaks of ulcers in the
45. 1. Thin pink drainage is expected in the ­Jackson
mouth. Spicy foods, alcohol, and tobacco
Pratt (JP) bulb.
are common irritants the client should
2. Guarding is a normal occurrence when
avoid.
touching a tender area on the abdomen and
Content – Medical: Integrated Nursing Process – does not require immediate intervention.
Evaluation: Client Needs – Physiological Integrity, 3. Tenderness around the surgical site is
­Physiological Adaptation: Cognitive Level – E
­ valuation:
a normal finding and does not require
Concept – Digestion.
intervention.
42. 1. The preoccupation with food, calories, and 4. Complaints of chills, sudden onset of
preparing meals are psychosocial behaviors ­fever, tachycardia, nausea, and hiccups are
suggesting the client has an eating disorder. symptoms of peritonitis, which is a life-
2. Clients who have anorexia nervosa have thin, threatening complication.
fine body hair. Content – Surgical: Integrated Nursing Process –
3. Iron-deficiency anemia causes clients to Assessment: Client Needs – Safe Effective Care
­experience a sore tongue. Environment, Management of Care: Cognitive
4. Thin, brittle hair occurs in clients with Level – Synthesis: Concept – Perioperative.
anorexia.
46. 1. Clients with hemorrhoids need to eat high-
Content – Medical: Integrated Nursing Process – fiber diets and increase fluid intake to keep
Assessment: Client Needs – Physiological Integrity, the stools soft and prevent constipation;
­Reduction of Risk Potential: Cognitive Level – ­Analysis:
therefore, the teaching is effective.
Concept – Nutrition.
2. Warm compresses or sitz baths decrease pain;
43. 1. The nurse should realize the client is anx- therefore, the teaching is effective.
ious about the diagnosis of cancer and the 3. Laxatives can be harsh to the bowel and
impending surgery. Therefore, the nurse are habit forming; they should not be
should be prepared to repeat information taken daily. Stool softeners soften stool
as necessary. The learning principle the and can be taken daily.
nurse needs to consider is “anxiety de- 4. Analgesic ointments, suppositories, and as-
creases learning.” tringents can be used to decrease pain and
2. Small manageable sessions increase learning, decrease edema; therefore, the teaching has
especially when the client is anxious. been effective.
3. Videos are not the best teaching tool for Content – Medical: Integrated Nursing Process –
adults. Short videos are useful for children. Evaluation: Client Needs – Physiological Integrity,
4. The nurse should assess the client’s readiness ­Physiological Adaptation: Cognitive Level – Synthesis:
and willingness to learn and not wait until the Concept – Bowel Elimination.
client asks questions about the surgery.
47. 1. The site of the surgery can cause embar-
Content – Surgical: Integrated Nursing Process – rassment when the nurse assesses the site;
Planning: Client Needs – Physiological Integrity, therefore, the nurse should establish a
Reduction of Risk Potential: Cognitive Level –
positive relationship.
Analysis: Concept – Cellular Regulation.
2. The lithotomy position is with the client’s
44. 1. The rate of the intravenous fluid is a col- legs in stirrups for procedures such as Pap
laborative nursing intervention because it re- smears and some surgeries such as transure-
quires an order from the health-care provider. thral resection of the prostate, not for the cli-
2. Encouraging the client to verbalize feel- ent who is postoperative hemorrhoidectomy.
ings about body changes assists the cli- 3. A tube is not placed in the client’s rectum af-
ent to accept these changes. This is an ter this surgery.
independent intervention not requiring a 4. The rectal sphincter does not need to be
health-care provider’s order. ­digitally dilated.

07_Colgrove_Ch07.indd 310 03/06/16 12:58 PM


C hapter 7  G astrointestinal D isorders 311

Content – Surgical: Integrated Nursing Process – 3. Expressing anxiety about the body changes is
Diagnosis: Client Needs – Physiological Integrity, a goal for an alteration in body image.
­Physiological Adaptation: Cognitive Level – Analysis: 4. Maintaining a balance in fluid is a goal for
­Concept – Bowel Elimination. a nursing diagnosis of risk for fluid deficit.
48. 1. Avoidance of fluids during meals will Content – Surgical: Integrated Nursing Process –
help prevent abdominal distention, which Diagnosis: Client Needs – Physiological Integrity,
causes symptoms of IBS. Do not confuse ­Physiological Adaptation: Cognitive Level – Analysis:
inflammatory bowel disease (IBD) and Concept – Skin Integrity.
irritable bowel syndrome (IBS). 51. In order of priority: 1, 4, 5, 2, 3.
2. Avoidance of flatus (gas)-forming foods helps 1. The nurse should assess the vital signs to
with the symptoms of IBD, not IBS. determine if the client is in hypovolemic
3. Clients with IBS do have altered bowel habits shock. The stem of the question does not
such as diarrhea and constipation, but peri- provide information indicating the client
anal care will not prevent IBS. is hypovolemic. The client’s perception of
4. IBS does have a psychological component, a large amount of blood may differ from
but a client recently diagnosed should be the nurse’s assessment.
taught other interventions before a psycholo- 4. The nurse should start the IV line to
gist is recommended. ­replace fluid volume.
Content – Medical: Integrated Nursing Process – 5. While the nurse is starting the IV, a
Planning: Client Needs – Physiological Integrity, blood sample for typing and crossmatch-
­Physiological Adaptation: Cognitive Level – Synthesis: ing should be obtained and sent to the
Concept – Bowel Elimination. laboratory.
49. 1. The nurse should not remove any 2. An N/G tube should be inserted so direct
­penetrating object in the abdomen; iced saline can be instilled to cause con-
removal could cause further internal striction, which will decrease the bleeding.
damage. 3. The iced saline lavage will help decrease
2. Removal of the knife could cause further in- bleeding.
ternal damage. Content – Medical: Integrated Nursing Process –
3. The client should be kept on the back and the Implementation: Client Needs – Safe Effective Care
knife should be stabilized. Environment, Management of Care: Cognitive
4. The nurse should stabilize the knife and no- Level – Analysis: Concept – Digestion.
tify Emergency Medical Services as quickly as 52. 1. This is metabolic acidosis; the client tak-
possible. ing many antacids would be in metabolic
Content – Medical: Integrated Nursing Process – alkalosis.
Implementation: Client Needs – Safe Effective Care 2. These are normal blood gas results.
­Environment, Management of Care: Cognitive Level – 3. This is metabolic alkalosis, a result of tak-
Application: Concept – Violence. ing many antacids.
50. 1. Intact skin around the stoma is the most 4. This is respiratory alkalosis, a result
appropriate outcome for the problem of of hyperventilation.
“impaired skin integrity.” Content – Medical: Integrated Nursing Process –
2. The client’s ability to change the ostomy bag Assessment: Client Needs – Physiological Integrity,
is a goal for a knowledge-deficit problem or ­Physiological Adaptation: Cognitive Level – Synthesis:
self-care. Concept – Digestion.

07_Colgrove_Ch07.indd 311 03/06/16 12:58 PM


07_Colgrove_Ch07.indd 312 03/06/16 12:58 PM
Endocrine
Disorders
A little knowledge that acts is worth infinitely more than much
knowledge that is idle.
8
—Kahlil Gibran

The endocrine system, along with the nervous system, controls or influences the activities of
other body systems. The major glands of the endocrine system—the pituitary, thyroid, para-
thyroid, adrenals, pancreas, and ovaries (in females) and testes (in men)—play a major role in
regulating activities of the body, including metabolism, fluid balance, reaction to stress, and
sexual function. As with organs in other systems, these glands are subject to many disorders,
some relatively minor and easily treated and others life threatening. This chapter focuses on
diabetes mellitus, one of the most common and major endocrine system disorders; on other
pancreatic disorders; and on adrenal, pituitary, and thyroid disorders. Disorders affecting the
glands involved in reproduction and sexual functioning are discussed in Chapter 10.

KEYWORDS ABBREVIATIONS
Addisonian crisis Adrenocorticotropic hormone (ACTH)
Aldosteronism Antidiuretic hormone (ADH)
Androgen Arterial blood gas (ABG)
Antidiuretic At bedtime (h.s.)
Endogenous Beats per minute (bpm)
Euthyroid Before meals (a.c.)
Exogenous Blood pressure (BP)
Fluid deprivation test Blood urea nitrogen (BUN)
Glucocorticoid Computed tomography (CT)
Hyperparathyroidism Diabetes insipidus (DI)
Hyperthyroidism Diabetes mellitus (DM)
Hypoparathyroidism Diabetic ketoacidosis (DKA)
Hypothyroidism Endoscopic retrograde cholangiopancreatogram
Iatrogenic (ERCP)
Ketone Health-care provider (HCP)
Kussmaul’s respirations Hyperosmolar hyperglycemic nonketotic syndrome
Mineralocorticoid (HHNS)
Pruritus Intake & output (I&O)
Tenesmus Intensive care unit (ICU)
Turgor Intravenous (IV)
Vasopressin Intravenous push (IVP)
Water challenge test Nasogastric (N/G)

313

08_Colgrove_Ch08.indd 313 02/06/16 8:01 PM


314 M ed -S urg S uccess

Nothing by mouth (NPO)


Percutaneous endoscopic gastrostomy (PEG)
Premature ventricular contraction (PVC)
Prothrombin time (PT)
Rule out (R/O)
Syndrome of inappropriate antidiuretic hormone
(SIADH)
Unlicensed assistive personnel (UAP)
Within normal limits (WNL)

PRACTICE QUESTIONS
Diabetes Mellitus 5. The nurse is discussing the importance of
exercising with a client diagnosed with type 2
1. An 18-year-old female client, 5′4″ tall, weighing diabetes whose diabetes is well controlled with diet
113 kg, comes to the clinic for a nonhealing and exercise. Which information should the nurse
wound on her lower leg, which she has had for include in the teaching about diabetes?
two (2) weeks. Which disease process should the 1. Eat a simple carbohydrate snack before
nurse suspect the client has developed? exercising.
1. Type 1 diabetes. 2. Carry peanut butter crackers when exercising.
2. Type 2 diabetes. 3. Encourage the client to walk 20 minutes three
3. Gestational diabetes. (3) times a week.
4. Acanthosis nigricans. 4. Perform warm-up and cool-down exercises.
2. The client diagnosed with type 1 diabetes has a 6. The nurse is assessing the feet of a client with
glycosylated hemoglobin (A1c) of 8.1%. Which long-term type 2 diabetes. Which assessment data
interpretation should the nurse make based on warrant immediate intervention by the nurse?
this result? 1. The client has crumbling toenails.
1. This result is below normal levels. 2. The client has athlete’s foot.
2. This result is within acceptable levels. 3. The client has a necrotic big toe.
3. This result is above recommended levels. 4. The client has thickened toenails.
4. This result is dangerously high. 7. The home health nurse is completing the
3. The nurse administered 28 units of Humulin admission assessment for a 76-year-old client
N, an intermediate-acting insulin, to a client diagnosed with type 2 diabetes controlled with
diagnosed with type 1 diabetes at 1600. Which 70/30 insulin. Which intervention should be
intervention should the nurse implement? included in the plan of care?
1. Ensure the client eats the bedtime snack. 1. Assess the client’s ability to read small print.
2. Determine how much food the client ate 2. Monitor the client’s serum prothrombin time
at lunch. (PT) level.
3. Perform a glucometer reading at 0700. 3. Teach the client how to perform a hemoglobin
4. Offer the client protein after administering A1c test daily.
insulin. 4. Instruct the client to check the feet weekly.
4. The client diagnosed with type 1 diabetes is 8. The client with type 2 diabetes controlled with
receiving Humalog, a rapid-acting insulin, by biguanide oral diabetic medication is scheduled
sliding scale. The order reads blood glucose for a computed tomography (CT) scan with
level: <150, zero (0) units; 151 to 200, three contrast of the abdomen to evaluate pancreatic
(3) units; 201 to 250, six (6) units; >251, contact function. Which intervention should the nurse
health-care provider. The unlicensed assistive implement?
personnel (UAP) reports to the nurse the client’s 1. Provide a high-fat diet 24 hours prior to test.
glucometer reading is 189. How much insulin 2. Hold the biguanide medication for 48 hours
should the nurse administer to the client? prior to test.
_________ 3. Obtain an informed consent form for the test.
4. Administer pancreatic enzymes prior to the test.

08_Colgrove_Ch08.indd 314 02/06/16 8:01 PM


C hapter 8  E ndocrine D isorders 315

9. The diabetic educator is teaching a class on 3. The nurse will monitor the client’s blood
diabetes type 1 and is discussing sick-day glucose levels four (4) times a day.
rules. Which interventions should the diabetes 4. The client will maintain normal kidney
educator include in the discussion? Select all function with 30-mL/hr urine output.
that apply.
13. The client diagnosed with type 2 diabetes
1. Take diabetic medication even if unable to eat
is admitted to the intensive care unit (ICU)
the client’s normal diabetic diet.
with hyperosmolar hyperglycemic nonketotic
2. If unable to eat, drink liquids equal to the
syndrome (HHNS) coma. Which assessment
client’s normal caloric intake.
data should the nurse expect the client to
3. It is not necessary to notify the health-care
exhibit?
provider (HCP) if ketones are in the urine.
1. Kussmaul’s respirations.
4. Test blood glucose levels and test urine
2. Diarrhea and epigastric pain.
ketones once a day and keep a record.
3. Dry mucous membranes.
5. Call the health-care provider if glucose levels
4. Ketone breath odor.
are higher than 180 mg/dL.
14. The elderly client is admitted to the intensive
10. The client received 10 units of Humulin R,
care department diagnosed with severe HHNS.
a fast-acting insulin, at 0700. At 1030 the
Which collaborative intervention should the
unlicensed assistive personnel (UAP) tells the
nurse include in the plan of care?
nurse the client has a headache and is really
1. Infuse 0.9% normal saline intravenously.
acting “funny.” Which intervention should the
2. Administer intermediate-acting insulin.
nurse implement first?
3. Perform blood glucometer checks daily.
1. Instruct the UAP to obtain the blood glucose
4. Monitor arterial blood gas (ABG) results.
level.
2. Have the client drink eight (8) ounces of 15. Which electrolyte replacement should the nurse
orange juice. anticipate being ordered by the health-care
3. Go to the client’s room and assess the client provider in the client diagnosed with diabetic
for hypoglycemia. ketoacidosis (DKA) who has just been admitted
4. Prepare to administer one (1) ampule 50% to the ICU?
dextrose intravenously. 1. Glucose.
2. Potassium.
11. The nurse at a freestanding health-care clinic
3. Calcium.
is caring for a 56-year-old male client who is
4. Sodium.
homeless and is a type 2 diabetic controlled with
insulin. Which action is an example of client 16. The client diagnosed with HHNS was admitted
advocacy? yesterday with a blood glucose level of 780 mg/
1. Ask the client if he has somewhere he can dL. The client’s blood glucose level is now 300
go and live. mg/dL. Which intervention should the nurse
2. Arrange for someone to give him insulin at implement?
a local homeless shelter. 1. Increase the regular insulin IV drip.
3. Notify Adult Protective Services about the 2. Check the client’s urine for ketones.
client’s situation. 3. Provide the client with a therapeutic diabetic
4. Ask the HCP to take the client off insulin meal.
because he is homeless. 4. Notify the HCP to obtain an order to
decrease insulin.
12. The nurse is developing a care plan for the
client diagnosed with type 1 diabetes. The 17. The client diagnosed with type 1 diabetes is
nurse identifies the problem “high risk for found lying unconscious on the floor of the
hyperglycemia related to noncompliance with bathroom. Which intervention should the nurse
the medication regimen.” Which statement is an implement first?
appropriate short-term goal for the client? 1. Administer 50% dextrose (IVP).
1. The client will have a blood glucose level 2. Notify the health-care provider.
between 90 and 140 mg/dL. 3. Move the client to the ICU.
2. The client will demonstrate appropriate 4. Check the serum glucose level.
insulin injection technique.

08_Colgrove_Ch08.indd 315 02/06/16 8:01 PM


316 M ed -S urg S uccess

18. Which assessment data indicate the client 23. Which arterial blood gas results should the
diagnosed with diabetic ketoacidosis is nurse expect in the client diagnosed with
responding to the medical treatment? diabetic ketoacidosis?
1. The client has tented skin turgor and dry 1. pH 7.34, Pao2 99, Paco2 48, HCO3 24.
mucous membranes. 2. pH 7.38, Pao2 95, Paco2 40, HCO3 22.
2. The client is alert and oriented to date, 3. pH 7.46, Pao2 85, Paco2 30, HCO3 26.
time, and place. 4. pH 7.30, Pao2 90, Paco2 30, HCO3 18.
3. The client’s ABG results are pH 7.29,
24. The client is admitted to the ICU diagnosed
Paco2 44, HCO3 15.
with DKA. Which interventions should the
4. The client’s serum potassium level is
nurse implement? Select all that apply.
3.3 mEq/L.
1. Maintain adequate ventilation.
19. The UAP on the medical floor tells the nurse 2. Assess fluid volume status.
the client diagnosed with DKA wants something 3. Administer intravenous potassium.
else to eat for lunch. Which intervention should 4. Check for urinary ketones.
the nurse implement? 5. Monitor intake and output.
1. Instruct the UAP to get the client additional
food.
2. Notify the dietitian about the client’s request. Pancreatitis
3. Request the HCP increase the client’s caloric
intake. 25. The client is admitted to the medical department
4. Tell the UAP the client cannot have with a diagnosis of rule-out (R/O) acute
anything else. pancreatitis. Which laboratory values should the
nurse monitor to confirm this diagnosis?
20. The emergency department nurse is caring for
1. Creatinine and (BUN).
a client diagnosed with HHNS who has a blood
2. Troponin and (CK-MB).
glucose of 680 mg/dL. Which question should
3. Serum amylase and lipase.
the nurse ask the client to determine
4. Serum bilirubin and calcium.
the cause of this acute complication?
1. “When is the last time you took your insulin?” 26. Which client problem has priority for the client
2. “When did you have your last meal?” diagnosed with acute pancreatitis?
3. “Have you had some type of infection lately?” 1. Risk for fluid volume deficit.
4. “How long have you had diabetes?” 2. Alteration in comfort.
3. Imbalanced nutrition: less than body
21. The nurse is discussing ways to prevent diabetic
requirements.
ketoacidosis with the client diagnosed with type
4. Knowledge deficit.
1 diabetes. Which instruction is most important
to discuss with the client? 27. The nurse is preparing to administer a.m.
1. Refer the client to the American Diabetes medications to clients. Which medication should
Association. the nurse question before administering?
2. Do not take any over-the-counter (OTC) 1. Pancreatic enzymes to the client who has
medications. finished breakfast.
3. Take the prescribed insulin even when unable 2. The pain medication, morphine, to the client
to eat because of illness. who has a respiratory rate of 20.
4. Explain the need to get the annual flu and 3. The loop diuretic to the client who has a
pneumonia vaccines. serum potassium level of 3.9 mEq/L.
4. The beta blocker to the client who has an
22. The charge nurse is making client assignments
apical pulse of 68 bpm.
in the intensive care unit. Which client should
be assigned to the most experienced nurse? 28. The client is diagnosed with acute pancreatitis.
1. The client with type 2 diabetes who has a Which health-care provider’s admitting order
blood glucose level of 348 mg/dL. should the nurse question?
2. The client diagnosed with type 1 diabetes 1. Bedrest with bathroom privileges.
who is experiencing hypoglycemia. 2. Initiate IV therapy of D5W at 125 mL/hr.
3. The client with DKA who has multifocal 3. Weigh the client daily.
premature ventricular contractions. 4. Low-fat, low-carbohydrate diet.
4. The client with HHNS who has a plasma
osmolarity of 290 mOsm/L.

08_Colgrove_Ch08.indd 316 02/06/16 8:01 PM


C hapter 8  E ndocrine D isorders 317

29. The nurse is completing discharge teaching 3. Assess the client’s intravenous site.
to the client diagnosed with acute pancreatitis. 4. Provide oral and nasal care.
Which instruction should the nurse discuss with 5. Monitor the client’s blood glucose.
the client?
35. The nurse is administering a pancreatic enzyme
1. Instruct the client to decrease alcohol intake.
to the client diagnosed with chronic pancreatitis.
2. Explain the need to avoid all stress.
Which statement best explains the rationale for
3. Discuss the importance of stopping smoking.
administering this medication?
4. Teach the correct way to take pancreatic
1. It is an exogenous source of protease, amylase,
enzymes.
and lipase.
30. The male client diagnosed with chronic 2. This enzyme increases the number of bowel
pancreatitis calls and reports to the clinic nurse movements.
he has been having a lot of “gas,” along with 3. This medication breaks down in the stomach
frothy and very foul-smelling stools. Which to help with digestion.
intervention should the nurse implement? 4. Pancreatic enzymes help break down fat in
1. Explain this is common for chronic the small intestine.
pancreatitis.
36. The client diagnosed with acute pancreatitis
2. Ask the client to bring in a stool specimen
is being discharged home. Which statement
to the clinic.
by the client indicates the teaching has been
3. Arrange an appointment with the HCP for
effective?
today.
1. “I should decrease my intake of coffee, tea,
4. Discuss the need to decrease fat in the diet so
and cola.”
this won’t happen.
2. “I will eat a low-fat diet and avoid spicy
31. The nurse is discussing complications of chronic foods.”
pancreatitis with a client diagnosed with the 3. “I will check my amylase and lipase levels
disease. Which complication should the nurse daily.”
discuss with the client? 4. “I will return to work tomorrow but take
1. Diabetes insipidus (DI). it easy.”
2. Crohn’s disease.
3. Narcotic addiction.
4. Peritonitis. Cancer of the Pancreas
32. The client is immediate postprocedure 37. The nurse is assessing a client with complaints
endoscopic retrograde cholangiopancreatogram of vague upper abdominal pain worse at night
(ERCP). Which intervention should the nurse but relieved by sitting up and leaning forward.
implement? Which assessment question should the nurse
1. Assess for rectal bleeding. ask next?
2. Increase fluid intake. 1. “Have you noticed a yellow haze when you
3. Assess gag reflex. look at things?”
4. Keep in supine position. 2. “Does the pain get worse when you eat a meal
33. The client diagnosed with acute pancreatitis is in or snack?”
pain. Which position should the nurse assist the 3. “Have you had your amylase and lipase
client to assume to help decrease the pain? checked recently?”
1. Recommend lying in the prone position with 4. “How much weight have you gained since you
legs extended. saw an HCP?”
2. Maintain a tripod position over the bedside 38. The nurse caring for a client diagnosed with
table. cancer of the pancreas writes the problem of
3. Place in side-lying position with knees flexed. “altered nutrition: less than body requirements.”
4. Encourage a supine position with a pillow Which collaborative intervention should the
under the knees. nurse include in the plan of care?
34. The client with an acute exacerbation of chronic 1. Continuous feedings via (PEG) tube.
pancreatitis has a nasogastric (N/G) tube. Which 2. Have the family bring in foods from home.
interventions should the nurse implement? 3. Assess for food preferences.
Select all that apply. 4. Refer to the dietitian.
1. Monitor the client’s bowel sounds.
2. Monitor the client’s food intake.

08_Colgrove_Ch08.indd 317 02/06/16 8:01 PM


318 M ed -S urg S uccess

39. The nurse is planning a program for clients at 3. Left lower abdominal cramps and
a health fair regarding the prevention and early tenesmus.
detection of cancer of the pancreas. Which 4. Nausea and coffee-ground emesis.
self-care activity should the nurse discuss as an
44. The client diagnosed with cancer of the
example of a primary nursing intervention?
head of the pancreas is two (2) days post-
1. Monitor for elevated blood glucose at random
pancreatoduodenectomy (Whipple’s procedure).
intervals.
Which nursing problem has the highest
2. Inspect the skin and sclera of the eyes for
priority?
a yellow tint.
1. Anticipatory grieving.
3. Limit meat in the diet and eat a diet low in fat.
2. Fluid volume imbalance.
4. Instruct the client with hyperglycemia about
3. Alteration in comfort.
insulin injections.
4. Altered nutrition.
40. The nurse and an unlicensed assistive personnel
45. The client had a total pancreatectomy and
(UAP) are caring for clients on an oncology
splenectomy for cancer of the body of the
floor. Which intervention should the nurse
pancreas. Which discharge instructions
delegate to the UAP?
should the nurse teach? Select all that apply.
1. Assist the client with abdominal pain to turn
1. Keep a careful record of intake and
to the side and flex the knees.
output.
2. Monitor the Jackson Pratt drainage tube to
2. Use a stool softener or bulk laxative regularly.
ensure it is draining properly.
3. Use correct insulin injection technique.
3. Check to see if the client is sleeping after pain
4. Take the pain medication before the pain gets
medication is administered.
too bad.
4. Empty the bedside commode of the client
5. Sleep with the head of the bed on blocks.
who has been having melena.
46. The client admitted to rule out pancreatic islet
41. The client diagnosed with cancer of the pancreas
tumors complains of feeling weak, shaky, and
is being discharged to start chemotherapy in
sweaty. Which priority intervention should be
the HCP’s office. Which statement made by
implemented by the nurse?
the client indicates the client understands the
1. Start an IV with D5W.
discharge instructions?
2. Notify the health-care provider.
1. “I will have to see the HCP every day for six
3. Perform a bedside glucose check.
(6) weeks for my treatments.”
4. Give the client some orange juice.
2. “I should write down all my questions so I can
ask them when I see the HCP.” 47. The home health nurse is admitting a client
3. “I am sure this is not going to be a serious diagnosed with cancer of the pancreas. Which
problem for me to deal with.” information is the most important for the nurse
4. “The nurse will give me an injection in my leg to discuss with the client?
and I will get to go home.” 1. Determine the client’s food preferences.
2. Ask the client if there is an advance directive.
42. The client is being admitted to the outpatient
3. Find out about insurance/Medicare
department prior to an endoscopic retrograde
reimbursement.
cholangiopancreatogram (ERCP) to rule out
4. Explain the client should eat as much as
cancer of the pancreas. Which preprocedure
possible.
instruction should the nurse teach?
1. Prepare to be admitted to the hospital after 48. The nurse caring for a client diagnosed with
the procedure for observation. cancer of the pancreas writes the nursing
2. If something happens during the procedure, diagnosis of “risk for altered skin integrity
then emergency surgery will be done. related to pruritus.” Which intervention
3. Do not eat or drink anything after midnight should the nurse implement?
the night before the test. 1. Assess tissue turgor.
4. If done correctly, this procedure will correct 2. Apply antifungal creams.
the blockage of the stomach. 3. Monitor bony prominences for
breakdown.
43. The client is diagnosed with cancer of the head
4. Have the client keep the fingernails short.
of the pancreas. Which signs and symptoms
should the nurse expect to assess?
1. Clay-colored stools and dark urine.
2. Night sweats and fever.

08_Colgrove_Ch08.indd 318 02/06/16 8:01 PM


C hapter 8  E ndocrine D isorders 319

Adrenal Disorders 54. The nurse is performing discharge teaching for


a client diagnosed with Cushing’s disease. Which
49. The nurse is admitting a client diagnosed with statement by the client demonstrates
primary adrenal cortex insufficiency (Addison’s an understanding of the instructions?
disease). Which clinical manifestations should 1. “I will be sure to notify my health-care
the nurse expect to assess? provider if I start to run a fever.”
1. Moon face, buffalo hump, and hyperglycemia. 2. “Before I stop taking the prednisone, I will be
2. Hirsutism, fever, and irritability. taught how to taper it off.”
3. Bronze pigmentation, hypotension, and 3. “If I get weak and shaky, I need to eat some
anorexia. hard candy or drink some juice.”
4. Tachycardia, bulging eyes, and goiter. 4. “It is fine if I continue to participate in
weekend games of tackle football.”
50. The nurse is developing a plan of care
for the client diagnosed with acquired 55. The charge nurse of an intensive care unit
immunodeficiency syndrome (AIDS) who has is making assignments for the night shift.
developed an infection in the adrenal gland. Which client should be assigned to the most
Which client problem is highest priority? experienced intensive care nurse?
1. Altered body image. 1. The client diagnosed with respiratory failure
2. Activity intolerance. who is on a ventilator and requires frequent
3. Impaired coping. sedation.
4. Fluid volume deficit. 2. The client diagnosed with lung cancer
and iatrogenic Cushing’s disease with
51. The nurse is planning the care of a client ABGs of pH 7.35, Pao2 88, Paco2 44, and
diagnosed with Addison’s disease. Which HCO3 22.
intervention should be included? 3. The client diagnosed with Addison’s disease
1. Administer steroid medications. who is lethargic and has a BP of 80/45, P 124,
2. Place the client on fluid restriction. and R 28.
3. Provide frequent stimulation. 4. The client diagnosed with hyperthyroidism
4. Consult physical therapy for gait training. who has undergone a thyroidectomy two
52. The client is admitted to rule out Cushing’s (2) days ago and has a negative Trousseau’s
syndrome. Which laboratory tests should the sign.
nurse anticipate being ordered? 56. The nurse writes a problem of “altered body
1. Plasma drug levels of quinidine, digoxin, image” for a 34-year-old client diagnosed with
and hydralazine. Cushing’s disease. Which intervention should be
2. Plasma levels of ACTH and cortisol. implemented?
3. A 24-hour urine for metanephrine and 1. Monitor blood glucose levels prior to meals
catecholamine. and at bedtime.
4. Spot urine for creatinine and white blood cells 2. Perform a head-to-toe assessment on the
(WBCs). client every shift.
53. The client has developed iatrogenic Cushing’s 3. Use therapeutic communication to allow the
disease. Which statement is the scientific client to discuss feelings.
rationale for the development of this diagnosis? 4. Assess bowel sounds and temperature every
1. The client has an autoimmune problem four (4) hours.
causing the destruction of the adrenal 57. The client diagnosed with Addison’s disease is
cortex. admitted to the emergency department after a
2. The client has been taking steroid day at the lake. The client is lethargic, forgetful,
medications for an extended period for and weak. Which intervention should the nurse
another disease process. implement?
3. The client has a pituitary gland tumor causing 1. Start an IV with an 18-gauge needle and
the adrenal glands to produce too much infuse NS rapidly.
cortisol. 2. Have the client wait in the waiting room until
4. The client has developed an adrenal gland a bed is available.
problem for which the health-care provider 3. Obtain a permit for the client to receive a
does not have an explanation. blood transfusion.
4. Collect urinalysis and blood samples for a
CBC and calcium level.

08_Colgrove_Ch08.indd 319 02/06/16 8:01 PM


320 M ed -S urg S uccess

58. The client diagnosed with Cushing’s disease has 3. The client’s vital signs are T 97.6ºF, P 88,
developed 1++ peripheral edema. The client R 20, and BP 130/80.
has received intravenous fluids at 100 mL/hr 4. The client has a 3-cm amount of dark-red
via IV pump for the past 79 hours. The client drainage on the turban dressing.
received intravenous piggyback (IVPB)
63. Which laboratory value should be monitored by
medication in 50 mL of fluid every six (6) hours
the nurse for the client diagnosed with diabetes
for 15 doses. How many mL of fluid did the
insipidus?
client receive? ________
1. Serum sodium.
59. The nurse manager of a medical-surgical unit 2. Serum calcium
is asked to determine if the unit should adopt a 3. Urine glucose.
new care delivery system. Which behavior is an 4. Urine white blood cells.
example of an autocratic style of leadership?
64. The nurse is discharging a client diagnosed with
1. Call a meeting and educate the staff on the
diabetes insipidus. Which statement made by the
new delivery system being used.
client warrants further intervention?
2. Organize a committee to investigate the
1. “I will keep a list of my medications in my
various types of delivery systems.
wallet and wear a Medic Alert bracelet.”
3. Wait until another unit has implemented
2. “I should take my medication in the morning
the new system and see if it works out.
and leave it refrigerated at home.”
4. Discuss with the nursing staff if a new delivery
3. “I should weigh myself every morning and
system should be adopted.
record any weight gain.”
60. The client diagnosed with Cushing’s disease has 4. “If I develop a tightness in my chest, I will call
undergone a unilateral adrenalectomy. Which my health-care provider.”
discharge instructions should the nurse discuss
65. The client is admitted to the medical unit with
with the client?
a diagnosis of rule-out diabetes insipidus (DI).
1. Instruct the client to take the glucocorticoid
Which instructions should the nurse teach
and mineralocorticoid medications as
regarding a fluid deprivation test?
prescribed.
1. The client will be asked to drink 100 mL of
2. Teach the client regarding sexual functioning
fluid as rapidly as possible and then will not
and androgen replacement therapy.
be allowed fluid for 24 hours.
3. Explain the signs and symptoms of infection
2. The client will be administered an injection
and when to call the health-care provider.
of antidiuretic hormone (ADH), and urine
4. Demonstrate turn, cough, and deep-breathing
output will be measured for four (4) to
exercises the client should perform every
six (6) hours.
two (2) hours.
3. The client will have nothing by mouth
(NPO), and vital signs and weights will be
Pituitary Disorders done hourly until the end of the test.
4. An IV will be started with normal saline,
61. The client diagnosed with a pituitary tumor and the client will be asked to try to hold
developed syndrome of inappropriate the urine in the bladder until a sonogram can
antidiuretic hormone (SIADH). Which be done.
interventions should the nurse implement?
1. Assess for dehydration and monitor blood 66. The nurse is caring for clients on a medical floor.
glucose levels. Which client should be assessed first?
2. Assess for nausea and vomiting and weigh 1. The client diagnosed with syndrome of
daily. inappropriate antidiuretic hormone (SIADH)
3. Monitor potassium levels and encourage fluid who has a weight gain of 1.5 pounds since
intake. yesterday.
4. Administer vasopressin IV and conduct a fluid 2. The client diagnosed with a pituitary tumor
deprivation test. who has developed diabetes insipidus (DI) and
has an intake of 1,500 mL and an output of
62. The nurse is admitting a client to the 1,600 mL in the last 8 hours.
neurological intensive care unit who is 3. The client diagnosed with syndrome of
postoperative transsphenoidal hypophysectomy. inappropriate antidiuretic hormone (SIADH)
Which data warrant immediate intervention? who is having muscle twitching.
1. The client is alert to name but is unable to tell 4. The client diagnosed with diabetes insipidus
the nurse the location. (DI) who is complaining of feeling tired after
2. The client has an output of 2,500 mL since having to get up at night.
surgery and an intake of 1,000 mL.

08_Colgrove_Ch08.indd 320 02/06/16 8:01 PM


C hapter 8  E ndocrine D isorders 321

67. The health-care provider has ordered 40 g/24 hr nurse is an example of the ethical principle of
of intranasal vasopressin for a client diagnosed autonomy?
with diabetes insipidus. Each metered spray 1. Discuss the information the client told the
delivers 10 g. The client takes the medication nurse with the health-care provider and
every 12 hours. How many sprays are delivered at significant other.
each dosing time? ______ 2. Explain it is possible the client could have
a seizure if he drank fluid beyond the
68. The nurse is planning the care of a client
restrictions.
diagnosed with syndrome of inappropriate
3. Notify the health-care provider of the client’s
antidiuretic hormone (SIADH). Which
wishes and give the client fluids as desired.
interventions should be implemented? Select all
4. Allow the client an extra drink of water and
that apply.
explain the nurse could get into trouble if the
1. Restrict fluids per health-care provider order.
client tells the health-care provider.
2. Assess level of consciousness every two
(2) hours.
3. Provide an atmosphere of stimulation. Thyroid Disorders
4. Monitor urine and serum osmolality.
5. Weigh the client every three (3) days. 73. The client is diagnosed with hypothyroidism.
69. The nurse is caring for a client diagnosed with Which signs/symptoms should the nurse expect
diabetes insipidus (DI). Which intervention the client to exhibit?
should be implemented? 1. Complaints of extreme fatigue and hair loss.
1. Administer sliding-scale insulin as ordered. 2. Exophthalmos and complaints of nervousness.
2. Restrict caffeinated beverages. 3. Complaints of profuse sweating and flushed
3. Check urine ketones if blood glucose is >250. skin.
4. Assess tissue turgor every four (4) hours. 4. Tetany and complaints of stiffness of the
hands.
70. The unlicensed assistive personnel (UAP)
complains to the nurse she has filled the water 74. The nurse identifies the client problem “risk
pitcher four (4) times during the shift for a client for imbalanced body temperature” for the
diagnosed with a closed head injury and the client diagnosed with hypothyroidism. Which
client has asked for the pitcher to be filled intervention should be included in the plan
again. Which intervention should the nurse of care?
implement first? 1. Discourage the use of an electric blanket.
1. Tell the UAP to fill the pitcher with ice cold 2. Assess the client’s temperature every two
water. (2) hours.
2. Instruct the UAP to start measuring the 3. Keep the room temperature cool.
client’s I&O. 4. Space activities to promote rest.
3. Assess the client for polyuria and polydipsia. 75. The client diagnosed with hypothyroidism is
4. Check the client’s BUN and creatinine levels. prescribed the thyroid hormone levothyroxine
71. The nurse is admitting a client diagnosed with (Synthroid). Which assessment data indicate the
syndrome of inappropriate antidiuretic hormone medication has been effective?
(SIADH). Which clinical manifestations should 1. The client has a three (3)-pound weight gain.
be reported to the health-care provider? 2. The client has a decreased pulse rate.
1. Serum sodium of 112 mEq/L and a headache. 3. The client’s temperature is WNL.
2. Serum potassium of 5.0 mEq/L and a 4. The client denies any diaphoresis.
heightened awareness. 76. Which nursing intervention should be included
3. Serum calcium of 10 mg/dL and tented tissue in the plan of care for the client diagnosed with
turgor. hyperthyroidism?
4. Serum magnesium of 1.2 mg/dL and large 1. Increase the amount of fiber in the diet.
urinary output. 2. Encourage a low-calorie, low-protein diet.
72. The male client diagnosed with syndrome of 3. Decrease the client’s fluid intake to
inappropriate antidiuretic hormone (SIADH) 1,000 mL/day.
secondary to cancer of the lung tells the nurse 4. Provide six (6) small, well-balanced meals
he wants to discontinue the fluid restriction and a day.
does not care if he dies. Which action by the

08_Colgrove_Ch08.indd 321 02/06/16 8:01 PM


322 M ed -S urg S uccess

77. The client is admitted to the intensive care should be taught to the client? Select all that
department diagnosed with myxedema coma. apply.
Which assessment data warrant immediate 1. Notify the HCP if a three (3)-pound weight
intervention by the nurse? loss occurs in two (2) days.
1. Serum blood glucose level of 74 mg/dL. 2. Discuss ways to cope with the emotional
2. Pulse oximeter reading of 90%. lability.
3. Telemetry reading showing sinus 3. Notify the HCP if taking over-the-counter
bradycardia. medication.
4. The client is lethargic and sleeps all the 4. Carry a medical identification card or bracelet.
time. 5. Teach how to take thyroid medications
correctly.
78. Which medication order should the nurse
question in the client diagnosed with untreated 82. The nurse is providing an in-service on thyroid
hypothyroidism? disorders. One of the attendees asks the nurse,
1. Thyroid hormones. “Why don’t the people in the United States get
2. Oxygen. goiters as often?” Which statement by the nurse
3. Sedatives. is the best response?
4. Laxatives. 1. “It is because of the screening techniques used
in the United States.”
79. Which statement made by the client makes
2. “It is a genetic predisposition rare in North
the nurse suspect the client is experiencing
Americans.”
hyperthyroidism?
3. “The medications available in the United
1. “I just don’t seem to have any appetite
States decrease goiters.”
anymore.”
4. “Iodized salt helps prevent the development
2. “I have a bowel movement about every
of goiters in the United States.”
three (3) to four (4) days.”
3. “My skin is really becoming dry and coarse.” 83. The nurse is preparing to administer the
4. “I have noticed all my collars are getting following medications. Which medication
tighter.” should the nurse question administering?
1. The thyroid hormone to the client who does
80. The 68-year-old client diagnosed with
not have a T3, T4 level.
hyperthyroidism is being treated with
2. The regular insulin to the client with a blood
radioactive iodine therapy. Which interventions
glucose level of 210 mg/dL.
should the nurse discuss with the client?
3. The loop diuretic to the client with a
1. Explain it will take up to a month for
potassium level of 3.3 mEq/L.
symptoms of hyperthyroidism to subside.
4. The cardiac glycoside to the client who has a
2. Teach the iodine therapy will have to be
digoxin level of 1.4 mg/dL.
tapered slowly over one (1) week.
3. Discuss the client will have to be hospitalized 84. Which signs/symptoms should make the nurse
during the radioactive therapy. suspect the client is experiencing a thyroid storm?
4. Inform the client after therapy the client will 1. Obstipation and hypoactive bowel sounds.
not have to take any medication. 2. Hyperpyrexia and extreme tachycardia.
3. Hypotension and bradycardia.
81. The nurse is teaching the client diagnosed
4. Decreased respirations and hypoxia.
with hyperthyroidism. Which information

CONCEPTS
The concepts covered in this chapter focus on metabolism and acid–base problems. Exemplars that are covered
are diabetes mellitus (DM) type 1 and type 2 and thyroid disorders. Interrelated concepts of the nursing process
and critical thinking are covered throughout the questions. The concept of critical thinking is presented in the
prioritizing or “first” questions.
85. The nurse is administering morning 2. The subcutaneous insulin to a client refusing
medications. Which medications should the blood glucose checks.
nurse administer question? 3. The levothyroxine PO to a client diagnosed
1. The oral carafate to a client who has not eaten with hypothyroidism.
breakfast. 4. The sliding scale insulin to a client whose
blood glucose level is 320 mg/dL.

08_Colgrove_Ch08.indd 322 02/06/16 8:01 PM


C hapter 8  E ndocrine D isorders 323

86. The nurse is administering medications at the beginning of the day shift. Which should the nurse
implement first?

Client Number: Diagnosis: Diabetes


Client Name: CBA 3456789 Allergies: NKDA Mellitus Type 2
MEDICATION ADMINISTRATION RECORD

Medication 0701–1500 1501–2300 2301–0700


Regular insulin 0730 1630
subcutaneously a.c. and h.s. 1130 2100
0–60 = 1 amp D50
61–150 = 0 units
151–300 = 5 units
301–450 = 10 units
Greater than 450 = Call HCP
Glargine (Lantus) 15 units SQ a.c. 0730
Metformin 500 mg PO bid 0800 1700
Signature/Initials:

1. The nurse should determine the blood 90. Which laboratory data would the nurse identify
glucose level. when discussing a client with uncontrolled
2. The nurse should combine the sliding scale diabetes mellitus type 2?
insulin with the glargine. 1.
3. The nurse should prepare to give both
Patient Normal
insulins and metformin. Results Values
4. The nurse should check the client’s health
record for new orders. Glucose 89 70–110 mg/dL
Hemoglobin A1C 5.6 4–6
87. The nurse identified a concept of metabolism
for a client diagnosed with diabetes mellitus type 2.
1. Which interventions should the nurse include
in the plan of care? Select all that apply. Patient Normal
1. Teach the client to perform self glucose Results Values
monitoring. Glucose 134 70–110 mg/dL
2. Instruct the client about complications of Hemoglobin A1C 7.1 4–6
high-glucose levels.
3. Instruct the client to inspect the feet daily. 3.
4. Explain the need to carry a source of quick-
acting proteins. Patient Normal
Results Values
5. Encourage the client to have regular eye exams.
Glucose 112 70–110 mg/dL
88. The client diagnosed with hyperthyroidism
Hemoglobin A1C 8.2 4–6
is complaining of being hot and cannot sit
still. Which should the nurse do based on the
4.
assessment?
1. Continue to monitor the client. Patient Normal
2. Have the UAP take the client’s vital signs. Results Values
3. Request an order for a sedative. Glucose 439 70–110 mg/dL
4. Insist the client lie down and rest.
Hemoglobin A1C 9.3 4–6
89. The concepts of nutrition and metabolism have
been identified for the client. Which referral
should the nurse include in the plan of care?
1. Physical therapy.
2. Social work.
3. Speech therapy.
4. Dietary.

08_Colgrove_Ch08.indd 323 02/06/16 8:01 PM


324 M ed -S urg S uccess

91. The nurse identified a concept of metabolism 94. The nurse is teaching the client diagnosed with
for a client diagnosed with diabetes. Which diabetes. Which should the nurse teach to limit
antecedent would be identified as placing the the complications of diabetes?
client at risk for diabetes? 1. Teach the client to keep the blood glucose
1. Nutrition. under 140 mg/dL.
2. Sensory perception. 2. Demonstrate how to test the urine for ketones.
3. pH regulation. 3. Instruct the client to apply petroleum jelly
4. Medication. between the toes.
4. Allow the client to eat meals as desired and
92. The client is diagnosed with hypothyroidism.
then take insulin.
Which assessment data support this diagnosis?
1. The client’s vital signs are: T 99.0, P 110, 95. Which interrelated concepts could be identified
R 26, and BP 145.80. as actual or potential for a 56-year-old male
2. The client complains of constipation and client diagnosed with diabetes mellitus type 2?
being constantly cold. Select all that apply.
3. The client has an intake of 780 mL and 1. Nutrition.
output of 256 mL. 2. Metabolism.
4. The client complains of a headache and has 3. Infection.
projectile vomiting. 4. Male reproduction.
5. Skin integrity.
93. The client diagnosed with diabetes complains
of a curtain being drawn across the eyes. Which 96. Which client would the nurse identify as being
should the nurse implement first? at risk for developing diabetes?
1. Assess the eyes using an ophthalmoscope. 1. The client who eats mostly candy and
2. Tell the client to keep the eyes closed. potatoes.
3. Notify the health-care provider (HCP). 2. The 22-year-old client who has been taking
4. Call the Rapid Response Team (RRT). birth control pills.
3. The client who has a cousin who has had
diabetes for two (2) years.
4. The 38-year-old female who delivered a
10-pound infant.

08_Colgrove_Ch08.indd 324 02/06/16 8:01 PM


PRACTICE QUESTIONS ANSWERS
AND RATIONALES

Diabetes Mellitus 4. An A1c of 13% is dangerously high; it


reflects a 300-mg/dL average blood glucose
1. 1. Type 1 diabetes usually occurs in young clients level over the past three (3) months.
who are underweight. In this disease, there is TEST-TAKING HINT: The test taker must know
no production of insulin from the beta cells normal and abnormal diagnostic laboratory
in the pancreas. People with type 1 diabetes values. Laboratory values vary depending on
are insulin dependent with a rapid onset of which laboratory performs the test.
symptoms, including polyuria, polydipsia, and Content – Medical: Integrated Nursing Process –
polyphagia. ­Assessment: Client Needs – Physiological Integrity, ­Reduction
2. Type 2 diabetes is a disorder usually occur- of Risk Potential: Cognitive Level – Analysis: Concept –
ring around the age of 40, but it is now be- Metabolism.
ing detected in children and young adults
3. 1. Humulin N peaks in six (6) to eight
as a result of obesity and sedentary life-
(8) hours, making the client at risk for
styles. Nonhealing wounds are a hallmark
hypoglycemia around midnight, which is
sign of type 2 diabetes. This client weighs
why the client should receive a bedtime
248.6 pounds and is short.
snack. This snack will prevent nighttime
3. Gestational diabetes occurs during pregnancy.
hypoglycemia.
4. Acanthosis nigricans (AN), dark pigmentation
2. The food intake at lunch will not affect the cli-
and skin creases in the neck, is a sign
ent’s blood glucose level at midnight.
of hyperinsulinemia. The pancreas is secreting
3. The client’s glucometer reading should be
excess amounts of insulin as a result of exces-
done around 2100 to assess the effectiveness of
sive caloric intake. It is identified in young
insulin at 1600.
children and is a precursor to the
4. Onset of Humulin N, an intermediate-acting
development of type 2 diabetes.
insulin, is two (2) to four (4) hours but it does
TEST-TAKING HINT: The test taker must be not peak until six (6) to eight (8) hours.
aware of kilograms and pounds. The stem is
TEST-TAKING HINT: Remember to look at the
asking about a disease process and acantho-
adjective or descriptor. Intermediate-acting
sis nigricans is a clinical manifestation of a
insulin provides the reader a clue: Anything
disease, not a disease itself. Therefore, the
with intermediate action, instead of longer
test taker should not select this as a correct
acting, is incorrect.
answer.
Content – Medical: Integrated Nursing Process –
Content – Medical: Integrated Nursing Process –
Implementation: Client Needs – Physiological Integrity,
­Diagnosis: Client Needs – Physiological Integrity,
Pharmacological and Parenteral Therapies: Cognitive
Reduction of Risk Potential: Cognitive Level – Analysis:
Level – Application: Concept – Metabolism.
Concept – Metabolism.
4. Three (3) units.
2. 1. The acceptable level for an A1c for a client with
The client’s result is 189, which is between 151
diabetes is between 6% and 7%, which corre-
and 200, so the nurse should administer three
sponds to a 120- to 140-mg/dL average blood
(3) units of Humalog insulin subcutaneously.
glucose level.
2. This result is not within acceptable levels for TEST-TAKING HINT: The test taker must be
the client with diabetes, which is 6% to 7%. aware of the way HCPs write medication or-
3. This result parallels a serum blood glucose ders. HCPs order insulin on a sliding scale ac-
level of approximately 180 to 200 mg/dL. cording to a range of blood glucose levels.
An A1c is a blood test reflecting average Content – Medical: Integrated Nursing Process –
blood glucose levels over a period of Implementation: Client Needs – Physiological Integrity,
three (3) months; clients with elevated Pharmacological and Parenteral Therapies: Cognitive
blood glucose levels are at risk for develop- Level – Application: Concept – Medication.
ing long-term complications.

325

08_Colgrove_Ch08.indd 325 02/06/16 8:01 PM


326 M ed -S urg S uccess

5. 1. The client diagnosed with type 2 diabetes who 3. Glycosylated hemoglobin is a serum blood
is not taking insulin or oral agents does not test usually performed in a laboratory, not
need extra food before exercise. in the client’s home. The hemoglobin Alc
2. The client with diabetes who is at risk for is p
­ erformed every three (3) months. Self-­
hypoglycemia when exercising should carry a monitoring blood glucose (SMBG) should be
simple carbohydrate, but this client is not at taught to the client.
risk for hypoglycemia. 4. The client’s feet should be checked daily, not
3. Clients with diabetes controlled by diet and weekly. In a week the client could have devel-
exercise must exercise daily at the same time oped gangrene from an injury the client did
and in the same amount to control the g ­ lucose not realize he or she had.
level. TEST-TAKING HINT: Always notice the age of a
4. All clients who exercise should perform client if it is provided because this is impor-
warm-up and cool-down exercises to help tant when determining the correct answer
prevent muscle strain and injury. for the question. Be sure to note the adverbs,
TEST-TAKING HINT: Options “1” and “2” apply such as “weekly” instead of “daily.”
directly to clients diagnosed with diabetes, Content – Medical: Integrated Nursing Process –
and options “3” and “4” do not directly ad- ­Planning: Client Needs – Physiological Integrity, Pharmaco-
dress clients diagnosed with diabetes. The logical and Parenteral Therapies: Cognitive Level – Synthesis:
reader could narrow the choices by either Concept – Metabolism.
eliminating or including the two similar 8. 1. High-fat diets are not recommended for cli-
options. ents diagnosed with diabetes, and food does
Content – Medical: Integrated Nursing Process – not have an effect on a CT scan with contrast.
Planning: Client Needs – Physiological Integrity, P
­ hysiological 2. Biguanide medication must be held for a
Adaptation: Cognitive Level – Synthesis: Concept –
test with contrast medium because it in-
Metabolism.
creases the risk of lactic acidosis, which
6. 1. Crumbling toenails indicate tinea unguium, leads to renal problems.
which is a fungus infection of the toenail. 3. Informed consent is not required for a CT
2. Athlete’s foot is a non–life-threatening ­fungal scan. The admission consent covers routine
infection. diagnostic procedures.
3. A necrotic big toe indicates “dead” ­tissue. 4. Pancreatic enzymes are administered when the
The client does not feel pain, does not pancreas cannot produce amylase and lipase,
realize the injury, and does not seek treat- not when the beta cells cannot ­produce insulin.
ment. Increased blood glucose levels de- TEST-TAKING HINT: The test taker could elimi-
crease the oxygen supply needed to heal nate option “1” because high-fat diets are
the wound and increase the risk for devel- not recommended for any client. Because
oping an infection. the stem specifically refers to the biguanide
4. Big, thick toenails are fungal infections and medication and CT contrast, a good choice
do not require immediate intervention by the addresses both of these. Option “2” discusses
nurse. both the medication and the test.
TEST-TAKING HINT: The test taker should Content – Medical: Integrated Nursing Process –
s­ elect the option indicating this is possibly a Implementation: Client Needs – Physiological Integrity,
life-­threatening complication or some type Reduction of Risk Potential: Cognitive Level – Application:
of assessment data the health-care provider Concept – Medication.
should be informed of immediately. Remem- 9. 1. The most important issue to teach clients
ber “warrants immediate intervention.” is to take insulin even if they are unable
Content – Medical: Integrated Nursing Process – to eat. Glucose levels are increased with
­Assessment: Client Needs – Safe Effective Care Environment, ­illness and stress.
Management of Care: Cognitive Level – Synthesis:
2. The client should drink liquids such as
Concept – Metabolism.
regular cola or orange juice, or eat regu-
7. 1. Age-related visual changes and diabetic lar gelatin, which provide enough glucose
retinopathy could cause the client to have to prevent hypoglycemia when receiving
difficulty in reading and drawing up insulin insulin.
dosage accurately. 3. Ketones indicate a breakdown of fat and must
2. The PT level is monitored for clients ­receiving be reported to the HCP because they can lead
Coumadin, an anticoagulant, which is not or- to metabolic acidosis.
dered for clients with ­diabetes, type 1 or 2.

08_Colgrove_Ch08.indd 326 02/06/16 8:01 PM


C hapter 8  E ndocrine D isorders 327

4. Blood glucose levels and ketones must be 4. The client needs the insulin to control the
checked every three (3) to four (4) hours, not diabetes, and talking to the HCP about tak-
daily. ing him off a needed medication is not an
5. The HCP should be notified if the blood example of advocacy.
glucose level is this high. Regular insulin TEST-TAKING HINT: Remember, the test taker
may need to be prescribed to keep the must understand what the question is asking
blood glucose level within acceptable and the definition of the terms.
range. Content – Medical: Integrated Nursing Process –
TEST-TAKING HINT: This is an alternate-type Implementation: Client Needs – Safe Effective Care
­question having more than one correct an- ­Environment, Management of Care: Cognitive Level –
swer. The test taker should read all options Application: Concept – Nursing Roles.
and determine if each is an appropriate 12. 1. The short-term goal must address the
intervention. response part of the nursing diagnosis,
Content – Medical: Integrated Nursing Process – ­ which is “high risk for hyperglycemia,”
Planning: Client Needs – Health Promotion and and this blood glucose level is within
­Maintenance: Cognitive Level – Synthesis: Concept –
acceptable ranges for a client who is
Nursing Roles.
noncompliant.
10. 1. The blood glucose level should be obtained, 2. This is an appropriate goal for a knowledge-
but it is not the first intervention. deficit nursing diagnosis. Noncompliance is
2. If it is determined the client is having a hypo- not always the result of knowledge deficit.
glycemic reaction, orange juice is appropriate. 3. The nurse is implementing an interven-
3. Regular insulin peaks in two (2) to four tion, and the question asks for a goal which
(4) hours. Therefore, the nurse should addresses the problem of “high risk for
think about the possibility the client hyperglycemia.”
is having a hypoglycemic reaction and 4. The question asks for a short-term goal and
should assess the client. The nurse should this is an example of a long-term goal.
not delegate nursing tasks to TEST-TAKING HINT: Remember, the nurs-
a UAP if the client is unstable. ing diagnosis consists of a problem related
4. Dextrose 50% is only administered if the to an etiology. The goals must address the
client is unconscious and the nurse suspects problem and the interventions must address
hypoglycemia. the etiology. The test taker should always
TEST-TAKING HINT: When answering a ques- remember a short-term goal is usually a goal
tion requiring the nurse to decide which met during the hospitalization, and the long-
intervention to implement first, all four op- term goal may take weeks, months, or even
tions are plausible for the situation but only years.
one answer should be implemented first. The Content – Medical: Integrated Nursing Process –
test taker must apply the nursing process; Diagnosis: Client Needs – Physiological Integrity,
assessment is the first step of the nursing ­Physiological Adaptation: Cognitive Level – Analysis:
process. Concept – Metabolism.
Content – Medical: Integrated Nursing Process – 13. 1. Kussmaul’s respirations occur with diabetic
Implementation: Client Needs – Safe Effective Care ketoacidosis (DKA) as a result of the break-
­Environment, Management of Care: Cognitive Level –
down of fat, resulting in ketones.
Synthesis: Concept – Medication.
2. Diarrhea and epigastric pain are not associ-
11. 1. This is an example of interviewing the client; ated with HHNS.
it is not an example of client advocacy. 3. Dry mucous membranes are a result of
2. Client advocacy focuses support on the the hyperglycemia and occur with both
client’s autonomy. Even if the nurse dis- HHNS and DKA.
agrees with his living on the street, it is 4. This occurs with DKA as a result of the
the client’s right. Arranging for someone breakdown of fat, resulting in ketones.
to give him his insulin provides for his TEST-TAKING HINT: The test taker must be
needs and allows his choices. able to differentiate between HHNS (type 2
3. Adult Protective Services is an organization diabetes) and DKA (type 1 diabetes), which
investigating any actual or potential abuse primarily is the result of the breakdown of
in adults. This client is not being abused fat and results in an increase in ketones caus-
by anyone. ing a decrease in pH, resulting in metabolic
acidosis.

08_Colgrove_Ch08.indd 327 02/06/16 8:01 PM


328 M ed -S urg S uccess

Content – Medical: Integrated Nursing Process – there is enough insulin to prevent fat break-
­Assessment: Client Needs – Physiological Integrity, down but not enough to lower blood glucose.
Reduction of Risk Potential: Cognitive Level – Analysis: 3. The client may or may not feel like eating,
Concept – Metabolism. but it is not the appropriate intervention
14. 1. The initial fluid replacement is 0.9% nor- when the blood glucose level is reduced to
mal saline (an isotonic solution) intrave- 300 mg/dL.
nously, followed by 0.45% saline. The rate 4. When the glucose level is decreased to
depends on the client’s fluid volume status around 300 mg/dL, the regular insulin in-
and physical health, especially of the heart. fusion therapy is decreased. Subcutaneous
2. Regular insulin, not intermediate, is the in- insulin will be administered per sliding
sulin of choice because of its quick onset and scale.
peak in two (2) to four (4) hours. TEST-TAKING HINT: When two (2) options are
3. Blood glucometer checks are done every one the opposite of each other, they can either be
(1) hour or more often in clients with HHNS eliminated or can help eliminate the other
who are receiving regular insulin drips. two options as incorrect answers. Options
4. Arterial blood gases are not affected in “2” and “3” do not have insulin in the an-
HHNS because there is no breakdown of swer; therefore, they should be eliminated as
fat resulting in ketones leading to metabolic possible answers.
acidosis. Content – Medical: Integrated Nursing Process –
TEST-TAKING HINT: The test taker should Implementation: Client Needs – Safe Effective Care
eliminate option “3” based on the word ­Environment, Management of Care: Cognitive Level –
“daily.” In the ICU with a client who is very Application: Concept – Metabolism.
ill, most checks are more often than daily. 17. 1. The nurse should assume the client
Remember to look at adjectives; “intermedi- is hypoglycemic and administer IVP
ate” in option “2” is the word eliminating dextrose, which will rouse the client
this as a possible correct answer. immediately. If the collapse is the result
Content – Medical: Integrated Nursing Process – ­ of hyperglycemia, this additional dextrose
Diagnosis: Client Needs – Physiological Integrity, will not further injure the client.
­Physiological Adaptation: Cognitive Level – Analysis: 2. The health-care provider may or may not
Concept – Metabolism. need to be notified, but this is not the first
15. 1. Glucose is elevated in DKA; therefore, the intervention.
HCP would not be replacing glucose. 3. The client should be left in the client’s room,
2. The client in DKA loses potassium from and 50% dextrose should be administered
increased urinary output, acidosis, cata- first.
bolic state, and vomiting. Replacement is 4. The serum glucose level requires a veni-
essential for preventing cardiac dysrhyth- puncture, which will take too long. A blood
mias secondary to hypokalemia. glucometer reading may be obtained, but the
3. Calcium is not affected in the client with DKA. nurse should first treat the client, not the ma-
4. The prescribed IV for DKA—0.9% normal chine. The glucometer only reads “low” after
saline—has sodium, but it is not specifically a certain point, and a serum level is needed to
ordered for sodium replacement. This is an confirm exact glucose level.
isotonic solution. TEST-TAKING HINT: The question is request-
TEST-TAKING HINT: Option “1” should be ing the test taker to select which interven-
eliminated because the problem with DKA is tion should be implemented first. All four
elevated glucose, so the HCP would not be options could be possible interventions, but
replacing it. The test taker should use physi- only one (1) intervention should be imple-
ology knowledge and realize potassium is in mented first. The test taker should select the
the cell. intervention directly treating the client; do
Content – Medical: Integrated Nursing Process – not select a diagnostic test.
Diagnosis: Client Needs – Safe Effective Care Environ- Content – Medical: Integrated Nursing Process –
ment, Management of Care: Cognitive Level – Analysis: Implementation: Client Needs – Safe Effective Care
Concept – Metabolism. ­Environment, Management of Care: Cognitive Level –
Synthesis: Concept – Metabolism.
16. 1. The regular intravenous insulin is continued
because ketosis is not present, as with DKA. 18. 1. This indicates the client is dehydrated, which
2. The client diagnosed with type 2 diabetes does not indicate the client is getting better.
does not excrete ketones in HHNS because

08_Colgrove_Ch08.indd 328 02/06/16 8:01 PM


C hapter 8  E ndocrine D isorders 329

2. The client’s level of consciousness can TEST-TAKING HINT: If the test taker does not
be altered because of dehydration and know the answer to this question, the test
­acidosis. If the client’s sensorium is taker could possibly relate to the phrase
intact, the client is getting better and “acute complication” and realize a medical
responding to the medical treatment. problem might cause this and select infec-
3. These ABGs indicate metabolic acidosis; tion, option “3.”
therefore, the client is not responding to Content – Medical: Integrated Nursing Process – ­
treatment. Assessment: Client Needs – Physiological Integrity,
4. This potassium level is low and indicates ­Reduction of Risk Potential: Cognitive Level – Analysis:
hypokalemia, which shows the client is not Concept – Metabolism.
responding to medical treatment. 21. 1. The American Diabetes Association is an
TEST-TAKING HINT: The phrase “responding excellent referral, but the nurse should
to medical treatment” is asking the test taker discuss specific ways to prevent DKA.
to determine which data indicate the client 2. The client should be careful with OTC medi-
is getting better. The correct answer will be cations, but this intervention does not help
normal data, and the other prevent the development of DKA.
three (3) options will be signs/symptoms of 3. Illness increases blood glucose levels;
the disease process or condition. therefore, the client must take insulin and
Content – Medical: Integrated Nursing Process – ­ consume high-carbohydrate foods such as
Assessment: Client Needs – Physiological Integrity, regular Jell-O, regular popsicles, and or-
­Reduction of Risk Potential: Cognitive Level – Analysis: ange juice.
Concept – Metabolism. 4. Vaccines are important to help prevent
19. 1. The client is on a special diet and should not illness, but regardless of whether the client
have any additional food. gets these vaccines, the client can still develop
2. The client will not be compliant with the diabetic ketoacidosis.
diet if he or she is still hungry. Therefore, TEST-TAKING HINT: The words “most impor-
the nurse should request the dietitian talk tant” in the stem of the question i­ndicate
to the client to try to adjust the meals so one (1) or more options may be appropriate
the client will adhere to the diet. instructions but only one (1) is the priority
3. The nurse does not need to notify the HCP intervention.
for an increase in caloric intake. The appro- Content – Medical: Integrated Nursing Process – ­
priate referral is to the dietitian. Planning: Client Needs – Physiological Integrity,
4. The client is on a special diet. The nurse ­Physiological Adaptation: Cognitive Level – Synthesis:
needs to help the client maintain compliance Concept – Metabolism.
with the medical treatment and should refer 22. 1. This blood glucose level is elevated, but not
the client to the dietitian.
life threatening, in the client diagnosed with
TEST-TAKING HINT: The test taker should se- type 2 diabetes. Therefore, a less experienced
lect the option attempting to ensure the client nurse could care for this client.
maintains compliance. The test taker should 2. Hypoglycemia is an acute complication of
remember to work with members of the mul- type 1 diabetes, but it can be managed by fre-
tidisciplinary health-care team. quent monitoring, so a less experienced nurse
Content – Medical: Integrated Nursing Process – could care for this client.
Implementation: Client Needs – Physiological Integrity, 3. Multifocal PVCs, which are secondary to
Basic Care and Comfort: Cognitive Level – Application: hypokalemia and can occur in clients with
Concept – Metabolism. DKA, are a potentially life-­threatening
20. 1. A client with type 2 diabetes usually is pre- emergency. This client needs an experi-
scribed oral hypoglycemic medications, not enced nurse.
insulin. 4. A plasma osmolarity of 280 to 300 mOsm/L
2. The client could not eat enough food to cause is within normal limits; therefore, a less expe-
a 680-mg/dL blood glucose level; therefore, rienced nurse could care for this client.
this question does not need to be asked. TEST-TAKING HINT: The test taker must select
3. The most common precipitating factor is the client with an abnormal, an unexpected,
infection. The manifestations may be slow or a life-threatening sign/symptom for his or
to appear, with onset ranging from 24 her disease process and assign this client to
hours to two (2) weeks. the most experienced nurse.
4. This does not help determine the cause of Content – Nursing Management: Integrated Nursing
this client’s HHNS. Process – Planning: Client Needs – Safe Effective Care

08_Colgrove_Ch08.indd 329 02/06/16 8:01 PM


330 M ed -S urg S uccess

Environment, Management of Care: Cognitive Level – acute pancreatitis, but these laboratory values
Synthesis: Concept – Nursing Roles. do not confirm the diagnosis.
23. 1. This ABG indicates respiratory acidosis, TEST-TAKING HINT: The test taker must be
which is not expected. able to identify at least two (2) laboratory
2. This ABG is normal, which is not expected. values that reflect each organ function prior
3. This ABG indicates respiratory alkalosis, to taking the NCLEX-RN. There is no test-
which is not expected. taking hint to help select the correct answer;
4. This ABG indicates metabolic acidosis, this is knowledge.
which is expected in a client diagnosed Content – Medical: Integrated Nursing Process –
with diabetic ketoacidosis. ­Assessment: Client Needs – Physiological Integrity, Reduc-
TEST-TAKING HINT: The test taker must know tion of Risk Potential: Cognitive Level – Comprehension:
Concept – Metabolism.
normal ABGs to be able to correctly answer
this question. Normal ABGs are pH 7.35 to 26. 1. The client will be NPO to help decrease pain,
7.45; Pao2 80 to 100; Paco2 35 to 45; HCO3 but it is not the priority problem because the
22 to 26. client will have intravenous fluids.
Content – Medical: Integrated Nursing Process – ­ 2. Autodigestion of the pancreas results in
Assessment: Client Needs – Physiological Integrity, severe epigastric pain, accompanied by
­Reduction of Risk Potential: Cognitive Level – Analysis: nausea, vomiting, abdominal tenderness,
Concept – Metabolism. and muscle guarding.
24. 1. The nurse should always address the 3. Nutritional imbalance is a possible client
airway when a client is seriously ill. problem, but it is not priority.
2. The client must be assessed for fluid 4. Knowledge deficit is always a client problem,
­volume deficit and then for fluid volume but it is not priority over pain.
excess after fluid replacement is started. TEST-TAKING HINT: The test taker should ap-
3. The electrolyte imbalance of primary con- ply Maslow’s hierarchy of needs when select-
cern is depletion of potassium. ing the priority problem for a ­client. After
4. Ketones are excreted in the urine; levels airway, pain is often priority.
are documented from negative to large Content – Medical: Integrated Nursing Process –
amount. Ketones should be monitored ­Diagnosis: Client Needs – Safe Effective Care E
­ nvironment,
frequently. Management of Care: Cognitive Level – Analysis:
5. The nurse must ensure the client’s fluid Concept – Metabolism.
intake and output are equal. 27. 1. Pancreatic enzymes must be administered
TEST-TAKING HINT: The test taker must select with meals to enhance the digestion of
all answer options that apply. Do not try to starches and fats in the gastrointestinal
outguess the item writer. In some instances (GI) tract.
all options are correct. 2. The client’s respiratory rate is within normal
Content – Medical: Integrated Nursing Process – limits; therefore, the morphine should be
Implementation: Client Needs – Safe Effective Care Envi- administered to the client who is having pain.
ronment, Management of Care: Cognitive Level – Analysis: 3. This is a normal potassium level; therefore,
Concept – Metabolism. the nurse does not need to question adminis-
tering this medication.
4. The apical pulse is within normal limits;
Pancreatitis therefore, the nurse should not question
administering this medication.
25. 1. These laboratory values are monitored for
clients in kidney failure. TEST-TAKING HINT: The test taker must de-
2. These laboratory values are elevated in termine if the assessment data provided in
clients with a myocardial infarction. the option are abnormal, unexpected, or
3. Serum amylase increases within two (2) to life-threatening to warrant questioning the
12 hours of the onset of acute pancreatitis administration of the medication.
to two (2) to three (3) times normal and Content – Medical: Integrated Nursing Process –
returns to normal in three (3) to four (4) Implementation: Client Needs – Physiological Integrity,
days; lipase elevates and remains Pharmacological and Parenteral Therapies: Cognitive
Level – Application: Concept – Medication.
elevated for seven (7) to 14 days.
4. Bilirubin may be elevated as a result of com- 28. 1. Bedrest will decrease metabolic rate, gastro-
pression of the common duct, and hypocal- intestinal secretion, pancreatic secretions, and
cemia develops in up to 25% of clients with pain; therefore, this HCP’s order should not
be questioned.

08_Colgrove_Ch08.indd 330 02/06/16 8:01 PM


C hapter 8  E ndocrine D isorders 331

2. The client will be NPO; therefore, initiating enzyme secretion and indicates impaired
IV therapy is an appropriate order. digestion and possibly an increase in the
3. Short-term weight gain changes reflect fluid severity of the pancreatitis. The client
balance because the client will be NPO and should see the HCP.
receiving IV fluids. Daily weight is an appro- 4. Decreasing fat in the diet will not help stop
priate HCP’s order. this type of stool.
4. The client will be NPO, which will TEST-TAKING HINT: This question requires
decrease stimulation of the pancreatic the test taker to have knowledge of the dis-
­enzymes, resulting in decreased au- ease process, but if the test taker knows the
todigestion of the pancreas, therefore exocrine function of the pancreas is part of
d­ ecreasing pain. the gastrointestinal system, the test taker
TEST-TAKING HINT: The test taker must de- might think altered stool is a cause for
termine which HCP’s order is not expected concern.
for the diagnosis. Sometimes, if the test taker Content – Medical: Integrated Nursing Process –
asks which order is expected, it is easier to Implementation: Client Needs – Physiological Integrity,
identify the unexpected or abnormal HCP Physiological Adaptation: Cognitive Level – Application:
order. Concept – Metabolism.
Content – Medical: Integrated Nursing Process – 31. 1. The client is at risk for diabetes mellitus
Implementation: Client Needs – Safe Effective Care (destruction of beta cells), not diabetes
­Environment, Management of Care: Cognitive Level –
insipidus, a disorder of the pituitary gland.
Application: Concept – Metabolism.
2. Crohn’s disease is an inflammatory disorder
29. 1. Alcohol must be avoided entirely because it of the lining of the gastrointestinal system,
can cause stones to form, blocking pancreatic especially of the terminal ileum.
ducts and the outflow of pancreatic juice, 3. Narcotic addiction is related to the
causing further inflammation and destruction frequent, severe pain episodes often
of the pancreas. occurring with chronic pancreatitis, which
2. Stress stimulates the pancreas and should be require narcotics for relief.
dealt with, but it is unrealistic to think a client 4. Peritonitis, an inflammation of the lining of
can avoid all stress. By definition, the absence the abdomen, is not a common complication
of all stress is death. of chronic pancreatitis.
3. Smoking stimulates the pancreas to re- TEST-TAKING HINT: The test taker may be
lease pancreatic enzymes and should be able to delete options based on normal ana-
stopped. tomical and physiological data. Diabetes
4. The client has acute pancreatitis, and pan- insipidus is a complication of the pituitary
creatic enzymes are only needed for chronic gland; Crohn’s disease is
pancreatitis. a disease of the GI tract; and the peritoneum
TEST-TAKING HINT: The test taker should is the lining of the abdomen. Therefore, op-
eliminate option “2” because of the word tions “1,” “2,” and “4” can be eliminated.
“all,” which is an absolute and there are Content – Medical: Integrated Nursing Process – ­
few absolutes in health care. The test taker Planning: Client Needs – Physiological Integrity,
should note the adjective “acute” in the stem, ­Physiological Adaptation: Cognitive Level – Synthesis:
which may help the test taker eliminate op- Concept – Metabolism.
tion “4” because enzymes are administered 32. 1. During this procedure, a scope is placed
for a chronic condition. down the client’s mouth; therefore, assessing
Content – Medical: Integrated Nursing Process – ­ for rectal bleeding is not an intervention.
Planning: Client Needs – Physiological Integrity, 2. The client’s throat has been anesthetized to
­Physiological Adaptation: Cognitive Level – Synthesis:
insert the scope; therefore, fluid and food are
Concept – Metabolism.
withheld until the gag reflex has returned.
30. 1. Any change in the client’s stool should be a 3. The gag reflex will be suppressed as a
cause for concern to the clinic nurse. result of the local anesthesia applied to
2. This is not necessary because the nurse the throat to insert the endoscope into
knows changes in stool occur as a complica- the esophagus; therefore, the gag reflex
tion of pancreatitis, and the client needs to must be assessed prior to allowing the cli-
see the HCP. ent to resume eating or drinking.
3. Steatorrhea (fatty, frothy, foul-­smelling 4. The client should be in a semi-Fowler’s or
stool) is caused by a decrease in pancreatic side-lying position to prevent aspiration.

08_Colgrove_Ch08.indd 331 02/06/16 8:01 PM


332 M ed -S urg S uccess

TEST-TAKING HINT: The test taker should ap- (outside) source of the pancreatic en-
ply the nursing process and select an option zymes protease, amylase, and lipase.
that addresses assessment—either “1” or “3.” 2. Pancreatic enzymes decrease the number of
The medical prefix endo- should help the test bowel movements.
taker ­select option “3” as the correct answer. 3. The enzymes are enteric coated and should
Content – Surgical: Integrated Nursing Process – not be crushed because the hydrochloric acid
Implementation: Client Needs – Safe Effective Care in the stomach will destroy the enzymes;
­Environment, Management of Care: Cognitive Level – these enzymes work in the small intestine.
Analysis: Concept – Metabolism. 4. Pancreatic enzymes help break down carbo-
hydrates, and bile breaks down fat.
33. 1. Lying on the stomach will not help to
decrease the client’s pain. TEST-TAKING HINT: Remember: ­Enzymes
2. This is a position used by clients with chronic break down other foods and the names end
obstructive pulmonary disease to help lung in -ase. The test taker must know the nor-
expansion. mal function of organs to identify correct
3. This fetal position decreases pain caused answers.
by the stretching of the peritoneum as a Content – Medical: Integrated Nursing Process – ­
result of edema. Diagnosis: Client Needs – Physiological Integrity,
4. Laying supine causes the peritoneum to ­Pharmacological and Parenteral Therapies: Cognitive
stretch, which increases the pain. Level – Analysis: Concept – Medication.

TEST-TAKING HINT: The test taker should think 36. 1. Coffee, tea, and cola stimulate gastric and
about where the pancreas is located in the pancreatic secretions and may precipitate
abdomen to help identify the correct answer. pain, so these foods should be avoided, not
Prone or supine causes the abdomen to be decreased.
stretched, which increases pain. 2. High-fat and spicy foods stimulate
Content – Medical: Integrated Nursing Process – gastric and pancreatic secretions and may
Implementation: Client Needs – Physiological Integrity, precipitate an acute pancreatic attack.
­Basic Care and Comfort: Cognitive Level – Analysis: 3. Amylase and lipase levels must be checked via
Concept – Metabolism. venipuncture with laboratory tests, and there are
no daily tests the client can monitor at home.
34. 1. The return of bowel sounds indicates the
4. The client will be fatigued as a result of de-
return of peristalsis, and the nasogastric
creased metabolic energy production and
suction is usually discontinued within
will need to rest and not return to work
24 to 48 hours thereafter.
immediately.
2. The client will be NPO secondary to the
chronic pancreatitis, and the client cannot eat TEST-TAKING HINT: The test taker should be
with a nasogastric tube. careful with words such as “decrease” because
3. The nurse should assess for signs of infec- many times the client must avoid certain foods
tion or infiltration. and situations completely, not decrease the
4. Fasting and the N/G tube increase the cli- intake of them. Only a few blood studies are
ent’s risk for mucous membrane irritation monitored at home on a daily basis—mainly
and breakdown. glucose levels—which should cause the test
5. Blood glucose levels are monitored taker to eliminate option “3.”
because clients with chronic pancreatitis Content – Medical: Integrated Nursing Process –
can develop diabetes mellitus. ­Evaluation: Client Needs – Safe Effective Care Environ-
ment, Management of Care: Cognitive Level – Synthesis:
TEST-TAKING HINT: This alternative-type
Concept – Metabolism.
question requires the test taker to select all
interventions appropriate for the client’s di-
agnosis. The test taker should evaluate each Cancer of the Pancreas
answer independently as to whether it is ap-
propriate or not. 37. 1. A yellow haze is a sign of a toxic level of
Content – Medical: Integrated Nursing Process – digoxin, with the client seeing through the
Implementation: Client Needs – Safe Effective Care yellow haze. Seeing a yellow haze is not the
­Environment, Management of Care: Cognitive Level – same as the client having jaundice. In jaun-
Analysis: Concept – Metabolism. dice, the skin and sclera are yellow, signs of
35. 1. Pancreatic enzymes enhance the digestion pancreatic cancer.
of starches (carbohydrates) in the gastro-
intestinal tract by supplying an exogenous

08_Colgrove_Ch08.indd 332 02/06/16 8:01 PM


C hapter 8  E ndocrine D isorders 333

2. The abdominal pain is often made worse 4. Instructing a client with hyperglycemia
by eating and lying supine in clients diag- (­diabetes mellitus) is an example of tertiary
nosed with cancer of the pancreas. nursing care.
3. The client does not know these terms, and the TEST-TAKING HINT: Even if the test taker
HCP should check these laboratory values. were not sure of the definition of primary,
4. Clients diagnosed with cancer of the pancreas ­secondary, or tertiary nursing interventions,
lose weight; they do not gain weight. “primary” means first. Only one answer op-
TEST-TAKING HINT: The test taker could ar- tion is preventive, and preventing something
rive at the correct answer by correlating comes before treating it.
words in the stem of the question and words Content – Medical: Integrated Nursing Process –
in the answer options—the abdomen with ­Planning: Client Needs – Health Promotion and Mainte-
eating and pain with pain. nance: Cognitive Level – Synthesis: Concept – Nursing Roles.
Content – Medical: Integrated Nursing Process – 40. 1. The UAP can help a client to turn to the
­Assessment: Client Needs – Physiological Integrity,
side and assume the fetal position, which
Reduction of Risk Potential: Cognitive Level – Analysis:
Concept – Cellular Regulation.
decreases some abdominal pain.
2. Monitoring a Jackson Pratt drain is a high-
38. 1. Tube feedings are collaborative interventions, level nursing intervention, which the UAP is
but the stem did not say the client had a feed- not qualified to implement.
ing tube. 3. Evaluation of the effectiveness of a prn medi-
2. Having family members bring food from cation must be done by the nurse.
home is an independent intervention. 4. The nurse should empty the bedside com-
3. Assessment is an independent intervention mode to determine if the client is continuing
and the first step in the nursing process. to pass melena (blood in the stool).
No one should have to tell the nurse to TEST-TAKING HINT: There are basic rules to
assess the client. delegation. The nurse cannot delegate as-
4. A collaborative intervention is to refer sessment, teaching, evaluation, medications,
to the nutrition expert, the dietitian. unstable clients, or situations requiring nurs-
TEST-TAKING HINT: The key word in the stem ing judgment.
is “collaborative,” which means another Content – Nursing Management: Integrated Nursing
health-care discipline must be involved. Only Process – Planning: Client Needs – Safe Effective Care
options “1” and “4” involve other members Environment, Management of Care: Cognitive Level –
of the health-care team. The test taker could Synthesis: Concept – Nursing Roles.
eliminate distracter “1” by rereading the
41. 1. This is routine for radiation therapy, but
stem and realizing the stem did not include
chemotherapy is given one (1) to three (3) or
the client having a feeding tube.
four (4) days in a row and then a period of
Content – Medical: Integrated Nursing Process – three (3) to four (4) weeks will elapse before
­Diagnosis: Client Needs – Physiological Integrity, Basic Care
the next treatment. This is called intermittent
and Comfort: Cognitive Level – Analysis:
Concept – Nutrition.
pulse therapy.
2. The most important person in the treat-
39. 1. Monitoring the blood glucose at random in- ment of the cancer is the client. Research
tervals, as done at a health fair, could identify has proved the more involved a client
possible diabetes mellitus or the presence of becomes in his or her care, the better the
a pancreatic tumor, but detecting a disease at prognosis. Clients should have a chance to
an early stage is secondary screening, not pri- ask questions.
mary prevention. 3. Cancer of any kind is a serious problem.
2. Inspecting the skin for jaundice is a secondary 4. Most antineoplastic medications are adminis-
nursing intervention. tered intravenously. Many of the medications
3. Limiting the intake of meat and fats in the can cause severe complications if adminis-
diet is an example of primary interven- tered intramuscularly.
tions. Risk factors for the development of TEST-TAKING HINT: The test taker can elimi-
cancer of the pancreas are cigarette smok- nate option “3” based on this statement be-
ing and eating a high-fat diet. By changing ing denial of the problem.
these behaviors, the client could possibly
Content – Medical: Integrated Nursing Process – ­
prevent the development of cancer of Evaluation: Client Needs – Physiological Integrity,
the pancreas. Other risk factors include ­Physiological Adaptation: Cognitive Level – Synthesis:
genetic predisposition and exposure to in- Concept – Cellular Regulation.
dustrial chemicals.

08_Colgrove_Ch08.indd 333 02/06/16 8:01 PM


334 M ed -S urg S uccess

42. 1. The client should stay in the outpatient de- 3. Pain is a priority but not over fluid volume
partment after the procedure for observation status.
unless the HCP determines a more extensive 4. Altered nutrition is an appropriate problem
work-up should be completed. but not priority over fluid volume shift. The
2. This is not the type of procedure where the client will be NPO with a nasogastric tube to
results warrant an emergency surgery. A car- suction and will be receiving total parenteral
diac catheterization sometimes results in an nutrition.
emergency surgery and the client is prepared TEST-TAKING HINT: The nurse should identify
for this possibility, but this is not the case all of the problems, but one—fluid volume
with an ERCP. imbalance—has the greatest priority because,
3. The client should be NPO after midnight if not addressed promptly and correctly, it
to make sure the stomach is empty to could lead to severe complications.
reduce the risk of aspiration during the Content – Surgical: Integrated Nursing Process –
procedure. ­Diagnosis: Client Needs – Safe Effective Care E
­ nvironment,
4. The possible blockage is of the duodenum, Management of Care: Cognitive Level – Analysis:
common bile duct, or pancreatic outlet. Concept – Cellular Regulation.
TEST-TAKING HINT: The nurse should never
45. 1. The client is being discharged. There is no
preface any instruction with “if done correctly” need for the client to continue recording in-
because this sets the nurse, HCP, and facility take and output at home.
up for a lawsuit. The client is NPO for any pro- 2. The client has undergone a radical and
cedure or surgery where the client will receive extensive surgery and will need narcotic
general or twilight sleep anesthesia. pain medication, and a bowel regimen
Content – Surgical: Integrated Nursing Process – should be in place to prevent constipation.
­Planning: Client Needs – Physiological Integrity, Reduction 3. Removal of the pancreas will create a
of Risk Potential: Cognitive Level – Synthesis: Concept –
diabetic state for the client. The client
Cellular Regulation.
will need insulin and pancreatic enzyme
43. 1. The client will have jaundice, clay-­colored replacement.
stools, and tea-colored urine resulting 4. The client should not allow pain to reach
from blockage of the bile drainage. above a “5” before taking pain medication
2. Night sweats and fevers are associated with or it will be more difficult to get the pain
lymphoma. under control.
3. Left lower abdominal cramps are associated 5. There is no reason for the client to sleep
with diverticulitis, and tenesmus is straining with the head of the bed elevated.
when defecating. TEST-TAKING HINT: The test taker might
4. Nausea and coffee-ground emesis are symp- choose option “3” by remembering the pan-
toms of gastric ulcers. creas secretes insulin. Option “4” is taught to
TEST-TAKING HINT: The test taker should re- all clients in pain.
member anatomical placement of organs. This Content – Surgical: Integrated Nursing Process – ­
eliminates options “3” and “4.” The pancreas Planning: Client Needs – Physiological Integrity,
empties pancreatic enzymes into the small ­Physiological Adaptation: Cognitive Level – Synthesis:
bowel in close proximity to where the com- Concept – Cellular Regulation.
mon bile duct enters the intestine to aid in the 46. 1. The client may need IV medication, but in
digestion of carbohydrates and fats. this case if it is needed, it is 50% dextrose.
Content – Medical: Integrated Nursing Process – ­ 2. The HCP might be notified, but the nurse
Assessment: Client Needs – Physiological Integrity, needs to assess the client first.
­Reduction of Risk Potential: Cognitive Level – Analysis:
3. These are symptoms of an insulin reac-
Concept – Cellular Regulation.
tion (hypoglycemia). A bedside glucose
44. 1. Clients diagnosed with cancer of the pancreas check should be done. Pancreatic islet
have a poor prognosis, but this is not the pri- tumors can produce hyperinsulinemia
ority problem at this time. or hypoglycemia.
2. This is a major abdominal surgery, and 4. Treating the client is done after the nurse
massive fluid volume shifts occur when knows the glucose reading.
this type of trauma is experienced by the TEST-TAKING HINT: The test taker should
body. Maintaining the circulatory system ­remember the function of the pancreas. This
without overloading it requires extremely leads the test taker to look for interventions
close monitoring. for hypoglycemia.

08_Colgrove_Ch08.indd 334 02/06/16 8:01 PM


C hapter 8  E ndocrine D isorders 335

Content – Medical: Integrated Nursing Process – Fever and irritability, along with hirsutism, are
Implementation: Client Needs – Safe Effective Care clinical manifestations of Cushing’s syndrome.
­Environment, Management of Care: Cognitive Level – 3. Bronze pigmentation of the skin, particu-
Synthesis: Concept – Cellular Regulation. larly of the knuckles and other areas of
47. 1. Food preferences are important for the care- skin creases, occurs in Addison’s disease.
giver to know because this will be the person Hypotension and anorexia also occur with
preparing meals for the client, but it is not Addison’s disease.
the highest priority. 4. Tachycardia, bulging eyes, and goiter are
2. Cancer of the pancreas has a poor clinical manifestations occurring with ­thyroid
prognosis; the nurse should determine disorders.
if the client has executed an advance TEST-TAKING HINT: This question contains
directive regarding his or her wishes. answer options referring to opposite dis-
3. This is important because of payment issues, eases, Addison’s disease and Cushing’s syn-
but it is not the highest priority. drome. If two options—in this case, options
4. Clients diagnosed with cancer frequently “1” and “2”—are appropriate for one of the
have anorexia, and explaining the client diseases, then these two can be ruled out as
should eat does not mean the client will eat. the correct answer.
TEST-TAKING HINT: The test taker needs to Content – Medical: Integrated Nursing Process –
know general information about the disease Assessment: Client Needs – Physiological Integrity,
process to answer this question, but option ­Reduction of Risk Potential: Cognitive Level – Analysis:
“2” is a good choice for many terminal dis- Concept – Metabolism.
eases. Remember to read the questions care- 50. 1. Altered body image is a psychosocial prob-
fully. The home health nurse is not arranging lem, which is not a priority over a potentially
meals for the client. lethal physical complication, and physical
Content – Medical: Integrated Nursing Process – changes occur over an extended period.
Implementation: Client Needs – Safe Effective Care 2. Activity intolerance will occur with adrenal
­Environment, Management of Care: Cognitive Level – gland hypofunction, but this is not a priority
Synthesis: Concept – Cellular Regulation. over dehydration.
48. 1. The client is at risk for poor nutrition and 3. Impaired coping can occur in clients with
malabsorption syndrome for which tissue tur- adrenal gland disorders, but it is not a priority
gor assessment is appropriate, but the over dehydration.
client problem is pruritus, or itching. 4. Fluid volume deficit (dehydration) can
2. The itching is associated with the cancer and lead to circulatory impairment and
not a fungus. hyperkalemia.
3. The client should be monitored for skin TEST-TAKING HINT: Assuming all of the prob-
breakdown, but pruritus is itching and an in- lems listed apply to the client diagnosed
tervention is needed to prevent skin with Addison’s disease, two are psychosocial
problems as a result of scratching. problems and two are physiological. Applying
4. Keeping the fingernails short will reduce Maslow’s hierarchy of needs, the two psycho-
the chance of breaks in the skin from logical problems can be ruled out as the highest
scratching. priority. Of the two options remaining, activity
TEST-TAKING HINT: The problem is “risk for intolerance is not life altering or threatening.
skin breakdown.” The etiology is “pruritus.” Content – Medical: Integrated Nursing Process –
Interventions address the etiology. Goals ­Diagnosis: Client Needs – Safe Effective Care E
­ nvironment,
­address the problem. Management of Care: Cognitive Level – Analysis:
Concept – Metabolism.
Content – Medical: Integrated Nursing Process –
Implementation: Client Needs – Physiological Integrity, 51. 1. Clients diagnosed with Addison’s disease
Basic Care and Comfort: Cognitive Level – Application: have adrenal gland hypofunction. The
Concept – Cellular Regulation. hormones normally produced by the gland
must be replaced. Steroids and androgens
are produced by the adrenal gland.
Adrenal Disorders
2. The client will have decreased fluid volume,
and fluid restriction exacerbates a crisis.
49. 1. A moon face, buffalo hump, and hyperglyce-
3. The client requires a quiet, calm, relaxed
mia result from Cushing’s syndrome, hyper-
atmosphere.
function of the adrenal gland.
4. The client walks with a stooped posture from
2. Hirsutism is hair growth where it normally
fatigue, but gait training is not needed.
does not occur, such as facial hair on women.

08_Colgrove_Ch08.indd 335 02/06/16 8:01 PM


336 M ed -S urg S uccess

TEST-TAKING HINT: To answer this question, of the immunosuppressive nature of the


the test taker must have knowledge of adre- disease.
nal gland function. 2. The client has too much cortisol; this client
Content – Medical: Integrated Nursing Process – should not be receiving prednisone, a steroid
Implementation: Client Needs – Safe Effective Care medication.
­Environment, Management of Care: Cognitive Level – 3. These are symptoms of hypoglycemia, which
­Application: Concept – Metabolism. is not expected in this client because this cli-
52. 1. The drugs quinidine, digoxin, and hydrala- ent has high-glucose levels.
4. The client is predisposed to osteoporosis and
zine can interfere with adrenal gland secre-
fractures. Contact sports should be avoided.
tions and cause hypofunction. Cushing’s
syndrome is adrenal gland hyperfunction. TEST-TAKING HINT: If the test taker is not
2. The adrenal gland secretes cortisol and aware of the disease problem, this question
the pituitary gland secretes adrenocorti- could be answered correctly because of com-
cotropic hormone (ACTH), a hormone mon standard discharge instructions—namely,
used by the body to stimulate the produc- notify the health-care provider of a fever.
tion of cortisol. Content – Medical: Integrated Nursing Process – ­
3. A 24-hour urine specimen for Evaluation: Client Needs – Safe Effective Care
17-hydroxycorticosterone and 17-ketosteroid ­Environment, Management of Care: Cognitive Level –
may be collected. Metanephrines and Synthesis: Concept – Metabolism.
catecholamines are urine collections for 55. 1. This client could be cared for by any nurse
pheochromocytomas. qualified to work in an intensive care unit.
4. Spot urinalysis and white blood cell count 2. These blood gases are within normal limits.
will not provide information on adrenal gland 3. This client has a low blood pressure and
functions. tachycardia. This client may be experi-
TEST-TAKING HINT: If the test taker is aware encing an addisonian crisis, a potentially
the adrenal gland produces cortisol, then life-threatening condition. The most expe-
there is only one answer option that ­refers to rienced nurse should care for this client.
cortisol. 4. A negative Trousseau’s sign is normal for this
Content – Medical: Integrated Nursing Process – P ­ lanning: client.
Client Needs – Physiological Integrity, Reduction of Risk TEST-TAKING HINT: The answer options
Potential: Cognitive Level – Synthesis: Concept – Metabolism. “1,” “2,” and “4” have expected or normal
53. 1. Cushing’s disease is not an autoimmune data. Only one option has abnormal data.
problem. Even if the test taker is unaware of addiso-
2. “Iatrogenic” means a problem has nian crisis, these are vital signs indicating
been caused by a medical treatment or potential shock.
­procedure—in this case, treatment with Content – Nursing Management: Integrated Nursing
steroids for another problem. Clients taking Process – Planning: Client Needs – Safe Effective Care
steroids over a period of time develop the Environment, Management of Care: Cognitive Level –
clinical manifestations of Cushing’s disease. Synthesis: Concept – Management.
Disease processes for which long-term 56. 1. Blood glucose levels do not address the prob-
steroids are prescribed include chronic lem of altered body image.
obstructive pulmonary disease, cancer, and 2. Head-to-toe assessments are performed
arthritis. to detect a physiological problem, not a
3. This could be a cause for primary Cushing’s psychosocial one.
syndrome. 3. Allowing the client to ventilate feelings
4. There is a known reason for the client to about the altered body image is the most
have iatrogenic Cushing’s syndrome. appropriate intervention. The nurse
TEST-TAKING HINT: This question requires the cannot do anything to help the client’s
test taker to know basic medical terminology. buffalo hump or moon face.
Content – Medical: Integrated Nursing Process – 4. Bowel sounds and temperature are physical
­Diagnosis: Client Needs – Safe Effective Care E
­ nvironment, symptoms.
Management of Care: Cognitive Level – Analysis: TEST-TAKING HINT: The intervention must
Concept – Metabolism. match the problem.
54. 1. Cushing’s syndrome/disease predisposes Content – Medical: Integrated Nursing Process –
the client to develop infections as a result Implementation: Client Needs – Psychosocial Integrity:
Cognitive Level – Application: Concept – Metabolism.

08_Colgrove_Ch08.indd 336 02/06/16 8:01 PM


C hapter 8  E ndocrine D isorders 337

57. 1. The client was exposed to wind and 3. Notifying the HCP if signs/symptoms of
sun at the lake during the hours prior to infection develop is an instruction given to
being admitted to the emergency depart- all surgical clients on discharge.
ment. This predisposes the client to 4. Turning and coughing is taught prior to
dehydration and an addisonian crisis. surgery, not at discharge.
Rapid IV fluid replacement is necessary. TEST-TAKING HINT: The test taker must
2. Sitting in the waiting area could cause the notice the adjectives; “discharge” tells the
­client to go into a coma and die. reader a time frame for the instructions. This
3. A blood transfusion is not an appropriate rules out option “4.”
­intervention for this client.
Content – Surgical: Integrated Nursing Process –
4. Laboratory specimens are not priority and Planning: Client Needs – Physiological Integrity,
calcium is not a problem in clients with ­Physiological Adaptation: Cognitive Level – Synthesis:
Addison’s disease. Concept – Metabolism.
TEST-TAKING HINT: This client is weak,
lethargic, and forgetful, indicating a di-
minished level of consciousness. The nurse Pituitary Disorders
should choose an action addressing the
problem. 61. 1. The client has excess fluid and is not
dehydrated, and blood glucose levels are
Content – Medical: Integrated Nursing Process –
Implementation: Client Needs – Safe Effective Care
not affected.
­Environment, Management of Care: Cognitive Level – 2. Early signs and symptoms are nausea and
Synthesis: Concept – Metabolism. vomiting. The client has the syndrome
of inappropriate secretion of antidiuretic
58. The client has received 8,650 mL of (against allowing the body to urinate) hor-
intravenous fluid. mone. In other words, the client is pro-
TEST-TAKING HINT: This is a basic addition ducing a hormone that will not allow the
problem. If the test taker has difficulty with client to urinate.
this problem, then a math review course 3. The client experiences dilutional hyponatre-
would be in order. mia, and the body has too much fluid already.
Content – Medical: Integrated Nursing Process – 4. Vasopressin is the name of the antidiuretic
Implementation: Client Needs – Physiological Integrity, hormone. Giving more increases the client’s
Pharmacological and Parenteral Therapies: Cognitive problem. Also, a water challenge test is per-
Level – Synthesis: Concept – Medication. formed, not a fluid deprivation test.
59. 1. An autocratic style is one in which the TEST-TAKING HINT: The syndrome’s name
person in charge makes the decision with- is confusing, with a double negative—
out consulting anyone else. “inappropriate” and “anti.” It is helpful to
2. This behavior is an example of a democratic put the situation in the test taker’s own
leadership style. words to remember which way the fluids are
3. This behavior is an example of laissez-faire being shifted in the body.
leadership style. Content – Medical: Integrated Nursing Process –
4. This behavior is an example of democratic Implementation: Client Needs – Safe Effective Care
leadership style. ­Environment, Management of Care: Cognitive Level –
Application: Concept – Cellular Regulation.
TEST-TAKING HINT: The test taker could
choose the correct answer if the test 62. 1. Neurological status is monitored every
taker knew terms such as “autocratic” and one (1) to two (2) hours. This client’s neuro-
“democratic.” logical status appears intact. Clients waking up
Content – Nursing Management: Integrated Nursing in an intensive care area may not be aware of
Process – Implementation: Client Needs – Safe Effective their surroundings.
Care Environment, Management of Care: Cognitive 2. The output is more than double the
Level – Comprehension: Concept – Management. intake in a short time. This client could be
60. 1. A unilateral adrenalectomy results in one developing diabetes insipidus, a complica-
adrenal gland still functioning. No hormone tion of trauma to the head.
replacement will be required. 3. These vital signs are within normal limits.
2. The client can still have normal physiologi- 4. A transsphenoidal hypophysectomy is per-
cal functioning, including sexual functioning, formed by surgical access above the gum line
with the remaining gland. and through the nasal passage. There is no
dressing. A drip pad is taped below the nares.

08_Colgrove_Ch08.indd 337 02/06/16 8:01 PM


338 M ed -S urg S uccess

TEST-TAKING HINT: Two (2) of the answer 65. 1. The client is not allowed to drink during
options contain normal data and would not the test.
warrant immediate intervention. Option “4” 2. This test does not require any medications
does not match the type of surgery. to be administered, and vasopressin will treat
Content – Surgical: Integrated Nursing Process – the DI, not help diagnose it.
Assessment: Client Needs – Physiological Integrity, 3. The client is deprived of all fluids, and if
­Reduction of Risk Potential: Cognitive Level – Synthesis: the client has DI the urine production will
Concept – Metabolism. not diminish. Vital signs and weights are
taken every hour to determine circulatory
63. 1. The client will have an elevated sodium
status. If a marked decrease in weight or
level as a result of low circulating blood
vital signs occurs, the test is immediately
volume. The fluid is being lost through the
terminated.
urine. Diabetes means “to pass through”
4. No fluid is allowed and a sonogram is not
in Greek, indicating polyuria, a symptom
involved.
shared with diabetes mellitus. Diabetes in-
sipidus is a totally separate disease process. TEST-TAKING HINT: The name of the test is a
2. Serum calcium is not affected by diabetes fluid deprivation test. Two (2) of the options
insipidus. require the administration of some type of
3. Urine glucose is monitored for diabetes fluid.
mellitus. Content – Medical: Integrated Nursing Process – P ­ lanning:
4. White blood cells in the urine indicate the Client Needs – Physiological Integrity, Reduction of Risk
presence of a urinary tract infection. ­Potential: Cognitive Level – Synthesis: Concept – Metabolism.
TEST-TAKING HINT: The test taker should 66. 1. Clients with SIADH have a problem with
not confuse diabetes insipidus and diabetes ­retaining fluid. This is expected.
mellitus. 2. This client’s intake and output are relatively
Content – Medical: Integrated Nursing Process – ­ the same.
Assessment: Client Needs – Physiological Integrity, 3. Muscle twitching is a sign of early sodium
­Reduction of Risk Potential: Cognitive Level – Analysis: imbalance. If an immediate intervention is
Concept – Metabolism. not made, the client could begin to seize.
64. 1. The client should keep a list of medications 4. The client has to get up all night to urinate,
being taken and wear a Medic Alert bracelet. so the client feeling tired is expected.
2. Medication for DI is usually taken every TEST-TAKING HINT: All of the answer options
eight (8) to 12 hours, depending on the contain expected information except
client. The client should keep the medica- option “3.”
tion close at hand. Content – Medical: Integrated Nursing Process – ­
3. The client is at risk for fluid shifts. Weighing Assessment: Client Needs – Safe Effective Care
every morning allows the client to follow the ­Environment, Management of Care: Cognitive Level –
fluid shifts. Weight gain indicates too much Analysis: Concept – Metabolism.
medication. 67. Two (2) sprays per dose.
4. Tightness in the chest could be an indicator 40 g of medication every 24 hours is to be
the medication is not being tolerated; if this given in doses administered every 12 hours.
occurs, the client should notify the health- First, determine number of doses needed:
care provider. 24 ÷ 12 = 2 doses
TEST-TAKING HINT: This is an “except” ques- Then, determine the amount of medication
tion. This means all answers except one will be to be given in each of those two (2) doses:
actions the client should do. If the test taker 40 ÷ 2 = 20 g of medication per dose
missed interpreting this from the stem, then Finally, determine how many sprays are
the test taker could jump to the first action needed to deliver the 20 mg when each spray
the client should do as the correct answer. delivers 10 g:
Content – Medical: Integrated Nursing Process – ­ 20 ÷ 10 = 2 sprays
Evaluation: Client Needs – Safe Effective Care TEST-TAKING HINT: The test taker should take
­Environment, Management of Care: Cognitive Level – each step of the problem one at a time and
Synthesis: Concept – Metabolism.
check the answer with the drop-down calcula-
tor if taking the examination on a computer.

08_Colgrove_Ch08.indd 338 02/06/16 8:01 PM


C hapter 8  E ndocrine D isorders 339

Content – Medical: Integrated Nursing Process – problem and the nursing process if it is a
Implementation: Client Needs – Physiological Integrity, question of this nature. Assessment is the
Pharmacological and Parenteral Therapies: Cognitive first step in the nursing process.
Level – Application: Concept – Medication.
Content – Medical: Integrated Nursing Process –
68. 1. Fluids are restricted to 500 to 600 mL per Implementation: Client Needs – Safe Effective Care
24 hours. ­Environment, Management of Care: Cognitive Level –
2. Orientation to person, place, and time Synthesis: Concept – Metabolism.
should be assessed every two (2) hours 71. 1. A serum sodium level of 112 mEq/L is
or more often. dangerously low, and the client is at risk
3. A safe environment, not a stimulating one, is for seizures. A headache is a symptom of a
provided. low-sodium level.
4. Urine and serum osmolality are moni- 2. This is a normal potassium level, and a
tored to determine fluid volume status. heightened level of awareness indicates drug
5. The client should be weighed daily. usage.
TEST-TAKING HINT: The test taker should no- 3. This is a normal calcium level and the client
tice numbers: Is assessing the client’s level of is fluid overloaded, not dehydrated, so there
consciousness every two (2) hours enough, would not be tented tissue turgor.
or is weighing the client every three (3) days 4. This is a normal magnesium level, and a large
enough? urinary output is desired.
Content – Medical: Integrated Nursing Process – TEST-TAKING HINT: The nurse must know
Implementation: Client Needs – Safe Effective Care common laboratory values.
­Environment, Management of Care: Cognitive Level –
Content – Medical: Integrated Nursing Process – ­
Analysis: Concept – Fluid and Electrolyte Balance.
Assessment: Client Needs – Physiological Integrity,
69. 1. Diabetes insipidus is not diabetes mellitus; ­Reduction of Risk Potential: Cognitive Level – Analysis:
sliding-scale insulin is not administered to the Concept – Fluid and Electrolyte Balance.
client. 72. 1. Discussing the information with others is not
2. There is no caffeine restriction for DI. allowing the client to decide what is best for
3. Checking urine ketones is not indicated. himself.
4. The client is excreting large amounts 2. This could be an example of beneficence
of dilute urine. If the client is unable to (to do good) if the nurse did this so the client
drink enough fluids, the client will quickly has information on which to base a decision
become dehydrated, so tissue turgor on whether to continue the fluid restriction.
should be assessed frequently. 3. This is an example of autonomy (the
TEST-TAKING HINT: Two (2) of the answer op- ­client has the right to decide for himself).
tions are appropriate for diabetes mellitus, 4. This is an example of dishonesty and should
not diabetes insipidus, and can be eliminated never be tolerated in a health-care setting.
based on this alone. TEST-TAKING HINT: The stem asks the test
Content – Medical: Integrated Nursing Process – taker about autonomy. Even if the test taker
Implementation: Client Needs – Safe Effective Care did not know the ethical principle, autonomy
­Environment, Management of Care: Cognitive Level – means the right of self-governance. Only one
Application: Concept – Metabolism. of the answer options could fit the definition
70. 1. The client should have the water pitcher of autonomy.
filled, but this is not the first action. Content – Medical: Integrated Nursing Process –
2. This should be done but not before assessing Implementation: Client Needs – Safe Effective Care
the problem. ­Environment, Management of Care: Cognitive Level –
3. The first action should be to determine Comprehension: Concept – Fluid and Electrolyte Balance.
if the client is experiencing polyuria and
polydipsia as a result of developing diabe-
tes insipidus, a complication of the head
Thyroid Disorders
trauma.
73. 1. A decrease in thyroid hormone causes de-
4. This could be done, but it will not give the
creased metabolism, which leads to
nurse information about DI.
fatigue and hair loss.
TEST-TAKING HINT: The nurse must apply 2. These are signs of hyperthyroidism.
a systematic approach to answering prior- 3. These are signs of hyperthyroidism.
ity questions. Maslow’s hierarchy of needs 4. These are signs of parathyroidism.
should be applied if it is a physiological

08_Colgrove_Ch08.indd 339 02/06/16 8:01 PM


340 M ed -S urg S uccess

TEST-TAKING HINT: Often if the test taker TEST-TAKING HINT: One way of determining
does not know the specific signs/symptoms the effectiveness of medication is to deter-
of the disease but knows the function of the mine if the signs/symptoms of the ­disease
system affected by the disease, some possible are no longer noticeable.
answers can be ruled out. Tetany and stiff- Content – Medical: Integrated Nursing Process –
ness of the hands are related to calcium, the Assessment: Client Needs – Physiological Integrity,
level of which is influenced by the parathy- ­Pharmacological and Parenteral Therapies: Cognitive
roid gland, not the thyroid gland; therefore, Level – Analysis: Concept – Medication.
option “4” can be ruled out. 76. 1. Fiber should be increased in the client
Content – Medical: Integrated Nursing Process – ­ diagnosed with hypothyroidism because the
Assessment: Client Needs – Physiological Integrity, client experiences constipation secondary
­Reduction of Risk Potential: Cognitive Level – Analysis:
to decreased metabolism.
Concept – Metabolism.
2. The client with hyperthyroidism should have
74. 1. External heat sources (heating pads, ­electric a high-calorie, high-protein diet.
or warming blankets) should be discour- 3. The client’s fluid intake should be increased
aged because they increase the risk of pe- to replace fluids lost through diarrhea and
ripheral vasodilation and vascular collapse. excessive sweating.
2. Assessing the client’s temperature every two 4. The client with hyperthyroidism
(2) hours is not needed because the tempera- has an increased appetite; therefore,
ture will not change quickly. The client needs well-balanced meals served several times
thyroid hormones to help increase the client’s throughout the day will help with the
temperature. client’s constant hunger.
3. The room temperature should be kept warm TEST-TAKING HINT: If the test taker
because the client will have complaints of be- knows the metabolism is increased with
ing cold. ­hyperthyroidism, then increasing the food
4. The client is fatigued and this is an appropri- intake is the most appropriate choice.
ate intervention but is not applicable to the
Content – Medical: Integrated Nursing Process –
client problem of “risk for imbalanced body Implementation: Client Needs – Physiological Integrity,
temperature.” Basic Care and Comfort: Cognitive Level – Application:
TEST-TAKING HINT: The test taker must Concept – Metabolism.
always know exactly what the question is 77. 1. Hypoglycemia is expected in a client with
­asking. Option “4” can be ruled out because myxedema; therefore, a 74-mg/dL blood
it does not address body temperature. If the ­glucose level is expected.
test taker knows the normal function of the 2. A pulse oximeter reading of less than
thyroid gland, this may help identify the an- 93% is significant. A 90% pulse oximeter
swer; decreased metabolism will cause the reading indicates a Pao2 of approximately
client to be cold. 60 on an arterial blood gas test; this is
Content – Medical: Integrated Nursing Process – severe hypoxemia and requires immediate
Diagnosis: Client Needs – Safe Effective Care E
­ nvironment, intervention.
Management of Care: Cognitive Level – Analysis:
3. The client with myxedema coma is in an ex-
Concept – Metabolism.
aggerated hypothyroid state; a low pulse is
75. 1. The medication will help increase the c­ lient’s expected in a client with hypothyroidism.
metabolic rate. A weight gain indicates not 4. Lethargy is an expected symptom in a client
enough medication is being taken to put the diagnosed with myxedema; therefore, this
client in a euthyroid (normal ­thyroid) state. does not warrant immediate intervention.
2. A decreased pulse rate indicates there is not TEST-TAKING HINT: The words “warrant imme-
enough thyroid hormone level; therefore, the diate intervention” means the test taker should
medication is not effective. select an option that is abnormal for the dis-
3. The client with hypothyroidism frequently ease process or a life-threatening symptom.
has a subnormal temperature, so a tem- Content – Medical: Integrated Nursing Process – ­
perature WNL indicates the medication is Assessment: Client Needs – Safe Effective Care
effective. ­Environment, Management of Care: Cognitive Level –
4. Diaphoresis (sweating) occurs with hyperthy- Synthesis: Concept – Metabolism.
roidism, not hypothyroidism.

08_Colgrove_Ch08.indd 340 02/06/16 8:01 PM


C hapter 8  E ndocrine D isorders 341

78. 1. Thyroid hormones are the treatment of may have to stay up to two (2) hours after the
choice for the client diagnosed with hypothy- treatment in the office.
roidism; therefore, the nurse should not ques- 4. If too much of the thyroid gland is destroyed
tion this medication. by the radioactive iodine therapy, the client
2. In untreated hypothyroidism, the medical may develop hypothyroidism and have to take
management is aimed at supporting vital thyroid hormone the rest of his or her life.
functions, so administering oxygen is an TEST-TAKING HINT: Some questions require
appropriate medication. the test taker to be knowledgeable of the in-
3. Untreated hypothyroidism is character- formation, especially medical treatments, and
ized by an increased susceptibility to the there are no specific hints to help the test
effects of most hypnotic and sedative taker answer the question.
agents; therefore, the nurse should ques- Content – Medical: Integrated Nursing Process –
tion this medication. Implementation: Client Needs – Physiological Integrity,
4. Clients with hypothyroidism become consti- Reduction of Risk Potential: Cognitive Level – Application:
pated as a result of decreased metabolism, so Concept – Metabolism.
laxatives should not be questioned by the nurse.
81. 1. Weight loss indicates the medication may
TEST-TAKING HINT: When a question asks
not be effective and will probably need to
which order the nurse should question, three be increased.
of the options are medications the nurse ex- 2. The client needs to know emotional highs
pects to administer to the client. Sometimes and lows are secondary to hyperthyroid-
saying, “The nurse administers this medica- ism. With treatment, this emotional labil-
tion,” may help the test taker ­select the cor- ity will subside.
rect answer. 3. Any over-the-counter medications (for
Content – Medical: Integrated Nursing Process – example, alcohol-based medications) may
Implementation: Client Needs – Safe Effective Care negatively affect the client’s hyperthy-
­Environment, Management of Care: Cognitive Level –
roidism or medications being used for
Application: Concept – Medication.
treatment.
79. 1. Decreased appetite is a symptom of hypothy- 4. This will help any HCP immediately know
roidism, not hyperthyroidism. of the client’s condition, especially if the
2. Constipation is a symptom of hypothyroidism. client is unable to tell the HCP.
3. Dry, coarse skin is a sign of hypothyroidism. 5. The client with hyperthyroidism will be
4. The thyroid gland (in the neck) e ­ nlarges on antithyroid medications, not thyroid
as a result of the increased need for medications.
­thyroid hormone production; an ­enlarged TEST-TAKING HINT: This alternate-type ques-
gland is called a goiter. tion instructs the test taker to select all the
TEST-TAKING HINT: If the test taker does not interventions that apply. The test taker must
know the answer, sometimes thinking about read and evaluate each option as to whether
the location of the gland or organ causing it applies or not.
the problem may help the test taker select or Content – Medical: Integrated Nursing Process – ­
rule out specific options. Planning: Client Needs – Physiological Integrity,
Content – Medical: Integrated Nursing Process – ­ ­Physiological Adaptation: Cognitive Level – Synthesis:
Evaluation: Client Needs – Physiological Integrity, Concept – Metabolism.
­Reduction of Risk Potential: Cognitive Level – Analysis:
82. 1. There is no screening for thyroid disorders,
Concept – Metabolism.
just serum thyroid levels.
80. 1. Radioactive iodine therapy is used to 2. This is not a true statement.
­­destroy the overactive thyroid cells. After 3. Medications do not decrease the development
treatment, the client is followed closely of goiters.
for three (3) to four (4) weeks until the 4. Almost all of the iodine entering the body
euthyroid state is reached. is retained in the thyroid gland. A defi-
2. A single dose of radioactive iodine therapy ciency in iodine will cause the thyroid
is administered; the dosage is based on the gland to work hard and enlarge, which is
client’s weight. called a goiter. Goiters are commonly seen
3. The colorless, tasteless radioiodine is ad- in geographical regions having an iodine
ministered by the radiologist, and the client deficiency. Most table salt in the United
States has iodine added.

08_Colgrove_Ch08.indd 341 02/06/16 8:01 PM


342 M ed -S urg S uccess

TEST-TAKING HINT: The nurse must know question around and say, “I should give this
about disease processes. There is no test- medication.”
taking hint to help with knowledge. Content – Medical: Integrated Nursing Process –
Content – Medical: Integrated Nursing Process – Implementation: Client Needs – Safe Effective Care
­Implementation: Client Needs – Health Promotion and ­Environment, Management of Care: Cognitive Level –
Maintenance: Cognitive Level – Application: Concept – ­Application: Concept – Medication.
Metabolism.
84. 1. These are signs of myxedema (hypothy-
83. 1. The thyroid hormone must be administered roidism) coma. Obstipation is extreme
daily, and thyroid levels are drawn every six constipation.
(6) months or so. 2. Hyperpyrexia (high fever) and heart rate
2. A blood glucose level of 210 mg/dL requires above 130 beats per minute are signs of
insulin administration; therefore, the nurse thyroid storm, a severely exaggerated
should not question administering this hyperthyroidism.
medication. 3. Decreased blood pressure and slow heart rate
3. This potassium level is below normal, are signs of myxedema coma.
which is 3.5 to 5.5 mEq/L. Therefore, the 4. These are signs/symptoms of myxedema
nurse should question administering this coma.
medication because loop diuretics cause TEST-TAKING HINT: If the test taker does not
potassium loss in the urine. have the knowledge to answer the question,
4. The digoxin level is within therapeutic the test taker should look at the options
range—0.8 to 2 mg/dL; therefore, the nurse closely. Options “1,” “3,” and “4” all have
should administer this medication. signs/symptoms of “decrease”—hypoactive,
TEST-TAKING HINT: When administering hypotension, and hypoxia. The test taker
medication, the nurse must know when to should select the option that does not match.
question the medication, how to know it is Content – Medical: Integrated Nursing Process – ­
effective, and what must be taught to keep Assessment: Client Needs – Physiological Integrity,
the client safe while taking the medica- ­Reduction of Risk Potential: Cognitive Level – Analysis:
tion. The test taker may want to turn the Concept – Metabolism.

08_Colgrove_Ch08.indd 342 02/06/16 8:01 PM


CONCEPTS

85. 1. Carafate is a mucosal barrier agent and consequences of not controlling the blood
should be administered on an empty stomach glucose.
so the medication can coat the mucosa. The 3. Diabetes affects all tissues in the body.
nurse would not question administering this The feet are particularly at risk for the
medication. ­development of foot sores.
2. The nurse cannot administer sliding- 4. The client should carry sources of quick-
scale insulin without knowing the cur- acting carbohydrates, not protein.
rent blood glucose. The nurse should talk 5. Diabetes can cause retinal changes and
with the client to try and obtain the cli- detachment.
ent’s cooperation and, if not, then notify TEST-TAKING HINT: The nurse should be ­
the HCP that the medication cannot be able to teach common information to the
administered. client.
3. Levothyroxine is an appropriate treatment for
Content – Medical: Integrated Nursing Process –
hypothyroidism. Planning: Client Needs – Physiological Integrity, ­Reduction
4. Sliding scale usually begins at 150 mg/dL; the of Risk Potential: Cognitive Level – Analysis: Concept –
nurse would not question administering this Metabolism.
medication.
88. 1. The nurse should continue to monitor the
TEST-TAKING HINT: The test taker must know
client. The behavior is expected for
normal and abnormal diagnostic laboratory
a client with hyperthyroidism.
values. Medications are administered per
2. The client’s vital signs are not indicated
sliding scale in response to blood glucose
because of the symptoms.
levels. The nurse must also recognize ac-
3. This behavior is expected for a client with
cepted treatments for diseases.
­hyperthyroidism. A sedative is not needed.
Content – Medical: Integrated Nursing Process – 4. The nurse cannot insist the client do
Implementation: Client Needs – Pharmacological Integrity, anything.
Reduction of Risk Potential: Cognitive Level – Analysis:
Concept – Metabolism. TEST-TAKING HINT: The test taker must know
­expected symptoms for disease processes.
86. 1. The nurse should know the blood glucose
Content – Medical: Integrated Nursing Process – ­
level before making any decision about Assessment: Client Needs – Physiological Integrity,
the administration of either insulin. ­Reduction of Risk Potential: Cognitive Level – Analysis:
Insulin is due at this time. Concept – Metabolism.
2. Lantus insulin is not combined with any other
insulin in the same syringe. 89. 1. Physical therapy is not indicated.
3. The nurse should prepare to give the insulins 2. Social work is not indicated.
but not before knowing the current blood 3. Speech therapy is not indicated.
glucose. 4. Metabolism involves the intake and utili-
4. This is not needed at this time. zation of nutrients; the dietitian should be
consulted.
TEST-TAKING HINT: The nurse must know
the rules of medication administration and TEST-TAKING HINT: As a coordinator of care,
must know the medications and when they the nurse must be aware of each discipline
are due. and how
it affects the client’s care.
Content – Medical: Integrated Nursing Process –
Implementation: Client Needs – Pharmacological and Content – Medical: Integrated Nursing Process –
­Parenteral Therapy: Cognitive Level – Analysis: Implementation: Client Needs – Physiological Integrity,
Concept – Metabolism. Reduction of Risk Potential: Cognitive Level – Analysis:
Concept – Metabolism.
87. 1. The client with diabetes should be taught
to perform self glucose monitoring. 90. 1. These results are both WNL; they do not in-
2. In order to maintain a healthy life- dicate type 2 diabetes.
style the client should be aware of the 2. These results are both slightly above normal
but they do not indicate type 2 diabetes.

343

08_Colgrove_Ch08.indd 343 02/06/16 8:01 PM


344 M ed -S urg S uccess

3. These results are both slightly above normal Content – Medical: Integrated Nursing Process – ­
but they do not indicate type 2 diabetes. Assessment: Client Needs – Physiological Integrity,
4. Both laboratory values are above the nor- ­Reduction of Risk Potential: Cognitive Level – Analysis:
mal range. The A1c indicates a lengthy Concept – Metabolism.
time (at least two [2] to three [3] months) 93. 1. The HCP and not the nurse should perform
that the blood glucose has been high. This this assessment. The nurse has an unusual and
would be supportive of uncontrolled type potentially life-changing issue identified.
2 diabetes. 2. Keeping the eyes closed will not change the
TEST-TAKING HINT: The test taker must know outcome of retinal detachment. This is an
normal and abnormal diagnostic laboratory ophthalmological emergency.
values. Laboratory values vary depending on 3. This is an emergency; this indicates
which laboratory performs the test. ­retinal detachment. The nurse should no-
Content – Medical: Integrated Nursing Process – ­ tify the HCP.
Assessment: Client Needs – Physiological Integrity, 4. The RRT will help to prevent a cardiac
­Reduction of Risk Potential: Cognitive Level – Analysis: or ­respiratory arrest. The HCP should be
Concept – Metabolism. notified to arrange for an ophthalmologist
consult.
91. 1. Nutrition encompasses obesity, and
obesity is a risk factor for developing TEST-TAKING HINT: The test taker should
diabetes mellitus type 2. recognize life-changing or life-threatening
2. Sensory perception may be a problem for cli- complications of a disease process. Failure to
ents who have diabetes because ophthalmo- immediately intervene can result in a “failure
logical issues occur as a result of high blood to rescue” situation.
glucose levels on a prolonged period of time Content – Medical: Integrated Nursing Process – ­
but are not antecedents. Intervention: Client Needs – Physiological Integrity,
3. The concept of pH is a situation that can ­Reduction of Risk Potential: Cognitive Level – Analysis:
­occur as a result of DM1 but not DM2 be- Concept – Metabolism.
cause acidosis results from lactic acid buildup 94. 1. To limit the complications of diabetes
from no insulin production from the pan- the client should keep the blood glucose
creas. Type 2 diabetes clients still produce levels under 140 mg/dL. This can be
some insulin. Insulin resistance is the issue in done with medications, diet, and exer-
type 2 diabetes. cise. Self glucose monitoring allows the
4. Medication is given to treat diabetes but not client to monitor the glucose levels.
to cause it. 2. Testing for urine ketones will not help to
TEST-TAKING HINT: The test taker must know keep the blood glucose level controlled.
risk factors for developing a disease process. 3. Petroleum jelly is rubbed on the feet but not
Content – Medical: Integrated Nursing Process – ­ between the toes.
Assessment: Client Needs – Physiological Integrity, 4. The client should administer sliding-scale
­Reduction of Risk Potential: Cognitive Level – Analysis: insulin when needed but not eat whatever the
Concept – Metabolism. client wishes. The client should still attempt
to control the amount of carbohydrates.
92. 1. The client with hypothyroidism has slowed
body processes so the temperature, pulse, TEST-TAKING HINT: The nurse must
and BP would be brady or lower. ­recommend measures to control or treat
2. All body processes slow as a result of de- d­ isease processes.
creased thyroid production. The client will Content – Medical: Integrated Nursing Process –
be constipated, cold, have thicker skin, low Implementation: Client Needs – Physiological Integrity,
temperature, and bradycardia. Reduction of Risk Potential: Cognitive Level – Analysis:
3. The intake and output would not be affected. Concept – Metabolism.
4. Hypothyroidism does not cause headaches or 95. 1. Obesity is included in the concept of
projectile vomiting. ­nutrition. Obesity is an antecedent of dia-
TEST-TAKING HINT: The test taker must know betes mellitus type 2.
basic signs and symptoms. The word “hypo” 2. Diabetes is a problem of glucose
in the name of the disease would help the metabolism.
test taker eliminate option “1.” 3. The client is at greater risk for develop-
ing infections resulting from the high

08_Colgrove_Ch08.indd 344 02/06/16 8:01 PM


C hapter 8  E ndocrine D isorders 345

circulating glucose levels. Bacteria utilize 96. 1. Eating sweets and high-carbohydrate foods
glucose for energy, as do mammals. can lead to obesity, but eating candy does not
4. Diabetes affects the ability of the blood cause diabetes.
vessels to respond to circulatory need. 2. Birth control pills do not increase the risk of
For a middle-aged male this can result in developing diabetes.
erectile dysfunction. 3. Type 2 diabetes can be more prevalent in
5. Skin integrity is an issue if a pressure sore families, but one cousin does not increase the
or a blister occurs on the feet. If not noted risk of diabetes for the client.
and treated early then an infection can re- 4. Research shows that women who deliv-
sult in amputation. ered a large infant have a greater risk for
TEST-TAKING HINT: The test taker must developing diabetes.
know the disease process and potential TEST-TAKING HINT: The test taker must know
complications. ­antecedents of developing disease processes.
Content – Medical: Integrated Nursing Process – ­ Content – Medical: Integrated Nursing Process – ­
Assessment: Client Needs – Physiological Integrity, Assessment: Client Needs – Physiological Integrity,
­Reduction of Risk Potential: Cognitive Level – Analysis: ­Reduction of Risk Potential: Cognitive Level – Analysis:
Concept – Metabolism. Concept – Metabolism.

08_Colgrove_Ch08.indd 345 02/06/16 8:01 PM


346 M ed -S urg S uccess

ENDOCRINE DISORDERS COMPREHENSIVE


EXAMINATION
1. The nurse is teaching a community class to people 4. Which signs/symptoms should the nurse expect
with type 2 diabetes mellitus. Which explanation to assess in the 31-year-old client who has a
explains the development of type 2 diabetes? sustained release of growth hormone (GH)?
1. The islet cells in the pancreas stop producing 1. An enlarged forehead, maxilla, and face.
insulin. 2. A six (6)-inch increase in height of the client.
2. The client eats too many foods high in sugar. 3. The client complaining of a severe headache.
3. The pituitary gland does not produce 4. A systolic blood pressure of 200 to 300 mm Hg.
vasopressin.
5. Which sign/symptom indicates to the nurse the
4. The cells become resistant to the circulating
client is experiencing hypoparathyroidism?
insulin.
1. A negative Trousseau’s sign.
2. The nurse is teaching the client diagnosed with 2. A positive Chvostek’s sign.
type 2 diabetes mellitus about diet. Which diet 3. Nocturnal muscle cramps.
selection indicates the client understands the 4. Tented skin turgor.
teaching?
6. Which laboratory data make the nurse suspect
1. A submarine sandwich, potato chips, and
the client with primary hyperparathyroidism is
diet cola.
experiencing a complication?
2. Four (4) slices of a supreme thin-crust pizza
1. A serum creatinine level of 2.8 mg/dL.
and milk.
2. A calcium level of 9.2 mg/dL.
3. Smoked turkey sandwich, celery sticks, and
3. A serum triglyceride level of 130 mg/dL.
unsweetened tea.
4. A sodium level of 135 mEq/L.
4. A roast beef sandwich, fried onion rings,
and a cola.
3. The nurse is preparing to administer sliding-scale insulin to a client with type 2 diabetes. The medication
administration record is as follows:
At 1130, the client has a blood glucometer level of 322. Which intervention should the nurse implement?
1. Notify the health-care provider.
2. Administer 10 units of regular insulin.
3. Administer five (5) units of Humalog insulin.
4. Administer 10 units of intermediate-acting insulin.

Client Number: Diagnosis:


Client Name: ABCD 1234567 Allergies: NKA Diabetes Mellitus
MEDICATION ADMINISTRATION RECORD

Medication 0701–1500 1501–2300 2301–0700


Regular insulin 0730 1630
subcutaneously a.c. and h.s. 1130 2100
0–60 = 1 amp D50
61–150 = 0 units
151–300 = 5 units
301–450 = 10 units
Greater than 450 = Call HCP
Metformin 500 mg PO bid 0800 1700
Signature/Initials:

08_Colgrove_Ch08.indd 346 02/06/16 8:01 PM


C hapter 8  E ndocrine D isorders 347

7. The nurse is assessing a client in an outpatient 13. Which signs/symptoms indicate the client
clinic. Which assessment data are a risk factor with hypothyroidism is not taking enough
for developing pheochromocytoma? thyroid hormone?
1. A history of skin cancer. 1. Complaints of weight loss and fine tremors.
2. A history of high blood pressure. 2. Complaints of excessive thirst and urination.
3. A family history of adrenal tumors. 3. Complaints of constipation and being cold.
4. A family history of migraine headaches. 4. Complaints of delayed wound healing and
belching.
8. The client is three (3) days postoperative
unilateral adrenalectomy. Which discharge 14. Which client problem is the nurse’s priority
instructions should the nurse teach? concern for the client diagnosed with acute
1. Discuss the need for lifelong steroid pancreatitis?
replacement. 1. Impaired nutrition.
2. Instruct the client on administration of 2. Skin integrity.
vasopressin. 3. Anxiety.
3. Teach the client to care for the suprapubic 4. Pain relief.
Foley catheter.
15. Which laboratory data indicate to the nurse
4. Tell the client to notify the HCP if the
the client’s pancreatitis is improving?
incision is inflamed.
1. The amylase and lipase serum levels are
9. Which psychosocial problem should be included decreased.
in the plan of care for a female client diagnosed 2. The white blood cell (WBC) count is
with Cushing’s syndrome? decreased.
1. Altered glucose metabolism. 3. The conjugated and unconjugated bilirubin
2. Body image disturbance. levels are decreased.
3. Risk for suicide. 4. The blood urea nitrogen (BUN) serum level
4. Impaired wound healing. is decreased.
10. The nurse is admitting a client to rule out 16. The client diagnosed with acute pancreatitis has
aldosteronism. Which assessment data support a ruptured pseudocyst. Which procedure should
the client’s diagnosis? the nurse anticipate the HCP prescribing?
1. Temperature. 1. Paracentesis.
2. Pulse. 2. Chest tube insertion.
3. Respirations. 3. Lumbar puncture.
4. Blood pressure. 4. Biopsy of the pancreas.
11. Which client history is most significant in the 17. Which signs/symptoms should the nurse expect to
development of symptoms for a client who has assess in the client diagnosed with an insulinoma?
iatrogenic Cushing’s disease? 1. Nervousness, jitteriness, and diaphoresis.
1. Long-term use of anabolic steroids. 2. Flushed skin, dry mouth, and tented skin
2. Extended use of inhaled steroids for asthma. turgor.
3. History of long-term glucocorticoid use. 3. Polyuria, polydipsia, and polyphagia.
4. Family history of increased cortisol 4. Hypertension, tachycardia, and feeling hot.
production.
18. Which risk factor should the nurse expect to find
12. The client is one (1) hour postoperative in the client diagnosed with pancreatic cancer?
thyroidectomy. Which intervention should the 1. Chewing tobacco.
nurse implement? 2. Low-fat diet.
1. Check the posterior neck for bleeding. 3. Chronic alcoholism.
2. Assess the client for the Chvostek’s sign. 4. Exposure to industrial chemicals.
3. Monitor the client’s serum calcium level.
4. Change the client’s surgical dressing.

08_Colgrove_Ch08.indd 347 02/06/16 8:01 PM


348 M ed -S urg S uccess

19. The nurse is discussing the endocrine system 25. In which area should the nurse administer the
with the client. Which endocrine gland secretes regular insulin to ensure the best absorption of
epinephrine and norepinephrine? the medication?
1. The pancreas.
2. The adrenal cortex.
3. The adrenal medulla.
4. The anterior pituitary gland.
20. Which question should the nurse ask
when assessing the client for an endocrine
dysfunction?
1. “Have you noticed any pain in your legs when

C
walking?”
2. “Have you had any unexplained weight loss?”
3. “Have you noticed any change in your bowel
movements?”

B
4. “Have you experienced any joint pain or

D
discomfort?”
21. Which nursing instruction should the nurse
discuss with the client who is receiving
glucocorticoids for Addison’s disease?
1. Discuss the importance of tapering A
medications when discontinuing
medication.
2. Explain the dose may need to be increased
during times of stress or infection.
3. Instruct the client to take medication on an
empty stomach with a glass of water.
4. Encourage the client to wear clean white
socks when wearing tennis shoes.
22. The client with chronic alcoholism has chronic
pancreatitis and hypomagnesemia. Which data 1. A
should the nurse assess when administering 2. B
magnesium sulfate to the client? 3. C
1. Deep tendon reflexes. 4. D
2. Arterial blood gases.
26. The client diagnosed with type 1 diabetes
3. Skin turgor.
mellitus received regular insulin two (2) hours
4. Capillary refill time.
ago. The client is complaining of being jittery
23. Which endocrine disorder should the nurse and nervous. Which interventions should
assess for in the client who has a closed head the nurse implement? List in order of priority.
injury with increased intracranial pressure? 1. Call the laboratory to confirm blood glucose
1. Pheochromocytoma. level.
2. Diabetes insipidus. 2. Administer a quick-acting carbohydrate.
3. Hashimoto’s thyroiditis. 3. Have the client eat a bologna sandwich.
4. Gynecomastia. 4. Check the client’s blood glucose level at the
bedside.
24. Which sign/symptom should the nurse expect
5. Determine if the client has had anything
in the client diagnosed with syndrome of
to eat.
inappropriate antidiuretic hormone (SIADH)?
1. Excessive thirst.
2. Orthopnea.
3. Ascites.
4. Concentrated urine output.

08_Colgrove_Ch08.indd 348 02/06/16 8:01 PM


C hapter 8  E ndocrine D isorders 349

27. The nurse in the intensive care unit is caring for a client 20 hours postoperative Whipple’s procedure for
cancer of the pancreas who has two (2) intravenous lines running via a central catheter and one (1) unit of
packed red blood cells in a peripheral line. The client’s intake and output record from the previous day is
as follows:

Oral Intravenous Urine Nasogastric Other (Specify)


Day One (in mL) (in mL) (in mL) Tube (in mL) (in mL)
0701–1500 1,275 1,250 2,000 250 Emesis 400
1501–2300 680 1,000 1,100 890
2301–0700 260 875 1,400 1,025
Total 2,215 3,125 4,500 2,165 400

Based on these findings, which action should the nurse implement?


1. Slow down the intravenous rates and notify the HCP immediately.
2. Assess the client for change in sensorium daily and stay in the room.
3. Continue to assess intake and output every one (1) hour.
4. Increase the blood rate to infuse rapidly and crossmatch for two (2) more units.

08_Colgrove_Ch08.indd 349 02/06/16 8:01 PM


ENDOCRINE DISORDERS COMPREHENSIVE
EXAMINATION ANSWERS AND RATIONALES

1. 1. This is the cause of type 1 diabetes mellitus. during adulthood, most commonly because
2. This may be a reason for obesity, which may of a pituitary tumor.
lead to type 2 diabetes, but eating too much 2. Gigantism occurs when GH hypersecretion
sugar does not cause diabetes. begins before puberty when the closure of the
3. This is the explanation for diabetes insipidus, epiphyseal plates occurs. Note the age of the
which should not be confused with ­diabetes client.
mellitus. 3. A severe headache is not a symptom of
4. Normally insulin binds to special recep- acromegaly.
tors sites on the cell and initiates a series of 4. High blood pressure is a sign of
reactions involved in metabolism. In type pheochromocytoma.
2 diabetes, these reactions are diminished Content – Medical: Integrated Nursing Process –
primarily as a result of obesity and aging. Assessment: Client Needs – Physiological Integrity, ­Reduction
Content – Medical: Integrated Nursing Process – of Risk Potential: Cognitive Level – Analysis: Concept –
Diagnosis: Client Needs – Safe Effective Care E
­ nvironment, Metabolism.
Management of Care: Cognitive Level – Analysis:
5. 1. A carpopedal spasm occurs when the blood
Concept – Nursing Roles.
flow to the arm is decreased for three (3)
2. 1. A submarine sandwich is on a bun-type bread minutes with a blood pressure cuff; a positive
and is usually six (6) to 12 inches long, and Trousseau’s sign indicates hypocalcemia, which
potato chips add fat and more carbohydrates to is a sign of hypoparathyroid function. A nor-
the meal. mal Trousseau means the body is functioning
2. Four (4) slices of pizza contain excessive num- as it should.
bers of carbohydrates, plus cheese and meats, 2. When a sharp tapping over the facial nerve
and whole milk is high in fat. elicits a spasm or twitching of the mouth,
3. Turkey is a low-fat meat. A sandwich usu- nose, or eyes, the client is hypocalcemic,
ally means normal slices of bread, and the which occurs in clients with hypopara-
client needs at least 50% carbohydrates in thyroidism. This is known as a positive
each meal. Celery sticks are not counted as ­Chvostek’s sign.
carbohydrates. 3. Muscle cramps makes the nurse suspect
4. The roast beef sandwich is high in carbohy- hypokalemia (low potassium).
drates, fried onion rings are high in fat, and a 4. Tented skin turgor makes the nurse ­suspect
regular cola is high in carbohydrates. ­dehydration, which occurs with hypernatremia.
Content – Medical: Integrated Nursing Process – Content – Medical: Integrated Nursing Process –
Evaluation: Client Needs – Physiological Integrity, Basic Assessment: Client Needs – Physiological Integrity,
Care and Comfort: Cognitive Level – Synthesis: Concept – Reduction of Risk Potential: Cognitive Level –
Metabolism. Analysis: Concept – Metabolism.

3. 1. The client’s blood glucose level does not war- 6. 1. A serum creatinine level of 2.8 mg/dL indi-
rant notifying the HCP. cates the client is in renal failure, which is a
2. According to the sliding scale, any blood complication of hyperparathyroidism. The
glucose reading between 301 and 450 formation of stones in the kidneys related
requires 10 units of regular insulin, which to the increased urinary excretion of cal-
is fast-acting insulin. cium and phosphorus occurs in about 55%
3. Humalog is rapid-acting insulin, but the ­order of clients with primary hyperparathyroid-
reads regular insulin. ism and can lead to ­renal failure.
4. Intermediate-acting insulin, NPH or ­ 2. This calcium level is within the normal range
Humulin N, is not regular insulin. of 9 to 10.5 mg/dL.
Content – Medical: Integrated Nursing Process – 3. This serum triglyceride level is within the
Implementation: Client Needs – Safe Effective Care ­normal range of 40 to 150 mg/dL in males
­Environment, Management of Care: Cognitive Level – and 30 to 140 mg/dL for females.
Application: Concept – Metabolism. 4. This sodium level is within the normal range
of 135 to 145 mEq/L.
4. 1. Acromegaly (enlarged extremities) ­occurs
when sustained GH hypersecretion begins
350

08_Colgrove_Ch08.indd 350 02/06/16 8:01 PM


C hapter 8  E ndocrine D isorders 351

Content – Medical: Integrated Nursing Process – 4. Blood pressure is affected by aldoste-


Assessment: Client Needs – Physiological Integrity, ronism, with hypertension being the
Reduction of Risk Potential: Cognitive Level – most prominent and universal sign of
Analysis: Concept – Metabolism. aldosteronism.
7. 1. A history of skin cancer is not a risk factor for Content – Medical: Integrated Nursing Process –
pheochromocytoma. Assessment: Client Needs – Physiological Integrity,
2. A history of high blood pressure is a sign of Reduction of Risk Potential: Cognitive Level –
this disease, not a risk factor for developing it. Analysis: Concept – Metabolism.
3. There is a high incidence of pheochromo- 11. 1. Anabolic steroids are used by individuals to
cytomas in family members with adrenal build muscle mass. Long-term use can lead to
tumors, and the von Hippel-Lindau gene psychosis or heart attacks.
is thought to be a primary cause. 2. Inhaled steroids do not have systemic effects,
4. Headaches are a symptom of this disease but which is described by iatrogenic Cushing’s
not a risk factor for it. disease.
Content – Medical: Integrated Nursing Process – 3. Iatrogenic Cushing’s disease is Cushing’s
Diagnosis: Client Needs – Health Promotion and disease caused by medical treatment—in
­Maintenance: Cognitive Level – Analysis: Concept – this case, by taking excessive steroids
Metabolism. resulting in the symptoms of moon
8. 1. Because the client has one adrenal gland face, buffalo hump, and other associated
remaining, the client may not need lifelong symptoms.
supplemental steroids. 4. Family history does not cause iatrogenic
2. Vasopressin is administered to clients with problems.
diabetes insipidus. Content – Medical: Integrated Nursing Process –
3. The client does not have a suprapubic cath- Assessment: Client Needs – Safe Effective Care
eter during this procedure. ­Environment, Management of Care: Cognitive Level –
4. Any inflammation of the incision indicates Analysis: Concept – Metabolism.
an infection and the client will need to 12. 1. The incision for a thyroidectomy allows
receive antibiotics, so the HCP must be the blood to drain dependently by gravity
notified. to the back of the client’s neck. Therefore,
Content – Surgical: Integrated Nursing Process – the nurse should check this area for hem-
Planning: Client Needs – Physiological Integrity, orrhaging, which is a possible complica-
­Physiological Adaptation: Cognitive Level – Synthesis: tion of any surgery.
Concept – Perioperative. 2. The Chvostek’s sign indicates hypocalcemia,
9. 1. This is not a psychosocial problem; it is a which is too early to assess for in this client.
physiological problem in clients diagnosed 3. Accidental removal of or damage to the para-
with Cushing’s syndrome. thyroid glands will not decrease the calcium
2. The client with Cushing’s syndrome has level for at least 24 hours.
body changes, including moon face, buf- 4. Surgeons prefer to change the surgical
falo hump, truncal obesity, hirsutism, and ­dressing for the first time.
striae and bruising, all of which affect the Content – Surgical: Integrated Nursing Process –
client’s body image. Implementation: Client Needs – Safe Effective Care
3. This is a psychosocial problem, but it is not ­Environment, Management of Care: Cognitive
one that commonly occurs in clients diag- Level – Application: Concept – Perioperative.
nosed with Cushing’s syndrome. 13. 1. Weight loss and fine tremors make the nurse
4. This is not a psychosocial problem; it is a suspect the client is taking too much thyroid
physiological problem, which does occur in hormone because these are symptoms of
clients diagnosed with Cushing’s syndrome. hyperthyroidism.
Content – Medical: Integrated Nursing Process – 2. Excessive thirst and urination are symptoms
­Diagnosis: Client Needs – Psychosocial Integrity: Cognitive of diabetes.
Level – Analysis: Concept – Metabolism. 3. If the client were not taking enough thy-
10. 1. The temperature is not affected by roid hormone, the client would exhibit
aldosteronism. symptoms of hypothyroidism such as con-
2. The pulse is not affected by this disorder. stipation and being cold.
3. The respirations are not affected by this 4. This indicates Cushing’s disease.
disorder.

08_Colgrove_Ch08.indd 351 02/06/16 8:01 PM


352 M ed -S urg S uccess

Content – Medical: Integrated Nursing Process – 3. These are signs/symptoms of hyperglycemia.


Evaluation: Client Needs – Physiological Integrity, 4. These are signs/symptoms of
­Pharmacological and Parenteral Therapies: Cognitive hyperthyroidism.
Level – Synthesis: Concept – Metabolism.
Content – Medical: Integrated Nursing Process –
14. 1. The client is NPO and can live without food Assessment: Client Needs – Physiological Integrity,
for a number of days as long as he or she Reduction of Risk Potential: Cognitive Level –
­receives fluids. Analysis: Concept – Metabolism.
2. The client is not on strict bedrest and can 18. 1. A history of smoking cigarettes is pertinent,
move about in the bed; therefore, skin integ- but a history of chewing tobacco is not.
rity is not a priority problem. In pancreatitis, 2. A diet high in fat, not low in fat, is a risk factor.
the tissue damage is internal. 3. Chronic alcoholism is not a risk factor, but
3. The client may be anxious, but psychosocial chronic pancreatitis is a risk factor.
problems are not priority. 4. Exposure to industrial chemicals or
4. The client with pancreatitis is in excruci- ­environmental toxins is a risk factor for
ating pain because the enzymes are pancreatic cancer.
autodigesting the pancreas; severe Content – Medical: Integrated Nursing Process –
abdominal pain is the hallmark symptom Assessment: Client Needs – Physiological Integrity,
of pancreatitis. Reduction of Risk Potential: Cognitive Level –
Content – Medical: Integrated Nursing Process – Analysis: Concept – Cellular Regulation.
Diagnosis: Client Needs – Safe Effective Care E
­ nvironment,
Management of Care: Cognitive Level – Analysis: 19. 1. The endocrine function of the pancreas is the
Concept – Metabolism. secretion of insulin and amylin.
2. The adrenal cortex secretes mineralocorti-
15. 1. These laboratory data are used to diag- coids, ­glucocorticoids, and gonadotrophins.
nose and monitor pancreatitis because 3. The adrenal medulla secretes the
amylase and lipase are the enzymes catecholamines epinephrine and
produced by the pancreas. norepinephrine.
2. Pancreatitis is not an infection of the pan- 4. The anterior pituitary gland secretes the
creas resulting from bacteria; such an infec- growth hormone.
tion causes an elevation in the WBCs. Content – Medical: Integrated Nursing Process –
3. Bilirubin is used to monitor liver problems. Diagnosis: Client Needs – Safe Effective Care E
­ nvironment,
4. BUN monitors kidney function. Management of Care: Cognitive Level – Analysis:
Content – Medical: Integrated Nursing Process – Concept – Metabolism.
Assessment: Client Needs – Physiological Integrity,
Reduction of Risk Potential: Cognitive Level – 20. 1. Leg pain when walking indicates intermittent
Analysis: Concept – Metabolism. claudication, which occurs with peripheral
vascular disease.
16. 1. A paracentesis is used to remove fluid from 2. Weight loss with normal appetite may
the abdominal cavity. ­indicate hyperthyroidism.
2. The pancreas lies immediately below 3. Changes in bowel movements may indicate
the diaphragm. When the cyst ruptures, colon cancer.
alkaline substances in the abdomen cause 4. Joint pain indicates a musculoskeletal or
fluid leaks at the esophageal diaphragmatic ­degenerative joint disease.
opening into the thorax. The fluid must be Content – Medical: Integrated Nursing Process –
removed to prevent lung collapse. Assessment: Client Needs – Physiological Integrity,
3. Lumbar puncture is used to diagnose Reduction of Risk Potential: Cognitive Level –
meningitis. Analysis: Concept – Metabolism.
4. Biopsies are performed to confirm a diagno-
sis; they are not used for treatment. 21. 1. The client will have to receive this medica-
tion the rest of his or her life, so this should
Content – Medical: Integrated Nursing Process –
not be discussed with the client.
Planning: Client Needs – Safe Effective Care Environment,
Management of Care: Cognitive Level – Synthesis: 2. During times of stress, the medication
Concept – Metabolism. may need to be increased to prevent
­adrenal insufficiency.
17. 1. Insulinoma is a tumor of the islet cells of 3. The medication should be taken with food to
the pancreas that produces insulin. The minimize its ulcerogenic effect.
signs/symptoms of an insulinoma are 4. Wearing white socks with tennis shoes is not
signs of hypoglycemia. an intervention pertinent to a client diag-
2. These are signs/symptoms of hyperglycemia. nosed with Addison’s disease.

08_Colgrove_Ch08.indd 352 02/06/16 8:01 PM


C hapter 8  E ndocrine D isorders 353

Content – Medical: Integrated Nursing Process – 25. 1. The anterior thigh is an appropriate area, but
Planning: Client Needs – Physiological Integrity, it does not provide the best absorption.
­Pharmacological and Parenteral Therapies: Cognitive 2. The abdominal area allows for the most
Level – Synthesis: Concept – Medication. rapid absorption of insulin and is the
22. 1. If deep tendon reflexes are hypoactive or recommended site.
absent, the nurse should hold the magne- 3. The deltoid is an appropriate area, but it does
sium and notify the health-care provider. not provide the most rapid absorption.
2. The arterial blood gases are not affected by 4. The gluteal buttocks area is primarily the best
the serum magnesium level. area for intramuscular injections.
3. The client’s skin turgor will not be affected Content – Medical: Integrated Nursing Process –
by the client’s serum magnesium level. Implementation: Client Needs – Safe Effective Care
4. The client’s capillary refill time is not ­affected ­Environment, Management of Care: Cognitive
by the client’s serum magnesium level. Level – Comprehension: Concept – Medication.
Content – Medical: Integrated Nursing Process – 26. In order of performance: 5, 2, 4, 1, 3.
Implementation: Client Needs – Physiological 5. Regular insulin peaks in two (2) to four
Integrity, Pharmacological and Parenteral Therapies: (4) hours; therefore, the nurse should sus-
Cognitive Level – Application: Concept – Medication. pect a hypoglycemic reaction if the client
23. 1. This is a tumor of the adrenal medulla. has not eaten anything.
2. Diabetes insipidus can be caused by brain 2. The antidote for insulin is glucose; there-
tumors or infections, pituitary surgery, fore, the nurse should give the client some
cerebrovascular accidents, or renal and type of quick-acting food source.
organ failure, or it may be a complication 4. The nurse should obtain the client’s blood
of a closed head injury with increased in- glucose level as soon as possible; this can
tracranial pressure. Diabetes insipidus is be done with a glucometer at the bedside.
a result of antidiuretic hormone (ADH) 1. Most hospitals require a confirmatory se-
insufficiency. rum blood glucose level. Do not wait for
3. Hashimoto’s thyroiditis causes results to give food.
hypothyroidism. 3. A source of long-acting carbohydrate
4. Gynecomastia is abnormal enlargement of and protein should be given to prevent
breast tissue in men. a reoccurrence of hypoglycemia.
Content – Medical: Integrated Nursing Process – Content – Medical: Integrated Nursing Process –
Assessment: Client Needs – Physiological Integrity, Implementation: Client Needs – Safe Effective Care Envi-
Reduction of Risk Potential: Cognitive Level – ronment, Management of Care: Cognitive
Analysis: Concept – Metabolism. Level – Analysis: Concept – Metabolism.

24. 1. Excessive thirst is a symptom of diabetes 27. 1. The IV rates are preventing the client from
insipidus, which is a deficiency of antidiuretic developing hypovolemic shock as a result
(ADH) hormone. of third spacing after a major abdominal
2. Orthopnea is difficulty breathing when in the surgery. The ICU nurse must be vigilant
supine position, which is not a sign/symptom for third spacing and fluid shifts following
of SIADH. major trauma to the body.
3. Ascites is excess fluid in the peritoneal cavity, 2. The sensorium must be assessed more fre-
which is not a sign/symptom of SIADH. quently than daily, and the nurse does not
4. Excess antidiuretic hormone (ADH) have to remain with the client continuously.
causes SIADH, which causes increased 3. The intake and output should be assessed
water reabsorption and leads to increased hourly to determine kidney perfusion and
fluid volume and scant, concentrated to determine when the fluids have stopped
urine. shifting to the surgical area.
Content – Medical: Integrated Nursing Process – 4. There is no evidence that the client has be-
Assessment: Client Needs – Physiological Integrity, gun to bleed and requires faster infusion and
Reduction of Risk Potential: Cognitive Level – another crossmatch.
Analysis: Concept – Fluid and Electrolyte Balance.

08_Colgrove_Ch08.indd 353 02/06/16 8:01 PM


08_Colgrove_Ch08.indd 354 02/06/16 8:01 PM
Genitourinary
Disorders
The possession of knowledge does not kill the sense of wonder and
mystery. There is always more mystery.
9
—Anaïs Nin

The genitourinary system includes the kidneys; ureters; bladder; urethra; and the ­associated
organ, the prostate gland. The urinary system organs are subject to acute and chronic in-
fections and functioning disorders/diseases and, for some organs, the formation of calculi
(stones). These disorders can lead to fluid and electrolyte disorders having wide-ranging
effects throughout the body. The prostate is also subject to disorders, a very common one
being benign prostatic hypertrophy. (Cancer of the prostate is discussed in Chapter 10.)
The nurse must know the normal laboratory values of the many tests used to assess the
function of these organs and recognize what independent and collaborative interventions
are necessary.

KEYWORDS ABBREVIATIONS
Anuria Acute renal failure (ARF)
Azotemia Benign prostatic hypertrophy (BPH)
Corrected output Bicarbonate (HCO3)
Creatinine Blood urea nitrogen (BUN)
Diffusion Chronic kidney disease (CKD)
Hyperkalemia Computed tomography (CT)
Intravesical End-stage renal disease (ESRD)
Oliguria Gastrointestinal (GI)
Osmolality Glomerular filtration rate (GFR)
Osmosis Health-care provider (HCP)
Intensive care department (ICD)
Intravenous (IV)
Intravenous piggyback (IVPB)
Intravenous push (IVP)
Licensed practical nurse (LPN)
Magnetic resonance imaging (MRI) scan
Nasogastric tube (NGT)
Nothing by mouth (NPO)
Nurse practitioner (NP)
Patient-controlled analgesia (PCA)
Premature ventricular contraction (PVC)
Prostate-specific antigen (PSA)
Rule out (R/O)
Transurethral resection of the prostate (TURP)

355

09_Colgrove_Ch09.indd 355 01/06/16 5:45 PM


356 M ed -S urg S uccess

Unlicensed assistive personnel (UAP)


Urinary tract infection (UTI)
When required, as needed (prn)
White blood cells (WBCs)

PRACTICE QUESTIONS
Acute Renal Failure (ARF) 6. The nurse is developing a plan of care for a client
diagnosed with ARF. Which statement is an
1. The nurse is admitting a client diagnosed with appropriate outcome for the client?
acute renal failure (ARF). Which question is 1. Monitor intake and output every shift.
most important for the nurse to ask during the 2. Decrease of pain by three (3) levels on a
admission interview? 1-to-10 scale.
1. “Have you recently traveled outside the United 3. Electrolytes are within normal limits.
States?” 4. Administer enemas to decrease hyperkalemia.
2. “Did you recently begin a vigorous exercise 7. The client diagnosed with ARF is admitted to
program?” the intensive care department and placed on a
3. “Is there a chance you have been exposed to a therapeutic diet. Which diet is most appropriate
virus?” for the client?
4. “What over-the-counter medications do you 1. A high-potassium and low-calcium diet.
take regularly?” 2. A low-fat and low-cholesterol diet.
2. The nurse is caring for a client diagnosed 3. A high-carbohydrate and restricted-protein diet.
with ARF. Which laboratory values are most 4. A regular diet with six (6) small feedings a day.
significant for diagnosing ARF? 8. The client diagnosed with ARF is placed on
1. BUN and creatinine. bedrest. The client asks the nurse, “Why do I have
2. WBC and hemoglobin. to stay in bed? I don’t feel bad.” Which scientific
3. Potassium and sodium. rationale supports the nurse’s response?
4. Bilirubin and ammonia level. 1. Bedrest helps increase the blood return to the
3. The nurse is caring for a client diagnosed with renal circulation.
rule-out ARF. Which condition predisposes the 2. Bedrest reduces the metabolic rate during the
client to developing prerenal failure? acute stage.
1. Diabetes mellitus. 3. Bedrest decreases the workload of the left side
2. Hypotension. of the heart.
3. Aminoglycosides. 4. Bedrest aids in reduction of peripheral and
4. Benign prostatic hypertrophy. sacral edema.
4. The client is diagnosed with ARF. Which 9. The nurse and an unlicensed assistive personnel
signs/symptoms indicate to the nurse the (UAP) are caring for clients on a medical floor.
client is in the recovery period? Select all that Which nursing task is most appropriate for the
apply. nurse to delegate?
1. Increased alertness and no seizure activity. 1. Collect a clean voided midstream urine
2. Increase in hemoglobin and hematocrit. specimen.
3. Denial of nausea and vomiting. 2. Evaluate the client’s eight (8)-hour intake and
4. Decreased urine-specific gravity. output.
5. Increased serum creatinine level. 3. Assist in checking a unit of blood prior to
hanging.
5. The client diagnosed with ARF has a serum 4. Administer a cation-exchange resin enema.
potassium level of 6.8 mEq/L. Which
collaborative treatment should the nurse
anticipate for the client?
1. Administer a phosphate binder.
2. Type and crossmatch for whole blood.
3. Assess the client for leg cramps.
4. Prepare the client for dialysis.

09_Colgrove_Ch09.indd 356 01/06/16 5:45 PM


C hapter 9  G enitourinary D isorders 357

10. The client is admitted to the emergency 14. The nurse in the dialysis center is initiating the
department after a gunshot wound to the morning dialysis run. Which client should the
abdomen. Which nursing intervention should nurse assess first?
the nurse implement first to prevent ARF? 1. The client who has hemoglobin of 9.8 g/dL
1. Administer normal saline IV. and hematocrit of 30%.
2. Take vital signs. 2. The client who does not have a palpable thrill
3. Place client on telemetry. or auscultated bruit.
4. Assess abdominal dressing. 3. The client who is complaining of being
exhausted and is sleeping.
11. The UAP tells the nurse the client with ARF has a
4. The client who did not take antihypertensive
white crystal-like layer on top of the skin. Which
medication this morning.
intervention should the nurse implement?
1. Have the assistant apply a moisture barrier 15. The male client diagnosed with CKD has
cream to the skin. received the initial dose of erythropoietin, a
2. Instruct the UAP to bathe the client in cool biologic response modifier, one (1) week ago.
water. Which complaint by the client indicates the
3. Tell the UAP not to turn the client in this need to notify the health-care provider?
condition. 1. The client complains of flu-like symptoms.
4. Explain this is normal and do not do anything 2. The client complains of being tired all
for the client. the time.
3. The client reports an elevation in his blood
12. The client diagnosed with ARF is experiencing
pressure.
hyperkalemia. Which medication should the
4. The client reports discomfort in his legs
nurse prepare to administer to help decrease the
and back.
potassium level?
1. Erythropoietin. 16. The nurse is developing a nursing care plan for
2. Calcium gluconate. the client diagnosed with CKD. Which nursing
3. Regular insulin. problem is priority for the client?
4. Osmotic diuretic. 1. Low self-esteem.
2. Knowledge deficit.
3. Activity intolerance.
Chronic Kidney Disease (CKD) 4. Excess fluid volume.
13. The nurse is caring for the client diagnosed 17. The client with CKD is placed on a fluid
with chronic kidney disease (CKD) who restriction of 1,500 mL/day. On the 7 a.m.
is experiencing metabolic acidosis. Which to 7 p.m. shift the client drank an eight (8)-ounce
statement best describes the scientific rationale cup of coffee, 4 ounces of juice, 12 ounces of tea,
for metabolic acidosis in this client? and 2 ounces of water with medications. What
1. There is an increased excretion of phosphates amount of fluid can the 7 p.m. to 7 a.m. nurse give
and organic acids, which leads to an increase to the client? _____________
in arterial blood pH.
18. The client diagnosed with CKD has a new
2. A shortened life span of red blood cells
arteriovenous fistula in the left forearm. Which
because of damage secondary to dialysis
intervention should the nurse implement?
treatments in turn leads to metabolic acidosis.
1. Teach the client to carry heavy objects with
3. The kidney cannot excrete increased levels of
the right arm.
acid because they cannot excrete ammonia or
2. Perform all laboratory blood tests on the
cannot reabsorb sodium bicarbonate.
left arm.
4. An increase in nausea and vomiting causes a
3. Instruct the client to lie on the left arm during
loss of hydrochloric acid and the respiratory
the night.
system cannot compensate adequately.
4. Discuss the importance of not performing any
hand exercises.

09_Colgrove_Ch09.indd 357 01/06/16 5:45 PM


358 M ed -S urg S uccess

19. The male client diagnosed with CKD secondary 24. The client receiving hemodialysis is being
to diabetes has been receiving dialysis for 12 discharged home from the dialysis center. Which
years. The client is notified he will not be placed instruction should the nurse teach the client?
on the kidney transplant list. The client tells the 1. Notify the HCP if oral temperature is 102°F
nurse he will not be back for any more dialysis or greater.
treatments. Which response by the nurse is most 2. Apply ice to the access site if it starts bleeding
therapeutic? at home.
1. “You cannot just quit your dialysis. This is not 3. Keep fingernails short and try not to scratch
an option.” the skin.
2. “You’re angry at not being on the list, and you 4. Encourage the significant other to make
want to quit dialysis?” decisions for the client.
3. “I will call your nephrologist right now so you
can talk to the HCP.”
4. “Make your funeral arrangements because you Fluid and Electrolyte Disorders
are going to die.”
25. The client is admitted to a nursing unit from a
20. The nurse is discussing kidney transplants with long-term care facility with a hematocrit of 56%
clients at a dialysis center. Which population is and a serum sodium level of 152 mEq/L. Which
less likely to participate in organ donation? condition is a cause for these findings?
1. Caucasian. 1. Overhydration.
2. African American. 2. Anemia.
3. Asian. 3. Dehydration.
4. Hispanic. 4. Renal failure.
21. The client receiving dialysis is complaining of 26. The client who has undergone an exploratory
being dizzy and light-headed. Which action laparotomy and subsequent removal of a large
should the nurse implement first? intestinal tumor has a nasogastric tube (NGT) in
1. Place the client in the Trendelenburg position. place and an IV running at 150 mL/hr via an
2. Turn off the dialysis machine immediately. IV pump. Which data should be reported to
3. Bolus the client with 500 mL of normal saline. the HCP?
4. Notify the health-care provider as soon as 1. The pump keeps sounding an alarm indicating
possible. the high pressure has been reached.
22. The nurse caring for a client diagnosed with 2. Intake is 1,800 mL, NGT output is 550 mL,
CKD writes a client problem of “noncompliance and Foley output is 950 mL.
with dietary restrictions.” Which intervention 3. On auscultation, crackles and rhonchi in all
should be included in the plan of care? lung fields are noted.
1. Teach the client the proper diet to eat while 4. Client has negative pedal edema and an
undergoing dialysis. increasing level of consciousness.
2. Refer the client and significant other to the 27. The client diagnosed with diabetes insipidus
dietitian. weighed 180 pounds when the daily weight
3. Explain the importance of eating the proper was taken yesterday. This morning’s weight
foods. is 175.6 pounds. One liter of fluid weighs
4. Determine the reason for the client not approximately 2.2 pounds. How much fluid
adhering to the diet. has the client lost? _______
23. The client diagnosed with CKD is receiving 28. The nurse writes the client problem of “fluid
peritoneal dialysis. Which assessment data volume excess” (FVE). Which intervention
warrant immediate intervention by the nurse? should be included in the plan of care?
1. Inability to auscultate a bruit over the fistula. 1. Change the IV fluid from 0.9% NS to D5W.
2. The client’s abdomen is soft, is nontender, and 2. Restrict the sodium in the client’s diet.
has bowel sounds. 3. Monitor blood glucose levels.
3. The dialysate being removed from the client’s 4. Prepare the client for hemodialysis.
abdomen is clear.
4. The dialysate instilled was 1,500 mL and
removed was 1,500 mL.

09_Colgrove_Ch09.indd 358 01/06/16 5:45 PM


C hapter 9  G enitourinary D isorders 359

29. The client is admitted with a serum sodium 34. The client has received IV solutions for three
level of 110 mEq/L. Which nursing intervention (3) days through a 20-gauge IV catheter placed
should be implemented? in the left cephalic vein. On morning rounds,
1. Encourage fluids orally. the nurse notes the IV site is tender to palpation
2. Administer 10% saline solution IVPB. and a red streak has formed. Which intervention
3. Administer antidiuretic hormone intranasally. should the nurse implement first?
4. Place on seizure precautions. 1. Start a new IV in the right hand.
2. Discontinue the intravenous line.
30. The telemetry monitor technician notifies the
3. Complete an incident record.
nurse of the morning telemetry readings. Which
4. Place a warm washrag over the site.
client should the nurse assess first?
1. The client in normal sinus rhythm with a 35. The nurse and an unlicensed assistive personnel
peaked T wave. (UAP) are caring for a group of clients. Which
2. The client diagnosed with atrial fibrillation nursing intervention should the nurse perform?
with a rate of 100. 1. Measure the client’s output from the
3. The client diagnosed with a myocardial indwelling catheter.
infarction who has occasional PVCs. 2. Record the client’s intake and output on the
4. The client with a first-degree atrioventricular I&O sheet.
block and a rate of 92. 3. Instruct the client on appropriate fluid
restrictions.
31. The client who is post-thyroidectomy complains
4. Provide water for a client diagnosed with
of numbness and tingling around the mouth
diabetes insipidus.
and the tips of the fingers. Which intervention
should the nurse implement first? 36. The client has been vomiting and has had
1. Notify the health-care provider immediately. numerous episodes of diarrhea. Which
2. Tap the cheek about two (2) cm anterior to laboratory test should the nurse monitor?
the earlobe. 1. Serum calcium.
3. Check the serum calcium and magnesium 2. Serum phosphorus.
levels. 3. Serum potassium.
4. Prepare to administer calcium gluconate IVP. 4. Serum sodium.
32. The nurse is caring for a client diagnosed with
diabetic ketoacidosis (DKA). Which statement Urinary Tract Infection (UTI)
best explains the scientific rationale for the
client’s Kussmaul’s respirations? 37. The client from a long-term care facility is
1. The kidneys produce excess urine and the admitted to the medical unit with a fever, hot
lungs try to compensate. flushed skin, and clumps of white sediment in
2. The respirations increase the amount of the indwelling catheter. Which intervention
carbon dioxide in the bloodstream. should the nurse implement first?
3. The lungs speed up to release carbon dioxide 1. Start an IV with a 20-gauge catheter.
and increase the pH. 2. Initiate antibiotic therapy IVPB.
4. The shallow and slow respirations will 3. Collect a urine specimen for culture.
increase the HCO3 in the serum. 4. Change the indwelling catheter.
33. The client is NPO and is receiving total 38. The nurse is inserting an indwelling catheter
parenteral nutrition (TPN) via a subclavian line. into a female client. Which interventions
Which precautions should the nurse implement? should be implemented? Rank in the order of
Select all that apply. performance.
1. Place the solution on an IV pump at the 1. Explain the procedure to the client.
prescribed rate. 2. Set up the sterile field.
2. Monitor blood glucose every six (6) hours. 3. Inflate the catheter bulb.
3. Weigh the client weekly, first thing in the 4. Place absorbent pads under the client.
morning. 5. Clean the perineum from clean to dirty with
4. Change the IV tubing every three (3) days. Betadine.
5. Monitor intake and output every shift.

09_Colgrove_Ch09.indd 359 01/06/16 5:45 PM


360 M ed -S urg S uccess

39. The nurse performs bladder irrigation through 44. The nurse is caring for a pregnant client
an indwelling catheter. The nurse instilled diagnosed with acute pyelonephritis. Which
90 mL of sterile normal saline. The catheter scientific rationale supports the client being
drained 710 mL. What is the client’s output? hospitalized for this condition?
________ 1. The client must be treated aggressively to
prevent maternal/fetal complications.
40. The nurse is examining a 15-year-old female
2. The nurse can force the client to drink
who is complaining of pain, frequency, and
fluids and avoid nausea and vomiting.
urgency when urinating. After asking the parent
3. The client will be dehydrated and there won’t
to leave the room, which question should the
be sufficient blood flow to the baby.
nurse ask the client?
4. Pregnant clients historically are afraid to take
1. “When was your last menstrual cycle?”
the antibiotics as ordered.
2. “Have you noticed any change in the color of
the urine?” 45. The nurse is discharging a client with a health-
3. “Are you sexually active?” care facility acquired urinary tract infection.
4. “What have you taken for the pain?” Which information should the nurse include
in the discharge teaching?
41. The client is reporting chills, fever, and left
1. Limit fluid intake so the urinary tract
costovertebral pain. Which diagnostic test should
can heal.
the nurse expect the HCP to prescribe first?
2. Collect a routine urine specimen for
1. A midstream urine for culture.
culture.
2. A sonogram of the kidney.
3. Take all the antibiotics as prescribed.
3. An intravenous pyelogram for renal calculi.
4. Tell the client to void every five (5) to six
4. A CT scan of the kidneys.
(6) hours.
42. The nurse is caring for a client with chronic
46. The nurse is preparing a plan of care for the
pyelonephritis. Which assessment data support
client diagnosed with acute glomerulonephritis.
the diagnosis of chronic pyelonephritis?
Which statement is an appropriate long-term
1. The client has fever, chills, flank pain, and
goal?
dysuria.
1. The client will have a blood pressure within
2. The client complains of fatigue, headaches,
normal limits.
and increased urination.
2. The client will show no protein in the urine.
3. The client had a group B beta-hemolytic strep
3. The client will maintain normal renal
infection last week.
function.
4. The client has an acute viral pneumonia
4. The client will have clear lung sounds.
infection.
47. The elderly client is diagnosed with chronic
43. The female client in an outpatient clinic is being
glomerulonephritis. Which laboratory value
sent home with a diagnosis of urinary tract
indicates to the nurse the condition has become
infection (UTI). Which instruction should the
worse?
nurse teach to prevent a recurrence of a UTI?
1. The blood urea nitrogen is 15 mg/dL.
1. Clean the perineum from back to front after a
2. The creatinine level is 1.2 mg/dL.
bowel movement.
3. The glomerular filtration rate is 40 mL/min.
2. Take warm tub baths instead of hot showers
4. The 24-hour creatinine clearance is
daily.
100 mL/min.
3. Void immediately preceding sexual intercourse.
4. Avoid coffee, tea, colas, and alcoholic
beverages.

09_Colgrove_Ch09.indd 360 01/06/16 5:45 PM


C hapter 9  G enitourinary D isorders 361

48. The clinic nurse is caring for a client diagnosed 53. Which nursing diagnosis is priority for the client
with chronic pyelonephritis who is prescribed who has undergone a TURP?
trimethoprim-sulfamethoxazole (Bactrim), 1. Potential for sexual dysfunction.
a sulfa antibiotic, twice a day for 90 days. 2. Potential for an altered body image.
Which statement is the scientific rationale for 3. Potential for chronic infection.
prescribing this medication? 4. Potential for hemorrhage.
1. The antibiotic will treat the bladder spasms
54. Which statement indicates discharge teaching
that accompany a urinary tract infection.
has been effective for the client who is
2. If the urine cannot be made bacteria free, the
postoperative TURP?
Bactrim will suppress bacterial growth.
1. “I will call the surgeon if I experience any
3. In three (3) months, the client should be rid of
difficulty urinating.”
all bacteria in the urinary tract.
2. “I will take my Proscar daily, the same as
4. The HCP is providing the client with enough
before my surgery.”
medication to treat future infections.
3. “I will continue restricting my oral fluid
intake.”
4. “I will take my pain medication routinely even
Benign Prostatic Hypertrophy if I do not hurt.”
49. The nurse emptied 2,000 mL from the drainage 55. The client is one (1) day postoperative
bag of a continuous irrigation of a client who TURP. Which task should the nurse delegate to
had a transurethral resection of the prostate the UAP?
(TURP). The amount of irrigation in the bag 1. Increase the irrigation fluid to clear clots from
hanging was 3,000 mL at the beginning of the the tubing.
shift. There was 1,800 mL left in the bag eight 2. Elevate the scrotum on a towel roll for
(8) hours later. What is the correct urine output support.
at the end of the eight (8) hours? _________ 3. Change the dressing on the first postoperative
day.
50. The nurse observes red urine and several
4. Teach the client how to care for the
large clots in the tubing of the normal saline
continuous irrigation catheter.
continuous irrigation catheter for the client
who is one (1) day postoperative TURP. Which 56. The client with a TURP who has a continuous
intervention should the nurse implement? irrigation catheter complains of the need to
1. Remove the indwelling catheter. urinate. Which intervention should the nurse
2. Titrate the NS irrigation to run faster. implement first?
3. Administer protamine sulfate IVP. 1. Call the surgeon to inform the HCP of the
4. Administer vitamin K slowly. client’s complaint.
2. Administer the client a narcotic medication
51. Which data support to the nurse the client’s
for pain.
diagnosis of acute bacterial prostatitis?
3. Explain to the client this sensation happens
1. Terminal dribbling.
frequently.
2. Urinary frequency.
4. Assess the continuous irrigation catheter for
3. Stress incontinence.
patency.
4. Sudden fever and chills.
57. The client who is postoperative TURP asks
52. Which intervention should the nurse include
the nurse, “When will I know if I will be able to
when preparing a teaching plan for the client
have sex after my TURP?” Which response is
with chronic prostatitis?
most appropriate by the nurse?
1. Sit in a warm sitz bath for 10 to 20 minutes
1. “You seem anxious about your surgery.”
several times daily.
2. “Tell me about your fears of impotency.”
2. Sit in the chair with the feet elevated for
3. “Potency can return in six (6) to eight
two (2) hours daily.
(8) weeks.”
3. Drink at least 3,000 mL of oral fluids,
4. “Did you ask your doctor about your
especially tea and coffee, daily.
concern?”
4. Stop broad-spectrum antibiotics as soon as
the symptoms subside.

09_Colgrove_Ch09.indd 361 01/06/16 5:45 PM


362 M ed -S urg S uccess

58. The client asks, “What does an elevated PSA test 62. The client diagnosed with renal calculi
mean?” On which scientific rationale should the is admitted to the medical unit. Which
nurse base the response? intervention should the nurse implement first?
1. An elevated PSA can result from several 1. Monitor the client’s urinary output.
different causes. 2. Assess the client’s pain and rule out
2. An elevated PSA can be only from prostate complications.
cancer. 3. Increase the client’s oral fluid intake.
3. An elevated PSA can be diagnostic for 4. Use a safety gait belt when ambulating
testicular cancer. the client.
4. An elevated PSA is the only test used to
63. The client with possible renal calculi is
diagnose BPH.
scheduled for a renal ultrasound. Which
59. The client returned from surgery after having intervention should the nurse implement for this
a TURP and has a P 110, R 24, BP 90/40, and procedure?
cool and clammy skin. Which interventions 1. Ask if the client is allergic to shellfish or
should the nurse implement? Select all that iodine.
apply. 2. Keep the client NPO eight (8) hours prior to
1. Assess the urine in the continuous irrigation the ultrasound.
drainage bag. 3. Ensure the client has a signed informed
2. Decrease the irrigation fluid in the continuous consent form.
irrigation catheter. 4. Explain the test is noninvasive and there is no
3. Lower the head of the bed while raising the discomfort.
foot of the bed.
64. Which clinical manifestations should the nurse
4. Contact the surgeon to give an update on the
expect to assess for the client diagnosed with a
client’s condition.
ureteral renal stone?
5. Check the client’s postoperative creatinine
1. Dull, aching flank pain and microscopic
and BUN.
hematuria.
60. The nurse is caring for a client with a TURP. 2. Nausea; vomiting; pallor; and cool, clammy
Which expected outcome indicates the client’s skin.
condition is improving? 3. Gross hematuria and dull suprapubic pain
1. The client is using the maximum amount with voiding.
allowed by the PCA pump. 4. The client will be asymptomatic.
2. The client’s bladder spasms are relieved by
65. The client diagnosed with renal calculi is
medication.
scheduled for a 24-hour urine specimen
3. The client’s scrotum is swollen and tender
collection. Which interventions should the nurse
with movement.
implement? Select all that apply.
4. The client has passed a large, hard, brown
1. Check for the ordered diet and medication
stool this morning.
modifications.
2. Instruct the client to urinate, and discard this
Renal Calculi urine when starting collection.
3. Collect all urine during 24 hours and place in
61. The laboratory data reveal a calcium phosphate appropriate specimen container.
renal stone for a client diagnosed with renal 4. Insert an indwelling catheter in client after
calculi. Which discharge teaching intervention having the client empty the bladder.
should the nurse implement? 5. Instruct the UAP to notify the nurse when the
1. Encourage the client to eat a low-purine diet client urinates.
and limit foods such as organ meats. 66. The client is diagnosed with an acute episode of
2. Explain the importance of not drinking water ureteral calculi. Which client problem is priority
two (2) hours before bedtime. when caring for this client?
3. Discuss the importance of limiting vitamin 1. Fluid volume loss.
D–enriched foods. 2. Knowledge deficit.
4. Prepare the client for extracorporeal shock 3. Impaired urinary elimination.
wave lithotripsy (ESWL). 4. Alteration in comfort.

09_Colgrove_Ch09.indd 362 01/06/16 5:45 PM


C hapter 9  G enitourinary D isorders 363

67. The client diagnosed with renal calculi is 72. The client is diagnosed with a uric acid stone.
scheduled for lithotripsy. Which postprocedure Which foods should the client eliminate from
nursing task is the most appropriate to delegate the diet to help prevent reoccurrence?
to the UAP? 1. Beer and colas.
1. Monitor the amount, color, and consistency of 2. Asparagus and cabbage.
urine output. 3. Venison and sardines.
2. Teach the client about care of the indwelling 4. Cheese and eggs.
Foley catheter.
3. Assist the client to the car when being
discharged home. Cancer of the Bladder
4. Take the client’s postprocedural vital signs.
73. The nurse is caring for clients on a renal surgery
68. Which statement indicates the client diagnosed unit. After the afternoon report, which client
with calcium phosphate renal calculi understands should the nurse assess first?
the discharge teaching for ways to prevent 1. The male client who just returned from a CT
future calculi formation? scan who states he left his glasses in the x-ray
1. “I should increase my fluid intake, especially department.
in warm weather.” 2. The client who is one (1) day postoperative
2. “I should eat foods containing cocoa and and has a moderate amount of serous drainage
chocolate.” on the dressing.
3. “I will walk about a mile every week and not 3. The client who is scheduled for surgery in
exercise often.” the morning and wants an explanation of the
4. “I should take one (1) vitamin a day with extra operative procedure before signing the permit.
calcium.” 4. The client who had ileal conduit surgery this
69. Which intervention is most important for the morning and has not had any drainage in the
nurse to implement for the client diagnosed with drainage bag.
rule-out renal calculi? 74. Which modifiable risk factor should the nurse
1. Assess the client’s neurological status every identify for the development of cancer of the
two (2) hours. bladder in a client?
2. Strain all urine and send any sediment to 1. Previous exposure to chemicals.
the laboratory. 2. Pelvic radiation therapy.
3. Monitor the client’s creatinine and BUN 3. Cigarette smoking.
levels. 4. Parasitic infections of the bladder.
4. Take a 24-hour dietary recall during the client
interview. 75. The client diagnosed with cancer of the bladder
is scheduled to have a cutaneous urinary
70. The client with a history of renal calculi calls the diversion procedure. Which preoperative
clinic and reports having burning on urination, teaching intervention specific to the procedure
chills, and an elevated temperature. Which should be included?
instruction should the nurse discuss with the 1. Demonstrate turn, cough, and deep breathing.
client? 2. Explain a bag will drain the urine from now on.
1. Increase water intake for the next 24 hours. 3. Instruct the client on the use of a PCA pump.
2. Take two (2) Tylenol to help decrease the 4. Take the client to the ICD so the client can
temperature. become familiar with it.
3. Come to the clinic and provide a urinalysis
specimen. 76. The client diagnosed with cancer of the bladder
4. Use a sterile 4 × 4 gauze to strain the client’s is undergoing intravesical chemotherapy. Which
urine. instruction should the nurse provide the client
about the pretherapy routine?
71. The client had surgery to remove a kidney stone. 1. Instruct the client to remain NPO after
Which laboratory assessment data warrant midnight before the procedure.
immediate intervention by the nurse? 2. Explain the use of chemotherapy in bladder
1. A serum potassium level of 3.8 mEq/L. cancer.
2. A urinalysis shows microscopic hematuria. 3. Teach the client to administer Neupogen, a
3. A creatinine level of 0.8 mg/100 mL. biologic response modifier.
4. A white blood cell count of 14,000/mm3. 4. Have the client take Tylenol, an analgesic,
before coming to the clinic.

09_Colgrove_Ch09.indd 363 01/06/16 5:45 PM


364 M ed -S urg S uccess

77. The nurse is planning the care of a postoperative 81. The client with a continent urinary diversion
client with an ileal conduit. Which intervention is being discharged. Which discharge
should be included in the plan of care? instructions should the nurse include in the
1. Provide meticulous skin care and pouching. teaching?
2. Apply sterile drainage bags daily. 1. Have the client demonstrate catheterizing the
3. Monitor the pH of the urine weekly. stoma.
4. Assess the stoma site every day. 2. Instruct the client on how to pouch the stoma.
3. Explain the use of a bedside drainage bag
78. The nurse and a licensed practical nurse
at night.
(LPN) are caring for a group of clients. Which
4. Tell the client to call the HCP if the
intervention should be assigned to the LPN?
temperature is 99°F or less.
1. Assessment of the client who has had a Kock
pouch procedure. 82. Which information regarding the care of a
2. Monitoring of the postop client with a WBC cutaneous ileal conduit should the nurse discuss
of 22,000/mm3. with the client?
3. Administration of the prescribed 1. Teach the client to instill a few drops of
antineoplastic medications. vinegar into the pouch.
4. Care for the client going for an MRI of the 2. Tell the client the stoma should be slightly
kidneys. dusky colored.
3. Inform the client large clumps of mucus are
79. The male client diagnosed with metastatic
expected.
cancer of the bladder is emaciated and refuses to
4. Tell the client it is normal for the urine to be
eat. Which nursing action is an example of the
pink or red in color.
ethical principle of paternalism?
1. The nurse allows the client to talk about not 83. The client is two (2) days
wanting to eat. postureterosigmoidostomy for cancer of the
2. The nurse tells the client if he does not eat, a bladder. Which assessment data warrant
feeding tube will be placed. notification of the HCP by the nurse?
3. The nurse consults the dietitian about the 1. The client complains of pain at a “3,”
client’s nutritional needs. 30 minutes after being medicated.
4. The nurse asks the family to bring favorite 2. The client complains it hurts to cough and
foods for the client to eat. deep breathe.
3. The client ambulates to the end of the hall
80. The client diagnosed with cancer of the bladder
and back before lunch.
states, “I have young children. I am too young
4. The client is lying in a fetal position and has a
to die.” Which statement is the nurse’s best
rigid abdomen.
response?
1. “This cancer is treatable and you should not 84. The female client diagnosed with bladder cancer
give up.” who has a cutaneous urinary diversion states,
2. “Cancer occurs at any age. It is just one of “Will I be able to have children now?” Which
those things.” statement is the nurse’s best response?
3. “You are afraid of dying and what will happen 1. “Cancer does not make you sterile, but
to your children.” sometimes the therapy can.”
4. “Have you talked to your children about your 2. “Are you concerned you can’t have children?”
dying?” 3. “You will be able to have as many children as
you want.”
4. “Let me have the chaplain come to talk with
you about this.”

09_Colgrove_Ch09.indd 364 01/06/16 5:45 PM


C hapter 9   G enitourinary D isorders 365

CONCEPTS
The concepts covered in this chapter focus on urinary elimination and fluid and electrolyte balance. Exemplars
that are covered are acute and chronic renal failure, urinary tract infection, and dehydration. Interrelated concepts
of the nursing process and critical thinking are covered throughout the questions. The concept of critical thinking
is presented in the prioritizing or “first” questions.

85. The nurse is monitoring the client’s laboratory values. Which laboratory report is diagnostic for a urinary
tract infection (UTI)?
1. Hemogram
Date: Today
Laboratory Test Client Values Normal Values
White Blood Cells 12.5 5–10 (103)
Red Blood Cells 3.8 M: 4.5–5.3 (106)
F: 4.1–5.1 (106)
Hemoglobin 11 M: 13–19 g/100 mL
F: 12–16 g/100 mL
Hematocrit 33% M: 37%–49%
F: 36%–46%
Platelets 250 150–400 (103)

2. Urinalysis
Client Values Normal Values
pH 5.5 4–8 (average 6)
Color Dark amber Amber yellow
Clarity Cloudy Clear
Specific Gravity 1.029 1.003–1.030
Osmolality 450 300–1300 mOsm/kg
Protein 0 0–trace
Glucose 0 None
Ketones 0 None
RBC 0 0–4 hpf
WBC 4 0–5 hpf

3. Urine Culture
Urine Culture Organism Sensitivity
5
48-hour result Greater than 10 Ceftriaxone
Escherichia coli Cefazolin
bacteria Imipenem-Cilastatin

4. Metanephrines
Client Value Normal Values
24-hour urine metanephrines 1.5 0.00–0.9

86. The nurse is caring for a client diagnosed with chronic renal failure (CRF). Which antecedents would the
nurse assess? Select all that apply.
1. Current diet.
2. Diabetes.
3. Hypertension.
4. Fluid restriction.
5. Race.

09_Colgrove_Ch09.indd 365 01/06/16 8:13 PM


366 M ed -S urg S uccess

87. The client is in the intensive care department (ICD) after a motor-vehicle accident in which the client lost
an estimated three (3) units of blood. Which action by the nurse could prevent the client from developing
acute renal failure?
1. Take and document the client’s vital signs every hour.
2. Assess the client’s dressings every two (2) hours.
3. Check the client’s urinary output every shift.
4. Maintain the client’s blood pressure greater than 100/60.
88. The nurse has identified the concept of urinary elimination for a client. Which information is most
important for the nurse to provide to the health-care provider the next day?

Intake and Output

Day One Oral Intravenous Urine Nasogastric Other (Specify)


(Shift Time) (in mL) (in mL) (in mL) Tube (in mL) (in mL)
0701–1500 2,100 1,000 435 Emesis 40
1501–2300 1,500 1,000 375
2301–0700 200 1,000 500
Total 3,800 3,000 1,310 40

1. The client vomited 40 mL on the day shift.


2. The client has an adequate oral intake, and IV fluids are not needed.
3. The client has had 6,800 mL intake and 1,350 mL output in the last 24 hours.
4. The client does not like to have to keep the urine for measurement.
89. The client diagnosed with chronic renal failure (CRF) is prescribed hemodialysis on Monday, Wednesday,
and Friday. Which interventions should the dialysis nurse implement? Select all that apply.
1. Weigh the client before and after each treatment.
2. Discuss the recommended fluid restriction.
3. Provide potato chips or pretzels as a snack.
4. Monitor the hemodialysis access site continuously.
5. Keep up a lively conversation during the treatments.
90. The nurse is administering morning medications. Which medication should the nurse question
administering?

Admitting Diagnosis: Acute


Client Name: ACC Renal Failure

Account Number: 678905 Med Rec #: 01 01 02 Allergies Penicillin

Date Medication 2301–0700 0701–1500 1501–2300


Yesterday Furosemide (Lasix) 80 mg PO daily 0900
K1 4.3
Yesterday Erythropoietin (Epogen) Sub Q daily 0900
times 3 days
Yesterday Multivitamin with iron PO daily 0900
Yesterday Levothyroxine (Synthroid) 0.75 mcg 0900
PO daily
Signature of Nurse:
Day Nurse RN (DN)

1. Furosemide.
2. Erythropoietin.
3. Multivitamin with iron.
4. Levothyroxine.

09_Colgrove_Ch09.indd 366 01/06/16 5:45 PM


C hapter 9  G enitourinary D isorders 367

91. The nurse identifies the concepts of elimination 94. The client diagnosed with a urinary tract
and immunity for a female client diagnosed infection has a blood pressure of 83/56 mm Hg
with a urinary tract infection. Which discharge and a pulse of 122 bpm. Which should the nurse
instructions should the nurse provide the client? implement first?
Select all that apply. 1. Notify the health-care provider (HCP).
1. Teach the client to wipe from front to back 2. Hang the IVPB antibiotic at the prescribed
after voiding. rate.
2. Encourage the client to drink cranberry juice 3. Check the laboratory work to determine if the
each morning. urine culture has been completed.
3. Inform the client that frequent episodes of 4. Increase the normal saline IV fluids from keep
incontinence are expected. open to 150 mL/hour on the IV pump.
4. Discuss the signs and symptoms of a recurrent
95. The nurse is developing a care map to care for
infection.
a client diagnosed with chronic renal failure
5. Have the client fill a container of water to sip
(CRF) on hemodialysis. Which interrelated
until at least 2,000 mL is consumed.
concepts should be included in the map? Select
6. Request that the client sit in a tub of warm
all that apply.
water twice a day for 25 minutes.
1. Fluid and electrolytes.
92. The client diagnosed with chronic renal failure 2. Hematologic regulation.
(CRF) is prescribed a 60-gm protein, 2,000-mg 3. Digestion.
sodium diet. Which food choices indicate the 4. Metabolism.
client understands the dietary restrictions? 5. Mobility.
1. A 4-ounce grilled chicken breast, broccoli, and 6. Nutrition.
small glass of unsweet tea.
96. The telemetry nurse is reviewing the laboratory
2. Baked potato with chopped ham and sour
results for a client. Which further assessment
cream, 12-ounce steak, and beer.
data should the nurse determine before
3. Double patty cheeseburger, french fries, and
notifying the health-care provider?
saccharin sweet Kool Aid.
4. Roast beef sandwich, potato chips, and soft
Laboratory Client
drink. Test Values Normal Values
93. The elderly client presents to the emergency K 2.3 3.5–5.5 mEq/L
department complaining of burning on urination Na 139 135–145 mEq/L
with an urgency to void, and a temperature of
Glucose 143 70–120 mg/dL
99.8°F. Which intervention should the nurse
implement first? Creatinine 1.5 M: 0.6–1.5 mg/dL
F: 0.6–1.1 mg/dL
1. Ask the client to provide a clean voided
midstream urine for culture. BUN 20 8–25 mg/dL
2. Insert an 18-gauge peripheral IV catheter and BNP 80 Less than 100 pg/mL
start normal saline fluids.
3. Arrange for the client to be admitted to the 1. Obtain the client’s 24-hour urine output.
medical unit. 2. Ask the unlicensed assistive personnel to get a
4. Initiate the ordered intravenous antibiotic blood glucose reading.
medication. 3. Assess the client’s telemetry reading.
4. Call the rapid response team (RRT).

09_Colgrove_Ch09.indd 367 01/06/16 5:45 PM


PRACTICE QUESTIONS ANSWERS
AND RATIONALES

Acute Renal Failure (ARF) of Risk Potential: Cognitive Level – Analysis: Concept –
Urine Elimination.

1. 1. Usually there are no diseases or conditions 3. 1. Diabetes mellitus is a disease that may lead to
warranting this question when discussing ARF. chronic renal failure.
2. Vigorous exercise will not impede blood flow 2. Hypotension, which causes a decreased
to the kidneys, leading to ARF. blood supply to the kidney, is one of the
3. Usually viruses do not cause ARF. most common causes of prerenal failure
4. Medications such as nonsteroidal anti- (before the kidney).
inflammatory drugs (NSAIDs) and some 3. Nephrotoxic medications are a cause of
herbal remedies are nephrotoxic; therefore, intrarenal failure (directly to kidney).
asking about medications is appropriate. 4. Benign prostatic hypertrophy (BPH) is a cause
of postrenal failure (after the kidney).
TEST-TAKING HINT: Asking about medications,
especially over-the-counter and herbal rem- TEST-TAKING HINT: The test taker must be
edies, during the admission interview is an cautious of adjectives (words describing some-
important intervention because many medica- thing); “prerenal” is the key to selecting the
tions are correct ­answer. The prefix pre- means “before.”
nephrotoxic and hepatotoxic. Content – Medical: Integrated Nursing Process –
­Diagnosis: Client Needs – Safe Effective Care Environment,
Content – Medical: Integrated Nursing Process –
Management of Care: Cognitive Level – Comprehension:
­Assessment: Client Needs – Safe Effective Care Environment,
Concept – Urine Elimination.
Management of Care: Cognitive Level – Analysis:
Concept – Urinary Elimination. 4. 1. Renal failure affects almost every system
2. 1. Blood urea nitrogen (BUN) levels reflect in the body. Neurologically, the client
the balance between the production and may have drowsiness, headache, muscle
excretion of urea from the kidneys. Cre- twitching, and seizures. In the recovery
atinine is a by-product of the metabolism period, the client is alert and has no s­ eizure
of the muscles and is excreted by the kid- activity.
neys. Creatinine is the ideal substance for 2. In renal failure, levels of erythropoietin are
determining renal clearance because it is decreased, leading to anemia. An increase in
relatively constant in the body and is the hemoglobin and hematocrit indicates the cli-
laboratory value most significant in diag- ent is in the recovery period.
nosing renal failure. 3. Nausea, vomiting, and diarrhea are com-
2. WBCs (white blood cells) are monitored for mon in the client with ARF; therefore, an
infection, and hemoglobin is monitored for absence of these indicates the client is in
blood loss. the recovery period.
3. Potassium (intracellular) and sodium (intersti- 4. The client in the recovery period has an
tial) are electrolytes and are monitored for a increased urine-specific gravity.
variety of diseases or conditions not specific to 5. The client in the recovery period has a
renal function. Potassium levels will increase decreased serum creatinine level.
with renal failure, but the level is not a diag- TEST-TAKING HINT: This is an alternate-type
nostic indicator for renal failure. question in which the test taker may choose as
4. Bilirubin and ammonia levels are laboratory many correct answers as warranted. The test
values determining the function of the liver, taker should not immediately assume that the
not the kidneys. option mentioning urine is the correct answer.
TEST-TAKING HINT: The nurse must know spe- The nurse must realize renal failure affects
cific laboratory tests for specific organ func- every body system.
tioning or conditions. This is memorizing, but Content – Medical: Integrated Nursing Process –
it must be done. ­Assessment: Client Needs – Safe Effective Care Environment,
Management of Care: Cognitive Level – Analysis:
Content – Medical: Integrated Nursing Process –
Concept – Urinary Elimination.
­Assessment: Client Needs – Physiological Integrity, Reduction

368

09_Colgrove_Ch09.indd 368 01/06/16 5:45 PM


C hapter 9  G enitourinary D isorders 369

5. 1. Phosphate binders are used to treat elevated TEST-TAKING HINT: The test taker must notice
phosphorus levels, not elevated potassium adjectives. A “therapeutic” diet should cause
levels. the test taker to eliminate option “4” because
2. Anemia is not the result of an elevated it is a regular diet.
potassium level. Content – Medical: Integrated Nursing Process –
3. Assessment is an independent nursing action, Implementation: Client Needs – Physiological Integrity, Basic
which is appropriate for the elevated potassium Care and Comfort: Cognitive Level – Application: Concept –
level, but the question asks for Urinary Elimination.
a collaborative treatment.
8. 1. Kidney function is improved about 40% when
4. Normal potassium level is 3.5 to 5.5 mEq/L.
recumbent, but this is not the scientific ratio-
A level of 6.8 mEq/L is life threatening and
nale for bedrest in ARF.
could lead to cardiac dysrhythmias. There-
2. Bedrest reduces exertion and the metabolic
fore, the client may be dialyzed to decrease
rate, thereby reducing catabolism and sub-
the potassium level quickly. This requires a
sequent release of potassium and accumu-
health-care provider order, so it is a collab-
lation of endogenous waste products (urea
orative intervention.
and creatinine).
TEST-TAKING HINT: Adjectives must be noted 3. This is a scientific rationale for prescribing
when reading the stem of the question and bedrest in clients with heart failure.
the answer options. 4. This is not the scientific rationale for prescrib-
Content – Medical: Integrated Nursing Process – ing bedrest. The foot of the bed may be el-
­Planning: Client Needs – Safe Effective Care Environment, evated to help decrease peripheral edema, and
Management of Care: Cognitive Level – Synthesis: bedrest causes an increase in sacral edema.
Concept – Urinary Elimination.
TEST-TAKING HINT: The test taker should not
6. 1. This is a nursing intervention, not a client jump to conclusions and select the only op-
outcome. tion with “renal” in the sentence. The nurse
2. This is a measurable client outcome, but acute must know normal anatomy and physiology
renal failure does not cause pain. of the body and be aware keeping someone in
3. Renal failure causes an imbalance of bed will not restore kidney function when the
­electrolytes (potassium, sodium, calcium, kidneys have failed.
phosphorus). Therefore, the desired ­client Content – Medical: Integrated Nursing Process – ­
outcome is electrolytes within normal Diagnosis: Client Needs – Physiological Integrity,
limits. ­Physiological Adaptation: Cognitive Level – Analysis:
4. A Kayexalate resin enema may be administered Concept – Urinary Elimination.
to help decrease the potassium level, but this is
9. 1. The UAP can collect specimens. Collecting
an intervention, not a client outcome.
a midstream urine specimen requires the
TEST-TAKING HINT: The nurse must be knowl- client to clean the perineal area, to urinate
edgeable of the nursing process. Client out- a little, and then collect the rest of the
comes are used to evaluate the planning part urine output in a sterile container.
of the nursing process. The outcomes must be 2. The UAP can obtain the client’s intake and
measurable, client focused, and realistic. output, but the nurse must evaluate the data
Content – Medical: Integrated Nursing Process – ­ to determine if interventions are needed or if
Diagnosis: Client Needs – Physiological Integrity, interventions are effective.
­Physiological Adaptation: Cognitive Level – Analysis: 3. Two registered nurses must check the unit of
Concept – Urinary Elimination. blood at the bedside prior to administering it.
7. 1. The diet is low potassium, and calcium is not 4. This is a medication enema and UAPs cannot
restricted in ARF. administer medications. Also, for this to be
2. This is a diet recommended for clients with ordered, the client must be unstable with an
cardiac disease and atherosclerosis. excessively high serum potassium level.
3. Carbohydrates are increased to provide for TEST-TAKING HINT: Nursing tasks not del-
the client’s caloric intake and protein is egated to a UAP include any task requiring
restricted to minimize protein breakdown nursing judgment, medication administration,
and to prevent accumulation of toxic waste teaching, evaluating, or assessing.
products. Content – Nursing Management: Integrated Nursing
4. The client must be on a therapeutic diet, and Process – Planning: Client Needs – Psychosocial Integrity:
small feedings are not required. Cognitive Level – Synthesis: Concept – Management.

09_Colgrove_Ch09.indd 369 01/06/16 5:45 PM


370 M ed -S urg S uccess

10. 1. Preventing and treating shock with blood 2. Calcium gluconate helps protect the heart
and fluid replacement will prevent acute from the effects of high potassium levels.
renal failure from hypoperfusion of the 3. Regular insulin, along with glucose, will
kidneys. Significant blood loss is expected drive potassium into the cells, thereby
in the client with a gunshot wound. lowering serum potassium
2. Taking and evaluating the client’s vital signs levels temporarily.
is an appropriate action, but regardless of the 4. A loop diuretic, not an osmotic diuretic, may
results, this will not prevent ARF. be ordered to help decrease the potassium
3. Placing the client on telemetry is an appro- level.
priate action, but telemetry is an assessment TEST-TAKING HINT: The test taker must be
tool for the nurse and will not prevent ARF. familiar with medical terms such as “hyper-
4. Assessment is often the first action, but as- kalemia” and know the rationale for adminis-
sessing the abdominal dressing will not help tering medications.
prevent ARF. Content – Medical: Integrated Nursing Process –
TEST-TAKING HINT: The test taker must read Implementation: Client Needs – Pharmacological and
the stem carefully and understand what the ­Parenteral Therapies: Cognitive Level – Application:
question is asking. Options “2,” “3,” and “4” Concept – Urinary Elimination.
are all forms of assessment and do not help
prevent ARF because they are not treatment. Chronic Kidney Disease (CKD)
Content – Medical: Integrated Nursing Process –
Implementation: Client Needs – Safe Effective Care 13. 1. There is a decrease in the excretion of phos-
­Environment, Management of Care: Cognitive Level –
phates and organic acids, not an increase.
Synthesis: Concept – Urinary Elimination.
2. The red blood cell destruction does not ­affect
11. 1. Moisture barrier cream will keep the crystals the arterial blood pH.
on the skin. 3. This is the correct scientific rationale for
2. These crystals are uremic frost result- metabolic acidosis occurring in the client
ing from irritating toxins deposited in the with CKD.
­client’s tissues. Bathing in cool water will 4. This compensatory mechanism occurs to
remove the crystals, promote client com- maintain an arterial blood pH between
fort, and decrease the itching resulting 7.35 and 7.45, but it does not occur as a
from uremic frost. result of CKD.
3. The client should be turned every two TEST-TAKING HINT: In option “1,” the test
(2) hours or more frequently to prevent skin taker should note “increased excretion”;
breakdown. CKD does not have any type of increase in
4. This may occur with ARF, and it does require excretion, so the test taker could eliminate
a nursing intervention. option “1.” Option “4” does not even men-
TEST-TAKING HINT: The nurse must know tion the renal system and a loss of hydro-
what is normal for specific disease processes, chloric acid results in a metabolic alkalosis,
and something coming out of the skin re- not acidosis, so the test taker can eliminate
quires some action even if the test taker is not this option.
familiar with the disease process. Option “4” Content – Medical: Integrated Nursing Process – ­
could be eliminated based on this test-taking Diagnosis: Client Needs – Safe Effective Care ­Environment,
strategy. The test taker should eliminate op- Management of Care: Cognitive Level – Analysis:
tion “3” because there are very few instances Concept – Urinary Elimination.
in which the client is not turned or moved; 14. 1. These laboratory findings are low but do
turning and movement are necessary to pre- not require a blood transfusion and often are
vent the development of pressure ulcers. ­expected in a client who is anemic secondary
Content – Medical: Integrated Nursing Process – to ESRD.
Implementation: Client Needs – Physiological Integrity, 2. This client’s dialysis access is compromised
­Basic Care and Comfort: Cognitive Level – Analysis:
and he or she should be assessed first.
Concept – Urinary Elimination.
3. It is not uncommon for a client undergoing
12. 1. Erythropoietin is a chemical catalyst dialysis to be exhausted and sleep through the
produced by the kidneys to stimulate red treatment.
blood cell production; it does not affect 4. Clients are instructed not to take their
­potassium level. antihypertensive medications before dialysis
to help prevent episodes of hypotension.

09_Colgrove_Ch09.indd 370 01/06/16 5:45 PM


C hapter 9  G enitourinary D isorders 371

TEST-TAKING HINT: The test taker must de- TEST-TAKING HINT: The test taker must read
termine which client’s situation is not normal the stem of the question and understand
or expected for the disease process, which in what the question is asking. This is a prior-
this question is CKD because all clients are ity question. This means all the options are
in the dialysis unit. pertinent problems for CKD, but only one
Content – Medical: Integrated Nursing Process – ­ is priority. Applying Maslow’s hierarchy of
Assessment: Client Needs – Safe Effective Care needs is one way to determine priorities:
­Environment, Management of Care: Cognitive Level – physiological problems are priority over
Analysis: Concept – Urinary Elimination. psychosocial problems, and life-threatening
15. 1. Flu-like symptoms are expected and tend to conditions take first priority.
subside with repeated doses; the nurse should Content – Medical: Integrated Nursing Process – ­
suggest Tylenol prior to the injections. Diagnosis: Client Needs – Safe Effective Care E
­ nvironment,
2. This medication takes up to two (2) to six (6) Management of Care: Cognitive Level – Analysis:
Concept – Urinary Elimination.
weeks to become effective in improving ane-
mia and thereby reducing fatigue. 17. 720 mL.
3. After the initial administration of erythro- The nurse must add up how many milliliters
poietin, a client’s antihypertensive medica- of fluid the client drank on the 7 a.m. to
tions may need to be adjusted. Therefore, 7 p.m. shift and then subtract that number
this complaint requires notification of from 1,500 mL to determine how much fluid
the HCP. Erythropoietin therapy is con- the client can receive on the 7 p.m. to 7 a.m.
traindicated in clients with uncontrolled shift. One (1) ounce is equal to 30 mL. The
hypertension. client drank 26 ounces (8 + 4 + 12 + 2) of
4. Long bone and vertebral pain is an expected fluid, or 780 mL (26 × 30) of fluid. There-
occurrence because the bone marrow is fore, the client can have 720 mL (1,500 − 780)
being stimulated to increase production of of fluid on the 7 p.m. to 7 a.m. shift.
red blood cells. TEST-TAKING HINT: The test taker must be
TEST-TAKING HINT: The test taker should se- knowledgeable of basic conversion factors.
lect the potentially life-threatening option or Use the drop-down calculator on the com-
a complaint requiring the medication to be puter examination to ensure accuracy in
adjusted or discontinued. The nurse should computations.
notify the HCP if the medication is causing Content – Medical: Integrated Nursing Process –
an adverse effect, not an expected side effect. Implementation: Client Needs – Safe Effective Care
Content – Medical: Integrated Nursing Process – ­ ­Environment, Management of Care: Cognitive Level –
Evaluation: Client Needs – Safe Effective Care Application: Concept – Urinary Elimination.
­Environment, Management of Care: Cognitive Level –
18. 1. Carrying heavy objects in the left arm
Synthesis: Concept – Urinary Elimination.
could cause the fistula to clot by putting
16. 1. Low self-esteem, related to dependency, undue stress on the site, so the client
role changes, and changes in body image, is should carry objects with the right arm.
a pertinent client problem, but psychosocial 2. The fistula should only be used for dialysis
problems are not priority over physiological access, not for routine blood draws.
problems. 3. The client should not lie on the left arm be-
2. Teaching is always an important part of the cause this may cause clotting by putting pres-
care plan, but it is not priority over a physi- sure on the site.
ological problem. 4. Hand exercises are recommended for new
3. Activity intolerance related to fatigue, fistulas to help mature the fistula.
­anemia, and retention of waste products TEST-TAKING HINT: The test taker must no-
is a physiological problem, but it is not a tice the adjectives, such as “left” and “right.”
­life-threatening problem. Options “2” and “3” have the nurse doing
4. Excess fluid volume is priority because of something to the arm with the fistula.
the stress placed on the heart and ­vessels,
Content – Medical: Integrated Nursing Process –
which could lead to heart failure, pulmo- Implementation: Client Needs – Safe Effective Care
nary edema, and death. ­Environment, Management of Care: Cognitive Level –
Analysis: Concept – Urinary Elimination.

09_Colgrove_Ch09.indd 371 01/06/16 5:45 PM


372 M ed -S urg S uccess

19. 1. The client does have the right to quit dialysis TEST-TAKING HINT: The Trendelenburg posi-
if he or she wants to. tion is often used as a distracter in questions,
2. Reflecting the client’s feelings and restat- and the nurse needs to know it is only used
ing them are therapeutic responses the in cases where blood needs to be shunted to
nurse should use when addressing the the brain.
­client’s issues. Content – Medical: Integrated Nursing Process –
3. This is passing the buck; the nurse should Implementation: Client Needs – Safe Effective Care
­address the client’s issues. ­Environment, Management of Care: Cognitive Level –
4. This may be true, but it is not therapeutic Synthesis: Concept – Urinary Elimination.
in attempting to get the client to verbalize 22. 1. Teaching is an intervention for knowledge
feelings. deficit, not noncompliance.
TEST-TAKING HINT: When asked to select a 2. Referring the client does not address the
­therapeutic response, the test taker should issue of noncompliance.
­select an option with some type of “feeling” 3. Noncompliance is a client’s choice, and
in the ­response, such as “angry” in option “2.” explaining interventions will not necessarily
Content – Medical: Integrated Nursing Process – make the client choose differently.
Implementation: Client Needs – Psychosocial Integrity: 4. Noncompliance is a choice the client has
Cognitive Level – Application: Concept – Urinary a right to make, but the nurse should de-
Elimination. termine the reason for the noncompliance
20. 1. Caucasians are composed of a multitude and then take appropriate actions based
of cultures but for the most part organ on the client’s rationale. For example, if
­donation is very likely, although individual the client has financial difficulties, the
preferences vary. nurse may suggest how the client can af-
2. Many in the African American culture ford the proper foods along with medica-
believe the body must be kept intact after tions, or the nurse may be able to refer
death, and organ donation is rare among the client to a social worker.
African Americans. This is also why a TEST-TAKING HINT: The test taker must al-
­client of African American ­descent will be ways clarify and understand exactly what the
on a transplant waiting list ­longer than question is asking the nurse to do. Answer
people of other races. This is b­ ecause of options “1,” “2,” and “3” have the nurse do-
tissue-typing compatibility. Remember, ing the talking; only option “4” is allowing
this does not apply to all African Ameri- the client to explain the lack of compliance.
cans; every client is an individual. Content – Medical: Integrated Nursing Process –
3. Asians as a culture participate in organ Implementation: Client Needs – Physiological Integrity,
donation. Basic Care and Comfort: Cognitive Level – Application:
4. Hispanics as a culture participate in organ Concept – Urinary Elimination.
donation. 23. 1. Peritoneal dialysis is administered through
TEST-TAKING HINT: The nurse must be aware a catheter inserted into the peritoneal cavity;
of cultural differences in health care. a fistula is used for hemodialysis.
Content – Medical: Integrated Nursing Process – ­ 2. Peritonitis, inflammation of the peritoneum,
Assessment: Client Needs – Safe Effective Care is a serious complication resulting in a hard,
­Environment, Management of Care: Cognitive Level – rigid abdomen. Therefore, a soft abdomen
Knowledge: Concept – Urinary Elimination. does not warrant immediate intervention.
21. 1. The nurse should place the client’s chair 3. The dialysate return is normally colorless
with the head lower than the body, which or straw-colored, but it should never be
will shunt blood to the brain; this is the cloudy, which indicates an infection.
Trendelenburg position. 4. Because the client is in ESRD, fluid must
2. The blood in the dialysis machine must be in- be removed from the body, so the output
fused back into the client before the machine should be more than the amount instilled.
is turned off. These assessment data require interven-
3. Normal saline infusion is a last resort because tion by the nurse.
one of the purposes of dialysis is to remove TEST-TAKING HINT: The words “warrant im-
excess fluid from the body. mediate intervention” should clue the test
4. Hypotension is an expected occurrence in taker into selecting an option with abnormal
clients receiving dialysis; therefore, the HCP or unexpected data for the client.
does not need to be notified.

09_Colgrove_Ch09.indd 372 01/06/16 5:45 PM


C hapter 9  G enitourinary D isorders 373

Content – Medical: Integrated Nursing Process – ­ Content – Medical: Integrated Nursing Process – ­
Assessment: Client Needs – Safe Effective Care Assessment: Client Needs – Physiological Integrity,
­Environment, Management of Care: Cognitive Level – ­Reduction of Risk Potential: Cognitive Level – Analysis:
Analysis: Concept – Urinary Elimination. Concept – Fluid and Electrolyte Balance.

24. 1. The client should not wait until the tem- 26. 1. The pump is alerting the nurse there is resis-
perature is 102°F to call the HCP; the client tance distal to the pump; this does not require
should call when the temperature is 100°F or notifying the HCP.
greater. 2. The client has an 1,800-mL intake and total
2. The client should apply direct pressure and output of 1,500 mL. The body has an in-
notify the HCP if the access site starts to sensible loss of approximately 400 mL/day
bleed, not apply ice to the site. through the skin, respirations, and other body
3. Uremic frost, which results when the skin functions. This does not warrant notifying
attempts to take over the function of the the HCP.
kidneys, causes itching, which can lead to 3. Crackles and rhonchi in all lung fields in-
scratching, possibly resulting in dicate the body is not able to process the
a break in the skin. amount of fluid being infused. This should
4. The nurse should encourage the client’s be brought to the HCP’s attention.
independence, not foster dependence by en- 4. Negative pedal edema and an increasing level
couraging the significant other to make the of consciousness indicate the client is not ex-
client’s decision. periencing a problem.
TEST-TAKING HINT: The test taker must read TEST-TAKING HINT: The question requires the
the question carefully. A temperature of test taker to distinguish nursing problems
102°F is usually not acceptable in any client. from client problems. Option “1” is a nurs-
Fostering dependence in any chronic illness ing problem and options “2” and “4” are
is not encouraged by the nurse and so the expected results, so the HCP does not need
test taker could eliminate option “4.” to be notified. Only one option “3” contains
Content – Medical: Integrated Nursing Process – abnormal or life-threatening information.
Planning: Client Needs – Physiological Integrity, Content – Medical: Integrated Nursing Process –
­Physiological Adaptation: Cognitive Level – Synthesis: Implementation: Client Needs – Safe Effective Care
Concept – Urinary Elimination. ­Environment, Management of Care: Cognitive Level –
Analysis: Concept – Fluid and Electrolyte Balance.

Fluid and Electrolyte Disorders 27. 2,000 mL has been lost.


First, determine how many pounds the ­client
25. 1. Clients who are overhydrated or have fluid has lost:
volume excess experience dilutional values of 180 − 175.6 = 4.4 pounds lost
sodium (135 to 145 mEq/L) and red blood Then, based on the fact that 1 liter of fluid
cells (44% to 52%). The levels are lower than weighs 2.2 pounds, determine how many
normal, not higher. ­liters of fluid have been lost:
2. Anemia is a low red blood cell count for a 4.4 ÷ 2.2 = 2 liters lost
­variety of reasons. Then, because the question asks for the
3. Dehydration results in concentrated answer in milliliters, convert 2 liters into
­serum, causing laboratory values to milliliters:
­increase because the blood has normal 2 × 1,000 = 2,000 mL
constituents but not enough volume to TEST-TAKING HINT: The test taker must be
­dilute the values to within normal range able to work basic math problems. This
or possibly lower. problem has several steps. Sometimes it is
4. In renal failure, the kidneys cannot excrete helpful to write out what is occurring at each
urine, and this results in too much fluid in the step, such as 4.4 divided by 2.2 kg per pound.
body. This can help the test taker realize if a step
TEST-TAKING HINT: The test taker must de- has been overlooked. Remember, on the
cide first if the values are high or low and NCLEX-RN use the drop-down calculator
then determine what is happening with body on the computer.
fluids in each process. Overhydration and Content – Medical: Integrated Nursing Process – ­
renal failure result in the same fluid shift, so Assessment: Client Needs – Safe Effective Care
these two (2) options (“1” and “4”) could be ­Environment, Management of Care: Cognitive Level –
excluded. Analysis: Concept – Fluid and Electrolyte Balance.

09_Colgrove_Ch09.indd 373 01/06/16 5:45 PM


374 M ed -S urg S uccess

28. 1. The nursing plan of care does not include 2. Fluctuations in rate are expected in clients
changing the HCP’s orders. diagnosed with atrial fibrillation, and a heart
2. Fluid volume excess refers to an isotonic rate of 100 is at the edge of a normal rate.
expansion of the extracellular fluid by an 3. Most people experience an occasional
abnormal expansion of water and sodium. ­premature ventricular contraction (PVC);
Therefore, sodium is restricted to allow this does not warrant the nurse assessing this
the body to excrete the extra volume. ­client first.
3. High blood glucose levels result in viscous 4. A first-degree block is not an immediate
blood and cause the kidneys to try to fix the problem.
problem by excreting the glucose through TEST-TAKING HINT: The test taker must know
increasing the urine output, which results in the normal data so the abnormal will be
fluid volume deficits. ­apparent. Normal heart rate is 60 to 100. The
4. If the FVE is the result of renal failure, then nurse should assess the client who has an
hemodialysis may be ordered, but this infor- ­abnormal or life-threatening condition first.
mation was not provided in the stem of the
Content – Medical: Integrated Nursing Process –
question. ­Evaluation: Client Needs – Affective Care Environment,
TEST-TAKING HINT: Option “1” is not a Management of Care: Cognitive Level – Analysis:
­nursing prerogative. The test taker should Concept – Fluid and Electrolyte Balance.
not read into the question.
31. 1. The HCP may need to be notified, but the
Content – Medical: Integrated Nursing Process – ­ nurse should perform assessment first.
Diagnosis: Client Needs – Safe Effective Care ­Environment, 2. These are signs and symptoms of
Management of Care: Cognitive Level – Analysis:
hypocalcemia, and the nurse can ­confirm
Concept – Fluid and Electrolyte Balance.
this by tapping the cheek to elicit the
29. 1. The client probably will be placed on fluid Chvostek’s sign. If the ­muscles of the
restriction. Fluids should not be encouraged cheek begin to twitch, then the HCP
for a client with a low sodium level (normal: should be notified ­immediately because
135 to 145 mEq/L). hypocalcemia is a medical emergency.
2. Hypertonic solutions of saline are 3% to 5%, 3. A positive Chvostek’s sign can indicate a
not 10%, because of the extreme nature of low calcium or magnesium level, but serum
hypertonic solutions. Hypertonic solutions laboratory levels may have been drawn hours
of saline may be used but very cautiously previously or may not be available.
­because, if the sodium levels are increased too 4. If the client does have hypocalcemia, this may
rapidly, a massive fluid shift can occur in the be ordered, but it is not implemented prior to
body, resulting in neurological damage and assessment.
heart failure. TEST-TAKING HINT: Assessment is the
3. The antidiuretic hormone (vasopressin) first step in the nursing process and is an
causes water retention in the body and ­appropriate option to select if the test taker
increases the problem. has difficulty when trying to decide between
4. Clients with sodium levels less than two options.
120 mEq/L are at risk for seizures as a
Content – Surgical: Integrated Nursing Process –
complication. The lower the sodium level, Implementation: Client Needs – Safe Effective Care
the greater the risk of a seizure. Environment, Management of Care: Cognitive Level –
TEST-TAKING HINT: The test taker must mem- Synthesis: Concept – Perioperative.
orize certain common laboratory values and
32. 1. Kussmaul’s respirations are the lung’s
understand how deviations in the electrolytes
attempt to maintain the narrow range of
affect the body.
pH compatible with human life. The respira-
Content – Medical: Integrated Nursing Process – tory system reacts rapidly to changes in pH.
Implementation: Client Needs – Safe Effective Care 2. Respiration is the act of moving oxygen and
­Environment, Management of Care: Cognitive Level –
carbon dioxide. Kussmaul’s respirations are
Application: Concept – Fluid and Electrolyte Balance.
rapid and deep and allow the client to exhale
30. 1. A client with a peaked T wave could be carbon dioxide.
experiencing hyperkalemia. Changes 3. The lungs attempt to increase the blood
in potassium levels can initiate cardiac pH level by blowing off the carbon dioxide
­dysrhythmias and instability. (carbonic acid).

09_Colgrove_Ch09.indd 374 01/06/16 5:45 PM


C hapter 9  G enitourinary D isorders 375

4. HCO3 (sodium bicarbonate) is an alkaline TEST-TAKING HINT: The question is asking for
(base) substance regulated by the kidneys and a first action, which means all of the options
is part of the metabolic buffer system, not a may be actions the nurse could implement,
respiratory system buffer. The excretion and but only one is priority. In general, priority
retention of carbon dioxide (CO2) are regu- actions are stop the problem, continue treat-
lated by the lungs and therefore a part of the ment, treat the problem, and then document.
respiratory buffer system. Content – Medical: Integrated Nursing Process –
TEST-TAKING HINT: Homeostasis is a delicate Implementation: Client Needs – Safe Effective Care
balance between acids and bases. The test Environment, Management of Care: Cognitive Level –
Synthesis: Concept – Fluid and Electrolyte Balance.
taker can discard option “1” by realizing
production of urine does not affect the 35. 1. The UAP can empty the catheter and mea-
respirations. sure the amount.
Content – Medical: Integrated Nursing Process – ­ 2. The UAP can record intake and output on
Diagnosis: Client Needs – Safe Effective Care Environ- the I&O sheet.
ment, Management of Care: Cognitive Level – Analysis: 3. The nurse cannot delegate teaching.
Concept – Fluid and ­Electrolyte Balance. 4. The client has a disease, but all the UAP is
33. 1. TPN is a hypertonic solution with enough being asked to do is take water to the client.
calories, proteins, lipids, electrolytes, and TEST-TAKING HINT: This is an example of an
trace elements to sustain life. It is admin- “except” question. Frequently, questions ask
istered via a pump to prevent too rapid which tasks can be assigned to the UAP, but
infusion. this question asks which action the nurse
2. TPN contains 50% dextrose solution; should implement. If the test taker does not
therefore, the client is monitored to read carefully, it is easy to jump to the first
ensure the pancreas is adapting to the option for actions the UAP can perform.
high glucose levels. Content – Medical: Integrated Nursing Process –
3. The client is weighed daily, not weekly, to Implementation: Client Needs – Safe Effective Care
monitor for fluid overload. ­Environment, Management of Care: Cognitive Level –
4. The IV tubing is changed with every bag Application: Concept – Nursing Roles.
­because the high glucose level can cause 36. 1. Serum calcium is decreased in conditions
­bacterial growth. such as osteoporosis or post–thyroid surgery
5. Intake and output are monitored to but not in vomiting and diarrhea.
­observe for fluid balance. 2. Serum phosphorus levels are altered in
TEST-TAKING HINT: Options “3” and “5” acute and chronic renal failure or diabetic
r­ efer to the same factor—namely, fluid level. ­ketoacidosis, among other conditions, but not
The test taker should then determine if the with acute fluid losses from the gastrointesti-
time factors are appropriate. Weekly weigh- nal tract.
ing is not appropriate, so option “3” can be 3. Clients lose potassium from the GI tract
eliminated. or through the use of diuretic medica-
Content – Medical: Integrated Nursing Process – tions. Potassium imbalances can lead to
Implementation: Client Needs – Safe Effective Care cardiac arrhythmias.
­Environment, Management of Care: Cognitive Level – 4. The body is not at risk from losing sodium
Analysis: Concept – Fluid and Electrolyte Balance. from these sources as it is with potassium.
34. 1. A new IV will be started in the right hand TEST-TAKING HINT: The nurse must recognize
­after the IV is discontinued. basic fluids and electrolytes in the body and
2. The client has signs of phlebitis and the the implications of excess or loss. The body
IV must be removed to prevent further holds onto sodium and releases potassium.
complications. Content – Medical: Integrated Nursing Process – ­
3. Depending on the health-care facility, this Assessment: Client Needs – Physiological Integrity,
may or may not be done, but client care ­Reduction of Risk Potential: Cognitive Level – Analysis:
comes before documentation. Concept – Fluid and Electrolyte Balance.
4. A warm washrag placed on an IV site some-
times provides comfort to the client. If this is
done, it should be done for 20 minutes four
(4) times a day.

09_Colgrove_Ch09.indd 375 01/06/16 5:45 PM


376 M ed -S urg S uccess

Urinary Tract Infection (UTI) 39. 620 mL of urine.


The amount of sterile normal saline is sub-
37. 1. The first action is to get a viable urine tracted from the total volume removed from
culture so the causative pathogen can be the catheter.
identified. An IV should be started, but this is TEST-TAKING HINT: This is a simple subtrac-
not the first action. tion problem, but the test taker must under-
2. Initiating an IV antibiotic is priority, but stand any fluid used to irrigate a body system
obtaining a culture is done first to make sure must be subtracted from the total volume
the HCP can treat the causative organism. in the suction device or catheter bag to get
3. This is not the first intervention because accurate information of the client’s fluid-
the culture will be obtained when the new balance status.
catheter has been inserted. Content – Medical: Integrated Nursing Process – ­
4. Unless the nurse can determine the Assessment: Client Needs – Safe Effective Care
catheter has been inserted within a few ­Environment, Management of Care: Cognitive Level –
days, the nurse should replace the Analysis: Concept – Urinary Elimination/Infection.
catheter and then get a specimen. This
40. 1. This could be asked with a parent in the
will provide the most accurate specimen
room, and the nurse should receive a ­truthful
for analysis.
answer.
TEST-TAKING HINT: In a question requiring 2. There is no reason the client should not
the test taker to choose a “first” action, the answer this question in the presence of the
test taker usually can order the choices 1, 2, parent.
3, 4. In this question, options “4,” “3,” “1,” 3. These are symptoms of cystitis, a bladder
and “2” should be the order of interventions. infection, which may be caused by sexual in-
Content – Medical: Integrated Nursing Process – tercourse as a result of the introduction of
Implementation: Client Needs – Safe Effective Care bacteria into the urethra during the physi-
­Environment, Management of Care: Cognitive Level – cal act. A teenager may not want to divulge
Synthesis: Concept – Urinary Elimination/Infection. this information in front of the parent.
38. In order of performance: 1, 4, 2, 3, 5. 4. This information could be obtained in front
1. The procedure should be explained to the of the parent.
client. TEST-TAKING HINT: The test taker must ana-
4. Incontinence pads should be placed under lyze the client’s age, 15, and determine which
the client before beginning the sterile part of the options might not be answered truth-
of the procedure. fully if the parents are present. In this ques-
2. The sterile field must be set up prior to tion, “asking the parent to leave the room” is
checking the bulb and cleaning the client’s the key to choosing the correct option.
perineum. Content – Medical: Integrated Nursing Process – ­
3. The bulb of the catheter should be tested Assessment: Client Needs – Physiological Integrity,
to make sure it will inflate and deflate ­Reduction of Risk Potential: Cognitive Level – Analysis:
prior to inserting the catheter into the Concept – Urinary Elimination/Infection.
client.
41. 1. Fever, chills, and costovertebral pain are
5. During the procedure, the perineum is
symptoms of a urinary tract infection
swiped with Betadine swabs from front to
(acute pyelonephritis), which requires a
back and also down the middle, then side
urine culture first to confirm the diagnosis.
to side with new swabs (clean to dirty).
2. A sonogram of the kidney might be ordered
TEST-TAKING HINT: This is an alternative-type if the client has recurrent UTIs to determine
question requiring the test taker to rank the if a physical obstruction is causing the recur-
options in the correct order. The test taker rent infections but not as the first diagnostic
must be knowledgeable of skills performed procedure.
by the nurse. 3. An intravenous pyelogram (IVP) is rarely
Content – Medical: Integrated Nursing Process – used to determine pyelonephritis because the
Implementation: Client Needs – Safe Effective Care Envi- results are negative 75% of the time in clients
ronment, Management of Care: Cognitive Level – Analysis: diagnosed with acute pyelonephritis.
Concept – Urinary Elimination/Infection. 4. A CT scan might be ordered if other tests
have not been conclusive.

09_Colgrove_Ch09.indd 376 01/06/16 5:45 PM


C hapter 9  G enitourinary D isorders 377

TEST-TAKING HINT: The question asks which 44. 1. A pregnant client diagnosed with a UTI
test should be ordered first, and the test will be admitted for aggressive IV anti-
taker should determine what the symptoms biotic therapy. After symptoms subside,
might be indicating. Fever and chills indicate the client will be sent home to com-
an infection. The anatomical position of the plete the course of treatment with oral
costovertebral angle (flank area between a rib medications.
and a vertebra) should alert the test taker to 2. The nurse cannot “force” a client to drink,
the kidney area of the body. A urine culture and forcing fluids could result in nausea and
is most likely to determine if a kidney infec- vomiting, not prevent it.
tion is present. 3. The client may or may not be dehydrated.
Content – Medical: Integrated Nursing Process – 4. Pregnant clients have a right to be concerned
­Planning: Client Needs – Physiological Integrity, Reduction about taking medications, but most are com-
of Risk Potential: Cognitive Level – Synthesis: Concept – fortable taking medications prescribed by the
Urinary Elimination/Infection. obstetrician.
42. 1. Fever, chills, flank pain, and dysuria are symp- TEST-TAKING HINT: In option “2” the nurse
toms of acute pyelonephritis, not chronic is “forcing” a client to do something, which
pyelonephritis. should be eliminated as a possible correct
2. Fatigue, headache, and polyuria as well answer. Option “4” is a broad generalization
as loss of weight, anorexia, and exces- about “all” pregnant clients and should be
sive thirst are symptoms of chronic discarded as a possible correct answer.
pyelonephritis. Content – Medical: Integrated Nursing Process –
3. Group B beta-hemolytic streptococcus Diagnosis: Client Needs – Safe Effective Care E
­ nvironment,
infections cause acute glomerulonephritis. Management of Care: Cognitive Level – Analysis:
4. Acute viral pneumonia is a cause of acute Concept – Urinary Elimination/Infection.
glomerlonephritis. 45. 1. The function of the urinary tract is to ­process
TEST-TAKING HINT: The key to this question fluids and wastes from the body. Limiting its
is the adjective “chronic.” The test taker functioning will increase the problem, not
must be aware disease processes may change help the problem.
over time do produce different effects. 2. A routine urine specimen is not a clean
Content – Medical: Integrated Nursing Process – ­ voided specimen and cannot be used for
Assessment: Client Needs – Physiological Integrity, culture.
­Reduction of Risk Potential: Cognitive Level – Analysis: 3. The client should be taught to take all
Concept – Urinary Elimination/Infection. the prescribed medication anytime a
­prescription is written for antibiotics.
43. 1. The perineum should be cleaned from front
4. The client should be taught to void every
to back after a bowel movement to prevent
two (2) to three (3) hours and to empty the
fecal contamination of the urethral meatus.
bladder completely. This prevents over-
2. The temperature of the water does not mat-
distension of the bladder wall and result-
ter, but the client should take showers instead
ing compromised blood supply, either of
of baths to prevent bacteria in the bathwater
which predisposes the client to developing
from entering the urethra.
a UTI.
3. Voiding immediately after, not before, sexual
intercourse used the action of the urine pass- TEST-TAKING HINT: Unless contraindicated by
ing through the urethra to the outside of a disease process, it is recommended for all
the body to flush bacteria from the urethra clients to drink six (6) to eight (8) glasses of
that might have been introduced during water each day; therefore, option “1” should
intercourse. be eliminated as a possible correct answer.
4. Coffee, tea, cola, and alcoholic beverages Option “2” has the adjective “routine,” which
are urinary tract irritants. could be eliminated because routine proce-
dures are usually not implemented when the
TEST-TAKING HINT: The test taker might jump
client is ill.
to option “3” as the correct answer if the test
taker did not read the word “preceding.” Content – Medical: Integrated Nursing Process – ­
Planning: Client Needs – Physiological Integrity,
Content – Medical: Integrated Nursing Process – ­
­Physiological Adaptation: Cognitive Level – Synthesis:
Planning: Client Needs – Physiological Integrity,
Concept – Urinary Elimination/Infection.
­Physiological Adaptation: Cognitive Level – Synthesis:
Concept – Urinary Elimination/Infection.

09_Colgrove_Ch09.indd 377 01/06/16 5:45 PM


378 M ed -S urg S uccess

46. 1. Blood pressure within normal limits is a 3. Clients who develop chronic infections may
short-term goal. never be free of the bacteria.
2. Lack of protein in the urine is a short-term 4. HCPs do not usually prescribe prn prescrip-
goal. tions for antibiotics.
3. A long-term complication of glomerulo- TEST-TAKING HINT: The question is asking
nephritis is it can become chronic if unre- why an HCP prescribes long-term use of
sponsive to treatment, and this can lead to antibiotics for a client with a chronic infec-
end-stage renal disease. Maintaining renal tion. Antibiotics treat bacterial infections.
function is an appropriate long-term goal. Based on this, option “1” can be eliminated.
4. Clear lung sounds indicate the client has been Option “3” promises “all,” which is false re-
able to process fluids and excrete them from assurance and can be eliminated. Option “4”
the body. Preventing pulmonary edema is a describes future infections, but the client
short-term goal. currently has an infection, so this option can
TEST-TAKING HINT: Answer options “1,” “2,” be eliminated.
and “4” all refer to body processes controlled Content – Medical: Integrated Nursing Process – ­
or treated immediately after assessment of Diagnosis: Client Needs – Physiological Integrity,
the problem. The stem is requesting a long- ­Pharmacological and Parenteral Therapies: Cognitive
term goal. Level – Analysis: Concept – Medication.
Content – Medical: Integrated Nursing Process – ­
Diagnosis: Client Needs – Safe Effective Care ­Environment,
Management of Care: Cognitive Level – Analysis: Benign Prostatic Hypertrophy
Concept – Urinary Elimination/Infection.
49. 800 mL.
47. 1. Normal blood urea nitrogen levels are 7 to
First, determine the amount of irrigation fluid:
18 mg/dL or 8 to 20 mg/dL for clients older
3,000 − 1,800 = 1,200 mL of irrigation fluid
than age 60 years.
Then, subtract 1,200 mL of irrigation fluid
2. Normal creatinine levels are 0.6 to 1.2 mg/dL.
from the drainage of 2,000 mL to determine
3. Glomerular filtration rate (GFR) is
the urine output:
approximately 120 mL/min. If the GFR
2,000 − 1,200 = 800 mL of urine output
is decreased to 40 mL/min, the kidneys
are functioning at about one-third TEST-TAKING HINT: The test taker should use
filtration capacity. the drop-down calculator for the NCLEX-RN
4. Normal creatinine clearance is 85 to examination.
125 mL/min for males and 75 to Content – Medical Integrated Nursing Process – ­
115 mL/min for females. Assessment: Client Needs – Safe Effective Care
­Environment, Management of Care: Cognitive Level –
TEST-TAKING HINT: The nurse must memo-
Analysis: Concept – Urinary Elimination.
rize common laboratory values. BUN and
creatinine levels are common laboratory 50. 1. The indwelling catheter should not be re-
­values used to determine status in a number moved because doing so may result in edema,
of diseases. Options “1” and “2” are normal which, in turn, may obstruct the urethra and
values and could be eliminated. Then, the not allow the client to urinate.
test taker could choose from only two (2) 2. Increasing the irrigation fluid will flush
options. out the clots and blood.
Content – Medical: Integrated Nursing Process – ­ 3. Protamine is the reversal agent for heparin,
Assessment: Client Needs – Physiological Integrity, an anticoagulant.
­Reduction of Risk Potential: Cognitive Level – Analysis: 4. Vitamin K is the reversal agent for the antico-
Concept – Urinary Elimination/Infection. agulant warfarin (Coumadin).
48. 1. Antibiotics may indirectly treat bladder TEST-TAKING HINT: The test taker should
spasms if the spasms are caused by an infec- ­eliminate options “3” and “4” because both
tion, but this is not the reason for prescribing are medications and the problem is with con-
the antibiotic in this manner. tinuous irrigation, which does not require
2. Some clients develop a chronic infection medications.
and must receive antibiotic therapy as a Content – Surgical: Integrated Nursing Process –
routine daily medication to suppress the Implementation: Client Needs – Safe Effective Care
bacterial growth. The prescription will be ­Environment, Management of Care: Cognitive Level –
refilled after the 90 days and continued. ­Application: Concept – Urinary Elimination.

09_Colgrove_Ch09.indd 378 01/06/16 5:45 PM


C hapter 9  G enitourinary D isorders 379

51. 1. Terminal dribbling is a symptom of BPH. ­ nvironment, Management of Care: Cognitive Level –
E
2. Urinary frequency is a sign of a UTI. Analysis: Concept – Urinary Elimination.
3. Stress incontinence occurs in women who 54. 1. This indicates the teaching is effective.
urinate when coughing, running, or jumping. 2. Clients do not need to take Proscar
4. Clients with acute bacterial prostatitis will postoperatively.
frequently experience a sudden onset of 3. There is no reason to restrict the client’s fluid
fever and chills. Clients with chronic pros- intake.
tatitis have milder symptoms. 4. Pain medication should be taken as needed.
TEST-TAKING HINT: The words “acute” and
TEST-TAKING HINT: If the test taker is not
“bacterial” should cue the test taker into the sure of the correct answer, selecting an
specific symptoms of infection. Symptoms ­option addressing notifying a health-care
for any infection are fever and chills. provider is usually an appropriate choice.
Content – Medical: Integrated Nursing Process – ­
Content – Medical: Integrated Nursing Process – ­
Assessment: Client Needs – Physiological Integrity,
Evaluation: Client Needs – Physiological Integrity,
­Reduction of Risk Potential: Cognitive Level – Analysis:
­Physiological Adaptation: Cognitive Level – Synthesis:
Concept – Urinary Elimination/Infection.
Concept – Perioperative.
52. 1. The client should sit in a warm sitz bath 55. 1. This intervention requires analysis and
for 10 to 20 minutes several times each should not be delegated.
day to provide comfort and assist with 2. Elevating the scrotum on a towel for
healing. support is a task that can be delegated to
2. Clients should avoid sitting for extended the UAP.
periods because it increases the pressure. 3. The surgeon changes the first dressing;
3. Oral fluids should be consumed to satisfy therefore, this cannot be delegated. A TURP
thirst but not to push fluids to dilute the does not have a dressing.
medication levels in the bladder. 4. The nurse is responsible for teaching.
4. Broad-spectrum antibiotics are administered
for 10 to 14 days and should not be stopped TEST-TAKING HINT: Teaching, assessing,
until all medications are taken by the client. evaluating, and intervening for clients who
are unstable cannot be delegated to an
TEST-TAKING HINT: The test taker must know
­unlicensed assistive personnel (UAP).
basic concepts when answering questions;
Content – Medical: Integrated Nursing Process –
this includes the need for the client to take
­Planning: Client Needs – Safe Effective Care Environment,
all prescribed antibiotics. If the test taker is Management of Care: Cognitive Level – Synthesis:
unsure of option “3,” drinking plenty of tea Concept – Perioperative.
and coffee should indicate this is an incorrect
answer because these are high in caffeine. 56. 1. The nurse should not call a surgeon until all
Content – Medical: Nursing Process – Planning: Client assessment is completed.
Needs – Physiological Integrity, Physiological Adaptation: 2. A pain medication should not be adminis-
Cognitive Level – Synthesis: Concept – Urinary tered until the cause of the problem is deter-
Elimination/Infection. mined and all complications are ruled out.
3. Telling a client that what he is experiencing
53. 1. TURPs can cause a sexual dysfunction, but is expected without assessing the situation is
if there were a sexual dysfunction, it is not dangerous.
priority over a physiological problem such as 4. The nurse should always assess any com-
hemorrhaging. plaint before dismissing it as a commonly
2. This is not a life-threatening problem. occurring problem.
3. This client has had this problem
preoperatively. TEST-TAKING HINT: When the question re-
4. This is a potentially life-threatening quires the test taker to decide which inter-
problem. vention should be first, assessment is usually
first. If the test taker has no idea which in-
TEST-TAKING HINT: A basic concept the test
tervention is correct, the test taker should
taker must know is, for most surgeries, the choose assessment.
highest priority problem is hemorrhaging.
Content – Surgical: Integrated Nursing Process –
Hemorrhaging is life threatening.
Implementation: Client Needs – Safe Effective Care
Content – Surgical: Integrated Nursing Process– ­ ­Environment, Management of Care: Cognitive Level –
Assessment: Client Needs – Safe Effective Care ­Application: Concept – Urinary Elimination.

09_Colgrove_Ch09.indd 379 01/06/16 5:45 PM


380 M ed -S urg S uccess

57. 1. The client wants information and the nurse the test taker should recognize the signs and
should provide facts. symptoms of shock.
2. The client wants information and the nurse Content – Surgical: Integrated Nursing Process –
should provide facts. Implementation: Client Needs – Safe Effective Care
3. This is usually the length of time clients ­Environment, Management of Care: Cognitive Level –
need to wait prior to having sexual inter- Application: Concept – Perioperative.
course; this is the information the client 60. 1. Using the maximum amount of medication
wants to know. does not indicate the client is achieving pain
4. The client may need to talk with his s­ urgeon, management.
but it should be after the nurse answers the 2. Bladder spasms are common, but being
client’s question. relieved with medication indicates the
TEST-TAKING HINT: The client is asking for condition is improving.
factual information and the nurse should 3. Scrotal edema and tenderness do not indicate
provide this information. Options “1” and “2” improvement.
are therapeutic responses addressing feel- 4. Clients are administered laxatives or stool
ings, and option “4” is passing the buck—the softeners to prevent constipation, which
nurse can discuss this with the client. could cause increased pressure.
Content – Surgical: Integrated Nursing Process – TEST-TAKING HINT: The stem asks which
Implementation: Client Needs – Safe Effective Care ­option indicates the client is improving.
­Environment, Management of Care: Cognitive Level –
Needing maximum medication (option “1”)
­Application: Concept – Urinary Elimination.
and scrotal edema (option “3”) do not in-
58. 1. An elevated PSA can be from urinary dicate the client is getting better. A bowel
­retention, BPH, prostate cancer, or movement has nothing to do with the
­prostate infarct. prostate.
2. An elevated PSA does not indicate only Content – Surgical: Integrated Nursing Process –
­prostate cancer. Assessment: Client Needs – Physiological Integrity,
3. PSA does not diagnose testicular cancer. ­Reduction of Risk Potential: Cognitive Level – Analysis:
4. An elevated PSA and digital examination are Concept – Urinary Elimination.
used in combination to diagnose BPH or
prostate cancer. Renal Calculi
TEST-TAKING HINT: Answer options “2” and
“4” have the word “only”; an absolute word 61. 1. This is appropriate for the client who has uric
should cause the test taker to eliminate them acid stones.
as possible answers. Options with words such 2. The nurse should recommend drinking
as “always,” “never,” and “only” are usually one (1) to two (2) glasses of water at night
incorrect. to prevent concentration of urine during
Content – Medical: Integrated Nursing Process – ­ sleep.
Diagnosis: Client Needs – Safe Effective Care ­Environment, 3. Dietary changes for preventing renal
Management of Care: Cognitive Level – Analysis: stones include reducing the intake of the
Concept – Cellular Regulation. primary substance forming the calculi.
59. 1. The nurse should assess the drain In this case, limiting vitamin D will inhibit
postoperatively. the absorption of calcium from the gas-
2. The client is hemorrhaging, so the nurse trointestinal tract.
should increase the irrigation fluid to clear 5. This is a treatment for an existing renal stone,
the red urine, not decrease the rate. not a discharge teaching intervention for a
3. The head of the bed should be lowered client who has successfully passed a renal
and the foot should be elevated to shunt calculus.
blood to the central circulating system. TEST-TAKING HINT: The test taker should
4. The surgeon needs to be notified of the ­remember to read the question carefully.
change in condition. The question asks for a “discharge teach-
5. These laboratory values assess kidney func- ing” intervention. This rules out option “4,”
tion, not the circulatory system, so this is not which is a treatment, as a potential answer.
an appropriate intervention. Content – Medical: Integrated Nursing Process – ­
TEST-TAKING HINT: When the test taker reads Planning: Client Needs – Physiological Integrity,
vital signs with the blood pressure decreased ­Physiological Adaptation: Cognitive Level – Synthesis:
Concept – Urinary Elimination.
and the pulse and respiratory rate elevated,

09_Colgrove_Ch09.indd 380 01/06/16 5:45 PM


C hapter 9  G enitourinary D isorders 381

62. 1. The client’s urinary output should be 4. Kidney stones and bladder stones may pro-
monitored, but it is not the first nursing duce no signs/symptoms, but a ureteral stone
intervention. always causes pain on the affected side be-
2. Assessment is the first part of the nurs- cause a ureteral spasm occurs when the stone
ing process and is priority. The renal colic obstructs the ureter.
pain can be so intense it can cause a vaso- TEST-TAKING HINT: Options “1” and “3” both
vagal response, with resulting hypotension have assessment data indicating bleeding.
and syncope. The test taker can usually eliminate these as
3. Increased fluid increases urinary output, possible answers or eliminate the other two
which will facilitate movement of the renal options not addressing blood. Renal stones
stone through the ureter and help decrease are painful; therefore, option “4” could be
pain, but it is not the first intervention. eliminated as a possible answer.
4. Ambulation will help facilitate movement of
Content – Medical: Integrated Nursing Process – ­
the renal stone through the ureter and safety Assessment: Client Needs – Physiological Integrity,
is important, but it is not the first intervention. ­Reduction of Risk Potential: Cognitive Level – Analysis:
TEST-TAKING HINT: Remember, if the question Concept – Urinary Elimination.
asks which intervention is first, all four (4)
65. 1. The health-care provider may order cer-
options may be appropriate for the client’s
tain foods and medications when obtain-
diagnosis but only one has priority. Assess-
ing a 24-hour urine collection to evaluate
ment is the first part of the nursing process
for calcium oxalate or uric acid.
and it is the first intervention a nurse should
2. When the collection begins, the client
implement if the client is not in distress.
should completely empty the bladder and
Content – Medical: Integrated Nursing Process – discard this urine. The test is started after
Implementation: Client Needs – Safe Effective Care the bladder is empty.
­Environment, Management of Care: Cognitive Level –
3. All urine for 24 hours should be saved
Analysis: Concept – Urinary Elimination.
and put in a container with preservative,
63. 1. An ultrasound does not require administra- refrigerated, or placed on ice as indicated.
tion of contrast dye. Not following specific instructions will re-
2. Food, fluids, and ordered medication are not sult in an inaccurate test result.
restricted prior to this test. 4. The urine is obtained in some type of urine
3. This is not an invasive procedure, so a signed collection device such as a bedpan, bedside
consent is not required. commode, or commode hat. The client is not
4. No special preparation is needed for this catheterized.
noninvasive, nonpainful test. A conductive 5. The nurse can delegate placing the urine
gel is applied to the back or flank and then output in the proper container to the UAP;
a transducer is applied, which produces therefore, the UAP does not need to notify
sound waves, resulting in a picture. the nurse when the client urinates.
TEST-TAKING HINT: The nurse must be aware TEST-TAKING HINT: This is an alternate-type
of preprocedure and postprocedure teach- ­question that has more than one correct an-
ing and care. The test taker must know the swer. The test taker must be knowledgeable
invasive and noninvasive diagnostic tests in of specific laboratory tests.
general. Ultrasound, computed tomography Content – Medical: Integrated Nursing Process –
(CT), and magnetic resonance imaging (MRI) Implementation: Client Needs – Safe Effective Care
are a few of the noninvasive diagnostic tests. ­Environment, Management of Care: Cognitive Level –
Content – Medical: Integrated Nursing Process – ­Application: Concept – Urinary Elimination.
Implementation: Client Needs – Physiological Integrity, 66. 1. The client’s fluid volume is increased and
Reduction of Risk Potential: Cognitive Level – Application:
there is usually not a fluid volume loss.
Concept – Urinary Elimination.
2. Knowledge deficit is important to help
64. 1. Dull flank pain and microscopic hematuria are prevent future renal calculi, but this is not
manifestations of a renal stone in the kidney. priority when the client is in pain, which
2. The severe flank pain associated with a will occur with an acute episode.
stone in the ureter often causes a sympa- 3. Impaired urinary elimination may occur, but
thetic response with associated nausea; it is not priority for the client with an acute
vomiting; pallor; and cool, clammy skin. episode of calculi.
3. Gross hematuria and suprapubic pain when 4. Pain is priority. The pain can be so severe
voiding are manifestations of a stone in the a sympathetic response may occur, caus-
bladder. ing nausea; vomiting; pallor; and cool,
clammy skin.

09_Colgrove_Ch09.indd 381 01/06/16 5:45 PM


382 M ed -S urg S uccess

TEST-TAKING HINT: Remember Maslow’s 2. Passing a renal stone may negate the need
­ ierarchy of needs: airway and pain are pri-
h for the client to have lithotripsy or a sur-
ority. No option mentions possible airway gical procedure. Therefore, all urine must
problems, so pain is priority. be strained, and a stone, if found, should
Content – Medical: Integrated Nursing Process – ­ be sent to the laboratory to determine
Diagnosis: Client Needs – Safe Effective Care ­Environment, what caused the stone.
Management of Care: Cognitive Level – Analysis: 3. These are laboratory studies evaluating
Concept – Urinary Elimination. ­kidney function, but they are not pertinent
when passing a renal stone. These values
67. 1. The urine must be assessed for bleeding and
do not elevate until at least half the kidney
cloudiness. Initially the urine is bright red,
­function is lost.
but the color soon diminishes and cloudiness
4. A dietary recall can be done to determine
may indicate an infection. This assessment
what types of foods the client is eating that
should not be delegated to a UAP.
may contribute to the stone formation, but it
2. Teaching cannot be delegated to a UAP. The
is not the most important intervention.
nurse should teach and evaluate the effective-
ness of the teaching. TEST-TAKING HINT: Remember, if the ques-
3. The UAP could assist the client to the car tion asks for “most important,” more than
once the discharge has been completed. one of the options could be appropriate but
4. The kidney is highly vascular. Hemorrhaging only one is most important. Assessment is
and resulting shock are potential complica- priority if the client is not in distress, but the
tions of lithotripsy, so the nurse should not test taker should make sure it is appropriate
delegate vital signs postprocedure. for the situation.
TEST-TAKING HINT: There are some basic Content – Medical: Integrated Nursing Process –
rules about delegation: the nurse cannot del- Implementation: Client Needs – Safe Effective Care
­Environment, Management of Care: Cognitive Level –
egate assessment, teaching, evaluation, or any
Analysis: Concept – Urinary Elimination.
task requiring judgment.
Content – Surgical: Integrated Nursing Process – 70. 1. The client needs to be evaluated for a
­Planning: Client Needs – Safe Effective Care Environment, possible urinary tract infection, which may
Management of Care: Cognitive Level – Synthesis: accompany renal calculi. Therefore, the clinic
Concept – Nursing Roles. nurse should not give advice without knowing
what is wrong with the client.
68. 1. An increased fluid intake ensuring 2 to
2. The nurse should not recommend any
3 L of urine a day prevents the stone-
medication (even Tylenol) unless the nurse is
forming salts from becoming concen-
absolutely sure what is wrong with the client.
trated enough to precipitate.
3. A urinalysis can assess for hematuria, the
2. Cocoa and chocolate are high in calcium and
presence of white blood cells, crystal frag-
should be avoided or the amount should be
ments, or all three, which can determine if
decreased to help prevent formation of cal-
the client has a urinary tract infection or
cium phosphate renal stones.
possibly a renal stone, with accompanying
3. Physical activity prevents bone absorption
signs/symptoms of UTI.
and possible hypercalciuria; therefore, the
4. The client needs to strain the urine if there
nurse should instruct the client to walk daily
is a possibility of renal calculi, which these
to help retain calcium in bone.
signs/symptoms do not support. Further
4. The renal calculi are caused by calcium;
diagnostic testing is needed to determine the
therefore, the client should not increase
presence of renal calculi.
­calcium intake.
TEST-TAKING HINT: Fever, chills, and burn-
TEST-TAKING HINT: This is a urinary problem
ing on urination require some type of as-
and fluid is priority. Therefore, the test taker
sessment. Therefore, the test taker should
should select an option addressing fluid, and
select an option that helps determine what
there is only one option addressing oral intake.
is wrong with the client and “3” is the only
Content – Medical: Integrated Nursing Process – ­
such option.
Evaluation: Client Needs – Physiological Integrity,
­Physiological Adaptation: Cognitive Level – Synthesis: Content – Surgical: Integrated Nursing Process –
Concept – Urinary Elimination. ­Planning: Client Needs – Safe Effective Care Environment,
Management of Care: Cognitive Level – Synthesis:
69. 1. Assessment is important, but the neurological Concept – Urinary Elimination/Infection.
system is not priority for a client with a uri-
nary problem.

09_Colgrove_Ch09.indd 382 01/06/16 5:45 PM


C hapter 9  G enitourinary D isorders 383

71. 1. This potassium level is within normal limits, placed in the ureters have not become
3.5 to 5.5 mEq/L. ­dislodged or blocked.
2. Hematuria is not uncommon after removal of TEST-TAKING HINT: Basic care of any post-
a kidney stone. operative client is to ensure urinary output.
3. A normal creatinine level is 0.8 to 1.2 mg/ Two of the options involve tasks that can
100 mL. be delegated or are not in the realm of the
4. The white blood cell count is elevated; nurse.
normal is 5,000 to 10,000/mm3. Content – Surgical: Integrated Nursing Process – ­
TEST-TAKING HINT: The nurse must know Assessment: Client Needs – Safe Effective Care
normal laboratory data and be able to apply ­Environment, Management of Care: Cognitive Level –
the normal and abnormal results to specific Analysis: Concept – Urinary Elimination.
diseases and disorders. 74. 1. The client has already been exposed; this
Content – Surgical: Integrated Nursing Process – ­cannot be undone.
Assessment: Client Needs – Physiological Integrity, 2. Pelvic radiation is prescribed for cancer in
­Reduction of Risk Potential: Cognitive Level – Analysis:
the abdomen. It is a life-saving procedure,
Concept – Perioperative.
but one of the risks of radiation therapy is the
72. 1. Beer and colas are foods high in oxalate, development of a secondary cancer.
which can cause calcium oxalate stones. 3. Cigarette smoke contains more than 400
2. Asparagus and cabbage are foods high in oxa- chemicals, 17 of which are known to cause
late, which can cause calcium oxalate stones. cancer. The risk is directly proportional to
3. Venison, sardines, goose, organ meats, the amount of smoking.
and herring are high-purine foods, which 4. Clients may be unaware of a parasitic
should be eliminated from the diet to help ­infection of the bladder for some time prior
prevent uric acid stones. to diagnosis, but it is not a risk factor for
4. Cheese and eggs are foods that help acidify ­cancer of the bladder.
the urine and do not cause the development TEST-TAKING HINT: The question asks for a
of uric acid stones. modifiable risk factor. Modifiable factors
TEST-TAKING HINT: The nurse has to be ­involve lifestyle changes, weight loss, tobacco
knowledgeable of foods included in specific use, and eating habits.
diets. This is memorizing, but the test taker Content – Medical: Integrated Nursing Process – ­
must have this knowledge to answer ques- Diagnosis: Client Needs – Safe Effective Care E
­ nvironment,
tions evaluating types of diets for specific Management of Care: Cognitive Level – Knowledge:
diseases and disorders. Concept – Cellular Regulation.
Content – Medical: Integrated Nursing Process – 75. 1. Any client undergoing general anesthesia
­Planning: Client Needs – Physiological Integrity, Basic Care should be taught to turn, cough, and deep
and Comfort: Cognitive Level – Synthesis:
breathe to prevent pulmonary complications.
Concept – Nutrition.
This is not specific to a urinary diversion
procedure.
Cancer of the Bladder 2. A urinary diversion procedure involves
the removal of the bladder. In a cutaneous
73. 1. This client does not need to be assessed first. procedure, the ureters are implanted in
A ward secretary can call the department and some way to allow for stoma formation on
check on the glasses. the abdominal wall, and the urine drains
2. A moderate amount of serous drainage is into a pouch. There are numerous meth-
expected after a surgery. Serous drainage is ods used for creating the stoma.
pale yellow body fluid. Sanguineous is the 3. Many clients with multiple types of proce-
term used to describe bloody drainage. dures use PCA pumps to control pain after
3. The nurse is not responsible for informing surgery.
the client about operative procedures. The 4. This should be done for any client who is ex-
surgeon should be notified to see this client pected to need intensive care postoperatively.
and provide the explanation. TEST-TAKING HINT: The test taker must no-
4. An ileal conduit is a procedure divert- tice the phrase “specific to the procedure” to
ing urine from the bladder and provides be able to correctly answer this question. All
an alternate cutaneous pathway for urine of the options are standard interventions for
to exit the body. Urinary output should major surgeries, but only one is specific to
always be at least 30 mL/hr. This client the procedure.
should be assessed to make sure the stents

09_Colgrove_Ch09.indd 383 01/06/16 5:45 PM


384 M ed -S urg S uccess

Content – Surgical: Integrated Nursing Process – ­ 2. This client has the laboratory symptoms
Planning: Client Needs – Health Promotion and of an infection; therefore, the nurse should
­Maintenance: Cognitive Level – Synthesis: Concept – assess and care for this client.
Urinary Elimination. 3. Antineoplastic medication is administered
76. 1. The client will have medication instilled only by a qualified registered nurse.
in the bladder, which must remain in the 4. It is in the scope of practice for the LPN
bladder for a prescribed length of time. to care for this client.
For this reason, the client must remain TEST-TAKING HINT: The client who is the least
NPO before the procedure. ill or the client having the least invasive pro-
2. Informed consent is important to do when cedure is the client who should be assigned
informing the client about chemotherapy, but to the LPN.
this is done when the client gives consent to Content – Nursing Management: Integrated Nursing
receiving intravesical chemotherapy, not as Process – Planning: Client Needs – Safe Effective Care
part of the pretherapy routine. Environment, Management of Care: Cognitive Level –
3. The advantage of administering chemo­ Synthesis: Concept – Nursing Roles.
therapy intravesically is systemic side effects
79. 1. This is therapeutic communication and is
of bone marrow suppression are avoided.
allowing the client autonomy, but it is not
Neupogen is used to stimulate the production
an example of paternalism.
of white blood cells so a client is not at risk
2. Paternalism is deciding for the client what
for developing an infection.
is best, similar to a parent making deci-
4. The procedure is not painful, so an analgesic
sions for a child. Feeding a client, as with
is not needed.
a feeding tube, without the client wishing
TEST-TAKING HINT: If the test taker is not to eat is paternalism.
aware of the term “intravesical,” then divid- 3. Consulting with a dietitian about the nutri-
ing the word into its components may be tional needs of a client is an appropriate nurs-
useful. Intra- means “into” and vesical means ing intervention, but it does not represent any
“bladder.” The test taker should choose ethical principle.
an option that has a direct effect on urine 4. This is an excellent intervention, but it does
production. not represent any ethical principle.
Content – Medical: Integrated Nursing Process – ­ TEST-TAKING HINT: The question asks for an
Planning: Client Needs – Physiological Integrity,
ethical principle, and only two of the options
­Pharmacological and Parenteral Therapies: Cognitive
could be considered to represent ethical
Level – Synthesis: Concept – Urinary Elimination.
principles. Option “1” is allowing the client a
77. 1. Urine is acidic, and the abdominal wall voice in the situation; the term “paternalism”
­tissue is not designed to tolerate acidic eliminates this option.
environments. The stoma is pouched so Content – Medical: Integrated Nursing Process –
urine will not touch the skin. Implementation: Client Needs – Safe Effective Care
2. Urinary diversion drainage bags are changed ­Environment, Management of Care: Cognitive Level –
every four (4) to five (5) days so the skin can Application: Concept – Nutrition.
remain intact; the bags should be clean but
80. 1. This is advising the client, a nontherapeutic
not sterile.
technique.
3. The urine will have the normal pH of all
2. This statement does not address the client’s
urine; it is not necessary to monitor the pH.
feelings.
4. The stoma should be assessed a minimum of
3. This is an example of restating, a thera-
every two (2) hours initially, then every four
peutic technique used to clarify the cli-
(4) hours.
ent’s feelings and encourage a discussion
TEST-TAKING HINT: The test taker should look of those feelings.
at time frames—daily and weekly. If the time 4. The stem did not say the client was dying.
frame is not sufficient, then the option can The stem said the client thinks he or she is
be eliminated as a possible correct answer. too young to die. A conversation to discuss
Content – Surgical: Integrated Nursing Process – the client’s death with the children may be
­Planning: Client Needs – Safe Effective Care Environment, premature.
Management of Care: Cognitive Level – Synthesis:
TEST-TAKING HINT: When the question
Concept – Urinary Elimination.
­requires a therapeutic response, the test
78. 1. Assessment cannot be assigned to an LPN, no taker should select an option addressing the
matter how knowledgeable the LPN. client’s feelings.

09_Colgrove_Ch09.indd 384 01/06/16 5:45 PM


C hapter 9  G enitourinary D isorders 385

Content – Medical: Integrated Nursing Process – Content – Surgical: Integrated Nursing Process – ­
Implementation: Client Needs – Psychosocial Integrity: Planning: Client Needs – Physiological Integrity,
Cognitive Level – Application: Concept – Cellular ­Physiological Adaptation: Cognitive Level – Synthesis:
Regulation. Concept – Urinary Elimination.

81. 1. A continent urinary diversion is a surgical 83. 1. A complaint of a “3” on a 1-to-10 pain scale is
procedure in which a reservoir is created expected after medication and does not war-
to hold urine until the client can self- rant notifying the HCP.
catheterize the stoma. The nurse should 2. Pain on coughing and deep breathing after
observe the client’s technique before surgery is expected.
discharge. 3. This indicates the client is able to ambulate
2. The purpose of creating a continent diversion and is doing activities needed to recover.
is so the client will not need a pouch. 4. The client is drawn up in a position that
3. Clients with cutaneous diversions that drain relieves pressure off the abdomen; a rigid
constantly use a bedside drainage bag at abdomen is an indicator of peritonitis, a
night, not those with continent diversions. medical emergency.
4. The client should be taught to notify the TEST-TAKING HINT: When the test taker is
HCP if the temperature is 100°F or greater. deciding on a priority question, the test taker
TEST-TAKING HINT: Options “2” and “3” are should decide if the situation is expected or
related to continuous drainage and could be if it is life threatening.
eliminated on this basis. The word “conti- Content – Surgical: Integrated Nursing Process – ­
nent” in the stem should key the test taker to Assessment: Client Needs – Safe Effective Care
the fact this diversion is a procedure in which ­Environment, Management of Care: Cognitive Level –
there is no continuous drainage of urine. Analysis: Concept – Urinary Elimination/Infection.
Content – Surgical: Integrated Nursing Process – ­ 84. 1. This client is asking for information and
Planning: Client Needs – Physiological Integrity, should be provided factual information.
­Physiological Adaptation: Cognitive Level – Synthesis:
The surgery will not make the client ster-
Concept – Urinary Elimination.
ile, but chemotherapy can induce meno-
82. 1. Vinegar will act as a deodorizing agent in pause and radiation therapy to the pelvis
the pouch and help prevent a strong urine can render a client sterile.
smell. 2. This is a therapeutic response, but the client
2. The stoma should be pink and moist at all asked for information.
times. A dusky color indicates a compromised 3. This is a false statement and lying to the
blood supply to the stoma, and the HCP client.
should be notified immediately. 4. This is outside the realm of a chaplain.
3. There will be mucus in the urine because of TEST-TAKING HINT: When the stem has the
the tissue used to create the diversion, but client asking for specific information, then
large clumps of mucus could occlude the the nurse should provide the correct infor-
stoma or ureters. mation. It is easy to confuse these questions
4. Urinary drainage should be a pale yellow to with ones requiring therapeutic responses.
amber color. The procedure does not change Content – Surgical: Integrated Nursing Process –
the color of the urine. Implementation: Client Needs – Safe Effective Care
TEST-TAKING HINT: A dusky color is never ­Environment, Management of Care: Cognitive Level –
normal when discussing body functioning. Application: Concept – Cellular Regulation.
There are very few procedures for which
bloody urine is a normal expectation.

09_Colgrove_Ch09.indd 385 01/06/16 5:45 PM


CONCEPTS

85. 1. This client has an elevated WBC but it only question but this is especially true of “Select
informs the nurse that an infection may be all that apply” questions.
present in the body. It is not diagnostic for Content – Medical: Integrated Nursing Process –
a UTI. Implementation: Client Needs – Safe Effective Care
2. Cloudy urine may indicate a UTI but the cul- ­Environment, Management of Care: Cognitive Level –
ture is definitive. Cloudy urine may contain Analysis: Concept – Urinary Elimination.
protein, WBCs, RBCs, bacteria, or noncellu- 87. 1. The nurse taking vital signs and document-
lar casts. ing them will not prevent acute renal failure
3. The urine culture has identified an infec- because action is not initiated that will di-
tious organism. This is the diagnostic test rectly affect the client’s health status because
for a UTI. of the results of the data. The nurse must
4. Twenty-four-hour urine specimens for meta- always initiate an intervention based on
nephrines indicate the presence of other abnormal data assessed.
diseases but not a UTI. Mild increases in 2. Assessing the client’s dressing will allow the
catecholamines in urine can be caused by nurse to be aware of bleeding but does not
stress such as operations, burns, or child- prevent acute renal failure.
birth. A marked increase in metanephrines 3. The urinary output is checked to ensure the
indicates a pheochromocytoma (a hereditary
kidneys are being perfused but there is no
tumor on the adrenal medulla or from certain
action that will maintain the perfusion in this
neuroblastomas).
option.
TEST-TAKING HINT: The test taker must pay 4. Maintaining the client’s blood pressure
attention to specific words. “Diagnostic” to greater than 100/60 ensures perfusion
means a test that specifically indicates with- of the kidneys. Acute renal failure oc-
out a doubt the cause of the disease process. curs when the kidneys have not been ad-
Content – Medical: Integrated Nursing Process – equately perfused. Vasopressor drips are
Implementation: Client Needs – Safe Effective Care used to maintain the BP.
­Environment, Management of Care: Cognitive Level –
TEST-TAKING HINT: The test taker must be
Analysis: Concept – Urinary Elimination.
aware of the purpose behind medications and
86. 1. The current diet should be one that limits the the results of inadequate administration of
complications of CRF; it is current, not an certain critical medications.
antecedent. Content – Medical: Integrated Nursing Process –
2. Diabetes is a leading cause of renal failure Implementation: Client Needs – Safe Effective Care
caused by the macrovascular changes that ­Environment, Management of Care: Cognitive Level –
occur when the blood glucose levels are Synthesis: Concept – Fluid and Electrolyte Balance.
high. 88. 1. The client had a small emesis 24 hours ago,
3. Hypertension is also a leading cause of re- which has not been repeated; this is not
nal failure because hypertension narrows pertinent information at this time.
the renal artery and decreases the blood 2. Whether or not IV fluids are indicated de-
flow to the kidney. pends on more than the oral intake. The
4. Fluid restriction is a recommended treatment
recommendation to discontinue the IV fluids
for CRF, not an antecedent.
might be indicated because the client is not
5. Race is an antecedent because genetics are
able to process all the fluid.
a risk factor for CRF. Non-Caucasians are 3. The client has a deficit output of almost
more at risk for developing CRF, especially 5,500 mL. This should be brought to the
when the client has a comorbid condition HCP’s attention to determine if renal
such as diabetes or hypertension. insufficiency is present.
TEST-TAKING HINT: When answering a “Select 4. Clients are frequently requested to do things
all that apply” question, the test taker must that give the health-care personnel informa-
look at each option independently of the other tion to assess what is occurring with the cli-
options. Each option becomes a true/false ent. It is not necessary for the client to “like”
question. The NCLEX-RN can test several saving the urine.
diseases or concepts in any multiple-choice
386

09_Colgrove_Ch09.indd 386 02/06/16 4:00 PM


C hapter 9  G enitourinary D isorders 387

TEST-TAKING HINT: The test taker must be 4. The nurse would not question administer-
able to read and interpret information from ing this medication. Nothing in the question
graphs to determine important information would lead the nurse to think that the order
to report. was incorrect.
Content – Medical: Integrated Nursing Process – TEST-TAKING HINT: The test taker/nurse
Implementation: Client Needs – Safe Effective Care should always remember the five (5) rights
­Environment, Management of Care: Cognitive Level – of medication administration. An incomplete
Synthesis: Concept – Fluid and Electrolyte Balance. medication order cannot be implemented
89. 1. These are called the pre- and post- until all the requirements of the order are
weights. The pre-weight is used to deter- present.
mine the amount of fluid to be removed Content – Medical: Integrated Nursing Process –
during the treatment and the post-weight Implementation: Client Needs – Pharmacological Therapy,
is used to determine if the goal was met. Management of Care: Cognitive Level – Analysis:
2. Clients experiencing renal failure are not Concept – Urinary Elimination.
processing the fluids in their body. Fluid 91. 1. A female client should be taught to wipe
restrictions are prescribed to allow for the meatus and vaginal area from front to
some fluid so the client does not become back to avoid contaminating the urethra
dehydrated but limited so the heart is not (urinary orifice) with fecal matter.
overtaxed, causing the client to go into 2. Cranberry juice is acidic and changes the
heart failure. pH of the urine, making the environment
3. Potato chips and pretzels are high in sodium less conducive to bacterial growth.
content and could increase a problem with 3. Incontinence is not expected for a client with
fluid retention. a UTI.
4. The client’s entire blood supply is being 4. The client should be taught about the
removed from the body and then returned signs and symptoms of a UTI so she can
after being filtered. The client could bleed know when to notify the HCP.
to death in a matter of minutes if the ac- 5. The client should increase the intake of
cess becomes dislodged. water to at least 200 mL/24 hours in or-
5. Most clients develop a routine of resting der to flush the bacteria from the urinary
during the treatments. The nurse should not system.
keep the client from being able to rest, read, 6. Sitting in a tub of water increases the risk of
or watch television. bacteria entering the urethra.
TEST-TAKING HINT: Asking about medications, TEST-TAKING HINT: The test taker must re-
especially over-the-counter and herbal rem- member basic nursing care. Several options
edies, during the admission interview is an are basic nursing interventions.
important intervention because many medi- Content – Medical: Integrated Nursing Process –
cations are nephrotoxic and hepatotoxic. Intervention: Client Needs – Safe Effective Care
Content – Medical: Integrated Nursing Process – ­Environment, Management of Care: Cognitive Level –
Intervention: Client Needs – Safe Effective Care Analysis: Concept – Urinary Elimination.
­Environment, Management of Care: Cognitive Level –
Synthesis: Concept – Urinary Elimination. 92. 1. This meal has a small portion of protein
and does not contain sodium if the client
90. 1. Clients diagnosed with renal failure are does not add salt.
frequently placed on a diuretic. The potas- 2. Ham and sour cream are high in sodium con-
sium level is WNL. The nurse would not tent, and 12 ounces of protein is too much for
question administering this medication. the client.
2. Erythropoietin is frequently prescribed 3. Double burger patties are too much protein,
for the anemia associated with renal and cheese is high in sodium.
failure; however, this is an incomplete 4. Potato chips are high in sodium, and the roast
order because no dosage is prescribed. beef could be too much protein.
The nurse should contact the HCP to de-
TEST-TAKING HINT: Dietary menus/questions
termine the dose to be administered. The
are some of the least favorite of many test
dose is determined by units/kg/dose.
takers. Basic rules include, if a choice exists
3. The nurse would not question administering
between grilled or fried foods, choose grilled,
this medication. The client should receive the
between chicken or beef, choose chicken.
multivitamin, and iron will assist in the pro-
Fresh vegetables are usually better than
duction of red blood cells.
canned or frozen.

09_Colgrove_Ch09.indd 387 01/06/16 5:45 PM


388 M ed -S urg S uccess

Content – Medical: Integrated Nursing Process – ­ produce erythropoietin to stimulate the


Evaluation: Client Needs – Safe Effective Care bone marrow to produce red blood cells.
­Environment, Management of Care: Cognitive Level – In addition, removal of the entire circulat-
Analysis: Concept – Fluid and Electrolyte Balance. ing blood three (3) times a week through
93. 1. Before the other options are performed the dialysis machine places stress on the
the nurse should have a urine culture red blood cells and they do not last as long
specimen sent to the laboratory for as in a normal body.
­culture. A culture is indicated from the 3. The CRF client does not have an issue with
symptoms. digestion unless he/she has a comorbid
2. The IV catheter will be required before ­condition that involves lack of mobility.
being able to initiate the antibiotic but 4. The CRF client does not have an issue with
getting the urine culture is first. metabolism unless he/she has a comorbid
3. The urine culture is first. condition that involves lack of mobility.
4. The urine culture is first so that treatment 5. The CRF client does not have an issue
can be initiated. with mobility unless he/she has a comorbid
condition that involves lack of mobility.
TEST-TAKING HINT: Basic nursing principles
6. Nutrition is an issue because the client
are applied in test questions, such as the re-
must adhere to a restricted diet to de-
sults of cultures will be skewed if the sample
crease the amount of toxic metabolites
is collected after antibiotics have been
not being eliminated through the kidneys.
initiated.
Content – Medical: Integrated Nursing Process – TEST-TAKING HINT: The test taker must re-
Implementation: Client Needs – Safe Effective Care member the entire client when determining
­Environment, Management of Care: Cognitive Level – the client needs/problems, including know-
Analysis: Concept – Urinary Elimination/Infection. ing the pathology of the disease process and
how it affects other systems in the body.
94. 1. The HCP should be notified but this delay
Content – Medical: Integrated Nursing Process – ­
could cost the client his/her life; this client is
Assessment: Client Needs – Safe Effective Care
in septic shock. ­Environment, Management of Care: Cognitive Level –
2. The IVPB will not treat the client as quickly Analysis: Concept – Fluid and Electrolyte Balance
as increasing the IV fluids. This would be the
second action to be performed by the nurse. 96. 1. The urine output would not be affected by
3. This is not the time to check the chart for laboratory results.
­information; it is the time for action. 2. The blood glucose is higher than normal but
4. This is septic shock and not fluid volume would not require the nurse to notify the
shock but the circulatory system is still HCP. The HCP can note the level on rounds.
compromised. Increasing the fluid volume The UAP does not need to collect a blood
will support the client’s BP until the IVPB glucose reading.
is infused. 3. The potassium level is at a critical level.
Low-potassium levels impact the cardiac
TEST-TAKING HINT: The test taker must
rhythm by causing a dysrhythmia. The
remember: “If in stress, do not assess; do
nurse should assess the telemetry reading
something that will treat the client.”
to determine if this is occurring.
Content – Medical: Integrated Nursing Process –
4. At this time the RRT is not needed.
Implementation: Client Needs – Safe Effective Care
­Environment, Management of Care: Cognitive Level – TEST-TAKING HINT: The test taker must be
Analysis: Concept – Urinary Elimination/Infection. able to read and interpret laboratory results.
Content – Medical: Integrated Nursing Process –
95. 1. The balance of fluids and electrolytes
­Assessment: Client Needs – Safe Effective Care Environment,
is regulated by the kidneys. Management of Care: Cognitive Level – Analysis:
2. Hematologic regulation is an interrelated Concept – Fluid and Electrolyte Balance.
concept because the client on dialysis
does not have a functioning kidney to

09_Colgrove_Ch09.indd 388 01/06/16 5:45 PM


C hapter 9  G enitourinary D isorders 389

GENITOURINARY DISORDERS
COMPREHENSIVE EXAMINATION
1. The elderly client being seen in the clinic has 6. The nurse is preparing the plan of care
complaints of urinary frequency, urgency, and for the client diagnosed with a neurogenic
“leaking.” Which priority intervention should flaccid bladder. Which expected outcome is
the nurse implement when interviewing the client? appropriate for this client?
1. Ensure communication is nonjudgmental and 1. The client has conscious control over bladder
respectful. activity.
2. Set the temperature for comfort in the 2. The client’s bladder does not become
examination room. overdistended.
3. Speak loudly to ensure the client understands 3. The client has bladder sensation and no
the nurse. discomfort.
4. Ensure the examining room has adequate 4. The client demonstrates how to check for
lighting. bladder distention.
2. The client is experiencing urinary incontinence. 7. Which nursing intervention is most important
Which intervention should the nurse implement? before attempting to catheterize a client?
1. Teach the client to drink prune juice weekly. 1. Determine the client’s history of catheter use.
2. Encourage the client to eat a high-fiber diet. 2. Evaluate the level of anxiety of the client.
3. Discuss the need to urinate every six (6) hours. 3. Verify the client is not allergic to latex.
4. Explain the importance of wearing cotton 4. Assess the client’s sensation level and ability
underwear. to void.
3. Which information indicates to the nurse the 8. Which client should the nurse not assign to a
client teaching about treatment of urinary UAP working on a surgical floor?
incontinence has been effective? 1. The client with a suprapubic catheter inserted
1. The client prepares a scheduled voiding plan. yesterday.
2. The client verbalizes the need to increase fluid 2. The client who has had an indwelling catheter
intake. for the past week.
3. The client explains how to perform pelvic floor 3. The client who is on a bladder-training
exercises. regimen.
4. The client attempts to retain the vaginal cone 4. The client who had a catheter removed this
in place the entire day. morning and is being discharged.
4. Which intervention should the nurse implement 9. The nurse is caring for an elderly client who
first for the client who has had an incontinent has an indwelling catheter. Which data warrant
episode? further investigation?
1. Palpate the client’s bladder to assess for urinary 1. The client’s temperature is 98°F.
retention. 2. The client has become confused and irritable.
2. Obtain a bedside commode for the client. 3. The client’s urine is clear and light yellow.
3. Assist the client with changing the wet clothes. 4. The client feels the need to urinate.
4. Request the UAP to change the client’s linens.
10. The nurse is observing the UAP providing direct
5. The elderly client recovering from a care to a client with an indwelling catheter.
prostatectomy has been experiencing stress Which data warrant immediate intervention by
incontinence. Which independent nursing the nurse?
intervention should the nurse discuss with 1. The UAP secures the tubing to the client’s leg
the client? with tape.
1. Establish a set voiding frequency of every 2. The UAP provides catheter care with the
two (2) hours while awake. client’s bath.
2. Encourage a family member to assist the client 3. The UAP puts the collection bag on the
to the bathroom to void. client’s bed.
3. Apply a transurethral electrical stimulator 4. The UAP cares for the catheter after washing
to relieve symptoms of urinary urgency. the hands.
4. Discuss the use of a “bladder drill,” including a
timed voiding schedule.

09_Colgrove_Ch09.indd 389 01/06/16 5:45 PM


390 M ed -S urg S uccess

11. Which intervention should the nurse implement 16. Which intervention should the nurse include
when caring for the client with a nephrostomy when assessing the client for urinary retention?
tube? Select all that apply.
1. Change the dressing only if soiled by urine. 1. Inquire if the client has the sensation of
2. Clean the end of the connecting tubing with fullness.
Betadine. 2. Percuss the suprapubic region for a dull sound.
3. Clean the drainage system every day with 3. Scan the bladder with the ultrasound scanner.
bleach and water. 4. Palpate from the umbilicus to the suprapubic
4. Assess the tube for kinks to prevent area.
obstruction. 5. Auscultate the two (2) lower abdominal
quadrants.
12. The client is 12 hours postoperative renal
surgery. Which data warrant immediate 17. The nurse is discussing how to prioritize care
intervention by the nurse? with the UAP. Which client should the nurse
1. The abdomen is soft, nontender, and rounded. instruct the UAP to see first?
2. Pain is not felt with dorsal flexion of the foot. 1. The immobile client who needs sequential
3. The urine output is 60 mL for the past compression devices removed.
two (2) hours. 2. The elderly woman who needs assistance
4. The client’s trough vancomycin level is ambulating to the bathroom.
24 mcg/mL. 3. The surgical client who needs help changing
the gown after bathing.
13. The nurse is teaching the female client
4. The male client who needs the intravenous
diagnosed with tuberculosis of the urinary
catheter discontinued.
tract prior to discharge. Which information
should the nurse include specific to this 18. The nurse is caring for the client recovering
diagnosis? from a percutaneous renal biopsy. Which data
1. Instruct the client to take the medication with indicate the client is complying with client
food. teaching?
2. Explain condoms should be used during 1. The client is lying flat in the supine position.
treatment. 2. The client continues oral fluids restriction
3. Discuss the need for follow-up chest x-rays. while on bedrest.
4. Encourage a well-balanced diet and fluid 3. The client uses the bedside commode to urinate.
intake. 4. The client refuses to ask for any pain
medication.
14. The nurse is assessing a client diagnosed with
urethral strictures. Which data support the 19. Which intervention should the nurse implement
diagnosis? for the client who has had an ileal conduit?
1. Complaints of frequency and urgency. 1. Pouch the stoma with a one (1)-inch margin
2. Clear yellow drainage from the urethra. around the stoma.
3. Complaints of burning during urination. 2. Refer the client to the United Ostomy
4. A diminished force and stream during voiding. Association for discharge teaching.
3. Report to the health-care provider any
15. The nurse is providing discharge teaching to the
decrease in urinary output.
client diagnosed with polycystic kidney disease.
4. Monitor the stoma for signs and symptoms of
Which statement made by the client indicates
infection every shift.
the teaching has been effective?
1. “I need to avoid any activity causing a risk for 20. The nurse is preparing the plan of care for
injury to my kidney.” a client with fluid volume deficit. Which
2. “I should avoid taking medications for high interventions should the nurse include in the
blood pressure.” plan of care? Select all that apply.
3. “When I urinate there may be blood streaks 1. Monitor vital signs every two (2) hours until
in my urine.” stable.
4. “I may have occasional burning when I 2. Measure the client’s oral intake and urinary
urinate with this disease.” output daily.
3. Administer mouth care when bathing the client.
4. Weigh the client weekly in the same clothing
at the same time.
5. Assess skin turgor and mucous membranes
every shift.

09_Colgrove_Ch09.indd 390 01/06/16 5:45 PM


C hapter 9  G enitourinary D isorders 391

21. Which outcome should the nurse identify for 26. The client diagnosed with a fluid and electrolyte
the client diagnosed with fluid volume excess? disturbance in the emergency department
1. The client will void a minimum of 30 mL is exhibiting peaked T waves on the STAT
per hour. electrocardiogram. Which interventions should
2. The client will have elastic skin turgor. the nurse implement? List in order of priority.
3. The client will have no adventitious breath 1. Assess the client for leg and muscle cramps.
sounds. 2. Check the serum potassium level.
4. The client will have a serum creatinine of 3. Notify the health-care provider.
1.4 mg/dL. 4. Arrange for a transfer to the telemetry floor.
5. Administer Kayexalate, a cation resin.
22. The nurse is caring for a client diagnosed
with rule-out nephrotic syndrome. Which 27. The clinic nurse is reviewing information
intervention should be included in the plan submitted by the UAP that states the presence
of care? of pediculosis pubis. Which area of the client’s
1. Monitor the urine for bright-red bleeding. body should the nurse assess?
2. Evaluate the calorie count of the 500-mg
protein diet. A
3. Assess the client’s sacrum for dependent
edema.
4. Monitor for a high serum albumin level.
23. The nurse is preparing a teaching care plan for
B
the client diagnosed with nephrotic syndrome.
Which intervention should the nurse include?
1. Stop steroids if a moon face develops.
2. Provide teaching for taking diuretics.
3. Increase the intake of dietary sodium.
4. Report a decrease in daily weight.
24. Which intervention is most important for the
nurse to implement for the client with a left
nephrectomy?
1. Assess the intravenous fluids for rate and C
volume.
2. Change surgical dressing every day at the
same time.
3. Monitor the client’s PT/PTT/INR level daily.
4. Monitor the percentage of each meal eaten.
25. The nurse is preparing the discharge teaching
plan for the male client with a left-sided
nephrectomy. Which statement indicates the
teaching is effective?
1. “I can’t wait to start back to work next week,
I really need the money.”
2. “I will take my temperature and if it is above D
101 I will call my doctor.”
3. “I am glad I won’t have to keep track of how 1. A
much I urinate in the day.” 2. B
4. “I am happy I will be able eat what I usually 3. C
eat, I don’t like this food.” 4. D

09_Colgrove_Ch09.indd 391 01/06/16 5:45 PM


GENITOURINARY DISORDERS COMPREHENSIVE
EXAMINATION ANSWERS AND RATIONALES
1. 1. Clients who have urinary incontinence are Content – Medical: Integrated Nursing Process –
often embarrassed, so it is the responsibil- Assessment: Client Needs – Safe Effective Care
ity of the nurse to approach this subject Environment, Management of Care: Cognitive
with respect and consideration. Level – Analysis: Concept – Urinary Elimination.
2. The temperature of the room is not pertinent 4. 1. The nurse may assess the bladder to deter-
to the nurse interviewing the client. mine if urine is being retained but not prior to
3. The nurse should not assume elderly clients changing the client’s wet clothes.
have hearing difficulty. If the client is “hard of 2. Having a bedside commode may or may not be
hearing,” the nurse should speak clearly and helpful to the client who is incontinent.
concisely but should not shout. 3. The nurse should first assist the client
4. The lighting of the room is not pertinent to in getting out of the wet clothes prior to
interviewing the client about incontinence. any other action. Wet clothes are embar-
Content – Medical: Integrated Nursing Process – rassing to the client and can lead to skin
Implementation: Client Needs – Safe Effective Care breakdown.
­Environment, Management of Care: Cognitive 4. The client’s linens need to be changed but not
Level – Application: Concept – Urinary Elimination. prior to changing the client’s wet clothes.
2. 1. Prune juice is given to prevent constipation Content – Medical: Integrated Nursing Process –
but should be taken daily, not weekly. Implementation: Client Needs – Safe Effective Care
2. Clients experiencing incontinence should Environment, Management of Care: Cognitive
eat a high-fiber diet to avoid constipation, Level – Application: Concept – Urinary Elimination.
which increases pressure on the bladder, 5. 1. Timed voiding is more helpful with neuro­
which may increase incontinence. genic disorders, such as those related to
3. Bladder training is used to assist with urinary diabetes.
incontinence by voiding every two (2) to three 2. A prompted voiding is useful with a client who
(3) hours, not every six (6) hours. does not have the cognitive ability to recognize
4. Cotton underwear may help decrease urinary the need.
tract infections but does not affect urinary 3. The use of transvaginal or transurethral
incontinence. electrical stimulation to stimulate the pelvic
Content – Medical: Integrated Nursing Process – floor muscles to contract is a collaborative
Implementation: Client Needs – Safe Effective Care intervention.
­Environment, Management of Care: Cognitive 4. Use of the bladder training drill is help-
Level – Application: Concept – Urinary Elimination. ful in stress incontinence. The client is
3. 1. Scheduled voiding allows the client to void instructed to void at scheduled intervals.
every two (2) to three (3) hours apart, and After consistently being dry, the interval
when the client has remained consistently is increased by 15 minutes until the client
dry, the interval is increased by about reaches an acceptable interval.
15 minutes. Content – Medical: Integrated Nursing Process –
2. The fluid intake should not be increased but Planning: Client Needs – Physiological Integrity, ­Physiological
should be adequate to prevent dehydration. Adaptation: Cognitive Level – Synthesis: Concept – Urinary
The majority of the fluid should be drunk Elimination.
early in the day to prevent nocturia. 6. 1. In the neurogenic flaccid bladder, the client
3. Pelvic floor (Kegel) exercises should be has lost the ability to recognize the need to
­performed two (2) to three (3) times daily with void; therefore, this is not a realistic expected
repetitions of 10 to 30 each session, but this outcome.
is recommended for stress incontinence, not 2. The treatment goal of the flaccid bladder is
­urinary incontinence. to prevent overdistention.
4. A series of vaginal weights can be used to in- 3. The sensation has been lost as a result of a
crease the muscle tone. The time is usually lower motor neuron problem; therefore, there
only 15 minutes, not all day.

392

09_Colgrove_Ch09.indd 392 01/06/16 5:45 PM


C hapter 9  G enitourinary D isorders 393

is no sensation to maintain and no discomfort, 9. 1. This temperature, 98°F, is within normal


so this is not a realistic goal. limits and does not require further
4. The client does not have to assess the bladder; investigation.
this is a nursing intervention. 2. When an elderly client’s mental status
Content – Medical: Integrated Nursing Process – changes to confused and irritable, the
Diagnosis: Client Needs – Safe Effective Care nurse should seek the etiology, which
Environment, Management of Care: Cognitive may be a UTI secondary to an indwelling
Level – Analysis: Concept – Urinary Elimination. catheter. Elderly clients often do not pres-
7. 1. To determine if the client has had a catheter ent with classic signs and symptoms of
in place previously assists with teaching and infection.
­alleviating anxiety, but it is not the most 3. The client’s urine should be clear and light
important intervention. yellow; therefore, this does not warrant fur-
2. Assessing the level of anxiety is helpful in ther investigation.
assisting the client but does not endanger the 4. The client often has a feeling of the need
client; therefore, it is not the most important to void when having an indwelling catheter,
intervention. but this comment does not warrant further
3. The nurse should always assess for allergies intervention.
to latex prior to inserting a latex catheter Content – Medical: Integrated Nursing Process –
or using a drainage system because, if the Assessment: Client Needs – Physiological Integrity,
client is allergic to latex, use of it could Reduction of Risk Potential: Cognitive Level –
cause a life-threatening reaction. Analysis: Concept – Urinary Elimination/Infection.
4. There are many reasons the client is catheter- 10. 1. The client’s catheter should be secured on the
ized regardless of the sensation and ability to leg to prevent manipulation, which increases
void. The nurse should not assess this until the the risk for a urinary tract infection. This
catheter is removed. action does not require intervention.
Content – Medical: Integrated Nursing Process – 2. The client with an indwelling catheter should
Implementation: Client Needs – Safe Effective Care receive catheter care with the bath and as
­Environment, Management of Care: Cognitive needed.
Level – Analysis: Concept – Urinary Elimination. 3. The drainage bag should be kept below
8. 1. This client requires the most skill and the level of the bladder to prevent reflux
knowledge because this client has the of urine into the renal system; it should
greatest potential for an infection; there- not be placed on the bed.
fore, the client should not be assigned 4. Hand hygiene is important before and after
to a UAP. handling any portion of the drainage system.
2. The UAP can care for a client with an indwell- Content – Medical: Integrated Nursing Process –
ing catheter because adherence to Standard Assessment: Client Needs – Safe Effective Care
Precautions is the only requirement for safe Environment, Management of Care: Cognitive
client care. Level – Analysis: Concept – Urinary Elimination.
3. The UAP cannot teach bladder training but 11. 1. The dressing should be routinely changed
can implement the strategies for the client on as often as daily or weekly.
a bladder-training program, such as taking the 2. When connecting the tubing to the drainage
client to the bathroom at scheduled times. bag, both ends should be cleaned with alcohol,
4. The UAP can care for this client because not Betadine.
noting if the client voided after removal of 3. The drainage system can be cleaned daily
the catheter is within the realm of the UAP’s with soap and water.
ability. 4. The nephrostomy tube should never
Content – Nursing Management: Integrated be clamped or have kinks because an
Nursing Process – Planning: Client Needs – Safe Effective obstruction can cause pyelonephritis.
Care Environment, Management of Care: Cognitive Content – Medical: Integrated Nursing Process –
Level – Synthesis: Concept – Management. Implementation: Client Needs – Safe Effective Care
Environment, Management of Care: Cognitive
Level – Application: Concept – Urinary Elimination.

09_Colgrove_Ch09.indd 393 01/06/16 5:45 PM


394 M ed -S urg S uccess

12. 1. The client who has renal surgery is at risk for occupations with risks for trauma should
paralytic ileus from the manipulation of the be avoided.
colon. A soft, rounded, and nontender abdo- 2. Antihypertensive medications should be taken
men does not require intervention. to protect the kidneys from further damage.
2. Pain felt with dorsal flexion of the foot in- 3. Blood should always be reported to the
dicates a deep vein thrombosis; therefore, a health-care provider, and hematuria is a sign
client who is asymptomatic does not require of polycystic kidney disease. Further evalua-
intervention. tion is needed.
3. The minimum of 30 mL/hr does not require 4. Burning during urination should be treated
intervention by the nurse. to prevent further damage to the kidneys and
4. The client who has restricted kidney renal system.
­function from surgery should be moni- Content – Medical: Integrated Nursing Process –
tored for damage as a result of the use of Evaluation: Client Needs – Physiological Integrity,
aminoglycoside antibiotics, such as vanco- ­Physiological Adaptation: Cognitive Level – Synthesis:
mycin, which are nephrotoxic. This level is Concept – Urinary Elimination.
high and warrants notifying the HCP.
16. 1. The nurse needs to assess the client’s
Content – Surgical: Integrated Nursing Process – ­sensation of needing to void or feeling of
Assessment: Client Needs – Safe Effective Care fullness.
Environment, Management of Care: Cognitive
2. A dull sound heard when percussing
Level – Synthesis: Concept – Perioperative.
the bladder indicates it is filled with urine.
13. 1. Antitubercular medications (rifampin and 3. A portable bladder scan is used to assess
INH) should be taken one (1) hour before or for the presence of urine, rather than us-
two (2) two hours after a meal. ing a straight catheter.
2. Clients who have been diagnosed with 4. A distended bladder can be palpated.
­tuberculosis of the renal tract should 5. Auscultation cannot assess the client’s bladder.
use condoms to prevent transmission Content – Medical: Integrated Nursing Process –
of the mycobacterium. If the infection is Assessment: Client Needs – Physiological Integrity,
located in the penis or urethra, abstaining Reduction of Risk Potential: Cognitive Level –
from sexual activity is recommended. Analysis: Concept – Urinary Elimination.
3. Follow-up chest x-rays are important for the
17. 1. The client who needs a sequential compres-
client with tuberculosis of the lung.
sion device removed is not urgent.
4. Maintaining a well-balanced diet and fluid
2. The elderly woman may have age-related
intake is important for recovery from any
changes (decreased bladder capacity, weak-
­illness and for a healthy lifestyle, but it is
ened urinary sphincter, shortened urethra)
not specifically for this diagnosis.
causing urinary urgency or incontinence.
Content – Medical: Integrated Nursing Process – The elderly client is at risk for falling
Planning: Client Needs – Physiological Integrity, while attempting to get to the bathroom,
­Physiological Adaptation: Cognitive Level – Synthesis:
so this client should be seen first.
Concept – Urinary Elimination.
3. Changing a gown does not affect the client’s
14. 1. Frequency and urgency are signs and safety.
symptoms of a urinary tract infection. 4. In many facilities this task cannot be
2. Clear yellow urethral drainage is urine. ­delegated, but the client’s safety is not
3. A complaint of burning during voiding is affected if the IV catheter is not immediately
a sign/symptom of urinary tract infection. discontinued.
4. The client with urethral strictures will Content – Nursing Management: Integrated
­report a decrease in force and stream Nursing Process – Planning: Client Needs – Safe
­during voiding. The stricture is treated by ­Effective Care Environment, Management of Care:
dilation using small filiform bougies. Cognitive Level – Synthesis: Concept – Management.
Content – Medical: Integrated Nursing Process – 18. 1. The client needs to lie flat on the back
Assessment: Client Needs – Physiological Integrity, to apply pressure to prevent bleeding.
Reduction of Risk Potential: Cognitive Level –
2. The client has oral intake withheld prior
Analysis: Concept – Urinary Elimination.
to the biopsy, not after the client is awake.
15. 1. Polycystic kidney disease poses an 3. The client must lie flat on the back, so us-
­increased risk for rupture of the kid- ing a bedside commode is not maintaining
ney, and, therefore, sports activities or bedrest.

09_Colgrove_Ch09.indd 394 01/06/16 5:45 PM


C hapter 9  G enitourinary D isorders 395

4. The client should request pain medication 3. The client with fluid volume excess has
if the client is in pain. This indicates the cli- too much fluid. Excess fluid is reflected
ent is not compliant with the client teaching. by adventitious breath sounds. Therefore,
Content – Surgical: Integrated Nursing Process – an expected outcome is to have no excess
Assessment: Client Needs – Safe Effective Care fluid, as evidenced by normal, clear breath
Environment, Management of Care: Cognitive sounds.
Level – Analysis: Concept – Urinary Elimination. 4. The creatinine is elevated in a client who is
dehydrated. The normal male should have a
19. 1. The nurse should maintain the drainage bag
creatinine of 0.6 to 1.2 mg/dL, and a female
with a one-eighth (1/8)-inch border around
client’s normal creatinine is between 0.5 and
the stoma.
1.1 mg/dL.
2. The United Ostomy Association is an ex-
cellent referral for information but not for Content – Medical: Integrated Nursing Process –
discharge teaching. The nurse retains the Assessment: Client Needs – Physiological Integrity,
Reduction of Risk Potential: Cognitive Level –
responsibility to teach information the client
Analysis: Concept – Fluid and Electrolyte Balance.
needs to know prior to discharge.
3. The output should be monitored to 22. 1. Hematuria is not a symptom of nephrotic
detect a decreased amount, indicating syndrome.
an obstruction from edema or ureteral 2. A calorie count may be helpful in the treat-
stenosis. Any decrease should be reported ment of this client, but a calorie count moni-
to the health-care provider. tors what the name implies—calories. The
4. The stoma should be monitored much more dietitian can calculate the amount of protein
frequently than once a shift. the client consumes, but this is a protein
Content – Surgical: Integrated Nursing Process – count.
Implementation: Client Needs – Safe Effective Care 3. The classic sign/symptom of nephrotic
Environment, Management of Care: Cognitive Level – syndrome is dependent edema located
Analysis: Concept – Urinary Elimination. on the client’s sacrum and ankles.
4. A low serum albumin level is expected for a
20. 1. Vital signs should be monitored every two
client diagnosed with nephrotic syndrome.
(2) hours until stable and more
frequently if the client is unstable. Content – Medical: Integrated Nursing
2. Intake and output should be monitored more Process– Diagnosis: Client Needs – Safe Effective Care
­Environment, Management of Care: Cognitive Level –
frequently than every 24 hours. Depending
Analysis: Concept – Fluid and Electrolyte Balance.
on the client’s condition, frequency may vary
from every hour to every four (4) hours. 23. 1. Steroid therapy should not be stopped
3. Mouth care should be given as often as abruptly if signs of toxicity occur, such as
needed. A minimum of care should be every moon face, because it may result in adrenal
eight (8) hours, not once a day when bathing insufficiency.
the client. 2. Treatment includes diuretics to eliminate
4. The client should be weighed daily, not dependent edema, usually in the ankles
weekly, at the same time wearing the same and sacrum. Medication teaching is an
clothing to ensure the reliability of this ­appropriate intervention.
indicator. 3. Sodium is restricted to prevent fluid retention.
5. Skin turgor and mucous membranes 4. A decrease in weight is expected if a diuretic
should be assessed every shift or more is administered; this indicates the medication
often depending on the client’s condition. is effective.
Content – Medical: Integrated Nursing Content – Medical: Integrated Nursing Process –
Process – Diagnosis: Client Needs – Safe Effective Care Planning: Client Needs – Physiological Integrity,
Environment, Management of Care: Cognitive Level – ­Physiological Adaptation: Cognitive Level – Synthesis:
Analysis: Concept – Fluid and Electrolyte Balance. Concept – Fluid and Electrolyte Balance.

21. 1. Voiding a minimum of 30 mL of urine each 24. 1. Assessing the rate and volume of intrave-
hour is appropriate for a client with fluid nous fluid is the most important interven-
­volume deficit. tion for the client who has one (1) kidney
2. Elastic skin turgor indicates the client has because an overload of fluids can result in
­adequate fluid volume status. This is an ex- pulmonary edema.
pected output for the client with fluid 2. A daily dressing change can be performed at
volume deficit. any time and is not the priority intervention.

09_Colgrove_Ch09.indd 395 01/06/16 5:45 PM


396 M ed -S urg S uccess

3. A client who has had surgery should not be 3. Hyperkalemia is a life-threatening situ-
receiving any type of anticoagulant therapy, ation because of the risk of cardiac dys-
so the nurse should not have to monitor this rhythmias; therefore, the nurse should
laboratory data. notify the health-care provider.
4. The nurse assesses the amount of food eaten, 5. Kayexalate is a medication that will help
but it is not the most important intervention. remove potassium through the gastroin-
Content – Surgical: Integrated Nursing Process – testinal system and should be adminis-
Implementation: Client Needs – Safe Effective Care tered to decrease the potassium level.
Environment, Management of Care: Cognitive Level – 4. The client should be monitored continu-
Application: Concept – Fluid and Electrolyte Balance. ously for cardiac dysrhythmias, so a trans-
fer to the telemetry unit is warranted.
25. 1. The client recovering from a nephrectomy
needs to refrain from strenuous or heavy Content – Medical: Integrated Nursing Process –
activities, and normal activities should not be Implementation: Client Needs – Safe Effective Care
Environment, Management of Care: Cognitive Level –
resumed until the client is given permission
Analysis: Concept – Fluid and Electrolyte Balance.
by the surgeon.
2. The client or family needs to contact the 21. 1. Pediculosis capitus is found on the head.
surgeon if the client develops chills, flank Head lice are found commonly in children in
pain, decreased urinary output, or fever. elementary schools.
3. The client needs to be informed of how to 2. Scabies are found on the forearms, and around
monitor the urinary output and which param- the waist and elastic lines of underwear.
eters should be reported to the surgeon. 3. The pubic lice are found in the pubic area
4. The client needs to follow any dietary or fluid and are commonly transmitted during
restriction the surgeon prescribes. sexual intercourse.
Content – Medical: Integrated Nursing Process – 4. The feet are not an area normally associated
Evaluation: Client Needs – Physiological Integrity, with body lice.
Physiological Adaptation: Cognitive Level – Synthesis: Content – Medical: Integrated Nursing Process –
Concept – Urinary Elimination/Infection. Assessment: Client Needs – Physiological Integrity,
Physiological Adaptation: Cognitive Level – Analysis:
26. In order of priority: 1, 2, 3, 5, 4.
Concept – Sexuality.
1. The nurse should assess to determine if
the client is symptomatic of hyperkalemia.
2. A peaked T wave is indicative of hyperka-
lemia; therefore, the nurse should obtain a
potassium level.

09_Colgrove_Ch09.indd 396 01/06/16 5:45 PM


Reproductive
Disorders
The aim of education is the knowledge not of fact, but of values.
10
—Dean William R. Inge

The organs of the male and female reproductive systems are subject to many disorders/diseases. Some are he-
reditary, some are related to the endocrine system and may involve an underproduction or overproduction of
hormones, and still others may be the result of infections or neoplastic growths. Whatever the etiology, the nurse
must be well informed about all the possible disorders/diseases and how to monitor and treat them, both indepen-
dently when allowed or under the direction of an HCP.

KEYWORDS     ABBREVIATIONS
Aneuploid Absolute neutrophil count (ANC)
Brachytherapy Acute respiratory distress syndrome (ARDS)
Chancre Anterior and posterior repair (A & P repair)
Colporrhaphy Blood pressure (BP)
DNA ploidy Breast self-examination (BSE)
Dysmenorrhea Deep vein thrombosis (DVT)
Dyspareunia Deoxyribonucleic acid (DNA)
Nulliparity Digital rectal examination (DRE)
Pessary Health-care provider (HCP)
Phimosis Hormone replacement therapy (HRT)
Human chorionic gonadotropin (hCG)
Human immunodeficiency virus (HIV)
Human papillomavirus (HPV)
Incision and drainage (I & D)
International normalized ratio (INR)
Intravenous (IV)
Intravenous piggyback (IVPB)
Low malignancy potential (LMP)
Luteinizing hormone–releasing hormone (LHRH)
Pelvic inflammatory disease (PID)
Prostate-specific antigen (PSA)
Prothrombin time (PT)
Rule out (R/O)
Sexually transmitted disease (STD)
Unlicensed assistive personnel (UAP)
Urinary tract infection (UTI)
White blood cells (WBCs)

397

10_Colgrove_Ch10.indd 397 01/06/16 5:45 PM


398 M ed -S urg S uccess

PRACTICE QUESTIONS
Breast Disorders 5. The client had a mastectomy for cancer of the
breast and asks the nurse about a TRAM flap
1. The client frequently finds lumps in her breasts, procedure. Which information should the nurse
especially around her menstrual period. Which explain to the client?
information should the nurse teach the client 1. The surgeon will insert a saline-filled sac under
regarding breast self-care? the skin to simulate a breast.
1. This is a benign process, which does not 2. The surgeon will pull the client’s own tissue
require follow-up. under the skin to create a breast.
2. The client should eliminate chocolate and 3. The surgeon will use tissue from inside the
caffeine from the diet. mouth to make a nipple.
3. The client should practice breast self- 4. The surgeon can make the breast any size the
examination monthly. client wants the breast to be.
4. This is the way breast cancer begins and the 6. The nurse is teaching a class on breast health to a
client needs surgery. group of ladies at a senior citizen’s center. Which
2. The client is diagnosed with breast cancer and risk factor is the most important to emphasize to
is considering whether to have a lumpectomy this group?
or a more invasive procedure, a modified radical 1. The clients should find out about their family
mastectomy. Which information should the nurse history of breast cancer.
discuss with the client? 2. Men at this age can get breast cancer also and
1. Ask if the client is afraid of having general should be screened.
anesthesia. 3. Monthly breast self-examination is the key to
2. Determine how the client feels about radiation early detection.
and chemotherapy. 4. The older a woman gets, the greater the chance
3. Tell the client she will need reconstruction with of developing breast cancer.
either procedure. 7. The client who is scheduled to have a breast
4. Find out if the client has any history of breast biopsy with sentinel node dissection states,
cancer in her family. “I don’t understand. What does a sentinel node
3. The client has undergone a wedge resection biopsy do?” Which scientific rationale should the
for cancer of the left breast. Which discharge nurse use to base the response?
instruction should the nurse teach? 1. A dye is injected into the tumor and traced to
1. Don’t lift more than five (5) pounds with the determine spread of cells.
left hand until released by the HCP. 2. The surgeon removes the nodes that drain the
2. The cancer has been totally removed and no diseased portion of the breast.
follow-up therapy will be required. 3. The nodes felt manually will be removed and
3. The client should empty the Hemovac drain sent to pathology.
about every 12 hours. 4. A visual inspection of the lymph nodes will be
4. The client should arrange an appointment with made while the client is sleeping.
a plastic surgeon for reconstruction. 8. The client who is four (4) months pregnant
4. Which recommendation is the American Cancer finds a lump in her breast and the biopsy is
Society’s (ACS) 2015 guideline for the early positive for Stage II cancer of the breast. Which
detection of breast cancer? treatment should the nurse anticipate the HCP
1. Beginning at age 18, have a biannual clinical recommending to the client?
breast examination by an HCP. 1. A lumpectomy to be performed after the baby
2. Beginning at age 30, perform monthly breast is born.
self-exams. 2. A modified radical mastectomy.
3. At age 45 through 54, receive a yearly 3. Radiation therapy to the chest wall only.
mammogram. 4. Chemotherapy only until the baby is born.
4. Beginning at age 50, have a breast sonogram
every five (5) years.

10_Colgrove_Ch10.indd 398 01/06/16 5:45 PM


C hapter 10  R eproductive D isorders 399

9. The client who had a right modified radical 14. Which intervention should the nurse include
mastectomy four (4) years before is being when teaching the client who is having an
admitted for a cardiac work-up for chest pain. anterior colporrhaphy to repair a cystocele?
Which intervention is most important for the 1. Discuss the need to perform perineal care
nurse to implement? every four (4) hours.
1. Determine when the client had chemotherapy 2. Discuss the care of an indwelling catheter for
last. at least one (1) month.
2. Ask the client if she received Adriamycin, an 3. Instruct the client how to care for the pessary
antineoplastic agent. inserted in surgery.
3. Post a message at the HOB for staff not to use 4. Teach the client how to perform Kegel
the right arm for venipunctures or BPs. exercises.
4. Examine the chest wall for cancer sites.
15. The nurse is assessing the client diagnosed with
10. The client is being discharged after a left wedge a rectocele. Which signs and symptoms should
resection. Which discharge instructions should the nurse expect? Select all that apply.
the nurse include? Select all that apply. 1. Rectal pressure.
1. Notify the HCP of a temperature of 100°F. 2. Flatus.
2. Carry large purses and bundles with the right 3. Fecal incontinence.
hand. 4. Constipation.
3. Do not go to church or anywhere with 5. Urinary frequency.
crowds.
16. What intervention should the nurse implement
4. Try to keep the arm as still as possible until
for a client diagnosed with a rectocele?
seen by the HCP.
1. Limit oral intake to decrease voiding.
5. Have a mammogram of the right and left
2. Encourage a low-residue diet.
breasts yearly.
3. Administer a stool softener daily.
11. The client who has had a mastectomy tells the 4. Arrange for the client to take sitz baths.
nurse, “My husband will leave me now since
17. Which statement indicates further instruction
I am not a whole woman anymore.” Which
is needed for the client with a cystocele?
response by the nurse is most therapeutic?
1. “I need to have a sonogram to diagnose this
1. “You’re afraid your husband will not find you
problem.”
sexually appealing?”
2. “I need to practice Kegel exercises to help
2. “Your husband should be grateful you will be
strengthen my muscles.”
able to live and be with him.”
3. “I lose my urine when I sneeze because of my
3. “Maybe your husband would like to attend a
cystocele.”
support group for spouses.”
4. “I can never have sexual intercourse again.”
4. “You don’t know that is true. You need to give
him a chance.” 18. Which specific complication should the nurse
assess for in the client with a uterine prolapse
12. The client has been diagnosed with cancer of the
recovering from an anterior and posterior
breast. Which referral is most important for the
repair?
nurse to make?
1. Orthostatic hypotension.
1. The hospital social worker.
2. Atelectasis.
2. CanSurmount.
3. Allen sign.
3. Reach to Recovery.
4. Deep vein thrombosis.
4. I CanCope.
19. Which information should the nurse include in
the discharge teaching for the client recovering
Pelvic Floor Relaxation Disorders from an abdominal hysterectomy?
1. The client should report any vaginal bleeding
13. Which question is most important for the or cramping to the surgeon.
nurse to ask the client with a cystocele who is 2. The client should start a vigorous exercise
scheduled to have a pessary inserted? routine to restore her muscle tone.
1. “Do you know if you are allergic to latex?” 3. The client should continue sitting in the
2. “When did you start having incontinence?” bedside chair at least six (6) hours daily.
3. “When was your last bowel movement?” 4. The client should soak in a warm tub bath
4. “Are you experiencing any pelvic pressure?” each night for one (1) hour.

10_Colgrove_Ch10.indd 399 01/06/16 5:45 PM


400 M ed -S urg S uccess

20. Which nursing task could be delegated to the nurse prepare the client to undergo to determine
unlicensed assistive personnel (UAP) for the the diagnosis?
client who had a total vaginal hysterectomy? 1. A bimanual vaginal exam.
1. Observe the color and amount of drainage on 2. A pregnancy test.
the client’s perineal pad. 3. An exploratory laparoscopy.
2. Maintain a current intake and output for the 4. An ovarian biopsy.
client each shift.
26. The client in the gynecology clinic asks the
3. Provide the client with a plan of
nurse, “What are the risk factors for developing
pharmacological pain management.
cancer of the cervix?” Which statement is the
4. Prepare the client for her discharge scheduled
nurse’s best response?
for the next day.
1. “The earlier the age of sexual activity and the
21. The nurse is formulating a care plan for a client more partners, the greater the risk.”
post–abdominal hysterectomy. Which nursing 2. “Eating fast foods high in fat and taking birth
diagnosis is appropriate for the client who has control pills are risk factors.”
developed a complication? 3. “A Chlamydia trachomatis infection can cause
1. Potential for urinary retention. cancer of the cervix.”
2. Potential for nerve damage. 4. “Having yearly Pap smears will protect you
3. Potential for intestinal obstruction. from developing cancer.”
4. Potential for fluid imbalance.
27. The nurse is admitting a client diagnosed with
22. The nurse is teaching the client diagnosed with Stage Ia cancer of the cervix to an outpatient
uterine prolapse. Which information should the surgery center for a conization. Which data
nurse include in the discussion? would the client most likely report?
1. Increase fluids and daily exercise to prevent 1. Diffuse watery discharge.
constipation. 2. No symptoms.
2. Explain there is only one acceptable treatment 3. Dyspareunia.
for uterine prolapse. 4. Intense itching.
3. Instruct the client to visually check the uterine
28. The client diagnosed with cancer of the uterus
prolapse daily.
is scheduled to have radiation brachytherapy.
4. Discuss limiting coughing and lifting heavy
Which precautions should the nurse implement?
objects.
Select all that apply.
23. The nurse is preparing the client for an insertion 1. Place the client in a private room.
of a pessary. Which information should the 2. Wear a dosimeter when entering the room.
nurse teach the client? 3. Encourage visitors to come and stay with
1. The pessary does not need to be changed. the client.
2. The client should clean the pessary routinely. 4. Plan to spend extended time with the client.
3. The pessary must be inserted in surgery. 5. Notify the nuclear medicine technician.
4. Estrogen cream is necessary for effective use
29. The postmenopausal client reveals it has
of a pessary.
been several years since her last gynecological
24. An elderly woman is diagnosed with pelvic examination and states, “Oh, I don’t need
relaxation disorder secondary to age-related exams anymore. I am beyond having children.”
changes. Which medication should the nurse Which statement should be the nurse’s
expect to administer? response?
1. Estrogen, a hormone. 1. “As long as you are not sexually active, you
2. Cervidil, a cervical ripening agent. don’t have to worry.”
3. Progesterone, a hormone. 2. “You should be taking hormone replacement
4. Pitocin, an oxytocic agent. therapy now.”
3. “You are beyond bearing children. How does
Uterine Disorders that make you feel?”
4. “There are situations other than pregnancy
25. The nurse is caring for a 30-year-old that should be checked.”
nulliparous client who is complaining of severe
dysmenorrhea. Which diagnostic test should the

10_Colgrove_Ch10.indd 400 01/06/16 5:45 PM


C hapter 10  R eproductive D isorders 401

30. The client has had a total abdominal 35. The nurse and an unlicensed assistive personnel
hysterectomy for cancer of the uterus. Which (UAP) are caring for clients on a gynecology
discharge instruction should the nurse teach? surgery floor. Which intervention cannot be
1. The client should take HRT every day to delegated to the UAP?
prevent bone loss. 1. Empty the indwelling catheter on the three
2. The client should practice pelvic rest until (3)-hour postoperative client.
seen by the HCP. 2. Assist the client who is two (2) days post-
3. The client can drive a car as soon as she is hysterectomy to the bathroom.
discharged from the hospital. 3. Monitor the peri-pad count on a client
4. The client should expect some bleeding after diagnosed with fibroid tumors.
this procedure. 4. Encourage the client who is refusing to get
out of bed to walk in the hall.
31. The client diagnosed with uterine cancer is
complaining of lower back pain and unilateral 36. The nurse is caring for a client diagnosed with
leg edema. Which statement best explains the uterine cancer who has been receiving systemic
scientific rationale for these signs/symptoms? therapy for six (6) months. Which intervention
1. This is expected pain for this type of cancer. should the nurse implement first?
2. This means the cancer has spread to other 1. Determine which antineoplastic medication
areas of the pelvis. the client has received.
3. The pain is a result of the treatment of uterine 2. Ask the client if she has had any problems
cancer. with mouth ulcers at home.
4. Radiation treatment always causes some type 3. Administer the biologic response modifier
of pain in the region. filgrastim (Neupogen).
4. Encourage the client to discuss feelings about
32. The client diagnosed with endometriosis
having cancer.
experiences pain rated a “5” on a 1-to-10 pain
scale during her menses. Which intervention
should the nurse teach the client? Ovarian Disorders
1. Teach the client to take a stool softener when
taking morphine, a narcotic. 37. The 24-year-old female client presents to the
2. Instruct the client to soak in a tepid bath for clinic with lower abdominal pain on the left
30 to 45 minutes when the pain occurs. side she rates as a “9” on a 1-to-10 scale. Which
3. Explain the need to take the nonsteroidal diagnostic procedure should the nurse prepare
anti-inflammatory drugs with food. the client for?
4. Discuss the possibility of a hysterectomy to 1. A computed tomography scan.
help relieve the pain. 2. A lumbar puncture.
3. An appendectomy.
33. The client is diagnosed with benign uterine
4. A pelvic sonogram.
fibroid tumors. Which question should the nurse
ask to determine if the client is experiencing a 38. The nurse is caring for a client newly diagnosed
complication? with Stage IV ovarian cancer. What is the
1. “How many periods have you missed?” scientific rationale for detecting the tumors at
2. “Do you get short of breath easily?” this stage?
3. “How many times have you been pregnant?” 1. The client’s ovaries lie deep within the pelvis
4. “Where is the location of the pain you are and early symptoms are vague.
having?” 2. The client has regular gynecological
examinations and this helps with detection.
34. The HCP has prescribed two (2) IV antibiotics
3. The client had a history of dysmenorrhea and
for the female client diagnosed with diabetes
benign ovarian cysts.
and pneumonia. Which order should the nurse
4. The client had a family history of breast
request from the HCP?
cancer and was being checked regularly.
1. Request written information on antibiotic-
caused vaginal infections.
2. Request yogurt to be served on the client’s
meal trays.
3. Request a change of one of the antibiotics to
an oral route.
4. Request L. acidophilus, a probiotic medication,
three (3) times a day.

10_Colgrove_Ch10.indd 401 01/06/16 5:45 PM


402 M ed -S urg S uccess

39. The female client presents to the gynecologist’s 43. The client diagnosed with ovarian cancer has
office for the fifth time with an ovarian cyst and had eight (8) courses of chemotherapy. Which
is scheduled for an exploratory laparoscopy. The laboratory data warrant immediate intervention
client asks the nurse, “Why do I need to have by the nurse?
another surgery? The other cysts have all been 1. Absolute neutrophil count of 3,500.
benign.” Which statement is the nurse’s best 2. Platelet count of 150  103.
response? 3. Red blood cell count of 5  106.
1. “Because eventually the cysts will become 4. Urinalysis report of 100 WBCs.
cancerous.”
44. The client diagnosed with ovarian cancer is
2. “All abnormal findings in the ovary should be
prescribed radiation therapy for regional control
checked out.”
of the disease. Which statement indicates the
3. “The surgery will not be painful and you will
client requires further teaching?
have peace of mind.”
1. “I will not wash the marks off my abdomen.”
4. “Are you afraid of having surgery? Would you
2. “I will have a treatment every day for
like to talk about it?”
six (6) weeks.”
40. The client has had an exploratory laparotomy 3. “Nausea caused by radiation therapy cannot
to remove an ovarian tumor. The pathology be controlled.”
report classifies the tumor as a “low malignancy 4. “I need to drink a nutritional shake if I don’t
potential” (LMP) tumor. Which statement feel like eating.”
explains the scientific rationale for this
45. The female client has a mother who died from
pathology report?
ovarian cancer and a sister diagnosed with
1. The client does not have cancer but will need
ovarian cancer. Which recommendations should
adjuvant therapy.
the nurse make regarding early detection of
2. The client would have developed cancer if the
ovarian cancer?
tumor had not been removed.
1. The client should consider having a
3. These borderline tumors resemble ovarian
prophylactic bilateral oophorectomy.
cancer but have better outcomes.
2. The client should have a transvaginal
4. The client has a very poor prognosis and has
ultrasound and a CA-125 laboratory test every
less than six (6) months to live.
six (6) months.
41. The 50-year-old female client complains of 3. The client should have yearly magnetic
bloating and indigestion and tells the nurse resonance imaging (MRI) scans.
she has gained two (2) inches in her waist 4. The client should have a biannual
recently. Which question should the nurse ask gynecological examination with flexible
the client? sigmoidoscopy.
1. “What do you eat before you feel bloated?”
46. The client has had a total abdominal
2. “Have you had your ovaries removed?”
hysterectomy for cancer of the ovary. Which
3. “Are your stools darker in color lately?”
diet should the nurse discuss when providing
4. “Is the indigestion worse when you lie down?”
discharge instructions?
42. The nurse writes a problem of “anticipatory 1. A low-residue diet without seeds.
grieving” for a client diagnosed with ovarian 2. A low-sodium, low-fat diet with skim milk.
cancer. Which nursing intervention is priority 3. A regular diet with fruits and vegetables.
for this client? 4. A full liquid-only diet with milkshake
1. Request the HCP to order an antidepressant supplements.
medication.
47. The nurse is preparing an educational
2. Refer the client to a CanSurmount volunteer
presentation for women in the community.
for counseling.
Which primary nursing intervention should
3. Encourage the client to verbalize feelings
the nurse discuss regarding the development of
about having cancer.
ovarian cancer?
4. Give the client an advance directive form to
1. Instruct the clients not to use talcum powder
fill out.
on the perineum.
2. Encourage the clients to consume diets with a
high-fat content.
3. Teach the women to have a lower pelvic
sonogram yearly.
4. Discuss the need to be aware of the family
history of cancer.

10_Colgrove_Ch10.indd 402 01/06/16 5:45 PM


C hapter 10  R eproductive D isorders 403

48. The nurse is caring for a client who is one (1) day 53. The client has undergone a bilateral
postoperative hysterectomy for cancer of the orchiectomy for cancer of the prostate. Which
ovary. Which nursing interventions should the intervention should the nurse implement?
nurse implement? Select all that apply. 1. Support the scrotal sac with a towel and
1. Assess for calf enlargement and tenderness. apply ice.
2. Turn, cough, and deep breathe every 2. Administer testosterone replacement
six (6) hours. hormone orally.
3. Assess pain on a 1-to-10 pain scale. 3. Encourage the client to place sperm in a
4. Apply sequential compression devices to legs. sperm bank.
5. Assess bowel sounds every four (4) hours. 4. Have the client talk to another man with
ejaculation dysfunction.

Prostate Disorders 54. The client diagnosed with cancer of the


prostate has been placed on luteinizing
49. Which is the American Cancer Society’s hormone–releasing hormone (LHRH) agonist
recommendation for the early detection of therapy. Which statement indicates the client
cancer of the prostate? understands the treatment?
1. A yearly PSA level and DRE beginning at 1. “I will be able to function sexually as always.”
age 50. 2. “I may have hot flashes while taking this
2. A biannual rectal examination beginning at drug.”
age 40. 3. “This medication will cure the prostate
3. A semiannual alkaline phosphatase level cancer.”
beginning at age 45. 4. “There are no side effects with this
4. A yearly urinalysis to determine the presence medication.”
of prostatic fluid. 55. The client is diagnosed with metastatic prostate
50. The client is diagnosed with early cancer of the cancer to the bones. Which nursing intervention
prostate. Which assessment data would the client should the nurse implement?
report? 1. Prepare for a transurethral resection of the
1. Urinary urgency and frequency. prostate.
2. Retrograde ejaculation during intercourse. 2. Keep the foot of the bed elevated at all times.
3. Low back and hip pain. 3. Place the client on a scheduled bowel
4. No problems have been noticed. regimen.
4. Discuss the client’s altered sexual functioning.
51. The 80-year-old male client has been diagnosed
with cancer of the prostate. Which treatment 56. Which could be a complication of cryotherapy
should the nurse discuss with the client? surgery for cancer of the prostate?
1. Radiation therapy every day for four (4) 1. The urethra could become scarred and cause
weeks. retention.
2. Radical prostatectomy with lymph node 2. The client could have ejaculation difficulties
dissection. and be impotent.
3. Diethylstilbestrol (DES), an estrogen, daily. 3. Bone marrow suppression could occur from
4. Penile implants to maintain sexual the chemotherapy.
functioning. 4. Chronic vomiting and diarrhea causing
electrolyte imbalance could occur.
52. The nurse writes a client problem of urinary
retention for a client diagnosed with Stage IV 57. The client is eight (8) hours post–transurethral
cancer of the prostate. Which intervention prostatectomy for cancer of the prostate. Which
should the nurse implement first? nursing intervention is priority at this time?
1. Catheterize the client to determine the 1. Control postoperative pain.
amount of residual. 2. Assess abdominal dressing.
2. Encourage the client to assume a normal 3. Encourage early ambulation to prevent DVT.
position for urinating. 4. Monitor fluid and electrolyte balance.
3. Teach the client to use the Valsalva maneuver
to empty the bladder.
4. Determine the client’s normal voiding pattern.

10_Colgrove_Ch10.indd 403 01/06/16 5:45 PM


404 M ed -S urg S uccess

58. The client scheduled for a radical prostatectomy 63. The client diagnosed with testicular cancer is
surgical procedure has an intravenous antibiotic scheduled for a unilateral orchiectomy. Which
medication ordered on call to surgery. The information is important to teach regarding
antibiotic is prepared in 100 mL of sterile sexual functioning?
normal saline. At what rate should the nurse 1. The client will have ejaculation difficulties
infuse via the IV pump to infuse the medication after the surgery.
in 30 minutes? ________ 2. The client will be prescribed male hormones
following the surgery.
59. The client diagnosed with cancer of the
3. The client may need to have a penile implant
prostate tells the nurse, “I caused this by being
to be able to have intercourse.
promiscuous when I was young and now I have
4. Libido and orgasm usually are unimpaired
to pay for my sins.” Which statement is the
after this surgery.
nurse’s most therapeutic response?
1. “Why would you think prostate cancer is 64. Which client has the highest risk for
caused by sex?” developing cancer of the testicles?
2. “You feel guilty about some of your actions 1. The client diagnosed with epididymitis.
when you were young?” 2. The client born with cryptorchidism.
3. “Well, there is nothing you can do about that 3. The client with an enlarged prostate.
behavior now.” 4. The client diagnosed with hypospadias.
4. “Have you told the HCP and been checked
65. The nurse is caring for a client who is eight
for an AIDS infection?”
(8) hours postoperative unilateral orchiectomy
60. The nurse is preparing the care plan for a 45-year- for cancer of the testes. Which intervention
old client who has had a radical prostatectomy. should the nurse implement?
Which psychosocial and physiological problem 1. Provide an athletic supporter before
should be included in the plan? ambulating.
1. Altered coping. 2. Encourage the client to delay use of pain
2. High risk for hemorrhage. medications.
3. Sexual impotence. 3. Place the client on a clear liquid diet for the
4. Risk for electrolyte imbalance. first 48 hours.
4. Monitor the PT/INR levels and have vitamin
K ready.
Testicular Disorders
66. The nurse and an unlicensed assistive personnel
61. The school nurse is preparing a class on (UAP) are caring for clients on a genitourinary
testicular cancer for male high school seniors. floor. Which nursing task can be delegated to
Which information regarding testicular self- the UAP?
examination should the nurse include? 1. Increase the drip rate on the Murphy drip
1. Perform the examination in a cool room irrigation set.
under a fan. 2. Check the suprapubic catheter insertion site
2. Any lump should be examined by an HCP as for infection.
soon as possible. 3. Encourage the two (2)-hour postoperative
3. Discuss having a second person confirm a client to turn and cough.
negative result. 4. Document the amount of red drainage in the
4. The procedure will cause mild discomfort if catheter.
done correctly. 67. The nurse is caring for a client with epididymitis
62. The nurse enters the room of a 24-year-old secondary to a chlamydia infection. Which
client diagnosed with testicular cancer. The discharge instruction should the nurse discuss?
fiancée of the client asks the nurse, “Will we 1. The sexual partner must be prescribed
be able to have children?” Which is the nurse’s antibiotics.
best response? 2. Delay sexual intercourse for a minimum of
1. “Your fiancée will be able to father children three (3) months.
like always.” 3. Expect the urine to have white clumps for one
2. “You will have to adopt children because he (1) to two (2) months.
will be sterile.” 4. Drainage from the scrotum is fine as long as
3. “You and he should consider sperm banking there is no fever.
prior to treatment.”
4. “Have you discussed this with your fiancée? I
can’t discuss this with you.”

10_Colgrove_Ch10.indd 404 01/06/16 5:45 PM


C hapter 10  R eproductive D isorders 405

68. The nurse is assessing a client with rule-out Sexually Transmitted Diseases
testicular cancer. Which assessment data support
the client having testicular cancer? 73. The occupational health nurse is preparing a
1. The client complains of pain when urinating. class regarding sexually transmitted diseases
2. There is a chancre sore on the shaft of the (STDs) for employees at a manufacturing plant.
penis. Which high-risk behavior information should
3. The client complains of heaviness in the be included in the class information?
scrotum. 1. Engaging in oral or anal sex decreases the risk
4. There is a red, raised rash on the testes. of getting an STD.
69. The 30-year-old male client diagnosed with 2. Using a sterile needle guarantees the client
germinal cell carcinoma of the testes asks the will not get an STD.
nurse, “What chance do I have? Should I end it 3. The more sexual partners, the greater the
all now?” Which response by the nurse indicates chance of developing an STD.
an understanding of the disease process? 4. If a condom is used, the client will not get a
1. “God does not want you to give up hope and sexually transmitted disease.
end it all now.” 74. The female client diagnosed with human
2. “There is a good chance for survival with papillomavirus (HPV) asks the nurse, “What
standard treatment options.” other problems can HPV lead to?” Which
3. “There may be little hope, but ending it all is statement is the most appropriate response by
not the answer.” the nurse?
4. “You have a 50/50 chance of living for at least 1. “HPV is transmitted during sexual
5 years.” intercourse.”
70. Which tumor marker information is used to 2. “HPV infection can cause cancer of the
follow the progress of a client diagnosed with cervix.”
testicular cancer? 3. “It has been known to lead to ovarian
1. CA-125. problems.”
2. Carcinogenic embryonic antigen (CEA). 4. “Regular Pap smears can help prevent
3. DNA ploidy test. problems.”
4. Human chorionic gonadotropin (hCG). 75. The male client presents to the public health
71. The client diagnosed with cancer of the testes clinic complaining of joint pain and malaise. On
calls and tells the nurse he is having low back assessment, the nurse notes a rash on the trunk,
pain that does not go away with acetaminophen, palms of the hands, and soles of the feet. Which
a nonnarcotic analgesic. Which action should action should the nurse implement next?
the nurse implement? 1. Determine if the client has had a chancre sore
1. Ask the client to come in to see the HCP for within the last two (2) months.
an examination. 2. Ask the client how many sexual partners he
2. Tell the client to use a nonsteroidal anti- has had in the past year.
inflammatory drug instead. 3. Refer the client to a dermatologist for a
3. Inform the client this means the cancer has diagnostic work-up.
metastasized. 4. Have the client provide a clean voided
4. Encourage the client to perform lower back– midstream urine specimen.
strengthening exercises. 76. The nurse is caring for a young adult client who
72. The charge nurse is making rounds on the has been diagnosed with gonorrhea. Which
genitourinary surgery floor. Which action statement reflects an understanding of the
by the primary nurse warrants immediate transmission of sexually transmitted diseases?
intervention? 1. Only lower socioeconomic income people are
1. The nurse elevates the scrotum of a client at risk for gonorrhea and syphilis.
who has had an orchiectomy. 2. The longer a client waits to become sexually
2. The nurse encourages the client to cough, active, the greater the risk for an STD.
although he complains of pain. 3. Females can transmit infectious diseases more
3. The nurse empties the client’s JP drain and rapidly than males.
leaves it rounded. 4. If a client is diagnosed with an STD, the client
4. The nurse asks the UAP to empty a catheter should be evaluated for other STDs.
drainage bag.

10_Colgrove_Ch10.indd 405 01/06/16 5:45 PM


406 M ed -S urg S uccess

77. The young female client is admitted with pelvic 81. Which laboratory test should the nurse expect for
inflammatory disease secondary to a chlamydia the client to rule out the diagnosis of syphilis?
infection. Which discharge instruction should be 1. Vaginal cultures.
taught to the client? 2. Rapid plasma reagin card test (RPR-CT).
1. The client will develop antibodies to protect 3. Gram-stained specimen of the urethral
against a future infection. meatus.
2. This infection will not have any long-term 4. Immunological assay.
effects for the client.
82. The client is diagnosed with tertiary syphilis.
3. Both the client and the sexual partner must be
Which signs and symptoms should the nurse
treated simultaneously.
expect the client to exhibit?
4. Once the infection subsides, the pain will go
1. Lymphadenopathy and hair loss.
away and not be a problem.
2. Warts in the genital area.
78. The nurse is assessing a male client for 3. Dementia and psychosis.
symptoms of gonorrhea. Which data support 4. Raised rash covering the body.
the diagnosis?
83. Which statement best describes the
1. Presence of a chancre sore on the penis.
responsibility of the public health nurse
2. No symptoms.
regarding sexually transmitted diseases?
3. A CD4 count of less than 200.
1. Notify the sexual partners of clients diagnosed
4. Pain in the testes and scrotal edema.
with an STD.
79. The nurse is working in a health clinic. Which 2. Determine the course of treatment for clients
disease is required to be reported to the public diagnosed with an STD.
health department? 3. Explain the legal aspects of STD reporting to
1. Pelvic inflammatory disease. a client diagnosed with an STD.
2. Epididymitis. 4. Analyze the statistics regarding STD
3. Syphilis. transmission and reporting the findings.
4. Ectopic pregnancy.
84. The nurse is admitting a client diagnosed with
80. The nurse is planning the care of a client trichomoniasis. Which assessment data support
diagnosed with pelvic inflammatory disease this diagnosis?
secondary to an STD. Which collaborative 1. Odorless, white, curdlike vaginal discharge.
diagnosis is appropriate for this client? 2. Strawberry spots on the vaginal surface and
1. Risk for infertility. itching.
2. Knowledge deficit. 3. Scant white vaginal discharge and dyspareunia.
3. Fluid volume deficit. 4. Purulent discharge from the endocervix and
4. Noncompliance. pelvic pain.

CONCEPTS
The concepts covered in this chapter focus on reproduction and sexuality. Exemplars that are covered are preg-
nancy and newborns, breast and prostate disorders, and sexually transmitted illnesses. Interrelated concepts of the
nursing process and critical thinking are covered throughout the questions. The concept of critical thinking is
presented in the prioritizing or “first” questions.
85. The outpatient clinic nurse is working with 86. The Gravida 7 Para 6 client delivered a
clients diagnosed with sexually transmitted 9-pound 4-ounce infant two (2) hours ago.
diseases (STD). Which long-term complication Which intervention is priority for the nurse to
should the nurse discuss with the clients about implement?
STDs? 1. Assess the client’s fundus every hour.
1. Stress the need for clients to completely finish 2. Assess the client’s voiding pattern every shift.
all antibiotics prescriptions. 3. Discuss birth control options with the client.
2. Inform the clients that, legally, many STIs 4. Discuss breastfeeding methods with the client.
must be reported to the health department.
3. Sexually transmitted diseases can result in
reproductive problems.
4. Discuss the myth that acquired
immunodeficiency syndrome is an STI.

10_Colgrove_Ch10.indd 406 01/06/16 8:16 PM


C hapter 10  R eproductive D isorders 407

87. The nurse in the gynecology clinic is assessing 89. The nurse is instructing a two (2)-week
a 14-year-old client who reports being sexually postpartum client who has red, tender breasts
active. Which information should the nurse after trying to breastfeed the infant. Which
teach the client? Select all that apply. should the nurse teach the client?
1. Inform the client that the nurse must tell the 1. Be sure the baby empties each breast when
parents of her being sexually active. feeding.
2. Teach the client about possible birth control 2. Apply warm, moist pack to the breasts for
options. comfort.
3. Instruct the client regarding sexually 3. Apply rubbing alcohol to the breast to treat
transmitted disease. the infection.
4. Demonstrate how a condom is applied correctly. 4. The baby must be given formula because the
5. Tell the client the importance of finishing all mother cannot breastfeed.
antibiotics.
90. The nurse is assessing a 34-year-old female
6. Discuss the importance of attending parenting
client with fibrocystic breasts. Which question
classes.
should the nurse ask the client during the
88. The 67-year-old male client reports difficulty assessment?
initiating a urinary stream, urinary frequency, 1. “Are your breasts more tender during your
and inability to completely empty the bladder. period?”
Which procedure would the nurse anticipate the 2. “Have you ever developed an allergy to
health-care provider (HCP) performing first? chocolate?”
1. Digital rectal examination (DRE). 3. “Can you tell me more about your feelings of
2. Prostate-specific surface antigen (PSA). having fibrocystic breast changes?”
3. Prostate ultrasound. 4. “Have you considered having a prophylactic
4. Biopsy of the prostate. mastectomy?”

91. The day nurse is administering the initial dose of medications to a newly admitted client at 0900. Which
medication should the nurse administer first?
1. Digoxin.
2. Furosemide.
3. Gentamycin.
4. Acetaminophen.

Client Name: D.S. Admitting Diagnosis: Prostatitis

Account Number: 98765-0 Med Rec #: 35-45-66 Allergies: NKDA

Date Medication 2301–0700 0701–1500 1501–2300


Today Digoxin 0.125 mg PO every day 0900
Today Furosemide 40 mg every 12 hours 0900
Today Gentamycin 80 mg 1000 1600
IVPB every 6 hours 2200
Today Acetaminophen 650 mg every
4 hours prn

Signature of Nurse:
Day Nurse RN/DN

10_Colgrove_Ch10.indd 407 01/06/16 5:45 PM


408 M ed -S urg S uccess

92. The client scheduled for chemotherapy is diagnosed with inflammatory breast cancer. Which intervention
should the nurse implement based on the complete blood count (CBC) report?

Complete Blood Count Client Value Normal Values


RBC (106) 4.8 Male: 4.7 to 5.1  106 cells/mm3
Female: 4.2 to 4.8  106 cells/mm3
Hemoglobin 12.4 Male: 13.2 to 17.3 g/dL
Female: 11.7 to 15.5 g/dL
Hematocrit 40 Male: 43% to 49%
Female: 38% to 44%
WBC (103) 7.8 4.5 to 11.0  103 cells/mm3
Platelet (103) 192 150 to 450  103/mm3
Differential
Granulocytes 75

1. Administer the chemotherapy as prescribed. 95. Which steps should the nurse provide clients
2. Notify the oncologist immediately. who choose to perform breast self-examination
3. Request urine and blood cultures. (BSE) according to the American Cancer
4. Prepare to administer one (1) unit of packed Society (ACS) guidelines? Rank in order of
red blood cells. performance.
1. Lie flat on the bed with a rolled towel placed
93. The nurse is teaching men about early detection
under the scapula; perform palpation of each
of prostate cancer according to the American
breast.
Cancer Society (ACS) guidelines. Which should
2. Pinch each nipple to see if fluid can be
the nurse teach the clients?
expressed.
1. Beginning at age 39 a Digital Rectal
3. With the breasts exposed, stand in front of
Examination (DRE) followed by a prostate-
a mirror and examine the breasts from front
specific antigen (PSA).
and each side.
2. Beginning at age 45 a rectal sonogram and,
4. In the shower, soap the breasts, and perform
if positive, followed by a Digital Rectal
palpation in a systematic manner on each
Examination (DRE).
breast.
3. Beginning at age 50 a prostate-specific
5. Find a private place where the self-
antigen (PSA) followed by a Digital Rectal
examination can be performed.
Examination (DRE).
4. Beginning at age 60 a Prostate-Specific 96. Which should the nurse teach the client
Antigen (PSA) followed, if positive, by a regarding Breast Health Awareness (BHA)
prostate biopsy. according to the American Cancer Society
(ACS). Select all that apply.
94. The client diagnosed with gestational diabetes
1. Women at high risk should talk to the HCP
delivered a 10-pound 5-ounce infant. Which is
about when to have a mammogram.
priority for the nursery nurse to monitor?
2. Beginning at age 45, to have a yearly
1. Failure to latch on to the breast during
mammogram.
feeding.
3. To perform a breast self-examination (BSE)
2. Jaundice and clay-colored stools.
bimonthly.
3. Parchment–like skin and lack of lanugo.
4. To get a sonogram of the breasts
4. Low blood glucose readings.
semiannually.
5. To have Magnetic Rresonance Imaging of the
breasts every five (5) years.

10_Colgrove_Ch10.indd 408 01/06/16 5:45 PM


PRACTICE QUESTIONS ANSWERS
AND RATIONALES

Breast Disorders Option “1” uses the word “afraid,” which is an


assumption; therefore, this option could be
1. 1. This is symptomatic of benign fibrocystic eliminated.
disease, but follow-up is always needed if the Content – Surgical: Integrated Nursing Process –
lumps do not go away when the hormone Planning: Client Needs – Safe Effective Care Environment,
­levels change. Management of Care: Cognitive Level – Synthesis:
2. Some practitioners suggest eliminating Concept – Cellular Regulation.
­caffeine and chocolate from the diet if the 3. 1. The client has had surgery on this side of the
breasts become tender from the changes, but body. Pressure on the incision should be lim-
there is no research supporting this to be ited until the client is released by the HCP
­effective in controlling the discomfort associ- to perform normal daily activities.
ated with fibrocystic breasts. 2. This is providing the client false hope. ­Cancer
3. The American Cancer Society no longer cells characteristically move easily in the
recommends breast self-examination (BSE) lymph or bloodstream to other parts of the
for all women, but it is advisable for women body. Microscopic disease cannot be deter-
with known breast conditions to perform mined by the naked eye.
BSE monthly to detect potential cancer. 3. A client who has a mastectomy might be dis-
4. The client may need a breast biopsy for charged with a Hemovac drain, but a wedge
­potential breast cancer at some point, but resection should not require one.
breast cancer develops when there is an altera- 4. The breast has not been removed; reconstruc-
tion in the DNA of a cell. tion is not needed.
TEST-TAKING HINT: The test taker could elimi- TEST-TAKING HINT: If the test taker did not
nate option “1” because of the clause “does know this answer, option “1” is information
not require follow-up.” The question is asking provided to any client who has had surgery on
about self-care, and only two (2) options—“2” the upper chest or arm.
and “3”—involve the client doing something.
Content – Surgical: Integrated Nursing Process –
The test taker should choose between these. Planning: Client Needs – Physiological Integrity, Physiologi-
Content – Medical: Integrated Nursing Process – cal Adaptation: Cognitive Level – Synthesis: Concept –
Planning: Client Needs – Health Promotion and Main- Cellular Regulation.
tenance: Cognitive Level – Synthesis: Concept – Cellular
Regulation. 4. 1. Unless there is a personal history of breast
cancer or a strong family history, clinical breast
2. 1. General anesthesia is used for either examinations should begin at age 30 years and
procedure. should be performed yearly.
2. The client should understand the treat- 2. If the client is going to perform breast self-
ment regimen for follow-up care. A examination (BSE), it should begin at age 18.
lumpectomy requires follow-up with The ACS no longer includes monthly BSE as
­radiation therapy to the breast and then part of its guidelines.
systemic chemotherapy. If the cancer is in 3. The ACS recommends a yearly mammo-
its early stages, this regimen has results gram for the early detection of breast cancer
equal to those with a modified radical beginning at age 45 and going to age 55 and
mastectomy. approximately every two (2) years after the
3. A lumpectomy removes only the tumor and a age of 55. Before age 45 and after age 55 it
small amount of tissue surrounding the tumor; should be a discussion between the woman
reconstruction is not needed. and her HCP to determine if more frequent
4. A history of breast cancer in the family is im- mammograms are warranted. A mammo-
material because this client has breast cancer. gram can detect disease that will not be
TEST-TAKING HINT: The test taker should use large enough to feel.
the nursing process to answer this question 4. Breast sonograms are performed to diagnose
and select an assessment intervention, which specific breast disease when a screening mam-
eliminates option “3” as a correct answer. mogram has shown a suspicious area.
409

10_Colgrove_Ch10.indd 409 01/06/16 5:45 PM


410 M ed -S urg S uccess

TEST-TAKING HINT: This is a knowledge-based TEST-TAKING HINT: The test taker cannot over-
question. The test taker might be swayed by look the age when it is given in a question.
the option about BSE, but the age must be “Senior citizen’s center” should alert the test
considered. taker to the older age group. The test taker
Content – Medical: Integrated Nursing Process – should decide what the age has to do with the
Planning: Client Needs – Health Promotion and Main- answer.
tenance: Cognitive Level – Synthesis: Concept – Cellular Content – Medical: Integrated Nursing Process –
Regulation. Planning: Client Needs – Health Promotion and Mainte-
nance: Cognitive Level – Synthesis: Concept – Promoting
5. 1. This is done for reconstruction of a breast
Health.
or augmentation of breast size, but it is not a
TRAM flap procedure, which uses the ­client’s 7. 1. A sentinel node biopsy is a procedure in
own tissue. which a radioactive dye is injected into the
2. The TRAM flap procedure is one in which tumor and then traced by instrumentation
the client’s own tissue is used to form the and color to try to identify the exact lymph
new breast. Abdominal tissue and fat are nodes the tumor could have shed into.
pulled under the skin with one end left 2. This is the older procedure in which the sur-
attached to the site of origin to provide geon removed the nodes thought to drain the
circulation until the body builds collateral tumor. There was no way of knowing if the
circulation in the area. surgeon was actually removing the affected
3. The plastic surgeon can rebuild a nipple from nodes.
pigmented skin donor sites or can tattoo the 3. The purpose of the procedure is not to rely on
nipple in place. guesswork in determining the extent of tumor
4. This is true of saline implants but not of involvement.
TRAM flaps. 4. Microscopic disease cannot be seen by the na-
TEST-TAKING HINT: If the test taker is taking a ked eye.
standard pencil-and-paper test and is not fa- TEST-TAKING HINT: The test taker could elimi-
miliar with this procedure, then skipping the nate options “3” and “4” if he or she were
question and returning to it at a later time is aware of the definition of sentinel, which
advisable. Another question might give a clue means “to watch over as a sentry.” This might
about the procedure. This is not possible on lead the test taker to determine that specific
the NCLEX-RN computerized examination. areas would have to be identified.
Content – Surgical: Integrated Nursing Process – Content – Surgical: Integrated Nursing Process –
Planning: Client Needs – Safe Effective Care Environment, ­Implementation: Client Needs – Safe Effective Care Envi-
Management of Care: Cognitive Level – Synthesis: ronment, Management of Care: Cognitive Level – Analysis:
Concept – Cellular Regulation. Concept – Cellular Regulation.

6. 1. Most women who develop breast cancer 8. 1. Waiting until the baby is born allows the can-
do not have a family history of the disease. cer to continue to develop and spread. This
Specific genes—BRCA-1 and BRCA-2—­ might be an option if the client were in the
implicated in the development of breast cancer third trimester, but not at this early stage.
have been identified, but most women with 2. A modified radical mastectomy is recom-
breast cancer do not have these genes. mended for this client because the client
2. Approximately 1,000 men are diagnosed every is not able to begin radiation or chemo-
year with breast cancer, but, as with women, it therapy, which are part of the regimen for
can occur at any age. Breast cancer in men fre- a lumpectomy or wedge resection. Many
quently goes undetected because men consider breast cancers developed during pregnancy
this a woman’s disease. are hormone sensitive and have the ideal
3. Mammograms can detect breast cancer earlier grounds for growth. The tumor should be
than breast self-examination and are the cur- removed as soon as possible.
rent recommendation by the American Cancer 3. Radiation therapy cannot be delivered to a
Society. pregnant client because of possible harm to
4. The greatest risk factor for developing the fetus.
breast cancer is being female. The second 4. Chemotherapy is not given to the client while
greatest risk factor is being elderly. By she is pregnant because of potential harm to
age 80, one (1) in every eight (8) women the fetus.
develops breast cancer.

10_Colgrove_Ch10.indd 410 01/06/16 5:45 PM


C hapter 10  R eproductive D isorders 411

TEST-TAKING HINT: The test taker should 5. The client has developed a malignancy in
eliminate options “3” and “4” because of one breast and is at a higher risk for de-
potential harm to the fetus but also because veloping another tumor in the remaining
each option has the word “only.” There are breast area.
very few “onlys” in health care. TEST-TAKING HINT: The test taker must de-
Content – Medical: Integrated Nursing Process – termine if the option of keeping the arm still
Diagnosis: Client Needs – Safe Effective Care Environ- is recommended. Most postoperative recom-
ment, Management of Care: Cognitive Level – Analysis: mendations require the client to move as
Concept – Cellular Regulation. much as possible.
9. 1. A client four (4) years postmastectomy should Content – Surgical: Integrated Nursing Process –
be finished with adjuvant therapy, which lasts Planning: Client Needs – Physiological Integrity, Physiologi-
from six (6) months to one (1) year. cal Adaptation: Cognitive Level – Synthesis: Concept –
2. The client may have received Adriamycin, Cellular Regulation.
which is a cardiotoxic medication, but know- 11. 1. This is restating the client’s ­feelings and is
ing this will not change the tests performed a therapeutic response.
or preparation for the tests. 2. This is not recognizing the client’s concerns
3. The nurse should post a message at and putting the nurse’s expectations on the
the head of the client’s bed to not use spouse.
the right arm for blood pressures or 3. This is problem-solving and could be offered,
laboratory draws. This client is at risk for but the therapeutic response is to restate the
lymphedema, and this is a lymphedema client’s feeling and encourage a conversation.
precaution. 4. The client may know this is true. The nurse
4. The chest wall is sometimes involved in is telling the client she has no reason for
breast cancer, but the most important inter- her feelings. Feelings are what they are and
vention is to prevent harm to the client. should be accepted as such.
TEST-TAKING HINT: The question is asking for TEST-TAKING HINT: When the question asks
an intervention common in the health-care for a therapeutic response, the test taker
industry. There are many breast cancer sur- should choose an option encouraging the
vivors who go on to develop unrelated prob- client to ventilate her feelings.
lems, but the nurse must still be aware of the Content – Surgical: Client Needs – Psychosocial Integrity:
lingering needs of the client. ­Cognitive Level – Application: Concept – Sexuality.
Content – Medical: Integrated Nursing Process –
­Implementation: Client Needs – Safe Effective Care Envi- 12. 1. The social worker assists clients in finding
ronment, Management of Care: Cognitive Level – Analysis: nursing home placement and financial arrange-
Concept – Cellular Regulation. ments and does some work with clients to dis-
cuss feelings, but this is not the best referral.
10. 1. It is a common instruction for any client 2. CanSurmount volunteers work with all types
who has had surgery to notify the HCP of clients diagnosed with cancer, not just
if a fever develops. This could indicate a ­clients with breast cancer.
postoperative infection. 3. Reach to Recovery is a specific referral
2. The client who has had a mastectomy is at program for clients diagnosed with breast
risk for lymphedema in the affected arm cancer.
because the lymph nodes are removed dur- 4. I CanCope is a cancer education program for
ing the surgery. The client should protect all clients diagnosed with cancer and their
the arm from injury and carry heavy ob- significant others.
jects with the opposite arm.
3. The client can attend church services and TEST-TAKING HINT: The question asks for the
large gatherings. This client had surgery, not most appropriate referral, and the test taker
chemotherapy, which would make the client should choose the one specific to breast
immunosuppressed. cancer.
4. The client should be taught arm-climbing ex- Content – Medical: Integrated Nursing Process – ­
ercises before leaving the hospital to facilitate Implementation: Client Needs – Safe Effective Care Envi-
maintaining range of motion. ronment, Management of Care: Cognitive Level –
Application: Concept – Cellular Regulation.

10_Colgrove_Ch10.indd 411 01/06/16 5:45 PM


412 M ed -S urg S uccess

Pelvic Floor Relaxation Disorders Reduction of Risk Potential: Cognitive Level – Analysis:
Concept – Female Reproduction.
13. 1. The client should be assessed for a­ llergies 16. 1. There is no reason to limit oral intake or to
to latex as a result of the ­composition of decrease voiding.
the pessary. 2. The client should be eating a high-fiber diet
2. These clients frequently have incontinence, to prevent constipation.
which is assessed prior to recommending a 3. Stool softeners and laxatives are used to
pessary. prevent and treat constipation, which is
3. A pessary is manually inserted to keep a pro- common with a rectocele. Because of
lapsed uterus in place. Asking about a bowel the positioning of the rectum, stool
movement is not an appropriate question. can stay in the rectal pouch, causing
4. This is a symptom experienced by a client constipation.
with a cystocele. 4. Sitz baths are not used to treat a rectocele.
TEST-TAKING HINT: If the test taker has no TEST-TAKING HINT: The test taker who is
idea what the answer is, then the test taker knowledgeable of medical terminology could
should apply Maslow’s hierarchy of needs, eliminate option “1,” which has the word
which in this question is safety. Only option “voiding,” because “rectocele” refers to the
“1” addresses a safety issue. Allergies are a rectum.
safety issue. Content – Medical: Integrated Nursing Process – ­
Content – Surgical: Integrated Nursing Process – Implementation: Client Needs – Safe Effective Care Envi-
Assessment: Client Needs – Physiological Integrity, Reduc- ronment, Management of Care: Cognitive Level –
tion of Risk Potential: Cognitive Level – Analysis: Application: Concept – Female Reproduction.
Concept – Female Reproduction.
17. 1. Sonograms and pelvic examinations are used
14. 1. Perineal care is given every shift or as needed. to diagnose cystoceles.
2. The client may have an indwelling catheter 2. Kegel exercises strengthen the pelvic floor
for two (2) to four (4) days postoperative but muscles.
not for a month. 3. Clients with cystoceles frequently have
3. A pessary is used in place of surgery. urinary incontinence when they cough,
4. The client should be taught how to per- sneeze, laugh, lift heavy items, or make
form Kegel exercises to strengthen the sudden jarring motions.
muscles. 4. Clients with cystoceles may have sexual
TEST-TAKING HINT: When selecting a correct intercourse unless contraindicated by
answer, the test taker should always look at ­another medical reason.
the adjectives, especially numbers such as TEST-TAKING HINT: This type of question is
four (4) in option “1” or one (1) in option “2.” confusing to test takers because the correct
Content – Surgical: Integrated Nursing Process – option provides incorrect information. Most
Planning: Client Needs – Physiological Integrity, Physiologi- of the time absolutes such as “never,” “only,”
cal Adaptation: Cognitive Level – Synthesis: Concept – and “always” make the option incorrect.
Female Reproduction. Option “4” has an absolute—“never”—but it
15. 1. A rectocele causes the rectum to be is the correct answer because all of the other
pouched upward, causing rectal pressure. options contain correct information the cli-
2. When the rectum pushes against the pos- ent has learned.
terior wall of the vagina, the result is flatus. Content – Medical: Integrated Nursing Process –
3. Clients with a rectocele experience fecal Evaluation: Client Needs – Physiological Integrity, Physi-
incontinence. ological Adaptation: Cognitive Level – Synthesis:
4. Clients with a rectocele frequently are Concept – Female Reproduction.
constipated. 18. 1. Orthostatic hypotension is not a surgery-
5. A client who has a rectocele does not experi- specific complication.
ence urinary frequency. 2. Atelectasis is a complication of general anes-
TEST-TAKING HINT: The test taker should learn thesia, not an A & P repair.
the terms. If the test taker does not know the 3. An Allen test is a physical examination evalu-
answer, the test taker can eliminate urinary ating arterial blood supply to the radial and
frequency ­because it is different from the ulnar arteries.
other options. It refers to the urinary tract, 4. Assessing for deep vein thrombosis (DVT)
whereas the others ­refer to bowel elimination. is performed on all clients having a vaginal
Content – Medical: Integrated Nursing Process – hysterectomy. After any surgery requiring
Assessment: Client Needs – Physiological Integrity, the client to be placed in the lithotomy

10_Colgrove_Ch10.indd 412 01/06/16 5:45 PM


C hapter 10  R eproductive D isorders 413

position, the client should be assessed for 4. An imbalance of fluid is a complication of


DVT. These clients are at a higher risk for several surgeries, not specifically a total
this complication. ­abdominal hysterectomy.
TEST-TAKING HINT: The test taker must have TEST-TAKING HINT: When the test taker does
knowledge of this surgical procedure to be not know the answer, the test taker should
able to answer this question. A good choice is consider the similarities and the differences
to choose an option assessed for most clients in surgeries. A total abdominal hysterectomy
who are immobile. has complications similar to all types of
Content – Surgical: Integrated Nursing Process – abdominal surgeries.
Assessment: Client Needs – Physiological Integrity, Reduc- Content – Surgical: Integrated Nursing Process –
tion of Risk Potential: Cognitive Level – ­Analysis: Diagnosis: Client Needs – Safe Effective Care Environ-
Concept – Female Reproduction. ment, Management of Care: Cognitive Level – Analysis:
Concept – Female Reproduction.
19. 1. The client should report any vaginal
bleeding or gastrointestinal changes such 22. 1. A uterine prolapse is not caused by consti-
as distention, cramping, or changes in pation; it is caused by a weakening of the
bowel habits. ­pelvic muscles. It is a protrusion of the uterus
2. Clients should rest and not start a vigorous through the vagina. It can pull on the vaginal
exercise program until the surgeon gives wall, bladder, and rectum.
permission. 2. There are multiple treatment modalities for
3. Clients should avoid activities increasing uterine prolapse. The selection of treatment
pelvic congestion, such as dancing, horseback is determined by the degree of the prolapse
riding, and sitting for long periods. and the medical history of the client.
4. Clients should avoid taking baths to help pre- 3. The protrusion from the vagina can be seen
vent infection of the incision site. in some cases, but this is not so with all
TEST-TAKING HINT: When the test taker is clients.
selecting possible correct answers, the test 4. Symptoms can be aggravated by coughing,
taker should carefully consider the descrip- sneezing, lifting heavy objects, standing
tive words. In option “2,” the adjective for prolonged periods, and climbing stairs.
“vigorous” should cause the test taker to TEST-TAKING HINTS: The test taker should
eliminate it, and in option “3” the words “six eliminate options containing words such as
(6) hours” should help eliminate it. “only,” “never,” “always,” “all,” or “most of
Content – Surgical: Integrated Nursing Process – the time.” If the test taker does not have a
Planning: Client Needs – Physiological Integrity, Physiologi- clue, “uterine” is not in the same body
cal Adaptation: Cognitive Level – Synthesis: Concept – system as constipation, so option “1” can
Female Reproduction. be eliminated.
20. 1. Observation is assessment, which cannot be Content – Surgical: Integrated Nursing Process –
delegated. Planning: Client Needs – Physiological Integrity, Physiologi-
cal Adaptation: Cognitive Level – Synthesis: Concept –
2. This nursing task can be delegated, but
Female Reproduction.
evaluation is the responsibility of the nurse.
3. Teaching cannot be delegated. 23. 1. Pessaries need to be changed to prevent
4. Teaching cannot be delegated. complications.
TEST-TAKING HINT: The nurse cannot del- 2. Clients do need to clean the pessary
egate a­ ssessment, teaching, evaluation, or an routinely.
unstable client to a UAP. 3. Pessaries are inserted in areas other than sur-
gery, such as in a health-care provider’s office.
Content – Surgical: Integrated Nursing Process –
Planning: Client Needs – Safe Effective Care Environ- 4. Hormone cream may be used, but usually an
ment, Management of Care: Cognitive Level – Synthesis: oral estrogen is prescribed.
Concept – Nursing Roles. TEST-TAKING HINTS: The test taker should
realize any removable device inserted into
21. 1. Urinary retention is a complication of a vagi-
the body requires routine changing at some
nal hysterectomy.
time; therefore, option “1” can be eliminated.
2. Nerve damage is a possible complication of
Option “4” is a medication and a pessary is a
improper positioning during surgery, but the
device, so option “4” can be eliminated.
client is “post” surgery.
3. Clients who have had a total abdominal Content – Surgical: Integrated Nursing Process –
hysterectomy are at risk for intestinal Planning: Client Needs – Physiological Integrity, Physiologi-
cal Adaptation: Cognitive Level – Synthesis: Concept –
obstruction.
Female Reproduction.

10_Colgrove_Ch10.indd 413 01/06/16 5:45 PM


414 M ed -S urg S uccess

24. 1. Estrogen changes the pelvic floor muscles 26. 1. Risk factors for cancer of the cervix
and lining of the uterus and may help im- ­include sexual activity before the age of
prove a pelvic relaxation disorder. 20 years; multiple sexual partners; early
2. Cervidil is used to prepare the cervix for childbearing; exposure to the human
delivery of a baby. It causes the cervix to papillomavirus; HIV infection; smoking;
shorten, soften, and dilate. and nutritional deficits of folates, beta
3. Progesterone is given for implantation of a carotene, and vitamin C.
fertilized ovum. 2. High-fat diets place clients at risk for some
4. Pitocin causes the uterus to contract. It is cancers but not for cervical cancer. The use of
used during the labor and delivery process. birth control pills may allow increased sexual
TEST-TAKING HINT: Options “2,” “3,” and “4” freedom because of the protection from preg-
are similar because they are all used dur- nancy, but it does not increase the risk for
ing a pregnancy or delivery. If the test taker cancer of the cervix.
doesn’t have a clue as to the correct answer, 3. Infections with the human papillomavirus are
the test taker should attempt to determine a risk factor for cancer of the cervix.
what answer is ­different from all the others. 4. Having a yearly Pap smear increases the chance
Content – Medical: Integrated Nursing Process –
of detecting cellular changes early, but it does
Planning: Client Needs – Physiological Integrity, Pharma- not decrease the risk for developing cancer.
cological and ­Parenteral Therapies: Cognitive Level – TEST-TAKING HINT: The test taker could dis-
Synthesis: Concept – ­Female Reproduction. card option “4” as a possible answer because
it is a yearly test for the early detection of
cervical ­cancer, not a risk factor.
Uterine Disorders Content – Medical: Integrated Nursing Process –
­Implementation: Client Needs – Health Promotion and
25. 1. A vaginal examination does not provide a ­Maintenance: Cognitive Level – Application: Concept –
definitive diagnosis to determine the cause Female Reproduction.
of the pain.
2. A pregnancy test is not usually ordered un- 27. 1. Diffuse, watery, foul-smelling discharge
less the client has a reason to think she may ­occurs at a much later stage.
be pregnant. Pregnancy temporarily allevi- 2. At this stage the client is asymptomatic
ates the symptoms of endometriosis because and the cancer has been determined by
neither ovulation nor menses occurs during a Pap smear.
pregnancy. 3. Dyspareunia is painful sexual intercourse; the
3. There is a high incidence of endometrio- client is asymptomatic.
sis among women who have never had 4. Intense itching occurs with vaginal yeast
children (nulliparity) and those who have infections.
children later in life. The most common TEST-TAKING HINT: The test taker could
way to diagnose this condition is through either choose option “2” because it is the
an exploratory laparoscopy. least presenting symptom or discard it. Stag-
4. The ovaries lie deep within the pelvic cavity. ing for all cancers starts with “0” or “1,”
To reach the ovaries, some form of abdomi- ­indicating the least detectable cancer.
nal procedure must be performed, such as a Content – Surgical: Integrated Nursing Process –
laparoscopy. However, the symptoms are not Assessment: Client Needs – Physiological Integrity, Reduc-
those of an ovarian cyst. tion of Risk Potential: Cognitive Level – Analysis:
TEST-TAKING HINT: The test taker could Concept – Female Reproduction.
eliminate answer option “1” because “diag- 28. 1. Brachytherapy is the direct implantation of
nosis” is in the stem. The stem is asking for radioactive seeds through the vagina into
a procedure providing a definitive diagnosis. the uterus. The client should be in a pri-
­Option “2” could be eliminated because, if vate room at the end of the hall to prevent
the client is menstruating (dysmenorrhea radiation exposure to the rest of the unit.
means “painful menstruation”), then the 2. Nurses wear a dosimeter registering the
­client is usually not pregnant. amount of radiation they have been ex-
Content – Medical: Integrated Nursing Process – posed to. When a certain level is reached,
Diagnosis: Client Needs – Physiological Integrity, Reduction the nurse is no longer allowed to care
of Risk Potential: Cognitive Level – Analysis: Concept – for clients undergoing internal radiation
Female Reproduction. therapy.

10_Colgrove_Ch10.indd 414 01/06/16 5:46 PM


C hapter 10  R eproductive D isorders 415

3. Visitors are limited while the radiation is 4. The client should not have any vaginal
in place. bleeding.
4. In this case, spending extra time with a client TEST-TAKING HINT: The test taker should
is not done. The nurse does only what must apply basic postoperative concepts when an-
be done and leaves the room. swering questions and realize bleeding is not
5. The nuclear medicine technician will expected postoperatively and safety should
­assist with the placement of the implants always be addressed.
and will deliver the implants in a lead- Content – Surgical: Integrated Nursing Process –
lined container. The technician will also Planning: Client Needs – Physiological Integrity, Physiologi-
scan any items (linens and wastes) leaving cal Adaptation: Cognitive Level – Synthesis: Concept –
the room for radiation contamination. Female Reproduction.
TEST-TAKING HINT: This is an alternate-type
31. 1. This pain indicates metastasis to the retro-
question, which requires the test taker to peritoneal region. If caught early, a complete
select more than one correct answer. The hysterectomy is usually the only therapy rec-
test taker must select all the correct answers ommended. This type of pain indicates the
to receive credit for the question. cancer is advanced and the prognosis is poor.
Content – Medical: Integrated Nursing Process – ­ 2. This pain indicates the cancer is in the
Implementation: Client Needs – Safe Effective Care Envi- retroperitoneal region and the prognosis
ronment, Management of Care: Cognitive Level –
is poor.
Application: Concept – Cellular Regulation.
3. Pain is not part of the treatment of cancer.
29. 1. This client is at risk for cancer of the ovary Surgery may cause pain, but most treatments
and uterus because of advancing age, regard- do not.
less of sexual activity, and should see an HCP 4. Radiation therapy does not always result in
yearly. pain; it depends on the area irradiated.
2. Hormone replacement therapy (HRT) is TEST-TAKING HINT: Option “4” has the abso-
not recommended for most postmenopausal lute word “always” and should be eliminated
clients because research has shown HRT in- as a correct answer. The stem is describing
creases the risk of myocardial infarctions and symptoms in regions other than the lower
cerebrovascular accidents (strokes). pelvis, so an educated choice is option “2.”
3. This is a therapeutic response and the client Content – Medical: Integrated Nursing Process –
did not state a feeling. Diagnosis: Client Needs – Physiological Integrity, Physi-
4. The client should have a yearly clinical ological Adaptation: Cognitive Level – Analysis: Concept –
examination of the breasts and pelvic area Cellular Regulation.
for the detection of cancer.
32. 1. The client taking a narcotic medication
TEST-TAKING HINT: If the stem is not ask-
should be placed on a bowel regimen, but
ing for a therapeutic response, then factual this client would not be prescribed narcotic
information should be provided to the cli- medication.
ent. This eliminates option “3” as a possible 2. A tepid bath for 30 to 45 minutes is not
answer. appropriate because the lukewarm water gets
Content – Medical: Integrated Nursing Process – cold. A heating pad to the abdomen some-
­Implementation: Client Needs – Health Promotion and times helps with the pain.
­Maintenance: Cognitive Level – Application: Concept –
3. The medication of choice for mild to
Female Reproduction.
moderate dysmenorrhea is an NSAID.
30. 1. Clients who are diagnosed with cancer of the NSAIDs cause gastrointestinal upset and
uterus have the ovaries removed to reduce should be taken with food.
hormone production. The client will not be 4. This may be an option eventually, but the
taking HRT. stem did not give an age nor state the client
2. Pelvic rest means nothing is placed in the has decided she does not want to get pregnant.
vagina. The client does not need a tampon TEST-TAKING HINT: The test taker should not
at this time, but sexual intercourse should read into the question. Option “4” is only
be avoided until the vaginal area has correct when more information is provided.
healed. The test taker must know about the scales
3. The sitting position a client assumes when used to rate pain, nausea, or depression. The
driving a vehicle places stress on the lower client’s report of midrange symptoms does
abdomen. The client should wait until the not indicate the need for routine narcotic
HCP releases her to drive. administration.

10_Colgrove_Ch10.indd 415 01/06/16 5:46 PM


416 M ed -S urg S uccess

Content – Medical: Integrated Nursing Process – TEST-TAKING HINT: The nurse cannot del-
Planning: Client Needs – Physiological Integrity, Physiologi- egate assessment. “Monitor” is a word in-
cal Adaptation: Cognitive Level – Synthesis: Concept – terpreted as “assess.” This is an “except”
Female Reproduction. question, so the test taker should not jump
33. 1. Benign fibroid tumors in the uterus cause the to option “1” as the ­correct answer because
client to bleed longer with a heavier flow, not the UAP can perform this task.
miss periods. Content – Medical: Integrated Nursing Process –
2. Many women delay surgery until anemia Planning: Client Needs – Safe Effective Care Environment,
has occurred from the heavy menstrual Management of Care: Cognitive Level – Synthesis:
flow. A symptom of anemia is shortness Concept – Nursing Roles.
of breath. 36. 1. This can be done to determine specific prob-
3. The number of pregnancies does not matter lems resulting from the specific side effects of
at this time; the client has a different problem. the medication, but it is not the first action.
4. The pain is in the pelvic region to low back, The nurse can ask general assessment ques-
where the uterus lies. tions to determine how the client is tolerating
TEST-TAKING HINT: This is a high-level ques- the treatments.
tion requiring the test taker to make several 2. The systemic side effects of chemotherapy
judgments before arriving at the answer. are not always apparent, and the develop-
First, the test taker must decide what hap- ment of stomatitis can be extremely dis-
pens when a client has fibroid tumors and tressing for the client. The nurse should
then which symptoms the c­ lient will exhibit. assess the client’s tolerance to treatments.
Content – Medical: Integrated Nursing Process – 3. This is done if the white blood cell count is
Assessment: Client Needs – Safe Effective Care Environ- low. The nurse assesses the WBC count and
ment, Management of Care: Cognitive Level – Analysis: then obtains an order from the HCP.
Concept – Female Reproduction. 4. This is an appropriate action but not before
assessing physical problems.
34. 1. The nurse does not require an order to teach.
Teaching is an independent nursing function. TEST-TAKING HINT: When prioritizing nursing
2. The nurse can request the dietitian to include interventions, the test taker should apply the
yogurt in the client’s calorie restrictions with- nursing process, and assessment is the first
out an order. step.
3. If the HCP has ordered an IV antibiotic, then Content – Medical: Integrated Nursing Process – ­
there is no reason to request a change to an Implementation: Client Needs – Safe Effective Care Envi-
oral route. ronment, Management of Care: Cognitive Level –
4. Female clients on antibiotics are at risk Synthesis: Concept – Cellular Regulation.
for killing the good bacteria, which keep
yeast infections in check. This is especially Ovarian Disorders
true in clients diagnosed with diabetes.
Lactobacillus acidophilus is a yeast replace- 37. 1. The client has symptoms of an ovarian cyst,
ment medication. usually diagnosed by a pelvic sonogram.
TEST-TAKING HINT: The test taker must be 2. The client has abdominal pain, not back or
aware of independent nursing functions. This neurological pain, which is when a lumbar
eliminates options “1” and “2.” puncture is performed.
Content – Medical: Integrated Nursing Process – 3. The appendix is in the right lower abdomen,
­Implementation: Client Needs – Physiological Integrity, not the left.
Pharmacological and Parenteral Therapies: Cognitive 4. Ovarian cysts are fluid-filled sacs located
Level – Application: Concept – Medication. on the surface of the ovary. A lower pelvic
35. 1. The unlicensed assistive personnel can empty sonogram is the preferred diagnostic tool.
the indwelling catheter and record the output. It is not invasive and usually not painful.
2. This is an appropriate assignment. TEST-TAKING HINT: The test taker could elim-
3. Monitoring a peri-pad count is done to inate options “2” and “3” by using knowledge
determine if the client is bleeding exces- of basic anatomy and physiology. The age
sively; the nurse should do this as part of of the client places this client in the typical
the assessment. age range for a benign ovarian cyst; before
4. All personnel should encourage the client to age 29 years, 98% of ovarian cysts are
ambulate. benign.

10_Colgrove_Ch10.indd 416 01/06/16 5:46 PM


C hapter 10  R eproductive D isorders 417

Content – Medical: Integrated Nursing Process – Content – Surgical: Integrated Nursing Process – ­
Diagnosis: Client Needs – Physiological Integrity, Reduction Implementation: Client Needs – Safe Effective Care Envi-
of Risk Potential: Cognitive Level – Analysis: Concept – ronment, Management of Care: Cognitive Level –
Female Reproduction. Application: Concept – Female Reproduction.

38. 1. The ovaries are anatomically positioned 40. 1. The client has a low-grade cancer occurring
deep within the pelvis, and because of in approximately 15% of ovarian tumors. The
this, signs and symptoms of cancer are affected ovary usually is removed, and the cli-
vague and nonspecific. Symptoms include ent may or may not require adjuvant therapy.
increased abdominal girth, pelvic pres- Women with this type of tumor are usually
sure, indigestion, bloating, flatulence, and younger than age 40 years.
pelvic and leg pain. Increasing abdomen 2. The tumor is classified as cancer. The follow-
size as a result of accumulation of fluid up care is not as extensive because of the
is the most common sign. Many women characteristics the tumor displays.
ignore the symptoms because they are so 3. These tumors are low-grade cancers with
nonspecific. fewer propensities for metastasis than
2. Regular gynecological examinations are rec- most ovarian cancers.
ommended, but this is advanced disease. 4. This client has a better prognosis than 85%
3. Dysmenorrhea is not a risk factor for devel- of clients diagnosed with ovarian cancer.
oping ovarian cancer. Any enlarged ovary TEST-TAKING HINT: The test taker could
should be evaluated, especially if the client eliminate option “4” because “low malig-
is postmenopausal, when the ovaries shrink nancy potential” and “poor prognosis” do
in size. not match. The statement in option “1” says
4. A family history of breast cancer is a cause for the client does not have cancer but will need
the client to be assessed regularly for breast therapy for cancer, so the test taker could
and ovarian cancer, but this is late disease. eliminate this option.
TEST-TAKING HINT: “Stage IV” should help Content – Medical: Integrated Nursing Process –
the test taker to eliminate options “2” and Diagnosis: Client Needs – Safe Effective Care Environ-
“4” ­because this client has advanced disease ment, Management of Care: Cognitive Level – Analysis:
and it is hoped regular checkups find prob- Concept – Female Reproduction.
lems early. 41. 1. This statement would be appropriate if not
Content – Medical: Integrated Nursing Process – for the abdominal girth change. This should
Diagnosis: Client Needs – Physiological Integrity, Reduction alert the nurse to some internal reason for
of Risk Potential: Cognitive Level – Analysis: Concept –
the change in girth. Ascites causes a change in
Cellular Regulation.
abdominal girth.
39. 1. All cysts do not become cancerous; 98% of 2. Ovarian cancer has vague symptoms of
ovarian cysts in clients younger than age 29 abdominal discomfort, but increasing
are benign, whereas in women older than abdominal girth is the most common
age 50, about half are benign. symptom. If the client has had the ovaries
2. Any abnormal ovary that cannot be removed, then the nurse could assess for
diagnosed with a transvaginal ultrasound another cause.
should be examined laparoscopically. 3. This could be assessing for a peptic ulcer, but
3. Any time the client has surgery, she should ulcers do not cause increasing abdominal girth.
be prepared to experience some pain. This is 4. This is a question to determine if the client
a false statement and could cause a breach in has gastroesophageal reflux, but this does not
the nurse–client relationship. cause increased waist size.
4. This is a therapeutic statement and the client TEST-TAKING HINT: The test taker must no-
is asking for information. tice all the symptoms the client is reporting.
TEST-TAKING HINT: The test taker should read Flatulence and bloating could be associated
the stem of the question carefully. Option with a number of problems, but these symp-
“1” has a form of absolute, “eventually will toms, along with increased waist size, narrow
become,” so it can be eliminated. Option “3” the possibilities.
is a false statement. Option “4” is a therapeu- Content – Medical: Integrated Nursing Process –
tic response and the stem asks the nurse to Assessment: Client Needs – Physiological Integrity, Reduc-
provide information. tion of Risk Potential: Cognitive Level – Analysis:
Concept – Female Reproduction.

10_Colgrove_Ch10.indd 417 01/06/16 5:46 PM


418 M ed -S urg S uccess

42. 1. An antidepressant may be needed at some 44. 1. The radiation markings on a client are there
time, but at this point the nurse should offer to guide the technician to irradiate only
his or her time and interest and encourage the area within the marks. The marks must
the client to discuss the feeling of having remain until the client has completed the
cancer. treatments.
2. CanSurmount volunteers are extremely help- 2. Radiation therapy is administered in fraction-
ful in talking about having cancer with the ated (divided) doses to allow for regeneration
client, but they do not provide counseling. of normal cells. Cancer cells do not regener-
The programs work based on the fact some- ate as rapidly as normal cells.
one who has had cancer and gone through 3. There are many medications prescribed
treatment can relate to the client about to for cancer or treatment-induced nausea.
begin treatment. The client should notify the HCP if
3. The nurse should plan to spend time with adequate relief is not obtained.
the client and allow the client to discuss the 4. Cancer treatments frequently interfere with
feelings of having cancer, ­dying, fear of the the client’s appetite, but supporting the nutri-
treatments, and any other concerns. tional status of the client is important.
4. The client will need to complete an advance TEST-TAKING HINT: The question is an
directive, but this action does not address the “except” question. All options except one (1)
client’s grieving process. will be statements indicating the client does
TEST-TAKING HINT: The test taker could understand the teaching. If the test taker
eliminate option “2” because a client is not missed the information making this an
referred to a volunteer for counseling. Only “except” question, finding two (2) options
one (1) option directly addresses the prob- with correct answers might clue the test
lem and requires the nurse to interact with taker to reread the stem.
the client. Content – Medical: Integrated Nursing Process –
Content – Medical: Integrated Nursing Process – Evaluation: Client Needs – Safe Effective Care Environ-
­Implementation: Client Needs – Psychosocial Integrity: ment, Management of Care: Cognitive Level – Synthesis:
Cognitive Level – Analysis: Concept – Cellular Regulation. Concept – Cellular Regulation.

43. 1. An absolute neutrophil count of 3,500 indi- 45. 1. This is appropriate information if the client is
cates the client has sufficient mature white in her mid- to late-30s and has completed her
blood cells, or granulocytes, to act as a family, but this is not discussing early detec-
defense against infections. tion of ovarian cancer.
2. A platelet count of 150,000 is within normal 2. A transvaginal ultrasound is a sonogram in
range (150  103 [1,000]  150,000). which the sonogram probe is inserted into
Thrombocytopenia is less than 100,000. the vagina and sound waves are ­directed
3. A red blood cell count of 5,000,000 toward the ovaries. The CA-125 tumor
is within normal limits (5  106 marker is elevated in several ­cancers. It is
[1,000,000]  5,000,000). nonspecific but, coupled with the sono-
4. A normal urinalysis contains one (1) to gram, can provide information about ovar-
two (2) WBCs. A report of 100 WBCs ian cancer for early diagnosis.
indicates the presence of an infection. A 3. Yearly MRI scans will not provide the infor-
clean voided specimen should be ­obtained mation the two (2) tests will, and every
and a urine culture should be done. This 12 months is too long an interval.
client should be prescribed antibiotics 4. A flexible sigmoidoscopy provides the HCP
immediately. with a visual examination of the sigmoid
TEST-TAKING HINT: The test taker should ­colon, not the ovaries.
memorize normal values for common labora- TEST-TAKING HINT: The test taker could elim-
tory tests. Urine will not have a large number inate option “4” because of the anatomical
of white blood cells unless there is a patho- site and option “3” because of the time fac-
logical process occurring. The kidneys filter tor. The test taker should ask, “If looking for
the blood but do not process the destruction early detection, at what interval should the
of blood cells. client see the HCP?”
Content – Medical: Integrated Nursing Process – Content – Medical: Integrated Nursing Process –
Assessment: Client Needs – Physiological Integrity, Reduc- ­Implementation: Client Needs – Health Promotion and
tion of Risk Potential: Cognitive Level – Analysis: Maintenance: Cognitive Level – Analysis: Concept –
Concept – Cellular Regulation. Cellular Regulation.

10_Colgrove_Ch10.indd 418 01/06/16 5:46 PM


C hapter 10  R eproductive D isorders 419

46. 1. This diet is appropriate for a client diagnosed 4. Sequential compression hose are used
with diverticulitis. prophylactically to prevent deep vein
2. This diet applies to a client with coronary ar- thrombosis.
tery disease and hypertension. 5. The client should be assessed for the
3. The client is not placed on a specific diet, ­return of bowel sounds.
but it is always a good recommendation to TEST-TAKING HINT: Option “2” has a time
include fruits and vegetables in the diet. frame in it, and the test taker should
4. There is no reason to limit the consistency of ask if the time frame is correct for the
the foods consumed to full liquids. intervention.
TEST-TAKING HINT: The test taker should rec- Content – Surgical: Integrated Nursing Process –
ognize option “3” as a recommended diet for ­Implementation: Client Needs – Safe Effective Care Envi-
all clients without a specific disease process ronment, Management of Care: Cognitive Level – Analysis:
limiting the types of foods consumed. Concept – Female Reproduction.
Content – Medical: Integrated Nursing Process –
Planning: Client Needs – Physiological Integrity, Basic Care
and Comfort: Cognitive Level – Synthesis: Concept – Prostate Disorders
Female Reproduction.
49. 1. The American Cancer Society rec-
47. 1. Research has shown the use of talcum ommends all men have a yearly
powder perineally increases the risk for prostate-specific antigen (PSA) blood
developing ovarian cancer, although there level, followed by a digital rectal examina-
is no explanation known for this occur- tion (DRE) beginning at age 50. Men in
rence. Other risk factors include a high- the high-risk group, including all African
fat diet, nulliparity, infertility, older age American men, should begin at age 45.
(70 to 80 years) has the greatest incidence, 2. A biannual (every 6 months) examination is
mumps before menarche, and family his- not recommended.
tory of ovarian cancer. 3. Alkaline phosphatase levels are performed
2. Nurses should never encourage a high-fat diet. on men with known prostate cancer to deter-
3. Only clients in a high-risk category should mine bone involvement. This is not a screen-
have routine sonograms. The time frame ing test.
for the high-risk group of clients is six (6) 4. This test is done if the client has signs and
months. This is not primary intervention; symptoms of prostatitis.
early detection is secondary intervention.
TEST-TAKING HINT: The nurse must be aware
4. This alerts the client to participate in ac-
of recommended screening guidelines for a
tivities detecting cancer early, a secondary
number of diseases. The test taker should
intervention.
carefully look at the time frame for the tests
TEST-TAKING HINT: The test taker could and the age of the client.
eliminate options “3” and “4” because of
Content – Medical: Integrated Nursing Process –
the word “primary” in the stem. Option “2” Planning: Client Needs – Health Promotion and Main-
could be eliminated because of the recom- tenance: Cognitive Level – Synthesis: Concept – Cellular
mendation of a high-fat diet. Regulation.
Content – Medical: Integrated Nursing Process –
Planning: Client Needs – Health Promotion and Mainte-
50. 1. Urgency and frequency are obstructive symp-
nance: Cognitive Level – Synthesis: Concept – Promoting toms and are late signs.
Health. 2. Retrograde ejaculation occurs when the
sperm are ejaculated into the urine; it occurs
48. 1. All clients who have had surgery are at in some cases of male infertility.
risk for developing deep vein thrombosis 3. Low back pain and hip pain are symptoms of
(DVT), and an enlarged, tender calf is a metastasis to the bone and are late symptoms.
sign of DVT. 4. In early-stage prostate cancer, the man
2. The client should be turned and encouraged will not be aware of the disease. Early
to cough and deep breathe at least every two detection is achieved by screening for the
(2) hours. cancer.
3. Clients who have had surgery should
TEST-TAKING HINT: The test taker should
be assessed for pain on a pain scale and
notice the word “early” in the stem of the
by observing for physiological markers
question and choose the option with the
­indicating pain.
least amount of symptoms.

10_Colgrove_Ch10.indd 419 01/06/16 5:46 PM


420 M ed -S urg S uccess

Content – Medical: Integrated Nursing Process – 53. 1. Elevating a surgical site and applying ice
Assessment: Client Needs – Physiological Integrity, Reduc- will reduce edema to the area.
tion of Risk Potential: Cognitive Level – Analysis: 2. The testes have been excised to remove the
Concept – Cellular Regulation. majority of male hormones. Replacing the
51. 1. Radiation therapy is considered aggressive hormones negates the purpose of the surgery.
therapy and is not recommended for the 3. Sperm banking is encouraged for younger
elderly client unless needed to alleviate pain men who want to have children in the future.
from bony metastasis. Prostate cancer is a cancer diagnosed in older
2. A radical prostatectomy with lymph node men, and sperm banking is not normally
dissection is extensive surgery and only rec- recommended.
ommended for clients with a life expectancy 4. Bilateral orchiectomy will render the
of greater than 10 years. ­client impotent but not with ejaculation
3. DES is a hormone preparation that sup- dysfunction.
presses the male hormones and slows the TEST-TAKING HINT: The test taker should try
growth of the tumor. Some men with a life to match the procedure to the answer option.
expectancy of less than 10 years choose The procedure removes hormone-producing
not to treat the cancer at all and will usu- ability, so option “2” could be eliminated be-
ally die from causes other than prostate cause it reverses the effects of the procedure.
cancer. Content – Surgical: Integrated Nursing Process – ­
4. Penile implants do not treat prostate cancer. Implementation: Client Needs – Safe Effective Care Envi-
Sexual functioning may or may not be im- ronment, Management of Care: Cognitive Level –
paired, depending on whether the client is Application: Concept – Cellular Regulation.
treated surgically by a radical prostatectomy.
54. 1. This hormone will suppress the production of
If so, then the client may be prescribed ­Viagra
male hormones, and the client will not func-
or Cialis. Eighty-year-old men are not candi-
tion sexually as always.
dates for a radical prostatectomy.
2. The client may have hot flashes because
TEST-TAKING HINT: The test taker should these drugs increase hypothalamic activ-
notice the age of the client. When an age is ity, which stimulates the thermoregulatory
provided in the question, it is significant. The centers of the body.
elderly do not tolerate many treatments well, 3. The medication decreases the growth rate of
so the test taker should choose the least in- the cancer, but it does not cure the cancer.
vasive treatment. 4. There are side effects with all medications.
Content – Medical: Integrated Nursing Process – These medications can cause gynecomas-
Planning: Client Needs – Safe Effective Care Environment, tia, hot flashes, cardiovascular effects, and
Management of Care: Cognitive Level – Synthesis: decreased sexual functioning. The LHRH
Concept – Cellular Regulation. agonists have fewer side effects than the
52. 1. The nurse should assess the client’s normal estrogens.
voiding pattern before taking any action. TEST-TAKING HINT: The test taker could
2. This is a good intervention, but it comes after eliminate options “3” and “4” because of the
assessment. promise of no side effects or a cure.
3. The Valsalva maneuver will help the client Content – Medical: Integrated Nursing Process –
to empty the bladder, but assessment comes Evaluation: Client Needs – Physiological Integrity, Phar-
before teaching. macological and Parenteral Therapies: Cognitive Level –
4. Determining the client’s normal voiding Synthesis: Concept – Cellular Regulation.
patterns provides a baseline for the nurse
55. 1. This intervention addresses the prostate can-
and client to use when setting goals.
cer but not the metastatic process of bone
TEST-TAKING HINT: Assessment is the first involvement.
step of the nursing process. In any ques- 2. There is no reason to keep the foot of the bed
tion requiring the test taker to choose the elevated.
first action, an answer o­ ption with the words 3. Bone metastasis is very painful, and the
“check,” “assess,” or “determine” should be client should be placed on a scheduled
considered as a possible answer. regimen of pain medication. Pain medica-
Content – Medical: Integrated Nursing Process – ­ tion slows peristalsis and causes consti-
Implementation: Client Needs – Safe Effective Care Envi- pation. The client should be placed on a
ronment, Management of Care: Cognitive Level – routine bowel management program to
Application: Concept – Cellular Regulation. prevent impactions.

10_Colgrove_Ch10.indd 420 01/06/16 5:46 PM


C hapter 10  R eproductive D isorders 421

4. This does not address the metastasis to the described in the procedure. Pain is a priority,
bone. but the test taker should read to see if an-
TEST-TAKING HINT: The test taker must other option has a higher priority.
decide which intervention addresses both the Content – Medical: Integrated Nursing Process – ­
cancer and the metastasis to the bone. Only Implementation: Client Needs – Safe Effective Care Envi-
one (1) option does this. ronment, Management of Care: Cognitive Level –
Synthesis: Concept – Perioperative.
Content – Medical: Integrated Nursing Process –
­Implementation: Client Needs – Physiological Integrity, 58. 200 mL/hr.
­Physiological Adaptation: Cognitive Level – Application: IV pumps work on the principle of number
Concept – Cellular Regulation. of milliliters per hour to infuse, and, un-
56. 1. Cryotherapy involves placing freezing less otherwise specified, IVPB medications
probes into the prostate to freeze the are infused over a 30-minute time frame.
cancer cells. An indwelling catheter is Sixty (60) minutes in one (1) hour divided
placed into the urethra, and warm water is by 30 = 2. Two (2) times the volume of
circulated through the catheter to try to 100 mL = 200 mL, the rate at which the
prevent the urethra from freezing. If the nurse should set the pump.
urethra scars, then the lumen will con- TEST-TAKING HINT: The test taker must know
strict, causing retention of urine. basic rules for medication administration and
2. Ejaculation difficulties are caused by be able to compute simple math equations.
­obstruction from tumor growth. Content – Surgical: Integrated Nursing Process –
3. Bone marrow suppression is caused by ­Implementation: Client Needs – Physiological Integrity,
­radiation or chemotherapy. Pharmacological and Parenteral Therapies: Cognitive
4. Cryotherapy does not cause chronic vomiting Level – Application: Concept – Medication.
or diarrhea. 59. 1. The nurse should never ask “why.” The client
TEST-TAKING HINT: The test taker should does not owe the nurse an explanation.
dissect the word “cryotherapy.” Cryo- means 2. The question asks for a therapeutic
“to freeze,” so cryotherapy indicates some response from the nurse. This response
form of cold therapy. Then the test taker is restating and clarifying.
should decide what cold therapy could cause 3. This is dismissive to the client’s feelings and
to happen in the area being treated. appears to agree with the client. There is no
Content – Surgical: Integrated Nursing Process – evidence cancer of the prostate is caused by
Diagnosis: Client Needs – Physiological Integrity, Reduction promiscuous behavior.
of Risk Potential: Cognitive Level – Analysis: Concept – 4. This is a problem-solving statement and does
Cellular Regulation. not address the client’s feelings.
57. 1. Pain does not have priority over a fluid and TEST-TAKING HINT: Because the question is
electrolyte imbalance. asking for a therapeutic response, the test
2. There is no dressing for a transurethral taker should address the client’s feelings.
resection. The body cavity is entered through However, the test taker should not use this
the penis. test-taking hint as an absolute because each
3. Early ambulation prevents several complica- question should be answered on its own merit.
tions, but the immediate complication is elec- Content – Medical: Integrated Nursing Process – ­
trolyte imbalance and fluid overload. Implementation: Client Needs – Psychosocial Integrity:
4. With irrigation of the surgical site Cognitive Level – Application: Concept – Cellular
through the indwelling three (3)-way Regulation.
catheter to prevent blood clots, fluids may 60. 1. This is a psychosocial problem.
be absorbed through the open surgical 2. This is a physiological problem.
site and retained. This can lead to fluid 3. This problem has both physiological
volume overload and electrolyte ­imbalance and psychosocial implications.
(hyponatremia). 4. This is a physiological problem.
TEST-TAKING HINT: This client is eight (8)
TEST-TAKING HINT: The test taker should read
hours postoperative, so the test taker could the stem carefully. The stem asks for a physi-
eliminate preventing DVT (option “3”). ological and psychosocial problem. Options
Option “2” could be eliminated if the “1,” “2,” and “4” can be sorted into only one
test taker looks at the surgical approach of the categories.

10_Colgrove_Ch10.indd 421 01/06/16 5:46 PM


422 M ed -S urg S uccess

Content – Medical: Integrated Nursing Process – 63. 1. The client usually will be able to maintain
Diagnosis: Client Needs – Safe Effective Care Environ- normal sexual functioning with the remain-
ment, Management of Care: Cognitive Level – Analysis: ing testicle. If not, testosterone may be pre-
Concept – Sexuality. scribed to improve functioning. Ejaculation
is not the problem; it is impotence (erectile
dysfunction).
Testicular Disorders 2. This will not be done until the HCP deter-
mines the remaining testicle is not able to
61. 1. The body temperature should be warm for
maintain adequate hormone production.
the scrotum to relax. The best place to per-
3. The client will be able to function sexually
form testicular self-examination is in a warm
with the remaining testicle alone or with pre-
to hot shower.
scribed male hormones.
2. The client may note a cordlike structure;
4. Sex drive (libido) and orgasms usually are
this is the spermatic cord and is normal.
unimpaired because the client still has one
Any lump or mass felt is abnormal and
testicle.
should be checked by an HCP as soon as
possible. TEST-TAKING HINT: The key to this question
3. The client can confirm his own negative is “unilateral.” The client will still have one
result; a negative result is no masses felt. functioning testicle.
4. The procedure is painless. If pain is elicited, Content – Surgical: Integrated Nursing Process –
then an HCP should examine the client. Planning: Client Needs – Physiological Integrity, Physi-
Cancer is usually painless; the presence of ological Adaptation: Cognitive Level – Synthesis:
pain may indicate an infection. Concept – Male Reproduction.

TEST-TAKING HINT: The test taker might 64. 1. Epididymitis is inflammation of the epididymis,
choose option “2” as an answer because any usually caused by bacteria descending from the
abnormality should be examined by an HCP. prostate or bladder. This does not increase the
Content – Medical: Integrated Nursing Process – risk of developing cancer of the testes.
Planning: Client Needs – Health Promotion and Mainte- 2. Cryptorchidism is the medical term for
nance: Cognitive Level – Synthesis: Concept – Promoting undescended testicle. The testicles may be
Health. in the abdomen or inguinal canal at birth.
This condition places the client at higher
62. 1. The usual treatment for testicular cancer is
risk for testicular cancer.
removal of the involved testicle followed by
3. Prostate enlargement occurs as men age,
radiation to the area and chemotherapy.
whereas the most common age range for
Every attempt is made to shield the remain-
testicular cancer is age 15 to 35.
ing testicle from the radiation, but sterility
4. Hypospadias is a congenital abnormality in
sometimes occurs.
which the urethral meatus is on the underside
2. With artificial insemination the client may
of the penis. This does not increase the risk
be able to father children, if the client has
for cancer.
banked his sperm.
3. Sperm banking will allow the client to TEST-TAKING HINT: This is a knowledge-
father children through artificial insemi- based question, but answer options “3” and
nation with the client’s sperm. “4” could be eliminated based on anatomical
4. The nurse is in the client’s room. The client’s position.
presence implies consent for the nurse to dis- Content – Medical: Integrated Nursing Process –
cuss his case with the fiancée. Diagnosis: Client Needs – Physiological Integrity, Reduction
of Risk Potential: Cognitive Level – Analysis: Concept –
TEST-TAKING HINT: The nurse must abide by
Cellular Regulation.
HIPAA rules. It is important for the nurse
to know and understand the confidentiality 65. 1. The scrotum will require support during
laws. Options “1” and “2” are opposites, so ambulation. An athletic supporter is de-
only one could be correct or neither may be signed to provide support in this area.
correct. ­Option “3” actually gives an option 2. The client should be encouraged to take pain
that lies b
­ etween the extremes. medications before the pain is at a high level
Content – Medical: Integrated Nursing Process – to help the pain medication be more effective.
­Implementation: Client Needs – Psychosocial Integrity: 3. The client can be on a regular diet as soon as
Cognitive Level – Application: Concept – Male the client is not nauseated from the anesthesia.
Reproduction. 4. This surgery does not increase bleeding times.

10_Colgrove_Ch10.indd 422 01/06/16 5:46 PM


C hapter 10  R eproductive D isorders 423

TEST-TAKING HINT: Option “2” can be elimi- 68. 1. Pain when urinating indicates a urinary tract
nated because it states the opposite of the infection.
instructions given to clients about pain 2. A chancre sore on the shaft of the penis indi-
medication. cates syphilis, a sexually transmitted disease.
Content – Surgical: Integrated Nursing Process – 3. Classic signs of cancer of the testes are a
­Implementation: Client Needs – Safe Effective Care Envi- mass on the testicle, painless enlargement
ronment, Management of Care: Cognitive Level – Analysis: of the testes, and heaviness of the scrotum
Concept – Perioperative. or lower abdomen.
66. 1. Increasing the drip rate on a continuous 4. There is no rash associated with cancer of the
bladder (Murphy) irrigation requires nursing testes.
judgment and cannot be delegated. TEST-TAKING HINT: The test taker could
2. In this situation, checking for infection is eliminate option “2” because the penis and
an assessment situation. The nurse cannot testes are separate body parts. The testicles
­delegate assessment. lie within the scrotum. Only one (1) option
3. The unlicensed assistive personnel can concerns that part of the body.
be asked to help a client turn, cough, and Content – Medical: Integrated Nursing Process –
deep breathe. This requires the UAP to Assessment: Client Needs – Physiological Integrity, Reduc-
perform an action only, not to use judg- tion of Risk Potential: Cognitive Level – Analysis:
ment or to assess. Concept – Cellular Regulation.
4. Red drainage in the catheter implies bleeding. 69. 1. The nurse should not impose any personal
The nurse should assess the amount of bleed- religious beliefs on the client.
ing occurring. 2. Testicular cancers have very good progno-
TEST-TAKING HINT: The test taker should ses, and even if the tumor returns, there is
choose the lowest level activity when choos- a good prognosis for extended survival.
ing an action to have unlicensed assistive 3. There is a great deal of hope to offer these
personnel perform. Blood should always be clients.
assessed. 4. This is giving the client incorrect information.
Content – Nursing Management: Integrated Nursing TEST-TAKING HINT: This is a question in
Process – Planning: Client Needs – Safe Effective Care which the test taker must know the informa-
Environment, Management of Care: Cognitive Level – tion. Rarely can a nurse provide a specific
Synthesis: ­Concept – Management. percentage of survival.
67. 1. Chlamydia is a sexually transmitted dis- Content – Medical: Integrated Nursing Process – ­
ease usually silent in the male partner, but Implementation: Client Needs – Psychosocial Integrity:
it can cause epididymitis. If both sexual Cognitive Level – Application: Concept – Cellular
partners are not treated, then the partner Regulation.
can reinfect the client. 70. 1. CA-125 is a tumor marker used for a number
2. Sexual intercourse can be resumed within a of cancers but not for testicular cancer.
couple of weeks as long as both partners have 2. CEA is a nonspecific tumor marker used for
been treated for the infection. a number of cancers, but it does not apply to
3. This indicates a urinary tract infection. testicular cancer.
A UTI is another cause of epididymitis. 3. The DNA ploidy test is used to determine
4. The scrotum is a closed cavity. There should the number of chromosomes and arrange-
not be any drainage from the scrotum. If there ment of chromosomes in a tumor cell. It is
is drainage, this could indicate a fistula and used for prognosis in some cancers, such as
requires immediate notification of the HCP. breast cancer, but not as a tumor marker to
TEST-TAKING HINT: The test taker should determine response to treatment.
always look at the time frame provided in 4. Tumor markers are substances synthe-
­either the stem or the answer options as a sized by the tumor and released into the
clue to answering the question correctly. The bloodstream. They can be used to follow
test taker should look closely at any option the progress of the disease. Testicular can-
that has a “month” time frame. cers secrete hCG and alpha-fetoprotein.
Content – Medical: Integrated Nursing Process – TEST-TAKING HINT: This is a knowledge-based
Planning: Client Needs – Physiological Integrity, Physi- question. If the test taker were aware of what
ological Adaptation: Cognitive Level – Synthesis: the test tells the HCP, then the DNA ploidy
Concept – Sexuality. test could be eliminated. The other options
are all ­tumor markers.

10_Colgrove_Ch10.indd 423 01/06/16 5:46 PM


424 M ed -S urg S uccess

Content – Medical: Integrated Nursing Process – 3. The more sexual partners, the greater the
Diagnosis: Client Needs – Physiological Integrity, Reduction risk for contracting an STD.
of Risk Potential: Cognitive Level – Analysis: Concept – 4. Condom use provides a barrier to con-
Cellular Regulation. tracting an STD, but it is not a guarantee.
71. 1. This information could signal the onset of The condom can break or come off during
symptoms of metastasis to the retroperi- intercourse.
toneum. The HCP should see the client TEST-TAKING HINT: In option “2” the word
and discuss follow-up diagnostic tests. “­guarantees” appears, and the nurse cannot
2. This information should be investigated and guarantee anything in dealing with health-
not put off. care issues. Option “4” is an absolute
3. This may or may not have occurred. Only statement—“will not get”—and can be
diagnostic tests can confirm metastasis. eliminated on this basis.
4. Low back exercises will not help the client if Content – Medical: Integrated Nursing Process –
this is metastatic cancer, and they could in- Planning: Client Needs – Health Promotion and Mainte-
crease the pain. nance: Cognitive Level – Synthesis: Concept – Promoting
TEST-TAKING HINT: The nurse cannot diag- Health.
nose the client as having metastasis, so 74. 1. This is a true statement, but it does not answer
­option “3” can be eliminated. As a rule, any the client’s question.
pain unrelieved with pain medication requires 2. Untreated HPV infection is a cause for
the client to see the HCP. developing cancer of the cervix.
Content – Medical: Integrated Nursing Process – 3. HPV infection does not invade the abdominal
­Implementation: Client Needs – Physiological Integrity, cavity and therefore does not cause ­ovarian
Reduction of Risk Potential: Cognitive Level – Analysis: cancer.
Concept – Cellular Regulation. 4. The Pap test was developed to note early cell
72. 1. This should be done for clients who have had changes in the cervix. It indirectly monitors
scrotal surgery. the effects of HPV, but it does not help pre-
2. Postoperative clients do not want to perform vent problems.
deep-breathing and coughing exercises be- TEST-TAKING HINT: The test taker should
cause it hurts, but they should be encouraged choose the answer for the question the client
to do so to prevent complications. is asking. Option “1” discusses transmission
3. The Jackson Pratt (JP) drain is a drain and option “4” discusses prevention; there-
­attached to a bulb, and the bulb should re- fore, these two (2) options could be eliminated
main compressed to apply gentle ­suction based on the stem of the question.
to the surgical site. Content – Medical: Integrated Nursing Process –
4. This is appropriate delegation. ­Implementation: Client Needs – Safe Effective Care Envi-
TEST-TAKING HINT: The test taker could ronment, Management of Care: Cognitive Level – Analysis:
eliminate option “4” by knowing the rules of Concept – Sexuality.
delegation. The nurse did not ask the UAP to 75. 1. These are signs of second-stage syphilis.
perform anything requiring nursing judgment The nurse should ask about the develop-
or assessment. Option “2” is just good nursing ment of a chancre sore, one of the first
practice and could be eliminated. The charge signs of a syphilis infection.
nurse does not need to stop this action. 2. This may be required of the public health
Content – Nursing Management: Integrated Nursing nurse for notification of the partners, but it
Process – Evaluation: Client Needs – Safe Effective Care is not required to assess this problem.
Environment, Management of Care: Cognitive Level – 3. This client does not need a dermatologist to
Synthesis: Concept – Perioperative. determine an STD infection. The HCP can
treat this infection.
4. A urine culture will not diagnose this disease.
Sexually Transmitted Diseases
TEST-TAKING HINT: If the test taker is aware
73. 1. Engaging in oral and anal sex increases the the symptoms are those of an STD, options
risk of contracting an STD. “3” and “4” can be eliminated.
2. Using a sterile needle for drug abuse ensures Content – Medical: Integrated Nursing Process –
the client will not get an STD from needle ­Implementation: Client Needs – Physiological Integrity,
sharing, but the client can still contract an Reduction of Risk Potential: Cognitive Level – Analysis:
STD from other risky behaviors. Concept – Sexuality.

10_Colgrove_Ch10.indd 424 01/06/16 5:46 PM


C hapter 10  R eproductive D isorders 425

76. 1. All socioeconomic levels of clients contract TEST-TAKING HINT: Two (2) answer options
STDs. mention male anatomy. If the test taker did
2. The longer the client abstains from sexual not know the information, then choosing
activity and the fewer partners the client has, between these two (2) options might be the
the less the risk of an STD. appropriate method of elimination.
3. Females and males can spread STDs equally. Content – Medical: Integrated Nursing Process –
Specific diseases may be asymptomatic in Assessment: Client Needs – Physiological Integrity, Reduc-
the sexes (in females, gonorrhea; in males, tion of Risk Potential: Cognitive Level – Analysis:
chlamydia) and they can transmit them Concept – Sexuality.
unknowingly. 79. 1. Pelvic inflammatory disease (PID) does not
4. If the client has one STD, there is a great have to be reported, but the cause of the PID
likelihood the client has another disease may need to be reported.
also. If one STD is found, the client should 2. There are causes for epididymitis other than
be monitored for others. an STD.
TEST-TAKING HINT: Option “2” does not make 3. Syphilis is an STD and therefore must
sense: If sexual activity is put off, there can- be reported to the appropriate health
not be an increased risk. Socioeconomic rea- department.
sons may be a reason for delaying treatment 4. An ectopic pregnancy may have numerous
of a disease, but diseases are not financially causes.
based and occur in all socioeconomic levels. TEST-TAKING HINT: Only one (1) answer option
Content – Medical: Integrated Nursing Process – is an STD. The other diseases/conditions may
Evaluation: Client Needs – Physiological Integrity, Physi- be caused by STDs, but they all have other
ological Adaptation: Cognitive Level – Synthesis: causes as well.
Concept – Sexuality.
Content – Medical: Integrated Nursing Process – ­
77. 1. Chlamydia does not cause an antigen–­ Implementation: Client Needs – Safe Effective Care Envi-
antibody reaction. ronment, Management of Care: Cognitive Level –
2. There are long-term problems associated Application: Concept – Sexuality.
with any STD. Chlamydia may have the 80. 1. Determining and diagnosing the risk for
long-term effects of chronic pain and in- infertility problems requires collaboration
creased risk for ectopic pregnancy, postpar- between the nurse and the HCP.
tum endometritis, and infertility. 2. The nurse is required to teach a client. This is
3. If both the client and the sexual partner an independent action.
are not treated simultaneously, the sexual 3. Fluid volume deficit is not an appropriate
partner can reinfect the client. nursing diagnosis for this client.
4. The client may develop chronic pelvic pain as 4. Noncompliance is an independent nursing
a result of the infection. problem.
TEST-TAKING HINT: Options “2” and “4” have TEST-TAKING HINT: The question requires the
a form of absolute. The words “any,” “will,” test taker to determine which are autono-
or “will not” are absolutes and in health care, mous functions of the nurse. The nurse does
there are very few absolutes. not have the capability to prescribe fertility
Content – Medical: Integrated Nursing Process – medications or treatments.
Planning: Client Needs – Physiological Integrity, Physi-
Content – Medical: Integrated Nursing Process –
ological Adaptation: Cognitive Level – Synthesis:
Diagnosis: Client Needs – Safe Effective Care Environ-
Concept – Sexuality.
ment, Management of Care: Cognitive Level – Analysis:
78. 1. A chancre sore is a symptom of syphilis, not Concept – Sexuality.
gonorrhea. 81. 1. Vaginal cultures are obtained to assess for
2. Gonorrhea is more likely to be asymptomatic gonorrhea and chlamydia.
in females. 2. The RPR test and the Venereal Disease
3. A CD4 count of less than 200 is a diagnostic Research Laboratory (VDRL) test are
indicator for acquired immunodeficiency diagnostic tests for syphilis.
syndrome (AIDS). 3. Gram stains of the vagina or urethral meatus
4. Pain in the testes and scrotal edema can of a male may be done for gonorrhea but not
indicate epididymitis, an inflammatory for syphilis.
process of the epididymis. This and ure- 4. An immunological assay may be done for
thritis are the most common presenting chlamydia but not for syphilis.
symptoms in a male with gonorrhea.

10_Colgrove_Ch10.indd 425 01/06/16 5:46 PM


426 M ed -S urg S uccess

TEST-TAKING HINT: The test taker must 3. The nurse can teach some information about
memorize the tests used to diagnose specific reporting, but the nurse is not qualified to
STDs and the symptoms differentiating one discuss all the legal aspects of reporting an
STD from another. STD.
Content – Medical: Integrated Nursing Process – 4. The nurse is not responsible for analyzing
Diagnosis: Client Needs – Physiological Integrity, Reduction statistics.
of Risk Potential: Cognitive Level – Analysis: Concept – TEST-TAKING HINT: Answer options “2,” “3,”
Sexuality. and “4” ask the nurse to take on roles not
82. 1. Lymphadenopathy and hair loss are symp- within the nurse’s expertise. The nurse must
toms of secondary syphilis, not tertiary know the Nurse Practice Act of the state
syphilis. where the nurse practices. No state allows
2. Genital warts are not signs of tertiary syphilis. the nurse to give legal or medical advice.
3. Aortitis and neurosyphilis (dementia, Content – Medical: Integrated Nursing Process –
psychosis, stroke, paresis, and meningitis) Diagnosis: Client Needs – Safe Effective Care Environ-
are the most common manifestations of ment, Management of Care: Cognitive Level –
tertiary syphilis. Comprehension: Concept – Sexuality.
4. A rash covering the body is a symptom of 84. 1. An odorless, white, curdlike vaginal discharge
gonorrhea. is a symptom of Candida albicans, a vaginal
TEST-TAKING HINT: The key word in this yeast infection.
question is “tertiary.” The test taker must 2. A strawberry spot on the vaginal wall or
decide which disease has three (3) distinct cervix, a fishy smelling vaginal discharge,
phases and then which symptoms accompany and itching are symptoms of trichomonas.
each phase. 3. Scant white vaginal discharge and dyspareu-
Content – Medical: Integrated Nursing Process – nia are symptoms of atrophic vaginitis.
Assessment: Client Needs – Physiological Integrity, Reduc- 4. Purulent discharge from the endocervix and
tion of Risk Potential: Cognitive Level – Analysis: pelvic pain are symptoms of cervicitis.
Concept – Sexuality. TEST-TAKING HINT: When studying for a
83. 1. The public health nurse is responsible for test covering similar diseases, the test taker
attempting to notify sexual partners of a should concentrate on the information that
client diagnosed with an STD of a poten- makes one different from another. Only one
tial infection and urging the partner to be STD has a characteristic strawberry spot.
tested for the disease and to receive treat- Content – Medical: Integrated Nursing Process –
ment. Health departments offer confiden- Assessment: Client Needs – Physiological Integrity, Reduc-
tial testing and treatment. tion of Risk Potential: Cognitive Level – Analysis.
2. An HCP will determine the course of treat- Concept –Reproduction: Concept – Sexuality.
ment for a client diagnosed with an STD.

10_Colgrove_Ch10.indd 426 01/06/16 5:46 PM


CONCEPTS

85. 1. A general rule when discussing antibiotics is Maintenance: Cognitive Level – Synthesis: Concept –
to teach clients to finish all of the prescrip- Female Reproduction.
tion, but this is not specific information 87. 1. The nurse does not have to inform the
­related to STIs. parents of the teenager’s disclosure of
2. Most STIs must be reported to comply with information.
public health laws, but this is not a long-term 2. The nurse should discuss birth control
complication of having an STI. and sexually transmitted diseases with the
3. Because of scarring of reproductive tissue, client. She is at risk for pregnancy and
infertility may be an issue resulting from STDs.
STI infection. 3. The nurse should discuss birth control
4. Acquired immunodeficiency syndrome and sexually transmitted diseases with the
(AIDS) is a sexually transmitted disease and client. She is at risk for pregnancy and
can also be transmitted by non–sexual blood STDs.
and body fluid exposure. 4. The male wears the most commonly used
TEST-TAKING HINT: The test taker can rule condoms but both partners are respon-
out option “1” because of the generalized sible for contraception and prevention
nature of the option; it is not specific to of STDs. This information will assist the
STDs. Option “2” does not address a long- client to know if the device is correctly
term complication for the client. And option applied and will have the best chance of
“4” requires the test taker to know the trans- preventing both pregnancy and STDs.
mission of the disease. 5. This would be information to provide if
Content – Medical: Integrated Nursing Process – the client had an STD, but is not needed at
Planning: Client Needs – Health Promotion and Mainte- this time.
nance: Cognitive Level – Synthesis: Concept – 6. The client is not pregnant at this time.
Reproduction/Sexuality.
TEST-TAKING HINT: The test taker could elim-
86. 1. A Gravida 7 Para 6 client has had seven (7) inate options “5” and “6” because the client
pregnancies and carried six (6) of those is not currently pregnant.
pregnancies to 20 weeks or longer. This Content – Medical: Integrated Nursing Process –
woman is at risk of having the fundus re- Planning: Client Needs – Health Promotion and Mainte-
main boggy and not diminishing in size, nance: Cognitive Level – Synthesis: Concept – Sexuality.
resulting in excessive bleeding. The nurse
88. 1. The DRE is performed after the PSA to
should assess the fundus hourly.
avoid false elevation of the PSA from the
2. Voiding should be assessed related to delivery
manipulation of the prostate during DRE.
time and position of the fundus, not just every
2. The PSA is drawn before the DRE to pre-
12 hours (a normal shift).
vent a false elevation due to the prostate
3. It is not priority immediately postdelivery to
being massaged during the DRE.
discuss birth control options.
3. Prostate ultrasound is performed if a nodule
4. If the client wishes to breastfeed her infant
is found or suspected and cannot be localized
at this time, with her history she can perform
on DRE. It would not be anticipated as the
this without teaching.
first action by the HCP.
TEST-TAKING HINT: The test taker could 4. A biopsy would be done if all the other data
eliminate option “2” because of the words are positive, but not first.
“every shift.” Timing words such as “hourly,”
TEST-TAKING HINT: The test taker could
“every day,” “­every 2 hours” can make an
eliminate options “3” and “4” because they
option correct or incorrect. The stem of the
are more invasive than a laboratory test or a
question said the delivery was two (2) hours
digital examination.
ago; this requires the nurse to determine
Content – Medical: Integrated Nursing Process –
which intervention should be performed
Planning: Client Needs – Health Promotion and Mainte-
immediately postdelivery.
nance: Cognitive Level – Synthesis: Concept – Sexuality.
Content – Medical: Integrated Nursing Process –
Assessment: Client Needs – Health Promotion and

427

10_Colgrove_Ch10.indd 427 02/06/16 5:50 PM


428 M ed -S urg S uccess

89. 1. Failure to have the baby to express all of 4. Acetaminophen is a mild analgesic or anti-
the milk produced will result in less milk pyretic medication; this could be adminis-
produced at future feedings and for the tered if needed after the antibiotic.
breasts to become sore and inflamed. TEST-TAKING HINT: The test taker could
2. Ice packs are usually applied to decrease the eliminate options “1” and “2” because the
discomfort. guidelines allow up to one (1) hour after
3. Rubbing alcohol will dry out the breast tis- the scheduled time to ­administer these
sues and the infant should not feed after alco- medications.
hol has been applied to the breast because of Content – Medical: Integrated Nursing Process –
absorption into the infant’s system. Planning: Client Needs – Pharmacological and Parenteral
4. The mother should be taught correct Therapies: ­Cognitive Level – Synthesis: Concept –
breastfeeding techniques, not to avoid Medication.
breastfeeding.
92. 1. The nurse should administer the chemo-
TEST-TAKING HINT: The test taker could
therapy as ordered because all listed labo-
eliminate option “3” because the infant could ratory values are WNL.
ingest the ­alcohol during feeding. 2. The laboratory data are WNL; the oncologist
Content – Medical: Integrated Nursing Process – can see the laboratory report on rounds.
­Implementation: Client Needs – Health Promotion and 3. The laboratory data are WNL; there is no
Maintenance: Cognitive Level – Synthesis: Concept –
need for cultures because an infection is not
Female Reproduction.
indicated.
90. 1. Mastalgia (breast pain and tenderness is 4. The laboratory data are WNL; no blood is
common during the menstrual cycle. This needed.
question assists the nurse in determin- TEST-TAKING HINT: The test taker should be
ing if the mastalgia is from the fibrocystic able to read and interpret reports so that
disease. appropriate nursing decisions can be made.
2. Some women find that avoiding chocolate Content – Medical: Integrated Nursing Process –
and stimulants (caffeine) reduces the discom- ­Implementation: Client Needs – Pharmacological and
fort of fibrocystic breasts, but it is not consid- Parenteral Therapies: Cognitive Level – Synthesis:
ered an allergy. Concept – Sexuality.
3. This is a therapeutic response, not an
­interviewing question to be asked during 93. 1. According to the ACS beginning at age 50 a
the a­ ssessment of the disease. PSA level followed by a DRE should be done
4. This is not precancerous; there is no reason yearly.
for a mastectomy. 2. According to the ACS beginning at age 50 a
PSA level followed by a DRE should be done
TEST-TAKING HINT: The test taker could
yearly.
eliminate option “3” because a therapeutic 3. According to the ACS beginning at age 50
response was not needed; the nurse needs to a PSA level followed by a DRE should be
ask a question that assists in gaining infor- done yearly.
mation about the disease being assessed. 4. According to the ACS beginning at age 50 a
Content – Medical: Integrated Nursing Process – PSA level followed by a DRE should be done
Assessment: Client Needs – Health Promotion and Mainte- yearly.
nance: Cognitive Level – Synthesis: Concept – Sexuality.
TEST-TAKING HINT: The test taker must have a
91. 1. Digoxin is a routine maintenance medication level of knowledge concerning basic nursing
and the nurse can begin the medication from activities and information to teach a client.
0800 to 1000, an hour before or after the Content – Medical: Integrated Nursing Process –
0900 time scheduled. ­Implementation: Client Needs – Health Promotion and
2. Furosemide is a routine maintenance medica- Maintenance: ­Cognitive Level – Comprehension:
tion and the nurse can begin the medication Concept – Sexuality.
from 0800 to 1000, an hour before or after
94. 1. Failure of the baby to latch onto the breasts is
the 0900 time scheduled.
not the priority for the nurse to assess.
3. An initial dose of IV antibiotic should be
2. There is nothing that indicates this baby will
considered a STAT or Now medication.
have jaundice and/or clay-colored stools.
The nurse should make sure that any or-
3. The baby would have parchment–like skin and
dered cultures are obtained and the medi-
lack lanugo if the baby were over 40 weeks
cation initiated within one (1) hour of the
prescription being written. gestation. This is not stated in the stem.

10_Colgrove_Ch10.indd 428 01/06/16 5:46 PM


C hapter 10  R eproductive D isorders 429

4. The neonate is high birth weight and the 2. After each breast is palpated the last thing
mother had gestational diabetes. This done is to pinch the nipples to see if there
infant had a high glucose content passing is a discharge.
through the placenta in utero and the in- TEST-TAKING HINT: The test taker must re-
fant’s pancreas has been producing insulin member basic guidelines for procedures and
to take care of the glucose content of the which steps are involved.
blood. The infant’s pancreas must adjust
Content – Medical: Integrated Nursing Process –
to lower levels of glucose in the system. ­Implementation: Client Needs – Health Promotion and
TEST-TAKING HINT: The test taker could ­Maintenance: Cognitive Level – Cognition: Concept –
eliminate option “2” because this is associ- Female Reproduction.
ated with liver or gallbladder issues and a
basic knowledge of the pathophysiology of 96. 1. Women at high risk for breast cancer
post–maturity gestation could eliminate should have the opportunity to have mam-
option “3.” mograms beginning earlier than recom-
mended for routine screening.
Content – Medical: Integrated Nursing Process –
Assessment: Client Needs – Health Promotion and Main-
2. According to 2015 guidelines, routine
tenance: Cognitive Level – Synthesis: Concept – Female screening mammograms should begin
Reproduction. at age 45 and be performed yearly until
age 55. At age 55 mammograms should be
95. In order of performance: 5, 3, 4, 1, 2. performed biennially.
5. Breast self-examination should be per- 3. Beginning in their 20s, women should be told
formed in a private area with good light- about the benefits and limitations of breast self-
ing and a comfortable temperature so the examination (BSE). Even those who choose not
woman is warm and can be consistent in to do BSE should be aware of how their breasts
the steps. normally look and feel and report any new
3. The first part of BSE is performed in breast changes to a health professional as soon
front of a mirror. The breasts are exam- as they are found. Finding a breast change does
ined from the front and each side, then not necessarily mean there is a cancer. BSE is
with arms down by the side, then raised, performed monthly, not bimonthly.
next with the arms on the hips and bend- 4. MRIs are performed on women with special
ing over. The woman is looking for dim- needs (breast implants, etc.) but are not per-
pling and irregularity in size or shape. formed on a routine basis.
4. The woman then gets into a warm shower
TEST-TAKING HINT: The test taker could
and palpates each breast when the breasts
eliminate option “3” based on the timing
are soapy and slippery.
“bimonthly.”
1. The last position the woman assumes is
lying flat on the bed with a towel behind Content – Medical: Integrated Nursing Process –
­Implementation: Client Needs – Health Promotion and
each scapula and palpation is performed.
Maintenance: Cognitive Level – Cognition: Concept –
Sexuality.

10_Colgrove_Ch10.indd 429 01/06/16 5:46 PM


430 M ed -S urg S uccess

REPRODUCTIVE DISORDERS
COMPREHENSIVE EXAMINATION
1. The nurse is reviewing the laboratory data on a male client. Which interpretation should the nurse make
regarding the prostate-specific antigen (PSA)?

Laboratory Test Client Value Normal Value


Prostate-specific antigen 6 mcg/L Male: less than 4 mcg/L
Female: less than 0.5 mcg/L

1. The client has early-stage prostate cancer.


2. The client should have more tests.
3. The client does not have prostate cancer.
4. The client has benign prostatic hypertrophy.
2. The nurse is performing the admission assessment 3. A dull pain in the left breast and tough, doughy
on a 78-year-old female client and observes feeling skin.
bilateral pendulous breasts with a stringy 4. Bloody discharge from the nipple and a hard
appearance. Which intervention should the nurse palpable mass.
implement?
5. The client has a diagnosis of rule-out Paget’s
1. Request a mammogram.
disease. Which test provides a definitive
2. Notify the HCP of the finding.
diagnosis of the disease?
3. Continue with the examination.
1. A breast biopsy.
4. Assess for peau d’orange skin.
2. A diagnostic mammogram.
3. The client is scheduled for a right breast biopsy 3. Ultrasound of the breast.
for a mass found in the tail of Spence. While the 4. Magnetic resonance imaging.
client is waiting in the holding area, the client asks
6. The nurse in the gynecology clinic is assessing the
the nurse, “Which lymph nodes will my surgeon
50-year-old client who has had four (4) children
take from my body?” Which area should the nurse
and is complaining of having lower abdominal
identify?
pressure and fatigue along with some urinary
incontinence. Which instruction should the nurse
teach the client?
1. Wear a peri-pad to keep from having an
A accident.
B
2. Try not to laugh or sneeze unless at home.
3. Discuss the pros and cons of a vaginal
hysterectomy.
4. Instruct to perform Kegel exercises.
7. The client is diagnosed with a rectovaginal
fistula which is to be managed medically. Which
information should the nurse teach the client
prior to discharge?
C D
1. Douche with normal saline.
1. A 2. Eat a low-residue diet.
2. B 3. Keep ice packs to the area.
3. C 4. Use an abdominal binder.
4. D
8. The client has an infected Bartholin’s cyst and
4. The client is diagnosed with left mastitis. Which the HCP has performed an incision and drainage
assessment findings should the nurse observe? (I&D) of the area. Which discharge instructions
1. Dimpling of the left breast when the client should the nurse teach the client?
raises the arm. 1. Complete all antibiotics as ordered.
2. A round lump in the left breast that is tender 2. Report any drainage immediately.
during menses. 3. Keep all water away from the area.
4. Lie prone as much as possible.

10_Colgrove_Ch10.indd 430 01/06/16 5:46 PM


C hapter 10  R eproductive D isorders 431

9. The client is diagnosed with vulvar cancer. 13. The nurse is discharging a client diagnosed
Which are the most common symptoms of with pelvic inflammatory disease (PID).
cancer of the vulva? Which statement by the client indicates an
1. Red, painful lesions. understanding of the discharge instructions?
2. Vulvar itching. 1. “I should expect pelvic pain after intercourse.”
3. Thin, white vulvar skin. 2. “I need to douche every day to prevent PID.”
4. Vaginal dryness. 3. “I will have a vaginal examination every
two (2) years.”
10. The nurse is teaching a class to older women
4. “My partner should use a condom if he is
concerning cancer of the uterus. Which situation
infectious.”
is a risk factor for developing endometrial
cancer? 14. The client has failed to conceive after many
1. Age 40 years or younger. attempts over a three (3)-year time period and
2. Perimenopausal bleeding. asks the nurse, “I have tried everything. What
3. Progesterone given with estrogen. should I do now?” Which statement is the
4. Truncal obesity. nurse’s best response?
1. “By ‘everything’ do you mean you have
11. The nurse is caring for a client diagnosed
consulted an infertility specialist?”
with uterine cancer who has received afterload
2. “You have tried everything. This must be hard
intracavitary radiation. Which precaution should
for you. Would you like to talk?”
the nurse implement?
3. “You should get on an adoption list because it
1. Wear rubber gloves to protect the nurse from
can take a long time.”
all exposure.
4. “You need to relax and not try so hard. It is
2. Allow any visitor the client wishes to see.
your nerves preventing conception.”
3. Minimize the amount of time spent with the
client. 15. The nurse writes a problem of “potential
4. Encourage the client to ambulate in the for complications related to ovarian
hallway. hyperstimulation” for a client who is taking
clomiphene (Clomid), an ovarian stimulant.
12. The nurse is caring for client A, who is
Which intervention should be included in the
postoperative after a breast biopsy. The
plan of care?
pathology report is posted on the chart. Which
1. Instruct the client to delay intercourse until
statement is the interpretation of the DNA
menses.
ploidy pathology report?
2. Schedule the client for frequent pelvic
1. This is Stage IV breast cancer with a poor
sonograms.
prognosis.
3. Explain the infusion therapy will take 21 days.
2. The cancer will respond to hormonal therapy.
4. Discuss that this may cause an ectopic
3. The chromosomes do not resemble normal
pregnancy.
human DNA.
4. The client should have a mastectomy as soon 16. The HCP orders cultures of the urethral
as possible. urine, bladder urine, and prostatic fluid. Which
instructions should the nurse teach to achieve
Client A Client Number: 001234567 the first two (2) specimens?
A 0.5-  1.2-cm specimen of tissue from the 1. Collect the first 15 mL in one jar and then the
right breast was evaluated with microscope next 50 mL in another.
examination. Margins were not clear. Tissue 2. Collect three (3) early-morning, clean voided
sample indicates high-grade invasive ductal cell urine specimens.
carcinoma. DNA ploidy by flow cytometry re- 3. Collect the specimens after the HCP
veals aneuploid characteristics. massages the prostate.
Findings: Invasive ductal cell carcinoma of the 4. Collect a routine urine specimen for analysis.
right breast with aneuploid findings.

10_Colgrove_Ch10.indd 431 01/06/16 5:46 PM


432 M ed -S urg S uccess

17. The nurse writes a client problem of “anxiety 22. The male client complains of mucus-like
related to potential sexual dysfunction” for a drainage from the rectum accompanied by rectal
client diagnosed with cancer of the prostate. pain and diarrhea. Which interview question
Which intervention should the nurse implement? should the nurse ask the client?
1. Tell the client to discuss his fears with 1. “Do you have difficulty trying to urinate?”
the HCP. 2. “Have you had rectal sexual intercourse?”
2. Talk to the wife about the client’s concerns. 3. “Do you eat a high-fiber diet and drink lots
3. Inform the client sexual functioning will not of fluids?”
be altered. 4. “Does the diarrhea alternate with
4. Provide a private area for the client to discuss constipation?”
his concerns.
23. The client is diagnosed with primary syphilis.
18. The male client is considering a vasectomy for Which symptoms should the nurse observe?
birth control. Which information should the 1. A chancre sore in the perineal area.
nurse teach the client? 2. A rash on the trunk and extremities.
1. Instruct the client to use hot packs to relieve 3. Blistering of the palms of the hands.
scrotal edema after the surgery. 4. Confusion and disorientation.
2. Tell the client to wear loose-fitting boxer
24. The nurse is discussing pelvic floor exercises
underwear after the surgery.
with a client. Which information should the
3. Explain initially an alternate form of birth
nurse teach?
control will be required.
1. Perform the exercises four (4) times per day.
4. Discuss potency will be diminished about
2. The exercises will prevent stress incontinence.
20% after a vasectomy.
3. Contract the perineal muscles and hold for
19. The 45-year-old male client has had a 10 seconds.
circumcision secondary to phimosis. Which 4. Contract the abdominal and buttock muscles
intervention should the nurse include in the to increase strength.
plan of care?
25. The county health department nurse is
1. Teach how to care for the glans to prevent
reviewing a client record and notes the RPR
recurrence of the phimosis.
laboratory results. Which question should the
2. Assess for pain on a scale of one (1) to 10.
nurse ask the client?
3. Perform wet-to-dry dressing changes daily.
4. Instruct the client to perform a monthly penis
Laboratory Client Normal
check for cancer. Test Result Finding
20. Which vaccination should the nurse recommend Rapid plasma Reactive for Nonreactive
to the postpubertal male to prevent orchitis? reagin (RPR) Treponema
1. Yearly flu injections. pallidum
2. Herpes varicella inoculations.
3. Mumps vaccination. 1. “When was your last tetanus shot?”
4. Rubella injections. 2. “Have you had a cold recently?’
3. “Do you have diabetes mellitus?”
21. The nurse is instructing a group of workers at 4. “Are you allergic to penicillin?”
an industrial plant regarding the transmission
of sexually transmitted diseases (STDs).
Which information should be included in the
presentation?
1. The same behaviors causing one STD could
lead to another.
2. Once clients have had an STD, they develop
immunity to it.
3. An infection with syphilis protects the client
from being infected with HIV.
4. Herpes simplex 1 is a totally different disease
from herpes simplex 2.

10_Colgrove_Ch10.indd 432 01/06/16 5:46 PM


C hapter 10  R eproductive D isorders 433

26. The nurse is teaching a class on Breast Health 27. The 32-year-old client diagnosed with an
Awareness. Which are the American Cancer ovarian cyst is complaining of acute pain. Which
Society’s recommended guidelines for the picture depicts the area where the client would
performance of breast self-examination (BSE)? be experiencing pain?
List in order of recommended performance. 1.
1. Visualize the breast from the front while
standing before a mirror.
2. Gently squeeze the nipple to express any fluid.
3. Turn to each side and view each breast in the
mirror.
4. Palpate each breast in a circular motion while
lying on the back.
5. Palpate each breast in a circular motion while
in the shower. 2.

3.

4.

10_Colgrove_Ch10.indd 433 01/06/16 5:46 PM


REPRODUCTIVE DISORDERS COMPREHENSIVE
EXAMINATION ANSWERS AND RATIONALES

1. 1. The client may have cancer of the prostate, but 4. 1. Dimpling of the breast indicates a tumor has
this test does not provide conclusive results. attached itself to the chest wall.
There are several reasons for the PSA to be 2. This indicates fibrocystic changes in the left
elevated, not just cancer. breast.
2. The PSA is elevated and more tests should 3. Mastitis is an infection of the breast occur-
be completed to determine the cause. PSA ring most often in women who are lactat-
levels are increased in benign prostatic ing. The breast becomes red and warm to
hypertrophy, urinary retention, prostatic touch. The skin becomes doughy and tough
infarct, and prostate cancer. in consistency, and the client develops a
3. Cancer cannot be eliminated as a diagnosis un- dull pain in the affected breast.
til other tests have been completed. 4. Bloody discharge indicates a tumor, benign or
4. This may be the actual diagnosis, but the cli- malignant.
ent should undergo more tests to confirm a Content – Medical: Integrated Nursing Process –
diagnosis. Assessment: Client Needs – Physiological Integrity, ­Reduction
Content – Medical: Integrated Nursing Process – of Risk Potential: Cognitive Level – A
­ nalysis: Concept –
Assessment: Client Needs – Physiological Integrity, ­Reduction Female Reproduction.
of Risk Potential: Cognitive Level – ­Analysis: Concept –
5. 1. Biopsy of the lesion is the only definitive
Cellular Regulation.
test for Paget’s disease, a form of breast
2. 1. These are normal findings in a postmenopausal cancer accounting for about 1% of all
breast and do not require a mammogram. The breast cancers.
client should have a mammogram yearly. 2. Mammography is the only test that routinely
2. These are normal findings in the postmeno- screens for breast cancer, but a definitive diag-
pausal breast so there is no need to notify the nosis is made by tissue identification.
HCP. 3. Ultrasound of the breasts can diagnose fluid-
3. These are normal findings in the post- filled cysts.
menopausal breast. Glandular tissue is 4. Magnetic resonance imaging can be done to
replaced with fibrous tissue, the breasts determine the extent of tumor involvement,
become pendulous, and the Cooper’s but tissue identification is the definitive test for
­ligaments become prominent. tumor diagnosis.
4. Peau d’orange skin occurs in advanced breast Content – Medical: Integrated Nursing Process –
cancer. Diagnosis: Client Needs – Physiological Integrity, ­Reduction
Content – Medical: Integrated Nursing Process – of Risk Potential: Cognitive Level – ­Analysis: Concept –
Implementation: Client Needs – Safe Effective Care Envi- Cellular Regulation.
ronment, Management of Care: Cognitive Level –
6. 1. The client would have determined the need
Application: Concept – Female Reproduction.
for protection without the nurse having to tell
3. 1. The tail of Spence is the upper outermost her to do so.
part of the breast, which extends toward 2. It is unrealistic to tell a client not to laugh or
the arm. The most likely lymph nodes to sneeze.
biopsy are the axillary nodes. 3. The client probably has a cystocele resulting
2. This is the mediastinal node area, which is on from childbirth. The corrective surgical repair
the opposite side of the breast. is a bladder suspension.
3. The internal mammary nodes are under the 4. Kegel exercises help to strengthen the pel-
breast, and the tail of Spence is at the top of vic muscles. They are recommended for all
the breast. women and should be performed 30 to 80
4. The parasternal nodes are on the opposite side times per day.
of the breast from the tail of Spence. Content – Medical: Integrated Nursing Process –
Content – Medical: Integrated Nursing Process – Planning: Client Needs – Physiological Integrity, Physiological
Implementation: Client Needs – Safe Effective Care Envi- Adaptation: Cognitive Level – Synthesis: Concept – Female
ronment, Management of Care: Cognitive Level – Reproduction.
Application: Concept – Female Reproduction.

434

10_Colgrove_Ch10.indd 434 01/06/16 5:46 PM


C hapter 10  R eproductive D isorders 435

7. 1. Cleansing douches are prescribed with tepid Content – Medical: Integrated Nursing Process –
water, not normal saline. Planning: Client Needs – Health Promotion and Main-
2. Measures to assist the client to heal with- tenance: Cognitive Level – Synthesis: Concept – Cellular
out surgical interventions include proper Regulation.
nutrition with a low-residue diet to mini- 11. 1. Rubber gloves should be worn to dispose of
mize contamination of the tissues with any soiled material, but they will not protect
feces, cleansing douches, enemas, and rest. the nurse from sealed radiation sources.
3. Warm perineal irrigations and controlled 2. Visitors who may be pregnant or who are
heat-lamp applications promote healing; ice younger than 18 years old should not be
vasoconstricts the area and delays wound ­allowed in the client’s room.
healing. 3. Afterload intracavitary radiation therapy
4. The client should wear perineal pads but not treatments are completed in the client’s
an abdominal binder. room after prepared applicators are
Content – Medical: Integrated Nursing Process – placed in surgery. This minimizes ex-
Planning: Client Needs – Physiological Integrity, Physiologi- posure of the health-care workers to
cal Adaptation: Cognitive Level – Synthesis: Concept – radiation. The nurse should plan care to
Female Reproduction. minimize exposure to the client and the
8. 1. The client has an infection and should radiation.
complete the ordered antibiotics. 4. The client is placed in a room at the far end
2. The client should be taught to expect some of the hallway, and in some facilities clients
drainage from the area because the area has on either side of the client’s room may have
been opened to allow for exudate to escape to be moved to limit exposure to radiation.
the body. The client is not allowed to leave the room
3. Routine hygiene is encouraged. until the radiation safety department clears
4. The client can assume any position of comfort. the client for discharge.
Content – Surgical: Integrated Nursing Process – Content – Medical: Integrated Nursing Process –
Planning: Client Needs – Physiological Integrity, Physiologi- Implementation: Client Needs – Safe Effective Care Envi-
cal Adaptation: Cognitive Level – Synthesis: Concept – ronment, Management of Care: Cognitive Level –
Female Reproduction. Application: Concept – Cellular Regulation.

9. 1. Red, painful lesions are symptoms of lichen 12. 1. Staging for breast cancer is completed using a
planus, which is a benign, although uncom- variety of measurements, including size of the
fortable, lesion. tumor and metastasis sites, but staging does
2. Cancer of the vulva may be asymptomatic, not involve the DNA characteristics.
but the client usually presents with persis- 2. The DNA ploidy tests are performed to
tent long-term itching. determine the response to all types of treat-
3. Thin, white vulvar skin indicates lichen ments. The more the cell represents normal
sclerosis. human DNA pairings (euploid), the better
4. Vaginal dryness is not associated with cancer the prognosis for the client.
of the vulva. 3. Aneuploid means the cells do not have
Content – Medical: Integrated Nursing Process –
human pairing characteristics. This
Assessment: Client Needs – Physiological Integrity, ­finding indicates the cells cannot be
­Reduction of Risk Potential: Cognitive Level – ­Analysis: ­expected to respond as normal human
Concept – Cellular Regulation. cells respond and the prognosis is not
good for the client.
10. 1. Clients at risk for uterine cancer are usually 4. The choice of surgical approach is deter-
55 years old or older. mined between the client and HCP. The
2. Postmenopausal bleeding places the client at tumor should be removed with enough tis-
risk, but perimenopausal bleeding is expected sue to have clear margins. Whether the cli-
as the ovaries begin to slow production of eggs. ent chooses a modified radical mastectomy,
3. Unopposed estrogen replacement therapy lumpectomy, or wedge resection depends on
predisposes women to developing uterine the choice of follow-up treatment.
cancer, but progesterone offsets the risk.
Content – Medical: Integrated Nursing Process –
4. Truncal obesity is one of the risk ­factors
Assessment: Client Needs – Physiological Integrity,
for developing endometrial cancer, ­Reduction of Risk Potential: Cognitive Level – ­Analysis:
although it has not been determined Concept – Cellular Regulation.
how this occurs.

10_Colgrove_Ch10.indd 435 01/06/16 5:46 PM


436 M ed -S urg S uccess

13. 1. Any pelvic pain after sexual exposure, child- this is the urethral urine specimen. Then,
birth, or pelvic surgery should be evaluated as the client voids the next 50 to 75 mL into
soon as possible. another sterile specimen cup; this is the
2. Douching reduces the natural flora, which bladder urine. If the client does not have
combats infecting organisms and may help acute prostatitis, the HCP can massage
infecting bacteria move upward into the the prostate and collect prostate fluid
uterus, but douching will not prevent PID. for culture, but if fluid is not expressed
3. The client should have a vaginal examination with massage, the next urine is sent for
at least once a year. analysis.
4. The client and partner should consistently 2. Early morning has no bearing on collecting
use a condom if there is any chance of prostatitis specimens.
transmission of any organism. 3. This is the third specimen and is obtained by
Content – Medical: Integrated Nursing Process – the HCP.
Evaluation: Client Needs – Physiological Integrity, 4. A routine specimen will not provide the in-
Physiological Adaptation: Cognitive Level – Synthesis: formation needed.
Concept – Female Reproduction. Content – Medical: Integrated Nursing Process –
14. 1. The nurse should investigate which fertil- Implementation: Client Needs – Physiological Integrity,
Reduction of Risk Potential: Cognitive Level – Synthesis:
ity measures have been attempted. There
Concept – Male Reproduction.
are many reasons for infertility, and only
a specialist in the area can identify the 17. 1. This is not an appropriate referral. The nurse
cause. should discuss the client’s fears.
2. This is a therapeutic response, but the client 2. The wife may need to be encouraged to talk
is asking for information. about her concerns, but this does not address
3. This is advising and not answering the client’s the client’s concerns.
question. 3. The client’s sexual functioning may be im-
4. The nurse cannot know this to be true, and paired depending on the treatment options
this does not address the client’s concern. chosen.
Content – Medical: Integrated Nursing Process – 4. Because the client may be sensitive and
Implementation: Client Needs – Safe Effective Care Envi- embarrassed about discussing problems
ronment, Management of Care: Cognitive Level – Analysis: related to genitalia and sexual functioning,
Concept – Female Reproduction. the nurse should provide a private area in
which to discuss these concerns.
15. 1. Clomid is begun on the fifth day of the men-
strual cycle and is taken for five (5) days; ovu- Content – Medical: Integrated Nursing Process –
lation should occur four (4) to eight (8) days Implementation: Client Needs – Psychosocial Integrity:
Cognitive Level – Application: Concept – Cellular
after—approximately from day nine (9) to day
Regulation.
13 of the cycle. Intercourse should be planned
for the optimum chance of conception. 18. 1. Ice packs, not hot packs, should be applied
2. Frequent sonograms are needed to moni- after surgery to reduce edema.
tor follicular stimulation. The ovaries are 2. The client should wear cotton, jockey-type
monitored to prevent overstimulation, underwear to provide support and added
which can cause ascites, pleural effusions, comfort.
and acute respiratory distress syndrome 3. The client will not be sterile until the
(ARDS). sperm stored distal to the surgery site
3. Clomid is not given by infusion. Gonadotropin- in the tubules have been ejaculated or
releasing hormone (GnRH) is given by reabsorbed by the body. Therefore, an
infusion. alternate form of birth control is needed
4. Ovarian stimulation does not cause ectopic for a certain period of time following the
pregnancy. vasectomy.
Content – Medical: Integrated Nursing Process – 4. There is no effect on the client’s potency after
Planning: Client Needs – Safe Effective Care ­Environment, a bilateral vasectomy.
Management of Care: Cognitive Level – Synthesis: Content – Surgical: Integrated Nursing Process –
Concept – Female Reproduction. Planning: Client Needs – Safe Effective Care Environment,
Management of Care: Cognitive Level – Synthesis:
16. 1. After cleansing the penis and retracting
Concept – Male Reproduction.
the foreskin (if present), the client voids
the first 15 mL in a sterile specimen cup;

10_Colgrove_Ch10.indd 436 01/06/16 5:46 PM


C hapter 10  R eproductive D isorders 437

19. 1. Phimosis is a tightness of the prepuce of the Proctitis is commonly associated with
penis preventing retraction of the foreskin anal-receptive intercourse with an
over the glans. Once the foreskin has been infected partner. The pathogens most
surgically removed, this is not a problem. frequently associated with proctitis are
2. There is considerable pain after an adult gonorrhea, chlamydia, herpes simplex,
circumcision, and the nurse should assess and Treponema pallidum.
for the pain. 3. A high-fiber diet does not cause these
3. A petroleum-saturated gauze is wrapped symptoms.
around the penis, not a wet-to-dry dressing. 4. Diarrhea alternating with constipation in-
4. Circumcision in infancy may prevent cancer dicates a possible rectal tumor and does not
of the penis. The older client who has a cir- cause these symptoms.
cumcision should be aware of the potential Content – Medical: Integrated Nursing Process –
for cellular changes, but the risk is greatly Assessment: Client Needs – Physiological Integrity,
reduced after the surgery. ­Reduction of Risk Potential: Cognitive Level – ­Analysis:
Content – Surgical: Integrated Nursing Process – Concept – Sexuality.
Diagnosis: Client Needs – Safe Effective Care E
­ nvironment, 23. 1. A chancre sore on the perineal area is a
Management of Care: Cognitive Level – Analysis:
symptom of primary syphilis.
Concept – Male Reproduction.
2. A rash on the trunk and extremities occurs in
20. 1. The flu vaccine will not prevent orchitis. secondary syphilis.
2. This is the chickenpox vaccine and will not 3. Blistering of the palms of the hand occurs in
prevent orchitis. secondary syphilis.
3. When postpubertal males contract 4. Tertiary syphilis occurs over a prolonged
mumps, one (1) in five (5) develops some period and includes symptoms of dementia,
form of orchitis (infection of the testes) psychosis, paresis, stroke, and meningitis.
within four (4) to seven (7) days after the Content – Medical: Integrated Nursing Process –
neck and jaw swell. The testes may show Assessment: Client Needs – Physiological Integrity,
atrophy after the infection, and the client ­Reduction of Risk Potential: Cognitive Level – ­Analysis:
may become sterile. Concept – Sexuality.
4. Rubella (measles) does not cause orchitis. 24. 1. Pelvic floor (Kegel) exercises should be
Content – Medical: Integrated Nursing Process – ­performed 30 to 80 times per day.
Planning: Client Needs – Health Promotion and Main- 2. The exercises will help reduce stress incon-
tenance: Cognitive Level – Synthesis: Concept – Male
tinence, but they may not relieve all stress
Reproduction.
incontinence.
21. 1. The behaviors leading to the development 3. Perineal muscles should be contracted and
of one STD could also lead to the devel- held for 10 seconds followed by 10 seconds
opment of another. of rest.
2. There is no antigen–antibody reaction devel- 4. The pelvic floor muscles are contracted with-
opment with STDs. A client can be reinfected out contracting the abdominal, buttock, or
multiple times. inner-thigh muscles.
3. There is no protection provided by one STD Content – Medical: Integrated Nursing Process –
from developing another, and, frequently, Planning: Client Needs – Physiological Integrity, Physiologi-
clients will have more than one STD cal Adaptation: Cognitive Level – Synthesis: Concept –
simultaneously. Female Reproduction.
4. Herpes simplex 1 and 2 are caused by the 25. 1. This is a test for syphilis and does not require
same virus. Herpes simplex 1 refers to oro- a tetanus injection.
labial lesions and herpes simplex 2 refers to 2. Having a cold does not affect the diagnosis of
genital lesions, which can be transferred from syphilis.
one area to the other. 3. Diabetes places a client at risk for many ill-
Content – Medical: Integrated Nursing Process – nesses, but sexual behavior places the client at
Planning: Client Needs – Safe Effective Care ­Environment, risk for a sexually transmitted disease.
Safety and Infection Control: Cognitive Level – Synthesis:
4. The test is positive for syphilis, and the
Concept – Promoting Health.
nurse anticipates the HCP ordering an
22. 1. The client has rectal symptoms, not urinary antibiotic; penicillin is the antibiotic of
tract symptoms. choice for syphilis. Syphilis is caused by
2. The client has described symptoms of the Treponema pallidum bacteria.
proctitis (inflammation of the rectum).

10_Colgrove_Ch10.indd 437 01/06/16 5:46 PM


438 M ed -S urg S uccess

Content – Medical: Integrated Nursing Process – Content – Medical: Integrated Nursing Process –
Assessment: Client Needs – Physiological Integrity, Implementation: Client Needs – Health Promotion and
­Reduction of Risk Potential: Cognitive Level – ­Analysis: Maintenance: Cognitive Level – Analysis:
Concept – Sexuality. Concept – Promoting Health.

26. The order should be: 1, 3, 5, 4, 2. 27. 1. The midabdominal area is where clients diag-
1. The first step in BSE is to visualize the nosed with peptic ulcers complain of pain.
breasts for symmetry while looking at a 2. The right upper quadrant is where clients
frontal view before the mirror. with gallbladder problems complain of pain.
3. The next step is to turn from side to side, 3. The lower abdominal quadrants are where
looking for any dimpling, puckering, or a client with an ovarian cyst will complain
asymmetry, in front of a mirror. of pain. The appendix is also located in
5. The client should palpate the breasts in the right lower quadrant.
a warm shower with the breasts soaped 4. The left upper abdominal quadrant is where
to allow for the fingers to glide over the clients with spleen infarcts or rupture, stom-
breast tissue. ach problems, or pancreas problems complain
4. After the shower, the client should lie on of pain.
the bed with a towel rolled up and placed Content – Surgical: Integrated Nursing Process –
under the shoulder to flatten the breast Assessment: Client Needs – Physiological Integrity,
tissue and palpate the breast. Physiological Adaptation: Cognitive Level – Synthesis:
2. The last step in BSE is to gently squeeze Concept – Female Reproduction.
the nipple to determine if there is ex-
pressed fluid.

10_Colgrove_Ch10.indd 438 01/06/16 5:46 PM


Musculoskeletal
Disorders
It is impossible for a man to learn what he thinks he already knows.
11
—Epictetus

Musculoskeletal injuries and disorders are common. Many, such as fractures, result from
­accidents; others, such as osteoarthritis, osteoporosis, and joint replacement, are often
­associated with the aging process; and still others, such as amputation, may result as a com-
plication of a specific disease process such as diabetes. Nurses must know how to assess these
clients, when to notify the health-care providers, and how to carry out the proper proce-
dures and administer the medications ordered.

KEYWORDS ABBREVIATIONS
Abduction Above-the-knee amputation (AKA)
Avulsion Activated partial thromboplastin time (aPTT)
Collaborative Activities of daily living (ADLs)
Comminuted Below-the-knee amputation (BKA)
Compound Blood pressure (BP)
Degenerative Computed tomography (CT)
Epiphyseal Continuous passive motion (CPM)
Greenstick Erythrocyte sedimentation rate (ESR)
Herniated nucleus pulposus Health-care provider (HCP)
Impacted Immediately (STAT)
Laminectomy Intravenous (IV)
Oblique Intravenous pyelogram (IVP)
Paresthesia Magnetic resonance imaging (MRI)
Pathological Nonsteroidal anti-inflammatory drugs (NSAIDs)
Primary care Osteoarthritis (OA)
Prostheses Patient-controlled analgesia (PCA)
Rhinitis Rule out (R/O)
Secondary care Total hip replacement (THR)
Tertiary care Total knee replacement (TKR)
Unlicensed assistive personnel (UAP)
When required, as needed (prn)

439

11_Colgrove_Ch11.indd 439 01/06/16 5:53 PM


440 M ed -S urg S uccess

PRACTICE QUESTIONS
Degenerative/Herniated Disk Disease 5. The client with a cervical neck injury as a result
of a motor-vehicle accident is complaining of
1. The nurse is caring for an elderly client diagnosed unrelieved pain after administration of a narcotic
with a herniated nucleus pulposus of L4-5. Which analgesic. Which alternative method of pain
scientific rationale explains the incidence of a control is an independent nursing action?
ruptured disk in the elderly? 1. Medicate the client with a muscle relaxant.
1. The client did not use good body mechanics 2. Heat alternating with ice applied by a physical
when lifting an object. therapist.
2. There is an increased blood supply to the back 3. Watch television or listen to music.
as the body ages. 4. Discuss surgical options with the health-care
3. Older clients develop atherosclerotic joint provider.
disease as a result of fat deposits. 6. The client diagnosed with cervical disk
4. Clients develop intervertebral disk degeneration has undergone a laminectomy.
degeneration as they age. Which interventions should the nurse implement?
2. The 34-year-old male client presents to the 1. Position the client prone with the knees
outpatient clinic complaining of numbness and slightly elevated.
pain radiating down the left leg. Which further 2. Assess the client for difficulty speaking or
data should the nurse assess? breathing.
1. Posture and gait. 3. Measure the drainage in the Jackson Pratt bulb
2. Bending and stooping. every day.
3. Leg lifts and arm swing. 4. Encourage the client to postpone the use
4. Waist twists and neck mobility. of narcotic medications.
3. The occupational health nurse is preparing an 7. The client is 12-hours post–lumbar laminectomy.
in-service for a group of workers in a warehouse. Which nursing interventions should be
Which information should be included to help implemented?
prevent on-the-job injuries? 1. Assess ability to void and log roll the client
1. Increase sodium and potassium in the diet every two (2) hours.
during the winter months. 2. Medicate with IV steroids and keep the bed in
2. Use the large thigh muscles when lifting and a Trendelenburg position.
hold the weight near the body. 3. Place sandbags on each side of the head and
3. Use soft-cushioned chairs when performing give cathartic medications.
desk duties. 4. Administer IV anticoagulants and place on O2
4. Have the employee arrange for assistance with at eight (8) L/min.
household chores. 8. The nurse is working with an unlicensed assistive
4. The occupational health nurse is planning personnel (UAP). Which action by the UAP
health promotion activities for a group of factory warrants immediate intervention?
workers. Which activity is an example of primary 1. The UAP feeds a client two (2) days
prevention for clients at risk for low back pain? postoperative cervical laminectomy a
1. Teach back exercises to workers after returning regular diet.
from an injury. 2. The UAP calls for help when turning to the
2. Place signs in the work area about how to side a client who is post–lumbar laminectomy.
perform first aid. 3. The UAP is helping the client who weighs
3. Start a weight-reduction group to meet at 300 pounds and is diagnosed with back pain
lunchtime. to the chair.
4. Administer a nonnarcotic analgesic to a client 4. The UAP places the call light within reach of
complaining of back pain. the client who had a disk fusion.

11_Colgrove_Ch11.indd 440 01/06/16 5:53 PM


C hapter 11   M usculoskeletal D isorders 441

9. The nurse is caring for clients on an orthopedic 14. The client is diagnosed with osteoarthritis.
floor. Which client should be assessed first? Which sign/symptom should the nurse expect
1. The client diagnosed with back pain who is the client to exhibit?
complaining of a “4” on a 1-to-10 scale. 1. Severe bone deformity.
2. The client who has undergone a myelogram 2. Joint stiffness.
who is complaining of a slight headache. 3. Waddling gait.
3. The client two (2) days post–disk fusion who 4. Swan-neck fingers.
has T 100.4, P 96, R 24, and BP 138/78.
15. The client diagnosed with OA is a resident
4. The client diagnosed with back pain who is
in a long-term care facility. The resident is
being discharged and whose ride is here.
refusing to bathe because she is hurting. Which
10. The nurse is administering 0730 medications instruction should the nurse give the unlicensed
to clients on a medical orthopedic unit. Which assistive personnel (UAP)?
medication should be administered first? 1. Allow the client to stay in bed until the pain
1. The daily cardiac glycoside to a client becomes bearable.
diagnosed with back pain and heart failure. 2. Tell the UAP to give the client a bed bath this
2. The routine insulin to a client diagnosed with morning.
neck strain and type 1 diabetes. 3. Try to encourage the client to get up and go
3. The oral proton pump inhibitor to a client to the shower.
scheduled for a laminectomy this a.m. 4. Notify the family the client is refusing to be
4. The fourth dose of IV antibiotic for a client bathed.
diagnosed with a surgical infection.
16. The client has been diagnosed with OA for the
11. The nurse writes the problem of “pain” for a last seven (7) years and has tried multiple medical
client diagnosed with lumbar strain. Which treatments and alternative treatments but still
nursing interventions should be included in the has significant joint pain. Which psychosocial
plan of care? Select all that apply. client problem should the nurse identify?
1. Assess pain on a 1-to-10 scale. 1. Severe pain.
2. Administer pain medication prn. 2. Body image disturbance.
3. Provide a regular bedpan for elimination. 3. Knowledge deficit.
4. Assess surgical dressing every four (4) hours. 4. Depression.
5. Perform a position change by the log roll
17. The client diagnosed with OA is prescribed a
method every two (2) hours.
nonsteroidal anti-inflammatory drug (NSAID).
12. The nurse working on a medical-surgical floor Which instruction should the nurse teach the
feels a pulling in the back when lifting a client client?
up in the bed. Which should be the first action 1. Take the medication on an empty stomach.
taken by the nurse? 2. Make sure to taper the medication when
1. Continue working until the shift is over and discontinuing.
then try to sleep on a heating pad. 3. Apply the medication topically over the
2. Go immediately to the emergency department affected joints.
for treatment and muscle relaxants. 4. Notify the health-care provider if vomiting
3. Inform the charge nurse and nurse manager blood.
on duty and document the occurrence.
18. Which client goal is most appropriate for a
4. See a private health-care provider on the
client diagnosed with OA?
nurse’s off time but charge the hospital.
1. Perform passive range-of-motion exercises.
2. Maintain optimal functional ability.
Osteoarthritis 3. Client will walk three (3) miles every day.
4. Client will join a health club.
13. The occupational health nurse is teaching a class 19. To which member of the health-care team
on the risk factors for developing osteoarthritis should the nurse refer the client diagnosed with
(OA). Which is a modifiable risk factor for OA who is complaining of not being able to get
developing OA? in and out of the bathtub?
1. Being overweight. 1. Physiatrist.
2. Increasing age. 2. Social worker.
3. Previous joint damage. 3. Physical therapist.
4. Genetic susceptibility. 4. Counselor.

11_Colgrove_Ch11.indd 441 01/06/16 5:53 PM


442 M ed -S urg S uccess

20. The nurse is discussing the importance of an Osteoporosis


exercise program for pain control to a client
diagnosed with OA. Which intervention should 25. The nurse is discussing osteoporosis with a
the nurse include in the teaching? group of women. Which factor will the nurse
1. Wear supportive tennis shoes with white socks identify as a nonmodifiable risk factor?
when walking. 1. Calcium deficiency.
2. Carry a complex carbohydrate while 2. Tobacco use.
exercising. 3. Female gender.
3. Alternate walking briskly and jogging when 4. High alcohol intake.
exercising.
4. Walk at least 30 minutes three (3) times a 26. The client diagnosed with osteoporosis asks the
week. nurse, “Why does smoking cigarettes cause my
bones to be brittle?” Which response by the
21. The HCP prescribes glucosamine and nurse is most appropriate?
chondroitin for a client diagnosed with OA. 1. “Smoking causes nutritional deficiencies,
What is the scientific rationale for prescribing which contribute to osteoporosis.”
this medication? 2. “Tobacco causes an increase in blood supply
1. It will help decrease the inflammation in to the bones, causing osteoporosis.”
the joints. 3. “Smoking low-tar cigarettes will not cause
2. It improves tissue function and retards your bones to become brittle.”
breakdown of cartilage. 4. “Nicotine impairs the absorption of calcium,
3. It is a potent medication which decreases the causing decreased bone strength.”
client’s joint pain.
4. It increases the production of synovial fluid in 27. Which signs/symptoms indicate to the nurse the
the joint. client has developed osteoporosis?
1. The client has lost one (1) inch in height.
22. The nurse is admitting the client with OA to 2. The client has lost 12 pounds in the last year.
the medical floor. Which statement by the 3. The client’s hands are painful to the touch.
client indicates an alternative form of treatment 4. The client’s serum uric acid level is elevated.
for OA?
1. “I take medication every two (2) hours for 28. The client is being evaluated for osteoporosis.
my pain.” Which diagnostic test is the most accurate
2. “I use a heating pad when I go to bed at when diagnosing osteoporosis?
night.” 1. X-ray of the femur.
3. “I wear a copper bracelet to help with my 2. Serum alkaline phosphatase.
OA.” 3. Dual-energy x-ray absorptiometry (DEXA).
4. “I always wear my ankle splints when I sleep.” 4. Serum bone Gla-protein test.

23. The client is complaining of joint stiffness, 29. Which foods should the nurse recommend
especially in the morning. Which diagnostic to a client when discussing sources of dietary
tests should the nurse expect the health-care calcium?
provider to order to R/O osteoarthritis? 1. Yogurt and dark-green, leafy vegetables.
1. Full-body magnetic resonance imaging scan. 2. Oranges and citrus fruits.
2. Serum studies for synovial fluid amount. 3. Bananas and dried apricots.
3. X-ray of the affected joints. 4. Wheat bread and bran.
4. Serum erythrocyte sedimentation rate (ESR). 30. Which intervention is an example of a
24. The nurse is caring for the following clients. secondary nursing intervention when discussing
After receiving the shift report, which client osteoporosis?
should the nurse assess first? 1. Obtain a bone density evaluation test.
1. The client with a total knee replacement who 2. Perform non–weight-bearing exercises
is complaining of a cold foot. regularly.
2. The client diagnosed with osteoarthritis who 3. Increase the intake of dietary calcium.
is complaining of stiff joints. 4. Refer clients to a smoking cessation program.
3. The client who needs to receive a scheduled
intravenous antibiotic.
4. The client diagnosed with back pain who is
scheduled for a lumbar myelogram.

11_Colgrove_Ch11.indd 442 01/06/16 5:53 PM


C hapter 11   M usculoskeletal D isorders 443

31. The female client diagnosed with osteoporosis 36. The client must take three (3) grams of calcium
tells the nurse she is going to perform swim supplement a day. The medication comes in
aerobics for 30 minutes every day. Which 500-mg tablets. How many tablets will the client
response is most appropriate by the nurse? need to take daily? _______
1. Praise the client for committing to do this
activity.
2. Explain to the client walking 30 minutes a day Amputation
is a better activity.
3. Encourage the client to swim every other day 37. The nurse instructs the client with a right BKA
instead of daily. to lie on the stomach for at least 30 minutes a
4. Discuss with the client how sedentary day. The client asks the nurse, “Why do I need
activities help prevent osteoporosis. to lie on my stomach?” Which statement is the
most appropriate statement by the nurse?
32. The client newly diagnosed with osteoporosis 1. “This position will help your lungs expand
is prescribed calcitonin by nasal spray. Which better.”
assessment data indicate to the nurse an adverse 2. “Lying on your stomach will help prevent
effect of the medication? contractures.”
1. The client complains of nausea and vomiting. 3. “Many times this will help decrease pain in
2. The client is drinking two (2) glasses of milk the limb.”
a day. 4. “The position will take pressure off your
3. The client has a runny nose and nasal itching. backside.”
4. The client has had numerous episodes of
nosebleeds. 38. The recovery room nurse is caring for a client
who has just had a left BKA. Which intervention
33. The nurse is teaching a class to pregnant should the nurse implement?
teenagers. Which information is most 1. Assess the client’s surgical dressing every
important when discussing ways to prevent two (2) hours.
osteoporosis? 2. Do not allow the client to see the residual
1. Take at least 1,200 mg of calcium supplements limb.
a day. 3. Keep a large tourniquet at the client’s bedside.
2. Eat foods low in calcium and high in 4. Perform passive range-of-motion exercises to
phosphorus. the right leg.
3. Osteoporosis does not occur until around
age 50 years. 39. The 62-year-old client diagnosed with type 2
4. Remain as active as possible until the baby diabetes who has a gangrenous right toe is being
is born. admitted for a below-the-knee amputation.
Which nursing intervention should the nurse
34. The 84-year-old client is a resident in a long- implement?
term care facility. Which intervention should 1. Assess the client’s nutritional status.
be implemented to help prevent complications 2. Refer the client to an occupational therapist.
secondary to osteoporosis? 3. Determine if the client is allergic to IVP dye.
1. Keep the bed in the high position. 4. Start a 22-gauge Angiocath in the right arm.
2. Perform passive range-of-motion exercises.
3. Turn the client every two (2) hours. 40. The male nurse is helping his friend cut wood
4. Provide nighttime lights in the room. with an electric saw. His friend cuts two fingers
of his left hand off with the saw. Which action
35. The client is taking calcium carbonate (Tums) should the nurse implement first?
to help prevent further development of 1. Wrap the left hand with towels and apply
osteoporosis. Which teaching should the nurse pressure.
implement? 2. Instruct the friend to hold his hand above his
1. Encourage the client to take Tums with at head.
least eight (8) ounces of water. 3. Apply pressure to the radial artery of the
2. Teach the client to take Tums with the left hand.
breakfast meal only. 4. Go into the friend’s house and call 911.
3. Instruct the client to take Tums 30 to
60 minutes before a meal.
4. Discuss the need to get a monthly serum
calcium level.

11_Colgrove_Ch11.indd 443 01/06/16 5:53 PM


444 M ed -S urg S uccess

41. A person’s right thumb was accidentally severed 3. Change the surgical dressing on the client
with an axe. The amputated right thumb was with a Syme’s amputation.
recovered. Which action by the nurse preserves 4. Ask the UAP to take the client to the physical
the thumb so it could possibly be reattached in therapy department.
surgery?
46. The client with a right AKA is being taught
1. Place the right thumb directly on some ice.
how to toughen the residual limb. Which
2. Put the right thumb in a glass of warm water.
intervention should the nurse implement?
3. Wrap the thumb in a clean piece of material.
1. Instruct the client to push the residual limb
4. Secure the thumb in a plastic bag and place
against a pillow.
on ice.
2. Demonstrate how to apply an elastic bandage
42. The Jewish client with peripheral vascular around the residual limb.
disease is scheduled for a left AKA. Which 3. Encourage the client to apply vitamin B12
question is most important for the operating to the surgical incision.
room nurse to ask the client? 4. Teach the client to elevate the residual limb at
1. “Have you made any special arrangements for least three (3) times a day.
your amputated limb?”
47. The 27-year-old client has a right above-the-
2. “What types of food would you like to eat
elbow amputation secondary to a boating
while you’re in the hospital?”
accident. Which statement to the rehabilitation
3. “Would you like a rabbi to visit you while you
nurse indicates the client has accepted the
are in the recovery room?”
amputation?
4. “Will you start checking your other foot at
1. “I am going to sue the guy who hit my boat.”
least once a day for cuts?”
2. “The therapist is going to help me get
43. The client is three (3) hours postoperative left retrained for another job.”
AKA. The client tells the nurse, “My left foot 3. “I decided not to get a prosthesis. I don’t
is killing me. Please do something.” Which think I need it.”
intervention should the nurse implement? 4. “My wife is so worried about me and I wish
1. Explain to the client his left leg has been she weren’t.”
amputated.
48. The 32-year-old male client with a traumatic left
2. Medicate the client with a narcotic analgesic
AKA is being discharged from the rehabilitation
immediately.
department. Which discharge instructions
3. Instruct the client on how to perform
should be included in the teaching? Select all
biofeedback exercises.
that apply.
4. Place the client’s residual limb in the
1. Report any pain not relieved with analgesics.
dependent position.
2. Eat a well-balanced diet and increase protein
44. The nurse is caring for a client with a right intake.
below-the-knee amputation. There is a large 3. Be sure to attend all outpatient rehabilitation
amount of bright red blood on the client’s appointments.
residual limb dressing. Which intervention 4. Encourage the client to attend a support
should the nurse implement first? group for amputations.
1. Notify the client’s surgeon immediately. 5. Stay at home as much as possible for the first
2. Assess the client’s blood pressure and pulse. couple of months.
3. Reinforce the dressing with additional
dressing.
4. Check the client’s last hemoglobin and
Fractures
hematocrit levels.
49. The client is taken to the emergency department
45. The nurse is caring for clients on a surgical unit. with an injury to the left arm. Which
Which nursing task is most appropriate for intervention should the nurse implement first?
the nurse to delegate to an unlicensed assistive 1. Assess the nailbeds for capillary refill time.
personnel (UAP)? 2. Remove the client’s clothing from the arm.
1. Help the client with a two (2)-day postop 3. Call radiology for a STAT x-ray of the
amputation put on the prosthesis. extremity.
2. Request the UAP double-check a unit of 4. Prepare the client for the application of a cast.
blood to be hung.

11_Colgrove_Ch11.indd 444 01/06/16 5:53 PM


C hapter 11   M usculoskeletal D isorders 445

50. The nurse is preparing the plan of care for the 3. Determine the client’s normal orientation
client with a closed fracture of the right arm. status.
Which problem is most appropriate for the 4. Monitor the client’s Buck’s traction.
nurse to identify?
55. The client admitted with a diagnosis of a
1. Risk for ineffective coping related to the
fractured hip who is in Buck’s traction is
inability to perform ADLs.
complaining of severe pain. Which intervention
2. Risk for compartment syndrome–related
should the nurse implement?
injured muscle tissue.
1. Adjust the patient-controlled analgesia (PCA)
3. Risk for infection related to exposed bone and
machine for a lower dose.
tissue.
2. Ensure the weights of the Buck’s traction are
4. Risk for complications related to
off the floor and hang freely.
compromised neurovascular status.
3. Raise the head of the bed to 45 degrees and
51. Which interventions should the nurse the foot to 15 degrees.
implement for the client diagnosed with an open 4. Turn the client on the affected leg using
fracture of the left ankle? Select all that apply. pillows to support the other leg.
1. Apply an immobilizer snugly to prevent
56. The nurse is providing discharge teaching to the
edema.
12-year-old with a fractured humerus and the
2. Apply an ice pack for 10 minutes and remove
parents. Which information should the nurse
for 20 minutes.
include regarding cast care?
3. Place the extremity in the dependent position
1. Keep the fractured arm at heart level.
to allow drainage.
2. Use a wire hanger to scratch inside the cast.
4. Obtain an x-ray of the ankle after applying the
3. Apply an ice pack to any itching area.
immobilizer.
4. Explain foul smells are expected occurrences.
5. Administer tetanus toxoid, 0.5 mL
intramuscularly, in the deltoid. 57. Which statement by the client diagnosed with
a fractured ulna indicates to the nurse the client
52. The nurse is caring for a client with a fractured
needs further teaching?
left tibia and fibula. Which data should the nurse
1. “I need to eat a high-protein diet to ensure
report to the health-care provider immediately?
healing.”
1. Localized edema and discoloration occurring
2. “I need to wiggle my fingers every hour to
hours after the injury.
increase circulation.”
2. Generalized weakness and increasing
3. “I need to take my pain medication before my
sensitivity to touch.
pain is too bad.”
3. Dorsalis pedal pulse cannot be located with a
4. “I need to keep this immobilizer on when
Doppler and increasing pain.
lying down only.”
4. Pain relieved after taking four (4) mg
hydromorphone, a narcotic analgesic. 58. The nurse is preparing the care plan for a
client with a fractured lower extremity. Which
53. The unlicensed assistive personnel (UAP)
outcome is most appropriate for the client?
reports a client with a fractured femur has
1. The client will maintain function of the leg.
“fatty globules” floating in the urinal. What
2. The client will ambulate with assistance.
intervention should the nurse implement first?
3. The client will be turned every two (2) hours.
1. Assess the client for dyspnea and altered
4. The client will have no infection.
mental status.
2. Obtain an arterial blood gas and order a 59. The nurse is caring for a client diagnosed with a
portable chest x-ray. fracture of the right distal humerus. Which data
3. Call the HCP for a ventilation/perfusion scan. indicate a complication? Select all that apply.
4. Instruct the UAP keep the client on strict 1. Numbness and mottled cyanosis.
bedrest. 2. Paresthesia and paralysis.
3. Proximal pulses and point tenderness.
54. The nurse is caring for an 80-year-old client
4. Coldness of the extremity and crepitus.
admitted with a fractured right femoral neck
5. Palpable radial pulse and functional
who is oriented 3 1. Which intervention should
movement.
the nurse implement first?
1. Check for a positive Homans’ sign.
2. Encourage the client to take deep breaths and
cough.

11_Colgrove_Ch11.indd 445 01/06/16 5:53 PM


446 M ed -S urg S uccess

60. An 88-year-old client is admitted to the 65. Which interventions should be included in the
orthopedic floor with the diagnosis of fractured discharge teaching for a client who had a total
pelvis. Which intervention should the nurse hip replacement? Select all that apply.
implement first? 1. Discuss the client’s weight-bearing limits.
1. Insert an indwelling catheter. 2. Request the client demonstrate use of assistive
2. Administer a Fleet’s enema. devices.
3. Assess the abdomen for bowel sounds. 3. Explain the importance of increasing activity
4. Apply Buck’s traction. gradually.
4. Instruct the client not to take medication
prior to ambulating.
Joint Replacements 5. Tell the client to ambulate with open-toed
house shoes.
61. The nurse is preparing the preoperative client 66. The nurse is caring for the client who has had
for a total hip replacement (THR). Which a total hip replacement. Which data indicate the
intervention should the nursing implement surgical treatment is effective?
postoperatively? 1. The client states the pain is at a “3” on a 1-to-
1. Keep an abduction pillow in place between 10 scale.
the legs at all times. 2. The client has a limited ability to ambulate.
2. Cough and deep breathe at least every four (4) 3. The client’s left leg is shorter than the right leg.
to five (5) hours. 4. The client ambulates to the bathroom.
3. Turn to both sides every two (2) hours to
prevent pressure ulcers. 67. The nurse is caring for a client six (6) hours
4. Sit in a high-seated chair for a flexion of less postoperative right total knee replacement. Which
than 90 degrees. data should the nurse report to the surgeon?
1. A total of 100 mL of red drainage in the
62. The client one (1) day postoperative total hip autotransfusion drainage system.
replacement complains of hearing a “popping 2. Pain relief after using the patient-controlled
sound” when turning. Which assessment data
analgesia (PCA) pump.
should the nurse report immediately to the
3. Cool toes, distal pulses palpable, and pale
surgeon? nailbeds bilaterally.
1. Dark red-purple discoloration. 4. Urinary output of 60 mL of clear yellow urine
2. Equal length of lower extremities. in three (3) hours.
3. Groin pain in the affected leg.
4. Edema at the incision site. 68. The client who had a total knee replacement
is being discharged home. To which
63. The nurse is discharging a client who had a total multidisciplinary team member should
hip replacement. Which statement indicates the nurse refer the client?
further teaching is needed? 1. The occupational therapist.
1. “I should not cross my legs because my hip
2. The physiatrist.
may come out of the socket.” 3. The recreational therapist.
2. “I will call my HCP if I have a sudden
4. The home health nurse.
increase in pain.”
3. “I will sit on a chair with arms and a firm seat.” 69. The nurse is caring for a client with a right total
4. “After three (3) weeks, I don’t have to worry knee repair. Which intervention should the
about infection.” nurse implement?
1. Monitor the continuous passive motion
64. The nurse finds small, fluid-filled lesions on the machine.
margins of the client’s surgical dressing. Which 2. Apply thigh-high TED hose bilaterally.
statement is the most appropriate scientific 3. Place the abductor pillow between the legs.
rationale for this occurrence? 4. Encourage the family to perform ADLs for
1. These were caused by the cautery unit in the the client.
operating room.
2. These are papular wheals from herpes zoster. 70. The nurse is caring for the client who had a
3. These are blisters from the tape used to right shoulder replacement. Which data warrant
anchor the dressing. immediate intervention?
4. These macular lesions are from a latex allergy. 1. The client’s hemoglobin is 8.1 g/dL.
2. The client’s white blood cell count is 9,000/mm3.
3. The client’s creatinine level is 0.8 mg/dL.
4. The client’s potassium level is 4.2 mEq/L.

11_Colgrove_Ch11.indd 446 01/06/16 5:53 PM


C hapter 11   M usculoskeletal D isorders 447

71. The nurse is assessing the client who is 72. The nurse is working on an orthopedic floor.
postoperative total knee replacement. Which client should the nurse assess first
Which assessment data warrant immediate after the change-of-shift report?
intervention? 1. The 84-year-old female with a fractured right
1. T 99˚F, HR 80, RR 20, and BP 128/76. femoral neck in Buck’s traction.
2. Pain in the unaffected leg during dorsiflexion 2. The 64-year-old female with a left total knee
of the ankle. replacement who has confusion.
3. Bowel sounds heard intermittently in four 3. The 88-year-old male post–right total hip
quadrants. replacement with an abduction pillow.
4. Diffuse, crampy abdominal pain. 4. The 50-year-old postop client with a
continuous passive motion (CPM) device.

CONCEPTS
The concepts covered in this chapter focus on mobility and functional ability. Exemplars that are covered are
arthritis, fractures, amputation, joint replacement, and degenerative disk disease. Interrelated concepts of the
nursing process and critical thinking are covered throughout the questions. The concept of critical thinking is
presented in the prioritizing or “first” questions.
73. The nurse identifies the concept of impaired Which should the nurse implement first when
functional ability for a client diagnosed with encouraging the client to participate in therapy?
rheumatoid arthritis. Which intervention should 1. Medicate the client for pain 30 minutes prior
the nurse implement? to the therapy.
1. Teach the client to apply antiembolism (TED) 2. Have the health-care provider make the client
hose. go to therapy.
2. Administer the nonsteroidal medication 3. Explain that insurance will not pay if the
before the morning meal. client does not participate in therapy daily.
3. Encourage the client to perform low-impact 4. Determine why the client refuses to
exercises daily. participate in therapy sessions.
4. Refer the client to occupational therapy for
76. The nurse is admitting a client with rheumatoid
gait training.
arthritis (RA) and the hands have the appearance
74. The client diagnosed with osteomyelitis of shown next. Which concept would the nurse
the left foot and ankle is being prepared for a identify as priority?
below-the-knee amputation. Which intervention
to improve the client’s functional ability is a
priority after rehabilitation?
1. Keep a large tourniquet at the bedside to stop
potential bleeding from the amputation site.
2. Place a pillow in the bed for the client to push
the stump against many times per day.
3. Take and document the client’s vital signs
every four (4) hours.
4. Have the dietary department send high-
protein, high-carbohydrate meals six (6)
times a day. 1. Mobility.
2. Functional ability.
75. The client in the rehabilitation hospital refuses 3. Coping.
to participate in physical therapy following 4. Rehabilitation.
surgery for repair of a fractured right femur
sustained in a motor-vehicle accident (MVA).
The client also fractured the left forearm.

11_Colgrove_Ch11.indd 447 01/06/16 5:53 PM


448 M ed -S urg S uccess

77. Which medication should the nurse question administering to a client diagnosed with rheumatoid arthritis
who has a comorbid condition of non-Hodgkin’s lymphoma?

Admitting Diagnosis: Rheumatoid


Client Name: Jane S. Arthritis/non-Hodgkin’s Lymphoma

Account Number: Med Rec #: Allergies

Date Medication 2301–0700 0701–1500 1501–2300


Today Celecoxib (Celebrex) 200 mg orally daily 0900
Today Filgrastim (Neupogen) subcu daily 0900
times 7 days
Today Adalimumab (Humira) 40 mg subcu 0900
every other week
Today Acetaminophen (Tylenol) 650 mg
orally every 4 hours prn
Signature of Nurse:
Day Nurse RN/DN

1. Celecoxib.
2. Filgrastim.
3. Adalimumab.
4. Acetaminophen.
78. The nurse identifies a concept of impaired mobility for a male client with degenerative disk disease. Which
assessment data best support this concept?
1. The client reports a history of chronic back pain and multiple back surgeries.
2. The client reports that taking NSAIDs caused the development of peptic ulcers.
3. The client reports a three (3)-year history of difficulty initiating a urinary stream.
4. The client states he fell a year ago and had to have a cast on the right arm for a month.
79. The nurse is caring for a client who has sustained a fracture of the right hip. Which data should be
reported to the orthopedic surgeon prior to surgery?

Complete Blood Count Client Value Normal Values


6 3
RBC (10 mm ) 4.78 Men: 4.5–5.3
Women: 4.1–5.1
(106 mm3)
Hemoglobin (g/100 mL) 14 Men: 13–18 g/100 mL
Women: 12–16/g100 mL
Hematocrit (%) 42 Men: 37%–49%
Women: 36%–46%
Platelet (103 mm3) 98 150–400 (103 mm3)
WBC (103) 7.8 5–10 (103)
Sedimentation rate (ESR) (mm/hr) 17 Men: 0–17 mm/hr
Westergren method Women: 1–25 mm/hr

1. The red blood cell count of 4.78 equals 4,780,000 when multiplied by 106.
2. The hemoglobin of 14 g/100 mL of blood.
3. The platelet count of 98. 98,000, when multiplies by 103.
4. The sedimentation rate of 17 mm/hr.

11_Colgrove_Ch11.indd 448 01/06/16 5:53 PM


C hapter 11   M usculoskeletal D isorders 449

80. The client who underwent a left above-the- 83. The client who has sustained a left-sided
knee amputation as a result of uncontrolled cerebrovascular accident (stroke) has residual
diabetes questions the nurse, asking, “Why did right-sided paralysis. The nurse identifies a
this happen to me? I have always been a good concept of impaired functional ability. Which
person.” Which is the nurse’s most therapeutic should be included in the care map? Select all
response? that apply.
1. “Tell me about how it feels to have caused this 1. Refer to the occupational therapist.
to happen to you.” 2. Assess the client for neglect of the right side.
2. “I know how you feel; having your leg cut off 3. Place the client in a room where the door is
is sad.” on the left side.
3. “Why do you think that you had to have your 4. Teach the client to call for assistance prior to
leg amputated?” getting out of bed.
4. “I can see you are hurting. Would you like to 5. Encourage the client to participate in physical
talk?” therapy daily.
81. The nurse writes a concept of “impaired 84. The 35-year-old client who sustained a crushing
mobility” for a client diagnosed with a fractured injury to the left hand and forearm is being
right hip. Which would the nurse include in the discharged. Which referral should the nurse
plan of care? Select all that apply. implement?
1. Request a physical therapy referral. 1. Refer the client to physical therapy at home.
2. Administer enoxaparin (Lovenox) 2. Refer the client to an assistive living facility.
subcutaneously. 3. Refer the client to a workforce commission
3. Utilize a gait belt when ambulating the client. for job training.
4. Assess the client’s pain levels on a 1-to- 4. Refer the client to the dietitian.
10 scale.
5. Provide a high-carbohydrate, high-fat,
high-sodium diet.
82. The nurse is admitting a female client who is
complaining of severe back pain radiating down
the left leg whenever she tries to ambulate. The
concepts of impaired mobility and comfort are
implemented on the care map. Which nursing
interventions should the nurse implement?
1. Assist the client when ambulating to the
bathroom and administer medications based
on the pain scale.
2. Place the client on strict bedrest and have the
client use a regular bedpan for elimination of
urine and feces.
3. Ambulate the client in the hallway at least
four (4) times per day and discourage the use
of pharmacological pain relief.
4. Request the health-care provider (HCP) to
assist the client in ambulating in the hallway
so the HCP can observe the client’s pain.

11_Colgrove_Ch11.indd 449 01/06/16 5:53 PM


PRACTICE QUESTIONS ANSWERS
AND RATIONALES

Degenerative/Herniated Disk Disease 2. These are instructions to prevent back in-


juries as a result of poor body mechanics.
1. 1. Back pain occurs in 80% to 90% of the popu- 3. Soft-cushioned chairs are not ergonomically
lation at different times in their lives. Although designed. Soft-cushioned chairs promote poor
not using good body mechanics when lifting body posture.
an object may be a reason for younger clients 4. This might help the client prevent back
to develop a herniated disk, it is not the reason ­injuries at home, but it does not prevent
most elderly people develop back pain. ­job-related injuries.
2. There is a decreased blood supply as the TEST-TAKING HINT: The question is asking for
body ages. information which will prevent on-the-job
3. Older clients develop degenerative joint dis- back injuries. Option “4” can be ruled out
ease. Fat does not deposit itself in the nucleus because of this. The two (2) electrolytes in
pulposus. option “1” are not associated with orthopedic
4. Less blood supply, degeneration of the disk, injuries or bones, thus ruling out this option.
and arthritis are reasons elderly people de- Content – Medical: Integrated Nursing Process – Planning:
velop back problems. Client Needs – Health Promotion and Maintenance:
TEST-TAKING HINT: The clue in this question is Cognitive Level – Synthesis: Concept – Nursing Roles.
“elderly.” The answer must address a problem 4. 1. Teaching back exercises to a client who has al-
occurring as a result of aging. ready experienced a problem is tertiary care.
Content – Medical: Integrated Nursing Process – ­ 2. Placing signs with instructions about how to
Diagnosis: Client Needs – Physiological Integrity, render first aid is a secondary intervention, not
­Physiological Adaptation: Cognitive Level – Analysis: primary prevention.
Concept – Mobility.
3. Excess weight increases the workload on
2. 1. Posture and gait will be affected if the the vertebrae. Weight-loss activities help to
­client is experiencing sciatica (pain radiat- prevent back injury.
ing down a leg resulting from pressure on 4. Administering a nonnarcotic analgesic to a cli-
the sciatic nerve). ent with back pain is an example of secondary
2. The client with pain and numbness is not able or tertiary care, depending on whether the cli-
to bend or stoop and should not be asked to do ent has a one-time problem or a chronic prob-
so. lem with back pain.
3. Leg lifts will not give the nurse the needed TEST-TAKING HINT: Primary care is any activity
information and could cause this client pain; which will prevent an illness or injury.
also, the lower extremity, not the upper ex- Content – Medical: Integrated Nursing Process – ­
tremity, is being assessed. Planning: Client Needs – Health Promotion and
4. Waist twists will not assess the mobility of the ­Maintenance: Cognitive Level – Synthesis: Concept –
lower extremity, and neck mobility is assessed Mobility.
if a cervical neck problem is suspected.
5. 1. This is an example of collaborative care.
TEST-TAKING HINT: Anatomical positioning and 2. This is an example of collaborative care.
the function of spinal nerves rule out options 3. This is distraction and is an alterna-
“3” and “4.” tive method often recommended for the
Content – Medical: Integrated Nursing Process – ­promotion of client comfort.
­Assessment: Client Needs – Safe Effective Care Environment, 4. Surgery is collaborative care.
Management of Care: Cognitive Level – Analysis:
Concept – Mobility. TEST-TAKING HINT: The question asks for an
alternative, independent type of care. Options
3. 1. Increased calcium, not potassium or sodium, “1,” “2,” and “4” are all collaborative care.
is helpful in preventing orthopedic injuries. If the test taker can find a common thread
Increasing sodium intake could prevent water among three of the options, then the correct
loss in a non–air-conditioned warehouse in the answer will be the other option.
summer months, not the winter months.

450

11_Colgrove_Ch11.indd 450 01/06/16 5:53 PM


C hapter 11   M usculoskeletal D isorders 451

Content – Medical: Integrated Nursing Process – 8. 1. Clients two (2) days postoperative laminec-
Implementation: Client Needs – Physiological Integrity, tomy should be eating a regular diet.
Basic Care and Comfort: Cognitive Level – Application: 2. The client who has undergone a lumbar
Concept – Comfort. laminectomy is log rolled. It requires four (4)
6. 1. “Prone” means on the abdomen. On the people or more to log roll a client.
abdomen with the knees flexed is an uncom- 3. The legs of any client diagnosed with back
fortable position, placing the spine in an un- pain can give out and collapse at any time,
natural position. but a large client diagnosed with back pain
2. The surgical position of the wound places is at increased risk of injuring the UAP as
the client at risk for edema of tissues in well as the client. The nurse should inter-
the neck. Difficulty speaking or breathing vene before the client or UAP becomes
should alert the nurse to a ­potentially life- injured.
threatening problem. 4. This action helps ensure safety for the client.
3. The drainage from a JP drain should be emp- TEST-TAKING HINT: This question is an
tied and monitored every shift. “­except” question. All the options but one
4. The client should be kept as comfortable contain interventions which should be
as possible. implemented.
TEST-TAKING HINT: The nurse must know the Content – Nursing Management: Integrated Nursing
meaning of the common medical term prone Process – Implementation: Client Needs – Safe Effective
and realize this would be uncomfortable for Care Environment, Management of Care: Cognitive
the client, thus eliminating option “1.” The Level – Application: Concept – Management.
time frame of “every day” makes option “3” 9. 1. Mild back pain is expected with this client.
wrong. 2. Lumbar myelograms require access into
Content – Surgical: Integrated Nursing Process – the spinal column. A small amount of cere-
Implementation: Client Needs – Safe Effective Care brospinal fluid may be lost, causing a mild
­Environment, Management of Care: Cognitive Level – headache. The client should stay flat in bed to
Analysis: Concept – Perioperative. prevent this from occurring.
7. 1. The lumbar nerves innervate the lower 3. This client is postoperative and now has
­abdomen. The bladder is in the lower a fever. This client should be assessed
abdomen. The client will be required to and the health-care provider should be
lie flat, and this is a difficult position for notified.
many clients, especially males, to be in to 4. A discharged client does not have priority
void. Clients are log rolled every 2 hours. over a surgical infection.
2. The client should be receiving IV pain medi- TEST-TAKING HINT: Options “1” and “2”
cation, not steroids. A Trendelenburg position contain assessment data expected for the
is head down. procedure.
3. Sandbags keep the neck still, but the surgical Content – Medical: Integrated Nursing Process – ­
area is in the lumbar region, so there is no Assessment: Client Needs – Safe Effective Care
reason the client cannot turn the head; also, ­Environment, Management of Care: Cognitive Level –
cathartic medications are harsh laxatives. Analysis: Concept – Assessment.
4. The client will be receiving subcutaneous an-
10. 1. This could be administered after breakfast
ticoagulant medications to prevent deep vein
if necessary. There is nothing in the action
thrombosis, but IV anticoagulant therapy is
of the medication requiring before breakfast
not warranted. Eight (8) L/min of oxygen is
medication administration.
high-flow oxygen and is used for a client in
2. Clients with type 1 diabetes are insulin
respiratory distress who does not have carbon
dependent. This medication should be
dioxide narcosis.
­administered before the client eats.
TEST-TAKING HINT: The test taker must note 3. This medication should be held until after
the adjective “lumbar”; this can rule out op- surgery.
tion “3.” Knowledge of medication classifica- 4. The client has already received three (3)
tions would rule out options “2” and “4.” doses of IV antibiotic. This medication could
Content – Surgical: Integrated Nursing Process – be given after the insulin.
Implementation: Client Needs – Safe Effective Care
TEST-TAKING HINT: The nurse must decide
­Environment, Management of Care: Cognitive Level –
which medication has priority by determining
Analysis: Concept – Perioperative.
the action of the medication, the route of ad-
ministration, and the diagnosis of the client.

11_Colgrove_Ch11.indd 451 01/06/16 5:53 PM


452 M ed -S urg S uccess

Content – Medical: Integrated Nursing Process – Drug 2. Increasing age is a risk factor, but there is
Administration: Client Needs – Physiological Integrity, nothing the client can do about getting older,
Pharmacological and Parenteral Therapies: Cognitive except to die.
Level – Application: Concept – Medication. 3. Previous joint damage is a risk factor, but it is
11. 1. An objective method of quantifying the not modifiable, which means the client can-
client’s pain should be used. not do anything to change it.
2. Once the nurse has determined the c­ lient 4. Genetic susceptibility is a result of family
is stable and not experiencing complica- genes, which the client cannot change; it is a
tions, the nurse can medicate the client. nonmodifiable risk factor.
3. A regular bedpan is high and could cause pain TEST-TAKING HINT: The adjective “modifi-
for a client diagnosed with back pain. The able” is the key to selecting the correct
­client should be given a fracture pan. answer. Only option “1” contains anything
4. There is no surgical dressing. the client has control over changing or
5. The client has not been to surgery, so log modifying.
rolling is not necessary. Content – Medical: Nursing Process – Planning: Client
TEST-TAKING HINT: Two of the options, “4” Needs – Health Promotion and Maintenance: Cognitive
and “5,” apply to postsurgical cases and could Level – Synthesis: Concept – Mobility.
be eliminated. 14. 1. Severe bone deformity is seen in clients
Content – Medical: Integrated Nursing Process – ­ ­diagnosed with rheumatoid arthritis.
Diagnosis: Client Needs – Physiological Integrity, 2. Pain, stiffness, and functional impairment
­Physiological Adaptation: Cognitive Level – Analysis: are the primary clinical manifestations of
Concept – Comfort.
OA. Stiffness of the joints is commonly
12. 1. The nurse should not continue working, and experienced after resting but usually lasts
this is self-diagnosing and treating. less than 30 minutes and decreases with
2. The nurse may go to the emergency depart- movement.
ment, but this is not the first action. 3. A waddling gait is usually seen in women in
3. The first action is to notify the charge their third trimester of pregnancy or in older
nurse so a replacement can be arranged children with congenital hip dysplasia.
to take over care of the clients. The nurse 4. Swan-neck fingers are seen in clients with
should notify the nurse manager or house rheumatoid arthritis.
supervisor. An occurrence report should TEST-TAKING HINT: The test taker can have
be completed, documenting the situation. difficulty distinguishing clinical manifesta-
This provides the nurse with the required tions of two similar sounding diseases, os-
documentation to begin a worker’s com- teoarthritis and rheumatoid arthritis. Both
pensation case for payment of medical bills. diseases involve the joints and cause pain and
4. The nurse has the right to see a private stiffness. Remember, rheumatoid arthritis
health-care provider in most states, but this is can permanently disfigure the client, leading
not the first action. to “bone deformity” and “swan-neck fingers.”
TEST-TAKING HINT: When the test taker is Content – Medical: Integrated Nursing Process –
determining a priority, then all of the an- ­Assessment: Client Needs – Physiological Integrity,
swers may be appropriate interventions, but ­Reduction of Risk Potential: Cognitive Level – Analysis:
only one is implemented first. The test taker Concept – Mobility.
should read the full stem, identifying the 15. 1. Clients with OA should be encouraged to
important words and making sure he or she move, which will decrease the pain.
understands what the question is asking. 2. A bed bath does not require as much move-
Content – Medical: Integrated Nursing Process – ment from the client as getting up and walk-
Implementation: Client Needs – Safe Effective Care ing to the shower.
­Environment, Management of Care: Cognitive Level – 3. Pain will decrease with movement, and
Application: Concept – Management.
warm or hot water will help decrease the
pain. The worst thing the client can do is
Osteoarthritis not move.
4. Notifying the family will not address the
13. 1. Obesity is a well-recognized risk factor client’s pain, and the client has a right to re-
for the development of OA and it is modi- fuse a bath, but the nursing staff must explain
fiable because the client can lose weight. why moving and bathing will help decrease
the pain.

11_Colgrove_Ch11.indd 452 01/06/16 5:53 PM


C hapter 11   M usculoskeletal D isorders 453

TEST-TAKING HINT: Allowing clients to stay in functional ability of the joints to ensure
bed only increases complications of immo- movement of the joints.
bility and will increase the client’s pain sec- 3. Most clients with OA are elderly, are over-
ondary to OA. Clients with chronic illnesses weight, and have a sedentary lifestyle, so
should be encouraged to be as independent walking three (3) miles every day is not a
as possible. The family should only be noti- ­realistic or safe goal.
fied if a significant situation has occurred. 4. Joining a health club is an intervention,
Content – Nursing Management: Integrated Nursing and the fact the client joins the health club
Process – Planning: Client Needs – Safe Effective Care doesn’t mean the client will exercise.
Environment, Management of Care: Cognitive Level – TEST-TAKING HINT: The test taker must re-
Synthesis: Concept – Comfort.
member a goal is the measurable outcome
16. 1. Pain is a physiological problem, not a psycho- of nursing interventions based on the client
social problem. problem/­diagnosis. Interventions are not
2. A client with OA does not have bone defor- goals; therefore, the test taker could elimi-
mities; therefore, body image disturbance is nate options “1” and “4” as possible answers.
not appropriate. Content – Medical: Integrated Nursing Process – ­
3. After seven (7) years of OA and multiple Diagnosis: Client Needs – Physiological Integrity,
treatment modalities, knowledge deficit is not ­Physiological Adaptation: Cognitive Level – Analysis:
appropriate for this client. ­Concept – Mobility.
4. The client experiencing chronic pain 19. 1. A physiatrist is a physician who specializes in
often experiences depression and physical medicine and rehabilitation, but the
hopelessness. nurse should not refer the client to this per-
TEST-TAKING HINT: The adjective “psycho- son just because the client is having difficulty
social” should help the test taker rule out with transfers.
option “1.” The test taker needs to read the 2. The social worker does not address this type
stem carefully. This client has had a problem of physical problem. Social workers address
for years, and, therefore, option “3” could be issues concerning finances, placement, and
ruled out as a correct answer. acquiring assistive devices.
Content – Medical: Integrated Nursing Process – 3. The physical therapist is able to help the
­Diagnosis: Client Needs – Psychosocial Integrity: Cognitive client with transferring, ambulation, and
Level – Analysis: Concept – Comfort/Mood. other lower extremity difficulties.
4. A counselor is not able to help the client
17. 1. This medication should be taken with food to
learn how to get in and out of the bathtub.
prevent gastrointestinal distress.
2. Glucocorticoids, not NSAIDs, must be TEST-TAKING HINT: The nurse must know the
­tapered when discontinuing. roles of all the health-care team members.
3. Topical analgesics are applied to the skin; Content – Medical: Integrated Nursing Process –
NSAIDs are oral or intravenous medications. ­Planning: Client Needs – Safe Effective Care ­Environment,
4. NSAIDs are well known for causing Management of Care: Cognitive Level – Synthesis:
­gastric upset and increasing the risk for ­Concept – Mobility.
peptic ulcer disease, which could cause 20. 1. Safety should always be discussed when
the client to vomit blood. teaching about exercises. Supportive
TEST-TAKING HINT: The worst-scenario option shoes will prevent shin splints. C ­ olored
is “4,” which has blood in the answer. If the test socks have dye and may cause ­athlete’s
taker did not know the answer, then selecting foot, which is why white socks are
an option with blood in it is most appropriate. recommended.
Content – Medical: Integrated Nursing Process – ­ 2. Clients with diabetes mellitus should carry
Planning: Client Needs – Physiological Integrity, complex carbohydrates with them.
­Pharmacological and Parenteral Therapies: Cognitive 3. Osteoarthritis occurs most often in weight-
Level – Synthesis: Concept – Medication. bearing joints. Exercise is encouraged, but
jogging increases stress on these joints.
18. 1. This is an intervention, not a goal, and “pas-
4. For exercising to help pain control, the client
sive” means the nurse performs the range of
must walk daily, not three (3) times a week.
motion, which should not be encouraged.
Walking at least 30 minutes three (3) times a
2. The two main goals of treatment for OA
week is appropriate for weight loss.
are pain management and optimizing

11_Colgrove_Ch11.indd 453 01/06/16 5:53 PM


454 M ed -S urg S uccess

TEST-TAKING HINT: The test taker can rule 4. An ESR is a diagnostic laboratory test for
out o
­ ption “3” as an answer because the stem rheumatoid arthritis, not osteoarthritis.
says “pain control”; option “1” is correct for TEST-TAKING HINT: If the test taker is guess-
any e
­ xercise program. ing which answer is correct and knows os-
Content – Medical: Integrated Nursing Process – ­ teo- means “bone,” the only option with any
Planning: Client Needs – Physiological Integrity, specific connection to bones is an x-ray. This
­Physiological Adaptation: Cognitive Level – Synthesis: selection is an educated guess.
­Concept – Safety.
Content – Medical: Integrated Nursing Process – D ­ iagnosis:
21. 1. NSAIDs or glucocorticoids help decrease Client Needs – Physiological Integrity, Reduction of Risk
­inflammation of the joints. ­Potential: Cognitive Level – Analysis: Concept – Mobility.
2. This is the rationale for administering 24. 1. A cold foot in a client who has had ­surgery
these medications. may indicate a neurovascular compromise
3. Narcotic and nonnarcotic analgesics help de- and must be assessed first.
crease the client’s pain. 2. A client with osteoarthritis is expected to
4. There is no medication at this time to help have stiff joints.
increase synovial fluid production, but sur- 3. A routine medication is not priority over a
gery can increase the viscosupplementation in potential complication of surgery.
the joint. 4. A routine diagnostic procedure does not have
TEST-TAKING HINT: There are some questions priority over a potential complication of surgery.
requiring the test taker to have the knowl- TEST-TAKING HINT: The test taker must take
edge, and there are no test-taking hints to a systematic approach when answering pri-
help with selecting the right answer. oritizing questions. First, the test taker must
Content – Medical: Integrated Nursing Process – ­ determine if any client is experiencing a
Diagnosis: Client Needs – Physiological Integrity, life-threatening or life-altering complication
­Pharmacological and Parenteral Therapies: Cognitive such as loss of a limb. The test taker must
Level – Analysis: Concept – Medication.
determine if the sign/symptom is expected
22. 1. Medication is a standard therapy and is not for the disease or condition.
considered an alternative therapy. Content – Medical: Integrated Nursing Process – ­
2. A heating pad is an accepted medical recom- Assessment: Client Needs – Safe Effective Care
mendation for the treatment of pain for cli- ­Environment, Management of Care: Cognitive Level –
ents with OA. Analysis: Concept – Assessment.
3. Alternative forms of treatment have not
been proved efficacious in the treatment
of a disease. The nurse should be non- Osteoporosis
judgmental and open to discussions about
25. 1. Calcium deficiency is a modifiable risk fac-
alternative treatment, unless it interferes
tor, which means the client can do something
with the medical regimen.
about this factor—namely, increase the intake
4. Conservative treatment measures for OA in-
of calcium—to help prevent the development
clude splints and braces to support inflamed
joints. of osteoporosis.
2. Smoking is a modifiable risk factor because
TEST-TAKING HINT: The test taker needs to the client can quit smoking.
read the stem carefully to be able to determine 3. A nonmodifiable risk factor is a factor
what the question is asking. There is only one the client cannot do anything to alter or
option with alternative-type treatment, which change. Approximately 50% of all women
is option “3”; options “1,” “2,” and “4” are ac- will experience an osteoporosis-related
cepted treatment options listed in a textbook. fracture in their lifetime.
Content – Medical: Integrated Nursing Process – 4. The client can quit drinking alcohol; there-
­Evaluation: Client Needs – Physiological Integrity, fore, this is a modifiable risk factor.
­Physiological Adaptation: Cognitive Level – Knowledge:
Concept – Mobility. TEST-TAKING HINT: The key word to answer-
ing this question is “nonmodifiable,” which
23. 1. MRIs are not routinely ordered for means the client cannot do anything to
­diagnosing OA. modify or change behavior to help prevent
2. There is no serum laboratory test to ­measure developing osteoporosis.
synovial fluid in the joints. Content – Medical: Integrated Nursing Process – ­
3. X-rays reveal loss of joint cartilage, which Planning: Client Needs – Health Promotion and
appears as a narrowing of the joint space ­Maintenance: Cognitive Level – Synthesis: Concept –
in clients diagnosed with OA. Fluid and Electrolyte Balance.

11_Colgrove_Ch11.indd 454 01/06/16 5:53 PM


C hapter 11   M usculoskeletal D isorders 455

26. 1. This is the rationale for heavy alcohol use 29. 1. The best dietary sources of calcium are
leading to the development of osteoporosis. milk and other dairy products. Other
2. Smoking decreases, not increases, blood sup- sources include oysters; canned sardines
ply to the bone. or salmon; beans; cauliflower; and dark-
3. Cigarette smoking has long been identified green, leafy vegetables.
as a risk factor for osteoporosis, and it doesn’t 2. These foods are high in vitamin C.
matter if the cigarettes are low tar. 3. These foods are high in potassium.
4. Nicotine slows the production of os- 4. These foods are recommended for a high-
teoblasts and impairs the absorption of fiber diet.
calcium, contributing to decreased bone TEST-TAKING HINT: A question about special
density. diets is a knowledge-based question, and the
TEST-TAKING HINT: The test taker must al- test taker must know which foods are in which
ways be aware of the words “increase” and type of diets. Foods high in calcium should be
“decrease” when selecting a correct answer. associated with milk products such as yogurt.
Content – Medical: Integrated Nursing Process – Content – Medical: Integrated Nursing Process –
Implementation: Client Needs – Health Promotion and ­Planning: Client Needs – Physiological Integrity, Basic
Maintenance: Cognitive Level – Application: Concept – Care and Comfort: Cognitive Level – Synthesis:
Fluid and Electrolyte Balance. Concept – Nutrition.
27. 1. The loss of height occurs as vertebral 30. 1. This is an example of a secondary nursing
bodies collapse. intervention, which includes screening for
2. Weight loss is not a sign of osteoporosis. early detection.
3. This may indicate rheumatoid arthritis but 2. The client should perform weight-bearing
not osteoporosis. exercises, which promote osteoblast activity
4. This is a sign of gout. helping to maintain bone strength and integ-
TEST-TAKING HINT: If the test taker is not rity. This is a primary nursing intervention.
sure of the answer and knows osteo- means 3. Increasing dietary calcium may be a primary
“bone,” the only answer related to bones is intervention to help prevent osteoporosis
the height of the client, related to the spine. or a tertiary intervention, which helps treat
Content – Medical: Integrated Nursing Process –
osteoporosis.
­Assessment: Client Needs – Physiological Integrity, 4. Smoking cessation is a primary intervention,
­Reduction of Risk Potential: Cognitive Level – Analysis: which will help prevent the development of
Concept – Fluid and Electrolyte Balance. osteoporosis.
TEST-TAKING HINT: The nurse must be knowl-
28. 1. Osteoporotic changes do not occur in the
bone until more than 30% of the bone mass edgeable of primary, secondary, and tertiary
has been lost. nursing interventions. Primary interventions
2. This serum blood study may be elevated af- are those which prevent the disease; second-
ter a fracture, but it does not help diagnose ary interventions are interventions such as
osteoporosis. screening the client for the disease with the
3. This test measures bone density in the goal of detecting it early; and tertiary in-
lumbar spine or hip and is considered to terventions are interventions implemented
be highly accurate. when the client has the disease.
4. This test is most useful to evaluate the effects Content – Medical: Integrated Nursing Process – ­
of treatment, rather than as an indicator of Planning: Client Needs – Health Promotion and
­Maintenance: Cognitive Level – Analysis: Concept –
the severity of bone disease.
Fluid and Electrolyte Balance.
TEST-TAKING HINT: Option “2” does not have
bone or x-ray in it; therefore, if the test taker 31. 1. Swimming is not as beneficial as walking in
did not know the correct answer, eliminating maintaining bone density because of the lack
this option is appropriate. If the test taker of weight-bearing activity.
knew osteoporosis is secondary to poor ab- 2. Weight-bearing activity, such as walking,
sorption of calcium, option “3” is an appro- is beneficial in preventing or slowing bone
priate selection for the correct answer. loss. The mechanical force of weight-­
Content – Medical: Integrated Nursing Process –
bearing exercises promotes bone growth.
­Diagnosis: Client Needs – Physiological Integrity, Reduction 3. Swimming is not as beneficial in maintain-
of Risk Potential: Cognitive Level – Analysis: Concept – ing bone density because of the lack of
Fluid and Electrolyte Balance. weight-bearing activity.
4. A sedentary lifestyle is a risk factor for the
development of osteoporosis.

11_Colgrove_Ch11.indd 455 01/06/16 5:53 PM


456 M ed -S urg S uccess

TEST-TAKING HINT: Sedentary activities in- 2. Range-of-motion (ROM) exercises will help
clude sitting and very low-activity exercises, prevent deep vein thrombosis or contractures,
which are risk factors in developing many but they do not help prevent osteoporosis.
diseases and disorders; therefore, option “4” 3. Turning the client will help prevent pressure
can be eliminated. ulcers but does not help prevent osteoporosis.
Content – Medical: Integrated Nursing Process – 4. Nighttime lights will help prevent the cli-
Implementation: Client Needs – Physiological Integrity, ent from falling; fractures are the number
Reduction of Risk Potential: Cognitive Level – Application: one complication of osteoporosis.
Concept – Fluid and Electrolyte Balance. TEST-TAKING HINT: The test taker should
32. 1. Nausea and vomiting may occur during initial realize the bed should be kept in a low posi-
stages of therapy, but they will disappear as tion at all times and should eliminate option
treatment continues. “1” as a possible answer; ROM exercises and
2. The client should be sure to consume ad- turning will help prevent complications of
equate amounts of calcium and vitamin D immobility, not osteoporosis.
while taking calcitonin. Content – Medical: Integrated Nursing Process –
3. Rhinitis (runny nose) is the most common Analysis: Client Needs – Safe Effective Care Environment,
side effect with calcitonin nasal spray along Management of Care: Cognitive Level – Analysis:
with itching, sores, and other nasal symptoms. Concept – Safety.
4. Nosebleeds are adverse effects and should 35. 1. There is no reason to take Tums with eight (8)
be reported to the client’s HCP. ounces of water. Tums are usually chewed.
TEST-TAKING HINT: If the test taker has no idea 2. Tums should not be taken with meals.
of the answer, an appropriate option to select 3. Free hydrochloric acid is needed for cal-
is an option with bleeding in it; bleeding is cium absorption; therefore, Tums should
abnormal and indicates an adverse effect. be taken on an empty stomach.
Content – Medical: Integrated Nursing Process – 4. To determine the effectiveness of calcium
Assessment: Client Needs – Physiological Integrity, supplements, the client must have a bone
­Pharmacological and Parenteral Therapies: Cognitive density test, not a serum calcium level
Level – Analysis: Concept – Medication. measurement.
33. 1. The National Institutes of Health (NIH) TEST-TAKING HINT: If unsure of the answer,
recommends a daily calcium intake of the test taker should not select an option
1,200 to 1,500 mg/day for adolescents, with an absolute-type word such as “only,”
young adults, and pregnant and lactating “always,” or “never.” There are very few
women. ­absolutes in health care.
2. The pregnant teenager should eat foods high Content – Medical: Integrated Nursing Process – ­
in calcium. Planning: Client Needs – Physiological Integrity,
3. Osteoporosis may not occur before age 50 ­Pharmacological and Parenteral Therapies: Cognitive
years, but taking calcium throughout the life Level – Synthesis: Concept – Medication.
span will help prevent it. Remember, teen- 36.    Six (6) tablets.
agers tend to focus on the present, not the 1,000 mg is equal to one (1) gram. There-
future, so the most important intervention to fore, three (3) grams is equal to 3,000 mg.
teach them is to take calcium supplements. If one (1) tablet is 500 mg, the client will
4. Activity will not help prevent osteoporosis in need six (6) tablets to get the total amount
the teenager; the teenager must take calcium of calcium needed daily:
supplements. 3,000 ÷ 500 = 6
TEST-TAKING HINT: The age of the client is TEST-TAKING HINT: The test taker must know
important when answering questions; devel- how to perform math calculations and must
opmental stages will help rule out or help be knowledgeable of conversions. Remem-
select the correct answer. ber to use the drop-down calculator on the
Content – Medical: Integrated Nursing Process – Plan- NCLEX-RN examination.
ning: Client Needs – Health Promotion and Maintenance: Content – Medical: Integrated Nursing Process –
Cognitive Level – Synthesis: Concept – Nursing Roles. Implementation: Client Needs – Physiological Integrity,
34. 1. The bed should be kept in the low position. Pharmacological and Parenteral Therapies: Cognitive
Preventing falls is a priority for a client diag- Level – Application: Concept – Medication.
nosed with osteoporosis.

11_Colgrove_Ch11.indd 456 01/06/16 5:53 PM


C hapter 11   M usculoskeletal D isorders 457

Amputation 4. An 18-gauge catheter should be started because


the client is going to surgery; the client may
37. 1. This position will decrease lung expansion. need a blood transfusion, which should be ad-
2. The prone position will help stretch ministered through an 18-gauge catheter.
the hamstring muscles, which will help TEST-TAKING HINT: The nurse must take into
prevent flexion contractures leading to account all the client’s comorbid conditions
problems when fitting the client for a (diabetes type 2) when selecting the correct
prosthesis. answer.
3. Lying on the back will not help decrease Content – Surgical: Integrated Nursing Process –
­actual or phantom pain. ­Implementation: Client Needs – Physiological Integrity,
4. This will help take pressure off the client’s Physiological Adaptation: Cognitive Level – Application:
buttocks area, but it is not why it is recom- Concept – Perioperative.
mended for a client with a lower extremity
40. 1. Wrapping the hand with towels is appropri-
amputation.
ate, but it is not the first intervention.
TEST-TAKING HINT: The test taker can elimi- 2. Holding the arm above the head will help
nate option “1” if visualizing the client in decrease the bleeding, but it is not the first
a prone position. This position will limit intervention.
expansion of the lung more than increase 3. Applying direct pressure to the artery
it. When trying to allow for expansion of above the amputated parts will help de-
the lungs, clients are placed with the head crease the bleeding immediately and is the
­elevated, a position the client in a prone first intervention the nurse should imple-
­position cannot achieve. ment. Then the nurse should instruct the
Content – Surgical: Integrated Nursing Process – client to hold the hand above the head,
Implementation: Client Needs – Physiological Integrity, apply towels, and call 911.
Physiological Adaptation: Cognitive Level – Application: 4. Calling 911 should be done, but it is not the
Concept – Mobility. first intervention.
38. 1. The client is in the recovery room, and the TEST-TAKING HINT: Remember, when the
dressing must be assessed more frequently stem asks the test taker to identify the first
than every two (2) hours. intervention, all four options will be prob-
2. The client must come to terms with the able interventions, but only one is the first
amputation; therefore, the nurse should intervention.
encourage the client to look at the resid- Content – Surgical: Integrated Nursing Process –
ual limb. Implementation: Client Needs – Safe Effective Care
3. The large tourniquet can be used if the ­Environment, Management of Care: Cognitive Level –
residual limb begins to hemorrhage either Synthesis: Concept – Trauma.
internally or externally.
41. 1. Placing the amputated part directly on ice
4. The nurse should encourage active, not
will cause vasoconstriction and necrosis of
­passive, range-of-motion exercises.
­viable tissue.
TEST-TAKING HINT: Remember to look at the 2. Warm water will cause the amputated part to
phrases describing the intervention, such as disintegrate and lose viable tissue.
“every two (2) hours” and “passive.” 3. Wrapping the amputated part in a piece of
Content – Surgical: Integrated Nursing Process – material will not help preserve the thumb so
Implementation: Client Needs – Safe Effective Care it can be reconnected.
­Environment, Management of Care: Cognitive Level – 4. Placing the thumb in a plastic bag will pro-
Application: Concept – Perioperative. tect it and then placing the plastic bag on
39. 1. For wound healing, a balanced diet with ice will help preserve the thumb so it may
adequate protein and vitamins is essential, be reconnected in surgery. Do not place
along with meals appropriate for type 2 the amputated part directly on ice because
diabetes. this will cause necrosis of ­viable tissue.
2. An occupational therapist addresses activities TEST-TAKING HINT: The test taker should
of daily living and usually addresses upper make sure he or she knows what the
extremity amputations. A referral to a physi- ­question is asking before selecting the op-
cal therapist is most appropriate to address tion. The question is asking “what will help
ambulating and transfer concerns. preserve the thumb?”—which is the key to
3. There is no type of intravenous dye used in answering this question.
this surgical procedure, so this answer is not
appropriate.

11_Colgrove_Ch11.indd 457 01/06/16 5:53 PM


458 M ed -S urg S uccess

Content – Surgical: Integrated Nursing Process – TEST-TAKING HINT: Remember, when the
Implementation: Client Needs – Safe Effective Care stem asks the test taker to identify the first
­Environment, Management of Care: Cognitive Level – intervention, all four options will be probable
Knowledge: ­Concept – Trauma. interventions but only one is the first inter-
42. 1. The Jewish faith believes all body parts vention. Also, the nurse should always assess
must be buried together. Therefore, many first. Remember the nursing process.
synagogues will keep amputated limbs Content – Surgical: Integrated Nursing Process –
­until death occurs. Implementation: Client Needs – Safe Effective Care
2. Specific foods are important but not while ­Environment, Management of Care: Cognitive Level –
the client is in the operating room. Analysis: Concept – Perfusion.
3. Spiritual issues are important for the nurse 45. 1. A client who is only two (2) days postopera-
to discuss with the client, but the operating tive amputation is not putting on a prosthesis.
room should be concerned with disposition of 2. Two (2) registered nurses must double-check
the amputated limb. a unit of blood prior to infusing the blood.
4. Addressing teaching issues is important, but 3. The surgical dressing is changed by the
the most important concern is disposition of surgeon or the nurse; Syme’s amputation is
the amputated limb. above the ankle, just removing the foot.
TEST-TAKING HINT: The nurse must always 4. The unlicensed assistive personnel (UAP)
address the cultural needs of the client, and could take a client to another ­department
when the test taker sees a specific culture in in the hospital.
the stem of a question, it is a prompt indicat- TEST-TAKING HINT: Remember, teaching, as-
ing this will be important when selecting the sessing, and evaluating cannot be delegated.
answer. Content – Nursing Management: Integrated Nursing
Content – Surgical: Integrated Nursing Process – Process – Planning: Client Needs – Safe Effective Care
Implementation: Client Needs – Safe Effective Care Environment, Management of Care: Cognitive Level –
­Environment, Management of Care: Cognitive Level – Synthesis: Concept – Management.
Application: Concept – Perioperative.
46. 1. Applying pressure to the end of the re-
43. 1. The client is three (3) hours postoperative sidual limb will help toughen the limb.
and needs medical intervention. Gradually pushing the residual limb
2. Phantom pain is caused by severing the against harder and harder surfaces is done
peripheral nerves. The pain is real to the in preparation for prosthesis training.
client, and the nurse needs to medicate 2. An Ace bandage applied distal to proximal
the client immediately. will help decrease edema and help shape the
3. Biofeedback exercises will not help address residual limb into a conical shape.
the client’s postoperative surgical pain. 3. Vitamin E oil will help decrease the angriness
4. Placing the residual limb below the heart of the scar, but it will not help with residual
(dependent) will not help address the client’s limb toughening.
pain and could actually increase the pain. 4. Elevating the residual limb will help decrease
TEST-TAKING HINT: The test taker needs to be edema, but it will also cause a contracture if
aware of adjectives such as “dependent.” The the residual limb is elevated after the first
nurse must know medical terms for position- 24 hours.
ing a client. TEST-TAKING HINT: The stem of the question
Content – Surgical: Nursing Process – Implementation: asks the test taker to choose a method of
­Client Needs – Physiological Integrity, Physiological Adapta- toughening the residual limb. Demonstrating
tion: Cognitive Level – Analysis: Concept – Comfort. how to apply an elastic bandage or elevating
44. 1. If the client is hemorrhaging, the surgeon the limb would not accomplish this, so op-
needs to be notified, but hemorrhaging has tions “2” and “4” could be eliminated from
not been determined. consideration.
2. Determining if the client is hemorrhaging Content – Surgical: Integrated Nursing Process –
is the first intervention. The nurse should Implementation: Client Needs – Physiological Integrity,
check for signs of hypovolemic shock: de- Physiological Adaptation: Cognitive Level – Application:
Concept – Mobility.
creased BP and increased pulse.
3. Reinforcing the dressing helps decrease 47. 1. This statement does not indicate acceptance;
bleeding, but the nurse must assess first. the client is still in the anger stage of grieving.
4. Checking the client’s laboratory results is an 2. Looking toward the future and problem-
appropriate intervention, but it is not the first solving indicate the client is accepting
intervention. the loss.

11_Colgrove_Ch11.indd 458 01/06/16 5:53 PM


C hapter 11   M usculoskeletal D isorders 459

3. At this young age, a client with an upper ex- 50. 1. The client may experience difficulty coping
tremity prosthesis needs to be thinking about depending on how much mobility the client
obtaining employment and living a full life. has after medical treatment, but it is not the
Getting a prosthesis is important to pursue most appropriate nursing diagnosis at this
this goal. time.
4. This statement does not indicate acceptance; 2. Compartment syndrome (edema within a
his wife will worry about the client’s life, muscle compartment) may occur, but there
which has been changed dramatically. are multiple complications the nurse should
TEST-TAKING HINT: Always notice when the be assessing for, so this is not the most appro-
age is given for the client. This will help priate nursing intervention.
guide the test taker to the correct answer. 3. The client has a closed fracture, so there is no
exposed bone or tissue.
Content – Surgical: Integrated Nursing Process –
­Evaluation: Client Needs – Physiological Integrity, 4. Assessing and preventing complications
­Physiological Adaptation: Cognitive Level – Synthesis: related to the neurovascular compro-
Concept – Self. mise is the most appropriate intervention
because, if there are no complications,
48. 1. Pain not relieved with analgesics could in- a closed fracture should heal without
dicate complications or could be phantom problems.
pain.
TEST-TAKING HINT: Physiological problems
2. A well-balanced diet promotes wound
are priority over psychosocial problems, so
healing, especially a diet high in protein.
the test taker could rule out option “1.”
3. The client must keep appointments in
outpatient rehabilitation to continue to Content – Medical: Integrated Nursing Process –
improve physically and emotionally. ­Diagnosis: Client Needs – Safe Effective Care Environment,
Management of Care: Cognitive Level – Analysis:
4. A support group may help the client ad-
Concept – Mobility.
just to life with an amputation.
5. The client should be encouraged to get out as 51. 1. An immobilizer should not be applied snugly.
much as possible and live as normal a life as There should be enough room to allow for
possible. edema and adequate perfusion of the
TEST-TAKING HINT: The test taker needs to tissues.
select all appropriate options. 2. Ice packs should be applied 10 minutes on
and 20 minutes off. This allows for
Content – Surgical: Integrated Nursing Process – ­
Planning: Client Needs – Physiological Integrity, vasoconstriction and decreases edema. Ice
­Physiological Adaptation: Cognitive Level – Synthesis: is a nonpharmacological pain management
Concept – Trauma. technique.
3. An injured extremity should be elevated
above the level of the heart to decrease edema
Fractures and pain.
4. An x-ray should be done before the immobi-
49. 1. The nurse should assess the nailbeds for lizer is in place, not after.
the capillary refill time. A prolonged time 5. Anytime trauma occurs, tetanus should be
(greater than three [3] seconds) indicates considered. In an open fracture, this is an
impaired circulation to the extremity. appropriate treatment.
2. Clothing may need to be removed but not TEST-TAKING HINT: This is an alternative-
before assessment. type question. When selecting all that
3. An x-ray will be done but is not the highest apply, it is important to consider the de-
priority action. scriptive words which make the options
4. A cast may or may not be applied, depending incorrect. Read adjectives and adverbs
on the type and location of the fracture. carefully. The terms “snugly,” “dependent,”
TEST-TAKING HINT: When the question asks and “after” make options “1,” “3,” and “4”
to prioritize nursing care, usually assessment incorrect.
is first. Assessment is an independent nursing Content – Medical: Integrated Nursing Process –
intervention. Implementation: Client Needs – Physiological Integrity,
Content – Medical: Integrated Nursing Process – Physiological Adaptation: Cognitive Level – Application:
Implementation: Client Needs – Safe Effective Care Concept – Mobility.
­Environment, Management of Care: Cognitive Level –
Synthesis: Concept – Assessment.

11_Colgrove_Ch11.indd 459 01/06/16 5:53 PM


460 M ed -S urg S uccess

52. 1. Localized edema and discoloration hours determine what is normal or usual for this
­after the injury are normal occurrences after a client.
fracture. 4. Checking the client’s Buck’s traction will not
2. Generalized weakness and increasing tender- address the problem of confusion. This will
ness are common and not life threatening. not address taking care of the orientation of
3. If the nurse cannot hear the pedal pulse the client.
with a Doppler and the client’s pain is TEST-TAKING HINT: The test taker needs to
increasing, the nurse should notify the understand what the question is asking.
health-care provider. These are signs of ­Although the client has a fractured hip, the
neurovascular compromise. orientation status is the unexpected symp-
4. Pain management is a desired outcome dem- tom which requires assessment.
onstrated by pain relieved after medication Content – Medical: Integrated Nursing Process –
administration. Implementation: Client Needs – Physiological Integrity,
TEST-TAKING HINT: The nurse should notify Reduction of Risk Potential: Cognitive Level – Synthesis:
the health-care provider of abnormal or un- Concept – Cognition.
expected assessment data; no pulse indicates 55. 1. The health-care provider orders the dosage
a neurovascular complication. All the other on a PCA. Unless a range of dosages or a new
options contain normal or expected data. order is obtained, a lower dose will not help
Content – Medical: Integrated Nursing Process – pain.
Implementation: Client Needs – Safe Effective Care 2. Weights from traction should be off the
­Environment, Management of Care: Cognitive Level –
floor and hanging freely. Buck’s t­ raction
Synthesis: Concept – Mobility.
is used to reduce muscle spasms preoper-
53. 1. The nurse should assess the client for atively in clients who have fractured hips.
signs of hypoxia from a fat embolism, 3. Raising the head of the bed or the foot will
which is what the nurse should anticipate alter the traction.
from “fatty globules” in the urine. 4. Turning the client to the affected side could
2. Arterial blood gases and portable chest x-ray increase pain rather than relieve it.
will be done, but they will not be done first. TEST-TAKING HINT: This intervention is a
3. A ventilation/perfusion scan is not the highest form of assessment, assessing the equipment
priority for the client. Assessment for compli- being used for the client’s condition. Re-
cations is priority. member to apply the nursing process.
4. The UAP should keep the client on strict Content – Medical: Integrated Nursing Process –
bedrest, but the nurse’s first intervention is Implementation: Client Needs – Physiological Integrity,
to assess the client. The client is unstable and Reduction of Risk Potential: Cognitive Level – Application:
the nurse should assess the client first, then Concept – Mobility.
maintain strict bedrest.
56. 1. The arm should be elevated above the heart,
TEST-TAKING HINT: If the test taker is unsure
not at heart level.
of the correct answer, always apply the nurs- 2. The nurse should instruct the child to not
ing process. Assessment is the first part of insert anything under the cast because it
the nursing process. could cause a break in the skin, leading to an
Content – Medical: Nursing Process – Implementation: infection.
Client Needs – Safe Effective Care Environment, Manage- 3. Applying ice packs to the cast will relieve
ment of Care: Cognitive Level – Synthesis: Concept –
itching, and nothing should be placed
Clotting.
down a cast to scratch. Skin becomes
54. 1. There is controversy over assessing for a ­fragile inside the cast and is torn eas-
positive Homans’ sign, but it is not the first ily. Alteration in the skin’s integrity can
intervention for a client who is oriented to ­become infected.
person only. 4. Smells indicate infection and should be
2. Encouraging the client to take deep breaths ­reported to the HCP.
and cough aids in the exchange of gases. TEST-TAKING HINT: A concept for any injury is
Mental changes are early signs of hypoxia in elevating it above the heart to decrease
the elderly client, but the nurse must first de- edema. Many times the test taker must apply
termine if mental changes have occurred. basic concepts to a variety of client condi-
3. The nurse is not aware of the client’s tions. Any foul smell is not expected in any
usual mental status so, before taking disease or condition.
any further action, the nurse should

11_Colgrove_Ch11.indd 460 01/06/16 5:53 PM


C hapter 11   M usculoskeletal D isorders 461

Content – Medical: Integrated Nursing Process – ­ TEST-TAKING HINT: This is an alternate-type


Planning: Client Needs – Physiological Integrity, question in which the test taker must select
­Physiological Adaptation: Cognitive Level – Synthesis: all options that apply. The test taker should
Concept – Mobility. remember the neuromuscular assessment,
57. 1. Protein is necessary for healing. which includes the 6 Ps—pulse, pain, pares-
2. By wiggling the fingers of the affected arm, thesia, paralysis, pallor, polar (cold).
the client can improve the circulation. Content – Medical: Integrated Nursing Process –
3. Pain medication should be taken prior to per- ­Assessment: Client Needs – Physiological Integrity,
ception of severe pain. Pain relief will require ­Physiological Adaptation: Cognitive Level – Analysis:
more medication if allowed to become severe. Concept – Mobility.
4. The immobilizer should be kept on at all 60. 1. Inserting an indwelling catheter is a good
times. This indicates the client does not intervention, but it is not the first interven-
understand the teaching and needs the tion. A tear or injury to the bladder should be
nurse to provide more instruction. suspected.
TEST-TAKING HINT: When selecting an an- 2. Administering a Fleet’s enema should not be
swer for questions such as this, the test taker implemented until internal bleeding has been
should remember to look for an untrue ruled out.
statement. This indicates teaching is needed. 3. Assessing the bowel sounds should be the
Content – Medical: Integrated Nursing Process – first intervention to determine if an ileus
­Evaluation: Client Needs – Physiological Integrity, has occurred. This is a common complica-
­Physiological Adaptation: Cognitive Level – Synthesis: tion of a fractured pelvis.
Concept – Mobility. 4. Buck’s traction is not used to treat a fractured
pelvis. It is used to treat a fractured hip.
58. 1. The expected outcome for a client with a
fracture is maintaining the function of the TEST-TAKING HINT: When prioritizing two
extremity. equal options, usually, assessing is the
2. Ambulation with assistance is not the best answer.
goal. Content – Medical: Integrated Nursing Process –
3. This is a nursing intervention, not a client Implementation: Client Needs – Safe Effective Care
goal. ­Environment, Management of Care: Cognitive Level –
4. Infection is not the highest priority problem Synthesis: Concept – Mobility.
for a client with a fracture.
TEST-TAKING HINT: The test taker must note the Joint Replacements
words “most appropriate” and look at the client
as a whole entity. With musculoskeletal prob- 61. 1. The abduction pillow should be kept between
lems, maintaining normal function or anatomi- the legs while in bed to maintain a neutral
cal function is the desired outcome. Remember, position and prevent internal rotation.
independence is priority for the client. 2. The client should deep breathe and cough
Content – Medical: Integrated Nursing Process – at least every two (2) hours to prevent atelec-
­Diagnosis: Client Needs – Safe Effective Care Environment, tasis and pneumonia.
Management of Care: Cognitive Level – Analysis: 3. The client will need to turn every two (2)
Concept – Mobility. hours but should not turn to the affected
59. 1. The nurse should assess for numbness side.
and mottled cyanosis, which might indi- 4. Using a high-seated toilet and chair will
cate nerve damage. help prevent dislocation by limiting the
2. The presence of paresthesia and paralysis flexion to less than 90 degrees.
indicates impaired circulation. TEST-TAKING HINT: Option “1” has the word
3. Pulses should be assessed but not proximal “all”; an absolute word such as this usually
to the fracture. Pulses distal to the fracture eliminates the option as a possible correct
should be assessed. Point tenderness should answer. Nursing usually does not have situa-
be expected. tions involving absolutes.
4. Coldness indicates decreased blood sup- Content – Surgical: Integrated Nursing Process –
ply. Crepitus indicates air in subcutaneous ­Implementation: Client Needs – Physiological Integrity,
tissue and is not expected. Basic Care and Comfort: Cognitive Level – Application:
5. Palpable radial pulses and functional move- Concept – Mobility.
ment do not indicate a complication has
occurred.

11_Colgrove_Ch11.indd 461 01/06/16 5:53 PM


462 M ed -S urg S uccess

62. 1. Bruising is common after a total hip margins of the dressing where the tape
replacement. was applied.
2. When a dislocation occurs, the affected 4. Skin reactions to latex are local irritations or
­extremity will be shorter. generalized dermatitis, not blisters.
3. Groin pain or increasing discomfort in the TEST-TAKING HINT: If the test taker does not
affected leg and the “popping sound” in- know the answer, the test taker might think
dicate the leg has dislocated, which should about the dressing because the lesions are on
be reported immediately to the HCP for a the side of the dressing. How is a dressing
possible closed reduction. anchored to the skin? Answer: with tape. The
4. Edema at the incision site is common, but test taker should choose the option having
an increase in edema or redness should be the word “tape.”
reported. Content – Surgical: Integrated Nursing Process –
TEST-TAKING HINT: The nurse should notify ­Diagnosis: Client Needs – Safe Effective Care Environment,
the surgeon of abnormal, unexpected, or Management of Care: Cognitive Level – Analysis:
life-threatening assessment data; if the test Concept – Perioperative.
taker did not have an idea of the answer, pain 65. 1. Clients need to understand the amount of
is always a good choice because pain means weight bearing to prevent injury.
something is wrong—it may be expected 2. Teaching the safe use of assistive devices
pain, but it may mean a complication. is necessary prior to discharge.
Content – Surgical: Nursing Process – Implementation: 3. Increases in activity should occur slowly
Client Needs – Physiological Integrity, Reduction of Risk to prevent complications.
­Potential: Cognitive Level – Synthesis: Concept – Mobility.
4. Using medication therapy, including anal-
63. 1. Clients should not cross their legs because the gesics, anti-inflammatory agents, or muscle
position increases the risk for dislocation. relaxants, should be taught so the client is
2. If the client experiences a sudden increase in comfortable while ambulating.
pain, redness, edema, or stiffness in the joint 5. The client should ambulate with well-fitted,
or surrounding area, the client should notify supported, closed-toed shoes such as a tennis
the HCP. shoe or walking shoe.
3. Clients should sleep on firm mattresses and TEST-TAKING HINT: The test taker should
sit on chairs with firm seats and high arms. apply basic concepts to all surgeries. Many
These will decrease the risk of dislocating the times the test taker may not be familiar with
hip joint. the specific surgery, but by using discharge
4. Infections are possible months after sur- teaching applicable to all clients, a choice can
gery. Clients should monitor tempera- be made.
tures and report any signs of infection. Content – Surgical: Integrated Nursing Process – ­
TEST-TAKING HINT: Note the stem is asking Planning: Client Needs – Physiological Integrity,
about the need for “further teaching.” This ­Physiological Adaptation: Cognitive Level – Synthesis:
means the test taker is looking for an unex- Concept – Mobility.
pected option. This is an “except” question. 66. 1. Minimal pain is expected in a postoperative
Sometimes, if the test taker will change the client but it does indicate surgical treatment
question and say “the client understands the is effective.
teaching,” then the option with an incorrect 2. The client should be able to ambulate with
statement is the answer. almost full mobility.
Content – Surgical: Integrated Nursing Process – 3. A shorter leg indicates a dislocation of
­Evaluation: Client Needs – Physiological Integrity, the hip.
­Physiological Adaptation: Cognitive Level – Synthesis: 4. The hip should have functional motion
­Concept – Perioperative.
and the client should be able to ambulate
64. 1. These are not burns from the cautery unit. to the bathroom. This indicates surgical
Such burns are located in or near the incision treatment has been effective.
site and are usually black. TEST-TAKING HINT: With musculoskeletal
2. Herpes simplex lesions occur in a linear pat- problems, functional movement is priority.
tern along a dermatome. Also note option “2” has the word “limited”
3. Fluid-filled blisters are from a reaction and “3” has “one leg shorter than the other,”
to the tape and usually occur along the both of which are negative outcomes, so the
test taker could eliminate these options.

11_Colgrove_Ch11.indd 462 01/06/16 5:53 PM


C hapter 11   M usculoskeletal D isorders 463

Content – Surgical: Integrated Nursing Process – ­ 3. Adductor pillows are used in clients with total
Assessment: Client Needs – Safe Effective Care hip replacements to maintain function hip
­Environment, Management of Care: Cognitive Level – alignment.
Analysis: Concept – Mobility. 4. The client should perform as many ADLs as
67. 1. Drainage in the first 24 hours can be expected possible. The client should maintain indepen-
to be 200 to 400 mL. When using an auto- dence as much as possible.
transfusion drainage system, the client’s blood TEST-TAKING HINT: The test taker should
will be filtered and returned to the client. remember to think about basic concepts
2. Pain relief with the PCA does not require of surgical care: Would an elastic hose be
­notifying the surgeon. placed over a new incision? Sometimes trying
3. Bilateral coolness of toes is not concerning to imagine what is actually occurring at the
because both feet are cool. Circulation is not bedside helps to eliminate some options.
restricted if pulses are present. Seeing pale Content – Surgical: Integrated Nursing Process –
pink nailbeds indicates blood loss during ­Implementation: Client Needs – Physiological Integrity,
surgery. Physiological Adaptation: Cognitive Level – Analysis:
4. The urinary output is not adequate; ­Concept – Mobility.
therefore, the surgeon needs to be noti-
70. 1. The client’s hemoglobin is near 8 g/dL,
fied. This is only 20 mL/hr. The minimum
which indicates the client requires a blood
should be 30 mL/hr.
transfusion. This information w ­ arrants
TEST-TAKING HINT: A concept the test taker ­intervention by the nurse.
will see throughout testing and throughout 2. This white blood cell count is within nor-
the nurse’s practice is 30 mL of urine output mal limits, so it does not warrant immediate
per hour is necessary to maintain kidney intervention.
function. 3. The creatinine level is within normal limits
Content – Surgical: Integrated Nursing Process – and does not warrant intervention.
Implementation: Client Needs – Safe Effective Care 4. The potassium level is within normal limits
­Environment, Management of Care: Cognitive Level – and does not require intervention by the
­Application: Concept – Perioperative. nurse.
68. 1. The occupational therapist addresses upper TEST-TAKING HINT: The test taker must be
extremity activities of daily living, swallowing knowledgeable of laboratory values. There
issues, and cognition. This is not an appropri- is no test-taking hint except memorize the
ate referral. values.
2. The physiatrist is a physician specializing in Content – Surgical: Integrated Nursing Process –
rehabilitation medicine who practices in a re- ­Assessment: Client Needs – Safe Effective Care Environ-
habilitation setting. ment, Management of Care: Cognitive Level – Synthesis:
3. The recreational therapist is used in psychiat- Concept – Perioperative.
ric settings, rehabilitation hospitals, and long-
71. 1. These vital signs are within normal limits.
term care facilities. The discipline is not seen
2. Pain with dorsiflexion of the ankle
in the home.
indicates deep vein thrombosis. This can
4. The home health-care nurse will be able
be from immobility or surgery; therefore,
to assess the client in the home and make
pain should be assessed in both legs.
further referrals if necessary.
3. Bowel sounds are normally intermittent.
TEST-TAKING HINT: The nurse should always 4. This type of pain should make the nurse
think about safety; therefore, the test taker suspect the client has flatus, which is not a
should select options addressing safety life-threatening complication and does not
issues. warrant immediate intervention.
Content – Surgical: Integrated Nursing Process – TEST-TAKING HINT: “Warrant immediate
Implementation: Client Needs – Safe Effective Care
intervention” means life threatening, abnor-
­Environment, Management of Care: Cognitive Level –
mal, or unexpected for the client’s condi-
­Application: Concept – Health-Care System.
tion. Pain with dorsiflexion of the ankle, the
69. 1. The CPM machine is used to ensure the Homans’ sign, may be life threatening if not
client has adequate range of motion in the treated immediately.
knee postoperatively. Content – Surgical: Integrated Nursing Process – ­
2. The TED hose are only applied to the unaf- Assessment: Client Needs – Safe Effective Care
fected leg, not the leg with the incision. ­Environment, Management of Care: Cognitive Level –
Synthesis: Concept – Perioperative.

11_Colgrove_Ch11.indd 463 01/06/16 5:53 PM


464 M ed -S urg S uccess

72. 1. This is a normal treatment of a fractured TEST-TAKING HINT: When deciding the an-
femoral neck. swer for this type of question, the test taker
2. This is an abnormal occurrence from this who does not know the answer should realize
information. This client should be seen three (3) choices have normal treatments for
first because confusion is a symptom of the disease process and one (1) option con-
hypoxia. tains different information, such as a symp-
3. This is a common treatment of a total hip tom, and choose the option that is different.
replacement. Content – Medical: Integrated Nursing Process – ­
4. This is a treatment used for total knee Assessment: Client Needs – Safe Effective Care
replacement. ­Environment, Management of Care: Cognitive Level –
Analysis: Concept – Cognition.

CONCEPTS

73. 1. Antiembolism hose are to prevent venous 4. A diet high in protein improves wound heal-
stasis, a circulatory issue, not for functional ing. The client’s diet should have sufficient
ability. calories for wound healing but not particu-
2. Nonsteroidal medications (NSAIDs) are larly high carbohydrates. These interventions
given with meals or food to prevent gastric help with tissue integrity and would healing,
distress and risk of bleeding ulcers. They are not functional ability.
not administered on empty stomachs. TEST-TAKING HINT: The test taker should read
3. Low-impact exercises improve the client’s the question carefully; the question is asking
range of motion in the joints and help to about what helps with functional ability. In
maintain functional ability. They should this question, the test taker can eliminate the
be performed on a daily basis. other three (3) options based on the fact that
4. Occupational therapists work on upper body they do not address functional ability. Even
activities and activities of daily living. Physical if the test taker does not “like” option “2,”
therapists work on the lower body and gait it is the odd man out so it should be the one
training as well as large muscle functioning. chosen.
TEST-TAKING HINT: The test taker could elim- Content – Medical: Integrated Nursing Process –
inate option “4” by knowing the function of Implementation: Client Needs – Physiological Integrity,
the different therapies, “1” can be eliminated Reduction of Risk Potential: Concept – Mobility.
by knowing the purpose of antiembolism
75. 1. The nurse should medicate the client for pain
hose, and “2” can be eliminated by knowing
prior to therapy if that is determined to be
basic nursing interventions for classifications
the cause for the client refusing to participate
of medications.
in therapy.
Content – Medical: Integrated Nursing Process – 2. The client can make his/her own deci-
Implementation: Client Needs – Physiological Integrity,
sions. The HCP cannot make the client do
Reduction of Risk Potential: Concept – Functional Ability.
anything.
74. 1. The tourniquet is used to prevent hemor- 3. The nurse should explain the rules of reha-
rhage from the residual limb. It does not im- bilitation coverage, but this is not first.
prove functional ability. 4. The nurse should first assess the
2. The client should push against a pillow situation to determine the reason the cli-
to toughen the stump and prepare it for ent does not wish to participate in therapy.
a prosthesis. This will assist the ­client in TEST-TAKING HINT: The test taker should re-
regaining functional ability and mobility. member the first step of the nursing process
3. Taking and documenting vital signs provides is assessment. There are many words that
the nurse with data to determine the stability can be used to indicate an assessment step.
of the client but does not improve functional Content – Medical: Integrated Nursing Process –
ability. ­Assessment: Client Needs – Health Promotion and Safety:
Concept – Functional Ability.

11_Colgrove_Ch11.indd 464 01/06/16 5:53 PM


C hapter 11   M usculoskeletal D isorders 465

76. 1. This picture does not indicate any deformity 3. Difficulty initiating a urinary stream usually
except the hands. indicates a male client has benign prostatic
2. This is a picture of swan neck fingers as- hypertrophy. The prostate is blocking the
sociated with rheumatoid arthritis. The urethra.
function of the client’s hands is priority. 4. A fall and wearing a cast on the arm do not
3. The client may have an issue with coping indicate degenerative disk (vertebra of the
with the RA, but a psychosocial need is back) disease.
not priority over an actual physiological need. TEST-TAKING HINT: The test taker could
4. Rehabilitation would be an interrelated con- eliminate option “4” if familiar with medi-
cept needed for this client, but determining cal terminology; disks refer to the back, not
the extent of functional impairment is prior- the arm. Option “2” is nonspecific as to the
ity first. reason for taking the NSAIDs and could be
TEST-TAKING HINT: The test taker should not eliminated.
read into the question; in option “1” mobil- Content – Medical: Integrated Nursing Process –
ity refers to the ability of the client to move ­Assessment: Client Needs – Physiological Integrity,
the body, not just the hands. The hands are ­Reduction of Risk Potential: Concept – Mobility.
the only body parts pictured. If an option
79. 1. The RBC is WNL; there is no reason for the
exists that is more specific to the picture
nurse to notify the surgeon.
(functional ability) then the test taker should
2. The hemoglobin is WNL; there is no reason
choose the one that is most closely related to
for the nurse to notify the surgeon.
the issue.
3. The client is going to surgery and bone
Content – Medical: Integrated Nursing Process – surgeries result in blood loss. This client’s
­Planning: Client Needs – Physiological Integrity, Reduction
platelet count is below 100,000, impair-
of Risk Potential: Concept – Functional Ability.
ing the client’s ability to clot. The nurse
77. 1. Celebrex is approved for the routine treat- should immediately notify the surgeon.
ment of arthritis and bone and joint dis- 4. The sedimentation rate is WNL; there is no
eases. The nurse would not question this reason for the nurse to notify the surgeon.
medication. TEST-TAKING HINT: When reading a graph
2. Clients undergoing treatment for cancer or a laboratory report, the test taker should
frequently require filgrastim to promote the read the normal ranges carefully in order to
production of WBCs due to the suppression identify what is abnormal.
of the bone marrow. The nurse would not Content – Medical: Integrated Nursing Process –
question this medication. ­Assessment: Client Needs – Physiological Integrity,
3. An adverse effect of adalimumab is the ­Reduction of Risk Potential: Concept – Clotting/ Mobility.
development of lymphoma. Because this
client has a lymphoma, suppressing the 80. 1. Placing blame is not therapeutic.
immune system even further could have 2. Unless the nurse has had exactly the same
disastrous results for the client. The situation happen to him/her, the nurse cannot
nurse would question administering this make this statement. It is not therapeutic.
medication. 3. Asking why is not therapeutic; the nurse
4. Acetaminophen would not be questioned; is ­requesting the client to defend his/her
the client has pain and the dose is a recom- feeling.
mended dose. 4. This is a broad opening statement
and offering self. Both are therapeutic
TEST TAKING HINT: The test taker must know
techniques. The client needs an oppor-
­basic medication guidelines.
tunity to verbalize the feeling associated
Content – Medical: Integrated Nursing Process – with loss.
Implementation: Client Needs – Pharmacological and
Parenteral Therapies: Concept – Functional Ability. TEST-TAKING HINT: When a question asks
for a therapeutic response by the nurse, the
78. 1. A history of low back pain and multiple nurse must give a response that does NOT
back surgeries indicates a history of disk lay blame or advise in any way and MUST
and back issues. encourage the client to discuss feelings.
2. The use of NSAIDS could have happened for
reasons other than degenerative disk disease.

11_Colgrove_Ch11.indd 465 01/06/16 5:53 PM


466 M ed -S urg S uccess

Content – Medical: Integrated Nursing Process – Content – Medical: Integrated Nursing Process –
­Assessment: Client Needs – Psychosocial Integrity: Implementation: Client Needs – Physiological Integrity,
Concept – Mobility. Reduction of Risk Potential: Concept – Mobility/Comfort.

81. 1. A physical therapist will assist the client to 83. 1. Occupational therapists work on upper
ambulate safely while protecting the hip body ability and activities of daily living as
from being displaced. well as increasing cognitive ability. This is
2. The client’s mobility is compromised, an excellent referral.
placing the client at risk for developing a 2. Clients who no longer have the use of a
deep vein thrombosis (DVT). Lovenox will side of the body will not realize when the
assist in preventing a DVT. arm or leg moves and this can be a safety
3. Health-care workers should use gait belts issue.
to provide support and stability when 3. The client may not realize that one-half of
­ambulating clients. the visual field has been impaired as a re-
4. Fractures of any bone are painful; pain sult of the stroke. If this has happened the
scales are useful in qualifying the amount client will not see things in the left visual
and type of pain being experienced by the fields. Remember that the nerve pathways
client. cross over at the base of the skull, so a left-
5. The client’s diet should be a well-balanced sided stroke produces issues for the body
diet with an emphasis on protein for wound below the neck on the opposite side of the
healing. stroke, but in the brain (visual fields) it
TEST-TAKING HINT: Knowledge of basic would be on the side of the stroke.
­nursing care is required when answering this 4. For safety this should be done for all
question. The use of a gait belt for ambu- clients.
lating and assessing pain are basic nursing 5. The nurse should encourage the client to
skills. participate in activity that increases the
client’s functional ability.
Content – Medical: Integrated Nursing Process –
Implementation: Client Needs – Physiological Integrity, TEST-TAKING HINT: When answering “Select
­Reduction of Risk Potential: Concept – Mobility. all that apply” questions, each option is read
independently of the others. Each option be-
82. 1. The nurse or nursing staff should assist
comes a true/false question.
the client to ambulate to the bathroom,
and pain medication should be adminis- Content – Medical: Integrated Nursing Process –
­Assessment: Client Needs – Physiological Integrity,
tered using the pain scale to quantify and
­Reduction of Risk Potential: Concept – Functional Ability.
qualify the pain level.
2. The client should have bathroom privileges; 84. 1. The client should be able at age 35 to per-
strict bedrest will place the client at risk for form exercises on his/her own.
pneumonia and DVT development. Move- 2. The client is 35 and should prepare to live
ment should be encouraged within safe life with the new limitations, not go into an
guidelines. A regular bedpan would place the assistive living facility.
client’s back in an awkward position and in- 3. The client needs to gain new skills to be-
crease the pain. come a productive member of society with
3. Pain is whatever the client says it is and oc- the new limitations. All states have oppor-
curs whenever the client says it does. The tunities for clients who have issues to be
nurse should not discourage the use of pain able to access training and assistance.
medications in the light of “severe” pain. 4. The client has a functional disability, not a
4. The HCP does not need to assist the client to dietary need.
ambulate in the hallway to observe the effect TEST-TAKING HINT: The test taker must read
of ambulation on the client’s pain. The HCP the words in the question and options. Ages
can ask the client to ambulate in the room with matter. This is a 35-year-old client who
the assistance of the UAP, nurse, or PT. should be in Erikson’s stage of Generativity
TEST-TAKING HINT: The test taker should read vs. Stagnation.
words in the stem of a question and in the Content – Medical: Integrated Nursing Process –
options—words matter. In option “2” “strict” ­Assessment: Client Needs – Psychological Integrity,
and “regular” make this an incorrect op- ­Potential: Concept – Functional Ability.
tion; in option “3” “discourage” makes it
incorrect. In option “4” HCPs do not do the
nurse’s job of ambulating.

11_Colgrove_Ch11.indd 466 01/06/16 5:53 PM


C hapter 11   M usculoskeletal D isorders 467

MUSCULOSKELETAL DISORDERS
COMPREHENSIVE EXAMINATION
1. The 50-year-old female client is being evaluated 6. Which priority intervention should the day
for osteoporosis. Which data should the nurse surgery nurse implement for a client who has had
assess? Select all that apply. right knee arthroscopy?
1. Family history of osteoporosis. 1. Encourage the client to perform range-of-
2. Estrogen or androgen deficit. motion exercises.
3. Exposure to secondhand smoke. 2. Monitor the amount and color of the urine.
4. Level and amount of exercise. 3. Check the client’s pulses distally and assess
5. Alcohol intake. the toes.
4. Monitor the client’s vital signs.
2. Which intervention should the nurse include for a
client diagnosed with carpal tunnel syndrome? 7. The client is prescribed Fosamax, a
1. Teach hyperextension exercises to increase bisphosphonate. Which information should the
flexibility. nurse teach?
2. Monitor safety during occupational hazards. 1. Take this medication with a full glass of water.
3. Prepare for the insertions of pins or screws. 2. Take with breakfast to prevent gastrointestinal
4. Monitor dressing and drain after the upset.
fasciotomy. 3. Use sunscreen to prevent sensitivity to
sunlight.
3. Which staff nurse should the charge nurse assign
4. This medication increases calcium
to the client recovering from a repair of the hallux
reabsorption.
valgus?
1. A new graduate nurse. 8. The school nurse is completing spinal screenings.
2. An experienced nurse. Which data require a referral to an HCP?
3. A nurse practitioner. 1. Bilateral arm lengthening while bending over
4. An unlicensed assistive personnel. at the waist.
2. A deformity which resolves when the head is
4. The client is scheduled for a computed
raised.
tomography (CT) scan. Which question is
3. Equal spacing of the arms and body at the
most important for the nurse to ask before the
waist.
procedure?
4. A right arm lower than the left while bending
1. “On a scale of 1 to 10, how do you rate your
over at the waist.
pain?”
2. “Do you feel uncomfortable in enclosed 9. The clinic nurse assesses a client with complaints
spaces?” of pain and numbness in the left hand and fingers.
3. “Are you allergic to seafood or iodine?” Which question should the nurse ask the client?
4. “Have you signed a permit for this procedure?” 1. “Do you smoke or use any type of tobacco
products?”
5. Two unlicensed assistive personnel (UAP) are
2. “Do you have to wear gloves when you are out
using the transfer board to move the client from
in the cold?”
the bed to the wheelchair. Which action should
3. “Do you do repetitive movements with your
the nursing take?
left fingers?”
1. Take no action because this is the correct
4. “Do you have tremors or involuntary
procedure for transferring a client.
movements of your hand?”
2. Instruct the UAPs not to use a transfer board
when moving the client.
3. Tell the UAPs to use the bed scale sling to
move the client to the chair.
4. Request the UAPs to stop and come to the
nurse’s station immediately.

11_Colgrove_Ch11.indd 467 01/06/16 5:53 PM


468 M ed -S urg S uccess

10. Which statement by the client prescribed 15. The client is postoperative open reduction and
calcitonin, a thyroid hormone, indicates to the internal fixation (ORIF) of a fractured femoral
nurse the teaching has been effective? neck. Which long-term goal should the nurse
1. “I should administer the mediation in a identify for the client?
different nostril each day.” 1. The client will maintain vital signs within
2. “I need to drink a lot of water when I take my normal limits.
medicine.” 2. The client will have a decrease in muscle
3. “I have to dilute the medication with vitamin spasms in the affected leg.
D before I take it.” 3. The client will have no signs or symptoms of
4. “This medication will help the calcium leave infection.
my bones.” 4. The client will be able to ambulate down to
the nurse’s station.
11. The client diagnosed with rule-out osteosarcoma
asks the nurse, “Why am I having a bone scan?” 16. The nurse is preparing to administer
Which statement is the nurse’s best response? subcutaneous Lovenox, a low molecular weight
1. “You seem anxious. Tell me about your heparin. Which intervention should the nurse
anxieties.” implement?
2. “Why are you concerned? Your HCP ordered 1. Monitor the client’s serum aPTT.
it.” 2. Encourage oral and intravenous fluids.
3. “I’ll have the radiologist come back to explain 3. Do not eat foods high in vitamin K.
it again.” 4. Administer in the anterolateral upper
4. “A bone scan looks for cancer or infection abdomen.
inside the bones.”
17. Which intervention should the nurse implement
12. The client is scheduled for a magnetic resonance for a client with a fractured hip in Buck’s
imaging (MRI) scan. Which intervention should traction?
the nurse delegate to the unlicensed assistive 1. Assess the insertion sites for signs and
personnel (UAP)? symptoms of infection.
1. Prepare the client by removing all metal 2. Monitor for drainage or odor from under the
objects. plaster covering the pins.
2. Inject the contrast into the intravenous site. 3. Check the condition of the skin beneath
3. Administer a sedative to the client to decrease the Velcro boot frequently.
anxiety. 4. Take weights off for one (1) hour every
4. Explain why the client cannot have any eight (8) hours and as needed.
breakfast.
18. The nurse is caring for a client in a hip spica
13. A client sustained a fractured femur in a motor- cast. Which intervention should the nurse
vehicle accident. Which data require immediate include in the plan of care?
intervention by the nurse? Select all that 1. Assess the client’s popliteal pulses every shift.
apply. 2. Elevate the leg on pillows and apply ice packs.
1. The client requests pain medication to sleep. 3. Teach the client how to ambulate with a
2. The client has eupnea and normal sinus tripod walker.
rhythm. 4. Assess the client for distention and vomiting.
3. The client has petechiae over the neck and
19. The nurse is discharge teaching for a client with
chest.
a short leg cast. Which statement indicates the
4. The client has a high arterial oxygen level.
client understands the discharge
5. The client has yellow globules floating in the
teaching?
urine.
1. “I need to keep my leg elevated on two
14. The nurse is caring for the client diagnosed pillows for the first 24 hours.”
with fat embolism syndrome. Which HCP order 2. “I must wear my sequential compression
should the nurse question? device all the time.”
1. Maintain heparin to achieve a therapeutic 3. “I can remove the cast for one (1) hour so
level. I can take a shower.”
2. Initiate and monitor intravenous fluids. 4. “I will be able to walk on my cast and not have
3. Keep the O2 saturation higher than 93%. to use crutches.”
4. Administer an intravenous loop diuretic.

11_Colgrove_Ch11.indd 468 01/06/16 5:53 PM


C hapter 11   M usculoskeletal D isorders 469

20. Which psychosocial problem should the nurse 25. Which information should the nurse teach the
identify for a client with an external fixator client regarding sports injuries?
device? 1. Apply heat intermittently for the first 48
1. Ineffective coping. hours.
2. Alteration in body image. 2. An injury is not serious if the extremity can be
3. Grieving. moved.
4. Impaired communication. 3. Only return to the health-care provider if the
foot becomes cold.
21. A client recovering from a total hip replacement
4. Keep the injury immobilized and elevated for
has developed a deep vein thrombosis. The
24 to 48 hours.
health-care provider has ordered a continuous
infusion of heparin, an anticoagulant, to infuse at 26. The emergency department nurse is caring for
1,200 units per hour. The bag comes with 20,000 a client with a compound fracture of the right
units of heparin in 500 mL of 0.9% normal ulna. Which interventions should the nurse
saline. At what rate should the nurse set the implement? List in order of priority.
pump? ______ 1. Apply a sterile, normal saline–soaked gauze to
the arm.
22. The nurse is caring for a client with a left
2. Send the client to radiology for an x-ray of
fractured humerus. Which data warrant
the arm.
intervention by the nurse?
3. Assess the fingers of the client’s right hand.
1. Capillary refill time is less than three (3)
4. Stabilize the arm at the wrist and the elbow.
seconds.
5. Administer a tetanus toxoid injection.
2. Pain is not relieved by the patient-controlled
analgesia. 27. The emergency department nurse is caring for
3. Left fingers are edematous and the left hand a 6-year-old child with a fractured forearm and
is purple. suspects the injury is the result of abuse. Which
4. Warm and dry skin on left fingers distal to the x-ray would confirm the suspicions for the
elastic bandage. nurse?
23. The client with a long arm cast is complaining 1. 2. 3. 4.
of unrelenting severe pain and feeling as if the
fingers are asleep. Which complication should
the nurse suspect the client is experiencing?
1. Fat embolism.
2. Compartment syndrome.
3. Pressure ulcer under the cast.
4. Surgical incision infection.
24. The elderly client is admitted to the hospital for
severe back pain. Which data should the nurse
assess first during the admission assessment?
1. The client’s use of herbs.
2. The client’s current pain level.
3. The client’s sexual orientation.
4. The client’s ability to care for self.

11_Colgrove_Ch11.indd 469 01/06/16 5:53 PM


MUSCULOSKELETAL DISORDERS COMPREHENSIVE
EXAMINATION ANSWERS AND RATIONALES

1. 1. Clients are more prone to have osteoporo- 4. 1. The assessment of the pain is important so the
sis if there is a genetic predisposition. client will be able to tolerate the procedure.
2. Clients who are deficient in either estrogen Pain is not a life-threatening problem but is a
or androgen are at risk for osteoporosis. quality-of-care issue.
3. Clients who smoke are more at risk for osteo- 2. This is an appropriate question for a client
porosis. Research does not show a correlation having a closed MRI, not a CT scan.
between osteoporosis and secondhand smoke. 3. This is the most important information
4. Regular, weight-bearing exercise p ­ romotes the nurse should obtain. Any client who is
healthy bones. allergic to seafood cannot be injected with
5. Clients who consume alcohol and have the iodine-based contrast. This contrast
­diets low in calcium are at a higher risk for could cause an allergic response, endanger-
osteoporosis. ing the client’s life.
Content – Medical: Integrated Nursing Process – 4. The general consent for admission to the
­Assessment: Client Needs – Physiological Integrity, Reduction hospital covers this procedure. A separate in-
of Risk Potential: Cognitive Level – Analysis: Concept – formed consent is not required.
Fluid and Electrolyte Balance. Content – Surgical: Integrated Nursing Process –
2. 1. Treatment for carpal tunnel syndrome does Assessment: Client Needs – Physiological Integrity, ­Reduction
of Risk Potential: Cognitive Level – ­Analysis: Concept –
not include hyperextension of the wrist.
Perioperative.
2. The nurse should monitor for potential in-
juries resulting from the alterations in mo- 5. 1. The UAPs are transferring the client
tor, sensory, and autonomic function of the ­correctly and safely, so no action should
first three (3) digits of the hand and palmar be taken. The UAPs are adhering to the
surface of the fourth. Patient Care Safety Standards by using
3. Surgery may be needed to release the com- ­approved equipment.
pression of the medial nerve, but pins and 2. The nurse should encourage the use of appro-
screws are used to hold the position. priate equipment designed to protect the client
4. Fasciotomy refers to the surgical excision of and the staff from injury.
strips of connective tissue. This is not appli- 3. The bed scale sling is inappropriate to use
cable in clients with carpal tunnel syndrome. when moving the client from the bed to a
Content – Medical: Integrated Nursing Process – wheelchair.
Planning: Client Needs – Physiological Integrity, P
­ hysiological 4. There is no reason for the nurse to stop the
Adaptation: Cognitive Level – Synthesis: Concept – UAPs because the task is being performed
Mobility. correctly.
3. 1. A new graduate is the best choice for this Content – Medical: Integrated Nursing Process –
client. The client’s surgery (correction ­Implementation: Client Needs – Safe Effective Care
­Environment, Management of Care: Cognitive Level –
of a hammer toe) is not a high-risk pro-
­Application: Concept – Safety.
cedure but requires assessment and pain
management. 6. 1. The nurse should not encourage range of mo-
2. This client does not need a more experienced tion until the surgeon gives permission for
nurse. flexion of the knee.
3. A nurse practitioner does not need to be as- 2. Urinary output is important postoperatively,
signed to this client. but monitoring is not priority over a neurovas-
4. The UAP is not assigned the responsibility of cular assessment.
managing the care of a client; the UAP works 3. Neurovascular assessment is priority be-
under the guidance of the nurse. cause this surgery has two (2) to three (3)
Content – Surgical: Integrated Nursing Process – small incisions in the knee area. The nurse
Planning: Client Needs – Safe Effective Care Environment, needs to make sure circulation is getting
Management of Care: Cognitive Level – Synthesis: past the surgical site.
Concept – Management. 4. Vital signs should be assessed, but the priority
is to maintain the neurovascular status of the
limb.
470

11_Colgrove_Ch11.indd 470 01/06/16 5:53 PM


C hapter 11   M usculoskeletal D isorders 471

Content – Surgical: Integrated Nursing Process – Imple- 4. Calcium should be retained in the bone to
mentation: Client Needs – Safe Effective Care Environ- maintain bone strength; medications are not
ment, Management of Care: Cognitive administered to encourage loss from the bone.
Level – Application: Concept – Perioperative.
Content – Medical: Integrated Nursing Process –
7. 1. The client needs to take this medication ­Evaluation: Client Needs – Physiological Integrity,
with a full glass of water and remain up- ­Pharmacological and Parenteral Therapies: Cognitive
right for at least 30 minutes to reduce the Level – Synthesis: Concept – Medication.
risk of esophagitis. 11. 1. This is a therapeutic technique, but the cli-
2. This medication should be taken before ent is asking for information. When a client
breakfast on an empty stomach. seeks information, the nurse should give in-
3. This medication does not cause formation first. Discussion of feelings should
photosensitivity. follow.
4. This medication decreases calcium reabsorp- 2. This nontherapeutic technique blocks com-
tion by decreasing the activity of osteoclasts. munication between the client and the nurse.
Content – Medical: Integrated Nursing Process – The nurse should avoid a response with the
Planning: Client Needs – Physiological Integrity, word “why,” which asks the client to explain
­Pharmacological and Parenteral Therapies: Cognitive or justify feelings to the nurse.
Level – Synthesis: Concept – Medication. 3. When the client requests information, the
8. 1. These are normal data and do not require nurse needs to provide accurate information,
intervention. not pass the buck.
2. If the screener suspects the client has scoliosis 4. This statement answers the client’s
while the client is bending over, the screener question.
asks the client to raise the head. An abnormal- Content – Medical: Integrated Nursing Process –
ity caused by scoliosis will not resolve. Implementation: Client Needs – Physiological
3. This indicates a normal occurrence and does Integrity, Reduction of Risk Potential: Cognitive
not need to be referred. Level – Application: Concept – Cellular Regulation.
4. Unequal arm length may indicate scolio- 12. 1. Metal objects such as jewelry and zippers
sis, and further assessment is needed by can interfere with the magnetic imaging
an HCP. and pose a danger to the client as a result
Content – Medical: Integrated Nursing Process – of the magnetic properties of the equip-
Assessment: Client Needs – Safe Effective Care ment. This intervention can be delegated
Environment, Management of Care: Cognitive Level – to the UAP.
Analysis: Concept – Mobility. 2. Injection of contrast is given in the radiology
9. 1. Assessing for smoking is evaluation for department.
­Raynaud’s disease. 3. UAPs are unable to administer medications in
2. Exposure to cold is appropriate to assess for hospitals.
Raynaud’s disease. 4. The nurse cannot delegate teaching to a UAP.
3. Repetitive movements are appropriate to Content – Medical: Integrated Nursing Process –
assess for carpal tunnel syndrome. ­Clients Planning: Client Needs – Safe Effective Care
with this disorder experience pain and Environment, Management of Care: Cognitive
numbness. Level – Synthesis: Concept – Nursing Roles.
4. Tremors or involuntary movements could 13. 1. The client requesting something for sleep
indicate Parkinson’s disease. is expected and does not require notifying
Content – Medical: Integrated Nursing Process – the HCP.
Assessment: Client Needs – Physiological Integrity, 2. Normal respirations and heart rate do not
­Reduction of Risk Potential: Cognitive Level – ­Analysis: require notifying the HCP.
Concept – Mobility. 3. Petechiae are macular, red-purple
10. 1. This medication is administered intrana- ­pinpoint bleeding under the skin. The
sally. Alternating nostrils will decrease the ­appearance of petechiae is a classic sign of
risk of nasal irritation. fat embolism syndrome.
2. This intervention should be implemented for 4. The arterial oxygen level would be low, not
Fosamax, a bisphosphonate, not calcitonin, a elevated. This sign does not warrant immedi-
thyroid hormone. ate intervention.
3. Clients do not dilute their medication. Vita- 5. Yellow globules in the urine are fat glob-
min D is not used as a diluent for medication. ules released from the bone as it breaks.
This should be reported immediately.

11_Colgrove_Ch11.indd 471 01/06/16 5:53 PM


472 M ed -S urg S uccess

Content – Medical: Integrated Nursing Process – 17. 1. Skeletal traction has a pin, screws, tongs, or
Assessment: Client Needs – Safe Effective Care wires inserted into the bone. There is no in-
­Environment, Management of Care: Cognitive sertion site in skin traction.
Level – Synthesis: Concept – Mobility. 2. Plaster traction is a combination of skeletal
14. 1. The HCP should prescribe heparin to treat a traction using pins and a plaster brace to
fat embolism. maintain alignment of any deformities.
2. The client should be hydrated to prevent 3. In Buck’s traction, a Velcro boot is used
platelet aggregation. to attach the ropes to weights to maintain
3. The nurse should monitor oxygen levels and alignment. Skin covered by the boot can
administer oxygen as needed to prevent fur- become irritated and break down.
ther complications. 4. Buck’s traction is applied preoperatively to
4. The nurse should question this order. prevent muscle spasms and maintain align-
This will decrease the client’s hydration ment, and the weights should not be removed
and may result in further embolism. unless assessing for skin breakdown.
Content – Medical: Integrated Nursing Process – Content – Medical: Integrated Nursing Process –
­Implementation: Client Needs – Safe Effective Care ­Implementation: Client Needs – Safe Effective Care
­Environment, Management of Care: Cognitive ­Environment, Management of Care: Cognitive Level –
Level – Analysis: Concept – Clotting. Analysis: Concept – Mobility.

15. 1. Vital signs remaining stable is a short-term 18. 1. The client’s popliteal pulse will be under the
goal, not a long-term goal. cast and cannot be assessed by the nurse; cir-
2. This is an expected short-term outcome for culation is assessed by the 6 Ps of the neuro-
a preoperative client with a fractured femoral vascular assessment.
neck. 2. Elevation should be used with an arm cast or
3. No signs/symptoms of infection is a short- a leg cast, but this is not possible with a spica
term goal for the nurse to identify in the cast.
hospital. 3. Clients with spica casts will not be able to
4. The discharge goal or long-term goal ambulate because the cast covers the entire
for this client is to return the client to lower half of the body.
­ambulatory status. 4. The nurse should assess the client for
Content – Surgical: Integrated Nursing Process – signs and symptoms of cast syndrome—
Diagnosis: Client Needs – Physiological Integrity, vomiting after meals, epigastric pain, and
­Physiological Adaptation: Cognitive Level – Analysis: ­ abdominal distention. This is caused by a
Concept – Mobility. partial bowel obstruction from compres-
sion and can lead to complete obstruction.
16. 1. An aPTT is used to determine therapeutic The client may still have bowel sounds
levels of unfractionated heparin. Labora- present with this syndrome.
tory studies such as aPTT are not monitored
Content – Medical: Integrated Nursing Process – ­
when administering subcutaneous Lovenox, a
Diagnosis: Client Needs – Physiological Integrity,
low molecular weight heparin. A therapeutic ­Physiological Adaptation: Cognitive Level – Analysis:
level will not be achieved as a result of a short Concept – Mobility.
half-life.
2. Oral fluids do not need to be increased 19. 1. This is a correct intervention. The leg
­because of this medication. should be elevated for at least the first
3. Vitamin K is the antidote for warfarin 24 hours. If edema is present, the client
(­Coumadin), an oral anticoagulant. It does needs to keep it elevated longer.
not affect Lovenox. 2. Sequential compression devices work to pre-
4. Administering the medication in the pre- vent deep vein thrombosis and the client does
scribed areas, the “love handles,” ensures not wear one of these at home.
safety and decreases the risk of abdominal 3. The client will not be able to remove the cast
trauma. for any reason. The cast must be cut off.
Content – Medical: Integrated Nursing Process – 4. Clients with casts can only ambulate if they
Implementation: Client Needs – Physiological have a walking cast or boot. This information
Integrity, Pharmacological and Parenteral Therapies: is not in the stem of the question.
Cognitive Level – Application: Concept – Medication.

11_Colgrove_Ch11.indd 472 01/06/16 5:53 PM


C hapter 11   M usculoskeletal D isorders 473

Content – Medical: Integrated Nursing Process – Content – Medical: Integrated Nursing Process –
­Evaluation: Client Needs – Physiological Integrity, Assessment: Client Needs – Safe Effective Care
­Physiological Adaptation: Cognitive Level – Synthesis: Environment, Management of Care: Cognitive
­Concept – Mobility. Level – Synthesis: Concept – Mobility.

20. 1. The client problem of ineffective coping is 23. 1. These are not signs/symptoms of a fat
usually not indicated for a client with an ex- embolism.
ternal fixator device, unless the stem provides 2. These are the classic signs/symptoms
more information about the client. of compartment syndrome.
2. Many clients with an external fixator 3. Clients in casts rarely develop pressure ­ulcers
have alterations in body image because and usually they are not
the large, bulky frame makes dressing painful.
difficult and because of scarring, which 4. Hot spots on the cast usually indicate an
occurs from the trauma and treatment. ­infection of the surgical incision under
The length of healing is prolonged, so the cast.
returning to the client’s normal routine is Content – Medical: Integrated Nursing Process –
delayed. ­Diagnosis: Client Needs – Physiological Integrity, Reduction
3. The client problem of grieving is usually not of Risk Potential: Cognitive Level – Analysis: Concept –
indicated for a client with an external fixator Mobility.
device, unless the stem of the question pro- 24. 1. This is a question the admitting nurse asks all
vides more information about the client. clients, but it is not the most important.
4. The client problem of impaired communica- 2. Pain assessment and management are
tion is usually not indicated for a client with the most important issues if the client is
an external fixator device, unless the stem breathing and has circulation. Lack
provides more information about the client. of pain management decreases the at-
Content – Medical: Integrated Nursing Process – tention of the client during the admis-
­Diagnosis: Client Needs – Psychosocial Integrity: sion process. Pain is called the fifth vital
Cognitive Level – Analysis: Concept – Self.
sign.
21. 30 mL/hr. 3. Sexual practices are included in the admission
Divide the amount of heparin by the volume forms, but they are not as important as pain
of fluid to get the concentration: management.
20,000 units ÷ 500 mL = 40 units of heparin 4. Assessing the client’s ability to perform
per 1 mL activities of daily living and self-care is im-
Divide the dose ordered by the concentration portant to prepare this client for discharge,
for the amount of milliliters per hour to set which begins on admission, but this is not
the pump: the most important at this time.
1,200 units/hr ÷ 40 units/mL = 30 mL/hr Content – Medical: Integrated Nursing Process –
Content – Medical: Integrated Nursing Process – ­Assessment: Client Needs – Physiological Integrity,
Implementation: Client Needs – Physiological Integrity, ­Reduction of Risk Potential: Cognitive Level – Analysis:
Pharmacological and Parenteral Therapies: ­Cognitive Concept – Comfort.
Level – Application: Concept – Medication.
25. 1. Ice should be applied intermittently for the
22. 1. This is a normal assessment finding and does first 48 hours. Heat can be used later in the
not require immediate action. recovery process.
2. Unrelieved pain should warrant inter- 2. Severe injury can be present even with some
vention by the nurse. Pain may indicate range of motion.
a complication or the need for pain 3. The client needs to return if the injury does
medication, but either way it warrants not improve and if the foot gets cold.
intervention. 4. The leg should be iced, elevated, and
3. Edema and a hematoma as a result of the ­immobilized for 48 hours.
injury are expected and do not warrant Content – Medical: Integrated Nursing Process –
­intervention by the nurse. Planning: Client Needs – Physiological Integrity,
4. The fingers distal to the Ace bandage in- ­Physiological Adaptation: Cognitive Level – Synthesis:
dicate adequate circulation and require no Concept – Mobility.
intervention.

11_Colgrove_Ch11.indd 473 01/06/16 5:53 PM


474 M ed -S urg S uccess

26. The order should be 4, 1, 3, 2, 5. 27. 1. A compound fracture is a fracture where the
4. The nurse first should stabilize the arm to bone protrudes through the skin; it is also
prevent further injury. called an open fracture, and the nurse would
1. A compound fracture is one in which the not suspect child abuse based on only the
bone protrudes through the skin. The type of fracture.
nurse should apply sterile, saline-soaked 2. A spiral fracture is a fracture that involves
gauze to protect the area from the intru- twisting around the shaft of the bone,
sion of bacteria. such as when an adult twists the arm of
3. The nurse should assess the client’s circu- a child. The nurse should suspect child
lation to the part distal to the injury. This abuse.
is done after the first two interventions 3. An oblique fracture is a fracture that remains
because life-threatening complications contained and does not break the skin. There
could occur if stabilization and protection are many reasons the child could have this
from infection are not addressed first. type of fracture other than child abuse.
2. An x-ray will be needed to determine the 4. A greenstick fracture is a fracture in which
extent of the injury. one side of the bone is broken and the other
5. A tetanus toxoid injection should be side is bent. There are many reasons other
­administered, but this can be done last. than child abuse that could account for this
Content – Medical: Integrated Nursing Process – type of fracture.
­Implementation: Client Needs – Safe Effective Care Content – Medical: Integrated Nursing Process –
­Environment, Management of Care: Cognitive ­Assessment: Client Needs – Physiological Integrity,
Level – Application: Concept – Mobility. ­Physiological Adaptation: Cognitive Level – Synthesis:
Concept – Mobility.

11_Colgrove_Ch11.indd 474 01/06/16 5:53 PM


Integumentary
Disorders
Knowledge is of two kinds: we know a subject, or we know where
we can find information upon it.
12
—Samuel Johnson

The integument, or skin—the largest organ in the body—is subject to many disorders.
Some, like the diseases/disorders that affect other body systems, are infectious; included are
bacterial, viral, and fungal infections of the skin. Burns and pressure ulcers also affect the
skin. This chapter discusses these disorders and other problems, including psoriasis, sebor-
rheic dermatosis, and contact dermatitis.

KEYWORDS ABBREVIATIONS
Escharotomy Acquired immunodeficiency syndrome (AIDS)
Keloid Arterial blood gases (ABGs)
Paraplegia Cerebrovascular accident (CVA)
Quadriplegia Emergency department (ED)
Fluid volume deficit (FVD)
Health-care provider (HCP)
Intravenous (IV)
Intravenous push (IVP)
Over-the-counter (OTC)
Sun protection factor (SPF)
Ultraviolet (UV) light
Unlicensed assistive personnel (UAP)
Within normal limits (WNL)

PRACTICE QUESTIONS
Burns 2. The client with full-thickness burns to 40% of
the body, including both legs, is being transferred
1. The client comes into the emergency department from a community hospital to a burn center.
in severe pain and reports that a pot of boiling hot Which measure should be instituted before the
water accidentally spilled on his lower legs. The transfer?
assessment reveals blistered, mottled red skin, and 1. A 22-gauge intravenous line with normal saline
both feet are edematous. Which depth of burn infusing.
should the nurse document? 2. Wounds covered with moist sterile dressings.
1. Superficial partial thickness. 3. No intravenous pain medication.
2. Deep partial thickness. 4. Ensure adequate peripheral circulation to both
3. Full thickness. feet.
4. First degree.

475

12_Colgrove_Ch12.indd 475 01/06/16 5:54 PM


476 M ed -S urg S uccess

3. The client has full-thickness burns to 65% of the 8. Which nursing interventions should be included
body, including the chest area. After establishing for the client who has full-thickness and deep
a patent airway, which collaborative intervention partial-thickness burns to 50% of the body?
is priority for the client? Select all that apply.
1. Replace fluids and electrolytes. 1. Perform meticulous hand hygiene.
2. Prevent contractures of extremities. 2. Use sterile gloves for wound care.
3. Monitor urine output hourly. 3. Wear gown and mask during procedures.
4. Prepare to assist with an escharotomy. 4. Change central lines once a week.
5. Administer antibiotics as prescribed.
4. The nurse is applying mafenide acetate
(Sulfamylon), a sulfa antibiotic cream, to a 9. The nurse is caring for a client with deep partial-
client’s lower extremity burn. Which assessment thickness and full-thickness burns to the chest
data would require immediate attention by the area. Which assessment data would warrant
nurse? notifying the health-care provider?
1. The client complains of pain when the 1. The client is complaining of severe pain.
medication is administered. 2. The client’s pulse oximeter reading is 95%.
2. The client’s potassium level is 3.9 mEq/L and 3. The client has T 100.4oF, P 100, R 24, and
sodium level is 137 mEq/L. BP 102/60.
3. The client’s ABGs are pH 7.34, Pao2 98, 4. The client’s urinary output is 50 mL in two
Paco2 38, and HCO3 20. (2) hours.
4. The client is able to perform active range-of-
10. The client is admitted with full-thickness and
motion exercises.
partial-thickness burns to more than 30% of the
5. The client is scheduled to have a xenograft to body. The nurse is concerned with the client’s
a left lower-leg burn. The client asks the nurse, nutritional status. Which intervention should the
“What is a xenograft?” Which statement by the nurse implement?
nurse would be the best response? 1. Encourage the client’s family to bring favorite
1. “The doctor will graft skin from your back to foods.
your leg.” 2. Provide a low-fat, low-cholesterol diet for the
2. “The skin from a donor will be used to cover client.
your burn.” 3. Monitor the client’s weight weekly in the
3. “The graft will come from an animal, same clothes.
probably a pig.” 4. Make a referral to the hospital social worker.
4. “I think you should ask your doctor about the
11. The client sustained a hot grease burn to the
graft.”
right hand and calls the emergency department
6. The intensive care unit (ICU) burn nurse is for advice. Which information should the nurse
developing a nursing care plan for a client with provide to the client?
severe full-thickness and deep partial-thickness 1. Apply an ice pack to the right hand.
burns over half the body. Which client problem 2. Place the hand in cool water.
has priority? 3. Be sure to rupture any blister formation.
1. High risk for infection. 4. Go immediately to the doctor’s office.
2. Ineffective coping.
12. The client is being discharged after being in
3. Impaired physical mobility.
the burn unit for six (6) weeks. Which strategies
4. Knowledge deficit.
should the nurse identify to promote the client’s
7. The nurse writes the nursing diagnosis mental health?
“impaired skin integrity related to open 1. Encourage the client to stay at home as much
burn wounds.” Which intervention would be as possible.
appropriate for this nursing diagnosis? 2. Discuss the importance of not relying on the
1. Provide analgesia before pain becomes severe. family for needs.
2. Clean the client’s wounds, body, and hair daily. 3. Tell the client to remember that changes in
3. Screen visitors for respiratory infections. lifestyle take time.
4. Encourage visitors to bring plants and flowers. 4. Instruct the client to discuss feelings only with
the therapist.

12_Colgrove_Ch12.indd 476 01/06/16 5:54 PM


C hapter 12   I ntegumentary D isorders 477

Pressure Ulcers 18. The wound care nurse documented a client’s


pressure ulcers on admission as 3.3 cm × 4 cm
13. The nurse in a long-term care facility is teaching stage II on the coccyx. Which information would
a group of new unlicensed assistive personnel. alert the nurse that the client’s pressure ulcer is
Which information regarding skin care should getting worse?
the nurse emphasize? 1. The skin is not broken and is 2.5 cm ×
1. Keep the skin moist by leaving the skin damp 3.5 cm with erythema that does not blanch.
after the bath. 2. There is a 3.2-cm × 4.1-cm blister that is red
2. Do not rub any lotion into the skin. and drains occasionally.
3. Turn clients who are immobile at least every 3. The skin covering the coccyx is intact but the
two (2) hours. client complains of pain in the area.
4. Only the licensed nursing staff may care for 4. The coccyx wound extends to the
the client’s skin. subcutaneous layer and there is drainage.
14. The nurse is caring for a client who has 19. The nurse and an unlicensed assistive personnel
developed stage IV pressure ulcers on the left (UAP) on a medical floor are caring for clients
trochanter and coccyx. Which collaborative who are elderly and immobile. Which action by
problem has the highest priority? the UAP warrants immediate intervention by
1. Impaired cognition. the nurse?
2. Altered nutrition. 1. The UAP elevates the head of the bed of a
3. Self-care deficit. client who can feed himself with minimal
4. Altered coping. assistance.
2. The UAP asks to take a meal break before
15. The nurse is caring for clients in a long-term turning the clients at the two (2)-hour time
care facility. Which is a modifiable risk factor for limit.
the development of pressure ulcers? 3. The UAP restocks the rooms that need
1. Constant perineal moisture. unsterile gloves before clocking out for the
2. Ability of the clients to reposition shift.
themselves. 4. The UAP mixes Thick-It into the glass
3. Decreased elasticity of the skin. of water for a client who has difficulty
4. Impaired cardiovascular perfusion of the swallowing.
periphery.
20. The nurse is caring for clients on a medical unit.
16. What is the scientific rationale for placing lift After the shift report, which client should the
pads under an immobile client? nurse assess first?
1. The pads will absorb any urinary incontinence 1. The 34-year-old client who is quadriplegic
and contain stool. and cannot move his arms.
2. The pads will prevent the client from being 2. The elderly client diagnosed with a CVA who
diaphoretic. is weak on the right side.
3. The pads will keep the staff from workplace 3. The 78-year-old client with pressure ulcers
injuries such as a pulled muscle. who has a temperature of 102.3o F.
4. The pads will help prevent friction shearing 4. The young adult who is unhappy with the
when repositioning the client. care that was provided last shift.
17. The paraplegic client is being admitted to a 21. The nurse is developing a plan of care for
medical unit from home with a stage IV pressure a client diagnosed with left-sided paralysis
ulcer over the right ischium. Which assessment secondary to a right-sided cerebrovascular
tool should be completed on admission to the accident (stroke). Which should be included in
hospital? the interventions?
1. Complete the Braden Scale. 1. Use a pillow to keep the heels off the bed
2. Monitor the client on a Glasgow Coma when supine.
Scale. 2. Order a low air-loss therapy bed immediately.
3. Assess for Babinski’s sign. 3. Prepare to insert a nasogastric feeding tube.
4. Initiate a Brudzinski flow sheet. 4. Order an occupational therapy consult for
strength training.

12_Colgrove_Ch12.indd 477 01/06/16 5:54 PM


478 M ed -S urg S uccess

22. The client who is debilitated and has developed 26. The female client admitted for an unrelated
multiple pressure ulcers complains to the nurse diagnosis asks the nurse to check her back
during a dressing change that he is “tired of because “it itches all the time in that one spot.”
it all.” Which is the nurse’s best therapeutic When the nurse assesses the client’s back,
response? the nurse notes an irregular-shaped lesion
1. “These wounds can heal if we get enough with some scabbed-over areas surrounding
protein into you.” the lesion. Which action should the nurse
2. “Are you tired of the treatments and needing implement first?
to be cared for?” 1. Notify the HCP to check the lesion on
3. “Why would you say that? We are doing our rounds.
best.” 2. Measure the lesion and note the color.
4. “Have you made out an advance directive to 3. Apply lotion to the lesion.
let the HCP know your wishes?” 4. Instruct the client to make sure the HCP
checks the lesion.
23. The nurse writes the problem “impaired skin
integrity” for a client with stage IV pressure 27. The nurse is caring for clients in an outpatient
ulcers. Which interventions should be included surgery clinic. Which client should be assessed
in the plan of care? Select all that apply. first?
1. Turn the client every three (3) to four (4) 1. The client scheduled for a skin biopsy who is
hours. crying.
2. Ask the dietitian to consult. 2. The client who had surgery three (3) hours
3. Have the client sign a consent for pictures of ago and is sleeping.
the wounds. 3. The client who needs to void prior to
4. Obtain an order for a low air-loss bed. discharge.
5. Elevate the head of the bed at all times. 4. The client who has received discharge
instructions and is ready to go home.
24. The client diagnosed with stage IV infected
pressure ulcers on the coccyx is scheduled for a 28. Which client is at the greatest risk for the
fecal diversion operation. The nurse knows that development of skin cancer?
client teaching has been effective when the client 1. The African American male who lives in the
makes which statement? northeast.
1. “This surgery will create a skin flap to cover 2. The elderly Hispanic female who moved from
my wounds.” Mexico as a child.
2. “This surgery will get all the old black tissue 3. The client who has a family history of basal
out of the wound so it can heal.” cell carcinoma.
3. “The surgery is important to allow oxygen to 4. The client with fair complexion who cannot
get to the tissue for healing to occur.” get a tan.
4. “Stool will come out an opening in my
29. The middle-aged client has had two (2) lesions
abdomen so it won’t get in the sore.”
diagnosed as basal cell carcinoma removed.
Which discharge instruction should the nurse
Skin Cancer include?
1. Teach the client that there is no more risk for
25. The school nurse is preparing to teach a health cancer.
promotion class to high school seniors. Which 2. Refer the client to a prosthesis specialist for
information regarding self-care should be prosthesis.
included in the teaching? 3. Instruct the client how to apply sunscreen to
1. Wear a sunscreen with a protection factor of the area.
10 or less when in the sun. 4. Demonstrate care of the surgical site.
2. Try to stay out of the sun between 0300 and
0500 daily.
3. Perform a thorough skin check monthly.
4. Remember caps and long sleeves do not help
prevent skin cancer.

12_Colgrove_Ch12.indd 478 01/06/16 5:54 PM


C hapter 12   I ntegumentary D isorders 479

30. The nurse is caring for a client diagnosed 34. Which client physiological outcome (goal) is
with squamous cell skin cancer and writes a appropriate for a client diagnosed with skin
psychosocial problem of “fear.” Which nursing cancer who has had surgery to remove the
interventions should be included in the plan lesion?
of care? 1. The client will express feelings of fear.
1. Explain to the client that the fears are 2. The client will ask questions about the
unfounded. diagnosis.
2. Encourage the client to verbalize the feeling 3. The client will state a diminished level of
of being afraid. pain.
3. Have the HCP discuss the client’s fear with 4. The client will demonstrate care of the
the client. operative site.
4. Instruct the client regarding all planned
35. The male client diagnosed with acquired
procedures.
immunodeficiency syndrome (AIDS) states that
31. The nurse and an unlicensed assistive personnel he has developed a purple-brown spot on his
(UAP) are caring for clients in a dermatology calf. Which action should the nurse do first?
clinic. Which task should not be delegated to the 1. Refer the client to an HCP for a biopsy of the
UAP? area.
1. Stock the rooms with the equipment needed. 2. Assess the lesion for size, color, and symmetry.
2. Weigh the clients and position the clients for 3. Discuss end-of-life decisions with the client.
the examination. 4. Report the sexually transmitted illness to the
3. Discuss problems the client has experienced health department.
since the previous visit.
36. The nurse participating in a health fair is
4. Take the biopsy specimens to the laboratory.
discussing malignant melanoma with a group of
32. The client is admitted to the outpatient surgery clients. Which information regarding the use of
center for removal of a malignant melanoma. sunscreen is important to include?
Which assessment data indicate the lesion is a 1. Sunscreen is only needed during the hottest
malignant melanoma? hours of the day.
1. The lesion is asymmetrical and has irregular 2. Toddlers should not have sunscreen applied to
borders. their skin.
2. The lesion has a waxy appearance with pearl- 3. Sunscreen does not help prevent skin cancer.
like borders. 4. The higher the number of the sunscreen, the
3. The lesion has a thickened and scaly more it blocks UV rays.
appearance.
4. The lesion appeared as a thickened area after
an injury. Bacterial Skin Infection
33. The client has had a squamous cell carcinoma 37. The client comes to the emergency department
removed from the lip. Which discharge complaining of pain in the left lower leg
instructions should the nurse provide? following a puncture wound from a nail in
1. Notify the HCP if a nonhealing lesion a board. The left lower leg is reddened with
develops around the mouth. streaks, edematous, and hot to the touch, and
2. Squamous cell carcinoma tumors do not the client has a temperature of 100.8o F. Which
metastasize. condition would the nurse suspect the client is
3. Limit foods to liquid or soft consistency for experiencing?
one (1) month. 1. Cellulitis.
4. Apply heat to the area for 20 minutes every 2. Lyme disease.
four (4) hours. 3. Impetigo.
4. Deep vein thrombosis.

12_Colgrove_Ch12.indd 479 01/06/16 5:54 PM


480 M ed -S urg S uccess

38. The client comes to the clinic complaining 43. The client is diagnosed with acne vulgaris.
of sudden onset of high fever, chills, and a Which psychosocial problem is priority?
headache. The nurse assesses a patchy macular 1. Impaired skin integrity.
rash on the trunk and a circular type of rash that 2. Ineffective grieving.
looks like an insect bite. Which question would 3. Body image disturbance.
be most appropriate for the nurse to ask during 4. Knowledge deficit.
the interview?
44. Which individual would most likely experience
1. “Do you live in an area where animals roam
the skin disorder pseudofolliculitis barbae
the street?”
(shaving bumps)?
2. “Have you been working in your garden
1. A male African American soldier.
lately?”
2. A female Caucasian hairdresser.
3. “Have you been deer hunting in the last
3. A male Asian food server.
week?”
4. A female Hispanic schoolteacher.
4. “Do you use sunscreen when you are outside?”
45. The female client calls the clinic and tells the
39. The school nurse is discussing impetigo with
nurse that she has a really big “boil” in the
the teachers in an elementary school. One of
perineal area that is causing a lot of pain. Which
the teachers asks the nurse, “How can I prevent
intervention should the nurse implement?
getting impetigo?” Which statement would be
1. Schedule an emergency appointment for the
the most appropriate response?
client.
1. “Wash your hands after using the bathroom.”
2. Instruct the client to apply warm, moist
2. “Do not touch any affected areas without
compresses to the area.
gloves.”
3. Determine if someone can squeeze the boil.
3. “Apply a topical antibiotic to your hands.”
4. Explain that this will resolve on its own.
4. “Keep the child with impetigo isolated in the
room.” 46. Which client would most likely be at risk for the
development of a carbuncle?
40. The client is admitted to the medical floor
1. The young male who is just beginning to
diagnosed with cellulitis of the left arm. Which
shave.
assessment data would warrant immediate
2. The female with a fair complexion.
intervention by the nurse?
3. The male who works out in the gym daily.
1. The client has bilaterally weak radial pulses.
4. The female diagnosed with diabetes mellitus.
2. The client is able to move the left fingers.
3. The client has a CRT less than three (3) 47. The female teacher comes to the school
seconds. nurse’s office and shows the nurse a rash on
4. The client is unable to remove the wedding her hands. The nurse tells the teacher she has
ring. probably contracted impetigo from one of the
students. Which intervention should the nurse
41. The nurse writes the client problem of “acute
implement?
pain and itching secondary to bacterial skin
1. Instruct the teacher to go to her HCP today.
lesions.” Which interventions should be included
2. Tell the teacher to wash her hands with soap
in the care plan? Select all that apply.
and water.
1. Keep humidity at less than 20%.
3. Encourage the teacher to rub vitamin E oil on
2. Maintain a cool environment.
the lesions.
3. Use a mild soap for sensitive skin.
4. Explain that the rash will go away in a few
4. Keep lesions covered at all times.
days.
5. Apply skin lotion after bathing.
48. The nurse is teaching a class on how to prevent
42. The nurse observes the unlicensed assistive
Lyme disease. Which intervention should be
personnel (UAP) squeezing the “blackheads” on
included in the discussion?
an elderly client. Which action should the nurse
1. Instruct the clients to wear dark clothes when
implement first?
hunting.
1. Notify the unit manager of witnessing this
2. Use a sunscreen of at least SPF 30 when
activity.
outside.
2. Instruct the assistant to stop this behavior.
3. Avoid dense undergrowth when in a wooded
3. Demonstrate the correct way to care for the
area.
skin.
4. Do not use any type of insect repellant when
4. Complete an incident report regarding the
deer hunting.
action.

12_Colgrove_Ch12.indd 480 01/06/16 5:54 PM


C hapter 12   I ntegumentary D isorders 481

Viral Skin Infection 53. The client is diagnosed with disseminated herpes
zoster secondary to AIDS. Which interventions
49. The nurse is discussing the prevention of herpes should the nurse implement? Select all that
simplex 2. Which intervention should the nurse apply.
discuss with the client? 1. Place the client in contact isolation.
1. Encourage the client to get the chickenpox 2. Administer a corticosteroid IVP.
immunization. 3. Assess the client’s pain on a 1-to-10 scale.
2. Do not engage in oral sex if you have a cold 4. Request that the client not have any visitors.
sore on the mouth. 5. Ensure that only nurses who have had
3. Wear nonsterile gloves when cleaning the chickenpox care for this client.
genital area. 54. Which statement by the client diagnosed with
4. Do not share any type of towel or washcloth chickenpox indicates that the client understands
with another person. the teaching?
50. The client is complaining of burning, stabbing 1. “I should put rubbing alcohol on the lesions
pain that radiates around the left rib cage twice a day.”
area. The nurse cannot find any type of skin 2. “I should not scratch myself if at all possible.
abnormality. Which action should the nurse It might lead to scarring.”
implement? 3. “I can go to work when my lesions have all
1. Transfer the client to the ED for a cardiac disappeared.”
work-up. 4. “I need to take all my antibiotics no matter
2. Inform the client that the nurse can’t see how I feel.”
anything. 55. The client with viral skin lesions is experiencing
3. Administer a nonnarcotic analgesic to the pruritus. Which statement would be an
client. appropriate long-term goal?
4. Ask the client if he or she has ever had 1. The client will refrain from scratching the
chickenpox. skin.
51. The client is diagnosed with herpes simplex 2. The client will maintain intact skin integrity.
2 and prescribed the antiviral medication 3. The client will have relief from itching.
valacyclovir (Valtrex). Which instructions should 4. The client will not develop a secondary
the nurse teach? bacterial infection.
1. This medication will prevent pregnancy and 56. The nurse is admitting an 88-year-old client
treat the virus. diagnosed with a viral skin infection. Which
2. This medication must be tapered when nursing task could the nurse delegate to the
discontinuing the medication. unlicensed assistive personnel?
3. This medication will suppress symptoms but 1. Measure and document the client’s skin
does not cure the disease. lesions.
4. This medication may cause the client’s urine 2. Apply the antihistamine cream to the lesions.
to turn orange. 3. Set up the isolation equipment for the client.
52. The nurse administered morphine 4. Determine if the client has prepared an
sulfate, a narcotic analgesic, IVP 45 minutes advance directive.
ago to a client diagnosed with herpes zoster. 57. The client is diagnosed with a viral infection and
On reassessment, the client complains the the HCP has prescribed an antiviral medication
pain decreased to a “5” on a 1-to-10 scale. Which to be administered by weight. The client weighs
intervention should the nurse implement? 220 pounds and the order reads 10 mg per
1. Turn on soft music and shut the blinds. kilogram per day to be administered in equally
2. Apply warm, moist heat to the lesions. divided doses every six (6) hours. How many
3. Notify the HCP for more pain medication. milligrams will be administered in one dose?
4. Encourage the client to ambulate with _____________
assistance.

12_Colgrove_Ch12.indd 481 01/06/16 5:54 PM


482 M ed -S urg S uccess

58. The 55-year-old client contracted chickenpox 63. The elderly client is admitted from the long-
from his grandchild. The client had to be term care facility diagnosed with congestive
hospitalized because of the seriousness of the heart failure. The client complains of severe
condition. Which complication is the client at itching on both hands and the nurse notes wavy,
risk for developing secondary to chickenpox? brown, threadlike lesions between the client’s
1. Deep vein thrombosis. fingers. Which comorbid condition would the
2. Varicella pneumonia. nurse suspect the client of having based on these
3. Pericarditis. assessment data?
4. Scarring of the skin. 1. Tinea capitis.
2. Herpes simplex 2.
59. The nurse is assessing a young mother who
3. Scabies.
came to the clinic complaining of sores on her
4. Psoriasis.
skin. Which assessment data would support that
the client has chickenpox? 64. The HCP prescribed Kwell lotion to be applied
1. Crops of lesions that have pus and reddened to the entire body. Which instructions should
base. the nurse teach the client concerning this
2. Oval scaling lesions that occur on the legs and medication?
arms. 1. Leave the lotion on for two (2) hours after
3. Severe itching of the scalp with tiny eggs applying it to the body.
visible. 2. Make sure that the skin is completely dry
4. Ringed red lesions on the face, neck, trunk, before applying the lotion.
and extremities. 3. Repeat total body lotion application daily for
at least one (1) week.
60. The long-term care nurse has received the a.m.
4. Put the lotion in the bathwater and soak for at
shift report. Which client should the nurse assess
least 20 minutes.
first?
1. The client who has not had a bowel 65. The nurse in the long-term care facility must
movement today. delegate a nursing task to an unlicensed assistive
2. The client who needs the indwelling catheter personnel. Which nursing task would be most
changed. appropriate to delegate?
3. The client with periorbital skin lesions. 1. Comb the nits out of the client’s hair.
4. The client with a stage I pressure ulcer. 2. Massage the reddened area on the hip.
3. Scrape the burrows to remove the scabies
mite.
Fungal/Parasitic Skin Infection 4. Apply antifungal lotion to the groin area.

61. The school nurse is assessing a teacher who has 66. The client has tinea pedis. Which intervention
pediculosis. Which statement by the teacher should the nurse teach to the client?
makes the nurse suspect that the teacher did not 1. Soak feet in a vinegar-and-water solution.
comply with the instructions that were discussed 2. Wear shoes without any type of socks.
in the classroom with the children? 3. Alternate shoes on a monthly basis.
1. “I used the comb to remove all the nits.” 4. Cut toenails straight across.
2. “I washed my hair with Kwell shampoo.” 67. The client with thick, crusty, yellow toenails is
3. “I removed all the sheets from my bed.” diagnosed with tinea unguium (onychomycosis)
4. “I had to fix my daughter’s hair with my and asks the clinic nurse what happens if he
brush.” can’t afford to take the medication the physician
62. The school nurse is discussing how to prevent prescribed. The nurse’s response will be based on
tinea cruris with the football players. Which which scientific rationale?
intervention should the nurse implement? 1. The toes will become gangrenous and may
1. Instruct the football players to wear tight, have to be amputated.
snug-fitting jock straps. 2. Over-the-counter antifungal creams can be
2. Explain the importance of wearing white substituted for the oral medication.
socks. 3. The toenail plate will separate and the entire
3. Teach the football players to not share brushes toenail may be destroyed.
or combs. 4. Take all the prescribed antibiotics or the
4. Discuss the need to dry the groin area infection may return.
thoroughly after bathing.

12_Colgrove_Ch12.indd 482 01/06/16 5:54 PM


C hapter 12   I ntegumentary D isorders 483

68. There is an outbreak of scabies in a long-term 71. The public health nurse is providing a class
care facility. Which instruction should the on skin disorders in the African American
infection control nurse provide to all client care community. Which information should the nurse
staff concerning the transmission of this parasitic include in the presentation?
infection? 1. People with dark skin suffer the same skin
1. Use only hand-washing foam when caring for conditions as people with light skin.
clients with scabies. 2. African American men are more likely to have
2. Wear gloves when providing hands-on care skin cancer than women.
for a client with scabies. 3. Dark-skinned individuals are less likely to
3. Wash all linen and clothes in cold water and form keloids after any type of surgery.
dry them outside in the sun. 4. Buccal mucosa of dark-skinned individuals is
4. Instruct clients to use plastic eating utensils usually a bluish-tinged color.
for meals.
72. Which skin condition would most likely occur
69. The nurse in a dermatology clinic is taking the in the highlighted areas?
history of a client. Which questions should the 1. Contact dermatitis.
dermatology nurse ask the client? Select all that 2. Herpes zoster.
apply. 3. Seborrheic dermatitis.
1. When did you first notice the skin problem? 4. Scabies.
2. What cosmetics or skin products do you use?
3. Have you experienced any loss of sensation?
4. What is your current and previous
occupation?
5. Do you experience any itching, burning, or
tingling?
70. The nurse is assessing the client diagnosed with
scabies. Which assessment technique would be
most appropriate?
1. Gently palpate the affected area using sterile
gloves.
2. Apply vinegar to the affected area to identify
the scabies.
3. Use a magnifying glass and a penlight to
visualize the skin.
4. Obtain a Doppler to assess the movement of
the mites.

12_Colgrove_Ch12.indd 483 01/06/16 5:54 PM


484 M ed -S urg S uccess

CONCEPTS
The concepts covered in this chapter focus on tissue integrity. Exemplars that are covered are pressure ulcers,
skin cancers, and skin infections. Interrelated concepts of the nursing process and critical thinking are covered
throughout the questions. The concept of critical thinking is presented in the prioritizing or “first” questions.
73. The nurse is staging a pressure ulcer on a 76. The nurse is admitting a male client who states
newly admitted client. Which would the nurse that he has been having a lot of pain in his right
document for this client? chest and side. The nurse observes the client’s
side and chest area and notes the vesicles seen
below. Which should the nurse implement?

1. Stage I.
2. Stage II.
3. Stage III.
4. Stage IV.
74. The nurse has written the concept of impaired
skin integrity for a client diagnosed with diabetes
mellitus type 2 who has an infected wound on
1. Allow the client’s preschool-aged
the left heel. Which interventions should the
grandchildren to visit if they have not had the
nurse implement? Select all that apply.
varicella vaccine.
1. Administer antibiotics via IVPB method.
2. Do not assign a nurse to care for the client
2. Perform wound dressing changes using
if the nurse has never had chickenpox or the
unsterile gloves.
varicella vaccine.
3. Monitor blood glucose levels.
3. Place the client in airborne precautions and
4. Assess the client’s culture daily.
have nuclear medicine decontaminate the
5. Encourage the client to comply with the
trays before sending them to be washed.
recommended diet.
4. Request a prescription for vancomycin, an
75. The nurse is assessing a client’s skin for aminoglycoside antibiotic, so the client’s
melanoma. The example below illustrates which infection will heal faster.
of the A, B, C, D, Es of skin cancer detection?
Select all that apply.

1. Asymmetry.
2. Borders.
3. Color.
4. Diameter.
5. Evolving.

12_Colgrove_Ch12.indd 484 01/06/16 5:54 PM


C hapter 12   I ntegumentary D isorders 485

77. The nurse is presenting an in-service to 79. The nurse admitting a client to a medical-
participants in a local health fair. Which surgical unit notes the lesion below. Which
information regarding the development of skin intervention should the nurse implement first?
cancers should the nurse teach?
1. The fairer the skin, the less the risk of
developing skin cancer.
2. Eating a diet high in fiber helps to minimize
the risk of skin cancer development.
3. Sun exposure at a beach is less dangerous than
at a stadium.
4. The participants should avoid sun exposure in
the afternoon hours.
78. The nurse identifies the concept of impaired
skin integrity for a pediatric client diagnosed
with impetigo on the arms. Which interventions
should the nurse implement?
1. Teach the parents to ensure the child takes all 1. Measure the lesion with a ruler.
the prescribed antibiotics. 2. Document the finding in the client’s medical
2. Give the parents a written excuse so the child record.
can go back to school. 3. Determine if the client has noticed the lesion
3. Encourage the parents to bathe the child in an before.
oatmeal bath for the itching. 4. Notify the health-care provider (HCP) of the
4. Apply topical lidocaine before debriding the lesion.
crusts from the lesions.

80. The clinic nurse is preparing to administer medications. Which safety precautions should the nurse employ
when administering the client’s medications?
1. Keep the head of the bed/chair elevated for 30 minutes after the application.
2. Teach the client not to eat solid foods for 24 hours after the medication is applied.
3. Have the client expose the area to sunlight for 30 minutes after the application.
4. Wear unsterile gloves when applying the 5-fluorouracil to the client’s lip.

Admitting Diagnosis: Basal Cell


Client Name: I. L. Sun F/47 Carcinoma of the Lip
Account Number: 22 22 222 Med Rec #:111 152 Allergies: Caine drugs
Date Medication 2301–0700 0701–1500 1501–2300
Today 5-Fluorouracil Cream apply 1000
topically to lip
Signature of Nurse:
Clinic Nurse RN/DN

12_Colgrove_Ch12.indd 485 01/06/16 5:54 PM


PRACTICE QUESTIONS ANSWERS
AND RATIONALES

Burns 3. 1. After airway, the most urgent need is


­preventing irreversible shock by replacing
1. 1. Sunburn is an example of this depth of burn; fluids and electrolytes.
a superficial partial-thickness burn affects 2. This is important, but it is not priority over
the epidermis and the skin is reddened and fluid volume balance, and this is not a collab-
blanches with pressure. orative intervention because the nurse can do
2. Deep partial-thickness burns are scalds and this independently.
flash burns that injure the epidermis, upper 3. Output must be monitored, but this is an inde-
dermis, and portions of the deeper dermis. pendent intervention.
This causes pain, blistered and mottled red 4. An escharotomy, an incision that releases scar
skin, and edema. tissue that prevents the body from being able
3. Full-thickness burns are caused by flame, to expand, enables chest excursion in circum-
­electric current, or chemical burns and include ferential chest burns. The client has not had
the epidermis, entire dermis, and sometimes time to develop eschar.
subcutaneous tissue and may also involve TEST-TAKING HINT: A collaborative interven-
­connective tissue, muscle, and bone. tion is an intervention that requires an HCP’s
4. First-degree burn is another name for a order or working with another discipline.
­superficial partial-thickness burn. Therefore, options “2” and “3” should be
TEST-TAKING HINT: The adjectives in the stem eliminated immediately.
are the most important words that assist the Content – Medical: Integrated Nursing Process –
test taker when selecting a correct answer. ­Planning: Client Needs – Safe Effective Care Environment,
Content – Medical: Integrated Nursing Process – Management of Care: Cognitive Level – Synthesis:
­Implementation: Client Needs – Safe Effective Care Concept – Skin Integrity.
­Environment, Management of Care: Cognitive Level – 4. 1. The client should be premedicated with an an-
Analysis: Concept – Skin Integrity. algesic because this agent causes severe burn-
2. 1. An 18-gauge catheter with lactated Ringer’s ing pain for up to 20 minutes after application.
infusion should be initiated to maintain a urine 2. Silver nitrate solution is hypotonic and acts as
output of at least 30 mL/hr. a wick for sodium and potassium. Also, these
2. Wounds should be covered with a clean, dry electrolytes are WNL and would not require
sheet. immediate intervention.
3. The client should be transferred with ad- 3. Sulfamylon is a strong carbonic anhydrase
equate pain relief, which requires intravenous inhibitor that may reduce renal buffering
morphine. and can cause metabolic acidosis. These
4. The client’s legs should have pedal pulses ABGs indicate metabolic acidosis and
and be warm to the touch, and the client therefore require immediate intervention.
must be able to move the toes. 4. The client being able to perform range-of-
TEST-TAKING HINT: Note the adjectives
motion exercises does not warrant i­ mmediate
“22-gauge” and “moist.” If the test taker is intervention; this is a very good result.
unsure of the correct answer, then the test TEST-TAKING HINT: “Require immediate at-
taker should determine which system is af- tention” means that the nurse must intervene
fected and see if that will help determine the independently or notify another health-care
right answer. A client’s extremities and a neu- provider. The test taker must know how to
rovascular assessment are similar; therefore, ­interpret ABGs, and, even if the test taker is
the test taker should select option “4.” not familiar with the medication, metabolic
Content – Medical: Integrated Nursing Process – acidosis requires intervention.
­Implementation: Client Needs – Safe Effective Care Content – Medical: Integrated Nursing Process –
­Environment, Management of Care: Cognitive Level – ­Assessment: Client Needs – Safe Effective Care Environment,
Analysis: Concept – Skin Integrity. Management of Care: Cognitive Level – Analysis:
Concept – Medication.

486

12_Colgrove_Ch12.indd 486 01/06/16 5:54 PM


C hapter 12   I ntegumentary D isorders 487

5. 1. This is the explanation for an autograft. TEST-TAKING HINT: The intervention addresses
2. This is the description of a homograft. the etiology of the nursing diagnosis “open
3. A xenograft or heterograft consists of skin burn wounds,” and the goal addresses the
taken from animals, usually porcine. ­response “impaired skin integrity.”
4. This is “passing the buck”; the nurse can Content – Medical: Integrated Nursing Process –
and should answer this question with factual ­Planning: Client Needs – Safe Effective Care Environment,
information. Safety and Infection Control: Cognitive Level – Synthesis:
TEST-TAKING HINT: The test taker should Concept – Skin Integrity.
eliminate options to help determine the cor- 8. 1. Hand washing is the number-one interven-
rect answers. Option “1” can be eliminated tion used to prevent infection, which is
because skin from self would be auto-, not ­priority for the client with a burn.
xeno-. Option “4” should be eliminated be- 2. Aseptic techniques minimize risk of cross-
cause the nurse should answer the question contamination and spread of bacteria.
and not pass the buck. 3. Aseptic techniques minimize risk of cross-
Content – Medical: Integrated Nursing Process – contamination and spread of bacteria.
­Implementation: Client Needs – Physiological Integrity, 4. Central lines are not changed unless they are
Reduction of Risk Potential: Cognitive Level – Application: no longer needed or the client has developed
Concept – Skin Integrity. an infection related to the line.
6. 1. Although this is a potential problem, it is 5. Antibiotics reduce bacteria.
priority because the body’s protective bar- TEST-TAKING HINT: Alternative-type questions
rier, the skin, has been compromised and require the test taker to choose all options
there is an impaired immune response. that apply. Infection is a priority for clients
2. This psychosocial client problem is important, with burns.
but in the ICU the first priority is preventing Content – Medical: Integrated Nursing Process –
infection so wound healing can occur. ­Planning: Client Needs – Safe Effective Care Environment,
3. Burn wound edema, pain, and potential joint Management of Care: Cognitive Level – Synthesis:
contractures can cause mobility deficits, but Concept – Skin Integrity.
the first priority is preventing infection so
9. 1. Severe pain would be expected in a client with
wound healing can occur.
these types of burns; therefore, it would not
4. Teaching is always important, but in the ICU
warrant notifying the health-care provider.
the priority is the physiological integrity of the
2. A pulse oximeter reading greater than 93% is
client.
WNL. Therefore, a 95% reading would not
TEST-TAKING HINT: The adjectives “intensive warrant notifying the health-care provider.
care” mean the client is critically ill; therefore, 3. The client’s vital signs show an elevated tem-
a physiological problem is priority and op- perature, pulse, and respiration, along with a
tions “2” and “4” can be eliminated. Although low blood pressure, but these vital signs would
actual is usually higher priority than potential, not be unusual for a client with severe burns.
in the case of a burn the risk for infection has 4. Fluid and electrolyte balance is the prior-
to be priority. ity for a client with a severe burn. Fluid
Content – Medical: Integrated Nursing Process – resuscitation must be maintained to keep
­Diagnosis: Client Needs – Safe Effective Care Environment, a urine output of 30 mL/hr. Therefore, a
Management of Care: Cognitive Level – Analysis: 25-mL/hr output would warrant immediate
Concept – Skin Integrity. intervention.
7. 1. Addressing pain will not address impaired skin TEST-TAKING HINT: The test taker must select
integrity. an answer that is not expected for the client’s
2. Daily cleaning reduces bacterial disease or condition when being asked which
colonization. data warrant immediate nursing intervention.
3. This intervention would be appropriate for a Content – Medical: Integrated Nursing Process –
“risk for infection” nursing diagnosis. ­Assessment: Client Needs – Safe Effective Care Environment,
4. Plants and flowers in water should be avoided Management of Care: Cognitive Level – Synthesis:
because stagnant water is a source for bacterial Concept – Skin Integrity.
growth.

12_Colgrove_Ch12.indd 487 01/06/16 5:54 PM


488 M ed -S urg S uccess

10 . 1. The client needs sufficient nutrients for chronic problem, such as getting back to
wound healing and increased metabolic normal life as soon as possible and being
requirements, and homemade nutritious independent, but also getting help when
foods are usually better than hospital needed and not expecting too much too
food. This also allows the family to feel soon.
part of the client’s recovery. Content – Medical: Integrated Nursing Process –
2. The client should be provided a high-­calorie, ­Planning: Client Needs – Psychosocial Integrity: Cognitive
high-protein diet along with vitamins. Level – ­Synthesis: Concept – Skin Integrity.
3. The client should be weighed daily, and the
goal is that the client loses no more than 5%
of preburn weight. Pressure Ulcers
4. The nurse would make a referral to a dieti-
tian, not a social worker. 13. 1. The skin should be kept dry. The skin should
be patted completely dry after each bath.
TEST-TAKING HINT: The nurse needs to be
2. Elderly people have decreased moisture in
knowledgeable of different types of diets;
the skin. Applying lotion restores moisture.
this requires memorization.
3. Clients should be turned at least every
Content – Medical: Integrated Nursing Process – one (1) to two (2) hours to prevent pres-
­Implementation: Client Needs – Physiological Integrity,
sure areas on the skin.
­Basic Care and Comfort: Cognitive Level – Analysis:
­Concept – Skin Integrity.
4. All employees in any health-care facility are
responsible for providing care within their
11. 1. Ice should never be applied to a burn because scope of services.
this will worsen the tissue damage by causing TEST-TAKING HINT: Option “2” has an abso-
vasoconstriction. lute “any” in it. The test taker can eliminate
2. Cool water gives immediate and striking this as an answer on this basis. Option “4”
relief from pain and limits local tissue has the absolute “only,” so this option can be
edema and damage. eliminated. Of the remaining options, option
3. Blisters should be maintained intact to pre- “3” can apply to all clients who are immobile.
vent infection.
Content – Medical: Integrated Nursing Process –
4. The client should be told to go to the ED, ­Planning: Client Needs – Safe Effective Care Environment,
not the doctor’s office, for burn care. Management of Care: Cognitive Level – Synthesis:
TEST-TAKING HINT: The test taker should ­Concept – Nursing Roles.
select an answer that directly cares for
14. 1. This can be an independent nursing problem
the client’s body. This eliminates options
or a collaborative one, but it does not relate
“3” (blisters have not formed yet) and “4.”
to pressure ulcers.
Therefore, the test taker has to decide
2. Altered nutrition is a collaborative prob-
­between cool water and ice.
lem involving the nurse, dietitian, and
Content – Medical: Integrated Nursing Process – HCP. The client will need a diet high in
­Implementation: Client Needs – Safe Effective Care
protein and vitamins if there is a chance
Environment, Management of Care: Cognitive Level –
Application: Concept – Skin Integrity.
for the client to heal.
3. Self-care deficit is an independent nursing
12. 1. The client should resume previous activities problem.
gradually and should not stay home; the cli- 4. Altered coping is an independent nursing
ent should go out and begin to live again. problem and does not relate to skin integrity.
2. The client should be honest with self, family, TEST-TAKING HINT: The stem gives two (2)
and friends about needs, hopes, and fears. clues to the answer—“collaborative” and
3. The client needs to know that it will take “pressure ulcers.” Collaborative means that
time to adjust to life after burns and that some other member of the health-care team
returning to work, family role, sexual must be involved, and pressure ulcers are the
­intimacy, and body image will take time. client’s problem. The correct answer must
4. The client should feel free to discuss feelings consider both of these variables.
with family, friends, and the therapist.
Content – Medical: Integrated Nursing Process –
TEST-TAKING HINT: Even if the test taker is ­Diagnosis: Client Needs – Safe Effective Care Environment,
not familiar with the disease process, there Management of Care: Cognitive Level – Analysis:
are certain interventions that go with any Concept – Skin Integrity.

12_Colgrove_Ch12.indd 488 01/06/16 5:54 PM


C hapter 12   I ntegumentary D isorders 489

15. 1. All the skin should be kept free of mois- 3. Assessment for Babinski’s sign would not be
ture. It is within the realm of nursing to attempted in a client who is paralyzed from
provide this service. Clients with constant the waist down. The nerve pathways are not
moisture on the skin are at high risk for working.
impaired skin integrity. 4. Brudzinski’s sign is used to assess for
2. The clients who are able to reposition meningitis.
­themselves would have decreased chances of TEST-TAKING HINT: The test taker must
developing pressure ulcers. This would not be memorize the specific diagnostic tools used
a risk factor. to assess clients.
3. Decreased elasticity occurs with aging, and it Content – Medical: Integrated Nursing Process –
is not modifiable. ­Implementation: Client Needs – Safe Effective Care
4. Impaired cardiovascular perfusion of the skin Environment, Management of Care: Cognitive Level –
in the periphery is not modifiable. Application: Concept – Skin Integrity.
TEST-TAKING HINT: The test taker must read
18. 1. This describes a stage I pressure ulcer, which
the stem carefully. Which situation can the would be an improvement of the client’s
nurse modify with nursing care? The nurse wound.
cannot modify the changes that occur with 2. This is a stage II pressure ulcer and is not a
aging or the sequelae that occur with disease significant change in the wound.
processes. 3. This implies that the skin has healed and is
Content – Medical: Integrated Nursing Process – ­ not at stage I.
Diagnosis: Client Needs – Health Promotion and 4. This is a stage III ulcer and is a worsening
­Maintenance: Cognitive Level – Knowledge: Concept –
of the client’s condition.
Skin Integrity.
TEST-TAKING HINT: The test taker could look
16. 1. The pads will absorb moisture and will at the description in the stem and then at
­protect the bed, but they also will keep the the descriptions in the options. Only one (1)
moisture next to the client’s skin, which option appears to have a more involved skin
­increases the risk of skin breakdown. condition.
2. The pads are made with plastic liners, which Content – Medical: Integrated Nursing Process – ­
tend to contain heat next to the client’s body, Assessment: Client Needs – Safe Effective Care
thereby increasing diaphoresis. ­Environment, Management of Care: Cognitive Level –
3. The pads are a help to lift the client but are Analysis: Concept – Skin Integrity.
not used to prevent workplace injuries. To
prevent workplace injuries, the staff must 19. 1. The head of a client eating in bed should be
practice good body mechanics. elevated. This is a correct action on the part
4. Lifting the client with a “lift” pad rather of the unlicensed assistive personnel.
than pulling the client against the sheets 2. It is important to turn bedfast clients
helps to prevent skin damage from fric- every one (1) to two (2) hours and to en-
tion shearing. courage them, if they are able, to make
minor readjustments to their position at
TEST-TAKING HINT: The stem asks for the
least every 15 minutes. Allowing the cli-
r­ ationale for using “lift” pads. Lifting a client ent to lie in the same position for at least
involves repositioning the client (option “4”). another 30 minutes before being turned
None of the other options mentions any kind should not be allowed.
of repositioning. 3. It is a courtesy to the oncoming staff to leave
Content – Medical: Integrated Nursing Process – the rooms equipped to care for the clients.
­Diagnosis: Client Needs – Physiological Integrity, Basic Care 4. Thick-It frequently is added to the liquids of
and Comfort: Cognitive Level – Analysis: Concept – Skin
a client who has difficulty swallowing.
Integrity.
TEST-TAKING HINT: This is an “except” ques-
17. 1. The Braden and Norton scales are tools tion. All but one (1) option will be actions
that identify clients at risk for skin prob- that are encouraged on the part of unlicensed
lems. This client should be ranked on this assistive personnel. The test taker could jump
scale, and appropriate measures should be to the c­ onclusion that option “1” is correct
initiated for controlling further damage to if the test taker did not pay attention to the
the skin. phrase “warrants immediate intervention.”
2. The Glasgow Coma Scale is a neurological Content – Medical: Integrated Nursing Process – ­
coma scale used to determine the depth of Evaluation: ­Client Needs – Safe Effective Care
neurological damage. ­Environment, Management of Care: Cognitive Level –
Synthesis: Concept – Skin Integrity.

12_Colgrove_Ch12.indd 489 01/06/16 5:54 PM


490 M ed -S urg S uccess

20. 1. The client who is a quadriplegic cannot 22. 1. The question asks for a therapeutic response.
move his or her arms or legs. “Quad” means This response addresses a physiological
four (4), and none of the four (4) extremi- problem and does not address the client’s
ties moves. This is expected for the client’s concerns.
problem. 2. This is restating and clarifying, both
2. Weakness on one side of the body is expected _therapeutic responses.
in clients who have experienced a CVA 3. The client does not owe the nurse an expla-
(stroke). nation for the client’s feelings. “Why” is not
3. The client has a fever indicating an infec- therapeutic.
tion. Clients with pressure ulcers fre- 4. This does not address the client’s feelings.
quently develop infections in the wounds, TEST-TAKING HINT: When the stem asks the
which can lead to further complications. test taker for a therapeutic response, the
4. This is a psychological problem and should correct answer must address the client’s
be addressed but not before assessing the feelings.
infection. Content – Medical: Integrated Nursing Process –
TEST-TAKING HINT: The test taker must ­Implementation: Client Needs – Psychosocial Integrity:
decide if the situation is expected for the Cognitive Level – Application: Concept –
disease process or if it is life threatening. Communication.
Physiological problems come before psycho- 23. 1. The client must be turned every one (1) to
logical problems, according to Maslow. two (2) hours.
Content – Medical: Integrated Nursing Process – 2. Clients with pressure ulcers usually are
Assessment: Client Needs – Safe Effective Care debilitated and have a poor nutritional
­Environment, Management of Care: Cognitive Level –
base for healing. An increase in protein
Analysis: Concept – Skin Integrity.
and vitamins is needed in the diet to pro-
21. 1. Using a pillow to suspend the heels off the mote healing.
bed when a client is supine ­prevents the 3. Clients must sign consents if they are rec-
development of pressure ­ulcers on the ognizable in the pictures. It is standard
heels. practice to document wounds by taking
2. Low air-loss therapy beds are expensive and an instant imaging picture and placing the
normally are provided only for clients who picture in the chart for reference by all con-
have stage III or stage IV impaired skin in- cerned staff. In this instance a consent is not
tegrity. An egg-crate mattress may be applied needed.
to the bed for pressure relief, but many hos- 4. A client with a stage IV pressure ulcer
pitals now have changed all of their regular needs a higher level of pressure reduc-
mattresses for ones that provide the same tion than a normal hospital mattress can
pressure reduction surface as an egg-crate provide.
mattress. 5. The head of the bed can be in any position
3. There is no indication that the client requires of comfort for the client, but the head should
tube feeding. not be elevated at “all” times because of
4. Physical therapists, not occupational thera- the pressure applied to the lower body
pists, work with clients on strength training. region.
Occupational therapists address activities of TEST-TAKING HINT: The test taker must notice
daily living deficits. time frames. Is three (3) to four (4) hours the
TEST-TAKING HINT: The test taker should not correct time frame for turning a client with
read into the question. The stem did not impaired skin integrity? Option “5” can be
mention any swallowing problem, only a mo- eliminated because of the word “all.”
bility problem. The correct answer must re- Content – Medical: Integrated Nursing Process –
late to the information provided in the stem. ­Planning: Client Needs – Physiological Integrity, Basic Care
Content – Medical: Integrated Nursing Process – and Comfort: Cognitive Level – Synthesis: Concept – Skin
­Diagnosis: Client Needs – Safe Effective Care Environment, Integrity.
Management of Care: Cognitive Level – Analysis:
Concept – Skin Integrity.

12_Colgrove_Ch12.indd 490 01/06/16 5:54 PM


C hapter 12   I ntegumentary D isorders 491

24. 1. A skin flap to graft an open wound is not a 26. 1. The nurse should complete an assessment
­fecal diversion. of the lesion prior to notifying the HCP to
2. This statement describes a débridement, not check it.
a fecal diversion. 2. This is part of assessing the lesion and
3. Hyperbaric chambers are used to increase ox- should be completed. The ABCDs of skin
ygenation to nonhealing wounds, but surgery cancer detection include the following:
does not increase oxygenation. (1) Asymmetry—Is the lesion balanced
4. A fecal diversion is changing the nor- on both sides with an even surface? (2)
mal exit of the stool from the body. A Borders—Are the borders rounded and
colostomy is created to keep stool from smooth or notched and indistinct? (3)
contaminating the wounds and causing Color—Is the color a uniform light brown
infection. or is it variegated and darker or reddish
TEST-TAKING HINT: The word fecal means purple? (4) Diameter—A diameter exceed-
“stool.” Only one option mentions anything ing 4 to 6 mm is considered suspicious.
to do with the stool. 3. This may help as a comfort measure, but it is
not the first and most important action.
Content – Medical: Integrated Nursing Process –
­Evaluation: ­Client Needs – Physiological Integrity, 4. Instructing the client to also notify the HCP
­Physiological Adaptation: Cognitive Level – Synthesis: to assess the lesion should be done but does
Concept – Skin Integrity. not have priority.
TEST-TAKING HINT: Assessment is the first
step in the nursing process. The test taker
Skin Cancer should have a systematic decision-making
model when determining a priority action.
25. 1. The lower the SPF number of sunscreen, the
Content – Medical: Integrated Nursing Process –
less protection. A sunscreen of SPF 15 is a ­Implementation: Client Needs – Safe Effective Care
minimum. Environment, Management of Care: Cognitive Level –
2. Clients should avoid sunlight in the after- Synthesis: Concept – Cellular Regulation.
noon, between 3 p.m. and 5 p.m. “Between
0300 and 0500” refers to morning, the middle 27. 1. This client has an unexpected situation
of the night. occurring and should be assessed before
3. The American Cancer Society recom- any stable client.
mends a monthly skin check using mirrors 2. This client’s surgery was three (3) hours ago
to identify any suspicious skin lesion for and the client should be stable and allowed
early detection. to rest.
4. Anything that prevents UV rays from reach- 3. This client can be seen after assessing the
ing the skin helps to prevent skin cancer. Hats ­client in option “1.”
with a full brim are preferred to baseball caps, 4. This client could be escorted to the door by
which leave the ears and back of the neck an unlicensed assistive personnel; another
exposed. nurse has already prepared the client for
discharge.
TEST-TAKING HINT: The test taker must no-
tice time frames. In option “2,” the use of TEST-TAKING HINT: Physiological problems
“between 0300 and 0500” instead of “be- usually have priority over psychological
tween 1500 and 1700” makes this option ones, but none of the other clients has a life-
incorrect. In option “1” the number of the threatening or life-altering situation. The
sunscreen makes this option wrong, and only unexpected situation is the client who is
in option “4” the word “not” is an absolute crying.
word that could eliminate this option. Content – Surgical: Integrated Nursing Process – ­
Content – Medical: Integrated Nursing Process – Plan- Assessment: Client Needs – Safe Effective Care
ning: Client Needs – Health Promotion and Maintenance: ­Environment, Management of Care: Cognitive Level –
Cognitive Level – Synthesis: Concept – Nursing Roles. Analysis: Concept – Assessment.

12_Colgrove_Ch12.indd 491 01/06/16 5:54 PM


492 M ed -S urg S uccess

28. 1. Darker skinned individuals have a lower risk 30. 1. The diagnosis of cancer is concerning for
of developing skin cancer. It is living in the ­clients; this is belittling the client’s concerns
southwestern regions of the United States and gives false reassurance.
where sun exposure is the greatest that in- 2. This is the most commonly written
creases skin cancer risk. therapeutic communication goal. This ad-
2. Hispanic clients have more melanin in the dresses the client’s concerns.
skin than Caucasian clients, and moving 3. The nurse is capable of discussing the client’s
from Mexico would have decreased the UV concerns. Many clients feel more comfortable
exposure. discussing fears with the nurse than with the
3. A family history of malignant melanoma HCP.
increases the risk of developing malignant 4. This should be done but does not directly
melanoma. Basal cell carcinoma is directly ­address the client’s problem.
related to sun exposure and does not have an TEST-TAKING HINT: The test taker must read
increased familial risk. the question carefully to decide what the
4. Clients with very little melanin in the skin question is asking. The answer must address
(fair-skinned) have an increased risk as a the problem of fear.
result of the UV damage to the underly- Content – Medical: Integrated Nursing Process –
ing membranes. Damage to the underly- ­Planning: Client Needs – Psychosocial Integrity: Cognitive
ing membranes never completely reverses Level – ­Synthesis: Concept – Cellular Regulation.
itself; a lifetime of damage causes changes
at the cellular level that can result in the 31. 1. The UAP can restock rooms.
development of cancer. 2. These activities can be performed by a
trained UAP.
TEST-TAKING HINT: If the test taker noticed
3. This is part of assessing the client and
the similarities in the clients in options “1”
cannot be delegated.
and “2”—darker skin and living in an area
4. This is appropriate delegation.
with less sun exposure or moving from an
area with greater sun exposure to an area TEST-TAKING HINT: This is an “except”
with less—the test taker could eliminate ­question. The test taker must be careful to
these two (2) options. Based on skin color, read the words in the stem of the question,
the fair-skinned client would be most at risk. such as “not be delegated.” All of the answers
except one should be activities that the UAP
Content – Surgical: Integrated Nursing Process –
Diagnosis: Client Needs – Physiological Integrity, Reduction can perform.
of Risk Potential: Cognitive Level – Analysis: Concept – Content – Medical: Integrated Nursing Process –
Cellular Regulation. ­Planning: Client Needs – Safe Effective Care Environment,
Management of Care: Cognitive Level – Synthesis:
29. 1. The client should be taught to complete Concept – Nursing Roles.
monthly skin checks to detect any future
lesions. 32. 1. Malignant melanomas are the most deadly
2. Basal cell lesions grow slowly and do not of the skin cancers. Asymmetry, irregu-
metastasize until the lesion has become very lar borders, variegated color, and rapid
large. Prostheses are usually not needed. growth are characteristic of them.
3. The area may need to have an antibiotic oint- 2. A waxy appearance and pearl-like borders are
ment, but sunscreen should not be applied characteristic of basal cell carcinoma.
until after the operative area has healed and 3. A thickened and scaly appearance describes
then only when the client is going to be in squamous cell carcinoma.
the sun. 4. A thickened area after an injury describes a
4. On discharge, all clients should receive in- benign condition called a keloid.
structions in the care of surgical incisions. TEST-TAKING HINT: This is a knowledge-based
TEST-TAKING HINT: If the test taker did not question, but it does indicate the differences
know the specific information regarding this in the types of skin lesions. The test taker
type of cancer surgery, then choosing an an- should concentrate on the differences when
swer that is appropriate for all surgeries is a studying for an examination.
good option. Content – Medical: Integrated Nursing Process –
Content – Surgical: Integrated Nursing Process – ­Assessment: Client Needs – Physiological Integrity, Reduc-
Planning: Client Needs – Physiological Integrity, tion of Risk Potential: Cognitive Level – Analysis:
­Physiological Adaptation: Cognitive Level – Synthesis: Concept – Cellular Regulation.
Concept – Cellular Regulation.

12_Colgrove_Ch12.indd 492 01/06/16 5:54 PM


C hapter 12   I ntegumentary D isorders 493

33. 1. The client should be aware of symptoms TEST-TAKING HINT: Assessment is the first
that indicate development of another step in the nursing process.
skin cancer. Squamous cell carcinoma can Content – Medical: Integrated Nursing Process –
develop in areas of the skin and mucous ­Implementation: Client Needs – Safe Effective Care
membranes. ­Environment, Management of Care: Cognitive Level –
2. Of deaths from squamous cell carcinomas, Analysis: Concept – Cellular Regulation.
75% occur because of metastasis. Even basal 36. 1. Sunscreen should be used whenever the client
cell carcinoma can metastasize but is usually is going to be exposed to UV rays.
so slow growing that surgical excision re- 2. Infants younger than age six (6) months
moves the cancer if the client does not delay should not be placed in the sun or have sun-
treatment. screen applied. This does not include the
3. The surgery was on the lip, not in the mouth. medical uses of a bili-light.
Food can be of a regular consistency. 3. Sunscreen blocks absorption of UV rays,
4. Applying heat to the area would increase which, when allowed to penetrate the skin,
circulation and edema, increasing the client’s cause damage to the layers of the skin. This,
discomfort. in turn, causes cellular changes, which over
TEST-TAKING HINT: Anatomical positioning time can develop into skin cancer.
(“lip”) could eliminate option “3.” An HCP 4. Sunscreen products range in numerical
should be notified for any nonhealing value from 4 to 50; the higher the num-
wound. ber of the sunscreen, the greater the UV
Content – Medical: Integrated Nursing Process – ­ protection.
Planning: Client Needs – Physiological Integrity, TEST-TAKING HINT: Option “1” has the ab-
­Physiological Adaptation: Cognitive Level – Synthesis:
solute word “only” and could be eliminated.
Concept – Cellular Regulation.
“Toddlers” is a key in this option. The test
34. 1. This is a psychological goal, not a taker must decide if it is appropriate for a
­physiological goal. toddler to have sunscreen applied to the skin.
2. This is a knowledge-deficit goal, not a Content – Medical: Integrated Nursing Process – ­
­physiological goal. Planning: Client Needs – Health Promotion and
3. Pain is a physiological problem; this is an ­Maintenance: Cognitive Level – Synthesis: Concept –
appropriate physiological goal. Cellular Regulation.
4. This is a teaching goal, not a physiological
goal.
Bacterial Skin Infection
TEST-TAKING HINT: The test taker must read
the question carefully to determine what the 37. 1. Cellulitis is a bacterial infection of the
stem is asking. All of the goals are appropri- subcutaneous tissue usually associated
ate for the client with skin cancer, but only with a break in the skin, and the nurse
one is a physiological goal. would suspect this with these signs/
Content – Surgical: Integrated Nursing Process – symptoms.
­Planning: Client Needs – Safe Effective Care Environment, 2. Lyme disease is caused by the bite of a tick,
Management of Care: Cognitive Level – Synthesis:
resulting in a bull’s eye–appearing lesion.
Concept – Cellular Regulation.
3. Impetigo is characterized by large, fluid-filled
35. 1. The client may need a biopsy, but the nurse blisters and is very contagious.
should assess the area before deciding to refer 4. Deep vein thrombosis signs/symptoms are
the client. a reddened, warm calf and pain on ambula-
2. This is the first step in deciding how to tion, but this condition is not caused by a nail
help the client. The nurse should assess puncture; it is caused by immobility.
the lesion to determine if it could be TEST-TAKING HINT: If the test taker does not
a Kaposi’s sarcoma tumor or a healing know the answer, the test taker should look
contusion. at the stem and identify that redness and
3. This is important for all clients, even those fever are signs of inflammation; -itis means
without a chronic illness, but the question “inflammation” and ­option “1” would be an
asks what should be done first. This is not the appropriate selection.
priority at this time. Content – Medical: Integrated Nursing Process –
4. AIDS is a reportable disease, but reporting is Assessment: Client Needs – Physiological Integrity,
not the priority intervention. ­Reduction of Risk Potential: Cognitive Level – Analysis:
Concept – Skin Integrity.

12_Colgrove_Ch12.indd 493 01/06/16 5:54 PM


494 M ed -S urg S uccess

38. 1. Animals roaming the streets is not as 3. A capillary refill time (CRT) less than
­important to ask as if the client has been to three (3) seconds is normal and would not
an area where there is a likelihood of being ­require immediate intervention.
exposed. 4. The client being unable to remove the
2. The client would most likely not be exposed wedding ring indicates that the arm is
to deer ticks while working in the garden. edematous and the ring must be removed
3. Deer ticks (Ixodes scapularis) are re- immediately or it may cause impaired cir-
sponsible for the spread of Lyme disease, culation to the left ring finger. This is a
which is what this client is experiencing dangerous situation.
based on the signs/symptoms. TEST-TAKING HINT: When the stem asks the
4. Sunscreen is important, but it will not p ­ rotect test taker to identify data that warrant imme-
the client from any type of insect bites. diate ­intervention, the test taker must select
TEST-TAKING HINT: Option “4” could be the option that is abnormal for the disease
­eliminated as a possible answer if the test process. All the options except “4” are nor-
taker realized that the other three options all mal neurovascular ­assessment data.
deal with some type of insect bite, which is Content – Medical: Integrated Nursing Process –
stated in the stem of the question. ­Assessment: Client Needs – Physiological Integrity,
Content – Medical: Integrated Nursing Process – ­Reduction of Risk Potential: C
­ ognitive Level – Analysis:
­Assessment: Client Needs – Physiological Integrity, Concept – Skin Integrity.
­Reduction of Risk Potential: C
­ ognitive Level – Analysis:
41. 1. Humidity should be kept high, around 60%,
Concept – Skin Integrity.
to prevent the skin from drying.
39. 1. Cleanliness and good hygiene prevent the 2. Coolness deters itching.
spread of impetigo, but washing hands after 3. Mild soaps contain no detergents, dyes, or
going to the bathroom will not prevent the fragrances to cause an increase in
spread. itching.
2. Lesions are extremely contagious and 4. Lesions should be left open to air as much
should not be touched, except when wear- as possible; cloth rubbing the lesions is
ing gloves. irritating.
3. The topical antibiotic ointment is applied 5. Effective hydration of the stratum cor-
after impetigo has developed. It will not help neum prevents compromise of the barrier
prevent impetigo. Usually the client with im- layer of the skin.
petigo is given systemic antibiotics. TEST-TAKING HINT: “Select all that apply”
4. The child is kept at home until he or she has questions require the test taker to make
taken antibiotics for 48 hours; the child is not a determination for each option individu-
isolated in the room. ally. Choosing one answer will not exclude
TEST-TAKING HINT: The test taker needs to another.
think about what the options are saying. Content – Medical: Integrated Nursing Process –
­Option “4” can be eliminated because a child ­Planning: Client Needs – Safe Effective Care E
­ nvironment,
cannot be isolated in a room with many other Management of Care: Cognitive Level – Synthesis:
children in the room. Washing hands after ­Concept – Skin Integrity.
using the bathroom (option “1”) is a general-
42. 1. The nurse is responsible for providing c­ orrect
hygiene tip encouraged for all individuals
information to the UAPs. This interven-
and is not specific to impetigo. The test taker
tion would be appropriate if confronting the
should look at the word “prevent” in the
­person has not altered the behavior.
stem; antibiotic ointment is only prescribed
2. This action could result in the client
for bacterial infections, so option “3” should
­developing a skin infection and should be
be ruled out as an answer.
stopped immediately; therefore, stopping
Content – Medical: Integrated Nursing Process – the behavior is the first intervention.
­Implementation: Client Needs – Safe Effective Care 3. The nurse must be a role model for the UAP
Environment, Safety and Infection Control: Cognitive
and demonstrate the correct way to care for
Level – Application: Concept – Skin Integrity.
the client’s skin.
40. 1. As long as the client has bilateral pulses, there 4. An incident report may need to be completed
is no need for the nurse to intervene. to document the situation, but it is not the
2. As long as the client is able to move the first intervention.
fingers, there is no need for the nurse to
intervene.

12_Colgrove_Ch12.indd 494 01/06/16 5:54 PM


C hapter 12   I ntegumentary D isorders 495

TEST-TAKING HINT: When the question asks 45. 1. A furuncle (boil) is not a life-threatening
the test taker to select the first intervention, emergency, and an appointment is not needed
all four (4) options may be possible actions, for this client.
but only one (1) should be done first. In this 2. Warm, moist compresses increase vascu-
situation, the behavior must be stopped prior larization and hasten the resolution of the
to performing any other interventions. furuncle.
Content – Medical: Integrated Nursing Process – 3. With staphylococcal infections, such as a boil,
­Implementation: Client Needs – Safe Effective Care it is important not to rupture the protective
Environment, Management of Care: Cognitive Level – wall of induration that localizes the infection.
­Application: Concept – Skin Integrity. 4. The client called the clinic to get help, and
this response does not help the client. A nurse
43. 1. Impaired skin integrity is a physiological
can recommend heat, ice, elevation, or some
problem, not a psychosocial problem.
type of independent treatment.
2. The nurse does not know if the client is
grieving over the acne. Do not read more TEST-TAKING HINT: The test taker can al-
into the question than is available. ways rule out options that don’t address the
3. Acne occurs on the face and neck. This is ­client’s needs, such as option “4.” Option “3”
the first impression that people get when can also be ruled out: There are very few
looking at the client; therefore, body “nevers,” but one is never squeeze or pop a
­image disturbance is the priority. boil or pimple.
4. Knowledge deficit is appropriate for this Content – Medical: Integrated Nursing Process –
­client, but it does not have priority over body ­Implementation: Client Needs – Safe Effective Care
image. Acne can be devastating to a client’s ­Environment, Management of Care: Cognitive Level –
self-image. Analysis: Concept – Skin Integrity.

TEST-TAKING HINT: The test taker must 46. 1. This client is not at increased risk for
r­ emember to observe adjectives. “Psycho- ­developing a carbuncle.
social” is the key word to answering this 2. This client is not at increased risk for
question. ­developing a carbuncle.
Content – Medical: Integrated Nursing Process – 3. This client is not at increased risk for devel-
­Diagnosis: Client Needs – Safe Effective Care Environment, oping a carbuncle.
Management of Care: Cognitive Level – Analysis: 4. A carbuncle is an abscess of the skin and
Concept – Skin Integrity. subcutaneous tissue and is an extension
of a furuncle. These are more likely to
44. 1. This disease is a bacterial inflammatory
occur in clients with underlying systemic
reaction that occurs predominantly on
diseases such as diabetes and hemato-
the faces and necks of curly-haired men
logic malignances and in clients who are
as a result of shaving. The sharp ingrow-
immunosuppressed.
ing hairs have a curved root that grows at
a more acute angle and pierces the skin. TEST-TAKING HINT: If the test taker did not
The only treatment is to not shave. Afri- know what a carbuncle was, he or she could
can American men who are in the military look at the options and note that only one
are allowed not to shave if this occurs. has a disease process, which might be the
2. This person will not get this disease. best choice for the correct answer.
3. This person will not get this disease. Content – Medical: Integrated Nursing Process –
4. This person will not get this disease. ­Diagnosis: Client Needs – Safe Effective Care ­Environment,
Management of Care: Cognitive Level – Knowledge:
TEST-TAKING HINT: If the test taker did not
Concept – Skin Integrity.
know the answer to this question, he or she
could examine the root words and endings: 47. 1. Systemic antibiotics are the treatment
Follicle refers to “hair” and -itis is an inflam- of choice for impetigo. Therefore, the
mation. The test taker should think about teacher must go to the HCP to get the
what type of hair each culture has and realize prescription today because impetigo is
that an African American has a different type highly contagious.
of hair than the other three options. 2. Washing hands is always a good practice,
Content – Medical: Integrated Nursing Process – Di- but the teacher needs medication for this
agnosis: Client Needs – Safe Effective Care Environment, ­bacterial infection.
Management of Care: Cognitive Level – Knowledge: Con- 3. Vitamin E oil does help prevent scarring, but
cept – Skin Integrity. it will not treat this bacterial infection.
4. Impetigo is not a rash; it is raised, crusty
lesions.

12_Colgrove_Ch12.indd 495 01/06/16 5:54 PM


496 M ed -S urg S uccess

TEST-TAKING HINT: The test taker must know TEST-TAKING HINT: In some instances, the
that impetigo is a bacterial infection requir- test taker simply must know about the dis-
ing medical treatment. Vitamins do not treat ease process. Sexually transmitted diseases
skin infections. Washing with an antiseptic are common, and the nurse should know
solution may help, but antibiotics are the about how to prevent, discuss, and treat
treatment of choice. them.
Content – Medical: Integrated Nursing Process – Content – Medical: Integrated Nursing Process –
Implementation: Client Needs – Safe Effective Care ­Planning: Client Needs – Safe Effective Care Environment,
­Environment, Safety and Infection Control: Cognitive Safety and Infection Control: Cognitive Level – Synthesis:
Level – Application: Concept – Skin Integrity. Concept – Skin Integrity.

48. 1. To help prevent deer tick bites, which spread 50. 1. The left rib cage area is not considered an
Lyme disease, the individual should wear area for revealing cardiac problems. The cli-
light-colored, tightly woven clothing with ent with a cardiac problem would have chest
long pants and long-sleeved shirts. These pain radiating to the left arm, sweating, and
­allow the person to see the tick better. nausea.
2. Sunscreen will not help prevent the spread of 2. This negates the client’s complaints and does
Lyme disease. not address the client’s concerns. The nurse
3. Staying on paths and avoiding dense un- should always further assess when the client is
dergrowth will help the person keep away in pain.
from tick-infested areas where the per- 3. The nurse should not administer pain medi-
son is more likely to be bitten by a tick cation unless the cause of the pain is known.
and perhaps subsequently develop Lyme The client is complaining of severe pain, and
disease. Tylenol (a nonnarcotic) will not be effective.
4. Insect repellant should be used; there is 4. The client’s description of the pain sug-
­special repellant, diethyltoluamide (DEET), gests shingles. Shingles is caused by
used to help prevent Lyme disease. herpes zoster, which is the same virus as
TEST-TAKING HINT: The test taker should be varicella-zoster, which causes chickenpox.
able to rule out sunscreen as an answer be- This virus is a retrovirus that never dies;
cause the stem says “disease” and sunscreen it becomes dormant and lives in the body
cannot prevent an infectious disease. Not along nerve pathways. During times of
wearing insect repellant (option “4”) would stress, it can erupt as herpes zoster, or
not be appropriate for any teaching except shingles. The pain usually occurs prior to
teaching about infants. the eruption of the vesicles.
Content – Medical: Integrated Nursing Process – ­ TEST-TAKING HINT: If the test taker does not
Planning: Client Needs – Health Promotion and know the answer, then the test taker should
­Maintenance: Cognitive Level – Synthesis: Concept – apply the nursing process. Option “4” is the
Nursing Roles. only option involving assessment, which is
the first part of the nursing process.
Content – Medical: Integrated Nursing Process –
Viral Skin Infection ­Implementation: Client Needs – Physiological Integrity,
Reduction of Risk Potential: Cognitive Level – Analysis:
49. 1. The virus that causes chickenpox is the Concept – Skin Integrity.
­varicella-zoster virus (human herpesvirus 3),
not the herpes simplex virus. 51. 1. Valtrex is not a birth control medication.
2. Herpes simplex 1 and 2 are caused by 2. Valtrex is usually prescribed for a set amount
the same virus. Herpes simplex 1 refers of time and does not need to be tapered.
to orolabial lesions and herpes simplex 3. Valtrex is an antiviral medication that sup-
2 ­refers to genital lesions, which can be presses the virus replication, but herpes is
transferred from one area to the other. a retrovirus, which means it never dies as
3. The question is asking which intervention the long as the host body is alive.
nurse should teach the client, and there is no 4. This medication does not cause the urine to
reason for the client to have to wear nonster- turn orange.
ile gloves for self-care. That is appropriate for TEST-TAKING HINT: Medications are fre-
the nursing personnel. quently advertised to the public; therefore,
4. Herpes simplex is not transmitted via towels the nurse must know about medications that
or washcloths. laypeople may be asking about.

12_Colgrove_Ch12.indd 496 01/06/16 5:54 PM


C hapter 12   I ntegumentary D isorders 497

Content – Medical: Integrated Nursing Process – ­ Content – Medical: Integrated Nursing Process –
Planning: Client Needs – Physiological Integrity, ­Implementation: Client Needs – Safe Effective Care
­Pharmacological and ­Parenteral Therapies: Cognitive ­Environment, Safety and Infection Control: Cognitive
Level – Synthesis: Concept – Medication. Level – Analysis: Concept – Skin Integrity.

52. 1. Diversionary techniques, including music 54. 1. Rubbing alcohol will cause tremendous pain
and television, are often used in conjunc- and will not help the lesions disappear.
tion with medication to manage pain. 2. The lesions are very irritating and the cli-
Shutting the blinds will help provide a ent will want to scratch them. Clients with
calm, quiet atmosphere. chickenpox should use calamine lotion,
2. Warm, moist heat will exacerbate the pain soak in oatmeal baths, and apply Benadryl
from the lesions; cool compresses would help. topical cream or take oral Benadryl.
3. The client was medicated 45 minutes ago and 3. The lesions are considered contagious until
would not be able to have more medication; they are dry and crusted over, usually in five
therefore, there is no reason to notify the (5) to seven (7) days. The client does not have
HCP. to wait until they have disappeared; this may
4. Ambulating will not help decrease the client’s take up to two (2) or three (3) weeks.
pain and could aggravate it. In addition, the 4. Chickenpox is a virus, and antibiotics are
client has been given morphine, which causes ­effective only with bacterial infections;
drowsiness and could lead to a safety issue. ­antibiotics are not prescribed.
TEST-TAKING HINT: The test taker could rule TEST-TAKING HINT: The test taker may ­realize
out option “4” with no knowledge of the that options with absolutes such as “all”
disease process because the nurse does not should not be selected as the right answer,
ambulate clients on morphine. The nurse but the test taker should notice that the “all”
often uses diversionary interventions when in option “2” is not used in the same manner
addressing pain issues. as to mean “always.”
Content – Medical: Integrated Nursing Process – Content – Medical: Integrated Nursing Process –
­Implementation: Client Needs – Physiological Integrity, ­Evaluation: ­Client Needs – Physiological Integrity,
Reduction of Risk Potential: Cognitive Level – Application: ­Physiological Adaptation: Cognitive Level – Synthesis:
Concept – Comfort. Concept – Skin Integrity.

53. 1. The zoster lesions are contagious, so the 55. 1. This would be a short-term goal, but it is
client should be in contact isolation. ­unrealistic to expect a client not to scratch
2. Corticosteroids decrease the inflamma- when experiencing severe itching.
tion, which helps with the healing process. 2. The client currently has lesions; therefore,
3. Assessment is always an appropriate the skin integrity is already compromised.
intervention. The key word is “maintain.”
4. The client can have visitors as long as they do 3. Relief of itching is a short-term goal for this
not have an infection that the client could get client. For a long-term goal, the nurse needs
and the visitors comply with isolation proto- to recognize the potential complications.
col. Only visitors who have had chickenpox 4. A major complication of pruritus (itching) is
should be allowed to visit. the development of a bacterial skin infection,
5. Herpes zoster is the same virus that which is secondary to the client scratching
causes chickenpox. It was thought for and allowing bacteria from the dirty hands
years that there were two separate viruses. or nails to enter compromised tissue.
Research has proven that the varicella TEST-TAKING HINT: The test taker must recog-
virus and zoster are the same; therefore, nize what symptoms the client is experiencing.
nurses who have not had chickenpox Lesions are open wounds; therefore, option
should not care for this client. “2” could be eliminated even if the test taker
TEST-TAKING HINT: This is an alternate-type did not know what the word “pruritus” means.
question where the test taker must select all Furthermore, the test taker must look at ad-
the interventions that are pertinent. If the jectives; the question asks for long-term goals
test taker is aware that corticosteroid medi- and the test taker should realize immediate
cations decrease inflammation, he or she behavior change (refrain from scratching) and
should select option “2” as a correct answer. pain relief are short-term goals.
Assessment (option “3”) is always an appro- Content – Medical: Integrated Nursing Process – ­
priate selection if the assessment data apply Planning: Client Needs – Physiological Integrity,
to the situation. ­Physiological Adaptation: Cognitive Level – Synthesis:
Concept – Skin Integrity.

12_Colgrove_Ch12.indd 497 01/06/16 5:54 PM


498 M ed -S urg S uccess

56. 1. The nurse cannot delegate assessment, which TEST-TAKING HINT: The question asks for a
is what measuring the lesions is doing, and complication; therefore, option “4” could be
the nurse must document the assessment. ruled out because it is expected; if the test
2. This is a medication and the nurse cannot taker knows that chickenpox is caused by var-
delegate medication administration. icella-zoster, then the most obvious answer is
3. The nurse can delegate the setup of option “2.”
equipment to the assistant. Content – Medical: Integrated Nursing Process –
4. Determining if a client has an advance direc- ­Diagnosis: Client Needs – Physiological Integrity, Reduction
tive should not be delegated to the assistant of Risk Potential: Cognitive Level – Comprehension:
because, if the client has questions, the nurse Concept – Skin Integrity.
must provide factual information.
59. 1. Chickenpox starts out with a macular rash.
TEST-TAKING HINT: The nurse must know The lesions appear in crops first on the
which nursing tasks can be delegated; assess- trunk and scalp, and then lesions move to
ment, teaching, evaluating, and medication the extremities. Then, they change to tear-
administration cannot be delegated to unli- drop vesicles with an ­erythematous base
censed assistive personnel. and become pustular. Finally, they dry.
Content – Medical: Integrated Nursing Process – 2. Oval scaling indicates fungal infections.
­Planning: Client Needs – Safe Effective Care E
­ nvironment, 3. Severe itching of the scalp with tiny eggs vis-
Management of Care: Cognitive Level – Synthesis: ible supports the diagnosis of lice.
­Concept – Skin Integrity. 4. Ringed red lesions support the diagnosis of
57. 250 mg per dose. ringworm.
First, determine the client’s weight in kilo- TEST-TAKING HINT: This is primarily a knowl-
grams; there are 2.2 pounds per kilogram, so: edge-based question. The test taker either
220 pounds ÷ 2.2 = 100 kilograms knows the signs and symptoms of different
Then, determine the total dosage to be given skin disorders or does not. One way to help
per day based on the HCP order of 10 mg identify the answer is to try to determine
per kilogram per day: what condition each option describes. For
100 × 10 = 1,000 milligrams per day ­example, “tiny eggs” should make the test
Then, determine how many doses are given in taker think of a parasite (lice).
a day if a dose is to be given every six (6) hours: Content – Medical: Integrated Nursing Process –
24 ÷ 6 = 4 doses ­Assessment: Client Needs – Physiological Integrity,
Finally, determine how many milligrams per ­Reduction of Risk Potential: Cognitive Level – Analysis:
each dose to administer 1,000 mg in 24 hours: Concept – Skin Integrity.
1,000 ÷ 4 = 250 mg per dose
60. 1. This client would not be priority; elderly
TEST-TAKING HINT: Math problems frequently ­clients frequently think they need a daily
require multiple steps. The test taker should bowel movement.
replace each number as the conversion is 2. A catheter change would not be priority.
made. The test taker must know conversion 3. Periorbital lesions may extend into the
factors and how to use the drop-down calcu- ­client’s eyes, which is an ophthalmic
lator on the computer. ­emergency, especially if it is herpes zoster.
Content – Medical: Integrated Nursing Process – 4. A stage I pressure ulcer requires changing the
­Implementation: Client Needs – Physiological Integrity, client’s position, which can be delegated.
Pharmacological and Parenteral Therapies: Cognitive
TEST-TAKING HINT: If the test taker knows
Level – Application: Concept – Medication.
medical terminology, then “periorbital”
58. 1. This is a complication of immobility, not (around the eye) would be priority.
­specifically chickenpox. Content – Medical: Integrated Nursing Process – ­
2. Varicella-zoster (human herpesvirus 3) is Assessment: Client Needs – Safe Effective Care
the causative agent for chickenpox, and ­Environment, Management of Care: Cognitive Level –
pneumonia is a potential complication in Analysis: Concept – Assessment.
adults.
3. Pericarditis (inflammation of the sac sur-
rounding the heart) is not a complication of
chickenpox.
4. Scarring of the skin is an expected sequela of
chickenpox, but it is not a complication.

12_Colgrove_Ch12.indd 498 01/06/16 5:54 PM


C hapter 12   I ntegumentary D isorders 499

Fungal/Parasitic Skin Infection 4. Psoriasis is a chronic, noninfectious inflam-


matory disorder of the skin, which results in
61. 1. This statement indicates the teacher under- scales.
stands the teaching about the spread of lice TEST-TAKING HINT: The nurse should be
(pediculosis). ­familiar with medical terminology, which
2. Kwell shampoo or Rid is prescribed when lice can often help rule out incorrect options.
are found in the hair. Capitus, for example, pertains to the head or
3. All linens and pertinent clothing should be scalp; therefore, option “1” could be elimi-
washed in hot water to help destroy the eggs. nated as a possible answer.
4. Sharing brushes is one of the main ways Content – Medical: Integrated Nursing Process –
that lice are spread. Therefore, this state- ­Diagnosis: Client Needs – Safe Effective Care Environment,
ment indicates the teacher did not comply Management of Care: Cognitive Level – Analysis:
with the instructions. Concept – Skin Integrity.
TEST-TAKING HINT: This is an “except” ques- 64. 1. The lotion should be left on for at least
tion. Therefore, three (3) of the options will 12 to 24 hours to be effective.
be a correct action for the problem and only 2. The application of Kwell lotion on wet
one (1) option will be an incorrect action. skin can lead to increased absorption of
The test taker should not jump to conclu- the lotion, which increases the possibility
sions after reading the first option. Always of central nervous system abnormalities,
read all the options thoroughly. including seizures.
Content – Medical: Integrated Nursing Process – 3. One application may cure the infestation of
Evaluation: Client Needs – Safe Effective Care lice or scabies, but another application may
­Environment, Management of Care: Cognitive Level – be needed in one (1) week; daily administra-
Synthesis: Concept – Skin Integrity. tion is contraindicated.
62. 1. To prevent tinea cruris (jock itch), the 4. This medication must be applied to dry skin
­football players should avoid wearing nylon to prevent possible fatal complications.
underwear, tight-fitting clothing, and wet TEST-TAKING HINT: The test taker should
bathing suits. always be aware of time frames; notice “two
2. This would be appropriate advice for tinea (2) hours,” “20 minutes,” and “daily.” These
pedis (athlete’s foot). are all terms that may help the test taker to
3. This would be appropriate advice to help choose the correct answer.
­prevent lice. Content – Medical: Integrated Nursing Process – ­
4. Tinea cruris (jock itch) results from a Planning: Client Needs – Physiological Integrity,
­fungal infection in warm, moist areas of ­Pharmacological and P­arenteral Therapies: Cognitive
the body. When such an infection occurs Level – Synthesis: Concept – Medication.
in the groin area, it is called tinea cruris.
65. 1. The assistant could use a fine-toothed
TEST-TAKING HINT: All the options discuss comb dipped in vinegar to remove any nits
prevention of something; therefore, the test in the hair of the client with lice.
taker must know about disease processes. 2. Reddened areas should not be massaged; that
Test-taking hints do not substitute for study- will lead to further damage.
ing and understanding the material. Remem- 3. Scraping the burrows is a diagnostic test and
ber that perfect test scores are few and far cannot be delegated to an assistant.
between. 4. Assistants cannot administer medication.
Content – Medical: Integrated Nursing Process – TEST-TAKING HINT: The test taker must ­apply
­Implementation: Client Needs – Safe Effective Care
universal delegation rules. Nurses cannot
Environment, Safety and Infection Control: Cognitive
Level – Application: ­Concept – Skin Integrity.
delegate medication administration or diag-
nostic tests. Some states allow medication
63. 1. Tinea capitis is ringworm of the scalp, which aides to administer routine medications
does not support this client’s signs/symptoms. to clients in long-term care facilities, but
2. A herpes simplex 2 lesion is in the genital the stem does not identify this employee
area. as a medication aide. Do not read into the
3. Scabies is an infestation of the skin by the question.
itch mite (Sarcoptes scabiei). The female Content – Nursing Management: Integrated Nursing
burrows into the superficial layer of skin, Process – Planning: Client Needs – Safe Effective Care
and burrows are found between the fin- Environment, ­Management of Care: Cognitive Level –
gers and on the wrist. Synthesis: Concept – Nursing Roles.

12_Colgrove_Ch12.indd 499 01/06/16 5:54 PM


500 M ed -S urg S uccess

66. 1. Soaking the feet will help remove the 68. 1. Foam or soap and water can be used to clean
crust, scales, and debris to reduce the in- the hands.
flammation in a client diagnosed with ath- 2. Because of the close living quarters,
lete’s foot. Vinegar is mildly acidic, which ­clients in long-term care facilities are at
helps remove crusts. a high risk for developing scabies. Clients
2. White cotton socks should be recommended may have poor hygiene as a result of lim-
to help prevent athlete’s foot. Colored socks ited physical ability, and the nursing staff
have dyes that irritate the skin, and cotton may transmit the parasite. Therefore, the
socks absorb moisture. nursing staff should wear gloves to provide
3. Several pairs of shoes should be alternated a barrier to the mites.
so that shoes can be completely dry before 3. Linens and clothing should be washed in hot
­wearing again. water and dried in a hot dryer cycle because
4. Cutting toenails straight across is correct, but the mites can survive up to 36 hours in linens.
it will not prevent or treat athlete’s foot. 4. Plastic eating utensils will not help prevent
TEST-TAKING HINT: There are basic interven- the spread of scabies.
tions that are taught about foot care in gen- TEST-TAKING HINT: If the test taker has no
eral, and trimming toenails is one of them. idea what the correct answer is, then the test
This could apply to many foot conditions. taker should eliminate options “1” and “3”
Wearing socks is always advisable because because of the words “only” and “all.” There
feet perspire. Not very many interventions are very few absolutes in the health-care
are done on a monthly b ­ asis—usually daily or profession.
weekly—so option “3” can be eliminated. Content – Medical: Integrated Nursing Process –
Content – Medical: Integrated Nursing Process – ­ ­Planning: Client Needs – Safe Effective Care Environment,
Planning: Client Needs – Physiological Integrity, Safety and Infection Control: Cognitive Level – Synthesis:
­Physiological Adaptation: Cognitive Level – Synthesis: Concept – Skin Integrity.
Concept – Skin Integrity.
69. 1. Dermatology is the study of the skin.
67. 1. This sequela may occur with a client diag- Therefore, asking about skin problems is
nosed with diabetes who develops a foot appropriate.
ulcer. 2. The nurse must differentiate between
2. Oral antifungal agents must be taken for a dermatitis, which could result from a
12 weeks because the fungal infection is cosmetic or other skin product, and a skin
­underneath the nail; topical medications infection.
would not reach the infection. 3. The skin is responsible for sensation, so
3. This is a condition of ringworm of the any loss would be significant.
toenail. The nurse must tell the client that 4. Many occupations, including those
the toenail will fall off if the client does ­involved in working with chemicals,
not take the medication, and it might fall predispose a client to abnormal skin
off anyway. conditions.
4. This is not a bacterial infection; therefore, 5. These are hallmark signs of skin
oral antibiotics will not be administered. It is abnormality.
a fungal infection and antifungal medication TEST-TAKING HINT: In “Select all that apply”
will be administered. questions, each option stands or falls on
TEST-TAKING HINT: Gangrene is usually a its own. The test taker should not try to
­circulation problem, not a toenail problem; second-guess the item writer thinking that
therefore, option “1” could be eliminated there is no way that all five (5) options will be
as a possible answer. Option “2” should be correct.
eliminated because nurses cannot prescribe Content – Medical: Integrated Nursing Process –
or change physicians’ orders. Of the two ­Assessment: Client Needs – Physiological Integrity,
remaining options, only “3” has the word ­Reduction of Risk Potential: Cognitive Level – Analysis:
toenail in it; therefore, the test taker should Concept – Skin Integrity.
select option “3” as the correct answer.
Content – Medical: Integrated Nursing Process –
­Implementation: Client Needs – Safe Effective Care
­Environment, Management of Care: Cognitive Level –
Analysis: Concept – Skin Integrity.

12_Colgrove_Ch12.indd 500 01/06/16 5:54 PM


C hapter 12   I ntegumentary D isorders 501

70. 1. This technique will not help diagnose scabies. should notice that options “2” and “3” have
2. Vinegar will not help visualize and identify “more likely” and “less likely,” which may
scabies. encourage the test taker to rule out both
3. A magnifying glass and a penlight are held ­answers and select option “1.”
at an oblique angle to the skin while a Content – Medical: Integrated Nursing Process – ­
search is made for small raised burrows, Planning: Client Needs – Health Promotion and
which indicate scabies. ­Maintenance: Cognitive Level – Synthesis: Concept –
4. A Doppler is used to obtain faint pulses, but it Nursing Roles.
will not help find a mite, which causes scabies.
72. 1. Contact dermatitis usually occurs where
TEST-TAKING HINT: The nurse must know cosmetics or perfume are applied or specific
about diagnostic equipment, and a Doppler clothing items touch the skin.
is commonly used in clients who have a non- 2. Herpes zoster in clients who are not im-
palpable pulse. Therefore, option “4” could munocompromised occurs along nerve roots
be eliminated. around the rib cage and back.
Content – Medical: Integrated Nursing Process – 3. Seborrheic dermatitis occurs in the midline
­Implementation: Client Needs – Safe Effective Care front and back of the body.
­Environment, Management of Care: Cognitive Level – 4. Scabies occurs around the waist, around
Analysis: Concept – Skin Integrity. the wrist, between the fingers, and in the
71. 1. This is correct information. axilla area.
2. Dark-skinned people are less likely to have TEST-TAKING HINT: There may be questions
skin cancer. that require the test taker to identify body
3. Dark-skinned people are more likely to have parts when selecting the correct answer. The
keloids (hypertrophied scar tissue) after test taker must know about skin conditions
surgery. to be able to answer the question.
4. Bluish-tinged buccal mucosa in anyone Content – Medical: Integrated Nursing Process –
­indicates decreased oxygenation. Diagnosis: Client Needs – Health Promotion and
TEST-TAKING HINT: The nurse should know ­Maintenance: Cognitive Level – Comprehension:
that any blue color on the body is abnormal, Concept – Skin Integrity.
thus ruling out option “4.” The test taker

CONCEPTS

73. 1. Stage I pressures are characterized by intact minimal disease and stage IV will show
skin with unblanchable redness. an ­increase in the disease symptoms. This
2. Stage II pressure ulcers are characterized by wound has depth and exposed muscle tissue.
partial-thickness loss of the dermis as a shal- Content – Medical: Integrated Nursing Process –
low open ulcer with a red-pink wound bed ­Assessment: Client Needs – Physiological Integrity,
without slough or as an intact unruptured Reduction of Risk Potential: Concept – Skin Integrity.
serum-filled blister.
74. 1. The wound is infected so antibiotics
3. Stage III pressure ulcers are characterized
should be administered. In the hospital,
by full-thickness tissue loss, subcutaneous fat
most antibiotics are administered via
may be visible but bone, tendon, and muscle
IVPB.
are not exposed. Slough may be present but
2. Dressing changes should be performed with
does not obscure the depth of tissue loss.
sterile, not unsterile, gloves.
4. Stage IV pressure ulcers are characterized
3. High blood glucose levels impair wound
by full-thickness tissue loss with exposed
healing and encourage bacterial growth.
bone, tendon, or muscle. Slough or es-
4. The client will not have daily cultures, and,
char may be present on some parts of the
after an antibiotic is initiated, the culture will
wound bed; often includes undermining
be skewed.
and tunnelling.
5. Controlling the glucose levels will assist
TEST-TAKING HINT: The test taker should the client to promote wound healing and
concentrate on the stages, I through IV. disease stabilization.
­Usually in health care, stage I will be

12_Colgrove_Ch12.indd 501 01/06/16 5:54 PM


502 M ed -S urg S uccess

TEST-TAKING HINT: When answering a “Select 77. 1. The fairer the skin, the greater the risk of
all that apply” question, the test taker must ­developing skin cancer.
look at each option independently of the 2. Dietary fiber does not impact the develop-
other options. Each option becomes a true/ ment of skin cancer.
false question. 3. Sun exposure is not less dangerous at a
Content – Medical: Integrated Nursing Process – beach—the exposure is basically the same. It
­Implementation: Client Needs – Physiological Integrity, depends on the length of time and amount of
Reduction of Risk Potential: Concept – Skin Integrity. exposure to the UV rays.
4. The afternoon hours between 3:00 pm
75. 1. Asymmetry means one-half of the area is
and 5:00 pm are the most dangerous times
not identical to the other half.
because of the heat and intensity of the
2. Borders describe irregular edges to the
rays. For people living in the Sun Belt, the
lesion.
time may be from 2:00 to 6:00 pm.
3. Color means that various shades or col-
ors are in the same lesion, including tan, TEST-TAKING HINT: The test taker must be
brown, black, or purple. aware of basic principles such as when the
4. Diameter refers to a lesion that is greater sun is hottest, resulting in greatest skin
than 6 mm (the diameter of a pencil ­exposure, and that people at a beach usually
eraser). This lesion from top to bottom are exposing more skin to the damaging rays
would be more than 6 mm and from side of the sun.
to side would be at least 6 mm. Content – Medical: Integrated Nursing Process –
5. Evolving is not correct because there is no ­Planning: Client Needs – Physiological Integrity, Reduction
way to see an evolving lesion from one static of Risk Potential: Concept – Skin Integrity.
picture. 78. 1. Impetigo is a group A beta hemolytic
TEST-TAKING HINT: When looking at a pic- streptococci or staphylococci infection
ture or diagram, the test taker must care- treated with antibiotics; the parents
fully ­observe exactly what is being pictured. should make sure that all medication is
“Evolving” cannot be shown with one taken as directed.
picture. 2. The child has a contagious infection and is
Content – Medical: Integrated Nursing Process – not allowed to return to school until the in-
­Assessment: Client Needs – Physiological Integrity, fection has resolved itself.
­Reduction of Risk Potential: Concept – Skin Integrity. 3. Oatmeal baths are useful to treat the pruritus
of poison ivy or oak but are not used for bac-
76. 1. Shingles, herpes zoster, is the same virus as
terial infections.
herpes varicella, the chickenpox virus. Chil-
4. The nurse would not débride the lesions.
dren who have not been vaccinated are at risk
Sometimes soap and water are used to soften
for contracting chickenpox from the client.
the crusts from the lesions.
2. Nurses who have no immunity from the
varicella (zoster) virus should not care for TEST-TAKING HINT: Standard instructions to
the client. all clients regarding antibiotics are to finish
3. The virus can be spread by droplets but do them all to treat the infection and prevent a
not require a negative airflow room ­(airborne regrowth of resistant bacteria from partially
precautions) and no nuclear wastes are in- treated infection.
volved so decontamination is not needed. Content – Medical: Integrated Nursing Process –
4. Acyclovir or another antiviral medication ­Implementation: Client Needs – Pharmacological Integrity,
may hasten recovery but an antibiotic is not ­Reduction of Risk Potential: Concept – Skin Integrity.
needed. 79. 1. The nurse should completely assess the
TEST-TAKING HINT: The test taker should be lesion before proceeding to document the
able to recognize common skin ailments and finding or notify the HCP.
make appropriate interventions based on the 2. Documenting the lesion would follow a full
assessment of the problem. assessment of the lesion.
Content – Medical: Integrated Nursing Process – 3. The lesion is present at this time; assessing it
­Implementation: Client Needs – Physiological Integrity, is first.
Reduction of Risk Potential: Concept – Skin Integrity. 4. The nurse should completely assess the ­lesion
before notifying the HCP.

12_Colgrove_Ch12.indd 502 01/06/16 5:54 PM


C hapter 12   I ntegumentary D isorders 503

TEST-TAKING HINT: When answering a “first” TEST-TAKING HINT: The test taker should
question the test taker should put the remember basic medication administration
­options in the order of how a nurse would guidelines. Unsterile gloves are required
perform the tasks. First, the nurse would whenever a nurse is applying topical medi-
­assess the lesion. Second (possible), deter- cations to prevent personal exposure to the
mine the length of time the client has been medication.
aware of its presence. Third or fourth is doc- Content – Medical: Integrated Nursing Process –
umentation and notifying the HCP. ­Assessment: Client Needs – Pharmacological Integrity,
Content – Medical: Integrated Nursing Process – Reduction of Risk ­Potential: Concept – Skin Integrity.
­Implementation: Client Needs – Physiological Integrity,
Reduction of Risk Potential: Concept – Skin Integrity.

80. 1. The cream should not be affected by the


­position of the head.
2. Once the cream has absorbed into the skin
the client can eat solid or liquid foods.
3. Exposure to sunlight is the most common
reason for developing a skin cancer; sun
­exposure is not recommended. Clients receiv-
ing 5-fluorouracil orally or intravenously can
become photosensitive.
4. The nurse should protect herself/­himself
from coming into contact with the
medication.

12_Colgrove_Ch12.indd 503 01/06/16 5:54 PM


504 M ed -S urg S uccess

INTEGUMENTARY DISORDERS
COMPREHENSIVE EXAMINATION
1. The nurse is caring for a client with complaints 5. The nurse is planning the care of a client
of a rash and itching on the face for one (1) diagnosed with psoriasis. Which psychosocial
week. Which intervention should the nurse problem should be included in the plan?
implement first? 1. Alteration in comfort.
1. Check for the presence of hirsutism on the 2. Altered body image.
face. 3. Anxiety.
2. Use the Wood’s light to visualize the rash 4. Altered family processes.
under the black light.
6. The elderly client diagnosed with poison ivy
3. Determine what OTC medications the client
is prescribed a Solu-Medrol (a steroid) dose pack.
has used on the rash.
Which intervention should the nurse teach the
4. Ask the client to describe when the rash first
client?
appeared.
1. Tell the client to return to the office in one
2. The nurse is assessing the client diagnosed (1) week for blood levels.
with psoriasis. Which data would support that 2. Instruct the client to take the medication
diagnosis? exactly as prescribed.
1. Appearance of red, elevated plaques with 3. Explain the medication should be taken on an
silvery white scales. empty stomach.
2. A burning, prickling row of vesicles located 4. Teach to stop the medication immediately if
along the torso. side effects occur.
3. Raised, flesh-colored papules with a rough
7. The nurse is teaching clients at a community
surface area.
center about skin diseases. Which information
4. An overgrowth of tissue with an excessive
about pruritus should the nurse include? Select
amount of collagen.
all that apply.
3. The nurse is preparing the plan of care for a client 1. Cool environments increase itching.
diagnosed with psoriasis. Which intervention 2. Use of soap increases itching.
should the nurse include in the plan of care? 3. Use hot water to rinse off soap.
1. Apply a thin dusting with Mycostatin, an 4. Apply mild skin lotion for hydration.
antifungal powder, over the area. 5. Blot gently, but completely dry the skin.
2. Cover the area with an occlusive dressing after
8. Which laboratory test should the nurse monitor
applying a steroid cream.
to identify an allergic reaction for the client
3. Administer acyclovir, an antiviral medication,
diagnosed with contact dermatitis?
to the affected areas six (6) times a day.
1. IgA.
4. Teach the client the risks and hazards of
2. IgD.
implanted radiation therapy.
3. IgE.
4. The nurse has completed the teaching plan for the 4. IgG.
client diagnosed with psoriasis. Which statement
9. Which client signs and symptoms indicate
indicates the need for further teaching?
contact dermatitis to the nurse?
1. “I will check my skin every day for redness with
1. Erythema and oozing vesicles.
tenderness.”
2. Pustules and nodule formation.
2. “I must take my psoralen medication two (2)
3. Varicosities and edema.
hours before my treatment.”
4. Telangiectasia and flushing.
3. “I will wear dark glasses during my treatment
and the rest of the day.”
4. “The coal-tar ointments and lotions will not
stain my clothes.”

12_Colgrove_Ch12.indd 504 01/06/16 5:54 PM


C hapter 12   I ntegumentary D isorders 505

10. The nurse is caring for the client diagnosed with 14. The nurse is preparing the plan of care for
contact dermatitis. Which collaborative a client diagnosed with Stevens-Johnson
intervention should the nurse implement? syndrome. Which interventions should the nurse
1. Encourage the use of support stockings. include? Select all that apply.
2. Administer a topical anti-inflammatory 1. Monitor intake and output every eight (8)
cream. hours.
3. Remove scales frequently by shampooing. 2. Assess breath sounds and rate every four (4)
4. Shampoo with lindane 1%, an antiparasitic, hours.
weekly. 3. Assess vesicles, erosions, and crusts frequently.
4. Perform the whisper test for auditory changes
11. The client had an allergic reaction to poison oak
daily.
two (2) weeks ago. He has returned to the clinic
5. Assess orientation to person, place, and time
with severe itching and weeping vesicles on the
every shift.
arms and legs. Which intervention should the
nurse implement? 15. Which expected outcome should the nurse
1. Obtain a sample of the drainage for culture include in the plan of care for the client
and sensitivities. diagnosed with seborrheic dermatitis?
2. Determine any allergic reactions to any 1. The client will have no further outbreaks.
medications taken recently. 2. The client will follow medical protocol.
3. Inquire how the poison ivy/oak plants were 3. The client will shampoo three (3) times a
destroyed. week.
4. Assess for any temperature elevation since the 4. The client will apply bacitracin twice daily.
last visit to the clinic.
16. The public health nurse is caring for a client
12. The client is complaining of severe itching diagnosed with leprosy (Hansen’s disease).
following a course of antibiotics. Which Which intervention should the nurse
independent nursing action should the nurse implement?
implement? 1. Explain the need for admission to the hospital.
1. Refer to an allergy specialist to begin 2. Administer dapsone, a sulfone, for one (1)
desensitization. month only.
2. Use a tar-preparation gel after each shower or 3. Instruct to use skin moisturizing lotion to
bath. control the symptoms.
3. Keep the covers tightly around the client at 4. Discuss the ways leprosy is transmitted to
night. other individuals.
4. Take baths with an OTC colloidal oatmeal
17. The health department nurse is caring for
preparation.
the client who has leprosy (Hansen’s disease).
13. The home health nurse is visiting an elderly Which assessment data indicate the client is
client who shows the nurse an area of rough skin experiencing a complication of the disease?
with a greasy feel and multiple papules. Which 1. Elevated temperature at night.
information should the nurse provide the client? 2. Brownish-black discoloration to the skin.
1. Contact the health-care provider immediately 3. Reduced skin sensation in the lesions.
for an appointment. 4. A high count of mycobacteria in the culture.
2. Tell the client this is a normal aging change
18. Which problem should the nurse identify for the
and no action should be taken.
client recently diagnosed with leprosy (Hansen’s
3. Tell the client to discuss this with the HCP at
disease)?
the next appointment.
1. Social isolation.
4. Have the client buy a wart remover kit at the
2. Altered body image.
store.
3. Potential for infection.
4. Alteration in comfort.

12_Colgrove_Ch12.indd 505 01/06/16 5:54 PM


506 M ed -S urg S uccess

19. The nurse is teaching the client diagnosed with 23. The nurse is caring for a client one (1) day
atopic dermatitis. Which information should the postoperative for facial reconstruction. Which
nurse include in the teaching? intervention should the nurse implement?
1. Discuss skin care using hydrating lotions and 1. Provide all activities of daily living.
minimal soap. 2. Allow the client to voice fears and concerns.
2. Tell the client the methods of treating 3. Monitor nutritional food and fluid intake.
secondary infection. 4. Assess signs and symptoms of infection.
3. Explain there are no adverse effects to using
24. The nurse and an unlicensed assistive personnel
topical corticosteroids daily.
(UAP) are caring for a client with a stage IV
4. Warn the client inhaled allergens have been
pressure ulcer. Which action by the UAP
linked to exacerbations.
warrants intervention by the nurse?
20. The nurse is working with clients in an aesthetic 1. The UAP turns the client every two (2) hours.
surgery center. Which intervention should 2. The UAP keeps the sheets wrinkle free.
the nurse implement for a client undergoing a 3. The UAP encourages the client to drink
chemical peel? high-protein drinks.
1. Teach the client to expect extreme swelling 4. The UAP places multiple diapers on the client.
after the procedure.
25. The nurse is caring for a male client diagnosed
2. Apply the chemical mixture directly to skin
with folliculitis barbae. Which information
after the face is cleansed.
should the nurse teach to prevent a
3. Administer general anesthesia to the client
reoccurrence?
prior to the procedure.
1. Tell the client to not shave the face.
4. Explain that there will be no pain or
2. Instruct the client to rub on astringent
discomfort during the procedure.
aftershave lotion.
21. The nurse is preparing the client scheduled for 3. Recommend the client apply hot packs for
a dermabrasion. Which information should the 20 minutes before shaving.
nurse include while teaching the client? 4. Teach the client to use an antibacterial soap
1. Erythema will go away within 24 hours. on the face.
2. Do not change the dressing until seen by the
26. The ED nurse is caring for a client admitted
HCP.
with extensive, deep partial-thickness and full-
3. Stay out of extreme cold or heat situations.
thickness burns. Which interventions should the
4. Avoid direct sunlight for three (3) days.
nurse implement? List in order of priority.
22. The nurse is caring for an elderly female client 1. Estimate the amount of burned area using the
preoperative for facial reconstruction. Which rule of nines.
client problem should the nurse include in the 2. Insert two (2) 18-gauge catheters and begin
preoperative plan of care? fluid replacement.
1. Loss of self-esteem. 3. Apply sterile saline dressings to the burned
2. Alteration in comfort. areas.
3. Ineffective airway clearance. 4. Determine the client’s airway status.
4. Impaired communication. 5. Administer morphine sulfate, IV.

12_Colgrove_Ch12.indd 506 01/06/16 5:54 PM


INTEGUMENTARY DISORDERS COMPREHENSIVE
EXAMINATION ANSWERS AND RATIONALES

1. 1. Hirsutism is an excessive amount of hair Content – Medical: Integrated Nursing Process –


growth in unexpected areas. This is not Planning: Client Needs – Physiological Integrity, Physiologi-
­associated with itching or a rash. cal Adaptation: Cognitive Level – Synthesis: Concept – Skin
2. A Wood’s light is used to examine certain in- Integrity.
fections. This would be used during the physi- 4. 1. The client needs to perform a complete in-
cal examination portion of the assessment. spection of skin to identify symptoms of gen-
3. The nurse should determine the previous eralized redness and tenderness. Treatments
treatment the client used, but it is not the first would be discontinued if they occur. The client
intervention. understands the teaching.
4. It is important to assess the rash as it ap- 2. Psoralen, a photosensitizing agent, is adminis-
peared. If the client treated the rash with tered two (2) hours before the ultraviolet light
an ointment or cream, its appearance may therapy to enhance the effects. The client un-
have changed. Many times the appearance derstands the teaching.
has changed from first onset and from the 3. The client will wear dark glasses to protect the
treatment. Assessment is the first part of eyes during the treatments and for the remain-
the nursing process. der of the day. Prior to the treatment, dark
Content – Medical: Integrated Nursing Process – glasses are not needed. The client understands
­Implementation: Client Needs – Safe Effective Care the teaching.
­Environment, Management of Care: Cognitive 4. Coal tar comes in lotions, ointments,
Level – Analysis: Concept – Skin Integrity. shampoos, and gels. They are used more in
2. 1. Most clients with psoriasis have red, raised the hospital setting than in home settings
plaques with silvery white scales. because of the staining and mess associ-
2. A burning, prickling row of vesicles located ated with their use. The client needs more
along the torso is the description of herpes teaching.
zoster. Content – Medical: Integrated Nursing Process –
3. Raised, flesh-colored papules with a rough ­Evaluation: Client Needs – Physiological Integrity,
­surface area is a description of warts. ­Physiological Adaptation: Cognitive Level – Synthesis:
4. An overgrowth of tissue with an excessive Concept – Skin Integrity.
amount of collagen is the definition of a keloid. 5. 1. Psoriasis can cause discomfort, itching, and
Content – Medical: Integrated Nursing Process – pain but it is not a psychosocial issue.
Assessment: Client Needs – Physiological Integrity, ­Reduction 2. Altered body image is a problem the nurse
of Risk Potential: Cognitive Level – ­Analysis: Concept – Skin should assess in clients with psoriasis. Any
Integrity. chronic skin disease affecting appearance
3. 1. Mycostatin is an antifungal medication and can cause psychosocial problems.
would not be useful to treat psoriasis. 3. Anxiety is not usually a problem in a client
2. Covering the affected area with an occlusive diagnosed with psoriasis. The main concern is
dressing enhances the steroid’s effectiveness. body image and discomfort.
This intervention should be limited to 12 4. The condition of psoriasis does not affect
hours to reduce systemic and local side effects. ­family processes.
3. Acyclovir is an antiviral medication and is used Content – Medical: Integrated Nursing Process –
for viral diseases. It would not be used for Planning: Client Needs – Psychosocial Integrity: ­Cognitive
psoriasis. Level – Synthesis: Concept – Skin Integrity.
4. Implanted radiation is a treatment for some
forms of cancer but not for psoriasis. Radiation
in the form of UV light therapy is sometimes
used to treat psoriasis.

507

12_Colgrove_Ch12.indd 507 01/06/16 5:54 PM


508 M ed -S urg S uccess

6. 1. Clients taking a steroid dose pack do not re- 10. 1. Support stockings are used for stasis dermati-
quire laboratory testing for a therapeutic level. tis, which is caused by impaired circulation.
2. The client should take the medication 2. Topical corticosteroids are administered
exactly as instructed. The number of pills to treat contact dermatitis, which comes
should be taken in a descending (tapering) from an allergic response to irritants. The
manner. irritant should be eliminated and topi-
3. Steroids by mouth can cause gastrointestinal cal anti-inflammatory creams should be
bleeding. They should be taken with food or administered.
after eating. 3. Seborrheic dermatitis is treated by frequent
4. Steroids should not be stopped suddenly; they cleaning with medicated shampoos and soaps
need to be tapered off. to remove the yellow scales.
Content – Medical: Integrated Nursing Process – 4. Lindane 1% shampoo is used to treat lice.
Planning: Client Needs – Physiological Integrity, The client shampoos the hair and rinses after
­Pharmacological and Parenteral Therapies: Cognitive Level – 10 minutes.
Synthesis: Concept – Medication. Content – Medical: Integrated Nursing Process –
7. 1. A cool environment makes itching decrease, ­Implementation: Client Needs – Safe Effective Care
­Environment, Management of Care: Cognitive
not increase.
Level – Application: Concept – Skin Integrity.
2. Soaps cause itching to increase. The c­ lient
should avoid soap when experiencing 11. 1. Collecting a sample for culture does not diag-
pruritus. nose poison oak or ivy. This is a reoccurrence
3. Tepid, cool water is better for the client who is of the allergic reaction to the poison oak.
itching. 2. These are the signs and symptoms of expo-
4. Mild lotion can help the skin stay hydrated. sure to poison oak, not to a medication.
5. The client should dry off completely after 3. Many people dispose of the poison oak
bathing and blot gently rather than rub plant in ways that spread the sap. Burning
vigorously. or pulling the plant without gloves can
Content – Medical: Integrated Nursing Process – cause another allergic reaction. Pets can
Planning: Client Needs – Health Promotion and spread the allergen on fur. Tools should
­Maintenance: Cognitive Level – Synthesis: Concept – Skin be cleaned prior to touching the skin.
Integrity. 4. Clients do not have temperature elevation
8. 1. Immunoglobulin A (IgA) protects against re- unless there is a secondary infection present.
spiratory, gastrointestinal, and genitourinary Content – Medical: Integrated Nursing Process –
infections. ­Implementation: Client Needs – Safe Effective Care
­Environment, Management of Care: Cognitive
2. IgD is a protein that is activated in collagen
Level – Analysis: Concept – Skin Integrity.
disease.
3. IgE is a protein responsible for allergic 12. 1. There is no indication for the need for the
reactions. client to be desensitized to the medication.
4. IgG is the major antibody for viruses, bacteria, The client should inform health-care provid-
and toxins. ers of any previous reactions to medications
Content – Medical: Integrated Nursing Process – prior to taking any medication.
Assessment: Client Needs – Physiological Integrity, ­Reduction 2. Tar solutions are used for psoriasis, not pruri-
of Risk Potential: Cognitive Level – ­Analysis: Concept – tus. Use of tar solutions would be a collabora-
Skin Integrity. tive intervention, rather than an independent
9. 1. Contact dermatitis presents with ­erythema intervention.
and small oozing vesicles. 3. Cool sleeping environments decrease itch-
2. Pustules and nodule formation indicate acne. ing. Warmth increases itching and should be
3. Stasis dermatitis presents with varicosities and avoided.
edema. 4. Soothing baths, such as colloidal baths or
4. Clients diagnosed with rosacea present with emollient baths, are helpful in treating
telangiectasia and periodic flushing. pruritus. Balneotherapy is a term used to
refer to therapeutic baths.
Content – Medical: Integrated Nursing Process –
Assessment: Client Needs – Physiological Integrity, Reduction Content – Medical: Integrated Nursing Process –
of Risk Potential: Cognitive Level – ­Application: Concept – ­Implementation: Client Needs – Safe Effective Care
Skin Integrity. ­Environment, Management of Care: Cognitive
Level – Application: Concept – Skin Integrity.

12_Colgrove_Ch12.indd 508 01/06/16 5:54 PM


C hapter 12   I ntegumentary D isorders 509

13. 1. An area that has a greasy, rough feel and mul- ­Physiological Adaptation: Cognitive Level – Synthesis:
tiple papules does not require an immediate Concept – Skin Integrity.
appointment with the HCP. 16. 1. Clients are treated on an outpatient basis by
2. Seborrheic keratosis is a common occurrence specialized clinics. In the United States, it is
in the elderly, but the skin lesion should be the health department’s responsibility to care
assessed by a health-care provider. for these clients and ensure that the clients
3. The client should discuss any suspicious are taking their medications.
area with the health-care provider. This is 2. Dapsone, a sulfone, will be used to treat lep-
not an emergency, but it should be assessed. rosy for several years up to the remainder of
4. An area wartlike in appearance, varying in the client’s life.
color from flesh tones to black, and having 3. Moisturizing lotion will not treat the infec-
a greasy, rough feel is probably a seborrheic tious process and therefore cannot control
keratosis. The home health nurse does not the symptoms.
have the authority to diagnose as a health- 4. Contrary to popular thought, leprosy,
care provider, and, therefore, the nurse should although contagious, usually requires a
not encourage the client to self-treat. prolonged exposure for the infection to
Content – Medical: Integrated Nursing Process – spread to another person. Touching the
­Implementation: Client Needs – Safe Effective Care lesions directly will increase the potential
­Environment, Management of Care: Cognitive for infection.
Level – Analysis: Concept – Skin Integrity.
Content – Medical: Integrated Nursing Process –
14. 1. The client with Stevens-Johnson syn- ­Implementation: Client Needs – Safe Effective Care
drome must be assessed for fluid volume ­Environment, Safety and Infection Control: Cognitive
deficit (FVD), the need for fluid replace- Level – Application: Concept – Skin Integrity.
ment, and renal failure. Intake and output 17. 1. The client does not usually have an elevated
monitors both. temperature.
2. Breath sounds and respiratory status 2. A side effect of the medication Dapsone is a
should be assessed because many clients discoloration of the skin from pink to brown-
develop respiratory failure and require ish black.
mechanical ventilation. 3. The decrease in sensation of the lesions
3. The client with Stevens-Johnson syndrome is the result of peripheral nerve damage.
has a combination of vesicles, e ­ rosions, and Leprosy is a peripheral nervous system
crusts at the same time. The skin should disease.
be assessed every eight (8) hours. 4. A high mycobacterium count would be
4. Hearing is not affected by Stevens-Johnson ­expected from the disease but would not be a
syndrome, but blindness can be a complica- complication.
tion; vision should be assessed for any changes.
Content – Medical: Integrated Nursing Process –
5. Neurological status is not compromised.
Assessment: Client Needs – Physiological Integrity,
Content – Medical: Integrated Nursing Process – ­Reduction of Risk Potential: Cognitive Level – ­Analysis:
Planning: Client Needs – Safe Effective Care E
­ nvironment, Concept – Skin Integrity.
Management of Care: Cognitive Level – Synthesis:
­Concept – Skin Integrity. 18. 1. The client diagnosed with leprosy (­Hansen’s
disease) may feel ostracized because of the
15. 1. Seborrheic dermatitis is a chronic skin dis- stigma of the disease. Historically, people
order with remissions and exacerbations. To have been isolated from society when diag-
have an expected outcome of no further out- nosed. Leprosy colonies were sites of treat-
breaks would not be realistic. ment for those diagnosed. Today much of
2. To control the disorder by following the the public is unaware of the presence of the
medical protocols would be realistic and disease. Clients are treated on an outpatient
appropriate. basis by health departments.
3. The client needs to shampoo daily or a mini- 2. Altered body image would occur in the late
mum of three (3) times each week for treat- stages of leprosy and may not occur at all if
ment. This is an intervention, not an outcome. treated in early stages.
4. To apply bacitracin would treat a bacterial in- 3. The client with leprosy already has an infec-
fection. This is an intervention, not a goal or tion so it is not potential.
expected outcome. 4. Clients with leprosy have a decreased sensa-
Content – Medical: Integrated Nursing Process – tion from peripheral nerve damage and have
Planning: Client Needs – Physiological Integrity, no discomfort.

12_Colgrove_Ch12.indd 509 01/06/16 5:54 PM


510 M ed -S urg S uccess

Content – Medical: Integrated Nursing Process – Content – Surgical: Integrated Nursing Process –
Diagnosis: Client Needs – Safe Effective Care Environment, Planning: Client Needs – Physiological Integrity,
Management of Care: Cognitive Level – Analysis: ­Physiological Adaptation: Cognitive Level – Synthesis:
Concept – Skin Integrity. ­Concept – Perioperative.

19. 1. Skin care must be meticulous. Minimal 22. 1. A loss of self-esteem can occur after a
soap and tepid water should be used when change in facial appearance through in-
showering or bathing. Lotions that do not jury, disease, or age-related changes.
irritate should be used to keep the skin 2. Edema and pain would be appropriate
hydrated. postoperatively.
2. The client needs to know the signs/­symptoms 3. The airway has not been compromised
indicating the need to notify the HCP, not preoperatively.
the methods of treatment. 4. Communication may be a problem postop-
3. There are adverse effects of the topical use of eratively but not prior to surgery.
corticosteroids. Content – Surgical: Integrated Nursing Process –
4. Research does not support a link between Diagnosis: Client Needs – Safe Effective Care ­Environment,
inhaled allergens and atopic dermatitis Management of Care: Cognitive Level – Analysis:
exacerbations. Concept – Perioperative.
Content – Medical: Integrated Nursing Process – 23. 1. The client should be able to perform most
Planning: Client Needs – Physiological Integrity, activities of daily living. The nurse must
­Physiological Adaptation: Cognitive Level – Synthesis:
­encourage independence in the client.
­Concept – Skin Integrity.
2. On the first day postoperative, the nurse’s
20. 1. After the first six (6) to eight (8) hours, ­priority would be physiological needs.
the client will have extreme edema caus- 3. Monitoring the client’s nutritional intake
ing the eyes to swell. This is expected. and fluid balance is important for healing.
2. The dermatologist will apply the chemical to 4. Assessing for infection is a responsibility of
begin the peeling procedure. the nurse but would occur later than one (1)
3. The client will be awake during the proce- day postoperative. This is an elective proce-
dure, but an analgesic and a tranquilizer can dure, which means the client would not have
be administered for sedation. Only certified had an infection prior to surgery.
registered nurse anesthetists (CRNAs) ad- Content – Surgical: Integrated Nursing Process –
minister general anesthesia. ­Implementation: Client Needs – Safe Effective Care
4. There is a sensation of burning during the ­Environment, Management of Care: Cognitive Level –
application of the chemical and for several Application: Concept – Perioperative.
days after the procedure. 24. 1. The client should be turned every two (2)
Content – Surgical: Integrated Nursing Process – hours, so this would not warrant intervention
­Implementation: Client Needs – Safe Effective Care by the nurse.
­Environment, Management of Care: Cognitive
2. Keeping the sheets wrinkle free helps with
Level – Application: Concept – Perioperative.
preventing skin breakdown. This would not
21. 1. Erythema can last from one (1) week to a warrant intervention by the nurse.
month. 3. Protein is needed for wound healing, so this
2. After 24 hours, the serum oozes from the intervention is appropriate for the UAP.
dressing, and the client needs to apply a pre- 4. Placing extra diapers saves the UAP from
scribed ointment to keep the area soft and changing the linens, but it keeps wet plas-
flexible. tic against the skin, leading to further skin
3. Extreme cold and heat, along with strain- breakdown. This action would warrant
ing and lifting heavy objects, should be intervention by the nurse.
avoided. Content – Medical: Integrated Nursing Process –
4. Direct sunlight should be avoided for three Evaluation: Client Needs – Safe Effective Care
(3) to six (6) months. Clients should be taught ­Environment, Management of Care: Cognitive Level –
to wear a sunscreen. Synthesis: Concept – Nursing Roles.

12_Colgrove_Ch12.indd 510 01/06/16 5:54 PM


C hapter 12   I ntegumentary D isorders 511

25. 1. Shaving is the cause of this condition, 26. In order of priority: 4, 2, 3, 1, 5.


and refraining from shaving is the only 4. Airway is always the first priority for any
cure. Special brushes are used. If the cli- process in which the airway might be
ent must shave, he should use a depilatory compromised.
cream or electric razor. 2. The nurse should start fluid resuscita-
2. Aftershave will not prevent folliculitis. tion as soon as possible before the client’s
3. Hot packs will not prevent folliculitis. blood pressure makes it more difficult to
4. Antibacterial soap is too strong for use on the establish an IV route.
face. Shaving is the cause of folliculitis. 3. Covering the open burns will prevent fur-
Content – Medical: Integrated Nursing Process – ther intrusion of bacteria.
Planning: Client Needs – Health Promotion and 1. Estimating the extent of the burned area
­Maintenance: Cognitive Level – Synthesis: Concept – Skin should be done but does not have prior-
Integrity. ity over airway, fluid replacement, and the
prevention of infection.
5. Pain is priority but not over determining
airway and fluid status and prevention of
infection.
Content – Medical: Integrated Nursing Process –
Implementation: Client Needs – Safe Effective Care
Environment, Management of Care: Cognitive
Level – Analysis: Concept – Skin Integrity.

12_Colgrove_Ch12.indd 511 01/06/16 5:54 PM


12_Colgrove_Ch12.indd 512 01/06/16 8:28 PM
Immune System
Disorders
The secret of joy in work is contained in one word—excellence. To
know how to do something well is to enjoy it.
13
—Pearl S. Buck

The immune system, which involves many tissues throughout the body, is subject to several
inflammatory disorders. Some involve genetic predisposition, some result from infectious
processes, and many are of uncertain etiology. This chapter includes questions on the most
common immune disorders—multiple sclerosis, Guillain-Barré syndrome, rheumatoid ar-
thritis, acquired immunodeficiency syndrome (AIDS), systemic lupus erythematosus, and
allergies—and on other disorders affecting the body’s immune system.

KEYWORDS ABBREVIATIONS
Alopecia Acquired immunodeficiency syndrome (AIDS)
Arthralgia Activities of daily living (ADLs)
Astringent Apical pulse (AP)
Cogwheel rigidity Arterial blood gases (ABGs)
Cutaneous Blood pressure (BP)
Demyelination Electromyogram (EMG)
Diplopia Erythrocyte sedimentation rate (ESR)
Dysarthria Guillain-Barré (GB) syndrome
Dysmetria Health-care provider (HCP)
Eradicated Human immunodeficiency virus (HIV)
Erythema Intramuscular (IM)
Exacerbation Intravenous (IV)
Hirsutism Intravenous piggyback (IVPB)
Ocular Intravenous push (IVP)
Plasmapheresis Licensed practical nurse (LPN)
Polymyositis Magnetic resonance imaging (MRI)
Pruritus Multiple sclerosis (MS)
Raynaud’s phenomenon Myasthenia gravis (MG)
Rhinitis Nonsteroidal anti-inflammatory drugs (NSAIDs)
Scleroderma Nothing by mouth (NPO)
Scotomas Nurse practitioner (NP)
Spasticity Osteoarthritis (OA)
Wheal Percutaneous endoscopic gastrostomy (PEG)
Pneumocystis pneumonia (PCP)
Range of motion (ROM)
Related to (R/T)
Respiratory rate (RR)
Rule out (R/O)

513

13_Colgrove_Ch13.indd 513 01/06/16 5:55 PM


514 M ed -S urg S uccess

Subcutaneous (SQ)
Systemic lupus erythematosus (SLE)
Ultraviolet (UV)

PRACTICE QUESTIONS
Multiple Sclerosis 4. The nurse enters the room of a client diagnosed
with acute exacerbation of multiple sclerosis and
1. The nurse is assessing a 48-year-old client finds the client crying. Which statement is the
diagnosed with multiple sclerosis. Which clinical most therapeutic response for the nurse
manifestation warrants immediate intervention? to make?
1. The client has scanning speech and diplopia. 1. “Why are you crying? The medication will
2. The client has dysarthria and scotomas. help the disease.”
3. The client has muscle weakness and spasticity. 2. “You seem upset. I will sit down and we can
4. The client has a congested cough and talk for awhile.”
dysphagia. 3. “Multiple sclerosis is a disease that has good
times and bad times.”
2. The client newly diagnosed with multiple 4. “I will have the chaplain come and stay with
sclerosis (MS) states, “I don’t understand how I you for a while.”
got multiple sclerosis. Is it genetic?” On which
statement should the nurse base the response? 5. The client diagnosed with multiple sclerosis is
1. Genetics may play a role in susceptibility to scheduled for a magnetic resonance imaging
MS, but the disease may be caused by a virus. (MRI) scan of the head. Which information
2. There is no evidence suggesting there is any should the nurse teach the client about the test?
chromosomal involvement in developing MS. 1. The client will have wires attached to the scalp
3. Multiple sclerosis is caused by a recessive gene, and lights will flash off and on.
so both parents had to have the gene for the 2. The machine will be loud and the client must
client to get MS. not move the head during the test.
4. Multiple sclerosis is caused by an autosomal 3. The client will drink a contrast medium
dominant gene on the Y chromosome, so only 30 minutes to one (1) hour before the test.
fathers can pass it on. 4. The test will be repeated at intervals during a
five (5)- to six (6)-hour period.
3. The 30-year-old female client is admitted with
complaints of numbness, tingling, a crawling 6. The 45-year-old client is diagnosed with primary
sensation affecting the extremities, and double progressive multiple sclerosis and the nurse writes
vision which has occurred two (2) times in the the nursing diagnosis “anticipatory grieving
month. Which question is most important for related to progressive loss.” Which intervention
the nurse to ask the client? should be implemented first?
1. “Have you experienced any difficulty with your 1. Consult the physical therapist for assistive
menstrual cycle?” devices for mobility.
2. “Have you noticed a rash across the bridge of 2. Determine if the client has a legal power of
your nose?” attorney.
3. “Do you get tired easily and sometimes have 3. Ask if the client would like to talk to the
problems swallowing?” hospital chaplain.
4. “Are you taking birth control pills to prevent 4. Discuss the client’s wishes regarding end-of-
conception?” life care.

13_Colgrove_Ch13.indd 514 01/06/16 5:55 PM


C hapter 13   I mmune S ystem D isorders 515

7. The home health nurse is caring for the client 11. The nurse writes the client problem of “altered
newly diagnosed with multiple sclerosis. Which sexual functioning” for a male client diagnosed
client issue is of most importance? with multiple sclerosis (MS). Which intervention
1. The client refuses to have a gastrostomy should be implemented?
feeding. 1. Encourage the couple to explore alternative
2. The client wants to discuss if she should tell ways of maintaining intimacy.
her fiancé. 2. Make an appointment with a psychotherapist
3. The client tells the nurse life is not worth to counsel the couple.
living anymore. 3. Explain daily exercise will help increase libido
4. The client needs the flu and pneumonia and sexual arousal.
vaccines. 4. Discuss the importance of keeping physically
calm during sexual intercourse.
8. The nurse and a licensed practical nurse
(LPN) are caring for a group of clients. Which 12. The nurse is admitting a client diagnosed with
nursing task should not be assigned to the LPN? multiple sclerosis. Which clinical manifestation
1. Administer a skeletal muscle relaxant to a should the nurse assess? Select all that apply.
client diagnosed with low back pain. 1. Muscle flaccidity.
2. Discuss bowel regimen medications with the 2. Lethargy.
HCP for the client on strict bedrest. 3. Dysmetria.
3. Draw morning blood work on the client 4. Fatigue.
diagnosed with bacterial meningitis. 5. Dysphagia.
4. Teach self-catheterization to the client
diagnosed with multiple sclerosis.
Guillain-Barré Syndrome
9. The male client diagnosed with multiple
sclerosis states he has been investigating 13. Which assessment data should the nurse assess
alternative therapies to treat his disease. in the client diagnosed with Guillain-Barré
Which intervention is most appropriate by the syndrome?
nurse? 1. An exaggerated startle reflex and memory
1. Encourage the therapy if it is not changes.
contraindicated by the medical regimen. 2. Cogwheel rigidity and inability to initiate
2. Tell the client only the health-care provider voluntary movement.
should discuss this with him. 3. Sudden severe unilateral facial pain and
3. Ask how his significant other feels about this inability to chew.
deviation from the medical regimen. 4. Progressive ascending paralysis of the lower
4. Suggest the client research an investigational extremities and numbness.
therapy instead.
14. Which statement by the client supports the
10. The client diagnosed with an acute exacerbation diagnosis of Guillain-Barré syndrome?
of multiple sclerosis is placed on high-dose 1. “I just returned from a short trip to Japan.”
intravenous injections of corticosteroid 2. “I had a really bad cold just a few weeks ago.”
medication. Which nursing intervention should 3. “I think one of the people I work with
be implemented? had this.”
1. Discuss discontinuing the proton pump 4. “I have been taking some herbs for more than
inhibitor with the HCP. a year.”
2. Hold the medication until after all cultures
have been obtained. 15. Which assessment intervention should the
3. Monitor the client’s serum blood glucose nurse implement specifically for the diagnosis of
levels frequently. Guillain-Barré syndrome?
4. Provide supplemental dietary sodium with the 1. Assess deep tendon reflexes.
client’s meals. 2. Complete a Glasgow Coma Scale.
3. Check for Babinski’s reflex.
4. Take the client’s vital signs.

13_Colgrove_Ch13.indd 515 01/06/16 5:55 PM


516 M ed -S urg S uccess

16. The health-care provider scheduled a lumbar 21. The client admitted with rule-out Guillain-
puncture for a client admitted with rule-out Barré syndrome has just had a lumbar puncture.
Guillain-Barré syndrome. Which preprocedure Which intervention should the nurse implement
intervention has priority? postprocedure?
1. Keep the client NPO. 1. Monitor the client for hypotension.
2. Instruct the client to void. 2. Apply pressure to the puncture site.
3. Place in the lithotomy position. 3. Test the client’s cerebrospinal fluid.
4. Assess the client’s pedal pulse. 4. Increase the client’s fluid intake.
17. Which priority client problem should be 22. The client diagnosed with Guillain-Barré
included in the care plan for the client diagnosed syndrome is having difficulty breathing and is
with Guillain-Barré syndrome? placed on a ventilator. Which situation warrants
1. High risk for injury. immediate intervention by the nurse?
2. Fear and anxiety. 1. The ventilator rate is set at 14 breaths per
3. Altered nutrition. minute.
4. Ineffective breathing pattern. 2. A manual resuscitation bag is at the client’s
bedside.
18. The nurse caring for the client diagnosed with
3. The client’s pulse oximeter reading is 85%.
Guillain-Barré syndrome writes the client
4. The ABG results are pH 7.4, Pao2 88,
problem “impaired physical mobility.” Which
Paco2 35, and Hco3 24.
long-term goal should be written for this
problem? 23. The client diagnosed with Guillain-Barré
1. The client will have no skin irritation. syndrome is on a ventilator. When the wife
2. The client will have no muscle atrophy. comes to visit, she starts crying uncontrollably,
3. The client will perform range-of-motion and the client starts fighting the ventilator
exercises. because his wife is upset. Which action should
4. The client will turn every two (2) hours while the nurse implement?
awake. 1. Tell the wife she must stop crying.
2. Escort the wife out of the room.
19. The client diagnosed with Guillain-Barré
3. Medicate the client immediately.
syndrome is on a ventilator. Which intervention
4. Acknowledge the wife’s fears.
will assist the client to communicate with the
nursing staff? 24. The client diagnosed with Guillain-Barré
1. Provide an erase slate board for the client syndrome is admitted to the rehabilitation unit
to write on. after 23 days in the acute care hospital. Which
2. Instruct the client to blink once for “no” and interventions should the nurse implement?
twice for “yes.” Select all that apply.
3. Refer to a speech therapist to help with 1. Refer the client to the physical therapist.
communication. 2. Include the speech therapist in the team.
4. Leave the call light within easy reach of the 3. Request a social worker consult.
client. 4. Implement a regimen to address pain control.
5. Refer the client to the Guillain-Barré
20. The client diagnosed with Guillain-Barré
Syndrome Foundation.
syndrome asks the nurse, “Will I ever get back
to normal? I am so tired of being sick.” Which
statement is the best response by the nurse? Myasthenia Gravis
1. “You should make a full recovery within a few
months to a year.” 25. Which ocular or facial signs/symptoms
2. “Most clients with this syndrome have some should the nurse expect to assess for the client
type of residual disability.” diagnosed with myasthenia gravis?
3. “This is something you should discuss with 1. Weakness and fatigue.
the health-care team.” 2. Ptosis and diplopia.
4. “The rehabilitation is short and you should be 3. Breathlessness and dyspnea.
fully recovered within a month.” 4. Weight loss and dehydration.

13_Colgrove_Ch13.indd 516 01/06/16 5:55 PM


C hapter 13   I mmune S ystem D isorders 517

26. The client is being evaluated to rule out 31. Which statement by the female client diagnosed
myasthenia gravis and being administered the with myasthenia gravis indicates the client needs
Tensilon (edrophonium chloride) test. Which more discharge teaching?
response to the test indicates the client has 1. “I will not have any menstrual cycles because
myasthenia gravis? of this disease.”
1. The client has no apparent change in the 2. “I should avoid people who have respiratory
assessment data. infections.”
2. There is increased amplitude of electrical 3. “I should not take a hot bath or swim in cold
stimulation in the muscle. water.”
3. The circulating acetylcholine receptor 4. “I will drink at least 2,500 mL of water a day.”
antibodies are decreased.
32. The client diagnosed with myasthenia gravis
4. The client shows a marked improvement of
is admitted to the emergency department with
muscle strength.
a sudden exacerbation of motor weakness.
27. Which surgical procedure should the nurse Which assessment data indicate the client is
anticipate the client with myasthenia gravis experiencing a cholinergic crisis?
undergoing to help prevent the signs/symptoms 1. The serum assay of circulating acetylcholine
of the disease process? receptor antibodies is increased.
1. There is no surgical option. 2. The client’s symptoms improve when
2. A transsphenoidal hypophysectomy. administering a cholinesterase inhibitor.
3. A thymectomy. 3. The client’s blood pressure, pulse, and
4. An adrenalectomy. respirations improve after IV fluid.
4. The Tensilon test does not show improvement
28. The client diagnosed with myasthenia gravis is
in the client’s muscle strength.
being discharged home. Which intervention has
priority when teaching the client’s significant 33. The client diagnosed with myasthenia gravis
others? is admitted with an acute exacerbation. Which
1. Discuss ways to help prevent choking interventions should the nurse implement?
episodes. Select all that apply.
2. Explain how to care for a client on a 1. Assist the client to turn and cough every two
ventilator. (2) hours.
3. Teach how to perform passive range-of- 2. Place the client in a high or semi-Fowler’s
motion exercises. position.
4. Demonstrate how to care for the client’s 3. Assess the client’s pulse oximeter reading
feeding tube. every shift.
4. Plan meals to promote medication
29. Which collaborative health-care team member
effectiveness.
should the nurse refer the client to in the late
5. Monitor the client’s serum anticholinesterase
stages of myasthenia gravis?
levels.
1. Occupational therapist.
2. Recreational therapist. 34. The wife of a client diagnosed with myasthenia
3. Vocational therapist. gravis is crying and shares with the nurse she just
4. Speech therapist. doesn’t know what to do. Which response is the
best action by the nurse?
30. The client with myasthenia gravis is undergoing
1. Discuss the Myasthenia Foundation with the
plasmapheresis at the bedside. Which assessment
client’s wife.
data warrant immediate intervention?
2. Refer the client to a local myasthenia gravis
1. The client’s BP is 94/60 and AP is 112.
support group.
2. Negative Chvostek’s and Trousseau’s signs.
3. Ask the client’s wife if she would like to talk to
3. The serum potassium level is 3.5 mEq/L.
a counselor.
4. Ecchymosis at the vascular site access.
4. Sit down and allow the wife to ventilate her
feelings to the nurse.

13_Colgrove_Ch13.indd 517 01/06/16 5:55 PM


518 M ed -S urg S uccess

35. The client with myasthenia gravis is prescribed 39. The nurse is developing a care plan for a client
the cholinesterase inhibitor neostigmine diagnosed with SLE. Which goal is priority for
(Prostigmin). Which data indicate the this client?
medication is effective? 1. The client will maintain reproductive ability.
1. The client is able to feed self independently. 2. The client will verbalize feelings of body-
2. The client is able to blink the eyes without image changes.
tearing. 3. The client will have no deterioration of organ
3. The client denies any nausea or vomiting function.
when eating. 4. The client’s skin will remain intact and have
4. The client denies any pain when performing no irritation.
ROM exercises.
40. The nurse is admitting a client diagnosed with
36. The client is diagnosed with myasthenia gravis. R/O SLE. Which assessment data observed by
Which intervention should the nurse implement the nurse support the diagnosis of SLE?
when administering the anticholinesterase 1. Pericardial friction rub and crackles in the
pyridostigmine (Mestinon)? lungs.
1. Administer the medication 30 minutes prior 2. Muscle spasticity and bradykinesia.
to meals. 3. Hirsutism and clubbing of the fingers.
2. Instruct the client to take with eight (8) 4. Somnolence and weight gain.
ounces of water.
41. The client diagnosed with an acute exacerbation
3. Explain the importance of sitting up for
of SLE is prescribed high-dose steroids. Which
one (1) hour after taking medication.
statement best explains the scientific rationale
4. Assess the client’s blood pressure prior to
for using high-dose steroids in treating SLE?
administering medication.
1. The steroids will increase the body’s ability to
fight the infection.
Systemic Lupus Erythematosus 2. The steroids will decrease the chance of the
SLE spreading to other organs.
37. The 26-year-old female client is complaining of 3. The steroids will suppress tissue
a low-grade fever, arthralgias, fatigue, and a facial inflammation, which reduces damage to
rash. Which laboratory tests should the nurse organs.
expect the HCP to order if SLE is suspected? 4. The steroids will prevent scarring of skin
1. Complete metabolic panel and liver function tissues associated with SLE.
tests. 42. The nurse enters the room of a female client
2. Complete blood count and antinuclear diagnosed with SLE and finds the client crying.
antibody tests. Which statement is the most therapeutic
3. Cholesterol and lipid profile tests. response?
4. Blood urea nitrogen and glomerular filtration 1. “I know you are upset, but stress makes the
tests. SLE worse.”
38. The client diagnosed with SLE is being 2. “Please explain to me why you are crying.”
discharged from the medical unit. Which 3. “I recommend going to an SLE support
discharge instructions are most important for group.”
the nurse to include? Select all that apply. 4. “I see you are crying. We can talk if you would
1. Use a sunscreen of SPF 30 or greater when in like.”
the sunlight. 43. The nurse is assessing a client with cutaneous
2. Notify the HCP immediately when lupus erythematosus. Which intervention should
developing a low-grade fever. be implemented?
3. Some dyspnea is expected and does not need 1. Use astringent lotion on the face and skin.
immediate attention. 2. Inspect the skin weekly for open areas or
4. The hands and feet may change color if rashes.
exposed to cold or heat. 3. Dry the skin thoroughly by patting.
5. Explain the client can be cured with 4. Apply anti-itch medication between the toes.
continued therapy.

13_Colgrove_Ch13.indd 518 01/06/16 5:55 PM


C hapter 13   I mmune S ystem D isorders 519

44. The nurse is caring for clients on a medical floor. 48. The nurse is discussing autoimmune diseases
Which client should be assessed first? with a class of nursing students. Which signs and
1. The client diagnosed with SLE who is symptoms are shared by rheumatoid arthritis
complaining of chest pain. (RA) and systemic lupus erythematosus (SLE)?
2. The client diagnosed with MS who is 1. Nodules in the subcutaneous layer and bone
complaining of pain at a “10.” deformity.
3. The client diagnosed with myasthenia gravis 2. Renal involvement and pleural effusions.
who has dysphagia. 3. Joint stiffness and pain.
4. The client diagnosed with GB syndrome who 4. Raynaud’s phenomenon and skin rash.
can barely move his toes.
45. The nurse and a female unlicensed assistive Acquired Immunodeficiency Syndrome
personnel (UAP) are caring for a group of clients
on a medical floor. Which action by the UAP 49. The school nurse is preparing to teach a
warrants immediate intervention by the nurse? health class to ninth graders regarding sexually
1. The UAP washes her hands before and after transmitted diseases. Which information
performing vital signs on a client. regarding acquired immunodeficiency syndrome
2. The UAP dons sterile gloves prior to (AIDS) should be included?
removing an indwelling catheter from a client. 1. Females taking birth control pills are
3. The UAP raises the head of the bed to a high protected from becoming infected with HIV.
Fowler’s position for a client about to eat. 2. Protected sex is no longer an issue because
4. The UAP uses a fresh plastic bag to get ice for there is a vaccine for the HIV virus.
a client’s water pitcher. 3. Adolescents with a normal immune system are
46. The client recently diagnosed with SLE asks not at risk for developing AIDS.
the nurse, “What is SLE and how did I get it?” 4. Abstinence is the only guarantee of not
Which statement best explains the scientific becoming infected with sexually transmitted
rationale for the nurse’s response? HIV.
1. SLE occurs because the kidneys do not filter 50. The nurse is admitting a client diagnosed with
antibodies from the blood. protein-calorie malnutrition secondary to AIDS.
2. SLE occurs after a viral illness as a result of Which intervention should be the nurse’s first
damage to the endocrine system. intervention?
3. There is no known identifiable reason for a 1. Assess the client’s body weight and ask what
client to develop SLE. the client has been able to eat.
4. This is an autoimmune disease that may have 2. Place in contact isolation and don a mask and
a genetic or hormonal component. gown before entering the room.
47. The client diagnosed with an acute exacerbation 3. Check the HCP’s orders and determine what
of SLE is being discharged with a prescription laboratory tests will be done.
for an oral steroid which will be discontinued 4. Teach the client about total parenteral
gradually. Which statement is the scientific nutrition and monitor the subclavian IV site.
rationale for this type of medication dosing? 51. The client diagnosed with AIDS is complaining
1. Tapering the medication prevents the client of a sore mouth and tongue. When the nurse
from having withdrawal symptoms. assesses the buccal mucosa, the nurse notes
2. So the thyroid gland starts working, because white, patchy lesions covering the hard and
this medication stops it from working. soft palates and the right inner cheek. Which
3. Tapering the dose allows the adrenal glands to interventions should the nurse implement?
begin to produce cortisol again. 1. Teach the client to brush the teeth and patchy
4. This is the health-care provider’s personal area with a soft-bristle toothbrush.
choice in prescribing the medication. 2. Notify the HCP for an order for an antifungal
swish-and-swallow medication.
3. Have the client gargle with an antiseptic-
based mouthwash several times a day.
4. Determine what types of food the client has
been eating for the last 24 hours.

13_Colgrove_Ch13.indd 519 01/06/16 5:55 PM


520 M ed -S urg S uccess

52. Which type of isolation technique is designed to 57. The client diagnosed with Pneumocystis
decrease the risk of transmission of recognized pneumonia (PCP) is being admitted to the
and unrecognized sources of infections? intensive care unit.Which HCP’s order should
1. Contact Precautions. the nurse implement first?
2. Airborne Precautions 1. Draw a serum for CD4 and complete blood
3. Droplet Precautions. count STAT.
4. Standard Precautions. 2. Administer oxygen to the client via nasal
cannula.
53. The nurse is describing the HIV virus infection
3. Administer trimethoprim-sulfamethoxazole, a
to a client who has been told he is HIV positive.
sulfa antibiotic, IVPB.
Which information regarding the virus is
4. Obtain a sputum specimen for culture and
important to teach?
sensitivity.
1. The HIV virus is a retrovirus, which means it
never dies as long as it has a host to live in. 58. Which intervention is an important psychosocial
2. The HIV virus can be eradicated from the consideration for the client diagnosed with
host body with the correct medical regimen. AIDS?
3. It is difficult for the HIV virus to replicate in 1. Perform a thorough head-to-toe assessment.
humans because it is a monkey virus. 2. Maintain the client’s ideal body weight.
4. The HIV virus uses the client’s own red blood 3. Complete an advance directive.
cells to reproduce the virus in the body. 4. Increase the client’s activity tolerance.
54. The client who has engaged in needle-sharing 59. The nurse on a medical floor is caring for clients
activities has developed a flu-like infection. An diagnosed with AIDS. Which client should be
HIV antibody test is negative. Which statement seen first?
best describes the scientific rationale for this 1. The client who has flushed, warm skin with
finding? tented turgor.
1. The client is fortunate not to have contracted 2. The client who states the staff ignores the call
HIV from an infected needle. light.
2. The client must be repeatedly exposed to 3. The client whose vital signs are T 99.9˚F,
HIV before becoming infected. P 101, R 26, and BP 110/68.
3. The client may be in the primary infection 4. The client who is unable to provide a sputum
phase of an HIV infection. specimen.
4. The antibody test is negative because the
60. The client diagnosed with AIDS is angry and
client has a different flu virus.
yells at everyone entering the room, and none
55. The nurse caring for a client who is HIV of the staff members wants to care for the client.
positive is stuck with the stylet used to start Which intervention is most appropriate for the
an IV. Which intervention should the nurse nurse manager to use in resolving this situation?
implement first? 1. Assign a different nurse every shift to the
1. Flush the skin with water and try to get the client.
area to bleed. 2. Ask the HCP to tell the client not to yell at
2. Notify the charge nurse and complete an the staff.
incident report. 3. Call a team meeting and discuss options with
3. Report to the employee health nurse for the staff.
prophylactic medication. 4. Tell one (1) staff member to care for the client
4. Follow up with the infection control nurse to a week at a time.
have laboratory work done.
56. The client on a medical floor is diagnosed with
HIV encephalopathy. Which client problem is
priority?
1. Altered nutrition, less than body
requirements.
2. Anticipatory grieving.
3. Knowledge deficit, procedures and prognosis.
4. Risk for injury.

13_Colgrove_Ch13.indd 520 01/06/16 5:55 PM


C hapter 13   I mmune S ystem D isorders 521

Allergies and Allergic Reactions 65. The client in the HCP’s office is complaining of
allergic rhinitis. Which assessment question is
61. The charge nurse observes the primary nurse important for the nurse to ask the client?
interacting with a client. Which action by the 1. “What time of year do the symptoms occur?”
primary nurse warrants immediate intervention 2. “Which over-the-counter medications have
by the charge nurse? you tried?”
1. The nurse explains the IVP diuretic will make 3. “Do other members of your family have
the client urinate. allergies to animals?”
2. The nurse dons nonsterile gloves to remove 4. “Why do you think you have allergies?”
the client’s dressing. 66. The client asks the nurse, “Which time of the
3. The nurse administers a medication without year is allergic rhinitis least likely to occur?”
checking for allergies. Which statement is the nurse’s best response?
4. The nurse asks the UAP for help moving a 1. “It is least likely to occur during the
client up in bed. springtime.”
62. The nurse in the emergency department is 2. “Allergic rhinitis is not likely to occur during
allergic to latex. Which intervention should the the summer.”
nurse implement regarding the use of nonsterile 3. “It is least likely to occur in the early fall.”
gloves? 4. “Allergic rhinitis is least likely to occur in
1. Use only sterile, nonlatex gloves for any early winter.”
procedure requiring gloves. 67. The client is highly allergic to insect venom and
2. Do not use gloves when starting an IV or is prescribed venom immunotherapy. Which
performing a procedure. statement is the scientific rationale for this
3. Keep a pair of nonsterile, nonlatex gloves in treatment?
the pocket of the uniform. 1. Immunotherapy is effective in preventing
4. Wear white cotton gloves at all times to anaphylaxis following a future sting.
protect the hands. 2. Immunotherapy will prevent all future insect
63. The client diagnosed with a bee sting allergy stings from harming the client.
is being discharged from the emergency 3. This therapy will cure the client from having
department. Which priority discharge any allergic reactions in the future.
instruction should be taught to the client? 4. This therapy is experimental and should not
1. Demonstrate how to use an EpiPen, an be undertaken by the client.
adrenergic agonist. 68. The client in the HCP’s office has a red, raised
2. Teach the client to never go outdoors in the rash covering the forearms, neck, and face and is
spring and summer. experiencing extreme itching which is diagnosed
3. Have the client buy diphenhydramine over as an allergic reaction to poison ivy. Which
the counter to use when stung. discharge instructions should the nurse teach?
4. Discuss wearing a Medic Alert bracelet when 1. Tell the client never to scratch the rash.
going outside. 2. Instruct the client in administering IM
64. The client comes to the emergency department Benadryl.
complaining of dyspnea and wheezing after 3. Explain how to take a steroid dose pack.
eating at a seafood restaurant. The client cannot 4. Have the client wear shirts with long sleeves
speak and has a bluish color around the mouth. and high necks.
Which intervention should the nurse implement 69. The nurse is developing a care plan for a
first? client diagnosed with allergic rhinitis. Which
1. Initiate an IV with normal saline. independent problem has priority?
2. Prepare to intubate the client. 1. Ineffective breathing pattern.
3. Administer oxygen at 100%. 2. Knowledge deficit.
4. Ask the client about an iodine allergy. 3. Anaphylaxis.
4. Ineffective coping.

13_Colgrove_Ch13.indd 521 01/06/16 5:55 PM


522 M ed -S urg S uccess

70. The nurse on a medical unit has received the 74. The client with RA has nontender, movable
morning shift report. Which client should the nodules in the subcutaneous tissue over the
nurse assess first? elbows and shoulders. Which statement is the
1. The client who has a 0730 sliding-scale scientific rationale for the nodules?
insulin order. 1. The nodules indicate a rapidly progressive
2. The client who received an initial dose of IV destruction of the affected tissue.
antibiotic at 0645. 2. The nodules are small amounts of synovial
3. The client who is having back pain at a fluid that have become crystallized.
“4” on a 1-to-10 scale. 3. The nodules are lymph nodes which have
4. The client who has dysphagia and needs to be proliferated to try to fight the disease.
fed. 4. The nodules present a favorable prognosis
and mean the client is better.
71. The nurse in the holding area of the operating
room is assessing the client prior to surgery. 75. The nurse is assessing a client diagnosed
Which information warrants immediate with RA. Which assessment findings warrant
intervention by the nurse? immediate intervention?
1. The client is able to mark the correct site for 1. The client complains of joint stiffness and the
the surgery. knees feel warm to the touch.
2. The client can only tell the nurse about the 2. The client has experienced one (1)-kg weight
surgery in lay terms. loss and is very tired.
3. The client is allergic to iodine and does not 3. The client requires a heating pad applied to
have an allergy bracelet. the hips and back to sleep.
4. The client has signed a consent form for 4. The client is crying, has a flat facial affect, and
surgery and anesthesia. refuses to speak to the nurse.
72. The client in the emergency department 76. The client diagnosed with RA who has been
begins to experience a severe anaphylactic prescribed etanercept, a tumor necrosis factor
reaction after an initial dose of IV penicillin, an alpha inhibitor, shows marked improvement.
antibiotic. Which interventions should the nurse Which instruction regarding the use of this
implement? Select all that apply. medication should the nurse teach?
1. Prepare to administer Solu-Medrol, a 1. Explain the medication loses its efficacy after
glucocorticoid, IV. a few months.
2. Request and obtain a STAT chest x-ray. 2. Continue to have checkups and laboratory
3. Initiate the rapid response team. work while taking the medication.
4. Administer epinephrine, an adrenergic 3. Have yearly magnetic resonance imaging to
blocker, SQ then IV continuous. follow the progress.
5. Assess for the client’s pulse and respirations. 4. Discuss the drug is taken for three (3) weeks
and then stopped for a week.

Rheumatoid Arthritis (RA) 77. The client diagnosed with RA has developed
swan-neck fingers. Which referral is most
73. The client diagnosed with RA is being seen appropriate for the client?
in the outpatient clinic. Which preventive 1. Physical therapy.
care should the nurse include in the regularly 2. Occupational therapy.
scheduled clinic visits? 3. Psychiatric counselor.
1. Perform joint x-rays to determine progression 4. Home health nurse.
of the disease. 78. The nurse is planning the care for a client
2. Send blood to the laboratory for an diagnosed with RA. Which intervention should
erythrocyte sedimentation rate. be implemented?
3. Recommend the flu and pneumonia vaccines. 1. Plan a strenuous exercise program.
4. Assess the client for increasing joint 2. Order a mechanical soft diet.
involvement. 3. Maintain a keep-open IV.
4. Obtain an order for a sedative.

13_Colgrove_Ch13.indd 522 01/06/16 5:55 PM


C hapter 13   I mmune S ystem D isorders 523

79. The 20-year-old female client diagnosed with 82. The nurse and a licensed practical nurse are
advanced unremitting RA is being admitted caring for clients in a rheumatologist’s office.
to receive a regimen of immunosuppressive Which task can the nurse assign to the licensed
medications. Which question should the nurse practical nurse?
ask during the admission process regarding the 1. Administer methotrexate, an antineoplastic
medications? medication, IV.
1. “Are you sexually active, and, if so, are you 2. Assess the lung sounds of a client with RA
using birth control?” who is coughing.
2. “Have you discussed taking these drugs with 3. Demonstrate how to use clothing equipped
your parents?” with Velcro fasteners.
3. “Which arm do you prefer to have an IV in 4. Discuss methods of birth control compatible
for four (4) days?” with treatment medications.
4. “Have you signed an informed consent for
83. The client with early-stage RA is being
investigational drugs?”
discharged from the outpatient clinic. Which
80. Which client problem is priority for a client discharge instruction should the nurse teach
diagnosed with RA? regarding the use of nonsteroidal anti-
1. Activity intolerance. inflammatory drugs (NSAIDs)?
2. Fluid and electrolyte imbalance. 1. Take with an over-the-counter medication for
3. Alteration in comfort. the stomach.
4. Excessive nutritional intake. 2. Drink a full glass of water with each pill.
3. If a dose is missed, double the medication at
81. The nurse is caring for clients on a medical floor.
the next dosing time.
Which client should the nurse assess first?
4. Avoid taking the NSAID on an empty stomach.
1. The client diagnosed with RA complaining of
pain at a “3” on a 1-to-10 scale. 84. The nurse is preparing to administer morning
2. The client diagnosed with SLE who has a medications. Which medication should the nurse
rash across the bridge of the nose. administer first?
3. The client diagnosed with advanced RA who 1. The pain medication to a client diagnosed
is receiving antineoplastic drugs IV. with RA.
4. The client diagnosed with scleroderma who 2. The diuretic medication to a client diagnosed
has hard, waxlike skin near the eyes. with SLE.
3. The steroid to a client diagnosed with
polymyositis.
4. The appetite stimulant to a client diagnosed
with OA.

CONCEPTS
The concepts covered in this chapter focus on immunity. Exemplars that are covered are Systemic Inflammatory
Response Syndrome (SIRS), Multi Organ Dysfunction Syndrome (MODS), and Acquired Immune Deficiency
Syndrome (AIDS). Interrelated concepts of the nursing process and critical thinking are covered throughout the
questions. The concept of critical thinking is presented in the prioritizing or “first” questions.
85. The nurse is caring for a client who has Systemic 2. Increased urine osmolality and decreased
Inflammatory Response Syndrome (SIRS) urine output.
following a major abdominal surgery. Which 3. Four plus pitting edema of the lower
signs and symptoms would the nurse observe extremities.
that indicate SIRS? Select all that apply. 4. Confusion, disorientation, delirium.
1. Bleeding times are increased and platelet 5. Heart rate 78, blood pressure 124/84, and
counts decreased. RR of 20.

13_Colgrove_Ch13.indd 523 01/06/16 5:55 PM


524 M ed -S urg S uccess

86. The client diagnosed with Multiple Organ 90. The concept of impaired immunity has been
Dysfunction Syndrome (MODS) is admitted to identified by the nurse as it applies to the client
the intensive care department. Which assessment diagnosed with Acquired Immune Deficiency
data are most important for the nurse to Syndrome (AIDS). Which interventions should
collect/monitor? the nurse implement?
1. Lung sounds, heart sounds, and blood 1. Keep fresh flowers and raw vegetables out of
pressure. the client’s room.
2. The client’s psychological response to the 2. Have the Unlicensed Assistive Personnel
illness. (UAP) assist with ADLs.
3. The client’s family’s expectations of the 3. Encourage the client to perform active range
hospitalization. of motion.
4. Amount of emesis, bile secretions, and mouth 4. Teach the client about the cardiovascular
ulcers. medications.
87. The nurse is caring for a client diagnosed with 91. The female client is homeless and pregnant. The
Systemic Inflammatory Response syndrome after client supports an IV drug habit by prostitution.
an extensive abdominal surgery. Which nursing Which data would be considered antecedents
interventions could prevent the development of (risk factor) for becoming HIV positive? Select
Multi Organ Dysfunction Syndrome (MODS)? all that apply.
1. Place the client on strict intake and output. 1. The client is pregnant.
2. Administer pain medication via patient- 2. The client is an intravenous drug abuser.
controlled analgesia. 3. The client has multiple sexual partners.
3. Keep the head of the bed elevated at all times. 4. The client does not have available health care.
4. Practice therapeutic communication. 5. The client does not have adequate bathroom
facilities.
88. The nurse is explaining Systemic Inflammatory
6. The client spends her money on nonessential
Response Syndrome (SIRS) to the client’s
items.
significant other. Which statement best
describes SIRS? 92. The client is diagnosed with Multi Organ
1. SIRS is a response of the body when it has Dysfunction Syndrome (MODS). Which is the
sustained a major burn or crushing injury in a most appropriate goal for the nurse to write
motor-vehicle accident. when planning the client’s care?
2. SIRS is a response by the body to some type 1. The client will maintain vital signs within
of injury or insult; the insult can be infectious normal limits during the next 24 hours.
or noninfectious in nature. 2. The client’s urine output will be maintained
3. SIRS only occurs when the body is to achieve output of 600 mL in the next
overwhelmed with an infectious organism 24 hours.
such as streptococcus bacteria. 3. The client will have elevated ALT, AST, and
4. SIRS occurs when the body is allergic to the GGT liver enzymes within the next 24 hours.
prescribed antibiotic and the body tries to 4. The client’s blood glucose reading will be 200
recover from the allergic response. to 240 mg/dL for the next 24 hours.
89. The client is known to be HIV positive. Which 93. The client diagnosed with Multi Organ
data indicate to the nurse that the client has Dysfunction Syndrome (MODS) has renal,
now progressed to the diagnosis of Acquired cardiovascular, and pulmonary dysfunction
Immune Deficiency Syndrome (AIDS)? issues. Which statement by the nurse indicates
1. The client’s CD4 count is 189. an understanding of the client’s prognosis?
2. The client has an Hgb of 9.4 and Hct of 29.1. 1. “As long as the client is maintained on a
3. The client’s chest x-ray show infiltrates. ventilator, then the prognosis can be up to
4. The client complains of a headache unrelieved 60% recovery.”
by Tylenol. 2. “The client will have less than a 2% potential
for recovery from the MODS.”
3. “When three or more body systems fail, the
mortality rate can be 70% to 80%.”
4. “More than one body system in failure
reduces the recovery rate to 80% to 90%.”

13_Colgrove_Ch13.indd 524 01/06/16 5:55 PM


C hapter 13   I mmune S ystem D isorders 525

94. The client diagnosed with Systemic Response 95. The nurse caring for a client diagnosed
Inflammatory Syndrome (SIRS) asks the nurse with Multi Organ Dysfunction Syndrome
what the diagnosis means. Which is the nurse’s (MODS) is preparing to administer morning
best response? medications. Which medication would the
1. SIRS is a localized response to major trauma nurse question?
that has occurred within the last three (3) 1. Cefazolin sodium IVPB every six (6) hours.
months. 2. Furosemide by mouth twice daily.
2. SIRS is a syndrome of potential responses to 3. Metoprolol IVP every four (4) hours and prn.
illness that has an optimum prognosis. 4. Acetaminophen by mouth every four (4)
3. SIRS is a respiratory response to the client hours prn.
having had a myocardial infarction or
pneumonia.
4. SIRS is a systemic response to a variety of
insults, including infection, ischemia, and
injury.
96. The nurse is preparing to administer medications to a client at 1600. Which medication would the nurse
administer first?
1. Vancomycin.
2. Furosemide.
3. Pantoprazole.
4. Acetaminophen.

Admitting Diagnosis: Acquired


Client Name: Jackson, James M/36 Immunodeficiency Syndrome

Account Number: 12 58 63 9 Med Rec #: 00 258 32 Allergies: Penicillin

Date Medication 2301–0700 0701–1500 1501–2300


Today Vancomycin 1,000 mg IVPB daily 1800
Today Furosemide 40 mg IVP twice daily 0900 1600
Today Pantoprazole oral daily 0900
Today Acetaminophen 650 mg every 4 hours
Signature of Nurse:
Day Nurse RN/DN

13_Colgrove_Ch13.indd 525 01/06/16 5:55 PM


PRACTICE QUESTIONS ANSWERS
AND RATIONALES

Multiple Sclerosis 3. These are clinical manifestations of MS


and can go undiagnosed for years because
1. 1. These are clinical manifestations of multiple of the remitting-relapsing nature of the
sclerosis and are expected. disease. Fatigue and difficulty swallowing
2. These are expected clinical manifestations of are other symptoms of MS.
multiple sclerosis. 4. Taking birth control medications should not
3. These are expected clinical manifestations of produce these symptoms or the pattern of
multiple sclerosis. occurrence.
4. Dysphagia is a common problem of clients TEST-TAKING HINT: This stem is somewhat
diagnosed with multiple sclerosis, and this involved. The test taker must be sure to un-
places the client at risk for aspiration pneu- derstand the important parts, which are the
monia. Some clients diagnosed with mul- client’s age, complaints, and occurrence of
tiple sclerosis eventually become immobile complaints. This should cause the test taker
and are at risk for pneumonia. to think about what these have in common.
TEST-TAKING HINT: This question is asking Content – Medical: Integrated Nursing Process –
the test taker to identify the assessment data ­Assessment: Client Needs – Physiological Integrity, ­Reduction
unexpected for the disease process. Respira- of Risk Potential: Cognitive Level – Analysis:
tory problems are high priority according Concept – Mobility.
to Maslow and often warrant immediate 4. 1. “Why” is requesting an explanation, and the
intervention. client does not owe the nurse an explanation.
Content – Medical: Integrated Nursing Process – 2. This is stating a fact and offering self.
­Assessment: Client Needs – Safe Effective Care Environment, Both are therapeutic techniques for
Management of Care: Cognitive Level – Synthesis: conversations.
Concept – Mobility. 3. The client did not ask about the nature of
2. 1. The exact cause of MS is not known, but MS. The client needs to be able to verbalize
there is a theory stating a slow virus is feelings.
partially responsible. A failure of a part of 4. This is “passing the buck.” Therapeutic com-
the immune system may also be at fault. munication is an integral part of nursing.
A genetic predisposition involving chro- TEST-TAKING HINT: The question is asking for
mosomes 2, 3, 7, 11, 17, 19, and X may be a therapeutic response. Therapeutic responses
involved. address feelings.
2. There is some evidence supporting a genetic Content – Medical: Integrated Nursing Process –
component involved in developing MS. Implementation: Client Needs – Psychosocial Integrity:
3. A specific gene has not been identified to know Cognitive Level – Application: Concept – Immunity.
if the gene is recessive or dominant.
4. The X chromosome, not the Y chromosome, 5. 1. This describes an evoked potential electroen-
may be involved. cephalogram (EEG).
2. MRI scans require the client to lie still and
TEST-TAKING HINT: Option “2” has the word not move the body; the client should be
“no” in it. Unless the test taker has absolute warned about the loud noise.
knowledge this is true, then an absolute word 3. The client does not drink any contrast me-
such as “no,” “never,” “all,” or “always” should dium. If contrast is used, it will be given IVP
rule out the option. for a CT scan.
Content – Medical: Integrated Nursing Process – 4. The test is performed at one time.
­Diagnosis: Client Needs – Safe Effective Care Environment,
TEST-TAKING HINT: The test taker must be
Management of Care: Cognitive Level – Analysis: Concept –
Neurological Regulation. knowledgeable about different tests and
­procedures and be able to teach about them to
3. 1. MS does not affect the menstrual cycle. the client. There are no test-taking hints to help
2. A rash across the bridge of the nose suggests remember protocols for procedures and tests.
systemic lupus erythematosus.

526

13_Colgrove_Ch13.indd 526 01/06/16 5:55 PM


C hapter 13   I mmune S ystem D isorders 527

Content – Medical: Integrated Nursing Process – 3. The LPN can draw blood.
­Planning: Client Needs – Physiological Integrity, Reduction of 4. The nurse should not assign assessing,
Risk Potential: Cognitive Level – Synthesis: teaching, or evaluation to the LPN. Evalu-
Concept – Communication. ating the client’s ability to perform self-
6. 1. The problem is grieving R/T loss of function- catheterization should not be assigned to
ing. Assistive devices will not prevent loss of the LPN.
functioning and do not address grieving. TEST-TAKING HINT: When deciding on assign-
2. A legal power of attorney is for personal prop- ing tasks, the test taker must be aware of the
erty and control of financial issues, which is capabilities of each classification of staff by
not the focus of the nurse’s care. A legal power licensure.
of attorney for health care may be appropriate. Content – Nursing Management: Integrated Nursing
3. The nurse should and must discuss end-of-life Process – Planning: Client Needs – Safe Effective Care
issues with the client and does not need to Environment, Management of Care: Cognitive Level –
contact the hospital chaplain. If or when the Synthesis: Concept – Nursing Roles.
client reveals spiritual needs, then the nurse
9. 1. The nurse should listen without being
could contact the chaplain.
judgmental about any alternative therapy
4. The client should make personal choices
the client is considering. Alternative
about end-of-life issues while it is possible
therapies, such as massage and relaxation,
to do so. This client is progressing toward
are frequently beneficial and enhance the
immobility and all the complications re-
medical regimen.
lated to it.
2. The nurse can discuss alternative therapy
TEST-TAKING HINT: This is a psychological with the client.
problem requiring a psychological answer. 3. This is not addressing the client’s concern
Option “1” is a physical intervention and of using alternative treatment.
therefore should be eliminated as a correct 4. Investigational therapies are treatments that
answer. The test taker should remember may have efficacy if proved by scientific
adjectives (“legal”) are important when an- methods. It is the health-care provider’s
swering questions. Option “3” is “passing responsibility to discuss these therapies with
the buck”; the test taker should be careful if the client.
thinking of selecting this type of option.
TEST-TAKING HINT: Two options—“3” and
Content – Medical: Integrated Nursing Process – “4”—don’t address the issue. The answer
Implementation: Client Needs – Psychosocial Integrity:
must address the client’s concern.
­Cognitive Level – Application: Concept –
Mobility. Content – Medical: Integrated Nursing Process –
Implementation: Client Needs – Safe Effective Care
7. 1. This issue is not a priority concern of a newly ­Environment, Management of Care: Cognitive Level –
diagnosed client with MS. Application: Concept – Medication.
2. This is not priority over a potential suicide
10. 1. Steroid medications increase gastric acid;
statement.
therefore, a proton pump inhibitor is an
3. A potential suicide statement is priority
­appropriate medication for the client.
for the nurse when caring for the client
2. Cultures are ordered prior to administering
with MS.
antibiotics, not steroids.
4. Flu and pneumonia vaccines are not priority.
3. Steroids interfere with glucose metabo-
TEST-TAKING HINT: When the test taker is pri- lism by blocking the action of insulin;
oritizing, a systematic approach must be used. therefore, the blood glucose levels should
Safety is priority, and a threat to a client’s life be monitored.
is priority. 4. Steroid medications cause the client to retain
Content – Medical: Integrated Nursing Process – sodium; therefore, a low-sodium diet should
­Assessment: Client Needs – Safe Effective Care Environment, be encouraged.
Management of Care: Cognitive Level – Analysis:
TEST-TAKING HINT: Steroid medications are
Concept – Communication.
some of the most common medications ad-
8. 1. The licensed practical nurse (LPN) can ministered by nurses. They are also among
administer a muscle relaxant. the most dangerous; therefore, the test taker
2. The licensed practical nurse can talk with a must know about steroids, their actions, side
health-care provider about medication the effects, and adverse effects.
LPN can give.

13_Colgrove_Ch13.indd 527 01/06/16 5:55 PM


528 M ed -S urg S uccess

Content – Medical: Integrated Nursing Process – TEST-TAKING HINT: The test taker should try
Implementation: Client Needs – Physiological Integrity, to remember at least one or two signs/symp-
Pharmacological and Parenteral Therapies: Cognitive toms of disease processes, and ascending
Level – Application: Concept – Medication. paralysis is an unusual sign/symptom specific
11. 1. This will assist the client and significant to this syndrome.
other to maintain a close relationship Content – Medical: Integrated Nursing Process – ­
without putting undue pressure on the Assessment: Client Needs – Physiological Integrity,
client. ­Reduction of Risk Potential: Cognitive Level – Analysis:
2. This is a real physical problem, not a Concept – Mobility.
psychological one. 14. 1. Visiting a foreign country is not a risk factor
3. The problem is impotence, not libido. for contracting this syndrome.
4. The problem is not psychosocial. It is a 2. This syndrome is usually preceded by a
­physical problem, and staying calm will respiratory or gastrointestinal infection
not help. one (1) to four (4) weeks prior to the
TEST-TAKING HINT: The test taker must onset of neurological deficits.
differentiate physical and psychological 3. This syndrome is not a contagious or a
problems. ­communicable disease.
Content – Medical: Integrated Nursing Process – 4. Taking herbs is not a risk factor for develop-
Implementation: Client Needs – Psychosocial Integrity: ing Guillain-Barré syndrome.
Cognitive Level – Application: Concept – Sexuality. TEST-TAKING HINT: There are some questions
12. 1. Muscle flaccidity is a hallmark symptom requiring the test taker to be knowledgeable
of MS. of the disease process. Herbs may aggravate
2. Lethargy is the state of prolonged sleepiness a disease process, but as a rule they do not
or serious drowsiness and is not associated cause disease processes, so option “4” can be
with MS. eliminated.
3. Dysmetria is the inability to control Content – Medical: Integrated Nursing Process – ­
muscular action characterized by over- Assessment: Client Needs – Physiological Integrity,
estimating or underestimating range of ­Reduction of Risk Potential: Cognitive Level – Analysis:
movement. Concept – Infection.
4. Fatigue is a symptom of MS. 15. 1. Hyporeflexia of the lower extremities is
5. Dysphagia, or difficulty swallowing, is the classic clinical manifestation of this
­associated with MS. syndrome. Therefore, assessing deep
TEST-TAKING HINT: These alternative-type ­tendon reflexes is appropriate.
questions are difficult because there are several 2. A Glasgow Coma Scale is used for clients
correct answers. The test taker gets credit only with potential neurological deficits and used
if the entire question is answered correctly. to monitor for increased intracranial pressure.
The test taker should read each answer option 3. Babinski’s reflex evaluates central nervous
carefully and rule it out as a potential correct ­system neurological status, which is not
answer before moving on to the next option. ­affected with this syndrome.
Content – Medical: Integrated Nursing Process – ­ 4. Vital signs are a part of any admission assess-
Assessment: Client Needs – Physiological Integ- ment but are not a specific assessment inter-
rity, ­Reduction of Risk Potential: Cognitive Level – vention for this syndrome.
­Comprehension: Concept – Mobility. TEST-TAKING HINT: Vital signs are general
­assessment skills and really do not help
­specifically diagnose a disease, except for blood
Guillain-Barré Syndrome pressure, which helps diagnose hyperten-
sion. The test taker should know the Glasgow
13. 1. These signs/symptoms, along with sleep
Coma Scale is for head or brain injuries.
­disturbances and nervousness, support the
diagnosis of Creutzfeldt-Jakob disease. Content – Medical: Integrated Nursing Process – ­
Assessment: Client Needs – Physiological Integrity,
2. These signs/symptoms support the diagnosis
­Reduction of Risk Potential: Cognitive Level – Analysis:
of Parkinson’s disease. Concept – Mobility.
3. These are signs/symptoms of trigeminal
neuralgia. 16. 1. The client does not need to be NPO prior to
4. Ascending paralysis is the classic symptom this procedure.
of Guillain-Barré syndrome. 2. The client should void prior to this proce-
dure to help prevent accidental puncture
of the bladder during the procedure.

13_Colgrove_Ch13.indd 528 01/06/16 5:55 PM


C hapter 13   I mmune S ystem D isorders 529

3. The lithotomy position has the client lying TEST-TAKING HINT: The adjective “long-term”
flat with the legs in stirrups, such as when should make the test taker eliminate option
Pap smears are obtained. “4” because the words “two (2) hours” are in
4. The pedal pulses should be assessed postpro- the goal. The word “perform” is an interven-
cedure, not prior to the procedure. tion, which is not a goal; therefore option “3”
TEST-TAKING HINT: The adjective “prepro- could be eliminated as the correct answer.
cedure” helps rule out option “4” as a pos- Content – Medical: Integrated Nursing Process – ­
sible correct answer; assessing pedal pulse is Planning: Client Needs – Physiological Integrity,
priority postprocedure. The test taker must ­Physiological Adaptation: Cognitive Level – Synthesis:
know terminology which describes position- Concept – Mobility.
ing, such as lithotomy, side-lying, supine, 19. 1. The ascending paralysis has reached the client’s
Trendelenburg, or prone. respiratory muscles; therefore, the client will not
Content – Medical: Integrated Nursing Process – be able to use the hands to write.
Implementation: Client Needs – Physiological Integrity, 2. The client will not be able to use the arms
Reduction of Risk Potential: Cognitive Level – Synthesis: as a result of the paralysis but can blink
Concept – Perioperative. the eyes as long as the nurse asks simple
17. 1. Safety is an important issue for the client, but “yes-or-no” questions.
this is not the priority client problem. 3. A speech therapist will not be able to help the
2. The client’s psychological needs are impor- client communicate while the client is on the
tant, but psychosocial problems are not prior- ventilator.
ity over physiological problems. 4. The ascending paralysis has reached the respi-
3. Clients with this syndrome may have choking ratory muscles; therefore, the client will not
episodes and are at risk for inability to swal- be able to use the hands to push the call light.
low as a result of the disease process, but this TEST-TAKING HINT: The test taker must real-
is not the priority nursing problem because ize all the options except option “3” are ways
weight loss is not an expected complication of to communicate with a client on the ventila-
this syndrome. tor. Options “1” and “4” both involve use of
4. Guillain-Barré syndrome has ascend- the hands, which might lead the test taker to
ing paralysis causing respiratory failure. eliminate these two options.
Therefore, breathing pattern is priority. Content – Medical: Integrated Nursing Process –
TEST-TAKING HINT: Knowledge of the disease Implementation: Client Needs – Safe Effective Care
process causes the test taker to select option ­Environment, Management of Care: Cognitive Level –
“4,” but applying Maslow’s hierarchy of needs Application: Concept – Mobility.
and choosing a client problem addressing 20. 1. Clients with this syndrome usually have a
airway is always a good option if the test full recovery, but it may take up to one
taker is not sure of the correct answer. (1) year.
Content – Medical: Integrated Nursing Process – 2. Only about 10% of clients are left with
Diagnosis: Client Needs – Safe Effective Care E
­ nvironment, ­permanent residual disability.
Management of Care: Cognitive Level – Analysis: 3. This is “passing the buck.” The nurse should
Concept – Mobility. answer the client’s question honestly, which
18. 1. This is an appropriate long-term goal for the helps establish a trusting nurse–client
client problem “impaired skin integrity.” relationship.
2. The client with Guillain-Barré syndrome 4. This indicates the nurse does not understand
will not be able to move the extremities; the typical course for a client diagnosed with
therefore, preventing muscle atrophy is an Guillain-Barré syndrome.
appropriate long-term goal. TEST-TAKING HINT: The test taker could elim-
3. The client will not be able to move the inate option “3” because this is “passing the
extremities. Therefore, the nurse will have to buck” and is usually not the best action of a
do passive range-of-motion exercises; this is nurse. The test taker needs to be knowledge-
an intervention, not a goal. able of the typical course of this syndrome to
4. This is a nursing intervention, not a goal, and be able to answer this question.
the client should be turned while sleeping Content – Medical: Integrated Nursing Process –
­unless the client is on a special immobility bed. Implementation: Client Needs – Psychosocial Integrity:
Cognitive Level – Application: Concept – Communication.

13_Colgrove_Ch13.indd 529 01/06/16 5:55 PM


530 M ed -S urg S uccess

21. 1. Very little cerebrospinal fluid is removed 23. 1. This action does not address the wife’s fears,
from the client. Therefore, hypotension is not and telling her to stop crying will not help
a potential complication of this procedure. the situation.
2. A bandage is placed over the puncture site, 2. Making the wife leave the room will further
and pressure does not need to be applied to upset the client and the client’s wife.
the site. 3. Medicating the client will not help the wife,
3. The laboratory staff, not the nurse, complete but if the nurse can calm the wife, then it is
tests on the cerebrospinal fluid; the nurse hoped the client will calm down.
could label the specimens and take them to 4. It is scary for a wife to see her loved one
the laboratory. with a tube down his mouth and all the
4. Increased fluid intake will help prevent a machines around him. The nurse should
postprocedure headache, which may occur help the wife by acknowledging her fears.
after a lumbar puncture. TEST-TAKING HINT: The test taker should se-
TEST-TAKING HINT: The test taker could lect the option addressing the wife’s needs
eliminate option “3” because nurses usually first. By addressing the wife’s needs, the cli-
do not perform tests on bodily fluids at the ent will calm down. The test taker should not
bedside. A basic concept in many procedures automatically select the option which medi-
is, if fluid is removed, it usually must be re- cates the client.
placed, which might cause the test taker to Content – Medical: Integrated Nursing Process –
select option “4.” Implementation: Client Needs – Safe Effective Care
Content – Surgical: Integrated Nursing Process – ­Environment, Management of Care: Cognitive Level –
Implementation: Client Needs – Safe Effective Care Application: Concept – Communication.
­Environment, Management of Care: Cognitive Level –
24. 1. The physical therapist is an important
Application: Concept – Perioperative.
part of the rehabilitation team who ad-
22. 1. The rate of ventilation is usually 12 to dresses the client’s muscle deterioration
15 breaths per minute in adults who are on resulting from the disease process and
ventilators, so this rate does not require immobility.
immediate intervention. 2. There is no residual speech deficit from
2. A manual resuscitation (Ambu) bag must be ­Guillain-Barré syndrome; therefore, this
at the client’s bedside in case the ventilator referral is not appropriate.
malfunctions; the nurse must bag the client. 3. The social worker could help with finan-
3. A pulse oximeter reading of less than 93% cial concerns, job issues, and issues con-
warrants immediate intervention; a 90% cerning the long rehabilitation time for
peripheral oxygen saturation indicates this syndrome.
a Pao2 of about 60 (normal, 80 to 100). 4. Pain may or may not be an issue with this
When the client is placed on the ventila- syndrome. Each client is different, but a
tor, this should cause the client’s oxygen plan needs to be established to address
level to improve. pain if it occurs.
4. These ABGs are within normal limits and do 5. This is an excellent resource for the client
not warrant immediate intervention. and the family.
TEST-TAKING HINT: The test taker must know TEST-TAKING HINT: The physical therapist and
specific norms for frequently performed tests social worker are two members of the reha-
for the client. Even if the test taker were not bilitation team who are always appropriate
knowledgeable of the ventilator-based deci- in long-term rehabilitation. Physical therapy
sions based on norms, the test taker could addresses complications of immobility; so-
ask, “Is a client with respiratory rate of 14 in cial workers help the client get back home.
respiratory failure or compromise?” Equip- Any resource referral is an appropriate
ment at the bedside probably does not war- intervention.
rant immediate intervention. Content – Medical: Integrated Nursing Process –
Content – Medical: Integrated Nursing Process – Implementation: Client Needs – Safe Effective Care
Assessment: Client Needs – Physiological Integrity, ­Environment, Management of Care: Cognitive Level –
­Reduction of Risk Potential: Cognitive Level – Synthesis: Application: Concept – Mobility.
Concept – Mobility.

13_Colgrove_Ch13.indd 530 01/06/16 5:55 PM


C hapter 13   I mmune S ystem D isorders 531

Myasthenia Gravis TEST-TAKING HINT: This is a knowledge-based


question, but the test taker may be able to
25. 1. These are musculoskeletal manifestations of eliminate options “2” and “4” if the test taker
myasthenia gravis. has a basic understanding of anatomy and
2. These are ocular signs/symptoms of MG. physiology and knows surgery involving the
Ptosis is drooping of the eyelid, and diplo- pituitary or adrenal glands does not help
pia is unilateral or bilateral blurred vision. prevent signs and symptoms of a muscular
3. These are respiratory manifestations of disorder.
myasthenia gravis. Content – Medical: Integrated Nursing Process –
4. These are nutritional manifestations of ­Planning: Client Needs – Safe Effective Care Environment,
myasthenia gravis. Management of Care: Cognitive Level – Synthesis:
Concept – Perioperative.
TEST-TAKING HINT: The keys to answering
this question are the adjectives “ocular” 28. 1. The client is at risk for choking; know-
and “facial.” This information should make ing specific measures to help the client
the test taker rule out options “1,” “3,” and helps decrease the client’s, as well as
“4” even if the test taker doesn’t know what the significant other’s, anxiety and pro-
“ptosis” or “diplopia” means. motes confidence in managing potential
Content – Medical: Integrated Nursing Process – complications.
Assessment: Client Needs – Physiological Integrity, 2. Clients diagnosed with MG may end up on a
­Reduction of Risk Potential: Cognitive Level – Analysis: ventilator at the end stage of the disease, but
Concept – Immunity. these clients would not be cared for at home;
26. 1. No change in the client’s muscles strength this would be a very unusual situation.
indicates it is not MG. 3. The client should be encouraged to perform
2. There is reduced amplitude in an electro- active range-of-motion exercises, but the
myogram (EMG) in a client with MG. most important intervention is treating chok-
3. The serum assay of circulating acetylcholine ing episodes.
receptor antibodies is increased, not de- 4. The client with MG doesn’t necessarily have
creased, in MG, and this test is only 80% to a feeding tube, and this information is not in
90% accurate in diagnosing MG. the stem.
4. Clients with MG show a significant TEST-TAKING HINT: The test taker should only
­improvement of muscle strength lasting consider the information in the stem of the
approximately five (5) minutes when Ten- question, which causes the test taker to
silon (edrophonium chloride) is injected. eliminate option “4.” If the test taker could
TEST-TAKING HINT: There are some knowl-
not decide between options “1” and “3,” the
edge-based questions, such as diagnostic test taker should apply Maslow’s hierarchy of
tests. needs; airway is priority.
Content – Medical: Integrated Nursing Process – ­
Content – Medical: Integrated Nursing Process –
Planning: Client Needs – Physiological Integrity,
­Diagnosis: Client Needs – Physiological Integrity, Reduction
­Physiological Adaptation: Cognitive Level – Synthesis:
of Risk Potential: Cognitive Level – Knowledge:
Concept – Mobility.
Concept – Mobility.

27. 1. There is a surgical option available. 29. 1. The occupational therapist assists the client
2. This surgery is performed in clients with with ADLs, but with MG the client has no
­pituitary tumors and is accomplished by problems with performing them if the client
going through the client’s upper lip though takes the medication correctly (30 minutes
the nasal passage. prior to performing ADLs).
3. In about 75% of clients with MG, the 2. A recreational therapist is usually in a
­thymus gland (which is usually inac- psychiatric unit or rehabilitation unit.
tive after puberty) continues to produce 3. A vocational therapist or counselor helps with
antibodies, triggering an autoimmune the client finding a job which accommodates
response in MG. After a thymectomy, the the disease process; clients with MG are
production of autoantibodies is reduced usually not able to work in the late stages.
or eliminated, and this may resolve the 4. Speech therapists address swallow-
signs/symptoms of MG. ing problems, and clients with MG are
4. An adrenalectomy is the surgery for a client ­dysphagic and at risk for aspiration.
diagnosed with Cushing’s disease, a disease in The speech therapist can help match
which there is an increased secretion of glu- food ­consistency to the client’s ability to
cocorticoids and mineralocorticoids. ­swallow, which enhances client safety.

13_Colgrove_Ch13.indd 531 01/06/16 5:55 PM


532 M ed -S urg S uccess

TEST-TAKING HINT: The test taker must be 32. 1. This is a diagnostic test done to diagnose MG.
aware of the responsibilities of the other 2. These assessment data indicate the client is
health-care team members. “Collaborative” experiencing a myasthenic crisis, which is
means working with another health-care the result of undermedication, missed doses
team discipline. of medication, or the development of an
Content – Medical: Integrated Nursing Process – infection.
­Planning: Client Needs – Safe Effective Care Environment, 3. The vital signs do not indicate if the client is
Management of Care: Cognitive Level – Synthesis: experiencing a cholinergic crisis.
Concept – Mobility. 4. The injection of edrophonium chloride
30. 1. Hypovolemia is a complication of plasma- (Tensilon test) not only diagnoses MG but
pheresis, especially during the procedure, helps to determine which type of crisis the
when up to 15% of the blood volume is in client is experiencing. In a myasthenic cri-
the cell separator. sis, the test is positive (the client’s muscle
2. Positive Chvostek’s and Trousseau’s signs (not strength improves), but in cholinergic
negative signs) warrant intervention and indi- crisis, the test is negative (there is no im-
cate hypocalcemia, which is a complication of provement in muscle strength), or the cli-
plasmapheresis. ent will actually get worse and emergency
3. This is a normal serum potassium level equipment must be available.
(3.5 to 5.5 mEq/L), which does not warrant TEST-TAKING HINT: This question requires
intervention, but the level should be moni- the test taker to be knowledgeable of the
tored because plasmapheresis could cause disease process, but this is an important con-
hypokalemia. cept the test taker must understand about
4. Ecchymosis (bruising) does not warrant myasthenia gravis.
immediate intervention. Signs of infiltration Content – Medical: Integrated Nursing Process – ­
or infection warrant immediate intervention. Assessment: Client Needs – Physiological Integrity,
­Reduction of Risk Potential: Cognitive Level – Analysis:
TEST-TAKING HINT: If the test taker has no
Concept – Mobility.
idea of the answer, then selecting signs of hy-
povolemia—hypotension and tachycardia—is 33. 1. Position changes promote lung expansion,
an appropriate selection if the question asks and coughing helps clear secretions from
which data warrant immediate intervention. the tracheobronchial tree.
Content – Medical: Integrated Nursing Process – 2. This position expands the lungs and alle-
Assessment: Client Needs – Physiological Integrity, viates pressure from the diaphragm.
­Reduction of Risk Potential: Cognitive Level – Analysis: 3. The respiratory system and pulse oximeter
Concept – Immunity. reading should be assessed more frequently
31. 1. MG has no effect on the ovarian function than every shift; it should be done every
and the uterus is an involuntary muscle, four (4) hours or more often.
not a skeletal muscle, so the menstrual 4. The medications should be administered
cycle is not affected. 30 minutes before the meal to provide
2. Infections can result in an exacerbation and ­optimal muscle strength for swallowing
extreme weakness. and chewing.
3. An extremely hot or cold environment may 5. There is no serum level available for medi-
cause an exacerbation of MG. cations used to treat MG; the client’s signs/
4. This will help the client mobilize and expec- symptoms are used to determine the effec-
torate sputum. tiveness of this medication.
TEST-TAKING HINT: An alternative-type ques-
TEST-TAKING HINT: This question is an
“­except” question and is asking the test taker tion requests the test taker to select more
to select the option which is not appropri- than one option as the correct answer. The
ate for the client’s disease process. Three (3) test taker must evaluate each option indi-
answers will be appropriate; sometimes if vidually to determine if it is correct. The
the test taker rethinks the question and asks, priority concerns for a client with MG are
“Which statements indicate the client under- respiration and eating.
stands the teaching?” this will help identify Content – Medical: Integrated Nursing Process –
the correct answer. Implementation: Client Needs – Safe Effective Care
­Environment, Management of Care: Cognitive Level –
Content – Medical: Integrated Nursing Process – ­ Analysis: Concept – Mobility.
Evaluation: Client Needs – Physiological Integrity,
­Physiological Adaptation: Cognitive Level – Synthesis:
Concept – Mobility.

13_Colgrove_Ch13.indd 532 01/06/16 5:55 PM


C hapter 13   I mmune S ystem D isorders 533

34. 1. This is an appropriate action by the nurse, Content – Medical: Integrated Nursing Process –
but it is not the best action. Implementation: Client Needs – Physiological Integrity,
2. Support groups are helpful to the client’s sig- Pharmacological and Parenteral Therapies: Cognitive
nificant others, but in this situation, it is not Level – Application: Concept – Medication.
the best action for the nurse.
3. A counselor is an appropriate intervention,
Systemic Lupus Erythematosus
but it is not the best action.
4. Directly addressing the wife’s feelings is
37. 1. SLE can affect any organ system, and these
the best action for the nurse in this situ-
tests are used to determine the possibility of
ation. All the other options can be done,
the liver being involved, but they are not used
but the best action is to address the wife’s
to diagnose SLE.
feelings.
2. No single laboratory test diagnoses SLE,
TEST-TAKING HINT: The test taker should but the client usually presents with mod-
select the option directly addressing and erate to severe anemia, thrombocytopenia,
helping the client or significant other. leukopenia, and a positive antinuclear
­Remember, if the word “best,” “most impor- antibody.
tant,” or “first” is in the stem, then all four 3. Female clients with SLE develop atheroscle-
(4) options could be possible interventions, rosis at an earlier age, but cholesterol and lipid
but only one is the highest priority. profile tests are not used to diagnose the disease.
Content – Medical: Integrated Nursing Process – 4. These tests may be done to determine SLE
Implementation: Client Needs – Psychosocial Integrity: infiltration in the kidneys but not to diagnose
Cognitive Level – Application: Concept – Communication. the disease itself.
35. 1. This medication promotes muscle con- TEST-TAKING HINT: A complete metabolic
traction, which improves muscle strength, panel is ordered for many different diseases;
which, in turn, allows the client to per- cholesterol and lipid panels are usually or-
form ADLs without assistance. dered for atherosclerosis, and BUN and
2. This medication does not affect secretions of glomerular filtration tests are specific to the
the eye. kidneys. Options “1,” “3,” and “4” could be
3. This medication does not help with the ruled out because they are specific to other
digestion of food. diseases or not specific enough.
4. This medication does not help with pain; Content – Medical: Integrated Nursing Process –
­clients with MG do not have muscle pain. ­Planning: Client Needs – Physiological Integrity, Reduction
TEST-TAKING HINT: The test taker must of Risk Potential: Cognitive Level – Synthesis:
Concept – Immunity.
know about the disease process to be able to
answer this question. Remember, when an- 38. 1. Sunlight or UV light exposure has been
swering pharmacology questions, the effec- shown to initiate an exacerbation of SLE,
tiveness of the medication is based on what so the client should be taught to protect
sign/symptom the client is experiencing. the skin when in the sun.
Content – Medical: Integrated Nursing Process – 2. A fever may be the first indication of an
Assessment: Client Needs – Physiological Integrity, exacerbation of SLE.
­Pharmacological and Parenteral Therapies: Cognitive 3. Dyspnea is not expected and could signal
Level – Analysis: Concept – Medication. ­respiratory involvement.
36. 1. This medication will increase muscle 4. Raynaud’s phenomenon is a condition in
strength to help enhance swallowing and which the digits of the hands and feet turn
chewing during meals. red, blue, or white in response to heat or
2. There is no need for the client to take this cold and stress. It occurs with some im-
medication with eight (8) ounces of water. mune inflammatory processes.
3. The client does not have to sit up after taking 5. SLE is a chronic disease and there is no
this medication. known cure.
4. These assessment data would not cause TEST-TAKING HINT: Dyspnea is an uncomfort-
the nurse to question administering this able sensation of not being able to breathe.
medication. Usually clients are not told this is normal
TEST-TAKING HINT: There are very few regardless of the disease process.
­medications given specifically on time, but Content – Medical: Integrated Nursing Process – ­
this medication is one of them. The blood Planning: Client Needs – Physiological Integrity,
pressure is checked prior to administering ­Physiological Adaptation: Cognitive Level – Synthesis:
Concept – Immunity.
antihypertensive medications.

13_Colgrove_Ch13.indd 533 01/06/16 5:55 PM


534 M ed -S urg S uccess

39. 1. SLE is frequently diagnosed in young women Content – Medical: Integrated Nursing Process – ­
and reproduction is a concern for these cli- Diagnosis: Client Needs – Physiological Integrity,
ents, but it is not the most important goal. ­Pharmacological and Parenteral Therapies: Cognitive
2. The client’s body image is important, but this Level – Analysis: Concept – Medication.
is not the most important. 42. 1. Unless the nurse has SLE and has been
3. SLE can invade and destroy any body sys- through the exact same type of tissue involve-
tem or organ. Maintaining organ function ment, then the nurse should not tell a client
is the primary goal of SLE treatment. “I know.” This does not address the client’s
4. Measures are taken to prevent breakdown, feelings.
but skin breakdown is not life threatening. 2. The nurse should never ask the client “why.”
TEST-TAKING HINT: When the question The client does not owe the nurse an expla-
asks for “priority,” the test taker should nation of his or her feelings.
­determine if one of the options has life- 3. Support groups should be recommended, but
threatening information or could result in a this is not the best response when the client is
serious complication for the client. crying.
Content – Medical: Integrated Nursing Process – 4. The nurse stated a fact, “You are crying,”
­Planning: Client Needs – Safe Effective Care Environment, and then offered self by saying “Would
Management of Care: Cognitive Level – Synthesis: you like to talk?” This addresses the non-
Concept – Immunity. verbal cue, crying, and is a therapeutic
response.
40. 1. SLE can affect any organ. It can cause
pericarditis and myocardial ischemia as TEST-TAKING HINT: The question asks for a
well as pneumonia or pleural effusions. therapeutic response, which means a feeling
2. Muscle spasticity occurs in MS, and brady- must be addressed. Therapeutic responses do
kinesia occurs in Parkinson’s disease. not ask “why,” so the test taker could rule
3. Hirsutism is an overgrowth of hair. Spotty out option “2.”
areas of alopecia occur in SLE, and clubbing Content – Medical: Integrated Nursing Process –
of the fingers occurs in chronic pulmonary or Implementation: Client Needs – Psychosocial Integrity:
cardiac diseases. Cognitive Level – Application: Concept – Communication.
4. Weight loss and fatigue are experienced by 43. 1. Moisturizing lotions, not astringents, are ap-
clients diagnosed with SLE. plied. Astringent lotions have an alcohol base,
TEST-TAKING HINT: The test taker must know which is drying to the client’s skin.
the signs and symptoms of disease processes. 2. The skin should be inspected daily for any
Content – Medical: Integrated Nursing Process – ­ breakdown or rashes.
Assessment: Client Needs – Physiological Integrity, 3. The skin should be washed with mild
­Reduction of Risk Potential: Cognitive Level – Analysis: soap, rinsed, and patted dry. Rubbing can
Concept – Immunity. cause abrasions and skin breakdown.
41. 1. Steroid medications mask the development 4. The stem does not tell the test taker the
of infections because steroids suppress the ­client is itching, and SLE does not have itch-
immune system’s response. ing as a symptom. Lotions are not usually
2. SLE does not metastasize, or “spread”; it does ­applied between the toes because this fosters
invade other organ systems, but steroids do the development of a fungal infection be-
not prevent this from happening. tween the toes.
3. The main function of steroid medications TEST-TAKING HINT: If the test taker did not
is to suppress the inflammatory response know what “astringent” meant, then the test
of the body. taker should skip this option and continue
4. Steroid medications can delay the healing looking for a correct answer. In option “2,”
process, theoretically making scarring worse. the time frame of weekly makes this option
TEST-TAKING HINT: Steroids are a frequently wrong.
administered medication class. The test taker Content – Medical: Integrated Nursing Process –
must know the common actions, side effects, Implementation: Client Needs – Physiological Integrity,
adverse effects, and how to administer the Basic Care and Comfort: Cognitive Level – Application:
Concept – Immunity.
medications safely.

13_Colgrove_Ch13.indd 534 01/06/16 5:55 PM


C hapter 13   I mmune S ystem D isorders 535

44. 1. Chest pain should be considered a prior- 4. There is evidence for familial and
ity regardless of the admitting diagnosis. ­hormonal components to the develop-
Clients diagnosed with SLE can develop ment SLE. SLE is an autoimmune disease
cardiac complications. process in which there is an exaggerated
2. Pain at a “10” is a priority but not above chest production of autoantibodies.
pain. TEST-TAKING HINT: The test taker could
3. Dysphagia is expected in clients diagnosed ­eliminate options “1” and “2” by referring to
with MG. basic anatomy and physiology and the func-
4. Clients diagnosed with GB syndrome have tion of the kidneys and endocrine system.
ascending muscle weakness or paralysis, Content – Medical: Integrated Nursing Process – ­
which could eventually result in the client Implementation: Client Needs – Safe Effective Care
being placed on a ventilator, but the problem ­Environment, Management of Care: Cognitive Level –
currently is in the distal extremities (the feet) Analysis: Concept – Immunity.
and is not priority over chest pain.
47. 1. Steroids are not addicting.
TEST-TAKING HINT: When the test taker
2. The adrenal gland, not the thyroid gland,
is deciding on which client has priority, a produces the glucocorticoid cortisol.
­potentially life-threatening condition is 3. Tapering steroids is important because
­always top priority. the adrenal gland stops producing corti-
Content – Medical: Integrated Nursing Process – ­ sol, a glucocorticosteroid, when the exog-
Assessment: Client Needs – Safe Effective Care enous administration of steroids exceeds
­Environment, Management of Care: Cognitive Level –
what normally is produced. The functions
Analysis: Concept – Immunity.
of cortisol in the body are to regulate
45. 1. The UAP should wash the hands before and glucose metabolism and maintain blood
after client care. pressure.
2. The UAP can remove an indwelling cath- 4. Tapering the dose is standard medical
eter with nonsterile gloves. This is a waste ­practice, not a whim of the HCP.
of expensive equipment. The nurse is re- TEST-TAKING HINT: Basic knowledge of
sponsible for teaching UAPs appropriate ­anatomy and physiology eliminates option
use of equipment and supplies and cost “2.” Tapering steroid medication is basic
containment. knowledge for the nurse administering a
3. Raising the head of the bed to a 90-degree steroid.
angle (high Fowler’s position) during meals Content – Medical: Integrated Nursing Process – ­
helps to prevent aspiration. Diagnosis: Client Needs – Physiological Integrity,
4. Using a clean plastic bag to access the ice ­Pharmacological and Parenteral Therapies: Cognitive
­machine indicates the assistant is aware of Level – Analysis: Concept – Medication.
­infection control procedures.
48. 1. Nodules and bony deformity are symptoms of
TEST-TAKING HINT: This is really an “except”
RA but not of SLE.
question—there will be three (3) options 2. Organ involvement occurs in SLE but not RA.
with desire actions and only one (1) needs to 3. Joint stiffness and pain are symptoms
change. ­occurring in both diseases.
Content – Medical: Integrated Nursing Process – ­ 4. Raynaud’s phenomenon and skin rashes are
Evaluation: Client Needs – Safe Effective Care associated with SLE.
­Environment, Management of Care: Cognitive Level –
Synthesis: Concept – Nursing Roles. TEST-TAKING HINT: There are a number of ill-
nesses sharing the same symptoms. The test
46. 1. The kidneys filter wastes, not antibodies, taker must be aware of the symptoms that
from the blood. distinguish one illness from another.
2. The problem is an overactive immune system, Content – Medical: Integrated Nursing Process –
not damage to the endocrine system. There is Assessment: Client Needs – Physiological Integrity,
no research supporting a virus as an initiating ­Reduction of Risk Potential: Cognitive Level – Analysis:
factor. Concept – Immunity.
3. SLE is an autoimmune disease character-
ized by exacerbations and remission. There is
empirical evidence indicating hormones may
cause the development of the disease, and
some drugs can initiate the process.

13_Colgrove_Ch13.indd 535 01/06/16 5:55 PM


536 M ed -S urg S uccess

Acquired Immunodeficiency Syndrome 51. 1. This client probably has oral candidiasis, a
fungal infection of the mouth and esophagus.
49. 1. Birth control pills provide protection against Brushing the teeth and patchy areas will not
unwanted pregnancy but they do not protect remove the lesions and will cause consider-
females from getting sexually transmitted dis- able pain.
eases. In fact, because of the reduced chance 2. This most likely is a fungal infection
of becoming pregnant, some women may find known as oral candidiasis, commonly
it easier to become involved with multiple called thrush. An antifungal medication is
partners, increasing the chance of contracting needed to treat this condition.
a sexually transmitted disease. 3. Antiseptic-based mouthwashes usually con-
2. There is no vaccine or cure for the HIV virus. tain alcohol, which is painful, for the client.
3. Adolescents are among the fastest-growing 4. The foods the client has eaten did not cause
population to be newly diagnosed with HIV this condition.
and AIDS. TEST-TAKING HINT: The client is complaining
4. Abstinence is the only guarantee the c­ lient of a “sore mouth.” The test taker must notice
will not contract a sexually transmitted all the important information in the stem
disease, including AIDS. An individual before attempting to choose an answer. How
who is HIV negative in a monogamous are brushing the area, an antiseptic mouth-
relationship with another individual who wash, or the foods eaten going to alleviate
is HIV negative and committed to a mo- the pain?
nogamous relationship is the safest sexual Content – Medical: Integrated Nursing Process –
relationship. Implementation: Client Needs – Physiological Integrity,
TEST-TAKING HINT: Answer option “1” is a Physiological Adaptation: Cognitive Level – Analysis:
form of an absolute, which could cause the Concept – Immunity.
test taker to eliminate this option. Option “4” 52. 1. Contact Precautions are a form of
is also an absolute—“only”—but it is a true ­transmission-based precautions used when
statement. There are some absolutes in the the infectious organism is known to be spread
health-care profession. by contact with a substance.
Content – Medical: Integrated Nursing Process – ­ 2. Airborne Precautions are used for bacteria,
Planning: Client Needs – Health Promotion and which are very small molecules carried at
­Maintenance: Cognitive Level – Synthesis: Concept – some distance from the client on air currents.
Nursing Roles. The bacterium which causes tuberculosis is
50. 1. The client has a malnutrition syndrome. an example of such bacteria. A special isola-
The nurse assesses the body and what the tion mask is required to enter the ­client’s
client has been able to eat. negative air pressure room.
2. Standard Precautions are used for clients 3. Droplet Precautions are used for organ-
diagnosed with AIDS, the same as for every isms causing flu or some pneumonias. The
other client. ­organisms have a larger molecule and “drop”
3. The nurse should check the orders but not within three (3) to four (4) feet. A normal
before assessing the client. ­isolation mask is used with this client.
4. The client will probably be placed on to- 4. Standard Precautions are used for all
tal parenteral nutrition and will need to be ­contact with blood and body secretions.
taught these things, but this is not the first TEST-TAKING HINT: Isolation procedures
action. are basic nursing knowledge, and the test
TEST-TAKING HINT: Assessment is the first taker must know, understand, and be able to
step in the nursing process. The nursing ­comply with all of the procedures.
process is a good place to start when setting Content – Fundamentals: Integrated Nursing Process –
priorities for the nurse’s actions. Diagnosis: Client Needs – Safe Effective Care Environment,
Content – Medical: Integrated Nursing Process – Safety and Infection Control: Cognitive Level – Analysis:
Implementation: Client Needs – Physiological Integrity, Concept – Safety.
­Basic Care and Comfort: Cognitive Level – Analysis:
Concept – Nutrition.

13_Colgrove_Ch13.indd 536 01/06/16 5:55 PM


C hapter 13   I mmune S ystem D isorders 537

53. 1. Retroviruses never die; the virus may 4. This is done at three (3) months and six (6)
­become dormant, only to be reactivated months after initial exposure.
at a later time. TEST-TAKING HINT: In questions asking the
2. “Eradicated” means to be completely cured or test taker to select the first action, all the
done away with. HIV cannot be eradicated. options could be appropriate interventions,
3. The HIV virus originated in the green but the test taker must decide which has the
monkey, in which it is not deadly. HIV in most immediate need and the most benefit.
­humans replicates readily using the CD4 cells Directly caring for the wound is of the most
as reservoirs. benefit.
4. The HIV virus uses the CD4 cells of the Content – Medical: Integrated Nursing Process – ­
immune system as reservoirs to replicate itself. Implementation: Client Needs – Safe Effective Care
TEST-TAKING HINT: If the test taker is not ­Environment, Safety and Infection Control: Cognitive
aware of the definition of a word, the indi- Level – Analysis: Concept – Safety.
vidual monitoring the test may be able to 56. 1. Altered nutrition may be a priority for
define the word, but this is not possible on a client with malnutrition, but HIV
the NCLEX-RN examination. Of the answer encephalopathy is a cognitive deficit.
options, option “1” has the most important 2. The client might grieve if the client still
information regarding prognosis and poten- has enough cognitive ability to understand
tial spread to noninfected individuals. the loss is occurring, but this is not the most
Content – Medical: Integrated Nursing Process – important consideration.
­Planning: Client Needs – Safe Effective Care Environment, 3. A client diagnosed with encephalopathy may
Safety and Infection Control: Cognitive Level – Synthesis:
not have the ability to understand instruc-
Concept – Immunity.
tions. The nurse should teach the significant
54. 1. The client may be in the primary infection others.
stage when the body has not had time to 4. Safety is always an issue with a client with
develop antibodies to the HIV virus. diminished mental capacity.
2. Repeated exposure to HIV increases the risk TEST-TAKING HINT: The test taker must have a
of infection, but it only takes one exposure to basis for deciding priority. Maslow’s hierar-
develop an infection. chy of needs lists safety as a high priority.
3. The primary phase of infection ranges Content – Medical: Integrated Nursing Process – ­
from being asymptomatic to severe Diagnosis: Client Needs – Safe Effective Care Environment,
flu-like symptoms, but during this time, Safety and Infection Control: Cognitive Level – Analysis:
the test may be negative although the Concept – Safety.
­individual is infected with HIV.
4. The client may or may not have a different 57. 1. Serum blood work, although ordered STAT,
virus, but this is not the reason the test is does not have priority over oxygenation of
negative. the client.
2. Oxygen is a priority, especially with a
TEST-TAKING HINT: Answer options “1” and
­client diagnosed with a respiratory
“4” assume the client is negative for the illness.
HIV virus. Therefore, these options should 3. It is extremely important to initiate IV anti-
be eliminated as correct answers unless the biotic therapy to a client diagnosed with an
test taker is completely sure the statement is infection as quickly as possible, but this does
correct. not have priority over oxygen.
Content – Medical: Integrated Nursing Process – 4. Culture specimens should be obtained prior
­Diagnosis: Client Needs – Physiological Integrity, Reduction to initiating antibiotic therapy, but oxygen
of Risk Potential: Cognitive Level – Analysis:
administration is still the first action.
Concept – Infection.
TEST-TAKING HINT: Airway, breathing, and
55. 1. The nurse should attempt to flush the providing oxygen to the tissues is the top pri-
skin and get the area to bleed. It is hoped ority in any nursing situation. If the cells are
this will remove contaminated blood from not oxygenated, they die.
the body prior to infecting the nurse. Content – Medical: Integrated Nursing Process – ­
2. The nurse should notify the charge nurse after Implementation: Client Needs – Safe Effective Care
flushing the area and trying to get it to bleed. ­Environment, Management of Care: Cognitive Level –
3. This should be done within four (4) hours Application: Concept – Infection.
of the exposure, not before trying to rid the
body of the potential infection.

13_Colgrove_Ch13.indd 537 01/06/16 5:55 PM


538 M ed -S urg S uccess

58. 1. Performing the head-to-toe assessment is a TEST-TAKING HINT: The test taker is being
nursing consideration, not a client consider- asked for the most appropriate method.
ation. This is a physiological intervention, not ­Option “4” can be discarded because of the
a psychosocial one. word “tell.” Option “3” gives the option for
2. Maintaining body weight is physical. multiple individuals to work together toward
3. Clients diagnosed with AIDS should be an outcome.
encouraged to discuss their end-of-life issue Content – Medical: Integrated Nursing Process –
with the significant others and to put those Implementation: Client Needs – Safe Effective Care
wishes in writing. This is important for all ­Environment, Management of Care: Cognitive Level –
clients, not just those diagnosed with AIDS. Application: Concept – Communication.
4. Activity tolerance is a physical problem.
TEST-TAKING HINT: All of the options except
Allergies and Allergic Reactions
one (1) focus on the physical care of the
­client. The stem asked the test taker to con- 61. 1. This is appropriate anytime the nurse is
sider a psychosocial need. administering a diuretic medication.
Content – Medical: Integrated Nursing Process – 2. A nurse uses nonsterile gloves to remove old
Implementation: Client Needs – Psychosocial Integrity: dressings, then washes the hands and sets up
Cognitive Level – Application: Concept – Grief and Loss.
the sterile field before donning sterile gloves
59. 1. Flushed warm skin with tented turgor in- to reapply the dressing.
dicates dehydration. The HCP should be 3. Checking for allergies is one (1) of the
notified immediately for fluid orders or five (5) rights of medication. Is it the right
other orders to correct the reason for the drug? Even if the drug is the one the HCP
dehydration. ordered, it is not the right drug if the
2. This is a concern but it can be taken care of client is allergic to it. The nurse should
after the client with the physical problem. always assess a client’s allergies prior to
3. The temperature is slightly elevated and the administering any medication.
pulse is one (1) beat higher than normal. This 4. The nurse should ask for assistance in moving
client could wait to be seen. a client in bed to prevent on-the-job
4. Many clients who have had sputum specimens injuries.
ordered are unable to produce sputum, but it TEST-TAKING HINT: The stem asks the test
does not warrant immediate intervention. taker to determine which is an incorrect ac-
TEST-TAKING HINT: This is an “except” tion. This is an “except” question. Three (3)
­ uestion asking the test taker to identify
q answers are actions the nurse should take.
abnormal data indicating a life-threatening Content – Medical: Integrated Nursing Process – ­
situation or a complication. Evaluation: Client Needs – Safe Effective Care
Content – Medical: Integrated Nursing Process – ­ ­Environment, Management of Care: Cognitive Level –
Assessment: Client Needs – Safe Effective Care Synthesis: Concept – Immunity.
­Environment, Management of Care: Cognitive Level – 62. 1. The nurse should use nonlatex gloves
Analysis: Concept – Immunity.
because of the latex allergy, but the gloves do
60. 1. This does not provide continuity of care for not have to be sterile.
the client. It does recognize the nurse’s posi- 2. The nurse must use gloves during procedures
tion, but it is not the best care for the client. and starting an IV. Not using gloves is a
2. The HCP should be asked to attend the care ­violation of Occupational Safety and Health
plan meeting to assist in deciding how to Administration standards and places the nurse
work with the client, but asking the HCP to at risk for developing illnesses.
“tell” the client to behave is not the best way 3. The nurse should be prepared to care for
to handle the situation. The client can always a client at all times and should not place
refuse to behave as requested. himself or herself at risk because the facil-
3. The health-care team should meet to dis- ity does not keep nonlatex gloves available
cuss ways to best help the client deal with in the rooms. The nurse should carry the
the anger being expressed, and the staff needed equipment (nonlatex gloves) with
should be consistent in working with the him or her.
client. 4. White cotton gloves are made of cloth
4. Telling a staff member to care for the client and do not provide the barrier against wet
for a week could result in a buildup of ani- substances.
mosity and make the situation worse.

13_Colgrove_Ch13.indd 538 01/06/16 5:55 PM


C hapter 13   I mmune S ystem D isorders 539

TEST-TAKING HINT: The test taker must be 65. 1. The symptoms are occurring at this time, so
aware of adjectives such as “sterile” in option asking what time of the year the symptoms
“1.” Basic concepts such as Standard Precau- occur is not an appropriate question.
tions should cause the test taker to eliminate 2. There are many over-the-counter reme-
option “2.” Option “4” has the word “all” dies available. Therefore, the nurse should
in it and could be eliminated as an answer assess which medications the client has
­because this is an absolute. tried and what medications
Content – Fundamentals: Integrated Nursing Process – the client is currently taking.
Implementation: Client Needs – Safe Effective Care 3. The client being allergic to animals was not
­Environment, Safety and Infection Control: Cognitive in the stem. Many clients diagnosed with
Level – Application: ­Concept – Immunity. ­allergic rhinitis are allergic to seasonal envi-
ronmental allergens such as pollen and mold.
63. 1. Clients who are allergic to bee sting
4. The client probably does not have any
venom should be taught to keep an
explanation for developing allergies.
EpiPen with them at all times and how to
use the device. This could save their lives. TEST-TAKING HINT: The test taker should not
2. It is unrealistic to think the client will never read into a question. Because animals were
go outdoors, but the client should be taught not mentioned in the stem, option “3” can be
to avoid exposure to bees whenever possible. eliminated. Many over-the-counter medica-
3. Over-the-counter diphenhydramine tions and herbal remedies are available to
­(Benadryl) is a histamine-1 blocker, but it is clients, and it is important for the nurse to
oral and not useful in this situation. determine what the client has been taking.
4. The client should wear a Medic Alert Content – Medical: Integrated Nursing Process – ­
bracelet, but it is not priority over ensuring Assessment: Client Needs – Physiological Integrity,
the client knows how to treat a bee sting. ­Reduction of Risk Potential: Cognitive Level – Analysis:
Wearing the bracelet does not ensure correct Concept – Immunity.
treatment of the bee sting. 66. 1. Tree pollen is abundant in early spring.
TEST-TAKING HINT: Answer option “2” is an 2. Rose and grass pollen are prevalent in early
absolute and should be eliminated as a summer.
­possible correct answer. 3. Ragweed and other pollens are prevalent in
Content – Medical: Integrated Nursing Process – ­ early fall.
Planning: Client Needs – Physiological Integrity, 4. Early winter is the beginning of deciduous
­Physiological Adaptation: Cognitive Level – Synthesis: plants becoming dormant. Therefore, al-
Concept – Immunity. lergic rhinitis is least prevalent during this
time of year.
64. 1. This intervention should be implemented,
but it is not the first action. TEST-TAKING HINT: The test taker could
2. This does address oxygenation but will take ­eliminate the three (3) options based on
time to accomplish, so this intervention is not the plant growing season if the test taker
the first action. ­realized allergic rhinitis can be caused by
3. The client is cyanotic with dyspnea and ­environmental plant pollens and molds.
wheezing. The nurse should administer Content – Medical: Integrated Nursing Process –
oxygen first. ­Diagnosis: Client Needs – Safe Effective Care E
­ nvironment,
4. The client may be allergic to iodine, a Management of Care: Cognitive Level – Knowledge:
component of many shellfish, but the first Concept – Immunity.
need of the client is oxygenation. 67. 1. Immunotherapy does not cure the prob-
TEST-TAKING HINT: The test taker must apply lem. However, if immunotherapy is done
some decision-making standard to determine following a reaction, it provides passive
what to do first. Maslow’s hierarchy of needs immunity to the insect venom (similar to
ranks oxygen as first. Of the two (2) options the way RhoGAM prevents a mother who
addressing oxygen, option “3” immediately is Rh negative from building antibodies
attempts to provide oxygen to the client. to the blood of a baby who is Rh positive).
Content – Medical: Integrated Nursing Process – This is the purpose for immunotherapy in
Implementation: Client Needs – Safe Effective Care clients who are allergic.
­Environment, Management of Care: Cognitive Level – 2. This is an untrue statement.
Synthesis: Concept – Immunity. 3. There is no cure for allergies to insect venom.
4. This therapy is standard procedure for clients
who have severe allergies to insect venom.

13_Colgrove_Ch13.indd 539 01/06/16 5:55 PM


540 M ed -S urg S uccess

TEST-TAKING HINT: Answer options “2” and 70. 1. This client should be seen but not before as-
“3” contain forms of absolutes such as “all” sessing for a possible anaphylactic reaction.
and “cure.” Rarely is anything absolute in 2. This client has received an initial dose of
health care. The test taker should be ab- antibiotic IV and should be assessed for
solutely sure of the correct answer before tolerance to the medication within
choosing any answer containing an absolute 30 minutes.
descriptive word or passage. The stem asks 3. Pain is a priority but not over a potential life-
for the rationale and option “4” is giving threatening emergency.
­advice, so it can be eliminated. 4. This client can be seen last. A delayed meal is
Content – Medical: Integrated Nursing Process – not life threatening.
Diagnosis: Client Needs – Safe Effective Care E
­ nvironment, TEST-TAKING HINT: The test taker should
Management of Care: Cognitive Level – Analysis: determine which client has the most press-
Concept – Immunity. ing need and rank the options in order.
68. 1. This is an unrealistic expectation for a client ­Life-threatening situations have priority.
diagnosed with poison ivy. The pruritus is Content – Medical: Integrated Nursing Process – ­
intense. Assessment: Client Needs – Safe Effective Care
2. The client should be instructed on how to use ­Environment, Management of Care: Cognitive Level –
the EpiPen, not IM Benadryl. Analysis: Concept – Immunity.
3. Clients with poison ivy are frequently pre- 71. 1. By the Joint Commission standards, clients
scribed a steroid dose pack. The dose pack must mark any surgical site to make sure the
has the steroid provided in descending operation is not done on the incorrect site,
doses to help prevent such as the right arm instead of the left arm.
adrenal insufficiency. 2. The client should understand the surgery in
4. This may cause the client to be warm, his or her own terms.
which increases the likelihood of itching. 3. Iodine is the basic ingredient in Betadine
TEST-TAKING HINT: Option “1” has the word (povidone-iodine), which is a common
“never,” which is an absolute word and can skin prep used for surgeries. Therefore,
be eliminated on this basis. Very few condi- the nurse should notify the surgeon if the
tions require the nurse to teach the client to client has an allergy to iodine.
take intramuscular (IM) injections; therefore, 4. The client should have a signed consent
option “2” could be eliminated as a possible for the surgery and the anesthesia prior to
answer. surgery.
Content – Medical: Integrated Nursing Process – TEST-TAKING HINT: The options involve basic
Planning: Client Needs – Physiological Integrity, concepts for surgical preparation, and aller-
­Physiological Adaptation: Cognitive Level – Synthesis: gies must be identified on the client as well
Concept – Immunity. as in the client’s chart.
69. 1. This can be an independent or collab- Content – Medical: Integrated Nursing Process – ­
orative nursing problem. It is an airway Assessment: Client Needs – Safe Effective Care
­problem and has priority. ­Environment, Management of Care: Cognitive Level –
2. Knowledge deficit is not a priority over the Analysis: Concept – Perioperative.
client with breathing problems. 72. 1. Steroid medications decrease inflamma-
3. Anaphylaxis is a collaborative problem. The tion and therefore are one of the treat-
nurse will need to start IVs, administer medi- ments for anaphylaxis.
cations, and possibly place the client on a 2. A STAT chest x-ray is not indicated at this
ventilator if the client is to survive. time.
4. Ineffective coping is a psychosocial problem; 3. The Rapid Response Team should be
it does not have priority over breathing. called because this client will be in respi-
TEST-TAKING HINT: The test taker must ap- ratory and cardiac arrest very shortly.
ply some problem-solving/decision-making 4. Because of its ability to activate a combi-
standard. In this case Maslow’s hierarchy of nation of alpha and beta receptors, epi-
needs is a good option. Airway has priority. nephrine is the treatment of choice for
Content – Medical: Integrated Nursing Process – anaphylactic shock.
­Diagnosis: Client Needs – Safe Effective Care E
­ nvironment, 5. The first step in initiating cardiopulmo-
Management of Care: Cognitive Level – Analysis: nary resuscitation is to assess for a pulse
Concept – Immunity. and respirations.

13_Colgrove_Ch13.indd 540 01/06/16 5:55 PM


C hapter 13   I mmune S ystem D isorders 541

TEST-TAKING HINT: This is an alternative-type TEST-TAKING HINT: The test taker can rule
question. If the test taker did not read the out option “3” with knowledge of anatomy
sentence “Select all that apply,” the fact there or physiology. Lymph nodes do not multiply;
are five (5), not four (4), options should alert they do form chains throughout the body.
the test taker to go back and read the stem Content – Medical: Integrated Nursing Process –
more closely. Each option must be decided Diagnosis: Client Needs – Safe Effective Care E
­ nvironment,
on for itself. The test taker cannot eliminate Management of Care: Cognitive Level – Comprehension:
one option based on the fact another option Concept – Immunity.
is correct. 75. 1. Joint stiffness and joints warm to the touch
Content – Medical: Integrated Nursing Process – are expected in clients diagnosed with RA.
Implementation: Client Needs – Physiological Integrity, 2. Clients diagnosed with RA have bilateral and
Pharmacological and Parenteral Therapies: Cognitive
symmetrical stiffness, edema, tenderness,
Level – Application: Concept – Medication.
and temperature changes in the joints. Other
symptoms include sensory changes, lymph
Rheumatoid Arthritis (RA) node enlargement, weight loss, fatigue, and
pain. A one (1)-kg weight loss and fatigue are
73. 1. This is done, but it will not prevent any expected.
disease from occurring. 3. The use of heat is encouraged to provide
2. This will follow the progression of the disease comfort for a client diagnosed with RA.
of RA, but it is not preventive. 4. The client has the signs and symptoms
3. RA is a disease with many immunological of depression. The nurse should attempt
abnormalities. The clients have increased to intervene with therapeutic conversation
susceptibility to infectious disease, such and discuss these findings with the HCP.
as the flu or pneumonia, and, therefore, TEST-TAKING HINT: The test taker should not
vaccines, which are preventive, should be automatically assume only physiological data
recommended. require immediate intervention. There will
4. Assessing the client does not address preven- be times when a psychological need will have
tive care. priority. Because options “1,” “2,” and “3”
TEST-TAKING HINT: The stem requires the are all expected in a client with RA, the psy-
test taker to determine what action is pre- chological need warrants intervention by the
ventive care for the client with RA. Only nurse.
­option “3” addresses preventive care. Content – Medical: Integrated Nursing Process – ­
Content – Medical: Integrated Nursing Process – Assessment: Client Needs – Physiological Integrity,
Implementation: Client Needs – Safe Effective Care ­Reduction of Risk Potential: Cognitive Level – Synthesis:
­Environment, Management of Care: Cognitive Level – Concept – Mood.
Application: Concept – Immunity. 76. 1. The drug does not lose efficacy, and clients
74. 1. The nodules may appear over bony promi- are removed from the drug when the body
nences and resolve simultaneously. They cannot tolerate the side effects.
appear in clients with the rheumatoid 2. The drug requires close monitoring to
factor and are associated with rapidly prevent organ damage.
­progressive and destructive disease. 3. MRI scans are not used to determine the
2. There is a proliferation of the synovial mem- progress of RA.
brane in RA, which leads to the formation of 4. There is no “off” period for the drug.
pannus and the destruction of cartilage and TEST-TAKING HINT: If the test taker is not
bone, but synovial fluid does not crystallize to aware of the medication being discussed,
form the nodules. ­option “2,” the correct answer, is information
3. The nodules are not lymph nodes. Lymph which could be said of most medications.
nodes may enlarge in the presence of disease, Content – Medical: Integrated Nursing Process – ­
but they do not proliferate (multiply). Planning: Client Needs – Physiological Integrity,
4. The nodes indicate a progression of the ­Pharmacological and Parenteral Therapies: Cognitive
disease, not an improving prognosis. Level – Synthesis: Concept – Medication.

13_Colgrove_Ch13.indd 541 01/06/16 5:55 PM


542 M ed -S urg S uccess

77. 1. Physical therapists work with gait training 2. Any individual older than age 18 years is con-
and muscle strengthening. Generally, the sidered an adult and does not need to discuss
physical therapist works on the lower half treatment with her parents unless she chooses
of the body. to do so.
2. The occupational therapist assists the 3. The medications can be administered on an
client in the use of the upper half of the outpatient basis, but if an inpatient has in-
body, fine motor skills, and activities of travenous therapy, then IV sites are changed
daily living. This is needed for the client every 72 hours and there is no guarantee an
with abnormal fingers. IV will last for four (4) days.
3. A counselor can help the client discuss 4. These are not investigational drugs and are
feelings about body image, loss of function, standard therapy approved by the American
and role changes, but the best referral is to College of Rheumatology and the Food and
the occupational therapist. Drug Administration.
4. The client may need a home health nurse TEST-TAKING HINT: The age of the client and
eventually, but first the client should be the fact the client is female could give the
assisted to remain as functional as possible. test taker an idea of the correct answer. This
TEST-TAKING HINT: The test taker must be is a client in the childbearing years.
aware of the roles of all the health-care team Content – Medical: Integrated Nursing Process –
members. The counselor (option “3”) can Assessment: Client Needs – Safe Effective Care
be ruled out as a possible correct answer ­Environment, Management of Care: Cognitive Level –
because swan-neck f­ ingers are a physical Analysis: Concept – Medication.
problem.
80. 1. Activity intolerance is an appropriate client
Content – Medical: Integrated Nursing Process – problem, but it is not priority over pain.
Implementation: Client Needs – Safe Effective Care 2. The client with RA does not experience fluid
­Environment, Management of Care: Cognitive Level –
and electrolyte disturbance.
Application: Concept – Immunity.
3. The client diagnosed with RA has chronic
78. 1. The client diagnosed with RA is generally pain; therefore, alteration in comfort is a
fatigued, and strenuous exercise increases priority problem.
the fatigue, places increased pressure on the 4. Clients diagnosed with RA usually experience
joints, and increases pain. anorexia and weight loss, unless they are tak-
2. The client should be on a balanced diet ing long-term steroids.
high in protein, vitamins, and iron for tissue TEST-TAKING HINT: The question is asking
­building and repair and should not require a for the priority problem, and pain is priority
mechanically altered diet. ­according to Maslow’s hierarchy of needs.
3. There is no specific reason for the client to Content – Medical: Integrated Nursing Process –
be ordered a keep-open IV; the client can Diagnosis: Client Needs – Safe Effective Care E
­ nvironment,
swallow needed medications. Management of Care: Cognitive Level – Analysis:
4. Sleep deprivation resulting from pain is Concept – Comfort.
common in clients diagnosed with RA.
A mild sedative can increase the client’s 81. 1. The client in pain should receive medication
ability to sleep, promote rest, and increase as soon as possible to keep the pain from be-
the client’s tolerance of pain. coming worse, but the client is not at risk for
a serious complication.
TEST-TAKING HINT: The test taker should be
2. A butterfly rash across the bridge of the nose
aware of adjectives leading to an option b ­ eing occurs in approximately 50% of the clients
eliminated—for example, the word diagnosed with SLE.
“strenuous” in o ­ ption “1.”
3. Antineoplastic drugs can be caustic to tis-
Content – Medical: Integrated Nursing Process – ­ sues; therefore, the client’s IV site should
Implementation: Client Needs – Safe Effective Care be assessed. The client should be assessed
­Environment, Management of Care: Cognitive Level –
for any untoward reactions to the medica-
Application: Concept – Comfort.
tions first.
79. 1. Immunosuppressive medications are 4. Scleroderma is a disease characterized by
­considered class C drugs and should not waxlike skin covering the entire body. This is
be taken while pregnant. These drugs expected for this client.
are teratogenic and carcinogenic, and the
­client is only 20 years old.

13_Colgrove_Ch13.indd 542 01/06/16 5:55 PM


C hapter 13   I mmune S ystem D isorders 543

TEST-TAKING HINT: Pain is a priority, but the a risk for the development of ulcers. The
test taker must determine if there is another client should take the NSAID with food.
client who could experience complications if TEST-TAKING HINT: Knowledge of medication
not seen immediately. administration is a priority for every nurse. It
Content – Medical: Integrated Process – Assessment: is especially important for the nurse to be fa-
­Client Needs – Safe Effective Care Environment, miliar with commonly used medications such
­Management of Care: Cognitive Level – Synthesis: as NSAIDs, which can be purchased over the
Concept – Immunity. counter and may be taken by the client in ad-
82. 1. Antineoplastic medications can be adminis- dition to prescription medications.
tered only by a registered nurse who has been Content – Medical: Integrated Nursing Process – ­
trained in the administration and disposal of Planning: Client Needs – Physiological Integrity,
these medications. ­Pharmacological and Parenteral Therapies: Cognitive
2. Assessment cannot be assigned to a licensed Level – Synthesis: Concept – Medication.
practical nurse. 84. 1. Pain medication is important and should
3. The licensed practical nurse (LPN) can be given before the client’s pain becomes
demonstrate how to use adaptive clothing. worse.
4. This is teaching requiring knowledge of 2. Unless the client is in a crisis, such as pulmo-
medications and interactions and should not nary edema, this medication can wait.
be assigned to an LPN. 3. Steroids do not have precedent over pain
TEST-TAKING HINT: The nurse cannot assign medication and should be administered with
assessment, evaluation, or teaching or any food.
medication requiring specialized knowledge 4. Clients diagnosed with OA are usually over-
or skills to administer safely. weight and do not require appetite stimulants.
Content – Medical: Integrated Nursing Process – The nurse should question this medication
­Planning: Client Needs – Safe Effective Care Environment, before administering the medication.
Management of Care: Cognitive Level – Synthesis: TEST-TAKING HINT: When determining
Concept – Nursing Roles. ­priorities, the test taker must employ some
83. 1. This is prescribing, and the nurse is not criteria to use as a guideline. According to
­licensed to do this unless the nurse has Maslow, pain is a priority.
become a nurse practitioner. Content – Medical: Integrated Nursing Process –
2. NSAIDs do not require a specific amount Implementation: Client Needs – Safe Effective Care
of water to be effective, unlike bulk laxatives. ­Environment, Management of Care: Cognitive Level –
3. The medication should be taken in the usual Application: Concept – Medication.
dose when the client realizes a dose has been
missed.
4. NSAID medications decrease prostaglan-
din production in the stomach, resulting
in less mucus production, which creates

13_Colgrove_Ch13.indd 543 01/06/16 5:55 PM


CONCEPTS

85. 1. SIRS involvement in the hematopoetic involvement in the inflammatory response


system includes prolonged bleeding times system.
and thrombocytopenia. 2. Pain control would be under a pain or com-
2. In the prerenal phase and intrarenal phase fort concept, not immunity.
of SIRS affecting the renal system, urine 3. Keeping the head of the bed elevated is to
osmolality increases but urine production allow for lung expansion, not to decrease an
decreases because of decreased glomeru- immune system response, and at “all times” is
lar filtration. not realistic.
3. The capillary membranes have a systemic 4. Therapeutic communication addresses a psy-
response to the insult, resulting in in- chosocial problem, not a physiological one.
creased permeability and leaking of fluid TEST-TAKING HINT: This question is asking
into the interstitial spaces (pitting edema). the test taker to identify the assessment data
4. The brain responds poorly to the in- unexpected for the disease process. The test
creased interstitial fluid and can result in taker should be very careful about choosing
confusion, disorientation, and delirium. an option with an absolute in the wording.
5. These vital signs are within normal limits. “Every” means there is no exception to the
This indicates a stable client, not one diag- situation that might apply.
nosed with SIRS.
Content – Medical: Integrated Nursing Process – ­
TEST-TAKING HINT: This question is asking Assessment: Client Needs – Safe Effective Care
the test taker to identify the assessment ­Environment, Management of Care: Cognitive Level –
data unexpected for the disease process. In Synthesis: Concept – Immunity.
“Select all that apply” questions, each option
88. 1. SIRS can occur from a burn but it can also
should be answered as a true/false question.
occur as a result of any insult that has a great
Content – Medical: Integrated Nursing Process – ­ impact on the body systems.
Assessment: Client Needs – Safe Effective Care 2. This is the definition of SIRS.
­Environment, Management of Care: Cognitive Level –
3. SIRS can occur from an infection but it can
Synthesis: Concept – Immunity.
also occur as a result of any insult that has a
86. 1. Lung sounds assess the respiratory great impact on the body systems.
­system, and heart sounds and blood pres- 4. SIRS can occur from an allergic response but
sure assess the circulatory/cardiovascular it can also occur as a result of any insult that
system. has a great impact on the body systems.
2. Psychological responses do not assess for TEST-TAKING HINT: This question is a basic
MODS. ­knowledge level definition. The test taker
3. The family’s expectations do not assess for should not refuse to choose an option be-
MODS. cause it seems too easy.
4. Emesis is not particularly associated with
Content – Medical: Integrated Nursing Process – ­
MODS and bile secretions and mouth ulcers Assessment: Client Needs – Safe Effective Care
have no correlation with MODS. ­Environment, Management of Care: Cognitive Level –
TEST-TAKING HINT: This question is ask- Cognitive: Concept – Immunity.
ing the test taker to identify the assess-
89. 1. The diagnosis of AIDS is determined by
ment data expected for the disease process.
predefined criteria: A CD4 count less than
­Physiological problems are high priority
200; a fungal infection candidiasis of the
according to Maslow and often warrant
bronchi, lungs, esophagus or Pneumocystis
­immediate intervention.
jiroveci pneumonia (PJP); disseminated
Content – Medical: Integrated Nursing Process – ­ extrapulmonary coccidioidomycosis; dis-
Assessment: Client Needs – Safe Effective Care seminated extrapulmonary histoplasmo-
­Environment, Management of Care: Cognitive Level –
sis; a viral issue, cytomegalovirus (CMV)
Synthesis: Concept – Immunity.
disease other than liver, spleen, or nodes;
87. 1. The intake and output will alert the nurse CMV retinitis herpes simplex virus with
to potential renal and cardiovascular chronic ulcers or bronchitis, pneumonia,

544

13_Colgrove_Ch13.indd 544 03/06/16 10:56 AM


C hapter 13   I mmune S ystem D isorders 545

or esophagitis; progressive multifocal leu- no guarantee the condom does not break,
koencephalopathy, extrapulmonary Cryp- resulting in shared fluids. The more part-
tococcus; protozoal toxoplasmosis of the ners, the greater the risk.
brain, chronic intestinal cryptosporidiosis; 4. Lack of available health care may delay treat-
bacterial Mycobacterium avium complex ment but is not a risk factor for developing an
(MAC) or Mycobacterium kansasii; or one HIV infection.
of the following opportunistic cancers: 5. Adequate bathroom facilities is not an ante-
invasive cervical cancer, Kaposi sarcoma, cedent for HIV infections. The HIV virus
Burkitt’s lymphoma, immunoblastic lym- dies six (6) minutes outside of a host body or
phoma, and primary lymphoma of the growth environment (petri dish).
brain. 6. Many people spend their money on nones-
2. See answer A. sential items; it is not an antecedent for HIV
3. See answer A. infection.
4. See answer A. TEST-TAKING HINT: The test taker should
TEST-TAKING HINT: This question is asking answer each option as a true/false question.
the test taker to identify the criteria of diag- One option does not eliminate another.
nosing AIDS, which is long and complicated. Content – Medical: Integrated Nursing Process – ­
The test taker should remember any issue Assessment: Client Needs – Safe Effective Care
that occurs as a result of a failing immune ­Environment, Management of Care: Cognitive Level –
system, opportunistic infections, and cancers. Synthesis: Concept – Immunity.
Content – Medical: Integrated Nursing Process – ­ 92. 1. Vital signs within normal limits indicate
Assessment: Client Needs – Safe Effective Care the client is stable and is a realistic mea-
­Environment, Management of Care: Cognitive Level –
surable goal.
Cognitive: Concept – Immunity.
2. Six hundred mL of urine in 24 hours
90. 1. Raw fruits and vegetables and fresh flow- averages 25 mL per hour, an inadequate
ers can harbor parasites and bacteria and amount of urine to indicate renal perfusion
should be kept out of the client’s room. (30 mL per hour).
2. This addresses the concept of functional 3. Liver enzymes indicating proper liver func-
­ability, not immunity. tion would be to maintain enzymes within
3. This addresses the concept of functional normal limits, not elevated.
­ability, not immunity. 4. These blood glucose readings are not
4. This addresses the concept of perfusion, not within normal limits, indicating the need for
immunity. ­intervention to bring the glucose down to a
TEST-TAKING HINT: This question is asking normal range with sliding scale.
the test taker to identify the assessment data TEST-TAKING HINT: This question is asking
unexpected for the disease process. Respira- the test taker to identify the assessment data
tory problems are high priority according ­expected for the stabilization or improve-
to Maslow and often warrant immediate ment of the ­client. The test taker should
intervention. work out the math to determine if the cli-
Content – Medical: Integrated Nursing Process – ent’s renal output falls within expected
Implementation: Client Needs – Safe Effective Care guidelines for adequate renal perfusion.
­Environment, Management of Care: Cognitive Level – Content – Medical: Integrated Nursing Process –
Synthesis: Concept – Immunity. Planning: Client Needs – Safe Effective Care Environment,
Management of Care: Cognitive Level – Synthesis:
91. 1. Pregnancy is a co-related condition but being
Concept – Immunity.
pregnant is not an antecedent for having an
HIV infection. 93. 1. This client is at high risk for a negative
2. Intravenous drug use does create a risk ­outcome, including death.
for becoming HIV positive. When the 2. This client has a 20% to 30% chance for
drug user shares the needle used to inject survival.
the drugs, then body fluids are directly 3. The prognosis for clients with MODS is
injected into the next person to use the poor with mortality rates between 70%
syringe and needle. and 80% if three or more systems fail.
3. Unprotected sex involves sharing of body 4. The rate of recovery is reduced to 20%
fluids, and even if using a condom, there is to 30%.

13_Colgrove_Ch13.indd 545 01/06/16 5:55 PM


546 M ed -S urg S uccess

TEST-TAKING HINT: This question is asking The nurse would question administer-
the test taker to identify data describing the ing a medication that decreases the blood
potential outcomes for a client. In order to pressure.
answer this question the test taker must have 4. This is not a high dose of acetaminophen
a working knowledge of the disease pro- and could be administered for mild pain. The
cess; however, if the test taker is not aware nurse would not question this medication.
of the information, reading “Multiorgan” TEST-TAKING HINT: This question is asking
could help to eliminate option “1” because the test taker to identify actions and side
ventilators do maintain life; but the longer effects of medications. It is important for the
a client remains on a ventilator, the worse nurse to recognize when a medication would
the prognosis. Hospital-acquired infections have untoward effects on the client.
­frequently occur with ventilator clients. Content – Medical: Integrated Nursing Process –
Content – Medical: Integrated Nursing Process – Implementation: Client Needs – Pharmacological and
Assessment: Client Needs – Safe Effective Care ­Parenteral Therapies: Cognitive Level – Synthesis:
­Environment, Management of Care: Cognitive Level – Concept – Immunity.
Synthesis: Concept – Immunity.
96. 1. It is 1600 and the vancomycin is not due for
94. 1. This is a systemic problem, not a localized two (2) hours. Vancomycin is nephrotoxic and
response. ototoxic; the nurse has a one (1)-hour window
2. SIRS untreated or unresponsive to treatment before and after the time due but two (2)
progresses to Multi Organ Dysfunction hours is too close to the previous dose.
Syndrome (MODS). 2. Furosemide is due now; it should be
3. SIRS is not limited to myocardial or the medication given first. A diuretic is
­pulmonary issues. administered to help the client excrete
4. This is the definition of SIRS. urine; it is scheduled for this time so that
TEST-TAKING HINT: This question is asking the effects have subsided before bedtime.
the test taker to know the definition of a 3. Pantoprazole should have been administered
disease process. The test taker may be able in the morning. Because it has not been ac-
to answer the question by identifying words knowledged as given, the nurse must research
in the name that describe what is occur- the reason for the client not receiving the
ring in the body, Localized versus the name medication before administering a potential
“­systemic” could eliminate option “1.” second dose.
Content – Medical: Integrated Nursing Process – 4. Acetaminophen is for mild pain; it could be
Assessment: Client Needs – Safe Effective Care administered second.
­Environment, Management of Care: Cognitive Level – TEST-TAKING HINT: This question is asking
Cognitive: Concept – Immunity. the test taker to identify actions and side
95. 1. MODS is frequently a result of sepsis; the ­effects of medications. It is important for the
nurse would not question an antibiotic. nurse to recognize when a medication would
2. MODS can involve development of capillary have untoward effects on the client.
permeability that allows fluids to “leak” from Content – Medical: Integrated Nursing Process –
the capillaries into the interstitial space; the Implementation: Client Needs – Pharmacological and
nurse would not question a medication that ­Parenteral Therapies: Cognitive Level – Synthesis:
encourages the fluid to return to the circula- Concept – Immunity.
tory system for excretion by the kidneys.
3. MODS’s effect on the circulatory ­system
includes a decreased blood pressure.

13_Colgrove_Ch13.indd 546 01/06/16 5:55 PM


C hapter 13   I mmune S ystem D isorders 547

IMMUNE SYSTEM DISORDERS


COMPREHENSIVE EXAMINATION
1. The client is prescribed a prick epicutaneous 3. Which referral should the nurse implement for a
test to determine the cause of hypersensitivity client with severe multiple allergies?
reactions. Which result indicates the client is 1. Registered dietitian.
hypersensitive to the allergen? 2. Occupational therapist.
1. The client complains of shortness of breath. 3. Recreational therapist.
2. The skin is dry, intact, and without redness. 4. Social worker.
3. The pricked blood tests positive for allergens.
4. The client diagnosed with an anaphylactic
4. A pruritic wheal and erythema occur.
reaction is admitted to the emergency department.
2. Which area of the body should the nurse assess to Which assessment data indicate the client is not
identify symptoms to support the early diagnosis responding to the treatment?
of Guillain-Barré syndrome? 1. The client has a urinary output of 120 mL in
1. A two (2) hours.
2. B 2. The client has an AP of 110 and a BP of 90/60.
3. C 3. The client has clear breath sounds and an RR
4. D of 26.
4. The client has hyperactive bowel sounds.
5. Which signs/symptoms should the nurse expect
A to assess in the client diagnosed with Sjögren’s
syndrome?
1. Complaints of dry mouth and eyes.
2. Complaints of peripheral joint pain.
B
3. Complaints of muscle weakness.
4. Complaints of severe itching.
6. Which intervention should the nurse implement
for the client diagnosed with systemic sclerosis
(scleroderma)?
C 1. Instill artificial tears four (4) times a day.
2. Apply moisturizers to the skin frequently.
3. Instruct the client on how to apply braces.
4. Encourage the client to decrease smoking.
7. The nurse is caring for a client with suspected
fibromyalgia. Which diagnostic test confirms the
diagnosis of fibromyalgia?
1. There is no diagnostic test to confirm
fibromyalgia.
2. A positive antinuclear antibody test.
3. A magnetic resonance imaging (MRI) shows
fibrosis.
D 4. A negative erythrocyte sedimentation rate
(ESR).

13_Colgrove_Ch13.indd 547 01/06/16 5:55 PM


548 M ed -S urg S uccess

8. The primary nurse is administering medications 11. The client is diagnosed with systemic lupus
to the assigned clients. Which client situation erythematosus (SLE). Which area of the body
requires immediate intervention by the charge in the figure below should the nurse assess for a
nurse? butterfly rash?
1. The client with congestive heart failure with 1. A
an apical pulse of 64 who received 0.125 mg 2. B
digoxin, a cardiac glycoside. 3. C
2. The client with essential hypertension who 4. D
received a beta blocker and has a blood
pressure of 114/80.
3. The client with myasthenia gravis who A
received the anticholinesterase medication 30
minutes late.
4. The client with AIDS who received
trimethoprim-sulfamethoxazole, an antibiotic, B
and has a CD4 cell count of less than 200.
9. Which interventions should the nurse discuss
with the female client who is positive for human
immunodeficiency virus (HIV)? Select all that
apply.
1. Recommend the client not to engage in
unprotected sexual activity.
2. Instruct the client not to inform past sexual
partners of HIV status.
3. Tell the client to not donate blood, organs, or C
tissues.
4. Suggest the client not get pregnant.
5. Explain the client does not have to tell health-
care personnel of HIV status.
10. Which sign/symptom should the nurse expect to
assess in the client who is in the recovery stage
of Guillain-Barré syndrome?
1. Decreasing deep tendon reflexes.
2. Drooping of the eyelids has resolved.
3. A positive Babinski’s reflex.
4. Descending increase in muscle strength.
D

12. Which nursing intervention should the nurse


include when teaching the client diagnosed with
polymyositis?
1. Explain the care of a percutaneous endoscopic
gastrostomy tube.
2. Discuss the need to take corticosteroids every
day.
3. Instruct to wear long-sleeved shirts when
exposed to sunlight.
4. Teach the importance of strict hand washing.
13. The client recently diagnosed with rheumatoid
arthritis is prescribed aspirin, a nonsteroidal
anti-inflammatory medication. Which comment
by the client warrants immediate intervention
by the nurse?
1. “I always take the aspirin with food.”
2. “If I have dark stools, I will call my HCP.”
3. “Aspirin will not cure my arthritis.”
4. “I am having some ringing in my ears.”

13_Colgrove_Ch13.indd 548 01/06/16 5:55 PM


C hapter 13   I mmune S ystem D isorders 549

14. Which sign/symptom makes the nurse suspect 19. The client with acquired immunodeficiency
the client has ankylosing spondylitis? syndrome (AIDS) dementia is referred to
1. Low back pain at night relieved by activity in hospice. Which intervention has highest
the morning. priority when caring for the client in the home?
2. Ascending paralysis of the lower extremities 1. Assess the client’s social support network.
up to the spinal cord. 2. Identify the client’s usual coping methods.
3. A deep ache and stiffness in the hip joints 3. Have consistent uninterrupted time with the
radiating down the legs. client.
4. Difficulty changing from lying to sitting 4. Discuss and complete an advance directive.
position, especially at night.
20. The client with multiple sclerosis is prescribed
15. The client diagnosed with multiple sclerosis the muscle relaxant baclofen (Lioresal). Which
is having trouble maintaining balance. Which statement by the client indicates the client needs
intervention should the nurse discuss with the more teaching?
client? 1. “This medication may cause drowsiness so
1. Discuss obtaining a motorized wheelchair for I need to be careful.”
the client. 2. “I should not drink any type of alcohol or take
2. Teach the client to stand with the feet slightly any antihistamines.”
apart. 3. “I will increase the fiber in my diet and
3. Encourage the client to narrow his or her base increase fluid intake.”
of support. 4. “I stopped taking the medication because
4. Explain the need to balance activity with rest. I can’t afford it.”
16. The nurse is caring for the client diagnosed with 21. Which intervention has the highest priority
acquired immunodeficiency syndrome (AIDS) when caring for a client diagnosed with
dementia. Which action by the unlicensed rheumatoid arthritis?
assistive personnel (UAP) requires immediate 1. Encourage the client to ventilate feelings
intervention by the nurse? about the disease process.
1. The UAP is helping the client to sit on the 2. Discuss the effects of disease on the client’s
bedside chair. career and other life roles.
2. The UAP is wearing sterile gloves when 3. Instruct the client to perform most important
bathing the client. activities in the morning.
3. The UAP is helping the client shave and 4. Teach the client the proper use of hot and
brush the teeth. cold therapy to provide pain relief.
4. The UAP is providing a back massage to the
22. Which statement indicates the female client
client.
with systemic lupus erythematosus (SLE)
17. Which assessment data should make the nurse understands the discharge instructions?
suspect the client has chronic allergies? 1. “I should wear sunscreen with at least a
1. Jaundiced sclera and jaundiced palms of 5 SPF.”
hands. 2. “I am not going to any activities with large
2. Pale, boggy, edematous nasal mucosa. crowds.”
3. Lacy white plaques on the oral mucosa. 3. “I should not get pregnant because I have
4. Purple or blue patches on the face. SLE.”
4. “I must avoid using hypoallergenic products.”
18. The client has had an anaphylactic reaction
to insect venom, a bee sting. Which discharge 23. Which is the highest priority nursing
instruction should the nurse discuss with the intervention for the client who is having an
client? anaphylactic reaction?
1. Take a corticosteroid dose pack when stung by 1. Administer parenteral epinephrine, an
a bee. adrenergic agonist.
2. Take antihistamines prior to outdoor 2. Prepare for immediate endotracheal
activities. intubation.
3. Use a cromolyn sodium (Intal) inhaler 3. Provide a calm assurance when caring for the
prophylactically. client.
4. Carry a bee sting kit, especially when going 4. Establish and maintain a patent airway.
outside.

13_Colgrove_Ch13.indd 549 01/06/16 5:55 PM


550 M ed -S urg S uccess

24. Which discharge instruction should the nurse 26. The client is experiencing an anaphylactic
implement for the client newly diagnosed with reaction to bee venom. Which interventions
myasthenia gravis (MG)? should the nurse implement? List in order of
1. Identify specific measures to help avoid priority.
fatigue and undue stress. 1. Establish a patent airway.
2. Instruct the client to pad bony prominences, 2. Administer epinephrine, an adrenergic
especially the sacral area. agonist, IVP.
3. Discuss complementary therapies to help 3. Start an IV with 0.9% saline.
manage pain. 4. Teach the client to carry an EpiPen when
4. Explain the possibility of having a outside.
splenectomy to help control the symptoms. 5. Administer diphenhydramine (Benadryl), an
antihistamine, IVP.
25. Which signs/symptoms make the nurse suspect
the most common opportunistic infection
in the female client diagnosed with acquired
immunodeficiency syndrome (AIDS)?
1. Fever, cough, and shortness of breath.
2. Oral thrush, esophagitis, and vaginal
candidiasis.
3. Abdominal pain, diarrhea, and weight loss.
4. Painless violet lesions on the face and tip of
nose.

13_Colgrove_Ch13.indd 550 01/06/16 5:55 PM


IMMUNE SYSTEM DISORDERS COMPREHENSIVE
EXAMINATION ANSWERS AND RATIONALES
1. 1. This is a sign of an anaphylactic reaction to an 4. 1. A urinary output of greater than 30 mL/hr is
allergen and will not happen during this test within normal limits and indicates the client is
because of the small amount of allergen used. responding to treatment.
2. This indicates a negative test and the client is 2. These vital signs indicate shock, which is a
not sensitive to the allergen. medical emergency and requires immediate
3. The skin reaction, not the blood pricked, indi- intervention.
cates a positive or negative test. 3. Clear breath sounds indicate response to treat-
4. During this test, a drop of diluted aller- ment, and although the RR is increased, this
genic extract is placed on the skin and then could be the result of anxiety or fear.
the skin is punctured through the drop. 4. The client’s bowel sounds are not significant
A positive test causes a localized pruritic data to determine the client’s response to
wheal and erythema, which occurs in five treatment.
(5) to 20 minutes. Content – Medical: Integrated Nursing Process –
Content – Medical: Integrated Nursing Process – Assessment: Client Needs – Physiological Integrity,
Assessment: Client Needs – Physiological Integrity, ­Reduction Reduction of Risk Potential: Cognitive Level – Analysis:
of Risk Potential: Cognitive Level – Analysis: Concept – Concept – Immunity.
Immunity.
5. 1. Sjögren’s syndrome is an autoimmune dis-
2. 1. The presenting symptom of a client with order causing inflammation and dysfunc-
Guillain-Barré syndrome is ascending pa- tion of exocrine glands throughout the
ralysis starting in the lower extremities. body. Dry mouth and eyes are some of the
2. Abdominal symptoms are not found early in signs/symptoms.
the diagnosis. 2. Peripheral joint pain may be a symptom
3. Chest symptoms are not found early in the of rheumatoid arthritis.
diagnosis. 3. Muscle weakness is a symptom of a variety of
4. Head symptoms are not found early in the disease processes and syndromes but not of
diagnosis. Sjögren’s syndrome.
Content – Medical: Integrated Nursing Process – 4. Severe itching is not a symptom of this
Assessment: Client Needs – Physiological Integrity, syndrome.
Reduction of Risk Potential: Cognitive Level – Content – Medical: Integrated Nursing Process –
Analysis: Concept – Mobility. Assessment: Client Needs – Physiological Integrity,
Reduction of Risk Potential: Cognitive Level – Analysis:
3. 1. A dietitian could help the client with any
Concept – Immunity.
necessary dietary changes for food allergies
and with ways to continue to meet nutri- 6. 1. Artificial tears are appropriate for a client
tional needs. ­diagnosed with Sjögren’s syndrome.
2. An occupational therapist addresses the 2. Nursing care addresses measures to main-
client’s ability to perform activities of daily tain skin integrity and moisturizers help
living. prevent dryness and cracking;
3. A recreational therapist works in a psychiatric once skin elasticity is lost, it cannot
setting or rehabilitation setting and assists with be regained.
the client’s therapeutic recreational activities. 3. Braces are not prescribed for the client with
4. A social worker addresses the client’s financial scleroderma.
needs. 4. The client should stop smoking, not just
Content – Medical: Nursing Process – Implementation: decrease smoking, because of the vasoconstric-
Client Needs – Safe Effective Care Environment, tive effect of nicotine and the respiratory ef-
­Management of Care: Cognitive Level – Application: fects of the disease.
Concept – Immunity. Content – Medical: Integrated Nursing Process –
Implementation: Client Needs – Safe Effective Care
­Environment, Management of Care: Cognitive Level –
Application: Concept – Immunity.

551

13_Colgrove_Ch13.indd 551 01/06/16 5:55 PM


552 M ed -S urg S uccess

7. 1. The diagnosis of fibromyalgia is based on Content – Medical: Integrated Nursing Process –


the history and physical assessment. There Planning: Client Needs – Physiological Integrity,
is no laboratory or diagnostic test for fibro- ­Physiological Adaptation: Cognitive Level – Synthesis:
myalgia. However, tests may be performed Concept – Immunity.
to rule out other diagnoses. 10. 1. This occurs in the acute stage of Guillain-
2. This test is not used to diagnose fibromyalgia. Barré syndrome.
3. An MRI is not used to diagnose fibromyalgia. 2. This indicates the client diagnosed with
4. An ESR does not support the diagnosis of myasthenia gravis is getting better.
fibromyalgia. 3. A positive Babinski’s reflex in an adult c­ lient
Content – Medical: Integrated Nursing Process – is abnormal and indicates neurological
Assessment: Client Needs – Physiological Integrity, deficits.
Reduction of Risk Potential: Cognitive Level – 4. The recovery stage may take from
Analysis: Concept – Immunity. several months to two (2) years, and mus-
8. 1. An apical heart rate of less than 60 warrants cle strength and function return in
intervention if the primary nurse gave the a descending order.
medication. Content – Medical: Integrated Nursing Process –
2. A blood pressure of less than 90/60 warrants Assessment: Client Needs – Physiological Integrity,
intervention if the primary nurse gave the Reduction of Risk Potential: Cognitive Level –
medication. Analysis: Concept – Mobility.
3. These medications must be administered 11. D. The client diagnosed with SLE does not have
exactly on time so increased strength can a butterfly rash on the feet.
occur during activity such as eating or C. The client does not have a butterfly rash on
grooming. There are very few medications the upper-thigh area.
administered exactly on time, but this is B. The client does not have a butterfly rash on
one of them. the chest area.
4. The client with AIDS receives prophylactic A. The client with SLE often has a reddened
treatment for Pneumocystis pneumonia (PCP) area over both cheeks known as a butterfly
when the CD4 count is less than 200 to 300. rash; it is diagnostic of a client with SLE.
Content – Medical: Integrated Nursing Process – Content – Medical: Integrated Nursing Process –
Evaluation: Client Needs – Safe Effective Care Assessment: Client Needs – Physiological Integrity,
Environment, Management of Care: Cognitive Level – Reduction of Risk Potential: Cognitive Level – Analysis:
Synthesis: Concept – Medication. Concept – Immunity.
9. 1. HIV is transmitted via sexual activity. 12. 1. The client is at risk for aspiration as a result
2. HIV is transmitted via sexual activity, and of muscle weakness, but modifications of
the client may have been HIV positive for dietary needs address this concern. The client
up to a year and not aware of it, so all past does not require a PEG tube.
sexual ­partners should be informed of the 2. Polymyositis is a systemic connective
HIV status. tissue disorder characterized by inflam-
3. Blood donations are screened and excluded mation of connective tissues and muscle
for this virus, as are organs/tissues from a fibers and is treated with long-term cor-
client with HIV, because the virus can be ticosteroid therapy. Adrenal insufficiency
transmitted to clients receiving the organ may occur if the client quits taking the
or tissue. corticosteroid.
4. HIV can be transmitted to the fetus from 3. Sunlight does not cause an exacerbation or
the pregnant woman with HIV. irritation of polymyositis.
5. The client should tell the HCP, especially 4. The client is not at risk for developing an
dentists, about the HIV status, but the infection, and an infection will not exacerbate
client does not have to tell health-care per- the client’s medical condition.
sonnel about the HIV status. Health-care Content – Medical: Integrated Nursing Process –
personnel should always follow Standard Planning: Client Needs – Physiological Integrity,
Precautions. ­Physiological Adaptation: Cognitive Level – Synthesis:
Concept – Immunity.

13_Colgrove_Ch13.indd 552 01/06/16 5:55 PM


C hapter 13   I mmune S ystem D isorders 553

13. 1. Aspirin should be taken with food to prevent 16. 1. This action is appropriate and does not
gastrointestinal upset. require any intervention by the nurse.
2. Daily aspirin is used as an anticoagulant; 2. The UAP should wear nonsterile gloves,
therefore, abnormal bleeding should be not sterile gloves. Wearing sterile gloves is
reported to the HCP. not cost effective.
3. Aspirin is used to reduce the inflammatory 3. The client has dementia, so helping the client
process and manage the signs and symptoms, with activities of daily living is ­appropriate to
but it does not stop the disease process. enable the client to maintain as much inde-
4. Tinnitus (ringing in the ears) is a sign of pendence as possible.
aspirin toxicity, and the client should be 4. This is an excellent intervention to help pre-
instructed to decrease the aspirin dosage vent skin breakdown; it is relaxing for the
or stop taking aspirin altogether. The cli- client and does not require intervention from
ent should be instructed to contact the the nurse.
health-care provider. Content – Medical: Integrated Nursing Process –
Content – Medical: Integrated Nursing Process – Evaluation: Client Needs – Safe Effective Care
Evaluation: Client Needs – Safe Effective Care Environment, Management of Care: Cognitive Level –
Environment, Management of Care: Cognitive Synthesis: Concept – Immunity.
Level – Synthesis: Concept – Medication.
17. 1. This may indicate a hemolytic reaction.
14. 1. Ankylosing spondylitis is a chronic inflam- 2. Pale, boggy, edematous nasal mucosa
matory arthritis that primarily affects the ­indicates chronic allergies.
spinal cord. The client complains of inter- 3. This may indicate hemolysis or immune
mittent bouts of low back pain with the deficiency.
pain worse at night, followed by morning 4. This may indicate Kaposi’s sarcoma.
stiffness relieved by activity. Content – Medical: Integrated Nursing Process –
2. Ascending paralysis makes the nurse suspect Assessment: Client Needs – Physiological Integrity,
Guillain-Barré syndrome. Reduction of Risk Potential: Cognitive Level – Analysis:
3. A deep ache and stiffness may indicate Concept – Immunity.
osteoarthritis, which occurs in weight-
18. 1. Corticosteroids may be used in both systemic
bearing joints.
and topical forms for many types of hyper-
4. This is not a symptom of ankylosing sensitivity responses but must be ordered by
spondylitis. a health-care provider and are not automati-
Content – Medical: Integrated Nursing Process – cally taken after a bee sting.
Assessment: Client Needs – Physiological Integrity, 2. Antihistamines are the major class of drugs
Reduction of Risk Potential: Cognitive Level – Analysis:
used to treat hypersensitivity responses, but
Concept – Mobility.
they are not taken prophylactically. They are
15. 1. Walkers or canes may be weighted to provide used when a reaction occurs.
support and balance for the client; a wheel- 3. This drug treats allergic rhinitis and asthma
chair should be used as a last resort. prophylactically. It does not help bee stings or
2. Standing with the feet slightly apart wid- insect bites.
ens the client’s base of support and helps 4. The kit usually includes a prefilled syringe
decrease balance problems. of epinephrine and an epinephrine nebu-
3. The client should widen his or her base of lizer, which allows prompt self-treatment
support by standing with the feet slightly for any future exposures to insect venom
apart. Narrowing the base of support does or other potential allergen exposure.
not help. Content – Medical: Integrated Nursing Process –
4. This intervention addresses fatigue, which Planning: Client Needs – Physiological Integrity,
does not cause balance problems. ­Physiological Adaptation: Cognitive Level – Synthesis:
Content – Medical: Integrated Nursing Process – Concept – Immunity.
Planning: Client Needs – Physiological Integrity, Basic Care
and Comfort: Cognitive Level – Synthesis:
Concept – Mobility.

13_Colgrove_Ch13.indd 553 01/06/16 5:55 PM


554 M ed -S urg S uccess

19. 1. This will help identify people who can help Content – Medical: Integrated Nursing Process –
support the client, but it is not the highest Implementation: Client Needs – Safe Effective Care
priority. Environment, Management of Care: Cognitive Level –
2. This will help the nurse identify methods Analysis: Concept – Comfort.
which worked previously in stressful situa- 22. 1. A sunscreen with an SPF of at least 15 should
tions and may help the client deal with this be used by the client with SLE.
disease. 2. The client with SLE is at risk for infec-
3. Developing a therapeutic relationship tions and should avoid large crowds.
with the client is priority because the cli- 3. Pregnancy is not contraindicated in most
ent probably has less than six (6) months women diagnosed with SLE.
to live. All the other interventions can be 4. The client with SLE should use hypoaller-
implemented, but establishing a thera- genic products and should not use irritating
peutic relationship will allow the nurse soaps, shampoos, or chemicals.
to discuss and implement additional Content – Medical: Integrated Nursing Process –
interventions. Evaluation: Client Needs – Safe Effective Care
4. An advance directive is important, and unless Environment, Management of Care: Cognitive Level –
the client is declared legally incompetent in Synthesis: Concept – Immunity.
a court of law, the client can complete an ad-
vance directive, but establishing a therapeutic 23. 1. Epinephrine is the drug of choice for an
relationship with the client is priority. anaphylactic reaction. It is a potent vasocon-
strictor and bronchodilator counteracting the
Content – Medical: Integrated Nursing Process –
effects of histamine, but this is not the prior-
­Implementation: Client Needs – Psychosocial Integrity:
Cognitive Level – Analysis: Concept – Immunity.
ity intervention.
2. This is an important intervention, but it is
20. 1. Muscle relaxants have sedative effects, so not the priority intervention.
appropriate safety measures should be taken. 3. Decreasing the client’s anxiety is important,
2. The client should avoid central nervous but it is not the priority intervention.
system depressants because they can increase 4. Establishing a patent airway is priority
the sedative effects of the medication. because facial angioedema, bronchospasm,
3. This will help prevent constipation, which is and laryngeal edema occur with an ana-
a side effect of this medication. phylactic reaction. Inserting a nasopharyn-
4. This medication must be tapered over one geal or oropharyngeal airway is the priority
(1) to two (2) weeks when discontinuing intervention to save the client’s life.
because sudden withdrawal may cause Content – Medical/Immune System: Integrated Nursing
_seizures and paranoid ideation. Process – Implementation: Client Needs – Safe Effective
Content – Medical: Integrated Nursing Process – Care Environment, Management of Care: Cognitive
Evaluation: Client Needs – Physiological Integrity, Level – Synthesis: Concept – Immunity.
­Pharmacological and Parenteral Therapies: Cognitive
Level – Synthesis: Concept – Medication.
24. 1. The client must use measures to help
­prevent fatigue, which increases the de-
21. 1. Rheumatoid arthritis is a chronic illness, and pletion of acetylcholine and causes muscle
verbalization of feelings is helpful in dealing weakness.
with disease processes, but it is not the high- 2. The client with MG is not on strict bedrest,
est priority intervention. and impaired skin integrity is not an expected
2. This helps the client accept the disease pro- complication of this disease process, espe-
cess and body changes and helps the client cially in the early stages.
to begin to identify strategies for coping 3. Pain is not an expected complaint of clients
with them, but it is not the highest priority diagnosed with MG.
intervention. 4. A thymectomy, not a splenectomy, may be
3. Helping the client prioritize activities helps recommended. Approximately 75% of clients
the client maintain independence as long as with MG have dysplasia of the thymus gland.
possible. Content – Medical: Integrated Nursing Process –
4. Pain is priority over psychological prob- Planning: Client Needs – Physiological Integrity,
lems and activity; remember Maslow’s ­Physiological Adaptation: Cognitive Level – Synthesis:
hierarchy of needs. Concept – Immunity.

13_Colgrove_Ch13.indd 554 01/06/16 5:55 PM


C hapter 13   I mmune S ystem D isorders 555

25. 1. Pneumocystis pneumonia (PCP) occurs 26. In order of priority: 1, 3, 2, 5, 4.


in approximately 75% to 80% of clients 1. Airway is always the first priority for any
diagnosed with AIDS. Signs/symptoms of process in which the airway might be
it include fever, cough, and shortness of compromised.
breath. 3. The nurse should start an IV so medica-
2. This is an opportunistic infection, but it is tions can be administered to treat the
not the most common infection. ­anaphylactic reaction.
3. These are signs/symptoms of Mycobacterium 2. Epinephrine is the drug of choice for the
avium complex (MAC), which affects up to treatment of anaphylaxis. The medication
25% of client’s with AIDS, but it is not the is administered every 10 to 15 minutes
most common opportunistic infection. until the reaction has subsided. Epineph-
4. These are signs/symptoms of Kaposi’s sar- rine is given for its vasoconstrictive action.
coma, which is the most common cancer 5. Benadryl, an antihistamine, is given to
associated with AIDS; it is not an infectious block histamine release, reducing capillary
disease. permeability.
Content – Medical: Integrated Nursing Process – 4. Teaching is important to prevent or treat
Assessment: Client Needs – Physiological Integrity, further reactions, but this will be done
Reduction of Risk Potential: Cognitive Level – Analysis: ­after the crisis is over.
Concept – Immunity. Content – Medical: Integrated Nursing Process –
Implementation: Client Needs – Safe Effective Care
­Environment, Management of Care: Cognitive Level –
Analysis: Concept – Immunity.

13_Colgrove_Ch13.indd 555 01/06/16 5:55 PM


13_Colgrove_Ch13.indd 556 01/06/16 5:55 PM
Sensory Deficits
The eye sees what the mind is prepared to comprehend.
14
—Henri Bergson

Some sensory deficits occur with normal aging, whereas others are the result of specific
disease processes. Problems involving the senses—sight, hearing, smell, taste, touch—arise
from many sources. Some are inflammatory or infectious, or both (otitis); some are the
result of a specific disease process (cataract, glaucoma); and still others are the result of the
normal aging process (decreased peripheral vision, decreased sense of smell). Many of these
diseases/disorders can be treated with appropriate medical-surgical interventions. Still oth-
ers can be addressed through safety precautions (smoke alarms) and assistive devices (hear-
ing aids). The nurse must be familiar with how to assess the sensory system, identify specific
problems, and help in the care of clients with these problems.

KEYWORDS ABBREVIATIONS
Amsler grid Head of bed (HOB)
Cataract Health-care provider (HCP)
Enucleation Intravenous (IV)
Glaucoma Laser-assisted in situ keratomileusis (LASIK)
Intraocular
Macular degeneration
Ménière’s disease
Myopia
Nephrotoxic
Nystagmus
Otitis
Otorrhea
Otoscope
Ototoxic
Perforation
Presbycusis
Ptosis
Retrobulbar
Snellen chart
Stapedectomy
Tinnitus
Tympanic membrane
Vertigo

557

14_Colgrove_Ch14.indd 557 01/06/16 5:56 PM


558 M ed -S urg S uccess

PRACTICE QUESTIONS
Eye Disorders 6. The employee health nurse is teaching a class
on “Preventing Eye Injury.” Which information
1. The client is diagnosed with glaucoma. Which should be discussed in the class?
symptom should the nurse expect the client to 1. Read instructions thoroughly before using
report? tools and working with chemicals.
1. Loss of peripheral vision. 2. Wear some type of glasses when working
2. Floating spots in the vision. around flying fragments.
3. A yellow haze around everything. 3. Always wear a protective helmet with eye
4. A curtain coming across vision. shield around dust particles.
4. Pay close attention to the surroundings so eye
2. The client is scheduled for right-eye cataract injuries will be prevented.
removal surgery in five (5) days. Which
preoperative instruction should be discussed with 7. The 65-year-old male client who is complaining
the client? of blurred vision reports he thinks his glasses
1. Administer dilating drops to both eyes for 72 need to be cleaned all the time, and he denies
hours prior to surgery. any type of eye pain. Which eye disorder should
2. Prior to surgery do not lift or push any objects the nurse suspect the client has?
heavier than 15 pounds. 1. Corneal dystrophy.
3. Make arrangements for being in the hospital 2. Conjunctivitis.
for at least three (3) days. 3. Diabetic retinopathy.
4. Avoid taking any type of medication which may 4. Cataracts.
cause bleeding, such as aspirin. 8. The nurse is administering eyedrops to the
3. The client is postoperative retinal detachment client. Which guidelines should the nurse adhere
surgery, and gas tamponade was used to flatten to when instilling the drops into the eye? Select
the retina. Which intervention should the nurse all that apply.
implement first? 1. Do not touch the tip of the medication
1. Teach the signs of increased intraocular container to the eye.
pressure. 2. Apply gentle pressure on the outer canthus of
2. Position the client as prescribed by the the eye.
surgeon. 3. Apply sterile gloves prior to instilling
3. Assess the eye for signs/symptoms of eyedrops.
complications. 4. Hold the lower lid down and instill drops into
4. Explain the importance of follow-up visits. the conjunctiva.
5. Gently pat the skin to absorb excess eyedrops
4. The 65-year-old client is diagnosed with macular on the cheek.
degeneration. Which statement by the client
indicates the client needs more discharge 9. The client has had an enucleation of the left
teaching? eye. Which intervention should the nurse
1. “I should use magnification devices as much as implement?
possible.” 1. Discuss the need for special eyeglasses.
2. “I will look at my Amsler grid at least twice a 2. Refer the client for an ocular prosthesis.
week.” 3. Help the client obtain a seeing-eye dog.
3. “I need to use low-watt light bulbs in my 4. Teach the client how to instill eyedrops.
house.” 10. The client diagnosed with glaucoma is
4. “I am going to contact a low-vision center to prescribed a miotic cholinergic medication.
evaluate my home.” Which data indicate the medication has been
5. The nurse who is at a local park sees a young effective?
man on the ground who has fallen and has a stick 1. No redness or irritation of the eyes.
lodged in his eye. Which intervention should the 2. A decrease in intraocular pressure.
nurse implement at the scene? 3. The pupil reacts briskly to light.
1. Carefully remove the stick from the eye. 4. The client denies any type of floaters.
2. Stabilize the stick as best as possible.
3. Flush the eye with water if available.
4. Place the young man in a high-Fowler’s
position.

14_Colgrove_Ch14.indd 558 01/06/16 5:56 PM


C hapter 14   S ensory D eficits 559

11. The client is scheduled for laser-assisted in 3. Instill ophthalmic drops in both ears and then
situ keratomileusis (LASIK) surgery for severe insert a cotton ball.
myopia. Which instruction should the nurse 4. Do not allow water to enter the ear for six
discuss prior to the client’s discharge from day (6) weeks.
surgery?
17. The client is diagnosed with Ménière’s
1. Wear bilateral eye patches for three (3) days.
disease. Which statement indicates the client
2. Wear corrective lenses until the follow-up
understands the medical management for this
visit.
disease?
3. Do not read any material for at least one (1)
1. “After intravenous antibiotic therapy, I will be
week.
cured.”
4. Teach the client how to instill corticosteroid
2. “I will have to use a hearing aid for the rest of
ophthalmic drops.
my life.”
12. The client comes to the emergency department 3. “I must adhere to a low-sodium diet,
after splashing chemicals into the eyes. Which 2,000 mg/day.”
intervention should the nurse implement first? 4. “I should sleep with the head of my bed
1. Have the client move the eyes in all elevated.”
directions.
18. The client is complaining of ringing in the ears.
2. Administer a broad-spectrum antibiotic.
Which data are most appropriate for the nurse
3. Irrigate the eyes with normal saline solution.
to document in the client’s chart?
4. Determine when the client had a tetanus shot.
1. Complaints of vertigo.
2. Complaints of otorrhea.
Ear Disorders 3. Complaints of tinnitus.
4. Complaints of presbycusis.
13. Which statement indicates to the nurse the 19. Which statement best describes the scientific
client is experiencing some hearing loss? rationale for the nurse holding the otoscope
1. “I clean my ears every day after I take a with the hand in a pencil-hold position when
shower.” examining the client’s ear?
2. “I keep turning up the sound on my 1. It is usually the most comfortable position to
television.” hold the otoscope.
3. “My ears hurt, especially when I yawn.” 2. This allows the best visualization of the
4. “I get dizzy when I get up from the chair.” tympanic membrane.
14. Which risk factors should the nurse discuss with 3. This prevents inserting the otoscope too far
the client concerning reasons for hearing loss? into the external ear.
Select all that apply. 4. It ensures the nurse will not cause pain when
1. Perforation of the tympanic membrane. examining the ear.
2. Chronic exposure to loud noises. 20. The nurse is preparing to administer otic
3. Recurrent ear infections. drops into an adult client’s right ear. Which
4. Use of nephrotoxic medications. intervention should the nurse implement?
5. Multiple piercings in the auricle. 1. Grasp the earlobe and pull back and out when
15. The nurse is caring for a client diagnosed with putting drops in the ear.
acute otitis media. Which signs/symptoms 2. Insert the eardrops without touching the
support this medical diagnosis? outside of the ear.
1. Unilateral pain in the ear. 3. Instruct the client to close the mouth and
2. Green, foul-smelling drainage. blow prior to instilling drops.
3. Sensation of congestion in the ear. 4. Pull the auricle down and back prior to
4. Reports of hearing loss. instilling drops.

16. The client diagnosed with chronic otitis media is 21. Which ototoxic medication should the
scheduled for a mastoidectomy. Which discharge nurse recognize as potentially life altering or
teaching should the nurse discuss with the threatening to the client?
client? 1. An oral calcium channel blocker.
1. Instruct the client to blow the nose with the 2. An intravenous aminoglycoside antibiotic.
mouth closed. 3. An intravenous glucocorticoid.
2. Explain the client will never be able to hear 4. An oral loop diuretic.
from the ear.

14_Colgrove_Ch14.indd 559 01/06/16 5:56 PM


560 M ed -S urg S uccess

22. Which teaching instruction should the nurse 24. The client is scheduled for ear surgery. Which
discuss with students who are on the high school statement indicates the client needs more
swim team when discussing how to prevent preoperative teaching concerning the surgery?
external otitis? 1. “If I have to sneeze or blow my nose, I will do
1. Do not wear tight-fitting swim caps. it with my mouth open.”
2. Avoid using silicone ear plugs while 2. “I may get dizzy after the surgery, so I must be
swimming. careful when walking.”
3. Use a drying agent in the ear after swimming. 3. “I will probably have some hearing loss after
4. Insert a bulb syringe into each ear to remove surgery, but hearing will return.”
excess water. 4. “I can shampoo my hair the day after surgery
as long as I am careful.”
23. The client comes to the clinic and is diagnosed
with otitis media. Which intervention should the
clinic nurse include in the discharge teaching?
1. Instruct the client not to take any over-the-
counter pain medication.
2. Encourage the client to apply cold packs to
the affected ear.
3. Tell the client to call the HCP if an abrupt
relief of ear pain occurs.
4. Wear a protective ear plug in the affected ear.

CONCEPTS
The concepts covered in this chapter focus on sensory perception. Exemplars that are covered are eye and ear
disorders. Interrelated concepts of the nursing process and critical thinking are covered throughout the questions.
The concept of critical thinking is presented in the prioritizing or “first” questions.
25. The nurse is observing the client administer the 26. The nurse is administering eardrops to a
prescribed eyedrops. Which intervention should six (6)-year-old client. Which indicates the nurse
the nurse implement? is aware of the correct method for instilling
eardrops to a child?
1. Pull the pinna upward only to instill the
eardrops.
2. Pull the pinna to a neutral position to instill
the eardrops.
3. Pull the pinna upward and backward prior to
instilling the drops.
4. Pull the pinna downward and forward to
instill the drops.

1. Praise the client for instilling the eyedrops as


recommended.
2. Remind the client to instill the eyedrops from
0.4 to 0.8 inch above the eye.
3. Ask the client if the eyedrops have been
warmed to room temperature.
4. Teach the client to instill the eyedrops in the
upper conjunctival sac.

14_Colgrove_Ch14.indd 560 01/06/16 5:56 PM


C hapter 14   S ensory D eficits 561

27. The nurse is instilling eye ointment. Which 30. The elderly client has undergone a right-eye
should the nurse perform prior to instilling the cataract removal with an intraocular implant.
medication depicted in the image? Which discharge instructions should the nurse
teach the client?
1. Have the client demonstrate placing the otic
drops in the ear.
2. Teach the client to instill the eyedrops as
prescribed.
3. Remind the client to keep the lights in the
home low at all times.
4. Encourage the client to sleep on two pillows
at night.
31. The nurse is assessing a client who has a
“pinpoint” pupil reaction bilaterally and the
1. Have the client close the eye tightly to rid the pupils do not constrict when the light is shown
eye of tears. on the eye. Which should the nurse document?
2. Place the nurse’s nondominant hand on the
client’s eyebrow.
3. Discard the first bead of ointment, then instill
the ointment.
4. Ask the client to look down toward the floor. 1 2 3 4 5 6 7 8 9 10

28. The nurse is assessing a client and performs


1. Pupillary response poor.
a whisper test. Which should the nurse
2. Pupils one (1) mm, equal and nonreactive to
implement? Rank in order of performance.
light.
1. Have the client cover the ear not being tested.
3. Pupils two (2) to three (3) mm and
2. Stand 12 to 24 inches to the side of the client.
nonconstrictive to light.
3. Explain to the client to repeat what the nurse
4. Pupils are barely open and don’t constrict to
says.
light.
4. Repeat the test for the opposite ear.
5. Ask the client if he/she is willing to participate 32. The emergency department nurse is assessing a
in the test. client who has a needle in the sclera of the right
eyeball just below the iris. Which should the
29. The nurse is caring for a client diagnosed
nurse implement first?
with a cerebrovascular accident (CVA). Which
1. Remove the needle with tweezers.
assessment information should the nurse
2. Notify an ophthalmologist to care for the
determine first when placing the client in the
client.
assigned room?
3. Stabilize the right eye and place a patch over
1. Determine if the client has loss of vision in
the left eye.
the same half of each visual field.
4. Irrigate the right eye to wash the needle out
2. Find out if the client prefers the bed by the
of the eye.
window or by the bathroom.
3. Request dietary to place the meat at 1200 on
each plate and vegetables at 0900 and 1500.
4. Request a physical therapy consult to assess
the client’s mobility issues.

14_Colgrove_Ch14.indd 561 01/06/16 5:56 PM


PRACTICE QUESTIONS ANSWERS AND RATIONALES

Eye Disorders intraocular pressure, e ­ ndophthalmitis, de-


velopment of another retinal detachment,
1. 1. In glaucoma, the client is often unaware he or loss of turgor in the eye.
or she has the disease until the client experi- 4. Follow-up visits are important, but this is
ences blurred vision, halos around lights, dif- not the first intervention the nurse should
ficulty focusing, or loss of peripheral vision. implement.
Glaucoma is often called the “silent thief.” TEST-TAKING HINT: When the question asks
2. Floating spots in the vision is a symptom of which intervention should be implemented
retinal detachment. first, all four (4) answer options are possible
3. A yellow haze around everything is a com- interventions but only one (1) should be im-
plaint of clients experiencing digoxin toxicity. plemented first. Remember to apply the nurs-
4. The complaint of a curtain coming across vi- ing process to help select the correct answer.
sion is a symptom of retinal detachment. Assessment is the first part of the nursing
TEST-TAKING HINT: The signs/symptoms of eye process.
disorders are confusing. The test taker must Content – Medical: Integrated Nursing Process –
know which complaints will be made by the Implementation: Client Needs – Safe Effective Care
client with a specific eye disorder. ­Environment, Management of Care: Cognitive Level –
­Application: Concept – Sensory Perception.
Content – Medical: Integrated Nursing Process –
­Assessment: Client Needs – Physiological Integrity, Reduction 4. 1. Magnifying devices used with activities such as
of Risk Potential: Cognitive Level – Analysis: Concept – threading a needle will help the client’s vision;
­Sensory Perception. therefore, this statement does not indicate the
2. 1. Dilating drops are administered every client needs more teaching.
10 minutes for four (4) doses one (1) hour prior 2. An Amsler grid is a tool to assess macular de-
to surgery, not for three (3) days prior to surgery. generation, often providing the earliest sign of
2. Lifting and pushing objects should be avoided a worsening condition. If the lines of the grid
after surgery, not prior to surgery. become distorted or faded, the client should
3. All types of cataract removal surgery are usu- call the ophthalmologist.
ally done in day surgery. 3. Macular degeneration is the most com-
4. To reduce retrobulbar hemorrhage, mon cause of visual loss in people older
any ­anticoagulation therapy is withheld, than age 60 years. Any intervention which
­including aspirin, nonsteroidal anti- helps increase vision should be included in
inflammatory drugs (NSAIDs), and the teaching, such as bright lighting, not
warfarin (Coumadin). ­decreased lighting.
TEST-TAKING HINT: The test taker must notice
4. Low-vision centers will send representatives
the adjectives; these descriptors are impor- to the client’s home or work to make recom-
tant when selecting a correct answer. The test mendations about improving lighting, thereby
taker should notice “preoperative” and “prior improving the client’s vision and safety.
to surgery.” TEST-TAKING HINT: This question is asking
Content – Surgical: Integrated Nursing Process – which statement indicates more teaching
­Planning: Client Needs – Physiological Integrity, Reduction is needed. Therefore, three (3) options will
of Risk Potential: Cognitive Level – Synthesis: Concept – ­indicate the client understands appropri-
­Sensory Perception. ate discharge teaching and only one (1) will
indicate the client does not understand the
3. 1. This should be done, but it is not the first
teaching.
­intervention the nurse should implement.
Content – Medical: Integrated Nursing Process –
2. The client will have to be specifically posi-
­Evaluation: Client Needs – Safe Effective Care Environment,
tioned to make the gas bubble float into the
Management of Care: Cognitive Level – Synthesis:
best position; some clients must lie face down Concept – Sensory Perception.
or on their side for days, but it is not the first
intervention. 5. 1. A foreign object should never be removed at
3. The nurse’s priority must be assessment the scene of the accident because this may
of complications, which include increased cause more damage.

562

14_Colgrove_Ch14.indd 562 01/06/16 5:56 PM


C hapter 14   S ensory D eficits 563

2. The foreign object should be stabilized to 4. A cataract is a lens opacity or cloudiness,


prevent further movement, which could resulting in the signs/symptoms discussed
cause more damage to the eye. in the stem of the question.
3. Flushing with water may cause further move- TEST-TAKING HINT: The test taker must
ment of the foreign object and should be know the signs/symptoms of eye disorders,
avoided. ­especially those commonly occurring in the
4. The person should be kept flat and not in a sit- e­ lderly. ­Option “2” could be ruled out
ting position because it may dislodge or cause because -itis means inflammation and none
movement of the foreign object. of the signs/symptoms is inflammatory.
TEST-TAKING HINT: In an emergency situation, Content – Medical: Integrated Nursing Process –
the first responder should first “do no harm.” ­Diagnosis: Client Needs – Safe Effective Care Environment,
The test taker should examine each option Management of Care: Cognitive Level – Analysis:
and decide what will happen if this option is Concept – Sensory Perception.
performed—will it help, harm, or stabilize the 8. 1. Touching the tip of the container to
client? If the test taker determines one (1) ac- the eye may cause eye injury or an eye
tion may not help, then stabilization becomes infection.
the priority. 2. Gentle pressure should be applied on the inner
Content – Medical: Integrated Nursing Process – canthus, not outer canthus, near the bridge of
Implementation: Client Needs – Safe Effective Care the nose for one (1) or two (2) minutes after
­Environment, Management of Care: Cognitive Level –
instilling eyedrops.
­Application: Concept – Sensory Perception.
3. The nurse should wash hands prior to and
6. 1. Instructions provide precautions and steps after instilling medications; this is not a sterile
to take if eye injuries occur secondary to procedure.
the use of tools or chemicals. 4. Medication should not be placed directly
2. The employee must wear safety glasses, not on the eye but in the lower part of the
just any type of glasses and especially not regu- eyelid.
lar prescription glasses. 5. Eyedrops are meant to go in the eye, not
3. A protective helmet is used to help prevent on the skin, so the nurse should use a clean
sports eye injuries, not work-related injuries. tissue to remove excess medication.
4. Eye injuries will not be prevented by paying TEST-TAKING HINT: This is an alternate-type
close attention to the surroundings. They are question requiring the test taker to select all
prevented by wearing protective glasses or eye the correct options. The test taker should not
shields. second-guess the question. All five (5) options
TEST-TAKING HINT: The test taker must make can be selected or only one (1). The test taker
sure what the question is asking and must pay should read each option, and if it is correct,
close attention to adjectives. An “employee select it.
health nurse” is in the workplace. If the test Content – Medical: Integrated Nursing Process –
taker is going to select an option with a word ­Implementation: Client Needs – Safe Effective Care
such as “always,” “never,” or “only,” he or she ­
Environment, Management of Care: Cognitive Level –
must be absolutely sure it is an intervention Knowledge: Concept – Sensory Perception.
never questioned. In health care, there are 9. 1. Special eyeglasses are not needed for an
very few absolutes. enucleation.
Content – Medical: Integrated Nursing Process – Planning: 2. An enucleation is the removal of the entire
Client Needs – Health Promotion and Maintenance: eye and part of the optic nerve. An ocular
Cognitive Level – Synthesis: Concept – Sensory Perception.
prosthesis will help maintain the shape of
7. 1. Corneal dystrophy is an inherited eye disorder the eye socket after the enucleation.
occurring at about age 20 and results in de- 3. The client had the left eye removed but is not
creased vision and the development of blisters; blind because he or she still has the right eye.
it is usually associated with primary open-angle 4. The eyeball was totally removed and a pres-
glaucoma. sure dressing was applied; therefore, there will
2. Conjunctivitis is an inflammation of the be no need to instill eyedrops.
­conjunctiva, which results in a scratching or TEST-TAKING HINT: In some questions, the test
burning sensation, itching, and photophobia. taker must know the definition of the word
3. Diabetic retinopathy results from deterioration (“enucleation”) to be able to apply it in a clinical
of the small blood vessels nourished by the situation.
retina; it leads to blindness.

14_Colgrove_Ch14.indd 563 01/06/16 5:56 PM


564 M ed -S urg S uccess

Content – Medical: Integrated Nursing Process – 4. Tetanus prophylaxis is recommended for


Implementation: Client Needs – Safe Effective Care ­
full-thickness ocular wounds.
­Environment, Management of Care: Cognitive Level –
TEST-TAKING HINT: If the test taker is not sure
­Application: Concept – Sensory Perception.
of the answer, the test taker should select the
10. 1. Steroid medication is administered to answer directly addressing the ­client’s condi-
decrease inflammation. tion. Options “1” and “3” directly affect the
2. Both systemic and topical medications are eyes, but when choosing between these two
used to decrease the intraocular pressure options, the test taker should ask, “How will
in the eye, which causes glaucoma. moving the eyes help treat the eyes?” and
3. Glaucoma does not affect the pupillary then eliminate option “1.”
reaction. Content – Medical: Integrated Nursing Process –
4. Floaters are a complaint of clients with ­retinal Implementation: Client Needs – Safe Effective Care
detachment. ­Environment, Management of Care: Cognitive Level –
TEST-TAKING HINT: To determine the ef- ­Application: Concept – Sensory Perception.
fectiveness of a medication, the nurse must
know the signs/symptoms of the disease pro-
cess. If the test taker knew glaucoma was the
Ear Disorders
result of an increase in intraocular pressure,
13. 1. Cleaning the ears daily does not indicate the
then the medication is effective if there was a
client has a hearing loss.
decrease in intraocular pressure.
2. The need to turn up the volume on the
Content – Medical: Integrated Nursing Process – television is an early sign of hearing
­Assessment: Client Needs – Physiological Integrity,
impairment.
­Pharmacological and ­Parenteral Therapies: Cognitive
3. Pain in the ears is not a clinical manifestation
Level – Analysis: Concept – ­Sensory Perception.
of hearing loss/impairment.
11. 1. The client does not have to wear eye patches 4. This statement may indicate a balance prob-
after this surgery. lem secondary to an ear disorder, but it does
2. The purpose of this surgery is to ensure the not indicate a hearing loss.
client does not have to wear any type of cor- TEST-TAKING HINT: If the test taker has no
rective lens. idea of the answer, option “2” is the only
3. The client can read immediately after this ­answer which has anything to do with sound.
surgery.
Content – Medical: Integrated Nursing Process –
4. LASIK surgery is an effective, safe, ­Assessment: Client Needs – Physiological Integrity,
­predictable surgery performed in day ­Reduction of Risk Potential: Cognitive Level – Analysis:
­surgery; there is minimal postoperative Concept – Sensory Perception.
care. ­Instilling topical corticosteroid drops
helps decrease inflammation and edema of 14. 1. The tympanic membrane is the eardrum,
the eye. and if it is punctured it may lead to hear-
ing loss.
TEST-TAKING HINT: Option “3” has the
2. Loud persistent noise, such as heavy
­absolute word “any,” so the test taker could
­machinery, engines, and artillery, over time
eliminate it. LASIK is a corrective surgery,
may cause noise-induced hearing loss.
and if the problem is corrected, then correc-
3. Multiple ear infections scar the tympanic
tive lenses should not be necessary.
membrane, which can lead to hearing loss.
Content – Surgical: Integrated Nursing Process – 4. Nephrotoxic means harmful to the kidneys;
Planning: Client Needs – Physiological Integrity,
ototoxic is harmful to the ears.
­Physiological Adaptation: Cognitive Level – Synthesis:
5. Multiple pierced earrings do not lead to ­hearing
Concept – Perioperative.
loss. The auricle (skin attached to the head) is
12. 1. Movement of the eye should be avoided until composed mainly of cartilage, except for the fat
the client has received general anesthesia; and subcutaneous tissue in the earlobe.
therefore, this is not the first intervention. TEST-TAKING HINT: This alternate-type
2. Parenteral broad-spectrum antibiotics are ­question requires the test taker to select
­initiated but not until the eyes are treated multiple correct answers. Many options can
first. be eliminated as incorrect answers when
3. Before any further evaluation or treat- the test taker knows medical terminology—
ment, the eyes must be thoroughly flushed nephro- means kidney-related—and normal
with sterile normal saline solution. anatomy of the body—auricle means “skin
­attached to the head.”

14_Colgrove_Ch14.indd 564 01/06/16 5:56 PM


C hapter 14   S ensory D eficits 565

Content – Medical: Integrated Nursing Process – TEST-TAKING HINT: Sleeping with the HOB
­Diagnosis: Client Needs – Safe Effective Care Environ- elevated is not a medical treatment; there-
ment, Management of Care: Cognitive Level – Knowledge: fore, option “4” can be eliminated as a
Concept – Sensory Perception. ­possible answer. The test taker must read the
15. 1. Otalgia (ear pain) is experienced by c­ lients stem carefully.
with otitis media. Content – Medical: Integrated Nursing Process – ­
2. A green, foul-smelling drainage supports the Evaluation: Client Needs – Safe Effective Care
diagnosis of external otitis, not of acute otitis ­Environment, Management of Care: Cognitive Level –
media. Synthesis: Concept – Sensory Perception.
3. A sensation of congestion in the ear supports 18. 1. Vertigo is an illusion of movement in which
serous otitis media. the client complains of dizziness.
4. Hearing loss supports a diagnosis of chronic 2. Otorrhea is drainage of the ear.
otitis media or serous otitis media. 3. Tinnitus is “ringing of the ears.” It is a
TEST-TAKING HINT: If the test taker were not subjective perception of sound with inter-
sure of the answer, the adjective “acute” in nal origins.
the stem should cause the test taker to think 4. Presbycusis is progressive hearing loss associ-
“pain,” which is included in option “1.” ated with aging.
Content – Medical: Integrated Nursing Process – TEST-TAKING HINT: The test taker who is
­Assessment: Client Needs – Physiological Integrity, ­familiar with medical terminology can rule
­Reduction of Risk Potential: Cognitive Level – Analysis: out options based on the understanding of
Concept – Sensory Perception. medical terms.
16. 1. The client should blow the nose with the Content – Medical: Integrated Nursing Process –
mouth open to prevent pressure in the eusta- ­Implementation: Client Needs – Safe Effective Care
chian tube. Environment, Management of Care: Cognitive Level –
2. There may be temporary deafness as a result Application: Concept – Sensory Perception.
of postoperative edema, but the hearing will 19. 1. This is not the rationale for holding the
return as the edema subsides. ­otoscope in this manner.
3. Ophthalmic drops are used in the eyes, not 2. Holding the otoscope in this manner does
the ears. Otic drops are used for the ears. not help visualize the membrane any better
4. Water should be prevented from enter- than holding the otoscope in other ways.
ing the external auditory canal because it 3. Inserting the speculum of the otoscope
may irritate the surgical incision and is a into the external ear can cause ear trauma
­medium for bacterial growth. if not done correctly.
TEST-TAKING HINT: The test taker must be 4. If the ear is inflamed, it may be impossible to
aware of adjectives. In option “3,” the test prevent hurting the client on examination.
taker should know “ophthalmic” refers to the TEST-TAKING HINT: The scientific rationale is
eye, which causes the test taker to eliminate the critical-thinking component of nursing;
this as a p
­ ossible answer. the test taker must understand the “why” of
Content – Surgical: Integrated Nursing Process – nursing interventions.
­Planning: Client Needs – Physiological Integrity, Reduction Content – Medical: Integrated Nursing Process –
of Risk Potential: Cognitive Level – Synthesis: Concept – ­Diagnosis: Client Needs – Safe Effective Care Environment,
Sensory Perception. Management of Care: Cognitive Level – Analysis:
17. 1. Antibiotics will not cure this disease. Surgery Concept – Sensory Perception.
is the only cure for Ménière’s disease, which 20. 1. This is not the correct way to administer
may result in permanent deafness as a result eardrops.
of the labyrinth being removed in the surgery. 2. The nurse must straighten the ear canal;
2. Ménière’s disease does not lead to deafness therefore, the outside of the ear must be
unless surgery is performed removing the moved.
labyrinth in attempts to eliminate the attacks 3. This will increase pressure in the ear and
of vertigo. should not be done prior to administering
3. Sodium regulates the balance of fluid eardrops.
within the body; therefore, a low-sodium 4. This will straighten the ear canal so the
diet is prescribed to help control the eardrops will enter the ear canal and
symptoms of Ménière’s disease. drain toward the tympanic membrane
4. Sleeping with the head of the bed elevated (eardrum).
will not affect Ménière’s disease.

14_Colgrove_Ch14.indd 565 01/06/16 5:56 PM


566 M ed -S urg S uccess

TEST-TAKING HINT: The test taker should no- Content – Medical: Integrated Nursing Process – ­
tice options “1” and “4” are opposite, which Planning: Client Needs – Health Promotion and
should clue the test taker into either elimi- ­Maintenance: Cognitive Level – Synthesis: Concept –
nating both or deciding one (1) of these two Sensory Perception.
(2) is the correct answer. Either way, the test 23. 1. Mild analgesics such as aspirin or acetamino-
taker now has a 50/50 chance of selecting the phen every four (4) hours as needed to relieve
correct answer. pain and fever are recommended; aspirin may
Content – Medical: Integrated Nursing Process – help decrease inflammation of the ear.
­Implementation: Client Needs – Physiological Integrity, 2. Heat applied to the affected ear is recom-
Pharmacological and Parenteral Therapies: Cognitive mended because heat dilates blood vessels,
Level – Application: Concept – Medication. promoting the reabsorption of fluid and
21. 1. Calcium channel blockers are not going to ­reducing edema.
­affect the client’s hearing. 3. Pain subsiding abruptly may indicate
2. Aminoglycoside antibiotics are ototoxic. spontaneous perforation of the tympanic
Overdosage of these medications can membrane within the middle ear and
cause the client to go deaf, which is why should be reported to the HCP.
peak and trough serum levels are drawn 4. Ear plugs should not be used in clients with
while the client is taking a medication of otitis media, but cotton balls could be used to
this type. These antibiotics are also very keep otic antibiotics in the ear canal.
nephrotoxic. TEST-TAKING HINT: The test taker must use
3. Steroids cause many adverse effects, but basic principles when answering questions.
­damage to the ear is not one of them. Cold causes constriction and heat dilates.
4. Administering an intravenous push loop Except for aspirin not being administered to
­diuretic too fast can cause auditory nerve children to prevent Reye’s syndrome, mild
damage, but an oral loop diuretic does not. analgesics can be administered for almost
TEST-TAKING HINT: The test taker must be any discomfort.
cautious of adjectives. The word “oral” in Content – Medical: Integrated Nursing Process – ­
­option “4” eliminates this option as a possible Planning: Client Needs – Physiological Integrity,
correct answer. ­Physiological Adaptation: Cognitive Level – Synthesis:
Concept – Sensory Perception.
Content – Medical: Integrated Nursing Process –
­Implementation: Client Needs – Physiological Integrity, 24. 1. Leaving the mouth open when coughing or
Pharmacological and Parenteral Therapies: Cognitive sneezing will minimize the pressure changes
Level – Application: Concept – Medication. in the middle ear.
22. 1. Tight-fitting swim caps or wetsuit hoods 2. Surgery on the ear may disrupt the client’s
should be worn because they prevent water equilibrium, increasing the risk for falling.
from entering the ear canal. 3. Hearing loss secondary to postoperative
2. Silicone ear plugs should be worn because edema is common after surgery, but the hear-
they keep water from entering the ear canal ing will return after the edema subsides.
without reducing hearing significantly. 4. Shampooing, showering, and immersing
3. A 2% acetic acid solution or 2% boric the head in water are avoided to prevent
acid in ethyl alcohol is effective in drying contamination of the ear canal; therefore,
the canal and restoring its normal acidic this comment indicates the client does not
environment. understand the preoperative teaching.
4. A bulb syringe with a Teflon catheter can be TEST-TAKING HINT: This is an “except” ques-
used to remove impacted debris from the ear, tion. The stem states “needs more teaching”;
but it is not used to remove excess water. therefore, three (3) of the options reflect an
TEST-TAKING HINT: If the test taker has no appropriate understanding of the teaching
idea what the correct answer is, the test taker and only one (1) indicates a misunderstand-
should evaluate the answer options to see if ing of the teaching.
two are similar. In this question, both options Content – Surgical: Integrated Nursing Process –
“1” and “2” say to not use ear protectors. Evaluation: Client Needs – Physiological Integrity, Physi-
Because there cannot be two correct answers, ological Adaptation: Cognitive Level – Synthesis: Concept –
these two could be eliminated as possible Perioperative.
correct answers.

14_Colgrove_Ch14.indd 566 01/06/16 5:56 PM


CONCEPTS

25. 1. The client is holding the dropper too high. 4. The client should look upward to reduce the
Eyedrops are instilled from one (1) to two amount of blinking during administration.
(2) cm (0.4 to 0.8 inch) above the eye. TEST-TAKING HINT: The test taker must re-
2. Eyedrops are instilled from one (1) to member basics of medication administration.
two (2) cm (0.4 to 0.8 inch) above the eye. Content – Medical: Integrated Nursing Process –
The client should not hold the dropper ­Implementation: Client Needs – Pharmacological Integrity,
too high. Reduction of Risk Potential: Cognitive Level – Cognition:
3. Eyedrops can be instilled at room tempera- Concept – Sensory Perception.
ture or chilled. If a client has trouble recog-
nizing if the drops have been instilled, the 28. In order of performance: 5, 3, 1, 2, 4
nurse can recommend refrigerating the drops 5. The client should be offered the oppor-
so the client can feel when the eyedrops have tunity to agree to being tested before any
been administered. further action is taken.
4. Eyedrops are administered in the lower 3. The nurse should give directions as to
­conjunctival sac. what the client is expected to do when he/
she hears what the nurse says.
TEST-TAKING HINT: The test taker must
1. The client covers the ear not being tested
­remember basics of medication administration. after the nurse has explained the test.
Content – Medical: Integrated Nursing Process – 2. The nurse should stand to the side but
­Implementation: Client Needs – Pharmacological Integrity, not until talking directly to the client.
Reduction of Risk Potential: Cognitive Level – Cognition:
4. One ear at a time is tested.
Concept – Sensory Perception.
TEST-TAKING HINT: This is a basic assessment
26. 1. The pinna should be pulled upward and tool to determine sensory perception. The
backward for all clients three (3) years of age test taker should memorize basic tests and
and older. Prior to three (3) years of age the normal results.
pinna is directed upward only. Content – Medical: Integrated Nursing Process –
2. The pinna should be pulled upward and ­Assessment: Client Needs – Physiological Integrity,
backward for all clients three (3) years of age ­Reduction of Risk Potential: Cognitive Level – Cognition:
and older. Prior to three (3) years of age the Concept – Sensory Perception.
pinna is directed upward only.
3. The pinna should be pulled upward and 29. 1. Homonymous hemianopsia (blindness
backward for all clients three (3) years of in the same half of each visual field) is a
age and older. Prior to three (3) years of common problem after a stroke. Clients
age the pinna is directed upward only. disregard objects in that part of the visual
4. The pinna should be pulled upward and field. The nurse would want to place the
backward for all clients three (3) years of age client in a room with the bed positioned
and older. Prior to three (3) years of age the so that the client will know when some-
pinna is directed upward only. one is entering the room.
2. Client preference can be taken into consider-
TEST-TAKING HINT: The test taker must re-
ation but is not a priority.
member basics of medication administration. 3. Requesting dietary to place foods in a certain
Content – Medical: Integrated Nursing Process – order will assist the client with visual distur-
Implementation: Client Needs – Pharmacological Integrity, bances to know where to find the food on the
Reduction of Risk Potential: Cognitive Level – Cognition:
plate but is not first.
Concept – Sensory Perception.
4. Physical therapy may need to assess the
27. 1. The client should close the eye gently after ­client, but it is not first.
the ointment is instilled in order for the TEST-TAKING HINT: The signs/symptoms of
­eyelid to spread the ointment over the eye. CVA vary and some of the clinical manifes-
2. The nurse’s nondominant thumb or fingers tations may not be immediately observable.
are placed on the cheekbone to pull the lid The brain impacts the entire body. The test
down to expose the conjunctival sac. taker must know which symptoms impact the
3. The first bead of ointment is considered client’s daily life.
contaminated and should be discarded.

567

14_Colgrove_Ch14.indd 567 03/06/16 11:07 AM


568 M ed -S urg S uccess

Content – Medical: Integrated Nursing Process – TEST-TAKING HINT: The signs/symptoms of


­Assessment: Client Needs – Physiological Integrity, eye disorders can be confusing but the nurse
­Reduction of Risk Potential: Cognitive Level – Synthesis: must communicate in professional terminol-
Concept – Sensory Perception. ogy that all medical personnel can interpret
30. 1. Otic drops go in the ear, not the eye. consistently.
2. Postoperatively the client will be prescribed Content – Medical: Integrated Nursing Process –
eyedrops for several weeks; the nurse Implementation: Client Needs – Physiological Integrity,
should teach the client to administer as Reduction of Risk Potential: Cognitive Level – Synthesis:
prescribed. Concept – Sensory Perception.
3. The light should be brighter for safety. 32. 1. The nurse should leave the needle where it is
4. The client does not need to sleep with the but try to make sure the client does not move
HOB elevated. the eye. The HCP will be the one to remove
TEST-TAKING HINT: The test taker must know the obstacle.
­basic instructions for postoperative clients. 2. The ophthalmologist will need to be notified
Content – Medical: Integrated Nursing Process – after the nurse has made sure that the client
­Implementation: Client Needs – Physiological Integrity, will not sustain further damage to the eye.
Reduction of Risk Potential: Cognitive Level – Analysis: 3. The nurse should try to stabilize the right
Concept – Sensory Perception. eye but not do anything that increases the
damage to it. The left eye is patched to
31. 1. Pupil response may be poor but this is not
keep it from moving to see what is going
professionally documented for a clear record
on and the right eye moves with it.
of the nurse’s observation.
4. The nurse should not do anything, including
2. Pinpoint describes the least amount of
irrigating the eye, that might move the needle
dilation noted, which is 1 mm. Bilateral
and create more damage.
means both sides, so equal describes
that one side is the same as the other. TEST-TAKING HINT: The emergency treatment
­Nonreactive describes the pupils not of eye disorders requires the nurse to stabi-
­constricting. This is clear and concise. lize the client until the HCP can take care of
3. Two to 3 mm is not pinpoint, and noncon- the situation.
strictive is not describing response to light. Content – Medical: Integrated Nursing Process –
4. This is something a layperson might say but ­Assessment: Client Needs – Physiological Integrity,
is not in professional terms. ­Reduction of Risk Potential: Cognitive Level – Analysis:
Concept – Sensory Perception.

14_Colgrove_Ch14.indd 568 01/06/16 5:56 PM


C hapter 14   S ensory D eficits 569

SENSORY DEFICITS COMPREHENSIVE


EXAMINATION
1. Which recommendation should the nurse 1. Ensure curtains are open when having the
suggest to an elderly client who lives alone when client read written material.
discussing normal developmental changes of the 2. Provide a variety of written material when
olfactory organs? discussing a procedure.
1. Suggest installing multiple smoke alarms in the 3. Assist the client when getting out of the bed
home. and sitting in the chair.
2. Recommend using a night-light in the hallway 4. Request a telephone for the hearing impaired
and bathroom. for all elderly clients.
3. Discuss keeping a high-humidity atmosphere
6. Which situation makes the nurse suspect the
in the bedroom.
client has glaucoma?
4. Encourage the client to smell food prior to
1. An automobile accident because the client did
eating it.
not see the car in the next lane.
2. The elderly male client tells the nurse, “My wife 2. The cake tasted funny because the client could
says her cooking hasn’t changed, but it is bland not read the recipe.
and tasteless.” Which response by the nurse is 3. The client has been wearing mismatched
most appropriate? clothes and socks.
1. “Would you like me to talk to your wife about 4. The client ran a stoplight and hit a pedestrian
her cooking?” walking in the crosswalk.
2. “Taste buds change with age, which may be
7. The client with a retinal detachment has just
why the food seems bland.”
undergone a gas tamponade repair. Which
3. “This happens because the medications
discharge instruction should the nurse include in
sometimes cause a change in taste.”
the teaching?
4. “Why don’t you barbecue food on a grill if you
1. The client must lie flat with the face down.
don’t like your wife’s cooking?”
2. The head of the bed must be elevated
3. The charge nurse is admitting a 90-year-old client 45 degrees.
to a long-term care facility. Which intervention 3. The client should wear sunglasses when
should the nurse implement? outside.
1. Ensure the client’s room temperature is cool. 4. The client should avoid reading for three (3)
2. Talk louder to make sure the client hears weeks.
clearly.
8. The nurse is conducting a Weber test on the
3. Complete the admission as fast as possible.
client who is suspected of having conductive
4. Provide extra orientation to the surroundings.
hearing loss in the left ear. Where should the
4. Which assessment technique should the nurse nurse place the tuning fork when conducting
implement when assessing the client’s cranial this test?
nerves for vibration?
1. Move the big toe up and down and ask in A
which direction the vibration is felt.
2. Place a tuning fork on the big toe and ask if the
vibrations are felt. B
3. Tap the client’s cheek with the finger and
determine if vibrations are felt.
4. Touch the arm with two sharp objects and ask C
if one (1) vibration or two (2) is felt.
D
5. Which intervention should the nurse include
when conducting an in-service to the ancillary 1. A
nursing staff on caring for elderly clients 2. B
addressing normal developmental sensory 3. C
changes? 4. D

14_Colgrove_Ch14.indd 569 01/06/16 5:56 PM


570 M ed -S urg S uccess

9. The student nurse asks the nurse, “Which type 14. The elderly client is complaining of abdominal
of hearing loss involves damage to the cochlea discomfort. Which scientific rationale should
or vestibulocochlear nerve?” Which statement is the nurse remember when addressing an elderly
the best response of the nurse? client’s perception of pain?
1. “It is called conductive hearing loss.” 1. Elderly clients react to pain the same way any
2. “It is called a functional hearing loss.” other age group does.
3. “It is called a mixed hearing loss.” 2. The elderly client usually requires more pain
4. “It is called sensorineural hearing loss.” medication.
3. Reaction to painful stimuli may be decreased
10. The client has undergone a bilateral
with age.
stapedectomy. Which action by the client
4. The elderly client should use the Wong scale
warrants immediate intervention by the nurse?
to assess pain.
1. The client is ambulating without assistance.
2. The client is sneezing with the mouth open. 15. Which instruction should the nurse discuss with
3. There is some slight serosanguineous the client when completing a sensory assessment
drainage. regarding proprioception?
4. The client reports hearing popping in the 1. Instruct the client to lie flat without a pillow
affected ear. during the assessment.
2. Instruct the client to keep both eyes shut
11. The female client tells the clinic nurse she
during the assessment.
is going on a seven- (7)-day cruise and is
3. During the assessment the client must be in a
worried about getting motion sickness. Which
treatment room.
information should the nurse discuss with the
4. Keep the lights off during the client’s sensory
client?
assessment.
1. Make an appointment for the client to see the
health-care provider. 16. Which signs/symptoms should the nurse expect
2. Recommend getting an over-the-counter to find when assessing the client with an acoustic
scopolamine patch. neuroma?
3. Discourage the client from taking the trip 1. Incapacitating vertigo and otorrhea.
because she is worried. 2. Nystagmus and complaints of dizziness.
4. Instruct the client to lie down and the motion 3. Nausea and vomiting.
sickness will go away. 4. Unilateral hearing loss and tinnitus.
12. The nurse writes the diagnosis “risk for 17. Which assessment technique should the nurse
injury related to impaired balance” for the use to assess the client’s optic nerve?
client diagnosed with vertigo. Which nursing 1. Have the client identify different smells.
intervention should be included in the plan of 2. Have the client discriminate between sugar
care? and salt.
1. Provide information about vertigo and its 3. Have the client read the Snellen chart.
treatment. 4. Have the client say “ah” to assess the rise of
2. Assess for level and type of diversional activity. the uvula.
3. Assess for visual acuity and proprioceptive
18. Which referral is most important for the nurse
deficits.
to implement for the client with permanent
4. Refer the client to a support group and
hearing loss?
counseling.
1. Aural rehabilitation.
13. The nurse is assessing the client’s cranial nerves. 2. Speech therapist.
Which assessment data indicate cranial nerve I 3. Social worker.
is intact? 4. Vocational rehabilitation.
1. The client can identify cold and hot on the
19. Which instruction should the nurse discuss with
face.
the female client with viral conjunctivitis?
2. The client does not have any tongue tremor.
1. Contact the HCP if pain occurs.
3. The client has no ptosis of the eyelids.
2. Do not share towels or linens.
4. The client is able to identify a peppermint
3. Apply warm compresses to the eyes.
smell.
4. Apply makeup very lightly.

14_Colgrove_Ch14.indd 570 01/06/16 5:56 PM


C hapter 14   S ensory D eficits 571

20. The client is two (2) hours postoperative right- 24. Which statement by the daughter of an
ear mastoidectomy. Which assessment data 80-year-old female client who lives alone
should be reported to the health-care provider? warrants immediate intervention by the nurse?
1. Complaints of aural fullness. 1. “I put a night-light in my mother’s bedroom.”
2. Hearing loss in the affected ear. 2. “I got carbon monoxide detectors for my
3. No vertigo. mother’s house.”
4. Facial drooping. 3. “I changed my mother’s furniture around.”
4. “I got my mother large-print books.”
21. Which behavior by the male client should make
the nurse suspect the client has a hearing loss? 25. The 72-year-old client tells the nurse food does
Select all that apply. not taste good anymore and he has lost a little
1. The client reports hearing voices in his head. weight. Which information should the nurse
2. The client becomes irritable very easily. discuss with the client?
3. The client has difficulty making decisions. 1. Suggest using extra seasoning when cooking.
4. The client’s wife reports he ignores her. 2. Instruct the client to keep a seven (7)-day
5. The client does not dominate a conversation. food diary.
3. Refer the client to a dietitian immediately.
22. The client with cataracts who has had
4. Recommend eating three (3) meals a day.
intraocular lens implants is being discharged
from the day surgery department. Which 26. The male client diagnosed with type 2 diabetes
discharge instructions should the nurse discuss mellitus tells the nurse he has begun to see
with the client? yellow spots. Which interventions should the
1. Do not push or pull objects heavier than nurse implement? List in order of priority.
50 pounds. 1. Notify the health-care provider.
2. Lie on the affected eye with two pillows at 2. Check the client’s hemoglobin A1c.
night. 3. Assess the client’s vision using the Amsler
3. Wear glasses or metal eye shields at all times. grid.
4. Bend and stoop carefully for the rest of 4. Teach the client about controlling blood
your life. glucose levels.
5. Determine where the spots appear to be in
23. The nurse is assessing the client’s sensory
the client’s field of vision.
system. Which assessment data indicate an
abnormal stereognosis test?
1. The client is unable to identify which way the
toe is being moved.
2. The client cannot discriminate between sharp
and dull objects.
3. The toes contract and draw together when
the sole of the foot is stroked.
4. The client is unable to identify a key in the
hand with both eyes closed.

14_Colgrove_Ch14.indd 571 01/06/16 5:56 PM


SENSORY DEFICITS COMPREHENSIVE
EXAMINATION ANSWERS AND RATIONALES

1. 1. The decreased sense of smell resulting Content – Medical: Integrated Nursing Process –
from atrophy of olfactory organs is a safety ­Implementation: Client Needs – Safe Effective Care
hazard, and clients may not be able to Environment, Management of Care: Cognitive Level –
smell gas leaks or fire, so the nurse should Application: Concept – Sensory Perception.
recommend a carbon monoxide detector 4. 1. This assesses proprioception, or position sense;
and a smoke alarm. This safety equipment direction of the toe must be evaluated.
is critical for the elderly. 2. Vibration is assessed by using a low-
2. Night-lights do not address the client’s sense frequency tuning fork on a bony promi-
of smell. nence and asking the client whether he or
3. High humidity may help with breathing, but it she feels the sensation and, if so, when the
does not help the sense of smell. ­sensation ceases.
4. The client’s sense of smell is decreased; there- 3. Tapping the cheek assesses for tetany, not
fore, smelling food before eating is not an ­cranial nerve involvement.
­appropriate intervention. 4. A two-point discrimination test evaluates
Content – Medical: Integrated Nursing Process – ­integration of sensation, but it does not assess
Planning: Client Needs – Physiological Integrity, Physiological for vibration.
Adaptation: Cognitive Level – Synthesis: Concept – Sensory Content – Medical: Integrated Nursing Process –
Perception. Assessment: Client Needs – Safe Effective Care Environment,
2. 1. The nurse needs to discuss possible causes Management of Care: Cognitive Level – Analysis: Concept –
with the client and not talk to the wife. Sensory Perception.
2. The acuity of the taste buds decreases with 5. 1. Adequate lighting without a glare should be
age, which could cause regular foods to provided when having the client read writ-
seem bland and tasteless. ten material; therefore, the curtains should be
3. Some medications may cause a metallic taste closed, not open.
in the mouth, but medication does not cause 2. The nurse should provide short, concise, and
foods to taste bland. concrete material, not a variety of material.
4. Telling the client to cook if he doesn’t like his 3. Because fewer tactile cues are received
wife’s food is an argumentative and judgmental from the bottom of the feet, the client may
response. get confused as to body position and loca-
Content – Medical: Integrated Nursing Process – tion. Safety is priority, and assisting the
Implementation: Client Needs – Physiological ­client getting out of bed and sitting in a
Integrity, Physiological Adaptation: Cognitive Level – chair is appropriate.
Application: Concept – Sensory Perception. 4. This is making a judgment. Not all elderly
3. 1. Because of altered temperature regulation, the clients are hard of hearing, and telephones for
client usually needs a warmer room tempera- the hearing impaired require special training
ture, not a cooler room temperature. for the user.
2. The nurse should use a low-pitched, Content – Medical: Integrated Nursing Process –
­normal-volume, clear voice. Talking louder or Planning: Client Needs – Health Promotion and
shouting only makes it harder for the ­client to ­Maintenance: Cognitive Level – Synthesis: Concept –
understand the nurse. ­Sensory Perception.
3. The elderly client requires adequate time to 6. 1. Loss of peripheral vision as a result of
receive and respond to stimuli, to learn, and glaucoma causes the client problems with
to react; therefore, the nurse should take time seeing things on each side, resulting in a
and not rush the admission. “blind spot.” This problem can lead to the
4. Sensory isolation resulting from visual and client having car accidents when switching
hearing loss can cause confusion, anxiety, lanes.
disorientation, and misinterpretation of 2. This is indicative of cataracts because clients
the new environment; therefore, the nurse with cataracts have blurred vision and cannot
should provide extra orientation. read clearly.

572

14_Colgrove_Ch14.indd 572 03/06/16 11:07 AM


C hapter 14   S ensory D eficits 573

3. This is indicative of cataracts because there is Content – Medical: Integrated Nursing Process –
a color shift to yellow–brown and there is re- Diagnosis: Client Needs – Safe Effective Care Environment,
duced light transmission. Management of Care: Cognitive Level – Knowledge:
4. This is indicative of macular degeneration, in ­Concept – Sensory Perception.
which the central vision is affected. 10. 1. Balance disturbance, or true vertigo,
Content – Medical: Integrated Nursing Process – rarely occurs with other middle-ear sur-
Evaluation: Client Needs – Safe Effective Care Environment, gical procedures, but it does occur for a
Management of Care: Cognitive Level – Synthesis: short time after a stapedectomy. Safety is
Concept – Sensory Perception. an important issue, and ambulating with-
7. 1. If gas tamponade is used to flatten the retina, out assistance requires intervention by the
the client may have to be specially positioned nurse.
to make the gas bubble float into the best posi- 2. Pressure changes in the middle ear will be
tion; clients must lie face down or on the side minimal if the client sneezes or blows the
for days. nose with the mouth open instead of closed.
2. The HOB should not be elevated after this 3. Slightly bloody or serosanguineous drainage
surgery. is normal after ear surgery.
3. There is no need for the client to wear 4. Popping and crackling in the operative ear is
sunglasses; this surgery does not cause normal for about three (3) to five (5) weeks
photophobia. after surgery.
4. The client does not need to avoid reading. Content – Surgical: Integrated Nursing Process –
Content – Surgical: Integrated Nursing Process – Assessment: Client Needs – Safe Effective Care Environment,
Planning: Client Needs – Physiological Integrity, Physiological Management of Care: Cognitive Level – Analysis:
Adaptation: Cognitive Level – Synthesis: Concept – Sensory Concept – Sensory Perception.
Perception. 11. 1. This is not a condition requiring an appoint-
8. 1. The tuning fork should be struck to pro- ment with the health-care provider.
duce vibrations and then placed midline 2. Anticholinergic medications, such as
between the ears on top of the head. ­scopolamine patches, can be recom-
2. The right temple area is not an appropriate mended by the nurse; this is not prescrib-
place to assess for conductive hearing loss. ing. Motion sickness is a disturbance of
3. The right occipital area is not the appropri- equilibrium caused by constant motion.
ate place to place the tuning fork; this is the 3. Motion sickness can be controlled with medi-
area behind the ear where the Rinne test is cation and it may not even occur. Therefore,
performed. discussing canceling the trip is not providing
4. The chin area is not the appropriate area to the client with appropriate information.
put the tuning fork. 4. This is providing the client with false infor-
Content – Medical: Integrated Nursing Process –
mation. Lying down may or may not help
Assessment: Client Needs – Safe Effective Care Environment, motion sickness. To be able to enjoy the
Management of Care: Cognitive Level – Knowledge: cruise, the client needs medication.
­Concept – Sensory Perception. Content – Medical: Integrated Nursing Process –
Planning: Client Needs – Physiological Integrity,
9. 1. Conductive hearing loss results from an exter- ­Physiological Adaptation: Cognitive Level – Synthesis:
nal ear disorder, such as impacted cerumen, or Concept – Sensory Perception.
a middle ear disorder, such as otitis media or
otosclerosis. 12. 1. This is appropriate for a diagnosis of
2. Functional (psychogenic) hearing loss is non- ­“knowledge deficit.”
organic and unrelated to detectable structural 2. This is appropriate for a diagnosis of
changes in the hearing mechanisms. It is usually “­deficient diversional activity” related to
a manifestation of an emotional disturbance. ­environmental lack of activity.
3. Mixed hearing loss involves both conductive 3. Balance depends on visual, vestibular,
loss and sensorineural loss. It results from and proprioceptive systems; therefore,
­dysfunction of air and bone conduction. the nurse should assess these systems for
4. Sensorineural hearing loss is described signs/symptoms.
in the stem of the question. It involves 4. This is appropriate for a diagnosis of
­damage to the cochlea or vestibulocochlear “­ineffective coping.”
nerve.

14_Colgrove_Ch14.indd 573 01/06/16 5:56 PM


574 M ed -S urg S uccess

Content – Medical: Integrated Nursing Process – 16. 1. Vertigo and otorrhea are not the signs/­
Diagnosis: Client Needs – Safe Effective Care symptoms of an acoustic neuroma.
Environment, Management of Care: Cognitive 2. Neither nystagmus, an involuntary rhythmic
Level – Analysis: Concept – Sensory Perception. movement of the eyes, nor dizziness is a sign
13. 1. Being able to identify cold and hot on the of an acoustic neuroma.
face indicates an intact trigeminal nerve, 3. Nausea and vomiting are not signs/­symptoms
cranial nerve V. of an acoustic neuroma.
2. Not having any tongue tremor indicates an 4. An acoustic neuroma is a slow-growing,
intact hypoglossal nerve, cranial nerve XI. benign tumor of cranial nerve VII. It usu-
3. No ptosis of the eyelids indicates an intact ally arises from the Schwann cells of the
oculomotor nerve (cranial nerve III), troch- vestibular portion of the nerve and results
lear nerve (IV), and abducens nerve (VI). in unilateral hearing loss and tinnitus with
Tests also assess for ocular motion, conjugate or without vertigo.
movements, nystagmus, and papillary reflexes. Content – Medical: Integrated Nursing Process –
4. Cranial nerve I is the olfactory nerve, ­Assessment: Client Needs – Physiological Integrity,
which involves the sense of smell. With Reduction of Risk Potential: Cognitive Level – Analysis:
the eyes closed, the client must identify Concept – Sensory Perception.
familiar smells to indicate an intact cranial 17. 1. This assesses cranial nerve I, the olfactory
nerve I. nerve.
Content – Medical: Integrated Nursing Process – 2. This assesses cranial nerve IX, the glossopha-
Assessment: Client Needs – Physiological Integrity, ryngeal nerve.
Reduction of Risk Potential: Cognitive Level – 3. This assesses cranial nerve II, the optic
Analysis: Concept – Sensory Perception. nerve, along with visual field testing and
14. 1. This is an inaccurate statement. ophthalmoscopic examination.
2. The elderly client usually requires less pain 4. This assesses cranial nerve X, the vagus nerve.
medication because of the effects of the nor- Content – Medical: Integrated Nursing Process –
mal aging process on the liver (metabolism) Assessment: Client Needs – Safe Effective Care Environment,
and renal system (excretion). Management of Care: Cognitive Level – Analysis:
3. Decreased reaction to painful stimuli is a Concept – Sensory Perception.
normal developmental change; therefore, 18. 1. The purpose of aural rehabilitation is to
complaints of pain may be more serious maximize the communication skills of the
than the client’s perception might indicate client who is hearing impaired. It includes
and thus such complaints require careful auditory training, speech reading, speech
evaluation. training, and the use of hearing aids and
4. The Wong scale is used to assess pain for the hearing guide dogs.
pediatric client, not the adult client. 2. A speech therapist may be part of the aural
Content – Medical: Integrated Nursing Process – rehabilitation team, but the most important
Diagnosis: Client Needs – Safe Effective Care Environment, referral is aural rehabilitation.
Management of Care: Cognitive Level – Analysis: 3. The client may or may not need financial
Concept – Sensory Perception. ­assistance, but the most important referral is
15. 1. The client should be in the sitting position aural rehabilitation.
during a sensory assessment. 4. The client may or may not need assistance
2. The eyes are closed so tactile, superficial with employment because of hearing loss,
pain, vibration, and position sense (pro- but the most important referral is the aural
prioception) can be assessed without the rehabilitation.
client seeing what the nurse is doing. Content – Medical: Integrated Nursing Process –
3. The sensory assessment can be conducted ­Implementation: Client Needs – Safe Effective Care
at the bedside; there is no reason to take the ­Environment, Management of Care: Cognitive
­client to the treatment room. Level – Application: Concept – Sensory Perception.
4. There is no reason the lights should be off 19. 1. The client should be aware eye pain (a sandy
during the sensory assessment; the client sensation and sensitivity to light) will occur
should close his or her eyes. with conjunctivitis.
Content – Medical: Integrated Nursing Process – 2. Viral conjunctivitis is a highly contagious
Planning: Client Needs – Physiological Integrity, eye infection. It is easily spread from one
­Physiological Adaptation: Cognitive Level – Synthesis: person to another; therefore, the client
Concept – Sensory Perception. should not share personal items.

14_Colgrove_Ch14.indd 574 01/06/16 5:56 PM


C hapter 14   S ensory D eficits 575

3. Cold compresses should be placed over the 22. 1. The client should not lift, push, or pull
eyes for about 10 minutes four (4) to five (5) objects heavier than 15 pounds; 50 pounds is
times a day to soothe the pain. excessive.
4. The client must not apply any makeup until 2. The client should avoid lying on the side of
the disease is over and should discard all old the affected eye at night.
makeup to help prevent reinfection. 3. The eyes must be protected by wearing
Content – Medical: Integrated Nursing Process – glasses or metal eye shields at all times
Planning: Client Needs – Physiological Integrity, following surgery. Very few answer op-
­Physiological Adaptation: Cognitive Level – Synthesis: tions with “all” will be correct, but if the
Concept – Infection. option involves ensuring safety, it may be
the correct option.
20. 1. Aural fullness or pressure after surgery is
4. The client should avoid bending or stooping
caused by residual blood or fluid in the mid-
for an extended period—but not forever.
dle ear. This is an expected occurrence after
surgery, and the nurse should administer the Content – Surgical: Integrated Nursing Process –
prescribed analgesic. Planning: Client Needs – Physiological Integrity,
­Physiological Adaptation: Cognitive Level – Synthesis:
2. Hearing in the operated ear may be reduced
Concept – Sensory Perception.
for several weeks because of edema, accumula-
tion of blood and tissue fluid in the middle ear, 23. 1. This is an abnormal finding for testing
and dressings or packing, so this does not need ­proprioception, or position sense.
to be reported to the health-care provider. 2. This is an abnormal finding for assessing
3. Vertigo (dizziness) is uncommon after this ­superficial pain perception.
surgery, but if it occurs the nurse should 3. This is a normal Babinski’s reflex in an adult
administer an antiemetic or antivertigo medi- client.
cation and does not need to report it to the 4. Stereognosis is a test evaluating higher
health-care provider. cortical sensory ability. The client is in-
4. The facial nerve, which runs through the structed to close both eyes and identify a
middle ear and mastoid, is at risk for in- variety of objects (e.g., keys, coins) placed
jury during mastoid surgery; therefore, in one hand by the examiner.
a facial paresis should be reported to the Content – Surgical: Integrated Nursing Process –
health-care provider. ­Assessment: Client Needs – Physiological Integrity,
Content – Surgical: Integrated Nursing Process – ­Reduction of Risk Potential: Cognitive Level – Analysis:
­Assessment: Client Needs – Physiological Integrity, Concept – Sensory Perception.
Reduction of Risk Potential: Cognitive Level – Analysis:
24. 1. With normal aging comes decreased
Concept – Perioperative.
­peripheral vision, constricted visual field,
21. 1. Voices in the head may indicate schizophre- and tactile alterations. A night-light
nia, but it is not a symptom of hearing loss. ­addresses safety issues and warrants praise,
2. Fatigue may be the result of straining to not intervention.
hear, and a client may tire easily when 2. Carbon monoxide detectors help ensure
listening to a conversation. Under these safety in the mother’s home, so this comment
circumstances, the client may become doesn’t warrant intervention.
­irritable very easily. 3. Decreased peripheral vision, constricted
3. Loss of self-confidence makes it increas- visual fields, and tactile alterations are
ingly difficult for a person who is hearing associated with normal aging. The client
impaired to make a decision. needs a familiar arrangement of furniture
4. Often it is not the person with the hear- for safety. Moving the furniture may cause
ing loss but a significant other who notices the client to trip or fall. The nurse should
hearing loss; hearing loss is usually gradual. intervene in this situation.
5. Many clients who are hearing impaired tend 4. As a result of normal aging, vision may
to dominate the conversation because, as long become impaired, and the provision of
as it is centered on the client, they can con- large-print books warrants praise.
trol it and are not as likely to be embarrassed Content – Surgical: Integrated Nursing Process –
by some mistake. Evaluation: Client Needs – Safe Effective Care
Content – Surgical: Integrated Nursing Process – Environment, Management of Care: Cognitive
­Assessment: Client Needs – Physiological Integrity, Level – Synthesis: Concept – Sensory Perception.
­Reduction of Risk Potential: Cognitive Level – Analysis:
Concept – Sensory Perception.

14_Colgrove_Ch14.indd 575 01/06/16 5:56 PM


576 M ed -S urg S uccess

25. 1. The acuity of taste buds decreases with which eye is affected, how long the client
age, which may cause a decreased appe- has been seeing the spots, and whether
tite and subsequent weight loss. Spices this ever occurred before.
or other seasonings may help the food 3. The Amsler grid is helpful in determining
taste better to the client. losses occurring in the visual fields.
2. This may be an appropriate intervention if 2. The hemoglobin A1c laboratory tests re-
excessive weight is lost or if seasoning the sults indicate glucose control over the past
food does not increase appetite, but it is not two (2) to three (3) months. Diabetic reti-
necessary at this time. nopathy is directly related to poor blood
3. The client does not need a dietary consult glucose control.
for food not “tasting good.” The nurse can 1. The health-care provider should be noti-
address the client’s concern. fied to plan for laser surgery on the eye.
4. This recommendation does not address the 4. The client should be instructed about
client’s comment about food not tasting good. controlling blood glucose levels, but this
Content – Surgical: Integrated Nursing Process – can wait until the immediate situation
Planning: Client Needs – Safe Effective Care Environment, is resolved or at least until measures to
Management of Care: Cognitive Level – Synthesis: ­address the potential loss of eyesight have
­Concept – Sensory Perception. been taken.
26. In order of priority: 5, 3, 2, 1, 4. Content – Surgical: Integrated Nursing Process –
5. The nurse should question the client ­Implementation: Client Needs – Safe Effective Care
­Environment, Management of Care: Cognitive
­further to obtain information such as
Level – Analysis: Concept – Sensory Perception.

14_Colgrove_Ch14.indd 576 01/06/16 5:56 PM


Emergency Nursing
The universe is full of magical things, patiently waiting for our
wits to grow sharper.
15
—Eden Phillpotts

Nurses in a medical-surgical unit may face emergency situations in the care of their clients.
An important concept is shock. A client in shock needs immediate intervention. Other emer-
gency situations occur because of outside factors—bioterrorism, disasters, physical abuse.
Nurses must know what to do immediately, when to notify the Rapid Response Team, when
to call a code, and what interventions are needed. In addition, because some emergency
situations involve large numbers of people or people who pose a threat, there are specific
public health and other governmental agencies that must be notified and whose rules must
be followed in specific situations.

KEYWORDS abbreviations
Corrosive Arterial blood gases (ABGs)
Epistaxis Automated external defibrillator (AED)
Gastric lavage Blood pressure (BP)
Blood urea nitrogen (BUN)
Cardiopulmonary resuscitation (CPR)
Emergency department (ED)
Health-care provider (HCP)
Intravenous (IV)
Intravenous piggyback (IVPB)
Intravenous push (IVP)
Licensed practical nurse (LPN)
Material safety data sheet (MSDS)
Nasogastric tube (NGT)
Nonsteroidal anti-inflammatory drugs (NSAIDs)
Post-traumatic stress disorder (PTSD)
Pulse (P)
Rapid response team (RRT)
Related to (R/T)
U.S. Environmental Protection Agency (EPA)

577

15_Colgrove_Ch15.indd 577 02/06/16 12:39 PM


578 M ed -S urg S uccess

PRACTICE QUESTIONS
Shock 7. The nurse caring for a client with sepsis writes
the client diagnosis of “alteration in comfort
1. The client diagnosed with hypovolemic shock has R/T chills and fever.” Which intervention should
a BP of 100/60. Fifteen minutes later the BP is be included in the plan of care?
88/64. How much narrowing of the client’s pulse 1. Ambulate the client in the hallway every shift.
pressure has occurred between the two readings? 2. Monitor urinalysis, creatinine level, and BUN
______ level.
3. Apply sequential compression devices to the
2. The client is admitted into the emergency lower extremities.
department with diaphoresis, pale clammy skin, 4. Administer an antipyretic medication every
and BP of 90/70. Which intervention should the four (4) hours PRN.
nurse implement first?
1. Start an IV with an 18-gauge catheter. 8. The nurse and an unlicensed assistive personnel
2. Administer dopamine intravenous infusion. (UAP) are caring for a group of clients on
3. Obtain arterial blood gases (ABGs). a medical floor. Which action by the UAP
4. Insert an indwelling urinary catheter. warrants intervention by the nurse?
1. The UAP places a urine specimen in a
3. The nurse is caring for a client diagnosed with biohazard bag in the hallway.
septic shock. Which assessment data warrant 2. The UAP uses the alcohol foam hand cleanser
immediate intervention by the nurse? after removing gloves.
1. Vital signs T 100.4˚F, P 104, R 26, and BP 3. The UAP puts soiled linen in a plastic bag in
102/60. the client’s room.
2. A white blood cell count of 18,000/mm3. 4. The UAP obtains a disposable stethoscope for
3. A urinary output of 90 mL in the last four (4) a client in an isolation room.
hours.
4. The client complains of being thirsty. 9. The elderly female client with vertebral fractures
who has been self-medicating with ibuprofen, a
4. The client diagnosed with septicemia has the nonsteroidal anti-inflammatory drug (NSAID),
following health-care provider orders. Which presents to the ED complaining of abdominal
HCP order has the highest priority? pain, is pale and clammy, and has a P of 110 and
1. Provide clear liquid diet. a BP of 92/60. Which type of shock should the
2. Initiate IV antibiotic therapy. nurse suspect?
3. Obtain a STAT chest x-ray. 1. Cardiogenic shock.
4. Perform hourly glucometer checks. 2. Hypovolemic shock.
5. The client is diagnosed with neurogenic shock. 3. Neurogenic shock.
Which signs/symptoms should the nurse assess in 4. Septic shock.
this client? 10. The client has recently experienced a myocardial
1. Cool, moist skin. infarction. Which action by the nurse helps
2. Bradycardia. prevent cardiogenic shock?
3. Wheezing. 1. Monitor the client’s telemetry.
4. Decreased bowel sounds. 2. Turn the client every two (2) hours.
6. The nurse in the emergency department 3. Administer oxygen via nasal cannula.
administered an intramuscular antibiotic 4. Place the client in the Trendelenburg position.
in the left gluteal muscle to the client with 11. The client diagnosed with septicemia is
pneumonia who is being discharged home. Which receiving a broad-spectrum antibiotic. Which
intervention should the nurse implement? laboratory data require the nurse to notify the
1. Ask the client about drug allergies. health-care provider?
2. Obtain a sterile sputum specimen. 1. The client’s potassium level is 3.8 mEq/L.
3. Have the client wait for 30 minutes. 2. The urine culture indicates high sensitivity to
4. Place a warm washcloth on the client’s the antibiotic.
left hip. 3. The client’s pulse oximeter reading is 94%.
4. The culture and sensitivity is resistant to the
client’s antibiotic.

15_Colgrove_Ch15.indd 578 02/06/16 12:39 PM


C hapter 15  E mergency N ursing 579

12. The nurse is caring for a client diagnosed with 16. The nurse is teaching a class on bioterrorism.
septic shock who has hypotension, decreased Which statement is the scientific rationale for
urine output, and cool, pale skin. Which phase of designating a specific area for decontamination?
septic shock is the client experiencing? 1. Showers and privacy can be provided to the
1. The hypodynamic phase. client in this area.
2. The compensatory phase. 2. This area isolates the clients who have been
3. The hyperdynamic phase. exposed to the agent.
4. The progressive phase. 3. It provides a centralized area for stocking the
needed supplies.
4. It prevents secondary contamination to the
Bioterrorism health-care providers.

13. The nurse in the emergency department has 17. The triage nurse in a large trauma center
admitted five (5) clients in the last two (2) hours has been notified of an explosion in a major
with complaints of fever and gastrointestinal chemical manufacturing plant. Which action
distress. Which question is most appropriate should the nurse implement first when the
for the nurse to ask each client to determine if clients arrive at the emergency department?
there is a bioterrorism threat? 1. Triage the clients and send them to the
1. “Do you work or live near any large power appropriate areas.
lines?” 2. Thoroughly wash the clients with soap and
2. “Where were you immediately before you got water and then rinse.
sick?” 3. Remove the clients’ clothing and have them
3. “Can you write down everything you ate shower.
today?” 4. Assume the clients have been decontaminated
4. “What other health problems do you have?” at the plant.

14. The health-care facility has been notified an 18. The nurse is teaching a class on biological
alleged inhalation anthrax exposure has occurred warfare. Which information should the nurse
at the local post office. Which category of include in the presentation?
personal protective equipment (PPE) should the 1. Contaminated water is the only source of
response team wear? transmission of biological agents.
1. Level A. 2. Vaccines are available and being prepared to
2. Level B. counteract biological agents.
3. Level C. 3. Biological weapons are less of a threat than
4. Level D. chemical agents.
4. Biological weapons are easily obtained and
15. The nurse is teaching a class on bioterrorism result in significant mortality.
and is discussing personal protective equipment
(PPE). Which statement is the most important 19. Which signs/symptoms should the nurse assess
fact for the nurse to share with the participants? in the client who has been exposed to the
1. Health-care facilities should keep masks at anthrax bacillus via the skin?
entry doors. 1. A scabby, clear fluid-filled vesicle.
2. The respondent should be trained in the 2. Edema, pruritus, and a 2-mm ulcerated
proper use of PPE. vesicle.
3. No single combination of PPE protects 3. Irregular brownish-pink spots around the
against all hazards. hairline.
4. The EPA has divided PPE into four levels of 4. Tiny purple spots flush with the surface of the
protection. skin.
20. The client has expired secondary to smallpox.
Which information about funeral arrangements
is most important for the nurse to provide to the
client’s family?
1. The client should be cremated.
2. Suggest an open casket funeral.
3. Bury the client within 24 hours.
4. Notify the public health department.

15_Colgrove_Ch15.indd 579 02/06/16 12:39 PM


580 M ed -S urg S uccess

21. A chemical exposure has just occurred at an 26. The unlicensed assistive personnel (UAP) is
airport. An off-duty nurse, knowledgeable about performing cardiac compressions on an adult
biochemical agents, is giving directions to the client during a code. Which behavior warrants
travelers. Which direction should the nurse immediate intervention by the nurse?
provide to the travelers? 1. The UAP has hand placement on the lower
1. Hold their breath as much as possible. half of the sternum.
2. Stand up to avoid heavy exposure. 2. The UAP performs cardiac compressions and
3. Lie down to stay under the exposure. allows for rescue breathing.
4. Attempt to breathe through their clothing. 3. The UAP depresses the sternum 0.5 to one
(1) inch during compressions.
22. The nurse is caring for a client in the prodromal
4. The UAP asks to be relieved from performing
phase of radiation exposure. Which signs/
compressions because of exhaustion.
symptoms should the nurse assess in the client?
1. Anemia, leukopenia, and thrombocytopenia. 27. Which intervention is most important for the
2. Sudden fever, chills, and enlarged lymph nurse to implement when participating in a
nodes. code?
3. Nausea, vomiting, and diarrhea. 1. Elevate the arm after administering
4. Flaccid paralysis, diplopia, and dysphagia. medication.
2. Maintain sterile technique throughout the
23. Which cultural issues should the nurse consider
code.
when caring for clients during a bioterrorism
3. Treat the client’s signs/symptoms; do not treat
attack? Select all that apply.
the monitor.
1. Language difficulties.
4. Provide accurate documentation of what
2. Religious practices.
happened during the code.
3. Prayer times for the people.
4. Rituals for handling the dead. 28. The CPR instructor is discussing an automated
5. Keeping the family in the designated area. external defibrillator (AED) during class. Which
statement best describes an AED?
24. The off-duty nurse hears on the television of
1. It analyzes the rhythm and shocks the client in
a bioterrorism act in the community. Which
ventricular fibrillation.
action should the nurse take first?
2. The client will be able to have synchronized
1. Immediately report to the hospital emergency
cardioversion with the AED.
department.
3. It will keep the health-care provider informed
2. Call the American Red Cross to find out
of the client’s oxygen level.
where to go.
4. The AED will perform cardiac compressions
3. Pack a bag and prepare to stay at the hospital.
on the client.
4. Follow the nurse’s hospital policy for
responding. 29. The nurse is caring for clients on a medical
floor. Which client is most likely to experience
sudden cardiac death?
Codes 1. The 84-year-old client exhibiting
uncontrolled atrial fibrillation.
25. The nurse finds the client unresponsive on the 2. The 60-year-old client exhibiting
floor of the bathroom. Which action should the asymptomatic sinus bradycardia.
nurse implement first? 3. The 53-year-old client exhibiting ventricular
1. Check the client for breathing. fibrillation.
2. Assess the carotid artery for a pulse. 4. The 65-year-old client exhibiting
3. Shake the client and shout. supraventricular tachycardia.
4. Notify the rapid response team.
30. Which health-care team member referral should
be made by the nurse when a code is being
conducted on a client in a community hospital?
1. The hospital chaplain.
2. The social worker.
3. The respiratory therapist.
4. The director of nurses.

15_Colgrove_Ch15.indd 580 02/06/16 12:39 PM


C hapter 15  E mergency N ursing 581

31. Which intervention is the most important for 36. The client in a code is now in ventricular
the intensive care unit nurse to implement when bigeminy. The HCP orders a lidocaine drip
performing mouth-to-mouth resuscitation on a at three (3) mg/min. The lidocaine comes
client who has pulseless ventricular fibrillation? prepackaged with two (2) grams of lidocaine in
1. Perform the jaw thrust maneuver to open the 500 mL of D5W. At which rate will the nurse set
airway. the infusion pump? _________
2. Use the mouth to cover the client’s mouth and
nose.
3. Insert an oral airway prior to performing Disasters/Triage
mouth to mouth.
4. Use a pocket mouth shield to cover the 37. Which situation warrants the nurse obtaining
client’s mouth. information from a material safety data sheet
(MSDS)?
32. The nurse is teaching CPR to a class. Which 1. The custodian spilled a chemical solvent in
statement best explains the definition of sudden the hallway.
cardiac death? 2. A visitor slipped and fell on the floor that had
1. Cardiac death occurs after being removed just been mopped.
from a mechanical ventilator. 3. A bottle of antineoplastic agent broke on the
2. Cardiac death is the time the HCP officially client’s floor.
declares the client dead. 4. The nurse was stuck with a contaminated
3. Cardiac death occurs within one (1) hour of needle in the client’s room.
the onset of cardiovascular symptoms.
4. The death is caused by myocardial ischemia 38. The triage nurse is working in the emergency
resulting from coronary artery disease. department. Which client should be assessed
first?
33. Which statement explains the scientific rationale 1. The 10-year-old child whose dad thinks the
for having emergency suction equipment child’s leg is broken.
available during resuscitation efforts? 2. The 45-year-old male who is diaphoretic and
1. Gastric distention can occur as a result of clutching his chest.
ventilation. 3. The 58-year-old female complaining of a
2. It is needed to assist when intubating the headache and seeing spots.
client. 4. The 25-year-old male who cut his hand with a
3. This equipment will ensure a patent airway. hunting knife.
4. It keeps the vomitus away from the health-
care provider. 39. The nurse is teaching a class on disaster
preparedness. Which are components of an
34. Which equipment must be immediately brought emergency operations plan (EOP)? Select all
to the client’s bedside when a code is called for a that apply.
client who has experienced a cardiac arrest? 1. A plan for practice drills.
1. A ventilator. 2. A deactivation response.
2. A crash cart. 3. A plan for internal communication only.
3. A gurney. 4. A preincident response.
4. Portable oxygen. 5. A security plan.
35. The nursing administrator responds to a code 40. According to the North Atlantic Treaty
situation. When assessing the situation, which Organization (NATO) triage system, which
role must the administrator ensure is performed situation is considered a level red (Priority 1)?
for legal purposes and continuity of care of the 1. Injuries are extensive and chances of survival
client? are unlikely.
1. A person is ventilating with an Ambu bag. 2. Injuries are minor and treatment can be
2. A person is performing chest compressions delayed hours to days.
correctly. 3. Injuries are significant but can wait hours
3. A person is administering medications as without threat to life or limb.
ordered. 4. Injuries are life threatening but survivable
4. A person is keeping an accurate record of the with minimal interventions.
code.

15_Colgrove_Ch15.indd 581 02/06/16 12:39 PM


582 M ed -S urg S uccess

41. Which statement best describes the role of the 46. The triage nurse has placed a disaster tag on
medical-surgical nurse during a disaster? the client. Which action warrants immediate
1. The nurse may be assigned to ride in the intervention by the nurse?
ambulance. 1. The nurse documents the tag number in the
2. The nurse may be assigned as a first assistant disaster log.
in the operating room. 2. The unlicensed assistive personnel documents
3. The nurse may be assigned to crowd control. vital signs on the tag.
4. The nurse may be assigned to the emergency 3. The health-care provider removes the tag to
department. examine the limb.
4. The LPN securely attaches the tag to the
42. The nurse in a disaster is triaging the following
client’s foot.
clients. Which client should be triaged as an
Expectant Category, Priority 4, and color black? 47. The father of a child brought to the emergency
1. The client with a sucking chest wound who is department is yelling at the staff and obviously
alert. intoxicated. Which approach should the nurse
2. The client with a head injury who is take with the father?
unresponsive. 1. Talk to the father in a calm and low voice.
3. The client with an abdominal wound and 2. Tell the father to wait in the waiting room.
stable vital signs. 3. Notify the child’s mother to come to the ED.
4. The client with a sprained ankle which may 4. Call the police department to come and
be fractured. arrest him.
43. Which federal agency is a resource for the nurse 48. A gang war has resulted in 12 young males being
volunteering at the American Red Cross who is brought to the emergency department. Which
on a committee to prepare the community for action by the nurse is priority when a gang
any type of disaster? member points a gun at a rival gang member in
1. The Joint Commission (JC). the trauma room?
2. Office of Emergency Management (OEM). 1. Attempt to talk to the person who has the gun.
3. Department of Health and Human Services 2. Explain to the person the police are coming.
(DHHS). 3. Stand between the client and the man with
4. Metro Medical Response Systems (MMRS). the gun.
4. Get out of the line of fire and protect self.
44. Which situation requires the emergency
department manager to schedule and conduct a
Critical Incident Stress Management (CISM)? Poisoning
1. Caring for a two (2)-year-old child who died
from severe physical abuse. 49. The parents bring their toddler to the ED
2. Performing CPR on a middle-aged male in a panic. The parents state the child had
executive who died. been playing in the kitchen and got into some
3. Responding to a 22-victim bus accident with cleaning agents and swallowed an unknown
no apparent fatalities. quantity of the agents. Which health-care agency
4. Being required to work 16 hours without should the nurse contact at this time?
taking a break. 1. Child Protective Services (CPS).
45. During a disaster, a local news reporter comes 2. The local police department.
to the emergency department requesting 3. The Department of Health.
information about the victims. Which action is 4. The Poison Control Center.
most appropriate for the nurse to implement? 50. Which is the primary goal of the ED nurse in
1. Have security escort the reporter off the caring for a client who has ingested poison?
premises. 1. Remove or inactivate the poison before it is
2. Direct the reporter to the disaster command absorbed.
post. 2. Provide long-term supportive care to prevent
3. Tell the reporter this is a violation of HIPAA. organ damage.
4. Request the reporter to stay out of the way. 3. Administer an antidote to increase the effects
of the poison.
4. Implement treatment prolonging the
elimination of the poison.

15_Colgrove_Ch15.indd 582 02/06/16 12:39 PM


C hapter 15  E mergency N ursing 583

51. The client has ingested a corrosive solution 55. The client presents to the ED with acute
containing lye. Which intervention should the vomiting after eating at a fast-food restaurant.
nurse implement? There has not been any diarrhea. The nurse
1. Administer syrup of ipecac to induce suspects botulism poisoning. Which nursing
vomiting. problem is the highest priority for this client?
2. Insert a nasogastric tube and connect to wall 1. Fluid volume loss.
suction. 2. Risk for respiratory paralysis.
3. Assess for airway compromise. 3. Abdominal pain.
4. Immediately administer water or milk. 4. Anxiety.
52. The male client was found in a parked car with 56. The client has ingested the remaining amount of
the motor running. The paramedic brought the a bottle of analgesic medication. The medication
client to the ED with complaints of a headache, comes 500 mg per capsule. Two (2) doses of two
nausea, and dizziness and the client is unable (2) capsules each have been used by another
to recall his name or address. On assessment, member of the family. The bottle originally had
the nurse notes the buccal mucosa is a cherry- 250 capsules. How many mg of medication did
red color. Which intervention should the nurse the client take? _______
implement first?
57. The nurse is providing first aid to a victim of a
1. Check the client’s oxygenation level with a
poisonous snake bite. Which intervention should
pulse oximeter.
be the nurse’s first action?
2. Apply oxygen via nasal cannula at 100%.
1. Apply a tourniquet to the affected limb.
3. Obtain a psychiatric consult to determine if
2. Cut an “X” across the bite and suck out the
this was a suicide attempt.
venom.
4. Prepare the client for transfer to a facility
3. Administer a corticosteroid medication.
with a hyperbaric chamber.
4. Have the client lie still and remove
53. A gastric lavage has been ordered for a client constrictive items.
who is comatose and who ingested a full bottle
58. The nurse is discharging a client diagnosed with
of acetaminophen, a nonnarcotic analgesic.
accidental carbon monoxide poisoning. Which
Which intervention should be included in the
statement made by the client indicates the need
procedure? Select all that apply.
for further teaching?
1. Place the client on the left side with the head
1. “I should install carbon monoxide detectors in
15 degrees lower than the body.
my home.”
2. Insert a small-bore feeding tube into the naris.
2. “Having a natural bright-red color to my lips
3. Have standby suction available.
is good.”
4. Withdraw stomach contents and then instill
3. “You cannot smell carbon monoxide, so it can
an irrigating solution.
be difficult to detect.”
5. Send samples of the stomach contents to the
4. “I should have my furnace checked for leaks
laboratory for analysis.
before turning it on.”
54. A vat of chemicals spilled onto the client. Which
59. The nurse and an unlicensed assistive personnel
action should the occupational health nurse
(UAP) are caring for clients on a medical unit.
implement first?
Which nursing task cannot be delegated to the
1. Have the client stand under a shower while
UAP?
removing all clothes.
1. Obtaining the intake and output on a client
2. Check the material safety data sheets for the
diagnosed with food poisoning.
antidote.
2. Performing a dressing change on the client
3. Administer oxygen by nasal cannula.
with a chemical burn.
4. Collect a sample of the chemicals in the vat
3. Assisting a client who overdosed on morphine
for analysis.
to the bedside commode.
4. Help a client with carbon monoxide poisoning
turn, cough, and deep breathe.

15_Colgrove_Ch15.indd 583 02/06/16 12:39 PM


584 M ed -S urg S uccess

60. The charge nurse is making assignments. 64. The adolescent female comes to the school nurse
Which client should be assigned to the most of an intermediate school and tells the nurse she
experienced nurse? thinks she is pregnant. During the interview, the
1. The client diagnosed with a snake bite who is client states her father is the baby’s father. Which
receiving antivenin. intervention should the nurse implement first?
2. The client who swallowed a lye preparation 1. Complete a rape kit.
and is being discharged. 2. Notify Child Protective Services.
3. The client who is angry the suicide attempt 3. Call the parents to come to the school.
did not work. 4. Arrange for the client to go to a free clinic.
4. The client who required skin grafting after a
65. The nurse in an outpatient rehabilitation facility
chemical spill.
is working with convicted child abusers. Which
characteristics should the nurse expect to
Violence, Physical Abuse, and Neglect observe in the abusers? Select all that apply.
1. The abuser calls the child a liar.
61. The female client presents to the emergency 2. The abuser has a tendency toward violence.
department with facial lacerations and 3. The abuser exhibits a high self-esteem.
contusions. The spouse will not leave the 4. The abuser is unable to admit the need for help.
room during the assessment interview. Which 5. The abuser was spoiled as a child.
intervention should be the nurse’s first action? 66. The nurse is teaching a class about rape prevention
1. Call the security guard to escort the spouse to a group of women at a community center.
away. Which information is not a myth about rape?
2. Discuss the injuries while the spouse is in the 1. Women who are raped asked for it by dressing
room. provocatively.
3. Tell the spouse the police will want to talk to 2. If a woman says no, it is a come on and she
him. really does not mean it.
4. Escort the client to the bathroom for a urine 3. Rape is an attempt to exert power and control
specimen. over the client.
62. The elderly male client is admitted to the 4. All victims of sexual assault are women; men
medical unit with a diagnosis of senile dementia. can’t be raped.
The client is 74 inches tall and weighs 54.5 kg. 67. The nurse working in the emergency
The client lives with his son and daughter-in-law, department is admitting a 34-year-old female
both of whom work outside the house. Which client for one of multiple admissions for spousal
referral is most important for the nurse to abuse. The client has refused to leave her
implement? husband or to press charges against him. Which
1. Adult Protective Services. action should the nurse implement?
2. Social worker. 1. Insist the woman press charges this time.
3. Medicare ombudsman. 2. Treat the wounds and do nothing else.
4. Dietitian. 3. Tell the woman her husband could kill her.
63. The nurse working in a homeless shelter 4. Give the woman the number of a women’s
identifies an adolescent female sexually shelter.
aggressive toward some of the males in the 68. The 84-year-old female client is admitted with
shelter. Which is the most common cause for multiple burn marks on the torso and under the
this behavior? breasts along with contusions in various stages
1. The client is acting in a learned behavior of healing. When questioned by the nurse, the
pattern to get attention. woman denies any problems have occurred.
2. The client had to leave home because of The woman lives with her son and does the
promiscuous behavior. housework. Which is the most probable reason
3. The client has a psychiatric disorder called the woman denies being abused?
nymphomania. 1. There has not been any abuse to report.
4. The client is a prostitute and is trying to get 2. The client is ashamed to admit being abused.
customers. 3. The client has Alzheimer’s disease and can’t
remember.
4. The client has engaged in consensual sex.

15_Colgrove_Ch15.indd 584 02/06/16 12:39 PM


C hapter 15  E mergency N ursing 585

69. Which question is an appropriate interview 71. The emergency department nurse writes the
question for the nurse to use with clients problem of “ineffective coping” for a client who
involved in abuse? has been raped. Which intervention should the
1. “I know you are being abused. Can you tell nurse implement?
me about it?” 1. Encourage the client to take the “morning-
2. “How much does your spouse drink before he after” pill.
hits you?” 2. Allow the client to admit guilt for causing the
3. “What did you do to cause your spouse to get rape.
mad?” 3. Provide a list of rape crisis counselors.
4. “Do you have a plan if your partner becomes 4. Discuss reporting the case to the police.
abusive?”
72. The nurse writes a nursing diagnosis of “risk for
70. The client who was abused as a child is injury as a result of physical abuse by spouse” for
diagnosed with post-traumatic stress disorder a client. Which is an appropriate goal for this
(PTSD). Which intervention should the nurse client?
implement when the client is resting? 1. The client will learn not to trust anyone.
1. Call the client’s name to awaken him or her, 2. The client will admit the abuse is happening
but don’t touch the client. and get help.
2. Touch the client gently to let him or her know 3. The client will discuss the nurse’s suspicions
you are in the room. with the spouse.
3. Enter the room as quietly as possible to not 4. The client will choose to stay with the spouse.
disturb the client.
4. Do not allow the client to be awakened at all
when sleeping.

CONCEPTS
The concepts covered in this chapter focus on clinical judgment. Exemplars that are covered are shock, disaster/
triage, and abuse/neglect. Interrelated concepts of the nursing process and critical thinking are covered through-
out the questions. The concept of critical thinking is presented in the prioritizing or “first” questions.
73. The nurse is admitting a client with the hemogram result listed next. Which priority intervention should
the nurse implement first?

Date: Today
Laboratory Test Client Values Normal Values
White Blood Cells 25.3 5–10 (103)
Red Blood Cells 4.8 M: 4.5–5.3 (106)
F: 4.1–5.1 (106)
Hemoglobin 10 M: 13–19 g/100 mL
F: 12–16 g/100 mL
Hematocrit 30% M: 37%–49%
F: 36%–46%
Platelets 250 150–400 (103)

1. Administer the prescribed antibiotic IVPB.


2. Start an intravenous line in the client’s left forearm.
3. Perform the admission assessment.
4. Have the laboratory draw blood cultures STAT.

15_Colgrove_Ch15.indd 585 02/06/16 12:39 PM


586 M ed -S urg S uccess

74. The nurse is preparing to administer medications to a newly admitted client with a hemoglobin of
5.3 g/mL. Which medication should the nurse administer first?
1. Normal saline.
2. Packed red blood cells unit #1.
3. Furosemide.
4. Diphenhydramine.

Admitting Diagnosis: Gastrointestinal


Client Name: Smith, John M/68 Bleeding

Account Number: 332 851 Med Rec #: 55-66-89 Allergies: NKDA

Date Medication 2301–0700 0701–1500 1501–2300


Today Normal Saline 1,000 mL
150 mL per hour continuous
Today Packed Red Blood Cells
2 units
Administer over 2 hours
each unit
Today Furosemide 40 mg
between units
Today Diphenhydramine 50 mg IVP
prior to each unit of PRBCs
Signature of Nurse: Medical Nurse RN
(MN)

75. The nurse responded to a cardiac arrest. On 76. The nurse is responding to a disaster call from
which sites should the nurse place the automated home following a multivehicle motor-vehicle
external defibrillator (AED) pads. Select all that accident. Which action should the nurse take
apply. first?
1. Go to the emergency department to triage the
clients coming in.
2. Assist the charge nurse to identify clients who
could be discharged.
3. Report to the command center for
assignment.
4. Pack a bag to be able to stay until the
emergency is over.
77. The charge nurse of the medical-surgical unit
A B secured the crash cart during the code. Which
intervention should the charge nurse implement
after transferring the client to the intensive
care unit?
1. Reassign the clients on the floor because one
is now gone.
E 2. Call the family of the client who coded and let
D C them know of the transfer.
3. Make sure the crash cart is restocked.
1. Site A. 4. Hold a unit meeting to determine if anything
2. Site B. could have been done differently during the
3. Site C. code.
4. Site D.
5. Site E.

15_Colgrove_Ch15.indd 586 02/06/16 12:39 PM


C hapter 15  E mergency N ursing 587

78. The charge nurse has been notified that a 80. The male client presents to the emergency
disaster has occurred and that all possible clients department stating he vomited a “large” amount
should be discharged so the floor can receive of bright red blood. Which should the nurse
the casualties. Which client should not be implement first?
discharged? 1. Start an intravenous line with an 18-gauge
1. The 13-year-old client who is scheduled for a needle.
tonsillectomy. 2. Have the UAP take the client’s vital signs.
2. The 42-year-old client scheduled for an 3. Ask the client to provide a stool specimen for
abdominal aorta aneurysm dissection. blood.
3. The 76-year-old client diagnosed with a 4. Send the client to radiology for an abdominal
pulmonary embolus whose INR is 2.9. CT scan.
4. The 80-year-old client who is refusing to
assist in activities of daily living.
79. The nurse is working at a facility where an Ebola
client has been admitted. Which action should
the nurse take?
1. Consult the nurse manager regarding the
infection-control standards to follow.
2. Resign immediately and leave the facility.
3. Watch the television news reports to identify
which station has the client.
4. Participate in a news report about the quality
of care provided at the hospital.

15_Colgrove_Ch15.indd 587 02/06/16 12:39 PM


PRACTICE QUESTIONS ANSWERS
AND RATIONALES

Shock 3. 1. These vital signs are expected in a client with


septic shock.
1. 16 mm Hg pulse pressure. 2. An elevated WBC count indicates an i­nfection,
The pulse pressure is the systolic BP minus the which is the definition of sepsis.
diastolic BP. 3. The client must have a urinary output of at
100 − 60 = 40 mm Hg pulse pressure in first least 30 mL/hr, so 90 mL in the last four (4)
BP reading hours indicates impaired renal perfusion,
88 − 64 = 24 mm Hg pulse pressure in second which is a sign of worsening shock
reading 4. The client being thirsty is not an uncommon
40 − 24 = 16 mm Hg pulse pressure complaint for a client in septic shock. This
narrowing. warrants immediate intervention.
A narrowing or decreased pulse pressure is an TEST-TAKING HINT: The words “warrant im-
earlier indicator of shock than a decrease in mediate intervention” mean the nurse must
systolic blood pressure. do something, which frequently is notify the
TEST-TAKING HINT: If the test taker is not health-care provider. Any client in shock will
aware of how to obtain a pulse pressure, the have signs and symptoms requiring the nurse
only numbers provided in the stem are sys- to intervene. In this question, the test taker
tolic and diastolic blood pressures. The test must determine priority and which data re-
taker should do something with the numbers. quire immediate intervention.
Content – Medical: Integrated Nursing Process – Content – Medical: Integrated Nursing Process –
­Assessment: Client Needs – Physiological Integrity, ­ ­Assessment: Client Needs – Physiological Integrity, ­Reduction
Reduction of Risk Potential: Cognitive Level – Analysis: of Risk Potential: Cognitive Level – Analysis: Concept –
Concept – Perfusion. Perfusion.

2. 1. There are many types of shock, but the 4. 1. The client’s diet is not priority when transcrib-
one common intervention which should be ing orders.
done first in all types of shock is to estab- 2. An IV antibiotic is the priority medication
lish an intravenous line with a large-bore for the client with an infection, which is
catheter. The low blood pressure and cold, the definition of sepsis—a systemic bacte-
clammy skin indicate shock. rial infection of the blood. A new order for
2. This blood pressure does not require dopa- an IV antibiotic should be implemented
mine; fluid resuscitation is first. within one (1) hour of receiving the order.
3. The client may need ABGs monitored, but this 3. Diagnostic tests are important but not pri-
is not the first intervention. ority over intervening in a potentially life-­
4. An indwelling catheter may need to be inserted threatening situation such as septic shock.
for accurate measurement of output, but it is 4. There is no indication this client has diabetes
not the first intervention. in the stem of the question, and glucose levels
are not associated with signs/symptoms of
TEST-TAKING HINT: This question asks for the
septicemia.
first intervention, which means all options
may be appropriate interventions for the cli- TEST-TAKING HINT: Remember, if the test taker
ent, but only one should be implemented first. can rule out two answers—options “1” and
Remember: When the client is in distress, do “4”—and cannot determine the right answer
not assess. between options “2” and “3,” select the option
Content – Medical: Integrated Nursing Process – directly affecting or treating the client, which
Implementation: Client Needs – Safe Effective Care is antibiotics. Diagnostic tests do not treat the
Environment, Management of Care: Cognitive client.
Level – Synthesis: Concept – Perfusion. Content – Medical: Category of Health Alteration –
Emergency: Integrated Nursing Process – Implementation:
Client Needs – Safe Effective Care Environment, Manage-
ment of Care: Cognitive Level – Application: Concept –
Perfusion.

588

15_Colgrove_Ch15.indd 588 02/06/16 12:39 PM


C hapter 15  E mergency N ursing 589

5. 1. The client diagnosed with neurogenic shock 7. 1. Ambulating the client in the hall will not
will have dry, warm skin, rather than cool, ­address the etiology of the client’s chills and
moist skin as seen in hypovolemic shock. fever; in fact, this could increase the client’s
2. The client will have bradycardia instead of discomfort.
tachycardia, which is seen in other forms 2. Monitoring these laboratory data does not
of shock. ­address the etiology of the client’s diagnosis.
3. Wheezing is associated with anaphylactic 3. Sequential compression devices help prevent
shock. deep vein thrombosis.
4. Decreased bowel sounds occur in the hyper­ 4. Antipyretic medication will help decrease
dynamic phase of septic shock. the client’s fever, which directly addresses
TEST-TAKING HINT: The test taker should iden- the etiology of the client’s nursing
tify the body system the question is address- diagnosis.
ing. In this case, neuro- indicates the question TEST-TAKING HINT: The test taker must
relates to the neurological system. With this know the problem “alteration in comfort” is
information only, the test taker could pos- ­addressed by the goal and the interventions
sibly rule out option “4,” which refers to the address the etiology, which is “chills
gastrointestinal system, and option “3,” which and fever.”
refers to the respiratory system. Although Content – Medical: Integrated Nursing Process –
bradycardia is in the cardiac system, the pulse ­Diagnosis: Client Needs – Safe Effective Care Environment,
rate is controlled by the brain. Management of Care: Cognitive Level – Analysis:
Content – Medical: Category of Health Alteration – Concept – Perfusion.
­Emergency: Integrated Nursing Process – Assessment: 8. 1. Specimens should be put into biohazard bags
Client Needs – Physiological Integrity, Reduction of Risk
prior to leaving the client’s room.
­Potential: Cognitive Level – Analysis: Concept – Perfusion.
2. This is the appropriate way to clean hands and
6. 1. It is too late to ask the client about drug aller- does not warrant intervention.
gies because the medication has already been 3. This is the appropriate way to dispose of soiled
administered. linens and does not warrant intervention.
2. Obtaining a specimen after the antibiotic has 4. Taking a stethoscope from a client who is in
been initiated will skew the culture and sen- isolation to another room is a violation of
sitivity results. It must be obtained before the infection-control principles.
antibiotic is started. TEST-TAKING HINT: This is an “except” ques-
3. Anytime a nurse administers a medication tion. The stem is asking which action warrants
for the first time, the client should be ob- intervention; therefore, the test taker must
served for a possible anaphylactic reaction, select the option indicating an inappropriate
especially with antibiotics. action by the unlicensed assistive personnel.
4. The client is being discharged and the nurse
Content – Medical: Integrated Nursing Process –
can encourage the client to do this at home, Implementation: Client Needs – Safe Effective Care
but it is not appropriate to do in the emer- Environment, Management of Care: Cognitive
gency department. Level – Synthesis: Concept – Nursing Roles.
TEST-TAKING HINT: The test taker must be
­observant of information in the stem. The
nurse has already administered the medica-
tion, and checking for allergies after the fact
will not affect the client’s outcome. This is a
violation of the five (5) rights; this medication
cannot be the right medication if the client is
allergic to it.
Content – Medical: Integrated Nursing Process –
­Implementation: Client Needs – Safe Effective Care
­Environment, Management of Care: Cognitive Level –
Application: Concept – Medication.

15_Colgrove_Ch15.indd 589 02/06/16 12:39 PM


590 M ed -S urg S uccess

9. 1. Cardiogenic shock occurs when the heart’s Content – Medical: Integrated Nursing Process –
ability to contract and pump blood is im- Implementation: Client Needs – Safe Effective Care
paired and the supply of oxygen to the heart Environment, Management of Care: Cognitive
and tissues is inadequate, such as occurs in Level – Application: Concept – Perfusion.
myocardial infarction or valvular damage. 11. 1. This is a normal potassium level (3.5 to
2. These client’s signs/symptoms make the 5.5 mEq/L); therefore, the nurse does not
nurse suspect the client is losing blood, need to notify the HCP.
which leads to hypovolemic shock, which 2. A culture result showing a high sensitivity to
is the most common type of shock and is an antibiotic indicates this is the antibiotic
characterized by decreased intravascular the client should be receiving.
volume. The client’s taking of NSAID 3. A pulse oximeter reading of greater than 93%
medications puts her at risk for hemor- indicates the client is adequately oxygenated.
rhage because NSAIDs inhibit prosta- 4. A sensitivity report indicating a resistance
glandin production in the stomach, which to the antibiotic being administered indi-
increases the risk of developing ulcers, cates the medication the client is receiv-
which can erode the stomach lining and ing is not appropriate for the treatment
lead to hemorrhaging. of the infectious organism, and the HCP
3. In neurogenic shock, vasodilation occurs as a needs to be notified so the antibiotic can
result of a loss of sympathetic tone. It can re- be changed.
sult from the depressant action of medication
TEST-TAKING HINT: The key words in option
or lack of glucose.
“2” are “high sensitivity,” and this should
4. Septic shock is a type of circulatory shock
make the test taker think this is a good thing.
caused by widespread infection.
In option “4,” the word “resistant” indicates
TEST-TAKING HINT: The test taker must look something wrong with the antibiotic and the
at the signs and symptoms and realize this need for intervention.
client is in shock. Tachycardia and hypoten- Content – Medical: Integrated Nursing Process –
sion with clammy skin indicate shock. The ­Assessment: Client Needs – Physiological Integrity,
additional information in the stem describes ­Reduction of Risk Potential: Cognitive Level – Analysis:
a particular medication, an NSAID, which Concept – Medication.
can cause a peptic ulcer.
12. 1. The hypodynamic phase is the last and
Content – Medical: Integrated Nursing Process –
irreversible phase of septic shock, charac-
­Assessment: Client Needs – Physiological Integrity,
­Reduction of Risk Potential: C
­ ognitive Level – Analysis:
terized by low cardiac output with vaso-
Concept – Perfusion. constriction. It reflects the body’s effort
to compensate for hypovolemia caused by
10. 1. Monitoring the telemetry will not prevent the loss of intravascular volume through
cardiogenic shock. It might help identify the capillaries.
changes in the hemodynamics of the heart, 2. In the compensatory phase of shock, the heart
but it does not prevent anything from rate, blood pressure, and respiratory rate are
occurring. within normal limits, but the skin may be
2. Turning the client every two (2) hours will cold and clammy and urinary output may be
help prevent pressure ulcers, but it will do decreased. However, this is the first phase of
nothing to prevent cardiogenic shock. all types of shock and is not specific to septic
3. Promoting adequate oxygenation of the shock.
heart muscle and decreasing the cardiac 3. The hyperdynamic phase, the first phase of
workload can prevent cardiogenic shock. septic shock, is characterized by a high car-
4. Placing the client’s head below the heart will diac output with systemic vasodilation. The
not prevent cardiogenic shock. This position BP may remain within normal limits, but the
can be used when a client is in hypovolemic heart rate increases to tachycardia and the
shock. ­client becomes febrile.
TEST-TAKING HINT: If the test taker has no 4. The progressive phase is the second phase
idea what the correct answer is, the test taker of all shocks. It occurs when the systolic BP
should apply Maslow’s hierarchy of needs, decreases to less than 80 to 90 mm Hg, the
which states oxygenation is most important. heart rate increases to greater than 150 beats
The test taker must know positions the per minutes, and the skin becomes mottled.
­client may be put in during different disor-
ders and diseases.

15_Colgrove_Ch15.indd 590 02/06/16 12:39 PM


C hapter 15  E mergency N ursing 591

TEST-TAKING HINT: There are some ques- TEST-TAKING HINT: If the test taker were
tions the test taker must know; they are ­totally unaware of the correct answer, then
called knowledge-based questions. The pre- the choice should be either option “1” or
fix hypo- is in the stem with “hypodynamics” option “4” because these are at either end
and matches the “hypo” of “hypotension”; of the spectrum. This gives the test taker a
this along with decreased urine output might 50/50 chance of selecting the correct answer,
help the test taker arrive at an answer. instead of a 25% chance.
Content – Medical: Integrated Nursing Process – Content – Medical: Integrated Nursing Process –
­Assessment: Client Needs – Physiological Integrity, Implementation: Client Needs – Physiological Integrity,
­Reduction of Risk Potential: Cognitive Level – Analysis: Reduction of Risk Potential: Cognitive Level – Knowledge:
Concept – Perfusion. Concept – Violence.

15. 1. Masks are kept at designated areas, not at


Bioterrorism ­every entry door.
2. This is a true statement, but it is not the most
13. 1. Power lines are not typical sources of biologi- important information; in an emergency
cal terrorism, which is what these symptoms ­situation, the respondent should use the
represent. equipment even if not trained.
2. The nurse should take note of any unusual 3. The health-care providers are not guaran-
illness for the time of year or clusters of teed absolute protection, even with all the
clients coming from a single geographical training and protective equipment. This is
location who all exhibit signs/symptoms of the most important information individu-
possible biological terrorism. als wearing protective equipment should
3. This might be appropriate for gastroenteritis know because all other procedures should
secondary to food poisoning but is not the be followed at all times.
nurse’s first thought to determine a biological 4. This is a true statement, but it is not the most
threat. The nurse must determine if the cli- important statement.
ents have anything in common. TEST-TAKING HINT: There are very few ques-
4. This is important information to obtain for tions where the test taker should select an
all clients but is not pertinent to determine a option with the word “all.” Option “3” is
biological threat. stating this is not an “always” situation. The
TEST-TAKING HINT: Option “4” is a question test taker should not automatically assume it
the nurse asks all clients; therefore, the test is not a possible answer until understanding
taker should eliminate it based on the spe- the context.
cific question. Power lines are electrical and Content – Medical: Integrated Nursing Process –
most bioterrorism threats involve chemical ­Planning: Client Needs – Safe Effective Care Environment,
or biological threats, so option “1” can be Safety and Infection Control: Cognitive Level – Synthesis:
eliminated. Concept – Violence.

Content – Medical: Integrated Nursing Process – 16. 1. This is not a rationale; this is a statement of
­Assessment: Client Needs – Physiological Integrity, what is done at the area.
­Reduction of Risk Potential: Cognitive Level – Analysis: 2. This separates the clients until decontamina-
Concept – Violence. tion occurs, but the question is asking for the
14. 1. Level A protection is worn when the scientific rationale.
highest level of respiratory, skin, eye, and 3. This is false statement—the supplies should
mucous membrane protection is required. not be kept in the decontamination area.
In this situation of possible inhalation of 4. Avoiding cross-contamination is a priority
anthrax, such protection is required. for personnel and equipment—the fewer
2. Level B protection is similar to Level A pro- the number of people exposed, the safer
tection, but it is used when a lesser level of the community and area.
skin and eye protection is needed. TEST-TAKING HINT: Options “1” and “2” are
3. Level C protection requires an air-purified not rationales.
respirator (APR), which uses filters or absor- Content – Medical: Integrated Nursing Process –
bent materials to remove harmful substances. Implementation: Client Needs – Safe Effective Care
4. Level D is basically the work uniform. Environment, Safety and Infection Control: Cognitive
Level – Analysis: Concept – Violence.

15_Colgrove_Ch15.indd 591 02/06/16 12:39 PM


592 M ed -S urg S uccess

17. 1. In most situations this is the first step, but 19. 1. Scabby, clear fluid-filled vesicles are charac-
with a potential chemical or biological ex- teristic of chickenpox.
posure, the first step must be the safety of 2. Exposure to anthrax bacilli via the skin
the hospital; therefore, the client must be results in skin lesions, which cause edema
decontaminated. with pruritus and the formation of mac-
2. This is the second step in the decontamina- ules or papules, which ulcerate, forming a
tion process. one (1)- to (3)-mm vesicle. Then a pain-
3. This is the first step. Depending on the less eschar develops, which falls off in
type of exposure, this step alone can one (1) to two (2) weeks.
­remove a large portion of exposure. 3. Irregular brownish-pink spots around the
4. This assumption could cost many people in hairline are characteristic of rubella.
the hospital staff, as well as clients, their lives. 4. Tiny purple spots flush with the skin surface
TEST-TAKING HINT: If the test taker wants to are petechiae.
­select option “4” as the correct answer, the TEST-TAKING HINT: This is a knowledge-based
test taker should be careful—assumptions question. The test taker should try to deter-
are dangerous. The test taker may want to mine which skin disease each answer option
choose option “1” because it involves as- describes to rule out the incorrect answers.
sessment, but exposure to a chemical agent Content – Medical: Integrated Nursing Process –
should be considered distress and an action ­Assessment: Client Needs – Physiological Integrity,
should be implemented first. ­Reduction of Risk Potential: Cognitive Level – Analysis:
Content – Medical: Integrated Nursing Process – Concept – Violence.
Implementation: Client Needs – Safe Effective Care 20. 1. Cremation is recommended because the
Environment, Safety and Infection Control: Cognitive
virus can stay alive in the scabs of the body
Level – Synthesis: Concept – Trauma.
for 13 years.
18. 1. Sources of biological agents include inhala- 2. An open casket might allow for the spread of
tion, insects, animals, and people. the virus to the general public; therefore, the
2. The only known vaccine against a possible nurse should not make this suggestion. The
bioterrorism agent is the smallpox vaccine, nurse should not tell the client’s family how
which is not available in quantities sufficient to make funeral arrangement for viewing.
to inoculate the public. 3. Burying the body quickly is the second best
3. Because of the vast range of agents, biologi- option for safety of the funeral home person-
cal weapons are more of a threat. A biological nel and anyone who could come in contact
agent could be released in one city and ­affect with the body. The quicker the burial, the
people in other cities thousands of miles safer the situation (if the family refuses
away. cremation).
4. Because of the variety of agents, the 4. The hospital, not the client’s family, must
means of transmission, and lethality of the ­notify the public health department.
agents, biological weapons, including an- TEST-TAKING HINT: Option “4” could be elimi-
thrax, smallpox, and plague, are especially nated because the test taker should know the
dangerous. hospital staff, not family members, must make
TEST-TAKING HINT: Answer option “1” should notifications to public health departments.
be eliminated because of the word “only.” The test taker should apply basic concepts
Even if the test taker has little knowledge of when answering questions. Infection-control
biological warfare, knowledge of the human concepts eliminate option “2” because an
body suggests a wide range of ways biological open casket allows for spread of the virus.
agents could be transmitted. Content – Medical: Integrated Nursing Process –
Content – Medical: Integrated Nursing Process – ­Planning: Client Needs – Safe Effective Care Environment,
­Planning: Client Needs – Safe Effective Care Environment, Safety and Infection Control: Cognitive Level – Synthesis:
Safety and Infection Control: Cognitive Level – Synthesis: Concept – Safety.
Concept – Violence.

15_Colgrove_Ch15.indd 592 02/06/16 12:39 PM


C hapter 15  E mergency N ursing 593

21. 1. The absence of breathing is death, and this is 23. 1. Language difficulties can increase fear and
neither a viable option nor a sensible recom- frustration on the part of the client.
mendation to terrified people. 2. Some religions have specific practices
2. Standing up will avoid heavy exposure ­related to medical treatments, hygiene,
because the chemical will sink toward the and diet, and these should be honored if
floor or ground. at all possible.
3. Staying below the level of the smoke is the 3. Prayers in time of grief and disaster are
instruction for a fire. important to an individual and actually can
4. Breathing through the clothing, which is have a calming affect on the situation.
probably contaminated with the chemical, 4. Caring for the dead is as important as
will not provide protection from the chemical caring for the living based on religious
entering the lung. beliefs.
TEST-TAKING HINT: If the test taker does not 5. For purposes of organization this may be
know the answer, the test taker should real- needed, but it is not addressing cultural
ize options “1” and “4” address breathing ­sensitivity and in some instances may violate
and options “2” and “3” address positioning cultural needs of the client and the family.
and one set of options should be eliminated, TEST-TAKING HINT: The stem asks the test
narrowing the choice to one (1) out of taker to address cultural needs, and these
two (2) options. are ­universal—in a bioterrorism attack or
Content – Medical: Integrated Nursing Process – with an individual in the hospital. The test
Implementation: Client Needs – Safe Effective Care taker should select options addressing cul-
Environment, Safety and Infection Control: Cognitive tural needs. Dishonoring cultural needs can
Level – Application: Concept – Safety. increase the client’s anxiety and increase
22. 1. Anemia, leukopenia, and thrombocytopenia, ­problems for the health-care team.
signs of bone marrow depression, are signs/ Content – Medical: Integrated Nursing Process –
symptoms the client experiences in the latent ­Planning: Client Needs – Safe Effective Care Environment,
Management of Care: Cognitive Level – Synthesis:
phase of radiation exposure, which occurs
­Concept – Violence.
from 72 hours to years after exposure. The
client is usually asymptomatic in the prodro- 24. 1. Many hospital procedures mandate off-duty
mal phase of radiation exposure. nurses should not report immediately so
2. Sudden fever, chills, and enlarged lymph ­relief can be provided for initial responders.
nodes are signs/symptoms of bubonic plague. 2. The nurse’s first responsibility is to the facil-
3. The prodromal phase (presenting symp- ity of employment, not the community.
toms) of radiation exposure occurs 48 to 3. This is a good action to take when the nurse
72 hours after exposure and the signs/ is notified of the next action. For example, if
symptoms are nausea, vomiting, diarrhea, the hospital is quarantined, the nurse may not
anorexia, and fatigue. Signs/symptoms report for days.
of higher exposures of radiation include 4. The nurse should follow the hospital’s
­fever, respiratory distress, and excitability. policy. Many times nurses will stay at
4. These are signs/symptoms of inhalation home until decisions are made as to where
botulism. the employees should report.
TEST-TAKING HINT: If the test taker knows the TEST-TAKING HINT: After looking at all the
definition of “prodromal,” which is an early options, the test taker should select the op-
sign of a developing condition or disease tion which best assesses the entire situation,
(prodrom is Greek for “running before”), which is following policy. There will be a
then the option with vague and nonspecific tendency for mass hysteria to occur in the
signs/symptoms should be selected as the community, but following the terrorist attack
correct answer. on 9/11, all hospitals and communities are
Content – Medical: Integrated Nursing Process – now required by Homeland Security to have
­Assessment: Client Needs – Physiological Integrity, a disaster preparedness plan in place. The
­Reduction of Risk Potential: Cognitive Level – Analysis: best action the nurse can take is to follow the
Concept – Safety. procedure and remain calm.
Content – Medical: Integrated Nursing Process –
Implementation: Client Needs – Safe Effective Care
Environment, Management of Care: Cognitive
Level – Application: Concept – Violence.

15_Colgrove_Ch15.indd 593 02/06/16 12:39 PM


594 M ed -S urg S uccess

Codes 4. Documentation is important but not priority


over treating the client.
25. 1. This is the third intervention based on the TEST-TAKING HINT: The phrase “most impor-
answer options available in this question. tant” in the stem is the key to answering this
2. This is the fourth intervention based on the question. All four (4) options are appropri-
options available in this question. ate interventions for the question, but only
3. This is the first intervention the nurse one (1) is the most important. The test taker
should implement after finding the client should remember to always select the ­option
unresponsive on the floor. directly affecting the client, and this may
4. The rapid response team is called if the c­ lient mean not selecting an assessment interven-
is breathing; a code would be called if the tion when the client is in distress.
­client were not breathing. Content – Medical: Integrated Nursing Process –
TEST-TAKING HINT: Options “1,” “2,” and Implementation: Client Needs – Safe Effective Care
“3” are all assessment interventions, which Environment, Management of Care: Cognitive
is the first step in the nursing process. Of Level – Application: Concept – Assessment.
these three (3) possible options, the test 28. 1. This is the correct statement explaining
taker should select the intervention easiest what an AED does when used in a code.
and fastest to determine if the client is alert, 2. The Life Pack on the crash cart must be used
which is to shake and shout at the client. to perform synchronized cardioversion.
Content – Medical: Integrated Nursing Process – 3. This is the explanation for a pulse oximeter.
Implementation: Client Needs – Safe Effective Care 4. This is not the function of the AED.
Environment, Management of Care: Cognitive
TEST-TAKING HINT: The test taker must know
Level – Application: Concept – Nursing Roles.
equipment to be able to answer this ques-
26. 1. This hand position will help prevent posi- tion. The test taker may be able to eliminate
tioning the hand over the xiphoid process, options based on knowledge of what other
which can break the ribs and lacerate the liver equipment does.
during compressions. Content – Medical: Integrated Nursing Process –
2. This is the correct two-rescuer CPR; ­Planning: Client Needs – Safe Effective Care Environment,
­therefore, no intervention is needed. Management of Care: Cognitive Level – Knowledge:
3. The sternum should be depressed one ­Concept – Nursing Roles.
and one-half (1.5) to two (2) inches during
29. 1. Atrial fibrillation is not a life-threatening
compressions to ensure adequate circula-
­dysrhythmia; it is chronic.
tion of blood to the body; therefore, the
2. Asymptomatic sinus bradycardia may be
nurse needs to correct the UAP.
­normal for the client, especially for athletes
4. The UAP should request another health-
or long-distance runners.
care provider to perform compressions when
3. Ventricular fibrillation is the most com-
exhausted.
mon dysrhythmia associated with sudden
TEST-TAKING HINT: The test taker must select cardiac death; ventricular fibrillation is
which option is an incorrect procedure for responsible for 65% to 85% of sudden
cardiac compressions. cardiac deaths.
Content – Medical: Integrated Nursing Process – ­ 4. “Supraventricular” means “above the ven-
Evaluation: Client Needs – Safe Effective Care tricle.” The atrium is above the ventricle, and
­Environment, Management of Care: Cognitive Level – atrial dysrhythmias are not life threatening.
Synthesis: Concept – Nursing Roles.
TEST-TAKING HINT: The test taker should
27. 1. This is an appropriate intervention, but it is know the left ventricle is responsible for
not the most important. pumping blood to the body (heart muscle
2. Sterile technique should be maintained as and brain) and could eliminate options
much as possible, but the nurse can treat a “1” and “4” as correct answers. The word
live body with an infection without using “asymptomatic” should cause the test taker
sterile technique; however, the nurse cannot to eliminate option “2” as the correct answer.
treat a dead body without an infection. Content – Medical: Integrated Nursing Process –
3. This is the most important intervention. ­Assessment: Client Needs – Physiological Integrity,
The nurse should always treat the client ­Reduction of Risk Potential: Cognitive Level – Analysis:
based on the nurse’s assessment and data Concept – Perfusion.
from the monitors; an intervention should
not be based on data from the monitors
without the nurse’s assessment.

15_Colgrove_Ch15.indd 594 02/06/16 12:39 PM


C hapter 15  E mergency N ursing 595

30. 1. The chaplain should be called to help 32. 1. This is not the definition of sudden cardiac
­address the client’s family or significant death; this is sometimes known as “pulling
others. A small community hospital does the plug” on clients who are diagnosed as
not have a 24-hour on-duty pastoral ser- brain dead.
vice. A chaplain is part of the code team in 2. This is not the definition of sudden cardiac
large medical center hospitals. death.
2. The social worker does not need to be 3. Unexpected death occurring within
­notified of a code. one (1) hour of the onset of cardiovascu-
3. The respiratory therapist responds to the lar symptoms is the definition of sudden
code automatically without a referral. The c­ ardiac death.
respiratory therapist is part of the code team 4. This is not the definition of sudden cardiac
and one (1) is on duty 24 hours a day, even in death.
a small community hospital. TEST-TAKING HINT: If the test taker re-
4. The director of nurses does not need to be lates the word “sudden” in the stem with
notified of codes, but possibly the house “­unexpected,” the best answer is option “3.”
­supervisor should be notified. The test taker must be aware of adjectives
TEST-TAKING HINT: The test taker must know and adverbs.
the roles of the multidisciplinary health-care Content – Medical: Integrated Nursing Process –
team to make appropriate referrals. The ­Diagnosis: Client Needs – Safe Effective Care Environment,
words “community hospital” are an impor- Management of Care: Cognitive Level – Analysis:
tant phrase to help determine the correct Concept – Nursing Roles.
answer.
33. 1. Gastric distention occurs from overventi-
Content – Medical: Integrated Nursing Process – lating clients. When compressions are per-
Implementation: Client Needs – Safe Effective Care formed, the pressure will cause vomiting,
Environment, Management of Care: Cognitive
which may cause aspiration into the lungs.
Level – Application: Concept – Nursing Roles.
2. The health-care provider does not require
31. 1. A jaw thrust is used for a possible fractured suctioning equipment to intubate.
neck. The nurse should use the head-tilt, 3. Nothing ensures a patent airway, except a
chin-lift maneuver to open the airway. correctly inserted endotracheal tube, and
2. The nurse should cover the client’s mouth ­suction is needed to clear the airway.
and nose with the nurse’s mouth when giv- 4. Suction equipment is for the client’s needs,
ing mouth-to-mouth resuscitation to an not the health-care provider’s needs.
infant but not when giving mouth-to-mouth TEST-TAKING HINT: Option “4” could be
resuscitation to an adult. According to the eliminated because the equipment is for the
American Heart Association 2010 Guidelines client, not for the nurse or health-care pro-
mouth to mouth is only performed with a viders. The word “ensures” in option “3” is
barrier device in place to protect the rescuer. an absolute word, so the test taker should be
3. An oral airway is not mandatory to do effec- cautious before selecting this option.
tive breathing; therefore, it is not the most
Content – Medical: Integrated Nursing Process –
important intervention. ­Diagnosis: Client Needs – Safe Effective Care Environment,
4. Nurses should protect themselves against Management of Care: Cognitive Level – Analysis:
possible communicable disease, such as Concept – Nursing Roles.
HIV and hepatitis, and should be pro-
tected if the client vomits during CPR. 34. 1. A ventilator is not kept on the medical-
surgical floors and is not routinely brought to
TEST-TAKING HINT: Unless the stem provides
the bedside. The client is manually ventilated
an age for the client, the client is an adult
until arriving in the intensive care unit.
­client; therefore, the test taker could elimi-
2. The crash cart is the mobile unit with the
nate option “2” because it is for an infant.
defibrillator and all the medications and
Content – Medical: Integrated Nursing Process – supplies needed to conduct a code.
Implementation: Client Needs – Safe Effective Care 3. The gurney, a stretcher, may be needed when
Environment, Management of Care: Cognitive
the client is being transferred to another unit,
Level – Analysis: Concept – Nursing Roles.
but it is not an immediate need, and in some
hospitals the client is transferred in the bed.
4. Oxygen is available in the room and portable
oxygen is on the crash cart, so it doesn’t need
to be brought separately.

15_Colgrove_Ch15.indd 595 02/06/16 12:39 PM


596 M ed -S urg S uccess

TEST-TAKING HINT: This is knowledge the test TEST-TAKING HINT: The test taker must be
taker must have. The crash cart is the primary familiar with basic nursing math and become
piece of equipment, and in most facilities there comfortable with the equations the test taker
is a person assigned to bring the crash cart to uses to compute dosage calculations.
the client’s bedside. Content – Medical: Integrated Nursing Process –
Content – Medical: Integrated Nursing Process – Implementation: Client Needs – Physiological Integrity,
Implementation: Client Needs – Safe Effective Care Pharmacological and Parenteral Therapies: Cognitive
Environment, Management of Care: Cognitive Level – Application: Concept – Medication.
Level – Application: Concept – Nursing Roles.

35. 1. This is providing immediate direct care to the Disasters/Triage


client and is not performed for legal purposes.
2. The key to answering the question is “legal,” and 37. 1. The MSDS provides chemical information
direct care is not performed for legal purposes. regarding specific agents, health informa-
3. This is providing immediate direct care to tion, and spill information for a variety of
the client and is not performed for legal pur- chemicals. It is required for ­every chemi-
poses. This is an occasion where someone cal found in the hospital.
else is allowed to document another nurse’s 2. This situation requires an occurrence or
medication administration. ­accident report.
4. The chart is a legal document, and the 3. Any facility administering antineoplastic
code must be documented in the chart agents (medications used to treat cancer) is
and provide information needed in the in- required to have specific chemotherapy spill
tensive care unit. kits available and a policy and procedure
TEST-TAKING HINT: Answer options “1,” “2,” included; in this situation the nurse already
and “3” have the nurse providing direct hands- knows the chemical involved.
on care. Option “4” is the only option address- 4. This requires a hospital variance report
ing documentation and should be selected as and notifying the employee health or
the correct answer because it is different. infection-control nurse.
Content – Medical: Integrated Nursing Process – TEST-TAKING HINT: If the test taker were not
Implementation: Client Needs – Safe Effective Care aware of an MSDS, the name tells the test taker
Environment, Management of Care: Cognitive to look for content in the answer options ad-
Level – Application: Concept – Nursing Roles.
dressing materials; therefore, options “2” and
36. 45 mL/hr. “4” could be eliminated as possible answers.
The test taker could remember the mne- Content – Medical: Integrated Nursing Process – ­
monic, which is “For 1 mg, 2 mg, 3 mg, Evaluation: Client Needs – Safe Effective Care
4 mg the rate is 15 mL, 30 mL, 45 mL, ­Environment, Management of Care: Cognitive Level –
60 mL.” If the test taker has not memorized Synthesis: Concept – Safety.
it, it is too late to figure it out in an emer- 38. 1. The child needs an x-ray to confirm the
gency situation. But for math purposes: First ­fracture, but the client is stable and does not
determine the number of milligrams of lido- have a life-threatening problem.
caine in the 500 mL of D5W: 2. The triage nurse should see this client
2 g × 1,000 mg = 2,000 mg per 500 mL first because these are symptoms of a
Then determine how many milligrams per myocardial infarction, which is potentially
milliliters: life threatening.
2,000 mg ÷ 500 mL = 4 mg/mL 3. These are symptoms of a migraine headache
Then find out how many milliliters must be and are not life threatening.
infused per minute to give the ordered dose 4. A laceration on the hand is priority, but not
of 3 mg/min. In algebraic terms: over a client having a myocardial infarction.
4 mg : 1 mL = 3 mg : x mL
TEST-TAKING HINT: The test taker should
Cross multiply and divide:
evaluate each option on a scale of one (1) to
x = 3/4 or 0.75
10, with one (1) being the least critical cli-
The number of milliliters to be infused in a
ent and 10 being life threatening. Option “2”
minute is 3/4 mL or 0.75.
rates a score of 10.
The infusion pump is set at an hourly rate, so
multiply 3/4 by 60 minutes: Content – Medical: Integrated Nursing Process – ­
3/4 × 60 = 45 Assessment: Client Needs – Safe Effective Care
­Environment, Management of Care: Cognitive Level –
The pump should be set at 45 mL/hr to
Analysis: Concept – Nursing Roles.
­infuse three (3) mg/min.

15_Colgrove_Ch15.indd 596 02/06/16 12:39 PM


C hapter 15  E mergency N ursing 597

39. 1. Practice drills allow for troubleshoot- 3. The people in this area are usually chaplains
ing any issues before a real-life incident or social workers, not direct client care per-
occurs. sonnel. In a disaster, direct care personnel
2. A deactivation response is important so cannot be spared for this duty.
resources are not overused, and the facil- 4. New settings and atypical roles for
ity can then get back to daily activities and nurses may be required during disasters;
routine care. medical-surgical nurses can provide first
3. Communication between the facility and aid and may be required to work in unfa-
­external resources and an internal communi- miliar settings.
cation plan are critical. TEST-TAKING HINT: The test taker should look
4. A postincident response is important to at traditional nursing roles requiring nursing
­include a critique and debriefing for all par- expertise and eliminate crowd control or rid-
ties involved; a preincident response is the ing in an ambulance.
plan itself. Be sure to read adjectives closely. Content – Medical: Integrated Nursing Process –
5. A coordinated security plan involving Implementation: Client Needs – Safe Effective Care
facility and community agencies is the Environment, Management of Care: Cognitive
key to controlling an otherwise chaotic Level – Application: Concept – Nursing Roles.
situation.
42. 1. This client should be classified as an Immedi-
TEST-TAKING HINT: The test taker must no-
ate Category, Priority 1, and color red. If not
tice adjectives such as “only” in option “3”
treated STAT, a tension pneumothorax will
and “preincident” in option “4.” These words
occur.
make these options incorrect. This question
2. This client has a very poor prognosis, and
requires the test taker to select more than
even with treatment, survival is unlikely.
one option as the correct answer.
3. This client should be classified as a Delayed
Content – Medical: Integrated Nursing Process – Category, Priority 2, and color yellow. This
­Planning: Client Needs – Safe Effective Care Environment, client receives treatment after the casualties
Management of Care: Cognitive Level – Synthesis:
requiring immediate treatment are treated.
Concept – Nursing Roles.
4. This client is a Minimal Category, Priority 3,
40. 1. This describes injuries color-coded black and color green. This client can wait days for
or Priority 4 and is called the Expectant treatment.
Category. TEST-TAKING HINT: If the test taker did
2. This is a description of injuries color-coded not know the definition of the categories,
green or Priority 3 and is called the Minimal ­looking at the word “black,” which has a con-
Category. notation of death, and the word “expectant”
3. These are injuries color-coded yellow or might lead the test taker to select the worst-
­Priority 2 and is called the Delayed Category. case scenario.
4. This is called the Immediate Category.
Content – Medical: Integrated Nursing Process –
Individuals in this group can progress ­Assessment: Client Needs – Physiological Integrity,
­rapidly to Expectant if treatment is ­Reduction of Risk Potential: Cognitive Level – Analysis:
delayed. Concept – Trauma.
TEST-TAKING HINT: This is basically a
43. 1. This organization mandates all health-care
­knowledge-based question, but often the
facilities to have an emergency operations
color “red” indicates a high priority.
plan, but it is a national agency, not a federal
Content – Medical: Integrated Nursing Process – agency.
­Planning: Client Needs – Safe Effective Care Environment, 2. Most cities and all states have an OEM,
Safety and Infection Control: Cognitive Level – Synthesis:
which coordinates the disaster relief efforts at
Concept – Trauma.
the state and local levels.
41. 1. The nurse should not leave the hospital area; 3. Federal resources include organizations
the nurse must wait for the casualties to come such as DHHS and the Department of
to the facility. Justice. Each of these federal departments
2. This is a position requiring knowledge of oversees hundreds of agencies, including
­instruments and procedures not common to the American Red Cross, which respond
the medical-surgical floor. to disasters.
4. MMRS teams are local teams located in cities
deemed to be possible terrorist targets.

15_Colgrove_Ch15.indd 597 02/06/16 12:39 PM


598 M ed -S urg S uccess

TEST-TAKING HINT: The question asks for a TEST-TAKING HINT: The nurse should address
federal agency. The word “metro” means the situation with the reporter and provide
“local”; therefore, option “4” could be elimi- an access for the information. Options “1,”
nated. All health-care providers should be “3,” and “4” do not help the reporter get
aware of the role of The Joint Commission in a­ ccurate information.
the hospital, so the test taker could eliminate Content – Medical: Category of Health Alteration –
option “1.” ­Emergency: Integrated Nursing Process – Implementation:
Content – Medical: Integrated Nursing Process – Client Needs – Safe Effective Care Environment, Safety and
­Planning: Client Needs – Safe Effective Care Environment, Infection Control: Cognitive Level – Analysis: Concept –
Management of Care: Cognitive Level – Synthesis: Nursing Roles.
­Concept – Nursing Roles.
46. 1. This is the correct procedure when tagging a
44. 1. CISM is an approach to preventing and client and does not warrant intervention.
treating the emotional trauma affecting 2. Vital signs should be documented on the tag.
emergency responders as a consequence The tag takes the place of the client’s chart,
of their job. Performing CPR and treat- so this does not warrant intervention.
ing a young child affects the emergency 3. The tag should never be removed from
personnel psychologically, and the death the client until the disaster is over or the
increases the traumatic experience. client is admitted and the tag becomes a
2. Caring for this type of client is an expected part of the client’s record. The HCP needs
part of the job. If the nurse finds this trau- to be informed immediately of the action.
matic enough to require a CISM, then the 4. The tag can be attached to any part of the
nurse should probably leave the emergency ­client’s body.
department. TEST-TAKING HINT: This question is asking the
3. This requires an intense time for triaging and test taker to identify an incorrect option for
caring for the victims, but without fatalities the situation. Sometimes asking which action is
this should not be as traumatic for the staff. appropriate helps identify the correct answer.
4. This is a dangerous practice because medica- Content – Medical: Integrated Nursing Process – ­
tion errors and other mistakes may occur as Evaluation: Client Needs – Safe Effective Care
a result of fatigue, but this is not a traumatic ­Environment, Management of Care: Cognitive Level –
situation. Synthesis: Concept – Nursing Roles.
TEST-TAKING HINT: The test taker should
47. 1. This will help diffuse the escalating situa-
examine the words “critical,” “incident,” and tion and attempt to keep the father calm.
“stress.” Each option should be examined 2. Sending the father to the waiting room does
to determine which is the most traumatic. not help his behavior and could possibly
Needless deaths of innocent children are make his behavior worse; loud and obnoxious
psychologically traumatic. behavior can become violent.
Content – Medical: Integrated Nursing Process – 3. This will not help the current situation and
Implementation: Client Needs – Safe Effective Care could make it worse because the nurse doesn’t
Environment, Management of Care: Cognitive
know the home situation.
Level – Analysis: Concept – Nursing Roles.
4. The nurse should notify hospital security be-
45. 1. The media have an obligation to report fore calling the police department.
the news and can play a significant positive TEST-TAKING HINT: The rule concerning
role in communication, but communication ­dealing with anger is to directly address the
should come from only one source—the client and diffuse the situation. There is only
­disaster command center. one option ­addressing this rule, option “1.”
2. Emergency operations plans will have Content – Medical: Integrated Nursing Process –
a designated disaster plan coordinator. Implementation: Client Needs – Psychosocial Integrity:
All public information should be routed Cognitive Level – Application: Concept – Nursing Roles.
through this person.
3. Client confidentiality must be maintained, 48. 1. This puts the nurse in a dangerous position
but the best action is for the nurse to help and might cause the death of the nurse.
the reporter get to the appropriate area for 2. This will escalate the situation.
information. 3. This is a dangerous position for the nurse to
4. This allows the reporter to stay in the emer- put himself or herself in.
gency department, which is inappropriate. 4. Self-protection is priority; the nurse is not
required to be injured in the line of duty.

15_Colgrove_Ch15.indd 598 02/06/16 12:39 PM


C hapter 15  E mergency N ursing 599

TEST-TAKING HINT: Self-protection is a 51. 1. Vomiting is never induced in clients who have
­ riority. There is no advantage to protecting
p ingested corrosive alkaline substances or pe-
others if the caregivers are also injured. The troleum distillates. More damage can occur to
only option protecting the nurse is to get out the esophagus and pharynx.
of the line of fire. 2. A gastric lavage may be done but not by
Content – Medical: Integrated Nursing Process – inserting an NGT and attaching it to wall
Implementation: Client Needs – Safe Effective Care suction.
Environment, Management of Care: Cognitive 3. Airway edema or obstruction can occur as
Level – Application: Concept – Nursing Roles. a result of the burning action of corrosive
substances.
4. Water or milk may be administered to
Poisoning dilute the substance if the airway is not
compromised.
49. 1. CPS should be contacted only if the nurse
TEST-TAKING HINT: This is an emergency
suspects an intentional administration of the
poison, but at this time determining which situation. If the test taker did not know the
poison the child has swallowed and the answer, Maslow’s hierarchy of needs puts
­antidote is the priority. a­ irway first.
2. The local police department is only notified if Content – Medical: Integrated Nursing Process –
the nurse suspects child abuse. Implementation: Client Needs – Safe Effective Care
Environment, Management of Care: Cognitive
3. The Department of Health does not need to
Level – Application: Concept – Trauma.
be notified.
4. The Poison Control Center can assist 52. 1. These are signs and symptoms of carbon
the nurse in identifying which chemical monoxide poisoning. Pulse oximetry is not a
has been ingested by the child and the valid test because the hemoglobin is saturated
antidote. with the carbon monoxide and a false high
TEST-TAKING HINT: The test taker should reading is being obtained.
a­ nalyze each option to determine what 2. These are signs and symptoms of carbon
information could be obtained. Then the monoxide poisoning. Symptoms include
test taker should put this information in skin color from a cherry red to cyanotic
order of priority. Even if the nurse suspects and pale, headache, muscular weakness,
child abuse, the priority is to help the child palpitations, dizziness, and confusion and
immediately. can progress rapidly to coma and death.
Content – Medical: Integrated Nursing Process –
Oxygen should be administered 100% at
Implementation: Client Needs – Safe Effective Care hyperbaric or atmospheric pressures to
Environment, Management of Care: Cognitive reverse hypoxia and accelerate elimination
Level – Application: Concept – Trauma. of the carbon monoxide.
3. This may be done, but it is not the first
50. 1. The primary goal for the ED nurse is to action.
stop the action of the poison and then 4. This may need to be done, but getting ­oxygen
maintain organ functioning. to the brain is first.
2. ED nurses do not provide long-term care.
TEST-TAKING HINT: Three (3) of the four (4)
3. Antidotes are administered to neutralize the
effects of poisons, not to increase the effects. options concern oxygenation. The test taker
4. Treatment is implemented to hasten the must then decide which of the three (3) has a
elimination of the poison. higher priority.
Content – Medical: Category of Health Alteration –
TEST-TAKING HINT: The test taker should
­Emergency: Integrated Nursing Process –
read each option carefully. ED nurse and Implementation: Client Needs – Safe Effective Care
“long-term care” don’t match. Increasing the Environment, ­Management of Care: Cognitive Level –
effects and prolonging the elimination of the Analysis: Concept – Trauma.
poison is damaging to the client.
Content – Medical: Integrated Nursing Process –
­Diagnosis: Client Needs – Safe Effective Care Environment,
Management of Care: Cognitive Level – Analysis:
Concept – Trauma.

15_Colgrove_Ch15.indd 599 02/06/16 12:39 PM


600 M ed -S urg S uccess

53. 1. The client should be placed on the left TEST-TAKING HINT: Usually oxygen is a prior-
side, which allows the gastric contents to ity, but in this scenario the client has dan-
pool in the stomach and decreases passage gerous chemicals on the skin. The stem did
of fluid into the duodenum during lavage. not tell the test taker the respirations were
After the placement of the orogastric a problem. It is important not to read into a
tube, the head is lowered to facilitate question.
removal of the gastric contents. Content – Medical: Integrated Nursing Process –
2. A large-bore tube is placed through the Implementation: Client Needs – Safe Effective Care
mouth into the stomach of a client who is Environment, Safety and Infection Control: Cognitive
comatose and an endotracheal tube is inserted Level – Analysis: Concept – Trauma.
into the airway prior to beginning lavage to 55. 1. Fluid volume loss is a concern because of the
prevent aspiration. potential for the client to go into hypovole-
3. Standby suction is an emergency measure mic shock, but this is not priority over airway.
to prevent aspiration in case the client 2. Clients with botulism are at risk for re-
vomits. spiratory paralysis, and this is the priority
4. Removing stomach contents before begin- problem.
ning the lavage helps to prevent overdis- 3. The client will be in pain and pain is a prior-
tention of the stomach and aspiration. ity, but it does not come before airway and
5. Samples of the first two (2) lavage wash- fluid volume.
ings should be sent to the laboratory to be 4. The client may be anxious, but a psychosocial
analyzed for chemical compounds. problem usually can be ranked after a physi-
TEST-TAKING HINT: When deciding on the cor- ological one in priority.
rect answers for an alternative-type question, TEST-TAKING HINT: Maslow’s hierarchy of
each option must be examined for its own needs lists airway as the highest priority.
merit. One good answer does not exclude
Content – Medical: Integrated Nursing Process –
another answer. In option “1,” anatomical po-
­Diagnosis: Client Needs – Safe Effective Care Environment,
sitioning can help the test taker determine if Management of Care: Cognitive Level – Analysis:
the position is a good one. In option “2,” the Concept – Trauma.
test taker must look at the adjective “small”
and ask if this facilitates or hinders irrigation 56. 123,000 mg of analgesic medication were
of the stomach. Even if the test taker did not consumed.
know what the circumstances are, option “3” The container originally contained
could be a good action. 250 ­capsules. Two (2) doses of two (2)
Content – Medical: Integrated Nursing Process – ­capsules each were removed.
Implementation: Client Needs – Safe Effective Care 2 × 2 = 4
Environment, Management of Care: Cognitive 250 capsules – 4 capsules = 246 capsules
Level – Analysis: Concept – Trauma. remaining
Each capsule contains 500 mg.
54. 1. The skin should be immediately drenched 246 capsules × 500 mg = 123,000 mg of
with water from a hose or shower. A con- medication consumed
stant stream of water is applied. Time
should not be lost by removing the clothes TEST-TAKING HINT: The test taker must not
first and then proceeding to rinsing with overlook a step in the problem. On the
water. If a dry powder form of white phos- NCLEX-RN, the test taker should check
phorus or lye spilled onto the client, it is ­answers with the drop-down calculator.
brushed off and then the client is placed Content – Medical: Integrated Nursing Process –
under the shower. Implementation: Client Needs – Safe Effective Care
2. The first action is to remove the poison from Environment, Management of Care: Cognitive
Level – Application: Concept – Medication.
the client’s skin and prevent further damage.
3. If the client becomes dyspneic, the nurse
administers oxygen while waiting for the
paramedics.
4. The vat should be labeled as to the chemical
contents per Occupational Safety and Health
Administration (OSHA) regulations, but if
not, then the nurse must determine which
chemicals are in the vat so the HCP can treat
the client appropriately.

15_Colgrove_Ch15.indd 600 02/06/16 12:39 PM


C hapter 15  E mergency N ursing 601

57. 1. Although this is seen as a first action in old 59. 1. UAPs can obtain intake and outputs, but
television westerns, it is not a recommended evaluating the information is the nurse’s
action for clients who have been bitten by a responsibility.
snake. This action will cause further damage 2. This is a sterile dressing change and
to the tissue by restricting blood flow to the should not be delegated.
tissue. 3. A UAP can assist clients to get up to the
2. This is an action seen in classic television ­bedside commode as long as the UAP is
programs and movies from the 1950s and knowledgeable about body mechanics.
1960s, but this is not the current treatment 4. The UAP can assist a client to turn and ask
for snakebite. If this is done, the rescuer will the client to cough and deep breathe.
suck the venom into the rescuer’s mouth and TEST-TAKING HINT: The task requiring knowl-
possibly be poisoned. edge of sterile technique is the one the nurse
3. Corticosteroid medications are contraindi- should perform. Any task requiring special-
cated in the first six (6) to eight (8) hours af- ized knowledge or nursing judgment cannot
ter the bite because they might interfere with be delegated.
antibody production and hinder the action of
Content – Medical: Integrated Nursing Process –
the antivenin. ­Planning: Client Needs – Safe Effective Care ­Environment,
4. The client should lie down, all restrictive Management of Care: Cognitive Level – Synthesis:
items such as rings should be removed, ­Concept – Nursing Roles.
the wound should be cleansed and covered
with a sterile dressing, the affected body 60. 1. Before administering antivenin, the af-
part should be immobilized, and the client fected body part must be measured, and
should be kept warm. it is remeasured every 15 minutes during
a four (4)- to six (6)-hour procedure. The
TEST-TAKING HINT: The test taker should
infusion is begun slowly and increased
not jump to what is depicted in the mass
after 10 minutes. The affected part is
media as the correct answer. Both options
measured every 30 to 60 minutes after the
“1” and “2” have answers portrayed in the
infusion and for 48 hours to detect symp-
media as the correct method of caring for
toms of compartment syndrome (edema,
snake bite. This should give the test taker a
loss of pulse, increased pain, and pares-
clue: If both options cannot be right, then
thesias). Allergic reactions to the antivenin
both are probably wrong.
are not uncommon and are usually the
Content – Medical: Integrated Nursing Process – result of a too-rapid infusion of the anti-
Implementation: Client Needs – Safe Effective Care venin. The most experienced nurse should
Environment, Management of Care: Cognitive
be assigned this client.
Level – Application: Concept – Trauma.
2. This client is beyond critical danger and is
58. 1. Installing carbon monoxide detectors in the being discharged, so a less experienced nurse
home is a recommended safety measure. could care for this client.
2. The lips should be pink, not bright red 3. This client has many needs, but anger is not a
or blue. This indicates a saturation of the priority over a physiological need.
hemoglobin with carbon monoxide. This 4. A less experienced nurse could care for this
client needs more instruction. client.
3. Because carbon monoxide is colorless and TEST-TAKING HINT: The test taker can rule
odorless, it can be dangerous. It is detected out options “2” and “3” because of the
with special detectors. ­discharge information and psychosocial
4. One of the major causes of accidental carbon ­versus physiological problem.
monoxide poisoning is a faulty furnace. Content – Nursing Management: Integrated Nursing
TEST-TAKING HINT: Three (3) options, “1,” Process – Planning: Client Needs – Safe Effective Care
“3,” and “4,” are about protecting the home Environment, Management of Care: Cognitive Level –
and the client from inhaling carbon monox- Synthesis: Concept – Nursing Roles.
ide. If the test taker did not know the answer,
a good choice is the different option.
Content – Medical: Integrated Nursing Process – ­
Evaluation: Client Needs – Safe Effective Care
­Environment, Management of Care: Cognitive Level –
Synthesis: Concept – Trauma.

15_Colgrove_Ch15.indd 601 02/06/16 12:39 PM


602 M ed -S urg S uccess

Violence, Physical Abuse, and Neglect 63. 1. Research suggests at least 67% of adoles-
cents who are runaways or homeless have
61. 1. This action could cause the spouse to become been abused in the home. This represents
violent. The security personnel should not at- a learned behavior pattern getting the
tempt to remove the spouse unless the client ­female adolescent attention.
wishes them to do so. 2. One reason adolescents of both sexes run
2. Injuries resulting from spousal abuse should away from home is abuse in the home. Noth-
be discussed without the abuser present. ing in the stem indicates the client was turned
3. This may or may not be true. The client will out of the home for any behavior.
have to prosecute, and many times the abused 3. This has the nurse medically diagnosing the
client will not do so. The client may feel re- client.
sponsible for the abuse, or may fear for her 4. This is a judgmental statement.
children’s lives or for her own, or there may be TEST-TAKING HINT: The test taker should not
a financial hold the spouse has over the client. read into the question or choose an option
Battered woman syndrome has many facets. allowing the nurse to function outside the
4. By escorting the client to a bathroom for scope of practice. Option “2” is assuming
any reason, the nurse can get the client facts not in the stem, and option “3” is asking
to a safe area out of the hearing of the the nurse to make a medical diagnosis.
spouse. This is the most innocuous way to Content – Medical: Integrated Nursing Process –
get the client alone. ­Diagnosis: Client Needs – Safe Effective Care Environment,
TEST-TAKING HINT: When dealing with a vio- Management of Care: Cognitive Level – Analysis:
lent person, the nurse should use discretion Concept – Violence.
to avoid the spouse erupting into violence 64. 1. The school nurse is not a Sexual Assault
directed against the nurse, client, or others Nurse Examiner (SANE) nurse, and this child
in the emergency department. thinks she is pregnant, suggesting the abuse
Content – Medical: Integrated Nursing Process – has been occurring for a period of time or at
Implementation: Client Needs – Safe Effective Care least in some months past. The child should
Environment, Management of Care: Cognitive be taken to a hospital for examination.
Level – Application: Concept – Violence. 2. Child Protective Services should be
62. 1. Adult Protective Services should be called ­notified to protect the child from further
only if it is determined willful neglect or abuse and to initiate charges against the
abuse of the client is occurring. father. An intermediate school nurse cares
2. The nurse should arrange for the social for children in the 4th, 5th, 6th, or 7th
worker to see the client and family to grades, depending on the school district.
­determine if some arrangements could be 3. This action brings the abuser to the school.
made to provide for the client’s safety and 4. Sending the child to a free clinic does not
for the client to be provided with nutri- negate the nurse’s responsibility to report sus-
tious meals while the adult children are pected child abuse.
at work. A long-term care facility or adult TEST-TAKING HINT: All 50 states require the
day care may be needed. nurse to report suspected child abuse. Child
3. The Medicare ombudsman is a person who Protective Services (CPS) is the advocate to
represents a Medicare client in a long-term notify. Nurses in a school clinic do not have
care facility. the appropriate facilities to perform rape
4. The dietitian could see this client to deter- ­examinations. Option “4” does not address
mine eating preferences (74 inches = 6 foot the abuse.
2 inches and 54.5 kg = 120 pounds), but the Content – Medical: Integrated Nursing Process –
most appropriate intervention is safety. Implementation: Client Needs – Safe Effective Care
TEST-TAKING HINT: The question asks for Environment, Management of Care: Cognitive
the test taker to determine a priority inter- Level – Application: Concept – Violence.
vention. The client is diagnosed with senile
dementia and is being left alone for hours of
the day. Safety is priority.
Content – Medical: Integrated Nursing Process –
Implementation: Client Needs – Physiological Integrity,
Basic Care and Comfort: Cognitive Level – Application:
Concept – Nursing Roles.

15_Colgrove_Ch15.indd 602 02/06/16 12:39 PM


C hapter 15  E mergency N ursing 603

65. 1. Frequently child abusers will deny the 67. 1. The nurse can encourage the client to press
child’s reports of abuse and say the child is charges but has no right to insist.
a habitual liar. 2. The nurse should treat the wound and may
2. Child abusers believe violence is an ac- find it frustrating the client will not press
ceptable way to reduce tension. They tend charges, but the nurse is obligated to provide
to have a low tolerance for frustration and the client information to help the client to
have poor impulse control. get to a safe place.
3. Child abusers have a tendency toward 3. The woman is more aware of this fact than
­feelings of helplessness and hopelessness. the nurse.
4. Child abusers tend to blame the child for 4. The nurse should help the client to de-
the abuse and not admit the problem is vise a plan for safety by giving the client
their own. the number of a safe house or a women’s
5. The child abuser may have been abused as shelter.
a child, but there is no evidence of the child TEST-TAKING HINT: The test taker could
abuser being spoiled as a child. eliminate option “3” based on common
TEST-TAKING HINT: This is an alternative-type sense; the client lives in an abusive situation
question. The test taker should examine each and realizes the abuser’s potential more than
option carefully to determine if it could be the nurse. Option “2” could be eliminated
a correct answer. Option “3” could be elimi- by the phrase “do nothing else.” Option “1”
nated because of the adjective “high” and “5” could be eliminated because of the prin-
could be eliminated because of the adjective ciple of nurses empowering their clients, not
“spoiled.” ­overpowering them, which is what has been
Content – Medical: Integrated Nursing Process – happening to the client already.
­Assessment: Client Needs – Physiological Integrity, Content – Medical: Integrated Nursing Process –
­Reduction of Risk Potential: Cognitive Level – Analysis: Implementation: Client Needs – Safe Effective Care
Concept – Violence. Environment, Management of Care: Cognitive Level –
Application: Concept – Violence.
66. 1. This is a myth believed by some people.
Many individuals are raped, ranging in age 68. 1. This client has signs of ongoing abuse such
from infants to the 90s, male and female, as multiple burns and contusions in different
­heterosexuals and homosexuals. No one asks stages of healing.
to be raped. 2. Many times the elderly are ashamed to
2. If a person says he or she is not interested in ­report abuse because they raised the
any type of sexual activity, it means “no” and abuser and feel responsible for their child
anything else is forced and it is rape. “No” becoming an abuser. The elder parent may
means “no.” It is considered rape if a prosti- feel financially dependent on the child or
tute says “no.” be afraid of being placed in a long-term
3. Rape is an act of violence motivated by the care facility. Forty-seven states have Adult
rapist desiring to overpower and control Protective Services (APS) created by the
the victim. states to protect elder citizens.
4. Men and children can be victims of rape. 3. There is no evidence provided in the stem
Sexual arousal and orgasm do not imply of the client not being mentally competent,
consent; it may be a pathological response to and there is evidence in the stem of physical
stimulation. abuse. This client is performing activities of
TEST-TAKING HINT: This is an “except” ques- daily living.
tion, which means three (3) of the options 4. Consensual sex does not involve the physical
will contain correct information. In this abuse noted in the assessment.
question, there are three (3) false statements TEST-TAKING HINT: The test taker could
about rape; this is a double-negative type of ­eliminate options “1,” “3,” and “4” by examin-
question. ing the stem and noting the physical abuse
Content – Medical: Integrated Nursing Process – occurring and by the fact the client is func-
­Planning: Client Needs – Safe Effective Care Environment, tioning by performing activities of daily living.
Safety and Infection Control: Cognitive Level – Synthesis: Content – Medical: Integrated Nursing Process –
Concept – Nursing Roles. ­Diagnosis: Client Needs – Psychosocial Integrity: Cognitive
Level – Knowledge: Concept – Violence.

15_Colgrove_Ch15.indd 603 02/06/16 12:39 PM


604 M ed -S urg S uccess

69. 1. Unless the nurse is being personally abused 71. 1. This plan for the client to take RU 486, or
in the same manner the client is being abused the “morning-after” pill, prevents pregnancy
and has seen the abuse taking place, the nurse from occurring, but it does not directly
cannot “know” the client is being abused. ­address coping skills.
2. Alcohol and drugs are implicated in the abuse 2. The client may talk about “what if I had
of many clients, but not all abusers use alco- not done . . .,” but the client is not guilty of
hol or drugs. ­causing the rape.
3. This is agreeing with the abuser about the 3. The client should be provided the phone
client causing the abuse. number of a rape crisis counseling center
4. This statement assesses the abused or counselor to help the client deal with
­client’s safety (or a plan for safety). the psychological feelings of being raped.
TEST-TAKING HINT: Option “3” could be elimi- 4. This is a legal issue.
nated because it blames the victim. O ­ ption “1” TEST-TAKING HINT: The test taker should read
can be eliminated because the nurse should the stem “ineffective coping” and eliminate
not tell the client “I know” unless the nurse the physiological problem in option “1” and
has proof or has been in the situation. the legal problem in option “4.”
Content – Medical: Integrated Nursing Process – Content – Medical: Integrated Nursing Process –
­Assessment: Client Needs – Physiological Integrity, Implementation: Client Needs – Safe Effective Care
­Reduction of Risk Potential: Cognitive Level – Analysis: Environment, Management of Care: Cognitive
Concept – Violence. Level – Application: Concept – Violence.
70. 1. Clients diagnosed with PTSD are easily 72. 1. The nurse should attempt to develop a re-
startled and can react violently if awak- lationship in which the client feels he or
ened from sleep by being touched. she can trust the nurse (males are abused by
2. Touching the client can cause the client to ­significant others too).
become afraid, to believe himself or herself 2. The first step in helping a client who has
to be under attack, and to react violently. been abused is to get the client to admit
The nurse should not touch a sleeping client the abuse is happening.
­diagnosed with PTSD. 3. This could cause the abuse to escalate.
3. If the client awakes with the nurse in the 4. This is what the nurse is trying to get the
room, the client could become fearful and ­client to avoid.
­react to the fear. TEST-TAKING HINT: Option “1” could be
4. There may be times when the nurse must eliminated because it is the opposite of what
awaken the client to determine if the client is the nurse tries to establish in a nurse–client
physically stable. relationship. Option “4” places the client in
TEST-TAKING HINT: Option “4” can be elimi- harm’s way.
nated because of the absolute statement “at Content – Medical: Integrated Nursing Process –
all.” Options “2” and “3” can be eliminated ­Diagnosis: Client Needs – Psychosocial Integrity: Cognitive
if the test taker thinks of how it feels to be Level – Analysis: Concept – Violence.
startled when perceiving another person
around him or her when the test taker was
not aware of the other person’s presence.
Content – Medical: Integrated Nursing Process –
Implementation: Client Needs – Safe Effective Care
Environment, Management of Care: Cognitive
Level – Application: Concept – Stress.

15_Colgrove_Ch15.indd 604 02/06/16 12:39 PM


CONCEPTS
73. 1. This should be considered a NOW medica- 75. 1. A is the right upper shoulder. The
tion but should not be the first intervention ­electrical activity of the client’s heart will
because cultures have been ordered. be read similar to Lead II on an ECG. Cur-
2. The IV line must be started before the IVPB rent delivered during defibrillation will be
can be administered, but it is not first. sent from the right shoulder through the
3. The admission assessment must be completed heart muscle to the left lower thoracic area.
in a timely manner, but it is actually the last 2. Site B is the left upper chest area and current
intervention of the four listed. Admission will not be consistent with the normal electri-
­assessments involve a thorough head-to-toe cal pattern of the heart.
assessment and can take time to complete. 3. Site C is the left lower thoracic area and
4. The cultures are the priority interven- the electrical activity of the client’s heart
tion because the antibiotics should not be will be read similar to Lead II on an
administered until the cultures have been ECG. Current delivered during defibril-
drawn. lation will be sent from the right shoulder
TEST-TAKING HINT: After looking at all the through the heart muscle to the left lower
options, the test taker should select the thoracic area.
­option that best addresses the abnormal lab- 4. Site D is the right lower thoracic area and the
oratory value, which is the high WBC. This electrical current passing from one electrode
client has an infection and must be treated as to the other is opposite of the normal electri-
rapidly as possible but performing cultures cal pattern of the heart.
first will give the HCP information about the 5. Site E is the right arm and the electrical
causative agent without skewing the results. ­activity will not reach the cardiac muscle.
Content – Medical: Integrated Nursing Process – TEST-TAKING HINT: After looking at all the
Implementation: Client Needs – Safe Effective Care options, the test taker should select the
Environment, Management of Care: Cognitive option(s) that apply(ies) like a true/false
Level – Application: Concept – Immunity. ­response. Is the pad placement correct
for an ECG?
74. 1. The normal saline can be initiated
to maintain the client’s circulatory Content – Medical: Category of Health Alteration –
status while the PRBCs are being ­Emergency: Integrated Nursing Process – Implementation:
Client Needs – Safe Effective Care Environment,
cross-matched.
Management of Care: Cognitive Level – Application:
2. The PRBCs are the third order to initiate. Concept – Safety.
3. Furosemide is not to be administered until
after the client has received the first unit of 76. 1. The nurse should wait to be assigned when
PRBCs. responding to a disaster; even if the nurse
4. Diphenhydramine is the second order to works in the emergency department he/she
­initiate because it is to be administered prior may be assigned to a different area for the
to the PRBCs. disaster.
TEST-TAKING HINT: After looking at all the 2. The nurse should not assume where he/she
options, the test taker should select the will be utilized during a disaster. The com-
­option which that best addresses the en- mand center administrative nurse will deter-
tire situation. This client has a perfusion/­ mine where the greatest need exists.
hematologic regulation issue. 3. The command center administrative nurse
will determine where the greatest need
Content – Medical: Integrated Nursing Process –
Implementation: Client Needs – Safe Effective Care
exists. The nurse reports there to receive
Environment, Management of Care: Cognitive assignments.
Level – Application: Concept – Hematologic Regulation. 4. The nurse should not delay responding to the
hospital to pack a bag.
TEST-TAKING HINT: After looking at all the
options, the test taker should think “How
will I know where the greatest need is?” The
answer is to ask for instructions, not assume
the answer.

605

15_Colgrove_Ch15.indd 605 02/06/16 12:39 PM


606 M ed -S urg S uccess

Content – Medical: Integrated Nursing Process – 79. 1. The nurse should not panic but follow the
Implementation: Client Needs – Safe Effective Care infection-control procedures as guided by
Environment, Management of Care: Cognitive the Centers for Disease Control (CDC).
Level – Application: Concept – Trauma. 2. This is not a professional response to a public
77. 1. The charge nurse would not reassign the health issue. The best action the nurse can
clients in the middle of the shift. If the nurse take is to follow the procedure and remain
who was caring for the client has time he/she calm.
can assist the other staff. 3. Television news reports are frequently
2. The HCP should notify the family of the ­inaccurate and biased.
­client’s arrest and current bed assignment. 4. The nurse should not discuss hospital situ-
3. The charge nurse must maintain a culture ations in a public venue; rather, the nurse
of readiness for any emergency. The crash should refer the news reporter to the Public
cart must be checked and restocked for a Relations Officer for the hospital.
future emergency. TEST-TAKING HINT: After looking at all the
4. A unit meeting can be held when the crash options, the test taker should select the
cart has been restocked and the floor has ­option which best assesses the entire situa-
settled down, but it is not the priority. tion, which is following policy. There will be
TEST-TAKING HINT: After looking at all the a tendency for mass hysteria to occur in the
options, the test taker should select the community but the CDC has plans for this
­option that best cares for the remaining type of emergency.
­clients on the unit. Content – Medical: Nursing Process – Implementation:
Content – Medical: Integrated Nursing Process – Client Needs – Safe Effective Care Environment,
Implementation: Client Needs – Safe Effective Care Management of Care: Cognitive Level – Application:
Environment, Management of Care: Cognitive Concept – Safety.
Level – Application: Concept – Management. 80. 1. The nurse should start an intravenous line
78. 1. The 13-year-old is having an elective surgery with a large-bore needle to administer
and could be rescheduled. fluids and blood if needed. This client may
2. Abdominal aortic aneurysm surgeries need rapid replacement of fluid volume.
are not scheduled until the aneurysm is 2. The nurse must assess the client to determine
at least 7 cm and a risk for rupture. This if the client is stable or unstable before being
­client should not be discharged. able to delegate this task to a UAP.
3. This client is on oral medication and the INR 3. This will be done but not first.
is in therapeutic range; this client could be 4. The nurse must assess the client to determine
discharged. if the client is stable or unstable before allow-
4. Not wanting to care for oneself is not a valid ing the client to go the radiology department.
reason to remain in the hospital. TEST-TAKING HINT: The nurse should initiate
TEST-TAKING HINT: After looking at all the an IV because in an emergency, this, followed
options, the test taker should select the op- with rapid fluid replacement, can stabilize an
tion which best assesses the entire situation. unstable client. Then the nurse must deter-
Option “1” describes an elective surgery and mine if the client is stable before delegating
could be postponed. The client described in activities or allowing the client to leave the ED.
option “3” is on oral medication, Coumadin, Content – Medical: Integrated Nursing Process –
with an INR that is within therapeutic range. Implementation: Client Needs – Safe Effective Care
And the client in option “4” has a psychoso- Environment, Management of Care: Cognitive
cial issue, and this does not warrant remain- Level – Application: Concept – Perfusion.
ing in the hospital.
Content – Medical: Integrated Nursing Process –
Implementation: Client Needs – Safe Effective Care
Environment, Management of Care: Cognitive
Level – Application: Concept – Management.

15_Colgrove_Ch15.indd 606 02/06/16 12:39 PM


C hapter 15  E mergency N ursing 607

EMERGENCY NURSING
COMPREHENSIVE EXAMINATION
1. The ED nurse is caring for a client diagnosed 6. A nurse is at the lake when a person nearly
with multiple rib fractures. Which data should the drowns. The nurse determines the client is
nurse include in the assessment? breathing spontaneously. Which data should the
1. Level of orientation to time and place. nurse assess next?
2. Current use and last dose of medication. 1. Possibility of drug use.
3. Symmetrical movement of the chest. 2. Spinal cord injury.
4. Time of last meal the client ate. 3. Level of confusion.
4. Amount of alcohol.
2. The nurse is caring for a client in the ED with
abdominal trauma who has had peritoneal lavage. 7. The ED nurse is caring for a male client admitted
Which intervention should the nurse include in with carbon monoxide poisoning. Which
the plan of care? intervention requires the nurse to notify the rapid
1. Assess for the presence of blood, bile, or feces. response team?
2. Palpate the client for bilateral femoral pulses. 1. The client has expectorated black sputum.
3. Perform Leopold’s maneuver every eight (8) 2. The client reports trying to kill himself.
hours. 3. The client’s pulse oximeter reading is 94%.
4. Collect information on the client’s dietary 4. The client has stridor and reports dizziness.
history.
8. The ED nurse is working triage. Which client
3. The elderly client is brought to the ED should be triaged first?
complaining of cramps, headache, and weakness 1. A client who has multiple injuries from a
after working outside in the sun. The telemetry motor-vehicle accident.
shows sinus tachycardia. Which intervention 2. A client complaining of epigastric pain and
should the nurse implement? nausea after eating.
1. Determine if the client is experiencing any 3. An elderly client who fell and fractured the left
thirst. femoral neck.
2. Administer D5W intravenously at 250 mL/hr. 4. The client suffering from a migraine headache
3. Maintain a cool environment to promote rest. and nausea.
4. Withhold the client’s oral intake.
9. The nurse is providing discharge teaching for
4. The ED nurse is caring for a client who suffered a the client with intermaxillary wiring to repair a
near-drowning. Which expected outcome should fractured mandible. Which statement by the client
the nurse include in the plan of care for this indicates teaching has been effective?
client? 1. Iced alcoholic drinks may be consumed by
1. Maintain the client’s cardiac function. using a straw.
2. Promote a continued decrease in lung 2. Only one (1) food item should be consumed at
surfactant. one (1) time.
3. Warm rapidly to minimize the effects of 3. Carbonated sodas should be limited to two
hypothermia. (2) daily.
4. Keep the oxygen saturation level above 93%. 4. Teeth can be brushed after tenderness and
edema subside.
5. The nurse is assessing the client who suffered
a near-drowning event. Which data require
immediate intervention?
1. The onset of pink, frothy sputum.
2. An oral temperature of 97˚F.
3. An alcohol level of 100 mg/dL.
4. A heart rate of 100 beats/min.

15_Colgrove_Ch15.indd 607 02/06/16 12:39 PM


608 M ed -S urg S uccess

10. The occupational health nurse is called to the 15. The ED nurse is caring for a client with
scene of a traumatic amputation of a finger. fractured pelvis and bladder trauma secondary to
Which intervention should the nurse implement a motor-vehicle accident. Which data are most
prior to sending the client to the ED? Select all important for the nurse to assess?
that apply. 1. Monitor the creatinine and BUN.
1. Rinse the amputated finger with sterile 2. Check urine output hourly.
normal saline. 3. Note the amount and color of the urine.
2. Place the amputated finger in a sealed and 4. Assess for bladder distention.
watertight plastic bag.
16. The school nurse is caring for a child with a
3. Place the amputated finger into iced saline
deep laceration. Which intervention should the
solution.
nurse implement first?
4. Wrap the amputated finger in saline-moistened
1. Clean with saline solution.
gauze dressings.
2. Apply direct pressure.
5. Replace the amputated finger on the hand and
3. Don nonsterile gloves.
wrap with gauze.
4. Notify the child’s parents.
11. The nurse is teaching the client home care
17. The ED receives a client involved in a
instructions for a reimplanted finger after a
motor-vehicle accident. The nurse notes a
traumatic amputation. Which information
large hematoma on the right flank. Which
should the nurse include in the teaching?
intervention should the nurse implement first?
1. Perform range-of-motion exercises weekly.
1. Insert an indwelling urinary catheter.
2. Smoking may be resumed if it does not cause
2. Take the vital signs every 15 minutes.
nausea.
3. Monitor the skin turgor every hour.
3. Protect the finger and be careful not to
4. Mark the edges of the bruised area.
reinjure the finger.
4. An elevated temperature is the only reason to 18. Which expected outcome is priority for the
call the HCP. nurse who is caring for a client with chest
trauma from a gunshot injury?
12. The ED nurse is caring for a client diagnosed
1. The client will have an absence of pain.
with frostbite of the feet. Which intervention
2. The client will maintain a BP of 90/60.
should the nurse implement?
3. The client will have symmetrical chest
1. Massage the feet vigorously.
expansion.
2. Soak the feet in warm water.
4. The client will maintain urine output of
3. Apply a heating pad to feet.
30 mL/hr.
4. Apply petroleum jelly to feet.
19. Which problem is most appropriate for the
13. A student reports to the school nurse with
nurse to identify for the client experiencing renal
complaints of stinging and burning from a wasp
trauma?
sting. Which intervention should the nurse
1. Infection of the renal tract.
implement?
2. Ineffective tissue perfusion.
1. Grasp the stinger and pull it out.
3. Alteration in skin integrity.
2. Apply a warm, moist soak to the area.
4. Alteration in temperature.
3. Cleanse the site with alcohol.
4. Apply an ice pack to the site. 20. The nurse is discharging a client from the ED
with a sutured laceration on the right knee.
14. The ED nurse is caring for a client who had a
Which information is most important for the
severe allergic reaction to a bee sting. Which
nurse to obtain?
discharge instructions should the nurse discuss
1. The date of the client’s last tetanus injection.
with the client?
2. The name of the client’s regular health-care
1. Instruct the client to wear a medical
provider.
identification bracelet.
3. Explain the sutures must be removed in 10 to
2. Apply corticosteroid cream to the site to
14 days.
prevent anaphylaxis.
4. Determine if the client has any drug or food
3. Administer epinephrine 1:10,000
allergies.
intravenously every three (3) minutes.
4. Teach the client to avoid attracting insects by
wearing bright colors.

15_Colgrove_Ch15.indd 608 02/06/16 12:39 PM


C hapter 15  E mergency N ursing 609

21. The nurse working in an outpatient clinic is 25. The ED nurse is completing the initial
caring for a client who is experiencing epistaxis. assessment on a client who becomes
Which intervention should the nurse implement unresponsive. Which intervention should the
first? nurse implement first?
1. Take the client’s blood pressure in both arms. 1. Assess the rate and site of the intravenous
2. Hold the nose with thumb and finger for fluid.
15 minutes. 2. Administer an ampule of sodium bicarbonate.
3. Have the client sit with the head tilted back 3. Assess the cardiac rhythm shown on the
and hold a tissue. monitor.
4. Prepare to administer silver nitrate, a 4. Prepare to cardiovert the client into sinus
cauterizing agent, with a packing applicator. rhythm.
22. The client with a temperature of 94˚F is being 26. The ED nurse is caring for a female client
treated in the ED. Which intervention should with a greenstick fracture of the left forearm
the nurse implement to directly elevate the and multiple contusions on the face, arms,
client’s temperature? trunk, and legs. The significant other is in the
1. Remove the client’s clothing. treatment area with the client. Which nursing
2. Place a warm air blanket over the client. interventions should the nurse implement?
3. Have the client change into a hospital gown. List in order of priority.
4. Raise the temperature in the room. 1. Determine if the client has a plan for safety.
2. Assess the pulse, temperature, and capillary
23. The ED nurse is caring for the client who
refill of the left wrist and hand.
has taken an overdose of cocaine. Which
3. Ask the client if she feels safe in her own
intervention should the nurse delegate to the
home.
unlicensed assistive personnel (UAP)?
4. Request the significant other wait in the
1. Evaluate the airway and breathing.
waiting room during the examination.
2. Monitor the rate of intravenous fluids.
5. Notify the social worker to consult on the
3. Place the cardiac monitor on the client.
case.
4. Transfer the client to the intensive care unit.
24. The client has been brought to the ED by
ambulance following a motor-vehicle accident
with a flail chest, an intravenous line, and a
Heimlich valve. Which intervention should the
nurse implement first?
1. Start a large-bore intravenous access.
2. Request a portable chest x-ray.
3. Prepare to insert chest tubes.
4. Assess the cardiac rhythm on the monitor.

15_Colgrove_Ch15.indd 609 02/06/16 12:39 PM


Emergency Nursing Comprehensive
Examination Answers and Rationales

1. 1. Orientation to person, place, and time should 3. 1. Elderly clients lose the defense mechanism of
be assessed on all clients, but this information increased thirst with dehydration. This does
will not provide specific information about the not accurately indicate fluid deficit.
chest trauma. 2. An intravenous fluid should be administered,
2. Current use of all medication and the last but the solution should correct fluid and elec-
doses should be assessed for all clients. trolyte imbalances. D5W does not replace
3. When a client suffers from multiple rib electrolytes lost, and 250 mL/hr could place
fractures, the client has an increased risk the client at risk for heart failure if the body
for flail chest. The nurse should assess the cannot adjust rapidly to the fluid replacement.
client for paradoxical chest wall movement 3. The nurse should encourage the client to
and, if respiratory distress is present, for rest and should maintain a cool environ-
pallor and cyanosis. ment to assist the client to recover from
4. The time of this last meal is important if the heat exhaustion. The elderly are more sus-
client were to have surgery or intubation ceptible to this condition.
planned. A nutritional assessment should be 4. If the client can tolerate oral fluids, the client
performed on all clients. should be encouraged to drink fluids to replace
Content – Medical: Integrated Nursing Process – electrolytes lost in excessive sweating.
Assessment: Client Needs – Physiological Integrity, Content – Medical: Integrated Nursing Process –
Reduction of Risk Potential: Cognitive Level – ­Implementation: Client Needs – Safe Effective
Analysis: Concept – Trauma. Care Environment, Management of Care: Cognitive Level –
Application: Concept – Thermoregulation.
2. 1. A diagnostic peritoneal lavage is performed
to assess the presence of blood, bile, and 4. 1. An expected outcome is a desired occurrence,
feces from internal bleeding induced by not a common event. Tachycardia is a common
injury. If any of these are present, surgery manifestation of a near-drowning event, but it
should be considered to explore the extent is not desired. A combination of physiological
of damage and repair of the injury. changes, hypothermia, and hypoxia put the cli-
2. Palpating the client’s peripheral pulses indi- ent at risk for life-threatening cardiac rhythms.
cates blood flow to the extremities. Femoral 2. Any near-drowning causes a decrease in al-
pulses are not necessarily assessed if all distal veolar surfactant, which results in alveolar
pulses are strong. collapse. A decrease in surfactant is not the
3. Leopold’s maneuver is performed on pregnant desired outcome.
clients to assess the position of the fetus. 3. The client needs to be rewarmed slowly to
4. Dietary history is information which is as- reduce the influx of metabolites. These me-
sessed but not in an emergency situation. tabolites, including lactic acid, remain in the
­Assessments need to be efficient and direct to extremities.
eliminate any time-wasting activities. 4. The oxygen level needs to be maintained
Content – Medical: Nursing Process – Planning: Client greater than 93%. The client needs as much
Needs – Safe Effective Care Environment, Management of support as necessary for this. ­Mechanical
Care: Cognitive Level – Synthesis: Concept – Trauma. ventilation with peak end-expiratory pres-
sure (PEEP) and high oxygen levels may be
needed to achieve this goal.
Content – Medical: Integrated Nursing Process – Diagno-
sis: Client Needs – Safe Effective Care
Environment, Management of Care: Cognitive
Level – Analysis: Concept – Trauma.

610

15_Colgrove_Ch15.indd 610 02/06/16 12:39 PM


C hapter 15  E mergency N ursing 611

5. 1. The onset of pinky, frothy sputum indi- 4. Stridor or dizziness indicates an occlusion
cates the client is experiencing pulmonary of the airway, which is a medical emer-
edema. This needs to be treated to prevent gency. The RRT is called when the client is
further decline in this client. experiencing a decline but is still breathing.
2. An oral temperature of 97˚F is in the lower Content – Medical: Integrated Nursing Process –
level of within normal limits. ­Implementation: Client Needs – Safe Effective Care
3. A blood alcohol of 100 mg/dL is an elevation ­Environment, Management of Care: Cognitive Level –
but should not be considered priority over Analysis: Concept – Trauma.
pulmonary edema. Treatments for elevations
8. 1. Injuries from a motor-vehicle accident can
in toxicology levels can be considered after the
be life threatening. This client should be
client is stable.
assessed first to rule out respiratory diffi-
4. A heart rate of 100 beats/min is tachycardia
culties and hemorrhage.
but not at a critical level. The nurse needs to
2. Epigastric pain with nausea after eating sounds
remember Maslow’s hierarchy of priority, and
like gallbladder disease. Pain has high priority
airway and breathing are physiological needs
but not over breathing and hemorrhage.
that take priority.
3. Elderly clients have special fluid and electro-
Content – Medical: Integrated Nursing Process – lyte issues after a fall. The cause of the fall
Assessment: Client Needs – Physiological Integrity, may be cardiac, but the question does not
Reduction of Risk Potential: Cognitive Level –
­indicate this.
Analysis: Concept – Trauma.
4. Migraine headaches are painful experiences,
6. 1. The use of drugs can alter the treatment of but they do not have a higher priority than
and recovery from the near-drowning event. breathing and hemorrhage.
This is information needed, but it is not prior- Content – Medical: Integrated Nursing Process –
ity at this time. ­Implementation: Client Needs – Safe Effective Care
2. An injury of the spinal cord should be consid- ­Environment, Management of Care: Cognitive Level –
ered and the spine should be assessed, but after Analysis: Concept – Trauma.
the client has been stabilized. The nurse does
9. 1. Alcoholic beverages should be avoided to
not complete an assessment of a potential spi-
­prevent nausea and vomiting. The client
nal injury before assessing oxygenation status.
should be taught where and how to cut wires
3. The nurse should assess the victim for
if vomiting occurs.
hypoxia. Signs and symptoms of hypoxia
2. A combination of foods should be blended into
include confusion or irritability and al-
a milk shake and consumed to maintain caloric
terations in level of consciousness, such as
intake and promote nutrition.
lethargy.
3. Carbonated sodas can cause foam in the back
4. The amount of alcohol ingestion will affect the
of the throat and may induce vomiting.
treatment, but this is not a higher priority than
4. Hygiene is helpful in healing. The mouth
oxygenation.
should be rinsed and an irrigation device
Content – Emergency: Integrated Nursing should be used frequently. Gentle brushing
Process – Assessment: Client Needs – Safe Effective Care and rinsing the mouth after each meal and
Environment, Management of Care: Cognitive Level –
at bedtime can begin after edema and ten-
Analysis: Concept – Trauma.
derness subside.
7. 1. The client diagnosed with carbon monoxide Content – Medical: Integrated Nursing Process –
poisoning frequently has black sputum from ­Evaluation: Client Needs – Physiological Integrity,
inhaling soot, so the RRT does not need to be ­Physiological Adaptation: Cognitive Level – Synthesis:
notified. Concept – Trauma.
2. The client admitting to attempting sui-
cide requires the client being placed on one
(1)-to-one (1) suicide precautions and psycho-
logical counseling.
3. A pulse oximeter reading of 94% indicates the
client is being well oxygenated and does not
require notifying the RRT.

15_Colgrove_Ch15.indd 611 02/06/16 12:39 PM


612 M ed -S urg S uccess

10. 1. The amputated finger and all tissue 13. 1. The stinger should not be grasped because
should be rinsed with sterile normal saline the wasp’s venom sac may release more toxin.
to remove dirt and sent to the ED with The stinger should be scraped in the opposite
the client. direction.
2. Place the finger and all tissue in a water- 2. Warmth increases the blood flow, which will
tight, sealed plastic bag to prevent loss increase the edema.
and contamination. 3. The site should be cleaned with soap and
3. The finger or other tissue should not be ­water, not alcohol.
placed on ice or in saline solution because this 4. The nurse should apply an ice pack to the
will cause severe damage to the tissue cells. site. The cold will decrease the blood flow
4. The finger should be wrapped in gauze and sensation. The ice should be applied
moistened with sterile normal saline. intermittently.
5. The finger should not be replaced on the Content – Medical: Integrated Nursing Process –
hand and wrapped with gauze in the field. ­Implementation: Client Needs – Safe Effective Care
The surgeon will determine if reattachment is ­Environment, Management of Care: Cognitive Level –
possible. ­Application: Concept – Trauma.
Content – Medical: Integrated Nursing Process – 14. 1. Clients who have severe reactions to
­Implementation: Client Needs – Safe Effective Care ­insect stings should wear identifying
­Environment, Management of Care: Cognitive Level –
bracelets to provide information. If the
­Application: Concept – Trauma.
client is unconscious, the bracelet can
11. 1. Exercises should be performed several times alert the health-care provider so treat-
each day, not weekly. ment can be started.
2. Smoking causes vasoconstriction, which will 2. Corticosteroid creams treat local reactions,
compromise the implanted finger’s survival. not systemic ones.
3. The client should take extra care to pro- 3. Epinephrine 1:10,000 is administered in-
tect the finger from injury. The periph- travenously during a code situation or for a
eral nerves protecting the finger require severe anaphylactic reaction to an allergen.
months to regenerate. This client is being discharged and may need
4. The client needs to report any signs of an EpiPen to carry at all times but not IV
­rejection of the finger, such as infection or epinephrine.
impaired circulation, not just an elevated 4. Bright-colored clothing attracts insects.
temperature. ­Clients who are allergic to insect stings
Content – Medical: Integrated Nursing Process – should learn how to avoid them to decrease
­Planning: Client Needs – Physiological Integrity, the risk. Flowery-smelling perfumes and
­Physiological Adaptation: Cognitive Level – Synthesis: ­lotions should also be avoided.
Concept – Trauma. Content – Medical: Integrated Nursing Process –
12. 1. Massaging or rubbing tissue with frostbite Planning: Client Needs – Physiological Integrity,
­Physiological Adaptation: Cognitive Level – Synthesis:
will cause further damage.
Concept – Trauma.
2. Soaking the feet in a warm bath of 107˚F
causes rapid continuous rewarming. 15. 1. The creatinine and BUN assess kidney func-
3. Heating pads are not used to rewarm tissue tion, but the nurse should assess bladder
with frostbite. Heating pads can cause tissue function by checking the amount and color of
damage from burns, especially in tissue with the urine.
impaired sensation. 2. Checking the urine output hourly is appro-
4. Petroleum jelly does not affect the tempera- priate data to assess but not the most impor-
ture of the tissue. tant for a client with bladder trauma.
Content – Medical: Integrated Nursing Process – 3. The amount and color of urine assists
­Implementation: Client Needs – Safe Effective Care with diagnosing the extent of injury. Color
­Environment, Management of Care: Cognitive of the urine indicates the presence of
Level – Application: Concept – Trauma. blood. The amount indicates whether the
urine is contained throughout the path-
way from bladder to urinary meatus.
4. The nurse should not palpate a client with
bladder trauma because it could cause further
damage.

15_Colgrove_Ch15.indd 612 02/06/16 12:39 PM


C hapter 15  E mergency N ursing 613

Content – Medical: Integrated Nursing Process – Content – Medical: Integrated Nursing Process –
Assessment: Client Needs – Physiological Integrity, Diagnosis: Client Needs – Safe Effective Care ­Environment,
Reduction of Risk Potential: Cognitive Level – Management of Care: Cognitive Level – Analysis:
Analysis: Concept – Trauma. Concept – Trauma.

16. 1. The laceration should be cleaned well to 19. 1. A potential for infection is an appropriate
­prevent infection. A sterile saline solution or nursing diagnosis, but there is no indication
water should be used. This is done after don- of infection from this question.
ning nonsterile gloves and applying pressure. 2. Bleeding results in an impairment of
2. The nurse should apply direct pressure to tissue perfusion. Because of the large
a deep laceration to stop the bleeding after amount of blood flow through the renal
donning nonsterile gloves. system, bleeding is a major problem.
3. The nurse must follow Standard Precau- 3. Skin integrity is not necessarily an issue in
tions in the school nurse setting by don- trauma. There is no indication from the
ning nonsterile gloves prior to caring for ­question the skin is not intact.
the client. 4. An alteration in temperature is not a problem
4. The school nurse must notify the parents but for this client unless infection occurs. This
not prior to taking care of the client. intervention is not indicated at this time.
Content – Medical: Nursing Process – Implementation: Content – Medical: Integrated Nursing Process –
Client Needs – Safe Effective Care Environment, ­Diagnosis: Client Needs – Safe Effective Care
­Management of Care: Cognitive Level – Application: Environment, Management of Care: Cognitive Level –
­Concept – Safety. Analysis: Concept – Trauma.

17. 1. Inserting an indwelling catheter may cause 20. 1. Any client who has not had a tetanus in-
further injury. Until the extent of injury is jection within five (5) years will need to
determined, prevention of further damage receive an injection as prophylaxis.
should have high priority. 2. The nurse may need to determine if the
2. Vital signs should be taken frequently to usual HCP can remove the sutures or the
assess for covert bleeding. The hematoma client should return to the ED for suture
in the flank area may indicate the pres- removal, but this is not the most important
ence of trauma to the kidney. Because of information.
the large amount of blood flow through 3. This information is important to teach the
the kidney, hemorrhage is a high risk. client, but preventing tetanus (lockjaw) is
3. Assessing skin turgor is important in deter- priority.
mining the fluid balance, but it is not higher 4. This client has been treated, so it is too late
priority than monitoring vital signs. to determine if the client has allergies.
4. The nurse could mark the bruised area to Content – Medical: Integrated Nursing Process –
better assess if the hematoma is enlarging, but ­Assessment: Client Needs – Physiological Integrity,
this is not the first intervention. Reduction of Risk Potential: Cognitive Level – Analysis:
Content – Medical: Integrated Nursing Process – Concept – Trauma.
­Implementation: Client Needs – Safe Effective Care 21. 1. The nurse should assess the client’s blood
­Environment, Management of Care: Cognitive
pressure but not prior to stopping the bleed-
Level – Analysis: Concept – Trauma.
ing. The most common cause of spontaneous
18. 1. Pain management is a goal for clients. At this epistaxis is hypertension.
time in the care of this client, it is not realistic 2. Most nosebleeds will stop after applying
to expect no pain. pressure on the nose between thumb and
2. Maintaining homeostasis is an appropriate index finger for 15 minutes.
outcome, but the priority is to maintain respi- 3. The nurse should position the client with
ratory status. Remember Maslow’s hierarchy the head tilted forward. This position will
of needs. prevent the client from swallowing the blood.
3. Symmetrical chest expansion indicates the The blood can be aspirated if the head is
client’s lungs have not collapsed and air is tilted back.
being exchanged. This is the client’s prior- 4. Most nosebleeds respond to pressure. If
ity outcome. ­pressure for 15 minutes does not stop the
4. A urine output of 30 mL/hr indicates the ­tissues bleeding, the health-care provider may need
are being adequately perfused and is an indica- to use electrocautery or silver nitrate. This is
tor of kidney functioning. Kidney function is performed by the HCP, not the nurse.
important but is not a priority over respiratory
status in a client with a gunshot wound.

15_Colgrove_Ch15.indd 613 02/06/16 12:39 PM


614 M ed -S urg S uccess

Content – Medical: Integrated Nursing Process – 25. 1. Assessing the site and rate is not the first
­Implementation: Client Needs – Safe Effective Care intervention.
Environment, Management of Care: Cognitive Level – 2. Sodium bicarbonate is not administered
Application: Concept – Trauma. ­unless indicated by arterial blood gases.
22. 1. Removing clothing causes further chilling. 3. The rhythm on the monitor should be
2. The warm air blanket blows warm air assessed. Many clients who become un-
over the client and is an active warming responsive have a lethal rhythm requiring
method. defibrillation immediately.
3. Hospital gowns have openings down the back 4. Cardioversion is not appropriate. Defibrilla-
and can increase chilling. tion may be needed.
4. Raising the temperature of the room will not Content – Medical: Integrated Nursing Process –
directly raise the client’s temperature. ­Implementation: Client Needs – Safe Effective Care
Content – Medical: Integrated Nursing Process – Environment, Management of Care: Cognitive Level –
­Implementation: Client Needs – Safe Effective Care Application: Concept – Assessment.
­Environment, Management of Care: Cognitive Level – 26. In order of priority: 4, 2, 3, 1, 5.
Analysis: Concept – Trauma. 4. This is done first before any action is
23. 1. Evaluation of airway and breathing is assess- taken to decrease suspicions on the part
ment and cannot be delegated. of the significant other. The nurse needs
2. Monitoring the rate of intravenous fluid is a to ask the client questions regarding the
part of administering a medication. Medica- injuries and may not get truthful answers
tion administration cannot be delegated. with the significant other in the room.
3. The UAP can attach leads to the client for 2. The nurse should assess the actual physi-
the cardiac monitor. cal problems before assessing the poten-
4. The nurse cannot delegate an unstable client tial abuse situation.
to the UAP. A client being transferred to the 3. This is one of the first questions the
intensive care unit is unstable. nurse should ask to determine if abuse is
Content – Medical: Integrated Nursing Process – occurring.
­Planning: Client Needs – Safe Effective Care Environment, 1. The nurse should determine if the ­client
Management of Care: Cognitive Level – Synthesis: has a plan to escape the violence. The
Concept – Trauma. nurse should provide the client with hot-
line numbers for safe houses.
24. 1. The client already has an intravenous access; 5. The nurse should refer the client to the
therefore, the nurse would not need to start social worker for further evaluation and
an intravenous line. referral needs.
2. A STAT chest x-ray will be done to evaluate
Content – Medical: Integrated Nursing Process –
the extent of the chest trauma, but it is not
­Implementation: Client Needs – Safe Effective Care
the first intervention. ­Environment, Management of Care: Cognitive Level –
3. The client will require a chest tube be- Analysis: Concept –Trauma.
cause the Heimlich valve is only tempo-
rary; therefore, the nurse should prepare
for this first.
4. Assessing the cardiac rhythm is important,
but the client is in distress and needs circula-
tory support, not further assessment.
Content – Medical: Integrated Nursing Process –
­Implementation: Client Needs – Safe Effective Care
­Environment, Management of Care: Cognitive Level –
Synthesis: Concept – Trauma.

15_Colgrove_Ch15.indd 614 02/06/16 12:39 PM


Perioperative Care
To repeat what others have said requires education; to challenge
it, requires brain.
16
—Mary Pettibone Poole

Surgery is a serious experience for clients. Before the surgery, in preoperative care, the nurse
must prepare clients for the specific surgery, telling them what to expect during the proce-
dure and afterward. During surgery, nurses help monitor the client and maintain the proper
condition of the operating room. The postoperative period is also important. The nurse
must assess the client frequently during the immediate postoperative period, ensuring that
airway, breathing, and circulation are maintained. The client also must be assessed for any
signs of complications, such as hemorrhage, evisceration, or infection. Because acute pain
is associated with most surgeries, the nurse also must assess the level and type of pain and
dispense pain-relieving medication as ordered by the health-care provider.

KEYWORDS ABBREVIATIONS
Anesthesia Association of Operating Room Nurses (AORN)
Anesthesiologist Blood pressure (BP)
Circulating nurse Centers for Disease Control and Prevention (CDC)
Evisceration Electrocardiogram (ECG)
Nurse anesthetist Health-care provider (HCP)
Intravenous (IV)
Medication administration record (MAR)
Nonsteroidal anti-inflammatory drug (NSAID)
Operating room (OR)
Partial thromboplastin time (PTT)
Patient-controlled analgesia (PCA)
Postanesthesia care unit (PACU)
Related to (R/T)
Unlicensed assistive personnel (UAP)

615

16_Colgrove_Ch16.indd 615 02/06/16 7:05 AM


616 M ed -S urg S uccess

PRACTICE QUESTIONS
Preoperative 5. The nurse is interviewing a surgical client in the
holding area. Which information should the nurse
1. The nurse requests the client to sign a surgical report to the anesthesiologist? Select all that
informed consent form for an emergency apply.
appendectomy. Which statement by the client 1. The client has loose, decayed teeth.
indicates further teaching is needed? 2. The client is experiencing anxiety.
1. “I will be glad when this is over so I can go 3. The client smokes two (2) packs of cigarettes a
home today.” day.
2. “I will not be able to eat or drink anything 4. The client has had a chest x-ray which does not
prior to my surgery.” show infiltrates.
3. “I can practice relaxing by listening to my 5. The client reports using herbs.
favorite music.” 6. Which task would be most appropriate for
4. “I will need to get up and walk as soon as the nurse to delegate to the unlicensed assistive
possible.” personnel (UAP)?
2. The nurse in the holding area of the surgery 1. Complete the preoperative checklist.
department is interviewing a client who requests 2. Assess the client’s preoperative vital signs.
to keep his religious medal on during surgery. 3. Teach the client about coughing and deep
Which intervention should the nurse implement? breathing.
1. Notify the surgeon about the client’s request to 4. Assist the client to remove clothing and
wear the medal. jewelry.
2. Tape the medal to the client and allow the 7. The nurse is assessing a client in the day surgery
client to wear the medal. unit who states, “I am really afraid of having
3. Request the family member take the medal this surgery. I’m afraid of what they will find.”
prior to surgery. Which statement would be the most therapeutic
4. Explain taking the medal to surgery is against response by the nurse?
the policy. 1. “Don’t worry about your surgery. It is safe.”
3. The nurse must obtain surgical consent forms for 2. “Tell me why you’re worried about your
the scheduled surgery. Which client would not be surgery.”
able to consent legally to surgery? 3. “Tell me about your fears of having this
1. The 65-year-old client who cannot read or surgery.”
write. 4. “I understand how you feel. Surgery is
2. The 30-year-old client who does not frightening.”
understand English. 8. The 68-year-old client scheduled for intestinal
3. The 16-year-old client who has a fractured surgery does not have clear fecal contents after
ankle. three (3) tap water enemas. Which intervention
4. The 80-year-old client who is not oriented to should the nurse implement first?
the day. 1. Notify the surgeon of the client’s status.
4. The nurse is preparing a client for surgery. Which 2. Continue giving enemas until clear.
intervention should the nurse implement first? 3. Increase the client’s IV fluid rate.
1. Check the permit for the spouse’s signature. 4. Obtain STAT serum electrolytes.
2. Take and document intake and output. 9. The nurse is caring for a male client scheduled for
3. Administer the “on call” sedative. abdominal surgery. Which interventions should
4. Complete the preoperative checklist. the nurse include in the plan of care? Select all
that apply.
1. Perform passive range-of-motion exercises.
2. Discuss how to cough and deep breathe
effectively.
3. Tell the client he can have a meal in the PACU.
4. Teach ways to manage postoperative pain.
5. Discuss events which occur in the
postanesthesia care unit.

16_Colgrove_Ch16.indd 616 02/06/16 7:05 AM


C hapter 16  P erioperative C are 617

10. The nurse is caring for a client scheduled for 14. The circulating nurse observes the surgical scrub
total hip replacement. Which behavior indicates technician remove a sponge from the edge of the
the need for further preoperative teaching? sterile field with a clamp and place the sponge
1. The client uses the diaphragm and abdominal and clamp in a designated area. Which action
muscles to inhale through the nose and exhale should the nurse implement?
through the mouth. 1. Place the sponge back where it was.
2. The client demonstrates dorsiflexion of the 2. Tell the technician not to waste supplies.
feet, flexing of the toes, and moves the feet in 3. Do nothing because this is the correct
a circular motion. procedure.
3. The client uses the incentive spirometer and 4. Take the sponge out of the room immediately.
inhales slowly and deeply so the piston rises to
15. The circulating nurse and the scrub technician
the preset volume.
find a discrepancy in the sponge count. Which
4. The client gets out of bed by lifting straight
action should the circulating nurse take first?
upright from the waist and then swings both
1. Notify the client’s surgeon.
legs along the side of the bed.
2. Complete an occurrence report.
11. The nurse is completing a preoperative 3. Contact the surgical manager.
assessment on a male client who states, “I am 4. Recount all sponges.
allergic to codeine.” Which intervention should
16. Which violation of surgical asepsis would
the nurse implement first?
require immediate intervention by the
1. Apply an allergy bracelet on the client’s wrist.
circulating nurse?
2. Label the client’s allergies on the front of the
1. Surgical supplies were cleaned and sterilized
chart.
prior to the case.
3. Ask the client what happens when he takes the
2. The circulating nurse is wearing a long-
codeine.
sleeved sterile gown.
4. Document the allergy on the medication
3. Masks covering the mouth and nose are being
administration record.
worn by the surgical team.
12. Which laboratory result would require 4. The scrub nurse setting up the sterile field is
immediate intervention by the nurse for the wearing artificial nails.
client scheduled for surgery?
17. The nurse identifies the nursing diagnosis “risk
1. Calcium 9.2 mg/dL.
for injury related to positioning” for the client in
2. Bleeding time two (2) minutes.
the operating room. Which nursing intervention
3. Hemoglobin 15 g/dL.
should the nurse implement?
4. Potassium 2.4 mEq/L.
1. Avoid using the cautery unit which does not
have a biomedical tag on it.
Intraoperative 2. Carefully pad the client’s elbows before
covering the client with a blanket.
13. Which activities are the circulating nurse’s 3. Apply a warming pad on the OR table before
responsibilities in the operating room? placing the client on the table.
1. Monitor the position of the client, prepare the 4. Check the chart for any prescription or over-
surgical site, and ensure the client’s safety. the-counter medication use.
2. Give preoperative medication in the holding 18. The circulating nurse is positioning clients for
area and monitor the client’s response to surgery. Which client has the greatest potential
anesthesia. for nerve damage?
3. Prepare sutures; set up the sterile field; and 1. The 16-year-old client in the dorsal
count all needles, sponges, and instruments. recumbent position having an appendectomy.
4. Prepare the medications to be administered 2. The 68-year-old client in the Trendelenburg
by the anesthesiologist and change the tubing position having a cholecystectomy.
for the anesthesia machine. 3. The 45-year-old client in the reverse
Trendelenburg position having a biopsy.
4. The 22-year-old client in the lateral position
having a nephrectomy.

16_Colgrove_Ch16.indd 617 02/06/16 7:05 AM


618 M ed -S urg S uccess

19. Which situation demonstrates the circulating Postoperative


nurse acting as the client’s advocate?
1. Plays the client’s favorite audio book during 25. The PACU nurse is receiving the client from
surgery. the OR. Which intervention should the nurse
2. Keeps the family informed of the findings of implement first?
the surgery. 1. Assess the client’s breath sounds.
3. Keeps the operating room door closed at all 2. Apply oxygen via nasal cannula.
times. 3. Take the client’s blood pressure.
4. Calls the client by the first name when the 4. Monitor the pulse oximeter reading.
client is recovering.
26. Which assessment data indicate the
20. The circulating nurse is planning the care for postoperative client who had spinal anesthesia is
an intraoperative client. Which statement is the suffering a complication of the anesthesia?
expected outcome? 1. Loss of sensation at the lumbar (L5)
1. The client has no injuries from the OR dermatome.
equipment. 2. Absence of the client’s posterior tibial pulse.
2. The client has no postoperative infection. 3. The client has a respiratory rate of eight (8).
3. The client has stable vital signs during 4. The blood pressure is within 20% of the
surgery. client’s baseline.
4. The client recovers from anesthesia.
27. The surgical client’s vital signs are T 98˚F, P 106,
21. Which nursing intervention has the highest R 24, and BP 88/40. The client is awake and
priority when preparing the client for a surgical oriented times three (3) and the skin is pale and
procedure? damp. Which intervention should the nurse
1. Pad the client’s elbows and knees. implement first?
2. Apply soft restraint straps to the extremities. 1. Call the surgeon and report the vital signs.
3. Prepare the client’s incision site. 2. Start an IV of D5RL with 20 mEq KCl at
4. Document the temperature of the room. 125 mL/hr.
22. The nursing manager is making assignments for 3. Elevate the feet and lower the head.
the OR. Which case should the manager assign 4. Monitor the vital signs every 15 minutes.
to the inexperienced nurse? 28. The PACU nurse administers Narcan, an opioid
1. The client having open-heart surgery. antagonist, to a postoperative client. Which
2. The client having a biopsy of the breast. client problem should the nurse include to the
3. The client having laser eye surgery. plan of care based on this medication?
4. The client having a laparoscopic knee repair. 1. Alteration in comfort.
23. The circulating nurse assesses tachycardia and 2. Risk for depressed respiratory pattern.
hypotension in the client. Which interventions 3. Potential for infection.
should the nurse implement? 4. Fluid and electrolyte imbalance.
1. Prepare ice packs and mix dantrolene sodium. 29. The 26-year-old male client in the PACU has
2. Request the defibrillator be brought into a heart rate of 110 and a rising temperature
the OR. and complains of muscle stiffness. Which
3. Draw a PTT and prepare a heparin drip. interventions should the nurse implement?
4. Obtain finger stick blood glucose Select all that apply.
immediately. 1. Give a back rub to the client to relieve
24. The nurse is planning the care of the surgical stiffness.
client having procedural sedation. Which 2. Apply ice packs to the axillary and groin areas.
intervention has highest priority? 3. Prepare an ice slush for the client to drink.
1. Assess the client’s respiratory status. 4. Prepare to administer dantrolene, a smooth-
2. Monitor the client’s urinary output. muscle relaxant.
3. Take a 12-lead ECG prior to injection. 5. Reposition the client on a warming blanket.
4. Attempt to keep the client focused.

16_Colgrove_Ch16.indd 618 02/06/16 7:05 AM


C hapter 16  P erioperative C are 619

30. Which data indicate to the nurse the client 36. The unlicensed assistive personnel (UAP)
who is one (1) day postoperative right total hip reports the vital signs for a first-day
replacement is progressing as expected? postoperative client as T 100.8˚F, P 80, R 24, and
1. Urine output was 160 mL in the past eight (8) BP 148/80. Which intervention would be most
hours. appropriate for the nurse to implement?
2. Paralysis and paresthesia of the right leg. 1. Administer the antibiotic earlier than
3. T 99.0˚F, P 98, R 20, and BP 100/60. scheduled.
4. Lungs are clear bilaterally in all lobes. 2. Change the dressing over the wound.
3. Have the client turn, cough, and deep breathe
31. The nurse and the unlicensed assistive personnel
every two (2) hours.
(UAP) are working on the surgical unit. Which
4. Encourage the client to ambulate in the hall.
task can the nurse delegate to the UAP?
1. Take routine vital signs on clients.
2. Check the Jackson Pratt insertion site. Acute Pain
3. Hang the client’s next IV bag.
4. Ensure the client obtains pain relief. 37. The client is complaining of left shoulder pain.
32. The charge nurse is making shift assignments. Which intervention should the nurse implement
Which postoperative client should be assigned first?
to the most experienced nurse? 1. Assess the neurovascular status of the left
1. The 4-year-old client who had a tonsillectomy hand.
and is able to swallow fluids. 2. Check the medication administration record
2. The 74-year-old client with a repair of the left (MAR).
hip who is unable to ambulate. 3. Ask if the client wants pain medication.
3. The 24-year-old client who had an 4. Administer the client’s pain medication.
uncomplicated appendectomy the previous 38. The nurse is caring for a client in acute pain as a
day. result of surgery. Which intervention should the
4. The 80-year-old client with small bowel nurse implement?
obstruction and congestive heart failure. 1. Administer pain medication as soon as the
33. Which statement would be an expected outcome time frame allows.
for the postoperative client who had general 2. Use nonpharmacological methods to replace
anesthesia? medications.
1. The client will be able to sit in the chair for 3. Use cryotherapy after heat therapy because it
30 minutes. works faster.
2. The client will have a pulse oximetry reading 4. Instruct family members to administer
of 97% on room air. medication with the PCA.
3. The client will have a urine output of 30 mL 39. Which situation is an example of the nurse
per hour. fulfilling the role of client advocate?
4. The client will be able to distinguish sharp 1. The nurse brings the client pain medication
from dull sensations. when it is due.
34. The postoperative client is transferred from the 2. The nurse collaborates with other disciplines
PACU to the surgical floor. Which action should during the care conference.
the nurse implement first? 3. The nurse contacts the health-care provider
1. Apply antiembolism hose to the client. when pain relief is not obtained.
2. Attach the drain to 20 cm suction. 4. The nurse teaches the client to ask for
3. Assess the client’s vital signs. medication before the pain gets to a “5.”
4. Listen to the report from the anesthesiologist. 40. Which statement should the nurse identify as
35. Which problem should the nurse identify the expected outcome for a client experiencing
as priority for client who is one (1) day acute pain?
postoperative? 1. The client will have decreased use of
1. Potential for hemorrhaging. medication.
2. Potential for injury. 2. The client will participate in self-care
3. Potential for fluid volume excess. activities.
4. Potential for infection. 3. The client will use relaxation techniques.
4. The client will repeat instructions about
medications.

16_Colgrove_Ch16.indd 619 02/06/16 7:05 AM


620 M ed -S urg S uccess

41. Which nursing intervention is the highest 45. Which nursing intervention is priority for the
priority when administering pain medication to client experiencing acute pain?
a client experiencing acute pain? 1. Assess the client’s verbal and nonverbal
1. Monitor the client’s vital signs. behavior.
2. Verify the time of the last dose. 2. Wait for the client to request pain medication.
3. Check for the client’s allergies. 3. Administer the pain medication on a
4. Discuss the pain with the client. scheduled basis.
4. Teach the client to use only imagery every
42. Which intervention is appropriate for the nurse
hour for the pain.
to delegate to the unlicensed assistive personnel
(UAP) when caring for the female client 46. The nurse is conducting an interview with a
experiencing acute pain? 75-year-old client admitted with acute pain.
1. Take the pain medication to the room. Which question would have priority when
2. Apply an ice pack to the site of pain. assisting with pain management?
3. Check on the client 30 minutes after she takes 1. “Have you ever had difficulty getting your
the pain medication. pain controlled?”
4. Observe the client’s ability to use the PCA. 2. “What types of surgery have you had in the
last 10 years?”
43. The nurse is administering an opioid narcotic to
3. “Have you ever been addicted to narcotics?”
the client. Which interventions should the nurse
4. “Do you have a list of your prescription
implement for client safety? Select all that
medications?”
apply.
1. Compare the hospital number on the MAR to 47. The nurse clears the PCA pump and discovers
the client’s bracelet. the client has used only a small amount of
2. Have a witness verify the wasted portion of medication during the shift. Which intervention
the narcotic. should the nurse implement?
3. Assess the client’s vital signs prior to 1. Determine why the client is not using the
administration. PCA pump.
4. Determine if the client has any allergies to 2. Document the amount and take no action.
medications. 3. Chart the client is not having pain.
5. Clarify all pain medication orders with the 4. Contact the HCP and request oral
health-care provider. medication.
44. Which technique would be most appropriate 48. Which problem would be most appropriate for
for the nurse to implement when assessing a four the nurse to identify for the client experiencing
(4)-year-old client in acute pain? acute pain?
1. Use words a four (4)-year-old child can 1. Ineffective coping.
remember. 2. Potential for injury.
2. Explain the 0-to-10 pain scale to the child’s 3. Alteration in comfort.
parent. 4. Altered sensory input.
3. Have the child point to the face which
describes the pain.
4. Administer the medication every four (4)
hours.

CONCEPTS
The concepts covered in this chapter focus on comfort. Exemplars that are covered are acute pain, infection, and
perioperative care. Interrelated concepts of the nursing process and critical thinking are covered throughout the
questions. The concept of critical thinking is presented in the prioritizing or “first” questions.

16_Colgrove_Ch16.indd 620 02/06/16 7:05 AM


C hapter 16  P erioperative C are 621

49. The nurse is preparing a client for surgery who will be receiving general anesthesia. Which medication
should the nurse question administering?
1. Metoprolol PO.
2. Cefazolin sodium IVPB.
3. EMLA cream topical.
4. Dabigatran etexilate.

Admitting Diagnosis:
Client Name: Operative Client Right Inguinal Hernia

Account Number: 234 567 890 Med Rec #: 12 122 12 Allergies: Penicillin

Date Medication 2301–0700 0701–1500 1501–2300


Today Metoprolol (Lopressor) 50 mg PO daily 0700
Today Cefazolin sodium (Ancef) 1,000 mg IVPB
on call to OR
Today EMLA cream topical prior to IV start 0700
Today Dabigatran etexilate (Pradaxa) 100 mg 0900
PO daily
Signature of Nurse: Day Nurse (RN) DN

50. The client has undergone an abdominal perineal resection of the colon for colon cancer with a left lower
quadrant colostomy. Which interventions should the nurse implement? Select all that apply.
1. Assess the stoma for color every four (4) hours and prn.
2. Encourage the client to turn, cough, and deep breathe every two (2) hours.
3. Maintain the head of the bed 30 to 40 degrees elevated at all times.
4. Auscultate for bowel sounds every four (4) hours.
5. Administer pain medications sparingly to prevent addiction.
51. The nurse is reviewing the pathology report of a client post–cervical neck node dissection. The health-care
provider has explained the results of the biopsy to the client. Which should the nurse implement?
1. Allow the client the opportunity to discuss feelings about the results.
2. Assess the client’s neck dissection dressing for bleeding.
3. Monitor the client’s white blood cell count for elevation.
4. Call the pathology department to verify the report is correct.

Specimen/Site Pathological Finding


Excisional biopsy of cervical Lymph Node #1
neck lymph nodes 9 cm times 5 cm; multinodularity noted; positive for Reed-Sternberg cells
Lymph Node #2
4 cm times 4 cm
Consistent with finding in Node #1
Lymph Node #3
3 cm times 2 cm
Consistent with finding in Node #1
Lymph Node #4
Findings consistent with Hodgkin’s lymphoma Stage 3

16_Colgrove_Ch16.indd 621 02/06/16 7:05 AM


622 M ed -S urg S uccess

52. The client returned to the medical surgical unit at 1800 following a two (2)-hour surgery and one (1)
hour in the postanesthesia recovery unit (PACU). The nurse is reviewing the client’s intake and output at
midnight. Which intervention should the nurse implement based on the recorded data?
1. Immediately place an indwelling catheter in the client.
2. Assess the client’s skin turgor in the abdominal area.
3. Recheck the client’s urinary output in two (2) hours.
4. Encourage the client to drink 500 mL of clear liquids.

Oral Intravenous Urine Nasogastric Tube Other (Specify)


Day One (in mL) (in mL) (in mL) (in mL) (in mL)
0701–1500 0 In OR
1501–2300 250 750 120 120
2301–0700
Total

53. The nurse is completing the preoperative checklist for the client who is scheduled for a laparoscopic
cholecystectomy. The preoperative complete blood count (CBC) results are on the chart. Which action
should the nurse implement first?
1. Check off that the CBC report is on the chart.
2. Notify the surgeon of the WBC.
3. Assess the client for dyspnea.
4. Teach the client to turn, cough, and deep breathe.

Laboratory Test Client Values Normal Values


White blood cell count (WBC) 12.3 4.5 to 11 × 103 cells/mm3
Red blood cell count (RBC) 6.8 Male: 4.7 to 5.1 × 106 cells/mm3
Female: 4.2 to 4.8 × 106 cells/mm3
Hemoglobin (Hgb) 14.4 Male: 13.2 to 17.3 g/dL
Female: 11.7 to 15.5 g/dL
Hematocrit (Hct) 41.8 Male: 43% to 49%
Female: 38% to 44%
Platelets 168 150 to 450 × 103/mm3

54. The nurse is receiving a client from the 55. The three (3)-day postoperative client is
postanesthesia care unit (PACU). Which complaining of unrelieved pain at the incision
interventions should the nurse implement? site one (1) hour after the administration of
Select all that apply. narcotic pain medication. Which action should
1. Ambulate the client to the bathroom to void. the nurse implement first?
2. Take the client’s vital signs to compare with 1. Check the MAR for another medication to
PACU data. administer.
3. Monitor all lines into and out of the client’s 2. Teach the client to use guided imagery to
body. relieve the pain.
4. Assess the client’s surgical site. 3. Assess the client for complications.
5. Push the client’s PCA button to treat for pain 4. Elevate the head of the client’s bed.
during movement.

16_Colgrove_Ch16.indd 622 02/06/16 7:05 AM


PRACTICE QUESTIONS ANSWERS
AND RATIONALES
Preoperative 3. 1. The 65-year-old client who cannot read can
mark an “X” on the form and is legally able
1. 1. The client will be in the hospital for a few to sign a surgical permit as long as the client
days. This is not a day-surgery procedure. understands the benefits, alternatives, and all
The client needs more teaching. potential complications of the surgery.
2. Clients are NPO (nothing by mouth) prior 2. The client who does not speak English can and
to surgery to prevent aspiration during and should have information given and questions
after anesthesia. The client understands the answered in the client’s native language.
teaching. 3. A 16-year-old client is not legally able to
3. Listening to music and other relaxing tech- give permission for surgery unless the
niques can be used to alleviate anxiety and adolescent has been given an emancipated
pain. This statement indicates the client status by a judge. This information was not
­understands the teaching. given in the stem.
4. Clients are encouraged to get out of bed as 4. A client is able to give permission unless deter-
soon as possible and progress until a return mined incompetent. Not knowing the day of
to daily activity is achieved. The client under- the week is not significant.
stands the teaching. TEST-TAKING HINT: Age in a stem or option
TEST-TAKING HINT: This question is ­asking the gives the test taker a clue as to the correct
test taker to identify the answer option which ­answer. The nurse must be aware of legal
is incorrect. Three (3) options will be appro- ­issues when caring for the client.
priate statements which indicate the client Content – Surgical: Integrated Nursing Process –
understands the teaching. ­Assessment: Client Needs – Safe Effective Care Environment,
Content – Surgical: Integrated Nursing Process – Management of Care: Cognitive Level – Analysis:
Evaluation: Client Needs – Physiological Integrity, ­Reduction Concept – Perioperative.
of Risk Potential: Cognitive Level – Synthesis: Concept – 4. 1. The client’s signature, not the spouse’s, should
Perioperative. be on the surgical permit.
2. 1. The surgeon does not need to be notified of 2. This would be important information if abnor-
the client’s request; this can be addressed by mal, but it is not the first intervention.
the nursing staff. 3. “On call” sedatives should be administered
2. The medal should be taped and the client ­after the surgical checklist is completed.
should be allowed to wear the medal be- 4. Completing the preoperative checklist has
cause meeting spiritual needs is essential to the highest priority to ensure all details are
this client’s care. completed without omissions.
3. The client should be allowed to bring the TEST-TAKING HINT: A client should never be
medal to surgery if the medal is taped to the sedated until the permit has been verified
client. and all legal issues are settled. The test taker
4. Hospital policies should be established for should not read into a question by inserting
the well-being of clients, and spiritual needs facts not in the stem. For example, the test
should be addressed. taker may think option “1” could be a correct
TEST-TAKING HINT: Because options “3” and answer if the client is confused, but the stem
“4” do not allow the client to wear the medal does not include this information.
to surgery, these can be eliminated as possible Content – Surgical: Integrated Nursing Process –
answers because they are both saying the same Implementation: Client Needs – Safe Effective Care
thing. ­Environment, Management of Care: Cognitive Level –
­Application: Concept – Perioperative.
Content – Surgical: Integrated Nursing Process –
­Implementation: Client Needs – Safe Effective Care
Environment, Management of Care: Cognitive Level –
Application: Concept – Perioperative.

623

16_Colgrove_Ch16.indd 623 02/06/16 7:05 AM


624 M ed -S urg S uccess

5. 1. Loose teeth or caries need to be reported to TEST-TAKING HINT: There are rules the test
the anesthesiologist so he or she can make taker should implement when answering these
­provisions to prevent breaking the teeth and types of questions. The test taker should not
causing the client to possibly aspirate pieces. select an option which asks the client “why,”
2. The nurse should report any client who is such as option “2,” or an option which states,
­extremely anxious, but the nurse can address “I understand,” such as option “4.”
the needs of a client experiencing expected Content – Surgical: Integrated Nursing Process –
surgical anxiety. Implementation: Client Needs – Psychosocial Integrity:
3. Smokers are at a higher risk for complica- ­Cognitive Level – Application: Concept – Perioperative.
tions from anesthesia.
8. 1. The nurse should contact the surgeon
4. No infiltrates on a chest x-ray is a normal
­because the client is at risk for fluid and
­finding and does not have to be reported.
electrolyte imbalance after three (3)
5. Herbs—for example, St. John’s wort, lico-
­enemas. Clients who are NPO, elderly
rice, and ginkgo—have serious interactions
­clients, and pediatric clients are more likely
with anesthesia and with bodily functions
to have these imbalances.
such as coagulation.
2. Administering more enemas will put the client
TEST-TAKING HINT: This question is an at further risk for fluid volume deficit and elec-
­alternate-type question requiring the test trolyte imbalance.
taker to select more than one (1) option as the 3. The IV may need to be increased, but the
correct answer. Safety is priority for a client nurse would need to fully assess the fluid status
undergoing surgery. of the client prior to initiating this interven-
Content – Surgical: Integrated Nursing Process – tion. Intravenous fluids would not directly af-
Implementation: Client Needs – Safe Effective Care fect the peristalsis of the gastrointestinal tract
­Environment, Management of Care: Cognitive Level – so that the bowels would move more rapidly.
­Application: Concept – Perioperative. 4. The electrolyte status may need to be assessed,
6. 1. The nurse should complete this form because but the nurse would need an order for this
it requires analysis, which cannot be delegated intervention.
to the UAP. TEST-TAKING HINT: Very few questions will
2. Nurses cannot delegate assessment. require the nurse to notify the health-care
3. The nurse cannot delegate teaching to a UAP. provider, but there will be some; the test taker
4. The UAP can remove clothing and jewelry. must know when a potential complication may
TEST-TAKING HINT: The nurse should ­consider occur. The nurse cannot prescribe or order
the knowledge and training of the person laboratory tests without a health-care pro-
­receiving the assignments. The nurse should vider’s order.
never delegate assessment, teaching, admin- Content – Surgical: Integrated Nursing Process –
istering medications, evaluation, or care of an Implementation: Client Needs – Safe Effective Care
unstable client to a UAP. ­Environment, Management of Care: Cognitive Level –
­Application: Concept – Perioperative.
Content – Surgical: Integrated Nursing Process –
­Planning: Client Needs – Safe Effective Care Environment, 9. 1. Passive means the nurse performs the range-
Management of Care: Cognitive Level – Synthesis: of-motion exercises. The client in the PACU
Concept – Nursing Roles. should do active range-of-motion exercises.
7. 1. This statement is giving false reassurance. 2. Coughing effectively aids in the removal of
2. “Why” is never therapeutic. The client does pooled secretions, which can cause pneu-
not owe the nurse an explanation. monia. Deep-breathing exercises keep the
3. This statement focuses on the emotion alveoli inflated and prevent atelectasis.
which that the client identified and is 3. The client having abdominal surgery will be
therapeutic. NPO until bowel sounds return, which will
4. This statement belittles the client’s fear, and no not occur in the PACU; therefore, the client is
person understands how another person feels. not given a meal.
4. The client’s postoperative pain should be
kept within a tolerable range.
5. These interventions help decrease the
­client’s anxiety.

16_Colgrove_Ch16.indd 624 02/06/16 7:05 AM


C hapter 16  P erioperative C are 625

TEST-TAKING HINT: This is an alternate-type Content – Surgical: Integrated Nursing Process –


question, which requires the test taker to ­Implementation: Client Needs – Safe Effective Care
select more than one (1) option as the cor- Environment, Management of Care: Cognitive Level –
rect answer. The nurse’s priority after surgery ­Application: Concept – Perioperative.
is to prevent postoperative complications. 12. 1. This laboratory value is within normal limits.
Content – Surgical: Integrated Nursing Process – 2. This laboratory value is within normal limits.
­Diagnosis: Client Needs – Safe Effective Care Environ- 3. This laboratory value is within normal limits.
ment, Management of Care: Cognitive Level – Analysis: 4. This potassium level is low and should be
Concept – Perioperative. reported to the health-care provider be-
10. 1. This is the correct way to perform deep- cause potassium is important for muscle
breathing exercises; therefore, no further function, including the cardiac muscle.
teaching is needed. TEST-TAKING HINT: There are some items,
2. This is the correct way to perform range- such as normal laboratory values, the test
of-motion exercises; therefore, no further taker must memorize. Laboratory values may
­teaching is needed. differ slightly between laboratories, but the
3. This is the way to use a volume incentive test taker must know them.
­spirometer; therefore, no further teaching Content – Surgical: Integrated Nursing Process – ­
is needed. Assessment: Client Needs – Safe Effective Care
4. The correct way to get out of bed postop- ­Environment, Management of Care: Cognitive Level –
eratively is to roll onto the side, grasp the Analysis: Concept – Perioperative.
side rail to maneuver to the side, and then
push up with one hand while swinging the
legs over the side. The client needs fur- Intraoperative
ther teaching.
13. 1. The circulating nurse has many responsi-
TEST-TAKING HINT: This is an “except” ques-
bilities in the OR, including coordinating
tion. Therefore, the test taker must select
the activities in the OR; keeping the OR
an option which shows the client does not
clean; ensuring the safety of the client;
understand the teaching. Sometimes flipping
and maintaining the humidity, lighting,
the question and asking which behavior indi-
and safety of the equipment.
cates the client understands the teaching will
2. This is the role of the nurse anesthetist or
help in answering this type of question.
anesthesiologist.
Content – Surgical: Integrated Nursing Process – ­ 3. This is the role of the scrub nurse or
Evaluation: Client Needs – Safe Effective Care technologist.
­Environment, Management of Care: Cognitive Level –
4. If there is an anesthesia technologist, this
Synthesis: Concept – Perioperative.
would be his or her role, or the nurse anes-
11. 1. This is an important step for the nurse to thetist and the anesthesiologist would assume
implement, but it is not the first intervention. the role.
2. This must be done, but it is not the first TEST-TAKING HINT: Options “2” and “4”
intervention. discuss anesthesia and an anesthesiologist,
3. The nurse should first assess the events which may lead the test taker to eliminate
which occurred when the client took this these as possible correct answers. Some
medication because many clients think a questions are knowledge-based questions
side effect, such as nausea, is an allergic which require the test taker to know the
reaction. ­information; this is an example of this type
4. This information must be put on the medica- of question.
tion administration record (MAR), but it is
Content – Surgical: Integrated Nursing Process –
not the first intervention. Implementation: Client Needs – Safe Effective Care
TEST-TAKING HINT: The stem is asking the ­Environment, Management of Care: Cognitive Level –
test taker to identify the first intervention. ­Application: ­Concept – Nursing Roles.
Therefore, all four (4) options could be
­interventions which should be implemented,
but assessment is the first part of the nursing
process, so option “3” is the correct answer.

16_Colgrove_Ch16.indd 625 02/06/16 7:05 AM


626 M ed -S urg S uccess

14. 1. Items which are on the edge of the sterile 16. 1. These are appropriate activities in a surgery
field are considered contaminated and should department; therefore, no intervention is
be removed from the field. required.
2. The technician is not wasting supplies; the 2. This is required to maintain surgical asepsis.
technician is following principles of asepsis. 3. This follows the principles of surgical asepsis.
3. The technician followed the correct pro- 4. According to the Centers for Disease
cedure. Sponges are counted to maintain Control and Prevention (CDC), the
client safety, so all sponges must be kept ­Association of Operating Room Nurses
together to repeat the count before the (AORN), and the Association for Practi-
incision site is sutured. The sponge must tioners in Infection Control, artificial nails
be removed, not used, and placed in a harbor microorganisms, which increase
­designated area to be counted later. the risk for infection.
4. Taking the contaminated sponge out of TEST-TAKING HINT: The adjective “artificial”
the room would cause a discrepancy in the in option “4” and the word “violation” in the
sponge count. stem should cause the test taker to select
TEST-TAKING HINT: When answering this option “4” as the correct answer if the test
question, the test taker must consider safety, taker had no idea which distracter to select.
which is always of the utmost importance Content – Surgical: Integrated Nursing Process – ­
during surgery. Sponge count is a basic Assessment: Client Needs – Safe Effective Care
­concept in operating room nursing theory. ­Environment, Safety and Infection Control: Cognitive
Content – Surgical: Integrated Nursing Process – Level – Synthesis: Concept – Safety.
Implementation: Client Needs – Safe Effective Care 17. 1. This would prevent an electrical injury, but
­Environment, Management of Care: Cognitive Level –
the interventions must address positioning,
­Application: Concept – Perioperative.
which is the etiology of the nursing diagnosis.
15. 1. When discrepancies occur in the count, it is 2. Padding the elbows decreases pressure
usually a simple mistake discovered with a so nerve damage and pressure ulcers are
recount. The surgeon will be notified if the ­prevented. This addresses the etiology of
count is wrong after a recount. the nursing diagnosis.
2. If an error is found to have been made, an 3. This would help to decrease hypothermia,
occurrence report will be completed, but it is but it does not address the etiology of the
not the first intervention. nursing diagnosis.
3. This would be done if a correct count is not 4. Checking the chart for medication use would
maintained, but it is not the first intervention. help prevent interactions between anesthesia
4. A recount of sponges may lead to the dis- and routine medications, but it does not
covery of the cause of the presumed error. address the etiology of the nursing diagnosis.
Usually it is just a miscount or a result of TEST-TAKING HINT: The test taker must be
a sponge being placed in a location other knowledgeable of nursing diagnosis and
than the sterile field, such as the floor or a the nursing process. The assessment data
lower shelf. support the response “risk for injury” and
TEST-TAKING HINT: When the test taker has the interventions address the etiology
no idea of the correct answer, the test taker “positioning.”
should apply the nursing process and choose Content – Surgical: Integrated Nursing Process –
the option which addresses assessment Implementation: Client Needs – Physiological Integrity,
because it is the first step in the nursing Basic Care and Comfort: Cognitive Level – Application:
process. Concept – Perioperative.
Content – Surgical: Integrated Nursing Process – 18. 1. The young client is not a high risk for nerve
Implementation: Client Needs – Safe Effective Care damage secondary to positioning.
­Environment, Management of Care: Cognitive Level –
2. The client’s age, along with positioning
Application: Concept – Perioperative.
with increased weight and pressure on the
shoulders, puts this client at higher risk.
3. This client is sitting in an upright position,
which would not put this client at risk for
nerve damage.
4. A younger client would not be at a high risk
for nerve damage when lying on the side.

16_Colgrove_Ch16.indd 626 02/06/16 7:05 AM


C hapter 16  P erioperative C are 627

TEST-TAKING HINT: Clients who are at high- 21. 1. This intervention prevents nerve damage
est risk for nerve damage are clients who from positioning, but it is not a higher prior-
are e
­ lderly, obese, or emaciated and clients ity than preventing the client from falling off
placed in positions which increase pressure the OR table.
on bony prominences. 2. This action would prevent the client from
Content – Surgical: Integrated Nursing Process – falling off the table, which is the highest
­Assessment: Client Needs – Physiological Integrity, priority.
­Reduction of Risk Potential: Cognitive Level – Analysis: 3. Preparing the incision site is not a higher pri-
Concept – Perioperative. ority than preventing the client from falling
19. 1. The client is not awake during surgery, so off the OR table.
playing a favorite audio book would not be an 4. The temperature of the room does not have a
example of client advocacy. higher priority than safety.
2. This would be a nice action to take, but it TEST-TAKING HINT: Client safety should
is not an example of client advocacy. ­always be a high priority. In order, the priori-
3. This would keep the client’s dignity by ties would be preventing a fall, preventing
maintaining privacy. With this action, the nerve damage, preventing infection, and then
nurse is speaking for the client while the documenting.
client cannot speak as a result of anesthe- Content – Surgical: Integrated Nursing Process –
sia; this is an example of client advocacy. Implementation: Client Needs – Safe Effective Care
4. Clients should be referred to by their last ­Environment, Management of Care: Cognitive Level –
name, rather than first, unless the client re- Analysis: Concept – Perioperative.
quests the staff to use his or her first name. 22. 1. Open-heart surgery is complex, and the care
This is not an example of client advocacy. of the client should be assigned to an experi-
TEST-TAKING HINT: The definition of a client enced nurse with special training.
advocate is a person designated to speak up 2. The case of a client having a biopsy of the
for the client’s rights when the client cannot. breast would be a good case for an inexpe-
Content – Surgical: Integrated Nursing Process – rienced nurse because it is simple.
­Implementation: Client Needs – Safe Effective Care 3. Laser eye surgery requires the nurse in the
Environment, Management of Care: Cognitive Level – OR to have additional training to operate the
­Application: Concept – Perioperative. equipment.
20. 1. This expected outcome addresses the 4. Additional training to be in the OR would
safety of the client while in the OR. be required for this case because special care
2. This would be an expected outcome in the to prevent infection is needed in orthopedic
postoperative period. cases.
3. The anesthesiologist or nurse anesthetist TEST-TAKING HINT: The test taker should
would monitor the client’s vital signs during select the option which requires the least
surgery. amount of additional training because the
4. This would be an expected outcome for the nurse is new to the operating room. Technol-
anesthesiologist or nurse anesthetist. ogy required for specific surgeries requires
TEST-TAKING HINT: The adjectives “intraop-
additional training.
erative” and “circulating” are the key words Content – Surgical: Integrated Nursing Process – ­
to answering this question. Safety is priority Planning: Client Needs – Safe Effective Care
­Environment, Management of Care: Cognitive Level –
in the operating room.
Synthesis: ­Concept –Management.
Content – Surgical: Integrated Nursing Process –
­Diagnosis: Client Needs – Safe Effective Care Environment, 23. 1. Unexplained tachycardia, hypotension, and
Management of Care: Cognitive Level – Analysis: elevated temperature are signs of malig-
Concept – Perioperative. nant hyperthermia, which is treated with
ice packs and dantrolene sodium.
2. A defibrillator would be needed if the client
were in ventricular tachycardia or ventricular
fibrillation.
3. These interventions would not be appropriate
for malignant hyperthermia.
4. This would be important if the client had
diabetes, but it does not address malignant
hyperthermia.

16_Colgrove_Ch16.indd 627 02/06/16 7:05 AM


628 M ed -S urg S uccess

TEST-TAKING HINT: The test taker could 26. 1. Loss of sensation in the L5 dermatome is
eliminate options based on basic concepts. ­expected from spinal anesthesia.
Option “2” could be eliminated because the 2. Absence of a posterior tibial pulse is indica-
client is not coding. Option “3” would in- tive of a block in the blood supply, but it is
crease bleeding, which would not be appro- not a complication of spinal anesthesia.
priate for a client having surgery. 3. If the effects of the spinal anesthesia
Content – Surgical: Integrated Nursing Process – move up rather than down the spinal cord,
Implementation: Client Needs – Safe Effective Care respirations can be depressed and even
­Environment, Management of Care: Cognitive Level – blocked.
­Application: Concept – Perioperative. 4. This is an expected outcome and does not
24. 1. Assessing the respiratory rate, rhythm, ­indicate a complication.
and depth is the most important action. TEST-TAKING HINT: The test taker must know
2. The nurse needs to monitor all systems, but normal rates for vital signs, and a respira-
monitoring the urine output would not be tory rate of eight (8) would be significantly
priority over monitoring breathing. low for any client and indicate a possible
3. Monitoring the client’s ECG is appropriate, complication.
but it is not priority. Content – Surgical: Integrated Nursing Process –
4. The client needs to be relaxed, not focused, ­Assessment: Client Needs – Physiological Integrity,
but this is not priority over respiratory status. ­Reduction of Risk Potential: Cognitive Level – Analysis:
Concept – Perioperative.
TEST-TAKING HINT: When the test taker must
prioritize nursing care, assessment is usu- 27. 1. The surgeon should be notified, but this is not
ally first. Using Maslow’s hierarchy of needs the first action; the client must be cared for.
to prioritize, assessment of respiration is 2. The postoperative client had lactated Ringer’s
always first. All sedation agents can depress infused during surgery. The rate should be in-
respirations. creased during hemorrhage—which the ­vital
Content – Surgical: Integrated Nursing Process – signs indicate is occurring—but potassium
Implementation: Client Needs – Safe Effective Care should not be added.
­Environment, Management of Care: Cognitive Level – 3. By lowering the head of the bed and rais-
­Application: Concept – Perioperative. ing the feet, the blood is shunted to the
brain until volume-expanding fluids can
be administered, which is the first inter-
Postoperative vention for a client who is hemorrhaging.
4. When signs and symptoms of shock are ob-
25. 1. The airway should be assessed first. When served, the nurse will monitor the vital signs
caring for a client, the nurse should fol- more frequently than every 15 minutes.
low the ABCs: airway, breathing, and
circulation. TEST-TAKING HINT: These are the signs of
2. After assessing the client’s airway and breath- ­hypovolemic shock. The test taker should se-
ing, the nurse can apply oxygen via a nasal lect the intervention which will directly affect
cannula if it is necessary. the client’s problem and can be implemented
3. The blood pressure is taken automatically the fastest to ensure the client’s safety.
by the monitor, but this is not priority over Content – Surgical: Integrated Nursing Process –
airway. Implementation: Client Needs – Safe Effective Care
4. The pulse oximeter is applied to the client’s ­Environment, Management of Care: Cognitive Level –
Analysis: Concept – Perioperative.
finger to obtain the peripheral oxygenation
status, but the nurse should assess the client’s 28. 1. Narcan does not cause pain for the client.
breathing first. 2. A client with respiratory depression
TEST-TAKING HINT: When the stem of the treated with Narcan can have another epi-
question asks the test taker to implement sode within 15 minutes after receiving the
a nursing intervention first, the test taker drug as a result of the short half-life of the
should think of assessment and then apply medication.
Maslow’s hierarchy of needs. 3. Infection would not be a concern immedi-
Content – Surgical: Integrated Nursing Process – ately after surgery.
Implementation: Client Needs – Safe Effective Care 4. Although the client may experience an im-
­Environment, Management of Care: Cognitive Level – balance in fluid or electrolytes, this problem
­Application: Concept – Perioperative. would not be of concern as a result of the
­administration of Narcan.

16_Colgrove_Ch16.indd 628 02/06/16 7:05 AM


C hapter 16  P erioperative C are 629

TEST-TAKING HINT: The test taker should be 31. 1. Taking the vital signs of the stable client
knowledgeable about medications commonly may be delegated to the UAP.
used in the postoperative period. If the test 2. Assessments cannot be delegated; “check” is a
taker had no idea of the answer, selecting an word which means “to assess.”
option ­addressing the airway is an appropri- 3. IVs cannot be hung by the UAP; this is
ate action. ­considered administering a medication.
Content – Surgical: Integrated Nursing Process – 4. Evaluating the client’s pain relief is a respon-
­Diagnosis: Client Needs – Safe Effective Care Environment, sibility of the RN.
Management of Care: Cognitive Level – Analysis: TEST-TAKING HINT: The test taker cannot
Concept – Perioperative. delegate assessment, teaching, or evaluating
29. 1. A back rub is a therapeutic intervention, but care of the client.
it is not appropriate for a life-threatening Content – Nursing Management: Integrated Nursing
complication of surgery. Process – Planning: Client Needs – Safe Effective Care
2. Ice packs should be applied to the axillary Environment, Management of Care: Cognitive Level –
and groin areas for a client experiencing Synthesis: Concept – Management.
malignant hyperthermia. 32. 1. The client appears stable; pediatric clients
3. The client would be NPO to prepare for can become unstable quickly, but the most
intubation, but an ice slush would be used to experienced nurse would not need to care for
irrigate the bladder and stomach per nasogas- this client.
tric tube. 2. A client with a fractured hip will be ambu-
4. Dantrolene is the drug of choice for lated by the physical therapist and this client
treatment. is stable, so the most experienced nurse does
5. Cooling blankets, not a warming blanket, are not need to care for this client.
used to decrease the fast-rising temperature. 3. A young client who had an appendectomy
TEST-TAKING HINT: This is an alternate-type would require routine postoperative care.
question, which requires the test taker to 4. An older client with a chronic disease
­select more than one (1) option as the correct would be a complicated case, requiring
answer. Malignant hyperthermia is a medical the care of a more experienced nurse.
emergency requiring immediate treatment. TEST-TAKING HINT: When questions ask for
Content – Surgical: Integrated Nursing Process – assignments for the most experienced nurse,
Implementation: Client Needs – Safe Effective Care the test taker should realize clients whose
­Environment, Management of Care: Cognitive Level – condition can change quickly, such as elderly
­Application: Concept – Perioperative. clients who have a complicated condition,
30. 1. Adequate urine output should be 30 mL/hr should be assigned to the most experienced
or at least 240 mL in an eight (8)-hour period. nurse.
2. Paralysis (inability to move) and paresthesia Content – Nursing Management: Integrated Nursing
(numbness and tingling) indicate neurovas- Process – Planning: Client Needs – Safe Effective Care
cular compromise to the right leg, which in- Environment, Management of Care: Cognitive Level –
dicates a complication and is not an expected Synthesis: Concept – Management.
outcome. 33. 1. The postoperative client is expected to
3. The client’s temperature and pulse are be out of bed as soon as possible, but this goal
slightly elevated and the BP is low, which is not specific to having general anesthesia.
does not indicate effective nursing care. 2. The anesthesia machine takes over
4. Lung sounds which are clear bilaterally in the function of the lungs during sur-
all lobes indicate the client has adequate gery, so the expected outcome should
gas exchange, which prevents postopera- directly reflect the client’s respiratory
tive complications and indicates effective status; the alveoli can collapse, causing
nursing care. atelectasis.
TEST-TAKING HINT: If the test taker does not 3. Urine output should be 30 mL/hr, but the
know the answer, then applying the testing expected outcome is not specific to general
rule that airway is priority would cause the anesthesia.
test taker to select option “4” as the correct 4. Sensation would be an outcome assessed ­after
answer. use of a spinal anesthesia or block, but it is
Content – Surgical: Integrated Nursing Process – not specific to general anesthesia.
­Assessment: Client Needs – Physiological Integrity,
­Reduction of Risk Potential: Cognitive Level – Analysis:
Concept –Perioperative.

16_Colgrove_Ch16.indd 629 02/06/16 7:05 AM


630 M ed -S urg S uccess

TEST-TAKING HINT: If the test taker has no want to change the surgical dressing for the
idea what the answer is, the test taker should first time.
apply Maslow’s hierarchy of needs and select 3. Having the client turn, cough, and deep
the option addressing the airway. This will breathe is the best intervention for the
not always result in the correct answer, but nurse to implement because, if a client
the rule can be followed if the test taker has has a fever within the first day, it is usually
no idea of the correct answer. caused by a respiratory problem.
Content – Surgical: Integrated Nursing Process – 4. The client is first-day postoperative, and am-
­Diagnosis: Client Needs – Safe Effective Care E
­ nvironment, bulating in the hall would not be appropriate.
Management of Care: Cognitive Level – Analysis: TEST-TAKING HINT: With clients who have
Concept – Perioperative. undergone surgery, the priority problems are
34. 1. Applying antiembolism hose may be appro- respiration and hemorrhaging. The test taker
priate, but it is not the first intervention. should select an option which addresses one
2. Attaching a drain would be appropriate but of these two (2) areas.
not before assessing the client. Content – Surgical: Integrated Nursing Process –
3. Assessing the client’s status after transfer Implementation: Client Needs – Physiological Integrity,
from the PACU should be the nurse’s first Physiological Adaptation: Cognitive Level – Application:
intervention. Concept – Perioperative.
4. Receiving reports is not the nurse’s first
intervention.
Acute Pain
TEST-TAKING HINT: The test taker should
apply the nursing process when answering 37. 1. The nurse should first assess the client
questions which require identifying the first for potential complications to determine
intervention. Assessment is the first step of if this is expected pain or pain requiring
the nursing process. notifying the health-care provider.
Content – Surgical: Integrated Nursing Process – 2. The nurse must check the MAR to determine
Implementation: Client Needs – Safe Effective Care when the last pain medication was adminis-
­Environment, Management of Care: Cognitive Level – tered, but it is not the first intervention.
Application: Concept – Perioperative. 3. The nurse must rule out complications which
35. 1. All clients who undergo surgery are at risk require medical intervention prior to medi-
for hemorrhaging, which is the priority cating the client.
problem. 4. The nurse should not administer any pain
2. The client is at risk for injury, but the prior- medication prior to ruling out complications
ity problem the first day postoperative is and checking the MAR.
hemorrhaging. TEST-TAKING HINT: The test taker should
3. A potential fluid imbalance would be for less ­apply the nursing process when answering
fluid as a result of blood loss and decreased the question and select an option that ad-
oral intake; it would not be for fluid volume dresses assessment.
excess. Content – Surgical: Integrated Nursing Process –
4. Infection would be a potential problem but Implementation: Client Needs – Safe Effective Care
not priority over hemorrhaging on the first ­Environment, Management of Care: Cognitive Level –
postoperative day. Application: Concept – Assessment.
TEST-TAKING HINT: Remember to apply the 38. 1. Pain medications should be administered
ABCs of care: airway, breathing, and circula- at the frequency ordered by the HCP, not
tion. The test taker must apply testing rules just when the client requests them, espe-
when answering questions which require cially for acute pain.
identifying priority problems. 2. Nonpharmacological methods should never
Content – Surgical: Integrated Nursing Process – replace medications, but they should be used in
­Diagnosis: Client Needs – Safe Effective Care E
­ nvironment, combination to help keep the client comfortable.
Management of Care: Cognitive Level – Analysis: 3. Cryotherapy (cold) is used immediately post-
Concept – Perioperative. operative or postinjury. Heat applications are
36. 1. Antibiotics need to be administered at the applied at a later time.
scheduled time. 4. Only clients should activate the PCA to
2. These data would not support the need to ­prevent overdosing.
change the dressing, and surgeons usually

16_Colgrove_Ch16.indd 630 02/06/16 7:05 AM


C hapter 16  P erioperative C are 631

TEST-TAKING HINT: Option “4” should be 3. Prior to giving any medication, the nurse
eliminated because a basic concept is the should assess any allergies, but it is not the
­client should be the person in control of the priority intervention.
pain, not a family member; pain is subjective. 4. The nurse should question the client to
Content – Surgical: Integrated Nursing Process – rule out complications and to determine
Implementation: Client Needs – Safe Effective Care which medication and amount would be
­Environment, Management of Care: Cognitive Level – most appropriate for the client. This is
­Application: Concept – Comfort. assessment.
39. 1. This exemplifies the role of provider of care, TEST-TAKING HINT: When questions require
and it does not address client advocacy. a priority answer, the test taker should look
2. This action is addressing the role of for an option which addresses assessment,
collaborator. but the test taker should remember there are
3. When the nurse contacts the HCP about many words which reflect assessment, such
unrelieved pain, the nurse is speaking as “discuss,” “determine,” “monitor,” or
when the client cannot, which is the defi- “obtain,” to name a few.
nition of a client advocate. Content – Surgical: Integrated Nursing Process –
4. This action is providing care to the client and Implementation: Client Needs – Physiological Integrity,
does not address client advocacy. Pharmacological and Parenteral Therapies: Cognitive
Level – Application: Concept – Medication.
TEST-TAKING HINT: One (1) of the most
­important roles of the nurse is to be a ­client 42. 1. Medication administration cannot be
advocate. The nurse must always identify prob- delegated to a UAP.
lems and follow through to their resolution. 2. This task does not require teaching,
Content – Surgical: Integrated Nursing Process – ­evaluating, or nursing judgment and
Implementation: Client Needs – Safe Effective Care therefore can be delegated.
­Environment, Management of Care: Cognitive Level – 3. Assessment cannot be delegated to a UAP.
Knowledge: Concept – Comfort. 4. Evaluation of teaching cannot be delegated to
a UAP.
40. 1. A decrease in use of pain medication does not
mean the client’s pain is managed; the client TEST-TAKING HINT: The terms “observe” and
may be concerned about possible addiction to “check” in options “3” and “4” are different
the pain medication. from the term “evaluate,” but reading the
2. Clients experiencing acute pain will not options, the tasks are clearly addressing the
be involved in self-care because of their evaluation step of the nursing process. Evalu-
reluctance to move, which increases the ation cannot be delegated to the UAP.
pain; therefore, participation indicates the Content – Nursing Management: Integrated Nursing
client’s pain is tolerable. Process – Planning: Client Needs – Safe Effective Care
3. Using relaxation techniques does not indicate Environment, Management of Care: Cognitive Level –
the client’s pain is under control. Synthesis: Concept – Management.
4. This would be an expected outcome of a 43. 1. This procedure ensures client safety by
knowledge-deficit problem. preventing medication from being given
TEST-TAKING HINT: The test taker must first to the wrong client.
determine what is the expected outcome, 2. This is a legal requirement, not a safety issue.
which should be “relief of pain,” and then de- 3. This intervention would prevent giving
termine which option addresses the client’s a narcotic to a client who is unstable or
relief of pain. compromised.
Content – Surgical: Integrated Nursing Process – 4. Determining allergies addresses client safety.
­Diagnosis: Client Needs – Safe Effective Care E
­ nvironment, 5. It would not be realistic to recheck all orders.
Management of Care: Cognitive Level – Analysis: TEST-TAKING HINT: This question specifically
Concept – Comfort. asks the test taker to identify interventions
41. 1. It is important to monitor vital signs, but it is for safely administering medication to the
not the priority intervention prior to admin- client. Therefore, options “2” and “5” could
istering the medication. be eliminated because they do not address
2. The nurse should verify the time the last dose the client’s safety. This is an alternate-type
was administered to determine the time the question requiring the test taker to select
next dose could be administered, but this is more than one (1) option as the correct
not the priority intervention. answer.

16_Colgrove_Ch16.indd 631 02/06/16 7:05 AM


632 M ed -S urg S uccess

Content – Surgical: Integrated Nursing Process – 3. Before asking this question, the nurse should
Implementation: Client Needs – Physiological Integrity, have specific information to suspect drug use.
Pharmacological and Parenteral Therapies: Cognitive 4. Discussing the client’s prescription medica-
Level – Application: Concept – Medication. tions is necessary, but asking for a list of
44. 1. The nurse should use words a four (4)-year- medications will not address the client’s pain
old child understands and remembers, but management.
this is not the best way to assess pain. TEST-TAKING HINT: Assessment, the first step of
2. A four (4)-year-old child cannot be expected the nursing process, of pain perception is indi-
to use the numeric pain scale because of lack cated when caring for a client with acute pain.
of cognitive abilities, and explaining it to the Content – Surgical: Integrated Nursing Process – ­
parents does not address the child’s pain. Assessment: Client Needs – Safe Effective Care
3. The Faces Scale is the best way to assess ­Environment, Management of Care: Cognitive Level –
pain in a four (4)-year-old child. Analysis: Concept – Comfort.
4. This does not assess the child’s pain, and ad-
47. 1. Assessing why the client is not using the
ministering the pain medication every four
medication is a priority and then, based on
(4) hours may compromise the child’s safety.
the client’s response, a plan of care can be
TEST-TAKING HINT: When age is listed, it is determined.
an indication the question is asking for age- 2. The fact a client is not using pain medication
specific information. The test taker should warrants the nurse determining the cause so
consider developmental levels for that par- appropriate action can be taken.
ticular age. 3. This may or may not be why the client is not
Content – Surgical: Integrated Nursing Process – using the PCA pump. The nurse must first
Implementation: Client Needs – Safe Effective Care determine why the client is not using pain
­Environment, Management of Care: Cognitive Level – medication.
Application: Concept – Comfort. 4. This may or may not be indicated, but until
45. 1. Assessing verbal and nonverbal cues is the nurse determines why the client is not
the priority intervention because pain is taking the medication, this action should not
subjective. be implemented.
2. Some clients are hesitant to ask for medica- TEST-TAKING HINT: Assessment is priority
tion or believe it is a sign of weakness to ask. when caring for a client. It is the first step
3. There are times when pain medications are of the nursing process, and if the test taker
given on a routine basis, but it is not the best is unsure of the correct answer, it is the best
answer because assessment takes priority. choice to select.
4. Alternative therapies, such as imagery, are Content – Surgical: Integrated Nursing Process –
used in combination with medications, but Implementation: Client Needs – Physiological Integrity,
they never replace medications. Pharmacological and Parenteral Therapies: Cognitive
TEST-TAKING HINT: Options such as option Level – Application: Concept – Medication.
“4” which have absolute words such as “only” 48. 1. This is a psychosocial problem, which is not
usually can be eliminated as a correct answer. appropriate for an acute physiological problem.
The test taker should remember to apply 2. A potential problem is not priority for a ­client
the nursing process, and the first step is in acute pain.
assessment. 3. Alteration in comfort is addressing the
Content – Surgical: Integrated Nursing Process – ­client’s acute pain.
Implementation: Client Needs – Physiological Integrity, 4. Altered sensory input does not address the
Basic Care and Comfort: Cognitive Level – Application: client’s acute physical pain.
Concept – Assessment.
TEST-TAKING HINT: The test taker should be
46. 1. The answer to this request would indicate ­familiar with NANDA’s list of client problems
if the client has had a negative experi- and nursing diagnoses, which includes altera-
ence which may influence the client’s pain tion in comfort for pain. Potential problems
management. do not have priority over actual problems.
2. Previous surgeries would be pertinent infor- Content – Surgical: Integrated Nursing Process –
mation but not for pain management. ­Diagnosis: Client Needs – Physiological Integrity, Basic
Care and Comfort: Cognitive Level – Analysis:
Concept – Comfort.

16_Colgrove_Ch16.indd 632 02/06/16 7:05 AM


CONCEPTS
49. 1. Even though this is an oral medication and 3. The client is not allowed to sit on the peri-
the client is NPO, beta blocker medications neal area for several days and should be main-
are not held for surgery. They are adminis- tained in a side-lying position when possible.
tered with a sip of water. Beta blockers can 4. The nurse should assess for bowel activity
cause rebound cardiac dysrhythmias if not at regularly scheduled intervals.
administered as routinely taken. Beta blocker 5. Pain medication is administered to control
medications should be tapered off to avoid the client’s pain; the nurse is concerned with
cardiac issues. The nurse would not question client comfort, not addiction. Poorly con-
this medication. trolled pain is more likely to result in drug-
2. An antibiotic medication would be expected seeking behavior than adequately treated
to be administered prophylactically to pre- pain.
vent infections postoperatively. This is a TEST-TAKING HINT: “Select all that apply”
cephalosporin, not penicillin. The nurse questions must be answered with each option
would not question this medication. considered a true/false freestanding ques-
3. Some facilities apply EMLA cream prior to tion. The test taker cannot use one option to
initiating the IV to decrease pain during the rule out another.
procedure. The nurse would not question this Content – Surgical: Integrated Nursing Process –
medication. Implementation: Client Needs – Safe Effective Care
4. Dabigatran is an anticoagulant; the nurse ­Environment, Management of Care: Cognitive Level –
would question this medication. The Application: Concept – Perioperative.
medication should have stopped five (5) to
seven (7) days preoperatively. The nurse 51. 1. The client will have been told that
would question this medication and the he/she has cancer. The nurse should
surgery may have to be postponed because allow the client to verbalize feelings.
the client is receiving an anticoagulant. 2. The nurse should routinely monitor any
dressing, but assessing the dressing does not
TEST-TAKING HINT: Because options “1” and
address the pathology report and its implica-
“4” are oral medications and clients who are tions for the client.
to receive general anesthesia are NPO, the 3. The report indicates a diagnosis of cancer,
test taker would choose from these options. not infection.
The nurse must know which medications 4. The pathology report is performed on the
have specific requirements, such as steroids actual cells removed from the tissues. The
and beta blockers are tapered off. report does not need to be verified.
Content – Surgical: Integrated Nursing Process –
TEST-TAKING HINT: The test taker could
Implementation: Client Needs – Safe Effective Care
­Environment, Management of Care: Cognitive Level – ­reason out the correct answer from some key
­Application: Concept – Perioperative. words in the report. Reed-Sternberg cells are
only associated with Hodgkin’s lymphoma,
50. 1. The colostomy stoma should be assessed but if the test taker did not know this then
to determine circulation to the stoma at “Stage 3” is usually used to designate the
least every four (4) hours. A purple or ­severity of the cancer being diagnosed.
bluish purple indicates that the circulation Content – Surgical: Integrated Nursing Process –
to the stoma is impaired and is a medical Implementation: Client Needs – Safe Effective Care
emergency. ­Environment, Management of Care: Cognitive Level –
2. This is an extensive surgery requiring Analysis: Concept – Perioperative.
the client to be under general anesthe-
sia for several hours. Turning, coughing,
and deep-breathing exercises done at
least every two (2) hours helps to prevent
pneumonia.

633

16_Colgrove_Ch16.indd 633 02/06/16 7:05 AM


634 M ed -S urg S uccess

52. 1. This client has been without food or water Content – Surgical: Integrated Nursing Process –
for several hours prior to the surgery, so the Implementation: Client Needs – Safe Effective Care
client needs fluid intake to urinate. Indwell- ­Environment, Management of Care: Cognitive Level –
ing catheters carry with them the risk for Synthesis: Concept – Perioperative.
a urinary tract infection. The nurse should 54. 1. The client should not be ambulated until the
continue to monitor the client’s output before nurse has a chance to assess for the client’s
taking this action. ability to ambulate safely.
2. Skin turgor should not be assessed for indi- 2. The nurse should assess the vital signs
cating a fluid volume deficit yet. The client from PACU with the current vital signs to
received fluids during surgery and would have be sure that the client is stable.
had an indwelling catheter to prevent incon- 3. The nurse should assess the intravenous
tinence during surgery. lines, indwelling catheters, and tubes upon
3. The nurse should wait and monitor the receiving the client.
client before taking another action. The 4. The nurse must assess the surgical site for
client has an IV running at 150 mL per bleeding to know if the client is actually
hour. The client should be given a chance stable or not.
to produce urine before rushing to take 5. Only the client should push the PCA pump’s
another action. button; otherwise the client may receive an
4. The client should be allowed to rest. It has overdose of medication.
been a long day of waiting for the surgery to
TEST-TAKING HINT: The client is “returning”
take place and then having the surgery. Push-
from PACU. This client may still be groggy
ing liquids at midnight is not advised, consid-
from anesthesia and should not be ambulat-
ering the need for rest.
ing until the nurse has assessed the client
TEST-TAKING HINT: Timing is important to and is aware the client is awake enough to
consider when answering this question— ambulate safely.
midnight, return from surgery, and PACU at Content – Surgical: Integrated Nursing Process –
1800. Basic knowledge of surgical procedure Implementation: Client Needs – Safe Effective Care
is the client is NPO for at least eight (8) ­Environment, Management of Care: Cognitive Level –
hours before general anesthesia. Application: Concept – Perioperative.
Content – Surgical: Integrated Nursing Process –
55. 1. This may be an appropriate intervention,
Implementation: Client Needs – Safe Effective Care
­Environment, Management of Care: Cognitive Level – but unrelieved pain three (3) days after the
Application: Concept – Perioperative. surgery may indicate a problem. The nurse
should assess the client first.
53. 1. The nurse should recognize the WBC count 2. A client in pain is not ready to learn. If nar-
is high, possibly indicating a current infec- cotic medication is not successful, something
tion. The HCP should be notified immedi- else may be occurring.
ately, and surgery may be cancelled because 3. The first step of the nursing process is to
a laparoscopic cholecystectomy is an elective assess. Pain unrelieved three (3) days post-
procedure. operative needs to be investigated.
2. The nurse should notify the surgeon. 4. Repositioning the client may or may not help,
Surgery may be cancelled because a lapa- but the nurse should assess the client.
roscopic cholecystectomy is an elective
TEST-TAKING HINT: The first step of the nurs-
procedure.
ing process is ASSESS. The test taker must
3. The client may have any of a number of in-
have some systematic method of problem
fections. The high WBC could indicate pneu-
solving. The test taker must also remem-
monia but it could indicate another type of
ber “if in stress—do not assess.” In other
infection.
words, the test taker has been given enough
4. The client may have any of a number of in-
information to implement an intervention
fections. The high WBC could indicate pneu-
immediately.
monia but it could indicate another type of
infection. Content – Surgical: Integrated Nursing Process – ­
Assessment: Client Needs – Safe Effective Care
TEST-TAKING HINT: The test taker should find ­Environment, Management of Care: Cognitive Level –
the abnormal data in the chart before decid- ­Application: Concept – Perioperative.
ing what action to take. If all the data pre-
sented are within normal limits, then option
“1” would be the answer.

16_Colgrove_Ch16.indd 634 02/06/16 7:05 AM


C hapter 16  P erioperative C are 635

PERIOPERATIVE CARE COMPREHENSIVE


EXAMINATION

1. Which client would the nurse identify as having 5. Which problem would be appropriate for the
the highest risk for developing postoperative nurse to identify for the preoperative client having
complications? an open reduction and internal fixation of the
1. The 67-year-old client who is obese, has right ankle?
diabetes, and takes insulin. 1. Alteration in skin integrity.
2. The 50-year-old client with arthritis taking 2. Knowledge deficit of postoperative care.
nonsteroidal anti-inflammatory drugs. 3. Alteration in gas exchange and pattern.
3. The 45-year-old client having abdominal 4. Alteration in urinary elimination.
surgery to remove the gallbladder.
6. The nurse and an unlicensed assistive personnel
4. The 60-year-old client with anemia who
(UAP) are caring for clients on a surgery unit.
smokes one (1) pack of cigarettes per day.
Which task would be most appropriate to
2. The nurse is completing the preoperative delegate to the UAP?
checklist on a client going to surgery. Which 1. Explain to the client how to cough and deep
information should the nurse report to the breathe.
surgeon? 2. Discuss preoperative plans with the client and
1. The client understands the purpose of the family.
surgery. 3. Determine the ability of the caregivers to
2. The client stopped taking aspirin three (3) provide postoperative care.
weeks ago. 4. Assist the client to take a povidone-iodine
3. The client uses the oral supplements licorice (Betadine) shower.
and garlic.
7. Which action by the client indicates to the nurse
4. The client has mild levels of preoperative
preoperative teaching has been effective?
anxiety.
1. The client demonstrates how to use the
3. Which statement explains the nurse’s incentive spirometer device.
responsibility when obtaining informed consent 2. The client demonstrates the use of the patient-
for the client undergoing a surgical procedure? controlled analgesia pump.
1. The nurse should provide detailed information 3. The client can name two (2) anesthesia agents
about the procedure. used during surgery.
2. The nurse should inform the client of any legal 4. The client ambulates down the hall to the
consultation needed. nurse’s station each hour.
3. The nurse should write a list of the risks for
8. Which intervention has priority for the nurse in
postoperative complications.
the surgical holding area?
4. The nurse should ensure the client is
1. Verify the surgical checklist.
voluntarily giving consent.
2. Prepare the client’s surgical site.
4. Which client outcome would the nurse identify 3. Assist the client to the bathroom.
for the preoperative client? 4. Restrain the client on the surgery table.
1. The client’s abnormal laboratory data will be
9. The client in the surgical holding area tells the
reported to the anesthesiologist.
nurse “I am so scared. I have never had surgery
2. The client will not have any postoperative
before.” Which statement would be the nurse’s
complications for the first 24 hours.
most appropriate response?
3. The client will demonstrate the use of a pillow
1. “Why are you afraid of the surgery?”
to splint while deep breathing.
2. “This is the best hospital in the city.”
4. The client will complete an advance directive
3. “Does having surgery make you afraid?”
before having the surgery.
4. “There is no reason to be afraid.”

16_Colgrove_Ch16.indd 635 02/06/16 7:05 AM


636 M ed -S urg S uccess

10. The unlicensed assistive personnel (UAP) 15. The circulating nurse notes a discrepancy in
can be overheard talking loudly to the scrub the needle count. What intervention should the
technologist discussing a problem which nurse implement first?
occurred during one (1) of the surgeries. Which 1. Inform the other members of the surgical
intervention should the nurse in the surgical team about the problem.
holding area with a female client implement? 2. Assume the original count was wrong and
1. Close the curtains around the client’s change the record.
stretcher. 3. Call the radiology department to perform a
2. Instruct the UAP and scrub tech to stop the portable x-ray.
discussion. 4. Complete an occurrence report and notify the
3. Tell the surgeon on the case what the nurse risk manager.
overheard.
16. The client in the surgery holding area identifies
4. Inform the client the discussion was not about
the left arm as the correct surgical site, but
her surgeon.
the operative permit designates surgery
11. The nurse is completing the preoperative to be performed on the right arm. Which
checklist. Which laboratory value should interventions should the nurse implement?
be reported to the health-care provider Select all that apply.
immediately? 1. Review the client’s chart.
1. Hemoglobin 13.1 g/dL. 2. Notify the surgeon.
2. Glucose 60 mg/dL. 3. Immediately call a “time-out.”
3. White blood cells 6 (× 103)/mm3. 4. Correct the surgical permit.
4. Potassium 3.8 mEq/L. 5. Request the client mark the left arm.
12. Which problem is appropriate for the nurse to 17. The nurse received a male client from the
identify for a client in the intraoperative phase of postanesthesia care unit. Which assessment data
surgery? would warrant immediate intervention?
1. Alteration in comfort. 1. The client’s vital signs are T 97˚F, P 108,
2. Disuse syndrome. R 24, and BP 80/40.
3. Risk for injury. 2. The client is sleepy but opens the eyes to his
4. Altered gas exchange. name.
3. The client is complaining of pain at a “5” on a
13. The client is in the lithotomy position during
1-to-10 pain scale.
surgery. Which nursing intervention should be
4. The client has 20 mL of urine in the urinary
implemented to decrease a complication from
drainage bag.
the positioning?
1. Increase the intravenous fluids. 18. The client received naloxone (Narcan), an opioid
2. Lower one leg at a time. antagonist, in the postanesthesia care unit.
3. Raise the foot of the stretcher. Which nursing intervention should the nurse
4. Administer epinephrine, a vasopressor. include in the care plan?
1. Measure the client’s intake and output hourly.
14. The circulating nurse observes the surgeon
2. Administer sleep medications at night.
tossing a bloody gauze sponge onto the sterile
3. Encourage the client to verbalize feelings.
field. Which action should the circulating nurse
4. Monitor respirations every 15 to 30 minutes.
implement first?
1. Include the sponge in the sponge count. 19. Which nursing task would be most appropriate
2. Obtain a new sterile instrument pack. to delegate to the unlicensed assistive personnel
3. Tell the surgical technologist about the (UAP) on a postoperative unit?
sponge. 1. Change the dressing over the surgical site.
4. Throw the sponge in the sterile trashcan. 2. Teach the client how to perform incentive
spirometry.
3. Empty and record the amount of drainage in
the JP drain.
4. Auscultate the bowel sounds in all four (4)
quadrants.

16_Colgrove_Ch16.indd 636 02/06/16 7:05 AM


C hapter 16  P erioperative C are 637

20. Which client assessment data are priority for 24. The client one (1) day postoperative develops an
the postanesthesia care nurse? elevated temperature. Which intervention would
1. Bowel sounds. have priority for the client?
2. Vital signs. 1. Encourage the client to deep breathe and
3. IV fluid rate. cough every hour.
4. Surgical site. 2. Encourage the client to drink 200 mL of
water every shift.
21. The male client in the day surgery unit
3. Monitor the client’s wound for drainage every
complains of difficulty urinating postoperatively.
eight (8) hours.
Which intervention should the nurse implement?
4. Assess the urine output for color and clarity
1. Insert an indwelling catheter.
every four (4) hours.
2. Increase the intravenous fluid rate.
3. Assist the client to stand to void. 25. Which statement made by the client who is
4. Encourage the client to increase fluids. postoperative abdominal surgery indicates the
discharge teaching has been effective?
22. The postoperative client complains of hearing
1. “I will take my temperature each week and
a “popping sound” and feeling “something
report any elevation.”
opening” when ambulating in the room. Which
2. “I will not need any pain medication when
intervention should the nurse implement first?
I go home.”
1. Notify the surgeon the client has had an
3. “I will take all of my antibiotics until they are
evisceration.
gone.”
2. Contact the surgery department to prepare
4. “I will not take a shower until my three
for emergency surgery.
(3)-month checkup.”
3. Assess the operative site and cover the site
with a moistened dressing. 26. The client diagnosed with appendicitis has
4. Explain this is a common feeling and tell the undergone an appendectomy. At two (2) hours
client to continue with activity. postoperative, the nurse takes the vital signs
and notes T 102.6˚F, P 132, R 26, and BP
23. The nurse received a report the elderly
92/46. Which interventions should the nurse
postoperative client became confused during the
implement? List in order of priority.
previous shift. Which client problem would the
1. Increase the IV rate.
nurse include in the plan of care?
2. Notify the health-care provider.
1. Risk for injury.
3. Elevate the foot of the bed.
2. Altered comfort level.
4. Check the abdominal dressing.
3. Impaired circulation.
5. Determine if the IV antibiotics have been
4. Impaired skin integrity.
administered.

16_Colgrove_Ch16.indd 637 02/06/16 7:05 AM


PERIOPERATIVE CARE COMPREHENSIVE
EXAMINATION ANSWERS AND RATIONALES

1. 1. This client has comorbid conditions— Content – Surgical: Client Needs – Safe Effective Care
advanced age, obesity, and diabetes—which ­Environment, Management of Care: Cognitive Level –
put this client at a higher risk for postop- Application: Concept – Perioperative.
erative complications. 4. 1. This would be an outcome for the health-care
2. This client’s risk factor of arthritis can make team, not the client.
positioning in surgery and movement in the 2. This would be an appropriate postoperative,
postoperative period more difficult, but this not preoperative, outcome.
does not put the client at greater risk for post- 3. This would be the expected outcome for
operative complications. The NSAIDs can be the client during the preoperative phase.
held a few days prior to surgery to decrease the After the teaching has been completed, the
problems associated with NSAIDs. client should be able to demonstrate how
3. The client with abdominal surgery may have to splint with the pillow while deep breath-
respiratory complications, but the client is not ing and coughing.
at as high risk as the older client with diabetes 4. This would not be an expected outcome for
and obesity. the preoperative client. All clients should be
4. The client’s smoking increases the risk of pul- encouraged to complete an advance directive,
monary complications and increases the blood but it is not required by law.
level of carboxyhemoglobin, but this client does
Content – Surgical: Integrated Nursing Process –
not have the problems of delayed healing and Diagnosis: Client Needs – Safe Effective Care Environment,
age the 67-year-old client with diabetes has. Management of Care: Cognitive Level – Analysis:
Content – Surgical: Integrated Nursing Process – Concept – Perioperative.
­Assessment: Client Needs – Physiological Integrity, ­Reduction
of Risk Potential: Cognitive Level – Analysis: Concept – 5. 1. This client problem would not be written until
Perioperative. after the surgery.
2. This would be an appropriate client prob-
2. 1. The surgeon should be notified if the client lem for the preoperative client who is
does not understand the surgical procedure, scheduled for ankle repair. Teaching is
not if the client understands. priority.
2. Aspirin should be stopped before surgery to 3. This would not be a problem for a client
help prevent bleeding. scheduled for surgery.
3. Licorice and garlic can interfere with co- 4. This would not be a problem for a client
agulation; therefore, the surgeon should be scheduled for surgery.
notified.
Content – Surgical: Integrated Nursing Process –
4. Mild levels of anxiety and apprehension before Diagnosis: Client Needs – Safe Effective Care Environment,
surgery are normal. Management of Care: Cognitive Level – Analysis:
Content – Surgical: Integrated Nursing Process – Concept – Perioperative.
­Implementation: Client Needs – Safe Effective Care
­Environment, Management of Care: Cognitive Level – 6. 1. Teaching cannot be delegated.
­Application: Concept – Perioperative. 2. Discussing the preoperative plans is part of the
planning process and cannot be delegated.
3. 1. The nurse is not responsible for explain- 3. Evaluation cannot be delegated to the UAP.
ing details of surgery. This is the surgeon’s 4. The UAP can assist a stable client to take a
responsibility. shower whether or not it is with Betadine.
2. The nurse is not responsible for informing the
Content – Nursing Management: Integrated Nursing
client of any need for legal representation. ­Process – Planning: Client Needs – Safe Effective Care
3. The surgeon is responsible for informing the ­Environment, Management of Care: Cognitive Level –
client of the risks and hazards of the surgery. ­Synthesis: Concept – Management.
4. The nurse is responsible for ensuring the
client voluntarily signs the surgical consent
form giving permission for the surgery
without coercion.

638

16_Colgrove_Ch16.indd 638 02/06/16 7:05 AM


C hapter 16  P erioperative C are 639

7. 1. The teaching is effective if the client 3. This is a nursing problem and the nurse
is able to demonstrate the use of the should handle the situation. The surgeon is
­spirometer prior to surgery. not in the chain of command of the UAP or
2. The patient-controlled analgesia pump would the scrub tech.
not be available prior to surgery because the 4. Telling the client the situation does not in-
pumps are charged to the client on a daily volve her surgeon is involving the client even
basis and the client would not be able to more in the overheard conversation.
­demonstrate how to use it. Content – Surgical: Integrated Nursing Process –
3. Determining allergies to anesthesia medica- ­Implementation: Client Needs – Safe Effective Care
tions is important prior to surgery, but the ­Environment, Management of Care: Cognitive Level –
nurse would not teach the specific medication ­Application: Concept – Management.
names.
11. 1. This hemoglobin is within normal limits and
4. This would demonstrate increased mobility
would not warrant immediate action.
and would be encouraged after surgery, but it
2. This glucose level indicates hypoglycemia,
would not determine if teaching was effective.
which requires medical intervention.
Content – Surgical: Integrated Nursing Process – 3. This white blood cell value is within normal
­Evaluation: Client Needs – Physiological Integrity, range and would not be reported.
­Reduction of Risk Potential: Cognitive Level – Synthesis:
4. This potassium level is within normal limits
Concept – Perioperative.
and would not require intervention.
8. 1. The surgical checklist is assessed when Content – Surgical: Integrated Nursing Process –
the client arrives in the surgery depart- ­Assessment: Client Needs – Physiological Integrity,
ment holding area where clients wait for a ­Reduction of Risk Potential: Cognitive Level – Analysis:
short time before entering the operating Concept – Perioperative.
room.
12. 1. This client problem would be appropriate
2. Preparing the surgical site is completed in the for a postoperative client or, in some circum-
surgery suite but not until the client and sur- stances, a preoperative client, but not for a
gery have been verified. client in surgery.
3. The client should have voided just prior to 2. This is a problem of long-term immobility
being transported to the surgery department. and would not apply during surgery.
4. Securing the client onto the surgical table 3. This problem would be appropriate for
would be important in the operating room, the intraoperative phase. The circulating
not in the holding area.
nurse would strap and carefully pad areas
Content – Surgical: Integrated Nursing Process – to prevent damage to tissues and nerves.
­Implementation: Client Needs – Safe Effective Care 4. The client is receiving oxygen or breathing
­Environment, Management of Care: Cognitive Level –
by the ventilator. The client should not have
Synthesis: Concept – Perioperative.
an alteration in gas exchange.
9. 1. The nurse should never ask the client Content – Surgical: Integrated Nursing Process –
“why.” The client does not owe the nurse an Diagnosis: Client Needs – Safe Effective Care Environment,
explanation. Management of Care: Cognitive Level – Analysis:
2. This response defends the hospital and does Concept – Perioperative.
not address the client’s feelings. 13. 1. The anesthesiologist, not the nurse in the op-
3. This response is therapeutic and erating room, manages the intravenous fluids.
­promotes communication of feelings. 2. The lithotomy position has both legs
4. This statement is closed-ended and will not elevated and placed in stirrups. The legs
encourage the continued discussion should be lowered one leg at a time to
of “fear.” prevent hypotension from the shift of the
Content – Surgical: Integrated Nursing Process – blood.
­Implementation: Client Needs – Safe Effective Care 3. Raising the foot of the bed would be a treat-
­Environment, Management of Care: Cognitive Level –
ment of hypotension, but not hypotension
­Application: Concept – Perioperative.
resulting from the lithotomy position.
10. 1. Closing curtains will not keep loud conversa- 4. Epinephrine, a vasopressor, is used during
tions from being overheard. codes to shunt blood from the periphery to
2. The UAP and scrub tech are violating the central circulation.
HIPAA and should be told to stop the Content – Surgical: Integrated Nursing Process –
conversation immediately. Implementation: Client Needs – Physiological Integrity,
Reduction of Risk Potential: Cognitive Level – Application:
Concept – Perioperative.

16_Colgrove_Ch16.indd 639 02/06/16 7:05 AM


640 M ed -S urg S uccess

14. 1. All sponges must be included in the sponge Content – Surgical: Integrated Nursing Process –
count, but it is not the first intervention. ­Implementation: Client Needs – Safe Effective Care
2. The circulating nurse should obtain another Environment, Management of Care: Cognitive Level –
sterile pack for the operation to continue, but Application: Concept – Perioperative.
it is not the first intervention. 17. 1. These are symptoms of hypovolemic
3. The circulating nurse should inform shock and require immediate intervention.
the surgical technologist of any break 2. This is a common response to anesthesia.
in sterile technique or field. This is the ­Clients are sleepy until the anesthesia
first intervention because the field is now wears off.
contaminated. 3. Pain management is required, but this does
4. This action is below standards of the Associa- not indicate a life-threatening complication.
tion of Operating Room Nurses and violates 4. Urine outputs should be monitored in the
the principles of sterility. The sponge is in- postoperative period, but indwelling cath-
cluded in the count and will not be discarded eter bags are emptied in the PACU prior to
until the end of the case and all sponges have transferring the client to the floor, so 20 mL
been accounted for. would not warrant immediate intervention.
Content – Surgical: Integrated Nursing Process – Content – Surgical: Integrated Nursing Process –
­Implementation: Client Needs – Safe Effective Care Assessment: Client Needs – Safe Effective Care
­Environment, Safety and Infection Control: Cognitive ­Environment, Management of Care: Cognitive Level –
Level – Application: Concept – Perioperative. Analysis: Concept – Perioperative.
15. 1. If the needle count does not correlate, the 18. 1. Narcan does not alter the urinary elimina-
surgical technologist and the other surgi- tion; therefore, this is not an appropriate
cal team members should be informed. ­intervention for this client.
After repeating the count, a search for the 2. Anesthesia may alter sleep patterns, but
missing needle should be conducted. this nursing intervention does not take into
2. Assuming the original count was wrong is ­account the need for Narcan to be adminis-
illegal and dangerous for the client. tered to the client.
3. If the needle is not located, an x-ray will be 3. This nursing intervention does not address
done, but this is not the first intervention. the use of Narcan.
4. If the missing needle is not located, an occur- 4. Narcan is given to reverse respiratory
rence report should be completed and sent depression from opioid analgesic medi-
to the risk manager, but this it is not the first cations and has a short half-life. The
intervention. client may experience a rebound respira-
Content – Surgical: Integrated Nursing Process – tory depression in 15 to 20 minutes, so
­Implementation: Client Needs – Safe Effective Care this nursing intervention of monitoring
­Environment, Safety and Infection Control: Cognitive respirations every 15 to 30 minutes is
Level – Application: Concept – Perioperative. appropriate.
16. 1. When the client in the holding area states Content – Surgical: Integrated Nursing Process –
the surgery site differs from the scheduled ­Planning: Client Needs – Pharmacological and ­Parenteral
surgery, the nurse should identify the cli- Therapies: Cognitive Level – Synthesis: Concept –
ent and review the client’s chart. Medication.
2. If there is a discrepancy, the nurse should 19. 1. The surgeon usually removes the first surgi-
notify the surgeon to explain the situation cal dressing. The nurse performs surgical
and resolve the issue. dressing changes so asepsis is maintained and
3. The Joint Commission surgical standards the incision can be assessed.
state a “time-out” period is called and 2. Nurses cannot delegate teaching and
everything stops until the discrepancy is assessment.
resolved. 3. Emptying the drainage devices and
4. The nurse should not change a permit. If recording the amounts on the bedside
an error is discovered, the nurse should ­intake and output forms can be delegated.
­correct the situation within legal and ethical 4. Listening to bowel sounds is assessing and
guidelines. cannot be delegated.
5. Clients are encouraged to mark the
­correct side or site with indelible ink.

16_Colgrove_Ch16.indd 640 02/06/16 7:05 AM


C hapter 16  P erioperative C are 641

Content – Surgical: Integrated Nursing Process – Content – Surgical: Integrated Nursing Process –
­Planning: Client Needs – Pharmacological and Parenteral ­Implementation: Client Needs – Safe Effective Care
Therapies: Cognitive Level – Synthesis: Concept – ­Environment, Management of Care: Cognitive Level –
Nursing Roles. ­Application: Concept – Perioperative.

20. 1. Bowel sounds should be assessed, but it is not 23. 1. Anytime the nurse has a client who is
priority for the surgical client in the PACU. disoriented, the nurse must initiate fall/
2. The postanesthesia care unit nurse should safety precautions.
follow the ABCs format described by the 2. Confusion would not indicate a change in
American Heart Association. “A” is for comfort level.
airway, “B” is for breathing, and “C” is for 3. Sudden confusion is usually not a circulation
circulation. Vital signs assess for hemo- problem.
dynamic stability; this is priority in the 4. Impaired skin integrity would not cause
PACU. confusion.
3. Intravenous fluids should be assessed ­after Content – Surgical: Integrated Nursing Process –
breathing and circulation have been assessed. Diagnosis: Client Needs – Safe Effective Care E
­ nvironment,
4. The surgical site should be assessed after Management of Care: Cognitive Level – Analysis:
the intravenous fluid rate is assessed. Concept – Perioperative.
Content – Surgical: Integrated Nursing Process – 24. 1. When a postoperative client develops a
­Assessment: Client Needs – Physiological Integrity, fever within the first 24 hours, the cause
­Reduction of Risk Potential: Cognitive Level – Analysis:
is usually in the respiratory system. The
Concept – Perioperative.
client should increase deep breathing
21. 1. This intervention is invasive, increases the and coughing to assist the client to ex-
client’s risk for infection, and should be the pand the lungs and decrease pulmonary
last resort. complications.
2. Increasing the IV fluids might increase the 2. Drinking fluid can bring down temperature,
amount of urine in the bladder and cause but 200 mL would not be a sufficient amount
­further discomfort. to accomplish this. Unless contraindicated,
3. Helping the male client to stand can offer the client should drink from one (1) to two
the assistance needed to void. The safety (2) L/day.
of the client should be ensured. 3. Wound infections may cause the fever later
4. Drinking more fluids helps to increase in the recovery but will usually not elevate
­urinary output but will not assist the client to within the first 24 hours after surgery.
empty the bladder. 4. A urinary tract infection may occur later but
Content – Surgical: Integrated Nursing Process – would probably not be the cause of elevated
Implementation: Client Needs – Physiological Integrity, temperature within 24 hours after surgery.
Basic Care and Comfort: Cognitive Level – Application: Content – Surgical: Integrated Nursing Process –
Concept – Perioperative. ­Implementation: Client Needs – Safe Effective Care
­Environment, Management of Care: Cognitive Level –
22. 1. The nurse should assess the client before
Synthesis: Concept – Perioperative.
­notifying the surgeon the client felt or heard
a “pop” and “something opening.” 25. 1. The client should check the temperature
2. The surgery department may or may not twice a day.
need to be notified. The incision should be 2. It is not realistic to expect the client to
assessed. ­experience no pain after surgery.
3. The nurse should assess the surgical site 3. This statement about taking all the
and, if the site has eviscerated, cover the ­antibiotics ordered indicates the teaching
opening with a sterile dressing moistened is effective.
with sterile 0.9% saline. This will pre- 4. Clients may shower after surgery, but not
vent the tissues from becoming dry and taking a tub bath for three (3) months after
infected. surgery is too long a time.
4. The nurse should not dismiss any complaint Content – Surgical: Integrated Nursing Process –
from a client without further assessment. Evaluation: Client Needs – Physiological Integrity,
­Physiological Adaptation: Cognitive Level – Synthesis:
Concept – Perioperative.

16_Colgrove_Ch16.indd 641 02/06/16 7:05 AM


642 M ed -S urg S uccess

26. In order of priority: 1, 3, 4, 5, 2. 5. The nurse should administer any IV


1. The nurse should increase the IV rate to antibiotics ordered after addressing ­
maintain the circulatory system function hypovolemia. The nurse will need this
until further orders can be obtained. information when reporting to the
3. The foot of the bed should be elevated to health-care provider.
help treat shock, the symptoms of which 2. The health-care provider should be
include elevated pulse and decreased ­notified when the nurse has the needed
blood pressure. Those signs and an el- information.
evated temperature indicate an infection Content – Surgical: Integrated Nursing Process –
may be present and the client could be ­Implementation: Client Needs – Safe Effective Care
developing septicemia. Environment, Management of Care: Cognitive Level –
4. The dressing should be assessed to deter- Analysis: Concept – Perioperative.
mine if bleeding is occurring.

16_Colgrove_Ch16.indd 642 02/06/16 7:05 AM


Cultural and Spiritual
Nursing and Alternative
Health Care
Remember that our nation’s first great leaders were also great
scholars.
17
—John F. Kennedy

The use of alternative therapies to treat specific disorders and to promote health and
well-being has increased in recent years. Many people use herbs, massage therapy, aroma-
therapy, rolfing, guided imagery, yoga, and numerous other techniques either alone or in
addition to the techniques and drugs used by conventional medicine. The nurse should be
aware of these practices and of the various health-related beliefs and traditions that people
of different cultures have.

KEYWORDS ABBREVIATIONS
Ayurveda Acquired immunodeficiency syndrome (AIDS)
Curandero Activities of daily living (ADLs)
Mezuzah Blood pressure (BP)
Nattuvidhyar Complementary and alternative medicine (CAM)
Rolfing Health-care provider (HCP)
Tallis International normalized ratio (INR)
Yom Kippur National Center for Complementary and Alternative Medicine (NCCAM)
Nonsteroidal anti-inflammatory drug (NSAID)
Unlicensed assistive personnel (UAP)

643

17_Colgrove_Ch17.indd 643 02/06/16 7:06 AM


644 M ed -S urg S uccess

PRACTICE QUESTIONS
1. The nurse is admitting the client to a medical 5. Which intervention should the nurse implement
unit. Which questions should the nurse when discussing health promotion activities for
specifically ask the client about the current use of a client with a German heritage who leads a
medications? Select all that apply. sedentary lifestyle?
1. “Have you used or do you currently use any 1. Refer the client to a support group for weight
type of herb?” loss.
2. “What over-the-counter medications do you 2. Teach the client to never drink alcoholic
take?” beverages.
3. “Are you allergic to any medications or foods?” 3. Help the client to identify a routine exercise
4. “When is the last time you took any program.
medication?” 4. Assist the client to make a list of foods which
5. “Do you take any prescription medications?” must be avoided.
2. The client who has had a cold for the last week 6. The elderly female client of Mexican heritage is
is complaining of congestion and a stuffy nose. upset and tells the nurse the unlicensed assistive
The client has been using medicated nasal spray personnel (UAP) complimented her grandchild’s
several times a day. Which information should the hair. Which intervention should the nurse
nurse teach the client about frequent use of nasal implement?
spray? 1. Ask the client why she is upset about the UAP’s
1. Nasal sprays are safe because they are available saying the child’s hair is pretty.
over the counter. 2. Take no action because the unlicensed assistive
2. Nasal sprays such as Afrin cause rebound personnel was being friendly.
congestion when used frequently. 3. Notify the psychologist of the client’s response
3. Nasal sprays have no adverse reactions, unlike to the UAP’s compliment.
over-the-counter oral decongestants. 4. Explain to the UAP about the cultural belief
4. Nasal sprays must be alternated to help prevent regarding the “evil eye.”
developing a tolerance to the spray.
7. The home health-care nurse is visiting an elderly
3. The client using contraceptive foam for birth African American female client who is talking
control has not had her menses for the last two loudly. The client weighs 102 kg, is 54 tall, and
(2) months. Which intervention should the nurse has a BP of 154/98. The client lives with her
implement first? daughter, son-in-law, and two grandchildren.
1. Perform a vaginal examination. Which intervention should the nursing implement?
2. Check the urine for glucose and protein. 1. Maintain extended direct eye contact during
3. Obtain vital signs to determine a baseline. the interview.
4. Request a pregnancy test for this client. 2. Address what is assumed to be the client’s anger
because she is talking loudly.
4. The female client with a cold is prescribed
3. Discuss the client’s care with the daughter and
warfarin (Coumadin), an anticoagulant, for
son-in-law.
chronic atrial fibrillation. The client calls the
4. Discuss a weight-loss program for the client.
clinic and tells the nurse she is bleeding and
bruising more than normal. Which information 8. The home health nurse is making the initial
indicates a need for further teaching? visit to a 42-year-old male client with terminal
1. The client reports taking echinacea, an herb. cancer. The client is first-generation American-
2. The client had an INR drawn last month. Vietnamese and lives with his parents, wife, and
3. The client uses acetaminophen, a nonopioid three children. Which behavior would best
analgesic, for pain. promote a therapeutic nurse–client relationship?
4. The client reads labels on packaged foods. 1. Use questions which require “yes” and “no”
answers when possible.
2. Touch the client’s head to assess warmth in case
of a fever.
3. Assume the client’s smile means the client
understands the teaching.
4. Initially address conversation to the eldest
family member.

17_Colgrove_Ch17.indd 644 02/06/16 7:06 AM


C hapter 17  C ultural and S piritual N ursing and A lternative H ealth C are 645

9. When interviewing the client of Hindu heritage, 14. Which form of complementary therapy should
which information should the nurse obtain to the nurse encourage as a relaxation technique for
plan culturally sensitive care? the anxious elderly client admitted for surgery?
1. Determine if the client has an advance 1. Meditation.
directive. 2. Deep breathing.
2. Assess the extent of the client’s use of 3. Rolfing.
ayurveda. 4. Scented candle.
3. Request an interpreter to obtain the client’s
15. The nurse is preparing the plan of care for
chief complaints.
the client diagnosed with migraine headaches.
4. Inquire if the client has allergies to any foods
Which information regarding complementary
or medications.
therapies should the nurse include in this plan?
10. The unlicensed assistive personnel (UAP) 1. Encourage the use of therapeutic touch.
notices a strange amulet pinned to the client’s 2. Purchase a textbook on autogenic therapy.
gown and offers to remove it. The client does 3. Explain how to perform massage therapy.
not want it removed. Which rationale should 4. Discuss the need to avoid aromatic oils.
the nurse give the UAP for allowing the client to
16. The rehabilitation nurse is caring for an elderly
continue to wear the amulet?
client who had a surgical repair of a fractured
1. The client is superstitious and may become
hip. The client states she misses her dog. Which
distressed if it is removed.
intervention should the nurse implement?
2. The amulet is a silly trinket which won’t hurt
1. Put the television on a show with animal
the client in any way.
performers.
3. The client brought the amulet because it gives
2. Arrange for the family to bring the client’s
the client emotional support.
dog for a visit.
4. The amulet is worthless and no one would try
3. Arrange for the client to have a visit from the
to steal it if it stays on the gown.
pet therapy dog.
11. After evaluating the meal tray of a Jewish client, 4. Share the client’s feelings are completely
the nurse notices the client ate none of the meal. understood by the nurse.
Which intervention should the nurse implement
17. The nurse is caring for a client in the
first?
rehabilitation unit who is hearing impaired, is
1. Request the client’s family bring meals the
anxious, and has elevated blood pressure. The
client can eat.
client enjoys pet therapy but is allergic to dogs
2. Contact the dietary department and request a
and cats. Which intervention should the nurse
kosher meal.
implement?
3. Notify the local rabbi to request meals be
1. Administer antihistamine medication prior to
provided for the client.
pet therapy.
4. Determine why the client is not eating any of
2. Spend extra time visiting at the client’s
the meal.
bedside.
12. The client approaches the nurse about a weight- 3. Arrange for a volunteer to bring the client a
loss program. Which complementary therapy radio.
should the nurse suggest to assist the client’s 4. Provide a colorful fish in a small bowl for the
plan? client.
1. Music therapy.
18. The nurse arranges for a dance movement
2. Hypnotherapy.
therapist to lead a group session for clients with
3. Hatha yoga.
osteoarthritis. Which statement best describes
4. Alexander technique.
the rationale for this intervention?
13. The nurse is incorporating complementary 1. Participants in dance movement form a
therapies in the routines of residents of a long- unique bond while moving in unison.
term care facility. Which information should the 2. Dancing will help increase the amount of
nurse consider when matching the clients with synovial fluid in the affected joints.
the therapy? Select all that apply. 3. Dance therapy helps decrease the client’s pain
1. The client’s preferences. in inflamed joints.
2. The client’s likes and dislikes. 4. Dancing causes the release of endorphins and
3. The benefits obtained from the therapy. helps the clients deal with pain.
4. The significant other’s concerns.
5. The client’s ability to perform therapies.

17_Colgrove_Ch17.indd 645 02/06/16 7:06 AM


646 M ed -S urg S uccess

19. The nurse is providing preoperative teaching 23. Which expected outcome should be included
about pain management techniques for the client in the plan of care for a postoperative client
having surgery. The client has a history of drug practicing guided imagery?
abuse. What should the nurse include in this 1. The client will have a decreased white blood
client’s plan of care? cell count.
1. The nurse should know these clients always 2. The client will report a decrease in pain using
require too much nursing care. the pain scale.
2. The client should select two (2) alternate 3. The client will have a urine output of 360 mL
therapies to replace medications. each shift.
3. The client should receive complementary 4. The client will have no drainage on the
therapies in addition to medications. surgical dressing.
4. The nurse should plan to administer a double
24. The nurse is teaching the client how to use
dose of medication to this client.
guided imagery. Which information would
20. Which intervention should the circulating nurse indicate the teaching has been effective?
implement for the preoperative elderly client 1. The client closes both eyes and makes a one-
who is anxious and diaphoretic? syllable sounding chant.
1. Play soft, slow-beat music in the operating 2. The client rubs the right arm in firm stroking
room. motions.
2. Encourage the client to take rapid, shallow 3. The client selects meaningful music which has
breaths. 50 beats per minute.
3. Continue to talk to the client to promote 4. The client visualizes a scene from a favorite
relaxation. place.
4. Open the door to the operating suite to
25. The client is complaining of nonspecific body
increase the airflow.
aches, congestion, and coughing. The client’s
21. The unlicensed assistive personnel (UAP) is blood pressure is elevated. Which intervention
bathing a client who is comatose and the radio should the nurse implement first?
is playing current rock music. The client has 1. Instruct the client to decrease salt in the diet.
tachycardia and elevated blood pressure. Which 2. Notify the HCP to request an
intervention should the nurse implement? antihypertensive medication.
1. No action by the nurse is indicated in this 3. Determine if the client takes any over-the-
situation. counter medication.
2. Instruct the unlicensed assistive personnel to 4. Discuss the long-term effects of
select baroque music. atherosclerosis and hypertension.
3. Administer medication to decrease the client’s
heart rate and blood pressure.
4. Assist the UAP to turn the client into the
lateral position.
22. The nurse is caring for the surgical client
who is experiencing pain at a “4” on a 1-to-10
pain scale and is receiving pain medication via
patient-controlled analgesia (PCA). Which
intervention should the nurse implement first to
assist the client’s pain management?
1. Notify the health-care provider to increase
the dosage of medication.
2. Inform the client to relax and the medication
will relieve the pain soon.
3. Assist the client to perform guided imagery to
increase relaxation.
4. Instruct the family member to continue to
push the button for the medication.

17_Colgrove_Ch17.indd 646 02/06/16 7:06 AM


C hapter 17  C ultural and S piritual N ursing and A lternative H ealth C are 647

CONCEPTS
The concepts covered in this chapter focus on cultural diversity and safety as well as alternative medicine.
­Exemplars that are covered are cultures, values, health beliefs, religion, spiritual distress, and alternative thera-
pies to western medicine. Interrelated concepts of the nursing process and critical thinking are covered through-
out the questions. The concept of critical thinking is presented in the prioritizing or “first” questions.

26. The nurse is caring for a client who is a recent 3. Ask the client why he thinks the amulet will
immigrant to the country. The family is present protect him during the hospital stay and then
in the room and one member of the family speaks respond accordingly.
English. Which should the nurse implement to 4. Tape the amulet to the client’s skin so that it
complete the admission assessment? will not become separated from him during
1. Ask the family to interpret the questions for surgery.
the client and provide the information.
28. The nurse is admitting a female client to the
2. Use hand gestures with the client to get and
medical-surgical unit when the client tells the
receive the needed information.
nurse she takes many vitamins to keep her
3. Notify the health-care provider that only
healthy. Which statement is the nurse’s most
limited information can be obtained about
appropriate response to the information
the client.
provided by the client?
4. Use the medical interpretation phone to ask
1. “Vitamins are just a way that the
the admission questions.
manufacturers bilk people out of money.”
27. The male client is wearing a silver amulet to 2. “You need to make sure that your health-care
protect from evil spirits. Which action should the provider knows which ones you take.”
nurse take when sending the client to surgery? 3. “If you take vitamins then be careful when
1. Tell the client that the amulet will be placed taking too much water-soluble vitamins.”
near him during surgery but cannot be 4. “Most people are deficient in getting enough
allowed to stay on him. vitamins. It is good you take supplements.”
2. Inform the client to send the amulet home
with the family because it is an infection
control risk in the hospital.
29. The nurse is preparing to administer morning medications. Which order would the nurse question?
1. Vitamin C.
2. Tamsulosin.
3. St. John’s wort.
4. Warfarin.

Admitting Diagnosis: Atrial


Client Name: I.M. Client       M/74 fibrillation

Account Number: 78 96 521 Med Rec #: 00 22 335 Allergies: NKDA

Date Medication 2301–0700 0701–1500 1501–2300


Yesterday Vitamin C 500 mg PO daily 0900
Yesterday Tamsulosin (Flomax) 0.4 mg PO twice a 0900 1800
day
Yesterday St. John’s wort 1 tablet daily 0900
Yesterday Warfarin (Coumadin) 1800
5 mg PO daily
Signature of Nurse: Day Nurse RN/DN

17_Colgrove_Ch17.indd 647 02/06/16 7:06 AM


648 M ed -S urg S uccess

30. The male client tells the nurse that he has been 33. The nurse is preparing to teach a class on
using acupuncture to control his chronic back cultural diversity. Which are examples of being
pain. Which is the nurse’s best response? culturally sensitive? Select all that apply.
1. Inform the client that insurance and Medicare 1. The female nurse has a male UAP perform
will not pay for acupuncture treatments. postmortem care for a Muslim client.
2. Ask the client to describe the pain before and 2. The nurse compliments a Hispanic mother’s
after the acupuncture treatments. infant but does not touch the infant.
3. Teach the client how to sterilize the needles 3. The nurse allows the undergarments worn by
before using them to relieve the pain. a Mormon client to be worn to radiology.
4. “You try to use guided imagery to control 4. The nurse takes the amulet off the comatose
your pain.” client and locks it up in the safe.
5. The nurse has the Jehovah’s Witness client
31. The client expresses anxiety prior to an
sign a Refusal to Permit Blood form.
upcoming procedure. Which should the nurse
implement first to assist the client? 34. The nurse is preparing to make morning rounds.
1. Call the health-care provider (HCP) for a Which client should the nurse see first?
sedative order. 1. The Muslim woman whose husband makes all
2. Notify the family the client is nervous about decisions about her care.
the procedure. 2. The German client who refuses pain
3. Encourage the client to take slow deep medication after an abdominal surgery.
breaths. 3. The Hispanic man who is complaining of
4. Tell the client not to think about the chest pain after eating a spicy meal.
procedure and it will be over. 4. The South Pacific Islands client who is angry
about the care being provided.
32. The nurse is preparing a female client for
surgery. The client tells the nurse that she is a
Jehovah’s Witness and is afraid that the staff will
administer blood to her during the procedure
while she is under anesthesia. Which is the
nurse’s best response to the client?
1. “I will place a notice on your chart and report
to the surgical team that you will not allow
blood or blood products to be given.”
2. “If you go into surgery then all interventions
there have to be administered in order for you
to have the desired results.”
3. “You do not want a blood transfusion? Tell me
why you would not want something that could
save your life.”
4. “Your health-care provider (HCP) must write
an order that you do not want blood or blood
products to be given.”

17_Colgrove_Ch17.indd 648 02/06/16 7:06 AM


PRACTICE QUESTIONS
ANSWERS AND RATIONALES

1. 1. The nurse should ask specifically about 3. 1. A vaginal examination would be performed
herbs because many clients do not con- ­after the pregnancy test result has been
sider them drugs and will not volunteer the obtained.
information. 2. The first intervention is to determine if the
2. The nurse should ask about the use of client is pregnant.
over-the-counter medications because 3. Baseline vital signs would be needed if the
many clients believe health-care provid- ­client were pregnant.
ers are only concerned about prescribed 4. The first intervention should be determin-
medications. ing if the client is pregnant. The results
3. The nurse should assess the client for any of a pregnancy test will determine which
previous reaction to medications. ­interventions should be implemented next.
4. The nurse should be aware of when the TEST-TAKING HINT: The test taker must key in
last medication was taken to determine the on the word “first,” which indicates all four (4)
­effectiveness of the medication and to en- interventions are appropriate but only one (1)
sure any further medication will not have should be implemented first.
an untoward effect. Content – Medical: Integrated Nursing Process –
5. The health-care team must be aware of all ­Implementation: Client Needs – Safe Effective Care
prescriptions so the medications may be ­Environment, Management of Care: Cognitive Level –
continued during hospitalization or discon- ­Application: Concept – Nursing Roles.
tinued if necessary.
4. 1. Echinacea combines with warfarin
TEST-TAKING HINT: This is an example of an
(­Coumadin) to increase bleeding time.
alternative-type question. There could be more It does not, however, alter the INR.
than one (1) correct answer. The test taker 2. Clients taking warfarin (Coumadin) should
should key into the descriptive words, such as have their INR tested routinely.
­“specifically,” in the stem of the question. 3. Clients taking anticoagulants should use
Content – Medical: Integrated Nursing Process – ­acetaminophen (Tylenol) for pain, rather than
­Assessment: Client Needs – Safe Effective Care Environment, aspirin or NSAIDs because these can cause
Management of Care: Cognitive Level – Analysis:
bleeding.
Concept – Medication.
4. Prepared foods can interact with medications
2. 1. All medications have side effects, even the and medical treatments, but not many foods
drugs which are available over the counter. cause bleeding and bruising.
2. Medicated nasal sprays, such as Afrin nasal TEST-TAKING HINT: Herbs often interfere
spray, cause the arterioles to constrict, re- with over-the-counter medications and pre-
sulting in increased congestion after several scription medications. Therefore, option “1”
days of use. would be an appropriate choice for the cor-
3. Medicated nasal sprays, as well as oral decon- rect ­answer if the test taker had no idea of the
gestants, have adverse effects. ­correct answer to this question.
4. Alternating several nasal sprays will not pre- Content – Medical: Integrated Nursing Process –
vent the client from developing a tolerance to ­Evaluation: Client Needs – Physiological Integrity,
the sprays. ­Pharmacological and Parenteral Therapies: Cognitive
TEST-TAKING HINT: If the test taker has no Level – Synthesis: Concept – Medication.
idea of the correct answer, option “2” has the
word “congestion” in the answer and the word
“congestion” is in the stem of the question.
This would be an appropriate option to select
as the correct answer.
Content – Medical: Integrated Nursing Process – ­
Planning: Client Needs – Physiological Integrity,
­Pharmacological and Parenteral Therapies: Cognitive
Level – Synthesis: Concept – Medication.

649

17_Colgrove_Ch17.indd 649 02/06/16 7:06 AM


650 M ed -S urg S uccess

5. 1. People with German heritage tend to be social, 7. 1. The nurse should realize maintaining direct
but nothing in the stem indicates the client is eye contact with some African American cli-
overweight. ents can be interpreted as aggressive behavior.
2. Unless contraindicated, a more realistic goal 2. Loud expression of needs does not mean the
would be to restrict the number of alcoholic client is angry.
beverages to two (2) per day. Beer is a frequent 3. The nurse should discuss the care with the cli-
beverage on many German tables. ent. Discussing her care with family members
3. All clients would benefit from a routine is a violation of HIPAA, and just because the
daily exercise program, which promotes client lives in their home, this does not mean
health, especially for those with a sedentary they are the client’s guardians.
lifestyle. 4. The nurse should discuss the importance
4. There is nothing in the stem indicating the of the management of ideal body weight
client has a specific disease process which because the client is overweight and
­warrants restricting foods. hypertensive.
TEST-TAKING HINT: The adjective “sedentary” TEST-TAKING HINT: The test taker must realize
is the key to answering this question correctly. 102 kg is 224 pounds, which is overweight for
The test taker should not become distracted a 5’4” woman. The test taker should not auto-
by excess verbiage in the stem. In option “2,” matically select option “2” because the word
the word “never” is an absolute term and “loudly” is in both the stem and the answer
makes this option incorrect. option. Speaking loudly could indicate a hear-
Content – Medical: Integrated Nursing Process – Plan- ing impairment or her normal speech.
ning: Client Needs – Health Promotion and Maintenance: Content – Medical: Integrated Nursing Process –
Cognitive Level – Synthesis: Concept – Nursing Roles. ­Implementation: Client Needs – Psychosocial Integrity:
Cognitive Level – Application: Concept – Promoting Health.
6. 1. The nurse should not ask the client “why”; the
client has a right to her feelings. 8. 1. Clients may use “yes” to answer questions to
2. An action should be implemented because the avoid conflict because of a desire to please the
client is upset and the client’s emotional state nurse.
should be addressed. 2. The head is considered sacred and should not
3. There is no reason for the nurse to refer the be touched. If it is medically necessary to touch
client to a psychologist. the head, the nurse should ask permission.
4. The UAP should be informed about “the 3. The client may smile for various reasons. In
evil eye” so the situation can be resolved. the Vietnamese culture, expression of emotions
By the UAP complimenting the child, the is viewed as a sign of weakness, and a stoic
grandmother might be afraid the child smile may be used to disguise negative emo-
will become ill. The situation should be tions. The nurse should not interpret a smile as
addressed, and the client may desire to understanding.
­perform a ritual to remove the “evil eye.” 4. The nurse should talk to the eldest family
TEST-TAKING HINT: The term “elderly” should member first. This is culturally acceptable
inform the test taker the question is age spe- behavior and will help establish a posi-
cific, and the word “Mexican” indicates the tive relationship with the client. However,
question is probably addressing a cultural is- addressing the conversation to the elder
sue. In answer options “1” and “2,” there is no family member does not mean the nurse
action addressing the client’s emotional state. discusses the client’s health without the
­client’s permission.
Content – Medical: Integrated Nursing Process –
­Implementation: Client Needs – Psychosocial Integrity: TEST-TAKING HINT: The question is asking for
Cognitive Level – Application: Concept – Diversity. an intervention which will help establish a
rapport with the client, and cultural influences
must be addressed, especially in the home.
Option “1” is assuming the client has de-
creased cognitive ability, which is information
not provided in the stem.

17_Colgrove_Ch17.indd 650 02/06/16 7:06 AM


C hapter 17  C ultural and S piritual N ursing and A lternative H ealth C are 651

Content – Medical: Integrated Nursing Process – 11. 1. The nurse could request the family bring
­Implementation: Client Needs – Psychosocial Integrity: meals, but this is not the first intervention.
­Cognitive Level – Analysis: Concept – Diversity. 2. The nurse should notify the dietary depart-
9. 1. An advance directive would not be informa- ment once it is determined why the client is
tion which would help provide culturally not eating the meals provided.
­sensitive nursing care. 3. Many local Jewish communities will provide
2. Ayurveda is the traditional health care of kosher meals for clients in the hospital if the
India, and the nurse should determine dietary department is unable to provide them,
if the client practices ayurveda or uses a but this is not the first intervention.
faith healer (nattuvidhyar). Many clients 4. The nurse should first assess and deter-
self-medicate with medications brought mine why the client is not eating. It could
from India. be because of the illness, medication, or
3. An interpreter would not help the nurse cultural beliefs.
provide culturally sensitive care but would TEST-TAKING HINT: Religious practices should
help the nurse understand what the client is be considered when clients are in the hos-
saying. pital. However, the nurse must first assess
4. Allergies to medications and environmental the situation to determine why the client is
elements are assessed in the initial interviews not eating. Remember, the first step of the
of all clients and would not reflect cultural ­nursing process is assessment.
aspects of care. Content – Medical: Integrated Nursing Process –
TEST-TAKING HINT: This question is request- ­Implementation: Client Needs – Physiological Integrity,
ing specific information about the culture Basic Care and Comfort: Cognitive Level – Application:
of the Hindu heritage. Options “1” and “4” Concept – Assessment.
are information the nurse would require of 12. 1. Music therapy has been used in pain manage-
any client and therefore could be eliminated ment and relaxation, but it is not used for
because the stem is asking about culturally weight loss.
sensitive care. 2. Hypnotherapy, or self-hypnosis, has been
Content – Medical: Integrated Nursing Process – used successfully in weight-loss programs.
­Assessment: Client Needs – Psychosocial Integrity: 3. There are several types of yoga exercises
­Cognitive Level – Analysis: Concept – Diversity. which increase relaxation and promote health,
10. 1. The nurse should not refer to the client’s but they do not assist with weight loss.
­beliefs in a negative manner. 4. The Alexander technique focuses on assisting
2. This statement is belittling to the client’s clients become aware of movements and the
­beliefs and would interfere with the nurse– relationship with health and performance, but
client relationship. it has not been used for weight loss.
3. Good luck charms, amulets, or medals TEST-TAKING HINT: The test taker should
provide clients with emotional support. be familiar with current treatments. Cur-
Folk remedies which are not harmful rent treatment options being advertised for
should be allowed while caring for the weight loss are hypnosis and self-hypnosis.
client. Content – Medical: Integrated Nursing Process –
4. Personal belongings are usually sent home to ­Implementation: Client Needs – Psychosocial Integrity:
prevent the loss of the items, but this state- ­Cognitive Level – Application: Concept – Health Care
ment would be false because the client values System.
the amulet.
TEST-TAKING HINT: A basic concept in nurs-
ing is the nurse should always try to support
the client’s cultural beliefs if they do not
harm the client or interfere with the medical
treatment.
Content – Medical: Integrated Nursing Process –
­Implementation: Client Needs – Psychosocial Integrity:
Cognitive Level – Application: Concept – Diversity.

17_Colgrove_Ch17.indd 651 02/06/16 7:06 AM


652 M ed -S urg S uccess

13. 1. Selection of various therapies offers the 15. 1. Therapeutic touch is used in the treat-
client the ability to have some control ment of migraine headaches by some cli-
over his or her care and may help maintain ents. The practitioner uses the hands to
independence. direct energy to correct imbalances which
2. Clients can select treatments which are cause the migraine headache.
pleasurable in addition to being effective. 2. Autogenic training is a method using ­mental
3. Many therapies decrease heart rate and relaxation to lead the body to healing.
blood pressure. Therefore, the nurse ­Because it may require several sessions by
should be knowledgeable so referrals can a certified trainer, the client would find it
be effective. ­difficult to learn this technique from a book.
4. The nurse should explain the benefits of the 3. Massage is the manual manipulation of the
therapy to the significant other to alleviate ­client’s tissue, which affects the entire body
concerns, but this would not be considered and produces generalized relaxation and
when the nurse is planning the therapy. a feeling of well-being, but the client can-
5. Many therapies require movement or a not perform massage therapy on himself or
specific cognitive ability to perform. If herself.
the client cannot perform the therapy, the 4. Aromatherapy has been linked to the relief
­client can become frustrated. of the symptoms of migraine headaches by
TEST-TAKING HINT: This is an example of an using certain essential oils such as rosemary,
­alternative-type question. The test taker chamomile, and lavender.
may have to select more than one (1) correct TEST-TAKING HINT: The test taker should be
answer. aware the client will be unable to perform
Content – Medical: Integrated Nursing Process – therapies on himself or herself. The test
­Planning: Client Needs – Safe Effective Care Environment, taker should be cautious when selecting an
Basic Care and Comfort: Cognitive Level – Synthesis: answer option which encourages the client to
­Concept – Nursing Roles. purchase a book explaining the therapy.
14. 1. Meditation takes practice and the ability to Content – Medical: Integrated Nursing Process – ­
focus. It is not the best way to help an elderly Planning: Client Needs – Physiological Integrity,
­Physiological Adaptation: Cognitive Level – Synthesis:
anxious client manage stress.
Concept – Comfort.
2. Deep breathing begins to relax the
muscles in the body, and this would be a 16. 1. The nurse should not ignore the client’s
method for an elderly client to learn and ­feelings of sadness and loneliness by turning
use to relax. on the television.
3. Rolfing is a holistic method of structural 2. The client is in a rehabilitation setting,
integration. The client would not be able to which often allows pet visits. Therefore,
perform this therapy independently because a the nurse should investigate having the
therapist is needed to perform it. family bring the dog to the facility. This is
4. Scented candles would be unsafe in a hospital being a client advocate.
setting. Aromatherapy using essential oils can 3. Pet therapy is being used to promote relax-
be used, but the client is admitted for surgery ation and a sense of well-being, but the client
and any scent postoperatively can initiate wants to see her own dog.
nausea. 4. The nurse should never claim to have a
TEST-TAKING HINT: The adjective “elderly” ­complete understanding of the client’s feel-
should cause the test taker to select the ings. This would belittle the client’s feelings.
­easiest and simplest way to help the client TEST-TAKING HINT: The test taker must be
relax. The test taker must pay close attention aware of adjectives, and “rehabilitation” in
to adjectives such as “elderly.” the stem indicates the client is not in an
Content – Medical: Integrated Nursing Process – acute care setting. Many rehabilitation units
­Implementation: Client Needs – Psychosocial Integrity: allow family pets to visit.
Cognitive Level – Analysis: Concept – Health Care System. Content – Medical: Integrated Nursing Process –
­Implementation: Client Needs – Psychosocial Integrity:
Cognitive Level – Application: Concept – Nursing Roles.

17_Colgrove_Ch17.indd 652 02/06/16 7:06 AM


C hapter 17  C ultural and S piritual N ursing and A lternative H ealth C are 653

17. 1. The nurse should not administer medication 19. 1. The nurse is responsible for ensuring the
so the client can visit with animals. The nurse ­client receives quality care regardless of
cannot control all of the client’s symptoms the client’s history or the amount of care
and this could present a danger to the client. required.
2. The nurse cannot realistically spend extra 2. The client should be encouraged to use
time visiting with the client when the nurse ­alternative therapies for pain management
has many clients to care for and duties to to supplement medication, not replace
complete. medication.
3. Arranging for the volunteer to bring a radio 3. The nurse should assist the client’s pain
for the client who is hearing impaired would management by using complementary
not be helpful because it would need to be therapies as well as medication but never
played loudly for the client to hear and would in place of pain medication.
interfere with other clients and staff. 4. A double dose of medication requires an
4. For clients who have allergies to pet HCP’s order, and, if administered, careful
­dander, the nurse can provide fish therapy. monitoring is required.
Studies indicate watching fish swim can TEST-TAKING HINT: The descriptive term
decrease blood pressure and promote a “preoperative teaching” can assist the test
calming effect. taker in eliminating options. Option “1”
TEST-TAKING HINT: The descriptive terms is judgmental and should be eliminated.
should guide the test taker to the correct ­Options “2” and “3” are opposites. Most of
­answer. The test taker should eliminate the time one (1) of these options is correct.
option “3” because of the words “hearing Content – Medical: Integrated Nursing Process –
­impaired” in the stem of the question. ­Diagnosis: Client Needs – Safe Effective Care Environment,
Content – Medical: Integrated Nursing Process – Management of Care: Cognitive Level – Analysis:
­Implementation: Client Needs – Psychosocial Integrity: Concept – Comfort.
Cognitive Level – Application: Concept – Nursing Roles.
20. 1. Studies show music can calm the anxious
18. 1. Dance movement therapy incorporates client when the music has a slow beat.
synchronized movement and creates an Faster beats or irregular rhythms tend to
experience shared by all the participants, irritate elderly clients.
which, in turn, contributes to the es- 2. Taking deep breaths may help decrease anxi-
tablishment of a bond between the par- ety. Rapid shallow breathing could cause the
ticipants. The sharing of emotions and client to hyperventilate and lead to dizziness.
experiences strengthens support-group 3. Talking makes some clients more anxious; a
relationships. quiet environment would assist this client to
2. Dance movements do not help increase the relax.
amount of synovial fluid in joints. 4. The increased airflow could increase the
3. Dance movements don’t help decrease pain; risk for client infection and will not help the
in fact, they may increase pain in some ­client’s diaphoresis.
clients. TEST-TAKING HINT: The test taker should
4. Endorphins are released by laughing, not have basic concepts in operating room
dancing, and their release does not necessarily ­nursing, and, often, music is played to help
help with pain. decrease the anxiety of the client as well as
TEST-TAKING HINT: The test taker should ap- the staff. Music can have a calming effect on
ply knowledge of anatomy and physiology to people.
help answer this question. Synovial fluid does Content – Medical: Integrated Nursing Process –
not increase for any reason; therefore, option ­Implementation: Client Needs – Psychosocial Integrity:
“2” should be eliminated as a correct answer. Cognitive Level – Application: Concept – Promoting
Inflamed joints respond to cold or heat; Health.
therefore, option “3” should be eliminated as
a correct answer.
Content – Medical: Integrated Nursing Process –
­Planning: Client Needs – Psychosocial Integrity: Cognitive
Level – ­Comprehension: Concept – Health Care System.

17_Colgrove_Ch17.indd 653 02/06/16 7:06 AM


654 M ed -S urg S uccess

21. 1. This situation indicates the nurse should 4. The expected outcome for the postoperative
take some type of action. The client has an client would be to have a clean, dry dress-
elevated heart rate and blood pressure. ing, but guided imagery would not affect the
2. Baroque music has a slower pace, and dressing.
studies have shown that client’s bodies TEST-TAKING HINT: The expected outcome
­attempt to synchronize with the beat and must reflect the goal of guided imagery.
rhythm of the sounds in the room. The Therefore, the test taker must have knowl-
client’s elevated vital signs could be a edge of why guided imagery is used to be
­result of the rock music in the room. able to answer this question.
3. Medications should be used if other methods Content – Surgical: Integrated Nursing Process –
are not effective. ­Diagnosis: Client Needs – Psychosocial Integrity: Cognitive
4. Changing the client’s position should be done Level – Analysis: Concept – Comfort.
routinely and may decrease discomfort, but
it will not affect the client’s pulse and blood 24. 1. This would be done during meditation.
pressure. 2. Rubbing the arm in firm stroking motions is
performing self-massage.
TEST-TAKING HINT: The test taker should
3. Music therapy is purposefully using music
recognize words in the questions can aid
which has 50 to 60 beats per minute.
in the selection of the correct answer. The
4. The client using guided imagery should
­description of “rock” music in the stem
use as many senses as possible to create
of the question and answer option “2” to
an image of a scene which has meaning
change the music should assist in selecting
for the client.
the correct answer.
TEST-TAKING HINT: Each answer option
Content – Medical: Integrated Nursing Process –
­Implementation: Client Needs – Safe Effective Care ­describes a type of complementary therapy.
Environment, Management of Care: Cognitive Level – The test taker should pay attention to the
Application: Concept – Nursing Roles. words “imagery” and “visualizes.” To visual-
ize is to use the imagination.
22. 1. The pain and evaluation of the medication
Content – Medical: Integrated Nursing Process – ­
should be assessed prior to contacting the Evaluation: ­Client Needs – Psychosocial Integrity:
health-care provider. ­Cognitive Level – Synthesis: Concept – Nursing Roles.
2. Telling the client to relax does not help when
the client is uncomfortable. 25. 1. Instructing on a low-salt diet may be needed,
3. Guided imagery can be used to increase but assessment should be the first interven-
relaxation and decrease the sensation of tion implemented.
pain. 2. Antihypertensive medications may be needed,
4. When clients are using a patient-controlled but assessment should be the first interven-
analgesia pump, family members should be dis- tion implemented.
couraged from administering the medication 3. The nurse should first assess the client to
for the client. The client can ­become overse- determine if the client is taking any over-
dated and develop respiratory depression. the-counter medication which may in-
crease the blood pressure. Decongestants
TEST-TAKING HINT: If the test taker wants to
that contain ephedrine or pseudoephed-
select the answer option which says “notify
rine elevate blood pressure and should be
the health-care provider,” the test taker must
used with caution.
evaluate the other options to make sure an-
4. Teaching about the complications of hyper-
other option does not include assessment
tension would be important but not before
data or an independent nursing intervention.
assessing the client.
Content – Surgical: Integrated Nursing Process –
­Implementation: Client Needs – Psychosocial Integrity: TEST-TAKING HINT: The test taker should use
Cognitive Level – Application: Concept – Comfort. the nursing process when answering ques-
tions which ask the test taker to select the
23. 1. Guided imagery is not used to prevent or first intervention. Assessment is the first
treat infection; therefore, assessing the white ­intervention of the nursing process.
blood cell count would not be appropriate.
Content – Medical: Integrated Nursing Process –
2. The postoperative client uses guided im- ­Implementation: Client Needs – Psychosocial Integrity:
agery to increase relaxation, which helps Cognitive Level – Analysis: Concept – Medication.
decrease the perception of pain.
3. This is a goal for a postoperative client, but it
is not based on the use of guided imagery.

17_Colgrove_Ch17.indd 654 02/06/16 7:06 AM


CONCEPTS ANSWERS AND RATIONALES

26. 1. Family may or may not understand the medi- 28. 1. This may be somewhat true in this country
cal questions. The client may not wish to but many people do benefit from taking a
have a family member be involved in knowing multivitamin daily. “Many” indicates the
the personal information required on the ad- ­client may be overdoing it, but challenging
mission assessment. the client without determining what is being
2. Hand gestures are inadequate for determin- taken can result in the client continuing to
ing the information required on an admission take the vitamins but next time will not tell
assessment. the health-care personnel.
3. The nurse is responsible for obtaining ac- 2. The HCP should be aware of all medica-
curate information. Not speaking a language tions, prescribed and over the counter,
is not a relevant excuse for not attempting to the client takes because they can interact
perform a thorough assessment. with one another and be inappropriate for
4. Hospitals are required by the Joint some diagnoses.
­Commission to have a method of com- 3. Water-soluble vitamins when taken to excess
municating to clients who are from other are eliminated from the body in the urine.
cultures and speak a language other than Fat-soluble vitamins can build up in the body
English. Most facilities pay a fee to have and create a problem.
medical interpreters available 24 hours 4. Most people could get sufficient vitamins
a day. daily if they eat a balanced diet. This state-
TEST-TAKING HINT: This is an example of a ment would be misleading to the client.
nurse being required to know rules and reg- TEST-TAKING HINT: The test taker must read
ulations that govern the practice of nursing. all words in the question and in the options
In this question the test taker could rule out carefully. Would a nurse tell a client that
options “2” and “3” because these options manufacturers “bilk” people? Basic guide-
violate basic rules of nursing practice. lines for discharge instructions encourage
Content – Medical: Integrated Nursing Process – ­ clients to make all HCPs aware of medica-
Assessment: Client Needs – Safe Effective Care tions being taken.
­Environment, Management of Care: Cognitive Level – Content – Medical: Integrated Nursing Process – ­
Analysis: Concept – Diversity. Assessment: Client Needs – Safe Effective Care
­Environment, Management of Care: Cognitive Level –
27. 1. The amulet is metal and cannot stay on
Analysis: Concept – Diversity/Alternative Medicine.
the client during surgery because the
cautery machine could create an electrical 29. 1. Vitamin C 500 mg is not an excessive dose
burn for the client. It should be allowed and the MAR does not indicate a problem
near the client and replaced on the client that the client might expect in taking the
as soon as it is safe to do so. vitamin.
2. The amulet can stay with the client and in the 2. Tamsulosin is a medication prescribed for a
room. Requesting the client to send it home client with benign prostatic hypertrophy. A
is not sensitive to his beliefs. 74-year-old man could be expected to have
3. The client believes it helps him; he does not mild to moderate BPH.
owe the nurse an explanation of his beliefs. 3. St. John’s wort is commonly taken by
4. Taping the amulet to the client’s skin could clients for depression. It has significant
result in an electrical burn during surgery. interactions with various prescription
TEST-TAKING HINT: The test taker must medications, including Coumadin. It will
­understand the equipment used in a hospital decrease the effectiveness of Coumadin.
and make adjustments to be culturally sensi- The nurse would question this m ­ edication
tive but maintain safety for the client. while the client is on Coumadin.
4. Coumadin is prescribed to prevent clot
Content – Medical: Integrated Nursing Process –
­Implementation: Client Needs – Safe Effective Care ­formation in clients with atrial fibrillation.
­Environment, Management of Care: Cognitive Level –
Analysis: Concept – Diversity/Safety.

655

17_Colgrove_Ch17.indd 655 02/06/16 7:06 AM


656 M ed -S urg S uccess

TEST-TAKING HINT: This is an example of an Content – Medical: Integrated Nursing Process –


alternative-type question. The test taker ­Implementation: Client Needs – Psychological Integrity:
must read all of the MAR to determine if the Cognitive Level – Analysis: Concept – Alternative Therapy.
medications are appropriate. Next the nurse 32. 1. The nurse should assure the client that
must realize that even though St. John’s wort her wishes will be respected and then
is an over-the-counter preparation it can still proceed to make sure that the informa-
impact other medications. There could be tion is provided to all other health-care
more than one (1) correct answer. The test personnel.
taker should key into the descriptive words, 2. An adult client has the legal right to make
such as “specifically,” in the stem of the decisions of this nature and the health-care
question. team is required to abide by those decisions.
Content – Medical: Integrated Nursing Process – 3. This is a poor attempt at a therapeutic
­Implementation: Client Needs – Pharmacological Care conversation; the client has given factual
Environment, Management of Care: Cognitive Level – information. “Why” is never therapeutic. A
Analysis: Concept – Safety. therapeutic conversation is not required in
30. 1. The client has been using the treatments and this question. The nurse can address the issue.
will know how to pay for the treatments. 4. The HCP would have to write an order to
2. This is an assessment question to deter- administer blood and blood products. An
mine the success of the treatment. order to refuse the blood does not have to be
3. The client would not sterilize the needles; written.
this is the responsibility of the acupuncturist. TEST-TAKING HINT: This is an example
4. The client is not asking for a different of knowing differences in cultures and
method to control the pain. religions.
TEST-TAKING HINT: The test taker could use Content – Medical: Integrated Nursing Process – ­
the nursing process to answer this question; Assessment: Client Needs – Safe Effective Care
assessment is the first step in the nursing ­Environment, Management of Care: Cognitive Level –
process. Analysis: Concept – Diversity.
Content – Medical: Integrated Nursing Process – ­ 33. 1. Persons of the opposite gender are not
Assessment: Client Needs – Safe Effective Care allowed to touch a deceased body in tradi-
­Environment, Management of Care: Cognitive Level – tional Muslim teachings.
Analysis: Concept – Diversity/Alternative Therapy. 2. In Hispanic cultures a person praising an
31. 1. The HCP may need to be notified of the infant should touch the baby to prevent
anxiety but not for a sedative order. This is ­giving the child the evil eye. Not touching
not assisting the client, but the first action is the child could be interpreted as trying to
to allow the client to cope with the situation. harm the infant.
The nurse should allow the client to verbal- 3. Some Mormon clients wear an undergar-
ize the anxious feelings and assist the client to ment as part of their religious beliefs.
cope with the situation. Working with the belief shows cultural
2. The client may ask the nurse to notify the sensitivity.
family but the nurse should not automatically 4. In most cases locking up valuables is a good
assume the client has a relationship with the thing but an amulet to protect from “evil”
family which would decrease the anxiety. spirits should be left on the client.
3. During anxiety situations a client’s breath- 5. Jehovah’s Witnesses believe that it is
ing increases in a “fight or flight” reaction. wrong to accept blood or blood products;
The nurse can assist the client to remain this indicates the nurse is being sensitive
calm and cope with the anxiety by coach- to their beliefs.
ing the client in deep-breathing exercises. TEST-TAKING HINT: This is an example of
4. The client cannot automatically turn off anx- an ­­alternative-type question. There will be
ious thoughts by telling himself/herself not to more than one (1) correct answer. The test
think about something. taker should read each option as a true/false
TEST-TAKING HINT: This is an example of question.
an alternative-therapy question. The nurse Content – Medical: Integrated Nursing Process –
should be open to nonpharmacological inter- ­Implementation: Client Needs – Safe Effective Care
ventions to assist the client to cope. This is ­Environment, Management of Care: Cognitive Level –
a temporary situation at this point, worrying Analysis: Concept – Diversity.
over a procedure.

17_Colgrove_Ch17.indd 656 02/06/16 7:06 AM


C hapter 17  C ultural and S piritual N ursing and A lternative H ealth C are 657

34. 1. This is an expected situation in the Muslim TEST-TAKING HINT: This is an example of a
culture. The nurse does not have to see this priority question. A physiological need is
client first. ­priority over psychosocial needs. If every
2. Many Germanic peoples are stoic when it other option is within normal limits or
comes to receiving pain medication. The expected for the disease process and not life
nurse does not have to see this client first. threatening, the psychosocial need can be
3. This client may have indigestion but he addressed first.
may be having cardiac pain. The nurse Content – Medical: Integrated Nursing Process – ­
should assess the pain and determine if it Assessment: Client Needs – Safe Effective Care
is cardiac related. Chest pain is considered ­Environment, Management of Care: Cognitive Level –
cardiac until proven to be from another Analysis: Concept – Critical Thinking.
source.
4. Being angry is a psychosocial need; a physi-
ological need is priority over a psychosocial
need.

17_Colgrove_Ch17.indd 657 02/06/16 7:06 AM


658 M ed -S urg S uccess

CULTURAL NURSING AND ALTERNATIVE


HEALTH CARE COMPREHENSIVE EXAMINATION

1. The client diagnosed with rheumatoid arthritis 5. The Mexican client requests a curandero to come
asks the nurse, “Is it all right for me to wear to the hospital to visit. Which statement explains
this copper bracelet to help cure my arthritis?” the role of the curandero in the Mexican culture?
Which statement by the nurse would be most 1. He is a person who treats muscle and joint
appropriate? problems using massage and manipulation.
1. “How do you know the copper bracelet will 2. She is a person who uses herbs, teas, and roots
help cure your arthritis?” to prevent or treat illnesses.
2. “The bracelet will not cure your arthritis, but if 3. She is a spiritualist who treats the person’s
it helps the pain, wear it.” condition caused by witchcraft.
3. “You should talk to your health-care provider 4. He is a person who receives his gift from God
before wearing the copper bracelet.” and treats traditional illnesses.
4. “I recommend not wearing the bracelet and
6. The home health nurse is visiting a Jewish client
taking your prescribed medications.”
who has a small container on the doorpost of the
2. The client asks the nurse about using herbs and house. Which term best describes this container?
special teas to decrease blood pressure. Which 1. A tallis.
question is most important for the nurse to 2. A mezuzah.
consider before addressing the client’s concerns? 3. A synagogue.
1. “How expensive is this treatment?” 4. Yom Kippur.
2. “Is this practice safe for the client?”
7. The home health nurse is assessing a client
3. “Do I have the knowledge to answer this
diagnosed with congestive heart failure who
question?”
has a slice of raw potato on a carbuncle on
4. “Is this within the scope of the state’s Nurse
the abdomen. Which action should the nurse
Practice Act?”
implement first?
3. The client is using acupuncture to help relieve 1. Explain the client should not put raw potatoes
severe back pain. Which client problem would the on the boil.
home health nurse identify in the plan of care? 2. Ask the client to explain why there is a raw
1. The likelihood of paralysis. potato on the boil.
2. The potential for infection. 3. Cleanse the wound and determine if there is an
3. The possibility of contracting AIDS. infection.
4. No resolution of the pain. 4. Leave the raw potato on the boil and take no
further action.
4. The male Navajo client comes to the clinic
complaining of chest pain and has a pouch filled 8. The female client is complaining of dyspepsia,
with objects around his neck. Which statement insomnia, and upper respiratory infection
best supports the nurse allowing the client to wear symptoms and has an elevated blood pressure.
the pouch? The client tells the nurse she recently moved to
1. This is a cultural practice shared by many the area to care for an ill parent. Which statement
Navajo clients, and the nurse should not best explains the client’s clinical manifestations?
remove it unless it interferes with the client’s 1. The client has a psychosomatic illness.
care. 2. The client’s immune system is altered by stress.
2. The client may get very upset and angry with 3. The client’s symptoms are caused by gastric
the nurse if the pouch is removed and lose faith reflux.
in the health-care system. 4. The client has essential hypertension.
3. The nurse should never ask any client to
remove any type of cultural objects from the
body.
4. This is a preventive measure prescribed by the
medicine man which wards off the evil of a
witch.

17_Colgrove_Ch17.indd 658 02/06/16 7:06 AM


C hapter 17  C ultural and S piritual N ursing and A lternative H ealth C are 659

9. The nurse assessing an elderly client diagnosed 13. The unlicensed assistive personnel asked the
with chronic obstructive pulmonary disease wound care nurse why a client has cabbage
notes purple, round, nontender areas on the leaves on the wound. Which statement would be
client’s back. Which question should the nurse the best response by the nurse?
ask the client? 1. “Cabbage leaves have antibacterial and anti-
1. “Have you had cupping performed?” inflammatory properties.”
2. “Do you always bruise this easily?” 2. “Some people are ignorant and will believe or
3. “Did someone living with you hurt you?” try anything to get better.”
4. “What have you done to cause these?” 3. “This is the placebo effect. If people believe
something will work, it will.”
10. The nurse is preparing a teaching plan for a
4. “The cabbage leaves will keep the wound
client diagnosed with chronic stress. Which
moist and clean until it heals.”
information should the nurse include in the
teaching care plan? Select all that apply. 14. The nurse at the Family Planning Clinic is
1. Discuss the relationship between stress and preparing a discussion for students attending
illness with the client. a community college about contraception
2. Define the terms “stress” and “relaxation” in and methods to prevent pregnancy. Which
words the client understands. information should the nurse include in the
3. Explain methods used to decrease stress which presentation?
the nurse has used. 1. Condoms should be used to prevent sexually
4. Refer the client to a practitioner of traditional transmitted diseases.
Chinese medicine. 2. Sexually active females should have vaginal
5. Provide Web site addresses for the client to examinations yearly.
investigate. 3. Hormonal therapy can be used orally,
subcutaneously, or topically.
11. The nurse is assisting the client diagnosed
4. All methods of conception need prescriptions
with chronic pain to identify methods of
from the health-care provider.
complementary and alternative medicine (CAM).
Which would indicate an expected outcome was 15. The occupational health nurse is caring for the
achieved? client with a superficial burn on the arm. The
1. The client will spend 18 out of 24 hours in client states a preference to use holistic natural
bed. medication. Which medicinal plant should the
2. The client will have an enhanced sensation of nurse recommend to the client?
pain. 1. Black cohosh.
3. The client will take his or her medication on a 2. Fennel.
routine basis. 3. Witch hazel.
4. The client will be able perform three (3) 4. Aloe vera.
ADLs without pain.
16. The client has been diagnosed with bronchitis.
12. The nurse and the unlicensed assistive personnel The client asks the nurse, “Can I use eucalyptus
are performing postmortem care on a client. oil in steam and breathe it in deeply?”
Two (2) of the family members are seen laughing Which response by the nurse would be most
about a memory of the client. What explanation appropriate?
of this behavior would be best for the nurse to 1. “I would not recommend using any type of
give the unlicensed assistive personnel? folk medicine.”
1. The two (2) family members probably did not 2. “The steam and the oil may help open up
love the client. your airway.”
2. The behavior is inappropriate for the 3. “This type of treatment could interfere with
situation and inexcusable. your other medications.”
3. The family members are using laughter to 4. “Inhaled eucalyptus oil is not as effective as
cope with the pain. rubbing it on the chest.”
4. This behavior is demonstrated only by
uneducated individuals.

17_Colgrove_Ch17.indd 659 02/06/16 7:06 AM


660 M ed -S urg S uccess

17. The client with a deficiency of the immune 3. “I will use the contraceptive patch because it
system questions the nurse about the use of has fewer side effects.”
ginseng. Which information should the nurse 4. “If the contraceptive ring comes out, I will
teach the client? have to get a new one.”
1. Take the ginseng daily to achieve a
22. Which method of birth control would the nurse
therapeutic level.
recommend for the homeless client who has had
2. Take ginseng with caffeine drinks to enhance
two (2) unplanned pregnancies?
its effects.
1. Prentif cavity-rim cervical caps.
3. Do not take ginseng continuously for more
2. Subdermal levonorgestrel implants.
than two (2) months.
3. Combination oral contraceptives.
4. There are no contraindications for any client
4. Depo-Provera injections intramuscularly.
taking ginseng.
23. Which information would be most important
18. The client with severe itching from an insect
for the public health department nurse to obtain
bite calls the clinic and asks the nurse, “What
before giving the client condoms?
do I need to do for the itching?” Which
1. Ask about the frequency of sexual activity.
intervention should the nurse implement for the
2. Find out the type of condom the client
client problem of alteration in comfort?
prefers.
1. Alternate Tylenol, an analgesic, with aspirin,
3. Determine if the client is allergic to latex.
an antiplatelet, every two (2) hours.
4. Discuss the client’s sexual history.
2. Cover the area with Benadryl gel every six (6)
hours as needed for itching. 24. The nurse and the unlicensed assistive personnel
3. Wash the area with soap and hot water and (UAP) are caring for a client who is anxious.
cover with gauze pads. Which task should the nurse delegate to the
4. Apply 1% hydrocortisone cream, a steroid, UAP?
every two (2) hours. 1. Teach the client how to deep breathe.
2. Give the client a back rub.
19. The client with anemia has been taking iron
3. Listen to the client’s concerns.
supplements. Which data would indicate to the
4. Instruct the client on how to perform guided
nurse the treatment has been effective?
imagery.
1. The client is able to prepare a menu high in
iron. 25. The nurse is teaching the client about relaxation
2. The client is able to perform activities of daily techniques. Which interventions should be
living. included in the teaching? Select all that apply.
3. The client has an increase in black tarry 1. Instruct the client to contract and relax the
stools. muscles.
4. The client has an increase in dystrophy of 2. Explain the importance of playing soothing
nails. music.
3. Use all five (5) senses when using guided
20. The nurse is teaching the elderly client
imagery.
about taking oral iron supplements. Which
4. Discuss the need to eliminate all stress in the
intervention should the nurse include in the
client’s life.
teaching?
5. Insist the client write about his or her feelings
1. Take the iron with an antacid to prevent a
in a journal.
bleeding ulcer.
2. If a dose is missed, take the dose with the next 26. The client is complaining of acute abdominal
dose. pain, and the next pain medication is not due for
3. The client should take the medication two (2) hours. Which interventions should the
between meals. nurse implement? List in order of priority.
4. The client should recline for 30 minutes after 1. Notify the health-care provider to increase
dosage. pain medication.
2. Auscultate for bowel sounds and palpate the
21. The nurse is teaching a 28-year-old client who
abdomen.
was recently prescribed birth control pills.
3. Instruct the client to visualize a pleasant
Which statement by the client indicates teaching
memory.
has been effective?
4. Turn on the radio to soft, easy-listening
1. “I will use another birth control method if I
music.
have to take antibiotics.”
5. Offer the client a therapeutic back massage.
2. “If I miss a pill, I will take it at the end of the
prescription.”

17_Colgrove_Ch17.indd 660 02/06/16 7:06 AM


CULTURAL NURSING AND ALTERNATIVE
HEALTH CARE COMPREHENSIVE EXAMINATION
ANSWERS AND RATIONALES

1. 1. This is challenging the client’s question and 3. AIDS is not a great concern because the health
will not help establish a therapeutic relation- department regulates the centers which per-
ship with the client. form acupuncture and there is no sharing of
2. The nurse should present facts and support the needles.
the client’s alternative health belief unless 4. The pain not being resolved is a concern, but
it hurts the client or makes the disease the potential for infection is priority.
process worse. Content – Medical: Integrated Nursing Process –
3. The nurse does not need to pass the buck to Diagnosis: Client Needs – Physiological Integrity,
the HCP to answer this question. ­Physiological Adaptation: Cognitive Level – Analysis:
4. Alternative medicine is often helpful to the Concept – Comfort.
client, and the nurse should support a client’s 4. 1. Navajo people practice preventive medi-
beliefs as much as possible if it does not hurt cine, and the nurse should support the
the client. client’s culture if it does not interfere with
Content – Medical: Integrated Nursing Process – the medical regimen.
Implementation: Client Needs – Physiological Integrity, 2. Most clients will not get upset and angry and
­Physiological Adaptation: Cognitive Level – Application:
lose faith if the nurse explains the importance
Concept – Nursing Roles.
of removing an object.
2. 1. The nurse can be concerned about the cost of 3. “Never” is an absolute word, and sometimes
the herbs and teas, but it is not the most im- objects must be removed, such as when going
portant question to consider. to surgery.
2. The client’s safety is priority, but if the nurse 4. This is the reason the object is worn, but it
does not have the knowledge, then the nurse is not the best statement to explain why the
cannot answer the question. nurse should not remove the pouch. Even if
3. Teaching about herbs and alternative ther- the nurse does not understand the reason for
apies is not a priority in nursing programs. the object, the nurse should respect and honor
Therefore, the nurse must determine if the practice if possible.
he or she has the knowledge to answer the Content – Medical: Integrated Nursing Process –
client’s question. The nurse should find a ­Implementation: Client Needs – Psychosocial Integrity:
reputable resource before answering this Cognitive Level – Application: Concept – Diversity.
question.
5. 1. This is a Mexican folk practitioner known as a
4. The nurse must always practice within the
sobadore.
scope of nursing, but providing correct and
2. This is a Mexican folk practitioner known as a
factual information is within the scope of the
yerbera or jerbera.
nurse’s practice.
3. This is a Mexican folk practitioner known as
Content – Medical: Integrated Nursing Process – an espiritista.
Implementation: Client Needs – Physiological I­ ntegrity,
4. Curanderos receive their gift from God or
Physiological Adaptation: Cognitive
Level – Analysis: Concept – Nursing Roles.
serve an apprenticeship. Some even pre-
scribe over-the-counter medications. They
3. 1. Acupuncture does not usually cause paralysis usually treat traditional illnesses not caused
because of the size of the needles/wires used. by witchcraft.
2. Acupuncture is a method of producing Content – Medical: Integrated Nursing Process –
analgesia or altering the function of a ­Implementation: Client Needs – Psychosocial Integrity:
system of the body by inserting fine, ­Cognitive Level – Application: Concept – Diversity.
wire-thin needles into the skin at specific
sites on the body. Infection does occur if a
sterile procedure is not used when insert-
ing the needles.

661

17_Colgrove_Ch17.indd 661 02/06/16 7:06 AM


662 M ed -S urg S uccess

6. 1. A tallis (or tallith) is a rectangular prayer shawl 2. This response assumes the areas are bruises
with fringes used during prayer. resulting from injury or illness.
2. A mezuzah is a small container with scrip- 3. This response assumes the areas are bruises
ture inside which is placed on the doorpost resulting from abuse, which could lead to
of the home; some wear the mezuzah as a false accusations and legal issues.
necklace. 4. This response sounds judgmental and blam-
3. A synagogue is a Jewish house of prayer; Jews ing, and the nurse should avoid responses
may pray alone, or they may pray as a group. which damage the client–nurse relationship.
4. Yom Kippur is a high holy day celebrated Content – Medical: Integrated Nursing Process –
in September or early October. On Yom Assessment: Client Needs – Safe Effective Care
Kippur one fasts for a day to cleanse and ­Environment, Management of Care: Cognitive Level –
purify oneself. Analysis: Concept – Nursing Roles.
Content – Medical: Integrated Nursing Process – 10. 1. The client should understand the rela-
­Implementation: Client Needs – Psychosocial Integrity: tionship between stress and illness.
Cognitive Level – Application: Concept – Diversity.
2. For effective teaching, the nurse should
7. 1. The nurse should be sensitive to folk ­medicine, use terms the client understands.
and if it does not hurt the client, the nurse 3. Methods the nurse has experienced
should be respectful and honor the practice. ­personally or has observed to be success-
2. The nurse should not make the client explain ful with other clients can aid the client’s
his or her folk medicine beliefs. selection.
3. The nurse should first assess the carbuncle 4. The nurse should not refer the client to alter-
to determine if the raw potato is making native health-care providers. This is imposing
the carbuncle worse; if it is not, the nurse the nurse’s cultural beliefs onto the client.
should support the client’s folk medicine. 5. The nurse should provide a variety of
4. The nurse should assess the carbuncle before ­referrals and resources to use to help
deciding not to take any further action. ­reduce and control stressful situations.
Content – Medical: Integrated Nursing Process – Content – Medical: Integrated Nursing Process –
­Implementation: Client Needs – Safe Effective Care Environ- ­Planning: Client Needs – Psychosocial Integrity: Cognitive
ment, Management of Care: Cognitive Level – ­Application: Level – Synthesis: Concept – Nursing Roles.
Concept – Nursing Roles.
11. 1. If the client spends more than three-fourths
8. 1. These are not typical signs and symptoms of a of the time in bed, then this would be a low
psychosomatic disorder. quality of life and would not be the expected
2. This would be the best explanation outcome.
because the client has experienced major 2. An expected outcome would be relief of pain
life changes—moving to another area and or enhanced pain management. Enhanced
assuming the role of caregiver to a parent. sensation of pain would be unrelieved pain.
Chronic unrelieved stress has been shown 3. CAM would not include prescription medica-
to decrease immunity, resulting in frequent tion, and taking medication is an interven-
upper respiratory tract infections and tion, not an expected outcome.
increased blood pressure. 4. A client performing activities of daily
3. Gastric reflux has been linked to asthma-type ­living (ADLs) without pain represents an
symptoms but not upper respiratory infections. appropriate expected outcome.
4. These signs/symptoms do not support the di- Content – Medical: Integrated Nursing Process –
agnosis of essential hypertension. Evaluation: Client Needs – Safe Effective Care
Content – Medical: Integrated Nursing Process – ­Environment, Management of Care: Cognitive Level –
Diagnosis: Client Needs – Safe Effective Care Environment, Synthesis: Concept – Nursing Roles.
Management of Care: Cognitive Level – Analysis:
12. 1. This behavior does not indicate lack of love;
Concept – Nursing Roles.
it is a coping mechanism.
9. 1. Cupping is an alternative treatment 2. Family members’ coping mechanisms should
­modality in which heated cups are applied not be criticized by staff.
to the body. They produce round, red or 3. Laughter during times of stress aids in
purple circles which may remain for as long coping with pain and increases relaxation.
as one (1) week. It may be used for clients 4. Laughter is used by many individuals as stress
with respiratory difficulty. relief, regardless of educational levels.

17_Colgrove_Ch17.indd 662 02/06/16 7:06 AM


C hapter 17  C ultural and S piritual N ursing and A lternative H ealth C are 663

Content – Medical: Integrated Nursing Process – 16. 1. The nurse should support the use of folk and
­Implementation: Client Needs – Safe Effective Care home remedies as long as they are not con-
­Environment, Management of Care: Cognitive Level – traindicated by the medical treatment.
­Application: Concept – Grief and Loss. 2. Eucalyptus has been used by clients
13. 1. Cabbage leaves have been shown to have with congestion and as an expectorant;
some antibacterial and anti-inflammatory steam helps open the airway and liquefies
properties. Softened leaves are applied to secretions.
wounds, ulcers, and arthritic joints. 3. This is an inhaled treatment which will pro-
2. This response is belittling the client’s beliefs vide the medication directly to the lungs and
and actions. will limit systemic effects.
3. Studies support the placebo effect, but this 4. The steam provides moisture to the lungs to
response does not answer the UAP’s question liquefy secretions, and the efficacy of rubbing
about the use of cabbage leaves. on the chest is provided by inhalation.
4. The leaves would not be the best way to keep Content – Medical: Integrated Nursing Process –
the wound moist and clean. Implementation: Client Needs – Physiological I­ ntegrity,
Pharmacological and Parenteral Therapies: Cognitive
Content – Medical: Integrated Nursing Process –
Level – Application: Concept – Medication.
­Implementation: Client Needs – Safe Effective Care
­Environment, Management of Care: Cognitive Level – 17. 1. Continuous, daily doses will not increase
Application: Concept – Medication. the effectiveness of the herb and could cause
14. 1. Condoms are helpful in preventing sexu- harm to the client.
ally transmitted diseases and in preventing 2. Ginseng is a stimulant; therefore, additional
pregnancy. stimulants such as caffeine should be avoided.
2. Sexually active females do need vaginal exam- 3. Clients should take ginseng intermit-
inations, but this statement does not address tently for safety. The client should omit
preventing pregnancy. the ­ginseng for two (2) or three (3) weeks
3. Routes of hormonal therapy include birth every two (2) months.
control pills, creams, patches, and injec- 4. There are contraindications to all medica-
tions. All address ways to help prevent tions. Ginseng elevates the blood pressure, so
conception. clients diagnosed with hypertension should
4. All methods of birth control do not require avoid ginseng.
a prescription. Condoms and contracep- Content – Medical: Integrated Nursing Process –
tive creams and ointments are sold over the Planning: Client Needs – Physiological Integrity,
counter. ­Pharmacological and Parenteral Therapies: Cognitive
Level – Synthesis: Concept – Medication.
Content – Medical: Integrated Nursing Process –
Planning: Client Needs – Health Promotion and 18. 1. Alternating Tylenol with aspirin is recom-
­Maintenance: Cognitive Level – Synthesis: Concept – mended for clients with fevers who do not
Nursing Roles. respond to Tylenol alone.
15. 1. Black cohosh is used for gynecological 2. Benadryl gel can be applied to the area
disorders. three (3) to four (4) times a day. The
2. Fennel is used as an expectorant or diuretic. ­medication is absorbed topically. If the
3. Witch hazel has anti-inflammatory proper- medication is applied more often, the
ties, but it is prepared with an alcohol base, ­client can develop systemic effects.
which would cause pain to the client when 3. Washing the area with soap and hot water
applying it to the burned area. will increase the itching.
4. Aloe vera has been used to promote 4. Hydrocortisone cream is applied three (3) to
wound healing. It has some antifungal four (4) times daily, not every two (2) hours.
properties and helps soothe the pain from Content – Medical: Integrated Nursing Process –
a superficial burn. Implementation: Client Needs – Physiological I­ ntegrity,
Pharmacological and Parenteral Therapies: Cognitive
Content – Medical: Integrated Nursing Process –
Level – Application: Concept – Medication.
Implementation: Client Needs – Physiological I­ ntegrity,
Pharmacological and Parenteral Therapies: Cognitive
Level – Application: Concept – Medication.

17_Colgrove_Ch17.indd 663 02/06/16 7:06 AM


664 M ed -S urg S uccess

19. 1. This outcome would be for a knowledge- Content – Medical: Integrated Nursing Process –
deficit problem. ­Evaluation: Client Needs – Physiological Integrity,
2. Clients with anemia have decreased ­Pharmacological and Parenteral Therapies: Cognitive
­energy and are unable to complete activi- Level – Synthesis: Concept – Medication.
ties of daily living. The ability to complete 22. 1. The Prentif cavity-rim cervical cap has a 40%
ADLs indicates the treatment has been failure rate for multiparous women.
effective. 2. Subdermal levonorgestrel implants are an
3. Green-black stools are an expected side ­effect effective method of birth control for up to
of taking iron supplements and can be con- five (5) years.
fused with tarry stools, which indicate blood 3. When taken as directed, oral contraceptives
in the stool. can be effective, but because this client has
4. Dystrophy of fingernails is an indication the had two (2) unplanned pregnancies, compli-
treatment is ineffective. ance should be questioned.
Content – Medical: Integrated Nursing Process – 4. Depo-Provera is one of the most effective
­Assessment: Client Needs – Physiological Integrity, methods of birth control but lasts for only
­Pharmacological and Parenteral Therapies: Cognitive three (3) months; therefore, this should not
Level – Analysis: Concept – Medication. be recommended for this client.
20. 1. Antacids should be taken one (1) hour before Content – Medical: Integrated Nursing Process –
or two (2) hours after taking iron. Implementation: Client Needs – Physiological I­ ntegrity,
2. Clients should not double up on doses be- Pharmacological and Parenteral Therapies: Cognitive
cause that may lead to gastrointestinal upset. Level – Application: Concept – Nursing Roles.
3. Food interferes with the absorption of 23. 1. The frequency of sexual activity would indi-
oral supplements, but it is recommended cate the number of condoms to be distrib-
clients take iron between meals to mini- uted, but it would not be the most important
mize gastrointestinal upset and maximize information.
iron absorption. 2. The public health department provides free
4. The client needs to remain in an upright condoms and is not concerned with the
position for 30 minutes after taking oral iron ­client’s style preference.
supplements to reduce esophageal irritation 3. The most effective condoms are made of
or corrosion. latex, but they should not be used if the
Content – Medical: Integrated Nursing Process – client is allergic to latex. The client should
Planning: Client Needs – Physiological Integrity, be instructed to ask the partner about
­Pharmacological and Parenteral Therapies: Cognitive ­latex allergies.
Level – Synthesis: Concept – Medication. 4. Sexual history should be discussed only if
21. 1. Antibiotics such as ampicillin and tetracy- the client has a sexually transmitted disease
cline increase the elimination of the oral because the public health department must
contraceptive by killing the flora in the notify all partners.
gastrointestinal tract. Clients should use Content – Medical: Integrated Nursing Process –
another method of birth control while ­Assessment: Client Needs – Physiological Integrity,
taking an antibiotic. ­Reduction of Risk Potential: Cognitive Level – Analysis:
2. The pills should be taken at the same time Concept – Nursing Roles.
each day. If the client misses one (1), that pill 24. 1. Deep breathing would aid the client to relax,
should be taken as soon as the client remem- but teaching cannot be delegated.
bers. If the client misses taking the pill two 2. This relaxing intervention can be
(2) days in a row, then two (2) pills should be ­delegated to the unlicensed assistive
taken each day for the next two (2) days. personnel.
3. The transdermal patch has the same side 3. Listening to why the client is anxious is part
­effects as oral birth control pills. The advan- of assessment, which cannot be delegated.
tage of the patch is it does not have to be 4. Guided imagery is a method which assists
taken every day. clients to relax, but teaching it cannot be
4. If the contraceptive ring comes out, it should delegated.
be washed with warm water and r­ eplaced. Content – Medical: Integrated Nursing Process –
One ring can be used for three (3) weeks. Planning: Client Needs – Safe Effective Care E
­ nvironment,
Management of Care: Cognitive Level – Synthesis:
Concept – Nursing Roles.

17_Colgrove_Ch17.indd 664 02/06/16 7:06 AM


C hapter 17  C ultural and S piritual N ursing and A lternative H ealth C are 665

25. 1. Progressive muscle relaxation is the 4. Soft music is a method of distraction


­systematic contracting and relaxing of which can be implemented quickly and
muscles. It helps aid in relaxation when may have a calming effect on the client.
the client feels anxious. 1. The nurse should notify the HCP
2. Soothing music with the rate of 50 to and ­discuss other possible medication
60 beats per minute can slow the client’s regimens which can be provided for
heart rate and relax the client. the client.
3. Guided imagery is helpful to decrease 5. According to the gate control theory of
anxiety. The more senses involved, the pain, flooding the brain with pleasurable
better the results of the guided imagery. sensations will block the transmission
4. Stress is not always harmful, and the absence of pain.
of all stress is death. 3. Clients in acute pain are not receptive to
5. Journaling is useful for some individuals, but being taught; therefore, this intervention
the nurse cannot insist or demand the client should be implemented as soon as the
do anything. pain is tolerable and may be used by the
Content – Medical: Integrated Nursing Process – client for future pain episodes.
­Planning: Client Needs – Psychosocial Integrity: Cognitive Content – Medical: Integrated Nursing Process –
Level – Synthesis: Concept – Nursing Roles. Implementation: Client Needs – Physiological Integrity,
Pharmacological and Parenteral Therapies: C
­ ognitive
26. In order of priority: 2, 4, 1, 5, 3. Level – Analysis: Concept – Comfort.
2. Any pain not relieved with prescribed pain
medication warrants assessment by the
nurse to rule out any complications.

17_Colgrove_Ch17.indd 665 02/06/16 7:06 AM


17_Colgrove_Ch17.indd 666 02/06/16 7:06 AM
End-of-Life
Issues
True friendship can afford true knowledge. It does not depend on
darkness and ignorance.
18
—Henry David Thoreau

End-of-life issues include some of the most sensitive issues a nurse must handle. Many times
a nurse will need to confront his or her own feelings and fears about death to be able to help
his or her clients. This chapter deals with advance directives, death and dying, chronic pain,
ethical/legal issues, and organ/tissue transplants.

ABBREVIATIONS
Advance directive (AD)
American Nurses Association (ANA)
Apical pulse (AP)
Blood pressure (BP)
Blood urea nitrogen (BUN)
Cardiopulmonary resuscitation (CPR)
Chronic obstructive pulmonary disease (COPD)
Do not resuscitate (DNR)
Electroencephalogram (ECG)
Health-care provider (HCP)
Health Insurance Portability and Accountability Act (HIPAA)
Human immunodeficiency virus (HIV)
Human leukocyte antigen (HLA)
Intravenous (IV)
Intravenous push (IVP)
Medication administration record (MAR)
Nonsteroidal anti-inflammatory drug (NSAID)
Pulse (P)
Respirations (R)
Transcutaneous electrical nerve stimulation (TENS)
Unlicensed assistive personnel (UAP)

667

18_Colgrove_Ch18.indd 667 02/06/16 1:32 PM


668 M ed -S urg S uccess

PRACTICE QUESTIONS
Advance Directives 5. Which client would be most likely to complete an
advance directive?
1. The client tells the nurse, “Every time I come in 1. A 55-year-old Caucasian person who is a bank
the hospital you hand me one of these advance president.
directives (AD). Why should I fill one of these 2. A 34-year-old Asian licensed practical nurse.
out?” Which statement by the nurse is most 3. A 22-year-old Hispanic lawn care worker.
appropriate? 4. A 65-year-old African American retired cook.
1. “You must fill out this form because Medicare 6. The client with an AD tells the nurse, “I have
laws require it.” changed my mind about my AD. I really want
2. “An AD lets you participate in decisions about everything possible done if I am near death since I
your health care.” have a grandchild.” Which action should the nurse
3. “This paper will ensure no one can override implement?
your decisions.” 1. Notify the health information systems
4. “It is part of the hospital admission packet and department to talk to the client.
I have to give it to you.” 2. Remove the AD from the client’s chart and
2. The nurse is presenting an in-service discussing shred the document.
do not resuscitate (DNR) orders and advance 3. Inform the client he or she has the right to
directives. Which statement should the nurse revoke the AD at any time.
discuss with the class? 4. Explain this document cannot be changed once
1. Advance directives must be notarized by a it is signed.
notary public. 7. The client has just signed an AD at the bedside.
2. The client must use an attorney to complete Which intervention should the nurse implement
the advanced directive. first?
3. Once the DNR is written, it can be used for 1. Notify the client’s health-care provider about
every hospital admission. the AD.
4. The health-care provider must write the DNR 2. Instruct the client to discuss the AD with
order in the client’s chart. significant others.
3. In which client situation would the AD be 3. Place a copy of the advance directive in the
consulted and used in decision making? client’s chart.
1. The client diagnosed with Guillain-Barré who 4. Give the original advance directive to the
is on a ventilator. client.
2. The client with a C6 spinal cord injury in the 8. The HCP has notified the family of a client in a
rehabilitation unit. persistent vegetative state on a ventilator of the
3. The client in end-stage renal disease who is in need to “pull the plug.” The client does not have
a comatose state. an AD or a durable power of attorney for health
4. The client diagnosed with cancer who has care, and the family does not want their loved one
Down syndrome. removed from the ventilator. Which action should
4. The nurse is moving to another state which is the nurse implement?
part of the multistate licensure compact. Which 1. Refer the case to the hospital ethics committee.
information regarding ADs should the nurse be 2. Tell the family they must do what the HCP
aware of when practicing nursing in other states? orders.
1. The laws regarding ADs are the same in all the 3. Follow the HCP’s order and “pull the plug.”
states. 4. Determine why the client did not complete
2. Advance directives can be transferred from an AD.
state to state.
3. A significant other can sign a loved one’s
advance directive.
4. Advance directives are state regulated, not
federally regulated.

18_Colgrove_Ch18.indd 668 02/06/16 1:32 PM


C hapter 18  E nd - of -L ife I ssues 669

9. The client asks the nurse, “When will the Death and Dying
durable power of attorney for health care take
effect?” On which scientific rationale would the 13. The spouse of a client dying from lung cancer
nurse base the response? states, “I don’t understand this death rattle. She
1. It goes into effect when the client needs has not had anything to drink in days. Where is
someone to make financial decisions. the fluid coming from?” Which is the hospice
2. It will be effective when the client is under care nurse’s best response?
general anesthesia during surgery. 1. “The body produces about two (2) teaspoons
3. The client must say it is all right for it to of fluid every minute on its own.”
become effective and enforced. 2. “Are you sure someone is not putting ice chips
4. It becomes valid only when the clients cannot in her mouth?”
make their own decisions. 3. “There is no reason for this, but it does
10. The male client requested a DNR per the AD, happen from time to time.”
and the HCP wrote the order. The client’s death 4. “I can administer a patch to her skin to dry up
is imminent and the client’s wife tells the nurse, the secretions if you wish.”
“Help him please. Do something. I am not 14. The nurse is discussing placing the client
ready to let him go.” Which action should the diagnosed with chronic obstructive pulmonary
nurse take? disease (COPD) in hospice care. Which
1. Ask the wife if she would like to revoke her prognosis must be determined to place the client
husband’s AD. in hospice care?
2. Leave the wife at the bedside and notify the 1. The client is doing well but could benefit
hospital chaplain. from the added care by hospice.
3. Sit with the wife at the bedside and encourage 2. The client has a life expectancy of six (6)
her to say good-bye. months or less.
4. Request the client to tell the wife he is ready 3. The client will live for about one (1) to two
to die, and don’t do anything. (2) more years.
11. Which situation would cause the nurse to 4. The client has about eight (8) weeks to live
question the validity of an AD when caring for and needs pain control.
the elderly client? 15. The client diagnosed with end-stage congestive
1. The client’s child insists the client make his or heart failure and type 2 diabetes is receiving
her own decisions. hospice care. Which action by the nurse
2. The nurse observes the wife making the demonstrates an understanding of the client’s
husband sign the AD. condition?
3. A nurse encouraged the client to think about 1. The nurse monitors the blood glucose four
end-of-life decisions. (4) times a day.
4. A friend witnesses the client’s signature on the 2. The nurse keeps the client on a strict fluid
AD form. restriction.
12. The nurse is aware the Patient Self-Determination 3. The nurse limits the visitors the client can
Act of 1991 requires the health-care facility to receive.
implement which action? 4. The nurse brings the client a small piece of
1. Make available an AD on admission to the cake.
facility. 16. The hospice care nurse is conducting a spiritual
2. Assist the client with legally completing a will. care assessment. Which statement is the
3. Provide ethically and morally competent care scientific rationale for this intervention?
to the client. 1. The client will ask all of his or her spiritual
4. Discuss the importance of understanding questions and get answers.
consent forms. 2. The nurse is able to explain to the client how
death will affect the spirit.
3. Spirituality provides a sense of meaning and
purpose for many clients.
4. The nurse is the expert when assisting the
client with spiritual matters.

18_Colgrove_Ch18.indd 669 02/06/16 1:32 PM


670 M ed -S urg S uccess

17. The nurse is caring for a dying client and the 22. The client has been in a persistent vegetative
family. The male client is Muslim. Which state for several years. The family, who have
intervention should the female nurse implement decided to withhold tube feedings because there
at the time of death? is no hope of recovery, asks the nurse, “Will the
1. Allow the wife to stay in the room during death be painful?” Which intervention should
postmortem care. the nurse implement?
2. Call the client’s imam to perform last rites 1. Tell the family the death will be painful but
when the client dies. the HCP can order medications.
3. Place incense around the bed, but do not 2. Inform the family dehydration provides a type
allow anyone to light it. of natural euphoria.
4. Do not touch the body, and have the male 3. Relate other cases where the clients have died
family members perform care. in excruciating pain.
4. Ask the family why they are concerned
18. The nurse writes a client problem of “spiritual
because they want the client to die anyway.
distress” for the client who is dying. Which
statement is an appropriate goal? 23. The family is dealing with the imminent
1. The client will reconcile self and the higher death of the client. Which information is
power of his or her beliefs. most important for the nurse to discuss when
2. The client will be able to express anger at the planning interventions for the grieving process?
terminal diagnosis. 1. How angry are the family members about the
3. The client will reconcile self to estranged death?
members of the family. 2. Which family member will be making decisions?
4. The client will have a dignified and pain-free 3. What previous coping skills have been used?
death. 4. What type of funeral service has been planned?
19. The hospice care nurse is planning the care of 24. The client who is terminally ill called the
an elderly client diagnosed with end-stage renal significant others to the room and said good-
disease. Which interventions should be included bye, then dismissed them and now lies quietly
in the plan of care? Select all that apply. and refuses to eat. The nurse understands the
1. Discuss financial concerns. client is in what stage of the grieving process?
2. Assess any comorbid conditions. 1. Denial.
3. Monitor increased visual or auditory abilities. 2. Anger.
4. Note any spiritual distress. 3. Bargaining.
5. Encourage euphoria at the time of death. 4. Acceptance.
20. The nurse is orienting to a hospice organization.
Which statement does not indicate a right of the Chronic Pain
terminal client? The right to:
1. Be treated with respect and dignity. 25. The nurse is assessing a client diagnosed with
2. Have particulars of the death withheld. chronic pain. Which characteristics would the
3. Receive optimal and effective pain nurse observe?
management. 1. The client’s blood pressure is elevated.
4. Receive holistic and compassionate care. 2. The client has rapid shallow respirations.
21. The client is on the ventilator and has been 3. The client has facial grimacing.
declared brain dead. The spouse refuses to 4. The client is lying quietly in bed.
allow the ventilator to be discontinued. Which 26. The client had a mastectomy and lymph
collaborative action by the nurse is most node dissection three (3) years ago and has
appropriate? experienced postmastectomy pain (PMP)
1. Discuss referral of the case to the ethics since. Which intervention should the nurse
committee. implement?
2. Pull the plug when the spouse is not in the 1. Have the client see a psychologist because the
room. pain is not real.
3. Ask the HCP to discuss the futile situation 2. Tell the client the pain is the cancer coming
with the spouse. back.
4. Inform the spouse what is happening is cruel. 3. Refer the client to a physical therapist to
prevent a frozen shoulder.
4. Discuss changing the client to a more potent
narcotic medication.

18_Colgrove_Ch18.indd 670 02/06/16 1:32 PM


C hapter 18  E nd - of -L ife I ssues 671

27. The male client diagnosed with chronic pain 31. The male client who has made himself a do not
since a construction accident which broke resuscitate (DNR) order is in pain. The client’s
several vertebrae tells the nurse he has been vital signs are P 88, R 8, and BP 108/70. Which
referred to a pain clinic and asks, “What good intervention should be the nurse’s priority
will it do? I will never be free of this pain.” action?
Which statement is the nurse’s best response? 1. Refuse to give the medication because it could
1. “Are you afraid of the pain never going away?” kill the client.
2. “The pain clinic will give you medication to 2. Administer the medication as ordered and
cure the pain.” assess for relief from pain.
3. “Pain clinics work to help you achieve relief 3. Wait until the client’ respirations improve and
from pain.” then administer the medication.
4. “I am not sure. You should discuss this with 4. Notify the HCP the client is unstable and
your HCP.” pain medication is being held.
28. The client diagnosed with cancer is experiencing 32. The charge nurse is making assignments on an
severe pain. Which regimen would the nurse oncology floor. Which client should be assigned
teach the client about to control the pain? to the most experienced nurse?
1. Nonsteroidal anti-inflammatory drugs 1. The client diagnosed with leukemia who has a
(NSAIDs) around the clock with narcotics hemoglobin of 6 g/dL.
used for severe pain. 2. The client diagnosed with lung cancer with a
2. Morphine sustained release, a narcotic, pulse oximeter reading of 89%.
routinely with a liquid morphine preparation 3. The client diagnosed with colon cancer who
for breakthrough pain. needs the colostomy irrigated.
3. Extra-Strength Tylenol, a nonnarcotic 4. The client diagnosed with Kaposi’s sarcoma
analgesic, plus therapy to learn alternative who is yelling at the staff.
methods of pain control.
33. The nurse and an unlicensed assistive personnel
4. Demerol, an opioid narcotic, every six (6)
(UAP) are caring for a group of clients in
hours orally with a suppository when the pain
a pain clinic. Which intervention would be
is not controlled.
inappropriate to delegate to the UAP?
29. The client is being discharged from the hospital 1. Assist the client diagnosed with intractable
for intractable pain secondary to cancer and pain to the bathroom.
is prescribed morphine, a narcotic. Which 2. Elevate the head of the bed for a client
statement indicates the client understands the diagnosed with back pain.
discharge instructions? 3. Perform passive range of motion for a client
1. “I will be sure to have my prescriptions filled who is bedfast.
before any holiday.” 4. Monitor the potassium levels on a client about
2. “There should not be a problem having the to receive medication.
prescriptions filled anytime.”
34. The client diagnosed with chronic back pain is
3. “If I run out of medications, I can call the
being placed on a transcutaneous electrical nerve
HCP to phone in a prescription.”
stimulation (TENS) unit. Which information
4. “There are no side effects to morphine I
should the nurse teach?
should be concerned about.”
1. The TENS unit will deaden the nerve
30. The client diagnosed with intractable pain is endings, and the client will not feel pain.
receiving an IV constant infusion of morphine, 2. The TENS unit could cause paralysis if the
a narcotic opioid. The concentration is 50 mg of client gets the unit wet.
morphine in 250 mL of normal saline. The IV 3. The TENS unit stimulates the nerves in the
is infusing at 10 mL/hr. The client has required area, blocking the pain sensation.
bolus administration of two (2) mg IVP × two 4. The TENS unit should be left on for an hour,
(2) during the 12-hour shift. How much and then taken off for an hour.
morphine has the client received during the
shift? _________

18_Colgrove_Ch18.indd 671 02/06/16 1:32 PM


672 M ed -S urg S uccess

35. The nurse is caring for clients on a medical floor. wrong with me?” Which response by the nurse
Which client should the nurse assess first after would support the ethical principle of veracity?
the shift report? 1. “I think you should talk to your doctor about
1. The client with arterial blood gases of pH your concerns.”
7.36, Paco2 40, HCO3 26, Pao2 90. 2. “What makes you think something is really
2. The client with vital signs of T 99˚F, P 101, wrong?”
R 28, and BP 120/80. 3. “Your family has requested you not be told
3. The client complaining of pain at a “10” on a your diagnosis.”
1-to-10 scale who can’t localize it. 4. “The doctor would never tell you incorrect
4. The client who is postappendectomy with information.”
pain at a “3” on a 1-to-10 scale.
40. The nurse is obtaining the client’s signature on
36. The female client in the oncology clinic tells a surgical permit form. The nurse determines
the nurse she has a great deal of pain but does the client does not understand the surgical
not like to take pain medication. Which action procedure and possible risks. Which action
should the nurse implement first? should the nurse take first?
1. Tell the client it is important for her to take 1. Notify the client’s surgeon.
her medication. 2. Document the information in the chart.
2. Find out how the client has been dealing with 3. Contact the operating room staff.
the pain. 4. Explain the procedure to the client.
3. Have the HCP tell the client to take the pain
41. The client is in the psychiatric unit in a medical
medications.
center. Which action by the psychiatric nurse is a
4. Instruct the client not to worry—the pain will
violation of the client’s legal and civil rights?
resolve itself.
1. The nurse tells the client civilian clothes can
be worn on the unit.
Ethical/Legal Issues 2. The nurse allows the client to have family
visits during visiting hours.
37. The nurse is teaching an in-service on legal 3. The nurse delivers unopened mail and
issues in nursing. Which situation is an example packages to the client.
of battery, an intentional tort? 4. The nurse listens to the client talking on the
1. The nurse threatens the client who is refusing telephone to a friend.
to take a hypnotic medication. 42. The client receiving dialysis for end-stage renal
2. The nurse forcibly inserts a Foley catheter in disease wants to quit dialysis and die. Which
a client who refused it. ethical principle supports the client’s right to die?
3. The nurse tells the client a nasogastric tube 1. Autonomy.
insertion is not painful. 2. Self-determination.
4. The nurse gives confidential information over 3. Beneficence.
the telephone. 4. Justice.
38. Which act protects the nurse against a 43. Which document is the best professional source
malpractice claim when the nurse stops at a to provide direction for a nurse when addressing
motor-vehicle accident and renders emergency ethical issues and behavior?
care? 1. The Hippocratic Oath.
1. The Health Insurance Portability and 2. The Nuremberg Code.
Accountability Act. 3. Home Health Care Bill of Rights.
2. The State Nurse Practice Act. 4. ANA Code of Ethics.
3. The Emergency Rendering Aid Act.
4. The Good Samaritan Act. 44. Which element is not necessary to prove nursing
malpractice?
39. The family has requested a client with terminal 1. Breach of duty.
cancer not be told of the diagnosis. The client 2. Identify the ethical issues.
tells the nurse, “I think something is really 3. Injury to the client.
wrong with me, but the doctor says everything 4. Proximate cause.
is all right. Do you know if there is something

18_Colgrove_Ch18.indd 672 02/06/16 1:32 PM


C hapter 18  E nd - of -L ife I ssues 673

45. The nurse is caring for a client who is confused 3. “No one knows who gets an organ. You just
and fell trying to get out of bed. There is no have to wait and pray.”
family at the client’s bedside. Which action 4. “The tissues must match or the body will
should the nurse implement first? reject the kidney and it will be wasted.”
1. Contact a family member to come and stay
50. The nurse is discussing the HCP’s
with the client.
recommendation for removal of life support
2. Administer a sedative medication to the client.
with the client’s family. Which information
3. Place the client in a chair with a sheet tied
concerning brain death should the nurse teach
around him or her.
the family?
4. Notify the health-care provider to obtain a
1. Positive waves on the electroencephalogram
restraint order.
(EEG) mean the brain is dead and any further
46. Which entity mandates the registered nurse’s treatment is futile.
behavior when practicing professional nursing? 2. When putting cold water in the ear, if
1. The state’s Nurse Practice Act. the client reacts by pulling away, this
2. Client’s Bill of Rights. demonstrates brain death.
3. The United States legislature. 3. Tests will be done to determine if any brain
4. American Nurses Association. activity exists before the machines are
turned off.
47. The nurse must be knowledgeable of ethical
4. Although the blood flow studies don’t indicate
principles. Which is an example of the ethical
activity, the client can still come out of the
principle of justice?
coma.
1. The nurse administers a placebo, and the
client asks if it will help the pain. 51. The client diagnosed with septicemia expired,
2. The nurse accepts a work assignment in an and the family tells the nurse the client is an
area in which he or she is not experienced. organ donor. Which intervention should the
3. The nurse refuses to tell a family member the nurse implement?
client has a positive HIV test. 1. Notify the organ and tissue organizations to
4. The nurse provides an indigent client with make the retrieval.
safe and appropriate nursing care. 2. Explain a systemic infection prevents the
client from being a donor.
48. The nurse is discussing malpractice issues in an
3. Call and notify the health-care provider of the
in-service class. Which situation is an example of
family’s request.
malpractice?
4. Take the body to the morgue until the organ
1. The nurse fails to report a neighbor who is
bank makes a decision.
abusing his two children.
2. The nurse does not intervene in a client who 52. The client has received a kidney transplant.
has a BP of 80/50 and AP of 122. Which assessment would warrant immediate
3. The nurse is suspected of taking narcotics intervention by the nurse?
prescribed for a client. 1. Fever and decreased urine output.
4. The nurse falsifies vital signs in the client’s 2. Decreased creatinine and BUN levels.
medical records. 3. Decreased serum potassium and calcium.
4. Bradycardia and hypotension.

Organ/Tissue Donation 53. The client received a liver transplant and is


preparing for discharge. Which discharge
49. The mother of a 20-year-old African American instruction should the nurse teach?
male client receiving dialysis asks the nurse, “My 1. The immune-suppressant drugs must be
son has been on the transplant list longer than tapered off when discontinuing them.
that white woman. Why did she get the kidney?” 2. There may be slight foul-smelling drainage
Which statement is the nurse’s best response? on the dressing for a few days.
1. “The woman was famous, and so more people 3. Notify the HCP immediately if a cough or
will donate organs now.” fever develops.
2. “I understand you are upset your son is ill. 4. The skin will turn yellow from the
Would you like to talk?” antirejection drugs.

18_Colgrove_Ch18.indd 673 02/06/16 1:32 PM


674 M ed -S urg S uccess

54. The pregnant client asks the nurse about 58. The experienced medical-surgical nurse is being
banking the cord blood. Which information oriented to the transplant unit. Which client
should the nurse teach the client? should the charge nurse assign to this nurse?
1. The procedure involves a lot of pain with a 1. The client who donated a kidney to a relative
very poor result. three (3) days ago and will be discharged in
2. The client must deliver at a large public the morning.
hospital to do this. 2. The client who had a liver transplantation
3. The client will be charged a yearly storage fee three (3) days ago and was transferred from
on the cells. the intensive care unit two (2) hours ago.
4. The stem cells can be stored for about four 3. The client who received a corneal transplant
(4) years before they ruin. four (4) hours ago and has developed a cough
and is vomiting.
55. The nurse is caring for a client who received
4. The client who had a pancreas transplantation
a kidney transplant from an unrelated cadaver
and has a fever, chills, and a blood glucose
donor. Which interventions should be included
monitor reading of 342.
in the plan of care? Select all that apply.
1. Collect a urine culture every other day. 59. The 6-year-old client diagnosed with cystic
2. Prepare the client for dialysis three (3) times fibrosis (CF) needs a lung transplant. Which
a week. individual would be the best donor for the
3. Monitor urine osmolality studies. client?
4. Monitor intake and output every shift. 1. The 20-year-old brother who does not have
5. Check abdominal dressing every four (4) cystic fibrosis.
hours. 2. The 45-year-old father who carries the cystic
fibrosis gene.
56. The client is three (3) hours post–heart
3. The 18-year-old who died in an MVA who
transplantation. Which data would support a
matches on four (4) points.
complication of this procedure?
4. The 5-year-old drowning victim who is a
1. The client has nausea after taking the oral
three (3)-point match.
antirejection medication.
2. The client has difficulty coming off the heart- 60. Which tissue or organ can be repeatedly donated
lung bypass machine. to clients needing a transplant?
3. The client has saturated three (3) ABD 1. Skin.
dressing pads in one (1) hour. 2. Bones.
4. The client complains of pain at a “6” on a 3. Kidneys.
1-to-10 scale. 4. Bone marrow.
57. The nurse and an unlicensed assistive personnel
(UAP) are caring for clients on a postoperative
transplant unit. Which task should the nurse
delegate to the UAP?
1. Assess the hourly outputs of the client who is
post–kidney transplantation.
2. Raise the head of the bed for a client who is
post–liver transplantation.
3. Monitor the serum blood studies of a client
who has rejected an organ.
4. Irrigate the nasogastric tube of the client who
had a pancreas transplant.

18_Colgrove_Ch18.indd 674 02/06/16 1:32 PM


C hapter 18  E nd - of -L ife I ssues 675

CONCEPTS
The concepts covered in this chapter focus on end-of-life issues including ethical, spirituality, legal, and comfort.
Exemplars that are covered are chronic pain, advance directives, decisions at the end of life, legal decisions, and
culture. Interrelated concepts of the nursing process and critical thinking are covered throughout the questions.
The concept of critical thinking is presented in the prioritizing or “first” questions.

61. The nurse is admitting a client to the medical- 64. The hospice nurse is admitting a client. Which
surgical unit. Which is required to be offered question concerning end-of-life care is most
to the client if the hospital accepts Medicare important for the nurse to discuss with the
reimbursement? client and family?
1. The opportunity to make an advance 1. Encourage the client and family to make
directive. funeral arrangements.
2. The client must be referred to a case manager. 2. Assess the client’s pain medication regimen
3. The client must apply for a Medicare for effectiveness.
supplement insurance. 3. Determine if the client has made an advance
4. The opportunity to discuss end-of-life issues. directive or living will.
4. Ask what durable medical equipment is in
62. The nurse pronounced Dr. Smith’s client to
place in the home.
be clinically dead. Which should the nurse
document on the client’s chart? 65. The client is dying and wants to talk to the nurse
1. Brain scan indicates no brain wave activity, about heaven. Which is the nurse’s best nursing
client pronounced deceased. Family refuses to action?
talk with organ bank. 1. Make a referral to the chaplain to come to see
2. Cardiac arrest noted, CPR initiated but the client.
unsuccessful. Pronounced dead. 2. Tell the client that nurses are not allowed to
3. Pulse, respirations, and blood pressure absent discuss spiritual matters.
at 0900, pronounced dead. Dr. Smith to sign 3. Ask the client to describe heaven and hell.
death certificate. 4. Allow the client to discuss the beliefs about
4. Client found without pulse, body cold to heaven.
touch. Pronounced deceased at 0900.
66. The male client in the long-term care facility
63. The nurse is caring for an 82-year-old female has been told that he will not live for many more
client who is crying and asking for her mother to months. The client has been estranged from
come to see her. Which statement represents the his daughter for years. He tells the nurse that
ethical principle of nonmalfeasance? he could die a happy man if he could talk to his
1. “You must miss your mother very much. Can daughter just one more time. Which statement is
you tell me about her?” the nurse’s best response?
2. “You are 82 years old. Your mother is dead 1. “You should not feel bad. Things will work
and can’t come see you.” out for the best before your death.”
3. “Why do you need your mother? Can I get 2. “What did you do to make your daughter not
something for you?” talk to you all this time?”
4. “Your mother would not want you to worry. 3. “If you would like I can try to contact your
I will tell her you want to see her.” daughter and ask her to come see you.”
4. “Tell me more about being unhappy that you
don’t have a relationship with your daughter.”
67. The nurse is caring for the family of the client
who has just died. Which is the nurse’s priority
action?
1. Be with the family.
2. Call the funeral home.
3. Notify the minister.
4. Fill out the death certificate.

18_Colgrove_Ch18.indd 675 02/06/16 1:32 PM


PRACTICE QUESTIONS ANSWERS
AND RATIONALES

Advance Directives 3. 1. A client diagnosed with Guillain-Barré


­syndrome is mentally competent, and being on
1. 1. Advance directives (AD) are not legally a ventilator does not indicate the client has lost
­required. It is a standard of the Joint Com- his or her decision-making capacity.
mission, and any facility which accepts federal 2. A client in the rehabilitation unit would be
funds must ask and offer the AD. alert, and spinal cord injuries do not cause the
2. ADs allow the client to make personal client to lose decision-making capacity.
health-care decisions about end-of-life 3. The client must have lost decision-making
­issues, including cardiopulmonary resusci- capacity as a result of a condition which
tation (CPR), ventilators, feeding tubes, and is not reversible or must be in a condition
other issues concerning the client’s death. specified under state law, such as a termi-
3. This is not a legal document guaranteed to nal, persistent vegetative state; an irrevers-
stand up in a court of law; therefore, the client ible coma; or as specified in the AD.
should make sure all family members know the 4. A client with Down syndrome may have some
client’s wishes. mental challenges, but unless the client has
4. It is part of the hospital admission require- been declared legally incompetent in a court of
ments, but it is not the reason why the client law, the client can complete an AD and partici-
should complete an AD. pate in his/her own case.
TEST-TAKING HINT: The test taker could TEST-TAKING HINT: If the test taker knows
eliminate option “1” because the nurse cannot what an AD is, then the words “end-stage”
make the client do anything. The client has a and “comatose” would lead the test taker to
right to say no. Option “3” is an absolute, and select option “3” as a correct answer. Remem-
unless the test taker knows for sure this is ber, clients with congenital or genetic dis-
correct information, the test taker should not orders are not incompetent, even if they are
select this option. mentally challenged.
Content – Medical: Integrated Nursing Process – Content – Medical: Integrated Nursing Process –
­Implementation: Client Needs – Safe Effective Care ­Implementation: Client Needs – Safe Effective Care
­Environment, Management of Care: Cognitive Level – ­Environment, Management of Care: Cognitive Level
Application: Concept – Legal. ­Application: Concept –Legal.

2. 1. This is not true; someone who is not family 4. 1. Individual states are responsible for specific
or directly involved in the client’s care must legal requirements for ADs.
­witness the AD, but the document does not 2. Moving from one state to another does not
have to be notarized. nullify or honor the AD; the nurse must be
2. This form can be filled out without the use of aware of the individual state’s requirements.
an attorney; copies of an AD can be obtained 3. Only the individual can complete and sign an
at hospitals or online from various sources. AD. The significant other may be asked to
3. The DNR order must be written on each implement the AD.
admission. 4. The state determines the definition of
4. The HCP writes the DNR order in the terms and requirements for an AD; individ-
client’s chart, and the client completes ual states are responsible for specific legal
the AD. requirements for ADs.
TEST-TAKING HINT: Options “1” and “2” TEST-TAKING HINT: The test taker should
­involve other legal entities outside the health- know the registered nurse must obtain a copy
care arena, which would make the test taker of the Nurse Practice Act of the state he or
eliminate them. she is practicing in. The test taker should
Content – Medical: Integrated Nursing Process – Planning: realize every state has different regulations
Client Needs – Safe Effective Care Environment, regarding ADs and other health-care issues.
­Management of Care: Cognitive Level – Synthesis: Option “4” is the only option which reflects
Concept – Nursing Roles. this thought.

676

18_Colgrove_Ch18.indd 676 02/06/16 1:32 PM


C hapter 18  E nd - of -L ife I ssues 677

Content – Medical: Integrated Nursing Process – 7. 1. The HCP should be made aware of the AD,
­Implementation: Client Needs – Safe Effective Care but this is not the first intervention.
­Environment, Management of Care: Cognitive Level – 2. This is the most important intervention
­Application: Concept – Legal. because the legality of the document is
5. 1. ADs are more frequently completed by sometimes not honored if the family mem-
white, middle- to upper-class individuals. bers disagree and demand other action. If
2. Many nurses do not have ADs, although they the client’s family is aware of the client’s
discuss them with clients daily. wishes, then the health-care team can sup-
3. Culturally, Hispanics allow their family port and honor the client’s final wishes.
­members to make decisions for them. 3. Copies of the AD should be placed in the chart
4. Many cultures, including the African American and given to significant others, the client’s at-
culture, often distrust the health-care system torney, and all health-care providers.
and believe necessary care will be withheld if 4. The original should be given to the client and
an AD is completed. a copy should be placed in the chart, but this is
not the first intervention.
TEST-TAKING HINT: If the test taker were not
aware of the research, the test taker could exam- TEST-TAKING HINT: This is a priority-setting
ine the occupations and ask themselves, “Which question, and the test taker should read all the
client would want to direct his or her own care answer options and try to rank them in order
and make his or her own decisions?” Nurses of priority.
may want this but many do not have ADs. Content – Medical: Integrated Nursing Process –
Content – Medical: Integrated Nursing Process – ­Implementation: Client Needs – Safe Effective Care
­Diagnosis: Client Needs – Psychosocial Integrity: Cognitive ­Environment, Management of Care: Cognitive Level –
Level – Analysis: Concept – Legal. Analysis: Concept – Legal.

6. 1. This department has nothing to do with the 8. 1. The ethics committee is composed of
AD. health-care workers and laypeople from the
2. The most appropriate action would be for the community to objectively review the situa-
nurse to have the client write on the AD he or tion and make a recommendation which is
she is revoking the document; the nurse can- fair to both the client and health-care sys-
not shred legal documents from the client’s tem. The family has the right to be present
chart. and discuss their feelings.
3. The client must be informed the AD can 2. The nurse is legally obligated to be a client
be rescinded or revoked at any time for any advocate.
reason verbally, in writing, or by destroy- 3. This action could create a multitude of
ing his or her own AD. The nurse cannot ­ramifications, including a lawsuit and possible
destroy the client’s AD, but the client can criminal charges.
destroy his or her own. 4. It really doesn’t matter at this point why the
4. This is an incorrect answer because the client client didn’t complete an AD; the client cannot
always has the right to change his or her mind. do it now.
TEST-TAKING HINT: Option “4” can be elimi- TEST-TAKING HINT: The test taker must be
nated by remembering statements with aware of the ethics committee and its role in
absolutes should not be selected as correct helping resolve ethical dilemmas. Any answer
answers unless the test taker knows for sure option which has the word “why” should be
the answer is evaluated closely before selecting it as the
correct. The client’s chart is a legal docu- correct answer. Removing the endotracheal
ment, and these papers cannot be shredded tube or turning off the ventilator (“pulling the
or altered by using anything that obscures the plug”) is a medical responsibility; therefore,
writing or by erasing information. option “3” could be eliminated as the correct
answer.
Content – Medical: Integrated Nursing Process –
­Implementation: Client Needs – Safe Effective Care Content – Medical: Integrated Nursing Process –
­Environment, Management of Care: Cognitive Level – ­Implementation: Client Needs – Safe Effective Care
­Application: Concept – Legal. ­Environment, Management of Care: Cognitive Level –
Application: Concept – Legal.

18_Colgrove_Ch18.indd 677 02/06/16 1:32 PM


678 M ed -S urg S uccess

9. 1. It is a power of attorney executed by a law- client’s own free will; an AD signed under
yer which allows a delegated other person to duress may not be valid.
make financial decisions. That document has 3. The nurse encouraging the client to think
nothing to do with a durable power of attor- about ADs is an excellent intervention and
ney for health care. would not make the AD invalid.
2. The client has not lost the capacity to make 4. A friend can sign the AD as a witness; this
decisions; therefore, a durable power of attor- would not cause the nurse to question its
ney cannot be used by the assigned person to validity.
make decisions. TEST-TAKING HINT: This is an “except” ques-
3. The client must not be able to make his or tion. The test taker could ask, “Which situ-
her own decisions before this document can ation is valid for an AD?” Remember, three
be used. answers are valid information for the AD and
4. The client must have lost decision-making only one is not. The test taker should read all
capacity as a result of a condition which answer options and not jump to conclusions.
is not reversible or must be in a condition Content – Medical: Integrated Nursing Process – ­
which is specified under state law, such as a Assessment: Client Needs – Safe Effective Care
terminal, persistent vegetative state; an ir- ­Environment, Management of Care: Cognitive Level –
reversible coma; or as specified in the AD. Analysis: Concept – Legal.
TEST-TAKING HINT: The test taker should not
12. 1. The Patient Self-Determination Act of
confuse a power of attorney and a durable 1991 requires health-care facilities which
power of attorney for health care. These are receive Medicare or Medicaid funding to
two separate, yet very important, documents make ADs available to clients on admis-
with similar names. sion into the facility.
Content – Medical: Integrated Nursing Process – 2. This act is not concerned with completing a
­Implementation: Client Needs – Safe Effective Care legal will.
­Environment, Management of Care: Cognitive Level –
3. Client care is not based on this act.
Analysis: Concept – Legal.
4. Consent forms are legal documents, which
10. 1. Only the client can revoke the AD. are not discussed in this act.
2. The wife should not be left alone, and the TEST-TAKING HINT: The test taker should
hospital chaplain may not be available for the examine the word “self-determination” in
client and his wife. the stem of the question, which matches the
3. At the time of death, loved ones become advance directive in option “1.” The words
scared and find it difficult to say good-bye. “legally,” “ethically,” and “morally” in options
The nurse should support the client’s deci- “2” and “3” apply to the nurse in the health-
sion and acknowledge the wife’s psycholog- care setting, not the client.
ical state. Research states hearing is the last Content – Medical: Integrated Nursing Process –
sense to go, and talking to the d ­ ying client ­Implementation: Client Needs – Safe Effective Care
is therapeutic for the client and the family. Environment, Management of Care: Cognitive Level –
4. The client is dying and should not be asked to Application: Concept – Legal.
exert himself for his wishes to be carried out.
TEST-TAKING HINT: Logic would suggest op-
tion “4” is not a viable answer. Leaving a Death and Dying
grieving spouse would not be appropriate in
any situation; therefore, the test taker should 13. 1. The respiratory tract cells produce liquid
eliminate option “2.” Option “1” denies the as a defense mechanism against bacteria
client’s autonomy and is not an ethical or a and other invaders. About nine (9) mL a
legal choice. minute are produced. The “death rattle”
can be disturbing to family members, and
Content – Medical: Integrated Nursing Process –
­Implementation: Client Needs – Safe Effective Care
the nurse should intervene but not with
Environment, Management of Care: Cognitive Level – suctioning, which will increase secretions
­Application: Concept – Grief and Loss. and the need to suction more.
2. This is a natural physical phenomenon and
11. 1. This is appropriate for completing an AD and should be addressed.
would not make the nurse question the valid- 3. There is an explanation.
ity of the AD. 4. The scopolamine patch applied to the skin
2. This is coercion and is illegal when sign- helps to limit the secretions, but this does not
ing an AD. The AD must be signed by the answer the question.

18_Colgrove_Ch18.indd 678 02/06/16 1:32 PM


C hapter 18  E nd - of -L ife I ssues 679

TEST-TAKING HINT: The test taker could 16. 1. The nurse is not able to provide all spiritual
eliminate option “3” because it states there answers to the client.
is no reason, option “4” because it does not 2. The nurse can explain physical aspects of
answer the question, and option “2” because death, but no one is able to tell the client
it is attempting to fix blame. with absolute knowledge what will happen to
Content – Medical: Integrated Nursing Process – the soul or spirit at death. The beliefs of the
­Implementation: Client Needs – Physiological Integrity, client may differ greatly from those of the
Physiological Adaptation: Cognitive Level – Analysis: nurse.
Concept – Grief and Loss. 3. Clients facing death may wish to find
14. 1. Hospice care does not assume care of a client meaning and purpose in life through a
with a prognosis of more than six (6) months higher power. This gives the clients hope,
and who is doing well. even if the life on earth will be temporary.
2. The HCP must think that, without life- 4. The nurse is not the expert but should be
prolonging treatment, the client has a life comfortable with his or her own beliefs to
expectancy of six (6) months or less. The be able to allow the client to discuss per-
client may continue receiving hospice care sonal ­beliefs and hopes. The experts would
if the client lives longer. be chaplains and spiritual advisers from the
3. The client may live this long, but the HCP c­ lient’s faith.
must think life expectancy is much shorter. TEST-TAKING HINT: The test taker should
4. Hospice will attempt to manage symptoms recognize the nurse’s expertise is not in the
of pain, nausea, and any other discomfort the spiritual realm, although the nurse is fre-
client is experiencing, but the life expectancy quently the one called on to perform the
is six (6) months. assessment and refer to the appropriate
TEST-TAKING HINT: This is a knowledge-
person.
based question requiring an understanding Content – Medical: Integrated Nursing Process –
of hospice. ­Assessment: Client Needs – Psychosocial Integrity:
­Cognitive Level – Analysis: Concept – Grief and Loss.
Content – Medical: Integrated Nursing Process –
­Diagnosis: Client Needs – Safe Effective Care Environment, 17. 1. No female is allowed to perform the post-
Management of Care: Cognitive Level – Analysis: mortem care on a male Muslim client; this
Concept – Grief and Loss. should be performed by a man.
15. 1. This would be basic care, but it does not indi- 2. Last rites are performed by a Catholic priest,
cate the nurse is aware of the client’s terminal not a Muslim imam.
prognosis. 3. Hindus use incense to pray, but Muslims do
2. This does not indicate an understanding of not.
the client’s terminal status. 4. Females, including the spouse, are not
3. The nurse should encourage visitors. There ­allowed to touch a male’s body after death.
is not much time left for making memories, The nurse should respect this and al-
which will assist those left behind in dealing low the male members of the family or
with the loss and allow the client time to say mosque to perform postmortem care.
good-bye. TEST-TAKING HINT: The question is requiring
4. The client may have diabetes, but the c­ ulturally sensitive knowledge. The test taker
­client is also terminal, and allowing some must be aware of the different beliefs of the
food for pleasure is understanding of the ­clients being cared for.
client’s life expectancy. Content – Medical: Integrated Nursing Process –
TEST-TAKING HINT: This question requires ­Implementation: Client Needs – Psychosocial Integrity:
the test taker to look not only at the disease ­Cognitive Level – Application: Concept – Grief and Loss.
processes but also at the descriptive words
“end-stage” and “hospice” and ask, “What do
these descriptors mean to the disease pro-
cess?” Not limiting the client in small ways
indicates the nurse is aware the client has a
limited time to live.
Content – Medical: Integrated Nursing Process –
­Implementation: Client Needs – Physiological Integrity,
Physiological Adaptation: ­Cognitive Level – Analysis:
­Concept – Grief and Loss.

18_Colgrove_Ch18.indd 679 02/06/16 1:32 PM


680 M ed -S urg S uccess

18. 1. The primary goal of spiritual care is to 20. 1. The client has the right to be cared for with
­allow the client to be able to reconcile respect and dignity.
himself or herself with a higher being, 2. The client has the right to discuss his or
maybe God. This goal is based on the her feelings and direct his or her care.
­belief that life comes from God, and to Withholding information would be lying
some degree for many people the process to the client.
of living includes some separation from 3. The client has the right to the best care avail-
God. In the Western world, 95% of the able and to have pain treated, regardless of
people claim some belief in God. the potential for hastening death.
2. This could be a goal for a diagnosis of anger, 4. All clients, even if they are not dying, have
but it does not recognize the spiritual aspect the right to holistic and compassionate care.
of the client. TEST-TAKING HINT: This is an “except” ques-
3. This would be a goal for altered family tion. All of the answer options except one
functioning. have correct information. The test taker
4. This is the physiological goal for any client should read the stem carefully to recognize
who is dying, but it is not a goal for spiritual this type of question.
distress. Content – Medical: Integrated Nursing Process –
TEST-TAKING HINT: The identified problem is ­Implementation: Client Needs – Safe Effective Care
“spiritual distress,” and the goal must have Environment, Management of Care: Cognitive Level –
information which addresses the spiritual. Application: Concept – Grief and Loss.
This would eliminate option “4.” Personal 21. 1. The nurse should discuss using the eth-
relationships with family members (option ics committee with the HCP to assist the
“3”) could also be eliminated. family in making the decision to terminate
Content – Medical: Integrated Nursing Process – life support. Many families feel there may
­Diagnosis: Client Needs – Psychosocial Integrity: Cognitive be a racial or financial reason the HCP
Level – Analysis: Concept – Grief and Loss.
wants to discontinue life support.
19. 1. The elderly are frequently on fixed 2. This would be an illegal act on the part of the
­incomes, and financial concerns are im- nurse and would destroy the nurse–­client re-
portant for the nurse to address. A social lationship with the family.
services referral may be needed. 3. The stem already indicates the spouse is
2. The elderly may have many comorbid aware of the situation.
conditions, which affect the type and 4. This is expressing a personal bias on the part
amount of medications the client can of the nurse.
­tolerate and the client’s quality of life. TEST-TAKING HINT: The test taker could
3. Visual and auditory senses decrease with age; ­eliminate option “2” based on the legal and
they do not increase. ethical issues. Option “3” is asking the HCP
4. The client may feel some spiritual distress to do something which has already been done.
at the terminal diagnosis. Even if the cli- Content – Medical: Integrated Nursing Process –
ent possesses a strong faith, the unknown ­Implementation: Client Needs – Safe Effective Care
can be frightening. Environment, Management of Care: Cognitive Level –
5. A type of euphoria may accompany de- Application: Concept – Nursing Roles.
hydration prior to death. This is a natural
22. 1. Death from dehydration occurs when the cli-
physiological occurrence the nurse should
ent is unable to take in fluids, but dehydration
recognize, but it is not an intervention the
is not painful.
nurse can implement.
2. Death from dehydration occurs when the
TEST-TAKING HINT: The test taker can decide client is unable to take in fluids. A natural
on three (3) of the answer options based on euphoria occurs with dehydration. This is
the descriptive word “elderly.” Option “5” is the body’s way of allowing comfort at the
not a nursing intervention. time of death.
Content – Medical: Integrated Nursing Process – 3. This is needless.
­Planning: Client Needs – Safe Effective Care Environment, 4. Families who make this decision usually do so
Management of Care: Cognitive Level – Synthesis: from a deep sense of love and commitment.
Concept – Grief and Loss.
It is an extremely difficult decision to make,
and the nurse should not condemn the family
decision.

18_Colgrove_Ch18.indd 680 02/06/16 1:32 PM


C hapter 18  E nd - of -L ife I ssues 681

TEST-TAKING HINT: The test taker could Chronic Pain


examine options “3” and “4” and eliminate
them based on the needless information or 25. 1. Blood pressure elevates in acute pain.
the nurse stepping outside of professional Chronic pain, by definition, lasts more than
boundaries. six (6) months, lasts far beyond the expected
Content – Medical: Integrated Nursing Process – time for the pain to resolve, and may have an
­Implementation: Client Needs – Psychosocial Integrity: unclear onset. Changes in vital signs
Cognitive Level – Analysis: Concept – Grief and Loss. result from the fight-or-flight response by the
23. 1. The family may or may not be angry and this body. The body cannot maintain this
would need to be addressed, but it is not the response and must adjust.
most important. 2. Rapid shallow respirations might be attrib-
2. Who makes the decisions is not as important uted to acute pain if it was painful to breathe.
as discovering which coping skills the family The client with a chest injury or pain will
uses when under stress. splint the area and slow the respirations or at-
3. The nurse should assess previous cop- tempt to breathe shallowly and rapidly.
ing skills used by the family and build on 3. Facial grimacing will occur in acute pain and
those to assist the family in dealing with is an objective sign the nurse can identify. Cli-
their loss. Coping mechanisms are learned ents with chronic pain may be laughing and
behaviors and should be supported if they still be in pain. Remember, pain is whatever
are healthy behaviors. If the client and the client says it is and occurs whenever the
family use unhealthy coping behaviors, client says it does.
then the nurse should attempt to guide 4. The client in chronic pain will have
the family to a counselor or support adapted to living with the pain, and lying
group. quietly may be the best way for the client
4. The type of funeral service may help the to limit the feeling of pain.
­family to grieve, but it is not the most impor- TEST-TAKING HINT: The test taker must
tant intervention. be able to differentiate between acute and
TEST-TAKING HINT: The test taker must pri- chronic pain. Options “1,” “2,” and “3” are
oritize the interventions listed. All of the in- objective symptoms of acute pain. If the test
terventions could be addressed in option “3.” taker were aware of this, then choosing the
only option left would be a good choice.
Content – Medical: Integrated Nursing Process –
­Planning: Client Needs – Psychosocial Integrity: Cognitive Content – Medical: Integrated Nursing Process –
Level – Synthesis: Concept – Grief and Loss. ­Assessment: Client Needs – Physiological Integrity,
­Reduction of Risk Potential: Cognitive Level – Analysis:
24. 1. The client is not denying death; the client has Concept – Comfort.
said good-bye.
26. 1. Pain is whatever the client says it is and
2. Anger is the second stage of the grieving pro-
­occurs whenever the client says it does. The
cess, but this client appears to have accepted
nurse should never deny the client’s pain
death.
exists.
3. There is no evidence of bargaining in the
2. This has been occurring for the past three
­client’s actions.
(3) years and does not mean the cancer has
4. The client has accepted the imminent
come back. Many clients will fear the cancer
death and is withdrawing from the signifi-
has r­ ecurred and delay treatment; denial is a
cant others.
potent coping mechanism.
TEST-TAKING HINT: There are five (5) stages 3. PMP is characterized as a constriction
to Dr. Elisabeth Kübler-Ross’s grieving pro- accompanied by a burning sensation or
cess, and some authorities list several more, prickling in the chest wall, axilla, or pos-
but this behavior could only be withdrawal or terior arm resulting from movement of
acceptance. the arm. Because of this, the client limits
Content – Medical: Integrated Nursing Process – movement of the arm and the shoulder
Assessment: Client Needs – Psychosocial Integrity: becomes frozen.
Cognitive Level – Analysis: Concept – Grief and Loss. 4. There are many problems associated with
long-term narcotic use. Other strategies
should be attempted prior to resigning
the client to a lifetime of taking narcotic
medications.

18_Colgrove_Ch18.indd 681 02/06/16 1:32 PM


682 M ed -S urg S uccess

TEST-TAKING HINT: The test taker could ­ harmacological and Parenteral Therapies: Cognitive
P
eliminate option “1” because it violates all Level – Synthesis: Concept – Comfort.
principles of pain management. Option “2” is 29. 1. Narcotic medications require handwritten
not in the realm of the nurse’s responsibility. prescription forms (Drug Enforcement
Content – Surgical: Integrated Nursing Process – Agency rules) which must be filled within
­Implementation: Client Needs – Safe Effective Care a limited time frame from the time the
Environment, Management of Care: Cognitive Level – prescription is written. Many local phar-
­Application: Concept – Comfort.
macies will not have the medication avail-
27. 1. This is a therapeutic response and the client able or may not have it in the quantities
is requesting information. needed. The client should anticipate the
2. Pain clinics do not cure pain; they do help needs prior to any time when the HCP
identify measures to relieve pain. may not be available or the pharmacy may
3. Pain clinics use a variety of methods to be closed.
help the client to achieve relief from pain. 2. There can be several reasons a legitimate pre-
Some measures include guided imagery, scription is not filled.
transcutaneous electrical nerve stimula- 3. Morphine needs a handwritten prescription
tion (TENS) units, nerve block surgery or on a triplicate form.
injections, or medications. 4. All medications have side effects; most
4. This is not an appropriate answer, even if the ­notably, narcotics slow peristalsis and cause
nurse is not sure. The nurse should attempt constipation.
to discover the information for the client and TEST-TAKING HINT: The test taker could
then give factual information. eliminate both options “1” and “2” because
TEST-TAKING HINT: The test taker should they are opposites. Option “4” is untrue of
answer a question with factual information. all medications.
If the stem asks for a therapeutic response, Content – Medical: Integrated Nursing Process –
then the test taker should choose one which ­Evaluation: Client Needs – Physiological Integrity,
addresses feelings. ­Pharmacological and Parenteral Therapies: Cognitive
Content – Medical: Integrated Nursing Process – Level – Synthesis: Concept – Medication.
­Implementation: Client Needs – Safe Effective Care 30. 28 mg of morphine.
Environment, Management of Care: Cognitive Level –  First, determine how many milligrams of
­Application: Concept – Comfort.
morphine are in each milliliter of saline:
28. 1. NSAIDs around the clock are dangerous 50 ÷ 250 mL = 0.2 mg/mL
­because of the potential for gastrointesti-  Then determine how many milliliters are
nal ulceration. NSAIDs are not the drug of given in a shift:
choice for cancer pain. 10 mL/hr × 12 hour = 120 mL infused
2. Morphine is the drug of choice for cancer 1 shift = 120 mL infused
pain. There is no ceiling effect, it metabo-  If each milliliter contains 0.2 milligram of
lizes without harmful by-­products, and morphine, then
it is relatively inexpensive. A sustained-  0.2 mg × 120 mL = 24 mg by constant
release formulation, such as MS Contin, infusion
is administered every six (6) to eight (8) Then determine the amount given IVP:
hours, and a liquid fast-acting form is ad- 2 × 2 = 4 mg given IVP
ministered sublingually for any pain which Finally, add the bolus amount to the amount
is not controlled. constantly infused:
3. Tylenol is not strong enough for this cli- 24 + 4 = 28 mg
ent’s pain. The maximum adult dose within a TEST-TAKING HINT: The nurse is responsible
24-hour period is four (4) g. Tylenol is toxic for being knowledgeable of all medications
to the liver in higher amounts. and the amount the client is receiving. The
4. Meperidine (Demerol) metabolizes into test taker can use the drop-down calculator
normeperidine and is not cleared by the body on the NCLEX-RN examination or ask the
rapidly. A buildup of normeperidine can cause examiner for scratch paper.
the client to seize. Content – Medical: Integrated Nursing Process –
TEST-TAKING HINT: The test taker must be Implementation: Client Needs – Physiological Integrity,
aware of medications and their uses. Pharmacological and Parenteral Therapies: Cognitive
Content – Medical: Integrated Nursing Process – ­ Level – Application: Concept – Medication.
Planning: Client Needs – Physiological Integrity,

18_Colgrove_Ch18.indd 682 02/06/16 1:32 PM


C hapter 18  E nd - of -L ife I ssues 683

31. 1. The client is in pain and has the right to have was determined for each option and realize
pain-control measures taken. option “2” needs the most experienced nurse.
2. The client is in pain. The American Content – Nursing Management: Integrated Nursing
Nurses Association Code of Ethics states Process – Planning: Client Needs – Safe Effective Care
clients have the right to die as comfortably Environment, Management of Care: Cognitive Level –
as possible even if the measures used to Synthesis: Concept – Nursing Roles.
control the pain indirectly hasten the im- 33. 1. The UAP could perform this function.
pending death. The Dying Client’s Bill of 2. The UAP could perform this function.
Rights reiterates this position. The client 3. The UAP could perform this function.
should be allowed to die with dignity and 4. The nurse should monitor any laboratory
with as much comfort as the nurse can work needed to administer a medication
provide. safely.
3. The client may be splinting to prevent the
pain from being too severe. The client’s respi- TEST-TAKING HINT: The rules for delegation
rations actually may improve when the nurse state assessment, teaching, evaluating, or
administers the pain medication. anything requiring nursing judgment cannot
4. The HCP is aware the client is unstable be delegated.
­because the HCP must write the DNR order Content – Nursing Management: Integrated Nursing
on the chart. There is no reason to withhold Process – Planning: Client Needs – Safe Effective Care
needed medication. Environment, Management of Care: Cognitive Level –
Synthesis: Concept – Nursing Roles.
TEST-TAKING HINT: The position of admin-
istering medication which could hasten a 34. 1. The TENS unit does not deaden nerve
client’s death is a difficult one and requires endings; this would be accomplished through
the nurse to be aware of ethical position local anesthesia.
statements. Nurses never administer medica- 2. The unit could stop functioning if it got wet,
tions for the purpose of hastening death but but this would not cause paralysis.
sometimes must administer medications to 3. The TENS unit works on the gate control
provide what nurses do best, comfort. theory of pain control and works by flood-
Content – Medical: Integrated Nursing Process – ing the area with stimulation and blocking
­Implementation: Client Needs – Safe Effective Care the pain impulses from reaching the brain.
Environment, Management of Care: Cognitive Level – 4. The TENS unit should be applied and left in
Synthesis: Concept – Comfort. place unless the client is showering.
32. 1. This hemoglobin is low but would be ex- TEST-TAKING HINT: A medical device which
pected for a client diagnosed with leukemia. causes paralysis so easily would not be ap-
A less experienced nurse could care for this proved for use by the general population,
client. Leukemia affects production of all cells so option “2” could be eliminated. The test
produced by the bone marrow—either there taker would need to be aware of the gate
is too much production of immature cells control theory of pain control to eliminate
overpowering the ability of the bone marrow the other options.
to use the pluripotent cells to produce other Content – Medical: Integrated Nursing Process – ­
needed blood cells or because the bone mar- Planning: Client Needs – Physiological Integrity,
row is not producing enough cells as needed. ­Physiological Adaptation: Cognitive Level – Synthesis:
Concept – Comfort.
It effectively produces a pancytopenia.
2. This represents an arterial blood gas 35. 1. These are normal arterial blood gases.
of less than 60%; this client should be 2. These temperature, pulse, and respiration
­assigned to the most experienced nurse. rates are only slightly elevated, and the blood
3. A client who needs a colostomy irrigated pressure is normal.
could be assigned to a less experienced nurse. 3. This is typical of clients with chronic pain.
4. Psychological problems come second to They cannot localize the pain and fre-
physiological ones. quently describe the pain as always being
TEST-TAKING HINT: This is a priority ques- there, as disturbing rest, and as demoral-
tion. The test taker should realize option “1” izing. This client should be seen, and ap-
is expected and may even be good for this propriate pain-control measures should
client; option “3” is expected and not life be taken.
threatening; and option “4,” although not 4. This is considered mild pain, and this client
­expected, is not life threatening. By doing can be seen after the client in chronic pain.
this, the test taker could then look at what

18_Colgrove_Ch18.indd 683 02/06/16 1:32 PM


684 M ed -S urg S uccess

TEST-TAKING HINT: Options “1” and “2” could Content – Fundamentals: Integrated Nursing Process –
be eliminated because the values are within Implementation: Client Needs – Safe Effective Care
normal limits or only slightly above normal. Environment, Management of Care: Cognitive Level –
Option “4” could be eliminated because Application: Concept – Legal.
three (3) is low on the 1-to-10 pain scale. 38. 1. The Health Insurance Portability and
Content – Medical: Integrated Nursing Process – ­ Accountability Act (HIPAA) is a federal act
Assessment: Client Needs – Safe Effective Care protecting the client’s privacy while receiving
­Environment, Management of Care: Cognitive Level – health care.
Analysis: Concept – Nursing Roles. 2. The state Nurse Practice Acts provides the
36. 1. This could be appropriate once the nurse laws which control the practice of nursing in
­assesses the situation further. each state.
2. The nurse should assess the situation 3. There is no such law as this act.
fully. The client may be afraid of becoming 4. The Good Samaritan Act protects health-
addicted or may have been using alterna- care practitioners against malpractice
tive forms of treatment, such as music claims for care provided in emergency
therapy, distraction techniques, acupunc- situations.
ture, or guided imagery. TEST-TAKING HINT: The test taker should be
3. This is not appropriate. It is in the nurse’s knowledgeable of the Good Samaritan Act
realm of responsibility to investigate the and its implications in the nurse’s profes-
­client’s reasons for not wanting to take pain sional career. The NCLEX-RN often asks
medication. questions on this act.
4. Chronic cancer pain does not resolve on Content – Fundamentals: Integrated Nursing Process –
its own. Implementation: Client Needs – Safe Effective Care
TEST-TAKING HINT: Option “1” is advising Environment, Management of Care: Cognitive Level –
without assessing. Assessment is the first Application: Concept – Legal.
step of the nursing process and should be 39. 1. This response does not support veracity.
implemented first in most situations unless 2. This response does not support veracity.
a direct intervention treats the client in an 3. The principle of veracity is the duty to
emergency. tell the truth. This response is telling the
Content – Medical: Integrated Nursing Process – ­client the truth.
­Implementation: Client Needs – Safe Effective Care 4. This response does not support veracity.
Environment, Management of Care: Cognitive Level –
TEST-TAKING HINT: The test taker must know
­Application: Concept – Assessment.
certain ethical principles, such as veracity,
beneficence, nonmalfeasance, fidelity, au-
Ethical/Legal Issues tonomy, and justice, to name a few. Without
knowing the definition of veracity, the test
37. 1. This is an example of assault, which is a mental taker would not be able to answer this ques-
or physical threat without touching the client. tion correctly.
2. When a mentally competent adult is Content – Fundamentals: Integrated Nursing Process –
forced to have a treatment he or she has Implementation: Client Needs – Safe Effective Care
refused, battery occurs. Environment, Management of Care: Cognitive Level –
3. This is fraud, a willful and purposeful mis- Application: Concept – Ethics.
representation which could cause harm to a 40. 1. The surgeon is responsible for explain-
client. ing the surgical procedure to the client;
4. This is called defamation, a divulgence of therefore, the nurse should first notify the
privileged information or communication. surgeon.
This is a violation of the Health Insurance 2. This information should be documented on
Portability and Accountability Act (HIPAA). the chart, but it is not the first intervention.
TEST-TAKING HINT: If the test taker knows 3. The operating room staff may or may not
battery is “bad,” it may lead to selecting need to be notified based on when or if the
­option “2,” which has “forcibly” in the stem. permit is being signed, but it is not the first
The test taker could attempt to eliminate intervention.
options based on knowledge. For example, 4. The nurse is not responsible for explaining
breaking confidentiality is a violation of the surgical procedure. The nurse is respon-
HIPAA; thus option “4” can be eliminated. sible for making sure the client understands
and for obtaining the consent.

18_Colgrove_Ch18.indd 684 02/06/16 1:32 PM


C hapter 18  E nd - of -L ife I ssues 685

TEST-TAKING HINT: The nurse is responsible experiments are conducted on human beings.
for getting the permit signed and on the This source does not provide direction for
chart prior to going to surgery, but the nurse the nurse addressing ethical issues.
is not responsible for explaining the proce- 3. This document informs clients and families
dure to the client. receiving home health care of the ethical con-
Content – Fundamentals: Integrated Nursing Process – duct they can expect from home care agencies
Implementation: Client Needs – Safe Effective Care and their employees when they are in the
­Environment, Safety and Infection Control: Cognitive home. This source is not the best professional
Level – Application: Concept – Nursing Roles. source for all nurses.
41. 1. Wearing their own clothes, keeping personal 4. The American Nurses Association (ANA)
items, and having a small amount of money Code of Ethics outlines to society the
are civil rights of clients in a psychiatric unit. values, concerns, and goals of the nursing
2. Seeing visitors is a civil right of the client. profession. The code provides direction
3. Receiving and sending unopened mail is a for ethical decisions and behavior by em-
civil right of the client, but any packages must phasizing the obligations and responsibili-
be inspected when the client is opening them ties which are entailed in the nurse–client
to check for sharp items, weapons, or any relationship.
type of medications. TEST-TAKING HINT: The test taker must be
4. This is a violation of the client’s rights. aware of the word “best” to be able to answer
The client has a right to have reason- this question. All four (4) answer options may
able access to a telephone and the op- or may not be potential answers, but the test
portunity to have private conversations by taker must select the option which addresses
telephone. all nurses. Option “3” should be eliminated
TEST-TAKING HINT: The test taker must be as a possible answer because it only addresses
aware of the client’s legal and civil rights. home health care.
The client in the psychiatric unit has the Content – Fundamentals: Integrated Nursing Process –
same rights as the client in the medical unit. Implementation: Client Needs – Safe Effective Care
Clients in a psychiatric hospital do not have Environment, Management of Care: Cognitive Level –
Application: Concept – Ethics.
to wear hospital gowns; they can wear their
own clothes. 44. 1. Breach of duty is one (1) of the four (4)
Content – Fundamentals: Integrated Nursing Process – elements necessary to prove nursing
Implementation: Client Needs – Safe Effective Care malpractice. It is failure to perform according
Environment, Management of Care: Cognitive Level – to the established standard of conduct.
Application: Concept – Legal. 2. This is one (1) of the four (4) steps in
42. 1. Autonomy implies the client has the right ­ethical decision making. It is not one
to make choices and decisions about his or (1) of the four (4) elements necessary to
her own care even if it may result in death prove nursing malpractice.
or is not in agreement with the health- 3. Failure to meet the standard of care resulting
care team. in an actual injury or damage to the client is
2. Self-determination is not an ethical principle. required to prove nursing malpractice.
3. Beneficence is the duty to actively do good 4. A connection must exist between conduct
for clients. and the resulting injury to prove nursing
4. Justice is the duty to treat all clients fairly. malpractice.
TEST-TAKING HINT: The test taker should be TEST-TAKING HINT: This is a knowledge-based
aware of ethical principles which mandate question, but the test taker should realize
a nurse’s behavior. Clients have rights, and ethical issues and legal issues are two differ-
­autonomy is an important principle which ent concerns and that malpractice is a legal
the nurse must ensure every client has. concern. The test taker should also know the
four (4) elements necessary to prove nursing
Content – Fundamentals: Integrated Nursing Process –
malpractice: (1) The nurse has a duty to the
Implementation: Client Needs – Safe Effective Care
­Environment, Management of Care: Cognitive Level – client. (2) The duty has been breached. The
Application: Concept – Legal. nurse failed to uphold a standard of care.
(3) There is some harm to the client. (4) The
43. 1. The Hippocratic Oath is the oath taken by breach of duty caused the harm.
medical doctors. Content – Fundamentals: Integrated Nursing Process –
2. The Nuremberg Code identifies the need for Implementation: Client Needs – Safe Effective Care
voluntary informed consent when medical

18_Colgrove_Ch18.indd 685 02/06/16 1:32 PM


686 M ed -S urg S uccess

Environment, Management of Care: Cognitive Level – 2. This is an example of nonmalfeasance, the


Comprehension: ­Concept – Legal. duty to prevent or avoid doing harm, whether
45. 1. This action should be taken, but this is not intentional or unintentional. Is it harmful for
the first action to keep the client safe. a nurse to work in an area where he or she is
2. This is a form of chemical restraint, and the not experienced?
nurse must have a health-care provider’s 3. This is an example of the ethical principle
order. of fidelity, the duty to be faithful to com-
3. This is a form of restraint and is against the mitments. It involves keeping promises and
law unless the nurse has a health-care pro- information confidential and maintaining
vider’s order. privacy.
4. The nurse must notify the health-care 4. Justice involves the duty to treat all clients
provider before putting the client in re- fairly, without regard to age, socioeco-
straints. Restraints are used in an emer- nomic status, or any other variables. Pro-
gency situation and for a limited time and viding safe and appropriate nursing care
must be for the protection of the client. to all clients is an example of justice.
TEST-TAKING HINT: The test taker must real- TEST-TAKING HINT: The test taker must be
ize when the stem asks which action is first; knowledgeable of ethical principles; they
more than one option may be appropriate for are part of the NCLEX-RN blueprint. The
the situation, but only one is implemented word “justice” should make the test taker
first. Restraining a client is considered bat- think about fairness, which might lead the
tery and is against the law unless the client test taker to select option “4” as the correct
is a danger to self and there is a health-care answer. The test taker should not automati-
provider’s order. cally think, “I don’t know the answer.” Think
about the words before selecting the correct
Content – Medical: Integrated Nursing Process –
answer.
Implementation: Client Needs – Safe Effective Care
Environment, Safety and Infection Control: Cognitive Content – Fundamentals: Integrated Nursing Process –
Level – Application: Concept – Nursing Roles. Implementation: Client Needs – Safe Effective Care
Environment, Management of Care: Cognitive Level –
46. 1. Nurse Practice Acts provide the laws which Application: Concept – Ethics.
control the practice of nursing in each
state. All states have Nurse Practice Acts. 48. 1. The law states child abuse or suspected child
2. The Client’s Bill of Rights, also known as abuse must be reported. The nurse is legally
“Your Rights as a Hospital Patient,” is a docu- responsible to report child abuse or sus-
ment which explains the client’s rights to par- pected child abuse. This is a legal issue, not
ticipate in his or her own health care; it does malpractice.
not address the nurse’s behavior. 2. Malpractice is a failure to meet the stan-
3. Each state, not the U.S. Congress, is respon- dards of care which results in harm to or
sible for writing and implementing the state’s death of a client. Failing to heed warnings
Nurse Practice Act. of shock is an example of malpractice.
4. The American Nurses Association is a volun- 3. Stealing narcotics is a legal situation, not
tary organization which provides standards of a malpractice issue. The nurse could have
care and a code of ethics. It addresses issues his or her nursing license revoked for this
in nursing, but it does not mandate the regis- illegal behavior.
tered nurse’s behavior. 4. Falsifying documents is against the law.
It is not a malpractice issue.
TEST-TAKING HINT: This is a knowledge-
based question which the test taker must TEST-TAKING HINT: The test taker must be
know. knowledgeable of malpractice. Legal issues
are dealt with by the laws of the state and
Content – Nursing Management: Integrated Nursing
federal government, and malpractice issues
Process – Implementation: Client Needs – Safe Effective
Care Environment, Management of Care: Cognitive are dealt with in the state Nurse Practice
Level – Application: Concept – Legal. Acts and in lawsuits in courts of law.
Content – Fundamentals: Integrated Nursing Process –
47. 1. This addresses the ethical principle of verac- Implementation: Client Needs – Safe Effective Care
ity. Should the nurse tell the client truthfully Environment, Management of Care: Cognitive Level –
a placebo will not help the pain? Application: Concept – Legal.

18_Colgrove_Ch18.indd 686 02/06/16 1:32 PM


C hapter 18  E nd - of -L ife I ssues 687

Organ/Tissue Donation Content – Medical: Integrated Nursing Process –


­Planning: Client Needs – Physiological Integrity, Reduction
of Risk Potential: Cognitive Level – Synthesis: Concept –
49. 1. There is a feeling during times of stress that
Grief and Loss.
organs may be distributed unfairly. Tissue
and organ banks use the United Network 51. 1. Many states require tissue and organ banks to
of O
­ rgan Sharing (UNOS) to be as fair as be notified of all deaths, but the systemic in-
possible in the allocation of organs and tis- fection eliminates this client from becoming a
sues. Organs will be given to the best match donor.
for the organ in the community where the 2. Septicemia is a systemic infection and will
donor dies. If no match is found in that area, prevent the client from donating t­issues
then the search for an HLA match will be or organs.
expanded to other areas of the country. The 3. There is no reason to notify the HCP.
recipient is chosen based on HLA match, not 4. If the client were to be an organ donor, then
fame or fortune. the client’s body would remain in the inten-
2. The client is asking for information, which sive care unit on the ventilator and with IV
the nurse should provide. medication support until the organ bank team
3. There is a definite method of allocation of arrives and takes the client to the o­perating
organs. room.
4. There are 27 known human leukocyte TEST-TAKING HINT: Option “3” could be
antigens (HLAs). HLAs have become ­eliminated from consideration because the
the principal histocompatibility system nurse should be able to handle this situa-
used to match donors and recipients. tion. Option “4” could be eliminated because
The greater the number of matches, the the client would have to stay on life support
less likely the client will reject the organ. if the organ bank were to retrieve viable
­Different races have different HLAs. organs.
TEST-TAKING HINT: Option “2” can be Content – Medical: Integrated Nursing Process –
eliminated because the client asked for Implementation: Client Needs – Safe Effective Care
­information. Option “1” can be eliminated Environment, Management of Care: Cognitive Level –
because the statement supports an unethical Application: Concept – Grief and Loss.
situation.
52. 1. Oliguria, fever, increasing edema,
Content – Medical: Integrated Nursing Process – h­ypertension, and weight gain are signs of
Implementation: Client Needs – Safe Effective Care organ rejection.
Environment, Management of Care: Cognitive Level –
2. A decrease in serum creatinine and BUN
Application: Concept – Communication.
would indicate the transplanted kidney is
50. 1. Positive brain waves on the EEG indicate functioning well.
brain activity, and the client is not brain dead. 3. Potassium and calcium are not monitored for
2. This is called the oculovestibular test. If the rejection.
client reacts, then it indicates brain activity 4. The client with a fever might have tachycar-
and the client is not brain dead. dia. Hypertension is a sign of rejection.
3. The Uniform Determination of Brain TEST-TAKING HINT: Option “2” could be
Death Act states brain death is deter- ­eliminated because of the word “decreased.”
mined by accepted medical standards If the test taker were aware of the role the
which indicate irreversible loss of all brain kidneys play in controlling blood pres-
function. Cerebral blood flow studies, sure, then option “4” could be eliminated.
EEG, and oculovestibular and oculoce- ­Decreased urine output in option “1” would
phalic tests may be done. make the most sense to choose because the
4. If the cerebral blood flow studies do not show kidneys produce urine.
acceptable blood flow to the brain, the client
Content – Surgical: Integrated Nursing Process –
will not come out of the vegetative state. ­Assessment: Client Needs – Physiological Integrity,
TEST-TAKING HINT: If the test taker examined ­Reduction of Risk Potential: Cognitive Level – Synthesis:
all answer options and did not understand Concept – Urinary Elimination.
options “1,” “2,” and “4,” then reading option
“3” again would prove it to be the best choice
because it simply states the machine won’t be
turned off u
­ ntil brain death has been proved.

18_Colgrove_Ch18.indd 687 02/06/16 1:32 PM


688 M ed -S urg S uccess

53. 1. The client must take an immune-suppressant anuria, oliguria, or polyuria and require
medication forever unless a rejection occurs, dialysis.
and then the client would die without another 3. Serum creatinine and BUN levels are moni-
transplant. tored, but there is no need to monitor the
2. Foul-smelling drainage would indicate urine osmolality.
infection and is not expected. This would 4. Hourly outputs are monitored and compared
be an emergency situation. with the intake of fluids.
3. Clients should be taught to notify the 5. The dressing is a flank dressing.
HCP immediately of any signs of an infec- TEST-TAKING HINT: The test taker should
tion. The immune-suppressant drugs will notice time frames. Anytime a specific time
mask the sign of an infection and superin- reference is provided, the test taker must
fections can develop. ­determine if the time frame is the appropri-
4. The skin turns yellow in liver failure; the ate interval for performing the activity. In
­antirejection drugs do not cause jaundice. option “4,” “every shift” is not appropriate.
TEST-TAKING HINT: Standard postoperative Content – Surgical: Integrated Nursing Process – ­
instructions include teaching the client to Planning: Client Needs – Physiological Integrity,
watch for any sign of an infection. Foul- ­Physiological Adaptation: Cognitive Level – Synthesis:
smelling drainage is never normal. Concept – Urinary Elimination.
Content – Surgical: Integrated Nursing Process – ­ 56. 1. The client would be NPO at this time and
Planning: Client Needs – Physiological Integrity, would be receiving parenteral antirejection
­Physiological Adaptation: Cognitive Level – Synthesis:
medications.
Concept – Perioperative.
2. The client would have been taken off the
54. 1. There is no pain associated with storing cord heart-lung bypass machine in the operating
blood. The blood is taken from the separated room.
placenta at birth. Forty to 150 mL of stem 3. Saturating three (3) dressing pads in one
cells can be retrieved from the umbilical vein. (1) hour would indicate hemorrhage.
2. All hospitals which have an obstetrics de- 4. Pain is expected and is not a complication of
partment should be able to assist with the the procedure.
collection of stem cells. The client should TEST-TAKING HINT: The test taker should no-
notify the HCP to be prepared with the kit to tice the time frame provided in the stem—in
obtain the specimens and to be able to send this case, three (3) hours after surgery. This
the stem cells to the Cord Blood Registry for could eliminate options “1” and “2.”
processing and storage. Content – Nursing Management: Integrated Nursing
3. There is an initial fee to process the Process – Planning: Client Needs – Safe Effective Care
stem cells and a yearly fee to maintain Environment, Management of Care: Cognitive Level –
the stored stem cells until needed. Stem Synthesis: Concept – Perioperative.
cells may be used by the infant in case of
a ­devastating illness or can be donated at 57. 1. Assessment is always the nurse’s responsibility
the discretion of the owner. and cannot be delegated. Hourly outputs are
4. This is true of stem cells which have been monitored to determine kidney function.
stored for more than 20 years. 2. The UAP can perform this function.
There is no nursing judgment required.
TEST-TAKING HINT: The test taker should
3. This requires nursing judgment and is out-
­recognize pain could not be associated with
side the UAP’s expertise.
tissue which is no longer a part of the body. 4. Irrigating a nasogastric tube for a client who
Content – Obstetrics: Integrated Nursing Process – has undergone a pancreas transplant should
­Planning: Client Needs – Safe Effective Care E
­ nvironment, be done by the nurse; this is a high-level
Management of Care: Cognitive Level – Synthesis:
nursing task.
­Concept – Pregnancy.
TEST-TAKING HINT: When asked to choose a
55. 1. Urine cultures are performed frequently task which can be delegated, the test taker
because of the bacteriuria present in the should determine which task requires the
early stages of transplantation. least amount of judgment and choose that
2. A cadaver kidney may have undergone option.
acute tubular necrosis and may not func- Content – Nursing Management: Integrated Nursing
tion for two (2) to three (3) weeks, dur- Process – Planning: Client Needs – Safe Effective Care
ing which time the client may experience Environment, Management of Care: Cognitive Level –
Synthesis: Concept – Management.

18_Colgrove_Ch18.indd 688 02/06/16 1:32 PM


C hapter 18  E nd - of -L ife I ssues 689

58. 1. This client is ready for discharge and is 2. The father would have only half of the
presumably stable. The client donated ­genetic makeup of the child.
the kidney and still has one functioning 3. There are at least 27 HLA types. A match
kidney. An experienced medical-surgical requires at least 7, and preferably 10 to
nurse could care for this client. 11 points.
2. This client must be observed closely for re- 4. This is not an acceptable match; the client
jection of the organ and is newly transferred would reject the organ.
from the intensive care unit; therefore, a TEST-TAKING HINT: If the test taker did not
more experienced nurse in transplant care know the rationale, then a choice between
should care for this client. options “1” and “2” would be the best option
3. This client has developed symptoms of a because of the direct familial relationships.
problem unrelated to the corneal transplant, Content – Surgical: Integrated Nursing Process –
but these symptoms will increase intracranial ­Planning: Client Needs – Safe Effective Care Environment,
pressure, resulting in indirect pressure to the Management of Care: Cognitive Level – Synthesis:
cornea. Therefore, a more experienced trans- Concept – Digestion.
plant nurse should care for this client.
4. This client is showing symptoms of organ 60. 1. Skin is taken from cadaver donors, so it is
rejection, which is a medical emergency and given once.
requires a more experienced transplant nurse. 2. Bones are taken from cadaver donors, so it is
given once.
TEST-TAKING HINT: The test taker should
3. A kidney can be donated while the donor is
choose the client with the fewest potential living or both can be donated as cadaver or-
problems. The nurse is experienced as a gans, but either way the donation is only once.
medical-surgical nurse, but transplant recipi- 4. The human body reproduces bone mar-
ents require more specialized knowledge. row daily. There is a bone marrow registry
Content – Nursing Management: Integrated Nursing for participants willing to undergo the
Process – Diagnosis: Client Needs – Safe Effective Care procedure to donate to clients when a
Environment, Management of Care: Cognitive Level –
match is found.
Analysis: Concept – Management.
TEST-TAKING HINT: The test taker could
59. 1. Living donors are able to donate some eliminate option “3” because the stem asks
organs. The kidneys, a portion of the liver, for repeated times and the client cannot live
and a lung may be donated, and the do- without kidney function. The client would
nor will still have functioning organs. An have to be placed on dialysis or he or she
identical twin is the best possible match. would die.
However, in the situation in this question, Content – Surgical: Integrated Nursing Process –
the identical twin would also have CF be- Implementation: Client Needs – Safe Effective Care
cause the genes would be identical. The Environment, Management of Care: Cognitive Level –
next best chance for a compatible match Analysis: Concept – Health Care System.
comes from a sibling with both parents in
common.

18_Colgrove_Ch18.indd 689 02/06/16 1:32 PM


CONCEPTS ANSWERS AND RATIONALES

61. 1. In the 1990s, Congress added the require- Content – Medical: Integrated Nursing Process –
ment for health-care facilities to offer cli- Implementation: Client Needs – Safe Effective Care
ents the opportunity to receive an advance Environment, Management of Care: Cognitive Level –
directive form and to be able to complete Application: Concept – Legal.
it to provide the health-care team with 63. 1. The nurse is caring for a client who is at
knowledge of the clients’ wishes. It was best disoriented; challenging this cogni-
added to a Medicare funding bill. tive deficiency will only create frustration
2. The client has to refuse or accept or alert the and anxiety in the client. Nonmalfeasance
facility of an intact document about advanced is the duty to prevent or do no harm. This
decisions made by the client, but referral to is a therapeutic response that validates the
a case manager is not attached to Medicare client’s concern but does not include lying
funding, to the client.
3. The client does not have to apply for supple- 2. This is veracity, to tell the truth.
mental insurance. 3. The client does not owe the nurse an expla-
4. The opportunity may include end-of-life nation of why she wishes to see her mother.
­issues but it is not limited to end of life; it “Why” is not appropriate in this situation.
does include issues of irreversible situations 4. This is the opposite of veracity; it is lying to
and surrogate decision makers. the client. If the nurse believes the client’s
TEST-TAKING HINT: The test taker could mother to be dead, then how will the nurse
eliminate option “3” because the nurse can- contact her?
not make the client do anything. The client TEST-TAKING HINT: The test taker could elim-
has a right to say no. inate option “2” because it is veracity and “4”
Content – Medical: Integrated Nursing Process – because it is lying.
­Implementation: Client Needs – Safe Effective Care Content – Medical: Integrated Nursing Process –
Environment, Management of Care: Cognitive Level – Implementation: Client Needs – Safe Effective Care
Application: Concept – Legal. Environment, Management of Care: Cognitive Level –
62. 1. Clinical death is the absence of pulse, respira- Application: Concept – Ethics.
tions, and blood pressure. It does not include 64. 1. The nurse could possibly help the family
radiology or other diagnostic tests. to guide them about the need for eventual
2. If cardiopulmonary resuscitation is unsuc- ­arrangements, but it is not appropriate ­during
cessful, the nurse cannot pronounce death. A the admission process.
physician must determine the reason for the 2. The client may or may not have pain; ­nothing
death. indicates pain is an issue in the stem of the
3. For it to be legal for a nurse to pronounce question.
death, the client must have a disease 3. Advance directives provide guidance for
process that could lead to death. The end-of-life care; the nurse needs this in-
physician must write a clear order that formation in order to plan the care per
the nurse can pronounce and be willing the client’s wishes.
to document the cause of death on the 4. This could be determined, but the priority is
death certificate. The observed clinical knowing the client’s wishes.
signs must be documented and the time
TEST-TAKING HINT: The test taker should
pronounced.
­recognize timing when reading a stem or
4. This is an incomplete entry.
option in a question. “On admission,” “every
TEST-TAKING HINT: The test taker could day,” “every two hours” will help to deter-
eliminate option “1” because clinical death mine a correct answer.
is the absence of clinical signs of life. Option Content – Medical: Integrated Nursing Process –
“3” is a complete documentation; the nurse Implementation: Client Needs – Safe Effective Care
states the facts without embellishment in Environment, Management of Care: Cognitive Level –
documentation. Application: Concept – Legal.

690

18_Colgrove_Ch18.indd 690 02/06/16 1:32 PM


C hapter 18  E nd - of -L ife I ssues 691

65. 1. Chaplains work with all faiths and are spiri- TEST-TAKING HINT: The test taker could
tual advisors. If the nurse feels comfortable eliminate option “1” because it is advising
with discussing heaven and if the client the client about how he should feel. Option
wishes to talk with the nurse, it is appropriate. “2” asks why and blames the client. Option
2. Nurses are not prohibited from discussing “4” does not address the client’s need.
spiritual issues with a client; the nurse should Content – Medical: Integrated Nursing Process –
not challenge the client’s personal beliefs. Implementation: Client Needs – Safe Effective Care
3. Hell is not what the client wants to talk Environment, Management of Care: Cognitive Level –
about. Application: Concept – Grief and Loss.
4. The nurse should allow the client to ver-
67. 1. When a death occurs the need is for the
balize his/her feeling regarding what to
nurse’s presence; just being there with
expect when death occurs.
the family is what will help the family
TEST-TAKING HINT: The nurse student is grieve.
taught in first-level courses to allow the cli- 2. The nurse may need to notify the funeral
ent to verbalize feeling; the test taker should home, but the family is the priority need.
recognize this as basic nursing skills. 3. If the family wants the minister to be called,
Content – Medical: Integrated Nursing Process – the nurse could do this, but, frequently, the
Implementation: Client Needs – Safe Effective Care family has a relationship with the minister
Environment, Management of Care: Cognitive Level – and will need to speak directly with the min-
Application: Concept – Spirituality. ister to arrange the services.
66. 1. This is false reassurance. 4. The death certificate is completed by the
2. The blame for the lack of communication physician signing it, not the nurse.
may not be the client’s; it could be all on the TEST-TAKING HINT: The test taker could
daughter. This is an accusatory statement. ­ liminate all options besides “1” because
e
3. The nurse is asking permission to divulge none of these will assist the grieving
the client’s location and health status to process.
the daughter; this is appropriate for com- Content – Medical: Integrated Nursing Process –
plying with HIPAA and is addressing the Implementation: Client Needs – Safe Effective Care
voiced concerns of the client. Environment, Management of Care: Cognitive Level –
4. The nurse can perform an intervention Application: Concept – Grief and Loss.
that directly affects the client’s situation. A
therapeutic conversation might be used if the
­client’s daughter is not willing to reconcile
with the client.

18_Colgrove_Ch18.indd 691 02/06/16 1:32 PM


692 M ed -S urg S uccess

END-OF-LIFE ISSUES
COMPREHENSIVE EXAMINATION
1. The 38-year-old client was brought to the 5. The hospice nurse is making the final visit to the
emergency department with CPR in progress wife whose husband died a little more than a year
and expired 15 minutes after arrival. Which ago. The nurse realizes the husband’s clothes are
intervention should the nurse implement for still in the closet and chest of drawers. Which
postmortem care? action should the nurse implement first?
1. Do not allow significant others to see the body. 1. Discuss what the wife is going to do with the
2. Do not remove any tubes from the body. clothes.
3. Prepare the body for the funeral home. 2. Refer the wife to a grief recovery support
4. Send the client’s clothing to the hospital group.
laundry. 3. Do not take any action because this is normal
grieving.
2. The primary nurse caring for the client who died
4. Remove the clothes from the house and dispose
is crying with the family at the bedside. Which
of them.
action should the charge nurse implement?
1. Request the primary nurse to come out in the 6. The nurse is giving an in-service on end-of life-
hall. issues. Which activity should the nurse encourage
2. Refer the nurse to the employee assistance the participants to perform?
program. 1. Discuss with another participant the death of a
3. Allow the nurse and family this time to grieve. client.
4. Ask the chaplain to relieve the nurse at the 2. Review the hospital postmortem care policy.
bedside. 3. Justify not putting the client in a shroud after
dying.
3. The nurse is discussing advance directives with
4. Write down their own beliefs about death and
the client. The client asks the nurse, “Why is this
dying.
so important to do?” Which statement would be
the nurse’s best response? 7. The 78-year-old Catholic client is in end-stage
1. “The federal government mandates this form congestive heart failure and has a DNR order.
must be completed by you.” The client has AP 50, RR 10, and BP 80/50, and
2. “This will make sure your family does what you Cheyne-Stokes respirations. Which action should
want them to do.” the nurse implement?
3. “Don’t you think it is important to let everyone 1. Bring the crash cart to the bedside.
know your final wishes?” 2. Apply oxygen via nasal cannula.
4. “Because of technology, there are many options 3. Notify a priest for last rites.
for end-of-life care.” 4. Turn the bed to face the sunset.
4. The client who is of the Jewish faith died during 8. The Hispanic client who has terminal cancer is
the night. The nurse notified the family, who requesting a curandero to come to the bedside.
do not want to come to the hospital. Which Which intervention should the nurse implement?
intervention should the nurse implement to 1. Tell the client it is against policy to allow faith
address the family’s behavior? healers.
1. Take no further action because this is an 2. Assist with planning the visit from the
accepted cultural practice. curandero.
2. Notify the hospital supervisor and report the 3. Refer the client to the pastoral care
situation immediately. department.
3. Call the local synagogue and request the rabbi 4. Determine the reason the client needs the
go to the family’s home. curandero.
4. Assume the family does not care about the
client and follow hospital protocol.

18_Colgrove_Ch18.indd 692 02/06/16 1:32 PM


C hapter 18  E nd - of -L ife I ssues 693

9. Which interventions should the nurse 13. The intensive care nurse is caring for a deceased
implement at the time of a client’s death? Select client who is an organ donor, and the organ
all that apply. donation team is en route to the hospital. Which
1. Allow gaps in the conversation at the client’s statement would be an appropriate goal of
bedside. treatment for the client?
2. Avoid giving the family advice about how to 1. The urinary output is 20 mL/hr via a Foley
grieve. catheter.
3. Tell the family the nurse understands their 2. The systolic blood pressure is greater than 90
feelings. mm Hg.
4. Explain this is God’s will to prevent further 3. The pulse oximeter reading remains between
suffering. 88% and 90%.
5. Allow the family time with the body in 4. The telemetry shows the client in sinus
private. tachycardia.
10. The male client asks the nurse, “Should I 14. The nurse is teaching a class on ethical
designate my wife as durable power of attorney principles in nursing. Which statement supports
for health care?” Which statement would be the the definition of beneficence?
nurse’s best response? 1. The duty to prevent or avoid doing harm.
1. “Yes, she should be because she is your next 2. The duty to actively do good for clients.
of kin.” 3. The duty to be faithful to commitments.
2. “Most people don’t allow their spouse to do 4. The duty to tell the truth to the clients.
this.”
15. Which action by the unlicensed assistive
3. “Will your wife be able to support your
personnel (UAP) would warrant immediate
wishes?”
intervention by the nurse?
4. “Your children are probably the best ones for
1. The UAP is holding the phone to the ear of a
the job.”
client who is a quadriplegic.
11. The client has been declared brain dead and is 2. The UAP refuses to discuss the client’s
an organ donor. The nurse is preparing the wife condition with the visitor in the room.
of the client to enter the room to say good-bye. 3. The UAP put a vest restraint on an elderly
Which information is most important for the client found wandering in the hall.
nurse to discuss with the wife? 4. The UAP is assisting the client with arthritis
1. Inform the wife the client will still be on the to open up personal mail.
ventilator.
16. The nurse is teaching a class on chronic pain
2. Instruct the wife to only stay a few minutes at
to new graduates. Which information is most
the bedside.
important for the nurse to discuss?
3. Tell the wife it is all right to talk to the client.
1. The nurse must believe the client’s report of
4. Allow another family member to go in with
pain.
the wife.
2. Clients in chronic pain may not show
12. Which client would the nurse exclude from objective signs.
being a potential organ/tissue donor? 3. Alternate pain-control therapies are used for
1. The 60-year-old female client with an chronic pain.
inoperable primary brain tumor. 4. Referral to a pain clinic may be necessary.
2. The 45-year-old female client with a
17. The client with chronic low back pain is
subarachnoid hemorrhage.
having trouble sleeping at night. Which
3. The 22-year-old male client who has been in
nonpharmacological therapy should the nurse
a motor-vehicle accident.
teach the client?
4. The 36-year-male client recently released
1. Acupuncture.
from prison.
2. Massage therapy.
3. Herbal remedies.
4. Progressive relaxation techniques.

18_Colgrove_Ch18.indd 693 02/06/16 1:32 PM


694 M ed -S urg S uccess

18. The client diagnosed with cancer is unable to 2. Assess for any spiritual distress.
attain pain relief despite receiving large amounts 3. Change the client’s position every two (2)
of narcotic medications. Which intervention hours.
should be included in the plan of care? 4. Turn on the radio to soothing music.
1. Ask the HCP to increase the medication.

19. The client diagnosed with chronic pain is laughing and joking with visitors. When the nurse asks the client
to rate the pain on a 1-to-10 scale, the client rates the pain as 10. According to the FACES® pain scale, how
would the nurse chart the client’s pain (See figure below)?
1. The client’s pain is between a zero (0) and two (2) on the FACES® scale.
2. The client’s pain is a “10” on a 1-to-10 pain scale.
3. The client is unable to accurately rate the pain on a scale.
4. The client’s pain is moderate on the pain scale.

20. The client diagnosed with diabetes mellitus 22. Which intervention should the nurse implement
type 2 wants to be an organ donor and asks the to provide culturally sensitive health care to the
nurse, “Which organs can I donate?” Which European-American Caucasian elderly client
statement is the nurse’s best response? who is terminal?
1. “It is wonderful you want to be an organ 1. Discuss health-care issues with the oldest
donor. Let’s discuss this.” male child.
2. “You can donate any organ in your body, 2. Determine if the client will be cremated or
except the pancreas.” have an earth burial.
3. “You have to donate your body to science to 3. Do not talk about death and dying in front of
be an organ donor.” the client.
4. “You cannot donate any organs, but you can 4. Encourage the client’s autonomy and answer
donate some tissues.” questions truthfully.
21. The client with multiple sclerosis who is 23. Which action by the primary nurse would
becoming very debilitated tells the home health require the unit manager to intervene?
nurse the Hemlock Society sent information on 1. The nurse uses a correction fluid to correct a
euthanasia. Which question should the nurse ask charting mistake.
the client? 2. The nurse is shredding the worksheet at the
1. “Why did you get in touch with the Hemlock end of the shift.
Society?” 3. The nurse circles an omitted medication time
2. “Did you know this is an illegal organization?” on the MAR.
3. “Who do you know who has committed 4. The nurse documents narcotic wastage with
suicide?” another nurse.
4. “What religious beliefs do you practice?”

18_Colgrove_Ch18.indd 694 02/06/16 1:32 PM


C hapter 18  E nd - of -L ife I ssues 695

24. Which action should the nurse implement for 26. The wife of a client receiving hospice care
the Chinese client’s family who are requesting to being cared for at home calls the nurse to
light incense around the dying client? report the client is restless and agitated. Which
1. Suggest the family bring potpourri instead of interventions should the nurse implement? List
incense. in order of priority.
2. Tell the client the door must be shut at all 1. Request an order from the health-care
times. provider for antianxiety medications.
3. Inform the family the scent will make the 2. Call the medical equipment company and
client nauseated. request oxygen for the client.
4. Explain the fire code does not allow any 3. Go to the home and assess the client and
burning in a hospital. address the wife’s concerns.
4. Reassure and calm the wife over the
25. The nurse is caring for the client who has active
telephone.
tuberculosis of the lungs. The client does not
5. Notify the chaplain about the client’s change
have a DNR order. The client experiences a
in status.
cardiac arrest, and there is no resuscitation mask
at the bedside. The nurse waits for the crash cart
before beginning resuscitation. According to the
ANA Code of Ethics for Nurses (see Table 18-1),
which disciplinary action should be taken against
the nurse?
1. Report the action to the State Board of Nurse
Examiners.
2. The nurse should be terminated for failure to
perform duties.
3. No disciplinary action should be taken against
the nurse.
4. Refer the nurse to the American Nurses
Association.

Table 18-1 The American Nurses Association Code of Ethics for Nurses

• The nurse practices with compassion and respect for the inherent dignity, worth, and uniqueattributes of every person.
• The nurse’s primary commitment is to the patient, whether an individual, family, group, community, or population.
• The nurse promotes, advocates for, and protects the rights, health, and safety of the patient.
• The nurse has authority, accountability, and responsibility for nursing practice; makes decisions; and takes action
consistent with the obligation to promote health and to provide optimal care.
• The nurse owes the same duties to self as to others, including the responsibility to promote health and safety, pre-
serve wholeness of character and integrity, maintain competence, and continue person and professional growth.
• The nurse, through individual and collective effort, establishes, maintains, and improves the ethical environment
of the work setting and conditions of employment that are conducive to safe, quality health care.
• The nurse, in all roles and settings, advances the profession through research and scholarly inquiry, professional
s­ tandards development, and the generation of both nursing and health policy.
• The nurse collaborates with other health professionals and the public to protect human rights, promote health
­diplomacy, and reduce health disparities.
• The profession of nursing collectively through its professional organizations, must articulate nursing values, maintain
the integrity of the profession, and integrate principles of social justic into nursing and health policy.

The American Nurses Association Code of Ethics for Nurses with Interpretative Statements. Copyright 2015, American Nurses Publishing,
A
­ merican Nurses Foundation/American Nurses Association, Washington, DC. Reprinted with permission.

18_Colgrove_Ch18.indd 695 02/06/16 1:32 PM


END-OF-LIFE ISSUES COMPREHENSIVE
EXAMINATION ANSWERS AND RATIONALES

1. 1. There is no reason the family members should 4. Technology now allows for the body to
not be able to see the client; this is important maintain life functions indefinitely in some
to allow the significant others closure. futile situations. ADs allow clients to make
2. This death should be reported to the medi- decisions, which hopefully will be honored
cal examiner because the death ­occurred at the time of their death.
less than 24 hours after hospital admission Content – Fundamentals: Integrated Nursing Process –
and an autopsy may be required. Therefore, Implementation: Client Needs – Safe Effective Care
the nurse must leave all tubes in place; the ­Environment, Management of Care: Cognitive Level –
medical examiner will remove the tubes. ­Application: Concept – Communication.
3. This is a medical examiner case, and the nurse 4. 1. Many of the Jewish faith do not believe in
should not prepare the body by removing viewing or touching the dead body. The
tubes or washing the body prior to taking the body is sent to the funeral home for burial
client to a funeral home. within 24 hours, and a closed casket is
4. The client’s clothing should be given to the preferred.
family or to the police if foul play is suspected. 2. The hospital supervisor does not need to be
Content – Medical: Integrated Nursing Process – notified the family did not want to come to the
­Implementation: Client Needs – Safe Effective Care hospital.
­Environment, Management of Care: Cognitive Level –
3. The nurse needs to take care of the client, not
­Synthesis: Concept – Grief and Loss.
the family, and should not call to request a
2. 1. The nurse is providing care for the family and rabbi to go visit the family.
should not have to leave the bedside. 4. The nurse must be aware of cultural differences
2. An employee assistance program is available at and not be judgmental.
many facilities for counseling employees who Content – Fundamentals: Integrated Nursing Process –
are having psychosocial issues, but this nurse is Implementation: Client Needs – Psychosocial Integrity:
being humane. Cognitive Level – Application: Concept – Diversity.
3. Crying was once considered unprofes-
5. 1. The nurse must first confront the wife
sional, but today it is recognized as simply
about moving on through the grieving
an expression of empathy and caring.
­process. After one (1) year, the wife should
4. The chaplain may come to the client’s room
be seriously thinking about what to do with
and offer support but should not relieve the
her husband’s belongings.
nurse who has developed a therapeutic nurse–
2. This is an appropriate intervention, but the
client relationship with the client.
nurse must first talk directly to the client.
Content – Nursing Management: Integrated Nursing 3. After one (1) year, the wife should be progress-
­Process – Implementation: Client Needs – Safe Effective
ing through the grieving process and needs
Care Environment, Management of Care: Cognitive Level –
Application: Concept – Grief and Loss.
encouragement to remove her husband’s
belongings.
3. 1. Advance directives (AD) are not mandated by 4. This will need to be done at some point, but
the federal government. The nurse must dis- it is not the nurse’s responsibility. This action
cuss this with the client, but the client does not is crossing professional boundaries unless the
have to complete it. wife asks the nurse to do this.
2. ADs can be overridden by the family because Content – Medical: Integrated Nursing Process –
the health-care provider is worried about be- ­Implementation: Client Needs – Psychosocial Integrity:
ing sued by family survivors. ­Cognitive Level – Application: Concept – Grief and Loss.
3. This response is not answering the client’s
question and it is argumentative.

696

18_Colgrove_Ch18.indd 696 02/06/16 1:32 PM


C hapter 18  E nd - of -L ife I ssues 697

6. 1. This activity will not help the nurse address his 3. The nurse should not tell the family he or she
or her own fear of death. understands; even if the test taker has lost a
2. This activity will not help the nurse address his loved one, the test taker should never select
or her own fear of death. an option which says the nurse understands
3. This activity will not help the nurse address his another person’s feelings.
or her own fear of death. 4. This is projecting the nurse’s personal reli-
4. Many nurses are reluctant to discuss death gious beliefs on the family and could cause
openly with their clients because of their more anger at God when the family needs to
own anxieties about death. Therefore, com- be able to draw on their own spiritual beliefs.
ing face to face with the nurse’s own mor- 5. The family needs time for closure, and al-
tality will address the fear of death. lowing the family to stay at the bedside is
Content – Medical: Integrated Nursing Process – meeting the family’s need to say good-bye.
­Planning: Client Needs – Psychosocial Integrity: Cognitive Content – Medical: Integrated Nursing Process –
Level – Synthesis: Concept – Communication. ­Implementation: Client Needs – Psychosocial Integrity:
Cognitive Level – Application: Concept – Grief and Loss.
7. 1. The client has a DNR; therefore, there is no
need to bring the crash cart to the bedside. 10. 1. The client can designate anyone he wishes to
2. The client has a DNR and the nurse needs to be the durable power of attorney.
help the client die peacefully. 2. This is not true; many spouses are designated as
3. The Catholic religion requires last rites the durable power of attorney for health care.
be performed immediately before or ­after 3. No matter who the client selects as
death. the power of attorney, the most impor-
4. The client is Catholic, and there is no specific tant a­ spect is to make sure the person,
way for the bed to be placed. whether it be the wife, child, or friend,
Content – Medical: Integrated Nursing Process – will honor the client’s wishes no matter
­Implementation: Client Needs – Psychosocial Integrity: what happens.
Cognitive Level – Application: Concept – Diversity. 4. The children must be at least 18 years old
and willing to honor the client’s wishes.
8. 1. The hospital should not prevent the client
from practicing his or her culture, and deny- Content – Medical: Integrated Nursing Process –
­Implementation: Client Needs – Psychosocial Integrity:
ing faith healers would be denying the client’s
Cognitive Level – Application: Concept – Grief and Loss.
spiritual guidance.
2. The nurse should support the client’s 11. 1. This is the most important action
­culture as long as it is not contraindicated ­because, when the wife walks in the room,
in the client’s care. This client is terminal; the client’s chest will be rising and falling,
therefore, allowing the curandero, who is the monitor will show a heartbeat, and the
a folk healer and religious person in the client will be warm. Many family members
Hispanic culture, would be appropriate. do not realize this and think the client is
3. There is no reason to refer this client to the still alive. The organs must be perfused
pastoral care department; the nurse can assist until retrieved for organ donation.
the client. 2. The wife should be encouraged to stay a
4. The nurse does not need to know why the short time and leave the facility before the
­client wants the curandero; the nurse should client is taken to the operating room, but it is
support the client’s request without prejudice. not the most important intervention.
Content – Medical: Integrated Nursing Process – 3. It is all right for the wife to talk to the cli-
­Implementation: Client Needs – Psychosocial Integrity: ent, but because the client is brain dead and
Cognitive Level – Application: Concept – Diversity. cannot hear her, it is not the most important
intervention.
9. 1. The nurse needs to be sensitive to the
4. It is all right for another family member to go
­family, and simply being present to support
into the room, but it is not the most impor-
the family emotionally is important; the
tant intervention.
nurse does not have to talk.
2. The nurse should avoid the impulse to give
advice; each person grieves in his or her
own way.

18_Colgrove_Ch18.indd 697 02/06/16 1:32 PM


698 M ed -S urg S uccess

Content – Medical: Integrated Nursing Process – 4. The client has a right to send and receive
­ lanning: Client Needs – Psychosocial Integrity:
P mail, and the UAP is helping the client open
Cognitive Level – Synthesis: Concept – Grief and Loss. the mail; therefore, this does not require im-
12. 1. Primary brain tumors rarely metastasize out- mediate intervention.
side the skull, and this client can be a donor; Content – Management: Integrated Nursing Process –
cancers other than primary brain ­tumors pre- Implementation: Client Needs – Safe Effective Care
vent organ/tissue donation. Environment, Management of Care: Cognitive Level –
2. This is an excellent potential donor because Synthesis: Concept – Management.
all other organs are probably healthy. 16. 1. The most important information for a
3. This is an excellent candidate because this is nurse caring for a client with acute or
a young person with a traumatic death, not a chronic pain is to believe the client. Pain
chronic illness. is subjective, and the nurse should not be
4. A male client who has been in prison judgmental.
is at risk for being HIV positive, which 2. This is a true statement because the client’s
­excludes him from being an organ/tissue sympathetic nervous system cannot remain
donor. in a continual state of readiness. This results
Content – Medical: Integrated Nursing Process – in no objective data to support the pain and a
Diagnosis: Client Needs – Psychosocial Integrity: Cognitive normal pulse and blood pressure. However, it
Level – Analysis: Concept – Ethics. is not the most important information a new
13. 1. The urinary output should be at least graduate should know.
30 mL/hr. 3. Transcutaneous electrical nerve stimulation
2. The systolic blood pressure must be (TENS), distraction, imagery, acupuncture,
maintained at this rate to keep the client’s and acupressure are all alternate pain thera-
organs perfused until removal. pies which may be used for chronic pain, but
3. The pulse oximeter should be greater than it is not the most important information the
93%. new graduate should know.
4. The client’s heart must be beating, but it 4. Pain clinics treat clients with chronic pain,
can be normal sinus rhythm or even sinus but it is not the most important information a
bradycardia. new graduate should know.
Content – Surgical: Integrated Nursing Process – Content – Medical: Integrated Nursing Process –
­Diagnosis: Client Needs – Physiological Integrity, Reduction Planning: Client Needs – Safe Effective Care Environment,
of Risk Potential: Cognitive Level – Analysis: Concept – Management of Care: Cognitive Level – Synthesis:
Grief and Loss. Concept – Comfort.

14. 1. This is the ethical principle of 17. 1. Acupuncture is an alternative therapy, but a
nonmalfeasance. nurse cannot teach it and the client cannot do
2. This is the ethical principle of this to himself or herself.
beneficence. 2. A client cannot perform massage therapy on
3. This is the ethical principle of fidelity. himself or herself.
4. This is the ethical principle of veracity. 3. The nurse should not prescribe herbal
remedies.
Content – Fundamentals: Integrated Nursing Process –
4. Progressive relaxation techniques involve
Implementation: Client Needs – Safe Effective Care
­Environment, Management of Care: Cognitive Level – visualizing a specific muscle group and
Application: Concept – Ethics. mentally relaxing each muscle; this can be
taught to the client, and it will allow the
15. 1. The client has a right to private phone con- client to relax, which will foster sleep.
versations but, because the client is a quad- Content – Medical: Integrated Nursing Process –
riplegic, holding the phone to the ear does Planning: Client Needs – Physiological Integrity,
not require immediate intervention. ­Physiological Adaptation: Cognitive Level – Synthesis:
2. This is the appropriate action for the UAP ­Concept – Comfort.
and should be praised.
3. Restraints are not allowed unless there
is a health-care provider’s order with
documentation by the nurse of the client
being a danger to himself or others. The
UAP’s putting the client in restraints war-
rants immediate intervention because it is
battery.

18_Colgrove_Ch18.indd 698 02/06/16 1:32 PM


C hapter 18  E nd - of -L ife I ssues 699

18. 1. The client is already receiving large amounts 21. 1. The nurse should not ask the client “why” he
of medication. The nurse should assess for or she does something; this is judgmental.
other causes of pain. 2. This answer option is giving erroneous infor-
2. Pain has many components, and spiritual mation because it is not illegal; it is an orga-
distress or psychosocial needs will affect nization which supports active euthanasia.
the client’s perception of pain; remember, 3. This question is not relevant to the situation.
assessment is the first step of the nursing 4. This question must be asked because
process. Judeo-Christian belief supports the view
3. Usually clients will naturally assume the most that suicide is a violation of natural law
comfortable position, and forcing them to and the laws of God. The tenets of the
move may increase their pain. Hemlock Society are in direct opposition
4. The client may or may not like this type to Judeo-Christian beliefs. If the client is
of music, but it would not be the first agnostic, then this organization may be
intervention. helpful to the client.
Content – Medical: Integrated Nursing Process – Content – Medical: Integrated Nursing Process –
Planning: Client Needs – Physiological Integrity, ­Implementation: Client Needs – Psychosocial Integrity:
­Pharmacological and Parenteral Therapies: Cognitive Cognitive Level – Application: Concept – Diversity.
Level – Synthesis: Concept – Comfort.
22. 1. Many Middle Eastern cultures practice this,
19. 1. The Faces pain scale was devised to help chil- but the Caucasian culture does not.
dren identify pain when they are unable to 2. Caucasians as a culture do not necessarily
understand the concept of numbers. The nurse have a preference, but this does not affect
can use this pain scale when caring for adults ­culturally sensitive health care.
who are unable to use the 1-to-10 numerical 3. Frequently Caucasians do not like to talk
scale. This client rated the pain at a 10. about death and dying, but this is an indi-
2. Pain is whatever the client says it is and vidual preference of the client and the nurse
­occurs whenever the client says it does. should allow the discussion.
Pain is a wholly subjective symptom, and 4. The western Caucasian society values
the nurse should not question the client’s autonomy and truth telling in individual
perception of pain. The client’s pain is a 10. decision making.
3. The client did rate the pain on the pain scale. Content – Medical: Integrated Nursing Process –
Laughing and talking with visitors may occur ­Implementation: Client Needs – Psychosocial Integrity:
with excruciating chronic pain. The client in Cognitive Level – Application: Concept – Diversity.
chronic pain must learn to adapt to pain and
try to live as normal a life as possible. 23. 1. The client’s chart is a legal document, and
if a mistake occurs, it should be corrected
4. The client rated the pain at a 10.
by marking one line through the entry in
Content – Medical: Integrated Nursing Process –
such a way the entry can still be read in a
­Implementation: Client Needs – Safe Effective Care
­Environment, Management of Care: Cognitive Level –
court of law. Erasing, using a correction
Application: Concept – Comfort. fluid, or obliterating the entry is illegal.
2. This is the correct method for disposing of
20. 1. This is not answering the client’s question. any paper which has client information on it
2. A client with type 2 diabetes has organ dam- which is not a part of the client’s permanent
age as a result of the high glucose over time; medical record.
therefore, most organs are not usable. 3. This is the correct method to indicate a
3. This is a false statement. The client does not medication was not administered to the cli-
have to will his or her body to science to be a ent; the circle means the person should go to
tissue/organ donor. the nurse’s notes to read the reason why the
4. The client can donate corneas, skin, and some medication was not administered.
joints, but organ donation from clients with 4. All narcotics not administered to the client
type 2 diabetes mellitus usually is not allowed. must be verified when being wasted and then
Content – Medical: Integrated Nursing Process – documented.
­Implementation: Client Needs – Safe Effective Care Content – Nursing Management: Integrated Nursing
­Environment, Management of Care: Cognitive Level – Process – Implementation: Client Needs – Safe Effective
Analysis: Concept – Metabolism. Care Environment, Management of Care: Cognitive
Level – Application: Concept – Management.

18_Colgrove_Ch18.indd 699 02/06/16 1:32 PM


700 M ed -S urg S uccess

24. 1. The nurse must support the client’s 4. The ANA cannot discipline nurses; it is a
­culture. Potpourri provides the scent ­voluntary nurse’s organization.
without having the burning incense, which Content – Medical: Integrated Nursing Process –
is against fire code, and thus is a compro- ­Implementation: Client Needs – Safe Effective Care
mise which supports the client’s culture. Environment, Management of Care: Cognitive Level –
2. Having the door shut does not matter; open Application: Concept – Ethics.
flames are not allowed in any health-care 26. In order of priority: 4, 3, 2, 1, 5.
facility. 4. The nurse should calm and reassure the
3. This is not necessarily true, and if it is part wife over the telephone.
of the cultural beliefs about dying, then the 3. The nurse should then visit the cli-
nurse should medicate the client if he or she ent immediately to assess the change in
becomes nauseated. condition.
4. This is a fact, but the nurse should attempt 2. Restlessness and agitation are symptoms
to compromise and support the client and of lack of oxygen. Therefore, calling the
­family’s cultural needs, especially at the time medical equipment company to send
of death. ­oxygen would be the next intervention.
Content – Medical: Integrated Nursing Process – 1. Terminal restlessness is difficult for the
­Implementation: Client Needs – Psychosocial Integrity: family to watch and the client to experi-
Cognitive Level – Application: Concept – Diversity.
ence, so antianxiety medications would be
25. 1. There is no need to report this action to the the next logical intervention.
state board; this is not malpractice. 5. Referral to the chaplain is needed ­because
2. This action does not warrant the nurse being death may be imminent.
terminated. Content – Medical: Integrated Nursing Process –
3. The Code states, “The nurse owes the ­Implementation: Client Needs – Safe Effective Care
same duty to self as to others, including ­Environment, Management of Care: Cognitive Level –
the responsibility to preserve integrity and Analysis: Concept – Nursing Roles.
safety.” Therefore, if the nurse realizes he
or she could contract TB if unprotected
mouth-to-mouth resuscitation is per-
formed, then not doing this action does
not violate the Code of Ethics.

18_Colgrove_Ch18.indd 700 02/06/16 1:32 PM


Pharmacology
Education makes a people easy to lead, but difficult to drive; easy
to govern, but impossible to enslave.
19
—Lord Brougham

This chapter contains test-taking hints specific to pharmacology-related questions. Many


of the general hints discussed in Chapter 1 provided with the answer rationales in the other
chapters are also helpful. Remember, however, test-taking hints are useful for discriminating
information and choosing among answer options, but they cannot substitute for knowledge.
Nurses must be familiar with medications—their specific uses, modes of administration, side
effects, possible adverse reactions, and ways to gauge their effectiveness in treating specific
disorders/diseases.

KEYWORDS ABBREVIATIONS
Agranulocytosis Apical pulse (AP)
Ataxia As needed (prn)
Doll’s eye test Beats per minute (bpm)
Echinacea By mouth (PO)
Mydriasis Computed tomography (CT)
Tetany Gastrointestinal (GI)
Health-care provider (HCP)
Hour of sleep (h.s.)
Intramuscular (IM)
Intravenous (IV)
Intravenous push (IVP)
Licensed practical nurse (LPN)
Medication administration record (MAR)
Nasogastric (N/G)
Nonsteroidal anti-inflammatory drug (NSAID)
Over-the-counter (OTC)
Supraventricular tachycardia (SVT)

701

19_Colgrove_Ch19.indd 701 02/06/16 4:02 PM


702 M ed -S urg S uccess

TEST-TAKING HINTS FOR PHARMACOLOGY


QUESTIONS
The test taker must know medications and memorize specific facts about the different medi-
cations, including their uses, dosages, and side effects. This knowledge is part of adminis-
tering medications safely. There are some specific tips to assist the test taker to learn about
medications, and they will apply to the 101 questions in this chapter.
It is important to learn the different classifications of drugs—for example, diuretics, an-
tibiotics, nonsteroidal anti-inflammatory drugs (NSAIDs). The test taker should learn the
actions, uses, side effects, adverse effects, possible interactions, and method of administra-
tion (for example, oral, intravenous, intramuscular) of these drugs. Generally speaking, the
various drugs in each classification will be similar in these factors.
Do not be too broad in the classifications. For example, do not combine all medications
administered for hypertension in the same category. Angiotensin-converting enzyme (ACE)
inhibitors, beta blockers, and calcium channel blockers, for example, are all used to treat
hypertension, but they are different categories of medications, acting differently in the body
and producing different effects. Diuretics and oral medications for diabetes mellitus fall into
different specific classifications and must be learned by the specific classification. Each clas-
sification has its own effects on the body, side effects, and adverse effects, and each has steps
the nurse must take before administering the medication.
When administering medications for a group of clients, the test taker must realize time is
a realistic problem. The nurse will be unable to look up 50 to 60 medications and administer
them all within the dosing time frame, so it is imperative that nurses learn the most common
medications.
One tip for learning the medications is for the test taker to complete handmade drug
cards. This is better than buying ready-made cards because, in making the card, the test
taker must involve more than one sense—reading, deciding which information to put on the
card, and writing the pertinent information. Using more than one sense will assist the test
taker to memorize the information.
A change has been made to the NCLEX examination that the student nurse and new
graduate must recognize: The examination has few to no classifications of medications and
no trade names are used in the questions. The student nurse should focus on learning the
medications solely by their generic name. The suffix of the word can give the test taker
a hint as to the drug classification, such as “ol,” atenolol or propranolol—Tenormin and
Inderal—respectively, are both beta blockers and as such have similar administration guide-
lines. Other suffixes include “il” for ACE inhibitors, “pam” for benzodiazepines, “toin” for
hydantoins, and “parin” for low molecular weight heparins.

Drug Cards
When the test taker is deciding which information is the most important to write on a drug
card, there are five (5) questions which can be used as a guide.
1. What is the scientific rationale for administering the medication?
The test taker should always ask “why” this intervention is being implemented.
• What classification is the medication the nurse is administering to the client?
• Why is this client receiving this medication?
• What action does the medication have in the body?
The answers to these questions provide the scientific rationale for administering the med-
ication. It is also important to remember in many cases a medication may be in a particular
classification but the client is receiving the medication for a different reason—for example,
the anticonvulsant depakote (Tegretol) is also administered as an antimania medication.

19_Colgrove_Ch19.indd 702 02/06/16 4:02 PM


C hapter 19  P harmacology 703

Example #1
Digoxin (Lanoxin) 0.25 mg PO
• The classification of this medication is a cardiac glycoside.
• The medication is administered to clients with congestive heart failure or rapid atrial
fibrillation.
• Cardiac glycosides increase the contractility of the heart and decrease the heart rate.

Example #2
Furosemide (Lasix) 40 mg IVP
• The classification of this medication is a loop diuretic.
• The medication is administered to clients with essential hypertension.
• This medication helps remove excess fluid from the body.
• Loop diuretics remove water from the kidneys along with potassium.
2. When should the administration of a medication be questioned?
• Does the medication have a therapeutic serum level?
• Which vital signs must be monitored?
• Which physiological parameters should be monitored when the medication is being
administered?
The answers to these questions will provide the nurse with information to base a decision
on which medication orders should be questioned.

Example #1
Digoxin (Lanoxin)
• Is the apical pulse less than 60 bpm?
• Is the digoxin level within the therapeutic range?
• Is the potassium level within normal range?

Example #2
Furosemide (Lasix)
• Is the potassium level within normal range?
• Does the client have signs/symptoms of dehydration?
• Is the client’s blood pressure below 90/60?
3. How can the nurse ensure the safety of the administration of medications?
• What interventions must be taught to the client to ensure the medication is adminis-
tered safely in the hospital setting?
• What interventions must be taught for taking the medication safely at home?

Example #1
Digoxin (Lanoxin)
• Explain to the client the importance of getting serum levels regularly.
• Teach the client to take the radial pulse and to not take the medication if the pulse is less
than 60.
• Tell the client to take the medication daily or as ordered and notify the HCP if not taking
the medication.

Example #2
Furosemide (Lasix)
• Teach the client about orthostatic hypotension.
• Instruct the client to drink water to replace insensible fluid loss.
• Because the medication is intravenous push (IVP), how many minutes should the medica-
tion be pushed over? What primary IV is hanging? Is it compatible with Lasix?

19_Colgrove_Ch19.indd 703 02/06/16 4:02 PM


704 M ed -S urg S uccess

4. What are the possible side effects and possible adverse reactions associated with a
specific medication?
• What are the side effects of this medication?
• What are the possible adverse reactions associated with this medication?
Side effects are not expected but are not unusual. Adverse reactions are any situations
which would require notifying the health-care provider or discontinuing the medication.

Example #1
Digoxin (Lanoxin)
• Inform the client of the signs of toxicity, which are nausea, vomiting, anorexia, and yellow
haze.

Example #2
Furosemide (Lasix)
• Side effects include dizziness and light-headedness.
• Adverse effects include hypokalemia and tinnitus (if Lasix is administered too quickly).

5. How can the effectiveness of a medication be monitored?

Example #1
Digoxin (Lanoxin)
• Have the signs/symptoms of congestive heart failure improved?
• Is the client able to breathe easier?
• How many pillows does the client need to sleep on when lying down?
• Is the client able to perform activities of daily living without shortness of breath?

Example #2
Furosemide (Lasix)
• Is the client’s urinary output greater than the intake?
• Has the client lost any weight?
• Does the client have sacral or peripheral edema?
• Does the client have jugular vein distention?
• Is the client’s blood pressure decreased?

Sample Drug Cards

Front of Card
Classification of Drug:       Route:
Action of Drug:
Uses:
Nursing Implications (When would I question giving the medication?)
How will I monitor to see if it is working?

Back of Card
Side Effects:
Teaching Needs:
Drug Names:
It is suggested the test taker complete these cards from a pharmacology textbook and not
a drug handbook because most test questions come from a pharmacology book.

19_Colgrove_Ch19.indd 704 02/06/16 4:02 PM


C hapter 19  P harmacology 705

Example #1: Digoxin

Front of Card
Classification of Drug: Cardiac Glycosides Route: PO/IV
Action: Positive inotropic action; increases force of ventricular contraction and thereby
increases cardiac output; slows the heart, allowing for increased filling time.
Uses: Congestive heart failure and rapid atrial cardiac dysrhythmias.
Nursing Implications: Check apical pulse for 1 full minute, hold if less than 60. Check
digoxin level (0.5–2 normal; greater than 2 is toxic). Check K+ level (3.5–5.5 mEq/L is
normal). Hypokalemia is the most common cause of dysrhythmias in clients receiving
digoxin. Monitor for S/S of CHF, crackles in lungs, I & O, edema. Question if the AP is
less than 60 or abnormal laboratory values.
IVP more than 5 minutes: maintenance dose 0.125–0.25 mg q day.
Effective: Breathing improves, activity tolerance improves, atrial rate decreases.

Back of Card
Side Effects: Toxic = yellow haze or nausea and vomiting, ventricular rate decreases. If a
diuretic is given simultaneously, might increase the likelihood of hypokalemia.
Teaching Needs: To take pulse and hold digoxin if it is less than 60 and notify HCP.
K+ replacement: Eat food high in K+ or may need supplemental K+.
Report weight gain of 3 lb or more.
Drug names: Digoxin (Generic)
Lanoxin
Lanoxicaps

Example #2: Furosemide

Front of Card
Classification of Drug: Loop Diuretic Route: PO/IVP
Actions: Blocks reabsorption of sodium and chloride in the loop of Henle, which prevents
the passive reabsorption of water and leads to diuresis.
Uses: CHF, fluid volume overload, pulmonary edema, HTN.
Nursing Implications: I & O, monitor K+ level, check skin turgor, monitor for leg cramps,
provide K+-rich foods or supplements, give early in the day to prevent nocturia.
If giving IVP: Give at prescribed rate (Lasix 20 mg/min), ototoxic if given faster.
Effective: Decrease in weight, output greater than intake, less edema, lungs sound clear.

Back of Card
Side Effects: Hypokalemia, muscle cramps, hyponatremia, dehydration.
Teaching Needs: Take early in the day.
Eat foods high in K+.
Drug Names: Furosemide (Lasix)
Bumetanide (Bumex)
Torsemide (Demadex)
Ethacrynic acid (Edecrin)

19_Colgrove_Ch19.indd 705 02/06/16 4:02 PM


706 M ed -S urg S uccess

MEDICATIONS ADMINISTRATION
IN A MEDICAL-SURGICAL SETTING
COMPREHENSIVE EXAMINATION
The test taker is encouraged to use the guidelines/test-taking hints given previously when taking the following
medication test. The test is comprehensive for medications administered in a medical-surgical setting.

1. The client asks the clinic nurse if he should take 5. The client diagnosed with asthma is prescribed
2,000 mg of vitamin C a day to prevent getting the mast cell inhibitor cromolyn. Which
a cold. On which scientific rationale should the statement by the client indicates the need for
nurse base the response? further teaching?
1. Vitamin C in this dosage will help cure the 1. “I will take two puffs of my inhaler before
common cold. I exercise.”
2. This vitamin must be taken with echinacea to 2. “I will rinse my mouth with water after taking
be effective. the medication.”
3. This dose of vitamin C is not high enough 3. “After inhaling the medication, I will hold my
to help prevent colds. breath for 10 seconds.”
4. Megadoses of vitamin C may cause crystals 4. “When I start to wheeze, I will use my inhaler
to form in the urine. immediately.”
2. The client recently has had a myocardial 6. The client diagnosed with methicillin-resistant
infarction. Which medications should the nurse Staphylococcus aureus (MRSA) is receiving the
anticipate the health-care provider recommending aminoglycoside antibiotic vancomycin. Peak and
to prevent another heart attack? trough levels are ordered for the dose the nurse
1. Vitamin K and a nonsteroidal anti- is administering. Which priority intervention
inflammatory drug. should the nurse implement?
2. Vitamin E and a daily low-dose aspirin. 1. Ask the client if he has had any diarrhea.
3. Vitamin A and an anticoagulant. 2. Monitor the aminoglycoside peak level.
4. Vitamin B complex and an iron supplement. 3. Determine if the trough level has been drawn.
4. Check the client’s culture and sensitivity report.
3. The client diagnosed with essential hypertension
calls the clinic and tells the nurse she needs 7. The nurse is caring for an elderly client who is
something for the flu. Which information should eight (8) hours postoperative hip replacement and
the nurse tell the client? is reporting incisional pain. Which intervention is
1. OTC medications for the flu should not be priority for this client?
taken because of your hypertension. 1. Assist the client to sit in the bedside chair.
2. If OTC medications do not relieve symptoms 2. Initiate pain medication at the lowest dose.
within three (3) days, contact the HCP. 3. Assess the client’s pupil size and
3. Tell the client to ask the pharmacist to accommodation.
recommend an OTC medication for the flu. 4. Monitor the client’s urinary output hourly.
4. Make an appointment for the client to receive 8. The client is diagnosed with pernicious anemia.
the influenza vaccine. Which health-care provider order should the
4. Which laboratory test should the nurse monitor nurse anticipate in treating this condition?
for the client receiving the intravenous steroid 1. Subcutaneous iron dextran.
Solu-Medrol? 2. Intramuscular vitamin B12.
1. Potassium level. 3. Intravenous folic acid.
2. Sputum culture and sensitivity. 4. Oral thiamine medication.
3. Glucose level. 9. The client with type 2 diabetes mellitus is
4. Arterial blood gases. prescribed glyburide (Micronase), a sulfonylurea.
Which statement indicates the client understands
the medication teaching?
1. “I should carry some hard candy when I go
walking.”
2. “I must take my insulin injection every morning.”
3. “There are no side effects I need to worry about.”
4. “This medication will make my muscles absorb
insulin.”

19_Colgrove_Ch19.indd 706 02/06/16 4:02 PM


C hapter 19  P harmacology 707

10. The unlicensed assistive personnel (UAP) 15. The client is receiving a continuous intravenous
reports the client’s glucometer reading is infusion of heparin, an anticoagulant. Based on
380 mg/dL. The client is on regular sliding-scale the most recent laboratory data:
insulin which reads:
PT 13.2 Control: 12.1
Glucometer Reading Units of Insulin
INR 1.3
Less than 150 0
PTT 72 Control: 39
151–250 5
251–350 8 Which action should the nurse implement?
351–450 10 1. Continue to monitor the infusion.
Notify the HCP
2. Prepare to administer protamine sulfate.
451+
3. Have the laboratory reconfirm the results.
How much insulin should the nurse administer 4. Assess the client for bleeding.
to the client? _________ 16. The elderly client is admitted to the emergency
11. The nurse administers 18 units of Humulin department from a long-term care facility. The
N, an intermediate-acting insulin, at 1630. client has multiple ecchymotic areas on the
Which priority intervention should the nurse body. The client is receiving digoxin, a cardiac
implement? glycoside; Lasix, a loop diuretic; Coumadin,
1. Monitor the client’s hemoglobin A1c. an anticoagulant; and Xanax, an antianxiety
2. Make sure the client eats the evening meal. medication. Which order should the nurse
3. Check the a.c. blood glucometer reading. request from the health-care provider?
4. Ensure the client eats a snack. 1. A STAT serum potassium level.
2. An order to admit to the hospital for
12. The nurse is administering the following observation.
1800 medications. Which medication should 3. An order to administer Valium intravenous
the nurse question before administering? push.
1. The sliding-scale insulin to the client who has 4. A STAT international normalized ratio (INR).
just been released to have the evening meal.
2. The antibiotic to the client who is one (1) day 17. The client with postmenopausal osteoporosis
postoperative exploratory abdominal surgery. is prescribed the bisphosphonate alendronate
3. Metformin (Glucophage), a biguanide, to the (Fosamax). Which discharge instruction should
client having a CT scan with contrast dye in the nurse discuss with the client?
the morning. 1. The medication must be taken with the
4. Protonix, a proton pump inhibitor, to the breakfast meal only.
client diagnosed with peptic ulcer disease. 2. Remain upright for at least 30 minutes after
taking medication.
13. The nurse is administering the long-acting insulin 3. The tablet should be chewed thoroughly
glargine (Lantus) to the client at 2200. The nurse before swallowing.
asks the charge nurse to check the dosage. Which 4. Stress the importance of having monthly
action should the charge nurse implement? hormone levels.
1. Ask the nurse why the insulin is being given
late. 18. The nurse is administering a.m. medications.
2. Check the MAR versus the dosage in the Which medication should the nurse administer
syringe. first?
3. Instruct the nurse to complete a medication 1. The daily digoxin to the client diagnosed with
error form. congestive heart failure.
4. Have the nurse notify the health-care provider. 2. The loop diuretic to the client with a serum
potassium level of 3.1 mEq/L.
14. The nurse is preparing to administer Synthroid, 3. The mucosal barrier Carafate to the client
a thyroid hormone replacement, to the client diagnosed with peptic ulcer disease.
diagnosed with hypothyroidism. Which 4. Solu-Medrol IVP to a client diagnosed with
assessment data would indicate the client is chronic lung disease.
receiving too much medication?
1. Bradypnea and weight gain.
2. Lethargy and hypotension.
3. Irritability and tachycardia.
4. Normothermia and constipation.

19_Colgrove_Ch19.indd 707 02/06/16 4:02 PM


708 M ed -S urg S uccess

19. The HCP ordered an angiotensin-converting 24. The client’s vital signs are T 99.2˚F, AP 59, R 20,
enzyme (ACE) inhibitor for the client diagnosed and BP 108/72. Which medication would the
with a myocardial infarction. Which statement nurse question administering?
best explains the rationale for administering this 1. Theo-Dur, a bronchodilator.
medication to this client? 2. Inderal, a beta blocker.
1. It will help prevent the development of 3. Ampicillin, an antibiotic.
congestive heart failure. 4. Cardizem, a calcium channel blocker.
2. This medication will help decrease the client’s
25. The client in end-stage renal disease is
blood pressure.
a Jehovah’s Witness. The HCP orders
3. ACE inhibitors increase the contractility of
erythropoietin (Epogen), a biologic response
the heart muscle.
modifier, subcutaneously for anemia. Which
4. They will help decrease the development of
action should the nurse take?
atherosclerosis.
1. Question this order because of the client’s
20. The client is receiving the angiotensin-converting religion.
enzyme (ACE) inhibitor enalapril (Vasotec). 2. Encourage the client to talk to his or her
When would the nurse question administering minister.
this medication? 3. Administer the medication subcutaneously
1. The client is not receiving potassium as ordered.
supplements. 4. Obtain the informed consent prior to
2. The client complains of a persistent irritating administering.
cough.
26. The elderly male client is admitted for acute
3. The blood pressure for two (2) consecutive
severe diverticulitis. He has been taking Xanax,
readings is 110/70.
a benzodiazepine, for nervousness three (3) to
4. The client’s urinary output is 400 mL for the
four (4) times a day prn for six (6) years. Which
last eight (8) hours.
intervention should the nurse implement first?
21. The nurse is preparing to administer the morning 1. Prepare to administer an intravenous
dose of digoxin, a cardiac glycoside, to a client antianxiety medication.
diagnosed with congestive heart failure. Which 2. Notify the HCP to obtain an order for the
data would indicate the medication is effective? client’s Xanax prn.
1. The apical heart rate is 72 beats per minute. 3. Explain Xanax causes addiction and he should
2. The client denies having any anorexia or nausea. quit taking it.
3. The client’s blood pressure is 120/80 mm Hg. 4. Assess for signs/symptoms of medication
4. The client’s lungs sounds are clear bilaterally. withdrawal.
22. The client diagnosed with multiple sclerosis 27. The nurse is administering an ophthalmic
(MS) is receiving Lioresal (baclofen), a muscle drop to the right eye. Which anatomical
relaxant. Which information should the nurse location would be correct when administering
teach the client/family? eyedrops?
1. The importance of tapering off medication
when discontinuing medication.
2. Baclofen may cause diarrhea, so the client
should take antidiarrheal medication.
3. The client should not be allowed to drive
A D
alone while taking this medication.
4. The need for follow-up visits to obtain a
monthly white blood cell count.
23. The nurse is administering digoxin, a cardiac C
glycoside, to the client with congestive heart B
failure. Which interventions should the nurse 1. A
implement? Select all that apply. 2. B
1. Check the apical heart rate for one (1) full 3. C
minute. 4. D
2. Monitor the client’s serum sodium level.
3. Teach the client how to take his or her radial
pulse.
4. Evaluate the client’s serum digoxin level.
5. Assess the client for buffalo hump and moon
face.

19_Colgrove_Ch19.indd 708 02/06/16 4:02 PM


C hapter 19  P harmacology 709

28. The nurse is administering the loop diuretic 33. The nurse is preparing to administer the
furosemide (Lasix) to the client diagnosed initial dose of an antibiotic in the emergency
with essential hypertension. Which assessment department. Which interventions should the
data would warrant the nurse to question nurse implement? Select all that apply.
administering the medication? 1. Assess for drug allergies.
1. The client’s potassium level is 4.2 mEq/L. 2. Collect needed specimens for culture.
2. The client’s urinary output is greater than 3. Check the client’s armband.
the intake. 4. Ask the client his or her birthday.
3. The client has tented skin turgor and dry 5. Draw peak and trough levels.
mucous membranes.
34. For which client should the nurse question
4. The client has lost two (2) pounds in the last
administering the muscarinic cholinergic agonist
24 hours.
oxybutynin (Ditropan)?
29. The client who has had a kidney transplant tells 1. The client diagnosed with overactive bladder.
the nurse he has been taking St. John’s wort, an 2. The client diagnosed with type 2 diabetes.
herb, for depression. Which action should the 3. The client diagnosed with glaucoma.
nurse take first? 4. The client diagnosed with peripheral vascular
1. Praise the client for taking the initiative to disease.
treat the depression.
35. The nurse is administering a topical ointment
2. Remain nonjudgmental about the client’s
to the client’s rash on the right leg. Which
alternative treatments.
intervention should the nurse implement first?
3. Refer the client to a psychologist for
1. Don nonsterile gloves.
counseling for depression.
2. Cleanse the client’s right leg.
4. Instruct the client to quit taking the
3. Check the client’s armband.
medication immediately.
4. Wash the hands for 15 seconds.
30. The nurse is administering an antacid to a client
36. The client is exhibiting multifocal premature
with gastroesophageal reflux disease. Which
ventricular contractions. Which antidysrhythmic
statement best describes the scientific rationale
medication should the nurse anticipate the HCP
for administering this medication?
ordering for this dysrhythmia?
1. This medication will suppress gastric acid
1. Adenosine.
secretion.
2. Epinephrine.
2. This medication will decrease the gastric pH.
3. Atropine.
3. This medication will coat the stomach lining.
4. Amiodarone.
4. This medication interferes with prostaglandin
production. 37. The client in the intensive care department
is receiving 2 mcg/kg/min of dopamine, an
31. The client is diagnosed with essential
inotropic vasopressor. Which intervention
hypertension and is receiving a calcium channel
should the nurse include in the plan of care?
blocker. Which assessment data would warrant
1. Monitor the client’s blood pressure every
the nurse holding the client’s medication?
two (2) hours.
1. The client’s oral temperature is 102˚F.
2. Assess the client’s peripheral pulses every shift.
2. The client complains of a dry, nonproductive
3. Use a urometer to assess hourly output.
cough.
4. Ensure the IV tubing is not exposed to the light.
3. The client’s blood pressure reading is 106/76.
4. The client complains of being dizzy when 38. The client is receiving thrombolytic therapy
getting out of bed. for a diagnosed myocardial infarction (MI).
Which assessment data indicate the therapy is
32. The client complains of leg cramps at night.
successful?
Which medication should the nurse anticipate
1. The client’s ST segment is becoming more
the HCP ordering to help relieve the leg
depressed.
cramps?
2. The client is exhibiting reperfusion
1. Quinine, an antimalarial.
dysrhythmias.
2. Soma, a muscle relaxant.
3. The client’s cardiac isoenzyme CK-MB is not
3. Ambien, a sedative-hypnotic.
elevated.
4. Darvon, an opioid analgesic.
4. The D-dimer is negative at two (2) hours
post-MI.

19_Colgrove_Ch19.indd 709 02/06/16 4:02 PM


710 M ed -S urg S uccess

39. The client with arthritis is self-medicating 43. The elderly client is in a long-term care
with aspirin, a nonsteroidal anti-inflammatory facility. If the client does not have a daily
medication. Which complication should the bowel movement in the morning, he requests a
nurse discuss with the client? cathartic, bisacodyl (Dulcolax). Which action is
1. Tinnitus. most important for the nurse to take?
2. Diarrhea. 1. Ensure the client gets a cathartic daily.
3. Tetany. 2. Discuss the complications of a daily cathartic.
4. Paresthesia. 3. Encourage the client to increase fiber in the diet.
4. Refuse to administer the medication to the
40. The client is receiving a loop diuretic for
client.
congestive heart failure. Which medication
would the nurse expect the client to be receiving 44. The client received Narcan, a narcotic
while taking this medication? antagonist, following a colonoscopy. Which
1. A potassium supplement. action by the nurse has the highest priority?
2. A cardiac glycoside. 1. Document the occurrence in the nurse’s notes.
3. An ACE inhibitor. 2. Prepare to administer narcotic medication IV.
4. A potassium cation. 3. Administer oxygen via nasal cannula.
4. Assess the client every 15 to 30 minutes.
41. The nurse is reviewing laboratory values
for the female client diagnosed with cancer. 45. The client diagnosed with chronic obstructive
Based on the laboratory report, which biologic pulmonary disease is being discharged and
response modifier would the nurse anticipate is prescribed the steroid prednisone. Which
administering to the client? scientific rationale supports why the nurse
1. Interferon. instructs the client to taper off the medication?
2. Neupogen. 1. The pituitary gland must adjust to the
3. Neumega. decreasing dose.
4. Procrit. 2. The beta cells of the pancreas have to start
secreting insulin.
Laboratory Client 3. This will allow the adrenal gland time to start
Test Values Normal Values functioning.
Red blood 4.11 M: 4.7–6.1 (× 106)/mm3 4. The thyroid gland will have to start producing
cells F: 4.2–5.4 (× 106)/mm3 cortisol.
Hemoglobin 12.2 M: 13.5–17.5 g/dL 46. The client is diagnosed with tuberculosis and
F: 11.5–15.5 g/dL prescribed rifampin and isoniazid (INH), both
Hematocrit 37 M: 40%–52% antituberculosis medications. Which instruction
F: 36%–48%
is most important for the public health nurse to
discuss with the client?
White blood 2 4.5–11 (× 103)/mm3
1. The client will have to take the medications
cells
for nine (9) to 12 months.
Platelets 160 140–400 (× 103)/mm3
2. The client will have to stay in isolation as long
as he or she is taking medications.
42. The client admitted with pneumonia is taking
3. Explain the client cannot eat any type of pork
Imuran, an immunosuppressive agent. Which
products while taking the medication.
question should the nurse ask the client
4. The urine may turn turquoise in color, but
regarding this medication?
this is an expected occurrence and harmless.
1. “Do you know this medication has to be
tapered off when discontinued?” 47. The employee health nurse is observing a
2. “Have you been exposed to viral hepatitis B student nurse administer a PPD tuberculin
or C recently?” test to a new employee. Which behavior would
3. “Why are you taking this medication, and warrant immediate intervention by the
how long have you taken it?” employee health nurse?
4. “Do you have a lot of allergies or sensitivities 1. The student nurse inserts the needle at a
to different medications?” 45-degree angle.
2. The student nurse cleanses the forearm with
alcohol.
3. The student nurse circles the injection site
with ink.
4. The student nurse instructs the employee to
return in three (3) days.

19_Colgrove_Ch19.indd 710 02/06/16 4:02 PM


C hapter 19  P harmacology 711

48. The female client diagnosed with herpes simplex 53. The female client with Trichomonas vaginalis
2 is prescribed valacyclovir (Valtrex), an antiviral. is prescribed metronidazole (Flagyl), an
Which information should the nurse discuss antibacterial medication. Which statement
with the client? indicates the client does not understand the
1. Do not get pregnant while on this medication; discharge teaching?
it will harm the fetus. 1. “I will not be able to drink any alcohol while
2. The medication does not prevent the taking this drug.”
transmission of the disease. 2. “My boyfriend will need to take this same
3. There are no side effects when taking this medication.”
medication by mouth. 3. “I cannot transmit the disease through oral sex.”
4. The client should get monthly liver function 4. “I must make sure I take all the pills no matter
study tests. how I feel.”
49. The client diagnosed with coronary artery 54. The client diagnosed with angina must receive
disease is prescribed an HMG-CoA reductase a two (2)-inch nitroglycerin paste (Nitro-Bid)
inhibitor to help reduce the cholesterol level. application. Which interventions should the
Which assessment data should be reported to nurse implement? Select all that apply.
the health-care provider? 1. Wear gloves when administering.
1. Complaints of flatulence. 2. Remove the old Nitro-Bid paper.
2. Weight loss of two (2) pounds. 3. Apply the paper on a hairy spot.
3. Complaints of muscle pain. 4. Put medication only on the legs.
4. No bowel movement for two (2) days. 5. Report any headache to the HCP.
50. The client with coronary artery disease is 55. The nurse is hanging 1,000 mL of IV fluids to
prescribed one (1) baby aspirin a day. Which run for eight (8) hours. The intravenous tubing
instructions should the nurse provide the client is a microdrip. How many gtt/min should the IV
concerning this medication? rate be set? ________
1. Take the medication on an empty stomach. 56. The client with osteoarthritis is prescribed a
2. Do not take Tylenol while taking this drug. nonsteroidal anti-inflammatory drug (NSAID).
3. If experiencing joint pain, notify the HCP. Which intervention should the nurse implement?
4. Notify the HCP if stools become dark and tarry. 1. Time the medication to be given with meals.
51. The nurse is preparing to administer phenytoin 2. Notify the HCP if abdominal striae develop.
(Dilantin), 100 mg intravenous push, to the 3. Do not administer if oral temperature is
client with a head injury who has an IV of D5W greater than 102˚F.
at 50 mL/hr. Which intervention should the 4. Monitor the liver function tests and renal
nurse implement? studies.
1. Flush the IV tubing before and after with 57. The client in the intensive care department
normal saline. has a nasogastric tube for continuous feedings.
2. Administer the medication if the Dilantin The nurse is preparing to administer nifedipine
level is 22 mcg/mL. (Procardia XL) via the N/G tube. Which
3. Push the Dilantin intravenously slowly over procedure should the nurse follow?
five (5) minutes. 1. Crush the medication and dissolve it in water.
4. Expect the intravenous tubing to turn cloudy 2. Administer and flush the N/G tube with
when infusing medication. cranberry juice.
52. The client diagnosed with epilepsy is being 3. Give the medication orally with pudding.
discharged from the hospital with a prescription 4. Do not administer the medication and notify
for phenytoin (Dilantin) by mouth. Which the HCP.
discharge instructions should the nurse discuss 58. The employee health nurse is discussing
with the client? hepatitis B vaccines with new employees.
1. The client should purchase a self-monitoring Which statement best describes the proper
Dilantin machine. administration of the hepatitis B vaccine?
2. The client should see the dentist at least every 1. The vaccine must be administered once a year.
six (6) months. 2. Two (2) mL of vaccine should be given in
3. The client should never drive when taking each hip.
this medication. 3. The vaccine is given in three (3) doses over a
4. The client should drink no more than one (1) six (6)-month time period.
glass of wine a day. 4. The vaccine is administered intradermally
into the deltoid muscle.

19_Colgrove_Ch19.indd 711 02/06/16 4:02 PM


712 M ed -S urg S uccess

59. The unlicensed assistive personnel (UAP) 63. The nurse administered an IV broad-spectrum
reported an intake of 1,000 mL and a urinary antibiotic scheduled every six (6) hours to the
output of 1,500 mL for a client who received a client with a systemic infection at 0800. At 1000,
thiazide diuretic this morning. Which nursing the culture and sensitivity prompted the HCP
task could the nurse delegate to the nursing to change the IV antibiotic. When transcribing
assistant? the new antibiotic order, when would the initial
1. Instruct the UAP to restrict the client’s fluid dose be administered?
intake. 1. Schedule the dose for 1400.
2. Request the UAP to insert a Foley catheter 2. Schedule the dose for the next day.
with an urometer. 3. Check with the HCP to determine when
3. Tell the UAP urinary outputs are no longer to start.
needed. 4. Administer the dose within one (1) hour of
4. Ask the UAP to document fluids on the the order.
bedside I & O record.
64. The client is receiving a continuous heparin
60. The charge nurse is observing the new graduate drip, 20,000 units/500 mL D5W, at 23 mL/min.
administering a fentanyl (Duragesic) patch to a How many units of heparin is the client
client diagnosed with cancer. Which action by receiving an hour? ________
the new graduate requires intervention by the
65. The client with epilepsy is prescribed
charge nurse?
carbamazepine (Tegretol), an anticonvulsant.
1. The new graduate documents the date and
Which discharge instruction should the nurse
time on the patch.
include in the teaching?
2. The new graduate removes the patch 24 hours
1. Wear SPF 15 sunscreen when outside.
after it is placed on the client.
2. Obtain regular serum drug levels.
3. The new graduate rotates the application site
3. Be sure to floss teeth daily.
on the client’s body.
4. Instruct the client to take tub baths only.
4. The new graduate checks the client’s name
band and date of birth. 66. The client diagnosed with bipolar disorder
has been taking valproic acid (Depakote), an
61. The 68-year-old client is admitted to the
anticonvulsant, for four (4) months. Which
emergency department with complaints of
assessment data would warrant the medication
slurred speech, right-sided weakness, and ataxia.
being discontinued?
The emergency department physician ordered
1. The client’s eyes are yellow.
thrombolytic therapy for the client. Which
2. The client has mood swings.
action should the nurse implement first?
3. The client’s BP is 164/94.
1. Administer thrombolytic therapy via protocol.
4. The client’s serum level is 75 mcg/mL.
2. Send the client for a STAT CT of the head.
3. Arrange for admission to the intensive care 67. The client is complaining of nausea, and the
department. nurse administers the antiemetic promethazine
4. Check to determine if the client is cross (Phenergan), IVP. Which intervention has
sensitive to the thrombolytic. priority for this client after administering this
medication?
62. The client is admitted to the burn unit and
1. Instruct the client to call the nurse before
prescribed pantoprazole (Protonix), a proton
getting out of bed.
pump inhibitor (PPI). Which statement
2. Evaluate the effectiveness of the medication.
best supports the scientific rationale for
3. Assess the client’s abdomen and bowel sounds.
administering this medication to a client with a
4. Tell the client not to eat or drink for at least
severe burn?
one (1) hour.
1. This medication will help prevent a stress
ulcer. 68. The client on bedrest is receiving enoxaparin
2. This medication will help prevent systemic (Lovenox), a low molecular weight heparin.
infections. Which anatomical site is recommended for
3. This medication will provide continuous administering this medication?
vasoconstriction. 1. The abdominal wall one (1) inch away from
4. This medication will stimulate new skin the umbilicus.
growth. 2. The vastus lateralis with a 23-gauge needle.
3. In the deltoid area subcutaneously.
4. In the anterolateral abdomen.

19_Colgrove_Ch19.indd 712 02/06/16 4:02 PM


C hapter 19  P harmacology 713

69. The male client comes to the emergency 74. The client diagnosed with migraine headaches is
department and reports he stepped on a rusty prescribed propranolol (Inderal), a beta blocker,
nail at home about two (2) hours ago. Which for prophylaxis. Which information should the
question would be most important for the nurse teach the client?
nurse to ask during the admission assessment? 1. Instruct to take the medication at the first sign
1. “What have you used to clean the puncture of headache.
site?” 2. Teach the client to take his or her radial pulse
2. “Did you bring the nail with you so we can for one (1) minute.
culture it?” 3. Explain this drug may make the client thirsty
3. “Do you remember when you had your last and have a dry mouth.
tetanus shot?” 4. Discuss the need to increase artificial light in
4. “Are you able to put any weight on your foot?” the home.
70. The nurse is administering carbidopa/levodopa 75. The client is experiencing supraventricular
(Sinemet) to the client. Which assessment tachycardia (SVT). Which antidysrhythmic
should the nurse perform to determine if the medication should the nurse prepare to
medication is effective? administer?
1. Assess the client’s muscle strength. 1. Atropine.
2. Assess for cogwheel movements. 2. Amiodarone.
3. Assess the carbidopa serum level. 3. Adenosine.
4. Assess the client’s blood pressure. 4. Dobutamine.
71. The client with coronary artery disease is 76. The client diagnosed with Parkinson’s disease is
prescribed atorvastatin (Lipitor) to help decrease taking levodopa (L-dopa) and is experiencing an
the client’s cholesterol level. Which intervention “on/off” effect. Which action should the nurse
should the nurse discuss with the client take regarding this medication?
concerning this medication? 1. Document the occurrence and take no action.
1. The client should eat a low-cholesterol, low- 2. Request the HCP to increase the dose of
fat diet. medication.
2. The client should take this medication with 3. Discuss the client’s imminent death as a result
each meal. of this complication.
3. The client should take this medication in the 4. Explain this is a desired effect of the
evening. medication.
4. The client should monitor daily cholesterol
77. The client in the emergency department
levels.
requires sutures for a laceration on the left leg.
72. The client is in end-stage renal disease and Which information is most pertinent prior to
is receiving sodium polystyrene sulfonate suturing the wound?
(Kayexalate) via an enema. Which data indicate 1. The client tells the nurse she has never had
the medication is effective? sutures.
1. The client has 30 mL/hr of urine output. 2. The spouse refuses to leave the room during
2. The serum phosphorus level has decreased. suturing.
3. The client is in normal sinus rhythm. 3. The client shares she is scared of needles.
4. The client’s serum potassium level is 4. The client reports hives after having dental
5 mEq/L. surgery.
73. The client has the following arterial blood gases: 78. The client diagnosed with diabetes insipidus
pH 7.19, Paco2 33, Pao2 95, and HCO3 19. is receiving vasopressin intranasally. Which
Which medication would the nurse prepare to assessment data indicate the medication is
administer based on the results? effective?
1. Intravenous sodium bicarbonate. 1. The client reports being able to breathe
2. Oxygen via nasal cannula. through the nose.
3. Epinephrine intravenous push. 2. The client complains of being thirsty all
4. Magnesium hydroxide orally. the time.
3. The client has a blood glucose of 99 mg/dL.
4. The client is urinating every three (3) to four
(4) hours.

19_Colgrove_Ch19.indd 713 02/06/16 4:02 PM


714 M ed -S urg S uccess

79. The nurse is administering an otic drop to the 84. The client diagnosed with diabetes mellitus
45-year-old client. Which procedure should the type 2 is scheduled for bowel resection in the
nurse implement when administering the drops? morning. Which medication should the nurse
1. Place the drops when pulling the ear down question administering to the client?
and back. 1. Ticlopidine (Ticlid), a platelet aggregate
2. Place the drops when pulling the ear up and inhibitor.
back. 2. Ticarcillin (Timentin), an extended-spectrum
3. Place the drops in the lower conjunctival sac. antibiotic.
4. Place the drops in the inner canthus and apply 3. Pioglitazone (Actos), a thiazolidinedione.
pressure. 4. Bisacodyl (Dulcolax), a cathartic laxative.
80. The male client with a chronic urinary 85. The client with type 2 diabetes is diagnosed
tract infection is prescribed trimethoprim- with gout and prescribed allopurinol (Zyloprim).
sulfamethoxazole (Bactrim). Which statement Which instruction should the nurse discuss
indicates the client needs more teaching? when teaching about this medication?
1. “I will drink six (6) to eight (8) glasses of water 1. The client will probably develop a red rash on
a day.” the body.
2. “I am going to have to take this medication 2. The client should drink two (2) to three (3)
forever.” liters of water a day.
3. “I can stop taking this medication if there is 3. The client should take this medication on an
no more burning.” empty stomach.
4. “I may get diarrhea with this medication, but I 4. The client will need to increase oral diabetic
can take Imodium.” medications.
81. The 54-year-old female client with severe 86. The client is receiving atropine, an anticholinergic,
menopausal symptoms is prescribed hormone to minimize the side effects of routine medications.
replacement therapy (HRT). Which secondary Which intervention will help the client tolerate
health screening activity should the nurse this medication?
recommend for HRT? 1. Teach the client about orthostatic
1. A Pap smear every six (6) months. hypotension.
2. A yearly mammogram. 2. Instruct the client to eat a low-residue diet.
3. A bone density test every three (3) months. 3. Encourage the client to chew sugarless gum.
4. A serum calcium level monthly. 4. Discuss the importance of daily isometric
exercises.
82. The LPN is administering 0800 medications to
clients on a medical floor. Which action by the 87. The client is showing ventricular ectopy, and
LPN would warrant immediate intervention by the HCP orders amiodarone (Cordarone)
the nurse? intravenously. Which interventions should the
1. The LPN scores the medication to give the nurse implement? Select all that apply.
correct dose. 1. Monitor telemetry continuously.
2. The LPN checks the client’s armband and 2. Assess the client’s respiratory status.
birth date. 3. Evaluate the client’s liver function studies.
3. The LPN administers sliding-scale insulin 4. Confirm the original order with another nurse.
intramuscularly. 5. Prepare to defibrillate the client at 200 joules.
4. The LPN is 30 minutes late hanging the IV
88. The HCP has ordered an intramuscular
antibiotic.
antibiotic. After reconstituting the medication,
83. The client in end-stage renal disease is receiving the clinic nurse must administer 4.8 mL of the
aluminium hydroxide (Amphojel). Which medication. Which action should the nurse
assessment data indicate the medication is implement first when administering this
effective? medication?
1. The client denies complaints of indigestion. 1. Inform the HCP the amount of medication is
2. The client is not experiencing burning on too large.
urination. 2. Administer the medication in the gluteal
3. The client has had a normal, soft bowel muscle.
movement. 3. Discard the medication in the sharps container.
4. The client’s phosphate level has decreased. 4. Divide the medication and give 2.4 mL in
each hip.

19_Colgrove_Ch19.indd 714 02/06/16 4:02 PM


C hapter 19  P harmacology 715

89. The client diagnosed with status asthmaticus 95. The nurse is evaluating the client’s home
is prescribed intravenous aminophylline, a medications and notes the client with angina is
bronchodilator. Which assessment data would taking an antidepressant. Which intervention
warrant immediate intervention? should the nurse implement because the client is
1. The theophylline level is 12 mcg/mL. taking this medication?
2. The client has expiratory wheezing. 1. Ask the client if there is a plan for suicide.
3. The client complains of muscle twitching. 2. Assess the client’s depression on a 1-to-10 scale.
4. The client is refusing to eat the meal. 3. Explain this medication cannot be taken
because of the angina.
90. To which client would the nurse question
4. Request a referral to the hospital psychologist.
administering the osmotic diuretic mannitol
(Osmitrol)? 96. The nurse is assessing the elderly client first
1. The client with 4+ pitting pedal edema. thing in the morning. The client is confused
2. The client with decorticate posturing. and sleepy. Which intervention should the nurse
3. The client with widening pulse pressure. implement first?
4. The client with a positive doll’s eye test. 1. Determine if the client received a sedative
last night.
91. The male client is self-medicating with the
2. Allow the client to continue to sleep and do
H-2 antagonist cimetidine (Tagamet). Which
not disturb.
complication can occur while taking this
3. Encourage the client to ambulate in the room
medication?
with assistance.
1. Melena.
4. Notify the health-care provider about the
2. Gynecomastia.
client’s status.
3. Pyrosis.
4. Eructation. 97. The nurse is preparing to administer 37.5 mg
of meperidine (Demerol) IM to a client who
92. The client is complaining of low-back pain and
is having pain. The medication comes in a
is prescribed the muscle relaxant carisoprodol
50-mg/mL vial. Which action should the nurse
(Soma). Which teaching intervention has
implement?
priority?
1. Notify the pharmacist to bring the correct
1. Explain this medication causes GI distress.
vial.
2. Discuss the need to taper off this medication.
2. Have another nurse verify wastage of
3. Warn this medication will cause drowsiness.
medication.
4. Instruct the client to limit alcohol intake.
3. Administer one (1) mL of medication to
93. The client diagnosed with adult-onset asthma the client.
is being discharged. Which medication would 4. Request the HCP to increase the client’s dose.
the nurse expect the health-care provider to
98. The client is to receive 3,000 mg of medication
prescribe?
daily in a divided dose every eight (8) hours.
1. A nonsteroidal anti-inflammatory medication.
The medication comes 500 mg per tablet. How
2. An antihistamine medication.
many tablets will the nurse administer at each
3. An angiotensin-converting enzyme inhibitor.
dose? __________
4. A proton pump inhibitor.
99. The 38-year-old client with chronic
94. The client is complaining of incisional pain.
asthma is prescribed a leukotriene receptor
Which intervention should the nurse implement
antagonist. Which is the scientific rationale for
first?
administering this medication?
1. Administer the pain medication STAT.
1. This medication is used prophylactically to
2. Determine when the last pain medication
control asthma.
was given.
2. This medication will cure the client’s chronic
3. Assess the client’s pulse and blood pressure.
asthma.
4. Teach the client distraction techniques to
3. It will stabilize mast cell activities and reduce
address pain.
asthma attacks.
4. It will cause the bronchioles to dilate and
increase the airway.

19_Colgrove_Ch19.indd 715 02/06/16 4:02 PM


716 M ed -S urg S uccess

100. The female nurse realizes she did not 101. The nurse has received the morning report
administer a medication on time to the client and has the following medications due or being
diagnosed with a myocardial infarction. Which requested. In which order should the nurse
action should the nurse implement? administer the medications? List in order of
1. Administer the medication and take no priority.
further action. 1. Administer furosemide (Lasix), a loop
2. Notify the director of nurses of the diuretic, IVP daily to a client diagnosed with
medication error. heart failure who is dyspneic on exertion.
3. Complete a medication error report form. 2. Administer morphine, a narcotic analgesic,
4. Report the error to the Peer Review IVP prn to a client diagnosed with lower
Committee. back pain who is complaining of pain at a
“10” on a 1-to-10 scale.
3. Administer neostigmine (Prostigmin), a
cholinesterase inhibitor, PO to a client
diagnosed with myasthenia gravis.
4. Administer lidocaine, an antidysrhythmic,
IVP prn to a client in normal sinus rhythm
with multifocal premature ventricular
contractions.
5. Administer vancomycin, an aminoglycoside
antibiotic, to a client diagnosed with a
Staphylococcus infection who has a trough
level of 14 mg/dL.

19_Colgrove_Ch19.indd 716 02/06/16 4:02 PM


MEDICATIONS ADMINISTRATION
IN A MEDICAL-SURGICAL SETTING
COMPREHENSIVE EXAMINATION
ANSWERS AND RATIONALES

1. 1. The normal recommended daily dose of 2. OTC medications should not be taken by the
vitamin C is 60 to 100 mg for healthy adults, client with essential hypertension.
but nothing cures the virus which causes the 3. The nurse should provide the information to
common cold. the client about what medications to take and
2. Echinacea is an herbal preparation thought to should not refer the client to the pharmacist.
limit the severity of a cold and is sold in OTC 4. It is too late for the flu vaccine because the
preparations, but it does not have to be taken ­client is already ill with the flu.
with vitamin C. Content – Pharmacology: Integrated Nursing Process –
3. This dose is already too high, and water- Implementation: Client Needs – Physiological Integrity,
soluble vitamins in excess of the body’s needs Pharmacological and Parenteral Therapies: Cognitive Level –
are excreted in the urine. Analysis: Concept – Perfusion.
4. Megadoses can lead to crystals in the urine, 4. 1. The potassium level is not affected by the
and crystals can lead to the formation of ­administration of steroids.
renal calculi (stones) in the kidneys. There- 2. Culture and sensitivity reports should be
fore, megadoses should not be taken be- ­monitored to determine if the proper anti­
cause there is no therapeutic value. biotic is being administered.
Content – Pharmacology: Integrated Nursing Process – 3. Steroids are excreted as glucocorticoids
Implementation: Client Needs – Physiological Integrity, from the adrenal gland and are responsible
Pharmacological and Parenteral Therapies: Cognitive Level –
for insulin resistance by the cells, which
Analysis: Concept – Medication.
may cause hyperglycemia.
2. 1. Vitamin K helps prevent clotting, and NSAIDs 4. There is no reason why the nurse would
are recommended for inflammatory disorders ­question administering a steroid based on an
and to relieve mild to moderate pain. arterial blood gas result.
2. Vitamin E is an antioxidant and is useful in Content – Pharmacology: Integrated Nursing Process –
the treatment and prevention of coronary Assessment: Client Needs – Physiological Integrity, Pharma-
artery disease, and aspirin is an antiplatelet cological and Parenteral Therapies: Cognitive Level –
which prevents platelet aggregation. Analysis: Concept – Medication.
3. Vitamin A is required for healthy eyes, gums, 5. 1. Cromolyn is used prophylactically to prevent
and teeth and for fat metabolism. Anticoagu- exercise-induced asthma attacks. It is adminis-
lants are prescribed for clients with a high risk tered in routine daily doses to prevent asthma
for clot formation. attacks.
4. Vitamin B complex is used for healthy function 2. Rinsing the mouth will help prevent the
of the nervous system, cell repair, and forma- growth of bacteria secondary to medication
tion of red blood cells; iron supplements are left in the mouth.
recommended for clients with iron-deficiency 3. Holding the breath for 10 seconds keeps the
anemia. medication in the lungs.
Content – Pharmacology: Integrated Nursing Process – 4. This medication is used to stabilize the
Planning: Client Needs – Physiological Integrity, Pharmaco- mast cells in the lungs. During an asthma
logical and Parenteral Therapies: Cognitive Level – Synthesis:
attack, the mast cells are already unstable;
Concept – Medication.
therefore, this medication will not be ef-
3. 1. OTC decongestant medications used for fective in treating the acute asthma attack.
the flu cause vasoconstriction of the blood This statement would require the nurse to
vessels, which would increase the client’s reteach about the medication.
hypertension and therefore should be Content – Pharmacology: Integrated Nursing Process –
avoided. The client should let the flu run Evaluation: Client Needs – Physiological Integrity, Pharma-
its course. cological and Parenteral Therapies: Cognitive Level –
Synthesis: Concept – Medication.

717

19_Colgrove_Ch19.indd 717 02/06/16 4:02 PM


718 M ed -S urg S uccess

6. 1. Diarrhea may indicate the client may have 2. This is an oral hypoglycemic medication.
a superinfection, but it is not the priority 3. There are side effects to every medication;
intervention at this time because the ­antibiotic this medication can cause hypoglycemia.
would still be administered. 4. The medication stimulates the pancreas to
2. The peak level is not drawn until one (1) hour produce more insulin, but it does not affect
after the medication has been infused. the muscles’ absorption of glucose.
3. The trough level must be drawn prior to Content – Pharmacology: Integrated Nursing Process –
administering this dose; therefore, it is the Evaluation: Client Needs – Physiological Integrity, Phar-
priority intervention. macological and Parenteral Therapies: Cognitive Level –
4. The culture and sensitivity (C&S) has already Synthesis: Concept – Medication.
been done because it is known the client has 10. 10 units.
MRSA. The nurse should administer the dosage for
Content – Pharmacology: Integrated Nursing Process – the appropriate parameters.
Implementation: Client Needs – Physiological Integrity,
Content – Pharmacology: Integrated Nursing Process –
Pharmacological and Parenteral Therapies: Cognitive Level –
Implementation: Client Needs – Physiological Integrity,
Analysis: Concept – Medication.
Pharmacological and Parenteral Therapies: Cognitive
7. 1. At eight (8) hours postoperative the client Level – Application: Concept – Medication.
should be on bedrest, and moving the client 11. 1. This test monitors the client’s average blood
to a chair will not help the incisional pain and glucose level over the previous three (3)
could cause hip dislocation. months.
2. Normal developmental changes in the
2. The evening meal would prevent hypoglyce-
organs of the elderly, especially the ­kidneys mia for regular insulin administered at 1630.
and liver, result in lower doses of pain
3. The before-meal (a.c.) blood glucose level
medication needed to achieve therapeutic
done at 1630 would not be affected by the
levels.
insulin administered after that time.
3. This is a neurological assessment, which is 4. The intermediate-acting insulin peaks six
not pertinent to the extremity assessment.
(6) to eight (8) hours after being adminis-
4. The urinary output would not affect the tered; therefore, the nighttime snack (h.s.)
administration of pain medication. will prevent late-night hypoglycemia.
Content – Pharmacology: Integrated Nursing Process –
Content – Pharmacology: Integrated Nursing Process –
Implementation: Client Needs – Safe Effective Care Envi-
Implementation: Client Needs – Physiological Integrity,
ronment, Management of Care: Cognitive Level – Analysis:
Pharmacological and Parenteral Therapies: Cognitive
Concept – Perioperative.
Level – Analysis: Concept – Medication.
8. 1. Iron dextran is administered for iron- 12. 1. The nurse would not question administering
deficiency anemia intravenously or intramus- insulin to a client about to eat.
cularly, not subcutaneously. 2. The client who is one (1) day postoperative
2. Vitamin B12 is administered for ­pernicious would be receiving a prophylactic antibiotic.
anemia because there is insufficient 3. Glucophage must be held 24 to 48 hours
­intrinsic factor produced by the rugae in prior to receiving contrast media (dye)
the stomach to be able to absorb and use because Glucophage, along with the con-
vitamin B12 from food sources. trast dye, can damage kidney function.
3. Folic acid is administered orally or intrave- 4. The client with peptic ulcer disease would
nously for folic acid deficiency, which is usually be ordered a proton pump inhibitor to help
associated with chronic alcoholism. decrease gastric acid production.
4. Thiamine is administered intravenously in
Content – Pharmacology: Integrated Nursing Process –
high doses to clients detoxifying from chronic
Implementation: Client Needs – Physiological Integrity,
alcoholism to prevent rebound nervous system Pharmacological and Parenteral Therapies: Cognitive
dysfunction. Level – Analysis: Concept – Medication.
Content – Pharmacology: Integrated Nursing Process –
Planning: Client Needs – Physiological Integrity, Pharmaco- 13. 1. This insulin is scheduled for bedtime.
logical and Parenteral Therapies: Cognitive Level – Synthesis: 2. The charge nurse should double-check the
Concept – Hematologic Regulation. dosage against the MAR to make sure the
client is receiving the correct dose; this in-
9. 1. This medication stimulates the pancreas sulin does not peak and works for 24 hours.
to secrete insulin. Therefore, the client is 3. There is not a medication error at this time.
at risk for developing hypoglycemic reac- 4. The HCP would only need to be notified if a
tions, especially during exercise. serious medication error has occurred.

19_Colgrove_Ch19.indd 718 02/06/16 4:02 PM


C hapter 19  P harmacology 719

Content – Pharmacology: Integrated Nursing Process – 2. Remaining in the upright position mini-
Implementation: Client Needs – Physiological Integrity, mizes the risk of esophagitis; the drug
Pharmacological and Parenteral Therapies: Cognitive should be taken with eight (8) ounces of
Level – Application: Concept – Nursing Roles. water.
14. 1. These are signs/symptoms of hypothyroidism, 3. The tablet should be swallowed, not chewed,
which indicates not enough medication. and should not be allowed to dissolve until it
2. These indicate not enough medication is is the stomach.
being administered. 4. There is no monthly hormone level to deter-
3. Irritability and tachycardia are signs/ mine the effectiveness of this medication; it is
symptoms of hyperthyroidism, which determined by a bone density test.
indicates the client is receiving too much Content – Pharmacology: Integrated Nursing Process –
medication. Planning: Client Needs – Physiological Integrity, Pharma-
4. Normothermia indicates a normal tempera- cological and Parenteral Therapies: Cognitive Level –
ture, which does not indicate hypothyroidism Synthesis: Concept – Medication.
or hyperthyroidism, and constipation is a sign 18. 1. A daily digoxin dose is not priority medication.
of hypothyroidism. 2. This potassium level is very low, and the
Content – Pharmacology: Integrated Nursing Process – nurse should not administer the loop diuretic.
Assessment: Client Needs – Safe Effective Care Environ- 3. The mucosal barrier must be adminis-
ment, Management of Care: Cognitive Level – Analysis: tered on an empty stomach; therefore, it
Concept – Metabolism. should be administered first.
15. 1. The therapeutic heparin level is 1.5 to 4. An IVP medication is not priority over
2 times the control, which is 58 to 78; administering a medication which must be
therefore, a PTT of 72 is within thera- given on an empty stomach.
peutic range so the nurse should continue Content – Pharmacology: Integrated Nursing Process –
to monitor the infusion. PT/INR are used Implementation: Client Needs – Safe Effective Care Envi-
to monitor the oral anticoagulant warfarin ronment, Management of Care: Cognitive Level – Analysis:
(Coumadin). Concept – Medication.
2. Protamine sulfate is the antidote for heparin 19. 1. Attempting to prevent CHF is the ra-
toxicity, but the client is in the therapeutic tionale for administering ACE inhibi-
range. tors to clients diagnosed with MIs. This
3. There is no need for the laboratory to medication is administered for a variety
reconfirm the results. of medical diagnoses, such as heart failure
4. The nurse would not need to assess for and stroke, and to help prevent diabetic
bleeding because the results are within the nephropathy.
therapeutic range. 2. ACE inhibitors are prescribed to help
Content – Pharmacology: Integrated Nursing Process – decrease blood pressure, but the stem states
Implementation: Client Needs – Physiological Integrity, the client has had an MI, not essential
Pharmacological and Parenteral Therapies: Cognitive hypertension.
Level – Analysis: Concept – Medication. 3. Cardiac glycosides such as digoxin, not ACE
16. 1. A STAT potassium level would be needed for inhibitors, increase the contractility of the
problems with digoxin or a diuretic, not for heart.
bleeding. 4. Antilipidemics, not ACE inhibitors, help
2. The nurse needs more information before decrease the development of atherosclerosis.
requesting an admission into the hospital. Content – Pharmacology: Integrated Nursing Process –
3. Valium IVP does not help bleeding. Implementation: Client Needs – Physiological Integrity,
4. Ecchymotic areas are secondary to bleed- Pharmacological and Parenteral Therapies: Cognitive
ing. The nurse should order an INR to Level – Analysis: Concept – Medication.
rule out warfarin (Coumadin) toxicity. 20. 1. ACE inhibitors may increase potassium
Content – Pharmacology: Integrated Nursing Process – ­levels. The client should avoid potassium salt
Implementation: Client Needs – Physiological Integrity, substitutes and supplements; therefore, the
Pharmacological and Parenteral Therapies: Cognitive nurse would not question the fact the client
Level – Analysis: Concept – Medication. is not receiving potassium supplements.
17. 1. The medication must be taken first thing in 2. An adverse effect of ACE inhibitors is the
the morning before breakfast on an empty possibility of a persistent irritating cough,
stomach; no food, juice, or coffee should be which might precipitate the HCP’s chang-
consumed for at least 30 minutes. ing the client’s medication.

19_Colgrove_Ch19.indd 719 02/06/16 4:02 PM


720 M ed -S urg S uccess

3. This blood pressure indicates the medication Content – Pharmacology: Integrated Nursing Process –
is effective. Implementation: Client Needs – Physiological Integrity,
4. A urinary output of 30 mL/hr indicates the Pharmacological and Parenteral Therapies: Cognitive
kidneys are functioning properly. Level – Analysis: Concept – Medication.
Content – Pharmacology: Integrated Nursing Process – 24. 1. The respiratory rate and pulse rate would not
Assessment: Client Needs – Physiological Integrity, Phar- affect the administration of this medication.
macological and Parenteral Therapies: Cognitive Level – 2. The apical heart rate (AP) of 59 would
Analysis: Concept – Medication. cause the nurse to question administering
21. 1. The apical heart rate is assessed prior to ad- this medication because beta blockers
ministering the dose, but it does not indicate decrease the sympathetic stimulation to the
the medication is effective. heart, thereby decreasing the heart rate.
2. Anorexia and nausea are signs of digoxin 3. These vital signs would not warrant the nurse
toxicity and do not indicate if the medication questioning administering an antibiotic.
is effective. 4. The blood pressure is higher than 90/60;
3. Digoxin has no effect on the client’s blood therefore, the nurse would not question ad-
pressure. ministering the calcium channel blocker.
4. Digoxin is administered for heart failure Content – Pharmacology: Integrated Nursing Process –
and dysrhythmias. Clear lung sounds Implementation: Client Needs – Physiological Integrity,
indicate the heart failure is being Pharmacological and Parenteral Therapies: Cognitive
­controlled by the medication. Level – Analysis: Concept – Medication.
Content – Pharmacology: Integrated Nursing Process – 25. 1. Epogen stimulates the client’s own bone
Assessment: Client Needs – Physiological Integrity, Phar- marrow to produce red blood cells; therefore,
macological and Parenteral Therapies: Cognitive Level – this is not a violation of the client’s religious
Analysis: Concept – Medication. beliefs about blood products.
22. 1. Abrupt discontinuation of baclofen is 2. There is no reason for the client to have
­associated with hallucinations, paranoia, problems receiving this medication because
and seizures. of religious beliefs, so the client does not
2. This medication causes constipation and need to talk to the minister.
­urinary retention. 3. This medication does not violate the
3. The client should not be allowed to drive at Jehovah Witnesses’ beliefs concerning
all when taking this medication because it receiving blood products; therefore, the
causes drowsiness, and the spasticity of MS nurse should administer the medication
makes driving dangerous for the client. via the correct route.
4. White blood cell levels do not need to be 4. This is not an invasive procedure or investi-
monitored because the drug does not cause gational medication, and, thus, informed con-
bone marrow suppression. sent is not needed.
Content – Pharmacology: Integrated Nursing Process – Content – Pharmacology: Integrated Nursing Process –
Planning: Client Needs – Physiological Integrity, Pharma- Implementation: Client Needs – Psychosocial Integrity:
cological and Parenteral Therapies: Cognitive Level – Cognitive Level – Analysis: Concept – Medication.
Synthesis: Concept – Medication. 26. 1. Because the client is NPO as a result of the ad-
23. 1. If the apical heart rate is less than 60, the mitting diagnosis, the client needs alternative
nurse should question administering this antianxiety medication to prevent withdrawal
medication. symptoms, but this is not the first intervention.
2. The client’s potassium level, not the sodium 2. The client will be NPO as a result of the
level, should be monitored. diverticulitis, and Xanax is administered
3. The client should be taught to monitor orally; therefore, another route of medication
the radial pulse at home and not to take administration is needed, but this is not the
the medication if the pulse is less than 60 first intervention.
because this medication will further 3. This is correct information, but it is not the
­decrease the heart rate. priority intervention.
4. The digoxin level should be between 4. Xanax has a greater dependence problem
0.8 and 2 ng/mL to be therapeutic. than all the other benzodiazepines; there-
5. The client with digoxin toxicity would fore, the nurse must assess for withdrawal
­complain of anorexia, nausea, and yellow symptoms first. Then the nurse can imple-
haze; buffalo hump and moon face would be ment the other interventions. The client
assessed for the client taking prednisone, a needs to be withdrawn slowly from the
glucocorticoid. benzodiazepines, but assessment is priority.

19_Colgrove_Ch19.indd 720 02/06/16 4:02 PM


C hapter 19  P harmacology 721

Content – Pharmacology: Integrated Nursing Process – 30. 1. This is the rationale for H-2 antagonists and
Implementation: Client Needs – Physiological Integrity, proton pump inhibitors.
Pharmacological and Parenteral Therapies: Cognitive 2. Antacids neutralize gastric acidity.
Level – Analysis: Concept – Medication. 3. This is the rationale for mucosal barrier agents.
27. 1. This is the outer canthus, and medications 4. Prostaglandin is responsible for production
are not administered to this area. of gastric acid. Antacids do not interfere with
2. The correct placement of ophthalmic prostaglandin production.
drops is to administer the medication in Content – Pharmacology: Integrated Nursing Process –
the lower conjunctival sac. Implementation: Client Needs – Physiological Integrity,
3. This is the sclera, and the correct placement Pharmacological and Parenteral Therapies: Cognitive
of eyedrops is in the lower conjunctival sac. Level – Analysis: Concept – Medication.
4. This is the inner canthus, where pressure can 31. 1. The client’s temperature would not affect the
be applied gently after instilling eyedrops to administration of this medication.
help prevent the systemic absorption of oph- 2. ACE inhibitors sometimes cause the client to
thalmic medications. develop a cough which requires discontinuing
Content – Pharmacology: Integrated Nursing Process – the medication, but this is a calcium channel
Implementation: Client Needs – Physiological Integrity, blocker.
Pharmacological and Parenteral Therapies: Cognitive 3. This blood pressure reading indicates the
Level – Application: Concept – Medication. client’s medication is effective.
28. 1. This potassium level is within normal limits; 4. This indicates orthostatic hypotension,
therefore, the nurse would administer the and the nurse should assess the client’s BP
medication. before administering the medication.
2. This indicates the medication is effective and Content – Pharmacology: Integrated Nursing Process –
the nurse should not question administering Assessment: Client Needs – Safe Effective Care Environ-
the medication. ment, Management of Care: Cognitive Level – Analysis:
3. This indicates the client is dehydrated Concept – Medication.
and the nurse should discuss this with the 32. 1. An unlabeled use for quinine is the pro-
HCP prior to administering another dose, phylaxis and treatment of nocturnal
which could increase the dehydration and leg cramps, which are associated with
could cause renal failure. arthritis, diabetes, varicose veins, and
4. This indicates the medication is effective. Daily arteriosclerosis.
weight changes reflect fluid gain and loss. 2. A muscle relaxant is prescribed for muscle
Content – Pharmacology: Integrated Nursing Process – spasms, and leg cramps are not always the
Assessment: Client Needs – Safe Effective Care Environ- result of muscle spasms.
ment, Management of Care: Cognitive Level – Analysis: 3. The question is addressing the relief of leg
Concept – Medication. cramps, and sleeping pills will not help leg
29. 1. The nurse should investigate any herbs a cramps.
client is taking, especially if the client has a 4. The client does not need an opioid analgesic
condition which requires long-term medica- because it may cause addiction; this type of
tion, such as antirejection medication. medication is given for acute pain for a short
2. The nurse should remain nonjudgmental but period. Prolonged use of Darvon compounds
must intervene if the alternative treatment also predisposes the client to renal cell
poses a risk to the client. carcinoma.
3. The client may need to be referred for Content – Pharmacology: Integrated Nursing Process –
psychological counseling, but it is not the Planning: Client Needs – Physiological Integrity, Pharma-
first action the nurse should take. cological and Parenteral Therapies: Cognitive Level –
4. St. John’s wort decreases the effects of Synthesis: Concept – Medication.
many medications, including oral contra- 33. 1. The nurse should always assess for
ceptives, antiretrovirals, and transplant allergies, but especially when administer-
immunosuppressant drugs. Rejection of ing antibiotics, which are notorious for
the client’s kidney could occur if the client allergic reactions.
continues to use St. John’s wort. 2. If specimens are not obtained for C&S
Content – Pharmacology: Integrated Nursing Process – prior to administering the first dose of
Implementation: Client Needs – Physiological Integrity, antibiotic, the results will be skewed.
Pharmacological and Parenteral Therapies: Cognitive
Level – Analysis: Concept – Medication.

19_Colgrove_Ch19.indd 721 02/06/16 4:02 PM


722 M ed -S urg S uccess

3. One (1) of the five (5) rights is to admin- 36. 1. Adenosine is ordered for supraventricular
ister the medication to the “right client.” tachycardia.
Checking the armband on the client 2. Epinephrine is administered during a code
with the MAR and medication is a way to vasoconstrict the periphery and shunt the
to ­ensure this. blood to the central circulating system.
4. The 2005 Joint Commission standards 3. Atropine is used for asystole or symptomatic
require two forms of identification prior sinus bradycardia.
to administering medications. The client’s 4. Amiodarone is the suggested drug of
armband and medical record number pro- choice for ventricular irritability according
vide one form of identifying information, to the ACLS algorithm to suppress ven-
and the client’s birthday is the second tricular ectopy.
form of identification in most health-care Content – Pharmacology: Integrated Nursing Process –
facilities. This is a nationwide emphasis Planning: Client Needs – Safe Effective Care Environment,
to help prevent medication errors. Management of Care: Cognitive Level – Synthesis:
5. The stem does not state it is an aminoglyco- Concept – Medication.
side antibiotic, and it is the initial dose, which 37. 1. The blood pressure must be continuously
means there is no medication in the system monitored more often, at least every 10 to
even if it were an aminoglycoside antibiotic. 15 minutes.
Content – Pharmacology: Integrated Nursing Process – 2. The peripheral pulses should be monitored
Implementation: Client Needs – Safe Effective Care more frequently than every shift, but dopamine
Environment, Safety and Infection Control: Cognitive
has no direct effect on the peripheral pulses.
Level – Application: Concept – Medication.
3. The client’s urine output should be
34. 1. This medication is prescribed for clients with monitored because low-dose dopamine is
an overactive bladder. administered to maintain renal perfusion;
2. There is no contraindication for a client with higher doses can cause vasoconstriction
type 2 diabetes receiving this medication. of the renal arteries.
3. These drugs cause mydriasis, which 4. Dopamine is not inactivated when exposed
increases the intraocular pressure, which to light.
could lead to blindness. Glaucoma is Content – Pharmacology: Integrated Nursing Process –
caused by increased intraocular pressure. Implementation: Client Needs – Safe Effective Care
4. There is no contraindication for a client with Environment, Management of Care: Cognitive Level –
peripheral vascular disease receiving this Application: Concept – Medication.
medication. 38. 1. The ST segment becoming more depressed
Content – Pharmacology: Integrated Nursing Process – indicates a worsening of the ­oxygenation of
Assessment: Client Needs – Safe Effective Care Environ- the myocardial tissue.
ment, Safety and Infection Control: Cognitive Level –
2. Reperfusion dysrhythmias indicate the
Analysis: Concept – Medication.
ischemic heart tissue is receiving oxygen
35. 1. The nurse should use nonsterile gloves to and is viable heart tissue.
apply ointment but should first wash his or 3. The creatine kinase CK-MB isoenzyme
her hands. elevates when there is necrotic heart tissue
2. The client’s leg should be cleansed prior to and does not indicate if thrombolytic therapy
administering a new application of ointment, is successful.
but it is not the first intervention. 4. D-dimer is used to diagnose pulmonary
3. The nurse should always check the client’s embolus.
armband, but it is not the first intervention Content – Pharmacology: Integrated Nursing Process –
when the nurse enters the room. Assessment: Client Needs – Physiological Integrity, Reduc-
4. Hand washing is the first intervention tion of Risk Potential: Cognitive Level – Analysis:
which must be done when the nurse Concept – Medication.
enters the client’s room before any con- 39. 1. Tinnitus, ringing in the ears, is a sign of
tact with the client; it is also the last inter- aspirin toxicity and needs to be reported
vention the nurse does after caring for the to the HCP; the aspirin should be stopped
client and leaving the room. immediately.
Content – Pharmacology: Integrated Nursing Process – 2. Diarrhea is a complication of many medica-
Implementation: Client Needs – Safe Effective Care tions but not with aspirin.
Environment, Safety and Infection Control: Cognitive
3. Tetany is muscle twitching secondary to
Level – Application: Concept – Medication.
hypocalcemia.

19_Colgrove_Ch19.indd 722 02/06/16 4:02 PM


C hapter 19  P harmacology 723

4. Aspirin does not cause paresthesia, which is Content – Pharmacology: Integrated Nursing Process –
numbness or tingling. Assessment: Client Needs – Physiological Integrity, Phar-
macological and Parenteral Therapies: Cognitive Level –
Content – Pharmacology: Integrated Nursing Process –
Analysis: Concept – Medication.
Planning: Client Needs – Physiological Integrity, Pharma-
cological and Parenteral Therapies: Cognitive Level – 43. 1. A daily cathartic is a colonic stimulant, which
Synthesis: Concept – Medication. results in dependency and a narrowing of
40. 1. Loop diuretics cause loss of potassium the lumen of the colon, which increases
in the urine output; therefore, the client constipation.
should be receiving potassium supple- 2. Although the client may think a medica-
ments. Hypokalemia can lead to life- tion for bowel movements is necessary, the
threatening cardiac dysrhythmias. nurse should teach the client this medica-
2. A cardiac glycoside, digoxin, is administered tion can cause serious complications, such
for congestive heart failure, but it is not nec- as dependency and narrowing of the colon.
essary when administering a loop diuretic. 3. Fiber will help increase the roughage, which
3. An ACE inhibitor is not prescribed along may help prevent constipation, but the most
with a loop diuretic. It may be ordered for important action is to empower the client to
congestive heart failure. make informed decisions about medications.
4. A potassium cation, Kayexalate, is ordered 4. The nurse should not refuse to administer the
to remove potassium through the bowel for medication; the nurse should talk to the client
clients with hyperkalemia. and, if needed, the HCP before administering
Content – Pharmacology: Integrated Nursing Process –
the medication.
Planning: Client Needs – Physiological Integrity, Pharma- Content – Pharmacology: Integrated Nursing Process –
cological and Parenteral Therapies: Cognitive Level – Implementation: Client Needs – Physiological Integrity,
Synthesis: Concept – Medication. Pharmacological and Parenteral Therapies: Cognitive
Level – Application: Concept – Medication.
41. 1. Interferon is administered to treat hepatitis
and some cancers, but it does not stimulate 44. 1. This should be documented in the client’s
the bone marrow. nurse’s notes because this is a prn medication,
2. Neupogen is a granulocyte-stimulating but it is not the priority medication.
factor which stimulates the bone marrow 2. The nurse would not administer another
to produce white blood cells (WBCs), narcotic, which is what caused the need for
which this client needs because the nor- Narcan in the first place.
mal WBC count is 4.5 to 11 (× 103)/mm3. 3. Oxygen will not help reverse respiratory
3. Neumega stimulates the production of plate- depression secondary to a narcotic overdose.
lets, but the client’s platelet count of 160 is 4. Narcan is administered when the client has
normal [100 to 400 (× 103)/mm3]. received too much of a narcotic. Narcan
4. Procrit stimulates the production of red has a short half-life of about 30 minutes
blood cells and hemoglobin, but a hemoglo- and the client will be at risk for respiratory
bin of 12.2 is normal for a woman (11.5 to depression for several hours; therefore, the
15.5 g/dL). nurse should assess the client frequently.
Content – Pharmacology: Integrated Nursing Process – Content – Pharmacology: Integrated Nursing Process –
Assessment: Client Needs – Physiological Integrity, Pharma- Implementation: Client Needs – Physiological Integrity,
cological and Parenteral Therapies: Cognitive Level – Pharmacological and Parenteral Therapies: Cognitive
Synthesis: Concept – Medication. Level – Analysis: Concept – Medication.

42. 1. This medication must be taken for life 45. 1. The pituitary gland is not directly affected
because the client has to have received some by the steroid and is not why the medication
type of transplant or have severe rheumatoid must be gradually tapered.
arthritis for it to be prescribed. 2. Steroids do not affect the pancreas’s produc-
2. Exposure to hepatitis does not have anything tion of insulin.
to do with receiving this medication. 3. When the client is receiving exogenous
3. Imuran is not a drug of choice for treating steroids, the adrenal glands stop produc-
pneumonia; therefore, the nurse must find ing cortisol, and if the medication is not
out why the client is taking it (either for a tapered, the client can have a severe
renal transplant or for severe rheumatoid hypotensive crisis, known as adrenal gland
arthritis). insufficiency or addisonian crisis.
4. Imuran does not affect the antigen–antibody 4. The adrenal gland, not the thyroid gland,
reaction. produces cortisol.

19_Colgrove_Ch19.indd 723 02/06/16 4:02 PM


724 M ed -S urg S uccess

Content – Pharmacology: Integrated Nursing Process – 49. 1. Flatulence (“gas”) is an expected side effect
Implementation: Client Needs – Physiological Integrity, which is not life threatening and does not
Pharmacological and Parenteral Therapies: Cognitive need to be reported to the HCP.
Level – Analysis: Concept – Medication. 2. A weight loss of two (2) pounds would not
46. 1. This medication is taken up to one (1) need to be reported to the HCP because this
year, and the public health department medication does not affect the client’s weight.
will pay for the medications and make 3. Muscle pain may indicate arthralgias, my-
sure the client complies because it is a ositis, or rhabdomyolysis, which are com-
public health risk. plications which would cause the HCP to
2. The client is in isolation until three (3) con- discontinue the medication because its
secutive early-morning sputum cultures are continued use may lead to liver failure.
negative, which is usually in about two (2) to 4. Not having a bowel movement may be
four (4) weeks. important to the client, but clients do not
3. Pork products do not interact with these have to have daily bowel movements.
medications. Content – Pharmacology: Integrated Nursing Process –
4. The client’s urine and all body fluids may Implementation: Client Needs – Physiological Integrity,
turn orange from the rifampin. Pharmacological and Parenteral Therapies: Cognitive
Level – Analysis: Concept – Medication.
Content – Pharmacology: Integrated Nursing Process –
Planning: Client Needs – Physiological Integrity, Pharma- 50. 1. Aspirin causes GI distress and should be
cological and Parenteral Therapies: Cognitive Level – taken with food.
Synthesis: Concept – Medication. 2. Tylenol is recommended for pain and can be
47. 1. This medication should be administered safely taken with a daily baby aspirin.
intradermally with the needle barely in- 3. Aspirin (acetylsalicylic acid [ASA]) does not
serted under the skin so a wheal (bubble) cause joint pain; in fact, it may provide some
forms after the injection. relief because of its anti-inflammatory action,
2. Cleansing the forearm with an alcohol swab but when aspirin is taken daily, it is an anti-
is standard procedure and would not warrant platelet medication.
immediate intervention. 4. ASA is known to cause gastric upset which
3. Circling the site is an appropriate interven- can lead to gastric bleeding, and dark, tarry
tion so, when the skin test is read and no stools may indicate upper GI bleeding.
reaction is occurring, the nurse will be able Content – Pharmacology: Integrated Nursing Process –
to document a negative skin test reading. Planning: Client Needs – Physiological Integrity, Pharma-
4. The skin test is read in three (3) days to cological and Parenteral Therapies: Cognitive Level –
determine the results. Synthesis: Concept – Medication.
Content – Pharmacology: Integrated Nursing Process – 51. 1. Dilantin will crystallize in the tubing
Evaluation: Client Needs – Physiological Integrity, Phar- and is not compatible with any IV fluid
macological and Parenteral Therapies: Cognitive Level – except normal saline. The IV tubing
Synthesis: Concept – Medication. must be flushed before the medication is
48. 1. This classification of medication is used to administered.
prevent transmission of this virus to the fetus 2. The therapeutic Dilantin level is 10 to
during pregnancy; therefore, it does not harm 20 mcg/mL; therefore, this is a toxic level.
the fetus and the client can be pregnant while 3. The medication is pushed at 50 mg/min.
taking this medication. 4. If the tubing turns cloudy, it means it is not
2. Only condoms reduce the risk or absti- compatible and the nurse must stop the IVP
nence will prevent transmission of the immediately and discontinue the IV.
herpes virus. Content – Pharmacology: Integrated Nursing Process –
3. There are side effects to every drug; this Implementation: Client Needs – Physiological Integrity,
one causes headache, dizziness, nausea, and Pharmacological and Parenteral Therapies: Cognitive
anorexia. Level – Application: Concept – Medication.
4. This medication does not directly affect the 52. 1. There is no machine for home use which
liver, and liver function tests (LFTs) are not monitors Dilantin levels. Levels are usually
required monthly. checked every six (6) months to one (1) year
Content – Pharmacology: Integrated Nursing Process – by venipuncture and laboratory tests.
Planning: Client Needs – Physiological Integrity, Pharma- 2. Dilantin causes gingival hyperplasia, and
cological and Parenteral Therapies: Cognitive Level – mouth care and dental care are priority to
Synthesis: Concept – Medication. help prevent rotting of the teeth.

19_Colgrove_Ch19.indd 724 02/06/16 4:02 PM


C hapter 19  P harmacology 725

3. Some states allow seizure-free clients with 3. The temperature does not affect the adminis-
epilepsy to drive, but some states don’t. The tration of this medication. NSAIDs would be
word “never” in this distracter should elimi- prescribed for fever.
nate it as a possible correct answer. 4. The liver and kidneys are responsible for
4. Alcohol should be strictly prohibited when metabolizing and excreting all medications,
taking anticonvulsant medications. but the tests are not routinely monitored for
Content – Pharmacology: Integrated Nursing Process – NSAIDs.
Planning: Client Needs – Physiological Integrity, Pharma- Content – Pharmacology: Integrated Nursing Process –
cological and Parenteral Therapies: Cognitive Level – Implementation: Client Needs – Physiological Integrity,
Synthesis: Concept – Medication. Pharmacological and Parenteral Therapies: Cognitive
Level – Application: Concept – Medication.
53. 1. Alcohol creates a disulfiram-like reaction to
the medication, which causes severe nausea, 57. 1. The XL means the medication is extended
vomiting, and extreme hypertension. release and cannot be crushed.
2. Trichomonas vaginalis is an asymptomatic sexu- 2. Whole capsules or tablets cannot be adminis-
ally transmitted disease in males. If the male tered through a feeding tube.
partner is not simultaneously treated, then he 3. The client has a feeding tube and is not able
can reinfect the female. to swallow; therefore, the nurse should not
3. This sexually transmitted disease can be administer the medications orally.
transmitted via oral routes. 4. Tablets which are enteric coated or
4. This is a concept which must be taught to all extended release cannot be crushed and
clients taking antibiotics: Take all the medica- administered via the N/G tube. This
tions as prescribed. would allow 24 hours worth of medication
Content – Pharmacology: Integrated Nursing Process – into the client’s system at one time. The
Evaluation: Client Needs – Physiological Integrity, Phar- nurse should ask the HCP to change the
macological and Parenteral Therapies: Cognitive Level – medication to a form which is not enteric
Synthesis: Concept – Medication. coated and not extended release. Then it
can be crushed and administered through
54. 1. If the nurse does not wear the gloves, the
the feeding tube.
nurse can absorb the medication and get
a headache. Content – Pharmacology: Integrated Nursing Process –
2. The old nitroglycerin paste must be removed Implementation: Client Needs – Physiological Integrity,
Pharmacological and Parenteral Therapies: Cognitive
because it could cause an overdose of the
Level – Application: Concept – Medication.
medication.
3. The paper should be applied to a clean, dry, 58. 1. The vaccine is administered in a series of
hairless area. three (3) injections and is reported to be
4. The medication can be placed on the chest, effective for life, but boosters may be given
arms, back, or legs. every five (5) years.
5. A headache is a common side effect and 2. This is the incorrect administration for
should not be reported to HCP. hepatitis B vaccine.
Content – Pharmacology: Integrated Nursing Process – 3. Hepatitis B is given in three (3) doses—
Implementation: Client Needs – Physiological Integrity, initially, then at one (1) month, and then
Pharmacological and Parenteral Therapies: Cognitive again at six (6) months.
Level – Application: Concept – Medication. 4. Hepatitis B vaccine is given intramuscularly
55. 125 gtt/min. in the deltoid muscle.
A microdrip is 60 gtt/mL. The formula for Content – Pharmacology: Integrated Nursing Process –
this dosage problem is as follows: Implementation: Client Needs – Physiological Integrity,
Pharmacological and Parenteral Therapies: Cognitive
1000 mL × 60 = 60,000 = 125 gtt/min
Level – Application: Concept – Medication.
480min     480
Content – Pharmacology: Integrated Nursing Process – 59. 1. An output greater than intake indicates the
Implementation: Client Needs – Physiological Integrity, medication is effective, and there is no need
Pharmacological and Parenteral Therapies: Cognitive to restrict the fluid intake.
Level – Application: Concept – Medication. 2. There is no reason to insert a Foley cath-
eter in the client who is urinating without
56. 1. This medication is harsh on the lining
difficulties.
of the stomach and should be taken with
3. As long as the client is receiving diuretics,
meals.
the client should be on intake and output
2. Abdominal striae occur with steroids, not
monitoring.
NSAIDs.

19_Colgrove_Ch19.indd 725 02/06/16 4:02 PM


726 M ed -S urg S uccess

4. The UAP can document the client’s fluid Content – Pharmacology: Integrated Nursing Process –
intake and output numbers on the bedside Implementation: Client Needs – Physiological Integrity,
record; this is one of the UAP’s duties. Pharmacological and Parenteral Therapies: Cognitive
Level – Analysis: Concept – Medication.
Content – Pharmacology: Integrated Nursing Process –
Planning: Client Needs – Physiological Integrity, Pharma- 63. 1. The new antibiotic must be started as soon as
cological and Parenteral Therapies: Cognitive Level – the medication arrives from the pharmacy.
Synthesis: Concept – Medication. 2. Waiting until the next day could cause serious
60. 1. This is a correct intervention when applying harm, with the client possibly going into sep-
a patch; therefore, the charge nurse would tic shock.
not have to intervene. 3. The HCP does not determine when the
2. The fentanyl patch takes about 24 hours medications are administered; this is a nurs-
to develop full analgesic effect; the patch ing intervention.
should be replaced every 72 hours. 4. A new IV antibiotic must be initiated as
3. The sites should be rotated to prevent irrita- soon as possible, at least within one (1)
tion to the skin. hour. A broad-spectrum antibiotic is or-
4. This is the correct way to administer all dered until C&S results are determined.
medications. Then, an antibiotic which will specifically
Content – Pharmacology: Integrated Nursing Process – target the infectious organism must be
Evaluation: Client Needs – Physiological Integrity, Phar- started immediately.
macological and Parenteral Therapies: Cognitive Level – Content – Pharmacology: Integrated Nursing Process –
Synthesis: Concept – Medication. Implementation: Client Needs – Physiological Integrity,
Pharmacological and Parenteral Therapies: Cognitive
61. 1. The nurse should prepare to administer the Level – Application: Concept – Medication.
medication, but it is not the first intervention.
2. A CT scan must be done to rule out a 64. 920 units/hr.
hemorrhagic CVA because, if it is a 20000 units ÷ 500 mL = 40 units/mL
hemorrhagic stroke, thrombolytic therapy 40units/mL × 23 mL/hr = 920 units/hr
will increase bleeding in the head. Content – Pharmacology: Integrated Nursing Process –
3. The client receiving thrombolytic therapy Implementation: Client Needs – Physiological Integrity,
will be in the ICD because the client needs Pharmacological and Parenteral Therapies: Cognitive
constant surveillance during therapy. H ­ eparin Level – Application: Concept – Medication.
will be started, but this is not the first 65. 1. Tegretol is photosensitive, but the client must
intervention. wear SPF of at least 30 to be protected.
4. The nurse should check to determine if the 2. This medication has a therapeutic level
client is allergic to medications, but in this which must be maintained to help prevent
situation the client must have a CT before seizures. The therapeutic range is from
any other action is taken. Cross sensitivity 6 to 12 mcg/mL.
usually occurs with antibiotics, not thrombo- 3. Dilantin, another anticonvulsant, causes
lytic therapy. hyperplastic gingivitis, but carbamazepine
Content – Pharmacology: Integrated Nursing Process – does not.
Implementation: Client Needs – Physiological Integrity, 4. The client with seizure disorder should only
Pharmacological and Parenteral Therapies: Cognitive take showers because, if a seizure occurs in
Level – Analysis: Concept – Medication. the bathtub, the client could drown.
62. 1. PPIs decrease gastric secretion and are Content – Pharmacology: Integrated Nursing Process –
prescribed for clients to prevent Curling’s Planning: Client Needs – Physiological Integrity, Pharma-
stress ulcer. PPIs are ordered for most cli- cological and Parenteral Therapies: Cognitive Level –
ents in the intensive care department, not Synthesis: Concept – Medication.
just clients with burns. 66. 1. Yellow eyes would indicate the client is
2. PPIs do not treat infections; antibiotics treat experiencing some type of hepatic toxicity,
infections. which would warrant the medication be-
3. PPIs do not cause continuous constriction. ing discontinued immediately. During the
Dopamine might do this. first few months of treatment, the client
4. Positive nitrogen balance accomplished is closely monitored for hepatic toxicity
through nutritional interventions will help because deaths have occurred.
promote tissue regeneration. 2. The medication dose may need to be in-
creased, but Depakote is administered to pre-
vent the mood swings.

19_Colgrove_Ch19.indd 726 02/06/16 4:02 PM


C hapter 19  P harmacology 727

3. The BP is slightly elevated, but it is not re- 70. 1. Sinemet does not affect the client’s muscle
lated to the medication. strength; it affects the smoothness of muscle
4. The therapeutic serum Depakote level is movement.
50 to 100 mcg/mL; therefore, the client is 2. Cogwheel motion (jerky, uneven move-
within therapeutic range. ments) is a symptom of Parkinson’s dis-
Content – Pharmacology: Integrated Nursing Process – ease, and if the client is not experiencing
Assessment: Client Needs – Safe Effective Care Environ- these types of movements, then the medi-
ment, Management of Care: Cognitive Level – Analysis: cation is effective.
Concept – Medication. 3. There is no such thing as a carbidopa thera-
peutic level. The client’s signs/symptoms
67. 1. Safety is priority when administering a
determine the effectiveness of the medication.
phenothiazine because it causes sedative-
4. The client’s blood pressure should be assessed
like effects.
to determine if the client is having hypoten-
2. Evaluation is not priority over safety.
sion, which is a side effect of the medication,
3. The nurse should have assessed the client’s
but this does not determine the effectiveness
GI system prior to administering the anti-
of the medication.
emetic, not after.
4. Withholding fluids/food is an appropriate Content – Pharmacology: Integrated Nursing Process –
intervention to help prevent emesis, but it is Assessment: Client Needs – Physiological Integrity, Phar-
macological and Parenteral Therapies: Cognitive Level –
not priority over safety after administering
Analysis: Concept – Medication.
this medication.
Content – Pharmacology: Integrated Nursing Process – 71. 1. This diet is recommended for clients with
Implementation: Client Needs – Physiological Integrity, coronary artery disease, but it is not an inter-
Pharmacological and Parenteral Therapies: Cognitive vention specific for this medication.
Level – Analysis: Concept – Medication. 2. This medication is taken once a day in the
evening.
68. 1. This is the correct area to administer subcu-
3. Atorvastatin (Lipitor) is taken at night to
taneous heparin, but not Lovenox.
enhance the enzymes which metabolize
2. This is in the client’s anterior thigh, which
cholesterol.
may be used for insulin administration but
4. There is no machine to test daily cholesterol
not for Lovenox, and a 25-gauge 1/2-inch
levels. The cholesterol level is checked every
needle is used to administer Lovenox.
three (3) to six (6) months.
3. This is the upper arm area, which is used for
subcutaneous insulin, but not Lovenox. Content – Pharmacology: Integrated Nursing Process –
4. Lovenox is administered in the “love Planning: Client Needs – Physiological Integrity, Pharma-
cological and Parenteral Therapies: Cognitive Level –
handles,” which is in the anterolateral
Synthesis: Concept – Medication.
abdomen; this helps prevent abdominal
wall trauma. 72. 1. The client in end-stage renal disease does not
Content – Pharmacology: Integrated Nursing Process – normally urinate, and urine output does not
Implementation: Client Needs – Physiological Integrity, determine if this medication is effective.
Pharmacological and Parenteral Therapies: Cognitive 2. Kayexalate does not affect phosphorus levels.
Level – Knowledge: Concept – Medication. 3. The client being in normal sinus rhythm is
good, but it does not determine if the medi-
69. 1. This may be a question the nurse asks, but it
cation is effective.
doesn’t matter because the nurse will clean
4. Kayexalate is a cation and exchanges
the site again.
sodium ions for potassium ions in the
2. The nail does not matter and it will
intestines, thereby lowering the serum
not be cultured; it is assumed the nail is
potassium level. Therefore, a serum
contaminated.
potassium level within normal limits
3. The tetanus shot must be received every
would indicate the medication is effective.
10 years to prevent tetany, also known as
Normal potassium levels are 3.5 to
“lockjaw.”
5.5 mEq/L.
4. Being able to walk on the foot is not a prior-
ity question. Determining the status of the Content – Pharmacology: Integrated Nursing Process –
tetanus shot is priority. Assessment: Client Needs – Physiological Integrity, Phar-
macological and Parenteral Therapies: Cognitive Level –
Content – Pharmacology: Integrated Nursing Process – Analysis: Concept – Medication.
Assessment: Client Needs – Safe Effective Care Environ-
ment, Safety and Infection Control: Cognitive Level –
Analysis: Concept – Medication.

19_Colgrove_Ch19.indd 727 02/06/16 4:02 PM


728 M ed -S urg S uccess

73. 1. The ABG results indicate metabolic 3. This effect does not mean the client is dying.
­acidosis, and the treatment of choice is It means the medication is wearing off.
­sodium bicarbonate. 4. This is not the desired effect of the
2. Oxygen is the treatment of choice for respira- medication.
tory acidosis. Content – Pharmacology: Integrated Nursing Process –
3. Epinephrine is administered in a code situation. Implementation: Client Needs – Physiological Integrity,
4. This is milk of magnesia, which is an antacid/ Pharmacological and Parenteral Therapies: Cognitive
laxative, but it is not the treatment for meta- Level – Analysis: Concept – Medication.
bolic acidosis. 77. 1. This information really doesn’t have bearing
Content – Pharmacology: Integrated Nursing Process – on the current situation.
Planning: Client Needs – Safe Effective Care Environment, 2. The spouse can stay in the room if able to
Management of Care: Cognitive Level – Synthesis:
stay calm and not upset the client.
Concept – Medication.
3. The nurse should address the client’s fear, but
74. 1. Propranolol is taken prophylactically, which it is not the most pertinent information.
means the client should take the medication 4. A local anesthetic will be administered
routinely whether the client has a headache to numb the area prior to suturing. The
or not. same classification of drugs is used to
2. Beta blockers decrease the heart rate. numb the mouth before dental proce-
If the radial pulse is less than 60 bpm, the dures, and this client may be allergic to
client should hold the medication and the numbing medication.
­notify the health-care provider. Content – Pharmacology: Integrated Nursing Process –
3. This medication will mask tachycardia in Assessment: Client Needs – Safe Effective Care Environ-
­clients with diabetes, an early symptom of ment, Safety and Infection Control: Cognitive Level –
­hypoglycemia. Thirst and dry mouth are Analysis: Concept – Medication.
signs of hyperglycemia, but this client does
78. 1. The medication is administered through the
not have diabetes.
nose, but it has no effect on the client’s ability
4. Beta blockers do not affect the client’s visual to breathe.
acuity; therefore, a change in light is not
2. Being thirsty all the time would indicate the
necessary.
medication is not effective.
Content – Pharmacology: Integrated Nursing Process – 3. Neither the medication nor the disease
Planning: Client Needs – Physiological Integrity, Pharma- process has anything to do with the glucose
cological and Parenteral Therapies: Cognitive Level –
level. A disease which affects the glucose level
Synthesis: Concept – Medication.
is diabetes mellitus, not diabetes insipidus.
75. 1. Atropine is used in clients with asystole or 4. Diabetes insipidus is characterized by the
symptomatic sinus bradycardia. client not being able to concentrate urine
2. Amiodarone is a Class C medication used for and excreting large amounts of dilute
ventricular dysfunction. urine. If the client is able to delay voiding
3. Adenosine is the drug of choice for clients for three (3) to four (4) hours, it indicates
with SVT. the medication is effective.
4. Dobutamine is used for clients in heart failure. Content – Pharmacology: Integrated Nursing Process –
Content – Pharmacology: Integrated Nursing Process – Assessment: Client Needs – Physiological Integrity, Phar-
Implementation: Client Needs – Safe Effective Care Envi- macological and Parenteral Therapies: Cognitive Level –
ronment, Management of Care: Cognitive Level – Analysis: Analysis: Concept – Medication.
Concept – Medication.
79. 1. This is the correct procedure for instilling
76. 1. Loss of effect of the medication occurs eardrops for children.
near the end of a dosing interval and indi- 2. “Otic” refers to the ear. Instilling ear-
cates the plasma drug level has declined to drops in the adult must be done by pull-
subtherapeutic value. This is an expected ing the ear up and back to straighten the
occurrence with the medication and the ­eustachian tube.
chronic nature of the disease. 3. This is the correct procedure for placing
2. Increasing the dose increases the peripheral ­ophthalmic drops in the eye.
action of the drug on the heart and vessels. 4. Pressure is applied to the inner canthus to
Because 75% of the drug never crosses the prevent eye medication from entering the
blood–brain barrier, the dose may not be systemic circulation.
increased.

19_Colgrove_Ch19.indd 728 02/06/16 4:02 PM


C hapter 19  P harmacology 729

Content – Pharmacology: Integrated Nursing Process – 83. 1. This is an antacid, but it is not being adminis-
Implementation: Client Needs – Physiological Integrity, tered to this client for that reason.
Pharmacological and Parenteral Therapies: Cognitive 2. The client is in end-stage renal disease
Level – Application: Concept – Medication. (ESRD), but burning on urination is not a
80. 1. The client should increase fluid intake to help sign of ESRD; it is a sign of urinary tract
flush the bacteria through the kidneys and infection.
bladder. 3. A side effect of this medication is constipation,
2. The client has a chronic UTI, which will but having a normal bowel movement does
require antibiotics on a daily basis to keep the not indicate the medication is effective.
bacteria count under control. 4. This medication decreases absorption
3. The key to answering this question is of phosphates in the intestines, thereby
the word “chronic,” which indicates a ­decreasing serum phosphate levels. The
­continuing problem; this statement normal phosphate level is 2.5 to 4.5 mg/dL.
would be ­appropriate for an acute UTI. Content – Pharmacology: Integrated Nursing Process –
4. Diarrhea is a sign of superinfection, which Assessment: Client Needs – Physiological Integrity, Phar-
occurs when the antibiotic kills the good macological and Parenteral Therapies: Cognitive Level –
flora in the bowel. However, the client must Analysis: Concept – Medication.
keep taking the antibiotic, and Imodium is an 84. 1. Any medication which will prolong bleed-
OTC antidiarrheal. ing, as a platelet aggregate inhibitor does,
Content – Pharmacology: Integrated Nursing Process – should not be administered to the client
Evaluation: Client Needs – Physiological Integrity, Phar- for at least two (2) to three (3) days prior
macological and Parenteral Therapies: Cognitive Level – to surgery.
Synthesis: Concept – Medication. 2. The nurse should not question administering
81. 1. A Pap smear is usually done yearly and is used an antibiotic before surgery, especially not
to detect cervical cancer; HRT does not in- before gastrointestinal surgery.
crease the risk. 3. This is a medication for type 2 diabetes and
2. The risk of developing breast cancer should be administered the day before the
increases when the client is receiving surgery.
HRT. 4. The client will be receiving medications to
3. A bone density test is used to detect osteopo- evacuate the bowel.
rosis, and HRT improves bone density. Content – Pharmacology: Integrated Nursing Process –
4. Calcium levels are not affected by HRT. Implementation: Client Needs – Safe Effective Care Envi-
Content – Pharmacology: Integrated Nursing Process – ronment, Management of Care: Cognitive Level – Analysis:
Planning: Client Needs – Physiological Integrity, Reduction Concept – Medication.
of Risk Potential: Cognitive Level – Synthesis: Concept – 85. 1. This rash indicates a sensitivity reaction, and
Medication. the medication may need to be discontinued
82. 1. One (1) of the five (5) rights is the correct permanently or the dose should be decreased.
dose, and some medications must be divided 2. Increased fluid intake minimizes the risk
prior to administering. of renal calculi formation.
2. One (1) of the five (5) rights is the correct 3. To minimize gastric irritation, the medication
client, and this is making sure it is the correct should be taken with food or milk.
client. 4. Allopurinol increases the effects of oral dia-
3. One (1) of the five (5) rights is the ­correct betic medications; therefore, the dose should
route. Insulin cannot be administered intra- be decreased.
muscularly. It must be administered subcu- Content – Pharmacology: Integrated Nursing Process –
taneously or intravenously; therefore, this Planning: Client Needs – Physiological Integrity, Pharma-
action warrants immediate intervention. cological and Parenteral Therapies: Cognitive Level –
4. One (1) of the five (5) rights is the right time, Synthesis: Concept – Medication.
and the LPN has 30 minutes to one (1) hour 86. 1. Atropine in this dosage will not cause
to administer medications depending on hos- ­orthostatic hypotension, but it will increase
pital policy; therefore, this would not require the pulse rate.
intervention by the nurse. 2. The client should increase the fiber in the diet
Content – Nursing Management: Integrated Nursing because this medication may cause constipation.
Process – Evaluation: Client Needs – Safe Effective Care 3. An expected side effect of anticholinergic
Environment, Management of Care: Cognitive Level – medication is a dry mouth, and chewing
Synthesis: Concept – Medication. gum will help relieve the dryness.

19_Colgrove_Ch19.indd 729 02/06/16 4:02 PM


730 M ed -S urg S uccess

4. Isometric exercises are muscle-building ex- 89. 1. The client’s drug level is within the therapeu-
ercises (weight lifting, or “pumping iron”), tic range of 10 to 20 mcg/mL.
which will not help the client tolerate this 2. Expiratory wheezing would be expected in
medication and should not be recommended the client with status asthmaticus and there-
for any client. fore would not warrant intervention.
Content – Pharmacology: Integrated Nursing Process – 3. Muscle twitching indicates the client is
Implementation: Client Needs – Physiological Integrity, receiving too much medication and may
Pharmacological and Parenteral Therapies: Cognitive experience a seizure.
Level – Application: Concept – Medication. 4. The client is having trouble breathing, and
eating requires energy. Therefore, the client
87. 1. Telemetry should be monitored during
may not want to eat a meal or the client may
therapy to ensure the client does not
not like the hospital food, which would not
develop worsening of dysrhythmias.
warrant immediate intervention.
2. The client taking amiodarone is at risk for
pulmonary toxicity and developing adult Content – Pharmacology: Integrated Nursing Process –
respiratory distress syndrome (ARDS); Assessment: Client Needs – Physiological Integrity, Pharma-
cological and Parenteral Therapies: Cognitive Level –
therefore, the nurse should monitor the
Synthesis: Concept – Medication.
client’s respiratory status.
3. When the client is receiving medications 90. 1. The client with pitting pedal edema is
intravenously, monitoring the liver and in fluid volume overload, which should
renal function is appropriate; this drug make the nurse question administering an
causes hepatomegaly. osmotic diuretic because this medication
4. Intravenous vasoactive medications are will pull more fluid from the tissues into
inherently dangerous; fatalities have the circulatory system, causing further
occurred from amiodarone, so the nurse fluid volume overload.
confirming the order with another nurse 2. An osmotic diuretic is administered for in-
is appropriate. creased intracranial pressure; therefore, a
5. The nurse should never defibrillate a client client who is exhibiting decorticate posturing
who has a heartbeat, and nothing in the stem would need this medication.
states the client is in ventricular fibrillation. 3. A widening pulse pressure indicates increased
Content – Pharmacology: Integrated Nursing Process – intracranial pressure; therefore, the client
Implementation: Client Needs – Safe Effective Care Envi- needs the osmotic diuretic.
ronment, Management of Care: Cognitive Level – Analysis: 4. The doll’s eye test indicates increased in-
Concept – Medication. tracranial pressure, which is why the HCP
would prescribe the osmotic diuretic.
88. 1. This medication amount is too much and
must be divided into two injections to be given Content – Pharmacology: Integrated Nursing Process –
safely, but the nurse can do this independently Assessment: Client Needs – Safe Effective Care Environ-
ment, Safety and Infection Control: Cognitive Level –
and does not need to notify the HCP.
Analysis: Concept – Medication.
2. The nurse should not administer 4.8 mL in
one (1) injection. No more than three (3) mL 91. 1. Melena is black, tarry stool, which should not
should be administered in an intramuscular occur from taking this medication.
injection. 2. Gynecomastia, or breast development in
3. There is no reason for the nurse to discard men, is a complication of this medication.
this medication. Divide the medication and 3. Pyrosis, or heartburn, is why the client would
give two (2) injections. be taking this medication.
4. The nurse should never administer more 4. Eructation, or belching, is not a complication
than three (3) mL in an intramuscular of this medication.
injection because a larger amount could Content – Pharmacology: Integrated Nursing Process –
cause damage to the muscle. The nurse Planning: Client Needs – Physiological Integrity, Pharma-
should divide the dose and administer two cological and Parenteral Therapies: Cognitive Level –
(2) injections. Synthesis: Concept – Medication.
Content – Pharmacology: Integrated Nursing Process – 92. 1. Muscle relaxants do not cause GI distress.
Implementation: Client Needs – Physiological Integrity, 2. Muscle relaxants, with the exception of
Pharmacological and Parenteral Therapies: Cognitive
baclofen, do not need to be tapered off.
Level – Analysis: Concept – Medication.
3. Initially muscle relaxants cause drowsi-
ness, so safety is an important issue.

19_Colgrove_Ch19.indd 730 02/06/16 4:02 PM


C hapter 19  P harmacology 731

4. As a safety precaution, the client should 4. The client taking an antidepressant medica-
avoid drinking alcohol while taking muscle tion does not automatically need a referral to
relaxants. a psychologist.
Content – Pharmacology: Integrated Nursing Process – Content – Pharmacology: Integrated Nursing Process –
Planning: Client Needs – Physiological Integrity, Pharma- Implementation: Client Needs – Physiological Integrity,
cological and Parenteral Therapies: Cognitive Level – Pharmacological and Parenteral Therapies: Cognitive
Synthesis: Concept – Medication. Level – Analysis: Concept – Medication.

93. 1. The client may be given a steroid, such as 96. 1. Many times, especially with elderly ­clients,
prednisone, but not an NSAID. sedatives extend the desired effects longer
2. An antihistamine is prescribed to decrease than expected; therefore, the nurse should
symptoms of a cold or the flu, but it is not check to see if the client received any
prescribed for asthma. sleeping medication.
3. An ACE inhibitor prevents deterioration of 2. The nurse should assess why the client is
heart muscle and kidneys, but it is not a drug sleepy and then allow the client to sleep if the
of choice for the respiratory system. sleepiness is a result of receiving a sedative
4. Because 80% to 90% of adult-onset the previous night.
asthma is caused by gastroesophageal 3. If an elderly client is confused and drowsy,
reflux ­disease, a proton pump ­inhibitor the client should not be allowed to ambulate,
would be prescribed to decrease acid even if assistance is being provided, because
­reflux into the esophagus and subsequent of safety issues.
aspiration. 4. The nurse must determine if this is an ex-
Content – Pharmacology: Integrated Nursing Process – pected occurrence or a decrease in neurologi-
Planning: Client Needs – Physiological Integrity, Pharma- cal function before notifying the health-care
cological and Parenteral Therapies: Cognitive Level – provider.
Synthesis: Concept – Medication. Content – Pharmacology: Integrated Nursing Process –
94. 1. The pain medication should be administered Implementation: Client Needs – Safe Effective Care Envi-
ronment, Management of Care: Cognitive Level – Analysis:
as soon as possible but not before assessing for
Concept – Medication.
complications which might be causing pain.
2. The nurse must not administer the medica- 97. 1. Medication does not always come in the exact
tion too close to the last dose, but this is amount of the HCP’s order.
not the first intervention the nurse would 2. Because this is a narcotic, the nurse pre-
implement. paring the medication must have someone
3. The first step of the nursing process is to to verify and document the wastage of
assess, and the nurse must determine if 12.5 mg of the Demerol.
this is routine postoperative pain the 3. This would be a medication error because the
client should have or if this is a compli- order is for 37.5 mg, not 50 mg.
cation which requires immediate inter- 4. This would not be an appropriate interven-
vention. Decreased blood pressure and tion. The nurse can safely and accurately
increased pulse indicates hemorrhaging. administer the prescribed dose to the client.
4. Teaching distraction techniques is an ap- If the pain is not controlled with the amount,
propriate intervention, but the nurse should then the HCP should be notified.
medicate the client. Content – Pharmacology: Integrated Nursing Process –
Content – Pharmacology: Integrated Nursing Process – Implementation: Client Needs – Safe Effective Care
Implementation: Client Needs – Safe Effective Care Environment, Safety and Infection Control: Cognitive
Environment, Safety and Infection Control: Cognitive Level – Application: Concept – Medication.
Level – Analysis: Concept – Assessment.
98. 2 tablets.
95. 1. Just because a client is taking an antidepres-  The nurse needs to determine how many
sant, this does not mean he or she is suicidal. doses are to be given in one (1) day (24 hours)
2. The nurse should determine if the client if doses are to be eight (8) hours apart.
is in a depressed state or if the medication 24÷ 8 = 3 doses
is effective, so the nurse should ask the If 3,000 mg are to be given in three (3) doses,
client to rate the depression on a 1-to-10 then determine how much is given in each
scale, with 1 being no depression and 10 dose:
being the most depressed. 3000 ÷ 3 = 1,000 mg per dose
3. Antidepressants must be tapered off because  If the medication comes in 500-mg tablets,
of rebound depression. then to give 1,000 mg, the nurse must give:
1000 ÷ 500 = 2 tablets

19_Colgrove_Ch19.indd 731 02/06/16 4:02 PM


732 M ed -S urg S uccess

Content – Pharmacology: Integrated Nursing Process – Content – Pharmacology: Integrated Nursing Process –
Implementation: Client Needs – Physiological Integrity, Implementation: Client Needs – Safe Effective Care
Pharmacological and Parenteral Therapies: Cognitive Environment, Safety and Infection Control: Cognitive
Level – Application: Concept – Medication. Level – Application: Concept – Medication.

99. 1. This medication decreases inflamma- 101. In order of priority: 4, 3, 2, 1, 5.


tion by stabilizing the leukotrienes in the 4. Although the lidocaine is a prn order,
lung which initiate an asthma attack. this client is exhibiting a life-threatening
2. Children may outgrow asthma attacks, dysrhythmia, multifocal premature ven-
whereas adult asthmatics can control their tricular contractions.
disease, but there is no cure for asthma at 3. The client diagnosed with myasthenia
this time. gravis must have this medication as close
3. This is the scientific rationale for mast cell to the specific time as possible. This
inhibitors. medication allows skeletal muscle to
4. This is the scientific rationale for function; if this medication is delayed,
bronchodilators. the client may experience respiratory
Content – Pharmacology: Integrated Nursing Process – distress.
Implementation: Client Needs – Physiological Integrity, 2. Pain is a priority and should be attended
Pharmacological and Parenteral Therapies: Cognitive to after administering medications to cli-
Level – Analysis: Concept – Medication. ents in life-threatening situations.
1. This client is symptomatic, and the loop
100. 1. Although many nurses will do this, the cor-
diuretic should relieve some of the symp-
rect and ethical action is to take responsi-
toms of dyspnea.
bility for the error and just be thankful the
5. Intravenous antibiotics are priority, but
client did not have a problem.
this client has received several doses of
2. There is a chain of command to report med-
the medication or there would not be a
ication errors, which includes the charge
trough level, so this client’s medication
nurse and the health-care provider, not the
could wait until the other medications
director of nurses.
have been administered.
3. The ethical and correct action is to re-
port and document the medication error; Content – Pharmacology: Integrated Nursing Process –
remember to always assess the client. Implementation: Client Needs – Safe Effective Care Envi-
ronment, Management of Care: Cognitive Level –
4. The Peer Review Committee would not be
Analysis: Concept – Medication.
involved in one medication error unless the
client died or a life-threatening complica-
tion occurred, or if the nurse has a pattern of
behavior with multiple medication errors.

19_Colgrove_Ch19.indd 732 02/06/16 4:02 PM


Comprehensive
Final Examination
There is no substitute for knowledge.
20
—Kathryn Colgrove, RN, MS

This book is designed to assist the test taker to recognize elements of test construction and
to be able to think critically to arrive at the correct answer. Many hints have appeared in the
previous chapters. Some are general hints that apply to preparing for class and subsequently
to taking examinations (Chapter 1), some are specific tips for the different types of questions
about disorders/diseases of the different body systems (Chapters 2–18), and some are spe-
cific to pharmacology (Chapter 19). The test taker should now apply these hints, use all the
knowledge gained in class and study, and take the comprehensive examination.

COMPREHENSIVE FINAL EXAMINATION


1. The 44-year-old female client calls the clinic and 4. The nurse is teaching a class on sexually
tells the nurse she felt a lump while performing transmitted diseases to high school sophomores.
breast self-examination (BSE). Which question Which information should be included in the
should the nurse ask the client? discussion?
1. “Are you taking birth control pills?” 1. Oral sex decreases the chance of transmitting a
2. “Do you eat a lot of chocolate?” sexual disease.
3. “When was your last period?” 2. Sexual activity during menses decreases
4. “Are you sexually active?” transmission of diseases.
3. Frequent sexual activity is necessary to transmit
2. Which problem is priority for the 24-year-old
a sexual disease.
client diagnosed with endometriosis who is
4. Unprotected sex puts the individual at risk for
admitted to the gynecological unit?
many diseases.
1. Hemorrhage.
2. Pain. 5. The nurse has taught Kegel exercises to the client
3. Constipation. who is para 5, gravida 5. Which information
4. Dyspareunia. indicates the exercises have been effective?
1. The client reports no SOB when walking up
3. The 28-year-old client diagnosed with testicular
stairs.
cancer is scheduled for a unilateral orchiectomy.
2. The client has no complaints of stress
Which intervention should have priority in the
incontinence.
client’s plan of care?
3. The client denies being pregnant at this time.
1. Encourage the client to bank his sperm.
4. The client has lost 10 lb in the last two (2)
2. Discuss completing an advance directive.
months.
3. Explain follow-up chemotherapy and radiation.
4. Allow the client to express his feelings
regarding having cancer.

733

20_Colgrove_Ch20.indd 733 02/06/16 5:04 PM


734 M ed -S urg S uccess

6. Which diagnostic procedure does the nurse 3. Determine if the client smokes cigarettes.
anticipate being ordered for the 27-year-old 4. Recommend the client not go outside.
female client who is reporting irregular menses 5. Teach about safety and fall precautions.
and complaining of lower left abdominal pain
12. The 33-year-old client had a traumatic
during menses?
amputation of the right forearm as a result of
1. Pelvic sonogram.
a work-related injury. Which referral by the
2. Complete blood count (CBC).
rehabilitation nurse is most appropriate?
3. Kidney, ureter, bladder (KUB) x-ray.
1. Physical therapist.
4. Computed tomography (CT) of abdomen.
2. Occupational therapist.
7. The client diagnosed with Stage IV prostate 3. Workers’ compensation.
cancer is receiving chemotherapy. Which 4. State rehabilitation commission.
laboratory value should the nurse assess prior to
13. The client has a fractured right tibia. Which
administering the chemotherapy?
assessment data warrant immediate intervention?
1. Prostate-specific antigen (PSA).
1. The client complains of right calf pain.
2. Serum calcium level.
2. The nurse cannot palpate the radial pulse.
3. Complete blood count (CBC).
3. The client’s right foot is cold to touch.
4. Alpha-fetoprotein (AFP).
4. The nurse notes ecchymosis on the right leg.
8. Which client should the charge nurse of the day
14. The nurse identifies the problem “high risk for
surgery unit assign to a new graduate nurse in
complications” for the client with a right total
orientation?
hip replacement who is being discharged from
1. The client who had an arthroscopy with an
the hospital. Which problem would have the
AP of 110 and BP of 94/60.
highest priority?
2. The client with open reduction of the ankle
who is confused.
3. The client with a total hip replacement who is
9%
being transferred to the ICU.
4. The client diagnosed with low back pain who
has had a myelogram.
Anterior–18%
9. The client in the long-term care facility has Posterior–18%
severe osteoarthritis. Which nursing task should
the nurse delegate to the unlicensed assistive
9% 9%
personnel (UAP)?
1. Feed the client the breakfast meal.
2. Give the client Maalox, an antacid.
3. Monitor the client’s INR results.
4. Assist the client to the shower room.
10. The primary nurse is applying antiembolism 1%
hose to the client who had a total hip
18% 18%
replacement. Which situation warrants
immediate intervention by the charge nurse?
1. Two fingers can be placed under the top of
the band.
2. The peripheral capillary refill time is less than
three (3) seconds.
3. There are wrinkles in the hose behind the
knees.
4. The nurse does not place a hose on the foot
with a venous ulcer.
11. The 54-year-old female client is diagnosed with
osteoporosis. Which interventions should the 1. Self-care deficit.
nurse discuss with the client? Select all that 2. Impaired skin integrity.
apply. 3. Abnormal bleeding.
1. Instruct the client to swim 30 minutes every 4. Prosthetic infection.
day.
2. Encourage drinking milk with added vitamin D.

20_Colgrove_Ch20.indd 734 02/06/16 5:04 PM


C hapter 20  C omprehensive F inal E xamination 735

15. The client has sustained severe burns on both 21. The client is newly diagnosed with epilepsy.
the anterior right and left leg and the anterior Which statement indicates the client needs
chest and abdomen. According to the rule of clarification of the discharge teaching?
nines, what percentage of the body has been 1. “I can drive as soon as I see my HCP for my
burned? _____ follow-up visit.”
2. “I should get at least eight (8) hours of sleep
16. The nurse is planning the care for the client
at night.”
with multiple stage IV pressure ulcers. Which
3. “I should take my medication every day even
complication results from these pressure ulcers?
if I am sick.”
1. Wasting syndrome.
4. “I will take showers instead of taking tub
2. Osteomyelitis.
baths.”
3. Renal calculi.
4. Cellulitis. 22. The nurse observes the unlicensed assistive
personnel (UAP) taking vital signs on an
17. The client comes to the clinic complaining of
unconscious client. Which action by the UAP
itching on the left wrist near a wristwatch. The
warrants intervention by the nurse?
nurse notes an erythematous area along with
1. The UAP uses a vital sign machine to check
pruritic vesicles around the left wrist. Which
the BP.
condition should the nurse suspect?
2. The UAP takes the client’s temperature orally.
1. Contact dermatitis.
3. The UAP verifies the blood pressure
2. Herpes simplex 1.
manually.
3. Impetigo.
4. The UAP counts the respirations for
4. Seborrheic dermatitis.
30 seconds.
18. Which diagnostic test should the nurse expect to
23. The client diagnosed with a brain tumor who
be ordered for the client who has a nevus which
had radiation treatment and developed alopecia
is purple and brown with irregular borders?
asks, “When will my hair grow back?” Which
1. Bone scan.
statement is the nurse’s best response?
2. Skin biopsy.
1. “Your hair should start growing back within
3. Carcinoembryonic antigen (CEA).
three (3) weeks.”
4. Sonogram.
2. “Are you concerned your hair will not grow
19. The client with a closed head injury is admitted back?”
to the neurointensive care department following 3. “It may take months, if your hair grows back
a motor-vehicle accident. Which goal is an at all.”
appropriate short-term goal for the client? 4. “It may take a couple of years for the hair to
1. The client will maintain optimal level of grow back.”
functioning.
24. Which assessment data indicate the treatment
2. The client will not develop extremity
for the client diagnosed with bacterial meningitis
contractures.
is effective?
3. The client’s intracranial pressure will not be
1. There is a positive Brudzinski’s sign and
greater than 15 mm Hg.
photophobia.
4. The client will be able to verbalize feelings of
2. The client tolerates meals without nausea.
anger.
3. There is a positive Kernig’s sign and an
20. The 25-year-old client who has a C6 spinal elevated temperature.
cord injury is crying and asks the nurse, “Why 4. The client is able to flex the neck without
did I have to survive? I wish I was dead.” Which pain.
statement is the nurse’s best response?
25. The client is being evaluated to rule out
1. “Don’t talk like that. At least you are alive and
Parkinson’s disease. Which diagnostic test
able to talk.”
confirms this diagnosis?
2. “God must have something planned for your
1. A positive magnetic resonance imaging (MRI)
life. Pray about it.”
scan.
3. “You survived because the people at the
2. A biopsy of the substantia nigra.
accident saved your life.”
3. A stereotactic pallidotomy.
4. “This must be difficult to cope with. Would
4. There is no test that confirms this diagnosis.
you like to talk?”

20_Colgrove_Ch20.indd 735 02/06/16 5:04 PM


736 M ed -S urg S uccess

26. The client diagnosed with a transient ischemic 31. The client diagnosed with end-stage congestive
attack (TIA) is being discharged from the heart failure is being cared for by the home
hospital. Which medication should the nurse health nurse. Which intervention should the
expect the HCP to prescribe? nurse teach the caregiver?
1. The oral anticoagulant warfarin (Coumadin). 1. Report any time the client starts having
2. The antiplatelet medication, a baby aspirin. difficulty breathing.
3. The beta blocker propranolol (Inderal). 2. Notify the HCP if the client gains more than
4. The anticonvulsant valproic acid (Depakote). 3 lb in a week.
3. Teach how to take the client’s apical pulse
27. The nurse has just received the shift assessment.
for one (1) full minute.
Which client should the nurse assess first?
4. Encourage the client to participate in
1. The client with encephalitis who has myalgia.
30 minutes of exercise a day.
2. The client who is complaining of chest pain.
3. The client who refuses to eat hospital food. 32. The client is diagnosed with aortic stenosis.
4. The client who is scheduled to go to the Which assessment data indicate a complication
whirlpool. is occurring?
1. Barrel chest and clubbing of the fingers.
28. Which client should the charge nurse on the
2. Intermittent claudication and rest pain.
substance abuse unit assign to the licensed
3. Pink, frothy sputum and dyspnea on exertion.
practical nurse (LPN)?
4. Bilateral wheezing and friction rub.
1. The client with chronic alcoholism who has
been on the unit three (3) days. 33. The client who has just received a permanent
2. The client who is complaining of palpitations pacemaker is admitted to the telemetry floor.
and has a history of cocaine abuse. The nurse writes the problem “knowledge
3. The client diagnosed with amphetamine deficit.” Which interventions should be included
abuse who tried to commit suicide. in the plan of care? Select all that apply.
4. The client diagnosed with cannabinoid abuse 1. Take tub baths instead of showers the rest of
who is threatening to leave AMA. his or her life.
2. Do not hold electrical devices near the
29. The telemetry nurse is monitoring the following
pacemaker.
clients. Which client should the telemetry nurse
3. Carry the pacemaker identification card at all
instruct the primary nurse to assess first?
times.
1. The client who has occasional premature
4. Count the radial pulse one (1) full minute
ventricular contractions (PVCs).
every morning.
2. The client post–cardiac surgery who has three
5. Notify the HCP if the pulse is 12 beats slower
(3) unifocal PVCs in a minute.
than the preset rate.
3. The client with a myocardial infarction who
had two (2) multifocal PVCs. 34. Which question should the nurse ask the client
4. The client diagnosed with atrial fibrillation who is being admitted to rule out infective
who has an AP of 116 and no P wave. endocarditis?
1. “Do you have a history of a heart attack?”
30. The nurse is teaching the client in a cardiac
2. “Have you had a cardiac valve replacement?”
rehabilitation unit. Which dietary information
3. “Is there a family history of rheumatic heart
should the nurse discuss with the client?
disease?”
1. No more than 30% of daily food intake
4. “Do you take nonsteroidal anti-inflammatory
should be fats.
medications?”
2. Eighty percent of calories should come from
carbohydrates. 35. The client diagnosed with arterial occlusive
3. Red meat should comprise at least 50% of disease is prescribed an antiplatelet medication,
daily intake. clopidogrel (Plavix). Which assessment data
4. Monounsaturated fat in the daily diet should indicate the medication is effective?
be increased. 1. The client’s pedal pulse is bounding.
2. The client’s blood pressure has decreased.
3. The client does not exhibit signs of a stroke.
4. The client has decreased pain when
ambulating.

20_Colgrove_Ch20.indd 736 02/06/16 5:04 PM


C hapter 20  C omprehensive F inal E xamination 737

36. The client diagnosed with atherosclerosis has 3. Broccoli, asparagus, and kidney beans.
coronary artery disease. The client experiences 4. Liver, cheese, and eggs.
sudden chest pain when walking to the nurse’s
42. The client diagnosed with stomach cancer
station. Which intervention should the nurse
has developed disseminated intravascular
implement first?
coagulopathy (DIC). Which collaborative
1. Administer sublingual nitroglycerin.
intervention should the nurse expect to
2. Apply oxygen via nasal cannula.
implement?
3. Obtain a STAT electrocardiogram.
1. Prepare to administer intravenous heparin.
4. Have the client sit in a chair.
2. Assess for frank hemorrhage from
37. The nurse and the unlicensed assistive personnel venipuncture sites.
(UAP) are caring for clients on a medical floor. 3. Monitor for decreased level of consciousness.
Which nursing task could be delegated to the 4. Prepare to administer total parenteral
UAP? nutrition.
1. Retake the BP on a client who received a
43. The nurse is administering 250 mL of packed
STAT nitroglycerin sublingual.
red blood cells with 50 mL of preservative. The
2. Notify the health-care provider of the client’s
client has no jugular vein distention and has
elevated blood pressure.
clear breath sounds. After the first 15 minutes, at
3. Obtain and document the routine vital signs
what rate should the nurse set the IV infusion
on all the clients on the floor.
pump? _______
4. Call the laboratory technician and discuss a
hemolyzed blood specimen. 44. The 24-year-old African American female client
tells the nurse she has a brother with sickle cell
38. The client with venous insufficiency tells the
disease. She is engaged to be married and is
nurse, “The doctor just told me about my
concerned about giving this disease to her future
disease and walked out of the room. What am I
children. Which information is most important
supposed to do?” Which statement is the nurse’s
to provide to the client?
best response?
1. Tell the client that she won’t pass this on if
1. “I will have your HCP come back and discuss
she has never had symptoms.
this with you.”
2. Encourage the client to discuss this concern
2. “One thing you can do is elevate your legs
with her fiancé.
above your heart while watching TV.”
3. Recommend that she and her fiancé see a
3. “You will probably need to have surgery
genetic counselor.
within a few months.”
4. Discuss the possibility of adopting children
4. “This will go away after you lose about
after she gets married.
20 pounds and start walking.”
45. The nurse is at home preparing for the 7 a.m. to
39. The client is admitted with rule-out leukemia.
7 p.m. shift and has the flu with a temperature of
Which assessment data support the diagnosis
100.4˚F. Which action should the nurse take?
of leukemia?
1. Notify the hospital the nurse will not be
1. Cervical lymph node enlargement.
coming into work.
2. An asymmetrical dark-purple nevus.
2. Go to work and wear an isolation mask when
3. Petechiae covering the trunk and legs.
caring for the clients.
4. Brownish-purple nodules on the face.
3. Request an alternative assignment not
40. The client diagnosed with non-Hodgkin’s involving direct client care.
lymphoma tells the nurse, “I am so tired. I just 4. Take over-the-counter cold medication and
wish I could die.” Which stage of the grieving report to work on time.
process does this statement represent?
46. The client is being admitted into the hospital
1. Anger.
with a diagnosis of pneumonia. Which HCP
2. Denial.
order should the nurse implement first?
3. Bargaining.
1. Initiate intravenous antibiotics.
4. Acceptance.
2. Collect a sputum specimen for culture.
41. The nurse writes the goal “the client will list 3. Obtain a clean voided midstream urinalysis.
three (3) food sources of vitamin B12” for the 4. Request a chest x-ray to confirm the
client diagnosed with pernicious anemia. Which diagnosis.
foods listed by the client indicate the goal has
been met?
1. Brown rice, dried fruits, and oatmeal.
2. Beef, chicken, and pork.

20_Colgrove_Ch20.indd 737 02/06/16 5:04 PM


738 M ed -S urg S uccess

47. Which medical client problem should the nurse 53. Which comment by the client diagnosed with
include in the plan of care for a client diagnosed rule-out Guillain-Barré (GB) syndrome is most
with cardiomyopathy? significant when completing the admission
1. Heart failure. interview?
2. Activity intolerance. 1. “I had a bad case of gastroenteritis a few
3. Paralytic ileus. weeks ago.”
4. Atelectasis. 2. “I never use sunblock and I use a tanning bed
often.”
48. The client comes to the emergency department
3. “I started smoking cigarettes about 20 years
complaining of pain in the right forearm. The
ago.”
nurse notes a large area of redness and edema
4. “I was out of the United States for the last
over the forearm, and the client has an elevated
2 months.”
temperature. Which condition should the nurse
suspect? 54. Which laboratory result warrants immediate
1. Cellulitis. intervention by the nurse for the female client
2. Intravenous drug abuse. diagnosed with systemic lupus erythematosus
3. Raynaud’s phenomenon. (SLE)?
4. Thromboangiitis obliterans. 1. A hemoglobin and hematocrit of 13 g/dL and
40%.
49. The client is performing breast self-examination
2. An erythrocyte sedimentation rate of 9 mm/hr.
(BSE) by the American Cancer Society’s
3. A serum albumin level of 4.5 g/dL.
recommended steps and has completed palpating
4. A white blood cell count of 15,000/mm3.
the breast. Which step is next when completing
the BSE? 55. The client diagnosed with gastroesophageal
1. Stand before the mirror and examine the breast. reflux disease (GERD) has undergone surgery
2. Lean forward and look for dimpling or for a hiatal hernia repair. The client has a
retractions. nasogastric tube in place. Intravenous fluid
3. Examine the breast using a circular motion. replacement is to be at 125 mL/hr plus the
4. Pinch the nipple to see if any fluid can be amount of drainage. The drainage from 0800
expressed. to 0900 is 45 mL. At which rate should the IV
pump be set for the next hour? ______
50. Which assessment information is the most
critical indicator of a neurological deficit? 56. Which assessment data indicate to the nurse the
1. Changes in pupil size. client has a conductive hearing loss?
2. Level of consciousness. 1. The Rinne test results in air-conducted sound
3. A decrease in motor function. being louder than bone-conducted.
4. Numbness of the extremities. 2. The client is unable to hear accurately when
conducting the whisper test.
51. The nurse is initiating a blood transfusion.
3. The Weber test results in the sound being
Which interventions should the nurse
heard better in the affected ear.
implement? Select all that apply.
4. The tympanogram results in the ticking watch
1. Assess the client’s lung fields.
heard better in the unaffected ear.
2. Have the client sign a consent form.
3. Start an IV with a 22-gauge IV catheter. 57. The client reports a twisting motion of the knee
4. Hang 250 mL of D5W at a keep-open rate. during a basketball game. The client is scheduled
5. Check the chart for the HCP’s order. for arthroscopic surgery to repair the injury.
Which information should the nurse teach the
52. The nurse is assessing the client with psoriasis.
client about postoperative care?
Which data support this diagnosis?
1. The client should begin strengthening the
1. Appearance of red, elevated plaques with
surgical leg.
silvery white scales.
2. The client should take pain medication
2. A burning, prickling row of vesicles located
routinely.
along the torso.
3. The client should remain on bedrest for
3. Raised, flesh-colored papules with a rough
two (2) weeks.
surface area.
4. The client should return to the doctor in
4. An overgrowth of tissue with an excessive
six (6) months.
amount of collagen.

20_Colgrove_Ch20.indd 738 02/06/16 5:04 PM


C hapter 20  C omprehensive F inal E xamination 739

58. The nurse is preparing the client newly 64. The client receiving a continuous heparin drip
diagnosed with asthma for discharge. Which complains of sudden chest pain on inspiration
data indicate the teaching about the peak and tells the nurse, “Something is really wrong
flowmeter has been effective? with me.” Which intervention should the nurse
1. “I can continue my usual activities without implement first?
medication if I am in the yellow zone.” 1. Increase the heparin drip rate.
2. “It takes one (1) to two (2) days to establish 2. Notify the health-care provider.
my personal best.” 3. Assess the client’s lung sounds.
3. “When I can’t talk while walking, I need to 4. Apply oxygen via nasal cannula.
take my quick-relief medicine.”
65. The nurse is assessing the client with a
4. “When I am in the red zone, I must take my
pneumothorax who has a closed-chest drainage
quick-relief medication and not exercise.”
system. Which data indicate the client’s
59. Which assessment data indicate the client has condition is stable?
developed a deep vein thrombosis (DVT) in the 1. There is fluctuation in the water-seal
left leg? compartment.
1. A negative Homans’ sign of the left leg. 2. There is blood in the drainage compartment.
2. Increased left-leg calf circumference. 3. The trachea deviates slightly to the left.
3. Elephantiasis of the left lower leg. 4. There is bubbling in the suction
4. Brownish pigmentation of the left lower leg. compartment.
60. The 85-year-old client diagnosed with severe 66. The client is admitted to the intensive care unit
end-stage chronic obstructive pulmonary diagnosed with rule-out adult respiratory distress
disease has a chest x-ray incidentally revealing syndrome (ARDS). The client is receiving
an eight (8)-cm abdominal aortic aneurysm 10 L/min of oxygen via nasal cannula. Which
(AAA). Which intervention should the nurse arterial blood gases indicate the client does not
implement? have ARDS?
1. Discuss possible end-of-life care issues. 1. pH 7.38, Pao2 82, Paco2 45, HCO3 26.
2. Prepare the client for abdominal surgery. 2. pH 7.35, Pao2 74, Paco2 43, HCO3 24.
3. Teach the client how to do pursed-lip 3. pH 7.45, Pao2 60, Paco2 45, HCO3 28.
breathing. 4. pH 7.32, Pao2 50, Paco2 55, HCO3 28.
4. Talk with the family about the client’s
67. The client has gastroesophageal reflux disease.
condition.
Which HCP order should the nurse question?
61. The unlicensed assistive personnel (UAP) 1. Elevate the head of the client’s bed with
notifies the nurse the client diagnosed with blocks.
chronic obstructive pulmonary disease is 2. Administer pantoprazole (Protonix) four (4)
complaining of shortness of breath and times a day.
would like his oxygen level increased. Which 3. A regular diet with no citrus or spicy foods.
intervention should the nurse implement? 4. Activity as tolerated and sit up in a chair for
1. Notify the respiratory therapist (RT). all meals.
2. Ask the UAP to increase the oxygen.
68. The client is diagnosed with an acute
3. Obtain a STAT pulse oximeter reading.
exacerbation of Crohn’s disease. Which
4. Tell the UAP to leave the oxygen alone.
assessment data warrant immediate attention?
62. Which psychosocial client problem should the 1. The client’s WBC count is 10 (× 103)/mm3.
nurse write for the client diagnosed with cancer 2. The client’s serum amylase is 100 units/dL.
of the lung and metastasis to the brain? 3. The client’s potassium level is 3.3 mEq/L.
1. Altered role performance. 4. The client’s blood glucose is 148 mg/dL.
2. Grieving.
69. Which information should the nurse discuss
3. Body image disturbance.
with the client to prevent an acute exacerbation
4. Anger.
of diverticulosis?
63. The client diagnosed with cancer of the larynx 1. Increase the fiber in the diet.
has had a partial laryngectomy. Which client 2. Drink at least 1,000 mL of water a day.
problem has the highest priority? 3. Encourage sedentary activities.
1. Impaired communication. 4. Take cathartic laxatives daily.
2. Ineffective coping.
3. Risk for aspiration.
4. Social isolation.

20_Colgrove_Ch20.indd 739 02/06/16 5:04 PM


740 M ed -S urg S uccess

70. The client diagnosed with peptic ulcer disease 74. The nurse is discussing funeral arrangements
is being discharged. Which nursing task can with the family of a deceased client whose
be delegated to a trained unlicensed assistive organs and tissues are being donated today.
personnel (UAP)? Which information should the nurse discuss
1. Complete the discharge instructions sheet. with family?
2. Remove the client’s saline lock. 1. The family can request an open casket funeral.
3. Clean the client’s room after discharge. 2. Your loved one must wear a long-sleeved shirt.
4. Check the client’s hemoglobin and 3. You might want to have a private viewing only.
hematocrit. 4. This will not delay the timing of the funeral.
71. The client diagnosed with colon cancer tells 75. The public health nurse is discussing hepatitis with
the nurse, “All I do is sit and watch TV all day. a client who is traveling to a third world country
I can barely go to the bathroom.” According to in one (1) month. Which recommendation should
the Oncology Nursing Society’s cancer fatigue the nurse discuss with the client?
scale, how would the nurse document the fatigue 1. A gamma globulin injection.
objectively? 2. A hepatitis A vaccination.
3. A PPD skin test on the left arm.
FATIGUE SCALE
Select the number that best describes how you feel today
4. A hepatitis B vaccination.
76. The client with chronic pancreatitis is admitted
with an acute exacerbation of the disease.
Which laboratory result warrants immediate
No Mild Moderate Extreme The Worst intervention by the nurse?
Fatigue Fatigue Fatigue Fatigue Fatigue
1. The client’s amylase is elevated.
0 1 2 3 4 5 6 7 8 9 10
2. The client’s WBC count is WNL.
1. Mild fatigue. 3. The client’s blood glucose is elevated.
2. Moderate fatigue. 4. The client’s lipase is within normal limits.
3. Extreme fatigue. 77. The client had abdominal surgery and is receiving
4. Worst fatigue ever. bag #5 of total parenteral nutrition (TPN) via a
72. The home health nurse must see all of the subclavian line infusing at 126 mL/hr. The nurse
following clients. Which client should the nurse realizes bag #6 is not on the unit and TPN bag
assess first? #5 has 50 mL left to infuse. Which intervention
1. The client who is postoperative from an open should the nurse implement?
cholecystectomy who has green drainage 1. Decrease the rate of bag #5 to a keep-open
coming from the T-tube. rate.
2. The client diagnosed with congestive heart 2. Prepare to hang a 1,000-mL bag of normal
failure who complains of shortness of breath saline.
while fixing meals. 3. When bag #5 is empty, convert to a heparin
3. The client diagnosed with AIDS dementia lock.
whose family called and reported that the 4. Infuse D10W at 126 mL/hr via the subclavian
client is vomiting “coffee grounds stuff.” line.
4. The client diagnosed with end-stage liver 78. Which priority problem should the clinic nurse
failure who has gained three (3) pounds and is identify for the client who is greater than ideal
not able to wear house shoes. body weight and weighs 87 kg?
73. Which data indicate to the nurse the client with 1. Risk for complications.
end-stage liver failure is improving? 2. Altered nutrition.
1. The client has a tympanic wave. 3. Body image disturbance.
2. The client is able to perform asterixis. 4. Activity intolerance.
3. The client is confused and lethargic.
4. The client’s abdominal girth has decreased.

20_Colgrove_Ch20.indd 740 02/06/16 5:04 PM


C hapter 20  C omprehensive F inal E xamination 741

79. Which assessment data indicate to the nurse 84. The client diagnosed with neurogenic diabetes
the client with diarrhea is experiencing a insipidus (DI) asks the nurse, “What is wrong
complication? with me? Why do I urinate so much?” Which
1. Moist buccal mucosa. statement by the nurse is most appropriate?
2. A 3.6-mEq/L potassium level. 1. “The islet cells in your pancreas are not
3. Tented tissue turgor. functioning properly.”
4. Hyperactive bowel sounds. 2. “Your pituitary gland is not secreting a
necessary hormone.”
80. The client with type 2 diabetes mellitus asks the
3. “Your kidneys are in failure and you are
nurse, “What does it matter if my glucose level is
overproducing urine.”
high? I don’t feel bad.” Which statement by the
4. “The thyroid gland is speeding up all your
nurse is most appropriate?
metabolism.”
1. “The high glucose level can damage your eyes
and kidneys over time.” 85. The client is admitted into the medical unit
2. “The glucose level causes microvascular and diagnosed with heart failure and is prescribed
macrovascular problems.” the thyroid hormone levothyroxine (Synthroid)
3. “As long as you don’t feel bad, everything will orally. Which intervention should the nurse
probably be all right.” implement?
4. “A high blood glucose level will cause you to 1. Call the pharmacist to clarify the order.
get metabolic acidosis.” 2. Administer the medication as ordered.
3. Ask the client why he or she takes Synthroid.
81. The client with type 1 diabetes asks the nurse,
4. Request serum thyroid function levels.
“What causes me to get dehydrated when my
glucose level is elevated?” Which statement 86. Which client should the nurse consider at risk
would be the nurse’s best response? for developing acute renal failure?
1. “The kidneys are damaged and cannot filter 1. The client diagnosed with essential
out the urine.” hypertension.
2. “The glucose causes fluid to be pulled from 2. The client diagnosed with type 2 diabetes.
the tissues.” 3. The client who had an anaphylactic reaction.
3. “The sweating as a result of the high glucose 4. The client who had an autologous blood
level causes dehydration.” transfusion.
4. “You get dehydrated with a high glucose
87. The client diagnosed with chronic renal
because you are so thirsty.”
failure is receiving peritoneal dialysis. Which
82. The client calls the clinic first thing in the assessment by the nurse warrants immediate
morning and tells the nurse, “I have been intervention?
vomiting and having diarrhea since last night.” 1. The dialysate return is cloudy.
Which response is appropriate for the nurse 2. There is a greater dialysate return than input.
to make? 3. The client complains of abdominal fullness.
1. Encourage the client to eat dairy products. 4. The client voided 50 mL during the day.
2. Have the client go to the emergency
88. Which action by the unlicensed assistive
department.
personnel (UAP) requires intervention by the
3. Request the client obtain a stool specimen.
nurse?
4. Tell the client to stay on a clear liquid diet.
1. The UAP used two (2) washcloths when
83. Which signs/symptoms should the nurse expect washing the perineal area.
to assess in the client diagnosed with Addison’s 2. The UAP emptied the indwelling catheter
disease? and documented the amount.
1. Hypotension and bronze skin pigmentation. 3. The UAP applied moisture barrier cream to
2. Water retention and osteoporosis. the anal area.
3. Hirsutism and abdominal striae. 4. The UAP is wiping the client’s perineal area
4. Truncal obesity and thin, wasted extremities. from back to front.

20_Colgrove_Ch20.indd 741 02/06/16 5:04 PM


742 M ed -S urg S uccess

89. The unlicensed assistive personnel (UAP) 93. The elderly client from the long-term care
empties the indwelling urinary catheter for facility is admitted into the hospital diagnosed
a client who is four (4) hours postoperative with septicemia. Which area of the body is the
transurethral resection of the prostate and most appropriate place for the nurse to assess
informs the nurse the urine is red with some the hydration status of the client?
clots. Which intervention should the nurse 1. A
implement first? 2. B
1. Assess the client’s urine output immediately. 3. C
2. Notify the HCP that the client has gross 4. D
hematuria.
3. Explain this is expected with this surgery.
4. Medicate for bladder spasms to decrease A
bleeding.
90. The client with a history of substance abuse
presents to the emergency department
complaining of right flank pain, and the
urinalysis indicates microscopic blood. Which
intervention should the nurse implement?
1. Determine the last illegal drug use.
2. Insert a #22 French indwelling catheter. B
3. Give the client a back massage.
4. Medicate the client for pain.
91. Which assessment data would make the nurse C
suspect the client has cancer of the bladder?
1. Gross painless hematuria.
D
2. Burning on urination.
3. Terminal dribbling.
4. Difficulty initiating the stream.
92. The client asks the nurse, “What are the risk
factors for developing multiple sclerosis?”
Which statement is a risk factor for multiple
sclerosis (MS)?
1. A close relative with MS may indicate a risk
for MS.
2. Living in the southern United States
predisposes a person to MS.
3. Use of tobacco product is the number-one 94. The student nurse accidentally punctured her
risk for developing MS. finger with a contaminated needle. Which action
4. A sedentary lifestyle can cause a person to should the student nurse take first?
develop MS. 1. Notify the infection control nurse.
2. Allow the puncture site to bleed.
3. Report to the emergency department.
4. Cleanse the site with Betadine.
95. Which psychosocial problem should the nurse
identify as priority for a client diagnosed with
rheumatoid arthritis?
1. Alteration in comfort.
2. Ineffective coping.
3. Anxiety.
4. Altered body image.

20_Colgrove_Ch20.indd 742 02/06/16 5:04 PM


C hapter 20  C omprehensive F inal E xamination 743

96. The client is admitted to the medical unit 99. The client is eight (8) hours postoperative
complaining of severe abdominal pain. Which small bowel resection. Which data indicate the
intervention should the nurse implement first? client has had a complication from the surgery?
1. Assess for complications. 1. A hard, rigid, boardlike abdomen.
2. Medicate for pain. 2. High-pitched tinkling bowel sounds.
3. Turn the television on. 3. Absent bowel sounds.
4. Teach relaxation techniques. 4. Complaints of pain at “6” on the pain scale.
97. The female client is admitted to the orthopedic 100. Which intervention will help prevent the nurse
floor with a spiral fracture of the arm and from being sued for malpractice throughout his
multiple contusions and abrasions covering the or her professional practice?
trunk of the body. Her husband accompanies 1. Keep accurate and legible documentation of
her. During the admission interview, which client care.
intervention is priority? 2. A kind, caring, and compassionate bedside
1. Notify the local police department of the manner at all times.
client’s admission. 3. Maintain knowledge of medications for
2. Provide privacy to discuss how the injuries disease processes.
occurred to the client. 4. Follow all health-care provider orders
3. Refer the client to the social worker for names explicitly.
of women’s shelters.
101. According to the nursing process, which
4. Ask the client if she prefers the husband to
interventions should the nurse implement
stay in the room.
when caring for a client diagnosed with a
98. Which interventions should the emergency right-sided cerebrovascular accident (stroke)
department nurse implement for a client who and who has difficulty swallowing? List the
has an AP of 122 and a BP of 80/50? Select all interventions in order of the nursing
that apply. process.
1. Put the client in reverse Trendelenburg 1. Write the client problem of “altered tissue
position. perfusion.”
2. Start an intravenous line with an 18-gauge 2. Assess the client’s level of consciousness and
catheter. speech.
3. Have the client complete the admission 3. Request dietary to send a full liquid tray
process. with Thick-It.
4. Cover the client with blankets and keep warm. 4. Instruct the UAP to elevate the head of the
5. Request the laboratory draw a type and bed 30 degrees.
crossmatch. 5. Note the amount of food consumed on the
dinner tray.

20_Colgrove_Ch20.indd 743 02/06/16 5:04 PM


COMPREHENSIVE FINAL EXAMINATION
ANSWERS AND RATIONALES

1. 1. Birth control pills regulate the hormones in 3. The client will not be undergoing chemo-
the body but will not cause changes in the therapy for at least six (6) weeks to allow the
breast tissue. client to heal; therefore, this is not a priority
2. There is a theory that chocolate increases intervention.
breast discomfort in women with fibrocystic 4. This is important, but when preparing the
breast changes. ­client for surgery, the priority intervention is
3. During the menstrual cycle, pregnancy, and to accomplish presurgical interventions.
menopause, variations in breast tissue occur Content – Medical: Integrated Nursing Process –
and must be distinguished from pathologi- ­Implementation: Client Needs – Safe Effective Care
cal disease. BSE is best performed on days ­Environment, Management of Care: Cognitive Level –
five (5) to seven (7) after menses, counting ­Synthesis: Concept – Male Reproduction.
the first day of menses as day one (1). 4. 1. Oral sex still involves mucous membrane–to–
4. Sexual manipulation of the breast does not mucous membrane contact and disease trans-
cause malignant changes in breast tissue. mission is possible; herpes simplex 2 is simply
Content – Medical: Integrated Nursing Process – herpes simplex 1 transferred to the genitalia.
­Assessment: Client Needs – Physiological Integrity, Reduction 2. This is a myth.
of Risk Potential: Cognitive Level – Analysis: Concept –
3. The more often the person engages in sexual
­Female Reproduction.
contact and the more partners he or she has,
2. 1. Anemia caused by endometriosis occurs over the more likely the person will contract an
time and is not an acute complication such as STD; however, one time is enough to contract
hemorrhaging. a deadly STD, such as AIDS.
2. Pain is the primary complaint of the client; 4. According to developmental theories,
the pain occurs as a result of ectopic tissue adolescents think they are invincible and
bleeding into the abdominal cavity during nothing will happen to them. This attitude
menses. leads adolescents to participate in high-risk
3. Endometriosis does not cause constipation, behaviors without regard to consequences.
and this would not be a priority problem. The Content – Medical: Integrated Nursing Process –
client may experience pain during a bowel Planning: Client Needs – Health Promotion and
movement. ­Maintenance: Cognitive Level – Synthesis: Concept –
4. Dyspareunia is pain during intercourse, and Promoting Health.
this client is in the hospital (and unlikely to be 5. 1. Kegel exercises do not have anything to do
having sex there). with activity endurance.
Content – Medical: Integrated Nursing Process – 2. Kegel exercises are exercises that
Diagnosis: Client Needs – Safe Effective Care ­Environment, strengthen the perineal muscles. Multiple
Management of Care: Cognitive Level – Analysis: Concept –
pregnancies weaken the pelvic muscles,
Female Reproduction.
­resulting in bladder incontinence; a report
3. 1. With a remaining testicle, the client will be of no stress incontinence indicates the
able to maintain sexual potency, but radia- ­Kegel exercises are effective.
tion and chemotherapy may cause the cli- 3. Kegel exercises do not affect pregnancy.
ent to become sterile. Therefore, banking 4. Kegel exercises do not have anything to do
his sperm will allow him to father a child with weight loss.
later in life. Content – Medical: Integrated Nursing Process –
2. Testicular cancer has a 90% cure rate with ­Evaluation: Client Needs – Physiological Integrity,
standard therapy; therefore, completing an ­Physiological Adaptation: Cognitive Level – Synthesis:
­advance directive is not priority. Concept – Female Reproduction.

744

20_Colgrove_Ch20.indd 744 02/06/16 5:04 PM


C hapter 20  C omprehensive F inal E xamination 745

6. 1. The pelvic sonogram, which visualizes the 9. 1. The nurse should encourage the client to
ovary using sound waves, is a diagnostic maintain independent functioning, and del-
test for an ovarian cyst, which would be egating the UAP to feed the client would be
suspected with the client’s signs/symptoms. encouraging dependence.
2. A CBC may be ordered to rule out appendi- 2. Although this is an over-the-counter medica-
citis, but this client does not have right lower tion, a UAP cannot administer any medica-
abdominal pain. tion to a client.
3. A KUB x-ray is ordered for a client with 3. The UAP cannot assess or evaluate any of the
­possible kidney stones. client’s diagnostic information.
4. A CT of the abdomen would not visualize con- 4. The UAP could assist the client to ambu-
tents in the pelvis. late to the shower room and assist with
Content – Medical: Integrated Nursing Process – morning care.
­Planning: Client Needs – Physiological Integrity, Reduction Content – Nursing Management: Integrated Nursing
of Risk Potential: Cognitive Level – Synthesis: Concept – Fe- Process – Planning: Client Needs – Safe Effective Care
male Reproductive. Environment, Management of Care: Cognitive Level –
Synthesis: Concept – Nursing Roles.
7. 1. PSA is a tumor marker monitored to
­determine the progress of the disease and 10. 1. This would not warrant intervention because
treatment, but it is not monitored prior to this indicates the hose are not too tight.
chemotherapy. 2. This indicates the hose are not too tight.
2. Serum calcium levels may be monitored to 3. There should be no wrinkles in the hose
determine metastasis to the bone, but it would after application. Wrinkles could cause
not be done prior to chemotherapy. constriction in the area, resulting in clot
3. The CBC is monitored to determine if the formation or skin breakdown; therefore,
client is at risk for developing an ­infection this would warrant immediate interven-
or bleeding as a result of side effects of the tion by the charge nurse.
chemotherapy medications. The chemo- 4. Antiembolism hose should not be put over
therapy could be held or decreased based a wound; they would restrict the circulation
on these results. to the wound and cause a decrease in wound
4. AFP is a tumor marker monitored to deter- healing.
mine the progress of the disease and treatment, Content – Nursing Management: Integrated Nursing
but it is not monitored prior to chemotherapy. Process – Evaluation: Client Needs – Safe Effective Care
Content – Medical: Integrated Nursing Process – Environment, Management of Care: Cognitive Level –
­Assessment: Client Needs – Physiological Integrity, ­Reduction Synthesis: Concept – Nursing Roles.
of Risk Potential: Cognitive Level – Analysis: Concept –
11. 1. The nurse should suggest walking daily be-
Cellular Regulation.
cause bones need stress to maintain strength.
8. 1. This client is showing signs/symptoms of 2. Vitamin D helps the body absorb calcium.
­hypovolemic shock and should not be assigned 3. Smoking interferes with estrogen’s
to an inexperienced nurse. ­protective effects on bones, promoting
2. Confusion could be a sign of many complica- bone loss.
tions after surgery, so this client should not be 4. Lack of exposure to sunlight results in
assigned to an inexperienced nurse. ­decreased vitamin D, which is necessary for
3. This client is being transferred to the ICU, calcium absorption and normal bone mineral-
which indicates the client is not stable; there- ization. The client should go outside.
fore, this client should not be assigned to an 5. The client is at risk for fractures;
inexperienced nurse. therefore, a fall could result in serious
4. A myelogram is a routine diagnostic test. complications.
With minimal instruction, an inexperienced Content – Medical: Integrated Nursing Process –
nurse could care for this client. Planning: Client Needs – Physiological Integrity,
Content – Nursing Management: Integrated Nursing ­Physiological Adaptation: Cognitive Level – Synthesis:
Process – Planning: Client Needs – Safe Effective Care Concept – Nursing Roles.
Environment, Management of Care: Cognitive Level –
Synthesis: Concept – Nursing Roles.

20_Colgrove_Ch20.indd 745 02/06/16 5:04 PM


746 M ed -S urg S uccess

12. 1. The physical therapist focuses on evaluating, Content – Surgical: Integrated Nursing Process –
diagnosing movement dysfunctions (injured Diagnosis: Client Needs – Safe Effective Care ­Environment,
tissues and structures), and treating these Management of Care: Cognitive Level – Analysis:
­issues. The PT helps restore movement and Concept – Mobility.
mobility. 15. 36%.
2. The occupational therapist focuses on  Each leg is 18%, with the anterior surface
evaluating and improving functional abili- (front) being 9%. Because the anterior of
ties to optimize independence and address both legs is burned (9% each), that would be
activities of daily living, which would be an 18%. That 18% plus the anterior surface of
appropriate referral. the trunk, which is 18%, totals 36% of the
3. Workers’ compensation is an insurance total body surface burned.
­provider for the employer and employee to Content – Medical: Integrated Nursing Process –
cover medical expenses and loss of wages. ­Assessment: Client Needs – Physiological Integrity,
This is not an appropriate referral by the ­Reduction of Risk Potential: Cognitive Level – Analysis:
­rehabilitation nurse. Concept – Skin Integrity.
4. The client may need this referral but after the
occupational therapist has worked with the 16. 1. Wasting syndrome occurs in clients with
client and determined the ability to perform protein-calorie malnutrition. This syndrome
skills. leads to the pressure ulcers not healing, but
it is not a complication of the pressure
Content – Medical: Integrated Nursing Process –
ulcers.
Implementation: Client Needs – Physiological Integrity,
Physiological Adaptation: Cognitive Level – ­Application:
2. Stage IV pressure ulcers frequently
Concept – Nursing Roles. ­extend to the bone tissue, predisposing
the client to developing a bone infection—
13. 1. The nurse would expect the client with a osteomyelitis—which can rarely be treated
fractured right leg to have pain but it would effectively.
not warrant immediate intervention. 3. Renal calculi may be a result of immobility,
2. The nurse would assess the client’s pedal or but they are not a complication of pressure
posterior tibial pulse for a client with a frac- ulcers.
tured right tibia. 4. Cellulitis is an inflammation of the skin,
3. Any abnormal neurovascular assessment which is not a complication of pressure ulcers.
data, such as coldness, paralysis, or pares- Content – Medical: Integrated Nursing Process –
thesia, warrant immediate intervention by Planning: Client Needs – Physiological Integrity,
the nurse. ­Physiological Adaptation: Cognitive Level – Synthesis:
4. Ecchymosis is bruising and would be ex- ­Concept – Skin Integrity.
pected in the client who has a fractured tibia.
17. 1. Contact dermatitis is a type of dermatitis
Content – Medical: Integrated Nursing Process –
caused by a hypersensitivity response. In
­Assessment: Client Needs – Physiological Integrity,
­Reduction of Risk Potential: Cognitive Level – Analysis:
this case, it is a hypersensitivity reaction
Concept – Mobility. to metal salts in the watch the client is
wearing. Anytime the nurse assesses red-
14. 1. The client is being discharged, so a self-care ness or irritation in areas where jewelry
deficit would not be a potential complication. (such as rings, watches, necklaces) or
2. The client is being discharged and is am- clothing (such as socks, shoes, or gloves)
bulating; therefore, impaired skin integrity are worn, the nurse should suspect con-
should not be a problem. tact dermatitis.
3. The client would have been taking a prophy- 2. Herpes simplex 1 virus occurs in oral or ­nasal
lactic anticoagulant but would not be at risk mucous membranes.
for abnormal bleeding. 3. Impetigo is a superficial infection of the skin
4. The client must inform all HCPs, espe- caused by staph or strep infection and occurs
cially the dentist, of the hip prosthesis on the body, face, hands, or neck.
because the client should be taking pro- 4. Seborrheic dermatitis is a chronic inflamma-
phylactic antibiotics prior to any invasive tion of the skin involving the scalp, eyebrows,
procedure. Any bacteria invading the body eyelids, ear canals, nasolabial folds, axillae,
may cause an infection in the joint, and and trunk.
this may result in the client having the
prosthesis removed.

20_Colgrove_Ch20.indd 746 02/06/16 5:04 PM


C hapter 20  C omprehensive F inal E xamination 747

Content – Medical: Integrated Nursing Process – 21. 1. This statement indicates the client does
Diagnosis: Client Needs – Safe Effective Care ­Environment, not understand the discharge teaching.
Management of Care: Cognitive Level – Comprehension: The client will not be able to drive until
Concept – Skin Integrity. the client is seizure free for a certain pe-
18. 1. A bone scan would not be ordered unless a riod of time. The laws in each state differ.
biopsy proves malignant melanoma. 2. Lack of sleep is a risk factor for having
2. This is an abnormal-appearing mole on seizures.
the skin, and the HCP would order a 3. Noncompliance with medication is a risk
­biopsy to confirm skin cancer. ­factor for having a seizure.
3. A CEA is a test used to mark the presence 4. If the client has a seizure in the bathtub, the
or prognosis of several cancers but not skin client could drown.
cancer. Content – Medical: Integrated Nursing Process –
4. A sonogram would not be ordered to ­Evaluation: Client Needs – Physiological Integrity,
­diagnose skin cancer. ­Physiological Adaptation: Cognitive Level – Synthesis:
­Concept – Neurologic Regulation.
Content – Medical: Integrated Nursing Process –
­Planning: Client Needs – Physiological Integrity, Reduction 22. 1. Using the vital sign machine to take the
of Risk Potential: Cognitive Level – Synthesis: Concept – ­client’s BP is an appropriate intervention.
Skin Integrity. 2. The body temperature of an unconscious
19. 1. This could be an appropriate long-term goal client should never be taken by mouth
for the client based on the extent of injury, because the client is unable to safely hold
but it is not an appropriate short-term goal. the thermometer.
2. This is an appropriate long-term goal to 3. Verifying the blood pressure manually is an
­prevent immobility complications, but it is appropriate intervention if the UAP questions
not an appropriate short-term goal. the automatic blood pressure reading. This
3. The worse-case scenario with a closed action should be praised.
head injury is increased intracranial pres- 4. Counting the respiration for 30 seconds and
sure resulting in death. An appropriate multiplying by two (2) is appropriate.
short-term goal would be the ICP re- Content – Medical: Integrated Nursing Process –
maining within normal limits, which is Evaluation: Client Needs – Safe Effective Care
5 to 15 mm Hg. ­Environment, Management of Care: Cognitive Level –
4. This is a psychosocial goal, which would not Synthesis: Concept – Nursing Roles.
be a short-term goal, and the client may not 23. 1. This is incorrect information for radiation
be angry. The stem did not indicate the client therapy. It is correct for chemotherapy.
is angry. 2. This is a therapeutic response, which does
Content – Medical: Integrated Nursing Process – not answer the client’s question.
Planning: Client Needs – Safe Effective Care E
­ nvironment, 3. Radiation therapy can cause permanent
Management of Care: Cognitive Level – Synthesis: damage to the hair follicles and the hair
­Concept – Neurologic Regulation. may not grow back at all; the nurse should
20. 1. This is negating the client’s feelings and will answer the client’s question honestly.
abruptly end any conversation the client may 4. This is not a true statement.
want or need to have. Content – Medical: Integrated Nursing Process –
2. This is imposing the nurse’s religious beliefs Implementation: Client Needs – Physiological I­ ntegrity,
on the client and these are clichés, which do Physiological Adaptation: Cognitive Level – Application:
not address the client’s feelings. Concept – Cellular Regulation.
3. This is explaining why the client survived, but
the client isn’t really asking for information.
The client is expressing and showing emo-
tions that must be addressed by the nurse.
4. This is a therapeutic response which
­allows the client to ventilate feelings.
Content – Medical: Integrated Nursing Process –
­Implementation: Client Needs – Psychosocial Integrity:
Cognitive Level – Application: Concept – Trauma.

20_Colgrove_Ch20.indd 747 02/06/16 5:04 PM


748 M ed -S urg S uccess

24. 1. A positive Brudzinski’s sign—flexion of the 27. 1. Myalgia is muscle pain, which is expected in a
knees and hip when the neck is flexed—­ client diagnosed with encephalitis.
indicates the presence of meningitis. There- 2. The client complaining of chest pain is
fore, the treatment is not effective. Sensitivity priority. Remember Maslow’s hierarchy of
to light is a common symptom of meningitis. needs.
2. This does not indicate whether the meningi- 3. Refusing to eat hospital food is not a priority.
tis is resolving. 4. The client going to the whirlpool is stable
3. Kernig’s sign—the leg cannot be extended and is not a priority over chest pain.
when the client is lying with the thigh flexed Content – Medical: Integrated Nursing Process –
on the abdomen—is a sign of meningitis. An Assessment: Client Needs – Safe Effective Care
elevated temperature indicates the client still ­Environment, Management of Care: Cognitive Level –
has meningitis. Analysis: Concept – Assessment.
4. The client does not have nuchal rigidity, 28. 1. The client should be assessed for delirium
which indicates the client’s treatment is tremens and should be assigned to a regis-
effective. tered nurse.
Content – Medical: Integrated Nursing Process – 2. Palpitations indicate cardiac involvement,
­Assessment: Client Needs – Physiological Integrity, and because the client has a history of cocaine
­Physiological Adaptation: Cognitive Level – Analysis:
abuse, this client should be assigned to a reg-
Concept – Neurologic Regulation.
istered nurse.
25. 1. An MRI is not able to confirm the diagnosis 3. This client is at high risk for injury to self and
of Parkinson’s disease. should be assigned to a registered nurse and
2. This is the portion of the brain where Parkin- be on one-to-one precautions.
son’s disease originates, but this area lies deep 4. The client has a right to leave against
in the brain and cannot be biopsied. medical advice (AMA), and marijuana
3. This is a surgery that relieves some of the abuse is not life threatening to him or
symptoms of Parkinson’s disease. To be eli- to others. Therefore, the LPN could be
gible for this procedure, the client must have ­assigned to this client.
failed to achieve an adequate response with Content – Nursing Management: Integrated Nursing
medical treatment. Process – Planning: Client Needs – Safe Effective Care
4. Many diagnostic tests are completed to Environment, Management of Care: Cognitive Level –
rule out other diagnoses, but Parkinson’s Synthesis: Concept – Nursing Roles.
disease is diagnosed based on the clinical
29. 1. An occasional PVC does not warrant inter-
presentation of the client and the pres-
vention; it is normal for most clients.
ence of two of the three cardinal mani-
2. Less than six (6) unifocal PVCs in one (1)
festations: tremor, muscle rigidity, and
minute is not life threatening.
bradykinesia.
3. Multifocal PVCs indicate the ventricle is
Content – Medical: Integrated Nursing Process – irritable, and this client is at risk for a car-
­Planning: Client Needs – Physiological Integrity, Reduction diac event such as ventricular fibrillation.
of Risk Potential: Cognitive Level – Synthesis: Concept –
4. Atrial fibrillation is not life threatening, and
Neurologic Regulation.
the nurse would expect the client not to have
26. 1. An oral coagulant is ordered if the TIA was a P wave when exhibiting this dysrhythmia.
caused by atrial fibrillation, and that informa- Content – Nursing Management: Integrated Nursing
tion is not presented in the stem. Process – Planning: Client Needs – Safe Effective Care
2. Atherosclerosis is the most common cause Environment, Management of Care: Cognitive Level –
of a TIA or stroke, and taking a baby aspi- Synthesis: Concept – Nursing Roles.
rin every day helps prevent clot formation
around plaques.
3. If the client had hypertension, a beta blocker
may be prescribed, but this information is not
in the stem.
4. Anticonvulsant medications are not
­prescribed to help prevent TIAs.
Content – Medical: Integrated Nursing Process –
Planning: Client Needs – Physiological Integrity,
­Pharmacological and Parenteral Therapies: Cognitive
Level – Synthesis: Concept – Medication.

20_Colgrove_Ch20.indd 748 02/06/16 5:04 PM


C hapter 20  C omprehensive F inal E xamination 749

30. 1. This is a correct statement. The recom- 33. 1. Once the chest incision heals, the client can
mended proportions of food are 50% shower or bathe, whichever the client prefers.
­carbohydrates, 30% or less from fat, and 2. Electrical devices may interfere with the
20% protein. functioning of the pacemaker.
2. Only 50% of the calories should come from 3. This alerts any HCP as to the presence of
carbohydrates. a pacemaker.
3. Red meat is an excellent source of protein but 4. The client should be taught to take the
should only comprise 20% of the diet, and radial pulse for one (1) full minute before
red meat is very high in fat. getting out of bed. If the count is more
4. Polyunsaturated fats, not the monounsatu- than five (5) bpm less than the preset rate,
rated fats, are the better fats. the HCP should be notified immediately
Content – Medical: Integrated Nursing Process – because this may indicate the pacemaker
­Planning: Client Needs – Physiological Integrity, Basic Care is malfunctioning.
and Comfort: Cognitive Level – Synthesis: Concept – 5. The client should notify the HCP if the pulse
Nutrition. is five (5) bpm less than the preset rate. This
may indicate pacemaker malfunction.
31. 1. The client diagnosed with CHF will be short
of breath on exertion and with activity. The Content – Medical: Integrated Nursing Process –
significant other should report difficulty Planning: Client Needs – Physiological Integrity,
­Physiological Adaptation: Cognitive Level – Synthesis:
breathing not subsiding with rest or stopping
Concept – Perfusion.
the activity.
2. Two (2) to three (3) pounds of weight gain 34. 1. Having a history of a myocardial infarction
reflects fluid retention as a result of heart is not a risk factor for developing infective
failure, which warrants notifying the HCP. endocarditis.
3. The caregiver must not administer the di- 2. This is why clients must receive prophy-
goxin if the radial pulse is less than 60 bpm. lactic antibiotic treatment before dental
The apical pulse is more difficult to assess in work and invasive procedures.
a client than the radial pulse. 3. A personal history of rheumatic fever, not a
4. The client in end-stage CHF is dying and family history, increases the risk of develop-
should not exercise daily; activity intolerance ing infective endocarditis.
as a result of decreased cardiac output is the 4. NSAIDs have no effect on the development
number-one life-limiting problem. of infective endocarditis.
Content – Medical: Integrated Nursing Process – Content – Medical: Integrated Nursing Process –
Planning: Client Needs – Physiological Integrity, ­Assessment: Client Needs – Physiological Integrity,
­Physiological Adaptation: Cognitive Level – Synthesis: ­Reduction of Risk Potential: Cognitive Level – Analysis:
Concept – Nursing Roles. Concept – Perfusion.

32. 1. Barrel chest and clubbing of the fingers are 35. 1. The client’s pedal pulse does not evaluate the
signs of chronic lung disease. effectiveness of this medication.
2. Intermittent claudication and rest pain are 2. This medication is not administered to help
signs of peripheral arterial disease. decrease blood pressure.
3. Pink, frothy sputum and dyspnea on 3. This medication inhibits platelet aggre-
­exertion are signs of congestive heart gation and is considered effective when
failure, which occurs when the heart can there is a decrease in atherosclerotic
no longer compensate for the strain of an events, an example of which is a stroke.
incompetent valve. 4. This medication will not help the pain associ-
4. Friction rub occurs with pericarditis, and ated with arterial occlusive disease.
­bilateral wheezing occurs with asthma. Content – Medical: Integrated Nursing Process –
Content – Medical: Integrated Nursing Process – ­Evaluation: Client Needs – Physiological Integrity,
­Assessment: Client Needs – Physiological Integrity, ­Pharmacological and Parenteral Therapies: Cognitive
­Reduction of Risk Potential: Cognitive Level – Synthesis: Level – Synthesis: Concept – Medication.
Concept – Perfusion.

20_Colgrove_Ch20.indd 749 02/06/16 5:04 PM


750 M ed -S urg S uccess

36. 1. Sublingual nitroglycerin is the medication 4. Brownish-purple nodules on the face indicate
of choice for angina, but it is not the first Kaposi’s sarcoma, a complication of AIDS.
intervention. Content – Medical: Integrated Nursing Process –
2. Applying oxygen is appropriate, but it is not ­Assessment: Client Needs – Physiological Integrity,
the first intervention. ­Reduction of Risk Potential: Cognitive Level – Analysis:
3. A STAT ECG should be ordered, but it is not Concept – Hematologic Regulation.
the first intervention. 40. 1. This statement does not represent the anger
4. Stopping the client from whatever activity stage of grieving.
the client is doing is the first interven- 2. This statement does not represent the denial
tion because this decreases the oxygen stage of grieving.
demands of the heart muscle and may 3. This statement does not represent the bar-
­decrease or eliminate the chest pain. gaining stage of grieving.
Content – Nursing Management: Integrated Nursing 4. This statement indicates the client is
Process – Implementation: Client Needs – Safe Effective ready to die and is in the acceptance stage
Care Environment, Management of Care: Cognitive
of the grieving process.
Level – Analysis: Concept – Perfusion.
Content – Medical: Integrated Nursing Process –
37. 1. This client is unstable and received medica- ­Diagnosis: Client Needs – Psychosocial Integrity: Cognitive
tion for chest pain. The nurse cannot delegate Level – Analysis: Concept – Grief and Loss.
any task for a client who is unstable.
41. 1. Brown rice, dried fruit, and oatmeal are
2. The UAP cannot notify the HCP because
sources of nonheme iron. Nonheme iron
UAPs are not allowed to take verbal or
comes from vegetable sources.
­telephone orders.
2. Beef, chicken, and pork are sources of heme
3. The UAP can take routine vital signs. The
iron or animal sources of iron.

You might also like